Sunteți pe pagina 1din 1080

Anul IV, Nr.

Ianuarie Iunie 2002

RECREATII ICE MATEMAT

REVIST| DE MATEMATIC| PENTRU ELEVI {I PROFESORI

= 1

Editura Crengu a Gld u IA I, 2002

Anul IV, Nr. 1

Ianuarie - Iunie 2002

RECREATII MATEMATICE

REVIST| DE MATEMATIC| PENTRU ELEVI {I PROFESORI

ei = 1

Apare cu sprijinul
FILIALEI IA I a SOCIET II de TIIN E MATEMATICE

IASI, 2002

Semnifica ia formulei de pe copert :


ntr-o form concis , formula e i matematicii: ARITMETICA GEOMETRIA ALGEBRA ANALIZA MATEMATIC

1 leag cele patru ramuri fundamentale ale

reprezentat de 1 reprezentat de reprezentat de i reprezentat de e

Redac ia revistei :
Petru ASAFTEI , Temistocle BRSAN, Dan BRNZEI, C t lin CALISTRU, Constantin CHIRIL , Constantin COCEA, Eugenia COHAL, Adrian CORDUNEANU, Paraschiva GALIA, Mihai GRTAN, Paul GEORGESCU, Dumitru GHERMAN (Pa cani), Gheorghe IUREA, Lucian Georges L DUNC , Gabriel MR ANU, Gabriel POPA, Dan POPESCU (Suceava), Florin POPOVICI (Bra ov), Maria RACU, Petru R DUCANU, Alin SPUM

Coordonatorul num rului : Temistocle BRSAN Tehnoredactare computerizat :


Ionu AILINC I, C t lin CALISTRU, Paul GEORGESCU, Gabriel POPA, Vlad ROTARIU i un grup de elevi de la Liceul Teoretic Gr. Moisil i Liceul Teoretic G. Ibr ileanu

Adresa redac iei:


Catedra de Matematic Universitatea Tehnic Gh. Asachi Ia i Bd. Carol I, nr.11, 6600, Ia i Tel. 032 213737 / int. 123 E-mail: acord@math.tuiasi.ro

EDITURA CRENGU A GLD U Toate drepturile rezervate ISSN 1582 - 1765 Bd. N. Iorga, Bl. K2, ap. 4 Tel. / Fax: 032 - 230598 IA I, 6600

Alin Spum Alin Spum


Cu mult triste e i p rere de r u anun m dispari ia n luna decembrie, anul 2001, a unuia dintre cei mai ata a i i entuzia ti membri ai redac iei revistei noastre. S-a n scut n Ia i , la 22 noiembrie 1974. n acest ora drag lui, a parcurs n mod str lucit toate treptele nv mntului, de la cel elementar i pn la cel superior: a absolvit Liceul de informatic Gr. Moisil i a ob inut licen a n matematic la Universitatea Al. I. Cuza. A ocupat prin concurs un post de preparator la Catedra de algebr , Facultatea de matematic a universit ii ie ene. Pasionat de matematic , s-a ndreptat spre domeniul Algebrei abstracte i aplica iilor acesteia: dup doi ani de masterat, s-a nscris la doctorat i a sus inut examenele i referatele programate. Lucra la teza de doctorat i urma s plece n Germania, beneficiind de o burs de studii, n momentul dispari iei sale. Timpul a pus cap t irului de proiecte n care se avnta i pentru realizarea c rora era att de nzestrat. A muncit cu mult pasiune i pricepere i a ob inut de timpuriu frumoase rezultate. Ca elev, a participat la fazele finale ale Olimpiadelor de matematic , Concursurilor rezolvitorilor Gazetei Matematice i multor concursuri interjude ene i a fost r spl tit cu premii i men iuni. Ca student, a fost premiat la mai multe edi ii ale Concursului studen esc Traian Lalescu i la Sesiunile de comunic ri tiin ifice studen e ti. Cadru didactic n nv mntul superior fiind, a participat cu regularitate la Conferin ele na ionale de Algebr din ultimii ani. A dorit mult ca Ia ul s aib o revist de matematic elementar i chiar a avut o ini iativ personal n acest sens. ntors n ar , dup o perioad de studii la Universitatea din Udine (Italia), s-a ata at imediat de revista Recrea ii matematice, care n acel moment preg tea apari ia primului s u num r. Cu competen i f r a- i menaja energia i timpul, a contribuit la cre terea calit ii revistei cu fiecare num r nou ap rut. Priceperea sa n tehnoredactarea pe calculator a fost o ans a revistei la primele sale apari ii. A fost neobosit n promovarea i distribuirea revistei, folosind orice prilej pe care-l avea: concursuri, tabere de matematic etc. Era att de tn r i era o prezen att de vie printre noi, membrii redac iei i colaboratorii revistei, nct vestea dispari iei lui a fost primit ca un fapt absurd i incredibil. Va r mne mereu n amintirea noastr ca un exemplu de pasiune i d ruire, h rnicie i modestie, entuziasm i competen . i nu avem alt mod mai potrivit de a cinsti memoria celui ce a fost Alin Spum , dect asigurnd apari ia revistei, la care a inut att de mult, f cnd-o ct mai interesant i atractiv .

REDAC IA REVISTEI

Petre Osmatescu ( 1925 2001 )


Petre Osmatescu s-a n scut n Romnia la 20 iunie 1925 n satul V deni, comuna Moldova, jude ul Cetatea Alb . Ca o nedreptate istoric , ora ul omonim din antichitate, nfiin at n secolul VI nainte de Christos sub denumirea de Tyras i f cnd parte din Moldova lui tefan cel Mare, este rebotezat ast zi Bielgorod Dnestrovski, iar comuna Moldova - Krutoiarovska i apar in amndou Ucrainei. coala primar a terminat-o n comuna natal n 1938 (care, la acea dat , mai apar inea Romniei). n perioada 1945-1947, urmeaz cursurile preg titoare de pe lng Institutul Pedagogic Ion Creang din Chi in u i apoi (1947-1951) cursurile institutului propriuzis, ncepndu- i cariera ca profesor de matematic i fizic la coala Medie nr. 2 din Orhei (1951-1956), unde este ales pre edintele sec iei metodice de fizic i matematic (19541956). n 1956, este transferat la Institutul Pedagogic din Tiraspol. n 1962, nfiin eaz la Tiraspol Seminarul de Topologie General , Geometrie Analitic i Func ii Reale pentru studen i i tinerele cadre. n 1964, i sus ine teza de doctorat n matematic i fizic . n 1965, devine conferen iar, iar n 1979 profesor la Catedra de matematici superioare nr. 1 de la Institutul Politehnic din Chi in u. Cu aceast ocazie, introduce metodele analizei vectoriale n cursul de analiz matematic . Tot n 1965, nfiin eaz i conduce Seminarul de Topologie General de la Universitatea de Stat din Moldova. El este membru al Societ ii Matematice din Moldova i Pre edinte al Societ ii East-West. A fost numit n 1996 Persoan Eminent a Educa iei Publice i a primit Mileniul 2000. Medalie de Onoare. El a fost fondatorul i organizatorul celor apte Simpozioane Tiraspoliene de Topologie i Aplica ii, pe care le-a editat. Cercet rile i rezultatele sale sunt de Topologie general i Teoria func iilor . A inut conferin e n Canada, Egipt, Elve ia, Germania, Romnia, Spania i Turcia. Un ntreg ciclu de conferin e l-a inut la Universitatea Tehnic din Ia i, cu care s-a semnat i un acord de cooperare interuniversitar . O maladie necru toare a ntrerupt firul vie ii sale n noiembrie 2001, punnd astfel cap t unei activit i prodigioase. Regretatul Profesor Petre Osmatescu , al c rui prieten am fost, s-a impus pe dou fronturi, care se ntrep trund : pe de o parte, pe plan matematic, n topologie, prin introducerea spa iilor subtile , unde elementul de baz este diviziunea punctului (ceva analog cu divizarea atomului din fizic ), teorie care i-a g sit aplica ii n economie, finan e i sociologie, ct i prin nfiin area la Chi in u a revistei Scripta Scientiarum Matematicarum, pe care a condus-o pn n ultima clip , iar pe de alt parte, pe plan romnesc, s-a dedicat luptei, prin mijloace culturale, pentru reunificarea Basarabiei cu Romnia, deziderat, care sper m s nu r mn doar un vis al nostru.

Prof. dr. Petru CARAMAN

Niels Henrik Abel 200 de ani de la na tere


A fost una dintre fiin ele de excep ie ce natura rareori d na tere n cursul unui secol. August Leopold Crelle N. H. Abel (1802-1829) s-a n scut la Finde ntr-o familie cu frumoase tradi ii. Moartea timpurie a tat lui i falimentul bunicului s u au dus familia la o existen precar . Se cunosc pu ine lucruri despre Abel. Ocupa iile lui favorite au fost teatrul i matematica. La 19 ani, student fiind, N. H. Abel propune o rezolvare algebric a ecua iei de gradul a cincilea. Aceasta con inea o eroare pe care tot el o g se te doi ani mai trziu. n Mmoires sur les l'quations algbriques o on dmontre l'impossibilit de la rsolution de l'quation gnrale du cinquime degr (1824), Abel pune cap t ncerc rilor de continuare a lucr rilor lui Tartalia, Cardano i Ferrari, ce au durat mai bine de un secol. n 1825 face o c l torie de studii n Germania, Fran a i Italia. La Berlin cunoa te pe A. L. Crelle, fondatorul revistei Journal fr die reine und angewandte Mathematik. Multe din lucr rile lui Abel au fost publicate n aceast revist . n iulie 1826 se afl la Paris, unde cunoa te pe cei mai importa i membri ai Academiei de tiin e: Cauchy, Laplace, Legendre .a. Preg te te i depune la acest nalt for de tiin cea mai important lucrare a sa Mmorie sur une proprit gnrale dune classe trs tendue des fonctions transcendentes. A a teptat zadarnic un r spuns din partea Academiei de tiin e, n particular din partea lui Cauchy i Legendre. Aici, n Paris, apar primele semne ale bolii sale. R spunsul mult a teptat vine la cteva luni dup moartea sa. n 1830 i s-a acordat postum, pentru aceast lucrare, marele premiu al Academiei de tiin e (mpreun cu Jacobi). Memoriul a disp rut pentru un timp i a fost publicat abia n 1841. Din Paris se ntoarce acas unde i tr ie te ultimii doi ani de via mpov rat de griji. Moare la numai 26 de ani, r pus de tuberculoz , boal incurabil n acea vreme. n cinci ani de crea ii originale i profunde, Niels Henrik Abel a reformat o parte a analizei matematice i a deschis drumuri noi n teoria grupurilor i n geometria analitic i algebric . A adus contribu ii n teoria ecua iilor algebrice i ecua iilor abeliene, teoria seriilor binomiale i a seriilor n general, teoria func iilor eliptice i, mai general, a func iilor algebrice. Un num r mare de no iuni i teoreme importante poart numele lui : integrale abeliene, grupuri abeliene, teorema Abel-Ruffini privind imposibilitatea rezolv rii prin radicali a ecua iilor algebrice de grad mai mare dect patru etc. Recent guvernul norvegian a hot rt crearea unui premiu care s poarte numele marelui matematician, Premiul Abel. Acesta este destinat domeniului matematicilor i se acord anual ncepnd cu anul 2002. Valoarea premiului este de 200.000.000 coroane norvegiene ( 27.100.000 euro). Crearea unui Premiu Abel a fost propus i n 1902 de c tre regele Oscar II al Suediei i Norvegiei. Datorit separ rii celor dou ri n 1905, proiectul a fost abandonat. Pn acum, cel mai prestigios premiu pentru matematicieni a fost medalia Fields, acordat din patru n patru ani, ncepnd din 1936. Premiul Abel este ca valoare i importan comparabil cu Premiul Nobel (ce nu se acord matematicienilor). Prof. dr. Temistocle BRSAN 4

ARTICOLE SI NOTE MATEMATICE


Asupra ipotezei lui Goldbach
Petru MINU
1

Una din problemele care au impulsionat considerabil dezvoltarea teoriei numerelor i care nu este nc rezolvat , n ciuda eforturilor f cute n ultimii 250 de ani de matematicieni dintre cei mai renumi i, este a a numita ipotez a lui Goldbach. Problema a fost pus pentru prima dat n coresponden a dintre Christian Goldbach i Leonhard Euler, la vremea respectiv matematicieni la Academia din Sankt Petersburg. La 7 iunie 1742, Goldbach i scrie lui Euler: Evident, orice num r (natural) este suma a trei numere prime. La 30 iunie 1742, Euler i r spunde: Consider ca o teorem pe deplin adev rat c orice num r par este suma a dou numere prime, de i nu pot s o demonstrez. Prin tradi ie s-a p strat sub denumirea de ipoteza lui Goldbach urm toarea afirma ie: Propozi ia 1. Orice num r natural par, mai mare ca 2 , este suma a dou numere prime (de exemplu, 4 = 2 + 2 , 6 = 3 + 3 , 8 = 3 + 5 , 10 = 3 + 7 = 5 + 5 etc.). A fost formulat i o ipotez mai tare: Propozi ia 2. Orice num r natural par, mai mare ca 6, este suma a dou numere prime diferite. Propozi ia 2 a fost verificat de Pipping pentru toate numerele pare pn la 100000. Teorema 1. Propozi ia 2 este echivalent cu afirma ia: orice num r natural mai mare ca 17 este suma a trei numere prime diferite. Demonstra ie. Observ m c dac n se reprezint sub forma n p q r , p, q, r prime, n cazul cnd n este impar toate trei numerele p, q, r sunt impare, iar n cazul cnd n este par unul dintre ele este par (deci 2) i celelalte dou impare. S mai observ m c nu exist numere n N care s admit patru reprezent ri de forma: n 3 3 r1 , n 5 5 r2 , n 7 7 r3 , n 11 11 r4 ( r1 , r2 , r3 , r4 prime). ntr-adev r, n caz contrar ar rezulta c r1 r4 16 , r2 r4 12 , r3 r4 8 . Ca urmare, dac r4 este de forma r4 3k , atunci r2 este multiplu de 3; dac r4 3k 1 ,

atunci r3 este multiplu de 3 i dac r4 3k 2 , atunci r1 este multiplu de 3, absurd. Presupunem c Propozi ia 2 este adev rat fie n 17 , n impar. Numerele pare n 3 , n 5 , n 7 , n 11 sunt mai mari ca 6 i se scriu sub forma unei sume de dou numere prime diferite: n 3 p1 q1 , n 5 p 2 q 2 , n 7 p 3 q 3 , n 11 p 4 q 4 . Conform observa iei de mai sus, din aceste patru reprezent ri ale lui n ca sume de trei numere prime exist cel pu in una n care to i termenii sunt diferi i. Dac n este par, n 2 este par i se poate scrie sub forma n 2 p q , unde p i q sunt dou numere prime impare diferite, deci n 2 p q .

Prof. dr. , Catedra de algebr , Univ. Al. I. Cuza, Ia i

Invers, presupunem c orice num r mai mare ca 17 este suma a trei numere prime diferite. Dac n este par, n 15 , avem n 2 par , n 2 17 , n 2 2 p q , deci n p q , p i q numere prime diferite. Pentru 6 n 15 avem: 8=3+5, 10=3+7, 12=5+7, 14=3+11. Cu aceasta demonstra ia este complet .
Teorema 2. Dac ipoteza lui Goldbach este adev rat , orice num r natural impar, mai mare ca 7, este suma a trei numere prime impare. Demonstra ie. Dac 2n 1 7 , 2n 1 3 2 n 1 4 i conform ipotezei lui Goldbach 2n 1 3 p q , p, q prime, impare. Deci 2n 1 3 p q . Observa ie. I.M. Vinogradov a demonstrat, n 1937, c orice num r natural impar

mai mare ca 3 3 este sum de trei numere prime. Consider m propozi ia:
Propozi ia 3. Orice num r natural impar, mai mare ca 7, este sum de trei numere prime impare. Pentru a ar ta c Propozi ia 3 este o teorem (propozi ie adev rat ) ar trebui

16

verificat c orice num r natural impar n, 7 numere prime impare.

33

16

se poate scrie ca o sum de trei

Teorema 3. Dac ipoteza lui Goldbach este adev rat , atunci orice num r ntreg impar n se poate reprezenta, ntr-o infinitate de moduri sub forma n p q r , unde p, q, r sunt numere prime. Demonstra ie. Fie n un num r ntreg impar. Putem alege, ntr-o infinitate de moduri, un num r prim impar r astfel nct n r 4 . Conform ipotezei lui Goldbach exist dou numere prime impare p i q astfel nct n r p q . Teorema 4. Orice num r natural mai mare ca 11 este suma a dou numere compuse. Demonstra ie. Dac n este par, n 4 este par (deci compus) i n 4 n 4 . Dac n este impar, n 9 este par (deci compus) i n 9 n 9 . Observa ie. G.H. Hardy i J.E. Littlewood au formulat ipoteza c orice num r natural n, suficient de mare este suma unui num r prim i a unui p trat : n p k 2 , p prim, k N . Ipoteza nu a putut fi nc confirmat sau infirmat . O alt ipotez a lui Hardy i Littlewood a devenit teorem prin demonstra ia dat de I.V. Linnik n 1959 : Teorema 5. Orice num r natural, suficient de mare, este suma ntre un num r prim i dou p trate: n p k 2 h 2 , p prim , k , h N . Demonstra ia acestei teoreme nu poate fi f cut cu mijloace elementare. 1. 2. 3. 4. Bibliografie D. A. Buh tab Teoria cisel, Moskva, 1960. C. Creang , C. Cazacu, P. Minu , Gh. Opai , C. Reischer Introducere n teoria numerelor, Editura Didactic i Pedagogic , Bucure ti , 1965. P. Minu Teoria numerelor. Capitole introductive, Ed. Crengu a Gld u, Ia i, 1997. W. Sierpinski Ce tim i ce nu tim despre numerele prime, Ed. tiin ific , Bucure ti, 1966.

Generalizarea teoremei de omologie a lui Barbilian


Constantin COCEA 1
Str lucitul matematician Dan Barbilian a demonstrat c dou triunghiuri echilaterale, de acela i centru, sunt triomoloage. Vom extinde n cele ce urmeaz acest rezultat. Are loc urm toarea Teorem . Fie ABC un triunghi cu centrul cercului nscris I; A 1 , B 1 , C 1 punctele de contact ale cercului nscris cu laturile, iar A 2 B 2C 2 un triunghi avnd centrul cercului circumscris n I i invers asemenea cu A 1B 1C 1 . Atunci dreptele AA 2 , BB 2 , CC 2 sunt concurente (adic triunghiurile ABC i A 2B 2C 2 sunt omologice) Demonstra ie. Fie 0, a, b, c afixele punctelor I , A, B, C , iar C (I,1) cercul nscris de raz unitate. Atunci: a1 b1 c1 1 (1)

Triunghiul A 2B 2C

fiind invers asemenea cu A 1 B 1C 1 , cu centrul cercului


C nct
a2 a1 , b 2 b1 , c 2 c1

circumscris I , rezult c exist

(2) n loc s demonstr m concuren a dreptelor AA 2 , BB 2 , CC 2 , vom demonstra (avnd


BC B2 C 2 , CA C 2 A2 i n vedere teorema lui Desargues) c punctele AB A2 B2 sunt coliniare. a1 . Cum BC este tangent cercului nscris, Panta complex a dreeptei IA1 este k IA1 a1 rezult c ecua ia lui BC este a1 (BC) z a1 z a1 sau z a1 z a1 2 0 . (3) a1

Se tie c raportul n care o dreapt de ecua ie (BC), afixele lui B i C fiind b respectiv c, este
R b b c c Prin urmare, innd seama de (2) i (3), avem : B2 C2 b1 a1 c 1 a1

0 mparte un segment

b1a1 2 c 1a1 2

. C2 A2

(4) i
A2 , ob inem: B2

Avnd n vedere expresiile analoage cu (4) ale rapoartelor

Profesor , Liceul Teoretic D.Cantemir, Ia i

1 C 2 A2 B 2 ceea ce probeaz coliniaritatea punctelor , , , deci triunghiurile ABC i A2 B 2 C 2 sunt omologice. Observa ie. Dac ABC este echilateral , I este centrul comun al triunghiurilor ABC i A 1B1C 1 i se ob ine:

B2

C 2 A2

Teorema lui BARBILIAN. Dou triunghiuri echilaterale de acela i centru, ABC i A2 B 2 C 2 , sunt n trei moduri omologice:
A B C , A2 B 2 C 2 A B C , B 2 C 2 A2 A B C . C 2 A2 B 2

Demonstra ia decurge din teorema anterioar , deoarece dou triunghiuri echilaterale sunt n trei moduri invers asemenea. Observa ie. Teorema se poate extinde i astfel: Teorem . Fie ABC un triunghi cu I a centrul cercului exnscris corespunz tor laturii BC, iar A1 , B1 , C1 punctele de contact ale acestui cerc exnscris cu laturile BC, CA respectiv AB. Fie A2 B 2 C 2 un triunghi invers asemenea cu triunghiul A 1B1C 1 avnd centrul cercului circumscris n I a . Atunci triunghiurile ABC si A2 B 2 C 2 sunt omologice. Consecin a 1. Fie ABC un triunghi, iar A 1 , B 1 , C1 punctele de contact ale cercului
nscris C (I,r) cu laturile. Paralelele prin B
1

i C1 la BC retaie cercul C n B 2 , C 2 . S

se arate c dreptele AA 1 , BB 2 , CC 2 sunt concurente. Demonstra ie. Triunghiurile A 1B1C 1 i A2 B 2 C 2 au acela i centru al cercului circumscris (punctul I) i sunt invers egale, deci invers asemenea.

Consecin a 2. Fie ABC un triunghi, iar A 1 , B1 , C1 punctele de contact ale cercului


nscris C (I,r) cu laturile. Not m cu A2 , B 2 , C 2 simetricele punctelor A 1 , B1 , C1 fa un diametru oarecare al cercului C . Atunci dreptele AA
2

de

, BB 2 , CC 2 sunt concurente.

Demonstra ie. Centrul cercului circumscris triunghiului A2 B2 C 2 triunghiurile A2 B2 C 2 i A 1B1C 1 sunt invers egale, deci invers asemenea. Bibliografie

este I, iar

1. D. Barbilian - I. Barbu - Pagini inedite, Editura Albatros, Bucure ti, 1981. 2. C. Cocea - Propriet i remarcabile ale triunghiurilor invers asemenea, S n elegem matematica, Bac u, 1992. 3. C. Cocea - Teoreme de triortologie i triparalelogie, 1992. 4. P. S. Modenov - Probleme de geometrie , Editura Nauka, Moscova, 1979. 5. N. Mih ileanu - Utilizarea numerelor complexe n geometrie, Ed. Tehnic , Bucure ti, 1968.

Asupra unor iruri de integrale


Iuliana GEORGESCU 1
Articolul de fa
b

i Paul GEORGESCU 2
n 1 de

prezint un mod de calcul al limitelor unor iruri x n

termen

general x n

f
a

x dx , cu f satisf cnd anumite ipoteze ce vor fi precizate ulterior. n


n 1,

particular, se pot determina limitele irurilor a n


/2

bn
/2 0

n 1,

cn

n 1,

dn
1

n 1
e

de termen

general a n

sin n xdx , bn

/2 0

cos n xdx , a n

tg n xdx , a n

ln n xdx .
lim a n 0.

Insist m mai nti asupra unei solu ii eronate date n [2] pentru faptul c
/2

Solu ie. Aplicnd teorema de medie, c


2 sin n c . Deoarece sin c

0, / 2 astfel nct
0

sin n xdx
sin n xdx 0. 0.

0,1 , avem lim sin n c


n

/2

0 i, deci, lim
n

Totu i, c nu este constant, ci depinde de n , i nu putem deduce c

lim sin n c

n cele ce urmeaz vom indica un mod de calcul al unor limite de acest tip.

Teorema 1. Dac
b

f : a, b
n

0,1 este continu , iar U f 0. x1 , x 2 , /k xk 1 , i

a, b ; f x

1 este

finit , atunci lim


n a

x dx

Demonstra ie. Presupunem c U f


Consider m I i
k

i fie
1, k

0 arbitrar, dar fixat.

a i , bi cu x i

Ii i l I i b , atunci I 1
k

i l Ii

= lungimea
a k , bk ).

intervalului I i (daca x1 Fie D1


i 1

a sau x k

a1 , b1 , respectiv I k

a i , bi

i D2

a, b \
i 1

a i , bi . Cum D 2 este o reuniune finit de


sup f x , iar M
x D2

intervale nchise i m rginite, i f este continu , exist M rezult c


D2

1 . De aici,

f
k

x dx

Mn

/ 2 . Deoarece f x
b

1,

a, b , se deduce c

f
D1

x dx
i 1

bi

ai

/ 2 . n consecin ,
a

x dx

k /k

/2 .

1 2

Profesor, Liceul cu Program Sportiv, Ia i Profesor, Liceul Teoretic Grigore Moisil, Ia i

Alegnd acum n 0
b

astfel nct M
n0 , deci lim
n a

n n

/2 f x dx

/k 0.

1,

n0

, se ob ine c

f
a

x dx

Observa ia 1. Ipoteza U f este finit din enun poate fi nlocuit cu U f este Jordan neglijabil , teorema r mnnd valabil cu aceea i demonstra ie. Corolar 1.
Dac
f : a, b 0,1 este continu
n

i strict cresc toare (sau strict


b

descresc toare), iar f b

1 (respectiv f a

1 ), atunci lim
a

x dx

0.
{a} .

Demonstra ie. Suntem n ipotezele teoremei , cu U f Aplica ia 1. Are loc rela ia


n

{b } , respectiv U f
n

lim a n

lim bn

lim c n

lim d n

0.

Solu ie. Se aplic Corolarul 1, inndu-se seama de monotonia func iilor de sub semnul de integral .
n cele ce vor urma vom nota cu D f mul imea punctelor de discontinuitate ale unei func ii f . Observ m c este valabil deasemenea urm toarea mbun t ire a Teoremei 1.

Teorema 2. Dac
b

f : a, b 0.

1, 1 , iar D f

iU f

sunt Jordan-neglijabile,

atunci lim
n a

x dx

Demonstra ie. Deoarece f este integrabil Riemann, f , f


integrabile Riemann. Fie acum jabil , k
k

i f

sunt de asemenea este Jordan-negli-

0 arbitrar, dar fixat. Deoarece D f

N* i I i
k

1 i k

astfel nct I i
k

a i , bi , D f
i 1

Ik i
i 1

bi

ai

/2.

Not m D1
i 1

a i , bi , D 2

a, b \
i 1

a i , bi . Mai departe,

f
D1

x dx

/ 2 , i cum

este continu pe D 2 ,
D2 b

x dx se poate majora cu ajutorul Teoremei 1. Rezult de


b n

aici c

lim
a

x dx

0 , deci lim
n a

x dx

0.

Aplica ia 2. Fie b
b

0 , iar

f x

sin

1 , x 0, b . x 0 ,x 0

Atunci f este integrabil

Riemann, iar

lim
a

x dx

0.

10

Solu ie.

Se observ

1 f

1,

0, b

i D f

0 . Atunci f este

integrabil Riemann, conform criteriului Lebesgue de integrabilitate Riemann. Mai departe, U f 2 2k 1 ; k N este de m sur Jordan nul i se poate aplica Teorema 2. Observa ia 2. Folosind no iuni de teoria m surii i integralei Lebesgue, mai precis teorema lui Egorov, se poate demonstra urm torul rezultat:
I continu , g : a, b R integrabil Riemann i Teorema 3. Fie f : a, b R , astfel nct g n f x g x x a, b , g n n 1 un ir de func ii continue, g n : I
b b

i gn f x

M,

a, b . Atunci
a

gn

f x dx
a

g x dx .

Aplica ia 3. Pentru a

0,b

2,g x

0, x 1, x

0, 2

, gn x
2

xn , f x

sin x ,

remarcnd c
n

sunt ndeplinite condi iile Teoremei 3, iar


0

g x dx
0.

0 , ob inem c

lim a n

0 . Analog deducem c

lim bn

lim c n

lim d n

i I. Paralescu, Problema 24154, G.M. 7-8/1999 ). Calcula i: 2 sin 2 x 1 dx b) lim dx . a) lim 2 2n n n 0 0 1 sin x sin x 1 sin x sin 2 n x Solu ie. Aplic m Teorema 3. Lu m sin 2 x , x 0, 2 2x 1 x 2 2 , gn x , f x sin x , g x , a) a 0 , b 1 sin 2 x 2 n 1 x x 1 / 3, 2 x
Aplica ia 4 (V. Drul
2

b) a

0, b

2 , gn x

1 1 x
2

2n

, f x

sin x , g x

1 1 x2, x 1 3, ,x

0, 2

Bibliografie 1. D. M. B tine u et al. Primitive i integrale , Ed. Brchi, Timi oara, 1998. 2. V. Schneider Culegere de probleme de analiz matematic , Ed. Hyperion, Craiova, 1993.

11

O generalizare a lemei lui Riemann


Dan POPESCU 1 i Florin POPOVICI 2
n aceast not sunt generalizate urm toarele dou rezultate: Teorema 1 (Lema lui Riemann [3]). Dac integrabil Riemann, atunci irurile
2 0

f : 0, 2
n N*

R este o func ie
2 0

f ( x) sin nx dx

i
2 0

f ( x) cos nx dx
n N*

sunt convergente i avem

lim
n

2 0

f ( x) sin nx dx 0

lim
n

f ( x) cos nx dx . i

Teorema 2 (Problema XII.19 [1]). Dac

f : 0, T

R este o func ie continu

g : 0,

0,

este o func ie continu


n

lim 0

i periodic de perioad T, atunci 1 T T f ( x) g (nx) dx f ( x) dx) 0 g ( x) dx) . T 0

Amintim mai nti un rezultat la care vom face referire.

Teorema 3[2]. Dac

f : a, b

R este o func ie integrabil

Riemann

a, b este o func ie bijectiv , derivabil i cu derivata integrabil Riemann, atunci func ia f este integrabil Riemann i are loc formula schimb rii de variabil
b f ( x)dx a 1 (b ) 1(a)

: c, d

f ( (t )) (t )dt .

Corolar.

Fie

R* astfel nct func ia de gradul nti

este o func ie bijectiv . Dac f : a, b Riemann atunci func ia f este integrabil Riemann i are loc formula
b f ( x)dx a 1 (b ) f( 1( a )

( x)

: c, d a,b , R este o func ie integrabil

(t ))dt.

Rezultatul principal este dat de

Teorema 4 (Lema lui Riemann generalizat ). Dac

f : 0, T

R este o func ie

integrabil Riemann i g : 0,

R este o func ie periodic de perioad T, astfel nct

restric ia g [0,T ] este integrabil Riemann, atunci avem:


n

1 T T lim 0 f ( x) g (nx) dx f ( x) dx) T 0

T g ( x) 0

dx) .

(1)

1 2

Profesor, Colegiul Na ional tefan cel Mare, Suceava, Profesor, Liceul Teoretic N. Titulescu ,Bra ov

12

Demonstra ie. Conform Corolarului, func ia dat de x

g (nx ) , x [0,T ] este

integrabil Riemann. Urmeaz c Pentru


x ni i T , n
ni

irul

T 0

f ( x) g (nx) dx
n N*
n

este corect definit.


x n1 x nn T ,

orice
0,1,

n N*

fie

0 x n0

unde 0,1, n

n . Conform primei teoreme de medie, pentru orice i

exist

inf f ( x ) x [ x n , i 1 , x ni ] , sup f ( x ) x [ x n , i 1 , x ni ]
xni xn, i 1

astfel nct
(2)

f ( x)dx

ni

x ni

x n, i 1

ni

T n
n xni g (nx)dx. ni x n, i 1

Evident, avem
T 0

f ( x) g (nx) dx

n x ni xn, i 1 i 1

f ( x) g (nx)dx

n x ni ( f ( x) xn, i 1 i 1

ni ) g (nx)dx i 1

(3)

Deoarece func ia g este periodic Riemann, rezult


g ( x) M,

T i restric ia g este integrabil [0,T ] func ia g este m rginit , deci exist M 0, astfel nct de perioad
n x ni x i 1 n, i 1 n

x [0, ) . Urmeaz c
n x ni ( f ( x) x i 1 n, i 1 ni ) g ( nx)

f ( x)

ni

g (nx) dx

n x ni x i 1 n, i 1

f ( x)

ni

dx M S

(f) s

(f).

unde

S n ( f ), s n ( f ) noteaz sumele Darboux superioar

i inferioar relative la

.
(4)

De aici rezult c

lim
n

n x ni ( f ( x ) x i 1 n, i 1

ni ) g ( nx)dx

0.

innd cont de rela ia (2) i de periodicitatea func iei g, se ob in egalit ile: T n n n 1 iT xni i g (nx )dx g (t )dt ni x ni n T g ( nx ) dx ni n,i 1 n (i 1)T i 1 i 1 i 1 i 1
n

(5)
T 0

1 Ti

n ni 1

T n

T 0

g (t )dt

1 T

T 0

g (t )dt
i 1

ni ( x n,i 1

x ni )

1 T

T 0

g (t )dt

f ( x)dx .

Din (3), (4) i (5) rezult c are loc (1).

Observa ie. Teoremele 1 i 2 sunt particulariz ri ale Teoremei 4. Bibliografie 1.D. Popescu - Problema XII.19, Recrea ii matematice 1/2001, p. 77. 2.F. Popovici, M. Bencze - Asupra schimb rii de variabile in integrala Riemann, G.M. metodic , 3/1996, 161-164. 3.Gh. Sire chi - Calcul diferen ial i integral, vol I, Ed. tiin ific i Enciclopedic , Bucure ti, 1985.

13

Cteva rela ii metrice deduse vectorial


Marian TETIVA 1
Ne propunem s prezent m n cele ce urmeaz o modalitate de deducere a unor rela ii metrice n triunghi sau n tetraedru cu ajutorul calculului vectorial. Ideea (simpl , dar eficient ) am ntlnit-o n lucrarea [3] i este exprimat n Propozi ia 1. Fie K, M, L trei puncte necoliniare. Pentru un punct P oarecare, urm toarele afirma ii sunt echivalente: a) Punctele L, M, P sunt coliniare; b) Exist numerele reale l, m astfel nct l+m = 1 i KP Demonstra ie. P
LM
l R astfel nct MP

l KL

m KM
l R astfel nct

l ML

KP KM l ( KL KM ) l R astfel nct KP l KL (1 l ) KM , q.e.d . Mai departe vom rezolva cteva probleme folosind rezultatul din Propozi ia 1. ncepem cu problema din [3] care ne-a condus la aceast not .

Problema 1. Fie ABC un triunghi, iar E, D puncte situate pe laturile AB, respectiv AC. Consider m punctul M, intersec ia dreptelor BD i CE; P i N sunt intersec iile dintre AM PN PM i BC, respectiv AM i DE. S se arate ca are loc egalitatea: 2 . NA MA AE AD AM AN Solu ie. S not m p= . Cum P BC , exist ,q , m , n EB DC NP MP

numerele x, y astfel nct

x+y=1,

AP

x AB

y AC . Din

AE

p EB rezult

imediat
AM

AE

p AB ; la fel , AD p 1

q AC , q 1

m n AP i AN AP . Egalitatea de m 1 n 1 mai sus poate fi scris n urm toarele forme echivalente : q 1 m 1 AM x AB y AD , m q n 1 AN n p 1 x AE p q 1 y AD, q

m 1 AM m

p 1 x AE p

y AC . innd seama de unicitatea scrierii unui vector n bazele

( AE , AD), ( AE , AC ) respectiv ( AB, AD) i de Propozi ia 1, ob inem egalit ile: m 1 m 1 p 1 q 1 n 1 p 1 q 1 x x y (2); y (3). Adunnd (2) x y (1); m m p q n p q

Profesor, Colegiul Na ional Gh. Ro ca Codreanu, Brlad

14

cu (3) i innd seama de (1) i de faptul c x+y=1 , ob inem c

2 m

1 , q.e.d. n

Problema 2 (rela ia lui Van Aubel). Cu nota iile din problema precedent , are loc AE AD AM . egalitatea EB DC MP Solu ie. Rela ia (2) de mai sus se scrie: 1
1 m 1 1 x 1 x , adic mx=p . Analog, p

din (3) ob inem my=q . Atunci

AE EB

AD DC

p q

m( x

y)

AM , q.e.d. MP

Problema 3 ([1]). P str m nota iile din Problema 1. n plus, consider m punctele T, p q TA SA S pe laturile AB, AC astfel nct . S se arate c M TS 1. , TB SC Solu ie. Vom avea, ca mai sus, AT
1 AB, AS 1 AC. Din rela ia

AP i

x AB

y AC rezult c

m 1 AM m

AT

AS . Conform Propozi iei 1

innd seama de unicitatea scrierii unui vector n baza AT , AS , avem: 1 p q p q m 1 1 1 x y 1 M TS x y 1 , q.e.d. m m m m m Dou cazuri particulare mai des ntlnite ale Problemei 3 sunt acelea n care punctul M este centrul de greutate G , respectiv centrul cercului nscris I pentru triunghiul ABC; cititorul se poate convinge c este valabil urm torul enun :
Problema 3. Fie ABC un triunghi i punctele T, S pe laturile AB, AC. Atunci: TB SC a) G TS 1; TA SA SC TB b) I TS a, unde a, b, c reprezint lungimile laturilor triunghiului. c b SA TA

Iat n continuare i variantele n spa iul cu trei dimensiuni ale chestiunilor discutate anterior; rezultatul similar celui din Propozi ia 1 este con inut n:
Propozi ia 2. Fie K, L, M, N patru puncte necoplanare. Un punct P este situat n acela i plan cu L, M, N dac i numai dac exist numerele reale l, m, n astfel nct

l+m+n=1 i KP

l KL m KM

n KN . Altfel spus, dac x,y,z sunt coordonatele vectorului

KP n baza, KL, KM , KN , atunci P LMN dac i numai dac x+y+z=1. Demonstra ia este aseman toare. Avem P LMN m, n R astfel nct LP m LM n LN
m, n R astfel nct KP KL m KM KL n KN KL

l , m, n R , l=1-m-n (deci cu l+m+n=1), astfel nct KP Folosind Propozi ia 2 putem rezolva:

l KL m KM

n KN , q.e.d.

15

Problema 4 ([3]). Fie ABCD un tetraedru i F, G, H puncte pe AB, AC, AD respectiv; fie M punctul comun planelor (FCD), (GBD), (HBC), iar P i N punctele de intersec ie ale PN PM dreptei AM cu planele (BCD) respectiv (FGH). Atunci are loc: . 3 NA MA AG AM AF AH Solu ie. Ca n Problema 1, s not m p , q , r , m , GC FB HD MP AN . innd cont de unicitatea scrierii unui vector ntr-o baz i de Propozi ia 2, n NP ob inem mai nti existen a numerelor reale x, y, z astfel nct x+y+z=1 i
AP x AB y AC z AC , iar apoi echivalen ele: n 1 n
m 1 m m 1 m x y p 1 x p r 1 z r

p 1 x p
y z 1 m

q 1 y q
1 m

r 1 z r
x m 1 ; p m x

1 n

x p

y q

z ; r
1 m y ; q

q 1 y z q

z 3 . De aici r m y z

x p

y q

z r

1 , adic rela ia din enun . n


m , deci am rezolvat i

De asemenea vom avea 1 x

p q r m

p q r

AF AG AH AM . FB GC HD MP n ncheiere propunem cititorului s rezolve problemele : Problema 6. P str m nota iile din Problemele 4 i 5; n plus fie punctele S, T, U pe AS AT AU , , . S se arate c : AB, AC, respectiv AD astfel nct SB TC UD p q r 1. M STU

Problema 5. Cu acelea i nota ii din Problema 4 avem

Caz particular. Centrul de greutate al tetraedrului se afl n planul (STU) dac i SB TC UD 1 ([4]). numai dac AS AT AU Problema 7. Fie ABCD un patrulater convex i punctele M ,N, P, Q pe laturile sale BN AQ AM DP AB, BC, CD, DA respectiv, astfel nct b . Fie S intersec ia a, NC QD MB PC MS QS dreptelor MP i NQ. S se calculeze rapoartele i (O frumoas solu ie sintetic a SN SP acestei probleme poate fi g sit n [2]). Bibliografie 1. M. Andronache Problema 4 (p. 158) , G.M. 4/2000. 2. D. Brnzei, R. Brnzei Metodica pred rii matematicii, Editura Paralela 45, 2000. 3. V.N. Dubrovski Solu ia problemei M1062, Kvant 1/1988. 4. Gh. Szllsy Problema C: 2275, G.M. 4/2000.

16

Inegalit i geometrice. Aplica ii


Dan- tefan MARINESCU 1 i Ioan ERDEAN 2

Scopul propus este de a demonstra, prin mijloace mai pu in folosite, dou inegalit i geometrice care au un num r mare de aplica ii. Propozi ia 1. Fie n N, n 2 i M, A1, A2, , An puncte din spa iu date. Dac a1, a2, , an R, atunci
n n

ai
i 1 i 1

a i MA i2
1 i
n

a i a j Ai A 2 . j
j n

(1)

Egalitatea se ob ine dac

i numai dac
i 1

ai MAi
n

0.

Demonstra ie. Avem:


i 1
n i 1 2 ai2 MA i

a i MA i
i 1
n

a i MA i

0 , de unde
0.

ai a j MAi MA j

0 sau
i 1

2 ai2 MA i 2

1 i j n

ai a j MA i MAj cos(A i MA j ) 1 i j n

Cum din teorema cosinusului n


2MA i MA j cos( A i MA j )
n i 1

MAiAj , eventual degenerat, avem


MA j2 A i A 2 , urmeaz c j

MA

2 i

a i2 MAi2

ai a j 1 i j n

MAi2

MA 2 j

a i a j Ai A2 j 1 i j n

i, deci, are loc inegalitatea (1).

Observa ie. Exist i alte demonstra ii ale inegalit ii (1) pentru cazul n care punctele sunt coplanare i a1, a2, , an>0. Urm torul caz particular este util n aplica ii : Corolar. Fie n 3, A1 A2 An un poligon i M un punct dat. Atunci ( ) a1, a2, , an R are loc inegalitatea (1). Aplica ia 1. Dac ABC este un triunghi oarecare i , , R, atunci ( + + ) 2R 2 a2 +b2 +c2 . L. Panaitopol Solu ie. n (1) lu m a1= , a2= , a3= i M punctul O (centru cercului circumscris). Ca un caz particular al acestei aplica ii se ob ine

a2 +b2

Aplica ia 2. Dac +c2 0. Aplica ia 3. Fie x, y, z


yz z
2

ABC

este un triunghi

R, atunci

R * . Atunci
zx z y
2

xy x z
2

1 1 16 xy

1 yz

1 . zx

D- t. Marinescu, I. erdean (etapa local 1998)


1 2

Profesor, Liceul teoretic Iancu de Hunedoara, Hunedoara Profesor, Colegiul Na ional Aurel Vlaicu, Or tie

17

Solu ie. n Aplica ia 1 se va lua =x, =y, =z i a=y+z, b=z+x, c=x+y. Din (x y )( y z )(z x ) abc , ob inem R etc. R 4S 4 xyz (x y z ) Aplica ia 4 . Pentru orice priamid triunghiular MABC avem : G.M. 2/1995, p.88 aMA 2 bMB 2 cMC 2 abc (a=BC, b=AC, c=AB). Solu ie. Se ob ine din (1) pentru n=3 i a1=a, a2=b, a3=c.
MA
4

Aplica ia 5. Fie ABC un triunghi i M un punct al planului s u. Atunci MB 4 MC 4 a 2b 2c 2 /(a 2 b 2 c 2 ) . I. Tomescu, G.M. 6/1972 Solu ie. Conform inegalit ii Cauchy Buniakovski Schwarz, avem:

MA4 MB 4 MC 4 a 2 b 2 c 2 aMA2 bMB 2 cMC 2 . . Combinnd aceasta cu inegalitatea din Aplica ia 4, deducem inegalitatea cerut .

Aplica ia 6 . Fie ABC un triunghi, P un punct n planul s u, iar PA=x, PB=y, PC=z. S se arate c ayz+bxz+cxy abc. C. Cocea Solu ie. Lu m n Corolar n=3, M=P, a1 a / x , etc. Ob inem:
a x b y ayz c z bxz xyz ax by cxy cz abc xyz a 2bc yz ayz ab 2c xz bxz abc 2 xy cxy abc ax by xyz abc. cz

Aplica ia 7. Dac A1 A2 An este un poligon i M un punct oarecare, atunci


n

n
i 1

MA

2 i

A i A j2 .
1 i j n

G.M. 5/1995, p.193


1, i 1, n n Corolar.

Solu ie. Lu m ai
n R
2 2

Aplica ia 8. Dac A1 A2, An este un poligon nscris n cercul C(O,R), atunci


Ai A2 . j
1 i j n

M. Chiri , M. Dinc Numere complexe

Solu ie. n Corolar se consider ai

1, i

1, n i punctul M n O.

Cel de-al doilea rezultat general este urm torul:


Propozi ia 2. Pentru orice n N, n 3 i orice x1, x2, , xn R avem: n n n 2 2 2 x k 2 x k 2 x k 1 x k 1 cos n n cos2 (x n 1 x 1 ) . n n k 1 k 1 k 1 Egalitatea are loc dac i numai dac x1= x2= = xn=1/2. Demonstra ie. Pentru n=3 inegalitatea revine la
3

(2)

2
k 1

2 xk

3
k 1

xk
k 1

xk xk

9 4

0 (cu x4= x1). Deoarece

2 xk

xk xk

18

avem:

2 xk

xk xk 1

xk

9 4

xk

xk

9 4

xk

3 2

0,

deci (2) are loc pentru n=3. Pentru n 4 inegalitatea (2) revine la 2 2 2 B 2 x k 2 x k 1 cos 2 x k x k 1 cos n n Cum

n sin2

0.

2 xk xk 1

2 xk

2 xk 1 , k

1, n , prin multiplicare cu cos


2 xk xk
1 cos

n 4) i apoi sumare dup k, ob inem: Ca urmare, avem: B


4
2 xk sin 2

2 n

2 (ce este n 2 2 . xk cos n n

0 pentru

2 xk 1 cos

2 n
n

2
n sin
2

xk 1 cos n

2 n
2

n sin 2

xk sin

2 xk

1 sin

0 q.e.d

Aplica ia 9. Se consdider triunghiul echilateral ABC de latur 1 i punctele 3 . A1 (BC), B1 (CA), C1 (AB). S se arate c A1B12 B1C 12 C 1A12 4 Concurs de matematic 1988 etapa final Solu ie. Notnd x1=BA1, x2=CB1, x3=AC1 i , conform teoremei cosinusului , inegalitatea revine la 3 2 2 2 , adic (2) 2 x1 x2 x3 2 x1 x2 x3 x1 (1 x2 ) x2 (1 x3 ) x3 (1 x1 ) 3 4 pentru n=3. Aplica ia 10. Pe laturile unui p trat ABCD de latur 1 se consider punctele M (AB), N (BC), P (CD), Q (DA) astfel nct MN 2 NP 2 PQ 2 QM 2 2. S se arate c MNPQ este p trat. C. N sturic , Cardinal 3/1991, p.55 Solu ie. Not m AM=x1 , BN=x2, CP=x3, DQ= x4. Egalitatea se scrie:
2 2 2 2 2 x1 x2 x3 x4 2 x1 x2 x3 x4 4 2 , adic n (2) pentru n=4 are loc egalitate, deci x1=x2=x3=x4=1/2 i M, N, P, Q, vor fi mijloacele laturilor p tratului ABCD. n consecin MNPQ este p trat.

Aplica ia 11. Fie A1 A2, An , n 4, un poligon regulat de latur 1 i P1 ( A1 A2), 2 P2 ( A2 A3), , Pn ( An A1). S se arate c P1P22 P2 P32 ... Pn P12 n cos . n R.M.T 1,2 / 1989 , Problema 6539 Solu ie. Notnd cu P1A2 =x1 ,P2A3 =x2 ,, PnA1 =xn , teorema cosinusului aplicat triunghiurilor P1A2P2 , P2A3P3 , ,PnA1P1 ne conduce la (n 2) (n 2) 2 2 2 , , Pn P2 xn (1 xn )2 2xn (1 xn ) cos P P2 x1 (1 x1 )2 2x1(1 x1 ) cos 1 1 n n n n n 2 2 2 2 Prin sumare ob inem: P P2 P2 P3 ... Pn P2 2 xk 2 xk 2 xk (1 xk ) cos n 1 1 n k 1 k 1 k 1

i, n conformitate cu (2), deducem inegalitatea cerut .


19

Asupra unei clase de iruri recurente


Dan POPESCU 1
Scopul acestui articol este prezentarea unei metode de abordare a unei clase de iruri recurente de primul ordin. Mai precis, dat irul ( x n ) n 1 prin
x1 D R

i xn

f ( xn ), n
n

N *,

(1)

unde f : D

D , se pune problema existen ei limitei


n

lim nx n n R .

n ipoteza c

lim x n
n

R {0} , irul (nx n ) n 1 are limita evident ; r mne de 0 . Rezultatul principal este cuprins n urm toarea
D i 0

investigat cazul n care lim x n


Teorem . Fie D

R * avnd originea ca punct de acumulare, func ia f : D


0 n

irul ( xn ) n 1 definit prin (1) astfel nct sunt ndeplinite condi iile: 1 lim x n
2 0 lim
x 0

f ( x) xf ( x )

R . Au loc implica iile:

(a) dac l

R {0} , atunci

lim nx n

1 ; l
x f ( x) xf ( x) 0, x V1 D,

(b) dac l=0 i exist atunci lim nxn


n

o vecin tate V1 a originii nct

;
x f ( x) xf ( x ) 0, x V2 D ,

(c) dac l=0 i exist o vecin tate V2 a originii nct atunci lim nxn
n

.
1 nx n scris sub forma
n 1

Demonstra ie. Se aplic criteriul Stolz-Cesro irului


1/ x n n

1/ x 1/ x n . n ipotezele impuse avem: lim n 1 n (n 1) 1 1


lim
n

lim

x n f (x n ) x f (x ) lim l R. . n x nf (x n ) xf (x )

Urmeaz c

1 l i afirma iile (a), (b), (c) decurg imediat. nx n Prezent m mai nti cteva aplica ii directe ale acestei teoreme. 1 (G.M. 11,12/1986, C: 649, M. Bencze). Fie irul ( xn ) n 1 definit prin x1 xn 1 ln(1 xn ), n N * . S se demonstreze c lim nx n 2 .
n

0 i

2
x1
1

(G.M. 3/1987, 21056, F. Dumitrel). Dac

irul ( x n ) n 1 se define te prin

0 i xn 1

xn 2

xn

N * , s se calculeze lim nx n .
n

Profesor, Colegiul Na ional tefan cel Mare, Suceava

20

3 (G.M. 10/1987, 21253, M. Lascu). Fie irul ( xn ) n 1 definit prin x1 0 i xn 1 ln(1 arctg ( xn )), n N * . S se arate c irul nx n n 1 este convergent la 2. 4 (R.M.T. 2/1987, 6256, V. Bivolaru). Se consider

irul ( xn ) n

nct x1

0 i

xn

2 xn

2xn ln( xn ) 1

, n N * . S se arate c

irul este convergent i s i se calculeze

limita. 5 (G.M. 4/1995, 23241, V. Nicula). Fie p N * , x 1 (0, 1 / p ) xn 1 xn (1 xn )(1 2 xn )...(1 px n ), n N * . S se calculeze limita lim nx n .
n

6 (G.M. 1/1997, 23668, A. Vernescu). Dac


xn
1

irul ( xn ) n

1 este definit prin x

0 i

1 1 , n N * , s se studieze natura 4 2 convergen , s se calculeze lim nx n .

xn

irului

nx n

n 1

i, n caz de

Condi ia 10 din teorem se verific u or pentru fiecare dintre irurile precedente. n privin a condi iei 2 0 , pentru func ia aferent
x ln(1 x ) 1. lim x 0 x ln(1 x )
1 , 2

f : (0, )

(0, ) indic m:

2. lim
x

x2 x 2 x
2

ln 2 ,

3. lim
x

x ln(1 arctg x ) 0 x ln(1 arctg x )

1 , 2

4. lim
x 0

x2 2 x ln(1 x ) x 2 x ln(1 x )
3

1 , 2

x x

5. lim
x

x x (1 x )(1 2x )...(1 px) p( p 1) 0 x 2(1 x )( 2x )...(1 px) 2 1

6. lim
x 0

1 1 4 2 1 1 4 2

1.

Observa ie. n privin a problemei 5, men ion m c pentru p= 1 se ob ine o problem din revista Matematika v kole, 5/1984. Prezent m acum cteva probleme care sunt sub inciden a Teoremei, fapt care nu este ns evident. 2 7 (G.M. 5,6/1988, 21458, M. Banyai). Fie k>0, x1>k i x n 1 x n /( x n k ), n n N * . S se calculeze lim . xn n Solu ie. Se constat c
irului ( y n ) n
1

( xn ) n 1 este cresc tor i lim xn


n

. Aplic m Teorema
n N*, i

definit prin yn

1 , xn

N * . Avem y n 1

y n (1 ky n ) ,

21

lim y n

0. Func ia f : (0,1) n xn lim ny n

(0,1) dat de f ( x ) 1 . k

x (1

kx ) are lim
x

f ( x) 0 xf ( x )

k.

. Ca urmare, lim
n

8 (G.M. 1/1989, 21668, M. Lascu). Fie


xn
1

irul ( x n ) n

1 definit

de x1

xn

1 , n xn

N * . S se calculeze lim xn

lim
n

xn 2n

(enun par ial).


2 2/xn,

Solu ie. Se constat c


yn
1 2 4 y n /( y n

. Not m y n

N * , i deducem c

4y n

4). Utiliznd Teorema, ob inem: lim


n

xn 2n

lim
n

1 ny n 0

1.

9
xn x
2 n

(G.M.

6,7/1990,

22115,

M.

P.

Mihail).

Dac

a, b, x1

/(a bx n ),

N * , s se calculeze lim
x 0

1
n xn

.
( yn ) n 1
2

Solu ie.
yn
1

Se

introduce
2

irul

prin
2

yn

2 xn,n

1.

Urmeaz

2 n

/(a b y n ) , n

1, f (x )

x /(a b x ) , x

0 , etc. Discu ie dup b i x1 .

10 (G.M. 11/1993, C: 1463, L. Panaitopol). Se consider


x1 1 i xn 1 xn xn 1, n N * .S se calculeze lim
n

irul (x n ) n 1 definit prin .


2 y n ,n

xn n2

Solu ie. Cu

yn

1/ x n , n xn
2

1 , ob inem y n

yn / 1 yn

1,

g sim, n cele din urm , lim

1 . 4 n n 11 (R.M.T. 2/1997, X127, V.Bivolaru). Fie irul ( xn ) n 1 definit prin x1 0 , x1 x2 ... xn x n 1 x n a x n , n N * , unde a>1. Dac y n , n N *, s ln n calculeze lim y n .
n

se

Solu ie. Cu criteriul Stolz-Cesro, avem: xn 1 lim nx n lim y n lim lim nx n ln n ln(n 1) n ln n ln(n 1) n n n ultima egalitate stabilindu-se cu ajutorul Teoremei (ca n Problema 3). 12 (G.M. 1/1998, C:2005, M. Bencze). Fie irul ( x n ) n
2 (1 x n ) x n 1 1 x n , n calculeze lim n(1 x n ). n 1

1 ln a 0 i

astfel ca x1

N * . S se demonstreze c

( x n ) n 1 este convergent i s se
2 y n ) /(2 y n ), n 1 ,

Solu ie. Consider m y n

1 xn,n

1 . Avem y n

(y n

i f (x ) (x x 2) /(2 x ), x 0 , etc. Limita este egal cu zero.


22

Un criteriu de concuren
Temistocle BRSAN

a dreptelor
1

n revista Recrea ii tiin ifice, IV (1886), pag. 48, este enun at urm toarea Problem . n orice triunghi, dreptele ce unesc picioarele n l imilor corespunz toare la dou laturi, picioarele bisectorelor al turate cu n l imile i picioarele perpendicularelor duse din centrul cercului nscris pe cele dou laturi trec prin acela i punct. n acela i volum, la pag. 118, este prezentat o solu ie sintetic a problemei, dat de N.Gr. B l nescu, elev la coala de Poduri i osele din Paris. ncepem prezenta not cu un criteriu de concuren a trei drepte determinate de punctele lor de intersec ie cu dou dintre laturile unui triunghi. Cu ajutorul acestuia vom da apoi o solu ie tehnic problemei de mai sus. Fie un triunghi ABC i dreptele d1, d2 i d3 ce intersecteaz n M, P i respectiv R dreapta AB i n N, Q i respectiv S dreapta AC. Consider m c pozi iile acestor puncte sunt determinate de rapoartele urm toare: MB NC PB QC RB SC m, n; p, q; r, s. (1) MA NA PA QA RA SA Men ion m c aceste puncte au pozi ii oarecare pe dreptele AB i AC, dar c , n cele ce urmeaz , vom exclude tacit anumite cazuri triviale, cum ar fi: dou dintre drepte sunt paralele sau coincid, cele trei drepte sunt concurente ntr-un punct situat pe AB sau AC, una dintre drepte trece prin vrful A etc. Propozi ia 1. Dreptele d1, d2 i d3 determinate de numerele m,n,p,q,r,s date de (1) sunt concurente dac i numai dac aceste numere ndeplinesc condi ia 1 1 1 (2) ps rq rn ms mq pn 0 m p r 0.
n q
n n 1

s
AC

Demonstra ie. Din (1) ob inem u or rela iile: m 1 1 AB ; NA AC , NB MA AB , MB m 1 n 1 m 1 Fie {X}= d1 d2 i X d 1 d 3 . Utiliznd AMN i teorema lui Menelaus relativ la transversalele PQ i RS i innd seama de (3), X i X coincid avem: d1, d2, d3 concurente
XM XN PB MB PA X 'M X 'N QA QA NA PM QA PA QN
RB MB RA SA

i analoagele. (3)

RM RA

SA SN
SA NA

Prof. dr., Catedra de matematic , Univ. Tehnic Gh. Asachi, Ia i

23

p p 1 1

m m 1 1

1 q 1 1 n 1 q 1

r r 1 1

m m 1 1

1 s 1 1 n 1 s 1

ps

rq

rn ms

mq

pn

0.

p 1

r 1

Solu ia Problemei. Fie d1, d2, d3 dreptele ce unesc picioarele n l imilor, picioarele bisectoarelor i, respectiv, punctele de contact ale cercului nscris (puncte pe dreptele AB i p b p c a cos B a cos C a a ,n ; p ,q AC). Atunci m ; r ,s i b cos A c cos A b c p a p a condi ia (2) se verific prin calcul direct. Observa ie. Fie A0 punctul de concuren a dreptelor din problem i B0, C0 punctele analoage acestuia. Vom ar ta ntr-o not urm toare c AA0, BB0, CC0 sunt concurente. n particular, dac M coincide cu B i S coincide cu C (i.e. m=s=0), vom ob ine: Propozi ia 2. Fie ABC un triunghi oarecare. Dreapta PQ trece prin punctul de intersec ie a cevienelor BN i CR dac i numai dac numerele p,q,n,r definite ca n (1) 1 1 1 p q (4) satisfac condi a 1 0 p r 0. r n n q 0 Observa ie. Acest ultim rezultat este cunoscut ([3, Teorema 3], [2, Teorema 1 o form diferit a condi iei] etc.). n locurile citate (ct i n alte locuri!) sunt indicate condi iile n care dreapta PQ trece prin diferite puncte remarcabile ale triunghiului: a a a cos B a cos C sin 2 A sin 2 A G( r n ,n ), H ( , ), O ( , ), 1), I ( r sin 2C b c b cos A c cos A sin 2B
p a p a p b p c ), ( , ), N ( , ) , unde K, , N sunt punctele p a p a p b p c b c2 lui Lemoine, Gergonne i respectiv Nagel. Se ob in rezultatele: QC PB QC PB 1. G PQ 1, 2. H PQ tgB tgC tgA , PA QA PA QA
K(
2

a2

a2

3. I 5. K 7. N

PQ
PQ PQ

PB PA
PB PA

QC QA
c2 QC QA

a,
a2 , QC

4. O 6.

PQ PQ

sin 2B

PB PA

sin 2C

QC QA

sin 2 A 1 p a

b2

1 PB p b PA

1 QC p c QA

( p a ) etc. PA QA Foarte adesea aceste condi ii sunt date pentru cazul restrictiv cnd P (AB) i Q (AC), fapt care justific renun area la lucrul cu segmente orientate.

( p b)

PB

( p c)

Bibliografie 1. Colec ia revistei Recrea ii tiin ifice (1883-1886). 2. C. Chiser Condi ii necesare i suficiente ca o dreapt s treac prin puncte importante dintr-un triunghi, G.M. 9/2000. 3. N. Oprea Un punct i o dreapt remarcabil din planul unui triunghi, G.M. 11/1996. 24

Cteva consecin e ale unei rela ii a lui Gergonne


Ioan S C LEANU 1
n triunghiul ABC consider m cevienele A A , B B , CC concurente n M. AM BM CM Not m x , y , z ; constat m c dac M este centrul de greutate MC MA MB al triunghiului, atunci x, y, z sunt numere naturale. Ne propunem s determin m toate punctele M Int ABC cu proprietatea urm toare:
rapoartele x, y , z corespunz toare lui M sunt numere naturale.

(*) n particular, vom a eza ntr-un cadru firesc i vom extinde rezultatul ob inut n [1]. Pornim de la urm toarea
Teorem (Gergonne).
concurente n M. Atunci
MA AA

n
MB BB

triunghiul
MC CC

ABC,

cevienele

AA ' , BB ' , CC ' sunt

1 (vezi [2]).

1 ; n mod z 1 1 . Vom presupune necesar, unul dintre termenii sumei din stnga trebuie s fie cel pu in 3 1 1 c , de unde x 2 . Cum x N*, rezult c x 1 sau x 2. x 1 3 1 1 1 Dac x 1 , atunci , i.e. ( y 1)(z 1) 4 i deoarece y 1 z 1 2

Cu nota iile ini iale, putem rescrie concluzia

( y , z ) {(2,5), (3,3), (5,2)} . 1 1 2 Dac x 2 , atunci . Unul dintre termenii din stnga trebuie s y 1 z 1 3 1 1 1 ; s presupunem c , adic y 2 . Dac y 1 , ob inem z 5 , fie cel pu in 3 y 1 3 iar dac y 2 , rezult z 2 .

y, z

N*, rezult c

n concluzie, ( x , y , z ) poate lua valorile (2,2,2), (1,3,3), (1,2,5), precum i toate permut rile posibile ale acestor triplete. Avem astfel 3 tipuri de puncte M cu proprietatea (*), pe care le vom nota corespunz tor M[2,2,2], M[1,3,3], M[1,2,5]. Ne propunem n continuare s caracteriz m geometric fiecare dintre aceste tipuri de puncte.
Propozi ia 1. Un punct M este de tipul M[2,2,2] dac de greutate al triunghiului ABC.
i numai dac este centrul

Profesor, Liceul Teoretic tefan cel Mare, Hrl u

25

Demonstra ie. Suficien a este evident . Pentru a demonstra necesitatea, aplic m teorema lui Menelaus n triunghiul AB M cu transversala BA C ; ob inem B 'C 1 BM B C A A BM 2 AA ' i atunci , adic B 1 . Din ipotez , 3, BB ' 3 CA 2 A'M B B CA A M este mijlocul lui [AC]. Rezult c M se afl situat pe mediana [ BB ' ] astfel nct
MB ' BB ' 1 , deci M este centrul de greutate al triunghiului ABC. 2

Propozi ia 2. Un punct M este de tipul M[1,3,3] dac i numai dac este mijlocul medianei [ AA ' ] (analog pentru M[3,1,3] i M[3,3,1]). C 'M B 'M 1 , deci. B ' C ' // BC Demonstra ie. Dac M[1,3,3], atunci MC MB 3 conform reciprocei teoremei lui Thales aplicat n triunghiul MBC. Rezult de aici c AC ' AB ' BA ' i folosind teorema lui Ceva ob inem c 1 , deci [ AA ] este median , A 'C C ' B B 'C iar M este mijlocul s u. Reciproc, dac M este mijlocul medianei AA , evident c x 1 . Din teorema lui AC ' AB ' , deci B ' C ' // BC , de unde C ' MB ' ~ CMB i Ceva ob inem c C ' B B 'C
AB 'C ' ~ ABC . Ob inem de aici c

1 y

1 z
AA C

B 'C ' BC

AB ' . Pe de alt parte, aplicnd AC


BMB ' , rezult
2 , i.e.

teorema lui Menelaus n triunghiul

cu transversala
CB ' B 'A

BA ' CB ' MA BA ' 1 MA 1 i cum , 1 , avem c BC B ' A MA ' BC 2 MA ' concluzie, y z 3 , deci M este de tipul M[1,3,3].

AB ' AC

1 . n 3

Propozi ia 3. Un punct M este de tipul M[1,2,5] dac i numai dac apar ine liniei mijlocii [C 1 B 1 ] i o mparte n dou segmente avnd raportul 2 (au loc i afirma iile analoage pentru M[1,5,2] etc.). Demonstra ie. A Fie M[1,2,5]; cum x 1 , rezult c M apar ine liniei mijlocii [C 1 B 1 ] . Ducem MT AC , B
T BC; atunci MB 1CT este paralelogram i deci MB 1 TC . Aplicnd teorema lui
BT BM 2, Thales n triunghiul BCB ' ob inem TC MB'

C1

B1

deci CT

1 BC 3

2 B 1C 1 , adic MB 3

2 B 1C 1 . 3

26

Reciproc, fie M evident c

C1 B1 astfel nct

MB 1 C 1M MB 1

2 . Cum M apar ine liniei mijlocii,

2 2 1 1 C 1B 1 BC BC , deci 3 3 2 3 BT BM BT 2 . Din teorema lui Thales aplicat n BCB ' rezult c 2 , adic TC MB ' TC y 2. Folosind acum rela ia lui Gergonne, se deduce c z 5 .

x 1 . Cu nota iile precedente, CT

Aplica ia 1. Centrul cercului circumscris triunghiului ABC are proprietatea (*) dac i numai dac triunghiul este echilateral. Demonstra ie. Presupunem c O are proprietatea (*). Dac , prin absurd, O B 1C 1 , atunci C1O AB i B 1O AC , deci A, B, C ar fi coliniare. Rezult c

O B 1C 1 i de aici urmeaz c O este de tipul O [2,2,2], adic O G, deci triunghiul ABC este echilateral. Reciproca este imediat .
Aplica ia 2. Centrul cercului nscris are proprietatea (*) dac i numai dac triunghiul ABC este echilateral. Demonstra ie. Presupunem c I are proprietatea (*). Dac , prin, absurd, I B 1C 1 , atunci < C1 IB < IBC i deci < C1 IB < C1 BI , adic triunghiul C1 BI este isoscel cu [ BC 1 ] [ C 1I ]. Analog, [ CB 1 ] [ B 1 I ], de unde BC 2 B1C1 2C1 I 2 B1 I 2 BC1 2CB1 AB AC , imposibil. Rezult c I este punct de tipul I[2,2,2], adic I G, deci triunghiul ABC este echilateral. Reciproca este evident . Observa ia 1. Acest rezultat este demonstrat pe o alt cale n [1]. Observa ia 2. Exist triunghiuri neechilaterale al c ror ortocentru are proprietatea (*). Caracterizarea acestora este dat mai jos (Aplica ia 4) i o propunem spre rezolvare, mpreun cu dou probleme ajut toare: Aplica ia 3. Fie triunghiul ABC, M si N mijloacele laturilor [AB], respectiv [AC] iar H intersec ia in l imilor AA ' i BB ' . Au loc echivalen ele:

1) 2)

BC 3

AB 5 AB

AC 2 2 AC

[ MN ]

i HN=2HM

HB HA 1 i HA ' HB '

2;

HA BC HA H este mijlocul lui [MN] 1 i 3. HA ' 2 HB ' 3 3 Aplica ia 4. Ortocentrul triunghiului ABC are proprietatea (*) dac i numai dac

laturile triunghiului sunt direct propor ionale fie cu 1,1,1, fie cu 3, 2 2 , 5 , fie cu 2,
3, 3.

Bibliografie 1. Iulica Georgescu Asupra unei clase de triunghiuri, G.M. 2-3/1982. 2. Liviu Nicolescu, Vladimir Boskoff Probleme practice de geometrie, Ed. Tehnic , Bucure ti, 1990.

27

Unele iruri monotone cu limita e sau e-1


Gheorghe COSTOVICI 1
n Propozi iile care urmeaz , vom demonstra prin metode elementare monotonia unor iruri cu limita e sau e-1. Propozi ia 1. irul e n (x ) (1 Demonstra ie. Din a n
a
a
1

bn

1 n ) este cresc tor, x 0 fixat. n x (a b )(a n a n 1b .... ab n 1 b n ) , n ipoteza

b
1

0 , se ob ine c
1 n x

a n [a (a b )(n 1)] b n 1 . Lund n aceast

inegalitate

i b

1 , g sim c n 1 x

e n (x )[1

x ] e n 1 ( x) , (n x)(n x 1)

care atrage e n (x ) e n 1 (x ) ,

n 1,
n

x
1 x

0 fixat.
)n
1

Propozi ia 2. irul a n (x ) (1

este descresc tor,


R , f (t )
1 t x ) t (t x

x (1
1 1)(t

( 1,0] fixat. 1 t x
x)

Demonstra ie. Consider m func ia f : [1, )


x ( 1,0] arbitrar fixat. Avem f (t )
f (t )[ln(1

)t

, cu

], t 1.

Folosind ln 1 y
t 1 x 1)(t

y,

y
x x 1)(t

0,

rezult

ln(1

1 t
f 't

(t
0, t

t 1 x 1)(t
1

1 x) i t x f este

(t

x)

(t

x)

0 , adic

deci

descresc toare. Atunci f ( n ) demonstra ia.

f (n 1), i.e. an (x ) an 1 (x ) , n 1 , ceea ce ncheie

Propozi ia 3. irul b n ( x )

(1

1 n x

) n este descresc tor,


(1 1 n x )n (1 x

x 1 fixat.

Demonstra ie. Observ m c b n ( x )

n 1 . irul e n ( x 1) este cresc tor pentru x 1 0 deci bn (x) este descresc tor pentru orice x 1 fixat.

1 1 , 1 e n (x 1) n ) n x 1 1 i are termeni pozitivi,

Propozi ia 4. irul c n (x ) (1 Demonstra ie. Observ m c

1 n x

)n

este cresc tor,

(0,1] .

Conf. dr., Catedra de matematic , Univ. Tehnic Gh. Asachi, Ia i

28

1 1 , n 1. 1 a n ( x 1) n 1 (1 ) n x 1 Conform cu Propozi ia 2, irul an (x 1) este descresc tor pentru x 1 ( 1,0] x (0,1] i are termenii pozitivi, deci c n (x ) este cresc tor pentru x (0,1] . c n ( x) (1
1

1 n ) n x

Propozi ia 5. irul E n p

(1

1 n ) n
p 2

este descresc tor,


p 2

p N*.

Demonstra ie. Din identitatea a n p b n p (a b)(a n p 1 a n


rezult
an
p

b ... ab n

bn

bn

(a b )(n
p

p )b n
p

p 1

. Punnd aici a

), a b 0, 1 1 i b 1 , 1 n n 1

p 1

se ob ine seama c

(1
1

1 n ) n

(1

1 n ) n 1
1

(1 va

n p ) pentru n 1 i p fixat. innd n (n 2) rezulta


(1 1 n ) n
p

n p n ( n 2)

1 , n 1

(1

1 n ) n 1

1 p

E n ( p)

E n 1 ( p ) , ceea ce ncheie demonstra ia. 1 n ) n


x

Propozi ia 6. irul E n (x ) (1

este descresc tor,

x 1 fixat. 1t ) t
x

Demonstra ie. Se consider func ia f : [1, )


fixat. Atunci f (t ) pentru y
f (t )[ln(1

R, f (t )

(1

, unde x 1

t x 1 ) ] i folosind inegalitatea ln(1 y ) y t t (t 1) t x 1 t x 1 x 1 0 , avem ln(1 ) , de unde f ' t 0 i t t (t 1) t t (t 1) t (t 1)

deci f este descresc toare pentru t 1 . Avem f (n) pentru n 1 , ceea ce ncheie demonstra ia.

f (n 1)

E n ( x)

E n 1 ( x)

Observa ie. Propozi ia 5 este un caz particular al Propozi iei 6, avnd ns o demonstra ie elementar . Propozi ia 7. irul u n ( x ) (1
1 n ) n
x

este cresc tor,


1 n ) n 1
1 x

0 fixat.

Demonstra ie. Putem scrie u n 1 (x ) (1


Conform Propozi iei 6, irul E n ( x are termenii pozitivi. Rezult

1 1 . 1 n x 1 E n (x 1) (1 ) n 1) este descresc tor pentru x 1 1 x 0 i ca irul u n 1 (x ) este cresc tor, adic

u 2 (x ) u 3 ( x ) ... u n (x ) ... . Dar u1 ( x ) =0, deci avem i u 1 (x ) u 2 (x ) , ceea ce ncheie demonstra ia.

29

Propozi ia 8. irul v n ( x ) (1

1 n ) n 1

este descresc tor,


R,
g (t ) (1

x t

0 fixat. 1 1 )t
x

Demonstra ie. Se consider func ia g : (0, )


x 0

, unde

fixat. Avem
y, 0

ln(1 y )

t x i folosind inegalitatea ] t (t 1) 1 t x 1 t x x y 1, g sim ln(1 ) 0 t 1 t (t 1) t 1 t (t 1) t (t 1) g (t ) g (t )[ln(1 1 t 1 )

pentru t (0, ). Deci g (t ) 0 , t (0, ) , adic g este descresc toare pe (0, ) . Vom avea g ( n ) g ( n 1) v n (x ) v n 1 (x ) , n 1 , ceea ce ncheie demonstra ia.
1 n x este cresc tor, x 1 fixat. ) n 1 1 n x 1 1 Demonstra ie. Avem v n ( x ) (1 , n 1. ) 1 n x n 1 E n (x ) (1 ) n Conform Propozi iei 6, irul E n (x ) este descresc tor pentru x 1 i are termenii

Propozi ia 9. irul v n ( x ) (1

pozitivi, deci v n x

este cresc tor,

1 fixat.

Observa ii. n demonstra ia Propozi iei 1, ne-am inspirat din [1;p. Relativ la irul E n (x ) , n [2; p. 223-224] se arat pe cale neelementar 1 1 descresc tor pentru x i cresc tor pentru x . In [3; p. 13-14] se arat 2 2 1 elementar ca irul E n ( ) este descresc tor. irurile studiate n Nota de 2 aplica ie n studiul unor serii de func ii.

24-25]. c este pe cale fa au

Bibliografie 1. Fr. Junker Hhere Analysis I, 1920. 2. G. Klambauer Problems and Propositions in Analysis, New York, 1979. 3. A. Vernescu iruri de numere reale, Bucure ti, 2000.

30

Studiu comparativ privind cteva medii uzuale


Claudiu- tefan POPA 1
Vom folosi n cele ce urmeaz urm toarele conven ii de notare: dac
mh
x, y 0, atunci

x2 y2 2 xy x y xy , m a , mp sunt mediile armonic , , mg x y 2 2 geometric , aritmetic , respectiv p tratic ale numerelor x i y. De asemenea, introducem
m x2 x y2 media ponderat a numerelor x i y cu ponderile reale x, respectiv y. n y

acest context, m h poate fi privit ca media ponderat a numerelor x i y cu ponderile y, respectiv x. Au loc urm toarele inegalit i ntre medii: mh m g ma m p m (1) cu egalitate pentru x
y.

Se observ u or prin calcul c m ma ma mh , (2) deci media aritmetic a numerelor x i y este medie aritmetic i pentru numerele m h i m. De asemenea, a a cum m g este i medie geometric ntre m h i m a , la fel m p este medie geometric ntre m a i m, deoarece Vom presupune n continuare x
B mp ma , C mg mh , D x2 2 m mp : y2 x y x2 x 2 y2 . y
ma

(3)
mg ,

y i, ca n figura de mai jos, not m A

A C B D A. 2 Demonstra ie. Pentru prima inegalitate, avem echivalent: A C m a m g 2 m g 2m h m a 2m h 3m g . 2 a b 4ab a b 4ab ns m a 2m h 2 2 2ab 3 ab 3m g . La fel, 2 2 a b a b

Propozi ie. Cu nota iile de mai sus, avem:

(4)

mg

mh

mp

ma

x 2

2 xy x y
2

x2 2
x2 y 2 2

y2

xy
1 x y

x y 2 2x y
1

x2 2

y2

xy

x2 2

y2

xy

x y 2

xy

Profesor, coala Alecu Russo, Ia i

31

x y

1
Pentru a ar ta c

x2 2
A

y2

xy

1 x y

x2 2

y2
2 mg

xy

B.

B, s observ m mai nti c m 2 p


2 (m p

2 2ma , rela ie ce rezult n

urma unui calcul de rutin ; deci m a numerelor m p


mp mg x

2 m g ) / 2 . Pe de alt parte, aplicnd (1)

i m g , avem c
y. Cum x

2 (m p

2 mg )/2

(m p
(m p

m g ) / 2 , cu egalitate pentru
m g ) / 2 , adic A

y, rezult c

ma

B.
ma )

n continuare s aplic m inegalitatea numerelor ma i m. .

xy

xy (echivalent cu m g
mp B ma m mp B

innd seama de (3) rezult c

D,

egalitatea fiind atins pentru cazul exceptat x y. n fine, inegalitatea D A rezult din faptul c A C (2)) i C B.

D (rela ie echivalent cu

Observa ia 1. Rezultatul demonstrat generalizeaz unele probleme ap rute n Gazeta Matematic . Astfel problemele x y xy E: 11 997. Dac x, y 0, demonstra i c xy . 4 x y Gh. Nec uleu i Ion Nec uleu E: 12 162. Dac x, y

0, s se arate c

x2 2

y2

xy .

Claudiu- tefan Popa

sunt simple transcrip ii ale inegalit ilor A

C , respectiv A

B.

Observa ia 2. Se pot ob ine inegalit i mai interesante, n care s apar mai multe dintre medii. Prezent m un exemplu: adunnd membru cu membru inegalit ile A B,
A C

ob inem

3A

A B C

xy

1 3

x2

y2

2 xy , x y

inegalitate a c rei demonstra ie direct este destul de laborioas .


Observa ia 3. Rela iile (4) arat c oricum am alege trei dintre numerele A, B, C, D, ele pot constitui laturile unui triunghi (n cazul x y ). Observa ia 4. Continund ideea, a ez m pe o ax numerele C B D A i comparnd lungimile intervalelor care apar, ob inem noi inegalit i n care apar mediile. n acest sens, problema: x y xy x y E: 12 177. Dac x, y 0, demonstra i c . xy 4 x y 2 Manuela Prajea este banal , ntruct se reduce la A C x y , inegalitate grosier .

32

NOTA ELEVULUI
Aplica ii ale monotoniei mediilor n raport cu ordinul lor
Codrin ANDREI
ordin

tefan RUSU 1

Fie date numerele x1, x2, ..., xn R*+ (n N*) i R. Se nume te media de a numerelor x1, x2, ..., xn num rul M (x1,...,xn) (pe scurt, M ) definit prin
x1 M
n

x2 n x2

xn xn ,

1/

, 0

(1)

x1

Observ m c
media aritmetic

M0= n x1 x 2 ... x n = Gn
ic M-1= 1
x1

este media geometric , M1=

x1 x 2 ... x n
n

=An -

n 1 1 =Hn - media armonic a numerelor date. ... x2 xn

Mai general, se nume te media ponderat de ordin a numerelor x1, x2, ..., xn cu ponderile (pozitive) p1, p2, ..., pn num rul M (x, p) (pe scurt, M ) definit prin
p1x 1 M (x , p ) p 2 x 2 ... p n x n p 1 p 2 ... p n
1/ ,

0 , 0

(2)

x 1 p1 x 2 p 2 ... x n p n

1 /( p1

p 2 ... p n )

( pentru p1=p2==pn=1, (2) devine (1) ). Vom prezenta mai nti un rezultat binecunoscut, anume :

Propozi ie. Mediile (ponderate sau nu) sunt monoton cresc toare n raport cu R, < M M . ordinul lor, adic , Demonstra ie. S presupunem mai nti c avem 0< < . Ca urmare, exist t >1 astfel nct = t . Utiliz m inegalitatea lui Jensen pentru func ia convex f (x)=x t, x 0, , i scriem :
p1 x1 p 2 x 2 ... p n x n p1 p 2 ... p n t t t t p1 x1 p 2 x 2 ... p n x n p1 p 2 ... p n

(3)

nlocuim n aceasta x1, x2, ..., xn respectiv cu x1 , x 2 ,..., x n ; ob inem :

Elevi, Liceul Teoretic Gr. Moisil, Ia i

33

t p1 x1 p1 p2 x2 p2 ... ... pn xn pn

t p1 x1 p1

t p2 x2 p2 ...

...

t pn xn pn

i , prin ridicare la puterea pozitiv Dac

1 t

deducem c M t >1 astfel nct

Mt , adic M

< <0, atunci exist

=t . Se procedeaz n mod

similar: n (3) se nlocuiesc x1, x2, ..., xn cu x1 , x 2 ,..., x n , se ridic la puterea negativ
1 ambii membri ai inegalit ii rezultate i se ob ine M t

Mt

, adic

M .

s tind la zero, n sfr it, dac n inegalitatea M < M ( 0< < ) facem ca vom ob ine M0 < M ( >0). Tot ca un caz limit se ob ine i M < M0 ( <0). Q.e.d. Indic m cteva aplica ii ale propozi iei de mai sus.
y y

1. S se rezolve ecua ia x Solu ie. Avem trei cazuri : 1 I. y=1. Ecua ia devine x x x
A adar perechile (x,1),
x

1 x

1 x

2x y , x

1 iy

0.

1 x

2 x i aceasta este verificat de orice x

1.

1 , sunt solu ii ale ecua iei date.

II. 0<y<1. Utiliznd inegalitatea My M1 pentru numerele distincte


1 x
y

1 x

1 x

ob inem :

1 x

1 x

1 y

2x y

, deci ecua ia dat

nu are solu ii n acest caz. III. y>1. Utiliz m de aceast dat inegalitatea M1 My pentru acelea i numere distincte i ob inem c membrul nti al ecua iei este strict mai mare ca 2x . Deci nu avem solu ii. Rezumnd, ecua ia dat are ca solu ii perechile (x,1),
x 1.

2. Fie a, b, c >0 cu abc=1. Demonstra i inegalitatea 2(a+b+c) a2b+ab2+a2c+


+ac2+b2c+bc2 (V. Popa, G : 615, R. M. Gala i, nr. 17-18).

Solu ie. Prin calcul direct constat m c a2b + ab2 +a2c +ac2+ b2c+ bc2-2 (a+b+c)
=(a+b+c)(ab+bc+ca-2)-3abc i, deci,avem de ar tat c (a+b+c)(ab+bc+ca-2) 3 (1) (am utilizat condi ia abc=1). Inegalitatea A G pentru numerele a, b, c revine la a+b+c 3 (2) iar G H conduce la 1 3 /(ab bc ca ) sau ab+bc+ca-2 1 (3) (s-a inut seama de rela ia

abc=1). Din (2) i (3) , prin nmul ire, se ob ine (1).

34

3. S se arate c

sin 2n cos 2n 1, 2 Solu ie. Partea dreapt se dovede te astfel : sin 2n


n 1

R i n
cos2n (sin2

N*.
cos2 )n 1 .

Pentru a dovedi inegalitatea din partea stng , utiliz m M1 Mn pentru numerele sin 2 cos 2 ; ob inem : sin 2n cos 2n 2[(sin 2 cos 2 ) / 2]n 1 / 2 n 1 , q.e.d.
4. Fie ABC un triunghi dreptunghic n A i num rul R. Au loc inegalit ile :
2
c
1 /2

(i) b c Solu ie. Avem


(b c )/2

/2

2
1/

a , dac M (b, c)
c2)/2

(ii) b 2 M 2 (b, c)
1/ 2

c
b

a , dac
/2

2.

(b 2

a/

21

Similar se procedeaz pentru a dovedi (ii).


5. Fie ABC un triunghi echilateral nscris n cercul C(O,R) i M C(O,R) , , R astfel nct 2, 2 4 i 4 au loc oarecare. Ar ta i c inegalit ile: (i) MA MB MC 3 2 / 2 R ; (ii ) MA MB MC 3 6 /4 R

(iii) 3 2
4

/2

MA

MB

MC

3 6

/4

Solu ie. Sunt cunoscute rela iile urm toare : MA2+MB2+MC2=6R2 i

MA +MB4+MC4=18R4 ([1], pp. 27-28). Ca urmare, M2(MA, MB, MC)=R 2 M4

i
M2

(MA,MB,MC)=R 4

6 . Pe de alt parte, avem : M

M2

M
/2

M4

M . Din M

urmeaz c MA ob in (ii) i (iii).

MB

MC =3(M )

3(M2) = 3 2

R , adic (i). Analog se

, ,

6. Fie ABCD un p trat nscris nscris n cercul C(O, r). Fie M C(O,r) i R astfel nct 2, 2 4 i 4 . Ar ta i c au loc inegalit ile :
MB MC MD
/2

(i) MA

4 (3/ 2)

/2

r ;

(ii ) MA

MB

MC

MD
/4

4 6

/4

(iii) 4 (3 / 2)
Solu ie.
MA MB MC inegalit ile cerute.
4 4

r
4

MA 24r
4

MB

MC
MA
2

MD
MB
2

4 6
MC
2

r
2

Se
4

utilizeaz MD

egalit ile:

MD

6r 2

[4] i , procednd ca n problema precedent , se ob in

Bibliografie 1.C. Cocea - 200 de probleme din geometria triunghiului echilateral, Ed. Gh. Asachi, Ia i, 1992 . 2.L. Pr an, C.-G. Lazanu -Probleme de algebr i trigonometrie, Ed. Facla, Timi oara,1983. 3.V. tef nescu, N. Deval - Asupra unor inegalit i ntre medii generalizate i aplica iile lor, G.M. 1/1985, 8-11. 4.G. i eica Culegere de probleme de geometrie, Ed. Tehnic , Bucure ti, 1956.

35

CHESTIUNI METODICE
Func ii care admit / nu admit primitive
Dumitru G LEAT
1

i Gabriel POPA

Ne propunem n ceea ce urmeaz o abordare ct mai intuitiv a unor probleme considerate n general ca fiind dificile, grupate n urm toarele clase: I. Dat o func ie, s se stabileasc faptul c admite primitive; II. Dat o func ie, s se arate c nu admite primitive; III. Dat o func ie a c rei expresie depinde de anumi i parametri, s se determine valorile acestora astfel nct func ia s admit primitive. n redactarea acestui articol am pornit de la constatarea c solu iile date n diverse c r i unora dintre problemele din [2], pag. 13-14, sunt artificiale i greu de urm rit de c tre elevi. Propunem n cele ce urmeaz o prezentare pe care o credem logic i adecvat pred rii la clas . I. (i) Orice func ie continu admite primitive.
1

Problema 1. Func ia f : R

R , f(x)=

1 x5

2 ex

,x ,x

0 0

admite primitive.
5 2

Solu ie. Substitu ia


5

1 x2

t conduce la

1 x5

5 2
5

dac x

0 i

1 x5

dac

0 ; lim f ( x) lim t = 0, de x 0 t et e f(0), adic f este continu n 0. Cum f este evident continu pe rezult c f continu pe R , deci f admite primitive. 1 x sin ,x R , f(x)= Problema 2. Func iile f : R x 0 ,x
x 0 t

0 , deci lim f ( x)

lim

t2

t2

unde

lim f ( x) = 0 =
x 0

(- , 0) i pe (0 , ),
0 0

g:R

R,

g(x)=

x cos 0

1 x

,x ,x

0 0

admit primitive.
0

adic

1 x 0 x x f este continu . Analog se procedeaz pentru g .

Solu ie. Pentru f, avem

x sin

0 , deci lim f ( x ) = 0 = f(0),


x 0

(ii) Anumitor func ii li se pot construi efectiv primitivele.


1 2

Profesor, Grup colar tefan Procopiu, Ia i Profesor, Liceul Teoretic Garabet Ibr ileanu, Ia i

36

Problema 3. Func iile f : R


1 1 sin x x 0

R , f(x)=

2 x sin

1 1 cos x x 0

,x ,x

0 0

i g:R

R,

g(x) =

2 x cos

,x ,x

0 0

admit primitive.
x 2 sin 0 1 x

Solu ie. Consider m func ia F ( x)


i F (x) = f(x),

,x ,x

0 0

, care este derivabil pe R

x R . S mai ar t m c F este derivabil n origine, iar F (0) = 0: 1 x 2 sin F ( x) F (0) x lim x sin 1 0 lim F ' (0) lim x 0 x 0 x 0 x 0 x x (vezi Problema 2), deci F este primitiv a lui f . Analog se procedeaz pentru g . 1 1 nx n 1 sin x n 2 cos ,x 0 Generalizare 1. Func iile fn : R R , fn(x)= i x x 0 ,x 0

g n: R

R , gn(x)=

nx n 1 cos

1 x 0

x n 2 sin

1 x

,x ,x

0 0

admit primitive, :

n 2.

Generalizare 1 (n 1) x n sin n fn(x)= x 0


i gn : R

2.
n cos 1 xn

Func iile
,x ,x 0 0 n sin 1 xn

fn

R , gn(x)=

(n 1) x n cos

1 xn 0

,x ,x

0 0

admit primitive,

1.

(iii) O combina ie liniar a unor func ii ce admit primitive, admite primitive. 1 sin ,x 0 R , f(x)= i g:R R , g(x)= Problema 4. Func iile f : R x 0 ,x 0
=
cos 0 1 x ,x ,x 0 0

admit primitive.

Solu ie. Func ia f se poate scrie ca o combina ie liniar cu coeficien ii 2, 1 a func iilor g din Problemele 2 i 3, deci admite primitive. Analog pentru g . x Observa ia 1. Dac F este o primitiv a func iei f, atunci func ia f ( ) admite

primitiva

37

Observa ia 2. Func iile f i g snt exemple de func ii necontinue (cu discontinuitate de specia a doua n origine) care admit primitive. 1 sin 2 ,x 0 x Problema 5. Func iile f : R R , f(x)= i g:R R , g(x)= 1 ,x 0 2
cos 2

1 x

,x ,x

1 2

admit primitive.
0

Solu ie. Avem f ( x)

1 2 (1 cos ) , x 2 x 1 ,x 2

=
0

1 2

2 1 cos x 2 0

,x ,x

0 0

i analog

pentru g.
Problema 6. Func iile f : R
cos3 0 1 x

R , f(x)=

sin 3 0

1 x

,x ,x

0 0

i g:R

R , g(x)=

,x ,x

0 0

admit primitive.
1 3 sin x 4 0 3 0 1 sin x 0 0 4

Solu ie. sin

3 x

3 sin

1 x

4 sin 3

1 , deci f ( x) x

,x ,x

,x ,x

0 0

fiecare dintre func iile din dreapta admi nd primitive cf. Problemei 4 i Observa iei 1. 1 sin 2 n 1 ,x 0 Generalizare. Func ia f : R R , f(x)= admite primitive x 0 ,x 0 (O.M.,etapa jude ean , 1982).
II. (i) Func iile care nu au P.D. nu admit primitive. n acest sens, innd seama de propriet ile unei func ii cu P.D., putem proceda n mai multe moduri: Dac Im f nu este interval, atunci f nu admite primitive; Dac f are discontinuit i de specia I , atunci f nu admite primitive. Problema 7. Func ia f : R R, f(x) = x nu admite primitive. Solu ie. Im f = Z , care nu este interval. Problema 8. Func ia f : R Solu ie. R , f(x) = x
x x
x k

nu admite primitive.
0, deci

Pentru

Z,

avem

lim f ( x ) 1, lim f ( x)
x k

are

discontinuit i de specia I.
Problema 9. Func ia h : (a,b) R , h(x)=

f ( x) , x g ( x) ,x

( a, b)

( a, b) / Q

, cu f i g

38

func ii continue distincte, nu admite primitive (O.M., etapa final , 1981). Solu ie. Cum f g, exist x0 (a, b) a.. f(x0) g(x0) . Cum R este separat Hausdorff, putem alege Vf V(f(x0)) i Vg V(g(x0)) a.. Vf Vg = . Deoarece f, g sunt continue, exist Uf ,Ug V(x0) (pe care le putem presupune intervale deschise) a.. x (a, b) Uf f(x) Vf i x (a, b) Ug g(x) Vg. Observ m acum c (a, b) Uf Ug este un interval, iar imaginea sa prin h este o reuniune de mul imi disjuncte incluse n Vf respectiv Vg ,care nu poate fi interval. Observa ie. Particulariznd convenabil func iile f i g, ob inem solu ii pentru problemele II.5, 6, pag. 13 din [2].
(ii) Pentru anumite func ii, putem demonstra efectiv c nu admit primitive. 1 1 1 sin cos ,x 0 R , f1(x)= nu admite Problema 10. Func ia f1 : R x x x 0 ,x 0

primitive (v. i Problema 3).


Solu ie. Observ m c
x sin 1 x sin x sin 1 x 1 1 cos , deci o eventual primitiv a x x ,x ,x 0 0

lui f 1 are n mod necesar forma F(x) = continu , atunci c1 = lim F ( x)


x 0

1 c1 x c2

. Cum F trebuie s fie

F (0)

c2 , deci c1 = c2. Func ia F trebuie s fie

F ( x ) F ( 0) derivabil , deci exist limita lim x 0 x 0 posibil. Urmeaz c f 1 nu admite primitive.

x sin
x

lim

1 x

lim sin
0

1 , ceea ce nu este x

(iii) Reducere la absurd.


sin

Problema 11. Func ia f : R

R , f(x) =

1 x

,x ,x

1 2

nu admite primitive.
0 sin 0 1 x ,x ,x 0 0

Solu ie. Presupunem c f ar admite primitive. Cum func ia g(x) =

admite primitive (v. Problema 4), atunci diferen a lor admite primitive, deci func ia h(x) = 0 ,x 0 1 f(x) g(x) = 1 admite primitive. ns Im h = 0, nu este interval , deci h nu ,x 0 2 2 admite primitive. Presupunerea f cut este fals . Observa ia 1. Problema de mai sus furnizeaz un exemplu de func ie cu P.D. care nu admite primitive. Observa ia 2. Putem da, folosind acest ra ionament, nenum rate exemple de func ii cu ramuri care nu admit primitive. De altfel, n unele culegeri de teste gril n vederea preg tirii a diverse examene, se ntlnesc probleme de forma: 39

Func ia f : R

R , f(x) =

sin a

1 x

,x ,x

0 0

admite primitive d.n.d. a =

Elevii trebuie obi nui i s simt valoarea potrivit a constantei nc nainte de a trece la rezolvarea propriu-zis a problemei !
III. Apar deseori aplica ii de tipul: Problema 12. Se consider func ia f : R R , f(x) =
ae x 1 x b x ,x ,x 0 0

cu

R. Determina i a, b a.. f s admit primitive. Metoda uzual de abordare a unor astfel de probleme este c utarea primitivelor lui f pe ramuri, considerarea unei primitive F sub forma general
ae x c1 c2 x xb (1 x 1) 2 ,x ,x c3 , x 0 0 0

a, b

F(x)=
2 x 1 ln

i determinarea apoi a constantelor a i b

a.. F s fie continu i derivabil . Propus pentru prima oar n clas , problema prime te ns de obicei urm toarea rezolvare: Pentru ca f s admit primitive, trebuie ca f s fie continu i putem afla a, b din continuitatea lui f . L snd la o parte confuzia care se face ntre condi ia necesar i cea suficient , n contextul dat un astfel de r spuns nu este chiar att de gre it. n 4 este enun at i par ial demonstrat urm torul rezultat (Problema 10, pag. 118):
Teorem . Fie un interval, x0 punct interior lui , iar f : R o func ie cu P.D. Dac f are limite laterale n x0, atunci f este continu n x0. Demonstra ia dat n 4 este valabil numai n cazul limitelor laterale finite (adic al discontinuit ilor de specia I), ns ea poate fi adaptat i n cazul general. i Presupunem, prin reducere la absurd, c f(x0 - 0) < f(x0) (limita putnd fi finit sau fie un num r ntre cele dou valori. Atunci V V(f(x0 0)), U V(x0) a.. x U , f(x) V. Considernd V = (f(x0 0), ), exist U pe care o putem lua x < x0 U = a, x0) a.. x U f(x) V ; altfel spus, f(x) < , x a, x0). Rezult c f nu ia valoarea pentru x a, x0 , fals. Revenind la Problema 12, f admite primitive, deci are P.D. n plus, f are limite ,b 1 laterale n 0 : f(0 0) = a, iar f(0 + 0) = 1 . Din teorema de mai sus, rezult c ,b 1 2 1 a= , b =1. 2 Bibliografie 1. V. Arsinte Probleme elementare de calcul integral , Ed. Univ. Bucure ti, 1995. 2. N. Boboc, I. Colojoar Analiz matematic , manual cl. a XII-a, E.D.P., 1999. 3. M. Ganga Elemente de analiz matematic , cl. a XII-a, Ed. Mathpress, 1999. 4. Gh. Gussi et al. Analiz matematic , manual cl. a XI-a, E.D.P., 1999.

40

CHESTIUNI COMPLEMENTARE MANUALELOR


Construc ii geometrice cu echerul
Eugenia COHAL 1
Nota Redac iei. S-au primit de la mai mul i elevi rezolvitori solu ii necorespunz toare la problema VII.20 din Rec.Mat-1/2001 : ace tia au confundat echerul abstract cu echerul colar ce are un unghi de 30 . Este binevenit aceast lucrare ce aduce clarific ri n aceast privin .

Geometria constructiv sau teoria construc iilor geometrice este o parte a geometriei fascinant prin problematica abordat , vechimea i istoria ei, printr-un num r de probleme celebre formulate n antichitate i care i-au g sit rezolvarea dup multe secole de cercet ri i cu utilizarea cuno tin elor de algebr i analiz matematic . Mul i i str luci i matematicieni au fost atra i de acest domeniu de care i-au legat numele prin contribu iile aduse. Rezolvarea problemelor de construc ii (geometrice) depinde de instrumentele admise a fi utilizate. Pe lng rigl i compas , de-a lungul timpului au fost folosite i alte instrumente : rigla bilateral , rigla cu etalon, distan ierul, echerul etc. sau au fost inventate instrumente speciale cu care puteau fi trasate anumite curbe. Fiecare instrument (sau grup de instrumente) se define te prin opera iile fundamentale ce pot fi efectuate cu acesta (acestea). n prezenta Not aten ia se concentreaz asupra unui singur instrument: echerul. Acesta trebuie imaginat ca dou rigle fixate sub un unghi drept n dou capete ale lor. I se mai spune echerul drept n contrast cu echerul oblic (col arul). Evident, echerul se deosebe te de instrumentul colar cu acela i nume. n fapt, echerul este definit de opera iile (construc iile) ce se pot face cu el pornind de la elementele date de problem (puncte, drepte etc.) ct i de la anumite elemente luate arbitrar. Opera iile fundamentale ce definesc echerul sunt n num r de trei: 1 construirea dreptei ce trece prin dou puncte i determinarea punctului de intersec ie a dou drepte ; 2 construirea perpendicularei la o dreapt ce trece printr-un punct; 3 construirea unui punct pe o drept din care dou puncte se v d sub un unghi drept. D m acum cteva construc ii ce pot fi f cute cu ajutorul echerului. 1. Construc ia paralelei la o dreapt d printr-un punct P exterior ei. Se construie te cu echerul d perpendicular pe d i care trece prin P (2 ). Apoi se construie te d perpendicular n P dreptei d (2 ). Evident, d este drepta ce trebuia construit . 2. Multiplicarea segmentelor. Pentru a dubla, tripla etc. un segment AB dat proced m astfel (fig. 1): Prin punctul A costruim dou drepte d i d1 (1 ), d oarecare iar d1 perpendicular pe d (2 ). Perpendiculara n B pe dreapta AB (2 ) taie dreptele d1 i d n punctele B1 , respectiv B2 (1 ). Fie d2 perpendiculara n B2 pe d (2 ). Perpendiculara n B1 pe drepta d1 se intersecteaz cu d2 n punctul C2 (2 , 1 ). Perpendiculara pe drepta AB ce trece prin C2
1

Profesor, Liceul teoretic Mihai Eminescu, Ia i

41

intersecteaz dreptele d1 i AB n punctele C1 i repectiv C. Ob inem similar punctele D2 , D1, D . a. m. d. Segmentele AB, BC, CD etc. sunt congruente ca proiec ii pe dreapta AB ale segmentelor congruente AB1, B1C1, C1D1 etc.
D1 C1 B1 d A C B C2 B2 Fi g. 1 D2 D d1

P
d2

Q R I

D Fi g. 2

O Fi g. 3

3. mp r irea segmentelor. S urm rim pe fig. 2, mp r irea segmentului AB n trei p r i congruente ; avem d AB (construc ia 1), segmeentele PQ, QR, RS congruente (construc ia 2) i AS BP {I } . Punctele C i D , ce mpart segmentul AB n trei p r i congruente, se ob in ca intersec iile dreptei AB cu dreptele RI i QI. njum t irea se face printr-o construc ie mai simpl : se proiecteaz pe AB centrul unui dreptunghi oarecare construit (cu echerul) pe segmentul AB. 4. Construc ia bisectoarei unui unghi . Procedeul este indicat de fig.3, unde s-au luat: A arbitrar, A ' simetricul lui A fat de O (construc ia 2), B se ob ine punnd vrful echerului pe latura Oy astfel ca laturile lui s treac prin punctele A i A ' (opera ia 3 ) i OM A ' B . Faptul c semidreapta OM este bisectoarea unghiului xOy decurge din congruen a unghiurilor marcate. Observa ie. Dat un triunghi ABC se pot construi cu echerul punctele H, G, O, I. 5. Construc ia centrului unui cerc. Fie A un punct pe cercul dat. Punem vrful echerului n A i marc m cu B i C punctele n care laturile lui intersecteaz cercul. Dreapta BC trece prin centrul cercului. Lu m pe cerc un alt punct A ' i ob inem o dreapt B ' C n acela i mod. Centrul cercului este punctul de intersec ie a acestor drepte. 6. Intersec ia unei drepte cu un cerc. Fie dat un cerc printr-un diametru AB (echivalent, prin centru i un punct al s u) i o dreapt d. Conform cu 3 , g sim cu echerul un punct P d din care diametrul AB se vede sub unghi drept. Atunci, P se afl i pe cerc. Al doilea punct de intersec ie c utat se poate construi (cu echerul) ca simetricul lui P n raport cu dreapta perpendicular pe d i care trece prin centrul cercului. Rezolvarea urm toarelor probleme (cu echerul) o propunem cititorilor ( eventual se g se te n [2] , p.73 ). 7. Date o dreapt d i un punct O d , s se determine un punct P d astfel nct OP AB , segmentul AB fiind dat. 8. Date o dreapt d i un punct P , s se duc prin P o dreapt care s formeze cu d un unghi de m rime egal cu a unui unghi cunoscut. Observa ie . Orice construc ie cu rigla i compasul poate fi efectuat cu echerul. Bibliografie 1. N.F. Cetveruhin Metodele construc iilor geometrice (l. rus ), Moscova, 1952. 2. A. Tth No iuni de teoria construc iilor geometrice, Ed. Did. i Ped., Bucure ti, 1963. 42

DIN ISTORIA MATEMATICII


Teorema celor patru culori
Silviana IONESEI 1
Cum se face c matematica produs prin excelen al gndirii umane, independent de experien poate fi att de admirabil adaptat obiectelor lumii reale? Albert Einstein Este binecunoscut faptul c marile probleme ale matematicii, cum ar fi Marea Teorem a lui Fermat sau Conjectura lui Goldbach, au contribuit enorm la dezvoltarea acestei tiin e. Din eforturile matematicienilor (timp de zeci sau chiar sute de ani) de a g si o rezolvare la ntreb ri n aparen simple s-au n scut noi discipline n matematic , cu aplica ii spectaculoase. Problema celor patru culori are toate valen ele unei probleme de mare carier : n primul rnd formularea ei este extrem de simpl , nu presupune cuno tin e matematice; n al doilea rnd, ea a r mas nerezolvat timp de peste un secol, fiind surprinz tor de grea i a suscitat preocuparea multor matematicieni de prestigiu. Iat cteva repere istorice. n 1852 un geograf din Edinburgh (istoria nu i-a re inut numele) l-a informat pe prietenul s u, student n matematici, c folose te cel mult patru culori pentru o hart mp r it n regiuni, f r ca dou regiuni vecine s aib aceea i culoare (preciz m c este vorba despre h r i plane, cu regiuni nchise, iar vecine sunt regiunile cu o linie de frontier comun ; dou regiuni care se ntlnesc ntr-un num r finit de puncte nu sunt considerate vecine). Tn rului matematician, pe nume Francis Guthrie, i-au pl cut cele aflate i a cerut informa ii mai ample, ns geograful l-a ncredin at c acest procedeu e foarte r spndit i aplicat pretutindeni din cauza economiei care-l prezint . R spunsul nu a fost mul umitor pentru Guthrie ; el i-a propus s demonstreze acest fapt dar nu a reu it. Fratele s u, Frederick, studia chimia la Londra i aflnd de problema care-l preocupa pe Francis a cerut ajutorul profesorului August De Morgan, dar nici acesta nu a g sit o demonstra ie satisf c toare. n c iva ani, problema a ajuns la mod printre matematicieni. Astfel, A. Cayley nefiind nici el capabil s demonstreze valabilitatea teoremei, a propus-o Societ ii Matematice din Londra. S trecem n revist cteva din rezultatele par iale ale demonstr rii teoremei. Faptul c trei culori nu sunt suficiente pentru colorarea oric rei h r i plane a fost repede constatat (vezi fig.1).
1

Profesor, Colegiul Na ional C. Negruzzi , Ia i

43

De Morgan a demonstrat c nu exist hart format din 5 regiuni astfel nct s fie dou cte dou vecine, deci aceasta poate fi colorat cu patru culori. A. B. Kempe, un avocat din Londra, membru al Societ ii Matematice din Londra i deosebit de pasionat de matematic a publicat n 1879 un articol n care sus inea c a demonstrat teorema. Ra ionamentul lui era deosebit de ingenios; el redusese problema la h r i normale, adic h r i n care nu exist ri nchise complet n alte ri i nici puncte

1 4
Fig.1

n care se ntlnesc mai mult de trei regiuni. De i ra ionamentul s-a dovedit incomplet, el con inea ideile de baz ce au condus la demonstra ia corect un secol mai trziu. Astfel, oric rei h r i i se poate asocia un graf n care fiecare regiune este reprezentat printr-un punct i dou puncte vor fi legate printr-o muchie dac i numai dac punctele corespund la dou regiuni vecine (vezi fig.2).

A A C E E E D Fig.2 B D C B

n acest mod problema color rii regiunilor de pe hart revine la problema color rii punctelor din graful asociat astfel nct punctele legate printr-o muchie s fie colorate diferit. Problema celor patru culori a contribuit la cercetari importante n teoria grafurilor, cum ar fi numerele cromatice ale grafurilor. Cu ajutorul unui graf special, matematicianul P. J. Heawood a ar tat n 1890 c demonstra ia lui Kempe are o eroare nu tocmai u or de nl turat.

44

Mai trziu, n 1913, matematicianul Ph. Franklin de la Massachusetts Institute of Technology ridic limita num rului de regiuni pentru care problema este rezolvat de la 5 la 21, iar n 1940 Winn reu e te s ajung la 35 de regiuni. Un alt rezultat deosebit de interesant se datoreaz lui P. J. Heawood, care i-a consacrat 60 de ani din via studierii problemei. Iat -l: probabilitatea de a g si o hart cu mai mult de 36 regiuni care s nu poat fi colorat cu patru culori este mai mic dect 10-10000 ! (de remarcat c 1010000 este un num r mai mare dect num rul atomilor din ntreaga galaxie ). O teorem a celor cinci culori (faptul c cinci culori sunt eficiente pentru a colora o hart ) a fost ob inut relativ u or; o demonstra ie elementar a acestui rezultat poate fi g sit n Despre numere i figuri de H. Rademacher i O. Toeplitz. Pe la mijlocul sec. XX s-a conturat ideea de rezolvare a problemei prin m rirea num rului de regiuni pentru care patru culori sunt suficiente i examinarea unor a a-zise configura ii inevitabile. Dac ar fi fost posibil s se produc toate aceste configura ii i s se arate c ele pot fi colorate cu patru culori, atunci demonstra ia ar fi fost complet . Cea mai eficient metod de producere a configura iilor s-a dovedit a fi un algoritm implementat pe calculator de W. Haken i K. Appel , Universitatea Illinois, SUA , care au lucrat aproape 1200 ore i, n fine, demonstra ia a fost ncheiat [1]. Un an mai trziu, folosind o alt procedur de reducere a configura iilor inevitabile, F. Allaise de la Universitatea Waterloo, Ontario, CA, a reu it s ob in demonstra ia teoremei n numai 50 de ore de dialog om-calculator. Entuziasmul firesc strnit n lumea matematicienilor de acest reu it neobi nuit pn atunci a fost temperat de voci sceptice care sus ineau c aceasta nu e o teorema de matematic n sensul clasic. Astfel, T. Tymoczko n articolul Problema celor patru culori i semnifica ia ei filozofic (Journal of Philosophy, 1979) afirm c teorema exprim un adev r a posteriori i nu a priori, ca orice adev r matematic. Argumentele sale se bazeaz n principal pe imposibilitatea de a verifica manual ( cu creionul ) demonstra ia, dat fiind faptul c nu exist un unic algoritm care s verifice toate programele posibile pe calculator. n replic , E. R. Swart scrie n [5] c inconvenientul semnalat de preopinentul s u este aparent, deoarece calculele foarte numeroase efectuate pe calculator erau de rutin , iar programul utilizat poate fi verificat. Sarcina calculatorului a fost cople itoare prin dimensiuni, sarcin pe care omul tia cum s o abordeze, dar n-ar fi putut-o termina niciodat . A fost prima situa ie memorabil n urma c reia lumea matematicienilor a trebuit s admit existen a unor demonstra ii par ial accesibile omului, ct i dreptul calculatorului de a ne sprijini n stabilirea adev rurilor matematice.

Bibliografie 1. K. Appel, W. Haken Every Planar Map Is Four Colorable , Bulletin of the American Mathematical Society, 82(1976), 711-712. 2. F. Cmpan Probleme celebre, Ed. Albatros, Bucure ti, 1972. 3. Gh. P un Din spectacolul matematicii, Ed. Albatros, Bucure ti, 1983. 4. L.A. Steen Mathematicians Today, Twelve Informal Essays, SpringerVerlag, 1978. 5. E.R Swart The Philosophical Implications of the Four-Color Problem, The American Mathematical Monthly, 87(1980), 697-707.

45

MATEMATICA N CLASELE PRIMARE


Introducerea opera iei de adunare la clasa I
Petru ASAFTEI 1
Introducerea opera iei de adunare la clasa I se poate face fie folosind reuniunea a dou mul imi disjuncte, fie folosind rigletele. n prezenta not metodic utiliz m prima variant , considernd c aceasta este mai puternic ancorat n experien a de via . Vom putea folosi astfel un vocabular mai bogat, contribuind astfel la o mai bun n elegere a opera iei de adunare. n predarea-nv area no iunilor cu con inut matematic la colarii mici, conform cerin elor psiho-pedagogice, trebuie s parcurgem urm toarele etape: I. Etapa oper rii cu mul imile de obiecte concrete (etapa perceptiv ). n aceast etap se realizeaz ac iunea nemijlocit cu obiecte concrete din mediul nconjur tor. Exemplu. Pe o farfurie punem trei mere i patru pere, apoi prin num rare constat m c avem apte fructe. (Se recomand s folosim cel pu in o fraz n care s fie inclu i termenii premerg tori opera iei de adunare ; exemplu : Copii, dac la trei mere ad ug m patru pere, ob inem apte fructe.) II. Etapa form rii reprezent rilor imaginativ-concrete (etapa semi-abstract ). n aceast etap se construiesc mul imi cu mere i pere (jetoane), apoi se face reuniunea lor (fig. 1). Aceast construc ie se efectueaz simultan la tabl i pe caiete. Fiec rei mul imi care intr n reuniune i ata m cifra care indic num rul de fructe (3, respectiv 4). Mul imii ob inut prin reuniune i ata m cifra 7 (prin num rare). Aceast etap putem 3 7 4 s o ncheiem cu o fraz de genul : Copii dac Fig. 1 punem la un loc 3 mere i 4 pere ob inem 7 fructe . III. Etapa scrierii i efectu rii adun rii (etapa abstract ). Acum reprezent m att merele ct i perele cu acelea i simboluri (stelu e, cercule e etc.) (fig. 2). Se explic elevilor c pentru a ar ta faptul c am pus la un loc o mul ime cu trei elemente i alta cu patru elemente se folose te semnul ( simbolul matematic ) ,,+ , numit plus, i scriem 3+4. Scrierea 3+4 se cite te trei plus patru sau trei adunat cu patru. Din etapele precedente, prin num rare, elevii au constatat c mul imea ob inut prin punere la un loc are apte elemente. Aten ion m elevii ca 3+4 i 7 reprezint tot att. Pentru a exprima acest lucru se folose te simbolul ,,=" i scriem 3+4=7 se cite te trei plus patru este egal cu apte . Spunem elevilor c scrierea 3+4 reprezint adunarea neefectuat a 3 3+4 4 numerelor 3 i 4 iar scrierea 3+4=7 nseamn Fig. 2 efectuarea adun rii numerelor 3 i 4. Explic m
1

Profesor, coala Normal Vasile Lupu, Ia i

46

elevilor c trecerea de la scrierea 3+4=7 s-a f cut prin opera ia de adunare. Trebuie s d m elevilor un algoritm eficient de efectuare a opera iei de adunare n acest stadiu. S scriem adunarea 5+2 i s num r m de la 5, nainte, nc dou numere n irul numerelor naturale. Ajungem astfel la num rul 7 i spunem elevilor c 7 este rezultatul adun rii numerelor 5 i 2. Scriem i n acest caz 5+2=7. Pentru verificarea corectitudinii rezultatului ob inut putem s apel m la procedeul punerii la un loc. n continuare putem s d m terminologia specific opera iei de adunare. Referindu-ne la ultima scriere 5+2=7 spunem elevilor c numerele 5 i 2 sunt numere care se adun i se numesc termenii adun rii, primul termen, respectiv, al doilea termen. Despre 7 spunem c este rezultatul adun rii sau totalul. Un caz special l reprezint adunarea cnd unul dintre termeni este zero. Folosim o nou reprezentare (fig. 3) din care elevii constat c dac punem la un loc o mul ime cu dou elemente i o mul ime f r nici un element, ob inem tot o mul ime cu dou elemente. Faptul c am pus elementele celor dou mul imi la un loc se scrie 2+0. Num rnd elementele noii mul imi 2 2+0=2 0 g sim 2 elemente. Cum 2+0 i 2 reprezint tot att Fig. 3 putem scrie 2+0=2. Spunem elevilor c 0 l las pe 2 neschimbat prin opera ia de adunare. Cu un demers asem n tor se pot justifica i scrierile 0+2=2 i 0+0=0. Deoarece 0 are aceste propriet i l numim element neutru la adunare. Suntem preg ti i s punem n eviden proprietatea de simetrie a rela iei de egalitate folosind rela ia tot att. Exemplu. Deoarece 7 reprezint tot att ca i 5+2, putem s scriem 7=5+2 i 5+2=7. Dup ce am consolidat algoritmul de adunare, putem s d m i cele dou propriet i importante ale ei: comutativitatea i asociativitatea. Propunnd elevilor s efectueze adun rile 5+4 i 4+5 vor constata c ob in acela i rezultat, num rul 9, deci 5+4 i 4+5 reprezint tot att. Din acest motiv scriem 5+4=4+5 . Spunem elevilor c rezultatul unei adun ri nu se schimb dac schimb m termenii ntre ei ; n aceasta const proprietatea de comutativitate. Pe baza propriet ii de comutativitate se introduce conceptul de prob a adun rii prin adunare. Exemplu. Propunem elevilor s efectueze adunarea 5+3. Num rnd de la 5, nainte, nc trei numere consecutive, vor ob ine rezultatul 8. Le spunem elevilor c putem verifica acest rezultat efectund adunarea 3+5, adic num rnd de la 3, nainte, nc cinci numere consecutive din irul numerelor naturale. Elevii vor constata c ob in acela i rezultat :8 . Pentru a justifica proprietatea de asociativitate, propunem elevilor s adune numerele 2, 4 i 3 astfel: nti s adune numerele 2 i 4 iar rezultatul ob inut s l adune cu 3, apoi s adune numerele 3 i 4 iar rezultatul s l adune cu 2. n primul caz elevii au efectuat (2+4)+3 iar n al doilea caz au efectuat (4+3)+2 care este tot att ca i 2+(4+3). Cum n ambele cazuri elevii au ob inut acela i rezultat, num rul 9, putem scrie (2+4)+3=(4+3)+2 i spunem elevilor c au utilizat asocierea, nti a lui 2 cu 4, a doua oar a lui 4 cu 3. Putem s introducem scrierea 2+4+3, numit sum cu trei termeni, nsemnnd (2+4)+3 sau (4+3)+2. Observa ii. 1) nv torul trebuie s explice rolul parantezelor n astfel de scrieri. 2) Dup ce elevii pot s scrie cu u urin aceste propriet i pe exemple numerice, se poate trece la scrierea literal a lor, f cndu-se men iunea c literele pot reprezenta numere.

47

CONCURSURI SI EXAMENE
Concurs de admitere 2001 , Ia i
Facultatea de Informatic , Universitatea Al. I. Cuza Elemente de analiz matematic
I. a) 1. Privitor la num rul de asimptote, care dintre afirma iile de mai jos pot fi adev rate pentru o func ie f: R R ? i) f are dou asimptote orizontale, dou asimptote oblice i o asimptot vertical ; ii) f are o asimptot orizontal , o asimptot oblic i o asimptot vertical ; iii) f are o asimptot orizontal , o asimptot oblic i nici o asimptot vertical ; iv) f are dou asimptote oblice, dou asimptote verticale i nici una orizontal . Justifica i r spunsul (r spunsurile) ales(e). 2. Care din propozi iile de mai jos exprim o proprietate adev rat privind continuitatea i derivabilitatea? i) Orice func ie continu ntr-un punct este derivabil n acel punct ; ii) Orice func ie derivabil ntr-un punct este continu n acel punct ; iii) Orice func ie continu pe intervalul [a,b] este derivabil pe intervalul (a,b) ; iv) Orice punct n care o func ie oarecare este derivabil i continu este punct de extrem al acelei func ii. 3. Pentru o func ie f : I R, I interval deschis, f derivabil pe I, fie a I un punct de minim local sau global. Atunci: i) f s' (a )
f d' (a ) ;

ii) f ' (a ) 0 ;

iii) f ' (a)

0;

iv) f ' (a) 0 .

4. Fie o func ie oarecare f :[a,b] R, a<b, f derivabil pe intervalul (a,b). Atunci: i) ntre dou r d cini consecutive ale lui f exist cel mult o r d cin a lui f ' ; ii) ntre dou r d cini consecutive ale lui f ' exist cel mult o r d cin a lui f ; iii) ntre dou r d cini consecutive ale lui f exist cel pu in o r d cin a lui f ' ; iv) ntre dou r d cini consecutive ale lui f ' exist cel pu in o r d cin a lui f . Justifica i r spunsul (r spunsurile) ales(e). 5. Care dintre afirma iile de mai jos sunt adev rate pentru orice ir de numere reale? i) Orice ir convergent este marginit ; ii) Orice ir convergent este monoton ; iii) Orice ir m rginit este convergent; iv) Orice ir monoton este convergent .
Not . R spunsurile se vor da numai pe teza cu col uri nnegrite, sub forma: I.a) 1.x, unde x { i, ii, iii, iv} etc. Fiecare ntrebare are cel pu in una din cele patru variante corect . Dac la o ntrebare sunt adev rate mai multe variante, atunci se vor indica toate variantele adev rate. Punctajul la o ntrebare nu se acord dac indica i i variante false. Acolo unde se cere, se va ad uga justificarea r spunsului dat.

b) Fie func ia f : R R, f (x )

1 1. S se studieze continuitatea func iei n punctul x=0. 2. S se studieze derivabilitatea func iei n punctul x=0. 3. S se comenteze rezultatele de la puntele I.b)1. i I.b)2. prin prisma r spunsului care l-a i dat la punctul I.a) 2 .

x, x x, x

0 . 0

48

II. a) Fie a R, a>0, fixat i


xn

xn

n 0

irul definit prin rela ia de recuren :


2. S se studieze convergen a acestui ir.
( x a)2 2b 2

1 xn 2

a , xn 1

n 1, cu x 0

b) S se reprezinte grafic func ia f : R R , f ( x) e

, a i b constante reale.

Algebra
III. a) Fie (G, ) un grup cu proprietatea c x y 2 x 2 y 2 pentru oricare x, y din G. Demonstra i c G este grup abelian. b) S se discute dup valorile parametrilor a Z 8 , b Z 8 i s se rezolve n Z8 ecua ia a x b 0. Fiecare caz identificat va fi ilustrat printr-o ecua ie (a=?, b=?) i prin solu iile

respective.
IV. a) Defini ia corpului comutativ. b) Fie a, b, c numere reale. Definim pe R legile de compozi ie: x y ax by 2 , oricare ar fi x i y numere reale; x y xy 2 x 2 y c , oricare ar fi x i y numere reale. Determina i a, b i c astfel nct (R, , ) s fie corp.

Facultatea de automatic

i calculatoare, Univ. Tehnic Gh. Asachi

1. Suma cuburilor r d cinilor trinomului 3x 2 3(a 1) x a 2 , a R, are valoarea maxim dac : 1 3 3 ; (d) a 0 ; (a) a (e) a [ 2, 1] . ; (b) a ; (c) a 4 4 4 2 b2 2.Mul imea r d cinilor ecua iei a a 2 b 2 , a, b C, ab 0, este: x x 1

(a) {0, 1};

(b)

a2 a2

i ab a 2 i ab , b2 a 2 b2
i ab b2
; (e)

(c)

a2 a
2

iab a 2 iab , b 2 a2 b 2

(d)

2a 2 a2

i ab 2a 2 , 2 b2 a

2a 2 a2
1

iab 2a 2 iab , 2 b2 a b2

3. Func ia f (a) (b) (c)

: (0, 1) (1, ) R, definit de f ( x ) x ln x , este: strict descresc toare pe (0, 1) i strict cresc toare pe (1, + ); strict cresc toare pe (0, 1) i strict descresc toare pe (1, + ); injectiv ; (d) constant ; (e) surjectiv . ax y 1 a cu a R. S se determine a, astfel nct sistemul s fie 4. Fie sistemul x ay 1 a compatibil determinat i solu ia s satisfac condi ia x 0, y 0.

49

(a)

2
(d)

(b) 1 a a 1 2; 2 a 2 1;

2 1; (e) 1

(c)
2 a

1
1

2
2.

a 1

1 5. Se consider irul ( xn ) , dat prin rela ia xn (1 xn 1 ) pentru n 1 , n care x0 2 este un num r real fixat. S se studieze monotonia irului i s se afle lim xn .
n

(a) ( xn ) cresc tor i lim xn


n

2;
1; 2

(b) ( xn ) cresc tor i lim xn


n

1;

(c) ( xn ) descresc tor i lim xn


n

(d) ( xn ) descresc tor i lim xn


n

2;

(e) ( xn ) cresc tor i lim x n


n

3 . 2
n care m este un parametru

6. Se d func ia f : R R prin rela ia

f ( x)

x x
2

m 1

real. S se afle valoarea lui m pentru care f are extrem n punctul x = 1 i s se precizeze natura acestui extrem. (b) m (a) m 3 , x 1 punct de maxim; 3, x 1 punct de minim; 2 1 (d) m (c) m 1 , x 1 punct de maxim; , x 1 punct de maxim; 2 2 (e) m 1 , x 1 punct de minim. e
7. Valoarea integralei
1

x dx 1)

2 0 ( x 1) ( x

este:

1 1 ; (c) 1 ( ( ln 2) ; (e) 2 ln 2) ; (d) ( 3) . 4 4 2 8 8. Se consider dreptunghiul ABCD unde AB=3BC. Pe latura CD se ia punctul E astfel c EC BC , iar AE DB={O}. Se cere m sura unghiului DOA. (c) 45o; (d) arcsin 5 ; (a) 30o; (b) arccos 5 ; (e) 60o. 3 3 9. S se determine solu iile ecua iei sin10 x cos10 x 29 cos 4 2 x . 16 3 (k Z) . (a) k ; (b) k ; (e) k k ; (d) k ; (c) 8 2 8 4 8 8 8 2 10. O m rgea se ob ine dintr-un corp sferic sfredelind o gaur de forma unui cilindru circular drept. tiind c n l imea cilindrului este 6 se cere volumul m rgelei. (b) 36 ; (c) 48 ; (d) 100 ; (e) 50 . (a) 24 ; 3 3
(a)
1
Not . Timp de lucru 3 ore. Fiecare problem are numai una din cele cinci variante corect .

3 ; (b) 2

50

Capacitate - teste preg titoare


Ion SECRIERU 1
Testul 1 A. 1. tiind c viteza luminii este 300 000 km/s, atunci distan a de 1 an-lumin este egal cu. km. 2. n timpul semestrului I al anului colar 2001-2002, un elev a ob inut la matematic notele 7, 7, 9, 7, 7, 7, iar la tez nota 8. Media la sfr itul semestrului este . 36% din 12 3. Valoarea raportului este .. 12% din 36 4. Un avion pleac de la Londra la ora 540 G.M.T. i ajunge la Bucure ti la ora 1200 ora local . Timpul n care a parcurs avionul distan a Londra-Bucure ti este.. are solu iile. ( x y )( x y ) ( x 1) 2 y 2 9 6. Fie punctele A(-2, -2), B(1, 2). Distan a AB este. 7. tiind c raza P mntului este 6400 km, atunci: a) lungimea Ecuatorului estekm; b) lungimea paralelei 450 este.km. 8. Un teren are forma unui dreptunghi cu lungimea de 150 m i l imea de 50 m. a) Suprafa a acestui teren este..m2; b) Costul aratului pentru un hectar este 1 000 000 lei; atunci aratul terenului cost lei. 9. O piramid patrulater regulat are latura bazei de 12 cm, iar unghiul diedru dintre o fa lateral i planul bazei este de 60 0. a) Aria sec iunii diagonale a piramidei estecm2; b) Volumul piramidei este.cm3. B. 10. La un concurs de matematic , valoarea cumulat a premiilor primilor patru clasa i a fost 3 200 000 lei. Premierea a fost f cut dup cum urmeaz : men iunea prime te o optime din sum , iar premiile III, II i I au fost r spl tite direct propor ional cu numerele 2,5; 5, respectiv 6,5. Ce sum a primit fiecare premiant? 11. Fie func iile f, g: R R, f(x) = ax+b, g(x) = 2 x 4 . a) Determina i func ia f tiind c graficul acesteia trece prin punctele A(1, 1), B(0, 2); b) Reprezenta i grafic, folosind acela i sistem de axe, func iile f i g; c) Determina i aria suprafe ei cuprinse ntre graficele celor dou func ii i axa Oy. 12. Fie trapezul isoscel ABCD nscris ntr-un cerc de raz 24. tiind c bazele AB i CD sunt de o parte i de alta a centrului cercului i subntind unghiuri de 120 0 , respectiv 60 0, s se calculeze: a) lungimile bazelor; b) aria i perimetrul trapezului;
1 2

i Cezar Marius ROMA CU 2

5. Sistemul

( x 1) 2

( y 3) 2

10

Profesor, Grupul colar Economic nr. 2 Virgil Madgearu, Ia i Profesor, coala Picioru Lupului, Ciurea (Ia i)

51

c) raportul volumelor corpurilor ob inute prin rotirea trapezului o dat n jurul bazei mici, apoi n jurul bazei mari. Testul 2 A. 1. Numerele naturale 4 x3 y divizibile cu 45 sunt 2. Solu ia inecua iei (x 3. Calculnd (1
2)(x 5) 4x 20 0 este ..

3 )( 4 2 3

2 3 ) , ob inem..
a b
3

4. Dac numerele naturale a, b sunt astfel nct a+b=.. 5. Pentru A= x


c
N; x 1 4 ; B= x R; x 1

4 7

i [a, b] =308, atunci

i C= x

Z;

x x

2 3

, avem

A B C . 6. Aria lateral a unui cilindru avnd ca sec iune axial un p trat este egal cu aria lateral a unui con echilater. Raportul volumelor este

7. Un romb are un unghi de 120 0 i n l imea 4 3 cm. a) Perimetrul rombului este.cm; b) Latura unui hexagon regulat echivalent cu rombul estecm. 8. ntr-un cub ABCDA B C D distan a de la vrful A la mijlocul E al laturii [BC] este 15 cm. a) Aria cubului estecm2; b) Volumul cubului este..cm3. 4 3 53 3 dintr-un num r ad ug m din alt num r ob inem ; din primul 9. Dac la 11 5 55 5 13 4 mai mare dect din al doilea num r. num r este cu 55 11 y x a) ..; 13 53 b) x + y = a b b c B. 10. Se dau propor iile i , cu a, b, c Z . 2 3 4 5 a b c a) S se determine k i p a a nct ; 8 k p b) Pentru k =12 i p=15, s se determine a, b, c n fiecare din urm toarele cazuri: (i) a+b+c=70; (ii) a2+b2+c2=433. 11. Pe laturile [BC] i [AD] ale paralelogramului ABCD se construiesc n exterior p tratele BCMN i ADPQ. S se demonstreze c : a) [BP] [ND]; b) PN, MQ, AC i BD sunt drepte concurente. 12. O piramid patrulater regulat SABCD are toate muchiile de lungimi egale. tiind

c apotema piramidei este de 6 3 cm , s se calculeze:

52

a) aria lateral i volumul piramidei; b) unghiul diedru format de planele (SAB) i (ABC); c) distan a de la centrul bazei la o fa a lateral . Testul 3 A. 1. Dac

xyz

yz , atunci xyz
n2 n 6 n 1
Z dac n

2. Pentru n Z, frac ia 3. Rezultatul calcului

{.}.

este . 3 2 2 3 2 2 4. ntre bazele trapezului ABCD exist rela iile ma=10 i mg=5; lungimea segmentului paralel cu bazele care trece prin punctul de intersec ie a diagonalelor este 5. ntr-un paralelogram ABCD (BC>CD), mediatoarea diagonalei [BD] intersecteaz dreptele AB i CD n E, respectiv F. Patrulaterul BFDE este 6. Lungimea i l imea paralelipipedului dreptunghic ABCDA B C D sunt valorile x a x a a ( 4a 3 x) absolute ale solu iilor ecua iei , a R+ , iar n l imea reprezint x a x a x2 a2 jum tate din lungime. Dac M, N sunt mijloacele segmentelor AD , respectiv A B , atunci d(M, N)=.
7. Sec iunea axial a unui con echilater are n l imea de 6 3 cm. a) Aria total a conului este..cm2; b) Volumul conului estecm3. 6 Z, 6 x 4 8. Fie mul imile A= x Z i B= x Z 2 3x 4 13 . x 1
Atunci A={..} i B={}. 9. Fie mul imile A={-2, 0, 2} i B={-2, 4, 10}. O func ie liniar f: A B are legea de compozi ie f(x)=. sau f(x)=. 3111 3 291 B. 10.a) S se compare numerele a= 333 i b= 453 . 5 5 b) S se arate c num rul a= 5 n 3 2 n 125 este divizibil cu 45 pentru orice n Fie func iile f, g: R R, f(x) = ax+b, g(x) = cx+d. a) S se determine f i g tiind c f x 1 3x 5 f 2 i f 2 x 3 g 2 x 5 ; b) Determina i valoarea parametrului real m pentru care ecua ia mx2+f(x)=0 are solu iile egale; afla i aceste solu ii. 12. Doi elevi privesc simultan i din aceea i parte un turn. Primul vede vrful turnului sub un unghi de 45 0, iar al doilea sub un unghi de 30 0. tiind c n l imea copiilor este de

3 2 2

3 2 2

1,50 m iar distan a de la turn pn la al doilea elev este 9 3 m, afla i: a) n l imea turnului; b) distan a dintre cei doi copii.

53

Testul 4 A. 1. S se determine numerele naturale a i b tiind c (a, b)=15 i a b=3150. 2. Afla i elementele mul imii A= abc abc
c N , tiind c a b c a.

3. G si i numerele naturale a, b, c tiind c a reprezint 20% din b, suma dintre a i c este 24, iar b i c sunt direct propor ionale cu 5 i 7. a c 4. Dac a2+b2+c2+14=4a+2b+6c, calcula i . b 5. Determina i func iile liniare f: R x R.
R cu proprietatea c f(x-2)

f(x+2)=x2-2x-3,

6. ntr-un triunghi ascu itunghic n care m B este cu 50 0 mai mare dect m C se construiesc n l imea AD i bisectoarea CE. tiind c m BAD i m BEA sunt direct propor ionale cu 3 i 4, s se calculeze m surile unghiurilor triunghiului ABC.

7. O prism din lemn are ca baz un p trat cu diagonala 12 2 cm. tiind c masa prismei este 30,24 kg, afla i masa unui cilindru din acela i material nscris n prisma dat . 8. Un trapez isoscel are aria 85 cm2 i n l imea 5 cm. tiind c diferen a bazelor trapezului este 24 cm, afla i perimetrul s u. 9. Un paralelogram are laturile de 10 cm, respectiv 14 cm, iar m sura unghiului ascu it este 60 0. Afla i lungimile n l imilor paralelogramului. B. 10. Pre ul unui produs a fost redus de dou ori succesiv, prima oar cu 10% , iar a doua oar cu 30%, ajungnd n final de 69 300 lei. S se afle: a) pre ul ini ial al produsului; b) pre ul dup dou scumpiri succesive cu 10% i 30%, pornind de la pre ul final; c) pre ul n dolari la un curs al zilei de 32 000 lei, n toate cele trei variante.
(x y 1) 2 (x y )( x y y ) 63 2 y ( y x)

11. S se rezolve sistemul

2x y 7

2y x 9 4

12. Apotema bazei unei prisme triunghiulare regulate este 3 , iar diagonala unei fe e laterale este 10 S se calculeze: a) aria total a prismei; b) n l imea unui tetraedru regulat ce are o fa echivalent cu o baz a prismei; c) raportul dintre volumul tetraedrului astfel determinat i volumul prismei. Testul 5 A.

............ x xy 3 y 2 2. Dac (a, b)=6, [a, b]=336 i a< 10, atunci a+b=.............. 1 1 3. Dac x, y, z sunt invers propor ionale cu , i 3, iar 2x-y+3z=9, atunci y+z 5 8 reprezint ............% din x.
2

1. tiind c x i y sunt direct propor ionale cu 2 i 3, atunci

3x 2

xy 2 y 2

54

4. n triunghiul isoscel ABC, [AB] [AC], m ( BAC ) =360, BC=4, iar BD este bisectoare cu D (AC). Atunci AB CD= 5. Cercurile C1(O1 , 3) i C2(O2 , 2) sunt tangente exterior, iar tangentele lor comune exterioare se intersecteaz n P. Atunci PO2 =................ 6. Aria unui dreptunghi este 60 cm2, iar lungimea este cu 7cm mai mare dect l imea. Lungimea diagonalei dreptunghiului este ................. 7. Un triunghi dreptunghic are o catet de 12 cm, iar suma dintre cealalt catet i ipotenuz este 24 cm. a) Lungimea ipotenuzei este.............cm; b) Aria triunghiului este ............cm2. 1 1 1 1 3 1 3 2 5 2 5 8. Fie A= , B= . 2 1 3 2 4 3 2 3 2 3 3 5 3 5 a) Num rul A este un num r...........; b) Produsul A B este egal cu 2x m 9. Se d ecua ia mx= 6 , m R. 2 a) Dac ecua ia are solu ia x=3, atunci m=.............; b) Pentru m=2, ecua ia are solu ia x=.............. 1 1 y y 3 y 2 3y 9 x x 1 x 1 B. 10. Fie E1(x)= . : ; E2(y)= 1 : 1 1 2 y 3 y 3y 5 x 1 x 1 1 x2 a) Aduce i expresiile la forma cea mai simpl ; 2 E1 ( x) E 2 ( y ) 4 5 b) Rezolva i sistemul . 1 E 1 ( x) E 2 ( y) 1 2 11. M surile unghiurilor A, B, C ale triunghiului ABC sunt direct propor ionale cu numerele 6,5; 5 i 0,5, iar mediatorea laturii [AC] taie latura [BC] n E. Afla i m BAE .

12. Fie VABC o piramid triunghiular regulat cu n l imea de 6 2 i unghiul diedru dintre o fa lateral i planul bazei de m sur 45 0. a) Afla i aria total a piramidei; b) Determina i aria lateral i volumul trunchiului ob inut prin sec ionarea

piramidei printr-un plan paralel cu baza aflat la o distan de 2 2 cm fa de vrf; c) Afla i raportul dintre volumele piramidei i cel al unui con circular drept n care poate fi nscris aceasta.

55

Bacalaureat - teste preg titoare


Gabriel MR ANU 1
Testul 1 I. 1. a) Se consider func iile f : A B, g : B C unde A, B, C sunt submul imi ale lui R. S se arate c : (i) dac g f este injectiv , atunci f este injectiv ; (ii) dac g f este surjectiv , atunci g este surjectiv . b) Fie f : (0, ) (0, ) o func ie cu proprietatea (f S se arate c : (i) f este o func ie bijectiv ; (ii) f (x 2 ) f 2 (x ), x (0, ) ;
(iii)
f (x ) f ( x ),
0

f )(x )

x 2, x

(0, ) .

(0, ) .

2. Fie irul ( x n ) n

definit prin : x 0

a) S se studieze monotonia b) S se studieze monotonia c) S se arate c irul ( x n ) n


2

0, x n 1 6 xn . i m rginirea irului ( x 2n ) n 0 . i m rginirea irului ( x 2n 1 ) n 0 . 0 este convergent i s se calculeze lim x n .


n
2

3. Fie elipsa de ecua ie :

x y 1 0. a 2 b2 a) S se arate c dreapta y mx n este tangent elipsei dac

i numai dac

a 2m2 n 2 . b) S se determine locul geometric al punctelor din plan din care se pot duce tangente perpendiculare la elips . 1 1 1 1 a b c x II. 1. Se consider determinantul , unde a, b, c, x R. 2 2 2 a b c x2 a 4 b4 c 4 x 4 a) Dezvolta i determinantul dup coloana a patra punnd rezultatul sub forma unui polinom p(x). b) Determina i coeficientul dominant i r d cinile polinomului p(x). c) Scrie i polinomul sub form de produs. 2. Fie func ia continu f : [a, b] R. S se arate c pentru orice cuplu de numere reale pozitive , se poate determina un num r c [a, b] , astfel nct f (a ) f ( b) ( ) f (c) . n

Profesor, Liceul Teoretic Gr. Moisil , Ia i

56

III. Fie G

( k , k ) , unde k R, k>0. Pe G se define te o opera ie: (x, y)

x y

k2(x y) k2 x y

a) Ar ta i c (G,

) este grup abelian.


f :G

b) Ar ta i c func ia
grupurile (G, ) i (R, +).

, f ( x)

k 1 ln 2k k

x este un izomorfism ntre x

IV. a) Fie f : R R o func ie continu


a nT a

i periodic de perioad principal T. S se arate c


n
T 0

f ( x ) dx

f ( x ) dx ,

R i n N.

b) Folosind punctul a) , s se calculeze : 1 sin n arccos x lim dx . 0 1 x2 0 1 Testul 2 I. 1. Se d o mul ime A care are n elemente. mp r im toate submul imile lui A n clase (disjuncte), punnd n aceea i clas toate submul imile lui A care au acela i num r de elemente. Care dintre aceste clase este cea mai numeroas ? 2. Fie f : R \ {1, 2} R, f ( x )
1
2

x 3x 2 k a) S se determine determine derivata de ordin n a lui x. b) S se calculeze lim a n


n

i an

n2

1 (k ) f ( n 3) . 0 k!

3. Se consider punctele A(-2,1), B(2,-1), C(-1,8). a) S se calculeze perimetrul triunghiului ABC. b) S se determine coordonatele punctului D , al patrulea vrf al paralelogramului ABCD , pentru care BC este o diagonal . c) S se scrie ecua ia cercului circumscris triunghiului ABC. II. 1. Fie familia de func ii de gradul al doilea :

f m ( x ) (m 1) x 2 m x 1, , M R\{-1} a) S se arate c parabolele asociate acestor func ii trec prin dou puncte fixe. b) S se determine locul geometric al punctelor descris de vrfurile parabolelor.
2. S se determine o primitiv a func iei f : [0, 1]
4 3 2

R , f (x )

ln( 1 x

1 x ).

III. Fie polinomul f Z6[X] , f 4 x 2x 2x 4x 3 . a) S se arate c f nu admite r d cini n Z6. b) S se arate c f se poate descompune n produsul a dou polinoame , din care unul de gradul nti. x ln x 0

IV.

Fie func ia f : D

R , f ( x)

0 1

1 0

1 . ln x

a) S se determine domeniul maxim de defini ie D i apoi s se studieze continuitatea i derivabilitatea func iei f.
57

b) S se arate c restric ia lui f la (0, ) este bijectiv

i apoi s se calculeze

f 1 (0), f 1 (0) . c) S se determine primitivele lui f pe R*. d).S se calculeze aria suprafe ei plane determinate de graficul func iei f , axa Ox i dreptele x=1, x=3.

Testul 3 I. 1. S se rezolve inecua ia : 2 3 x

8
2

3 2x
x .
n

8 3 2

0.

2. Fie x R \ {1} i S ( x ) x x a) S se calculeze S(x) i S (x ) . b) Folosind a) s se calculeze T ( x )

x 2x2

3x 3
n

nx n .

c) S se calculeze limita irului (a n ) n 1 cu a n

k
k

k 12

II. 1. Fie M mul imea matricilor de tipul (m, n) n care toate elementele sunt egale cu 1 sau (1) i astfel nct produsul numerelor din fiecare linie, respectiv coloan s fie egal cu (1). S se calculeze num rul de elemente al mul imii M . 2. a) Enun a i teorema lui Lagrange. b) Aplica i teorema lui Lagrange func iei f : [e, ] R, f ( x ) ln x . c) Demonstra i inegalitatea : e III. 1. Fie G
f : (b, ) R f ( x)
e

.
R*, b 0.

b ( x b) ,

a) S se arate c (G , ) este grup abelian. b) S se demonstreze c (G , ) ( R, ) . 2. Fie P Z[X] un polinom pentru care P(0), P(1) sunt impare. S se arate c ecua ia P(x)=0 nu are r d cini ntregi. IV. 1. Fie func ia f : I
n

R , I interval inclus n R a I i func ia f este derivabil n a.


1 ) n f (a 2 ) n f (a
1 0

S se calculeze lim n f ( a
2. Fie irul ( a n ) n
1

k ) kf ( a ) , unde k N* este fixat. n x (1 x ) n dx , n N*.

cu termenul general a n
n

a) S se determine an. b) S se calculeze bn c) S se calculeze lim bn .


n

ak .
k 1

Testul 4 I. 1. Fie x1 , x 2 , , x n , un ir de numere reale nenule. S se arate c acest ir este o progresie aritmetic , dac i numai dac pentru orice n 2 , avem rela ia :

58

1 x1 x 2

1 x 2 x3

1 xn 1 xn

n 1 . x1 x n

2. Se consider

irul ( x n ) n 0 , x n n 2n arctgn . a) S se arate c ( x n ) n 0 este un ir monoton i m rginit. b) S se calculeze lim x n .


n

3. S se demonstreze c produsul distan elor unui punct oarecare al hiperbolei


x2 a2 y2 b2 1 0 , la cele dou asimptote , este egal cu a 2b2 a2 b2

II. 1. Determina i p r ile stabile finite ale lui Z n raport cu nmul irea. Este R\Q parte stabil a lui R n raport cu adunarea , respectiv cu nmul irea ? x 1 t 2. Fie func ia f : [0, x ] R , f ( x ) ln dt , x (0, 1) . 0 1 t a) S se calculeze f ( x ) . f ( x) b) S se calculeze lim . x 0 arcsin x 2 III. Fie (G , )

i (G, ) dou grupuri, f : G

un morfism de grupuri i mul imea

Ker f

{x

G f ( x)

e } , e fiind elementul neutru al lui G .

a) S se arate c Kerf este subgrup al lui G. b) S se arate c morfismul f este injectiv dac

i numai dac Kerf={e}.


1 , 2 1 x {1, , 2
1 0

x 3 , x [0,1] \ {1,

IV. 1. Fie func ia g : [0, 1]

R , g ( x)

3x ,

1 } n n N*. 1 } n

S se arate c g este integrabil pe [0, 1] i s se calculeze


2. Se consider func ia f : (0, ) R, f ( x )

g ( x )dx .

p ln x , n N*. x x S se calculeze S(m) , aria suprafe ei m rginit de graficul func iei f, axa Ox, i dreptele x=1 , x=m (m >1). S se determine m, astfel nct S ( m ) p ln m 1 / 2 .

Testul 5 I. 1. Fie irul de numere complexe ( z n ) n 1 definit prin z1 i, z2 i, zn zn 1 q (zn 1 zn 2) pentru n 2 , q fiind un num r complex dat diferit de 1. a) Calcula i z3 i z4. b) Ar ta i c
zn zn 1 2 iq n 2

c) Ar ta i c afirma iile z n echivalente.

. 1 q i i q este r d cin de ordinul n-1 a unit ii sunt

i deduce i c

zn

z1

2i

1 qn 1

59

2. a) Defini i no iunea de punct de inflexiune al unei func ii i da i interpretarea geometric . b) S se determine punctele de inflexiune ale func iei : f : R R, f (x) 3. Fie elipsa (E) :
3

x 1 3 x 1.

x2 y2 1 0. 4 3 a) S se scrie ecua iile tangentelor la elips , duse din punctul A(-16, 9). b) Dac se noteaz cu T1, T2 punctele de contact ale celor dou tangente de la punctul a) cu elipsa (E) , s se determine ecua ia dreptei T1T2. c) Calcula i distan a de la punctul A la dreapta T1T2.

II.

1. a) Rezolva i n R inecuatia

1 x2 x

3 . 4 6 7 1 7
2002

b) S se g seasc rangul celui mai mare termen din dezvoltarea 2. a) Ar ta i c func ia f : R R, f ( x )

1, x Q nu admite primitive pe R. 0, x R \ Q

b) G si i o func ie g : R R care nu admite primitive pe R , astfel nct func ia compus f g s admit primitive pe R (f fiind func ia de la punctul a)). III. 1.Ar ta i c polinomul f x 2 3 x 3 divide polinomul, g ( x 1)3n 2. Fie a , b Zn, i ecua ia a x b . Ar ta i c : a) dac (a, n)=1, atunci ecua ia a x b admite solu ie unic .
b) dac (a, n)=d > 1 i d nu divide pe b, atunci ecua ia a x
2

x 2, n N*.

b nu are solu ii.

IV. 1. Fie func ia f : [0, )

R , f ( x)

x ln x 0,

x,

(0, ) x 0

a) Studia i continuitatea i derivabilitatea lui f pe [0, ) b) Studiati variatia func iei f i reprezenta i graficul ei. c) Ar tati c restric ia lui f la [e, ) are invers , f- 1, i studia i derivabilitatea lui f- 1 . d) Calcula i f ( e 3 ) i ( f 1 ) ( 2e 2 ) . e) Calculati aria suprafe ei plane m rginit de graficul func iei f , axa Ox, i dreptele
x e, x e2 .

2. Fie f : (0, ) a) S se arate c b) S se arate c

R, f ( x ) e x 1 x 2 f ( x ) 0, x 0. f ( x ) 0, x 0.
1 x2 0

x4 .

c) S se demonstreze c :

e dx

1,43 .

60

PROBLEME SI SOLUTII
Solu iile problemelor propuse n nr. 1/2001
Clasele primare P.7. Crizantema are cu 38 timbre mai pu in dect colega ei, Maria. Cte timbre trebuie s mai cumpere Crizantema pentru a avea cu cel mult 4 timbre n plus fa de Maria? Crizantema Mironeanu, elev , Ia i Solu ie. Pentru a o egala pe Maria, Crizantema mai are nevoie de 38 timbre. Pentru a o dep i cu cel mult 4 timbre, ea trebuie s mai cumpere, pe lng cele 38, nc 1,2,3 sau 4 timbre. Deci Crizantema trebuie s mai cumpere 39,40,41 sau 42 timbre. P.8. S se arate c din numerele 1,2,3,,10 nu se pot forma dou iruri de numere, cu acela i num r de numere, astfel nct adunnd numerele din fiecare ir s ob inem sume egale. Maria Mursa, elev , Ia i Solu ie. Suma numerelor 1,2,3,,10 este 55. Dac ar exista cele dou iruri de numere cu sume egale, atunci cele dou sume adunate ar trebui s dea 55. Acest lucru este imposibil deoarece nu exist dou numere naturale egale care s dea suma 55. P.9. Afla i vrsta n prezent a tat lui unui b iat tiind c b iatul are 7 ani, iar atunci cnd b iatul va avea vrsta tat lui, tat l va avea 55 ani. nv. Elena Marchitan, Ia i Solu ie. S figur m cele dou vrste innd cont de rela iile dintre ele. Not m cu t vrsta tat lui i cu f vrsta fiului. 7 Se observ c vrsta de 55 de ani este format din f vrsta fiului i din dublul diferen ei dintre vrsta tat lui t i a fiului n prezent. 55 ani 1. Care este dublul diferen ei dintre cele dou vrste n prezent? 55 7 = 48 2. Care este diferen a dintre cele dou vrste n prezent? 48 : 2 = 24 3. Care este vrsta tat lui n prezent? 24 + 7 = 31 R : 31 P. 10. George i-a propus s citeasc n cinci zile o carte ce are 42 file. Num rul filelor citite n primele trei zile este reprezentat de numere pare consecutive. n a patra i a cincea zi a citit 12 file. tiind c n ultima zi a citit de dou ori mai mult dect n ziua precedent , s se afle cte file a citit George n fiecare zi. nv. Geta Dragnea, Ia i Solu ie. S figur m num rul de file citite de George n fiecare zi I zi 2 a II-a zi (42-12) file 2 a III-a zi a IV-a zi 12 file a V-a zi 1. Cte file a citit n primele trei zile? 42 12 = 30 61

2. Care este triplul num rului de file citit n prima zi? 30 6 = 24 3. Cte file a citit n prima zi? 24 : 3 = 8 4. Cte file a citit n a doua zi? 8 + 2 = 10 5. Cte file a citit n a treia zi? 10 + 2 = 12 6. Cte file a citit n a patra zi? 12 : 3 = 4 7. Cte file a citit n a cincea zi? 4 . 2 =8 R : 8 file, 10 file, 12 file, 4file, 8 file. P. 11. C i spectatori au fost asear la Teatrul Na ional Vasile Alexandri, din Ia i, dac la balcon au fost 160 de spectatori, la loj un sfert din restul spectatorilor, iar la stal cu 80 spectatori mai mult dect la loj i balcon mpreun ? nv. Rodica Agrici, Ia i Solu ie. S figur m reparti ia spectatorilor, n fiind num rul total. n balcon loj loj 80+160 80 + 160 = 240 240 . 2 = 480 480 + 160 = 640 R : 640 spectatori P.12. Mo Cr ciun mparte daruri elevilor clasei a IV-a. Dac ar da fiec rui copil cte 2 pachete, ar r mne n sac 2 pachete. Dac ar oferi fiec rui copil cte 3 pachete, ar r mne 9 copii f r daruri. Cte pachete are Mo Cr ciun n sac? nv. F nic Dragnea, Ia i Solu ie. Figur m cele dou situa ii din problem folosind simbolurile E (elev) i P (pachet) Primul rnd de simboluri sugereaz fiecare elev a primit cte 2 pachete 2 pachete au r mas nerepartizate. PP PP PP PP PP Al doilea rnd de simboluri sugereaz E E ...............E E ......... E , PP c primii elevi au primit cte 3 pachete iar 9 elevi ultimii 9 elevi nu au nici un pachet. Putem PP PP PP PP E ......... E E E ......... E PP PP PP PP E considera c al treilea pachet a fost oferit P P P .......... ......... PP ..... 9 elevi , ultimilor 9 elevi. Acum putem scrie: 1.C i elevi au primit cte 3 pachete? 2 . 9 + 2 . 1 = 20 9elevi 2.Cte pachete are Mo Cr ciun? 20 . 3 = 60 R : 60 pachete P.13. Scriitorul Ion Creang a publicat povestea Capra cu trei iezi n 1875. Se spune c pe atunci capra ar fi avut o vrst egal cu dublul sumei vrstelor iezi orilor ei, anii acestora fiind exprima i prin numere naturale consecutive. Peste un an, cnd s-a ab tut necazul asupra caprei, lupul avea vrsta egal cu dublul sumei vrstelor de atunci ale iezilor, iar to i cinci aveau mpreun 40 ani. Ce vrst avea fiecare n anul public rii acestei pove ti? nv. Mihai Agrici, Ia i Solu ie. Figur m datele corespunz toare anilor 1875 i 1876. 1. Ct reprezint jum tate din restul spectatorilor? 2. C i spectatori au stat la loj i stal? 3. C i spectatori au fost la teatru? restul spectatorilor stal

EE

EE E

I1 I2 I3 C

1 1 1 1 1 1 1 1

de 9 ori segmentul I1 i nc 9 ani

62

1 I1+1 1 I2+1 1 40 ani I3+1 1 1 1 1 1 1 1 C+1 1 1 1 1 1 1 1 1 1 1 1 1 L+1 Unde I1, I2, I3, C, L reprezint vrstele celor cinci vie uitoare n anul 1875. Analiznd figurarea corespunz toare anului 1876, putem scrie: 1. Care este num rul segmentelor ce reprezint vrsta I1? 3 . 1+ 2 . 6 = 15 2. C i ani reprezint aceste segmente? 40 25 = 15 3. Ce vrst avea mezinul? 15 : 15 = 1 4. Ce vrst avea iedul mijlociu? 1+1=2 5. Ce vrst avea iedul cel mare? 2+1=3 6. Ce vrst avea capra? (1+2+3) . 2 = 12 7. Ce vrst avea lupul? (2+3+4) . 2 1 = 17 R : 1an, 2ani, 3ani, 12ani, 17ani. Clasa a V-a V.16. Un automobilist vede la un moment dat pe kilometrajul de la bord num rul 12921. Dup dou ore de mers cu vitez constant , pe kilometraj a ap rut urm torul num r care se cite te la fel n ambele sensuri. Afla i viteza de deplasare a automobilului. Gabriel Mr anu, Ia i Solu ie. Urm torul num r care se cite te la fel n ambele sensuri este 13031. n dou ore de mers automobilistul a parcurs cu vitez constant distan a de 110 km = 13031 km 12921 km, deci viteza sa a fost de 55 km/h.

V.17. S se arate c fiecare termen al irului: 19204, 9012004, 900120004, ... este un p trat perfect. Constantin Chiril , Ia i Solu ia 1. Se observ c 9120 = 3022, 9012004 = 30022 etc. n general, se pare c 900.....01200.....04 300.....02 2 . Se verific prin ridicare la p trat c egalitatea este
p p 1 p 1

adev rat ; ca urmare, numerele din ir sunt p trate perfecte. Solu ia 2 (Schibinschi Greta, Boto ani). Scriem: 90.....0120.....04 9 10 2 p 4 12 10 p 2 4 9 10 2 p 4 6 10 p
p p 1

6 10 p
2

4 30.....02 2 .
p 1

3 10 p

(3 10 p

2) 2(3 10 p

2)

(3 10 p

2) 2

V.18. Ar ta i c , dac suma a n numere naturale nenule este un num r prim, atunci aceste numere sunt prime ntre ele. Cristiana Artenie, elev , Ia i Solu ie. Fie p = a1 +a2 +...+an, p num r prim i d = (a1, a2, ... , an). Atunci d |a1, d | a 2, , d | an i deci d | a1+a2 + ... + an, adic d | p. Ca urmare sau d = 1, sau d = p; ar t m c nu putem avea d = p i de aici va rezulta concluzia. ntr-adev r, dac d = p, atunci numerele de i vom avea pa1 pa 2 ... pa n p sau date se scriu: a1 pa1 ,..., a n pa n a1 a 2 ... a n 1 , ceea ce nu se poate! 63

V.19. Fie a, b, p, q N*. Afla i valorile pe care le poate lua num rul (1+A)B, unde A a b a b a b ... i B b a b a b a ...
p termeni q termeni

Cristiana Constanda, elev , Ia i Solu ie. Observ m c num rul A are valorile 0, a, a + b sau b, dup cum p = M4, p = M4 + 1, respectiv p = M4 + 3. Ca urmare, 1+A {1,1+a,1+a+b, 1+b}. n mod analog, B {0, b, a+b, a}. n consecin , avem: (1+A)B {1, (1+a)a, (1+b)b, (1+a)a+b, (1+b)a, (1+b)b, (1+b)a+b, (1+a+b)0, (1+a+b)b, (1+a+b)a+b}. V.20. S se aranjeze 12 puncte pe 6 drepte astfel nct pe fiecare dreapt s fie situate 4 puncte (indica i cel pu in dou aranjamente de acest fel). Andrea Balla, elev , Bra ov Solu ie. Reproducem trei dintre numeroasele solu ii primite de redac ie.

Clasa a VI-a VI.16. Fie a i b dou numere ntregi. Ar ta i echivalen a afirma iilor urm toare: 1 1000a+b 43; 2 a+4b 43; 3 11b-8a 43; 4 7b-9a 43. Gheorghe Costovici, Ia i Solu ie. Vom ar ta c 1 2 3 4 1 . Avem: 1 1000a+b = 43k (2343+11)a+b = 43k 2343a+11a+b = 43k 42343a+44a+4b = 443k 42343a+43a+(a+4b) = 443k a+4b 43; 2 a+4b = 43k -8a32b = -843k (11b8a) 43b = -843k 11b-8a 43; 3 11b8a = 43k 99b72a = 943k 43b+56b72a = 943k 43b+8(7b 9a) = 943k 7b9a 43; 4 7b9a = 43k 1357b1359a = 13543k 945b1215a = 13543k 946bb 1000a 215a = 13543k 43(22b5a)(1000a+b) =13543k 1000a+b 43. VI.17. Fie E = 24n+2 + 34n + 44n + 52n + 62n, n 1) Ar ta i c E nu este p trat perfect. 2) Afla i n astfel nct E 9. N.

Cristiana Constanda, elev , Ia i Solu ie. 1) Dac n = 0, atunci E = 8 i nu este p trat perfect. Dac n 1, atunci ultima cifr a num rului E este ultima cifr a sumei 4+1+6+5+6 = 22, deci E se termin n 2 i de aici rezult c nu poate fi p trat perfect. 2) Dac n = 0, E = 8 nu se divide prin 9. Dac n 1, atunci 62n 9 i deci avem:

64

24n+2 +44n+52n 9 2n+223n + 162n + (9-4)2n 9 E 9 n n+2 2n n n (-1) 2 +2 +2 (9-1) 9 (-1)n2n4+2n2n+(-1)n2n

2n+2(9-1)n + 22n(9-1)2n + 42n 9 9 5(-1)n+2n 9.

(5+26k) 9 5+(9-1)2k 9 (5+1) 9, Dac n = 6k (k 1), atunci 5(-1)n+2n 9 fapt care nu este adev rat. La fel se arat c E nu se divide cu 9 dac n = 6k+1, n = 6k+3 sau n = 6k+4 (k 0). Dac n = 6k+2 sau n = 6k+5 (k 0), atunci E se divide cu 9, deoarece: 5+4(9-1)2k 9 (5+4) 9 etc. 5 (-1)6k+1+26k+2 9 V.18. S se descompun n factori primi num rul S dat de: S = 123456789 + 234567891 + 345678912 + ... +12345678. Paraschiva Brsan, Ia i Solu ie. Scriind reprezentarea n baza 10 al fiec rui termen din suma S, ob inem : S = (1+2+ ... +9)108 + (1+2+ ... +9)107 + ... + (1+2+ ... +9) = =45(108+107+ ... +10 + 1) = 45[106(102+10+1) + 103(102+10+1) + (102+10+1)]= = 45(102+10+1)(106+103+1) = 3251111001001 = 34537333667, aceasta fiind descompunerea n factori primi a num rului S. VI.19. S se afle cinci numere ra ionale tiind c :a) suma lor este 351; b) primele trei sunt invers propor ionale cu primele trei numere prime; c) ultimele trei sunt direct propor ionale cu 7, 11 i 13. Cristiana Artenie, elev , Ia i. Solu ie. Fie a, b, c, d i e cele cinci numere. Condi iile din enun se scriu: 5c c d e a+b+c+d+e = 351, 2a = 3b = 5c i . Din ultimele rela ii deducem c a , 7 11 13 2 5c 11c 13c b , d , e i nlocuind n prima egalitate ob inem o ecua ie cu necunoscuta 3 7 7 c etc. VI.20. Fie a, b N i c Q direct propor ionale cu p1, p2, p3, unde p1 < p2 < p3 sunt numere prime. a) Ar ta i c c N*. b) Determina i p1, p2, p3 dac a+b < 35 = c. Gheorghe Iurea, Ia i a b c a deducem ap2 = bp1 i c p 3 . Din prima egalitate Solu ie. a) Din p1 p 2 p3 p1

rezult
N*.

p1 | ap2 i deci p1 | a, adic

a p1

N*. Folosind a doua egalitate, ob inem c

b) Avem: a+b < 35 = c k (p1+ p2) < 35 = kp3 35 = kp3 i p1+ p2 < p3 p3 {5, 7} i p1+p2 < p3. Dac p3 = 3, atunci p1 = 2 i p2 = 3 (deoarece p1 < p2 < p3) i nu avem p1+p2 < p3. R mne p3=7, caz n care inegalitatea p1+p2 < p3 este ndeplinit numai de numerele prime p1 = 2, p2 = 3.
Clasa a VII-a VII.16. S se cerceteze care dintre elementele mul imii A={(x,y) Q Q; 4x2+12x+9 + 2 |y -25|=0} apar in graficului func iei f(x) = -2x+2, x R. 65

Cristiana Constanda, elev , Ia i Solu ie. Avem: A ={(x,y) Q Q; (2x+3)2+|y2-25| = 0} = {(x,y) Q Q; 2x+3 = 0 i y2-

25 = 0} = {(x,y) Q Q; x =

3 2

i y = 5 }=

(-

3 2

, - 5), (-

3 2

, 5) . Cum f(

3 2

) = 5, numai

al doilea punct al lui A apar ine graficului lui f.


[2, + ). Ar ta i c (x2+y)(y2+z)(z2+x) 27xyz. [3, + ). Ar ta i c (x2+y)(y2+z)(z2+x) 64xyz. Lucian Tu escu, Craiova Mai general, pentru orice n N fixat are loc inegalitatea: (x2+y)(y2+z)(z2+x) (n+1)3xyz, x, y, z [n, + ). (1) (pentru n = 2 i n = 3 se ob in inegalit ile din enun ). Solu ia 1 (n maniera autorului). Cum x n x2 nx x2+y nx+y i cum nx+y = x++x+y (n+1) n 1 x n y , ob inem x2+y (n+1) n 1 x n y . Analog, y2+z

VII.17. a) Fie x, y, z b) Fie x, y, z

(n+1) n 1 y n z , yn+x (n+1) n 1 z n x . Prin nmul irea membru cu membru a ultimelor trei inegalit i se ob ine (1). Solu ia 2 (n maniera mai multor elevi din Bra ov). Are loc x2+y (n+1)x, x, y [n, + ) c ci x2 + y (n+1)x x2 (n+1)x + n + (y-n) 0 (x-1)(x-n) + (y-n) 0. nmul ind membru cu membru inegalitatea x2 + y (n+1)x cu analoagele ei, ob inem (1).
VII.18.
2 2

S
2

se

determine
2

numerele
2 2

reale
x
2

x
2

pentru

expresia

x y ( x a ) ( y a ) + ( x b) ( y a) i s se afle apoi aceast valoare minim .

( y c) (a,b,c

R+) este minim

Cristiana Artenie, elev , Ia i Solu ie. n raport cu un sistem Oxz de axe coordonate figur m punctele O(0,0),
A(-a,a), B(b,a), C(0,c) i M(x,y). Deoarece
( x b) 2 ( y a ) 2 =MB i x2 ( y c) 2 =MC, x2 y 2 = MO, ( x a) 2 ( y a ) 2 = MA,

problema revine la determinarea minimului expresiei E = MO+MA+MB+MC, atunci cnd M este un punct oarecare din plan. OC Avem inegalit ile: MO + MC ( MOC), MA+MB AB ( MAB), deci E AB+OC, cu egalitate pentru M (OC) (AB), adic M coincide cu P(0,a). Ca urmare, Emin se ob ine pentru x = 0, y = a i are valoarea Emin = 2a+b+|a c|.
VII.19. Se consider un triunghi dreptunghic isoscel ABC cu vrful n A i se noteaz
cu E i F punctele de intersec ie ale cercurilor C(C, punctele B, E i F sunt coliniare.
3 1 BC) i C(A, BC). S se arate c 4 4

66

Adrian Zanoschi, Ia i

Solu ie. Fie AB = AC = a. Avem: AE = =AF 3BC 3a 2 BC a 2 = = i CE = CF = = . Se 4 4 4 4 observ de aici c 2a 2 18a 2 AE2 + AC2 = =EC2, deci AEC a2 = 16 16 este dreptunghic n A, adic AE AC , de unde rezult c E AB. Analog se arat c F AB, deci B, E, F sunt coliniare. VII.20. S se mpart cu ajutorul unui echer negradat un segment [AB] n trei p r i de lungimi egale. Constatin Cocea, Ia i Solu ie. Vezi E. Cohal - Construc ii geometrice cu echerul, p.41 din acest num r. Clasa a VIII-a VIII.16. Fie n N* i An= x1 x 2 ...x n . Definim func ia d: An An N prin d(x, y) =
Card i / i 1, n , x i y i , unde x = x1 x 2 ...x n i y = y1 y 2 ... y n . S se arate c d este o distan , adic satisface condi iile: 1) d(x, y) = 0 x = y; 2) d(x,y) = d(y,x), x,y An; 3) d(x,y) d(x,z)+d(z,y), x,y,z An. Petru Asaftei, Ia i Solu ie. Verific m pe rnd 1), 2) i 3): 1. d(x, y) = 0 x1 = y1, x2 = y2, ... , xn = yn x = y. 2. d(x, y) = d(y, x), x,y An, c ci perechile de numere (x, y) i (y, x) au acelea i cifre distincte. 3. S presupunem c d(x, y) = k = Card{i1, i2, , ik}(deci xi1 y i1 , , xin y in i xj = yj pentru j {i1, ..., ik}) i fie z = z1 , z 2 , ... , z n . Fie p num rul indicilor din {i1, ..., ik} pentru care coincid cifrele lui z i x i fie q num rul acelora pentru care coincid cifrele lui z i y. z i1 x i1 etc., rezult c Evident, avem p+q k. Deoarece z i1 xi1 z i1 y i1 i z i1 y i1 d(x, z) k-q i d(z, y) verificat 3). k-p. Ca urmare, d(x, z) + d(z, y) (0,1), tiind c num rul real

2k- (p+q)

k = d(x, y), adic este

VIII.17. Ar ta i c 2x5 + x3 1= 0. Solu ia 1. Avem: 2


5

este o solu ie a ecua iei

Dumitru Neagu, Ia i

1=0

(2 +1) = 1

=
2

1
2

0<
1

<1

0<

< 1 (s-a utilizat faptul c 0, zero nefiind solu ie a ecua iei date). Solu ia 2. Dac x 0, atunci 2x5+x31 -1 < 0, deci x 0 nu este solu ie a ecua iei 67

date. Dac x 1, atunci 2x5 +x3 1 2 + 1 1 > 0, deci x Cum num rul este solu ie a acestei ecua ii, rezult c
VIII.18. Dac x,y
x+ y [0,1], atunci avem:
1 1 x2 1 1 y2 1

1 nu este solu ie a ecua iei date. (0,1).

1 x2 1

1 y2 .

Constantin Cocea, Ia i Solu ie. Ridicnd inegalitatea dat la p trat (avem voie!), ob inem: x2+y2+2xy

(1+ 1 y 2 - 1 x 2 - (1 x 2 )(1 y 2 ) ) + (1- 1 y 2 + 1 x 2 - (1 x 2 )(1 y 2 ) )+2xy, echivalent cu (1-x2) + (1-y2)


VIII.19. S
x N/ x
2

2 (1 x 2 )(1 y 2 ) , adev rat conform inegalit ii mediilor.


2 tiind c mul imea

se determine numerele naturale n


N

x 1 n

{1,2,..., n} este format dintr-un singur element.

Cristinel Mortici, Constan a Solu ie. Fie n 2 un num r cu proprietatea dorit i fie x {1, 2, , n} unicul num r pentru care n | x2+x+1. Cum n+1-x {1, 2, , n} i n | (n+1-x)2 (n +1-x)-1 (dup cum se vede din egalitatea (n+1-x)2 (n+1-x)-1= n(n-2x+1) + (x2x+1), din unicitatea lui x rezult n 1 i atunci n este impar: n = 2k+1, x = k+1. n consecin , n | x2 c n+1-x=x, deci x = 2 2k + 1 | k2+k1 2k + 1 | 4k2+4k4 2k+1 | (2k+1)2 5 x1 2k+1 | (k+1)2 (k+1)1 2k + 1 | 5 k {1, 5}. Se verific direct c numai n = 5 satisface cerin ele problemei. VIII.20. Fie cubul ABCD A B C D tangenta unghiului dintre B O i A D .
cu O centrul fe ei A B C D . Calcula i

Cristiana Constanda, elev , Ia i Solu ie. Dublnd cubul ca n figur se ob ine paralelipipedul EFCDE F C D , n care F B A D (ambele paralele cu E A ). Observ m c <( A D , O B ) = <( F B , O B ) =
=< F B O = . = O ' B '2 B ' B 2 Pentru
a 6 2

laturile

triunghiului

F BO

avem:

F B =a 2 ,

OB =

i O F = O 'P 2

F 'P 2 F BO ,

a 10 (unde p este mijlocul 2

muchiei B C ) . Aplicnd teorema cosinusului n ob inem: unde = 1


10a 4
2

= 2a2 + =
3 . 6

6a a 6 -2 a 2 cos , de 4 2

cos
3 36

Ca

urmare,
11 .

sin

33 i deci tg = 6

68

Clasa a IX-a IX.16. n ipoteza c ecua ia cu coeficien i reali x 3 ax 2 bx c 0 are r d cinile x1 , x 2 , x 3 reale, s se demonstreze echivalen a: x i 0 , i 1, 2, 3 , a 0 , b 0 , c 0 , ab c . Adrian Corduneanu, Ia i Solu ie. Conform rela iilor lui Vite, avem: x1 x 2 x 3 a , x1 x 2 x 2 x 3
x 3 x1
2

b , x1 x 2 x 3

(1)

b)( x a) ab c (2). Ecua ia din enun se poate scrie i sub forma ( x Dac x i 0 , i 1, 2, 3 , atunci din (1) rezult c a 0 , b 0 i c 0 . Tot din (1) se (x 2 x 3 ) 0 . De aici i din egalitatea ob inut punnd x1 n poate deduce c x1 a locul lui x n (2) deducem c ab c 0 . Reciproc, s presupunem c sunt ndeplinite condi iile din enun privind coeficien ii. Din (1) avem x1 x 2 x 3 c 0 i atunci nu toate numerele x1 , x 2 , x 3 pot fi negative. Fie
x1 0 ; rezult c are loc i x 2 x 3 0 (3). Pe de alt parte, din ab c i din (2) cu x 1 n (x 2 x 3 ) , deci x 2 x 3 0 (4). Din locul lui x , se ob ine c x1 a 0 . ns x1 a (3) i (4) decurge c x 2 0 i x 3 0 .

IX.17. Ar ta i c ecua ia {x 2 } { y 2 } {z 2 } are o infinitate de solu ii n Q \ Z ({a} reprezint partea frac ionar a num rului real a). R. B rbulescu i M.B. Ion, elevi, Lucian Tu escu, prof. , Craiova Solu ie (dat de un grup de elevi din Bra ov). Scriem egalitatea 3 2 4 2 5 2 sub

forma:

3 7n

4 7n
3 7n

5 7n
2

, n N* .
3 7n
2

Deoarece:

,
, 4 7
n

4 7n

4 7n

5 7n

5 7n

,
n

N* , N* .

urmeaz c tripleta x, y, z =

3 7
n

5 7n

este o solu ie a ecua iei din enun

IX.18. Determina i func iile

f: R

R pentru care avem: f(x 3 + 3x 2 + 3x) x

f 3 (x) + 3f 2 (x) + 3f(x) x

R. Generalizare.

Gabriel Popa, Ia i Solu ie. Vom rezolva direct urm toarea problem mai general : Dac : R R este o func ie surjectiv i strict cresc toare, s se determine func iile x f x , x R. (1) f: R R pentru care: f( x

Din ipotez , este inversabil de monotonia lui , au loc:

i avem

t Atunci, innd seama i

69

(1)

f x

x, x f x , x

R R

f t
1

t,

t x

R R

f x,

f x

x
3

f x,

n cazul particular considerat, cresc toare i surjectiv , deci f(x) =

x
1

x3
3

x2

x
x

x
R.

i este evident strict

(x)= x 1 1 ,

IX.19. Ar ta i c n orice triunghi are loc inegalitatea: (a8 +b8 + c8 +3) R4 Solu ie. Se tie c n orice triunghi au loc R pentru triunghiul echilateral. Atunci:
a8 b8 c8 3 R4
b4

8r2a2b2c2.

Mihai Bogdan Ion, elev, Craiova 2r i a 4 b 4 c 4 16S 2 , cu egalitate


a8 b8 c8 R 4

a8 1 b8 1 2 2
2 16 S 2 R 4

c8 1 4 R 2
2 a 2b 2c 2

R 2 4 r 2 8r 2 a 2 b 2 c 2 . R2 IX.20. Pe laturile AB i AC ale tringhiului ABC se consider punctele M, respectiv N, astfel nct BM CN = k (constant). Dac B i C sunt fixe i m A este constant , s se

2 a4

c4 R4

afle locul geometric al mijlocului segmentului MN.


C t lin Calistru, Ia i Solu ie. Fie P, Q, R, S respectiv mijloacele segmentelor [MN], [BC], [BN], [CM]. Laturile patrulaterului PRQS sunt linii mijlocii n triunghiurile NMB, BNC, CMB, respectiv MNC i cum BM CN =k, rezult c PRQS este romb de latur k . n plus,
2

unghiurile acestui romb au m suri constante ( , respctiv - ), deci indiferent de pozi ia punctului A, diagonala PQ a rombului are lungime constant

QP

k cos

. Cum Q este fix, rezult c P descrie

2
r k cos

, delimitat de dreptele CA1 i CA2 2 corespunz toare pozi iilor limit ale punctului A pe arcul capabil de unghi (BA k, CA k). Dac A descrie arcul simetric din semiplanul inferior, P parcurge un alt arc de cerc, simetricul primului fa de BC. un arc de cerc de centru Q i raz
Clasa a X-a X.16. S se determine num rul func iilor f : 1,2,
n

1,1 cu proprietatea c

f(i) = k, k, n N , n
i 1

2, k < n.

70

Solu ie. Fie X

Petru Asaftei, Ia i {1,2,...,n} mul imea acelor elemente a c ror imagine prin f este 1.
n

Evident c X
n

, altfel
i 1

f i

k . Not m x

card X . Atunci n k . 2

k
i 1

f i
i X

f (i )
j X

f ( j)

n x

2x

n k

Dac n i k au parit i diferite, nu exist func ii ca n enun . Pentru n i k de aceea i paritate, func iile c utate sunt bine determinate de submul imea X; num rul lor va fi deci
n k

egal cu num rul submul imilor cu n k elemente ale mul imii 1,2,...,n , adic
2

Cn2 .

X.17. Fie f : R

0,

definit prin : a,b N\{0,1}, a b.


Ma,b |
bx, x R \Q

x f(x) = a , x Q

Not m cu Ma,b mul imea acestor func ii i cu I = { f f surjectiv }.

Ma,b | f injectiv }, S ={ f

1) I ,S i I = S. 2) dac (a, b) = 1, atunci f nu-i nici injectiv i nici surjectiv . 3) este adevarat reciproca afirma iei de la punctual 2)? Dumitru Gherman, Pa cani

Solu ie. Vom demonstra c f surjectiv f injectiv r Q * astfel nct b a r . a) f surjectiv f injectiv . ntr-adev r, s presupunem prin absurd c f nu este injectiv ; atunci x1 Q, i x 2 R\Q astfel nct f(x1) = f(x2), de unde a x1 b x2 , adic x1 x 2 log a b , deci log a b R\Q, prin urmare ecua ia f x b nu are solu ie, ceea ce contrazice faptul c f este surjectiv . r Q* astfel nct b a r . Vom proceda tot prin reducere la b) f injectiv i, ca urmare, absurd; presupunem c b a r , r Q*. Atunci log b a Q f log b a b logb a
*

f 1 . Cum f injectiv , urmeaz c


a
r

log b a 1 , imposibil.
0,

c) r Q astfel nct b
f(x) y . Dac

f surjectiv . Pentru y

, consider m ecua ia

log a y

Q, atunci x1

log a y este solu ie a acestei ecua ii; n caz

1 log a y R \ Q este solu ie a ecua iei. r Acum cerin ele problemei sunt imediate. Pentru a=2, b=4, avem f I iar I=S conform primei echivalen e dovedite. Punctul 2) rezult din ultima echivalen , iar 1. reciproca sa este fals : de exemplu, f I pentru a=2 i b=6, ns ( 2,6 )

contrar, x

71

X.18. Fie ABCD un patrulater convex, orientat pozitiv, ale c rui vrfuri au afixele a, b, c, d .S se arate c ABCD este p trat dac i numai dac d a i (b a) i a+c=b+d. Adrian Corduneanu, Ia i Solu ie. Prin transla ia z z - a , ob inem patrulaterul A B C D congruent cu ABCD, unde A 0 , B b a , C c a , D d a . Avem: ABCD p trat ABCD
p trat
A B C D paralelogram i OD se ob ine din OB printr-o rota ie de unghi

2
sens direct

c a

(b a) (d a). i d a

i(b a)

a+c=b+d i. d a

i(b a) .

X.19. Fie x, y, z, a, b, c C. Not m = xa + yb + zc, = xb + yc + za i = xc + ya + zb. S se arate c dac numerele complexe , , sunt afixele vrfurilor unui triunghi echilateral, atunci cel pu in unul din tripletele (x, y, z) i (a, b, c) reprezint afixele vrfurilor unui triunghi echilateral. Constantin Cocea, Ia i Solu ie. Concluzia problemei se ob ine imediat folosind urm toarele observa ii :
(i) u, w, z sunt afixele vrfurilor unui triunghi echilateral dac
2 2

i numai dac u +

w + z uw uz wz = 0; (ii) Are loc rela ia (ce se poate verifica printr-un calcul de rutin ): 2 2 2 x 2 y 2 z 2 xy xy yz a 2 b 2 c 2 ab bc ac . X.20. Stabili i natura triunghiului n care au loc simultan rela iile: 2 cos A cos B 1 = 2 2 cos A + 2, 2 cos B cos C 1 = 2 2 cos B + 2, 2 cos C cos A 1 = 2 2 cosC + 2. Neculai Roman, Mirce ti (Ia i) Solu ie. Cu nota iile x 2cos A , y = 2 cos B i z = 2 cos C , ipoteza problemei se scrie 2xy x 2 2 , 2 yz y 2 2 , 2zx z 2 2 . Avem: 2xy x 2 y 2 i analoagele i atunci
se ob ine c 2 y 2 , 2 z 2 , 2 x 2 . Deci, 1 2 cos B , 1 2 cos C , 1 2 cos A , de unde m( A ) 60, m( B ) 60, m( C ) 60. Deoarece m() + m( B ) + m( ) = 180, inegalit ile precedente se transform n egalit i, adic triunghiul ABC este echilateral.

Clasa a XI-a XI.16. Fie numerele a, b C i matricele A, B M n (C) astfel nct aAB bBA I n , In fiind matricea unitate de ordin n 1 . S se demonstreze c det AB BA 0 sau exist

Un

{z C | z n

1} nct a b

0.

Dan Popescu, Suceava Solu ie. Din ipotez ob inem c a AB BA a b BA I n , deci are loc egalitatea a AB BA I n a b BA . Analog b AB BA I n a b AB . Pe de alt parte, det I n tAB det I n tBA t C, c ci matricele AB i BA au acela i polinom caracteristic. Rezult c det a AB BA det b AB BA . Presupunnd c det( AB BA) 0,

72

urmeaz c i atunci

an
ab
n

( b) n , rela ie care mpreun cu ipoteza asigur c ab


1 , ceea ce ncheie demonstra ia. x1
2

0 (deci b

0)

XI.17. Fie 0

x2 i

0,1 . S se arate c

irul x n

n 1

dat de

x 2n

x 2n 1 x1 n , x 2n 2

x 2 n x 1 n 1 , n 1 , este convergent. 2

Gheorghe Costovici, Ia i Solu ie. Pornind de la observa ia c 0 x1 x 3 x 4 x 2 , se arat prin induc ie matematic faptul c 0 x 2n -1 x 2n 1 x 2n 2 x 2n , n 1 . Atunci sub irul x 2n -1 n 1

este cresc tor i m rginit superior de x2 ,deci este convergent la l1, sub irul x 2n descresc tor i m rginit inferior de x1, deci este convergent la l2 i n plus l 1 la limit n rela iile de recuren , ob inem c a adar x n n 1 este un ir convergent.
99

n 1

este

l 2 . Trecnd

1 l1 = l1 l 2-

, deci

1 l1

1 l2

, adic l1= l2,

XI.18. S se demonstreze inegalitatea


k 1

e arcsin k 100

1999 99
7

Lucian-Georges L dunc , Ia i Solu ie. Plecnd de la inegalitatea cunoscut sin x


x, x

(0,1] i folosind monotonia


arcsin x

func iei arcsin, rezult c x < arcsin x, x alt parte, se arat imediat c
ex 1 x

(0,1 ), de unde e
2

ex , x

0,1 . Pe de

x x2 , x 0 , deci e arcsin x 1 x , x 2 2 (0,1) . Facem, pe rnd, x 1 100, 2 100 , , 99 100 i sum m inegalit ile ob inute. Se
99

deduce c
k 1

e arcsin k 100

99

1 100

99

k
k 1

1 2 100
2

99

k2
k 1

99 1999 1200

99 1999
7

XI.19. Determina i func iile f: R

R continue n x0 = 1 i care verific rela ia:


f ( x) ( a 1) 2 ( x 1) , x R,

f ( a 2 x a 2 1) 2 f (ax a 1) unde a>1 este un num r real dat.

Dumitru - Dominic Bucescu, Ia i Solu ie. Cu substitu ia x 1 t , ecua ia func ional dat devine: f ( a 2 t 1) 2 f ( at 1) f (t ) (a 1) 2 t , t R Evident c func ia identic 1R verific (1); fie f o alt solu ie. Avem ( f 1R )(a 2 t 1) 2( f 1R )(at 1) ( f 1R )(t ) 0, t R , echivalent
2

(1) cu

g(a t ) 2 g (at ) g (t ) 0 t R, unde func ia g: R R, g (t ) ( f 1R )(t 1) este continu n t 0 0 . Definim h : R R , h t g at g t , func ie continu n t 0 0 , cu proprieta ile: h 0 0 , h at h t 0 , t R. Inductiv, ob inem

73

ht

1 t a ht

1 a2
n

t 1 an

h t

1 an

t
1 an t

R,

t
h0

N*
0, t

i f cnd n

, rezult c 0, t

lim h

h lim
n

R, deci
1

g at
h0

gt

R. Relund ra ionamentul anterior ob inem c g t


1R c c

c , deci g

t an R . Reciproc, aceste func ii verific evident rela ia (1).


n

g lim

XI.20. ntr-un plan dat se consider punctele fixe A, A' i punctul mobil P. Se noteaz cu P' proiec ia punctului P pe mediatoarea segmentului A A' i cu P" simetricul lui P fa de dreapta A A'. S se afle locul geometric al punctului P tiind c dreapta P'P" este paralel cu dreapta lui Euler a triunghiului PAA'. Paraschiva Brsan, Ia i Solu ie. Fie G = PO P'P", unde O este mijlocul lui AA'. Din OGP' ~ PGP", OG OP 1 ob inem , deci G este centrul de greutate al PAA'. Cum dreapta lui Euler GP PP 2 con ine punctul G, rezult din ipotez c aceasta esta chiar P'P". Deoarece centrul cercului circumscris AA' P este intersec ia dreptei lui Euler cu o mediatoare, urmeaz c P' este punctul din plan egal dep rtat de P, A, A'. Atunci P'P 2=P'A2=P'O 2+OA 2. Raportnd planul la un reper ortogonal cu originea n O i avnd pe AA' drept ax a absciselor, fie x, y coordonatele lui P. Rela ia precedent conduce la x 2 y 2 a 2 , unde AA'=a. Deci locul geometric al punctului P este hiperbola echilater de semiaxe a i avnd ca axe de simetrie dreptele AA' i mediatoarea segmentului [AA']. Clasa a XII-a XII.16. Cerceta i dac exist func ii continue f : R* R* pentru care F (2x 1 / x ) 2F (x ) F (1 / x ), x R , unde F este o primitiv a func iei f. Gabriel Popa, Ia i Solu ie. Pentru nceput, s observ m c rela ia din enun are sens, ntruct 2x 1 / x 0 x R * , adic putem defini F (2x 1 / x ) . Cum F este derivabil , derivnd membru cu membru rela ia dat , se ob ine: (2 1/ x 2 )f (2x 1/ x )
2f (x )F (1/ x ) (2 / x 2 )F (x ) f (1/ x ), x R*. Lund n aceast egalitatea x 1 R*, ob inem

f (3)

0 Im f , deci nu exist func ii f cu propriet ile cerute.

XII.17. Dac func ia f : R R* este continu , impar


T

i periodic de perioad

principal T, calcula i I
0

f ( f ( x) kx)dx, k

Z.

Dumitru Gherman, Pa cani Solu ie. Cu schimbarea de variabil x=T- u ob inem:

74

I
T

f ( f (T u ) k (T u ))du
0

f ( f ( u ) ku kT )du
0

f ( f (u ) ku )du

T 0

f ( f (u ) ku )du

0.

XII.18. Calcula i I Solu ie.


I J x 2000 x
2668

x 2000 x 2668 1

dx

i J

dx , x (0, ) . x 2668 1 Lucian-Georges L dunc , Ia i

x 666

x 666

dx

x 666 1 / x 668 x
1334

1/ x

1334

dx

1 667

x 667 1 / x 667 x 667 1 / x 667


C.
2

dx

2 x 667 1 / x 667 arctg 1334 2

...

x 1/ x 2 Formnd un sistem din rela iile ob inute, afl m valorile lui I i J.

1 667

x 667 1 / x 667
667 667 2

dx

x 667 1 / x 667 2 ln 2668 x 667 1 / x 667

2 2

C .

XII.19. Dac T > 0 este perioad pentru func ia continu f : [0, ) g : [0,T] R este continu , s se demonstreze rela ia:
T n

R i

lim g (x )f (nx )dx


0

1 T

f (x )dx
0 0

g (x )dx .

Dan Popescu, Suceava Solu ie. A se vedea nota D. Popescu i F. Popovici O generalizare a lemei lui Riemann , n acest num r al revistei, p.12. XII.20.
n 1

Fie (G, ) un grup comutativ de ordin n.

se arate c : ( ai )n
i 1

n1

(
i 1

ai )

n 2

e , unde e, a1 , a 2 ,
Solu ie.
n 1

sunt elementele grupului G, e- elementul neutru. Cristian Fr sinaru, Ia i Deoarece G n , rezult c ain e, i 1, n 1 (fie din teorema lui
an
1

Lagrange, fie cf. pb. R-4, cap.III, 3 din manualul n vigoare n cazul comutativ), deci
(
i 1

ai ) n

e (1). Aplica ia f : G

G , f ( x)

x 1 , x G , este injectiv
{e
1

i cum G este

finit, rezult c

f este bijectiv . Atunci: {e , a1 ,..., a n 1 } G


1

, a1 1 ,..., a n 11 } i deci
n 1 i 1

are loc egalitatea e a 1 .... a n

a1 1 ... a n 11 , ceea ce implic (

ai ) 2

e (2). Din

(1) i (2), concluzia problemei urmeaz imediat.

75

Probleme propuse 1
Clasele primare
P.24. Afla i numerele a, b, c, d tiind c verific n acela i timp urm toarele egalit i: a + 3 = b, b + 3 = c, c + 3 = d, a + 3 = 10. ( Clasa I ) nv. Maria Racu, Ia i P.25. Un elev din clasa I, fixnd un num r din irul numerelor naturale, constat c suma numerelor din fa a lui nu este mai mic dect 55, iar suma aceasta adunat cu num rul fixat nu dep e te pe 66. Despre ce num r este vorba? ( Clasa I ) Lumini a Popa, elev , Ia i P.26. Pe trei borcane de compot, unul de cire e, altul de vi ine i al treilea cu amestec de cire e i vi ine, toate etichetele au fost puse gre it. Sco nd un singur fruct dintr-un singur borcan, determina i con inutul fiec ruia. ( Clasa a II-a ) *** P.27. S se scrie num rul 31 folosind cele patru opera ii aritmetice i numai cifra 3 (se cer cel pu in dou solu ii). ( Clasa a II-a ) Andrea Balla, elev , Bra ov P.28. Cte pagini are o carte dac pentru paginarea ei s-a folosit cifra 9 de 117 ori? ( Clasa a III-a ) Crizantema Mironeanu, elev , Ia i P.29. Ioana i Alina au cules mpreun 165 de nuci. Ioana a cules mai pu ine nuci dect Alina; ea face un calcul i observ c triplul diferen ei dintre num rul nucilor culese de ele reprezint tocmai num rul nucilor culese de Alina. Cte nuci a cules fiecare fat ? ( Clasa a III-a) nv. Maria Racu, Ia i P.30. Ar ta i c dintre oricare patru numere naturale diferite, mai mici dect 1000000, se pot alege dou a c ror diferen s se mpart exact la 3. ( Clasa a IV-a ) Roxana Bolocan, elev , Ia i P.31. O veveri descoper un alun nc rcat cu fructe i i face provizii pentru iarn transportnd la scorbura sa alternativ: o dat dou alune, o dat trei alune. Dup ce transport 47 de alune, face o pauz pentru a se odihni. S se calculeze ce distan a parcurs veveri a n total, dac de la alun la scorbura ei este o distan de x hm x dam x m, unde x are ca valoare cel mai mic num r natural posibil. ( Clasa a IV-a ) nv. Mihai Agrici, Ia i P.32. Un p rinte i mparte averea astfel: la primul copil 10 milioane plus o cincime din rest, la al doilea copil 20 de milioane plus o cincime din noul rest, la al treilea 30 de milioane plus o cincime din noul rest i a a mai departe. S se afle suma mp r it de p rinte, precum i num rul copiilor, tiind c to i au mo teniri egale. ( Clasa a IV-a ) Mihai Grtan, Ia i a0b c0d V.26. S se determine cifrele distincte i e*f nenule a, b, c, d, e, f, g pentru care rezultatul g** nmul irii al turate este cel mai mare posibil: g***f Ioan S c leanu, Hrl u V.27. Trei apicultori au tras mpreun 700 kg miere de albine. Cnd au mp r it mierea, primul apicultor a luat jum tate, al doilea jum tate din rest, al treilea jum tate din
1

Clasa aV-a

Se primesc solu ii pn la data de 15. 01. 2003.

76

noul rest, apoi opera iunea se repet pn se mparte toat mierea. S se afle ct miere a luat fiecare. C t lin-Cristian Budeanu, Ia i 2001 2001 2002 2002 i N2 3 sunt numere divizibile cu 5. V.28. Ar ta i c N1 3 2 2 Dorina Carapanu, Ia i V.29. S se afle numerele abc pentru care abc ac b 2 . Roman a Ghi i Ioan Ghi , Blaj V.30. Dac xi, i= 1,500 , sunt numere naturale nedivizibile cu 5, atunci num rul
N 4x 14
8 8x 2 12 12x 3

...

2000 2000x 500 este divizibil cu 5.

Tamara Culac, Ia i

Clasa aVI-a
VI.26. Fie A 4a 6b c, B 4a 3b c, C (A, B)=23, ar ta i c (A, B, C)=23.
3a 11b 28c , unde a,b,c Z . Dac

Cristiana Constanda, elev , Ia i VI.27. S se rezolve n Z sistemul 3x 2 y 8;


x y 1; 3x 1 1. . Mihai Cr ciun, Pa cani

2) ... 2n 2 4 ... 2n . Dumitru - Dominic Bucescu, Ia i VI.29. n triunghiul ascu itunghic ABC, bisectoarea interioar a unghiului B intersecteaz nal imea din A n E. Fie F (DC astfel nct AE=EF. Ar ta i c BE AF. Tamara Culac, Ia i VI.30. Pe ipotenuza (BC) a triunghiului dreptunghic ABC se consider punctele N i M astfel nct BN=AB, CM=AC. Dac P i Q sunt proiec iile punctelor M i N pe dreptele AN, respectiv AM, demonstra i c segmentele (MP), (NQ) i (PQ) se pot constitui n laturile unui triunghi. C t lin Calistru, Ia i

VI.28. S se rezolve n N ecua ia 1 2 2 3 n (n 1) (n 1) ( n

Clasa a VII-a
VII.26. Determina i a VII.27. Determina i a
2 x1 (a 1)x1

Q tiind c

Q. Gheorghe Iurea, Ia i

R astfel nct sistemul

a2 x 2, 4 s admit numai solu ii ntregi.

...

2 , xn

(a 1)x n

a2 4

2 x n , x n (a 1)x n

a2 4

x1

C t lin Calistru, Ia i VII.28. Fie zece numere naturale nenule care au suma egal cu 55. S se arate c printre ele exist trei care pot fi lungimile laturilor unui triunghi. Adrian Zanoschi, Ia i VII.29. Fie ABCD un p trat, O centrul s u, iar M i P mijloacele segmentelor (OA),

77

respectiv (CD). S se arate c triunghiul BMP este dreptunghic isoscel.258 Constantin Cocea i Dumitru Neagu, Ia i VII.30. Fie ABCD un p trat de latur 1 i punctele M (AD), N (BC), 4 4 1 i AM BN . {P}=BM AN. Dac SDCNPM = , demonstra i c 1 AM BN 2 3 9 Emil Vasile, Ploie ti

Clasa aVIII-a
VIII.26. Demonstra i c ecua ia (t 2 1) x 2 n Z Z.
4t 2 x 4t 2

0 are numai dou solu ii

VIII.27. Determina i x,y Z pentru care frac ia


(n leg tur cu E: 9314* din G.M. 11-12/1987).

x x2 xy

Mihai Cr ciun, Pa cani 5 este echivalent cu 19 y2

Gabriel Popa, Ia i VIII.28. Fie a,b numere naturale de parit i diferite. Afla i valorile lui n pentru care Sn=an+ an-1b + an-2b2 ++ abn-1 + bn este divizibil cu a+b. Mihaela Predescu, Pite ti VIII.29. Se consider piramida triunghiular regulat VABC cu latura bazei a, iar 3a . muchia lateral 2a. Fie M mijlocul lui (VA), iar N un punct pe (VB) astfel nct VN= 4 Afla i distan a de la V la planul (MNC). Adrian Corduneanu, Ia i VIII.30. Fie A, B, C, D patru puncte necoplanare astfel nct AB
4 73 , CD 4 29 . Not m cu E, F mijloacele segmentelor (AB), respectiv (CD). S se arate c mijloacele segmentelor (AF), (BF), (CE), (DE) sunt vrfurile unui paralelogram i s se

calculeze aria acestuia tiind c are o latur de lungime 194 . Roman a Ghi

i Ioan Ghi , Blaj

Clasa a IX-a
IX.26. Dac a
0,

, s se rezolve ecua ia

IX.27. S se determine func iile f, g : 0,


aditiv i g y g f x f x g y , x, y 0,

a a . Discu ie. x x x D. M. B tine u-Giurgiu, Bucure ti 0, , unde g este surjectiv i

IX.28. S se determine func iile f : R


R fixat, iar func ia g

Ioan S c leanu, Hrl u R pentru care ( f f f) x x ,


n ori

1R este monoton .

Mihail Bencze, Bra ov IX.29. S se arate c n orice triunghi ABC are loc inegalitatea

78

3 3 lb lc 3R p ( p 3 3abc) . hb hc 2r Viorel Cornea i Dan tefan Marinescu, Hunedoara IX.30. n patrulaterul ABCD consider m punctele R i S pe diagonala BD, n AR BC , interioarele triunghiurilor ABC, respectiv ACD. Not m M CR AB , N

3 la ha

AS

CD

i Q
BM n MC
n

CS

AD .

tiind c
4, n

AM 2 MB 2
N.

BM 2 MC 2

CP 2 PD 2

DQ 2 QA 2

4 , s se

arate c

AM n MB
n

CP n PD
n

DQ n QA n

C t lin Calistru, Ia i

Clasa a X-a
X.26. Fie ecua ia x 4 S1 x 3 Sx 2 mx m 1 0 , unde S este aria unui triunghi neechilateral ABC , iar S1 este aria triunghiului A1 B1C1 determinat de punctele de intersec ie ale bisectoarelor interioare cu cercul circumscris triunghiului ABC . S se determine m R tiind c ecua ia admite un num r impar de r d cini n 0,1 . X.27. Fie r
arate c dac P z
1,

, D

C; z

Dumitru Gherman, Pa cani i P C X , P X aX 2 bX c . S se


D.

D, z

D , atunci a, b, c
z
2

D.M. B tine u-Giurgiu, Bucure ti X.28. Rezolva i ecua ia z 2 (2


1) z (2
z 1

1) 1 0 , z

C\R.

Emil Vasile, Ploie ti X.29. Un motan scoate cu ajutorul unui pahar un num r de pe ti ori dintr-un acvariu. C i pe ti ori trebuie s con in acvariul astfel nct motanul s aib matematic speran a c va scoate 5 dintre ei? Gabriel Popa, Ia i X.30. Fie M 1,2,..., n . S se afle num rul de k-uple A1 , A2 ,..., Ak de submul imi
k k

ale lui M astfel nct


i 1

Ai

i Card (
i 1

Ai )

l, l

n fixat.

Lucian-Georges L dunc , Ia i

Clasa a XI-a
XI.26. Fie A M n R . Dac XI.27. Fie a
0,1
Tr (t A A (t A )* A * ) 2n det A , atunci t A A * . Iuliana Georgescu i Paul Georgescu, Ia i un ir de numere reale astfel nct

i xn
2 xn

n 0

2 a max x n 1 ,

1 2 xn 2

2 xn

N* .

Ar ta i c

irul x n

n 0

este convergent i determina i limita sa. Aurel Muntean, Sibiu

79

XI.28. S se determine p
n

R pentru care limita irului a n

n 1 definit

prin termenul

general a n
k 1

np k k2 1

este finit

i nenul . Constantin Chiril , Ia i

XI.29. S se arate c

n lim n ln ln n

1 1 n1 ..... 1 2 n

1.

Marian Tetiva, Brlad XI.30. Fie f : R R o func ie discontinu i care are proprietatea lui Darboux. Dac exist o func ie g : R R R astfel nct f x y g f x , y , pentru orice x, y R , . atunci func ia f nu are limit la tefan Alexe, Pite ti

Clasa a XII-a
XII.26. Se consider mul imea M
1 a2 a2 a 2 sau A = Q sau A = R. Ar ta i c a2 1 a2 a 2 a 2 a 2 ;a 1 A , unde A = Z

XII.27. Fie G ,

M , este grup; este acesta izomorf cu A * , ? Gheorghe Costovici, Ia i un grup cu Z G e i H un subgrup netrivial al lui G . S se
1

demonstreze c exist x, y G \ H , x y de grup care nu are aceast proprietate.

, astfel nct xy

H i yx

H . Da i exemplu

Ovidiu Munteanu, student, Bra ov XII.28. Calcula i


n

tgx dx, x

0,

, pentru n

2,3,4 .

XII.29. Fie f : R

R o func ie continu
n n

it
t

0 . Pentru a, b
t a b

Daniel Jinga, Pite ti 0 , s se calculeze

lim
k 1

kt

k 1t nt

f n

ka k 1 b

Mihail Bencze, Bra ov


1

XII.30. S se arate c
0

ex

ln 1 x 1 x2
2

dx

e . ln 2, 8 16

Cristian Moan , Craiova

80

   
  
  
 
     
 
      

  


 



  
 
   
   
       
 ! 
  


 


 " # + " #   


   

   

    " +  ! 
 "   # $% ' $ 
 %
    %!  '
  


   
 
  (        
 % %  %!    (        
 % %  %! )    
 
     %
  *      

 
 
   %
        +  
           
   ,

       ! 
 
  )     # $% () -
 
! '    +


+

+  +


+

+  +

= + 

(( .! '  

  



*+ , #  - = /00/ 

(. $   
  (0

(1
   (3 

 / 

+# $% 
 
  # %  #    %
 
 + + $ # $% $ 
 1  
 ,2 
 ,2 ')     

  ! ,  2
 

2  # $% $ 
   "   3


 

[ ] + [  ] +  + [ ] =


  $  #

# $%

   0   + / +

   4   +  

 4

2  
 
 # # $% ( 5     


 ,2   2 
   ) 7   6   + 
,2    * 
!
 2
 " /- 
# %
 ( 5  

 '1 
  ,   
 
  %  
     

  '    
 %  
     

  ' 
$ 
    )   /"

#

  $ 


      

  = %         3       

2  / !5 7
# $% 7 8 -
   
    % ( 5 
 ,2
    )  %    
       
 ,2   9


 {  } =   %
 ,2    
  !
   %   
  '  { } =     
     8 
   # " % (' 5   
 ,2  2 

  
  % 

    "   %
! ! ,   


   ')

   
  ! 
 $     "  " 
 

    
 
  " + "  + " " =     ,  ,     
  ,   "9 
# $% .) $ 
 

  ,
  

   '   
 2    
 
  
 2
 ')
    % 
  ')

     
  !  2  %
 $ 
      

 
   ,  '   
 
 

 " 

   ' 2
 

2+  # $%

()



 
< 5
# 

#   =

 = :; 




 
 " 



 
" 


"

+


  

+


+  +

= 

=

+  +


7 < <
 ,2 
 4   
{ } =    {#} =    
    >        

()

4
"  # # 2        %   
      % #  

$ 
> 
# ?@  < 5
 ,2  %  ?     (  ) = {    }    ( ) =    + + 4 3 {    }    (  ) = {    }  $ 
       4 
# *  %  A$%B % $ % ( ' & (  )  ( ' &  <' 5 
 ,2  ,2 () + ') + &) = ( ( + ' + &)    )   
 ,
 
 ,2  
  
! 
 

  
 ,2     2 
 * 
!
    %   # $% @   $ 
 1  
,
 
    
 <()  5 


   
  
  

#

 
,
 
    

    
  
  
 
,
           
    
 / 

+# $%  <(( $ 
! '     " = "  + A0  
  $  # # $% <(. 5  

 '1+  : ;3B  C      (  
   +  (  + C    -

!  " (   
 (    " +   "  +  "    ,

  * +,  *D "  C 2
 % 
5 7
* 
 # ?
/   + + - . +   <(0 5  4[E] . = . + .  >  % 
 $ 


  

 
    ! ' %

)

+ ( )

>

<(1 -

   

  

+ + * # 2  /  % . +      . +    (.) = 

 


!
  
 

   
   ,

 
  

)      + 

  
    
    
 
  ( ] 
     %    
' ,   "9 
# $%   
 %
 '
,        $ 
  

  $ 
  %   %

. +

<(3 5 4 =  % ( $ )  )    


 ={ +$ }

 # $%

< % %  = {  ( [  ] [  ] +     F }  
 1 /  <(  5 / ( )  [  % ] % 
      
 

  
 1   
 <( 5  /
/   $ [  % ]   )

/ ( ) =  % / = /

/ (

/$ [ + % / ( $ ) ] = $( + % $ )  5 7
 D #  % 


 ! ' % / ( [  ) [  )    
'  

/

( )

[  ) 



 


 

 

/  

 /     

  > 
% 
# 2     )  + 0    '
 %

(

+ 0)   (  ) 

  0 {  $ 


<( 5   (  ) 

 }

 + 1    '
 

  1  % (  + 1)   (  ) +

#"

) = {  + 1+ 1 2 }   ,
 

  =   
   

  
%   
    ,

   )  * 
$
 #  %  ,
 ,
  
(   )  888      3 =   888     3 =   % 3    
 < 
!  </
# $% / ( [ ] 4   
 
'   $ <.) 5 4  >   $  
  
    4 ( [ ] 4  4 ( ) = / ( )     
 
   / (") / 5"   " *  2+ -# "  %  <(' 5 3

( )

<$>, *E*"4$<F4 8$<$<E$ $5$  >>*


6  
 
 H %
 H 6 6 K  LM   9 N ? % 6 Q  R
  LM     
N ? % 6 Q  R
  LJ  . ' N ? % 6 Q  R
  LJ  . ' N ? % 6 Q  R
  L*
 ?
  N ? % 6 U '
  R2  L*2  2N ? % 6 U '
  L  ? 9 ! N ? % 6 Q  R
  L*
 ?
  N ? % 6 Q  R
  LJ  . ' N ? % 6 Q   
 LI ' T  N ? % 6 *
 K 
J 
,  ? % 6 *
 K 
J 
,  ? % 6 K  LR D 
 N ? % 6 K  LR D 
 N ? % 6 K  LR D 
 N ? % 6 9 , >  L9 >,
!!N ? % 6 K 
  ? % 6 Q  R
  L  ? 9 ! N ? % 6 ?   

 ? % 6 Q  R
  L*
 D  N ? % 6 Q  R
  L*
 D  N ? %



4# > I>J J  48 *.?*I.K P  A R?DISJ *2


,2 A R.> 5?. ?  A T<.$> -
2' A" T<.$> R    A0 T<.$> D
 A/ 9.->U 
 A4 D?.I> > AA U9   >   A# 9 ?K D2  >
 A8 -Q*S?U D
 # -QVJKU M
  # 9IMQ9?U9 D
,
 # 9IMQ9?U9 
#" D?SQ9SJ M   #0 $?D?. 9  #/ $?D?. 9
  #4 -MQ?U9 9
  #A S?M$ D2  ## $U$>U * #8 *JI.*J$9U ? 
Q '     
 

#0

Pagina rezolvitorilor
BRA OV
coala general nr.5. Clasa a V-a. POSTEUC Raluca: P(12,21), V(21-23); POSTEUC Bogdan: P(12,21), V(21-23). Liceul Teoretic N. Titulescu. Clasa a IX-a. BOR O Olivia-Dana: VII.16, VIII(17, 23,24), IX.17; BOSCORNEA Ionu Bogdan: VII.16, VIII(17,23,24), IX.17; BURSUCANU Lucian: VII.16, VIII(17,23,24), IX.17; CEN U Nicoleta Veronica: VII.16, VIII(17,23,24), IX.17; EDU Vladimir: VII.16, VIII(17,23,24), IX.17; GHEORGHE Liana Elena: VII.16, VIII(17,23,24), IX.17; HALIPPA Andra: VII.16, VIII(17,23,24), IX.17; LITR Anatolie: VII.16, VIII(17,23,24), IX.17; MOARC S Liviu George: VII.16, VIII(17,23,24), IX.17; ONCIOIU Cristina: VII.16, VIII(17,23,24), IX.17; PAN Andreea: VII.16, VIII(17,23,24), IX.17; PASCAL Andreea: VII.16, VIII(17,23,24), IX.17; PLATON Alexandru: VII.16, VIII(17,23,24), IX.17; R CHITEANU Lavinia: VII.16, VIII(17,23,24), IX.17; RUSEN Adrian Nicolae: VII.16, VIII(17,23,24), IX.17; OIMU Adelina: VII.16, VIII(17,23,24), IX.17; TUDOR Ioana: VII.16, VIII(17,23,24), IX.17.Clasa a X-a. BRA OVEANU Adina: VIII.24, IX.23, X(20,22-24); CHELMEA Andreea: VIII(17,24), IX.23,X(22,23); CHI Cristina: VIII.24, IX.23, X(20,22-24); CURCAN Diana: VIII.24, IX.23, X(20,22-24); DINU Cristina: VIII.24, IX.23, X(20,22-24); DI OR Mihaela: VIII(17,24), IX.23, X(22,23); ELEKE Alexandra: VIII(17,24), IX.23, X(22,23); EPURE Alexandru: VIII.24, IX.23, X(20,22-24); EZARU Lucian: VIII.24, IX.23, X(20,22-24); GAVRIL Ramona: VIII(17,24), IX.23, X(22,23); HO OLEANU Cristina: VIII.24, IX.23, X(20,22-24); JEGAN Alina: VIII(17,24), IX.23, X(22,23); LUNGU Cezar: VIII(17,24), IX.23, X(22,23); NEAGOE L cr mioara: VIII(17,24), IX.23, X(22,23); OANCEA Vlad: VIII.24, IX.24, X(20,22,23); OSTAFI Andrei: VIII.24, IX.23, X(20,22-24); PAHON U Radu: VIII(17,24), IX.23, X(22,23); PETRI ORU Doris-Georgiana: VIII.24, IX.23, X(20,22-24); PU CA U Andreea: VIII.24, IX.23, X(20,22-24); PU COIU Lucian: VIII(17,24), IX.23, X(22,23); SAVEI Ionela: VIII(17,24), IX(17,23), X(22,23); TODORU Anca: VIII.24, IX.23, X(20,22-24); UIC Tatiana: VIII(17,24), IX.23, X(22,23); V R REANU Andrei: VIII.24, IX.23, X(22-24). Clasa a XII-a. BCLEA Timea: X.24, XI(22,23,25), XII(20,24): C PRARU Andrei: X(20,24), XI(22,25), XII.20; CERCELEA Rare : X.24, XI(22,25), XII(20,24); DOBO Claudiu: X(22-24), XI(22,23,25), XII.24; DUMITRU Andrei: X(20,24), XI(22,23,25), XII(20,24); FER STOARU Valentin: X(20,23), XI(22,23,25), XII(20,24); FORSEA Adrian: X(22-24), XI(22,23,25), XII(20,24); GHERGHE Mihai: X(20,24), XI(22,25), XII.20; IORDACHE Florin: X(20,23,24), XI(22,23,25), XII(20,24); MU OIU Rare : X(23,24), XI(22,23,25), XII(20,24); N STAS Gabriel: X(22,23), XI(22,25), XII.20; RO CA Silviu: X(20,24), XI(22,23,25), XII(20,24); ROTOP NESCU Roxana: X(20,22-24), XI.25, XII.20; AG U Marian: X(23,24), XI(22,23,25), XII(20,24); RU Ana-Maria: X(20,22-24), XI(22,25), XII.20.

CRAIOVA
Colegiul Na . Fra ii Buze ti. Clasa a V-a. TU ESCU Anca- tefania:V(2125),VI.22.

IA I
coala nr. 3 Alexandru Vlahu . Clasa a IV-a (nv. GALIA Gabriela). CHITIC Andreea: P(14-18,20); CIOBANU Diana Luciana: P(14-18,20); SOFICU Corina-Maria: P(14-17,19,20); VRABIE Corina-Roxana: P(14-16,19,20). coala nr. 7 Nicolae Tonitza. Clasa a IV-a ( nv. MELINTE Rodica). ANDRONIC 85

Mihaela: P(14, 16-23); DUMITRIU Alexandru: P(14, 16-19).Clasa a IV-a (nv. TUDOSE Elena). BOTEZ Alin: P(14-23); CHILCO Claudia: P(14-23); CHIRIAC Alexandra:P(1423); CIUBOTARIU Remus: P(14-18,21); IONESCU Mihai-Bogdan: P(14-21,23); ZORICI Iulian: P(14-18,20-23). Clasa a IV-a (nv. PANAINTE Adriana). CURC Bogdan P(1418,21); ONU Adina: P(14,15,17,19-21); SRGHE Nicoleta: P(14,15,17, 19-21); TRUFANDA Alexandru: P(14-17,20,21). coala nr. 11 Otilia Cazimir. Clasa a III-a. (nv. PRIAL D-tru). TIBA Marius: P(14-22). coala nr.17 Ion Creang Clasa a VI-a B DI Vasilic Valentin: V(16, 19-21, 23),VI.17; CASIAN Manuela: P(9,12), V(16,18) VI.17; IFTODE Andreea P(9,12), V(16,18,20,21); LUPU Andreea-M d lina: P(9,12), V(20,21), VI.17; SOFRONEA Gabriela: P.9, V(20,21), VI(16,17); TANANA Irina-Eliza: P(9,12), V(16,19,21) VI.17. coala nr. 22 B.P. Ha deu Clasa a IV-a (nv. TRZIORU Iuliana). BARAN LigiaMaria P(14-23); B L U A Bogdan-Alexandru: P(14-23); CHIHAIA Mihai: P(14-18, 2023); MACOVEI Smaranda-Teona: P(14-23); PRODAN Andreea: P(14-23); RAI Bogdan: P(14-23); SILION C t lina: P(14-23); SPNU Drago : P(14-23). coala nr. 23 Titu Maiorescu .Clasa a II-a (nv. CHIRIL Beatrice). TUDORACHE Alexandru-Gabriel: P(7-11, 14-18, 20,21). coala nr. 43 Dimitrie Sturdza. Clasa a IV-a (nv. OBREJA Rodica). OPRIC Adelina: P(14,16,17,21,22) Colegiul Na ional C. Negruzzi. Clasa a IX-a. ARTENIE Cristiana: VII(16,22), VIII(17, 18, 20-24), IX(16,18,22,24). Liceul Teoretic Gr. Moisil Clasa a VII-a. COSTIN Ciprian : V(21-23, 25), VI(2123,25), VII(22,23,25), G(2,3). Clasa a XII-a. ALECSANDRESCU Ioana: XI(21-24), XII.24. Liceul Teoretic Garabet Ibr ileanu Clasa a V-a . BABIUC Sanda: P(9,12,13,21), V(16,20,21,23); BUDEANU tefana: P(9,13,22), V(16,20,21); FUIOREA Bogdan: P(9,12,21-23), V.16; GAVRILESCU Ioana: P(10,19,20,22), V.16; TURCHIN Ariadna: P(9,12,13,21-23), V(20,21); UNGUREANU Drago : P(9,10,13,21,22), V(20,21). Clasa a VI-a. PETRU C Magda: P(9,12,13), V(19,21). Clasa a VII-a. CHIRVASE Ariadna : V.23, VI(22,23,25), VII.22; JUVERDEANU George: V(21,23), VI(22,23,25), VII.23; PRUTEANU Irina: : V.23, VI(22,25), VII(22,23); STANCIU Filip: V(16, 20, 21, 23), VI(18,19,22). Liceul Teoretic M. Eminescu. Clasa a VII-a. AVRAM Mircea: V(21-23,25), VI(23,25), VII.22; CIUCANU Radu: V(21-23,25), VI.25; D N IL MIHAI: V(21-23), VI(22,23,25), VIII.23; E ANU Ruxandra: V(16,21,23,25), VI.19, VIII.23; GRAMSCHI Raluca: V(21,23,25), VI(19,25), VIII.23; TOFAN Andrei: V(21-23,25), VI(23,25); TUDOSE tefan: V(16,20,21,23,25), VI.25; TURLIUC C lin Rare : V(22-25), VI(21,25); Clasa a VIII-a. BUZDUGA Alexandru: VI(21,25), VIII(21,24,25).

Redac ia revistei Recrea ii matematice acord cte un premiu n c r i elevilor: COSTIN Ciprian (Lic. Teoretic Gr. Moisil): 1/2001(8pb), 2/2001(8pb), 1/2002(12pb). CIUCANU Radu (Lic. M. Eminescu): 1/2001(8pb), 2/2001(6pb), 1/2002(5pb). D N IL Mihai (Lic. M. Eminescu): 1/2001(5pb), 2/2001(7pb), 1/2002(7pb). TOFAN Andrei (Lic. M. Eminescu): 1/2001(6pb), 2/2001(8pb), 1/2002(6pb). TURLIUC C lin Rare (Lic. M. Eminescu): 1/2001(6pb), 2/2001(6pb), 1/2002(6pb). 86

Revista RECREA II MATEMATICE apare de dou ori pe an (la datele de 1 martie i 1 septembrie) i se adreseaz elevilor, profesorilor, studen ilor i tuturor celor pasiona i de matematicile elementare. n aten ia tuturor colaboratorilor Materialele trimise redac iei spre publicare (note i articole, chestiuni de metodic , probleme propuse etc.) trebuie prezentate ngrijit, clar i concis; ele trebuie s prezinte interes pentru un cerc ct mai larg de cititori. Se recomand ca textele s nu dep easc patru pagini. Evident, ele trebuie s fie originale i s nu fi ap rut sau s fi fost trimise spre publicare altor reviste. Problemele originale destinate rubricii Probleme propuse vor fi redactate pe foi separate cte una pe fiecare foaie, cu enun i demonstra ie/rezolvare, fiind nso ite de numele autorului, coala i localitatea unde lucreaz /nva . Redac ia revistei va decide asupra oportunit ii public rii materialelor primite. n aten ia elevilor Numele elevilor care vor trimite redac iei solu ii corecte la exerci iile i problemele din rubrica Probleme propuse vor fi men ionate n Pagina rezolvitorilor. Elevii vor ine seama de urm toarele reguli: 1. Pot trimite solu ii la minimum cinci probleme propuse n num rul prezent i cel anterior al revistei; pe o foaie va fi redactat solu ia unei singure probleme. 2. Elevii din clasele VI-XII au dreptul s trimit solu ii la problemele propuse pentru clasa lor, pentru orice clas mai mare, din dou clase mai mici i imediat anterioare. Elevii din clasa a V-a pot trimite solu ii la problemele propuse pentru clasela a IV-a, a V-a i orice clas mai mare, iar elevii claselor I-IV pot trimite solu ii la problemele propuse pentru oricare din clasele primare i orice clas mai mare. Orice elev poate trimite solu ii la problemele de concurs (de tip G i L). 3. Vor fi men ionate urm toarele date personale: numele i prenumele, clasa, coala i localitatea. 4. Plicul cu probleme rezolvate se va trimite prin po t (sau va fi adus direct) la adresa Redac iei: Prof. dr. Temistocle Brsan Catedra de Matematic Universitatea Tehnic Gh. Asachi Ia i Bulevardul Carol I nr. 11, 6600, Ia i (pentru Recrea ii Matematice) E-mail: tbi@math.tuiasi.ro

CUPRINS
Alin Spum .1 Petre Osmatescu (1925 2001).2 Academicianul Petru Soltan la 70 ani..3 Niels Henrik Abel 200 de ani de la na tere4 ARTICOLE I NOTE MATEMATICE

P. MINU Asupra ipotezei lui Goldbach.5 C. COCEA Generalizarea teoremei de omologie a lui Barbilian.7 I. GEORGESCU, P. GEORGESCU Asupra unor iruri de integrale...9 D. POPESCU, F. POPOVICI O generalizare a lemei lui Riemann.12 M.TETIVA Cteva rela ii metrice deduse vectorial....14 D.- T. MARINESCU, I. ERDEAN Inegalit i geometrice. Aplica ii...17 D. POPESCU Asupra unei clase de iruri recurente....20 T. BRSAN Un criteriu de concuren a dreptelor...23 I. S C LEANU Cteva consecin e ale unei rela ii a lui Gergonne....25 GH. COSTOVICI Unele iruri monotone cu limita e sau e-1 ..28 C.- T. POPA Studiu comparativ privind cteva medii uzuale.31 NOTA ELEVULUI C. ANDREI, T. RUSU Aplica ii ale monotoniei mediilor n raport cu ordinul lor..33 CHESTIUNI METODICE D. G LEAT , G. POPA Func ii care admit / nu admit primitive..36 CHESTIUNI COMPLEMENTARE MANUALELOR E. COHAL Construc ii geometrice cu echerul....41 DIN ISTORIA MATEMATICII S. IONESEI Teorema celor patru culori..43 MATEMATICA N CLASELE PRIMARE P. ASAFTEI Introducerea opera iei de adunare la clasa I ..46 CONCURSURI I EXAMENE

Concurs de admitere 2001, Ia i...48 Capacitate teste preg titoare (I. SECRIERU, C.M. ROMA CU)...51 Bacalaureat teste preg titoare (G. MR ANU)...56 PROBLEME I SOLU II

Solu iile problemelor propuse n nr.1/2001....61 Probleme propuse..76 Probleme pentru preg tirea concursurilor...81 Pagina rezolvitorilor...85

Recreaii tiinifice - cea nti brazd t s t a a

La 15 ianuarie 1883 apare la Iai primul numr al revistei "Recreaii tiin s a t s ce va dura timp de ase ani, cu cte un numr pe lun. s a a Obiectivele urmrite, racordate la cerinele din acele timpuri ale nvm a t ata "de o nelepciune ce face cinste fondatorilor revistei" [6] i apar expuse t s claritate n "Ctr cetitori" - cuvtul de nceput din 15 ianuarie 1883: a a

Apariia revistei "Recreaii tiinifice" este strns legat de condiiile ist t t s t a t epoc. Unirea Principatelor i Proclamarea Independenei Romniei, c a s t 1

structurale din timpul domniei lui Al. I. Cuza i apoi a regelui Carol I au s politic i legislativ al formrii statului romn modern i afirmrii acestuia. s a s a romnesc, ntocmai ca i societatea romneasc n general, a trecut prin m s a frmntri: nfiinarea celor dou universiti din Iai i Bucureti, reforma n a a t a at s s s din 1964, multele regulamente menite s organizeze reeaua de coli i a t s s programele acestora etc. Pe de alt parte, liceniaii romni ai universitilor apusene (din Fran a t t at sau Italia) odat revenii n ar realizau imediat faptul c, n climatul ex a t t a a fi putut ntreprinde cercetri originale proprii. Acestora le revenea obligai a t important pentru acel moment de a contribui la edificarea nvmntu a ata de a ine lecii i a elabora manuale i cursuri n limba romn, de a pre t t s s a viitoare ce urma s fac pasul ctre creaia tiinific original. a a a t s t a a Fondatorii reviste sunt: N. Culianu, C. Climescu i I. Melik - profesori s de tiine din Iai, G. I. Lucescu i V. Paladi - profesori de matematic la L s t s s a din Iai, G. I. Roiu i I. D. Rallet - profesori de matematic la Scoala Mi s s s a G. Zarifopol - profesor de fizic i chimie la Scoala Militar din Iai, I. V. P as a s de matematic la Scoala Normal "Vasile Lupu" din Iai i I. M. Dospinescu a a s s matematic la Gimnaziul " tefan cel Mare" din Iai. a S s a t Este prima revist din tar cu prol stiinic, materialele publicate acop a ramuri ale tiinei: matematic, zic, chimie, mineralogie, geograe, ast s t a a mograe etc. Se adreseaz cu precdere elevilor din colile secundare, a a s profesorilor. Majoritatea fondatorilor erau liceniai n matematic sau profesau acea t t a se explic astfel faptul c "Recreaiile stiinifice" au un coninut predomina a a t t t multe numere de revist au un coninut exclusiv matematic. a t Un numr are n medie 25 pagini; doar n anii II, V i VI numerele 7 i 8 a s s au fost tiprite mpreun, ca o singur revist de 40 pagini. Structura d a a a a numr este: articole, probleme rezolvate i probleme propuse. Date in a s care a aprut, materialele publicate n paginile revistei, articole i problem a s ate sau erau prelucrri din tratatele i revistele de circulaie din acea vrem a s t de matematic vizeaz diversele ei ramuri: aritmetic, algebr, geomet a a a a analitic, descriptiv i diferenial), mecanic teoretic i astronomie, mat a as t a a as arial, istoria matematicii, problemele nvmntului matematic. Problem a ata trezit un viu interes printre elevii din Iai i din toat ara; plecai la studii s s at t unii dintre fotii colaboratori ai revistei au continuat s trimit soluii. Pen s a a t elevilor, redacia public soluiile corecte primite i menioneaz numele tut t a t s t a au dat-o. Sunt tiprite liste de rezolvitori n ordinea numrului de problem a a "Ctr cetitori" din nr. 1/1885, redacia revistei apreciaz c rezultatele ob a a t a a t doi ani sunt pozitive: "[...] rezultatele la care am ajuns snt n destul de O micare n aceast direcie, ntre elevii emineni din scoalele noastre, s a t t s-a determinat. Un numr nsemnat de tineri ne trimet regulat soluii, din a t 2

destul de ingenioase". Nu ne propunem s facem o analiz a coninutului revistei (v. [6],[ a a t selecta doar cteva aspecte ce considerm c sunt semnificative i interesa a a s C. Climescu, sufletul revistei "Recreaii tiinifice", a publicat multe t s t domenii diverse. n ciclul "Cteva curbe celebre i importante", ncep s ncheiat n vol.III, expune principalele curbe plane clasice: cisoida lui Dioc cicloide, spirale etc. I. Melik public n nr. 3/1883 articolul "Despre scrierea numerelor cu li a G. I. Roiu traduce (dup o ediie italian) prima carte a "Elementelor s a t a public n vol.II i III ale revistei. Precizm c traducerea complet n lim a s a a a "Elementelor" a fost fcut mult mai trziu de Victor Marian i publicat a a s a Gazetei Matematice n trei volume, 1939-1941. I. D. Rallet contribuie cu articole variate: maxime i minime geometri s ale patrulaterului circumscriptibil, formulele fundamentale ale trigonometr terminani, echilibrul unui punct material etc. t G. I. Lucescu public printre altele un studiu amplu i documentat des a s timpului i calendar, iar I. V. Praja abordeaz chestiuni de aritmetic (proble s a a . a.), geometrie (transversale, poli i polare .a.), analiz etc. s s s a Revista "Recreaii tiinice" a reuit s atrag colaborarea unor emin t s t s a a din acele timpuri: Miltiade Tzony - Facultatea de tiine din Iai, a publica s t s probleme", ce este prima culegere de probleme de mecanic teoretic din a a leme); P. Tanco - profesor din Nsud, cu chestiuni de lozoa matematicii aa Constantin Gogu - Universitatea din Bucureti, cu cteva scrisori despre c s Solomon - inginer, cu chestiuni de istoria matematicii din antichitate; pro V. Buureanu (mineralogie) i August Scriban (geograe) .a. t s s Nu puini sunt aceia care, n drum ctre o strlucit carier, au fos t a a a a lor activi rezolvitori ai "Recreaiilor tiinice": Ermil Pangrati - profesor t s t descriptiv i rector al Universitii din Bucureti, Anastasie Obreja - crea as at s chimie organic din Iai, Vasile Cristescu - unul din cei patru "stlpi" ai G a s atice, Dimitrie Pompeiu - ilustrul matematician romn, Petre Culianu, Gr i muli alii. s t t Rezolvitorii erau mai ales elevi ai liceelor i colilor militare sau studen s s t din toate colurile rii (de atunci!): Dorohoi, Bacu, Brlad, Focani, Bucu t ta a s Apar i rezolvitori cu profesii mai deprtate de matematic: un preot din s a a profesor de limba francez din Bacu, o persoan ce semneaz cu "Vrfu a a a a Cu o munc susinut i mari sacrificii materiale, redactorii au scos a t a s asigurat revistei un nivel de calitate nalt. Ei sunt pe deplin contieni s t obinute i de faptul c prin munca lor au apropiat momentul apariiei lucr t s a t a Acest lucru rezult fr echivoc din cuvntul redaciei "Ctr cetitori" la n a aa t a a al VI-lea:

Greutile materiale fac ca odat cu publicarea numrului din decembr at a a exact dup ase ani, revista s nceteze s apar. Se cuvine s menionm c as a a a a t a principal al "Recreaiilor tiinifice" a fost C. Climescu. Pe coperta reviste t s t an este scris: "Redacia si Administraia la Dl. C. Climescu, Profesor la t t t Stiine, Strada Butu 22". Aceeai adres apare i n casetele ce urmeaz t s a s a numr din ultimul an de apariie (cum se poate vedea i n reproducerea d a t s 4

Foti rezolvitori ai "Recreaiilor tiinifice" i vor aduce aminte cu re s t s t s aceasta. Peste timp, mari matematicieni romni vor avea cuvinte de ap curajul, sacrificiile i fapta celor care "au tras cea nti brazd". s a a Peste nici apte ani de la dispariia lor, "Recreaiile tiinifice" i afl s t t s t s a n "Gazeta matematic", ai crei fondatori i colaboratori au tiut i au re a a s s s imense greuti i obstacole i s fac din visul cercetrii originale o rea at s s a a a i-a serbat un veac de existen nentrerupt i este mereu tnr. s ta as aa

Prof. dr. Temistocle B

Recreaii tiinifice t s t prezen n contiina posteritii ta s t at

Rezolvirea problemelor este unul din cele mai bune stimulente pentr cineva ctre studiul matematicilor. Experiena noastr personal ne pr a t a a acesta. Mai muli dintre noi datoresc acest gust revistei Recreaii Sti t t aprut n timp de 6 ani la Iai i pe care noi ncercm a o continua. a s s a Redacia ["Gazetei matema t

Pe cnd la Bucureti se petreceau aceste prefaceri de societi tiinifice s at s t tiine din Iai se hotrsc s scoat o revist tiinific, i anume Recrea s t s aa a a as t a s care a aprut cu mari greuti ase ani, de la 1883-1888. Revista consacra a at s parte matematicilor i propunea probleme pentru folosul liceenilor. Ace s contribuit mult la rspndirea gustului pentru studiul matematicilor la noi a Ion Ion

Cea dinti ncercare de a iei din acest impas, de a rupe ineria, de a s t curent de preocupare tiinic i de a crea astfel un nceput de atmosfer s t as a voltrii tiinei matematice, a fost fcut la Iai prin publicarea Recreaiilo a s t a a s t t Gheorghe Ti

Omagiu pios primilor pionieri ai studiilor matematice la noi; omagiu pi reviste matematice Recreaii Stiinice i vrednicilor ei fondatori i t t s s omagiu Gazetei Matematice i acelor care au ntemeiat-o i susinut-o pr s s t lor pn azi. a Gr. G. Stratile

"Recreaiile tiinifice" au aprut n 1883 viznd obiective de o nelep t s t a t cinste fondatorilor revistei. [...] Discret i oarecum neateptat cum a apru s s a decembrie 1888, prima revist tiinific romneasc menit s deschid d as t a a a a a originale. Ilie Popa [6 5

Pe lng nvmntul matematic secundar i superior, o contribuie a ata s t dezvoltarea tiinei matematice la noi n ar i a interesului faa de a s t t a s t au adus-o, n perioada aceasta de pregtire, cele dou reviste care au a a i Bucureti: "Recreaii tiinifice", care a durat din 1883 pn n 188 s s t s t a matematic", nfiinat la 15 septembrie 1895. a t a George St. Ando n ara noastr, publicaiile periodice destinate propagrii gustului i com t a t a s tru propunerea i rezolvarea de probleme matematice aniverseaz un secol s a n 1883 a aprut la Iai, prin dragostea i devotamentul unui grup de a s s seam, profesori universitari sau secundari, ingineri, fizicieni, medici, revi a Stiinifice", creia Universitatea din Iai i Societatea de Stiine Matema t a s s t Romnia i-au aniversat centenarul apariiei la sfritul anului 1983. t s Nicolae Teodo Revista i-a ntrerupt activitatea pe neateptate. Ea a reuit s tre s s s a interes pentru matematici n rndul elevilor i studeniolor. Mare parte din c s t formai de "Recreaii tiinifice" au devenit profesori i ingineri cu o serioa t t s t s matematicile elementare. Nicolae Mihilean a 1. 2. 3.

4. 5. 6. 7. 8. 9. 10. 11.

Bibliografie Recreaii tiinice (1883 - 1888) - colecia revistei. t s t t *** - Introducere, Gazeta matematic, an.I, nr.1, septembrie 1895 a este autorul acestei "Introduceri" - sarcin ncredinat de redacia G.M.) a t a t I. Ionescu - Constituirea, administrarea si redactarea Gazetei Matema aprut n volumul Gazeta Matematic, 1895-1935. Istoric - nvtminte (v a a a a Biblioteca Gazetei Matematice, vol. XI, Bucureti, 1935. s t t Gh. Tieica - Rolul Gazetei Matematice n dezvoltarea stiinei matema nia, ibidem. Srbtorirea celor 40 ani ai "Gazetei matematice". Cuvntarea D-lui profeso a a lescu, G.M. XLI (1936), 361-374. I. Popa - Recreaii stiinifice - precursoare a Gazetei Matematice, t t A, 9/1955, 492-493. I. Popa - Dezvoltarea matematicii, aprut n Contribuii la istoria dezvoltr a t a din Iai, vol. II, Bucureti, 1960. s s t a G. St. Andonie - Istoria matematicii n Romnia, vol. I, Ed. Stiinic, Bu N. Teodorescu .a. - Probleme din Gazeta Matematic, Ed. Tehnic, B s a a (citat din Prefa, semnat de acad. N. Teodorescu, preedintele S.S.M.R ta a s Gh. Banta - O pagin din istoria matematicii romneti: centenarul revi s a s t stiinice", Probleme de istoria i lozoa tiinei, vol. X, Acad. R.S.R., l s s t N. Mihileanu - Revistele de matematici elementare din Romnia (pn a a Gil, Zalu, 1995. a 6

Cteva curbe celebre i importante s 1. Cisoida lui Diocles1


Se d un cerc, pe care se ia un punct A; fie AB a y diametrul ce trece prin acest punct i T T 0 tangenta s n B; prin A se duce o secant care taie cercul n H a Q i tangenta n G; pe secant se ia, cu ncepere de s a la punctul A, o lungime AM egal cu HG - poria t M unea de secant dintre cerc i tangent - ; locul a s a geometric al punctelor M este Cisoida lui Diocles. A P S nsemnm prin R raza cercului; originea de coa a ordonate s fie A; direcia diametrului AB s fie luat a t a a ca ax de x i perpendiculara n A pe acest diametru a s s fie axa de y. a Fie M un punct al Cisoidei, ale crui coordonate snt a x = AP i y = M P . Prin ipotez avem AM = HG, s a de unde rezult AP = DB. n triunghiul dreptunghiu AHB, avem a apoi triunghiurile asemenea AM P i AHD dau s HD AD HD 2R x = sau = . MP AP y x Eliminnd HD ntre relaiunile (1) i (2), avem t s (2R x) y 2 = x3 sau x3 + xy 2Ry 2 = 0. HD2 = AD DB = x (2R x) ;

Aceasta este ecvaia Cisoidei. t Curba este simetric n privirea axei de x, cci la fiecare valoare dat a a a punde pentru y dou valori egale i de semne contrare. Ea se compune di a s indefinite, egale ntre ele, situate de o parte i de alta a axei de x. s Originea A este un punct de napoiere de specia ntia. n adevr, dac a a la teoria punctelor multiple, tim c coordonatele unui asemenea punct, sa s a curbei i ntilor derivate pariale; avem s a t f (x, y) = x3 + xy 2 2Ry 2 ,
0 fx = 3x2 + y 2 ,

coordonatele punctului A snt x = 0 i y = 0, care substituite n aceste re s f (0, 0) = 0,


1

0 fy = 2xy 4R

0 f0 = 0,

0 f0 = 0,

Articol preluat din "Recreaii Stiinice", an II (1884), nr. 1, 19-23. t t Ciclul "Cteva curbe celebre i importante" cuprinde 9 lecii prezente n numerele d s t care sunt expuse principalele curbe plane: Cisoida lui Diocles (care deschide ciclul), conc de, melcul lui Pascal , strofoida, ovalele lui Cassini, cicloide, epicicloide, conice i spiral s S-au pstrat termenii de matematic din textul original, dar s-au f cut modificri n a a a a i ortografiei (astfeli - astfel, valor - valori, da - dau, adec - adic, sa - s-a etc). s u u a a

ceea ce nsemneaz c originea este un punct duplu; apoi dac formm e a a a a telor n acest punct gsim y 2 = 0; adic tangentele n acest punct se con a a de x. Aadar, originea este un punct duplu de napoiere i de ntia spec s s a este de o parte i de alta a tangentei. s Cisoida admite o asimptot paralel cu axa de y, cci tim c aceste a a a s a capt egalnd cu zero coeficienii celei mai nalte puteri a lui y, ceea ce d aa t de unde x = 2R, adic tangenta la cercul T T 0 este asimptot Cisoidei. a a Asimptote neparalele cu axele nu snt. Cercul dat este numit cercul director al Cisoidei. Newton a dat Cisoidei urmtoarea descripie mecanic. a t a Fie un punct fix A i o dreapt fix T T 0 ; s a a din A se duce perpendiculara AD pe dreapta fix; apoi se imagineaz un unghi drept care a a se mic astfel c una din laturile lui trece prin s a a M punctul A, iar extremitatea celeilalte laturi, - A O R luat egal cu AD -, se razim pe dreapta a a a fix; dac G este vrful unghiului drept i H a a s G extremitatea laturii a doua, punctul M din mijlocul laturii GH descrie Cisoida. S lum mijlocul O al dreptei AD i din punca a s tul D ca centru cu DO ca raz s descriem un cerc; s unim AH, apoi a a a mijlocul laturii GH. Vom demonstra mai nti c dreptele AH i OM snt paralele ntre e a s triunghiurile dreptunghe AGH i ADH snt egale cci AH = AH, apo s a conform enunciului. De aici rezult AG = DH i fiindc unghiurile ARG a s a egale ca opuse la vrf, apoi rezult c i triunghiurile dreptunghe AGR a a s egale ntre ele. Din egalitatea acestor dou din urm triunghiuri rezult GR = RD i a a a s = GM , apoi mai rezult OR = RM i AO = HM i prin urmare drepte a s s snt paralele. Fiindc punctul O este mijlocul dreptei AO, din paralelismul acestor dr a c OM taie pe HD ntr-un punct care-i la mijlocul dreptei HD. a S artm acum c triunghiurile HIM i DIM snt egale; n adevr, a a aa a s a HM = DO = DN ; apoi ungh.N ID = ungh.HIM ; pe urm succesiv a = ungh.M OR = ungh.HAD = ungh.AHG = ungh.HM I, aadar s fiindc OI = IK, apoi rezult IM = IN i prin urmare OM = N K. a a s Aadar, n micarea unghiului drept AGH, punctul M descrie o Cisoid s s director este acel descris din D ca centru cu DO ca raz. a

Cisoida a fost imaginat de Diocles (500 a. Ch.) pentru a rezolvi prob a medii proporionale1 . Iat cum se rezolvete aceast problem. t a s a a
1

Sublinierele din aceast fraz nu apar i n textul original. a a s

Ecvaia (3) fiind rezolvit n privirea lui y d: t a a x2 . y=p x (2R x) Lum semnul + naintea radicalului fiindc considerm ramura de deasupr a a a S nsmnm prin z ordonata punctului de pe cerc al criu abscis est a a a a a 2 z = x (2R x) . Comparnd ecvaiile (4) i (5) avem raporturile t s 2R x z x = = . z x y Fie a i b liniile ntre care se cere a se afla dou medii proporionale. s a t b a S nmulim terminii raporturilor (6) prin , ceea ce d a t y (2R x) b bz bx y y y = = . bz bx b y y S lum pe Cisoid un punct astfel ca s avem a a a a (2R x) b = a, y ceea ce revine a considera punctul comun Cisoidei, - reprezentat prin ec a dreptei - reprezentat prin ecvaia (8) -; atunci raporturile (7) se pot scrie a t bz bx a y y = = . bz bx b y y bx a bz = i s = , vom avea = = . Aceste trei raportur S punem a y y b bz bx i ecvaii, din care vom scoate pe i . Astfel cantitile t s at s vor f y y aceste vor fi cele dou medii proporionale ntre a i b. a t s Dac Cisoida este construit, pentru a gsi punctul de pe ea cu ajutoru a a a rezolvi problema, trebuie s construim dreapta (8), care este o dreapt a a punctul B, pentru aceea lum o lungime BK = a, rdicm perpendicu a a a dreapta BL este dreapta (8). Punctul M n care aceast dreapt taie a a punctul cutat, i avem a s AP = x, M P = y, QP = z; QP i b prin urmare cele dou medii proporionale ntre a i b vor fi b a t s s MP (Va urma)

C. CLIME
9

Scrierea numerelor cu litere chirilice2

Toate popoarele, afar de vechii chinezi i de un trib puin cunoscut de a s t Aristot3 au adoptat sistema de numeraie zecimal, n care numerele sn t a perioade de cte zece uniti. Baza, n aceast sistem de numeraie, es at a a t zece uniti de un ordin oarecare trebuiesc ca s formeze o unitate de o at a mai nalt. Scrierea oricrui numer se face astzi numai cu zece semne, numite cif a a nou reprezint pe cele dinti nou numere ntregi, i care snt : a a a s 1 unu, 2 doi, 3 trei, 4 patru 5 cinci, 6 ese, s 7 epte, s 8 opt,

9 nou

Semnul al zecelea este 0 sau zero, prin care se arat lipsa de unit a at oarecare. S-a admis c orice cifr pus la stnga unei alte cifre reprezint unit a a a a a mai mari dect aceasta din urm, adic uniti de ordinul imediat superior a a at dnd acestor semne, diferite locuri se poate exprima, ntr-un chip nu se po orice numer, fie orict de mare numerul unitilor din care el se compune. at Semnele acestea le-am mprumutat, pe la mijlocul secolului al X, de l dnii pare c le-au luat de la indieni, adevraii ntemeietori ai tiinei ct s a a t s t n vechime, era obeciul de a se reprezenta numerele cu litere. Grecii nsemnau numerele cu literele din alfabetul lor; diferite ordine de u zeci, sute, . . . , se deosebeau unele de altele prin accente puse deasupra lit Romanii, pentru scrierea numerelor, ntrebuinau epte semne principa t s I unu, V cinci, X zece, L cincizeci, C o sut, a D cinci sute,

M o mi

Cu ajutorul acestor litere, i prin chipul combinrei lor fcut dup ni s a a a a statornicite, se pot scrie numere din o mie i orice numer de sute, zeci i u s s n crile romneti vechi, n cronici precum i n crile religioase, at s s at hrisoave, se gsesc numere scrise cu litere chirilice. a n reprezentarea chirilic a numerelor, unimilor, zecile i sutele se nseam a s se vede n tabela de mai la vale, fiecare cu cte un semn deosebit, o liter a chirilic avnd deasupra lor un semn, poate spre a arta c litera este luat a a a Astfel, pentru a scrie un numer oarecare, mai mic dect o mie, n no trebuiesc 27 semne. Aceleai litere, avnd semnul s n stnga lor i puin mai jos, serves s t unitile din clasa miilor, adic unimile de mii, zecile de mii i sutele de m at a s Alte rnduri de semne convenionale, puse alturi cu aceleai litere, ar t a s spre a reprezenta unitile din celelalte clase mai nalte, clasa milioanelor, at
2 3

Articol preluat din "Recreaii Stiinifice", an I (1883), nr. 3, 57-60. t t Bossut, Istoria general a matematicilor. a

10

11

. . . ; totui, numerele exprimnd milioanele precum i ordine mai nalte urm s s n cuvinte. Caracteristic este modul cum se potrivete de bine scrierea numerelor n s lic cu numirea lor. Se tie c, numerele cuprinse ntre zece i douzeci prezi a s a s a n ceea ce privete numirea lor, de la regula general ntemeiat pe modu s a a Pe cnd la numerele de la douzeci nainte, compuse din zeci i unimi, s a s nti zecile i apoi unimile, dac snt , ca de exemplu n: patruzeci i patr s a s nou etc., la numerele cuprinse ntre zece i douzeci, se enun mai nti u a s a ta zecile: treisprezece, optsprezece etc. n reprezentarea chirilic a numerelo a ntocmai dup cum se enun: unitile se scriu nainte sau dup zeci, dup a ta at a numirea numerului. Este lesne de vzut c numerele exprimate cu litere se pot supune c a a numerele scrise cu cifre arabe; singura deosebire st ntr-aceea c calculul a a nti, cu atta mai lung i mai greu, cu ct numerul va fi mai mare. s

I. M. M

12

Acad. Radu Miron la a 75-a aniversar

Academician profesor doctor docent Radu Miron.....ce se mai poa n cteva rnduri!? Si totui...profesorul Radu Miron este preferatul s raii de studeni ai Facultii de Matematic din Iai. Prezena carism t t at a s t teatru, reuete sa captiveze n mod natural auditoriul. Cu mult calm s s remarcabila, cele mai ntunecate capitole ale matematicii se limpezes tra opac din spatele oglinzii capt o transparena de cristal chiar i a aa t s disimulat n student. Si totui...profesorul R. Miron arunca peste noi, ca un prestidigit s imensa care este opera sa, opera care conine peste 250 de lucrari t t s bibliografice, mongrafii etc. Trind peste 50 de ani n atmosfera Seminarului Matematic Al. My a varat laborator de creaie tiinica n cmpul abstract al matematicii, t s t demica de nalta inuta, profesorul R. Miron este un continuator al acelei t aur a matematicienilor romni. Menionm aici numele lui Alexandru M t a torul Seminarului Matematic, Octav Mayer, Gheorghe Vranceanu, G Mendel Haimovici, Adolf Haimovici, Dimitrie Mangeron, Constantin C Popa, Gheorghe Gheorghiev . a. s Remarcat de profesori nc din primii ani de studenie este num a t anul II, iar dup absolvirea facultii este ncadrat ca cercettor la a at a Matematica, Filiala Iai a Academiei. In 1957 i susine teza de do s s t tlul Problema geometrizarii sistemelor mecanice neolonome, sub con demicianului Mendel Haimovici, lucrare publicat n ntregime n rev a Cercetri Matematice. a Parcurgnd ntreaga ierarhie universitara este numit n 1969 profes de Geometrie a Facultaii de Matematica din Universitatea Al. I. Cuza t director al Institutului de Matematic. Decan al Facultii de Matema a at 1972-1976, conductor de doctorat din 1972, ef al Catedrei de Geome a s al Consiliului Profesoral i al Senatului, profesorul R. Miron a desfau s s activitate didactica i educativa. s Activitatea de cercetare tiinific a profesorului R. Miron este bine s t a lumea ntreag. Este stabilit i recunoscut contribuia sa important a as a t la dezvoltarea Geometriei Difereniale moderne i a aplicaiilor ei n Fiz t s t Profesorul R. Miron a creat i dezvoltat n matematica noi ramuri s conguraiilor Myller, teoria invarianta a spaiilor Finsler, spaii La t t t Lagrange generalizate, teoria subspaiilor Lagrange, geometria spaiilor t t ordin superior, spaii Hamilton, spaii Hamilton generalizate, teoria t t spaiilor brate, teoria lagrangeana a relativitatii si electromagnetism t Einstein si Maxwell. A rezolvat multe probleme deschise ca: prelungire riemanniene, nsleriene, lagrangeene, spaii Finsler de ordin superior t Cercetarile iniiate i dezvoltate de profesorul R.Miron au avut un t s asupra specialitilor n geometrie i nu numai. Profesorul Makoto Mat s s Universitatea din Kyoto, n cartea sa Fundamentele Geometriei Fin 13

Finsler speciale se refer la conceptul de reper Miron i ecuaii fund a s t reperului Miron. Profesorul Masao Hashiguchi atribuie numele de "s unui caz remarcabil de spaii Hamilton, introduse pentru prima dat t a R. Miron, iar G. S. Asanov, de la Universitatea din Moscova, aplica mode ale profesorului R. Miron n cosmologie i obine cea mai bun devia s t a t periheliului planetelor Marte, Venus i Mercur. Bazndu-se pe teoria s R. Miron, G. Beil (S.U.A.) a obinut o bun teorie gauge i P. L. Anton t a s a aplicat-o n biologie. Profesorul R. Miron a creat n Romnia o coal de matematic d s a a care a cooperat pe parcursul multor ani cu oameni de tiina din Ja s t S.U.A., Germania, Italia, Anglia, Canada, Ungaria, Egipt etc. Un nu doctoranzi din ara i strainatate (Japonia, Italia, Ungaria, Vietnam t s titlul de doctor n matematic sub conducerea d-sale. a Avnd un renume deosebit n lumea Matematicii, profesorul R. Miro tat ca visiting professor la prestigioase universiti ca: Universitatea at (Japonia, 1988, 1990, 1992 ), Bary (Italia, 1987 ), Freiburg si Mnche 1975, 1990 ), Edmonton (Canada, 1992 ). Profesorul R. Miron a publicat o parte din lucrrile sale n colab a ometri japonezi: M. Matsumoto, M. Hashiguchi, Y. Ichijio, S. Kikuch be, S. Ikeda sau cu membrii ai Seminarului Naional de Geometrie F t grange, iniiat de dnsul n 1980 la Universitatea din Braov. t s Este primul preedinte al Societii Balcanice a Geometrilor cons s at iativa sa i a profesorului G. Tsagas de la Universitatea Aristotel din t s (Grecia) n 1994-1995. Instituto per la Ricerca di Base din Italia i fesorului R. Miron titlul de Full Professor in the Division of Mathe I.R.B.. Profesorului R. Miron i s-a acordat Premiul Ministerului Educaie t miul Gh.Tieica al Academiei (1968) si a fost ales membru al Acade t (1991). Profesorul R. Miron a scris n colaborare cu profesorul M. Anast de pionerat Geometria spaiilor Lagrange: teorie si aplicaii , publica t t Kluwer Academic (S.U.A.) n prestigioasa serie Fundamental Theorie Mai amintim dou titluri importante de monografii care concentreaz a a matice ale profesorului R.Miron: Geometria spaiilor Lagrange de or t Aplicaii n Mecanica si Fizica (Kluwer Academic, 1997) i Geome t s Finsler de ordin superior (Hadronic Press, U.S.A., 1998). Impreun cu profesorul P. L. Antonelli de la Universitatea din Alber a R. Miron este editorul crii Geometrie Finsler si Lagrange.Aplicai at t Biologie, publicata deasemeni de Kluwer Academic n 1996. Chiar daca pe 3 octombrie 2002, profesorul Radu Miron a mpl vrst de 75 de ani, sunt convins c "surprizele matematice" vor co a a uimeasc. a

Prof. dr. Alexandru N

14

Numere prime din progresii aritmetic


Petru Minu1 t

Un numar natural p, p > 1, se numete numar prim daca nu are s nafara de 1 i p. s Lem. Un numar natural n, n > 1, are un divizor prim. a Demonstraie. Fie M mulimea tuturor numerelor naturale care t t ai lui n diferii de 1. M 6= , deoarece n M . n M exist un numr t a a mai mic, p. Artm, prin reducere la absurd, c p este prim. Presupun aa a compus: p = ab, 1 < a < p. Din a | p i p | n rezulta ca a | n. Am gasit s lui n mai mic ca p ceea ce contrazice alegerea lui p.

Teorema 1. n mulimea numerelor naturale exista o infinitate de n t Demonstraie. Exist numere prime. De exemplu 2, care nu po t a divizori n afar de 1 i 2. Folosim metoda reducerii la absurd. Presupu a s o mulime finita de numere prime n N, P = {p1 , p2 , . . . , pk }. Conside t ajutator N = p1 p2 . . . pk + 1. Deoarece N > 1, exista un numar prim p P rezult c p | p1 p2 . . . pk . Dac dou numere sunt multipli de p, at a a a a lor este multiplu de p. Rezult c p | 1, ceea ce implic p = 1 i contraz a a a s numarului prim. Observaie. Teorema 1 o gsim enunat i demonstrat pentru p t a t as a opera lui Euclid "Elemente" (sec. III . Ch.) i este cunoscuta sub s teorema lui Euclid. Se cunosc numeroase demonstraii ale acestei teore t

Singurul numar prim par este 2. Aranjam numerele impare n doua 3, 7, 11, 15, . . . , 4k 1, . . . 1, 5, 9, 13, . . . , 4k + 1, . . .

Constatam ca n aceste iruri (progresii aritmetice), mergnd pna la ter s tot mai mare, gsim noi termeni care sunt numere prime. Dac lum i a a a s aritmetice, de exemplu: 3, 13, 23, 33, 43, 53, 63, 73, 83, . . . 2, 7, 12, 17, 22, 27, 32, 37, 42, 47, . . .

constatm acelai lucru. Este uor de demonstrat c n progresia (1) exis a s s a de numere prime.

Teorema 2. Exista o infinitate de numere prime de forma p = 4k Demonstraie. Procedam prin reducere la absurd. Am pus dej t cteva numere prime de aceast form (irul (1)). Presupunem c exi a a s a a finit de numere prime de acest fel: p1 , p2 , . . . , pn . Construim numrul = 4p1 p2 . . . pn 1. Deoarece N > 1, exist p prim, p | N . Orice numr a a de 2 este de forma p = 4k 1 sau p = 4k + 1. Daca toi divizorii primi t de forma p = 4k + 1, numrul N este de forma N = 4h + 1, deci 4 | N a ceea ce implic 4 | 2. Contradicie! Exist divizori primi ai lui N de form a t a
1

Prof. dr., Univ. "D. Cantemir", Tg. Mure s

15

p un asemenea divizor. Rezult c p {p1 , p2 , . . . , pn }, deci p | 4p1 p2 a a p | N rezult p = 1. Contradicie! Presupunerea c exist un numr fin a t a a a prime de forma p = 4k 1 nu poate fi adevrat. a a Teorema 2 se generalizeaza dupa cum urmeaza: Teorema 3. Pentru orice numar natural n, n 6= 0, exista o infinita prime p de forma p = nk 1, k N. Demonstraie. Pentru n = 1, {nk 1 | k N} = N {1} i a t s remei este adevrat (teorema lui Euclid). Pentru n = 2, {nk a a = {1, 1, 3, 5, 7, . . . } i armaia teoremei este adevrat (exist o in s t a a a mere prime impare). Pentru demonstraia teoremei n cazul n > 2 vom folosi lema urma t Lem. Pentru orice numar natural n, n > 1, avem: a Y 1r<n r 1 (mod n) .
(r,n)=1

proprietatea c r2 1 (mod n) avem r (n r) 1 (mod n). Rezult a a k a nti al congruenei (5) este congruent cu (1) , unde 2k este numru t proprietatea r2 1 (mod n) (r i n r au ambii aceasta proprietate s demonstrata. Revenim la demonstraia teoremei n cazul n > 2. Vom arta pri t a absurd, c exist o infiniate de numere prime p de forma p = nak 1 a a a este produsul numerelor naturale mai mici ca n i prime cu n luat s sau dupa cum produsul acestor numere este congruent cu +1 sau Presupunem c exist un numr finit de numere p de forma p = nak1: a a a Considerm numrul ajuttor N = nap1 p2 . . . ps 1. N > 1 deoarece a a a s = 0, N = na 1 > n 1 1. Exista p prim, p | N ; p este de forma (r, N a) = 1. Din p = N au + r nu + r (mod n) rezulta ca p i nu + s rest la mprirea cu n. Rezult c r < n i nu putem avea 1 < r < n at a a s rezulta c r | n sau r | a, deci r | p i contrazicem faptul c p este prim. a s a p este de forma p = nau 1. Daca toi divizorii lui N ar fi de forma p t ar fi i el de aceasta forma. Prin urmare, exista un divizor prim p al lu s p = nau 1. Rezult c p {p1 , p2 , . . . , ps }, p | nap1 p2 . . . ps i cum p a a s p | 1, deci p = 1 i contrazicem definiia numrului prim. s t a Teorema 2 se obine din Teoremei 3 lund n = 4. Din Teorema 3 rez t o infinitate de numere prime p de forma p = 6k 1, k N sau p = 8 .a.m.d. s Pentru a arta c progresia (2) conine o infinitate de numere prim a a t urmatoarea 2 Lem. Oricare ar fi numarul natural n, n > 1, numarul (n!) + a primi si acetia sunt de forma p = 4k + 1. s 16

Demonstraie. Pentru n = 2 congruena este evidenta. Pentru n t t 1 r < n, (r, n) = 1, tim ca exista soluie unica pentru congruena rx s t t Deci, exist un singur r0 , 1 r0 < n, (r0 , n) = 1 astfel nct rr0 1 a membrul nti al congruenei (5) nlocuim produsele rr0 cu 1 pentru t Q Q care r0 6= r. Rezulta ca (r,n)=1 r r. Observam ca (r,n)=1
r2 1(mod n)

Demonstraie. Pentru n > 1, numrul (n!) + 1 este impar, m t a 2 Exist p | (n!) + 1, p 6= 2. Deci p este de forma p = 4k + 1 sau p = 4k a 2 2(2k+1) a +1, adic a este de forma p = 4k+3, din p | (n!) +1 rezult p | (n!) p i apoi p | (n!) + n!. Conform cu mica teorema a lui Fermat p | (n!)p s p | 2n!, deci p n, p | n! ceea ce implica p | 1, contradicie! t Teorema 4. Exista o infinitate de numere prime p de forma p = 4 Demonstraie. Folosim din nou metoda lui Euclid. Presupunem t numr finit de numere prime de forma 4k + 1: p1 = 5 < p2 < < ps a numrul ajuttor N = [(p1 p2 . . . ps )!]2 + 1. Acesta admite, conform lem a a prim p de forma p = 4k + 1 i ajungem din nou la contradicia p | 1. s t Pentru generalizarea Teoremei 4 avem nevoie de cteva chestiuni pre
2h

k un numr natural, k 1. Ecuaia xk 1 = 0 are rdcinile xh = e k a t a a Q 2h i sin , h = 0, 1, . . . k 1. Consideram polinomul Fn (x) = (h,n)=1 k unde produsul se face dupa numerele h {0, 1, . . . , n 1} care sunt Gradul lui Fn (x) este (n) ( (n) = numarul numerelor naturale ma prime cu n, este cunoscut sub numele de funcia indicatoare a lui Euler a t Q c xk 1 = n | k Fn (x) (produsul se face dup divizorii pozitivi a a a xk 1 = Fk (x) Gk (x), unde Gk (x) este cel mai mic multiplu comun al xn 1, n | k, n < k, avnd coeficientul termenului de grad cel mai t s 1. Deoarece Gk (x) este un polinom cu coeficieni ntregi, atunci i F polinom cu coeficieni ntregi. Observam ca pentru orice numar ntre t avem Fk (x) Gk (x) 6= 0. Lema 1. Fie n un divizor propriu al k ( n 6= 1, n 6= k). Pentru lui xk 1 ntreg x, x 6= 1, avem: xn 1, n | k. x 1 n Demonstraie. Notam k = nd, x 1 = y. Vom avea: t
d

(y + 1) 1 xk 1 1 = y d1 + Cd y d2 + + d d (mod = xn 1 y xk 1 Daca = xn 1, n , din | y rezulta | d i, cum d | k, rezulta s x 1 Lema 2. Fie x Z, x 6= 1. Orice divizor prim, comun lui Fk este un divizor al lui k. Demonstraie. Fie p prim, p | Fk (x), p | Gk (x). Din p | Gk (x) rez t Q n N , n | k, n < k, astfel nct p | Fn (x) (deoarece Gk (x) = n | k dac un numr prim divide un produs atunci el divide cel puin unul d a a t xk 1 xk 1 i p | xn1 , n . Co s Din p | xn 1 i p | Fk (x) rezulta p | n s x 1 x 1 1, p | k. Teorema 5. Pentru orice numar natural k, k 1, exista o infinita prime de forma p = nk + 1, n N. Demonstraie. Pentru k = 1 enunul teoremei este adevrat (teo t t a clid). Pentru k > 1, artm mai nti c exist numere prime de form aa a a 17

Pentru x = ky, y Z vom avea: Fk (x) Gk (x) = xk 1 1 (mod a a a ecuaiile Fk (x) = 1 au un numr finit de rdcini, putem alege y t Fk (x) 6= 1. Exist numere prime p care sunt divizori ai lui Fk (x). D a (p | k p | x p |1), rezulta (conform Lemei 2) ca p - Gk (x) i dec s oricare ar fi numrul natural n, n | k, n < k. Deci xn 6 1 (mod n), n a xk 1 (mod n). Fie n = (k, p 1). Exist dou numere ntregi s i a a s k s p1 t n x 1 (mod p). Nu putem n = sk+t (p 1). Rezulta ca x = x deci n = k. Conform cu mica teorema a lui Fermat xp1 1 (mod p p 1 este multiplu de k. ntr-adevr, dac este cea mai mic putere n a a a pozitiva a lui x astfel nct x 1 (mod p), atunci xa 1 (mod p) b a = b, atunci xa = x 1 (mod p). Daca xa 1 (mod p), a = b + nu putem avea > 0 deoarece xa xr 1 (mod p) i contrazicem a s Deci, p 1 = nk, adic p = nk + 1. a a a Fie p1 un numr prim de forma p1 = n1 k + 1. Lum k1 = p1 k. Co pari a demonstraiei exista un numar prim p2 de forma p2 = np1 k + 1, t t orice numar prim p1 de forma p1 = nk + 1 exista un numar prim p2 de a p2 > p1 . Rezult c exist o infinitate de numere prime p de forma a a a n N. Enunul cel mai general, care cuprinde drept cazuri particulare to t prezentate, l constitue teorema urmtoare, cunoscut n literatura ma a a denumirea de teorema lui Dirichlet. Teorema 6. Oricare ar fi numerele l Z si k N , (l, k) = aritmetica l, l + k, l + 2k, . . . l + nk, . . . conine o infinitate de numere prime. t Condiia (l, k) = 1 este necesar . Dac (l, k) = d > 1, toi terme t a a t sunt multipli de d. Demonstraia Teoremei 6, n cazul general, nu p t prin metode ale matematicii elementare. Problema numrului de numere prime dintr-o progresie aritmetic a pentru prima oar n 1775 de Leonard Euler n cazul particular l = a sa "Thorie des nombres" A. M. Legendre a dat o demonstraie Teore t pe o ipotez, care ulterior s-a dovedit a fi fals. Prima demonstraie a a t fost dat n 1837 de Lejeune P. G. Dirichlet care a creat un aparat an a (seriile Dirichlet). Demonstraia lui Dirichlet este considerat actul t a teoriei analitice a numerelor.

Bibliografie 1. I. Creang, C. Cazacu, P. Minu, Gh. Opai, C. Reischer - Introdu a t t numerelor, Editura didactic i pedagogic, Bucureti, 1965. as a s 2. Hua Loo Keng - Introduction to Number Theory, Springer Verlag, B berg, 1982. 3. P. Minu - Teoria numerelor. Capitole introductive, Editura "Creng t Iai, 1997. s 4. C. P. Popovici - Teoria numerelor, Ed. didactic i pedagogic, Bucur as a 5. W. Sierpinski - Ce stim si ce nu stim despre numerele prime, Editu Bucureti, 1966. s 18

Recreaie matematic i nu numai t as


Horea BANEA1

Este cunoscuta urmatoarea problema - joc: Sa se descompuna poligo alaturata prin doua linii drepte astfel nct din poligoanele obinute pri t se realizeze un patrat. Soluia este indicat n figur. t a a

Sugerat de aceasta, propunem urmtoarea problem: a a Dintr-o bucata de carton, de forma poligonului de mai sus, printr-o o dreapta si realipirea bucatilor obinute se realizeaza diferite figuri d t poligoane convexe. Sa se gaseasca toate situaiile distincte. Sa se calcu t laturilor poligoanelor obinute. t n legtur cu enumerarea propus facem urmtoarele precizri: a a a a a Situaii distincte sunt cele n care T = taietura i/sau P = poli t s difera ntre ele. Cazurile n care cu aceeai T obinndu-se componente simetr s t realiza acelai P n diferite moduri, vor fi considerate doar variante e s aceleiai situaii cu excepia cazurilor n care alipirea aceleiai compo s t t s la alt segment al componentei de baza (=cea mai mare) care vor fi c situaii distincte. t Cnd T este variabil, obinndu-se acelai tip de P dar cu dimens a t s depinznd de un parametru, se considera ca o singura situaie, dar cazur t ale parametrului care conduc la P cu anumite particularitai se enumer t cazul general. Nu se enumer, fiind socotite variante echivalente, figurile obinute a t rea pe verso a ntregii figuri obinute ntr-un caz. t Pentru a uura urmarirea efectuarii taieturilor indicam grus parea lor n raport cu anumite puncte remarcabile prin care au fost duse: M : T116 ; N : T1722 ; P : T23 ; Q : T25,26 . Enumerarea lungimilor laturilor se face ncepnd cu cea superioara, n sens matematic. Justicarea calculelor, bazndu-se doar pe teorema lui Pitagora i pe asemnarea triunghiurilor, se las p s a a Trapez dreptunghic variabil: p 5, y, 4y 2 8y + 29, 2 y; Triunghi dreptunghic: 5,

29, 2

Conf. dr., Univ. Transilvania, Braov s

19

Patrulater inscriptibil:

19 4 , , 5 5

4 3 29 4 2 2 Pentagon: 3, , , , 5 5 5 5 Trapez dreptunghic variabil: p 3 + x, 4x2 + 4x + 5, 2 x,

Hexagon: p 2x + 2 3, 4 x2 , , 2 x, x x 4 3 x 1, 84, x + x 2, 75x2 Obs. Nu se consider i P5 . as Hexagon: p p 2x + 2 3, 4 x2 , , 2 x, x x ca la P6 . Obs. Aceeai ca la P6 . s 2 8 Trapez dreptunghic: 3, , 13, 3 3

Trapez dreptunghic ortogonal: 4,

x+2 Pentagon: 3, 2 1 x, ,2 x x 0, 89 , 4x3 + x2 3x Obs. Nu se considera i P5 . s

Trapez dreptunghic circumscripti 5+ 5 5 5 , 3, , 2; x = 2 2 Trapez dreptunghic: 7/2, 2 2, 3/ Obs. Realizat prin alipirea alto faa de P5 . t Trapez dreptunghic: 3, 5, 2, 2.

20

Trapez dreptunghic variabil: p 3 x, 4x2 4x + 5, 2 + x, 2; 5 5 Dreptunghi: , 2, , 2. 2 2 Obs. Are variante echivalente.

Trapez dreptunghic variabil: p x, 4x2 20x + 29, 5 x, 2; 0

Trapez dreptunghic circumscripti 5 5 5+ 5 x= , 3, 2 2 Trapez dreptunghic ortogonal: 1,

Triunghi dreptunghic:

29, 5, 2.

Trapez dreptunghic variabil: p 4y 2 8y + 29, y, 5, 2 y; 0 Paralelogram: 5 5 10, , 10, . 3 3

Trapez isoscel:

2 8 10, , 10, . 3 3

Triunghi dreptunghic: 5, 2 5,

Trapez: 4, 2 5, 1, 5.

21

Pentagon: 3,

1 1 , 2 5, , 5. 2 2

Pentagon: 9+2 3 3 3 6+2 3 , , , 2 3 3 3 Pentagon: 3 3 6+2 3 6 3 3, , , 3 3 3 Pentagon: 3, 2 2, 1, 2, 2. Obs. Cele dou triunghiuri i a s locurile. Sunt i variante echivale s 5 5 Dreptunghi: , 2, , 2. 2 2 5 5 , 5, , 5. 2 2 7 3 , 5, , 5. 2 2 5 3 5 , , 2 2

Paralelogram:

Trapez isoscel:

Patrulater inscriptibil:

5 5 3 5 1 Pentagon: , , , , 2. 2 2 2 2

Hexagon inscriptibil cu axa de si 1, 2, 1, 3, 1, 2. Hexagon cu centru de simetrie: 2, 2, 1, 2, 2, 1.

22

Hexagon cu centru de simetrie: 1, 2, 2, 1, 2, 2.

Hexagon inscriptibil cu axa de si 1, 2, 1, 2, 1, 3. 5, 1, 2.

Pentagon: 3, 1,

Pentagon: 3, 2 2, 2, 3 2, Obs. Cele doua truinghiuri i s locurile. Sunt i variante echivale s Pentagon cu axa de simetrie: 2, Obs. Are variante echivalente.

Pentagon variabil cu axa de sime p p 3x, 4x2 4x + 2, 4x2 4x 1/2 < x < 1. Hexagon cu centru de simetrie: 2, 1, 2, 2, 1, 2.

Obs. Are variante echivalente. Hexagon inscriptibil cu axa de si 3, 1, 2, 1, 2, 1. Obs. Are variante echivalente. Dreptunghi: 5, 1, 5, 1. Obs. Are variante echivalente.

Dreptunghi: 5, 1, 5, 1. Obs. Are variante echivalente. * * * Aceast list lung de poligoane (adresm cititorilor provocarea d a a a a altele remarcabile) permite utilizarea ei n diferite moduri adaptate la n tinelor celor carora li se adreseaza i la scopurile urmarite de propunat t s plu, propunem urmatoarele: 1) S se gseasc doar situaiile care dau triunghiuri sau patrulat a a a t 23

particulare: trapeze . a.) s 2) n ce situaie se obine figura cu perimetrul maxim sau minim? t t 3) Ce tipuri distincte de poligoane se pot obine? (triunghi dreptu t unghi, paralelogram, trapez, trapez isoscel, trapez dreptunghic, trapez c bil, trapez ortodiagonal, patrulater inscriptibil, pentagon, pentagon cu trie, hexagon, hexagon cu centru de simetrie, hexagon cu ax de simet a n considerare doar particularitile clasice nelund n considerare p at ca: pentagon cu doua unghiuri drepte . a.). s 4) Care tietur d cel mai mare numr de variante echivalente? a a a a obin acelai poligon; de exemplu T20 d 16 variante cci dac notm t a s a a a a decupate prin I cel de sus i II cel de jos, respectiv prin F (faa) i V ( s t s ele pot fi aezate pentru a forma pentagonul n cele doua poziii S (s s t astfel:

S : IF IF IV IV IIF IIV IIF IIV J : IIF IIV IIF IIV IF IF IV IV i toate aceste combinaii nc o dat numrate dac ntoarcem ntreg p s t a a a a verso. 5) Daca se considera o singura faa a cartonului care situaii nu se t t (De exemplu T2 cu P4 ). 6) Care situaii duc la poligoane congruente? (De exemplu P35 , P39 t 7) De ce nu se pot realiza poligoane convexe cu mai mult de ase la s 8) Rezolvarea aproximativa a ecuaiilor de gradul 3 i 4 care au apa t s pot constitui un pretext pentru a prezenta la o activitate suplimentar a Cardano, respectiv Ferrari). 9) Relativ la taieturi: care este cea mai mica? cea mai mare? cea poligonul n doua bucai echivalente (de aceeai arie)? s t * * * n ncheiere sa revenim la problema niiala adaptnd-o la mulimea t t mai sus i anume: s Se poate ca printr-o singur tietur n linie dreapt s realizm din a a a a a a un ptrat? a Raspunsul este armativ doar daca ndoim n prealabil cartonul aa c s cat mai jos (dupa bisectoarea unghiului drept format de cele doua taietu n soluia sa). t

Dar aceast problem cu ndoire i tiere poate iniia o alt RECREA a a s a t a MATICA.

24

Asupra unor perechi de iruri liniar recur s


D. M. BATINETU-GIURGIU 1

n aceasta nota matematica vom evidenia proprietaile unor pere t t fiecare satisfcnd o anumit recurena liniar omogen de ordinul al d a a t a a cieni constani. t t Spunem ca un ir (xn )n0 de numere reale, satisface o recurena li s t omogena de ordinul al doilea, daca exista a, b R, b 6= 0 i exista s nct: xn+2 + axn+1 + bxn = 0, n k. n funcie de valorile coeficienilor a, b R i de condiiile iniiale t t s t t xk+1 = v R se obin diferite iruri, dintre care unele cunoscute i de e t s s De exemplu, dac a = b = 1, k = 0 se obine recurena: a t t xn+2 xn+1 xn = 0, n N, pe care o vom numi recurena Fibonacci-Lucas. t Daca n recurena (2) consideram x0 = F0 = 0, x1 = F1 = 1, xn = t sirul lui Fibonacci care satisface recurena: t Fn+2 = Fn+1 + Fn , n N. Dac n recurena (2) considerm x0 = L0 = 2, x1 = L1 = 1, xn = a t a sirul lui Lucas, ir care satisface recurena: s t Ln+2 = Ln+1 + Ln , n N. Daca n (1) luam a = 2, b = 1, k = 1 se obine recurena liniara: t t xn+2 2xn+1 + xn = 0, n N xn+2 xn+1 = = x2 x1 = r

numit recurena progresiilor aritmetice de raie r R. a t t n fine, dac n recurena (1) lum a = 2, b = 1, k = 0 obin a t a t liniara cu coeficieni constani de tip Pell : t t xn+2 2xn+1 xn = 0, n N. Dac n recurena (6) considerm x0 = P0 = 0, x1 = P1 = 1, xn = a t a sirul lui Pell, care satisface recurena: t Pn+2 = Pn + 2Pn+1 , n N. De asemenea, daca n (6) luam x0 = Q0 = 1, x1 = Q1 = 1, xn = sirul lui Pell asociat (Qn )n0 , ir care satisface recurena: s t Qn+2 = 2Qn+1 + Qn , n N. Mai departe, vom enuna i demonstra unele propoziii care scot t s t anumite proprieti pe care le verific unele perechi formate dintr-un i at a s recurena (2) i un ir care verifica recurena (5). t s s t Propoziia 1. Daca (xn )n0 satisface recurena (2) n care x0 t t x1 = d R , iar (un )n1 este o progresie aritmetica de raie r R, a t + n X uk xk = un xn+2 r (xn+3 x4 ) x2 u1 , n N .
k=1
1

Profesor, Colegiul Na ional Matei Basarab, Bucureti t s

25

Demonstraie. Vom demonstra afirmaia prin metoda induciei m t t t Pentru n = 1 relaia (9) devine u1 x1 = u1 x3 r (x4 x4 ) x2 u t = u1 (x3 x1 ) = u1 x2 ceea ce arat c pentru n = 1, enunul este adev a a t Pentru n = 2 relaia (9) devine u1 x1 + u2 x2 = u2 x4 r (x5 x t u2 (x4 x2 ) = u1 (x1 + x2 ) + r (x5 x4 ) u2 x3 = (u1 + r) x3 = u2 x deduce c enunul este adevrat i pentru n = 2. a t a s Presupunem c enunul este adevrat pentru n 2, (adic relaia ( a t a a t cata) i sa demonstram ca ea este verificata i pentru n + 1. Avem de s s
n+1 X k=1

uk xk = un+1 xn+3 r (xn+4 x4 ) x2 u1 .

ntr-adevar, relaia (10) este echivalenta cu t n X uk xk + un+1 xn+1 = un+1 xn+3 r (xn+4 x4 ) x2 u1
k=1

relaie care (n condiiile verificarii condiiei (9)) este echivalenta cu: t t t

un xn+2 r (xn+3 x4 ) x2 u1 + un+1 xn+1 = un+1 xn+3 r (xn+4 x

un+1 (xn+3 xn+1 ) = un xn+2 + r (xn+4 xn+3 ) xn+2 un+1 = un xn+2 + rxn+2 = (un + r) xn+2 = un+1 xn+ de unde (daca inem seama ca xn R , n N ) deducem ca relaia t t + varata. Conform principiului induciei matematice rezulta ca enunul t t pentru orice n N i astfel propoziia este demonstrat. s t a Observaie. Dac x0 = 0 = F0 , x1 = 1 = F1 iar (un )n1 est t a aritmetica de raie r din relaia enunului deducem ca t t t n X uk Fk = un Fn+2 r (Fn+3 F4 ) u1 ,
k=1

adic am obinut Problema C:2310 propus de Florin Rotaru n a t a p.360. Dac aici lum r = 0 i un = 1, n N deducem c irul a a s a s verifica relaia t n X Fk = Fn+2 1, n N .
k=1

t atunci sirul (un )n1 este o progresie aritmetica de raie r. Demonstraie. Vom face i aici demonstraia prin metoda inducie t s t t Pentru n = 2 relaia enunului devine: t t u1 x1 +u2 x2 = u2 x4 r (x5 x4 )x2 u1 u1 x1 +r (x4 + x3 x4 )+u1 x2

Propoziia 2. Daca (xn )n0 este un sir de numere reale strict t satisface recurena (2) iar sirul (un )n1 are proprietatea ca exista r t nct n X uk xk = un xn+2 r (xn+3 x4 ) x2 u1 , n N ,
k=1

u1 (x1 + x2 ) + rx3 = u2 x3 u1 x3 + rx3 = u2 x3 (u1 + r) x3 de unde dac inem seama c xn > 0, n N deducem c u2 = u1 + r at a a 26

Presupunem c un = un1 + r, n 2 i s demonstrm c un+ a s a a a ntr-adevr, pentru n + 1 relaia (11) se scrie: a t


n+1 X k=1

uk xk = un+1 xn+3 r (xn+4 x4 ) x2 u1

k=1

n care dac inem seama de relaia (11) obinem: at t t

n X

uk xk + un+1 xn+1 = un+1 xn+3 r (xn+4 x4 ) x2 u

atunci xn+2 = xn+1 + xn , n N. Demonstraie. Procedm i acum prin inducie matematic. Con t a s t a lui pentru n = 0, avem x2 = x1 + x0 . Pentru n = 1 relaia (12) d t u1 x3 r (x4 x4 ) x2 u1 u1 x1 + u1 x2 = u1 x3 de unde, dac in a t un > 0, n N , obinem c x3 = x2 + x1 , adic armaia enunului e t a a t t i pentru n = 1. Presupunem c xk+2 = xk+1 + xk , k = 0, n i s dem s a s a xn+3 = xn+2 + xn+1 . Dac n (12) nlocuim n cu n 1 deducem c a a
n1 X k=1

un+1 (xn+3 xn+1 ) = un xn+2 + r (xn+4 xn+3 ) un+1 xn+2 = un xn de unde prin simplificare cu xn+2 > 0, n N deducem ca un+1 = un Conform principiului induciei matematice rezult c un+1 = un + t a a ceea ce arat c (un )n1 este o progresie aritmetic de raie r. a a a t t Propoziia 3. Daca (un )n1 este o progresie aritmetica de raie r t iar (xn )n0 este un sir de numere reale astfel nct x0 0, x1 > 0, x2 daca n X uk xk = un xn+2 r (xn+3 x4 ) x2 u1 , n N ,
k=1

un xn+2 r (xn+3 x4 ) x2 u1 + un+1 xn+1 = un+1 xn+3 r (xn+4 x

uk xk = un1 xn+1 r (xn+2 x4 ) x2 u1 , n 2.

un xn + un1 xn+1 r (xn+2 x4 ) x2 u1 = un xn+2 r (xn+3 x4 un (xn + xn+1 xn+2 ) + r (xn+3 xn+2 xn+1 ) = 0, dar xn + xn+1 xn+2 = 0 n baza ipotezei de inducie i deci rmne t s a r (xn+3 xn+2 xn+1 ) = 0 xn+3 = xn+2 + xn+1 .

Daca n (12) inem seama de (14) obinem ca t t n1 X uk xk = un xn+2 r (xn+3 x4 ) x2 u1 un xn +


k=1

Conform principiului induciei matematice rezulta ca xn+2 = xn+1 + x t astfel propoziia este demonstrata. t Bibliografie s 1. M. D. Btineu - Siruri, Editura Albatros, Bucureti, 1979. a t 2. Gazeta Matematica, Colecia 1895-2001. t 27

Asupra unei probleme de construcie t

Anca TIMOFTE, Alexandru TURCANU 1

Punctul de plecare al acestei note a fost una dintre problemele rezolvare absolvenilor clasei a VII-a n cadrul Concursului "Recrea t t tice" din 27 august 2002. Enunul acestei probleme este urmtorul: t a

Fie dat un segment [M N ]. Construii cu rigla si compasul un p t astfel nct M [AB], AM = M B, iar N [AC], AN = 3N C. (D construciile care trebuie efectuate.) (Gabriel Popa) t

Prezentm n continuare soluia dat de autorul problemei, aa cum a t a s baremul de corectare: A Soluia 1. Sa presupunem problema rezolvata i fie P t s mijlocul segmentului [BC]. Atunci N P kBD, P M kAC (ca \ linii mijlocii), deci N P M P i m(M P B) = 45 . Daca a s a 2 M este lungimea laturii patratului, atunci M P = , NP = 2 a 10 1 a 2 , deci M N = , de unde N P = M N . = 4 4 5 B Construcia. Vom lua ca unitate un segment u de t lungime egala cu cea a segmentului [M N ].

u 5 u 5 5u u u x

x=

u 1 Ca n figurile de mai sus, construim un segment de lungime u. 5 1 cercul de diametru [M N ] cu cercul de centru N i raza u, obinn s t 5 Construim triunghiul dreptunghic isoscel de ipotenuz [M P ] i aflm a a s a al patratului. Apoi, A i C sunt simetricele lui B faa de M , respectiv s t se obine ca intersecie a paralelelor duse prin A i C la BC, respectiv t t s Demonstrarea faptului c ABCD astfel determinat este ptrat cu a a dorite, este imediat. Evident c problema are soluie, unic pn la a a t a a planului. Vom da mai jos nc dou soluii ale acestei probleme. Prima are av a a t folosete nici un punct auxiliar; este ns necesar o bun cunoatere a s a a a s
1

Elevi, Scoala nr.7 "Octav Bncil", Botoani a a s

28

cu rigla i compasul. A doua are la baz un raionament mai elaborat, s a t numai construcii la nivelul manualelor. t Soluia 2. Pe figura i notaiile din prima soluie, aplicm teorem t s t t a n 4AM N : MN2 \ = AM 2 + AN 2 2AM AN cos(M AN ) = !2 a 2 3a 2 5a2 a 3a 2 2 + = = 2 2 4 2 4 2 8

2 10 MN. deci a = 5

Rezulta urmatoarea construcie: determinam un segment de lungime t unde u = M N . Vrful A al patratului este la intersecia arcului ca t a construit pe [M N ] drept coard, cu cercul de centru M i raz . Afl a s a 2 fiind simetricul lui A faa de M etc. t Soluia 3. Presupunem problema rezolvata t A i aplicm teorema lui Menelaus n 4ABC cu s a transversala M N P ; obinem: t AM BP CN M =1 MB P C NA BP N = 3 BC = 2P C. PC Aplicnd acum Menelaus n 4M N P cu transver- B sala C N A, gsim: a MN P C BA M N =1 = 1 M N = N P. CB AM N P NP Sa observam ca 4DAM 4DCP (C.C.), de unde M D = DP i AD s \ Ultima relaie arat c t a a \ \ \ \ \ \ m(M DP ) = m(M DC) + m(CDP ) = m(M DC) + m(ADM ) = m(A

aadar 4M DP este dreptunghic isoscel. Fie {T } = M P CD; teorema s PT PC 1 a semanarii aplicata n 4P BM cu CT kBM arata ca = = PM PB 3 Construcia. Aflm P ca simetric al lui M faa de N . Intersect t a t diametru [M P ] cu mediatoarea acestui segment, determinnd vrful D PT Aflm punctul T [M P ] care mparte segmentul n raportul a = PM intersecia dreptei DT cu semicercul de diametru [DP ] aflat n semipla t de dreapta DP ce conine punctul N . Vrfurile A i B ale patratului t s acum cu uurina. s t Observaie. Problema se poate generaliza considernd c punctele t a luate astfel nct AM = m M B i AN = n N C. s

29

Cteva aplicaii ale teoremei lui Case t


Marius PACHI TARIU 1

O generalizare remarcabil a teoremei lui Ptolemeu este teorema l a numete distana tangeniala dintre cercurile C1 i C2 , notata d12 , lungim s t t s lor comune exterioare. Teorema lui Casey. Daca cercurile C1 , C2 , C3 , C4 sunt tangen terior sau toate exterior) la cercul C, ordinea punctelor de tangena t numerotarea acestor cercuri, atunci are loc relaia: t d12 d34 + d23 d41 = d13 d24 .

Rezultatul rmne adevrat dac cercurile Ci (toate sau o parte a a a genereaz n puncte sau dac cercul C devine dreapt. a a a Aplicaia 1. Fie ABC un triunghi nscris n cercul C si cercur t tangente la C interior precum si laturilor (BC), (CA) si respectiv (AB A si C1 , B si C2 , C si C3 sa fie de pari diferite faa de BC, CA, r t t Notam cu l1 , l2 , l3 lungimile tangentelor din A, B, C la cercurile C1 , C2 , Are loc echivalena: t b+c c+a a+b , l2 = , l3 = . d12 = d23 = d31 l1 = 2 2 2 A Soluie. Observam mai nti ca C1 , C2 , C3 sunt tangente t la laturi n mijlocul acestora. Aplicnd teorema lui Casey pentru cercurile C i A, C2 , C1 , C3 ; C i B, C3 , C2 , C1 ; C i s s s C3 C, C1 , C3 , C2 , obinem: t B b c c a a b d13 + d12 = l1 d23 , d12 + d23 = l2 d13 , d23 + d13 = l3 d12 . 2 2 2 2 2 2 Din acestea, rezult imediat implicaia "". Invers, dup a t a b+c nlocuirea lui l1 cu etc., aceste relaii se scriu: t 2 b (d13 d23 ) = c (d23 d12 ) , c (d12 d13 ) = a (d13 d23 ) , a (d23 d12 ) = d12 d13 d23 d12 d13 d23 0 i.e. = = = a b c a+b+c (suma numrtorilor fiind nul). Deducem c d12 d13 = 0, d23 d12 = 0, aa a a deci d12 = d23 = d31 , q.e.d. b Aplicaia 2. Fie ABC un triunghi nscris n cercul C si cu m(A t 0 0 0 C (O , R ) cercul tangent la C interior si la laturile [AB] si [AC]. S 4 0 R = r, unde r este raza cercului nscris n triunghiul dat. 3 Soluie. Fie {X} = AB C 0 , {Y } = AC C 0 i l = AX = t s b = AY = XY (4AXY este echilateral, caci m(A) = 60 ). X Relativ la C i cercurile A, C, C 0 , B aplicm teorema lui s a Casey: B b (c l) + c (b l) = al,
1

Elev, cl. a IX-a, Colegiul Na ional, Iai t s

30

2bc bc bc 2bc 4 = = r= r = r. Pe a+b+c p S bc sin A 3 XY l 4 0 0 0 R =OX = = . Ca urmare, R = r. 2 sin 60 3 3 Aplicaia 3. Cercurile Ci (Oi , ri ), i = {1, 2, 3, 4} sunt tangente (n t merotarii) la cercul C (O, r) si, pentru orice i {1, 2, 3, 4}, Ci este ta si Ci+1 ( C1 ind C4 , iar C5 ind C1 ). Atunci, n condiia ca puncte t ct si O, O3 , O4 sunt coliniare, avem: a) 4r1 r2 r3 r4 = (r r1 ) (r r2 ) (r r3 ) (r r4 ), daca Ci sunt tang la C; b) 4r1 r2 r3 r4 = (r + r1 ) (r + r2 ) (r + r3 ) (r + r4 ), daca Ci sunt tang la C. Soluie. Se stabilete uor c dou cercuri de raze a t s s a a i b tangente exterior au lungimea a tangentei comune s exterioare data de d = 2 ab. Ca urmare, teorema lui Casey ne conduce la relaia: t O4 2 r1 r2 2 r3 r4 + 2 r2 r3 2 r1 r4 = d13 d24 . de unde obinem c l = t a

Datorit coliniaritii punctelor O, O1 O3 , avem d2 = a at 13 = (2r r1 r3 )2 (r1 r3 )2 , adica d2 = 4 (r r1 ) (r r3 ); 13 analog d2 = 4 (r r2 ) (r r4 ). nlocuind n relaia precet 24 dent obinem formula de la punctul a). Punctul b) se dovedete n mod a t s Aplicaia 4. Fie dat un cerc C si pe el punctele A si B. De o pa t a dreptei AB consideram cercurile C1 , C2 tangente interior la C si tan [AB] n punctele X si respectiv Y . Sa se determine poziia punctelor X t 1 pentru care d12 = AB. 2 Soluie. Cu teorema lui Casey aplicat lui C i cert a s curilor C1 , A, C2 , B, obinem AX BY +AY BX = AB d12 . t Condiia din enun este echivalenta cu t t 1 AX BY + AY BX = AB AB C1 2 2AX BY + 2AY BX = (AX + BX) (AY + BY ) C AX BY + AY BX AX AY BX BY = 0 (AX BX) (BY AY ) = 0 AX = BX sau AY = BY,

adica unul dintre punctele X i Y trebuie sa fie mijlocul coardei [AB] (c s fi oriunde pe [AB]) pentru a fi ndeplinit condiia problemei. a t

Bibliografie. 1. M. Drguin - Despre utilitatea unui rezultat prea puin folosit: teorem a s t G.M. 12/1995, 716-720. 2. N. Roman - Asupra unor probleme date la O.I.M., G.M. 3/2000, 99-1

31

Extinderi de inele i corpuri s o posibil lecie de recapitulare final a t a


Dumitru GHERMAN 1

Pentru ca recapitularea sa aiba eficiena, trebuie ca n organizarea t de unele principii: la recapitulare nu se parcurge din nou ntreaga materie;

trebuie sa se urmareasca, pe ct este posibil, realizarea unei lega versele ramuri ale matematicii colare; s

recapitularea trebuie s aduc elemente noi, probleme care pot fi a a prelucrarea creatoare a cunotinelor anterioare; s t

se are n vedere stimularea lucrului individual al elevului, folosin indicata de profesor i / sau cautnd noi surse; s

n cele ce urmeaz, vom prezenta un proiect didactic pentru o pos a recapitulare finala la clasa a XII-a. Nu ne propunem sa rezolvam toa sau sa demonstram toate teoremele ce vor aparea; majoritatea aparin f t i poate fi consultat bibliografia. s a I. Inelul ntregilor ptratici. Fie d un numr ntreg liber de pt a a a o h i n Z d = x C | x = m + n d , m, n Z . h i 1) Z d ; +, este un subinel al corpului numerelor complexe, c de integritate. h i m n 2) Z d este izomorf cu inelul matricelor de forma , dn m raport cu operaiile i t h uzualehcu matrice. i as a 3) Inelele Z d i Z d 0 sunt izomorfe dac i numai dac d = s n primul rnd c un izomorfism f ntre cele a inele invariaz eleme a dou a atunci el este bine determinat de valoarea f d ). h i 4) Subinelele unitare ale lui Z d sunt de forma o n An = a + bn d | a, b Z , n N. h i 5) Definim aplicaia norma N : Z d Z, N m + n d = m2 t notam cu x = m n d conjugatul ntregului patratic x = m + n d , are proprietihasemntoare modulului: N (x) = x x, N (xy) = N (x a a t i a aici, x U Z d N (x) U (Z) = {1}. 6) Grupul multiplicativ al elementelor inversabile din Z [i] este U (Z [i
1

recapitularea trebuie s in cont de structura i cerinele exame at a s t

Profesor, Liceul Teoretic Mihail Sadoveanu, Pacani s

32

7) Fie C\Q astfel nct mulimea A = {m + n | m, n Z} t de operaiile uzuale din C. Dac A are exact patru elemente inver t a A = Z [i]. 8) Daca d {2, 3, 5}, atunci U (Z [d ]) conine o infinitate de eleme t gsi n U (Z [d ]) elemente pozitive orict de mici (este suficient s gs a a a element, considernd apoi puterile acestuia i conjugatele lor). s Problemele 1-6 sunt rezolvate n [3]; problema 7 a fost propus de a la etapa finala a Olimpiadei de Matematica n 1997, iar 8 poate fi gasita examenului de bacalaureat din ultimii ani.

II Corpul numerelor ptratice. Fie d un numar ntreg libe a definim n o Q d = z C | z = a + b d, a, b Q . 1) Q d ; +, este subcorp al lui C (inversul elementului ne 1 ab d Q d , deoarece a2 db2 6= 0; altfel, este 2 2 a db a d = R\Q!). / b a b d este izomorf cu mulimea matricelor de forma t , 2) Q db a formeaz corp n raportcu operaiile uzuale. a t 3) Corpurile Q as d i Q d0 sunt izomorfe dac i numai dac s gurele automorsme ale corpului Q d sunt aplicaia identica i cea t s ambele invariind elementele lui Q. 4) Daca un subcorp K C este astfel nct End K = {f, g} i f ( s x Q, atunci exist un ntreg liber de ptrate d 6= 1 pentru care K = a a 5) Dac f Q [x], atunci f () = f (z), z Q a z d ; de aic orice polinom cu coeficieni raionali, are eventualele radacini din Q t t conjugate. Problemele 1-3 pot fi gasite n [3], problema 4 a fost propusa la e Olimpiadei de Matematic din 1988 de ctre Marcel Tena, iar 5 poat a a urmnd pas cu pas demonstrarea unor rezultate analoage din manuale

III Extinderi ptratice. a Corpuri pitagorice. Fie r Q astfel + definim ca mai sus corpul Q ( r), care este subcorp al lui R din poziti 1) Q ( r) este cel mai mic subcorp al lui R care include Q { r}. 2) Putem gndi pe Q ( r) ca un Q-spaiu vectorial, definind nm t torilor" din Q ( r) cu "scalari" din Q prin restricionarea nmulirii t t t Q ( r) (de fapt, din R). Dimensiunea acestui spaiu vectorial este 2 a s {1, r} (a se gsi i alte baze!). 3) Polinomul f = X 2 r Q [X] este ireductibil peste Q, dar adm r n Q ( r); orice alt polinom g Q [X] care admite rdcina r se a a Spunem c f este polinomul minimal al lui r. a 33

d se 4) Din puncte de vedere geometric, extinderea lui Q la Q compasul. De exemplu, r este abscisa unuia dintre punctele de i 1 1 cercului de centru O i raz (r + 1) cu dreapta y = (r 1). s a 2 2 5) Considernd r , r2 , Q , definim Qr1 = Q r1 , Qr2 = Q 1 + t r , . . . ; spunem ca am adjuncionat la Q, pe rn , Q = Qrn1 rn n r1 , r2 , . . . , rn , . . . . Am construit astfel irul de extinderi s Q Qr1 Qr2 Qrn R.

merelor pitagorice; se arata ca aceasta nu depinde de alegerea irului r1 , s i ca formeaza un subcorp al lui R, nchis la operaiile aritmetice i s t s rdcinii ptrate i care este "cel mai mic" (n sensul incluziunii) cu a a a a s eti. at IV Construcii cu rigla i compasul. t s 1) Daca L este un subcorp al lui R iar (D) este o drepta ce trece prin avnd coordonatele n L L, atunci ecuaia dreptei are coeficieni di t t un cerc care are centrul de coordonate din L L i trece printr-un as s are coeficienii ecuaiei sale din L. t t 2) Fie L subcorp n R, iar M (x, y) un punct n plan; spunem ca structibil cu rigla si compasul plecnd de la L dac el poate fi obinut p a t de drepte i cercuri avnd coecienii n L. Dac M este un astfel de s t a fie x, y L, fie exist u L, u > 0 astfel nct x, y L ( u). a 3) Numim numar constructibil cu rigla si compasul un numar real donat a unui punct constructibil. Se arat c orice numr real cons a a a a totodat i numr pitagoric. Ca o consecina, polinomul minimal al unu as a t structibil are gradul putere a lui 2. De aici rezult imposibilitatea dub a triseciei unghiului si cuadraturii cercului (pentru amanunte, v.[1],[2],[ t 4) Corpul ordonat K R se bucur de un anumit tip de comple a mit completitudine euclidiana : cercetat cu rigla i compasul, nu se va a s niciodat c lipsete vreun punct. Tocmai confuzia dintre aceast co a a s a i completitudinea Cantor - Dedekind a lui R a ntrziat soluionarea s t clasice ale antichitii. at

Cum Q este mulime numrabil, putem alege r1 , r2 , . . . , rn , . . . astf t a a numr obinut, pornind de la Q, prin efectuarea unui numr finit de adu a t a nmuliri, mpariri i extrageri de radacini patrate (un astfel[ numa t de t s numar pitagoric) s aparin unui anumit Qrn . Notm K = a t a a Qrn
n

Bibliografie 1. T. Brsan - Trisecia unghiului, Recreaii Matematice, 2/2001, 38-41. t t 2. E. Moise - Geometrie elementara dintr-un punct de vedere superior, E reti, 1980. s 3. C. Nia, T. Spircu - Probleme de structuri algebrice, Ed. Tehnica, Bu t 4. I. Tofan, C. Volf - Algebra - Inele, Module, Teorie Galois, Ed. Matrix reti, 2001. s 34

Metode i procedee de rezolvare a problem s de maxim sau de minim


Gheorghe CROITORU 1

Ne propunem n cele ce urmeaza sa prezentam, prin exemple, o seri i metode prin care pot fi soluionate problemele de maxim sau de minim s t pe mai multe ci a unei aceleiai probleme va permite cititorului s com a s a acestora, precum i s aleag contextul cel mai potrivit pentru aplica s a a alteia dintre ele. a3 Problema 1. Sa se afle minimul expresiei E (x) = x2 + , x x a R+ este dat. Soluia 1. Aplicnd inegalitatea mediilor, obinem c t t a r a3 a3 a3 a3 3 3 x3 3 x3 + + x3 + a3 2 2 = 3 2a 2 2 E (x) = = x x x 2 a3 a egalitatea fiind atins atunci cnd x3 = a , i.e. x = . Urmeaz 3 2 2 2 3 3 2a = . 2 Soluia 2. Se tie c, dac x, y R i suma x + y = const, atu t s a a + s x y m n x y (m, n N ) este maxim pentru = ; dual, dac xm y n = a m n x y suma x + y este minima pentru = (aceste afirmaii se extind la u t m n de termeni / factori i la cazul n care exponenii sunt din Q ). s t + 3 2 a = a6 = const, urmeaza ca E (x) are valoare m ntruct x2 x 3 3 2a2 a3 a a = t cnd x2 = x = . Se obine Emin = E . 3 3 2x 2 2 2 Soluia 3. Fie funcia f : (0, ) R, t t a x 0 f (x) = E (x). Extremele acestei funcii se pot t 3 2 gsi folosind prima derivat. Avem c f 0 (x) = a a a f0 0 3 3 2x a a f & Em = , care se anuleaza pentru x = . 3 x2 2 Tabelul de variaie este prezentat alturat. t a sin x Problema 2. Aflai valorile extreme ale expresiei E (x) = t cos x (Paul Georgescu, Gabriel Popa, Problema 24739, G.M. 9/2002) x t Soluia 1. Pentru x 6= (2k + 1) , k Z, notnd t = tg , ob t 2 2 3t + 2t 3 , t R. Pentru a afla mulimea valorilor lui E, fie y = t = 2+3 t t t R, deci t2 (y + 3) 2t + (3y + 3) = 0, unde t R. Se impune condii
1

Profesor, Liceul Teoretic Al. I. Cuza, Iai s

35

# 2 3 2 3 ce conduce la y 2 , 2 + . Pe de alt parte, E ((2k + a 3 3 ! 2 3 2 3 2 3 , 2 + , iar Emax 2 . Urmeaz c Emin = 2 a a 3 3 3 (se dovedete uor faptul ca aceste valori extreme sunt efectiv atinse). s s Soluia 2. Definim punctele M (cos x, sin x), A (2, 3); atunci E (x t panta dreptei AM . Pentru x R, M parcurge cercul trigonometric i c s exteriorul acestui cerc, urmeaz c valorile extreme ale lui E (x) sunt a a cnd AM este una dintre tangentele duse din A la cerc. Fie d : y 3 ecuaia unei drepte prin A; aceasta este tangent la C (0, 1) cnd dis t a Folosind formula care d distana de la un punct la o dreapt, obinem a t a t ( ) 2 3 |2m + 3| = 1 3m2 + 12m + 8 = 0 m 2 3 m2 + 1 2 3 2 3 n concluzie, Emin = 2 , iar Emax = 2 + . 3 3 Soluia 3. Putem evident utiliza derivata nti n studiul funciei a t t siei E (x). Problema 3. Dintr-o bara metalica de forma cilindrica se obine pr t bara paralelipipedica. Sa se determine dimensiunile dreptunghiului de s nct pierderea de material sa fie minima. Soluia 1. Notnd cu x i y dimensiunile dreptunghiului i cu R ra t s s problema revine la a gsi maximul funciei f (x, y) = xy, n condi a t t x2 + y 2 = 4R2 . nsa, cum x, y sunt pozitive, produsul xy este max produsul x2 y 2 . Deoarece suma x2 + y 2 este constanta, x2 y 2 este maxim a = y 2 = 2R2 . Dimensiunile dreptunghiului cutat sunt, prin urmare, x Soluia 2. Studiul funciei f (x, y) = xy pentru x, y > 0, x2 + y 2 = t t studiul funciei g(x) = x 4R2 x2 , x (0, 2R) i acesta se face apelnd t s Soluia 3. Cnd se cauta extremele unei funcii f (x, y), ntre varia t t o legtur de forma (x, y) = 0, se aplic n general metoda multip a a a Lagrange. Dac f, sunt de clas C 1 , considerm funcia auxiliar a a a t a F (x, y) = f (x, y) + (x, y) , R.

"

Punctul (x0 , y0 ) din domeniul lui f este punct de extrem al acestei fu numai dac (x0 , y0 ) este soluie a sistemului a t
0 Fx (x, y) = 0, 0 Fy (x, y) = 0,

(x, y) = 0,

(x0 , y0 ) = 0, (x0 , y0 ) = 0 nu sunt satisfacute simult iar egalitaile t n cazul nostru, F (x, y) = xy + x2 + y 2 4R2 i avem de rezolv s y + 2x = 0, x + 2y = 0, x2 + y 2 4R2 = 0,

0 x

0 y

neadmind soluiile pentru care 2x = 2y = 0. Obinem imediat c x = t t t a Problema 4. Doua orae A, B sunt situate respectiv la 10 km si s ru rectiliniu, iar proiecia lungimii AB pe direcia rului este de 20 k t t orae trebuie alimentate cu apa de la o uzina amplasata pe marginea r s 36

poziia uzinei pentru care lungimea conductelor ce o leaga de cele dou t minima. Soluia 1. Dac A00 este simetricul lui A faa de direcia rului, iar M t a t t uzinei, evident ca [AM ] [A00 M ], deci AM + M B = A00 M + M B. Acea sum este minim cnd A00 , M, B sunt puncte coliniare; punctul M a a a uzinei astfel obinut se caracterizeaz prin congruena unghiurilor t a t M A, respectiv M B cu normala la direcia rului (v. figura 1). t Notnd x = A0 M , din 4M AA0 4M BB 0 obinem ca t 0 0 AA 10 AM x = = x = 8 (km). BB 0 M B0 15 20 x
B B A A x M B A TA N M TB

Fig.1

Fig.2

Soluia 2. Un fir inextensibil de lungime suficient de mare este fixa t printr-un inel M ce poate culisa pe direcia d i apoi printr-un mic scrip t s Capatul liber are ataata o greutate P , care la echilibru se va afla ct m s sol, minimiznd astfel lungimea M A+M B. n aceasta poziie de echilib t TA i TB care acioneaz n fire sunt egale n modul, paralelogramul fore t a t s deci punctul cutat M este determinat din nou de congruena unghiur a t Soluia 3. Locul geometric al punctelor X pentru care XA + XB t o elipsa de focare A i B. Considernd fasciculul de elipse omofocale (d s B), punctul cutat M este dat de intersecia cu d a acelei elipse din fa a t tangent la d. Proprietatea optic a elipsei asigur din nou congruen a a a t i . s q Soluia 4. Cu notaiile din Soluia 1, AM = 100 + x2 , M B = 22 t t t q 2 + 225+(20 i avem de determinat minimul funciei f (x) = 100 + x s t Not. Exista multe alte procedee i metode de abordare a proble a s trem; menionm, pentru importana lor, metodele programrii liniare t a t a grafurilor. Pentru alte aplicaii, poate fi consultat bibliograa. t a

Bibliografie. 1. M. Cerchez - Aplicaii ale matematicii n practica, E.D.P., Bucureti, t s 2. A. Leonte, C. Niculescu - Culegere de probleme de algebra si analiza Ed. "Scrisul Romnesc", Craiova, 1981. 3. C. Udrite, E. Tnsescu - Minime si maxime ale funciilor reale de v s a a t Ed. Tehnic, Bucureti, 1980. a s 4. Gazeta Matematica (colecie). t 37

Comentarii asupra unui exerciiu t


SU Dan PLAE 1
19 | (5a + 4b) 19 | (9a + 11b) .

Exerciiu. Fie numerele naturale nenule a, b. Sa se demonstreze e t

19 | (55a + 44b) 19 | [19a + 4 (9a + 11b)] 19 | 4 (9a + 11b) 2) 19 | (9a + 11b) 19 | 5 (9a + 11b) 19 | (45a + 55b) 19 | [19b + 9 (5a + 4b)] 19 | 9 (5a + 4b)
(19,9)=1

Soluie (tip culegere). 1) 19 | (5a + 4b) 19 | 11 (5a + 4b) t


(19,4)=1

19

19 | (5a + 4b) .

Comentariu metodic Se poate pune, firesc, ntrebarea: prin care raionament s-a ajuns la t expresia 5a + 4b trebuie nmulita cu 11, iar 9a + 11b cu 5? Prezent t urmeaz un punct de vedere n aceast privina. a a t Determinam numerele naturale n, x, y, x 6= 0 astfel nct sa avem: n (5a + 4b) = 19 (xa + yb) + z (9a + 11b) . Comparnd coeficienii lui a i b obinem: t s t 19x + 9z = 5n . 19y + 11z = 4n

nmulind prima relaie cu 4 i a doua cu 5 i scznd membru cu mem t t s s a 76x 95y = 19z, de unde, prin nprirea la 19, rezult: at a 4x 5y = z.

Dm lui x i y acele valori pentru care expresia x + y + z este minim i a s as malitatea expresiei precedente nu este necesara ci recomandata pentru calculelor!). Astfel, considernd x = 1, y = 0 obinem z = 4. Rezult n t a egalitatea: 11 (5a + 4b) = 19a + 4 (9a + 11b) . n mod analog, pentru demonstrarea implicaiei reciproce, determin t naturale n, x, y, z 6= 0 astfel nct s avem: a n (9a + 11b) = 19 (xa + yb) + z (5a + 4b) . Efectund calculele, se obine z = 11x + 9y. Rezult, lund x = 0, y t a i n = 5. Deci, avem: s 5 (9a + 11b) = 19b + 9 (5a + 4b) .

Observaie. Raionamentele sunt valabile i pentru numere ntreg t t s n ncheiere, propunem 1) 2) 3)


1

cititorilor demonstrarea urmtoarelor echiva a 11 | (2a + 5b) 11 | (3a + 2b) , 23 | (2a + 3b) 23 | (9a + 2b) , 19 | (11a + 2b) 19 | (18a + 5b) .

Profesor, Scoala Normal Vasile Lupu, Iai a s

38

Cteva probleme privind triplete pitagor


Mircea CR AREANU 1 SM

Subiectul triplete pitagoreice are o istorie bogat, fiindu-i dedica a ticole (a se vedea n acest sens capitolul IV din [2], unde la pagina 189 cteva tabele cu astfel de triplete). Definiie. Tripletul de numere naturale nenule (x, y, z) cu max t numete pitagoreic daca x2 + y 2 = z 2 . s

Se tie c forma general a unui triplet pitagoreic ([1], [2, ex.5.8 s a a este: x = 2 2 , y = 2, z = 2 + 2 , cu , numere naturale nenule i prime ntre ele, adic (, ) = 1. s a

n cele ce urmeaz prezentm cteva generalizri ale unor rezultate a a a triplete pitagoreice, rezultate aate n bibliografia romna.

1. 60 | xyz. Demonstraie. Avem xyz = 2 2 2 2 + 2 = 2 4 t (i) Divizibilitatea cu 3 (5). Dac sau este multiplu de 3 am te a (5) nu, conform teoremei lui Fermat, avem 2 2 1 (mod 3) 4 4 de unde rezulta divizibilitatea cu 3 i 5. s (ii) Divizibilitatea cu 4. Deoarece (, ) = 1 cel mult unul dintre par. 2 2 2l 1 (ii2 ) = 2k, = + 1 xyz = 4k (2l + 1) 4k 4l 4l 1 4k + 4l2 + 4l + 1 2 (ii2 ) = 2k + 1, = 2l + 1 xyz = 16 (2k + 1) (2l + 1) k + k 2k 2 + 2k + 2l2 + 2l + 1 . n concluzie avem i divizibilitatea cu 4. s

Observaie. Divizibilitatea cu 4 constituie Problema C:827, G.M.-1 t Augustin Stan, iar divizibilitatea cu 5 Problema E:6303, G.M.-8/197 n [2] la pagina 171 este citat P. Lenthric ca fiind autor al acestui rez anului 1830!

2. z si orice putere a sa este suma a doua patrate diferite. Demonstraie. Pentru z avem concluzia datorit relaiei (1) cu (, t a t tru puterile lui z aplicam Problema E:5888 , G.M.-5/1977, autor Stef (pentru rezolvare a se vedea G.M.-10/1977, p.405-406): Dac un numr natural este suma a k ptrate diferite atunci oric a a a este suma a k ptrate diferite. a

3. Se cer lungimile catetelor unui triunghi dreptunghic aa nct p s fie de p ori perimetrul, cu p un numar prim dat.
1

Lector dr., Facultatea de matematic, Univ. Al. I. Cuza, Iai a s

39

Demonstraie. Din xy = p (x + y + z) rezult 2 2 2 = p t a adic ( ) = p. Cum p este numr prim rezult c avem soluiile a a a a t (1, p) , (p, 1) deci (, ) = (p + 1, 1) , (p + 1, p). n concluzie avem: 2 (i) x = (p + 1) 12 = p (p + 2), y = 2 (p + 1), (ii) x = (p + 1)2 p2 = 2p + 1, y = 2p (p + 1).

Observaie. Pentru p = 2 se obine Problema OG:111, G.M.-1 t t Valer Pop.

4. (G.M.-5/1979, Problema O:35, Bucur B. Ionescu) Exista triple cu x, y, z numere prime? Soluie. Din y = 2 rezult singura posibilitate = = 1 dar at t a x = 2 2 = 0, imposibil. Deci rspunsul este negativ. a 5. (G.M.12/1979, Problema E:6736 , I. Joldi) x + y z | xy. s + Demonstraie. x + y + z = 2 ( + ), iar xy = 2 2 2 . t

6. (Problema 7.8, [3, p.190 + p.199]) x2 xy + y 2 este suma a dou 2 Demonstraie. x2 xy+y 2 = 2 2 2 2 2 +42 2 = t +2 ( )2 .

7. Daca p si q sunt numere naturale nenule si prime ntre ele, ecuaia diofantica p2 x2 + q 2 y 2 = 2p2 q 2 z 2 . t Soluie. Considernd x = qu i y = pv obinem u2 + v 2 = 2z 2 de t s t 2 2 uv u+v u+v uv i deci z2 = + s = 2, = 2 2 , z = 2 2 2 2 concluzie: x = q 2 + 2 2 , y = p 2 2 2 , z = 2 + cu (, ) = 1.

Observaie. Pentru p = 2, q = 3 se obine Problema 5.9 din [3, p. t t

Bibliografie 1. V. Claudian - Analiza diofantica, G.M.-1/1970, 1-9. 2. L. E. Dickson - History of the theory of numbers, vol. II - Diophant Chelsea, N. Y., 1952. 3. P. Radovici-Mrculescu - Probleme de teoria elementara a numerelor a Seria Culegeri de probleme de matematic i zic, Bucureti, 1986. as a s

40

Cteva aplicaii ale inegalitii Cauchy-Buni t at

Ioana CRACIUN si Gheorghe CRACIUN 1

0 (ai bj aj bi )2 = a2 b2 2 (ai bj ) (aj bi ) + a2 b2 , i, j = 1, i j j i Sumam dupa j i obinem: 0 a2 S b2 2ai bi S (ab) + b2 S a2 , i = i s t 2 2 i a acum dup i: 0 S a2 S b2 2S 2 (ab) + S a S b , adic S 2 (ab) a q.e.d. Aplicaii t 1. Fie patratul ABCD si M , N doua puncte pe cercul nscris n ac se arate ca 1 AABCD (AM AN + BM BN + CM CN + DM DN 3 Fie P, R, S, T mijloacele laturilor AB, BC, CD,DA. Din teorema m M A2 + M B 2 AB 2 M B2 + M C 2 B , M R2 = MP2 = 2 4 2 2 2 2 2 2 MC + MD MD + MA CD D , MT 2 = M S2 = 2 4 2 Adunnd obinem t 2 2 2 2 2 2 M P + M R + M S + M T = M A + M B + M C 2 + M D2 Triunghiurile P M S i T M R sunt dreptunghice, deci s M P 2 + M S 2 = P S 2 = AB 2 i M T 2 + M R2 = T R2 = AB 2 . s Adunnd obinem M P 2 +M S 2 +M T 2 +M R2 = 2AB 2 , deci t M A2 +M B 2 +M C 2 +M D2 = 3AB 2 (egalitate cunoscuta). Analog, N A2 + N B 2 + N C 2 + N D2 = 3AB 2 . Folosind inegalitatea Cauchy-Buniakowski, avem:

Daca a1 , a2 , . . . an si b1 , b2 , . . . bn sunt numere reale, n N , n 2, inegalitatea Cauchy-Buniakowski: 2 (a1 b1 + a2 b2 + + an bn ) a2 + a2 + + a2 b2 + b2 + 1 2 n 1 2 a2 an a1 cu egalitate daca si numai daca sau ai = bi = 0, = = = b1 b2 bn n n P 2 P ak , S (ab) = ak bk . Avem: Demonstraie. Notam S a2 = t
k=1 k=1

M A2 + M B 2 + M C 2 + M D2 N A2 + N B 2 + N C 2 + N D2
2

9AB 2 (AM AN + BM BN + CM CN + DM DN )2 3AABCD AM AN + BM BN + CM CN + DM D

(AM AN + BM BN + CM CN + DM DN ) sau

2. Fie a1 , a2 , . . . an R. Sa se ae x1 , x2 , . . . xn stiind ca a1 x1 + n n P 2 P 2 2 2 2 +an xn = ai i x1 + x2 + + xn = ai . s


i=1 i=1
1

Profesori, Plopeni (Prahova)

41

Conform inegalitii C-B, avem: at

cu egalitate daca xi = kai , i = 1, n. Tinnd cont de condiiile din e t n n n P 2 2 P 2 P 2 c a ai ai ai , care are loc cu semnul de egalita
i=1 i=1 i=1 i=1 i=1

(a1 x1 + a2 x2 + + an xn )2 a2 + a2 + + a2 x2 + x2 + 1 2 n 1 2

Dac notm Ai Mi = xi , iar Ai Ai+1 = ai , aceast ultim relaie a a a a t 2 2 2 2 2 +x2 + + xn = (a1 x1 ) + (a2 x2 ) + + (an xn ) sau, efectu x2 +x2 + +x2 = x2 +x2 + +x2 +a2 +a2 + +a2 2 (a1 x1 + a2 x2 + 1 2 n 1 2 n 1 2 n 1 2 2 a + a + + a2 . Aplicnd ineg a1 x1 + a2 x2 + + an xn = n 2 1 2 avem: (a1 x1 + a2 x2 + + an xn )2 x2 + x2 + + x2 a2 + a2 + 1 2 n 1 2 egalitate daca i numai daca ai = kxi , i = 1, n. n consecina, rezulta c t s 2 2 2 1 2 2 2 2 2 2 a + a2 + + an x1 + x2 + + xn a1 + a2 + + 2 1 1 2 a1 + a2 + + a2 x2 + x2 + + x2 , 2 n 1 2 n 4 adic relaia dorit. Pentru a avea egalitate punem condiia ai = k a t a t gasim k = 2. Deci M1 , . . . Mn sunt pe mediatoarele laturilor poligonu inscriptibil, iar M va fi centrul cercului circumscris lui. ( a+b 4. Sa se determine suma elementelor mulimii M = abc | p t 3 (a2 + 2 Inegalitatea (a + b + c)2 3 a + b2 + c2 se deduce imediat cu a a+b+c litii C-B. Deci p at 1, cu egalitate pentru a = b = 3 (a2 + b2 + c2 ) ca M = {111, 222, 333, . . . , 999} i 111 + 222 + + 999 = 111 (1 + 2 s = 111 45 = 4995. 42

t Observaie. Pentru n = 3 i a1 = 2, a2 = 2, a3 = 3 obinem prob t s (Alfred Eckstein, G.M. - 11/2000). 3. Fie A1 , A2 , . . . An un poligon convex si M un punct n interi proiecteaza M pe laturile A1 A2 , A2 A3 , . . . , An A1 n punctele M1 , M2 , . arate ca: 1 A1 A2 2 + A2 A3 2 + + A A1 M1 2 + A2 M2 2 + + An Mn 2 4 Fie i {1, 2, 3, . . . , n} fixat. n triunghiurile M Ai Mi i M Ai+1 Mi s rema lui Pitagora: Mi M 2 = Ai M 2 Ai Mi 2 = Ai+1 M 2 Ai+1 Mi2 An+1 = A1 ). Scriind a doua egalitate pentru i = 1, n i sumnd, gasim s n n X X Ai Mi 2 = Ai+1 Mi 2 .
i=1 i=1

relaia precedent este o egalitate i rezult c xi = kai , i = 1, n. nloc t a s a a n n P 2 P 2 condiie din enun, obinem: t t t ai = k 2 ai , deci k2 = 1 i xi = ai , s

Concursul "Recreaii Matematice" t


Ediia a II-a, Iai, 27 August 2002 t s Clasa a VII-a

1. Fie x, y, z (0, ) cu x + y + z = 1. Sa se determine cele mai m care le pot lua expresiile xy 2 yz 2 zx 2 xy yz zx E= + + . + + ; F = z x y z x y a s s t Cornel Noan, Focani i Lucian Tues s 2. Fie n N i x = n2 + n. a) S se arate c x R\Q i s se afle [x]. a a s a b) S se determine primele dou zecimale de dup virgul ale a a a a n = 20022002 . Cornel Noa 3. Fie dat un segment [M N ]. Construii numai cu rigla i compa t s ABCD astfel nct M [AB], AM = M B, iar N [AC], AN = 3N toate construciile care trebuie efectuate.) t Gabrie

Clasa a VIII-a

1. Daca suma, produsul i ctul a doua numere iraionale sunt, fie s t raionale, calculai suma cuburilor celor doua numere. t t Claudiu- tefan Popa, Iai (Recreaii Matemat S s t 2. S se rezolve n Z Z ecuaia x2 (y + 1) + y 2 (x + 1) + 1 = 0. a t Gabrie 3. Fie ABCA0 B 0 C 0 un trunchi de piramida oarecare. Notam cu G, G greutate ale bazelor, iar {D} = BC 0 CB 0 , {E} = AC 0 CA0 , {F } = A se arate c dreptele BE, CF i GG0 sunt concurente. a s Dan B

Clasa a IX-a

1. S se rezolve n R ecuaia xn 3 [x] + 2 = 0, n N . a t Cornel Noa

2. S se arate c pentru orice (0, 2), exist n Z astfel nct a a a Gheorghe t 3. Fie Db , Dc , Fa , Ea puncte de tangena ale cercurilor exnscrise tr cuitunghic ABC cu dreptele suport ale laturilor, astfel nct B (Dc C) t A B B (AFa ), C (AEa ). Sa se arate ca: Dc Fa kDb Ea sin2 = sin2 2 2 Temistocle

Clasa a X-a

1. Fie P R [X], P (X) = X n + an1 X n1 + + a1 X + a0 . a) S se arate c dac toate rdcinile polinomului sunt reale i m a a a a a s n 2, atunci (1) P (1) + an1 + 2n 1. 43

b) S se arate c dac toate rdcinile polinomului sunt reale, pozit a a a a a 2n + 2n2 an2 + 2n4 an4 + . . . dect 2, atunci n1 < 1. 2 an1 + 2n3 an3 + 2n5 an5 + . . . Carmen Nejneru i Vlad Ma s 2. Sa se arate ca cos n arctg 2 2 Q, n N. Gheorghe 3. Fie A, B dou puncte fixate, iar M un punct variabil n plan. a imaginile lui M prin rotaiile n jurul punctului A, respectiv B, de ungh t 0 0 a a s t a t a . Dac vectorul A B pstreaz aceeai direcie, artai c M parcur a 2 Reciproca este adevarata? Gabrie

Clasa a XI-a

1. a polinoame, fiecare avnd cte o rdcin a a a Fie P, Q R [X] dou 1 1 4 4 P a t a + x + Q (x) = Q + x + P (x) , x R, artai c 2002 2002 Lucian Tues t 2. Fie A Mn (R) astfel nct exist m N, m > n 3 i a s pentru care Am+1 Am A + In = On . Sa se arate ca |det A| = 1. Lucian-Georges Ldunc, Iai (Recreaii Matema a a s t 3. Determinai funciile continue f : (0, 1) (0, ) pentru care f t t xy , x, y (0, 1). =f 2xy x y + 1 Lucian L

44

Concurs de admitere 2002, Iai s

Facultatea de Informatic, Universitatea "Al. I. C a

Analiz matematic a a 1. Fie (an )nN , (bn )nN doua iruri de numere reale. s i) Dac (an )nN converge ctre a i (bn )nN converge ctre b, ce se a a s a despre convergena irului a0 , b0 , a1 , b1 , . . . , an , bn , . . . ? S se justifice r t s a (1)n i bn = (1)2n+1 + n+2 n, n N, s se studiez a s ii) Dac an = a n+1 irului a0 , b0 , a1 , b1 , . . . , an , bn , . . . . s 2. Fie funcia f : [1, 1] R, f (x) = ln x2 + 1 . S se arate c: t a a i) |f (x2 ) f (x1 )| < |x2 x1 |, x1 , x2 [1, 1], x1 6= x2 ; ii) exist un singur x0 (1, 1), astfel nct f (x0 ) = x0 . a

Algebr a 1. Fie (G, ) i (, ) dou grupuri. S se demonstreze c dac f s a a a a izomorfism, atunci i f 1 : G este izomorfism. s 2. Fie dat q Q . Sa se arate ca: i) funcia f : Z Q, f (k) = q k , este morfism de la grupul (Z, t (Q , ); ii) dac q {1, 1}, atunci exist un subgrup al lui (Q , ) izom a / a (Z, +). Sa se precizeze acest subgrup. 3. Fie f Z3 [X], f = b X X 2 X n1 , n N . Aratai c 1 t 2, as a prin X b dac i numai dac n este multiplu al lui 3. 4. S se descompun n factori ireductibili peste Q, R i respectiv a a s g = X 4 + X 3 X 2 2X 2, tiind ca g se divide prin X , unde es s de ordinul trei a unitaii. t

Algebr - colegiu a 1. Fie A M2 (R), t (A) suma elementelor de pe diagonala princip A i det (A) determinantul matricii A. Sa se arate ca: s i) A2 t (A)A+det (A)I2 = O2 , unde I2 i O2 sunt matricea unita s matricea nul din M2 (R); a ii) daca A2 = 2 , atunci t (A) 0. O = a11 a12 a13 2. i) Fie A = a21 a22 a23 M3 (R). S se scrie explicit toi a t a31 a32 a33 apar n expresia determinantului matricii A i care sunt de forma (1)i+ s ii) fiind un parametru real, sa se discute i sa se rezolve sistemul s 2x 3y z = 1 . 4x + 6y + 2z = 2 a t a 3. Fie H = x + y 2 | x, y Q, x2 2y 2 = 1 . Artai c H este a lui R n raport cu nmulirea i c toate elementele lui H sunt sim t s a raport cu operaia indus. t a 4. Sa se determine polinoamele f, g Z [X], de gradul 1, astfel nc 2 X + 2X + 2 f + X 2 + 3X + 3 g = 1. 45

Fac. de Electronic i Telecomunicaii, Univ. Tehnic " as t a


Matematic a

1. Ce relaie exista ntre numerele reale A = n n+1 i B = (n + 1) t s a) A > B; b) A = B; c) A < B; d) A B; e) A B k2 k 2. Mulimea soluiilor inecuaiei C16 > C16 este t t t a) {4, 5, . . . , 9}; b) ; c) {17, 18, 19}; d) {10, 11, . . . , 16}; e) {1 3. Fie inecuaia logx x + 30 1. Soluiile acestei inecuaii sunt t t t a) x (, 5]; b) x [6, ); c) x (1, 6]; d) x (1, ); Z 3 dx 4. Fie I (a) = , a R i L = lim I (a). Atunci: s a2 |x a| + 1 1 a) L = 2; b) L = 2 ln 2; c) L = 4 ln 2; d) L = 8 ln 2; e) L = x + y + 2z = 1 x + (2 1) y + 3z = 1 5. Sistemul cu , R este com x + y + ( + 3) z = 2 1 terminat pentru a) R, = 1; b) = 0, = 5 sau R, = 1; c) = 0, d) = 0, = 1 sau R, = 5; e) R, R. sin2 x cos2 x 6. Sa se afle soluiile ecuaiei t t = cos x (1 + tg x) sin x (1 + ctg x) a) x = k ; b) x = 2k ; c) x ; d) x = k ; e) x = 2 3 6 (1 + i)2002 7. Numrul complex a n este real pentru n N de forma (1 i) a) n N; b) n = 4k; c) n = 4k + 1; d) n = 4k + 2; e) n = 4k + 8. Se consider polinoamele f, g R [X], f = X 2n X n + X 4 + 1, g a Sa se determine restul r al mparirii polinomului f la polinomul g. t a) r = (n + 4) X n 2; b) r = nX; c) r = (n + 2) X + n d) r = (n + 4) X + n 2; e) r = 2nX. x2 9. Se consider funcia f care satisface relaia: 2f a t t +f x+1 pentru orice x R\ {1, 2}. Valoarea derivatei f (n) (2) este n! n! n! n! ; d) (1)n n ; e a) (1)n n+1 ; b) n+1 ; c) (1)n 3 3 2 3n 3 xn + 1 10. Funcia f : D R, D R, f (x) = 3 t , n N, are cel puin t x +1 i nu admite asimptote orizontale sau oblice pentru vertical s a a) n = 2k + 1, k N; b) n N; c) n < 4; d) n = 4; e) n = 2 Matematic - ingineri a

Fac. de Automatic i Calculatoare, Univ. Tehnic "G as a

1. Valorile parametrului real a pentru care rdcinile x1 , x2 ale ec a a 2 a2 + 2a + 1 x + a2 + 1 = 0 satisfac x1 + x2 4x1 x2 sunt: a) a > 0; b) a = 0; c) a = 1; d) a = 1; e) a 6= 2. 2. Fie polinoamele f = X 2n X n + X 4 + 1, n > 4 i g = (X s determine restul mpririi lui f la g. at 46

a) (n + 4) X n 2; b) nX + n 2; c) (n + 2) X n d) (n + 4) X + n 2; e) (n 4) X + n 2. 1 1 3. Daca a este o radacina a ecuaiei x2 + x + 1 = 0 i A = 1 a t s 2 1 a 3 0 0 2 0 0 1 0 a) A2 = 0 0 3; b) A2 = 0 0 2; c) A2 = 0 0 0 3 0 0 2 0 0 1 2 2 0 3 3 0 d) A2 = 0 2 2; e) A2 = 0 3 3. 2 0 2 3 0 3 1 4. Legea de compoziie x y = (x + y xy + 1) este o lege de gr t 2 pe mulimea: t a) R; b) R\ {0}; c) R\ {0, 1}; d) R\ {1}; e) R\ {1}. 1 1 1 . 5. Sa se afle lim + + + n 2n + 1 2n + 2 2n + n 3 1 3 a) ln 2; b) ln ; c) ; d) e; e) 1 + ln . 2 2 2 2 x+m 6. Se d funcia f : R\ {2} R, definit prin f (x) = a t a e x+2 este parametru real. Sa se precizeze valorile lui m pentru care f are do extrem. a) m [2, 6]; b) m (, 2/3]; c) m (2/3, 6); d) m (, 2 e) m (, 2/3) (6, ). Z 1 2 x + 7. S se determine parametrul a, astfel nct s avem 1 < a a 2 0 x + 2 3 2 3 a) 3 < a < 3 + ; b) 3 < a < 3; c) 3 < a < 3 1 + 7 . d) 3 < a < ; e) 3 < a < 6 2 8. Se d un triunghi de arie S i de laturi a, b, c. Fie M, N, P proiec a s cercului nscris pe laturi. Se cere aria 4M P . N abc (a + b + c) S2 3 3S 2 3 a+b+c 2 S ; b) ; c) ; d) a) 2 ; e) ab 3abc 16S 3abc (a + b + c) 9. S se rezolve ecuaia sin (2x + 1) = cos (2x 1). a t a) +k ; b) +k; c) +k ; d) (1)k +k ; e) (1)k 8 2 8 8 2 8 2 8 10. Unghiul diedru dintre doua fee adiacente ale unui octaedr t a a a msura, exprimat n radiani, egal cu: 2 5 3 3 ; c) ; d) 2 arccos ; e) 2 arcsin . a) ; b) 2 3 8 3 3 Matematic - colegiu a 1. Soluiile sistemului t xy + x + y = 11 sunt: x2 y + xy 2 = 30 47

a) (x, y) {(2, 3) , (3, 2)}; b) (x, y) {(1, 5) , (5, 1) c) (x, y) {(2, 3) , (1, 5)}; d) (x, y) {(3, 2) , (5, 1)}; e) (x, y) {(2, 3) , (3, 2) , (1, 5) , (5, 1)}. 2. Sa se rezolve inecuaia log2 (9 2x ) > 3 x. t a) x < 8; b) 0 < x < 3; c) 2 x < 4; d) x > 3; e) nu exist < a 1 0 1 3. Dac matricea A = 0 1 0 satisface A3 = aA2 + bA, atunc a 1 0 1 a) (a, b) = (3, 2); b) (a, b) = (3, 3); c) (a, b) = (2, 2); d) (a, b) = e) (a, b) = (2, 3). 4. Pe R se definete legea de compoziie prin relaia: x y = xy + s t t x, y R. Sa se determine a, b R astfel nct legea sa fie comutativa 1 1+ 5 1 ,b = a) a = 1, b = ; b) a = 0, b = 0 sau a = 1, b = ; c) a = 2 2 2 1 5 1 5 ,b = ; d) a = 4, b = 2; e) nu exist solu a a= 2 4 r 2 n +1 n+1 ln . 5. Sa se afle lim n n+2 n 1 a) ; b) 1; c) e; d) e; e) . 2 x2 + mx + 2 6. Se considera funcia f (x) = 2 t , unde m R este un p x + 2x + m se determine m, astfel nct domeniul ei de definiie s fie R i s admi t a s a puncte de extrem. a) m(1, 2)(2, ); b) m(2, ); c) m(3, ); d) m(1, 2); e) m R xdx , x (a, ) (a 6= 0). 7. Sa se calculeze (x + a)3/2 1 a + C; b) 2 + C; c) a) 2 x+a x+a x+a x+a x + 2a 2x+a+1 + C; e) 2 + C. d) 3 x+a x+a 8. ntr-un patrulater convex se cunosc diagonalele d1 , d2 i un ungh s Se cere aria patrulaterului ale carui vrfuri sunt mijloacele laturilor cel 1 1 2 1 d + d2 d1 d2 sin a) d1 d2 cos ; b) d1 d2 sin ; c) 2 4 4 4 1 1 1 d) d1 d2 (sin + cos ); e) (d1 + d2 )2 sin . 4 16 9. Se dau numerele x = cos 3, y = tg 3, z = ctg 3. Atunci a) x < y < z; b) y < x < z; c) z < y < x; d) x < z < y; e) z 10. Locul geometric al centrelor sferelor ce trec prin dou puncte d a este a) o sfera; b) o dreapta; c) doua drepte perpendiculare; d) u e) doua plane perpendiculare.

48

Capacitate - teste pregtitoare a

Testul 1 (prof. Gheorghe TIMOHE) I. 1. Valorile raionale ale numerelor x i y pentru care x t s +y 2 + 2 3 = 4 2 3 sunt ......................... 2. Descompus n produs de doi factori expresia E (x) = x2 9y 2 a egala cu ......................... 98 97 96 95 94 93 99 + + + + + + + 3. Fie E = 98 97 96 95 94 93 92 unde [x] - partea ntreag a numrului x R; atunci E = ................. a a p p 4. Soluia n N a inecuaiei 3 + 2 2 x + 3 2 2 x < 2 este .... t t 5. Daca numerele prime x i y satisfac relaia 2x + 3y = 16, s t ........................., y = ......................... 6. ntr-un vas n form de cub se toarn 6 l ap, ceea ce reprez a a a capacitatea vasului. Diagonala cubului este egala cu ......................... 7. Dimensiunile unui paralelipiped dreptunghic sunt direct propor t i 10, iar diagonala are lungimea de 10 2 cm. Volumul paralelipipedulu s 8. Fie ABCD un trapez cu bazele AB = 13 cm i CD = 15 cm, M s nct M D = 2M A i M N kAB, N [BC]. s BN a) Valoarea raportului este ......................... b) M N = ......... BC 9. Fie AC i BC doua tangente la un cerc C (O, R) n A i B, m( s s AO = 8 cm. a) m(AOB) = ......................... b) lungimea coardei AB = ........ II. 1. a) Reprezentai ntr-un sistem de coordonate carteziene to t (x, y) care verifica relaiile |x 1| = 3 i |x y| 3. t s b) Fie E (x) = (3 |x| 1) / |x|. Determinai x R pentru care E (x t 2. Demonstrai c suma distanelor de la vrfurile unui triunghi t a t exterioar lui este egal cu suma distanelor de la mijloacele laturilor t a a t aceeai dreapta. s 3. Pe planul patratului ABCD de latura a se ridica perpendicular Fie M i N proieciile lui A pe (OBC) i (OCD), iar P i L proiecii s t s s t pe (AOD), respectiv M H. S se determine: a) M P i AL; b) ungh a s planelor (AM H) i (ABC); c) poziia punctului Q pe OC astfel nc s t 4OBD sa fie minim. Testul 2 (prof. Gheorghe TIMOHE)

I. 1. Propoziia "n N, n (n + 3) + 3 este numar prim" este ....... t 1 1 2 1 + ++ . Dintre S i este mai mare .... s 2. Fie S = 100 101 299 3 3. Soluiile ecuaiei ||3x 1| 12| = sunt ......................... t t 4 ab, a (b, (a + b) /2) = ............... 4. a) Dac 0 < b < a, atunci a 1999 2000 , 2 Z, atunci card A = ......................... b) Dac A = 2 a |x + 2| + |y 3| = 7 5. Rezolvnd sistemul , obinem (x, y) .... t |x + 2| + 3 = y 3 6. Fie ABCD - trapez de baze AB = 7 cm, CD = 2 cm i latur s 49

BC = 4 cm i AD = 3 cm. Msura unghiului dintre AD i BC s a s ......................... grade. 7. Fie ABCDA0 B 0 C 0 D0 - cub. Atunci m( (A0 C 0 , AD0 )) = ........... 8. Aria 4ABC este 268 cm2 . Aria triunghiului format de mijloa sale este ......................... 9. Fie 4ABC cu m(A) = 60 , m(B) = 80 , (AA0 , (BB 0 , (CC 0 unghiurilor 4ABC (A0 , B 0 , C 0 aparin cercului circumscris 4ABC). t a) m(B 0 C 0 A0 ) = ......................... b) m(A0 B 0 C 0 ) = ................... II. 1. Fie f : R R, f (x) = 3x + 1, g : R R, g (x) = x + 1. a) Calculai aria suprafeei determinat de graficele funciilor i Ox t a t s t b) A = (x, y) | x, y R , f (x + y) f 2 xy . Reprezentai gr t + lui A. c) B = n N | |g (n)| 8 2 . Determinai suma elementelor din t 2. Fie numerele a = 2+1 3 2 . . . 100 + 99 10 21 3 + 2 . . . 100 99 101 + 100 . b= a) Artai c a + b > 2. b) Comparai numerele (1 a) (1 + b) i b a t a t s 3. Tetraedrul ABCD se secioneaz cu un plan paralel cu mu t a [CD], AB = a, CD = b. Planul intersecteaz muchiile [BD] , [BC] , [ a N, M, P i respectiv Q. s a) Demonstrai c patrulaterul M N P Q este paralelogram. t a BM b) Daca = x, x > 0, exprimai aria paralelogramului M N P Q t MC a, b, x i masura unghiului dintre AB i CD. s s c) Determinai poziia punctului M [BC] pentru care aria par t t M N P Q este maxim. a

Testul 3 (prof. Lidia BOSNCIANU )

1. Cel mai mic numar natural nenul divizibil cu 88 i care poa s produsul a trei numere naturale consecutive este ......................... 2. Dac a, b, c, d R, i cd = 1, valoarea minim a expresiei a2 + b a s a +2ab 2ac 2ad 2bc 2bd + 14 este ......................... 3. Numerele 5 x, 2x + 3 i 5x 2, x R, reprezinta lungimile s triunghi. Atunci x ......................... 2 2 4. Ecuaiile (m 1) (x + 2)+9 = (m 1) 3 (m 1) (x + 2), (m t = 3 (3x + m + 5), m R sunt echivalente daca m ......................... 5. Fie f : R R, f (x) = 2 |x| f (1) 3, x R. Atunci f (x) = 6. Fie ABCD un dreptunghi, M A (ABC), N mijlocul lui (BC), = 6 cm i M D = 4 3 cm, atunci d (M, DN ) = ......................... s 1 7. Fie 4ABC oarecare, M (BC) astfel nct M C = BC, N 3 1 AP nct AN = AC, iar AM BN = {P }. Atunci = ..................... 4 PM 8. ABCDA0 B 0 C 0 D0 este un paralelipiped dreptunghic cu perime cm. Daca diagonala paralelipipedului are lungimea de 20 cm i formeaz s lateral un unghi cu msura de 30 , volumul [ABCDA0 B 0 C 0 D0 ] este .... a a 9. Fie un con circular drept cu seciunea axial un triunghi isoscel t a 50

i perimetrul 18 cm. s a) Msura unghiului corespunztor sectorului de cerc obinut prin a a t conului este ......................... b) Volumul conului este ......................... II. 1. S se gseasc ultimele dou cifre ale numrului a a a a a 2n 2n 2n 2n+1 2n+2 2n+3 2n+1 A= 2 +2 +2 +2 + 42n+2 + 42n 4 + 4 + 4

22n+1 + 22n+3 , n N . 4n + 4n+1 a 2. Fie x1 , x2 , . . . , xn R cu x1 x2 . . . xn = 1. S se demonstreze in + (x1 + x2 + 1) (x2 + 2 x2 + 1) . . . (xn + n xn + 1) > 3 4 n (n 3. V ABCD este o piramida patrulatera regulata avnd toate muchii a. Fie Q mijlocul lui (CV ) i (C) cercul nscris n triunghiul V BC. s a) S se arate c (BDQ) (V BC). a a b) Dac M este un punct oarecare pe (C) i N (BD), s se determ a s a minima posibila pentru (M N ).

5. Fie f : R R, f (x 1) = 3x + 1 f (2), x R. Atunci f (x 6. Fie M mijlocul laturii (AB) a dreptunghiului ABCD, M N A \ i 8M N = AC. Atunci m(BAC) = ......................... s b b 7. Fie 4ABC cu m(B)m(C) = 40 . Fie AD BC i (AE bisectoa s \ D, E (BC). Daca (AD i (AE mpart BAC n trei unghiuri cu m s proporionale cu 1, 2, 3, atunci masurile unghiurilor 4ABC sunt ........ t 8. ntr-un cilindru cu diametrul de 4 cm i nlimea de 25 cm se s at bile cu raza de 20 mm. a) Care este numarul maxim de bile ce ncap n cilindru? b) Cte procente din volumul cilindrului ocupa bilele? 9. ntr-o piramid patrulater regulat, seciunea diagonal este a a a t a dreptunghic isoscel. Atunci At /Al = ......................... II. 1. Fie a, b Z. Aratai ca a b2 + a2 + a = 0 a = b = 0. t x2 x3 . . . xn + x1 x3 . . . xn + + x1 x2 2. Rezolvai n N ecuaia y = t t x1 x2 . . . xn (x1 + x2 + + xn 3. Fie 4ABC dreptunghic isoscel, (AB) (AC) i AB = a. Se s (ABC), CS = a. a) Calculai aria totala i volumul piramidei SABC n funcie de a t t s \ b) Aflai m((SAB) , (ABC)). t c) Daca CP SB i Q este mijlocul muchiei (SA), sa se demonstr s laterul ABP Q este inscriptibil. 51

631 . Atunci a = .................. 1. Fie a = 2 22 23 2100 162 a 2. Stiind c 6 3 + bb + 2c = 493, atunci abc = ......................... a a13 a 3. Dac a = , a, b N , atunci cea mai mare valoare pentru b b+7 q ab + 36 / ab 36 N = .......... 4. Mulimea A = ab N | t
1

Testul 4 (prof. Lidia BOSNCIANU )

Soluiile problemelor propuse n nr. 1/2 t


Clasele primare

P.24. Aflai numerele a, b, c, d stiind ca verifica n acelai timp urm t s litati: a + 3 = b, b + 3 = c, c + 3 = d, a + 3 = 10. ( Clasa I ) nv. Maria Soluie. Din ultima relaie aflm pe a, a = 10 3 = 7, apoi b = t t a c = 10 + 3 = 13, d = 13 + 3 = 16.

P.26. Pe trei borcane de compot, unul de ciree, altul de viine si s s amestec de ciree si viine, toate etichetele au fost puse greit. Sco s s s t fruct dintr-un singur borcan, determinai coninutul fiecaruia. t t ( Clasa a II-a) Soluie. Se scoate un fruct din borcanul cu eticheta CV. Dac t a cireaa, atunci n borcanul cu eticheta V nu putem avea numai viin s s ciree. Rezulta ca avem ciree i viine. n acest caz avem coresponde s s s s coninut CV C, C V, V CV. Dac fructul extras este viin, t a s a corespondena CV V, C CV, V C. t

P.25. Un elev din clasa I, fixnd un numar din sirul numerelor natu ca suma numerelor din faa lui nu este mai mica dect 55, iar suma ace t cu numarul fixat nu depaseste pe 66. Despre ce numar este vorba? ( Clasa I ) Luminia Popa t Soluie. Suma numerelor din faa numarului cautat poate fi 55 t t doua suma poate fi 66, 65, 64, . . . . Daca a doua suma nu este 66, at mare valoare posibil a numrului cutat este 65 55 = 10. Suma p a a a numere nenule este 45, ceea ce nu corespunde datelor problemei. Deduc suma este 66. Daca prima suma nu este 55, atunci cea mai mare va a numarului cautat este 66 56 = 10 i iarai ajungem la o contradic s s cutat este 66 55 = 11. a

P.28. Cte pagini are o carte daca pentru paginarea ei s-a folosit c ori? ( Clasa a III-a) Crizantema Mironeanu Soluie. De la pagina 1 la pagina 100 se folosete cifra 9 de 20 ori. t s de la pagina 1 la pagina 600 se folosete cifra 9 de 120 de ori. Pentru a s ori cifra 9 trebuie sa eliminam paginile: 600, 599, 598. Cartea are 597 p

P.27. Sa se scrie numarul 31 folosind cele patru operaii aritme t cifra 3 (se cer cel puin doua soluii). t t ( Clasa a II-a) Andrea Balla, el Soluie. 1) [(3 + 3) 3 3] (3 3 : 3) + 3 : 3 = 15 2 + 1 = 31. t 2) [(3 + 3 : 3) 3 (3 3 : 3)] 3 + 3 : 3 = 10 3 + 1 = 31.

P.29. Ioana si Alina au cules mpreuna 165 de nuci. Ioana a cule nuci dect Alina; ea face un calcul si observa ca triplul diferenei di t nucilor culese de ele reprezinta tocmai numarul nucilor culese de Alina 52

P.32. Un parinte i mparte averea astfel: la primul copil 10 m s cincime din rest, la al doilea copil 20 de milioane plus o cincime din al treilea 30 de milioane plus o cincime din noul rest si aa mai depar s suma mparita de parinte, precum si numarul copiilor, stiind ca toi t t egale. ( Clasa a IV-a) Mihai G Soluie. Din faptul ca primii doi copii au primit sume egale rezulta c t 4 s 5 = 10, adic R1 R2 = 50. Al doilea copil primete din suma de a 5 4 ce nseamna ca din R1 depaete pe R2 cu 20 milioane. Avem R1 s s 5 t 4 R1 : 5 R2 = 20 de unde rezulta R1 : 5 = 30. Obinem R1 = 5 S = 10 + 150 = 160 (milioane). Deci suma mparita este de 160 mili t copil a primit 10 + 150 : 5 = 40 (milioane). Numrul copiilor este 160 : a

P.31. O veveria descopera un alun ncarcat cu fructe si i face p t s iarna transportnd la scorbura sa alternativ: o data doua alune, o da Dupa ce transporta 47 de alune, face o pauza pentru a se odihni. Sa s distana a parcurs veveria n total, daca de la alun la scorbura ei este t t x hm x dam x m, unde x are ca valoare cel mai mic numar natural po ( Clasa a IV-a) nv. Mihai Soluie. Numarul x nu poate fi 0. nseamna ca x este 1. Distana d t t la alun este de 1hm 1dam 1m= 111m. Pentru prima grup de 5 alune pa a Alun-Scorbur-Alun-Scorbur, deci 3 111m. Pentru fiecare grup de a a a restul de 42, parcurge traseul Scorbura-Alun-Scorbura-Alun-Scorbura, Deoarece sunt 8 grupe, veveria va parcurge 8 4 111m. Pentru restul t parcurge traseul Scorbur-Alun-Scorbur, deci 2 111m. a a n total veveria parcurge (3 + 32 + 2) 111m = 37 111m = 4107m t

P.30. Aratai ca dintre oricare patru numere naturale diferite, m t 1 000 000, se pot alege doua a caror diferena sa se mparta exact la 3. t ( Clasa a IV-a) Roxana Bolocan Soluie. La mprirea cu 3 resturile posibile sunt 0, 1, 2. nseamn t at doua numere din cele patru vor da acelai rest la mparirea cu 3. Fi s t b = 3d + r, a > b. Avem a b = 3c 3d = 3 (c d).

cules fiecare fata? ( Clasa a III-a) nv. Maria Soluie. Examinnd textul se constat c Alina are trei pri ia t a a at pari din cele cinci pari egale. Alina a cules 165 : 5 3 = 33 3 = 99 (n t t a cules 165 99 = 66 (nuci).

Clasa a V-a
V.26. Sa se determine cifrele distincte si nenule a, b, c, d, e, f, g pentru care rezultatul nmulirii alaturate este t cel mai mare posibil: Ioan Sacleanu, Hrlu a a Soluie. Avem ad = e < 10 i ac = g < 10. Deoarece t s 53

g g

cifrele a, b, c, . . . , g sunt distincte, rezult c ad, ac {1 2, 1 3, . . . , 1 a a 2 4, 3 1, 3 2}. Cum a 6= 1 (cci altfel am avea d = e) i c 6= 1, d 6= a s similare), urmeaz c ad, ac {2 3, 2 4, 3 2}. Ca urmare, pentru ca a a enun, sa fie maxim, luam g = 8, e = 6, a = 2, d = 3, c = 4. Au ramas d t b i f . Observm c b, f {1, 5, 7, 9}. Dar b 6= 1 (cci altfel f = 3 = s a a a (altfel f = 5 = b). Deci b = 7 i f = 1 sau b = 9 i f = 7. n consecina s s t mai mare este 209 403 = 84227 i cifrele cutate sunt: a = 2, b = 9, s a e = 6, f = 7, g = 8.

V.27. Trei apicultori au tras mpreuna 700 kg miere de albine. Cn mierea, primul apicultor a luat jumatate, al doilea jumatate din rest, al tate din noul rest, apoi operaiunea se repeta pna se mparte toata m t afle cta miere a luat fiecare. Ctlin-Cristian Bu a a Soluie. Se observ c primul apicultor ia de dou ori mai mult m t a a a a doilea, iar al doilea de doua ori mai multa dect al treilea. Daca x notea de miere luat de al treilea apicultor, atunci al doilea ia 2x, iar primul a relaia x + 2x + 4x = 700. Rezult c x = 100; deci al treilea ia 100 kg t a a doilea 200 kg, iar primul 400 kg.

V.29. Sa se afle numerele abc pentru care abc = ac b2 . Romana Ghia i Ioan t t s Soluie. Valorile b = 0 i b = 1 nu sunt posibile deoarece condiia t s t scrie a0c = 0 i respectiv a1c = ac i aceste egalitai sunt false. Nici s s t valoare posibil, cci 100a + 20 + c = 40a + 4c 60a + 20 = 3c, ceea c a a fals (3c poate fi cel mult 27). Pentru b = 3 avem: 100a + 30 + c = 90a + 9c 5a + 15 = 4c. U . implic c . 5, adic c = 5, precum i a = 1. Obinem c abc = 135 es a . a s t a

V.28. Aratai ca N1 = 32001 + 22001 si N2 = 32002 22002 sunt num t cu 5. Dorina Ca Soluie. Ultima cifra a unui numar de forma 24n este 6, iar a unuia t este 1. Prin urmare, 22000 = 24500 se termina n 6, iar 22001 = 22000 a cifr 2. La fel obinem c 32001 se termin n 3. Ca urmare, N1 se te a t a este, deci, divizibil cu 5. n privina numarului N2 , observam ca 32002 se termina n 9, 22002 t . N n 9 4 = 5. Deci, N . 5. .
2 2

problemei. Aratam ca nu putem avea b 4. ntr-adevar, abc = acb2 100a+ (10a + c) 10b = 10ab2 100a + b2 c c 10b = 10 b2 10 (1). Dac b 4, atunci b2 10 > 0, b2 1 > 0 i din (1) rezult c 10b a s a a (am minorat a cu 1 i c cu 0). Constatm c aceast inegalitate nu-i s a a a valorile b = 4, 5, . . . , 9. Numarul abc = 135 este singura soluie. t V.30. Daca xi , i = 1, 500 , sunt numere naturale nedivizibile 54

numarul N = 4x4 + 8x8 + 12x12 + + 2000x2000 este divizibil cu 5. 1 2 3 500 Tamara Soluia I (a autorului). Numrul xi , i = 1, 500, este de una t a M5 + 1, M5 + 2, M5 + 3, M5 + 4. Aratam ca x4 = M5 + 1. ntr i 4 (M5 + 1) = (M5 + 1) (M5 + 1) (M5 + 1) (M5 + 1) = M5 +1, (M5 + 2)4 = M5 + 1, (M5 + 3)4 = M5 + 34 = M5 + 1, (M5 + 4) = M5 + 4 Evident, avem i faptul ca numerele x8 , x12 , . . . , x2000 sunt de forma M s i i i N = 4 (M5 + 1) + 8 (M5 + 1) + + 2000 (M5 + 1) = M5 + 4 (1 + 2 + 501 500 = M5 + 2 500 501 = M5 . = M5 + 4 2 Soluia II (dat de eleva Tuescu Anca Stefania, Craiova). t a t 8 4 N = 4x1 4 + 4 + 8x2 8 + 8 + + 2000 x2000 2000 + 20 500

N = 4 x4 1 + 8 x8 1 + + 2000 x2000 1 + 4 (1 + 2 + 1 2 500 501 500 Cum 4 (1 + 2 + + 500) = 4 = 2 501 500, rezult c a a 2 al numrului N se divide cu 10. Pe de alt parte, pentru orice num a a nu-i divizibilcu 5, avem: U (x) {1, 2, 3, 6, 7, 8, 9}, U x2 {1, 4, 6 4, {1, 6}, U x8 {1, 6} Ca x2 . etc. urmare, U x4 1 , U 8 1 , . . , U 1 4 {0, 5}; iar U 4 x1 1 , U 8 x8 1 , . . . , U 2000 x2000 1 { 2 500 n consecina, toi termenii din scrierea (1) a lui N sunt divizibili c t t . N . 10. .

Clasa a VI-a

VI.26. Fie A = 4a + 6b c, B = 4a 3b c, C = 3a 11b a, b, c Z. Daca (A, B) = 23, aratai ca (A, B, C) = 23. t Cristiana Constanda . Soluie. Nu trebuie s artm, de fapt, dect c C . 23. Avem c t a aa a . . . . deci 9b . 23 i cum (9, 23) = 1, atunci b . 23. Din A = 4a + 6b c . 23, u . s . . . . 23, deci c = 4a 23k, k Z. n aceste condiii, C = 3a c 4a c . a t VI.27. Sa se rezolve n Z sistemul: 3x + 2y 8;

= 3a 11b 28 (4a 23k) = 115a 11b + 28 23k, fiecare termen f de 23.

x y 1; 3x Mihai Crci a

y = 3x 1 3 (1 + y) 1 = 2 + 3y 2y 2 y 1; deoarece y y+1 i singurele soluii convenabile sunt (0 y {1, 0, 1, 2}. nsa x = s t 3 VI.28. Sa se rezolve n N ecuaia t 1 2 + 2 3 + + n (n + 1) (n + 1) (n + 2) 2n = 2 + 4 55

Soluie. Avem: y = 3x 1 8 2y 1 = 7 2y 3y t

Dumitru - Dominic B

1 [k (k + 1) (k + 2) 3 n care dm valori lui k de la 1 la n, obinem prin sumare 12+23+ a t 1 = n (n + 1) (n + 2). Scaznd n ambii membri ai ecuaiei date suma 1 t 3 obinem echivalent: t Soluie. Plecnd de la identitatea k (k + 1) = t

VI.30. Pe ipotenuza (BC) a triunghiului dreptunghic ABC se considera punctele N si M astfel nct BN = AB, CM = AC. Daca P si Q sunt proieciile punctelor t M si N pe dreptele AN , respectiv AM , demonstrai ca t segmentele (M P ), (N Q) si (P Q) se pot constitui n laturile unui triunghi. Ctlin Calistru, Iai a a s Soluie. Fie {R} = M P N Q ortocentrul 4AM N ; t atunci ARBC. Avem: b \ [ \ m(BAM ) = m(BAR) m(M AR) = 90 m(B) b \ \ 90 m(AM C) = m(AM C) m(B) =

n (n + 1) n (n + 1) (2n + 1) n (n + 1) (n + 2) 2n (2n + 1) = = 3 2 2 6 n+1 altfel spus = 1, ceea ce antreneaza n = 5. 6 VI.29. n triunghiul ascuitunghic ABC, bisectoarea interioara a t intersecteaza nalimea AD n E, D [BC]. Fie F (DC astfel nc t Aratai ca BEAF . t Tamara Soluie. Fie E 0 [AB] astfel nct EE 0 AB. Cum t b E se afla pe bisectoarea lui B, este egal departat de la 0 turile unghiului: ED = EE . Atunci 4AEE 0 4F ED \ \ a a (C.I.), deci AEE 0 DEF , de unde rezult c E 0 , E, F sunt coliniare, adica F E 0 AB. Urmeaza ca E este or tocentrul 4ABF , aadar BEAF . s

b b b \ \ \ = m(M AC) m(B) = m(M AR) + 90 m(C) m(B) = m(M 1 b \ \ \ Analog se arata ca i m(CAN ) = m(N AR), deci m(M AN ) = m(A) = s 2 4P AM i 4QAN sunt triunghiuri isoscele, de unde M P = AP i s s Urmeaza ca (M P ), (N Q), (P Q) se pot constitui n laturile unui triu 4AP Q.

p a + 2 2 Q. Gheorghe p Soluie. Fie x = a + 2 2 Q. Atunci a + 2 = 2 + 2x 2 t 1 9 a, x Q, urmeaza ca a = 2 + x2 i 1 = 2x. De aici, x = i a = . R s s 2 4 VII.26. Determinai a Q stiind ca t 56

Clasa a VII-a

a=

Cum dorim ca sistemul s aib soluii ntregi, rezult a = 2k, k Z. a a t a este imediat c x1 = x2 = = xn = k constituie soluie a sistemulu a t VII.28. Fie zece numere naturale nenule care au suma egala cu 55 ca printre ele exista trei care pot fi lungimile laturilor unui triunghi. Adrian Za Soluie. S ordonm cresctor numerele: 1 a1 a2 t a a a c ai+1 < ai + ai+2 , i = 1, 8; ar mai trebui s artm c exist u a a aa a a {1, 2, . . . , 8} pentru care ai+1 > ai+2 ai . Pentru aceasta, sa presupune ai+1 ai+2 ai , i = 1, 8, i.e. ai+2 ai + ai+1 , i = 1, 8. Av a3 a1 + a2 1 + 1 = 2, a4 a2 + a3 1 + 2 = 3 i n continuare a5 s a7 13, a8 21, a9 34, a10 55. Atunci a1 +a2 + +a10 1+1+ adica 55 143, absurd. VII.29. Fie ABCD un patrat, O centrul sau, iar M si P mijloacel (OA), respectiv (CD). Sa se arate ca triunghiul BM P este dreptungh Constantin Cocea i Dumitru s Soluia I (dat de elevul Mihul Andrei, Iai). Fie t a s M 0 , M 00 proieciile punctului M pe BC i respectiv CD. Avem: t s 1 1 0 0 00 s M M = (AB + OO ) i M M = (AD + OP ) (ca linii mi2 2 0 s s jlocii n trapezele ABO O i ADP O) i deci M M 0 = M M 00 . Evident, M 0 B = M 00 P . Deducem c 4BM 0 M 4P M 00 M , a MM deci M B M P i BM M 0 P\ 00 . Ultima relaie conduce s \ t 0M M 0M M \ M\ 00 . Unghiul M\ 00 ind drept, urmeaz a la BM P ca BM P este unghi drept i, deci, 4BM P este dreptunghic isoscel. s \ Soluia II. Fie a latura ptratului. Avem c P A2 = t a a 5a2 . Aplicnd teorema medianei n 4P AO AD2 + DP 2 = 4 10a2 10a2 i n 4BOA, obinem P M 2 = , BM 2 = , deci s t 16 16 2 5a i atunci lungimile s P M = BM . Pe de alt parte, BP 2 = a 4 P M , M B, P B sunt numere pitagoreice, de unde concluzia. VII.30. Fie ABCD un patrat de latura 1 si punctele M (AD) 1 , demonstrai ca 1 < AM + t {P } = BM AN . Daca SDCN P M = 2 57

p 9 2 2+1 1 avem ca a + 2 2 = 2 = Q. 4 2 2 VII.27. Determinai a R astfel nct sistemul t a2 a2 x2 +(a + 1) x1 + = x2 , . . . , x2 +(a + 1) xn1 + = xn , x2 +(a + 1 1 n1 n 4 4 sa admita numai soluii ntregi. t Ctlin C a a Soluie. Adunnd membru cu membru ecuaiile, efectund reduceri t t n X a 2 obinem c t a xi + = 0, de unde n mod necesar x1 = x2 = 2 i=1

Emil Vas Soluie. Fie x = AM , y = BN , z = P P 0 (unde P P 0 AB). t 1 Din ipoteza, SABM +SABN SABP = , prin urmare x+yz = 2 z BP 0 P 0A xy x+y z + = 1, deci z = , 1. Dar + = x y AB AB x+y 4 unde am inut seama de inegalitatea ntre media armonica i t s x+y , adica cea aritmetica. Rezulta ca x + y = 1 + z 1 + 4 3 xy x + y . Pe de alt parte, x + y a = 1 implic a 4 x+y 4 1 4 xy = (x + y) (x + y 1) = ; am folosit faptul evident ca x + 3 3 9

AM BN

4 . 9

VIII.26. Demonstrai ca ecuaia t2 + 1 x2 + 4t2 x + 4t2 5 = 0 ar t t soluii n Z Z. t Mihai Crci a 2 a Soluie. Ecuaia se scrie echivalent t2 (x + 2) = t x2 . Dac x t t t x2 = 0, deci t = 4. Daca x Z\ {2}, atunci t2 (x + 2)2 t, iar tx tatea este atinsa daca i numai daca t = x = 0. n concluzie, S = {(0, 0 s

Clasa a VIII-a

t VIII.27. Determinai a R stiind ca ecuaia x4 2x3 + 3x2 2x t singura soluie reala. t Gabrie Soluie. Dac P (x) este expresia din membrul stng al ecuaiei, p t a t

P (x) = x4 2x3 + 3x2 2x + a = x2 (1 x)2 2x (1 x) +

i de aici se observa ca P (x) = P (1 x), x R. Cum ecuaia are o s t 1 a t real x0 , trebuie ca x0 = 1 x0 , adic x0 = . nlocuind, obinem n a 2 2 7 7 1 7 2 =0 x x+ c a = a . Dac a = a , ecuaia devine x t 16 16 2 4 1 singura soluie real (dubl) x = . t a a 2 VIII.28. Fie a, b numere naturale prime ntre ele. Aflai valorile t care Sn = an + an1 b + an2 b2 + + abn1 + bn este divizibil cu a + Mihaela Prede Soluie. Dac n impar, atunci Sn are un numr par de termeni i a t a a s (an + bn )+ an1 b + abn1 + = (a + b) an1 an2 b + + bn1 . an3 an4 + + bn3 + = (a b) A, deci Sn . a + b. Dac + . b atunci Sn = a an1 + an2 b + + bn1 + bn i cum n 1 impar, s . . . . divide cu a + b. Rezulta ca Sn . a + b bn . a + b; vom arata ca aceasta este imposibil n condiiile date. Avem c (b, a + b) = 1, deoarece a t a d | a + b, atunci d | a i d | b, adic d | (a, b), deci d | 1. Urmeaz c n de s a a a 58

lor, numerele b i a + b nu au nici un factor prim comun, afirmaie vala s t numerele bn i a + b. n concluzie, pentru n par, Sn nu se divide cu a + s

VIII.29. Se considera piramida triunghiulara regulata V ABC cu l iar muchia laterala 2a. Fie M mijlocul lui (V A), iar N un punct pe 3a nct V N = . Aflai distana de la V la planul (M N C). t t 4 Adrian Cordu a 6 . Fo Soluie. Cu teorema medianei n 4V AC, obinem CM = t t 2 a 31 . Din teorema cosinusulu lui Stewart n 4V BC, rezult CN = a 4 a obinem M N = . Cunoatem prin urmare laturile 4CM N i cu formu t s s 2 2 10a 15 . Calculnd acum volumul tetraedrul aflm aria sa: SCMN = a 128 a3 doua moduri, cnd folosim drept baza 4V M N gasim ca VV MN C = 6 lum drept baz 4CM N , a a volumul fiind cunoscut, obinem c distan t a a 165 . planul (M N C) este h = 25

VIII.30. Fie A, B, C, D patru puncte necoplanare astfel nct A CD = 4 29. Notam cu E, F mijloacele segmentelor (AB), respect se arate ca mijloacele segmentelor (AF ), (BF ), (CE), (DE) sunt v paralelogram si sa se calculeze aria acestuia stiind ca are o latura de lu Romana Ghia i Ioan t t s Soluie. Fie M, N, P, Q mijloacele segmentelor (EC), (AF ), (ED t [BF ]. n 4ECD, (M P ) este linie mijlocie, iar (EF ) este mediana; i (EF ) se njumataesc. Raionnd analog n 4F AB, deducem ca ( s t t se njumtesc. Rezult c M P i N Q sunt concurente n mijlocu a at a a s se njumtesc, deci M, N, P, Q sunt coplanare i M N P Q este para a at s 1 1 O centrul acestuia. Avem ca OQ = AB = 73, OP = CD = 4 4 P Q = 194, aria 4OP Q se poate calcula cu formula lui Herron sau afl 1 1 obinem SOP Q = i deci SMON = . Cum (P O) este mediana n 4 t s 2 2 1 1 ca SP ON = SP OQ = i deci SOMQ = . n final, SM N P Q = 2. s 2 2

a a = {x} + [x] {x D. M. Btineu-Giurgiu a t Soluie. Pentru existena numitorilor, x [0, 1) i x Z. Ec t t / s / a echivalent ([x] {x}) 1 = 0, iar prima paranteza nu se [x] {x} a Ramne ca [x] {x} = a, deci {x} = , unde n = [x] este din N deoa n IX.26. Daca a (0, ), sa se rezolve ecuaia [x] + t 59

Clasa a IX-a

a , n N , n > [a]. n IX.27. Sa se determine funciile f, g : [0, ) [0, ), unde g este t aditiva si g (y) + g (f (x)) = f (x + g (y)), x, y [0, ). Ioan Scle a a t Soluie. Fie y0 [0, ) pentru care g (y0 ) = 0; atunci obinem c t f (x), x [0, ) (1). Prin urmare, g (y) + f (x) = f (x + g (y)), relaie care pentru x = 0 arata ca f (g (y)) = f (0) + g (y), y [0, ) t este surjectiv, pentru orice z [0, ), gsim y [0, ) astfel nct g a a (2) deducem f (z) = f (0) + z, z [0, ). nlocuind n (1) i folosin s este aditiva, obinem ca g (z) = z, z [0, ). t {x} 0. Atunci x = n +

= x + , R fixat, iar funcia g = f 1R este monotona. t Mihail Ben Soluie. Aplicnd f n ambii membri ai egalitaii din enun obinem t t t t = f (x) + , x R, de unde g (x + ) = f (x + ) (x + ) = f (x) x R.Cum g monoton, rezult c g constant: g (x) = k, x a a a a f (x) = x + k, x R, deci (f f f ) (x) = x + nk = x + . Urm | {z } adic f (x) = x + a , x R. n IX.29. Sa se arate ca n orice triunghi ABC are loc inegalitatea 3 3R p l3 l3 la + b + c p (p3 3abc) ha hb hc 2r
n ori

IX.28. Sa se determine funciile f : R R pentru care (f f t | {z

no

Viorel Cornea i Dan Stefan Marinescu, s Soluie. Din egalitatea Cauchy-Buniakowski-Schwarz, obinem ca t t X 3 2 X X 1 la 6 . la ha h2 a 1 Se tie c OG2 = R2 a2 + b2 + c2 0, deci a2 +b2 +c2 9R2 i atu s a s 9 2 2 2 2 X 1 a +b +c 9R = . Acum al doilea factor: h2 4S 2 4S 2 a r 2bc A p (p a) p la = cos bc = p (p a), deci b+c 2 bc X X 3 6 (p a) = p3 3p3 3p2 (a + b + c) + 3p a2 + b2 + c2 a la p3 Un calcul de rutin arat c a3 + b3 + c3 = 3p a2 + b2 + c2 + 3abc a a a X 3 6 rezulta ca la p3 p 3abc . Revenind n (1), deducem concluzia 60

IX.30. n patrulaterul ABCD consideram punctele R si S pe diag interioarele triunghiurilor ABC, respectiv ACD. Notam {M } = CR

= AR BC, {P } = AS CD si {Q} = CS AD. Stiind ca +

AM n BN n CP n DQn DQ2 = 4, sa se arate ca + + + = 4, n N QA2 M Bn N Cn P Dn QAn Ctlin C a a Soluie. Aplicnd teorema lui Ceva n 4ABC i 4ACD i combi t s s AM BN CP DQ obinute, rezult c t a a = 1. Putem scrie: M B N C P D QA s BN 2 CP 2 DQ2 AM 2 BN 2 CP 2 D AM 2 + + + =44 M B2 N C2 P D2 QA2 M B 2 N C 2 P D2 Q AM BN deci este atins egalitatea n inegalitatea mediilor; atunci a = = MB NC Concluzia este acum imediat. a

AM 2 B + M B2 N

Clasa a X-a

X.26. Fie ecuaia x4 S1 x3 + Sx2 + mx m 1 = 0, unde S e t triunghi neechilateral ABC, iar S1 este aria triunghiului A1 B1 C1 d punctele de intersecie a bisectoarelor interioare cu cercul circumscri t t ABC. Sa se determine m R stiind ca ecuaia admite un numar impa n (0, 1). Dumitru Gherm Soluie. Dac notm f (x) = x4 S1 x3 + Sx2 + mx m 1, atun t a a ecuaia data sa admita un numar impar de radacini n (0, 1) este echival t f (1) < 0, adica (m 1) (S S1 ) < 0 (). Sa determinam acum sem A+B A+C 2 2 s SS1 . Avem S = 2R sin A sin B sin C i S1 = 2R sin sin 2 2 A A B C B C = 2R2 cos cos cos , de unde rezult c SS1 = 8 sin sin sin a a 2 2 2 2 2 2 tate daca i numai daca triunghiul ABC este echilateral. Deci, n condii t s s t t are loc S < S1 i atunci, avnd n vedere relaia (), obinem m ( X.27. Fie r [1, ), D = {z C; |z| r} si P C [X], P ( +bX + c. Sa se arate ca daca P (z) D, z D, atunci a, b, c D. D.M. Btineu-Giurgiu a t Soluie. Fie 1, i 2 rdcinile ecuaiei x3 = 1. Deoarece 1, , 2 t s a a t s c P (1) = a + b + c D, P () = a2 + b +c D i P 2 = a + a De aici, obinem c 3c = P (1) + P () + P 2 , 3a = P (1) + P ( t a i = P (1) + 2 P P 2 . Folosind aceste at () + s 3b egaliti deducem = P (1) + P () + P 2 |P (1)| + |P ()| + P 2 3r i, analog s 3 |b| 3r. Deci, a, b, c D. 2 X.28. Rezolvai ecuaia z 2 2|z| 1 + z 2|z1| 1 + 1 = 0, z t t Emil Vas 2 s Soluie. Fie z = a + bi (a, b R, b 6= 0), 2|z| 1 = R i 2|z1| t Cu aceste notaii ecuaia noastr devine: a2 b2 + 2abi + (a + bi) t t a 2 2 2 2 t 2ab 2 + b = a b + a + 1 = 0, de unde obinem a b + 2 2a +b 1 a2 + b2 = 1 (). 61

Deoarece funcia f (t) = t (2t 1) este strict cresctoare pe (0, t a ecuaia () are o soluie unic i anume a2 + b2 = 1. De aici deduc t t a s i nlocuind n egalitatea 2a + = 0, obinem = 2a, adic 2 2 s t a Dac facem notaia c = 1 2a, cum |a| 1, rezult c 3. Pe de a a t a 2 1+c = c 0, obinem c 2, deci c = 2 1+c 2 3 > 25/3 > 3. C t care am ajuns arata ca ecuaia data nu are soluie. t t X.29. Un motan scoate cu ajutorul unui pahar un numar de pe un acvariu. Ci petiori trebuie sa conina acvariul astfel nct mo t s s t matematic sperana ca va scoate 5 dintre ei? t Gabrie Soluie. Notam cu n numarul de petiori din acvariu. Fie X varia t s s care ia ca valori numrul de petiori care se afl n paharul motan a s s a culm pk = P ({X = k}). Deoarece sunt n total 2n cazuri egal posib a submulimilor unei mulimi cu n elemente), dintre care sunt favorab t t k Cn s t pk = n . Aadar, tabloul de repartiie al variabilei aleatoare X este: 2 0 1 2 ... n 0 1 2 n X : Cn Cn Cn Cn . ... n n n n 2 2 2 2 Motanul poate spera c va extrage un numr de petiori egal cu speran a a s s t (sau media) variabilei X, adica n n X X kC k n2n1 n n E (x) = m = pk xk = = = . 2n 2n 2 k=1 k=1 Prin urmare, avem n/2 = 5, deci n = 10. X.30. Fie M = {1, 2, . . . , n}. Sa se afle numarul de k-uple (A1 , A k ! k [ \ Ai = M si Card Ai = l, l submulimi ale lui M astfel nct t
i=1 i=1

= (1, 1, . . . , 1)}) = l. Cum numrul de moduri n care l elemente din a l l prin f n (1, 1, . . . , 1) este Cn , rezult c rspunsul problemei este Cn a a a 62

s = (j1 , j2 , . . . , jk ) N, unde ji = 1 daca j Ai i ji = 0 daca exemplu, f (3) = (1, 1, 0, . . . , 0, 1) daca i numai daca 3 A1 , 3 A s n a t 3 A3 , A4 , . . . , Ak1 ). Numrul acestor funcii este 2k 1 , dar nu / ultima condiie din ipotez. t a Observam ca j A1 A2 Ak daca i numai daca f (j) = ( s k ! \ N . Deci, condiia Card t Ai = l este echivalenta cu Card ({j
i=1

Lucian-Georges L a Soluie. Fie N = {(0, 0, . . . 0, 1), (0, 0, . . . , 1, 0), . . . , (1, 1, . . . t mulimea k-uplelor formate din 0 i 1, fr elementul (0, 0, . . . , 0, 0). Mu t s aa 2k 1 elemente. A partiiona M = {1, 2, . . . , n} n k submulimi (A1 , A2 , . . . , Ak t t k [ Ai = M este totuna cu a defini o funcie f : M N prin l t
i=1

Clasa a XI-a

t de unde, notnd cu C = A t (A ), obinem tr (C t C) = 0. Ultima rela cu observaia C Mn (R), ne conduce la condiia C = On , adica tA = t t XI.27. Fie a [0, 1) si (xn ) n0 un sir de numere reale astfel nc 1 2 2 2 2 x + xn1 , n N . xn a max xn1 , 2 n Aratai ca sirul (xn ) n0 este convergent si determinai limita sa. t t Aurel Mun 1 2 x + x2 Soluie. Daca max x2 , t = x2 , atunci x2 n1 n1 n1 n 2 n 1 2 x + x2 |xn | k1 |xn1 |, unde k1 = a [0, 1). Daca max x2 , n1 n 2 n a 2 a 2 a 1 2 2 x x2 x + x2 x + x2 = n1 , avem xn n1 sau 1 2 n 2 n 2 n 2 n1 r a a s x2 i deci |xn | k2 |xn1 |, unde k2 = [0, 1). 2 a n1 2a Fie k = max {k1 , k2 }. Deoarece k [0, 1) i |xn | k |xn1 | k 2 |x s k n |x0 |, rezult c irul (xn )n0 converge la zero. a as XI.28. Sa se determine p R pentru care limita sirului (an )n n X np p este finita si nenula. termenul general an = k + k2 1 k=1 Constantin Soluie. Avem succesiv: t n n X X np p 2 p n k+1 k = = an = 2 k + k2 1 k=1 k=1 n np 2 np 2 X = k+1 k = n+11 = 2 2 k=1 ! r 1 2 p+ 1 1 = 1+ . n 2 2 n n 1 Deci, irul (an ) are limita finita nenula daca i numai daca p = . s s 2 63

XI.26. Fie A Mn (R). Daca tr (tA A + (tA) A ) = 2n det A, at Iuliana Georgescu i Paul Geo s Soluie. Se tie ca tr (X + Y ) = tr X + tr Y ( R), tr (XY t s XX = (det X) In i (tX) = t (X ), X, Y Mn (R). Egalitatea d s astfel: tr tA A + tA A = tr A A + tA tA t t t tr A A + A t (A ) = tr A A + A (A ) tr A tA A + A tA t (A ) = 0 tr A t (A ) tA A = 0,

! 1 1 n 1 + + + 1 = 1. 2 n Marian Te n (un 1) Soluie. Scriem termenul general xn sub forma: xn = t ln an r 1 1 1 1 n i bn = ln n. Din un = 1 + + + , an = 1 + + + s 2 n 2 n r 1 1 < n 1 + + + < n n, n 2 deducem c lim un = 1. Atunci, a a n 2 n n XI.29. Sa se arate ca lim n ln ln n lim

n (un 1) un 1 un 1 = lim = lim = n n ln un n ln [1 + (un 1)] ln an an = 1. n consec Folosind criteriul lui Stolz-Cesro, gsim c lim a a n bn ln (an /bn ) ln an = lim + 1 = 1. De aici i din (1), obinem c s t a lim n ln bn n ln bn XI.30. Fie f : R R o funcie discontinua si care are proprietatea t Daca exista o funcie g : R R R astfel nct f (x + y) = g (f (x) , y) t x, y R, atunci funcia f nu are limita la . t Stefan Al Soluie. Dac f ar fi injectiv, cum f are proprietatea lui Darbou t a a c f este continu, ceea ce contrazice ipoteza. Deci f nu este injectiv i a a as a, b R, a < b astfel nct f (a) = f (b). Aadar, avem f (a + x) = s = g (f (b) , x) = f (b + x), x R, de unde rezult c f (x) = f (x + b a a adic f este periodic i T = b a este o perioad a ei. Cum f est a a s a rezult c f nu este constant i deci exist , R, 6= astfel nct a a as a Considernd irurile xn = + nT i yn = + nT , n N, care tind s s f (xn ) = f ( + nT ) = f () f () i f (yn ) = f ( + nT ) = f () s ce demonstreaz c f nu are limit la +. a a a

a2 a2 1 a2 XII.26. Se considera multimea M = a2 1 + 2 a 2 a 1 a 2 a 2 unde A = Z sau A = Q sau A = R. Aratai ca (M, ) este grup; este a t cu A , ? + Gheorghe Co Soluie. Notm cu M (a), a A, un element oarecare al mulimii t a t M (a) M (b) = M (a + b) (), a, b A, rezulta ca nmulirea este lege t ie intern pe M . Folosindu-ne de relaia () i avnd n vedere c a t a t s a asociativ i comutativ pe A rezult c nmulirea este asociativ i c as a a a t as M . Mai mult, se observ c M (0) este element neutru pentru nmul a a orice M (a) M admite un simetric, i anume, M (a) M . Prin u s este grup comutativ. n fine, se verific uor c funcia f : M A , f a s a t + este un izomorfism ntre grupurile (M, ) i A , . s + 64

Clasa a XII-a

XII.27. Fie (G, ) un grup cu Z (G) 6= {e} si H un subgrup netri

Sa se demonstreze ca exista x, y G\H, x 6= y 1 , astfel nct xy H Dai exemplu de grup care nu are aceasta proprietate. t Ovidiu Munteanu, stud Soluie. Oricare ar fi x G\H i oricare ar fi u H\{e}, avem y = t s (ntr-adevr, dac y = x1 u H, atunci y 1 H i deci x = uy 1 a a s este fals). De aici, deducem c xy = u H. Deoarece yx = x1 u a mai demonstrm c exist x G\H i u H\{e} astfel nct x1 ux a a a s Z (G) = {a G | ab = ba, b G} 6= {e}, rezulta ca exista x G\H astfel nct xu = ux, deci x1 ux = u H. Cu aceasta prima parte a este ncheiat. a Pentru a doua parte, considerm S3 = e, , , 2 , , 2 i H a s observ c nu exist x, y G\H astfel nct xy = yx = . ntr-a a a a prin absurd, ar exista x, y G\H astfel nct xy = yx = , atunci x1 x = , deci x = x. Cum nu comuta cu nici un element din G\ ca ultima egalitate este falsa. Z n , pentru n {2, 3, 4}. tg xdx, x 0, XII.28. Calculai t 2 Daniel Ji Z Z n n Soluie. Notnd In = t tg xdx, Jn = ctg xdx i efectun s Z Z tn tn2 n tg x = t, obinem: In = n t dt, Jn = n dt. 1 + t2n 1 + t2n Pentru n = 3, Z Z Z 1 u 1 u+1 t3 t2 =u 3 dt = du = I3 = 3 + 1 + t6 2 1 + u3 2 u + 1 u2 u + 1 Pentru n = 2, considerm a 1 1 Z Z 2 Z t2 1 2 t t 1 t t dt = 2 I2 J2 = 2 dt = 2 2 1 1 + t4 1 t2 t2 + 2 t t t Prin urmare, tg x + ctg 1 tg x ctg x +C, I2 J2 = ln I2 +J2 = 2 arctg 2 2 2 tg x + ctg i se calculeaz uor valoarea lui I2 . s a s Pentru n = 4, procedam la fel: Z 4 Z 1 1 t t2 t2 dt = I4 J4 = 4 dt = 4 8 1 1+t t4 + 4 t 0 1 Z Z t du t = 4 dt = 4 4 2 4 4u2 + u 1 1 t +2 4 t t t 65

XII.29. Fie f : R R o funcie continua si t > 0. Pentru a, t calculeze n t X k t (k 1)t b a+b 1 a . k (k 1) lim f n n nt k=1 Mihail Ben t t 0 1 nt , , . . . , t o diviziune a intervalului [ Soluie. Fie n = t nt nt n k t (k 1)t nt (n 1)t = max 0 pentru n . kn k = nt nt k{0,1,...,n} puncte intermediare media geometrica ponderata a punctelor de divizi 1 a+b ! t a t b t (k 1) b a+b k = nt ka (k 1) , k= k = nt nt Atunci n (f, ) =

n t a+b X k t (k 1)t . f nt k a (1 k)b nt k=1 Z 1 Cum f : [0, 1] R este continu, rezult c lim n (f, ) = a a a f (x) n 0 Z t h i 2 ln (1 + x) e XII.30. Sa se arate ca ex 2 dx 8 ln 2, 16 ln 2 . (1 + x2 ) 0 Cristian Moan

Deoarece ex 1 + x2 , x R, avem c f (x) 1; deoarece f este cresc a a 3 x2 2x e e (cci f 0 (x) = a 0, x [0, 1]), rezult c f (x) f (1) = . C a a 2 )2 2 (1 + x Z Z 1 e 1 ln (1 + x) ln (1 + x) dx I dx 1 + x2 2 0 1 + x2 0 Z 1 ln (1 + x) i rmne de artat c J = dx = ln 2. s a a a 1 + x2 8 0 ntr-adevar, cu schimbararile x = tg t i t = u, vom avea s 4 Z 0 Z /4 ln (1 + tg t) dt = ln 1 + tg u du J = 4 0 /4 Z /4 Z /4 2 ln ln 2du J, du = = 1 + tg u o 0 de unde J = ln 2. 8

Soluie. Notm cu I integrala din enun i fie f : [0, 1] R, f ( t a ts

66

Soluiile problemelor pentru pregtirea concu t a din nr. 1/2002


A. Nivel gimnazial

G6. Daca un numar natural se poate scrie ca suma a doua pa nenule distincte, atunci orice putere a sa se poate scrie, de asemenea doua patrate perfecte nenule.

Soluie. Fie a = b2 + c2 , cu b, c N , iar n N . Dac n = 2 t a 2 k 2 k 2 2k 2k 2 k k b + c = a b + a c , cu a b, a c N . Daca a =a a=a demonstra mai nti afirmaia pentru k = 1. ntr-adevr, a2 = b4 + t a 2 2 = b2 c2 + (2bc) , cu B = b2 c2 , C = 2bc N . Pentru k 2 2 an = a2k2 a2 = a2(k1) B 2 + C 2 = ak1 B + ak1 C , cu ak1 B G7. Aratai ca numarul aa . . . a (2001 cifre) nu poate fi patrat pe t ar fi cifra a n baza 10.
n

Soluie. Afirmaia este adevrat n cazul general al unui num t t a a n 2 cifre. Numerele 22 . . . 2, 33 . . . 3, 77 . . . 7 i 88 . . . 8 nu pot fi pa s din cauza ultimei cifre. Cum orice ptrat perfect este fie de forma 4k a 4k + 1, k N, nu pot fi ptrate perfecte numerele 11 . . . 1, 55 . . . 5, 66 . . a n sfrit, dac 44 . . . 4 = 4 11 . . . 1 ar fi ptrat perfect, atunci 11 . . . s a a perfect, absurd. 3n (18n + 13) 28 este fracie t G8. Determinai n Z pentru care t 3n + 1 Dumitru - Dominic B Soluie. Deoarece 3n (18n + 13) 28 = (3n + 1) (18n + 7) 35, fr t simplific prin d N\ {0, 1} dac d este un divizor al lui 35. Pentru d a a ca 3n + 1 = 5l, l Z, ecuaie diofantica cu soluia particulara l = 1, t t generala l = 1+3k, n = 2+5k, k Z. Pentru d = 7, gasim l = 3p+ p Z. n concluzie, valorile cutate ale lui n sunt {5k 2 | k Z}{7p a G9. Se dau trei fiicuri de monede aezate vertical, asupra carora p s s una dintre operaiile O1 : luam cele doua monede de deasupra unui fiic t s peste altul, sau O2 : luam cele doua monede de deasupra unui fiic si le s una peste fiecare dintre celelalte doua fiicuri. s a) Gasii o condiie necesara pentru ca, dupa un numar de operaii, t t t t sa conina la fel de multe monede; t b) Aratai ca aceasta condiie nu este suficienta daca este permisa o t t raie, nsa este suficienta n cazul n care sunt permise amndoua. t Gabrie Soluie. Deoarece numrul total de monede rmne constant pe p t a a turii operaiilor, acest numr trebuie s fie n mod necesar un multi a t a a mare sau egal cu 6. Presupunnd ca distribuia iniiala a monedelor este (3, 2, 1), n con t t este permis o singur operaie, se arat c egalizarea celor trei fiicuri nu a a t a a s considernd toate micrile ce pot fi efectuate. n cazul n care ambele s a 67

permise, aeznd n mod repetat cte dou monede din fiicul cel m s a s cel mai mic, ajungem fie ca fiicurile s se egalizeze, fie ca n vrfurile s a O o situaie de tipul (3, 2, 1). n aceast situaie, succesiunea (3, 2, 1) t a t (2, 2, 2) rezolv problema. a

1, . . . , 1. Urmeaza ca u1 = u2 = = un+1 = 1, de unde x1 = x2 =

G10. Pentru n N, n 2, rezolvai ecuaia t t r r r n+1 n+1 n+1 x1 + x2 + . . . xn + x1 + x2 + + n n n Mihai Tot r n+1 Soluie. Cu notaiile ui = t t xi , x = 1, n, un+1 = x1 + n obinem ca u1 + u2 + + un+1 = n + 1, iar u2 + u2 + + u2 = n t 1 2 n+1 (u1 + u2 + + un+1 )2 = (n + 1) u2 + u2 + + u2 , deci este atinsa 1 2 n inegalitatea Cauchy-Buniakowski-Schwarz aplicata numerelor u1 , u2 , .

t G11. Rezolvai n N N ecuaia x2 + y 2 = 5445. t Daniela Iosub Soluie. Vom folosi urmatorul rezultat din teoria numerelor: daca p t un numar prim si p | a2 + b2 , a, b N , atunci p | a si p | b. Din ipote i 11 | x2 + y 2 , x, y N , iar 3 sau 11 sunt numere prime de forma s urmare, 33 | x i 33 | y, deci x = 33l, y = 33m, l, m N . nlocuin s obinem ca l2 + m2 = 5, l, m N , adica (l, m) {(1, 2) , (2, 1)}. De t {(33, 66) , (66, 33)}. G12. Sa se determine n, m N pentru care 1 + 2 + ...[ Adrian Za Soluie. Pentru n = 1, m N relaia data se verifica. Cautam sol t t Nu putem avea m 2, deoarece h i h i 1 + 2 + . . . n 1 + 2 + + n < n n < n2

Ramne de cercetat cazul m = 1; se observa ca n = 2 i n = 3 dau solu s t iar pentru n 4 obinem t h i h i 1 + 2 + . . . n 1 + 1 + 1 + 4 + + n > 3 + 2 (n 3) =

s G13. Aratai ca numerele 18n si 2n + 18n , n N, au acelai numa t Gheorghe Soluie. S presupunem prin reducere la absurd c 18n are k cifre t a a are mai mult de k cifre, deci 2n + 18n 10k > 18n 10k1 . Evident, k mparind prin 2n aceasta inegalitate, obinem: t t 1 + 9n 5k 2kn > 9n 2kn 5k = 1 + 9n ,
n

adic nu mai gsim soluii. n concluzie, (n, m) {(1, a) | a N } { a a t

deoarece 2kn 5k N. Pe de alt parte, 1 + 9n = 1 + (8 + 1) = a 2kn 5k = M 4 + 2, de unde k n = 1; relaia (1) devine 2 5n+1 = t 68

a adic 9n > 10 5n = 2 5n+1 , deci (2) nu este verificat pentru n 4. a obinut ncheie demonstraia. t a t G14. Sa se arate ca nu exista nici un triunghi dreptunghic avnd ca raionale, iar ipotenuza egala cu 2001. t Constantin x2 z2 Soluie. Pentru a arta c ecuaia 2 + 2 = 2001 nu are solu t a a t t y t 2 2 2 suficient s demonstrm c ecuaia m + n = 2001p a a a t (1) nu are Folosind rezultatul amintit n soluia problemei G11 i observnd c 3 | 2 t s a ca n mod necesar m in sunt multipli de 3; m = 3m1 , n = 3n1 , m1 , n1 s (1) devine 3 m2 + n2 = 667p2 i cum (3, 667) = 1, urmeaza ca p = s 1 1 Dup nlocuire, m2 + n2 = 2001p2 (2). a 1 1 1 Dac presupunem c ecuaia (1) admite soluii, fie o asemenea solui a a t t t Din (2) se obine nsa o noua soluie cu p1 < p, contradicie! Urmeaza t t t soluii n N , de unde concluzia. t Not. Metoda folosit se numete metoda coborrii i a fost utiliza a a s s strarea Marii Teoreme a lui Fermat n cazurile n = 3 i n = 4. s G15. Sa se arate ca E(x, y, z) 3, daca E(x, y, z) = x2 2x sin z p + y 2 2y sin z 6 cos z + 10, x, y, z R. Cristiana Artenie Soluie. Se observa ca t q q E (x, y, z) = (x sin z)2 + (2 cos z)2 + (y sin z)2 + (3 cos z)2

Numerele n = 0, 1, 2, 3 nu verific (2), iar pentru n 4 avem c a a n 9 = (1, 8)n (1, 8)4 = (3, 24)2 > (3, 2)2 = 10, 24 > 10 5

unde M (cos z, sin z), P (2, x), Q (3, y), x, y, z R. Punctul M parcur tate C, iar punctele P i Q parcurg dreptele verticale d1 : x = 2, respect s Minimul lui E (x, y, z) se atinge pentru {M } = C[Ox, {P } = d1 Ox, { n acest caz E (x, y, z) = 3, de unde concluzia. G16. Fie M un punct n interiorul triunghiului echilateral ABC \ M A2 = M B 2+M C 2 2M BM C ; calculai masura unghiului BM C. t Corneliu Brdea a at Soluie. n general, vom arata ca daca M A2 = t 2 \ = M B + M C 2 2M B M C cos , atunci m(BM C) = t a = + 60 . n situaia problemei date, va rezulta c \ m(BM C) = 105 . Fie D n semiplanul determinat de BC opus lui A ast\ fel nct 4M BD este echilateral. Atunci m(ABM ) = \ = 60 m(M BC), de unde 4ABM \ = m(CBD) 4CBD (LUL), deci AM = DC. Cum M D = M B, relaia de mai sus se scrie CD2 = M D2 +M C 2 2M D t \ \ M C cos , ceea ce arat c m(DM C) = , adic m(BM C) = 60 + a a a 69

G17. Fie ABCD un patrulater convex ce nu are diagonalele perpend D1 proieciile punctelor B, respectiv D pe AC, iar A1 si C1 proieciile t t 2 BD SBB1 DD1 respectiv C pe BD. Sa se arate ca si SABCD cos2 = 2 SCC1 AA1 AC = SBB1 DD1 SCC1 AA1 . Claudiu- tefan S Soluie. Avem c SABCD = SACD + SACB = t a AC BB1 BB1 + DD1 AC DD1 + = AC. Pe = 2 2 2 de alt parte, BB1 DD1 este trapez sau paraleloa gram (BB1 , DD1 B1 D1 ) cu nalimea [B1 D1 ], t BB1 + DD1 deci SBB1 DD1 = Atunci B1 D1 . 2 B1 D1 SBB1 DD1 = . SABCD AC B1 O BO B Observm c 4DOD1 4BOB1 i de aici a a s = , adic a D1 O DO D SBB1 D D1 O \ a a deci B1 D1 = BD = BD cos(AC, BD). Rezult c SABCD DO \ Analog se obine ca SCC1 AA1 = AC \ t cos(AC, BD). cos(AC, BD) SABCD BD apoi nmulind membru cu membru ultimele dou egaliti, gsim rela t a at a cluzie.

b G18. Fie ABC un triunghi cu m(A) 90 . Pe latura (BC) punctele M si N astfel nct AM si AN sa fie simetrice faa de bisectoa t A. Cercul circumscris triunghiului AM N intersecteaza laturile AB

respectiv F . Daca {I} = BF CE si {P } = AI BC, demonstrai ca t Florin Nico \ Soluie. Din ipotez, EAM N AF , deci n t a \

cercul C avem c EM F N , de unde EF kM N . a Fie {D} = AP EF ; atunci 4AED 4ABP i s ED AD 4AF D 4ACP i va rezulta c s a = = BP AP DF ED BP = , i.e. = (1). Din asemnrile a a PC FD CP 4EID 4CIP i 4DIF 4P IB obinem, ca s t CP ED = (2). Din (1) i (2) urmeaza s mai sus, FD BP ca (BP ) (CP ). BC Presupunem prin reducere la absurd ca AP < , adica AP < BP 2 b s b \ [ [ Atunci m(BAP ) > m(B) i m(P AC) > m(C), deci m(\) + m(P A BAP b b b b +m(C), de unde m(A) > 180 m(A), i.e. m(A) > 90 , ceea ce contr problema este astfel rezolvat. a 70

G19. Fie A1 A2 A3 un triunghi echilateral nscris n cercul C(O, R

= 6R2 6Rr = constant, unde (1) se justific prin aplicarea teoremei lui Pitagora a t n triunghiurile dreptunghice P Pi Oi , iar (2) prin relaia lui Leibniz. G20. Sa se arate ca pentru orice alegere a 12 numere naturale cons pot numerota muchiile unui cub astfel ca suma numerelor aflate pe tre au un vrf comun sa fie aceeai pentru toate vrfurile cubului (nu se s doua muchii cu acelai numar). Sa se arate ca este posibila numerot s daca se aleg convenabil 12 numere dintre oricare 13 numere naturale c Constantin Soluie. Fie n + 1, n + 2, . . . , n + 12, n N i sa presupunem pr t s suma numerelor de pe oricare trei muchii adunate adiacente este s. Ob t = 2[(n + 1) + (n + 2) + + (n + 12)], de unde, dup calcule, gsim 2s = a a Am ajuns evident la o contradicie, deoarece n stnga avem un num t dreapta unul impar. Pentru partea a doua, fara a restrnge generalitatea, putem considera numerele 1, 2, . . . , 13; cazul general se reduce imediat la acesta. Fie c numrul pe care l vom elimia na. Cu raionamentul de mai sus, obinem 4s = 91 c t t 91 c i cum s = s N, n mod necesar c {3, 7, 11}, 4 deci s {22, 21, 20}. Pentru c = 7, vom da o aezare a s numerelor 1, 2, 3, 4, 5, 6, 8, 9, 10, 11, 12, 13 care s respecte a cerinele problemei: fiecare pereche de numere simetrice t faa de 7 de forma (p, 14 p) se scriu pe muchii simetrice faa de cen t t astfel nct suma ntr-un vrf s fie 21. a

s Ci (i = 1, 2, 3) de aceeai raza r, tangente interior cercului C n vrfu punzatoare. Sa se arate ca pentru orice P C(O, R) are loc relaia t t constant, unde ti (i = 1, 2, 3) este lungimea tangentei dusa din P la ce Temistocle Soluie. Fie Oi centrul cercului Ci , i = 1, 3. Evit dent c 4O1 O2 O3 este echilateral, iar centrul su este a a punctul O. Avem: X X X (1) X 2 2 (2) P Oi r2 = 3r2 + t2 = P Pi2 = P Oi = i X 2 2 = 3r2 + 3P O2 + OOi = 3r2 +3R2 +3 (R r) =

B. Nivel liceal

L6. Fie x1 , x2 , . . . , xn , n N\{0, 1}, numere reale cu proprietatea x1 x2 xn + + + = 1, S x1 S x2 S xn Pn unde S = i=1 xi . Aratai ca t x3 x3 x3 S2 1 2 n + + + . S x1 S x2 S xn n Rzvan Brbulescu, el a a 71

Soluie. Din relaia dat n ipotez, deducem succesiv: t t a a

b L7. n triunghiul ABC, m(A) > 60 , consideram medianele CN , 0 toarele BE, CE . Notam {P } = CN BE, {Q} = CE 0 BN 0 . Arata P si Q nu pot fi ambele pe nalimea din A. t Ioan Scle a a Soluie. Sa presupunem prin absurd ca P i Q aparin nalimii (A t t s t teorema lui Menelaus n 4ABD cu transversala N P C, apoi teorem n acelai triunghi, obinem s t

x2 xn Sx1 Sx2 S x1 + + + =1 + + + S x1 S x2 S xn S x1 S x2 S xn (S xn ) + x2 x1 (S x1 ) + x2 x2 (S x2 ) + x2 1 2 n + + + S x1 S x2 S x2 n x2 x2 x2 1 2 n + x2 + + + xn + =S x1 + S x1 S x2 S xn x2 x2 x2 n 1 2 + + + =0 S x1 S x2 S xn x3 x3 x3 1 2 n + x2 + + + x2 + =0 x2 + 1 2 n S x1 S x2 S xn x3 x3 x3 1 2 n + + + = x2 + x2 + + x2 1 2 n S x1 S x2 S xn r x1 + x2 + + xn x2 + x2 + + x2 1 2 n , de unde rezulta imed nsa n n

AN BC DP BC BD a c cos B a =1 =1 =1 N B CD P A CD BA b cos C c b a cos C a Repetnd raionamentul n 4ACD, obinem c t t a = , deci c cos B c a2 = bc. nsa a2 = b2 + c2 2bc cos A, de unde b2 + c2 bc (1 + 2 cos 2 b b urmare s (1 + 2 cos A) + 1 = 0 i cum = 4 cos2 A + 4 cos A c c b 1 , deducem ca R, absurd. cos A 1, / 2 c Not. Soluie corect s-a primit de la Marius Pachiariu, elev, Iai a t a t s L8. Fie triunghiul ABC si M Int ABC, M A C (M BC) M B C (M CA) = {M, B1 }, M C C (M AB) = {M, C1 }. Sa se arate

M A1 M B1 M C1 + + 6. MA MB MC Neculai Roman, M \ \ \ Soluie. Fie x = m(BM A1 ), y = m(CM B1 ), z = m(AM C1 ); e t +y + z = 180 . Dac R1 este raza cercului prin M, B, C, avem: A1 B a A1 C = 2R1 sin z, BC = 2R1 sin (x + z) = 2R1 sin y. Aplicnd teorema 72

n patrulaterul inscriptibil M BA1 C, obinem succesiv: t

M A1 BC = M B A1 C + M C A1 B M A1 sin y = M B sin z + M M A1 M B sin z M C sin x = + . MA M A sin y M A sin y Scriind relaiile analoage i adunndu-le, concluzia urmeaza imediat t s 1 a + 2, (0, ). a n cazul particular M = O, obinem inegalitatea remarcabil t a OA1 + OB1 + OC1 6R,

unde R este raza cercului circumscris 4ABC. S mai observm c e a a a atins n triunghiul echilateral. a L9. Fie ABC un triunghi ascuitunghic cu a b c si u, v, t u v w. Daca uGA + vGB + wGC = (u + v + w) R, unde G es greutate al triunghiului, iar R este raza cercului circumscris, atunci tri este echilateral. Paul Georgescu i Gabrie s Soluie. Fie f : P R, f (M ) = u |zM zA | + v |zM zB | + t 2z0 + zH , din inegalitatea modulului obinem c f (G t a Deoarece zC = 3 1 + f (H). Din ipotez, f (G) = f (O), deci f (H) f (O). Pe de alt p a a 3 h i [0, 1], gasim ca inegalitatea lui Jensen funciei concave cos : 0, t 2 uA + f (H) = 2R (u cos A + v cos B + w cos C) 2R (u + v + w) cos u+ 1 Din inegalitatea lui Cebev, uA + vB + wC (u + v + w) (A + s 3 f (H) 2R (u + v + w) cos = f (O). Am obinut c f (H) = f (O) t a 3 atins egalitatea n inegalitile Jensen i Cebev, adic 4ABC este a at s s a L10. a) Fie n N , n 2. Sa se arate ca exista o progresie numere naturale care nu are nici un termen de forma xn , x N. b) Daca o progresie aritmetica de numere naturale conine un term t xn , x N, atunci sa se arate ca progresia conine o infinitate de terme t forma. Adrian Za Soluie. a) S demonstrm c progresia aritmetic ak = 4k+2, k t a a a a nici un termen de forma xn , x N. ntr-adevr, acest fapt rezult din a a (4m)n = M4 , (4m + 1)n = M4 + 1, (4m + 2)n = M4 , (4m + 3)n = M4 b) Fie o progresie de numere naturale cu raia r care conine un term t t n Numrul natural (x + r) este termen al progresiei, deoarece a (x + r)n = xn + nxn1 r + + rn = xn + nxn1 + + rn
r

Analog se demonstreaz c orice numr de forma (x + kr) , k N, e a a a progresiei. 73

t L11. Sa se rezolve n N ecuaia 2 3x = 3 2y + 174. Daniela Iosub Soluie. Cum x, y N , atunci a = x 1, b = y 1 sunt num t mparind ecuaia prin 6, obinem ecuaia echivalenta 3a = 2b + 29, a, b t t t t b b 3a = (3 1) + 29, deci 3a = M3 + (1) + 29, prin urmare b trebu b = 2k, k N (deoarece este evident c b = 0 nu convine). Obinem a t i.e. (4 1)a = 4k +29, de unde M4 +(1)a = 4k +29, adic a trebuie s a af l N (a = 0 nu convine). n aceste condiii, ecuaia devine 3l 2k t t i cum 29 este prim, iar 3l 2k < 3l + 2k , gsim c 3l 2k = 1, 3l + 2k = a a s astfel format nu are soluii n N i atunci ecuaia iniial nu are soluii t s t t a t

L12. Fie ABCD un patrulater convex; notam {O} = AC BD lui (AB), N mijlocul lui (CD). Pentru propoziiile P1 : ABCD insc t OM CD; P3 : ON AB, sa se arate ca: a) P1 P2 P3 ; b) P c) P3 P1 P2 (n legatura cu problema C:2265 din G. M. 3/2000). Viorel Cornea i Dan Stefan Marinescu, s Soluie. a) Daca ABCD inscriptibil, din puterea punctului O f t circumscris obinem c OA OC = OB OD. Avem succesiv: t a 1 OA + OB OD OC = 0 P2 OM CD = 0 2 OA OD OA OC + OB OD OB OC = 0 OA OD OB OC + (OA OC OB OD) = 0 OA OD OB OC (OA OC OB OD) = 0 OA OB OD + OC = 0 2 AB ON = 0 AB b) Se procedeaza analog. c) Dac OM CD, se obine relaia (1). Din ON AB deducem a t t OD OB + OC OB OD OA + OA OC = 0

Adunnd (1) i (2), gsim c OA OC = OB OD, adic ABCD este s a a a

L13. Fie P R [X], P (X) = a0 X n + a1 X n1 + + an1 X + a a0 > 0 si cu toate radacinile pozitive si subunitare. Sa se arate ca (n n + (1) an > 0. Gheorghe Molea, Curt a1 n n a Soluie. Avem: (n 1) a0 + a1 + (1) an > 0 (1) t a0 a P Q r n xi n xi < n 1, unde xi (0, 1), i = 1, n, suntQadacinile i=1 i=1 Prin inducie completa, se dovedete uor inegalitatea: 1 n (1 t s s i=1 bi (0, 1), i = 1, n, n 2. Lum n aceasta bi = 1 xi , i = 1, n, i obinem: a s t 1 q.e.d.
i=1 n Y

xi <

n X i=1

(1 xi ) 1

i=1

n Y

xi < n

n X i=1

xi

n X i=1

xi

i=1

n Y

74

X+1 1 1 2 X 2 +2 2 L14. Pentru n N consideram polinomul Pn (X)= ... ... ... n n n n (n + 1) a) Aratai ca zero este radacina multipla de ordin a aces t 2 b) Daca n este par, Pn nu are radacini reale nenule, iar daca n este o singura radacina reala nenula, care este simpla si situata n interval Temistocle Soluie. Considernd ultima linie ca o sum de dou linii, avem: t a a X + 1 1 ... 1 2 X2 + 2 . . . 2 ... ... ... Pn (X) = X n Pn1 (X) + n . . . n 1 n 1 . . . X n1 + (n 1) 1 1 ... 1 Scznd ultima linie nmulit respectiv cu 1, 2, . . . , n1 din celelalte, ob a t a de recurena t Pn (X) = X n Pn1 (X) + nX n(n1)/2 , din care se deduce, prin calcule de rutin, c a a Pn (X) = X n(n1)/2 Qn (X)

cu Qn (X) = X n + X n1 + 2X n2 + + (n 1) X + n. Afirmaia a) rezulta direct din (2) i (3). Afirmaia b) n cazul t s t scriind polinomul Qn sub forma 1 n2 3 3 n n1 + X n2 + X n3 + X n + X Qn (X) = X + X 2 2 2 5 n4 5 n6 n n3 4 + X + 5X n5 + X X + (n 3) X 3 + + + 2 2 2 n1 2 + X + (n 1) X + n 2 i observnd ca parantezele patrate au discriminantul nul, iar cele r s negativ. Dac n este impar, verificm mai nti c Q0 are valori pozitive p a a a m ntr-adevr, procedm ca mai sus, observnd c a a a Q0 (X) = nX n1 + (n 1) X n2 + 2 (n 2) X n3 + + (n 2) 2X n 1 (n 1) n3 n1 n2 + X n3 + = nX + 1 (n 1) X + X 2 3 (n 3) n5 3 (n 3) n3 n4 + X n + + 3 (n 3) X + X X 2 2 (n 4) 4 2 (n 4) 4 4 3 + X + (n 4) 4X + X + X 2+ 2 2 (n 2) 2 2 + X + (n 2) 2X + (n 1) . 2 75

Partea a doua a armaiei b) rezult din faptul c Qn (x) > 0 pe t a a Qn (2) Qn (1) < 0 i funcia x Qn (x) este strict cresctoare pent s t a L15. Fie R\Z si (cn )n1 un sir convergent de numere reale. S sirul (xn )n1 definit prin xn = {n + cn } nu este monoton. Iuliana Georgescu i Paul Geo s Soluie. S observm c {x} {y} {x y} = {x} {y}. Pr t a a a s (xn )n1 este crescator. Atunci, deoarece acest ir este marginit, el va i xn+1 xn 0. Pe de alta parte, xn+1 xn = {(n + 1) + cn+1 } s = { + cn+1 cn } .Dar + cn+1 cn i R\Z, deci { + c s {} > 0. Prin urmare, xn+1 xn {} > 0. Absurd. L16. a) Fie a < b si M = {f : [a, b] [a, b] ; f monotona}. Ara f M cu f (x) 6= x, x [a, b] si ca orice asemenea funcie nu are pr t Darboux. b) Demonstrai ca f M , c [a, b] astfel nct f (c) [a + b f (c)] = t Stefan Al Soluie. a) Funcia f : [a, b] [a, b] definit prin f (x) = a + t t a x [a, (a + b) /2) i f (x) = a, dac x [(a + b) /2, b], satisface condii s a t Presupunem ca exista o funcie f M fara puncte fixe i cu proprie t s boux pe [a, b]. Fiind monotona, f poate avea discontinuitai doar de t avnd proprietatea lui Darboux, f nu are nici discontinuiti de acest at c f este continu pe [a, b] i tot aa este i g : [a, b] R, g (x) = f ( a a s s s g (a) g (b) = [f (a) a] [f (b) b] 0, deducem ca c [a, b] astfel n adica f (c) = c. Atunci c este un punct fix al funciei f , ceea ce co t supunerea fcut. a a b) Fie f M . Presupunem c f este cresctoare i notm E = {x [a a a s a Observam ca E este nevida i marginita (a f (a) i E [a, b]). s s c = sup E i, evident, c [a, b]. Din x c, x E, deducem ca x s x E. Deci f (c) este un majorant al mulimii E i avem c f t s este cresctoare, rezult c f (c) f (f (c)), de unde deducem c f (c) a a a a f (c) c. Aadar, f (c) = c (1). s Daca f este descrescatoare, atunci h : [a, b] [a, b], h (x) = a + cresctoare i, procednd c mai sus, d [a, b] astfel nct h (d) = d, a s a = a + b d (2). Din (1) i (2) rezulta ca f M ecuaia [f (x) x] [a + b x f s t f (x) [a + b f (x)] = x (a + b x) are soluii n [a, b], deci c [a, b t f (c) [a + b f (c)] = c (a + b c). L17. Fie A un numar real pozitiv si f : [0, ) [0, ) o func pentru care f (0) = 0 si |f 0 (x)| Af n (x), x [0, ), unde n es natural dat, n 1, iar f n = f f f . Atunci f este identic nula Sorin Pupa s Soluie. Este suficient s artm c f este identic nul pe oric t a aa a a forma [0, ], > 0. Presupunem ca > 0 astfel nct f nu-i identic n adic avem M > 0, unde M = sup f (x). Cu teorema creterilor fi a s
x[0,]

x [0, ] are loc relaia f (x) = xf 0 (c), unde c (0, x); deci f (x) t 76

xA |f n (c)| AM , x [0, ]. Ca urmare, f (x) AM , x [0 M AM sau A 1. Fie = (0, 1 , 2 , . . . , m1 , ) o diviziune a intervalului [0, ] c (). Daca f nu-i identic nula pe [0, 1 ], atunci M1 = sup f (x) > 0
x[0,1 ]

L18. Fie A Mn (Z), n N, n 2 astfel nct In + sA este (In + sA)1 Mn (Z) pentru orice s {1, 2, . . . , n} . a) Sa se arate ca In + kA este inversabila pentru orice k Z si (I Mn (Z); b) Daca A2 = On , sa se arate ca G = {In + kA; k Z} este grup nmulirea matricelor si sa se determine toate subgrupurile lui G. t Marian Ione Soluie. a) Se arata uor afirmaia: C Mn (Z) este inversabila t t s det C = 1. Fie P (x) = det (In + xA), grad P n, P Z (X). s {0, 1, 2, . . . , 2n}, avem: Cs = In + sA este inversabil n Mn (Z) a = 1 P (s) = 1. Prin urmare, exist cel puin n + 1 numere s a t valoare 1 sau cel puin n + 1 numere s n care P este 1. t Consideram ca exista u1 , u2 , . . . , un+1 {0, 1, 2, . . . , 2n} astfel n = P (u2 ) = = P (un+1 ) = 1; analog se procedeaz n cellalt ca a a Q (X) = P (X) 1, de grad cel mult n, se anuleaz pentru n + 1 valori d a Q = 0 i P = 1. Rezulta ca det (In + kA) = 1, k Z i, n consecin s s dorita. b) Dac Cs = In + sA, Ct = In + tA, s, t Z, atunci, n ipoteze a Cs Ct = Cs+t . Se verific uor c G este grup n raport cu nmulir a s a t i ca (G, ) ' (Z, +) prin f : Z G, f (k) = In + kA. Deoarce sub s (Z, +) sunt de forma H = mZ, m 0, rezulta ca subgrupurile lui (G {In + mkA; k Z} cu m N. L19. Fie H un subgrup al grupului altern (A2002 , ). Daca 1 2 . . . 1999 2000 2001 2002 = H 1 2 . . . 1999 2001 2002 2000

obinem f (x) 1 AM1 , x [0, 1 ]; deducem ca M1 1 AM1 , ad t ceea ce contrazice (). n concluzie f este identic nula pe [0, 1 ]. n par f (1 ) = 0 i cu intervalul [1 , 2 ] procedm la fel ca i cu [0, 1 ] etc. s a s Dup un numr finit de pai, deducem c f este identic nul pe nt a a s a a [0, ]. Presupunerea iniiala facuta este falsa. n concluzie, f este id t [0, ).

si 1 H, A2002 , sa se arate ca H = A2002 . Lucian-Georges L a Soluie. Se tie c grupul altern (An , ), n 3, este generat de ciclu t s a 3. Pentru a demonstra ca H = A2002 este sucient sa aratam ca orice ast afla n H. Pentru aceasta, fie (, , ) un 3-ciclu oarecare (, , {1, . . . a . . . b . . . 2000 2001 2002 , unde a, b, a0 , b0 s Fie = . . . a0 . . . b0 . . . a a fel nct A2002 . Se constat c (, , ) = 1 H. 77

L20. Fie a R, a > 1. Se considera funcia f : [1, a] R de doua o t Sa se arate ca daca funcia g : [1, a] R, g (x) = xf 0 (x) este monoton t atunci Z a f (t) a ln a f dt. t 1 Marcel Chiria t Soluie. Cum g este funcie crescatoare, rezulta ca g 0 0, adica f 0 (x t t 0, x [1, a]. Fie funcia h : [0, 1] R, h (x) = f (ax ). Avem: h0 (x) = ax t 00 h (x) = ax ln2 a [f 0 (ax ) + ax f 00 (ax )] 0, x [1, a], de unde rezu convexa. Conform inegalitaii lui Jensen, avem: t x1 + x2 + + xn h (x1 ) + h (x2 ) + + h (xn ) h , x1 , x2 , . . . n n k s t Pentru xk = , k = 1, n i trecnd la limita pentru n obinem n Z 1 Z 1 1 h h (t) dt, adic f a a f (ax ) dx. 2 0 0 n ultima relaie efectuam schimbarea ax = t i obinem inegalitatea ce t s t

LISTA MEMBRILOR FILIALEI IA I a S. S


- continuare din nr. 1/2000, 1/2001 i 1/2002 s 90. GALL Eduard 91. URSACHE Felicia-Camelia 92. LAMATIC Lidia-Carmen 93. MACSIMIUC Delia 94. FARCA ANU Ana-Corina S 95. BAICAN Tatiana 96. BUCATARU Mihaela 97. BUCATARU Ion 98. CRETU Ines 99. ASIMINOAIEI Ana 100. NAZARIE Elena 101. PSLARU Margareta Adriana 102. BOTRCA Mihaela Lucian-Georges 103. LADUNCA 104. GO MAN Neculai S 105. ONICIUC Carmen-Elena 106. LUPULEASA Iuliana 107. STIURCA Ecaterina 108. ANITA Alice 109. PREDA Anioara s
1

Inginer, S.C. Easten, Iai s Scoala gen. nr.36, Iai s Grupul Scolar Agricol Holboca Scoala "Otilia Cazimir", Iai s Scoala gen. nr.36, Iai s Colegiul "C.Negruzzi", Iai s Colegiul "E.Racovia", Iai t s Fac. de matematic, Univ. "Al. a Scoala gen. nr.42, Iai s Liceul de chimie, Iai s Liceul de chimie, Iai s Scoala prof. speciala, Tg. Frum Scoala gen. nr.10, Iai s Liceul de informatic "Gr.Moi a Scoala "G.Ibrileanu", Tg.Fru a Scoala nr.6 "M.Busuioc", Pac s

Grupul Scolar "M.Sturza", Ia s Colegiul Naional, Iai t s Scoala "D.D.Ptrcanu", Tom a as

Lista va continuat n numerele urmtoare. a a

78

Probleme propuse
Clasele primare

P.44. Un vecinul al unui vecin al numarului 81 este egal cu un vecin al numrului 77. Despre ce numr este vorba? a a ( Clasa I ) Mihaela Rusu

P.45. Adunnd trei numere naturale a, b, c obinem suma 62. Prim t mai mare dect al treilea i mpreun au suma 12. Care sunt cele trei s a ( Clasa a II-a) nv. Maria

P.46. Mihai, Dan i Petru practica fiecare un alt fel de sport i a s s fotbal sau volei. Mihai i voleibalistul locuiesc n acelai bloc. Cel care s s cel care joac fotbal l-au urmrit pe Petru la un meci. Ce sport practi a a ( Clasa a II-a) Adina Dohotaru

P.47. Diferena a doua numere este 48. Aceasta diferena este cu t t dect jumtatea unuia dintre ele. Determinai numerele. a t ( Clasa a III-a) nv. Rodica Rot

P.48. Un agricultor mparte un teren n trei parcele. n fiecare an, f este cultivat numai cu una din culturile: gru, porumb sau legume. a anul 2003, agricultorul se hotrte ca pe fiecare parcel s fie alt cultu a as a a a consecutivi. a) Care este primul an dupa 2003 n care se repeta culturile pe cele b) Se poate preciza care este ordinea culturilor pe cele trei parcele ( Clasa a III-a) Andreea Surugiu

P.49. La un moment dat, cernd unei persoane anul naterii, acea s "anul acesta mplinesc 25 ani, iar dac a scrie toate numerele nce a s terminnd cu anul naterii i apoi toate numerele ncepnd cu 1 i term s s s n care ne am mi-ar trebui 13710 cifre. n ce an ne aam cnd am pu a ( Clasa a III-a) Prof. Ctlin - Cristian Bu a a

P.50. a) Cte numere trebuie adugate irului 1, 2, 4, 5, 7, 8, . . . , 97 a s obine toate numerele de la 1 la 98? t b) Efectuai 1 + 2 + 4 + 5 + 7 + 8 + + 97 + 98 2 (3 + 4 + 5 + t ( Clasa a IV-a) Georgiana Ciobanu

P.51. Produsul a dou numere naturale este 913 368. Unul din num a unitailor i cifra zecilor mai mare ca 2 i mai mica dect 8. Daca la s s t marim cifra zecilor cu 2 i micoram cifra unitailor cu 1, obinem un p s s t t 951 425. Aai cele dou numere. t a ( Clasa a IV-a) nv. Elena Z a

P.52. n trei cutii sunt 212 bile. Din prima cutie se scoate un numar doua de 2 ori mai mult i nc dou bile, din a treia se scoate ct triplul s a a bile scos din a doua cutie. n fiecare cutie rmne un numr de bile ega a a total al bilelor scos din cele trei cutii la un loc. Cte bile au fost n fiec ( Clasa a IV-a) nv. Maria

P.53. Efectund o singur cntrire, s se ia 475g dintr-un kilog a a a 79

utiliznd dou greuti, una de 200g i cealalt de 150g. a at s a ( Clasa a IV-a) Prof. Petru A

Clasa a V-a

V.36. Fie n un numr impar, iar a1 , a2 , . . . , an , n N numere c a la n dau cturi distincte i resturi distincte. Artai c valoarea min s a t a S = a1 + a2 + + an este multiplu de 12. Drago Ungurean s 333331 222221 i b = V.37. Comparai fraciile a = t t s . 333334 222223 Maria Co

V.38. S se arate c 2a + 2b + 2c + 2d + 2e 6= 2003, a, b, c, d, e N a a Irina Ispas, st

V.39. Sa se determine numerele prime p1 < p2 < p3 < p4 astfel n p1 + p2 + p3 + p4 , p3 p2 , p4 p3 sa e, de asemenea, prime. Petru

V.40. Este posibila o partiionare a mulimii {1, 2, . . . , 12n + 9} n t t mulimi disjuncte, fiecare cu cte trei elemente, astfel nct n fiecare su t element s fie suma celorlaltor dou? a a Titu Zvonaru

Clasa a VI-a

VI.36. Fie k N, k 3. Aratai ca printre valorile naturale ale t . . n + k, exist cel puin trei ptrate perfec 2 a t a adevrat propoziia n + k . a a t Claudiu Stefan

VI.37. Numerele 1160, 1604 i 2270 dau acelai rest la mprirea s s at mparitorul n. t Cristian

VI.38. Demonstrai c nu exist numere naturale x, y, z direct pro t a a trei numere naturale consecutive, astfel nct x + y + z sa fie numar pr Alexandru Negrescu, ele

VI.39. Radu i Mihai joaca de mai multe ori un joc n urma carui s primete a puncte, iar cel care pierde primete b puncte (a, b N , a s s scorul final este 6149 n favoarea lui Radu, iar Mihai a ctigat 4 partid s Adrian Za b VI.40. Fie 4ABC cu m(A) = 120 . Perpendiculara n C pe AC mediatoarea lui [AB] n D; notam {E} = CD AB. Sa se arate ca AB \ = 90 i BE = 2AB. i numai dac m(BDE) s a s Ioan Scle a a

Clasa a VII-a
VII.36. Sa se arate ca

1 + n

2 + + n

2n 1 < 2n 1, n n Ctlin C a a

VII.37. Artai c n baza de numeraie 7 printre numerele ce se s a t a t 0, 1, 2 exist o infinitate care sunt ptrate perfecte i o infinitate ce nu a a s 80

perfecte. Aceste afirmaii rmn valabile dac se folosesc cifrele 3, 5, 6? t a a Ruxandra Ioana Vlcu VII.38. Fie a, b, c cifre nenule, a 6= c. Sa se arate ca daca

ab cbb (termenii primei fracii coninnd cte 2003 cifre b), atunci b = a + c. t t Mihaela Bu

VII.39. Dac x < y < z sunt lungimile laturilor unui triunghi a atunci xn + y n 6= z n , n N, n 3. Dumitru b = 60 , iar M VII.40. Fie ABC un triunghi ascuitunghic cu m(A) t \ = 150 . Notm cu P, Q, R proieciile lui M p astfel nct m(BM C) a t respectiv AB. Sa se arate ca 4P QR este dreptunghic. Constantin

Clasa a VIII-a

VIII.36. Determinai cardinalul minim al unei mulimi B pentru t t defini funcii f : R B astfel nct f (1) < 0 i f (xy) = f (x) f (y) t s Iulia Zanoschi

VIII.37. If a, b, c (0, ) prove the following inequalities: a) (a + b + c)3 a3 + b3 + c3 24 where abc = 1; 3 8 3 3 3 3 where ab + bc + ac = 1. b) (a + b + c) a + b + c 3 Zdravko Starc, Vrac s

VIII.38. Fie n N fixat. Artai c exist o infinitate de nume a t a a astfel nct x2n + y 2n + z 2n = x2n+1 + y 2n+1 + z 2n+1 . Lucian Tues t

VIII.39. Fie ABCD un patrulater strmb cu [AD] [BC]. Sa se dreptele paralele d1 , d2 , d3 , d4 astfel nct A d1 , B d2 , C d3 dist (d1 , d4 ) = dist (d2 , d3 ). Horia Mihail Teodoresc

VIII.40. Fie ABCDA0 B 0 C 0 D0 un cub, iar O (BB 0 ). Dreptele intersecteaz (ABC) n E, respectiv F , iar AO i CO intersecteaz (A a s a respectiv F 0 . a) Aratai ca EF E 0 F 0 nu depinde de poziia lui O; t t b) Artai c SBB 0 E0 E SABCD i determinai O pentru care se atin a t a s t Monica N

Clasa a IX-a

y IX.36. Determinai x < 0 < y astfel nct xy + = y 3 5y + 2. t x Cezar Lupu, elev

t IX.37. Pentru x [1, ), n N , demonstrai inegalitatea n n+1 + 1 (x 1) 2nxn (x 1) . x 81

Marius Pachiari t

xn+1 y n+1 z n+1 + n + n x + y + z, x, y, z > yn z x Gigel Buth, 1 2 1 + q = IX.39. S se rezolve ecuaia q a t [x] [x + 2 [x]3 3 3 [x] [x + 1]3 Daniel Ji IX.40. Fie M 6= G n planul 4ABC i D, E, F mijloacele la s [CA] i respectiv [AB]. Considerm punctele X, Y, Z astfel nct X s a Y E = mY M , ZF = mZM , m 6= 1. 3 a) Dac m 6= , atunci AX, BY, CZ sunt concurente n S, cu SG = a 2 3 b) Dac m = , atunci AX, BY, CZ sunt paralele cu GM . a 2 Virgil Nicula IX.38. S se arate c a a

Clasa a X-a

X.36. S se rezolve inecuaia alogb x + xlogb x a + b, unde a, b (1 a t Daniela Dodan X.37. Fie a, b (0, 1) (1, ) i funcia injectiva f : (0, ) R s t a a funcia g : R R, g (x) = f (ax ) + f (bx ) este constant. S se arate c t exist funcii f care satisfac ipotezele problemei. a t Dan Popes X.38. Fie a, b, c, d R cu a > b > c > d. S se arate c a, b, c, d sun a a 3 ad aritmetica daca i numai daca (a b) (b c) (c d) = . s 3 A. V. Miha 0 0 0 0 X.39. Fie ABCDA B C D un paralelipiped dreptunghic cu dimens AD = b, AA0 = c. Dac M Int A0 B 0 C 0 D0 , notm cu , , msuri a a a pe care AM le face cu AB, AD i respectiv AA0 . S se arate c s a a AM < a cos + b cos + c cos < AC 0 . Ctlin C a a X.40. a) Pentru x, y, z 0, demonstrai inegalitatea t p x + y + x + z + y + z xy + xz + yz 3 6xyz. b) Cu notaiile uzuale, n orice triunghi are loc inegalitatea t 2 2 2 a b + ( a c) + b c 9 R . 2 2 r 4 a+ b+ c

Marian Te

Clasa a XI-a

XI.36. Fie D, M doua matrice nesingulare de ordin n, D diag a a triunghiular. Dac D = t M DM , s se arate c M este tot o matri a a avnd 1 pe diagonala principal. a Adrian Cordu 82

(xn )n1 R are proprietatea c xn xn+1 (1 + xn yn+1 ), n 1, ar a + 1 este convergent. xn n1 Gheorghe Molea, Curt XI.40. Fie x0 [1, 1]; ataic pentru orice n N, ecuaia 3x ar t a t 1 1 t as are o singur soluie xn+1 , . Demonstrai c irurile (xn )n0 a t 2 2 sunt convergente i calculai limitele lor. s t Marian Te

XI.37. Fie A M3 (C) astfel nct det (A + tA) = 0, unde 2 ( 1)2 det A. Sa se arate ca det (A + tA) = Marian Ionescu, Piteti i Lucian Tues s s t XI.38. S se determine funciile continue f : [0, ) [0, ) a t f (f (x)) + 2f (x) = 3x, x 0. Mihail Ben n P XI.39. Fie irul (yn )n1 astfel nct irul s s yi este conv
i=1 n1

Clasa a XII-a

XII.36. Sa se determine n N, n 2 pentru care ecuaia x2 = x + t unic n Zn ; rezolvai ecuaia n acest caz. a t t Andrei N XII.37. Fie (G, +) un subgrup al grupului (R, +). S se determin a cresctoare de la (G, +) la (R, +). a Dan Stefan Marinescu i Viorel Cornea, s XII.38. Determinai funciile derivabile f, g : R R astfel nct f 0 ( t t i g 0 (x) = f (x) x, x R. s Gheorghe XII.39. Fie f, g : (0, ) R astfel nct lim f (x) = lim g ( x x Z 1 g(n) f (x) x lim = R. S se calculeze lim f (n) a dx, unde x g (x) n 0 x+ Adrian Sandovici, Pi XII.40. Fie f : [0, 1] R o funcie derivabil cu derivata continu t a f (x) 0 exist i este finit. S se ara as a a xf (x) f (x), x [0, 1], iar lim x0 x x>0 Z 1 Z 1 f (x) f (x) dx, f (1) min 2 dx . x 0 0 Marcel Chiria t

83

Probleme pentru pregtirea concursuri a


A. Nivel gimnazial

G36. Fie x, n N astfel nct x divide 10n 1, ns x nu divide 1 a . m k < n. Sa se arate ca x divide 10 1 daca i numai daca m . n. s . N. N. H

G37. 2n muzicieni (n > 2) particip la un festival. La fiecare con a dintre ei cnta iar ceilali asculta. Sa se determine numarul minim de c t nct fiecare muzician sa-i asculte pe toi ceilali. t t Titu Zvonaru

G38. Mulimea A Z are cinci elemente. Adunnd n toate mod t cte trei elemente din mulime, obinem urmtoarele 10 sume: 3, 6, t t a 15, 16, 18, 20. Determinai mulimea A. (n legtur cu o problem d t t a a a Iugoslavia.) Gabrie

G39. Fie xi R, i = 1, n, unde n 2003, astfel nct x1 (n + 1) x2 + nx3 n 1 .................................... xn2 (n + 1) xn1 + nxn n 1 xn1 (n + 1) xn + nx1 n 1 n2 xn (n + 1) x1 + nx2 2n 1. Dac x1 = 1, s se calculeze x2003 . a a Romeo 2 G40. Comparai numerele reale a i b, tiind c a 14a + b2 + 6b t s s a Bogdan Rducan a

G41. Daca 0 < x y z, sa se arate ca x z x y x x2 y 2 z y z z 3 + + + + +1+ 2 + 2 + z y x y z x z x y z x Ovidiu Pop,

G42. Determinai a, b R dac [x] + [x + a] = [bx], x R. t a Gheorghe d G43. Fie xOy un unghi oarecare i P un punct n interiorul su. s a punctele A, B [Ox cu A (OB) i C, D [Oy cu C (OD) as s unghiurile P AB i P CD sa fie echilaterale. Aratai ca dreptele OP , A s t concurente daca i numai daca P se afla pe bisectoarea unghiului dat. s Temistocle

G44. Fie V ABC o piramida, iar G centrul de greutate al 4AB ce trece prin G taie dreptele V A, V B, V C n A0 , B 0 i respectiv C 0 . S s VA VB VC + + = 3. V A0 V B0 V C0 Constantin

G45. Fie SABC un tetraedru n care 4ABC nu este echilatera ile [SA] , [SB] , [SC] nu sunt toate congruente. Demonstrai ca exista t s A1 , B1 , C1 , A2 , B2 , C2 pe dreptele SA, SB, SC, BC, AC i respectiv AB trulaterele A1 B1 A2 B2 , B1 C1 B2 C2 i A1 C1 A2 C2 s fie trapeze izoscele ( a s 84

A1 C1 kA2 C2 , B1 C1 kB2 C2 ) dac i numai dac as a 2 2 2 2 2 SA AB AC + SB BC BA2 + SC 2 CA2 CB 2 Daly Marciuc,

B. Nivel liceal

L36. Fie 4ABC i M triunghiul sau median. Daca P este un p s t interiorul sau pe laturile lui M, iar A0 , B 0 , C 0 sunt interseciile drepte 1 AP BP CP CP cu laturile BC, CA i respectiv AB, atunci < s 4 AA0 BB 0 CC 0 Marian Ione s s L37. Fie cercurile C1 , C2 i C astfel nct C1 i C2 sunt tangente exte cercurile C1 i C2 sunt tangente interior lui C n B, respectiv C. Tang s interioar cercurilor C1 i C2 taie cercul C n A i A1 , dreapta AB taie a s s 1 2 1 = AC taie C2 n L. Sa se arate ca + . DA DA1 KL Neculai Roman, Mi L38. Fie 4ABC i punctele D, D0 BC conjugate armonic n rapo s a s B i C. Cercul circumscris 4ADD0 intersecteaz AB n M i AC n s b c, dac M N BC, atunci [AD i [AD0 sunt bisectoarele unghiului A a a s b exterioar) sau m(A) = 90 . a Temistocle an L39. Determinai toate numerele naturale nenule n pentru care t p ptrat perfect, unde a, p N . a Mihai 2 2 L40. Fie A, B Mn (Z) astfel nct det A B + AB este impar c A + B este inversabil pentru orice Q. a a Marian Ursre a L41. Demonstrai c grupul simetric S32 nu are elemente de ordin t a Paul Georgescu i Gabrie s L42. Fie (A, +, ) un inel finit cu cel puin 5 t elemente i cu 1 + 1 s a a Fie M = x A | x2 = 1 , I = x A | x2 = x . S se arate c card M < card A /2. Ovidiu Muntea L43. Determinai polinoamele P R [X] pentru care P (z) C\R, t Gheorghe s L44. Fie n 2 numar natural, iar f0 , f1 , f2 , . . . un ir de polin 0 prin: f0 = (X + 1)n , fp+1 = X fp , p 0. Definim nca hp = fp X p1 p1 n + + (1) p1 f1 , p 1, unde k = i1 i2 . . . ik , k
1i1 <<ik n

sunt sumele simetrice fundamentale ale numerelor 1, 2, . . . , n. S se ara a hp = n (n 1) . . . (n p + 1) X p (X + 1)np , p = 1, 2, . . . Marian Te L45. Fie f : [0, ) [0, ) continua. Daca funcia F : [0, ) t Z Z x 1 f (t) dt este marginita, sa se arate ca lim n xf (nx) dx = 0. = n 0 0 Adrian Za 85

Pagina rezolvitorilor

BOTO ANI S Scoala nr. 7 "Octav Bancila". Clasa a VIII-a. NEGRESCU Alexa 28,30,32,34), VII(26,32), VIII(26,27,32,34), G(7,11,32). BRA OV S Scoala generala nr. 5. Clasa a VI-a. POSTEUCA Raluca: V(26,3 35), VII(34). Clasa a VII-a. POSTEUCA Bogdan: V(26,31,33), VI(34 Scoala generala nr. 20. Clasa a VII-a. BOERIU Adela: VI(31-35) Liceul "N. Titulescu". Clasa a IX-a. ANDRA Cristian: VII(26 S VIII(32); BORICEAN Mihai: VII(26,29,32,34,35), VIII(32,33); BUR VII(26,29,32,34,35), VIII(27,32,33); CIOBOTA Andreea: VII(26,29,32 (32,33); CIOBOTA Cristian: VII(26,29,32,34,35), VIII(32,33); CMPE VII(26,29,32,34,35), VIII(27,32,33); COSTEA Rodica: VII(26,29,34), FERAR Achim: VII(26,29,32,34,35), VIII(27,32,33); FUNDUREANU VII(26,29,32,34,35), VIII(32,33); GHILIFTOIU Mirela: VII(26,29,32, CEA Ovidiu: VII(26,29,32,34,35), VIII(27,32, 33); MANEA George: VI 35), VIII(32); MIHALCEA Ctlin: VII(26, 29,32,34,35), VIII(32); M aa hai: VII(26,29,32,34,35), VIII(27,32,33); MUNTEAN Alexandru: VII VIII(32); MUNTEANU Luminia: VII(26,32,24), VIII(32,33); PUCHEA t VII(26,29,32, 34,35), VIII(27,32,33); R CU Laura: VII(26,27,29,34,35) S VLAD Daniel: VII(26,29,32,34,35), VIII(27). CRAIOVA Colegiul Naional "Fraii Buzeti". Clasa a VI-a. TUTESCU A t t s V(31,35), VI(31,32,34), VII(31,32), G(21); Clasa a VIII-a. DINU Lav VIII(31,32), G(21,23,26). FOC ANI S Colegiul Naional "Unirea". Clasa a VIII-a. SECARA Andree t VII(31,32), VIII(32,34,35), G(21,23,25). HRLAU (IA I) S Liceul Teoretic "Stefan cel Mare". Clasa a VII-a. ANTOCI Bogd 30), VI(29,33,34,35), VII(29,32); BURICAN Bogdan Alexandru: V(28,3 VII(29); MIHULCA Lucian: V(27,28,30,32), VI(29,34,35), VII(29,31,32 Marian-Drago: V(27,28,30,32,33), VI(29,33-35), VII(29); ROTARU L s 28,32), VI(29,32-35), VII(29). IA I S Colegiul Naional "C.Negruzzi". Clasa a VI-a. RO U Eugeni t S VI(31-35), VII(32), VIII(32), G(21,23,27). Clasa a X-a. IACOB Alin: L(21,28,29). Liceul "Garabet Ibraileanu". Clasa a VI-a. BUDEANU Stefan V(31-33), VI(31); FUIOREA Bogdan: P(42), V(31,32), VI(31,32); U Drago: P(42), V(31-33), VI(31,32). Clasa a VIII-a. ANDRIESCU s S 35), VII(32), VIII(34); BRANI TEANU Stefana:VI(35), VII(31,32), G VERDEANU George: VI(33,34), VII(32,35), VIII(34); MORO ANU M S 35), VII(31,32); TANASE Ioana: VI(33-35), VII(31,32); TUDORACHI 86

35), VII(31,32), VIII(32). Clasa a X-a. TONU Constantin: VIII X(33), G(32,33). Liceul Teoretic "M.Eminescu". Clasa a VIII-a. AVRAM Mircea: 35), VII(26,28,29,32),VIII(27); CIUCANU Radu: VI(27,34,35), VII(26 (27); DANAILA Mihai: VI(27,33-35), VII(26,29,32), VIII(27); DU A S (27,34), VII(31,32), VIII(33); TOFAN Andrei: VI(27,34,35), VII(26,29, TUDOSE Stefan: VI(27,34,35), VII(26,32), VIII(27); TURLIUC Rar 35), VII(29); GRAMSCHI Raluca: VI(26,27,34,35) (4 probleme). Cl DUMITRESCU Roxana: VII(26,29,31,32), VIII(31,32,34,35), IX(31). Scoala "G.Cobuc". Clasa a II-a (nv. GALIA Paraschiva). CIO s Ctlina: P(24,27,34-36); MIHAILESCU Laura-Ioana: P(24,27,34-36) aa Constantin: P(24,27,34-36). Clasa a II-a (nv. RACU Maria). BARA P(24,27,34-36); BURLACU Claudiu: P(24,27,34-36); CALOIAN Andre 36); CALIN Georgiana: P(24,27,34-36); CRACIUN Mdlina: P(24,27, a a GAN Crina-Alexandra: P(24,27,34-36); MOISA Bogdan: P(24,27,34-36 Razvan: P(24,27,34-36); RUSU Flavia: P(24,27,34-36). Scoala "Al. Vlahua". Clasa a IV-a (nv. MAXIM Gabriela). CIOC t Florin: P(34-42); MUNTEANU Ioana-Alexandra: P(34-42); SOFICU P(35-37,39,40); STURZU Tudor-Nicolae: P(33-42). Scoala "Alexandru cel Bun". Clasa a II-a (nv. SPNU Doinia) t Ionu-Mihai: P(24,25,27,33-35); COJOCARIU Oana-Alexandra: P(24, t COJOCARU Veronica: P(24,25,27,33-36); DAMIAN Daniel: P(24,25,27 REA Roxana-Maria: P(24,25,27,33-37); FURTUNA Marta: P(24, 25,27 NIE Ioana: P(24,25,27,34-36); IVANOV Alla: P(24,25,27,33-36); M Mihai: P(24,25,27,33-36); MIHAILA Tofana-Maria: P(24,25,27,33-37 Ciprian: P(24,25,27,33-37); PATRA C Ilinca: P(24,25,27,33,36,37); R S dru: P(24,25,27,33-37); SRBU Silviu Alexandru: P(24,25,27,33,35, Andreea: P(24,25,27,33-37); URSU Gina-Ioana: P(24,25,27,33-36). Scoala "B.P.Hadeu". Clasa a IV-a (nv. STEFAN Liviu). PIN s Liviu: P(24-31); PINTILIE Nicoleta: P(24-30); STERBULEAC Dan 31). Clasa a IV-a (nv. TRZIORU Iuliana). CHIHAIA Mihai-Seb 43); RAITA Bogdan: P(34-43); SILION Catalina: P(34-42); SPNU D P(33-43). Scoala "N.Tonitza". Clasa a III-a (nv. MARCU Monica). BUTNA P(35-37,39,40); ONUTA Alin: P(35-37,39,40). Clasa a IV-a (nv. Loredana: P(34,38-42); BONCU Andrei: P(34,38 Elena). ANDRU CA S Scoala "T.Maiorescu". Clasa a III-a (nv. CHIRILA Beatrice). T Alexandru-Gabriel: P(34-41). Scoala "O.Cazimir". Clasa a IV-a (nv. PRIALA Dumitru). T P(38-43). PLOIE TI S Colegiul Naional "I.L.Cargiale". Clasa a VI-a. JELEA Anc t V(21,23,28) (soluiile au fost primite nainte de apariia nr. 1/2002). t t

87

IMPORTANT

n scopul unei legaturi rapide cu redacia revistei, pot fi utiliz t toarele adrese e-mail: tbi@math.tuiasi.ro, popagabriel@ Pe aceast cale colaboratorii pot purta cu redacia un dialog a t materialele trimise acesteia, procurarea numerelor revistei etc La problemele de tip L se primesc soluii de la orice iubitor de m t elementare (indiferent de preocupare profesionala sau vrsta ) dintre soluiile acestor probleme - ce sunt publicate n revist t a an - va fi urmat de numele tuturor celor care au rezolvat-o. a Adresm cu insistena rugmintea ca materialele tri a t a vistei s nu fie (s nu fi fost) trimise i altor publica a a s t

Redacia revistei "Recreaii matematice" acord cte o di t t a premiu n cri urmtorilor elevi: at a ANDRIESCU Alina (Lic. "G. Ibraileanu", cl. a VIII-a): 2/2001 ( (5pb), 1/2003 (5pb); S BRANI TEANU Stefana (Lic. "G.Ibraileanu", cl. a VIII-a): 2/2001 (11pb), 1/2003 (5pb); BUDEANU Stefana (Lic. "G. Ibrileanu", cl. a VI-a): 1/2002 (6pb), a 1/2003 (6pb); CHIHAIA Mihai - Sebastian ( c. "B. P. Hadeu", cl. a IV-a): 1 S s 2/2002 (9pb), 1/2003 (10pb); JUVERDEANU George (Lic. "G. Ibraileanu", cl. a VIII-a): 1/2002 (5pb), 1/2003 (5pb); RAITA Bogdan ( c. "B. P. Hadeu", cl. a IV-a): 1/2002 (10pb), 2 S s 1/2003 (10pb); SILION Ctlin ( c. "B. P. Hadeu", cl. a IV-a): 1/2002 (10pb), 2 a a S s 1/2003 (9pb); SPNU Drago - Andrei ( c. "B. P. Hadeu", cl. a IV-a): 1/2002 (1 s S s (9pb), 1/2003 (11pb); TUDORACHE Alexandru - Gabriel ( c. "T. Maiorescu", cl. a I S (12pb), 2/2002 (7pb), 1/2003 (8pb); TUDOSE Stefan (Lic. "M. Eminescu", cl. a VIII-a): 1/2001 (5pb), 1 1/2003 (5pb); TUTESCU Anca Stefania (Colegiul Naional "Fraii Buzeti", Craio t t s 1/2002 (6pb), 2/2002 (8pb), 1/2003 (8pb); UNGUREANU Bogdan (Lic. "G. Ibraileanu", cl. a VI-a): 1/2002 ( (7pb), 1/2003 (6pb), autor al problemei V.36

Crile au fost oferite de revista "Recreaii matematic a t t

88

Al V-lea Congres internaional t al matematicienilor romni


Piteti, 22 - 28 iunie, 2003 s

Incepnd cu anul 1929, s-au organizat, pna n prezent, cinci cong naional ale matematicienilor romni. t Primul Congres a avut loc n anul 1929 la Cluj avnd ca promotor Sergescu, dar avnd concursul celor mai reputai matematicieni romn t perioada. Al II-lea Congres a avut loc la Turnu Severin n 1932, bucurnd primul de participarea unor mari matematicieni ai timpului, Paul Montel Denjoy, Waclaw Sierpinski i alii. s t Al III-lea Congres s-a desfaurat la Bucureti n 1945, cu o participar s s a matematicienilor straini, date fiind condiiile dificile de la sfritul celui d t s rzboi mondial. a Cel de-al IV-lea Congres a fost organizat tot la Bucureti, cu o pre s ciala, n anul 1956. Statul romn a facut un efort special i prestigiul s bucurau matematicienii romani (toi formai n marile coli din occident) t t s invitarea unui numr nsemnat de matematicieni strini, de faim inter a a a Din Frana au participat Jacques Hadamard i Arnaud Denjoy, din t s Masuo Hukuhara, din Germania - W. Blaschke, din Statele Unite - Ei i S. Eilenberg, din Uniunea Sovietica - I. Vekua, din Polonia - K. Kura s T. Wazewski. A fost un prilej de rentlnire a matematicienilor romni din lui Grigore Moisil, Gheorghe Vrnceanu, Miron Nicolescu, Simion a Nicolae Teodorescu, Tiberiu Popoviciu cu fotii lor mentori sau cole s Dupa o ntrerupere de 47 de ani, s-a organizat cel de-al V-lea Congr versitatea din Piteti, al crei Rector dr. Gheorghe Barbu este el nsui ma s a s (format la Iai i Bucureti). Domnia sa i-a asumat sarcinile dicile ale s s s s congresului, eveniment organizat sub egida Academiei Romne, a Univer cureti si a Institutului de Matematica "S. Stoilow" al Academiei Romn s zarea congresului a fost reuit, datorit n primul rnd comitetului local s a a zare, autoritilor locale i sprijinului acordat de la Bucureti. Ca o nou at s s ristica a acestui congres, subliniem prezena unui numar nsemnat de mat t romni care-i desfaoara acum activitatea n ari straine (a se vedea Liber s t s matica, vol. XXIII, n care se afl numele i adresele a peste 300 de mat a s romni ce dein catedre n universiti din strintate, pe toate continente t at a a "Ziarul de Azi" din Piteti, n timpul desfaurarii congresului, a publicat s s relatari i informaii privind participarea unor reputai matematicieni stra t t s a multor matematicieni romni care au activat sau activeaz n alte ari. a t din ultima categorie vom aminti academicienii Nicolae Cristescu i Nic s culeanu, Sergiu Klainerman (Princeton), Dan Burghelea (Columbu Daniel Tataru (Berkeley, CA), Henri Moscovici (Columbus - Ohio), M (Tel Aviv), Radu Theodorescu (Laval, Canada). Spre deosebire de multe alte congrese sau conferine cu participare inter t congresele internaionale ale matematicienilor romni au fost ntotdeauna t

zate printr-o larg reprezentare a tuturor domeniilor de baz din cerceta a a matica. Astfel, cele peste 400 de comunicari anunate pentru Congresu t au fost distribuite n 15 secii, ncepnd cu Logica, Algebra i Teoria n t s mergnd pn la Istoria si Filozofia matematicii i Pedagogia matematic a s reprezentate Geometria, Analiza clasica si moderna, Ecuaiile diferenia t t controlului optimal, Teoria probabilitatilor si Statistica matematica, Cerceta tionala, Mecanica i Astronomia, Fizica matematica. Lucrarile s-au desfa s n plenul congresului (ncepnd cu edina de deschidere la care Ambasado s t la Bucureti, E. S. Philippe tienne, el nsui matematician i admirabi s s s a captivat audiena), precum i n numeroase secii pe specialitai. t s t t Pe lnga matematicienii straini care au participat la Congres, venind Unite, Canada, Frana, Germania, Rusia, Ungaria, Italia i alte ari, t s t remarcm prezena destul de nsemnat a matematicienilor din Republica a t a Este destul de dicil sa prezentam o vedere de ansamblu asupra desfau s gresului al V-lea al matematicienilor romni, data ind varietatea domen date de ctre participani. Vom sublinia totui faptul c programul si de a t s a lucrarilor congresului s-au ncadrat n standardele internaionale. O cri t adus organizatorilor a fost aceea ca data congresului a coincis cu multiple academice, cum ar fi: examenele studeneti, examenul de licena i altel t s t s acesta, muli doritori din ar de a participa au fost abseni. t t a t Vom ncheia subliniind faptul c acest al V-lea Congres a ilustrat a matematicii romneti, ncadrarea ei reuita n comunitatea matematica i s s nala. S sperm c urmtorul congres va avea loc dup o perioad n a a a a a a ndelungat ca pn acum. a a

Constantin CORDUNEAN University of Texas at Arlin

Observatorul din Iai 90 de ani de la nfiin s

nfiinarea observatoarelor astronomice din Bucureti (n 1908) i ap t s s (n 1913) face parte dintr-un proces mai amplu de modernizare a nv at universitar i a cercetrii tiinifice, proces impulsionat de Legea Haret d s a s t care se va maturiza n condiiile social-politice i culturale din Romnia t s nca din momentul nfiinarii n 1860 a Universitii din Iai, n t at s "seciei tiinelor pozitive din facultatea de filozofie" sunt prevzute i t s t a s mecanic i astronomie, dar catedrele aferente vor cpta fiina mai t a s a a t legea nvaamntului din 1864, care se pune n aplicare ncepnd cu d t febr. 1865, se creeaza Facultatea de tiine, desprinsa din Facultatea d s t i avnd trei secii distincte: fizic, matematic i tiine naturale; una d s t a as s t catedre ale noii faculti este cea de geodezie teoretica si astronomie. La 15 at este numit profesor titular al acestei catedre Neculai Culianu, care o pn n 1906, anul pensionrii sale. N. Culianu trece licena n tiine m a a t s t la Sorbona, este atras de astronomie i de Observatorul din Paris, cun s i rmne prieten pentru toat viaa cu astronomul francez Camille Fla s a a t N. Culianu este autor al unui Curs de cosmografie pentru liceu (doua edi 1902). Universitatea din Iai a primit, chiar din momentul nfiinarii, de la s t de Medici si Naturaliti din Iai un bun instrument de observaii astrono s s t aparinuse poetului moldovean Costache Conachi i pe care motenitorii t s s acesteia. Dup nfiinarea Catedrei de astronomie (n 1864) au fost achiziion a t t instrumente; ele au fost depozitate ntr-o cmrua a vechiului local al un a a t nct nici nu puteau fi artate studenilor. Cu toate insistenele nu s-a r a t t ndelungat obinerea fondurilor pentru construirea unui observator astron t

Constantin Popovici este liceniat al Facultii de tiine din Iai (190 t at s t s la Paris cu o burs "Adamachi" unde obine din nou licena n matematic a t t apoi doctoratul la Sorbona (1908) n domeniul ecuaiilor difereniale. n t t numit la Catedra de geometrie analitica a universitaii ieene, iar n 1910 s t n Frana pentru specializare n astronomie i documentare n privina t s t viitorului observator din Iai. Se rentoarce i este numit n 1911 la C s s astronomie, geodezie i mecanica cereasca, Universitatea din Iai. s s C. Popovici este fondatorul Observatorului astronomic din Iai, am s dealul Copou; piatra de temelie a cldirii a fost pus la 12 sept. 1912, ia a a s-a fcut la mijlocul lui decembrie 1913. a C. Popovici este primul director al Observatorului (n perioada 1913-193 le instrumente intrate n dotarea acestuia au fost cele provenite de la ca astronomie ninat de N. Culianu. Prin strdaniile lui C. Popovici i a t a s i colaboratorului su, Vintil Siadbei, au fost achiziionate noi instr s a a t luneta meridiana, un ecuatorial Ressel, doua cronometre (pentru timpul m sideral), un fotometru Gra i altele necesare procesului didactic. s n anul 1938 Catedra de astronomie este transformat ntr-o con a C. Popovici se transfer la Bucureti. n perioada 1938-1944, Vintil a s a suplinit conferina de astronomie. t

Ca urmare a evacurii Observatorului, prilejuit de cel de-al doilea rzbo a a a o buna parte a aparaturii din dotarea acestuia s-a deteriorat sau a fost su

n 1948 Victor Nadolschi ocupa prin concurs conferina de astronomi t directorul Observatorului din Iai, funcie deinuta pna n anul 1966. V. s t t este un eminent continuator al lui C. Popovici i al lui V. Siadbei. s organizeaz i relanseaz activitatea i pune bazele astronomiei fotogra a s a s V. Nadolschi achiziioneaza un astrograf Zeiss (1956), un fotometru fotoelec t un ecuatorial Zeiss cotit (1960), un aparat pentru masurat cliee (1963), s zenital Meopta (1963) etc. ncepnd cu anul 1966 activitatea didactic i de cercetare este coor a s Iulian Breahn, absolvent al Universitaii din Bucureti, secia de astro a s t t Din 1966 funcioneaza n cadrul Observatorului din Iai un atelier de t s fina i un laborator electronic necesare ntreinerii i cercetrii. A fost a t s a s un orologiu cu cuar care, completat ulterior cu alte anexe, constituie i t s un cronograf digital de precizie. n anul 1980 a fost achiziionat un planetariu Zeiss destinat nvaam t t tronomiei, care a fost instalat n incinta Universitii din Iai. Studeni at s t posibilitatea de a-i nsui mai uor o multitudine de fenomene privind cin s s s dinamica sistemului planetar al Soarelui. Planetariul a atras pna n prez mii de vizitatori. Cu prilejul eclipsei totale de Soare din 11 august 1999 s-a achiziionat u t CCD (dispozitiv cu cuplaj de sarcin) de performana i o camera Astro a t s nregistrari continue de imagini. n perioada 1951-1999, pe lnga Observator i prin grija personalului s funcionat o staie seismica. Observaiile efectuate de aceasta au pus n t t t dou focare seismice: unul la circa 25 km dincolo de Prut i al doilea n zo a s Zorleni. Activitatea de cercetare desfaurata pe lnga Observatorul din Iai s-a c s s n peste 140 lucrri. C. Popovici a generalizat legea Newton-Coulomb prin a rea unei fore neconservative, rezultat dintr-o combinaie a gravitaiei n t a t t cu presiunea luminii. V. Siadbei obine rezultate noi privind traiectoriile t i cometelor i face observaii asupra eclipselor de Luna i Soare, stabilind s s t s simple pentru calculul acestora. V. Nadolschi s-a preocupat de teoria grupurilor de pete solare, continu tradiia observrii eclipselor i are a t a s a fi pus bazele astronomiei fotograce la Iai. Abordarea unor teme din s radioastronomiei s-a dovedit deosebit de dicila, dei s-au depus efortur s pentru crearea bazei materiale necesare unei astfel de cercetri. a Cu toate c de-a lungul timpului au fost de nlturat multe dicult a a at cole, la Observatorul din Iai s-a reuit sa se desfaoare o activitate car s s s cunoscut n ara i n strainatate. Aceste armaii sunt dovedite i de acord t s t s lui de membru al Uniunii Astronomice Internaionale urmtorilor astron t a Constantin Popovici, Vintila Siadbei, Victor Nadolschi i Iulian Breahna. s

Redacia revistei t

Marea teorem a lui Fermat pentru polinoa a


Temistocle BRSAN 1

1. Odata cu caderea Constantinopolului (1453), muli nvaai biza t t t ndreptat spre Europa de Vest aducnd cu ei manuscrise preioase - m t care supravieuiser devastrii Bibliotecii din Alexandria se adunaser d t a a a timpului n aceasta capitala a lumii. Prin hazardul mprejurarilor, ase din cele 13 volume ale Aritmeticii lu s au ajuns n Frana. nvatul i amatorul de matematic francez Claud t at s a Bachet de Mziriac i d seama de importana crii lui Diofant i publ s a t at s o versiune n limba latina a Aritmeticii, care cuprinde peste o suta de p rezolvarile detaliate ale lui Diofant. Pentru Pierre Fermat (1601-1665) Aritmetica lui Diofant a fost c l-a pus n contact cu bogatele cunotine ale popoarelor antice n direc s t numerelor i sursa de inspiraie pentru noi i subtile probleme pe care s s t s formula. Fermat obinuia sa noteze pe marginile carii lui Diofant come s t cule i schie de demonstraii. Nu s-a preocupat s-i publice rezultatele s t t as straiile, dar se amuza comunicndu-i rezultatele altor matematicieni ai t t s provocndu-i la rezolvarea acestora. n Cartea a II-a a Aritmeticii, Fermat gasete informaii bogate relativ l t s pitagoreice, adic trei numere naturale ce verific ecuaia lui Pitagora a a t x2 + y 2 = z 2 .

Stia c Euclid demonstrase c exist o infinitate de astfel de triplete. Ce se a a a ns, dac n loc de (1) se consider ecuaia a a a t xn + y n = z n ,

unde n 3? Rspunsul lui Fermat, notat ca observaie pe marginea crii l a t at este cu totul surprinztor: nu exista nici o soluie a ecuaiei (2) cu nume a t t nenule, daca n = 3, 4, . . . . Urmeaza notat urmatorul comentariu: Cuius rei demonstrationem mirabilem sane detex hanc marginis exig caperet [4]. (Ma aflu n posesia unei demonstraii minunate a acestei afir t marginea paginii este prea strmta pentru a o cuprinde.)

Aceasta extraordinara descoperire, care astazi poarta numele de Marea a lui Fermat, ct i alte rezultate, ar fi putut sa ramna necunoscute lumi s ticienilor i s se piard, dac, dup moartea lui Fermat, fiul su cel mai s a a a a a fi examinat nsemnrile scrise de tatl su pe margini i n-ar fi publicat a a a s lui Diofant coninnd si observaiile lui Pierre de Fermat (Toulouse, 1670 t t Pe parcursul ctorva secole, cele mai sclipitoare mini de matematicien t cat i i-au adus contribuia la rezolvarea acestei enigme (i, totodat, s s t s a lsat de Fermat: Euler, Sophie Germain, Dirichlet, Legendre, Lam a a Kummer . a. Drumul ce duce la demonstrarea Marii teoreme a lui Fe s
1

Prof. dr., Catedra de matematic, Univ. Tehnic "Gh. Asach", Iai a a s

presrat cu reuite pariale, ambiii, nfrngeri, decepii, orgolii, intrigi, te a s t t t sinucidere etc. [4]. n anul 1995, dupa opt ani de munca nentrerupta, n completa izola colegii si i pstrnd o discreie total asupra cercetrilor sale, englezu a s a t a a Wiles pune capt enigmei de peste 350 de ani: Marea teorema a lui F a demonstrata! Demonstraia data de Wiles este, nsa, accesibila unui numa t de specialiti; n fapt, Wiles pentru a atinge scopul a dovedit justeea C s t Taniyama - Shimura utiliznd o aparatur matematic modern i sostic a a as eliptice, forme modulare, reprezentri Galois . a. [5]. a s

x (X) = [p (X)] [q (X)] , y (X) = 2p (X) q (X) , z (X) = [p (X)] + i verificam direct ca tripleta (x (X) , y (X) , z (X)) este o soluie a ecua s t C [X]. Similar cu Marea teorem a lui Fermat se formuleaz a a

2. Este cunoscut faptul c inelul Z al numerelor ntregi i inelul C [X a s noamelor cu coecieni numere complexe au proprieti asemntoare. t at a a apare ca fireasca problema rezolvarii ecuaiilor (1) i (2) n C [X]. t s n privina ecuaiei (1) constatm uor, ca i n cazul numeric, c are o t t a s s a de soluii: p, q C [X], lum t a
2 2 2

Teorema lui Fermat pentru polinoame ([3], [5]). Daca n este n 3, atunci ecuaia (2) nu are soluii n C [X] cu polinoame neconstant t t prime.

Surprinztor, spre deosebire de Marea teorem a lui Fermat, pentru ace a a se cunoate o demonstraie elementara i simpla, accesibila unui elev de lic s t s tatul este cunoscut din sec. al XIX-lea i a fost demonstrat utiliznd cun s geometrie algebric. Demonstraia elementar la care ne-am referit se s a t a o teorem de dat recent datorat matematicienilor W. Stothers (1981 a a a a pendent, R. C. Mason (1983), teorema foarte importanta i n sine. Sun s cteva (puine!) pregatiri. t Fie p C [X] un polinom neconstant avnd rdcinile a1 , a2 , . . . , ak c a a de multiplicitate respective m1 , m2 , . . . , mk ; deci p se scrie sub forma k Y (X ai )mi , C . p (X) =
i=1

Notm gradul polinomului p i numrul rdcinilor sale distincte cu deg p a s a a a s a n0 (p), adic deg p = m1 + m2 + + mk , n0 (p) = k. Menionm c, dac p, q C sunt neconstante, avem t a a a deg (pq) = deg p + deg q, n0 (pq) n0 (p) + n0 (q) , cu egalitate dac i numai dac p i q sunt relativ prime. as a s Derivata formal a polinomului p dat de (3) este a p0 (X) = [m1 (X a1 ) + mk (X a1 )
m1 1 m1

(X a2 )

m2

(X ak1 )

mk1

(X ak ) (X ak )

mk

mk 1

+ + ]

i, ca urmare, cel mai mare divizor comun al polinoamelor p i p0 are form s s (p, p0 ) = (X a1 )m1 1 (X a2 )m2 1 (X ak )mk 1 . Atunci de unde obinem relaia t t deg p = deg (p, p0 ) + n0 (p) .

deg (p, p0 ) = (m1 1) + (m2 1) + + (mk 1) = deg p n0 (p

Teorema Mason - Stothers. Fie p, q, r C [X] neconstante si rela Daca are loc egalitatea p + q = r, atunci max {deg p, deg q, deg r} n0 (pqr) 1.

Demonstraie (dat de Noah Snyder [3], p.30). Vom ncepe cu dou t a a utile. Mai nti, n prezena condiiei p + q = r, polinoamele p, q, r sunt rel t t daca i numai daca sunt prime doua cte doua. Apoi, ntruct enunul teo t s simetric n p, q, r (cci putem scrie egalitatea i sub forma p + q + r a s restrngem generalitatea dac vom presupune c polinomul r are grad a a ridicat. Ca urmare, inegalitatea de demonstrat se scrie deg r n0 (pqr) 1. Avem
0

p0 q pq 0 = p0 (p + q) p (p0 + q 0 ) = p0 r pr0 . s Constatm c (p, p ) i (q, q 0 ) divid membrul stng, iar (r, r0 ) divide memb a a deci i pe cel stng. Cum p, q, r sunt prime doua cte doua, urmeaza ca s (p, p0 ) (q, q 0 ) (r, r0 ) divide p0 q pq 0 . n consecina, t sau, datorita relaiei (4) i analoagelor ei, t s deg p n0 (p) + deg q n0 (q) + deg r n0 (r) deg p + deg q deci deg r n0 (p) + n0 (q) + n0 (r) 1. Cum p, q, r sunt prime doua cte doua, obinem n final t care este tocmai relaia (50 ) de demonstrat. t deg r n0 (pqr) 1,

deg (p, p0 ) + deg (q, q 0 ) + deg (r, r0 ) deg (p0 q pq 0 ) deg p + deg q

Demonstraia Teoremei lui Fermat pentru polinoame. Presu t ecuaia (2) pentru n 3 ar avea o soluie (x (X) , y (X) , z (X)) cu polinoa t t stante relativ prime. Aplicm teorema Mason - Stothers polinoamelor p(X) a n n q (X) = [y (X)] i r (X) = [z (X)] . Obinem t s deg [x (X)]n n0 ([x (X)]n [y (X)]n [z (X)]n ) 1 sau n deg x (X) n0 (x (X) y (X) z (X)) 1.

Tinnd seama c x (X), y (X) i z (X) sunt prime dou cte dou i de a s a a s n0 (p) deg p, p C [X], vom avea n deg x (X) n0 (x (X)) + n0 (y (X)) + n0 (z (X)) 1 deg x (X) + deg y (X) + deg z (X) 1. n deg x (X) deg x (X) + deg y (X) + deg z (X) 1,

Obinem astfel inegalitatea t

precum i inegalitaile analoage scrise pentru y (X) i z (X), care adunate s s t adic a n (deg x (X) + deg y (X) + deg z (X)) 3 (deg x (X) + deg y (X) + deg z

(n 3) (deg x (X) + deg y (X) + deg z (X)) 3. Evident, dac n 3, aceast relaie ne conduce la o absurditate, ceea a a t demonstraia. t

3. Analogia care exist ntre inelele Z i C [X] pune n mod firesc a s "translarii" teoremei Mason - Stothers de la polinoame la numerele nt nct Marea teorem a lui Fermat s poat fi demonstrat elementar. a a a a D. Masser i J. Oesterle (1986) au ajuns la aa - numita conjectu s s urmare a unor consideraii de geometrie algebric i teoria funciilor modu t as t n legatura cu teorema Mason - Stothers). k Q mi Daca m N are descompunerea n factori primi m = pi , atunci i=1 k Q pi . radicalul lui m numarul N0 (m) =
i=1

Conjectura abc ([2], [3]). Dat > 0, exista o constanta C () a pentru orice ntregi a, b, c nenuli si relativ primi cu a + b = c avem inega max {|a| , |b| , |c|} C () (N0 (abc))
1+

Aceast conjectur spune c, dac n descompunerea numerelor a, a a a a factori primi cu exponeni mari, aceti factori sunt compensai prin factori t s t muli, dar cu exponentul 1. t S enunam acum aa - numita a t s

Teorema lui Fermat asimptotic. Exista un ntreg pozitiv n1 cu pr a ca, daca n n1 , atunci ecuaia (2) nu are soluii cu x, y, z ntregi si xy t t

Cu aceleai argumente ca n cazul polinoamelor se poate dovedi urmat s

Teorem ([2], [3]). Conjectura abc implica Teorema lui Fermat asim a Demonstraie. Fie date x, y, z pozitive i relativ prime astfel nc t s (x, y, z) sa fie soluie a ecuaiei (2), adica xn + y n = z n . t t Notm a = xn , b = y n i c = z n i observm c a s s a a Utiliznd conjectura abc obinem t xn C () (xyz)1+ , N0 (abc) = N0 (xn y n z n ) = N0 (xyz) xyz. y n C () (xyz)1+ ,

z n C () (xyz)1+

Prin nmulire, rezult c t a a de unde i cum xyz > 2, obinem s t (xyz) [C ()] (xyz)
n 3 3+3

(n 3 3) log (xyz) 3 log C () n<

3 log C () + 3 + 3. log 2 Notm a 3 log C () + 3 + 3 . n1 = log 2 Urmeaz c ecuaia (2) nu are soluii ce verific condiiile specificate dac a a t t a t ceea ce trebuia demonstrat.

Observaie. Aceast cale nu ofer o demonstraie a Marii teoreme a l t a a t ntr-adevar, numarul n1 definit de (6) depinde de C () (putem consider C (1) pentru a fixa ideile). Determinarea efectiv a constantei C () nu e a cut. Dac, de exemplu, C (1) s-ar putea efectiv determina, atunci dem a a Marii Teoreme a lui Fermat s-ar reduce la un numr finit de cazuri, care a abordate prin calcul direct.

4. Interesul pentru Marea teorema a lui Fermat nu s-a stins nici dup strarea ei. Au ramas ntrebari fara raspuns, sunt formulate altele noi. Da nu a dat dect o demonstraie eronat, care ar putea fi natura greelii fc t a s a aceast demonstraie ar fi corect, care este acel argument ingenios produ a t a lui Fermat ce a scapat attor matematicieni ilutri? Este posibila o dem s elementara, accesibila i unor persoane cu cunotine obinuite de matem s t s s n 1966, Andrew Beal instituie un premiu pentru demonstrarea sau aa - numitei Conjecturi Beal, care este o generalizare a problemei lui Fer s Ecuaia xp + y q = z r , p, q, r numere ntregi mai mari ca 2, nu are nic t cu x, y, z ntregi pozitivi si relativ primi ([6], [1]). Bibliografie 1. 2. 3. 4. 5. 6.

A. Corduneanu - Despre Marea teorema a lui Fermat, Recreaii M t 1 (1999), nr.1, 37-39. S. Lang - Old and new conjectured diophantine inequalities, Bull. AMS, 37-75. S. Lang - Math Talks for Undergraduates, Springer, 1999. S. Singh - Marea teorema a lui Fermat, Humanitas, Bucureti, 1998. s A. Wiles - Modular elliptic curves and Fermats Last Theorem, Annals 142 (1995), 443-551. *** - Beals Conjecture, The New Zealand Math. Mag., 35 (1998), n

De la o problem cu matrice la transform a a elementare


Marian TETIVA1

1. Introducere. Problema la care ne referim n titlu este urmatoarea

Sa se arate ca nu exista matrice patratice X, Y Mn (C) astfel nct XY In ind matricea unitate de ordinul n.

Este o problem cunoscut, care poate fi ntlnit n mai multe ma a a a culegeri, care s-a dat la concursuri etc. i nu este tocmai simpl: un elev m s a ntotdeauna descurajat de enunuri de tipul "sa se arate ca exista /nu t Mai mult, n aceasta situaie nu prea avem alta cale de abordare n afara t utilizeaz noiunea de urm a unei matrice i proprietile sale. Istoria a t a s at este cam aa: prin anii 70 ai secolului trecut ea era propus la olimpiad s a a 80 a patruns n manuale pentru ca n anii 90 sa ajunga a fi parte din di de bacalaureat sau admitere la facultate; aceasta spune ceva despre felul evoluat programele nvamntului matematic elementar n Romnia. N at c elevul mediu din ziua de azi se afl n acelai impas ca i cel de acum do a a s s treizeci de ani (sau poate chiar mai rau) atunci cnd este confruntat cu probleme. De aceea aceasta nota i se adreseaza, dar numai daca este cu interesat de matematic. a Amintim c urma matricei A = (aij )1i, jn Mn (C) este, prin a numarul Tr (A) = a11 + a22 + + ann (suma elementelor situate pe " principal" a matricei). Sunt cunoscute urmtoarele proprieti ale urmei a a at 1 Tr (A + B) = Tr (A) + Tr (B), A, B Mn (C), 2 Tr (A) = Tr (A), C, A Mn (C), 3 Tr (AB) = Tr (BA), A, B Mn (C). Primele dou egaliti se mai pot scrie condensat n forma Tr (A a at = Tr (A) + Tr (B), oricare ar fi , C i A, B Mn (C) i expr s s ritatea urmei: Tr : Mn (C) C este aplicaie liniara (sau morsm de t vectoriale). De aici deducem Tr (XY Y X) = Tr (XY ) Tr (Y X) = 0 pe X, Y Mn (C) i aceasta explic de ce egalitatea din enun nu poate avea s a t nici o pereche de matrice X, Y : matricea XY Y X are urma nul, deci n a egala cu In , a carei urma este n. Remarcm c matricea In din enun poate fi nlocuit cu orice matrice a a t a n avnd urma nenul, enunul i rezolvarea rmnnd valabile; problem a t s a uor reformulat astfel: s a

Daca pentru o matrice A Mn (C) exista X, Y Mn (C) astfel XY Y X, atunci Tr (A) = 0.

Atunci se nate n mod natural ntrebarea dac reciproca acestei afir s a adevrat, adic se pune problema valabilitii urmtorului enun: a a a at a t

Fie A o matrice patratica de ordin n cu elemente numere complexe. D matricei A este nula, atunci exista X, Y Mn (C) astfel nct A = XY
1

Profesor, Colegiul Na ional "Gh. Roca Codreanu", Brlad t s

n cele ce urmeaz ne propunem s rezolvm aceast problem; mai a a a a a aratam ca raspunsul la ntrebare este afirmativ. Ideea rezolvarii este sa cautam nite matrice Y, Z Mn (C) astfel nct s fi scris n forma A = Z Y ZY 1 (Y fiind inversabil, desigur); atunci pr a a fi rezolvat: e suficient s alegem X = ZY 1 i avem A = ZY 1 Y Y a a s = XY Y X. Aici cititorul poate avea o nemulumire: de unde i pna unde aceste m t s n locul lui Xi Y din enun? S remarcm c din proprietatea 3 rezult s t a a a a 4 Tr (A) =Tr CAC 1 , C Mn (C), C inversabil (se dovedete im a s A = AC 1 C). Matricele de forma A i CAC 1 sunt asemenea, iar pr s 4 spune ca acestea au aceeai urma. s Desigur, problemele abia ncep. Sunt necesare cteva pregatiri.

2. Matrice asemenea i transformri elementare. Fie K un corp c s a cititorul mai puin familiarizat cu aceast noiune abstract poate cons t a t a notaie pentru unul dintre corpurile numerice uzuale Q, R sau C. t

Dou matrice X, Y Mn (K) se numesc matrice asemenea ( sau simi a exista U Mn (K) cu det U 6= 0 astfel nct Y = U XU 1 (vom nota Cititorul poate verifica uor faptul c relaia de asemnare (similaritate) es s a t a de echivalena pe mulimea Mn (K). t t

Transformarile elementare care se fac asupra unei matrice sunt, n cele mai simple modificari care nu i afecteaza rangul, adica interschimba linii (sau coloane), adunarea unei linii (coloane) nmulite cu un numar ( t element al corpului K) la alt linie (respectiv coloan), sau chiar nmu a a linii (coloane) cu un numr nenul. Una din cele mai simple aplicaii a a t calculul rangului unei matrice; de asemenea, se pot folosi aceste transform rezolvarea sistemelor de ecuaii liniare. t S ncepem prezentarea transformrilor elementare cu aa numitele m a a s mentare; legtura va aprea curnd. Vom nota cu Eij matricea ptratic a a a n peste corpul K ale carei elemente sunt toate nule, cu excepia element t linia i i coloana j care este egal cu 1. Se verica uor ca matricele Eij , s s 1 j n formeaz o baz a spaiului vectorial Mn (K) peste K, precum a a t Eij Ekl = 0n , j 6= k i Eij Ejl = Eil . s Se numesc matrice elementare urmatoarele tipuri de matrice patratice n, cu elemente din K: 1) Matricele Tij (a) = In +aEij ; aici a K i i, j {1, 2, . . . n} sunt ind s Matricea Tij (a) se obine din matricea unitate fcndu-i o singur modi t a a mentul de pe linia i i coloana j devine a. Se constata imediat ca (vezi pr s matricelor Eij ) Tij (0) = In , Tij (a) Tij (b) = Tij (a + b) , Tij (a) GLn (K) , Tij (a)
1

unde GLn (K) = {U Mn (K) | det U 6= 0} - grupul general liniar de ord corpul K. Deci {Tij (a) | a K} formeaz, pentru i 6= j fixate, un grup a grupul (K, +). Sa vedem ce efect are nmulirea unei matrice oarecare cu t

= Tij (a) , a, b K,

Tij (a). Fie A = (akl )1k,ln o matrice din Mn (K), care mai poate n X akl Ekl . Atunci A=
k,l=1

Tij (a) A = (In + aEij ) =


n X

X n
l=1

akl Ekl

akl Ekl +

k,l=1

k,l=1 n X

k,l=1

n X

akl Ekl +

k,l=1

n X

aakl Eij E

aajl Eil .

a Ce nseamn asta? nseamn c elementele matricei Tij (a) A rmn a a a ale matricei A, cu excepia celor de pe linia i: aici, n locul elementulu t acum ail + aajl , adic matricea Tij (a) A se obine din A prin adunarea a t a liniei j nmulite cu a, cu alte cuvinte nmulirea la stnga cu o matr t t realizeaz o transformare elementar a matricei A. De asemenea, se poate a a acelai fel c matricea ATij (a) se obine din A prin adunare la coloana j s a t i nmulite cu a. t 2) Matricele Qij = Tij (1) Tji (1) Tij (1) (i, j {1, 2, . . . , n} , i 6= j) n categoria matricelor elementare. Avem

t deci Qij este matricea care se obine din matricea unitate prin schimbar elemente: elementele de pe diagonala principala, de pe linia i, coloana i i s j, coloana j se nlocuiesc cu zerouri; n locul elementului de pe linia i i s avem 1, iar n locul celui de pe linia j i coloana i se gsete 1. La fel c s a s s calculm a a X n Qij A = (In Eii Ejj Eij + Eji ) akl Ekl = =
k,l=1 n X

Qij = Tij (1) Tji (1) Tij (1) = (In Eij ) (In + Eji ) (In Eij ) = = (In + Eji Eij Eii ) (In Eij ) = In + Eji Eij Eii Eij E = In Eii Ejj Eij + Eji ,

akl Ekl =

k,l=1

k,l=1

aadar, matricea Qij A se obine din A prin nlocuirea liniei i, respectiv j s t nmulita cu 1, respectiv cu linia i. Asemanator, se poate observa ca schi t care le produc asupra lui A nmulirea cu matricea Qij la dreapta sunt ur t coloana i se nlocuiete cu coloana j, iar coloana j se nlocuiete cu coloana s s cu 1. Sa mai spunem ca, fiind produs de matrice inversabile, Qij este, de matrice inversabila; avem Q1 = Tij (1) ij i, deci, s Q1 ij = Qij .
1

n X

n X

akl Eii Ekl


n X l=1

k,l=1

n X

akl Ejj Ekl


n X l=1

akl Ekl

ail Eil

ajl Ejl

k,l=1 n X l=1

k,l=1 n X

akl Eij Ekl +


n X l=1

k,l=1

n X

ajl Eil +

ail Eil ;

Tji (1)

Tij (1)

= Tij (1) Tji (1) Tij (1)

3) Un alt tip de matrice elementare sunt matricele Di (d) = In + ( 1 i n, d K fiind nenul; o astfel de matrice se obine din matric t modificndu-i un singur element: n locul lui 1 de pe linia i i coloana i pu s e greu de vzut c nmulirea unei matrice A oarecare cu Di (d) la stnga a a t la dreapta) i modific doar linia (respectiv coloana) i, anume o nlocuiete a s cu linia (respectiv coloana) i nmulita cu d. De asemenea, Di (d) este inv t 1 are inversa Di (d) = Di d1 . 4) Vom folosi matricele Pij pe care le definim prin Pij = Di (1) Qij = Qij Dj (1) = In Eii Ejj + Eij + Eji .

Verificai aceste egaliti! Observai forma matricei Pij i efectul su la t at t s a t matricea APij (respectiv Pij A) se obine din A prin interschimbarea liniilor coloanelor) i i j. Si nu n ultimul rnd, aratai ca Pij este inversabila i P s t s

3. Rezolvarea problemei. Sa demonstram aadar urmatoarea s Propoziie. Fie K un corp comutativ infinit si A Mn (K) o matr t urma este zero. Atunci exista matricele X, Y Mn (K) astfel nct A = X Demonstraie. Pentru nceput vom presupune ca nu exista nici o su t a mulimii {a11 , a22 , . . . , ann } a elementelor de pe diagonala principal a t a pentru care suma elementelor s fie nul, desigur, cu excepia ntregii mu a a t vedea imediat la ce ne folosete aceast presupunere, iar la sfrit ne vom d s a s nu este prea restrictiva). Deoarece, conform ipotezei, avem a11 + a22 + se pot determina elementele a1 , a2 , . . . , an K astfel nct

E clar ca, datorita ipotezei suplimentare pe care am facut-o, oricare d elementele a1 , a2 , . . . , an sunt distincte; vom folosi acest lucru mai departe Putem scrie pe A n forma A = B C, unde a1 a2 a12 . . . a1,n1 0 ... 0 0 a21 0 a2 ... 0 0 a3 . . . a2,n1 ... ... ... . . . , C = . . . . . . . . . ... B = ... n1,1 an1,2 . . . an1 0 0 a 0 ... an an2 . . . an,n1 an 0 0 ... 0 an1

a11 = a1 a2 , a22 = a2 a3 , . . . , an1,n1 = an1 an , ann = an

i B 0 C 0 . Folosind faptul c asemnarea matricelor este o relaie de ec s a a t obinem c B C. t a Sa le luam pe rnd. Ne amintim ca nmulirea unei matrice cu matr t stnga (respectiv la dreapta) schimba ntre ele liniile (respectiv coloanele 1 acelei matrice. De aceea, pentru M Mn (K), matricea Pij M Pij =

Aa cum am artat la nceput, pentru rezolvarea problemei ar fi s s a artm c matricele B i C sunt asemenea. n acest scop, vom arta c aa a s a a C C 0 , unde a1 a2 0 ... 0 0 ... 0 0 a2 . . . 0 a3 . . . 0 , C0 = 0 B0 = . . . . . . . . . . . . . . . . . . . . . . . . 0 0 . . . an 0 0 . . . a1

are schimbate ntre ele elementele de pe diagonala principal situate pe a coloanele) i i j; de asemenea, mai sunt afectate i celelalte elemente de p s s coloanele i i j. Aceasta nu are nsa importana n cazul unor matrice p s t sau C 0 , la care toate elementele din afara diagonalei principale sunt zero 1 Pij B 0 Pij = Pij B 0 Pij este o matrice care difer de B 0 doar prin acee a schimbat ntre ele locurile doua elemente de pe diagonala principala, an t aj . Cum orice permutare e produs de transpoziii, e clar ca dupa un n de asemenea transformri o putem aduce pe B 0 la orice form n care pe a a principal apar elementele a1 , a2 , . . . , an permutate cumva (i n rest, ze a s particular, B 0 este asemenea cu C 0 . Sa aratam acum ca B B 0 (i nu vom mai face demonstraia pentru C s t fiind ntru totul asemntoare). ncepem prin a observa urmtorul calcul a a a 1 0 0 1 0 0 0 = = , a 1 1 a ( ) + 0 daca a este ales convenabil, adica daca a = / ( ); desigur, asta se numai n cazul n care 6= . Un calcul asemntor se poate face i pentru matrice de ordin n. a a s considera Tij (a) BTij (a)1 = Tij (a) BTij (a), unde i > j, elementul aij (i, j) se nlocuiete cu a (aj ai ) + aij i a poate fi ales astfel nct acest s s s devin nul (cci am presupus c ai 6= aj ). Mai sunt afectate i celelalte el a a a liniei i (la care se adun linia j nmulit cu a) i ale coloanei j (la care a t a s coloana i nmulita cu a). Remarcam ca aceste transformari oricum nu po t zerourile de deasupra diagonalei principale, care ramn intacte, i nici elem s pe diagonala principal. a 1 Acum e clar ce avem de fcut: mai nti calculm T21 (a) BT21 (a) ca a a un a bine ales reprezinta o matrice B1 asemenea cu B care are n pozii t zero (asta daca nu era dinainte; de altfel se poate vedea uor ca, daca s atunci a care ne trebuie este a = 0 deci T21 (a) = T21 (0) este, de fapt identica). Apoi, pentru aceasta matrice calculam T31 (a) B1 T31 (a)1 , care anumit a este o matrice asemenea cu B1 (deci i cu B) i are 0 n pozii s s t poate vedea c elementul 0 obinut la pasul anterior nu va fi afectat. C a t astfel, lucrnd cu matrice de forma Ti1 (a) pn cnd toate elementele de a coloana "de sub" a1 devin zerouri, apoi trecem i facem zerouri pe coloan s "sub" a2 , folosind transformari de tip T32 (a) , . . . , Tn2 (a) (adica nmulim t la stnga i cu inversele lor la dreapta; la fiecare pas similaritatea ma s pstreaz), n ordine, alegnd, desigur, de fiecare dat valoare care trebuie a a a Elementele nule obinute pe prima coloana nu vor fi afectate, la fel cele de t deasupra diagonalei principale, Tot aa vom proceda pna cnd, la urma s la o matrice care are partea de deasupra diagonalei principale neschimb diagonala principal, iar sub diagonala principal are numai zerouri, adic a a la B 0 i la concluzia dorita ca aceasta este asemenea cu B. s n concluzie, am aratat ca matricele B i C sunt asemenea, deci am a s unde ne-am propus: exist V GLn (K) astfel nct C = V BV 1 ; at a = B C = B V BV 1 i notnd X = BV 1 , Y = V avem A = XY Y s Demonstraia ar fi ncheiata, daca n-ar mai fi un mic amanunt de la t

facem cu ipoteza suplimentar pe care am impus-o (i de care, s-a dovedit p a s avem mare nevoie, caci daca elementele de pe diagonala nu sunt distincte, n alege pe a astfel nct Tij (a) sa produca un zero n locul lui aij )? Raspunsu de greu i arat, cum spuneam, c restricia dat de aceast ipotez nu este s a a t a a a de ... restrictiv. E suficient s mprim mulimea elementelor de pe di a a at t submulimi disjuncte doua cte doua, fiecare dintre acestea avnd suma e t 0 i fiecare nemaiavnd alta submulime (stricta) pentru care suma eleme s t 0. S numim b1 , . . . , bk elementele unei asemenea submulimi (a cror s a t a aadar, zero); pentru acestea putem determina c1 , c2 , . . . , ck astfel nct b1 s b2 = c2 c3 , . . . , bk1 = ck1 ck , bk = ck c1 . Mai mult, oricare d c1 , c2 , . . . , ck sunt distincte doua cte doua i proprietaile lor se pastrea s t a nlocuim cu c1 + t, c2 + t, . . . , ck + t, t K. Gsim cte o grupare de asem distincte dou cte dou pentru fiecare submulime de b-uri a mulimii e a a t t de pe diagonala principala, iar apoi alegem cte un t pentru fiecare astfel nct toate c-urile sa fie distincte doua cte doua (ceea ce sigur se poate fa n care corpul K este infinit; gndii-v de ce!). Mai departe totul decu t a deoarece putem scrie matricea noastr ca diferena a dou matrice, una in a t a superior triunghiulara, fiecare dintre aceste elemente sunt distincte doua Propoziia este complet demonstrat. t a

Noi ne-am propus s rezolvm problema n cazul corpurilor uzuale d a a Q, R, C, de aceea ipoteza pe care am facut-o asupra infinitaii corpulu t deranjeaz foarte mult; totui, se prea poate ca aceast presupunere s a s a a legat de rezolvarea pe care am dat-o aici i s nu fie esenial. Aada a s a t a s ntrebarea daca este valabil enunul propoziiei demonstrate si n cazul t t finit. n ncheiere, s mai spunem c nu exist nici o pretenie de originalitat a a a t rarea acestei note; este foarte posibil ca aceast soluie s fie cunoscut, at a t a a autorul nu are nici un fel de referina pentru problema discutat, pe care t a doar din folclor (n urma cu civa ani aceasta problema mi-a fost comuni t viu grai" de ctre un elev, actualmente student strlucit al Facultii de M a a at din Bucureti; aa c i mulumesc pe aceast cale lui Drago Deliu, care s s a t a s s caut s rezolv aceast problem, cutri din care s-a nscut i aceast a a a a a a a s a

Recreaii matematice
1. S se ndeprteze patru segmente din gura alturat a a a a (alctuit din ase ptrate) astfel nct noua gur s e fora a s a a a mata din trei patrate. Not. Soluia problemei se poate gsi la pagina 39. a t a

Trei perle ale olimpiadelor de matematic a


Gabriel DOSPINESCU 1

Problemele propuse la testele de selecie pentru OIM sau la fazele na t t diverse ari se remarca prin profunzimea (i uneori simplitatea) ideilor care t s n cele ce urmeaz, vom rezolva trei probleme propuse la astfel de teste a n anii 2002 i 2003, demonstrnd dificultatea rezolvrii problemelor de "m s a elementara", precum i tendina accentuata de a mbina algebra, teoria nu s t analiza matematica n actul de concepere i rezolvare a unor asemenea perl s tice.

1. Un prim exemplu este urmatoarea problema propusa la unul din selecie pentru OIM n anul 2002, in Vietnam. n rezolvare vom folosi d t rezultate legate de ecuaia de gradul al doilea. Dup cum se tie, multe t a s dificile se rezolv relativ uor folosind trinomul de gradul al doilea (metoda a s Vom da doar doua exemple, fara a insista prea mult. a b c 1) Aratai ca daca numarul d = + + este ntreg, iar a, b, c su t bc ca ab naturale, atunci d este 1 sau 3. 2) Aratai ca, daca numerele naturale distincte si nenule a1 , a2 , . . . , a t a2 + a2 + + a2 = na1 a2 an , atunci ele sunt prime ntre ele doua ct 1 2 n ncercai sa rezolvai aceste doua probleme nainte de abordarea probl t t

perfect. Soluie. Vom folosi trinomul pentru a crea soluii pentru anumite ecua t t tice, deci n mod constructiv. m Y Ar fi bine s dispar a a a2 . Deci, s scriem expresia sub forma a i
i=1 i=1 m Y

PROBLEMA 1. Sa se demonstreze ca exista un numar m 2002 si m n Y X a2 4 a2 sa naturale nenule a1 , a2 , . . . , am , distincte, astfel nct i i
i=1 i=1

a2 i

Pentru a "scapa" i de 4, luam k = 2. Aadar am adus problema la o s s "acceptabila" (dar nu mai puin dificila): t Aratai ca exista m 2002 si a1 , a2 , . . . , am N distincte astfel nct t 1 + a2 + a2 + + a2 = a1 a2 am . 1 2 m

n X i=1

a2 i

m Y

i=1

ai k

!2

Sa cautam m astfel nct m 3 dintre necunoscutele ecuaiei (1) sa fie t revine la ecuaia t x2 + y 2 + z 2 + m 2 = xyz. Privind aceasta ecuaie ca una de gradul al doilea n z, vom ncerca sa t criminantul nul. Deci x2 y 2 = 4 x2 + y 2 + m 2 . Luam x = 2a, y = 2b
1

Student, Facultatea de Matematic-Informatic, Bucureti a a s

m = 4 a2 1 b2 1 2. S concluzionm: putem alege b > a > 2002 a a putem lua m = 4 a2 1 b2 1 2 > 2002. Atunci ecuaia (2) va ave t (x, y, z) = (2a, 2b, 2ab). Rezult c ecuaia (1) are soluia (2a, 2b, 2ab, 1 a a t t Dar putem scrie (1) i sub forma s 12 1 2a 2b 2ab + (2a)2 + (2b)2 + (2ab)2 + m 3 = 0.

Din relaiile lui Vite rezult c i 2a 2b 2ab 1 este soluie a ecua t a as t care n loc de 1 punem t. Aadar am redus cu o unitate numrul celor m s a am obinut o noua soluie a ecuaiei (1): (2a, 2b, 2ab, 2a 2b 2ab 1, 1, 1, t t t Analog, scriem
2 2 2

12 1 2a 2b 2ab (2a 2b 2ab 1) + (2a) + (2b) + (2ab) + Deci obinem o alta soluie a ecuaiei (1), cu numar i mai mic de 1: t t t s + (2a 2b 2ab 1) + m 4 = 0.
2

Astfel, rezulta ca putem elimina pe rnd fiecare 1 din m-upla (2a, 2b, 2ab, 1 Riguros, aceasta nseamna ca folosind succesiv relaiile lui Vite, obinem t t a upl (x1 , x2 , . . . , xk , 1, 1, . . . , 1) n care este clar c 2a = x1 < 2b = x2 < 2 a < < xk . La sfrit (cci dup cel mult m pai am eliminat toi de 1), s a a s t soluie (a1 , a2 , . . . , am ) a ecuaiei (1), n care a1 < a2 < < am . Aceas t t va satisface condiiile enunului. t t 2. Continuam cu o frumoasa problema propusa la ultima runda a poloneze n anul 2003. Simplitatea soluiei care urmeaza nu are nsa nici t cu dificultatea problemei, cci multe metode de atacare a problemei nu a un rezultat.

(2a, 2b, 2ab, 2a 2b 2ab 1, 2a 2b 2ab (2a 2b 2ab 1) 1, 1, 1, . .

PROBLEMA 2. Determinai polinoamele cu coecieni ntregi f t t etatea ca pentru orice n natural avem f (n) | 2n 1. Soluie. Evident, problema ar fi banala daca s-ar demonstra ca exista o t de numere n pentru care 2n 1 este numar prim. Dar, dupa cum vom vedea accept i soluii mai "blnde". as t Cum este clar c nu putem afla prea multe despre divizorii i factorii p a s 2n 1, vom ncerca sa lucram cu divizori ai numerelor de forma f (n). Pr care ne vine n minte, innd seama ca f are coecieni ntregi, este t t rezultatul urmtor: m n | f (m) f (n). Deci, va trebui s cut a a a a astfel nct f (m) | f (n). Dup cutri mai mult sau mai puin lungi, a a a t f (n) = n + f (n) n | f (n + f (n)) f (n). Deci f (n) | f (n + f (n)). n acest moment, jumatate din problema este rezolvata. ntr-adevar, f cu f , putem presupune c f are coeficientul dominant pozitiv. Atunc a astfel nct pentru n > M s avem f (n) N. Fixm un n > M . Avem f (n a a f (n) | f (n + f (n)) | 2n+f (n) 1 = (2n 1) 2f (n) + 2f (n) 1 (evident, n + f deci f (n) | 2f (n) 1. Daca am putea demonstra ca singurul numar natura care n | 2n 1 este 1, atunci ar rezulta c pentru n > M avem f (n) = a ar fi constanta 1. Dar faptul c n | 2n 1 implic n = 1 este binecunoscu a a de simplu. Sa presupunem ca n > 1 i sa luam p cel mai mic factor prim s

Atunci este clar c (n, p 1) = 1. Dar p | n | 2n 1 i p | 2p1 1 (t a s Fermat). Deci p | 2n 1, 2p1 1 . Se tie ca irul (xn )n1 , xn = 2n s s Mersenne (adica (xm , xn ) = x(m,n) ). Rezulta ca p | (xn , xp1 ) = x(n,p1) contradicie. Aadar n = 1 i f este constanta 1. Cum, dac f este soluie t s s a t f este soluie, deducem c polinoamele cerute sunt constantele 1 i 1. t a s

3. ncheiem scurta incursiune prin matematica elementara cu o problem de dificila, propusa la un test de selecie n Vietnam, 2002. Dificultatea t const mai ales n faptul c admite multe soluii (care nici nu se intrezresc a a t a frumuseea const n mbinarea algebrei cu analiza matematic i teoria t a as Nu exageram daca afirmam ca urmatoarea problema este una dintre cele m i frumoase probleme referitoare la polinoame, propuse la vreun concurs pe s

PROBLEMA 3. Determinai toate polinoamele p Z [X] propr t cu exista un polinom q Z [X] pentru care q 2 (X) = X 2 + 6X + 10 p2 (X) Soluie. Evident, orice t rezolvitor "srguincios" va scrie relaia din t forma q 2 (X 3) = X 2 + 1 p2 (X 3) 1 i va nota f (X) = p(X 3 s = q (X 3). Deci 2 X + 1 f 2 (X) = g 2 (X) + 1. Aici este ns punctul de oprire, cci orice ncercare ulterioar de rezolva a a a Ca de obicei, vom putea presupune ca f i g au coecienii dominani po t t s putem schimba f cu f sau g cu g, fr a se modica nimic). Deci exist aa nct pentru orice n > M s avem f (n) , g (n) N. a Apelm acum la teoria numerelor. Este binecunoscut faptul c toate a a numere naturale ale ecuaiei Pell x2 + 1 = 2y 2 sunt date de t 2n1 2n1 2n1 + 1 2 1 2 1+ 2 1+ 2 xn = , yn = 2 2 2

Rezulta ca irul de numere ntregi (2an 1 k (2n 1))n>M este conver s staionar. t Aadar, exist n0 > M astfel nct pentru n > n0 s a 2an 1 k (2n 1) = u, pentru o constant ntreag u. Ca urmare, pent a a avem 2n1 ! k(2 k(2n1)+u 2n1 + 1 2 + 1 2 1+ 2 1+ 2 = g 2 2

Ce se ntmpla daca substituim xn n (1)? Obinem g 2 (xn ) + 1 = 2 (y t Da, i perechea (g (xn ) , yn f (xn )) este soluie a ecuaiei Pell i aceasta se s t t s pentru orice n > M . Deci exist irurile (an )n>M , (bn )n>M astfel nct g (x as yn f (xn ) = ybn . Acum ncepe partea analizei matematice. Fie grad g = k , grad f = m. 2an 1k(2n1) 2xan = lim lim 1 + 2 k(2n1) = n n 1+ 2 k xn g (xn ) = 2 lim (2n1) = finit. n xk n 1+ 2

k u u 1 1 k x 1+ 2 + 1 2 x x x= g 2 2 n o 2n1 | n > n0 . Aducnd la acela pentru orice x din mulimea 1 + 2 t n (2), obinem o identitate polinomiala adevarata pentru o infinitate de t variabilei, deci (2) este adevrat pentru orice x nenul. Dup ce aducem a a a numitor i egalam coeficienii dominani n (2), deducem ca 2k1 1 + s t t unde k este coeficientul dominant al lui g. Dar aceasta implic u = 0 a pentru orice x nenul, avem k 1 1 k x + x x x= g . 2 2 Dac notm x = t + a a

Rezult c a a

t2 + 1, din (3) obinem c pentru orice t avem t a k k t + 1 + t2 + t t2 + 1 g(t) = . 2 Lum n (1) x = i i obinem c g 2 (i) = 1. Deci, folosind (4), obinem a s t a t adic k este impar. Din (4) i (1) rezult prin calcul c a s a a " k k #2 X + X2 + 1 + X2 + 1 X 2 f (X) = , k impar. 2 X2 + 1

Cum f este polinom i are coeficientul dominant pozitiv, deducem din (5) s k k X + X2 + 1 + X2 + 1 X f (X) = . 2 X2 + 1 Dar, daca f verifica (1), atunci i f verifica aceeai relaie. Mai mult, s s t din membrul drept al relaiei (6) are coeficieni ntregi. Rezulta ca exista d t t de polinoame care verific relaia (1) a t k k X + X2 + 1 + X2 + 1 X , k impar. 2 X2 + 1

n sfrit, obinem c polinoamele p cerute se obin din polinoamele (7) s t a t X cu X + 3.

Ce-ar mai fi de adugat dup prezentarea acestor trei nestemate din a a sfrit al problemelor elementare de matematic? Slefuite cu rbdarea b s a a cele trei probleme adaug o palet de lumini ncepnd cu actul creator al a a lor i terminnd cu soluiile propuse. Fiecare dintre noi are nevoie de asem s t iar aceast scurt prezentare se nscrie pe aceast linie. a a a

n legtur cu o problem de concurs a a a


Dan Stefan MARINESCU 1

La etapa finala a Olimpiadei de matematica din anul 1989 prof. univ. dr panu a propus urmatoarea problema: Z b atunci pentru fiecare n N exista n numere distincte Z b ca n (b a) f (x) dx = . 1 1 1 a + + + f (x1 ) f (x2 ) f (xn ) Daca f : [a, b] R este o funcie integrabila, continua pe (a, b) si t
a x1 , x2 , . . . , xn

f(

f (x) f (0) ; evident h est f (1) f (0) pe [0, 1], derivabila pe (0, 1) i h (0) = 0, h (1) = 1. s Pentru orice k {1, 2, . . . , n 1} consideram funcia continua hk : [ t k P hk (x) = h (x) i . Cum h1 (0) = 1 < 0, h1 (1) = h (1) 1 = 1 Demonstraie. Fie h : [0, 1] R, h (x) = t
i=1

(enu Enunul i o soluie a problemei pot fi aflate n [3]. n cele ce urm t s t prezenta o generalizare a acestei frumoase probleme. Pentru ceea ce ne-am propus, avem nevoie de Propoziia 1. Fie f, g : [0, 1] R doua funcii cu urmatoarele propri t t i) f , g continue pe [0, 1], ii) f , g derivabile pe (0, 1), iii) f (1) 6= f (0) si g 0 (x) 6= 0, x (0, 1). Atunci pentru orice n N si orice 1 , 2 , . . . , n > 0 cu 1 + 2 + exista x1 , x2 , . . . , xn (0, 1) cu x1 < x2 < < xn astfel nct n X g 0 (xi ) g (1) g (0) i 0 = . f (xi ) f (1) f (0) i=1

conchidem, din continuitatea funciei h1 , ca exista c1 (0, 1) cu h1 (c t h (c1 ) = 1 . Analog, h2 (c1 ) = h (c1 ) 1 2 = 2 < 0, h2 (1) = h (1) = 1 1 2 > 0, de unde acelai raionament conduce la existena unui c s t t astfel nct h2 (c2 ) = 0 h (c2 ) = 1 + 2 . Inductiv, gasim 0 < c1 < c < cn1 < 1 astfel nct h (c1 ) = 1 , h (c2 ) = 1 + 2 , ...

, h (cn1 ) = 1 + 2 + . . . + n

s t s Fie c0 = 0 i cn = 1, atunci pentru orice k {1, 2, . . . , n} funciile h condiiile din teorema lui Cauchy pe intervalul [ck1 , ck ]; ca urmare, de t exista x1 (c0 , c1 ), x2 (c1 , c2 ), . . . , xn (cn1 , cn ) astfel nct 0 h (x1 ) h (c1 ) h (c0 ) h0 (x2 ) h (c2 ) h (c1 ) h0 (xn ) h (cn ) = , 0 = , ..., 0 = 0 (x ) g 1 g (c1 ) g (c0 ) g (x2 ) g (c2 ) g (c1 ) g (xn ) g (cn ) de unde, mpreun cu (3), avem: a
1

Profesor, Liceul Teoretic "Iancu de Hunedoara", Hunedoara

h0 (x1 ) 2 h0 (x2 ) n 1 = 0 , = 0 , ..., g (c1 ) g (c0 ) g (x1 ) g (c2 ) g (c1 ) g (x2 ) g (cn ) g (cn1 ) ceea ce, innd seama de faptul ca 1 , 2 , . . . , n > 0, conduce la relaiile t t g 0 (x1 ) g 0 (x2 ) g 0 (xn ) 1 0 = g (c1 )g (c0 ) , 2 0 = g (c2 )g (c1 ) , . . . , n 0 = g( h (x1 ) h (x2 ) h (xn ) n X g 0 (xi ) De aici, prin adunare, obinem t i 0 = g (cn ) g (c0 ) = g (1) g h (xi ) i=1 0 f (x) h0 (x) = , x (0, 1), conchidem c are loc relaia (2). a t f (1) f (0) Corolarul 1 [1]. Daca f : [0, 1] R este o funcie continua pe [0, 1], t pe (0, 1), f (0) = 0, f (1) = 1, atunci pentru orice n N si orice k1 , k2 , . n X ki exista x1 , x2 , . . . , xn (0, 1) distincte doua cte doua astfel nct f 0 (x i=1 (vezi i [2], [4]). s n .X Demonstraie. Consideram n Propoziia 1, i = ki t t ki , pe

a n > 0 cu 1 + 2 + + n = 1, exist n numere distincte x1 , x2 , . . . , astfel ca Z b n X g (xi ) Z b i g (x) dx f (x) dx. = f (xi ) a a i=1 Z x Demonstraie. Fie f1 , g1 : [0, 1] R, f1 (x) = t f ((1 t) a + tb) d Z x 0 g ((1 t) a + tb) dt. n mod evident, f1 i g1 sunt bine denite, veric a s = Z b Z 0 1 1 g (x) dx, f1 (1)f1 (0) = din Propoziia 1 i g1 (1)g1 (0) = t s ba a ba De unde exist c1 < c2 < < cn din (0, 1) astfel nct a Rb Rb n n X g 0 (ci ) X g ((1 ci ) a + ci b) g (x) dx g (x) 1 a i 0 i = Rb = Ra b f1 (ci ) f ((1 ci ) a + ci b) f (x) dx f (x) i=1 i=1 a a a Lund xi = (1 ci ) a + ci b, i = 1, n, evident c x1 < x2 < < xn formula (4). Observaie. Egalitatea (1) se obine lund n (4) 1 = 2 = = t t g (x) = 1, x [a, b].

i=1 i {1, 2, . . . , n} i g : [0, 1] R, g (x) = x. s Corolarul 2. Fie f, g : [a, b] R dou funcii integrabile, continue a t Z b f (x) dx 6= 0 i g (x) 6= 0, x (a, b). Atunci, pentru fiecare n N i s s a

Bibliografie 1. G. G. Z. Giang - Problema 1125, Math. Mag. 2. P. Orno - Problema 1053, Math. Mag. 3. I. Tomescu (coordonator) - Probleme date la olimpiadele de matemat licee (19501990), Ed. tiinifica, Bucureti, 1992. s t s 4. *** - Problema C:1791, G. M. 3/1996.

Asupra unei probleme propuse la O. I. M. Neculai ROMAN 1


La O. I. M. n anul 1982 a fost propus problema B3 GB: a Fie ABC un triunghi si P un punct n interiorul lui astfel ca P AC Fie L, M picioarele perpendicularelor din P pe BC, CA respectiv. Fie lui [AB]. Sa se demonstreze ca DL = DM .

Enunul i o soluie a acestei probleme se poate gsi n [1], pag. 322 i t s t a s blema are i o soluie mai simpl, accesibil i elevului de gimnaziu i c s t a as s a fi cunoscuta. De asemenea, vom arata ca problema are loc pentru o mu variat de puncte din planul triunghiului. n acest scop, vom demonstra u a

Teorem. Fie ABC un triunghi, D mijlocul lui [AB] si punctele a 0 dreptele AC, respectiv BC astfel ca C (AA ) si C (BB 0 ). Fie P n planul triunghiului si L, M picioarele perpendicularelor din P pe B tiv AC. Sa se demonstreze ca daca P Int (ACB) Int (A0 CB 0 P AC P BC sau daca P Int (BCA0 ) Int (ACB 0 ) astfel ca m ( +m (P BC) = 180 atunci DM = DL. Demonstraie. Fie punctele D0 i D00 mijloacele segmentelor [P A], t s [P B] (fig. 1, 2 i 3). s
A
A
A

D M

D D

D B L

P CB
C D M A

L B L P
P D

Fig. 1 Avem M D0 =

Fig. 2

Fig. 3

PA a = DD00 , ([M D0 ] mediana corespunztoare ipo 2 00 4AM P i [DD ] linie mijlocie n 4AP B). s Deci [M D0 ] [DD00 ] . Analog, [LD00 ] [DD0 ] . 0 00 Din P D DD paralelogram, rezulta ca 0 P D D P D00 D
1

Profesor, Scoala "V. Alecsandri", Mirceti, Iai s s

Din P Int (ACB) Int (A0 CB 0 ) astfel ca P AC P BC (fig. 1 i 2 s ca P D0 M P D00 L (teorema unghiului exterior). Daca P Int (BCA0 ) astfel ca m (P AC) + m (P BC) = 180 (fig. P AC P BL i deci P D0 M P D00 L (teorema unghiului exteri s P Int (ACB 0 ) astfel ca m (P AC)+m (P BC) = 180 , atunci P BC i deci P D0 M P D00 L. s n concluzie, P D0 M P D00 L. Din relaiile (3) i (4) rezult c t s a a M D0 D LD00 D.

Acum din relaiile (1), (2) i (5) rezulta ca 4M D0 D 4DD00 L, de und t s [DM ] [DL] i deci DM = DL. s

Teorema reciproc. Fie ABC un triunghi, D mijlocul lui [AB] si pu a B 0 pe dreptele AC respectiv BC astfel ca C (AA0 ) si C (BB 0 ). Fie p M pe dreptele BC respectiv AC astfel ca DM = DL. Perpendicularele n AC respectiv BC se ntlnesc n P . Sa se demonstreze afirmaiile: t a) daca P Int (ACB) Int (A0 CB 0 ), atunci P AC P BC; b) daca P Int (ACB 0 )Int (BCA0 ), atunci m (P AC)+m (P BC Demonstraie. a) Fie punctele D0 i D00 mijloacele segmentelor [P A t s [P B]. Se arata uor ca 4M D0 D 4DD00 L, de unde rezulta ca s Din P D0 DD00 paralelogram, rezult a Din relaiile (6) i (7) obinem t s t M D0 D DD00 L. P D0 D P D00 D. P D0 M P D00 L.

a) Daca P Int (ACB)Int (A0 CB 0 ), atunci din (8) rezulta P AC (fig.1 i 2). s b) Dac P Int (BCA0 ) (fig. 3), atunci din relaia (8) rezult c P AC a t a a i, deci, m (P AC) + m (P BC) = 180 . s Dac P Int (ACB 0 ), atunci din relaia (8) rezult: P BC P a t a m (P AC) + m (P BC) = 180 .

Bibliografie 1. I. Cuculescu - Olimpiadele internaionale de matematica ale el t Tehnic, Bucureti, 1984. a s

Asupra unei ecuaii funcionale t t


Loredana AGORE 1
Scopul acestei note este rezolvarea ecuaiei funcionale t t n mulimea funciilor f : R R sau f : R R sau f : R R. t t + n ecuaia (1) este cuprins ecuaia lui Cauchy t a t f (x + y) = f (x) + f (y) ,

f (axy + x + y) = bf (x) f (y) + c [f (x) + f (y)] + d (a, b, c, d R

ale crei soluii se numesc funcii aditive, precum i urmtoarele ecuaii a t t s a t sunt reductibile la ecuaia lui Cauchy: t f (x + y) = f (x) f (y) , f (xy) = f (x) + f (y) , f (xy) = f (x) f (y) .

Rezolvarea n detaliu a ecuaiilor (2) (5) se poate gasi n [1]. Tot n [1], t studiat i ecuaia funcional obinut considernd n (1) a 0, c = 1, d as t t a t a Rezolvarea ecuaiei funcionale (1) const n reducerea ei, potrivit cu t t a a, b, c i d, la una dintre ecuaiile (2) (5). Distingem cteva cazuri. s t

I a 6= 0, b 6= 0. nmulim ecuaia (1) cu b i apoi punem rezultatul o t t s forma (ax + 1) 1 (ax + 1) (ay + 1) 1 + c = bf +c bf a a (ay + 1) 1 bf + c + bd + c a t1 2 s Cu notaiile = bd + c c , u = ax + 1, v = ay + 1 i g(t) = bf t a ecuaia (6) se scrie t g (uv) = g (u) g (v) + . Dac bd = c2 c, adic = 0, atunci (7) este de tipul (5) i se reduce a a s lui Cauchy [1]. Soluiile se exprim cu funciile aditive sau sunt funcii co t a t t Daca bd 6= c2 c, deci 6= 0, luam = v = 1 n (7) i obinem g (1) = [ s t u q 1 1 De aici, g (1) = t 1 1 4 = 1 (1 2c)2 4bd , cu condiia 2 2 1 bd c2 c + . Pe de alt parte, ecuaia (7) cu v = 1 devine a t 4 g (u) = g (1) g (u) = g (u) g (1) + g (u) = 1 g (1) (evident, g (1) 6= 1). Ca urmare,ecuaia (1) are, n cazul considerat, d t q 1 1 date de f (x) = [g (1) c] = (1 2c) (1 2c)2 4bd . b 2b
1

Elev, cl. a XI-a, Colegiul Na ional "Mihai Viteazul", Bucureti a t s

II a = 0, b 6= 0. Ecuaia (1) devine t i se poate scrie n forma s

f (x + y) = bf (x) f (y) + c [f (x) + f (y)] + d

sau, notnd = bd + c c2 i g (t) = bf (t) + c, t R, s

bf (x + y) + c = [bf (x) + c] [bf (y) + c] + bd + c c2 g (x + y) = g (x) g (y) + .

Dac bd = c2 c, atunci (10) este de tipul (3) etc. a Daca bd 6= c2 n (8) luam x = y = 0 i obinem f (0) = b [f (0)]2 +2cf c, s t q 1 2 deci f (0) = (1 2c) (1 2c) 4bd (n mod necesar, (1 2c)2 2b 1 4 0). Dar, daca n (8) luam y = 0 i apoi grupam convenabil, a s [1 bf (0) c] f (x) = cf (0) + d [1 bf (0) c] f (x) = [1 bf (0) c] f (0) f (x) = f (0) (1 bf (0) c 6= 0 bd 6= c2 c). Avem dou soluii: a t q 1 2 f (x) = f (0) = (1 2c) (1 2c) 4bd . 2b III a 6= 0, b = 0. n acest caz, (1) se scrie f (axy + x + y) = c [f (x) + f (y)] + d.
() ()

Dac c = 1, punem (11) n forma a (ax + 1) (ay + 1) 1 (ax + 1) 1 (ay + 1) f +d = f +d + f a a a t 1 + d, t R, sau, notnd u = ax + 1, v = ay + 1 i g (t) = f s a g (uv) = g (u) + g (v) , care este o ecuaie de tipul (4). t Dac c 6= 1, pentru x = y = 0 luat n (11) obinem a t Cum, pentru y = 0 n (11), avem

f (0) = 2cf (0) + d (1 2c) f (0) = d.


(12)

f (x) = cf (x) + cf (0) + d f (x) = cf (x) + f (0) cf (0) d (12) (1 c) f (x) = (1 c) f (0) f (x) = f (0) f (x) = 1 2c 1 dac mai presupunem n plus c 6= . Este banal verificarea faptului c, n a a a 2 d impuse, f (x) = este soluie a ecuaiei (11). t t 1 2c 1 Daca c = , avem de rezolvat ecuaia t 2 1 f (axy + x + y) = [f (x) + f (y)] + d. 2

IV a = 0, b = 0. Este vorba de ecuaia t f (x + y) = c [f (x) + f (y)] + d. Daca c = 1, (14) se poate scrie f (x + y) + d = [f (x) + d] + [f (x) + d] , care este o ecuaie Cauchy n g (t) = f (t) + d, t R. t Dac c 6= 1, lum x = y = 0 n (14) i obinem, ca i n cazul a a s t s relaiile echivalente (12). Se continua tot ca n cazul amintit i se ob t s t 1 d , x R, dac c 6= . a f (x) = f (0) = 1 2c 2 1 Dac c = , (14) se scrie a 2 1 f (x + y) = [f (x) + f (y)] + d. 2 Lund x = y = 0, constatm c d = 0. Punnd n (15) d = 0 i fix a a s 1 obinem f (x) = [f (x) + f (0)]. Deci f (x) = f (0) = constant, x R, t 2 este soluie a ecuaiei (15). t t

1 [f (0) + f (0)] + d, deci d = 0. Lu 2 1 1 1 f (x) + f , de un y = n (13) cu d = 0, vom avea f (0) = a 2 a ca f (x) = k (constant), x R. Se verifica uor ca aceasta funcie este t s soluie pentru orice k R. t Pentru x = y = 0, obinem f (0) = t

Bibliografie 1. V. Pop - Ecuaii funcionale. Ecuaii clasice si probleme, Ed. Mediam t t t Napoca, 2002.

Recreaii matematice

2. Un calator, care nu avea la el dect un lan cu apte verigi de aur, t s ntr-o zi la un han. El se nelege cu hangiul sa-l plateasca pentru fiecare zi t la han cte o verig de aur. Dac st apte zile i plata trebuie fcut n a a as s a a care este numrul minim de tieturi care trebuie fcute n lan pentru a p a a a t preul convenit? (Se accepta ca, atunci cnd este cazul, hangiul sa dea c t ca rest un numar de verigi (posibil toate!) pe care le-a primit deja.) 3. Care este eroarea n "demonstraia" de mai jos a egalitaii 3 = 0? t t 2 3 x x+1=0 | x x (1) = 0 x3 x2 + x = 0 x3 = 1 2 3 x = 1 x x x =0 Punnd x = 1 n x2 x + 1 = 0, obinem 3 = 0. t Not. Soluiile problemelor 2 i 3 se pot gsi la pagina 39. a t s a

Asupra unei inegaliti condiionate at t


Cezar LUPU 1

La OBM - 2001 a fost dat problema urmtoare: a a Fie a, b, c numere reale strict pozitive astfel nct a + b + c abc. Sa s a2 + b2 + c2 abc 3. Cristinel Mortici, Soluia autorului utilizeaza metoda reducerii la absurd. Presupunem t inegalitatea contrar, adic a2 + b2 + c2 < abc 3. Aplicnd inegalitate a a 1 1 2 2 Schwarz, obinem abc 3 > a2 +b2 +c2 (a + b + c) (abc) , de unde a t 3 3 Pe de alt parte, aplicnd a inegalitatea mediilor, avem abc 3 > a2 + 3 2 b2 c2 i, deci, abc > 3 3. Se obine astfel o contradicie. 3 a t t s Alte soluii ale acestei probleme sunt prezentate n [1] i [2]. t s 1. Problema de mai sus poate fi ntrit astfel: a a Problema 1. Sa se arate ca, daca a, b, c > 0 si a + b + c > a ab + bc + ca abc 3. 1 1 Soluia I. Ipoteza i concluzia se pot scrie n felul urmtor: t s a + + ab bc 1 1 1 2 a + + 3. Utilizm binecunoscuta inegalitate (x + y + z) 3 (xy + a b c 2 1 1 1 1 1 1 x, y, z R (1) pentru a obine t 3 + + + + , din c a b c ab bc ac 1 1 1 ca + + 3. a b c Soluia II. Inegalitatea ceruta rezulta direct din inegalitatea ab + t p 3abc (a + b + c), a, b, c R+ (2) (aceasta se reduce la (1), daca nota y = bc i z = ca). s Problema 2. Se considera a, b, c > 0 astfel nct a + b + c abc. Sa s bc 3 ca ab + + . Ce a2 (b + c) b2 (c + a) c2 (a + b) 2 X Soluie. Pentru prescurtarea scrierii folosim nsumarea ciclica t . A 2 X (bc)2 X 1 1 (bc + ca + ab) ab + bc + bc = = a2 (b + c) abc a (b + c) abc 2 (bc + ca + ab) 2abc X (bc)2 2 (s-a folosit (bc + ca + ab) 2 (bc + ca + ab) , adevrat confo a a a (b + c) litaii Cauchy-Schwarz). t 2. Avnd ca punct de plecare inegalitatea condiionata data la OB t obine inegaliti geometrice ntr-un triunghi. S observm mai nti c a t at a a a Daca a, b, c sunt lungimile laturilor unui triunghi oarecare nscris ntr raza egala cu unitatea, atunci a + b + c abc. n [2] sunt date patru demonstraii. Reproducem una dintre ele. t abc 4RS = abc i S = pr conduc la relaia s t = 2Rr. Utiliznd in a+b+c lui Euler i faptul c R = 1, obinem inegalitatea dorit. s a t a
1

Elev, cl. a X-a, Colegiul Na ional "Mircea cel Btrn", Constan a t a t

Problema 5. n orice triunghi nscris ntr-un cerc de raza 1 are loc u a b c 3 inegalitate: + + . Ce bc (b + c) ca (c + a) ab (a + b) 2 (a + b + c) Soluie. Utiliznd rezultatul din Problema 3 i inegalitatea Cauchy t s X (a bc)2 (a bc + b ca + c ab)2 3a2 b2 c2 , de putem scrie: 2 (a + b + c) 2 (a + b + c) b+c 2 2 2 t a mprire cu a b c , obinem inegalitatea dorit. at 3. Propunem spre rezolvare urmatoarele probleme: 1. Se consider a, b, c trei numere reale strict pozitive astfel nct a + b a b2 c2 3 3 a2 + + . Cezar Lu Aratai ca t bc (b + c) ca (c + a) ab (a + b) 2 2. Fie ABC un triunghi oarecare nscris ntr-un cerc de raza egala cu 3 AB BC CA, P Int (ABC). Ce c P A + P B + P C a 3 Bibliografie 1. M. Blun i M. Becheanu (prezentare de) - A 18-a OBM, 3-9 mai 2001 a as GM - 5-6/2001, 229-236. 2. Cezar Lupu - Asupra unei probleme de concurs, Rev. Mate(matic), 200 a

Cu inegalitatea Cauchy-Schwarz sau utiliznd inegalitatea (2) de mai su X 1 9 9 9 X p . Este sucient ca p bc 3 (ab + bc + ca) 3 (ab + bc + bc sau, echivalent, ab + bc + ca 9R2 . Aceasta decurge din ab + bc + ca a i faptul cunoscut c ntr-un triunghi are loc a2 + b2 + c2 9R2 . s a Problema 4. Sa se arate ca n orice triunghi nscris ntr-un cerc de a 1 1 1 b c cu 1 are loc inegalitatea + + + 32 + + . Ce bc ca ab a b c Soluie. Este binecunoscut inegalitatea a2 + b2 + c2 4S 3 + t a 2 2 + c) +(c a) (Finsler i Hadwiger, 1938). Este echivalenta cu 2 (ab + (b s a2 + b2 + c2 4S 3 i innd seama c R = 1, conduce la inegalitate s t a

Ca urmare, n condiia impus triunghiului, are loc i in t a s a2 + b2 + c2 abc 3. De altfel, aceasta din urma rezulta direct din inegalitate a2 + b2 + c2 4S 3 (Weitzenbck, 1919) pentru R = 1. O ntrire a acestor inegaliti este dat de a at a Problema 3. orice triunghi este satisfacuta inegalitatea n b c ab + bc + ca bc + ca + ab 4S 3. n particular, daca R = a ab + bc + ca a bc + b ca + c ab abc 3. X X Soluie. Prima parte a dublei inegaliti se dovedete astfel: t at s ab 1 1 1 1 1 1 + + + + A2 + B 2 + C 2 AB + BC + CA a b c ca ab bc 1 1 1 s A = , B = i C = ), care este adevarata. a c b X X Demonstrm acum partea a doua, adic a a a bc 4S 3 sau

O metod de demonstrare a concurenei unor a t


Gabriel POPA, Paul GEORGESCU 1

Vom exemplifica n cele ce urmeaza aplicabilitatea unei metode de dem concurenei unor drepte, prea puin utilizata n contextul introducerii noii t t colare de geometrie. s Date dou puncte A, B avnd vectorii de poziie rA i respectiv rB , v a t s poziie al unui punct al dreptei AB este de forma t rM = rA + (1 ) rB , R

(ecuaia vectoriala a dreptei AB). Avnd o dreapt ataat unui triunghi, v t a s a poziie al unui punct curent M al su poate fi exprimat funcie de vectorii t a t ai vrfurilor i de un parametru real . Considernd nca o dreapta (cu p s notat ), pentru a afla punctul comun celor doua drepte vom avea de r sistem liniar n i . s Dac dorim s probm concurena a trei drepte, le vom intersecta a a a t doua i vom urmari daca vectorii de poziie ai punctelor obinute coincid t t s presupune, n general, un important volum de calcule, nsa este "sigura" n plus faa de alte metode, poziionarea punctului de concurena. t t t

Problema 1. Fie ABC un triunghi si M, N (BC), P, Q (AC), R, puncte astfel nct BM = CN = CP = AQ = AR = BS = x, unde < min {AB, BC, CA}. Daca A1 , B1 , C1 sunt respectiv mijloacele segment (RN ), (M Q), aratai ca dreptele AA1 , BB1 , CC1 sunt concurente. t Constant SB Soluie. Punctul S mparte segmentul orientat BA n raportul t SA atunci cx x x cx rS = rB + rA = rB + rA . c cx c c bx x s Analog, rP = rC + rA i atunci b b 1 x (b + c) cx bx rA + rB + rC . rA1 = (rS + rP ) = 2 2bc 2c 2b Vectorul de poziie al unui punct curent X al dreptei AA1 va fi t x (b + c) (c x) (b rX = rA1 + (1 ) rA = + (1 ) rA + rB + 2bc 2c unde R. Cu totul analog, vectorul de poziie al unui punct curent Y t BB1 va fi (c x) x (a + c) (a x) rY = rA + + (1 ) rB + rC . 2c 2ac 2a
1

Profesori, Colegiul Na ional i Liceul de Informatic "Gr. Moisil", Iai t s a s

Intersecia celor dou drepte se obine rezolvnd sistemul t a t x (b + c) + (1 ) = (c x) 2bc 2c (c x) x (a + c) = + (1 ) 2c 2ac (b x) (a x) = . 2b 2a Sistemul este compatibil determinat, cu soluia t 2bc (a x) ; = 2 x (a + b + c) 2x (ab + bc + ac) + 3abc 2ac (b x) = 2 . x (a + b + c) 2x (ab + bc + ac) + 3abc s Punctul comun al dreptelor AA1 i BB1 este T , unde 1 rT = 2 [a (b x) (c x) rA x (a + b + c) 2x (ab + bc + ac) + 3abc + b (a x) (c x) rB + c (a x) (b x) rC ] .

Scriind acum ecuaia vectorial a dreptei CC1 i aflnd intersecia acestei t a s t obinem acelai punct T . Urmeaza ca AA1 , BB1 , CC1 sunt concurente. t s

Observaii. t 1) Calcule foarte asemanatoare rezolva problema L.25.a) din R. M. autor Constantin Cocea. Legat de punctul b) al acestei probleme, c notele aprute n R. M. T. numerele 2/1991 i 1/1996, putem observa c a s calculul vectorial ajut la simplificarea soluiilor (a se vedea i [4]). a t s 2) n [6] se demonstreaza concurena nalimilor i bisectoarelor unu t s t folosind aceasta metoda; aceste demonstraii au constituit punctul de ple t ticolului de faa. t 3) Calculele pot fi simplificate atunci cnd, din considerente geometri anumite simetrii verificate de punctul de concurena. t

Problema 2. Fie H ortocentrul 4ABC, M , N si P mijloacele latur [CA] respectiv [AB], iar A1 (AH), B1 (BH), C1 (CH) a AA1 BB1 CC1 = = . Sa se arate ca dreptele A1 M , B1 N si C1 P sunt c A1 H B1 H C1 H Gabriel Popa, Paul G Soluie. Raportam planul la un reper cu originea n centrul cercului c t A1 H t a = triunghiului i fie rA , rB , rC vectorii de poziie ai vrfurilor. Dac s AA1 1 rH = rA + rB + rC , rM = (rB + rC ) , 2 1 k 1 1 rA1 = rH + rA = rA + rB + rC . 1+k 1+k 1+k 1+k A1 Q Cautam un punct Q (A1 M ) astfel nct = l, iar rQ sa se exprim QM

funcie de rA , rB i rC : t s 1 l 1 l l 1 1 rA1 + rM = rA + + rB + + rQ = 1+l 1+l 1+l 2 1+k 2 1+k l 1+k 1 2k pentru + =1l= , obinem ca rQ = t (rA + rB + rC 2 1+k 1+k 1 + 3k 0 00 s t cautam Q (B1 N ) i Q (C1 P ) care sa se exprime simetric funcie de a a rC ; vom gsi c Q0 = Q00 = Q, deci cele trei drepte sunt concurente.

Problema 3. Laturile (AB), (BC), (AC) ale triunghiului ABC sun cercului nscris de centru I n punctele C1 , A1 respectiv B1 . Daca B2 es laturii (AC), demonstrai ca dreptele B1 I, A1 C1 si BB2 sunt concurente t Olimpiad Rep. a Soluie. Funcie de vectorii de poziie ai vrt t t furilor 4ABC, vectorii de poziie ai punctelor care t A apar n problem sunt: a B1 a+bc a+cb rA1 = rB + rC ; C1 B 2a 2a I b+ac b+ca rB1 = rA + rC ; 2b 2b c+ab c+ba rC1 = rA + rB ; B A1 2c 2c 1 a b c rB2 = (rA + rC ) ; rI = rA + rB + r 2 a+b+c a+b+c a+b+c Fie X un punct pe IB1 ; atunci (b + a c) (1 ) a + rA + rX = rB1 + (1 ) rI = 2b a+b+c (1 ) b (b + c a) (1 ) c + rB + + rC , R. a+b+c 2b a+b+c Cautam o valoare a lui pentru care rX sa aiba o exprimare simetrica n deci (b + a c) (1 ) a (b + c a) (1 ) c b + = + = 2b a+b+c 2b a+b+c a+ pentru aceasta valoare a lui , a+cb b a+cb rX = rA + rB + rC . 2 (a + c) a+c 2 (a + c) Fie acum Y punct pe A1 C1 ; atunci c+ab rY = rA1 + (1 ) rC1 = (1 ) rA + 2c (a + c b) (a + b c) (1 ) (c + b a) + + rB + rC 2a 2c 2a Cautnd o valoare pentru astfel nct rY sa aiba o exprimare simetrica a i, pentru aceast valoare, obinem = t s a a+c a+cb b a+cb rA + rB + rC , rY = 2 (a + c) a+c 2 (a + c)

adic Y = X. S observm n final c, datorit simetriei n rA i rC , aces a a a a a s afla i pe mediana BB2 . s

Probleme propuse 1. Fie GA , GB , GC , GD centrele de greutete ale feelor tetraedrului A t M un punct interior tetraedrului. Daca A0 , B 0 , C 0 , D0 sunt situate re semidreptele (M GA , (M GB , (M GC , (M GD , n exteriorul tetraedrului, a M GA M GB M GC M GD = = = , s se arate c dreptele AA0 , BB 0 , a a GA A0 GB B 0 GC C 0 GD D0 sunt concurente. Gabriel Popa, Paul G 2. Fie ABC un triunghi nscris n cercul C, A1 , B1 , C1 punctele de pe C opuse vrfurilor, iar GA , GB , GC centrele de greutate ale triunghiurilor A1 B respectiv C1 AB. Aratai ca dreptele AGA , BGB , CGC sunt concurente nt t situat pe dreapta lui Euler a 4ABC. Gabriel Popa, Paul G

3. Fie M n interiorul 4ABC. Bisectoarele interioare ale unghiuri \ \ CM A, AM B taie laturile [BC], [CA], respectiv [AB] n A1 , B1 , respectiv arate c AA1 , BB1 i CC1 sunt concurente. a s Gheorgh 4. Fie D, E, F punctele de tangena ale cercului nscris n 4ABC t [BC], [CA], respectiv [AB]. Paralela prin E la AB taie F D n Q, iar pa D la AB taie EF n T . S se arate c dreptele CF , DE i T Q sunt conc a a s Marce 5. Fie tetraedrul ABCD i punctele M (AB), N (CD), P (BC), s AM DN BP AQ astfel nct = , = . Notam {A1 } = BN DP , {B1 } = MB NC PC QD {C1 } = BQ DM , {D1 } = AP CM . Sa se arate ca dreptele AA1 , BB DD1 sunt concurente. Bibliografie

1. C. Cocea - Problema L.25, R. M. T. - 2/1990. 2. C. Cocea - Problema X.8, R. M. T. - 1/1996. 3. P. Georgescu, G. Popa - Structuri fundamentale n algebra liniara, geom toriala si geometria analitica, Ed. MatrixRom, Bucureti, 2003. s 4. G. Popa - Aplicaii ale dimensiunii dreptei vectoriale, planului vectorial si t vectorial, Matematica pentru elevi, Galai, 17-18/2001. t 5. G. Popa, P. Georgescu - Dreapta lui Euler privita ca loc geometric, Rec tematice - 2/2002. 6. E. Murgulescu, N. Donciu - Culegere de probleme de geometrie analiti reniala (vol. I), E. D. P., 1971. t 7. *** - A 46-a Olimpiada de Matematica a Rep. Moldova, R. M. T - 3/2

Teorema ariciului i cteva aplicaii s t


Dumitru MIHALACHE 1

n aceasta not ne propunem s prezentm un rezultat mai puin ve a a a t literatura matematic romneasc din ultimii ani, precum i o aplicai a a s t neateptata a sa; credem ca cititorii interesai vor gasi destule alte prob s t sa-l foloseasca. Vom prezenta teorema ariciului n mod gradat (denu justicat de aspectul conguraiilor ce vor aprea); n plan mai nti pentr a t a i apoi pentru poligon oarecare, iar n spaiu pentru tetraedru i pe urm s t s a poliedru arbitrar, cu demonstraii ntre care exista analogii. t Propoziia 1. Fie ABC un triunghi si i, j, k versori perpendiculari t BC, CA, respectiv AB, ndreptai spre exteriorul triunghiului. Cu notai t t are loc egalitatea ai + bj + ck = 0. Demonstaia I. Notam S = ai + bj + ck; avem ca S ai = a2 + ab i j t b Deoarece unghiul dintre i i j are masura 180 m(C), obinem ca i j = s t procednd la fel, analoagele. Folosind identitatea b cos C + c cos B = a, g a

Similar, S bj = 0, prin urmare S este ortogonal pe doi vectori necoliniari, d Demonstraia II. Construim, ca n gur, t a F reprezentani cu originea n A ai vectorilor bj i ck, t s fie acetia AE, respectiv AD; fie nca F al patrulea s D vrf al paralelogramului construit pe aceti vectori. Se s observ atunci c 4ABC 4EF A (L.U.L.), deci a a \ AF = BC = a i F AE \ De aici, m(M AC) = s [ ACB. ck [ m(EAF ) = 90 m(ACB), [ \ = 180 m(CAE) A \ adic m(AM C) = 90 , unde {M } = AF BC. a Urmeaza ca AF este ortogonal pe BC, are lungimea a i sens opus lui i, deci AF = ai. Pe de alt parte, s a AF = AE + AD = bj + ck, de unde concluzia. Sa observam ca putem considera ca teorema ariciu- B M lui a fost demonstrata cu prima metoda (sau cu alta, vom vedea c mai exist); atunci relaia ai + bj + ck = 0 conduce, avnd a a t figura de mai sus, la AF = ai, i.e. AF BC, AF = BC, plus o condi sensul. Prin urmare, putem afirma c, din punct de vedere logic, teorem a pentru triunghi este echivalent cu urmtorul enun (pb. 45, pg. 49 din [2 a a t Problema 1. Se considera 4ABC pe ale carui laturi [AB] si [AC] s iesc n exterior patratele ABGD si ACKE. Daca O este mijlocul lui D AO = BC/2 si AO BC. Altfel spus, mediana din A n 4ADE este nlime n 4ABC; atenie c at t \ Si nca un amanunt: nu trebuie ignorat cazul n care unul din unghiurile A este drept sau obtuz.
1

S ai = a2 ab cos C ac cos B = a (a b cos C c cos B) = 0.

Profesor, Colegiul Na ional "Gh. Roca Codreanu", Brlad t s

Propoziia 2. Fie A1 A2 . . . An un poligon cu laturile de lungimi A t A2 A3 = a2 , . . . , An A1 = an . Pentru fiecare k = 1, n, pe latura de lung construiete un versor ik orientat spre exteriorul poligonului; s a1 i1 + a2 i2 + + an in = 0. Demonstraie. S remarcm c, dei poate s t a a a s a a4 i4 nu fie convex, se subnelege ca poligonul nu trebuie t sa aiba autointersecii; va convingei uor ca pentru o t t s A4 "fundia" format cu dou laturi opuse ale unui drept a a tunghi i cu diagonalele sale, proprietatea nu are loc s A5 (asta daca reuii sa stabilii care este interiorul i care st t s este exteriorul ei!). mprim poligonul n triunghiuri cu interioarele at disjuncte, prin diagonale care nu se intersecteaz. a5 i5 A1 a Aplicam apoi Propoziia 1 fiecarui triunghi, nsumam t relaiile obinute i concluzia urmeaza daca inem t t s a1 i t seama de faptul c pe laturile comune pentru cte dou a a triunghiuri (diagonale ale poligonului!) sunt construii cte doi vectori cu s t Bineneles, demonstraia poate capata i o forma mai tehnica, utilizn t t s induciei matematice; lsm acest demers n seama cititorului. S spunem t aa a aceste demonstraii deoarece n cazul poliedrelor se va observa o temeinic t n argumentare. Cazul plan poate fi rezolvat mult mai simplu, chiar n u forma mai generala: 0 Propoziia 2 . Fie A1 A2 . . . An un poligon si v1 , v2 , . . . , vn vectori n t orientai catre exteriorul poligonului, nct, pentru fiecare k = 1, n, vk are t ct [Ak Ak+1 ] si formeaza un acelai unghi cu Ak Ak+1 . Atunci v1 +v2 + s Demonstraie (aflata de autor de la prof. Marian Tetiva, Brlad t servm c v1 , v2 , . . . , vn se obin din A1 A2 , A2 A3 , . . . , respectiv An A a a t rotaie de acelai unghi . Cum A1 A2 + A2 A3 + + An A1 = 0, la fel t s s + + vn = 0. Pentru teorema ariciului n spaiu avem nevoie de urmtoarea t a Lem. Fie A1 A2 A3 A4 un tetraedru; notam cu Sk aria feei opuse vr a t cu hk unghiul feelor de arii Sh si Sk , format spre interiorul tetraedrul t are loc egalitatea S1 = S2 cos 12 + S3 cos 13 + S4 cos 14 . Demonstraie. Considerm nti c A1 se proiecteaz pe planul (A t a a a punctul H interior 4A2 A3 A4 . Atunci SHA3 A4 = S2 cos 12 , SHA2 A4 = SHA2 A3 = S4 cos 14 i SHA3 A4 + SHA2 A4 + SHA2 A3 = S1 , de unde concluz s namentul este analog n cazul n care H Int A2 A3 A4 . / S observm analogia cu egalitatea b cos C + c cos B = a din cazul triu a a Propoziia 3. Cu notaiile din lema, fie versorii ik , k = 1, 4, ortogona t t t pe feele de arii Sk si orientai spre exteriorul tetraedrului. Atunci t S1 i1 + S2 i2 + S3 i3 + S4 i4 = 0.

Demonstraia pe care o dm este dup [3] i decurge la fel cu aceea t a a s iei 1. Notam aadar S = S1 i1 + S2 i2 + S3 i3 + S4 i4 i, folosind Lema t s s

2 S S1 i1 = S1 S1 S2 cos 12 S1 S3 cos 13 S1 S4 cos 14 = 0 i nc trei relaii analoage. Fiind ortogonal pe trei vectori necoplanari, s a t este n mod necesar 0. Propoziia 4. Fie un poliedru convex cu ariile feelor S1 , S2 , . . . , S t t Pe planul feei de arie Sk se construiete versorul ik perpendicular, ori t s exteriorul poliedrului. Are loc relaia S1 i1 + S2 i2 + + Sn in = 0. t Demonstraie. Partiionam poliedrul n tetraedre cu interioarele disj t t care dou tetraedre avnd n comun cel mult o faa. Pentru fiecare tetra a t struim vectorii perpendiculari pe planele feelor, spre exterior, de lungim t ariile feelor respective. Aplicam pentru fiecare tetraedru Propoziia 3 i in t t s t ca pe fiecare faa a tetraedrelor care nu este faa a poliedrului iniial sunt t t t doi vectori care se anuleaz reciproc. a O alta demonstraie a teoremei ariciului pentru tetraedre poate fi ga t i utilizeaz produsul vectorial, iar o frumoas demonstraie n cazul gen s a a t n [4], bazat pe ideea c suma proieciilor vectorilor pe orice dreapt a a t a (pb. M119 din Kvant). n spaiu, o demonstraie analoag cu cea a Pro t t a nu se poate gasi. Folosind Propoziia 3, putem obine valabilitatea urmtorului enun t t a t credem c ele sunt echivalente), care reprezint extinderea n spaiu a Pro a a t Problema 2. Cu notaiile din lema, construim punctul B2 de ceala t planului (A1 A2 A3 ) dect A2 si astfel nct A1 B2 (A1 A3 A4 ), iar A1 B2 e 0 ric egal cu S2 . Analog construim B3 si B4 , apoi paralelipipedul A1 B2 B3 B4 B 0 0 pe vectorii A1 B2 , A1 B3 , A1 B4 . Atunci A1 A1 (A2 A3 A4 ) si A1 A1 = S1 n ncheiere, propunem rezolvarea urmtoarelor probleme: a 1. Deducei, cu teorema ariciului, ca fiecare latura a unui poligon este t dect suma celorlaltor laturi; generalizare n spaiu. Este reciproca adev t a 2. Demonstrai teoremele cosinusurilor pentru tetraedru: t

c a

2 2 2 2 S1 = S2 + S3 + S4 2S2 S3 cos 23 2S2 S4 cos 24 2S3 S4 cos 3 2 2 2 2 S1 + S2 2S1 S2 cos 12 = S3 + S4 2S3 S4 cos 34 .

Artai c aceste egaliti sunt valabile i dac S1 , S2 , S3 , S4 sunt lungimi a t a at s a unui patrulater (redefinind hk ). 3. Rezolvai Problema 2 sintetic (sau pe orice alt cale) i obinei astfel t a s t t a logic dintre Problema 2 i Propoziia 3. t a s t 1. 2. 3. 4.

Bibliografie D. Brnzei, S. Ania, C. Cocea - Planul si spaiul euclidian, Ed. A t t Bucureti, 1986. s J. Hadamard - Lecii de geometrie elementara. Geometrie plana, Ed t Bucureti, 1960. s M. Miculia - Introducere n geometria tetraedrului, Ed. Mined, Iai, 199 t s Probleme din revista KVANT (traduse i selectate de H. Banea), E. D. P., s 1983.

Numrul polinoamelor ireductibile din Zp [ a

Elena ROGOJINA1 , Lucian-Georges LADUNCA

este inversabil peste corpul Zp ); este deci suficient sa gasim numarul po normate (monice) ireductibile, prin nmulirea acestui numr cu p1 aflnd t a la problem. a Numarul polinoamelor de forma f = X 3 + aX 2 + bX + c, a, b, c Z Ca n [2], sa vedem nti cte dintre aceste polinoame sunt reductibile. P reductibile din Zp sunt fie de forma f = (X i) (X j) (X k), i, j, k forma f = (X i) X 2 + mX + n , i, m, n Zp i X 2 + mX + n ireduc s Zp . Prima dificultate care trebuie depaita n trecerea de la p = 5 la caz s este numrarea polinoamelor de primul tip: observm c numrul lor es a a a a numrul tipurilor de cuvinte de lungime 3 formate cu elementele mulim a t este dat de numrul combinrilor cu repetiie a a t p (p + 1) (p + 2) 3 3 Cp = Cp+2 = . 6 Aflm acum cte polinoame normate ireductibile de grad 2 peste Zp exi a p = 2, n Z2 [X] exista patru polinoame de grad 2, dintre care singurul iredu X 2 + X + b Fie p 3 prim; n Zp [X] exista p2 polinoame de forma X 2 + 1. 4n a dintre care sunt reductibile cele pentru care = m2 b este ptratul un p+1 a Zp . Numarul acestor "patrate perfecte" este . ntr-adevar (v., de 2 2 b [2], Problema 12, Spring 1977), Q (p) = {0}{x Zp | x = a , a Z } = p unde f : Z Z , f (a) = a2 este morfism de grupuri. Deoarece p 3, ecua p p are exact doua soluii, b i p 1, deci Ker f = {b p 1} i cum Im f t 1s [ 1, [ s = p1 p1 p+1 atunci Card (Im f ) = , prin urmare Card Q (p) = +1= 2 2 2 p (p + 1 2 perechilor (m, n) Zp pentru care este "patrat perfect" este 2 p+1 fiecare valoare dat lui m, n ia a valori, dat fiind faptul c b este in a4 2
1 2

Problema 3 propus la Berkeley Preliminary Exams, Fall 1985, cere det a numarului polinoamelor ireductibile de grad 3 si coecientul dominant b 1 ([2], p. 230). Mai general, Problema 150 din G. M. (seria A), nr. 1/2003, minarea numarului polinoamelor ireductibile de grad 3 din Zp [X], p prim Popa, [3]).n nota de faa vom urmri rezolvarea acestor probleme i vom t a s poate fi aflat numarul polinoamelor ireductibile de grad n din Zp [X], p p n P S observm mai nti c polinomul a a a ak X k este ireductibil peste k=0 n1 P 1 numai dac polinomul X k + a an ak X k este ireductibil (unde evident
k=0

Student, Universitatea "Ovidius", Constan a a t Profesor, Liceul de Informatic "Gr. C. Moisil", Iai a s

Zp , p fiind impar). Prin urmare, numrul polinoamelor normate ireductib a p (p + 1) p (p 1) 2 este p2 = , relaie adevrat i pentru p = 2. t a as 2 2 n final, numarul polinoamelor normate ireductibile de grad 3 din Z p (p + 1) (p + 2) p (p 1) p (p 1) (p + 1) p3 p = . 6 2 3 Evident, aceasta metoda de numarare este sortita eecului n cazul s polinoamelor ireductibile de grad n din Zp [X], dat fiind faptul ca exista cele dou tipuri de polinoame reductibile din cazul n = 3. Modalitatea de a problemei poate fi urmrit detaliat n [1], pp. 188-191 i folosete rezultat a a s s de teoria corpurilor; vom prezenta mai jos numai desfaurarea ideilor. s Pentru un polinom normat ireductibil de gradul d din Zp [X], are loc e
n

f | X p X d | n. n Se arata ca polinomul X p X nu are radacini multiple, deci n descompun produs de polinoame normate ireductibile nu exist factori care s se repete a a echivalenei anunate, aceast descompunere cuprinde ca factori toate P t t a p normate ireductibile din Zp [X] al cror grad divide pe n, de unde pn = a

d|n

am notat cu (k, p) numarul polinoamelor normate ireductibile de grad k d Aplicaia : N {1, 0, 1}, (1) = 1, (n) = (1)r daca n este t r numere prime distincte i (n) = 0 dac n > 1 i n nu este liber de s a s numete funcia lui Mbius. Aceast funcie aritmetic are proprietatea s t a t a orice aplicaie f : N C, avem ca t n X n X (d) F f (n) = F (d) = , d d d|n d|n P f (d); relaia de mai sus poarta numele d t unde f : N C, F (n) =
d|n

Aadar, numarul polinoamelor normate ireductibile de grad n din Z s 1P (d) pn/d . n cazul particular n = 3, avem n n|d p (p 1 1 3 1X 3/d 3 (d) p = (1) p + (3) p = p p = (3, p) = 3 3 3
d|3

de inversiune a lui Mbius. Aplicnd aceasta formula funciei f : N t = n (n, p), avem c F (n) = pn i atunci a s 1 X n d 1X (n, p) = (d) pn/d . p = n d n
d|n n|d

adic regsim rezultatul problemei [3]. a a

Bibliografie 1. T. Albu, I. D. Ion - Itinerar elementar n algebra superioara, Ed. ALL, 1997. 2. C. Costara, D. Popa - Berkeley Preliminary Exams, Ed. Ex Ponto, C 2000. 3. G. Popa - Problema 150, G. M. (seria A), nr. 1/2003.

Funciile lui Smarandache proprieati eleme t t

Prezenta Not este rezultatul unei selecii din m a t trimis Redaciei de ctre Minh Perez, Rehoboth, NM, S t a

Funcia Smarandache apare n literatura matematic cu mult timp n u t a istorice pot fi gsite n J. Sndor - The Smarandache function introd a than 80 years ago! Octogon Mathematical Magazine, 9 (2001), no. 2, F. Smarandache redescopera i cerceteaza aceasta funcie i are meri t s s generat un curent de preocupri n privina acesteia. a t

n cele ce urmeaz, sunt adunate o serie de proprieti ale funciei Sm a at t i ale unor generalizari ale ei. s 1. Daca p este prim, atunci S (p) = p. Reciproca este adevarata? ( Begay) Soluie. Dac p este prim, atunci r! nu este divizibil cu p pentru r < t a alt parte, p! se divide cu p i, cum este cel mai mic numr cu aceast p a s a a rezult c S (p) = p. Reciproca nu este adevrat: lsnd la o parte cazul a a a a a cu 1 neprim, avem contraexemplul S (4) = 4. Pot fi gasite alte contraexem 2. Daca n este liber de patrate, iar p este cel mai mare factor prim di punerea sa, atunci S (n) = p. (Leonardo Motta) Soluie. Fie n = a b . . . p descompunerea n factori primi a lu t a < b < < p. Atunci p! conine n scrierea sa toi divizorii primi ai t t S (n) p. Pentru r < p, observam ca r! nu se divide cu p, deci S (n) p ca S (n) = p, ceea ce doream. n n n particular, S (n) = p = (deoarece n scrierea n = p q, av q 2 (T. Yau) 3. Daca p este prim, atunci S (pp ) = p2 . (Alecu Stuparu) Soluie. Deoarece S (pp ) trebuie s se divid cu p, iar p este prim, t a a S (pp ) trebuie s fie un multiplu nenul al lui p, fie acesta kp . Mai mult, fiin a se divide cu pp , trebuie sa avem kp p (se vede ca p (p 1)! se divide cu nu i cu pp ). Atunci p2 este cel mai mic numar al carui factorial se divide s unde concluzia. Asemntor pot fi definite a doua si a treia funcie Smarandache: S2 ( a a t mai mic numr natural pentru care S2 (n)!! se divide cu n (unde m!! est a numerelor nenule cel mult egale cu m, de aceeai paritate ca i m); S3 ( s s mai mic numar natural pentru care S3 (n)!!! se divide cu n (unde m!!! est numerelor nenule cel mult egale cu m, care dau acelai rest ca i m la mpr s s at 4. Daca n 3 este un numar par liber de patrate, iar p este cel mai m prim din descompunerea sa, atunci S2 (n) = 2p. (Gilbert Johnson) Soluie. Fie n = 2 a b . . . p, cu 2 < a < b < < p numere pr t S2 (n) = 2p k, unde 1 k < 2p, atunci (2p k)!! nu se divide prin p. E

Definim funcia Smarandache S (n) pe mulimea N prin: S (1) = 1, t t n 2, S (n) este cel mai mic numar natural pentru care S (n)! se divide c

(2p)!! = 2 4 . . . (2a) . . . (2b) . . . (2p) se divide la n i, cum este ce s numar cu aceasta proprietate, urmeaza concluzia. 5. Fie p numar prim impar; sa se determine S2 pk+2 , unde p = 2k + Godunov) Soluie. Ca n rezolvarea problemei 3, se arat c S2 pk+2 = p2 . t a a 6. Daca n este multiplu nenul al lui 3, atunci S3 (n) este tot mult (K. L. Ramsharan) Soluie. Fie m = S3 (n); daca m nu ar fi multiplu de 3, atunci m!!! = m t (m 6) . . . nu s-ar divide nici el cu 3 i atunci m!!! nu se divide cu n. R s S3 (n) este multiplu de 3. 7. Sa se rezolve ecuaia diofantica S2 (x) = p, unde p este un nu t (Gilbert Johnson) Soluie. Pentru p prim fixat, vom determina numarul de numere t astfel nct S2 (x) = p. Avem c p!! se divide cu x, iar p este cel mai mic a aceast proprietate. Cum p este prim, x trebuie s fie multiplu de p. a a a) Daca p = 2, atunci x = 2. b) Daca p > 2, atunci x este produsul dintre p i o combinaie de 0, 1 sau s t p3 0 dintre factorii 3, 5, . . . , p 2. Notnd k = t , avem Ck = 1 soluie cu 2 1 2 factor (x = p), Ck soluii cu doi factori (x = p 3, p 5, . . . , p (p 2)), Ck t 0 1 trei factori etc. Numrul total de soluii este cel mult egal cu Ck +Ck + + a t

Recreaii matematice
Soluiile problemelor enunate la paginile 15 i 26. t t s 1. nlturnd segmentele marcate se obine o figur format a t a a din trei ptrate. a

2. Cu o tietur fcut n veriga a treia obinem trei buci de lan fo a a a a t at t o veriga, doua verigi i patru verigi. n prima zi calatorul plaete o veriga s t s da bucata formata din doua verigi i ia napoi o veriga; a treia zi da hangi s izolat; a patra zi d bucata din patru verigi i primete ca rest celelalte a a s s de la hangiu; a cincea zi d iari veriga izolat; a asea zi d bucata din d a as a s a i ia veriga napoi; n sfrit, n a aptea zi da hangiului i veriga ramasa. s s s s Aadar, este suficient o singur n lan pentru a putea fi fcut plata s a a t a a zilnic.

3. Iat interpretarea corect a calculului efectuat: dac exist o solu a a a a ecuaiei x2 x + 1 = 0, aceasta poate fi 1. Egalitatea 3 = 0, obinu t t x = 1 n ecuaie, arata ca 1 nu este soluie i, deci, ecuaia nu are solu t t s t

n legtur cu o problem de aritmetic a a a a propus la BAC99. a


Romana GHITA si Ioan GHITA1 t

n august 1999, la bacalaureat, profilul pedagogic, a fost propus prob a ntr-un depozit erau 185 t carbuni, iar n altul 237 t. Din primul dep cte 15 t carbuni pe zi, iar din al doilea cte 18 t pe zi. Dupa cte zile a 1 depozitul al doilea de 1 ori mai mult carbune dact n primul? 2 2 Soluie ([1]). Dac n cel de-al doilea depozit ar fi din 237 t (adic t a 3 2 a a a din el s-ar scoate zilnic din 18 t (adic 12 t), dup numrul de zile cerut 3 ar fi egale. Diferena de 185 t 158 t = 27 t este anulata de diferena de t t cantitile scoase zilnic n 9 zile. at 2 Considerm c aceast soluie necesit unele claricri. nti, a a a t a a rep 3 1 versul lui 1 . Apoi, diferena de 3 t nu este cea dintre 18 i 15, ci dintr t s 2 coincidena ntre diferenele de tone scoase zilnic este una nefericita. t t a Practic, dintr-o cantitate x aflat n depozitul I se scade zilnic cte a, cantitate y aflat n depozitul II se scade zilnic b (x < y, a < b). Ne i a m > numrul p de zile dup care n depozitul II rmne o cantitate de a a a n n n mare dect cea ramasa n depozitul I (a > b, x > y). m m n n Daca n depozitul II ar fi y i s-ar scoate zilnic b, dupa cele p zile s m m n n n cantitatea C = y p b = (y pb), egal cu cea rmas n prim a a a m m m m n (deoarece y pb = (x pa) din ipoteza problemei). Diferena y t es n my n n n de diferena a b n x y : a b zile. t m m m Putem formula probleme asemntoare, cu condiia de a alege datele a a t nct s fim condui la operaii cu numere naturale. a s t

Probleme propuse. 1. ntr-o tabr colar sunt 792 elevi, iar n alta 531. Din fiecare tab aas a din 5 n 5 minute grupuri de cte 36 i respectiv 23 elevi n drumeii. s t minute n prima tabara se vor afla de 9/7 ori mai muli elevi dect ntr-a t 2. n doua couri se gasesc 405 i respectiv 800 bomboane. n fiecare s s cte 15, respectiv 32 bomboane. Dup cte zile n coul al doilea vor fi cu a s multe bomboane dect n primul? Bibliografie 1. Gh. Andrei i colab. - Admiterea 1999, Ed. GIL, Zalu, 1999. s a
1

Profesori, Col. Na . "I. M. Clain", Blaj t

Concursul "Recreaii Matematice" t


Ediia a III-a, Iai, 28 August 2003 t s Clasa a VII-a
1. Rezolvai n N N ecuaia t t

a b + = 1. b+1 a+1 Alexandru Negrescu, Botoani (RecMat s 2. Un triunghi are doua mediane perpendiculare, iar suma lungimilo stant. S se determine maximul ariei triunghiului. a a Mihai Gavrilu t 3. Fie XOY un unghi oarecare i P un punct n interiorul lui. Se s punctele A, B OX cu A (OB) i C, D OY cu C (OD) astfel nct s rile P AB i P CD s fie echilaterale. Artai c, dac dreptele OP , AD s a a t a a concurente, atunci P se afla pe bisectoarea unghiului XOY . Temistocle Brsan, Iai (RecMat s

Clasa a VIII-a

1. Fie n N fixat. Aratai ca exista o infinitate de numere x, y, z Z a t x2n + y 2n + z 2n = x2n+1 + y 2n+1 + z 2n+1 . Lucian Tuescu, Craiova (RecMat t 2. Gasii ntregii pozitivi n, x1 , x2 , . . . , xn astfel nct x1 + x2 + + t i produsul x1 x2 . . . xn sa fie maxim. s Agnes Constantinescu, 0 0 0 0 3. Fie ABCDA B C D un cub. Cubul este patat cu cafea pe ma jumatate din suprafaa lui totala. Aratai ca exista doua puncte pe suprafa t t coliniare cu centrul cubului care nu sunt ptate cu cafea. a Valerica Bena, Iai i Mugur Roca t s s s

pentru orice n natural. Stiind ca f (0) = 0 i f (1) = 1, calculai f (2003). s t Andrei Ned 3. Fie ptratul ABCD, E mijlocul lui (AB), M (CD), N (AD) a a BM k EN . S se arate c M N este tangenta cercului C (S, r) nscris n p a a Nicu M

2 1 1 = 1. Rezolvai n R ecuaia q t t + q , un [x] [x + 2] 3 3 3 2 [x] 3 [x + 1] [x] partea ntreag a lui x. a Daniel Jinga, Piteti (RecMat s 2. Fie f : R R o funcie care satisface t 2 n + 3n + 3 f (n + 2) 2 n2 + n 1 f (n + 1) + n2 n + 1 f (n

Clasa a IX-a

Clasa a X-a

1. Fie a, b (0, 1) (1, ) i funcia injectiv f : (0, ) R astfel nc s t a g : R R, g (x) = f (ax ) + f (bx ) este constanta. Sa se arate ca ab = 1 s

funcii f care satisfac ipotezele problemei. t Dan Popescu, Suceava (RecMat

2. Sa se afle locul geometric al imaginilor numarului complex z = (0, ).

Mihai Gavrilu t 3. Un triunghi de arie S se proiecteaz pe trei plane perpendiculare a a dou. Dac ariile proieciilor sunt S1 , S2 , respectiv S3 , s se demo a a t S S1 + S2 + S3 < S 3. Gheorghe Iu

Clasa a XI-a

Silviu Boga 4. O pat de ulei curge pe un ru. La un moment dat ea intersecte a unui fir de telegraf. Sa se demonstreze ca exista un moment n care um mparte pata de ulei n doua poriuni de aceeai arie. t s Vlad Marti

1. Fie D, M dou matrice nesingulare de ordin n, D diagonal, iar M a a lar. Dac D = t M DM , s se arate c M este tot o matrice diagonal, a a a a a pe diagonala principala. Adrian Corduneanu, Iai (RecMat s 2. Fie (xn )n1 , (yn )n1 dou iruri de numere naturale mai mari ca a s xn yn pxn pyn arate c lim a = 0 lim = 0, unde pn este al n-lea nu n n yn pyn Gabriel Mr s 3. n tetraedrul ABCD se consider notaia (ab) = m [ (ABC; ABD a t punztoare muchiei AB i analoagele, corespunztoare la celelalte muchii. a s a 1 cos (cd) cos (bd) cos (bc) cos (cd) 1 cos (ad) cos (ac) = 0. cos (bd) cos (ad) 1 cos (ab) cos (bc) cos (ac) cos (ab) 1

Clasa a IX-a (BARAJ)


1. Determinai funciile f : R R care verific egalitatea t t a xf x3 + x + 1 + f x3 + 3x2 4x + 3 = x + 1, x R.

Silviu Boga 2 3 2. t A x 4 Se 3dau 2mulimile: = x + 4 | x Z , B = x + x | x = x + x + x + x | x Z , D = 2x | x Z . Determinai mulim t t B D. Andrei Nedelcu, Iai (RecMat s

Concursul interjudeean "Octav Onicescu t

Ediia a VII-a, 31 oct. - 2 nov. 2003, Botoani t s

Aceast ediie a Concursului de matematic "Octav Onicescu" a cuno a t a ticipare numeroas i entuziast, antrennd elevi din 5 judee: Botoani, Ia a s a t s s Vaslui i Vrancea. s Ceea ce particularizeaz n mod deosebit acest concurs este faptul c se p a a rezolvare aceleai subiecte pentru toi participanii de la clasa a IX-a pn la cla s t t a Subiectele propuse nu sunt axate pe materia studiat de fiecare elev la nivelul su a a ci ncearc s pun n valoare abilitile matematice pure ale concurenilor. a a a at t Deschiderea festiv a concursului i premierea s-au desfurat n Aula Mag a s as "A. T. Laurian" din Botoani, iar alturi de elevi i profesori au participat i s a s s locale. De partea organizatoric s-a ocupat I. S. J. Botoani i C. N. "A. T. L a s s Sarcina elaborrii subiectului de concurs a revenit, ca n fiecare an, dom a sori Adrian Bo an i Adrian Panaete, iar misiunea corectrii lucrrilor sc t s a a brilor catedrei de matematic de la C. N. "A. T. Laurian". Preedintele com a s prof. univ. dr. Eugen Popa de la Facultatea de Matematic, Universitatea " a din Iai. s

Publicm n continuare problemele propuse concurenilor i lista prem a t s

1. Fie a1 , a2 , a3 , . . . , a2003 numerele 1, 2, 3, . . . , 2003 n alta ordin ca macar doua din numerele |a1 1|, |a2 2|, . . . , |a2003 2003| sunt eg 2. De pe o tabla de ah 7 7 scot un patrat; aratai ca patratele ram s t a) nu pot fi acoperite cu 24 de dominouri 1 2 daca patratul scos e A b) pot fi acoperite cu 24 de dominouri dac ptratul scos e D4 i a a s acoperire cu numr minim de dominouri orizontale (justicare). a 3. Dac n este natural, gsii restul mpririi lui 10n prin 999 i ar a a t at s a numr natural divizibil cu 999 are mcar 3 cifre nenule. Cte numere cu c a a cifre fiecare au fix 3 cifre nenule i se divid cu 999? s 4. Cte ptrate ale unei table de ah 340 121 sunt tiate n interior a s a diagonalele tablei? Dar pentru o tabla 340 120? 5. Ali Baba i cei 40 de hoi stau n cerc n jurul focului i vor s s t s a mod egal 4100 de galbeni care iniial se afla mparii la ntmplare la c t tt ei (posibil la unul singur). Ali Baba bate din palme i la comanda lui fiec s 41 d un galben vecinului din stnga sa, dac acesta are mai puin dec a a t vecinul are egal sau mai mult nu primete nimic!). Dac nu au realizat s a Ali Baba bate din palme din nou etc. Justificai ca dupa un timp sumele s t (toi 100 de galbeni). t

Premiaii sunt: premiul I - Chiril Cezar (C. N. "M. Eminescu", t a premiul II - Istrate Carmen Maria (C. N. "Unirea", Focani), prem s Pachiariu Marius (Colegiul Naional Iai). Au fost acordate 21 meniu t t s t

1 Selec iuni din materialul trimis redac iei de ctre elevul Alexandru Negrescu i p t t a s Tomi a , C. N. "A. T. Laurian", Botoani t s

Concurs de admitere 2003, Iai s


Facultatea de Informatic, Universitatea "Al. I. Cuza" a

Algebr a I. 1. Se da matricea A M3 (R), unde M3 (R) este inelul matricelor 0 0 1 de ordin 3 cu elemente reale, A = 1 0 0. S se arate c A3 = I3 i a a s 0 1 0 relaia (A I3 ) A2 + A + I3 = 0. t 2. Fie S3 o permutare din grupul simetric de grad 3, astfel nc (e noteaz permutarea identic). Demonstrai c exist k {1, 2, 3} a a a t a a (k) = k. 1 3. Demonstrai ca polinomul P = X 3 + X + 1 este ireductibil n Q [X t 2 II. 1. Fie G un grup cu n elemente, n N . Aratai ca n orice coloan t operaiei lui G apar n elemente distincte. t 2. Fie (Z, +, ) inelul numerelor ntregi. Determinai toate morfisme t f : Z Z. 3. Fie (C, +, ) corpul numerelor complexe. Sa se arate ca f : C C de f (z) = z este izomorfism de corpuri (z noteaza conjugatul numarului com

Analiz matematic a a 1 1 1 I. 1. Fie Hn = 1 + + + + , n N . Demonstrai ca irul (Hn t s 2 3 n nemarginit. 2. Fie f : R R o funcie continua i marginita. Demonstrai ca exis t t s astfel nct f (x0 ) = x0 . 1 3. Fie f : R \ {1} R, f (x) = . Sa se calculeze f (n) (0), und x+1 (n) noteaz derivata de ordin n a funciei f . a t iar f II. Pentru n N considerm fn : R R, fn (x) = xn + xn1 + + a f3 (x) f2 (x) a) Reprezentai grafic funcia g : D R, g (x) = t t , un f1 (x) domeniul maxim de definiie al funciei g. t t t b) Aratai ca pentru orice n N , ecuaia fn (x) = 0 are o unica sol t un , n intervalul [0, 1]. c) Demonstrai c irul (un )nN este convergent. t as d) S se determine lim un . a
n

Fac. de Electronic i Telecomunicaii, Univ. Tehnic "Gh as t a

1 1 1 1 a) 1 n n b) 2 n+1 n+1 2 3 2 3 3 1 1 e) n 2 2 2 3n

13 3 a 1. Rangul termenului din dezvoltarea care l conine p t + 3 3 a a) 8 b) 6 c) 3 d) 4 e) 9 n k P k k 2. Suma 2 + 3 /6 este egala cu


k=1

c)

3 1 1 + n+ 2 2 2 3n

d)

1 + 2n 3

3. Se consider inecuaia (m + 1) e2x + 2 (m + 1) ex + m > 0, x a t m R este un parametru. Valorile lui m pentru care inecuaia este verifica t sunt a) (, 0] b) [0, +) c) [1, 0] d) (0, 1) e) (, 1) 4. Mulimea tuturor valorilor m R astfel ca sistemul t mx + y z = 0 x + (m + 1) y + z = 2 + m m2 x 2y mz = 2 + 3m m2

sa fie compatibil este a) {2} b) {2, 1, 2} c) R \ {2, 1} d) R \ {1} e) R \ {2, 10 5. Numarul complex z, care satisface |z| + z = este 2i 3 3 1 3 a) 2 + i b) 2 + 5i c) 2i d) + 3i e) + 2i 2 2 2 2 x sin x a s 6. S se determine m astfel ca l = lim a m s fie finit i difer x0 (1 cos x) Sa se precizeze i valoarea lui l. s 3 2 2 2 3 3 c) m = , l = b) m = 3, l = d) m = , a) m = , l = 4 3 2 3 2 3 1 e) m = 2, l = 6 2 ln x 1 este 7. Mulimea valorilor funciei f : (0, ) R, f (x) = t t x 2 2 2 b) (0, ) c) R d) (0, e) e) 1/e , a) , 1/e 8. Daca x1 , x2 , . . . , xn sunt radacinile ecuaiei xn + 1 = 0, atunci valoa t 1 1 1 + + + este 1 x1 1 x2 1 xn 2 a) n/2 b) n c) n d) n (n + 1) e) n Z 2 x3 max x , arctg x dx. 9. Sa se calculeze I = 3 1 1 5 5 2 arctg 3 a) arctg 2 + ln 3 b) + 2 arctg 2 ln 5 c) 12 2 12 5 5 2 arctg 2 + ln 3 e) + arctg 2 d) 11 12 1 1 10. S se afle soluia ecuaiei arcsin a t t + arcsin = . x1 x + 2 p 1 p p p p a) 3 + 5 b) 3 + 5 c) 2 + 5 d) 2 + 5 e) 1 +

Fac. de Automatic i Calculatoare, Univ. Tehnic "Gh. as a

1. S se determine parametrul real m astfel nct ecuaia x2 + 2mx + (m a t s admit rdcinile reale x1 i x2 verificnd x1 < 1 < x2 . a a a a s ! ! 1 17 1 + 17 5 a) m , , b) m , c) m 2 2 3 ! 1 17 3 d) m e) m , 2 5

Care din urmtoarele afirmaii este fals? a t a a) Dac a = 0, sistemul este incompatibil b) Dac a = b, sistemul es a a tibil nedeterminat c) Exist a, b R, b 6= 0 astfel nct sistemul are sol a d) Daca a 6= 0 i a 6= b, sistemul este incompatibil e) Daca a = 1 i b 6= s s sistemul este incompatibil 4. Fie M = (, 1) (1, ) i legea de compoziie intern pe M s t a x y = 3ax + by + xy, x, y M , unde a, b R, b 6= 0. S se afle a i b a a s (M, ) sa fie grup abelian i sa se precizeze simetricul x0 al unui elemen s x M. 1 x x 1 a) a = , b = 1, x0 = b) a = 1, b = 3, x0 = c) a = , b = 1, x 3 x+1 x+1 3 1 x 1 1 d) a = 1, b = , x0 = e) a = , b = 1, x0 = 3 x+1 3 x+1 n 5. Se d irul definit prin relaia xn+1 = xn + (a) , n N , x1 a s t 0 < a < 1. Care din urmtoarele afirmaii este adevrat: a t a a a) irul este strict cresctor cu limita + b) irul este strict descr s a s a limita c) irul nu este monoton, dar are limita s d) irul este str s a+1 tor cu limita 1 e) irul nu este monoton, deci nu are limita s 1 3 5 1 + + + 6. Se d f : R\ {0, 2, 4, 6} R, f (x) = + a x x2 x4 x6 punctelor n care graficul funciei intersecteaza axa Ox este t a) 0 b) 1 c) 2 d) 3 e) 4 1 a 7. Fie ecuaia diferenial y 0 + y = 6x, x > 0. S se precizeze interva t t a x care y (x) > 0, unde y (x) este soluia care satisface condiia y (1) = 1. t t 1 3 2, c) x , a) x (1, 2) b) x d) x (2, 3) e) x 3 2 0 0 0 8. Se dau triunghiurile ABC i A B C ce au centrele de greutate G i G s s 0 vectorul GG este egal cu 1 1 2 a) (AA0 + BB 0 + CC 0 ) b) (AA0 + BB 0 + CC 0 ) c) (AB + BC + CA 3 4 3 1 1 B 0 C 0 + C 0 A0 ) d) (AB 0 + BA0 + AC 0 + CA0 + BC 0 + CB 0 ) e) (AB 0 + B 6 3 9. S se determine mulimea punctelor din planul complex care sunt a t 2 numerelor z care verific ecuaia z 2 z |z| + |z| = 0. a t a) doua drepte perpendiculare b) un cerc cu centrul n origine c) do paralele d) dou semidrepte e) dou cercuri concentrice a a 1 1 1 10. Numrul soluiilor ecuaiei arctg a t t + arctg arctg 2 x1 x+1 x 1 a) 1 b) 2 c) 3 d) 4 e) o infinitate

2. Aflai numrul termenilor raionali din dezvoltarea binomial t a t a a) 1 b) 2 c) 3 d) 4 e) 5 3. Fie sistemul x+y+z =1 ax + ay + 2az = b a, b R, b 6= 0. 2 a x + a2 y + 2a2 z = b2

3 7

Soluiile problemelor propuse n nr. 1 / 20 t


Clasele primare

P.44. Un vecin al unui vecin al numarului 81 este egal cu un vecin al al numarului 77. Despre ce numar este vorba? ( Clasa I ) Mihaela Rusu, e Soluie. Acest numr trebuie s fie mai mare ca 77 i mai mic dect 81 t a a s se afl n secvena 77 81. Este vorba despre numrul 79. a t a P.45. Adunnd trei numere naturale a, b, c obinem suma 62. Primul n t mai mare dect al treilea si mpreuna au suma 12. Care sunt cele trei nu ( Clasa a II-a) nv. Maria R Soluie. Numarul b = 62 12 = 50. Perechea (a, c) poate fi: (12, t (10, 2); (9, 3); (8, 4) sau (7, 5). Tripletul (a, b, c) poate lua valorile: (12, 50, 0 (10, 50, 2); (9, 50, 3); (8, 50, 4) sau (7, 50, 5). P.46. Mihai, Dan si Petru practica fiecare un alt fel de sport si anu fotbal sau volei. Mihai si voleibalistul locuiesc n acelai bloc. Cel care joa s cel care joaca fotbal l-au urmarit pe Petru la un meci. Ce sport practica f ( Clasa a II-a) Adina Dohotaru, e Soluie. Din textul problemei se deduce c Petru nu joac volei sau f t a a el joac tenis. Mihai i voleibalistul locuiesc n acelai bloc. Aceasta n a s s Mihai nu joaca volei. Soluia problemei este: Petru joaca tenis, Mihai joac t Dan joaca volei. P.47. Diferena a doua numere este 48. Aceasta diferena este cu 22 t t dect jumatatea unuia dintre ele. Determinai numerele. t ( Clasa a III-a) nv. Rodica Rotaru Soluie. Fie a b = 48. Avem dou cazuri: 1) 48 = b : 2 + 22 de und t a b = 52 i a = 100. 2) 48 = a : 2 + 22 de unde obinem a = 52 i b = 4. s t s P.48. Un agricultor mparte un teren n trei parcele. n fiecare an, fiec este cultivata numai cu una din culturile: gru, porumb sau legume. n anul 2003, agricultorul se hotaraste ca pe fiecare parcela sa fie alta cultu ani consecutivi. a) Care este primul an dupa 2003 n care se repeta culturile pe cele tre b) Se poate preciza care este ordinea culturilor pe cele trei parcele n an ( Clasa a III-a) Andreea Surugiu, e Soluie. Presupunem c n anul 2003 avem ordinea (gru, legume, t a n anul 2004 putem avea (legume, porumb, gru) sau (porumb, gru, le anul 2005 putem avea (porumb, gru, legume) sau (legume, porumb, gru avem din nou ordinea (gru, legume, porumb). Raspunsul la a) este anu Ordinea culturilor se mai repet n 2009, 2012, 2015, 2018. Nu putem preci a culturilor n anul 2019. P.49. La un moment dat, cernd unei persoane anul naterii, aceasta s "anul acesta mplinesc 25 ani, iar daca a scrie toate numerele ncepn s terminnd cu anul naterii si apoi toate numerele ncepnd cu 1 si term s

anul n care ne aflam mi-ar trebui 13710 cifre. n ce an ne aflam cn ntrebarea? ( Clasa a III-a) Prof. Ctlin - Cristian Bude a a Soluie. Pentru scrierea numerelor de la 1 999 sunt necesare 2889 cifr t c anul naterii nu poate fi format din trei cifre. ntr-adevr, 2 2889 + 25 n a s a n 4. Anul naterii este de forma abcd. Fie x numarul cifrelor pentr s numerelor de la 1 la abcd. Transpunnd n ecuaie ceea ce a spus persoana t x + (x + 4 25) = 13710, cu soluia x = 6805. Pentru scrierea numerelor t la abcd sunt necesare 6805 2889 = 3916 cifre, ceea ce nseamn c de la 1 a a sunt 3916 : 4 = 979 numere. nseamna ca anul abcd este 1978. ntrebarea a n anul 1978 + 25 = 2003. P.50. a) Cte numere trebuie adaugate sirului 1, 2, 4, 5, 7, 8, . . . , 97, 9 obine toate numerele de la 1 la 98? t b) Efectuai 1 + 2 + 4 + 5 + 7 + 8 + + 97 + 98 2 (3 + 4 + 5 + t ( Clasa a IV-a) Georgiana Ciobanu, e Soluie. a) Lipsesc numerele: 3, 6, 9, . . . 96 care pot fi scrise: 3 1, 3 2, t 3 32. Se observa ca lipsesc 32 numere. b) Expresia de calculat se poate scrie:

P.51. Produsul a doua numere naturale este 913 368. Unul din numer unitatilor si cifra zecilor mai mare ca 2 si mai mica dect 8. Daca la ac marim cifra zecilor cu 2 si micoram cifra unitatilor cu 1, obinem un p s t cu 951 425. Aflai cele doua numere. t ( Clasa a IV-a) nv. Elena Zrn a Soluie. Fie a i b numerele cutate. Obinem t s a t

1 + 2 + (4 3) + (5 3) + (7 4) + (8 4) + + (97 34) + (98 = 1 + 2 + (1 + 2 + 3 + 4 + + 63 + 64) = 3 + 64 65 : 2 = 3 + 2080 =

(a + 20 1) b = 951425 ab + 19b = 951425 913368 + 19b = 951425 b = 2003 a = 913368 : 2003 = 45

P.52. n trei cutii sunt 212 bile. Din prima cutie se scoate un numar d a doua de 2 ori mai mult si nca doua bile, din a treia se scoate ct triplul de bile scos din a doua cutie. n fiecare cutie ramne un numar de b numarul total al bilelor scos din cele trei cutii la un loc. Cte bile au fost cutie? ( Clasa a IV-a) nv. Maria R Soluie. Notm cu p numrul bilelor scos din prima cutie. Rezult c t a a a a cutie ramn 9p + 8 bile. Deducem ca n toate cutiile au fost 36p + 32 bil 36p + 32 = 212, de unde p = 5. n cele trei cutii au fost 58, 65, respectiv P.53. Efectund o singura cntarire, sa se ia 475 g dintr-un kilogram utiliznd doua greutati, una de 200 g si cealalta de 150 g. ( Clasa a IV-a) Prof. Petru Asa Soluie. Utilizm o balana cu brae egale. Distribuim kilogramu t a t t i cte una din cele dou greuti, pe cele dou talere, pn realizm s a at a a a echilibru. Pe fiecare taler vom avea 675 g. Masa cautata este pe talerul

afl greutatea de 200 g: 675 g 200 g = 475 g zahr. a a

Clasa a V-a

V.36. Fie n un numar impar, iar a1 , a2 , . . . , an , n N numere care la n dau cturi distincte si resturi distincte. Aratai ca valoarea minim t S = a1 + a2 + + an este multiplu de 12. Drago Ungureanu, s Soluie. Conform ipotezei, avem: a1 = nc1 +r1 , a2 = nc2 +r2 , . . . , an = t unde {r1 , r2 , . . . , rn } = {0, 1, 2, . . . , n 1}. Astfel, suma n (n 1) S = a1 + a2 + + an = n (c1 + c2 + + cn ) + 2 este minim dac {c1 , c2 , . . . , cn } = {0, 1, 2, . . . , n 1}, deci a a n (n 1) n (n 1) n (n 1) (n + 1) Smin = n + = . 2 2 2 . . . Cum n este impar, rezulta ca (n 1) (n + 1) . 8, deci Smin . 4. Pe de a . . . 3. Pr deoarece n, n 1, n + 1 sunt numere consecutive, rezulta ca S .
min

Smin este multiplu de 12.

V.37. Comparai fraciile a = t t

Maria Cojoc 3 1 2 1 i = 1 + s . Cum 3 222221 > Soluie. Avem = 1 + t a 333331 b 222221 1 1 rezult c > , deci b > a. a a a b V.38. Sa se arate ca 2a + 2b + 2c + 2d + 2e 6= 2003, a, b, c, d, e N. Irina Ispas, stud Soluie. Presupunem c exist cinci numere naturale a b c d t a a nct 2a + 2b + 2c + 2d + 2e = 2003. Daca a 6= 0, atunci termenul din stnga al egalitaii (1) este par i atun s t contradicie. Pentru a = 0, relaia (1) devine: 2b + 2c + 2d + 2e = 2002. t t 210 = 1024, rezult c numai e ar putea avea, eventual, valoarea 10. a a Daca e = 10, atunci 2b + 2c + 2d = 978. n acest caz, daca b, c, d 2b + 2c + 2d 3 256 < 978. Aadar, d = 9 i 2b + 2c = 466, ceea ce nu es s s Dac toate numerele b, c, d, e sunt strict mai mici ca 10, se observ c a a trei numere pot fi 9 (altfel avem 2b + 2c + 2d + 2e 4 512 > 2002) i cel s trebuie sa fie 9 (deoarece, n caz contrar, avem 2b + 2c + 2d + 2e < 29 + 29 + < 2002). Prin urmare, c = d = e = 9 i atunci 2b = 2002 3 29 = 476, ab s V.39. Sa se determine numerele prime p1 < p2 < p3 < p4 astfel nc p1 + p2 + p3 + p4 , p3 p2 , p4 p3 sa fie, de asemenea, prime. Petru M Soluie. Deoarece p1 + p2 + p3 + p4 este un numar prim mai mare ca t c el este impar i atunci unul dintre numerele p1 , p2 , p3 , p4 trebuie s fi a s a p1 = 2. Cum p2 , p3 i p4 sunt impare, nseamn c p3 p2 i p4 p3 su a a s s avnd n vedere ca sunt prime, rezulta ca p3 p2 = p4 p3 = 2. De aici

222221 333331 si b = . 333334 222223

c p3 = p2 + 2 i p4 = p2 + 4. Se observ c p1 = 2, p2 = 3, p3 = 5 i p a s a a s o soluie a problemei (2 + 3 + 5 + 7 = 17 este numar prim). Daca p2 > t p2 = 3k + 1 sau p2 = 3k + 2, k N . n cazul p2 = 3k + 1, avem p3 = 3 nu este prim, iar n cazul p2 = 3k + 2, avem p4 = 3k + 6, care nu este prim p1 = 2, p2 = 3, p3 = 5, p4 = 7 este singura soluie. t V.40. Este posibila o partiionare a mulimii {1, 2, . . . , 12n + 9} n 4 t t mulimi disjuncte, fiecare cu cte trei elemente, astfel nct n fiecare subm t element sa fie suma celorlaltor doua? Titu Zvonaru, B Soluia I. Fie {a, b, c} o mulime astfel nct a = b + c. De aici, t t elementele mulimii {a, b, c} sunt ori toate pare, ori dou impare i unul pa t a s pentru ca sa fie posibila o partiie ca n problema, trebuie ca mulimea data t t un numr par de numere impare. Deoarece mulimea dat are 6n + 5 nume a t a rezult c partiionarea nu este posibil. a a t a Soluia II. S presupunem c ar fi posibil o partiie n condiiile impus t a a a t t fiecare din cele 4n + 3 submulimi de trei elemente are suma elementelor e t numr par, deci suma elementelor mulimii {1, 2, . . . , 12n + 9} ar trebui s a t a (12n + 10) (12n + 9) par. Cum, 1 + 2 + + 12n + 9 = = (6n + 5) (12n 2 este un numar impar, rezulta ca partiionarea ceruta nu este posibila. t

Clasa a VI-a
VI.36. Fie k N, k 3. Aratai ca printre valorile naturale ale lui t . adevarata propoziia n2 + k . n + k, exista cel puin trei patrate perfecte. t . t

Claudiu Stefan P Soluie. Din n2 + k = n2 k 2 + k2 + k = (n k) (n + k) + k 2 + k, t . . n2 + k . n + k dac i numai dac k 2 + k . n + k. Cum A = k + 1, k2 + k . as a . k, putem lua n + k din mulimea A i atunci obinem n 0, 1, k 2 . Astfel t s t trei patrate perfecte care verifica cerina problemei. t VI.37. Numerele 1160, 1604 si 2270 dau acelai rest la mparirea prin s t mparitorul n. t Cristian L Soluie. Conform ipotezei, avem: 1160 = nc1 +r, 1604 = nc2 +r, 2270 t unde r < n i r, c1 , c2 , c3 N. Scznd aceste egaliti dou cte dou s a at a a 444 = n (c2 c1 ), 666 = n (c3 c2 ) i 1110 = n (c3 c1 ), deci n este divi s al numerelor 444, 666, 1110. Cum (444, 666, 1110) = 222 rezulta ca n 37, 74, 111, 222}, valori care verific ipoteza problemei. a VI.38. Demonstrai ca nu exista numere naturale x, y, z direct propor t trei numere naturale consecutive, astfel nct x + y + z sa fie numar prim Alexandru Negrescu, elev, Soluie. Dac presupunem contrariul, avem t a x y z x+y+z = = = , cu n N . n n+1 n+2 3n + 3 De aici, obinem c 3y = x + y + z, deci 3 | x + y + z, care mpreun cu t a a x + y + z este prim ne conduce la concluzia ca x + y + z = 3 i deci y = 1. s

x 1 n = , adic x = a , care nu aparine lui N. t n n+1 n+1 VI.39. Radu si Mihai joaca de mai multe ori un joc n urma caruia c primete a puncte, iar cel care pierde primete b puncte ( a, b N , a > s s scorul final este 61 49 n favoarea lui Radu, iar Mihai a ctigat 4 par s a si b. Adrian Zano Soluie. Dac notm cu x numrul partidelor ctigate de Radu, avem t a a a s = 61, 4a + xb = 49, de unde obinem ca (x + 4) (a + b) = 110. De aici t vedere ca x + 4 9 i a + b 3, rezulta ca x + 4 = 22 i a + b = 5 sau x s s i a + b = 10 sau x + 4 = 10 i a + b = 11. n primul caz, avem x = 18, s s xa + 4b este un numr par, diferit de 61, deci aceast situaie nu convine. P a a t la fel, constatam ca nici al treilea caz nu convine. n al doilea caz, gasim x i b = 3, care este soluia problemei. s t b VI.40. Fie 4ABC cu m(A) = 120 . Perpendiculara n C pe AC int mediatoarea lui [AB] n D; notam {E} = CD AB. Sa se arate ca A \ = 90 si BE = 2AB. daca si numai daca m(BDE) Ioan Scleanu a a Soluie. Fie M mijlocul lui AB. t Presupunem ca AB = 2AC. n acest caz D \ \ rezult c AM = AC, deci CDA = ADM = a a \ = M DB = . Cum suma unghiurilor patrulaterului DM AC este 360 , obinem ca t C \ = 90 . Triunghiul DAB = 30 , deci BDE \ este isoscel i are unghiul BDA de 60 , adica s este echilateral i, prin urmare, DA = AB. n B s M A \ [ plus DBA = 60 , deci AEC = 30 . Atunci 4ACD 4ACE (C.U.), de unde AD = AE. n concluzie, BA = AD = BE = 2AB. \ Fie acum BDC = 90 i A mijlocul lui BE. Cum AC k BD, rezult c s a a 1 [ s linie mijlocie n triunghiul BDE, deci AC = BD. Din CAE = 60 i C 2 obinem ca DBA = 60 , deci triunghiul DBA este echilateral, ceea ce c t \ 1 concluzia BD = AB. Aadar, avem AC = AB sau AB = 2AC. s 2 n relaia (1), gsim t a

Clasa a VII-a
VII.36. Sa se arate ca

2n 1 < 2n 1, n N n Ctlin Cali a a Soluia I (un grup de elevi de r Colegiul Naional din Iai i A t la t s s 1 + k/n k+n k < = , k = 1, 2 Negrescu, elev, Botoani). Avem s n 2 2n urmare, r r r 1 1 1 2 2n 1 + + + < 2n 1 + (1 + + (2n 1)) n n n 2 n

1 + n

2 + + n

1 1 (2n 1) 2n = 2n 1 + = 2n 1. 2 n 2 Soluia II. r t Membrul r stnga al inegalitii date se poate scrie gru din at nk n+k menii de forma , , k {1, 2, . . . , n 1}. n acest fel, obi t nr n n paranteze i termenul s = 1. Deoarece n nk+n+ r !2 r r r nk n+k nk n+k n =2+2 + <2+2 n n n n 2 r r nk n+k + < 2, de unde concluzia. rezult c a a n n VII.37. Aratai ca n baza de numeraie 7 printre numerele ce se scriu t t 0, 1, 2 exista o infinitate care sunt patrate perfecte si o infinitate ce nu su perfecte. Aceste afirmaii ramn valabile daca se folosesc cifrele 3, 5, 6? t Ruxandra Ioana Vlcu, e 10 Soluie. Se observa ca 100 {z . 01 2 = (7n + 1)2 = 72n +27n +1 = | . . t | . . } (7)
n+1 cifre 2n+1 cifre 2n+1 cifre

trat perfect pentru nici un n N , deoarece este cuprins ntre (7n + 1)2 i s Daca n N, atunci putem scrie n = 7k + r, unde k, r N, r < 7. n2 = 7k 0 + r0 , cu r0 {0, 1, 2, 4}, rezult c nici un ptrat perfect scris n a a a se termin cu 3, 5 sau 6. Prin urmare, rspunsul la ultima ntrebare este a a VII.38. Fie a, b, c cifre nenule, a 6= c. Sa se arate ca daca

este ptrat perfect, n N , iar 10 . . . 020 . . . 2 (7) = | . . . 020 . . . 1 (7) + 1 n a 10 {z } {z } |

2n

abb . . cbb . . (termenii primei fracii coninnd cte 2002 cifre b), atunci b = a + c. t t Mihaela Buc a Soluie. Daca notam n = | {z . 1 , avem succesiv: t 11 . . }
2002 cifre 2003

VII.39. Daca x < y < z sunt lungimile laturilor unui triunghi dr atunci xn + y n 6= z n , n N, n 3. Dumitru Ne Soluie. Din relaia z > y > x, rezult c z n2 > y n2 i z n2 > xn t t a a s ar fi n 3. De aici, obinem ca, pentru orice n 3, avem: t z n = z n2 z 2 = z n2 x2 + y 2 > xn2 x2 + y n2 y 2 = xn + y

102003 (a + c) 100nb + 10nb 10 (c + a) = 0 2002 1 90bn = 0 (a + c) 10 9n 90bn = 0 (a + c) 10 10

a 10 + 10nb + c 10a + c = c 102003 + 10nb + a 10c + a 2 102003 a c2 + 100nb (c a) + 10nb (a c) + 10 c2 a2 = 0

b VII.40. Fie ABC un triunghi ascuitunghic cu m(A) = 60 , iar M t

\ astfel nct m(BM C) = 150 . Notam cu P , Q, R proieciile lui M pe B t respectiv AB. Sa se arate ca 4P QR este dreptunghic. Constantin Co A Soluie. Deoarece patrulaterele M P BR i M P CQ t s \ \ \ sunt inscriptibile, avem: M P R = RBM = 90 RM B 60 \ = QCM = 90 QM C. Astfel, obinem: \ \ i M P Q s t \ \ \ \ \ RP Q = M P R + M P Q = 180 ( RM B + QM C ) = \ \ = 180 (360 RM Q BM C ) = = 180 (360 120 150 ) = 180 90 = 90 .
R M B P

Clasa a VIII-a

VIII.36. Determinai cardinalul minim al unei mulimi B pentru c t t defini funcii f : R B astfel nct f (1) < 0 si f (xy) = f (x) f (y), t Iulia Zanoschi, e Soluie. Vom demonstra c mulimea B trebuie s aib cel puin tre t a t a a t i c exist o funcie care are codomeniul B format din trei elemente i n s a a t s restul condiiilor din enun. t t Avem f (1) = f ((1) (1)) = f (1) f (1) > 0. Pe de alt parte, d a f (0 (1)) = f (0) f (1), rezult c f (0) [f (1) 1] = 0, deci f (0) = a a urmare, f (1), f (0) i f (1) sunt trei numere distincte, ceea ce nseamna s cel puin trei elemente. n fine, se observa ca f : R {1, 0, 1}, de t 1, x < 0 0, x = 0 , verific toate condiiile cerute. f (x) = a t 1, x > 0 VIII.37. If a, b, (0, ) prove the following inequalities: c a) (a + b + c)3 a3 + b3 + c3 24 where abc = 1; 3 8 3 3 3 3 b) (a + b + c) a + b + c where ab + bc + ac = 1. 3 Zdravko Starc, Vrac, Serbia and Mo s Soluie. a) Se tie ca, oricare ar fi numerele a, b, c, are loc egalitatea: t s (a + b + c)3 a3 + b3 + c3 = 3 (a + b) (b + c) (c + a) . Avnd n vedere identitatea (1) i inegalitatea mediilor, putem scrie: s 3 3 (a + b + c) a + b3 + c3 = 3 (a + b) (b + c) (c + a) 3 2 ab 2 bc 2 ca = 24abc = 24. b) Soluia I (Irina Mustaa, eleva, Iai). Prin nmulirea ultimelor do t t t s teze din partea dreapt a relaiei (1) i a t s tinnd cont c ab + bc + ca = 1 a s (a + b + c)3 a3 + b3 + c3 = 3 (a + b) c2 + 1 ; similar, avem i (a + 3 2 3 3 3 3 3 3 s a + b + c = 3 (b + c) a + 1 i (a + b + c) a + b + c = 3 (c + a Prin adunarea acestora avem (a + b + c)3 a3 + b3 + c3 = (a + b) c2 + 1 + (b + c) a2 + 1 + (c + a) = 2 (a + b + c) + ab (a + b) + bc (b + c) + ca (c + a) = = 2 (a + b + c) + (a + b + c) (ab + bc + ca) 3abc, adica

3 (a + b + c) a3 + b3 + c3 = 3 (a + b + c) 3abc. Observam ca din 3 = 3 (ab + bc + ca) (a + b + c)2 rezulta ca a + b + c 1 3 2 b2 c2 deducem c abc . Revenind la (2) v a din 1 = ab + bc + ca 3 a 3 3 3 1 8 3 3 (a + b + c) a + b3 + c3 3 3 = . 3 3 Soluia II (Marius Pachiariu, elev, Iai). Cum ab + ac + bc = 1, av t t s 2 3 3 3 a + b + c 3abc = (a + b + c) a + b2 + c2 ab ac bc = i h 2 3 = (a + b + c) (a + b + c) 3 (ab + ac + bc) = (a + b + c) 3 (a + b 8 Astfel, inegalitatea de la punctul b) se va scrie 3 (a + b + c) 3abc 3 8 3 a + b + c abc . 9 Pentru a justifica inegalitatea (2), vom demonstra dubla inegalitate: r ab + bc + ca a+b+c 3 abc, a, b, c > 0. 3 3 Pentru prima parte a relaiei (3), observam ca t 2 a+b+c ab + bc + ca 2 (a + b + c) 3 (ab + ac + bc) 3 3 evident adevarata. Pentru partea a doua, folosim inegalitatea mediilor: r r q 2 ab + bc + ca 3 3 3 ab bc ca = abc = abc. 3 Revenind la inegalitatea (2), avem: r !3 r 8 3 ab + bc + ca ab + bc + ca = a + b + c abc 3 3 3 9 Soluia III (data de autor). Din inegalitatea lui Carlson: t r r ab + bc + ca 3 (a + b) (b + c) (c + a) , a, b, c > 0 8 3 i identitatea (1), rezult c: s a a (a + b + c)3 a3 + b3 + c3 = 3 (a + b) (b + c) (c + a) r !3 1 8 3 ab + bc + ca =38 = 38 . 3 3 3 3 (a b)2 + (b c)2 + (c a)2 0,

VIII.38. Fie n N fixat. Aratai ca exista o infinitate de numere t astfel nct x2n + y 2n + z 2n = x2n+1 + y 2n+1 + z 2n+1 . Lucian Tuescu t

Soluie. Dac lum z = y, atunci din relaia dat, obinem: t a a t a t 2n 2n 2y = x (x 1) . O soluie a acestei din urm ecuaii putem gsi alegnd x 1 = 2a2n t a t a 2n ntr-adevr, n acest caz egalitatea (1) devine 2y 2n = 2a2n + 1 a 2a2n 2n gsim y = a 2a 1 . Deci, exist o infinitate de numere cu propriet a + a x = 1 + 2a2n , y = a 2a2n + 1 , z = a 2a2n + 1 , a Z . VIII.39. Fie ABCD un patrulater strmb cu [AD] [BC]. Sa se co dreptele paralele d1 , d2 , d3 , d4 astfel nct A d1 , B d2 , C d3 , D dist (d1 , d4 ) = dist (d2 , d3 ). Horia Mihail Teodorescu, Soluie. Fie d o dreapt care face unghiuri egale cu AD i BC (evide t a s gasi o astfel de dreapta). Dreptele d1 , d2 , d3 i d4 , duse prin A, B, C s D i paralele cu d, satisfac condiiile problemei. ntr-adevar, daca notam s t picioarele perpendicularelor din A i B pe d4 , respectiv d3 avem c 4AED s a (I. U.), deci AE = BF , adic dist (d1 , d4 ) = dist (d2 , d3 ). a VIII.40. Fie ABCDA0 B 0 C 0 D0 un cub, iar O (BB 0 ). Dreptele A intersecteaza (ABC) n E, respectiv F , iar AO si CO intersecteaza (A E 0 , respectiv F 0 . a) Aratai ca EF E 0 F 0 nu depinde de poziia lui O; t t t b) Aratai ca SBB 0 E 0 E SABCD si determinai O pentru care se atinge t Monica Ned Soluie. a) Cum (A0 B 0 C 0 ) k t E (ABC) i (EOF ) (A0 B 0 C 0 ) = A0 C 0 , s C B (EOF ) (ABC) = EF , rezulta ca F 0 0 0 0 EF k A C , deci 4A OC 4EOF , D A de unde deducem c a EF EO BO O = = 0 . (1) A0 C 0 OA0 BO Analog, putem demonstra ca 4AOC C B 4E 0 OF 0 , deci 0 0 0 0 EF EO BO = = . (2) A D AC OA OB 0 0 EF E F = 1, deci EF E 0 F 0 = AC 2 = const. Din (1) i (2), obinem s t AC A0 C 0 0 0 BE x BE ax i b) Fie B 0 O = x. Atunci, avem = s = . De aici a ax a x BB 0 (B 0 E 0 + BE) a2 x ax a2 SBB 0 E 0 E = = + 2 = a2 = 2 2 ax x 2 x a Egalitatea are loc dac i numai dac as a = 1, adic x = , ceea ce n a ax 2 O este mijlocul segmentului [BB 0 ].

Clasa a IX-a

y IX.36. Determinai x < 0 < y astfel nct xy + = y 3 5y + 2. t x Cezar Lupu, elev, C

Soluie. Ecuaia dat este echivalent cu: t t a a 2 1 x + + 5 = y2 + . x y 1 Cum x < 0, rezult c x + + 5 2 + 5 = 3, cu egalitate numai pentr a a x Pe de alta parte, avnd n vedere ca y > 0, putem scrie: r 2 1 1 1 1 2 2 y + = y + + 3 3 y 2 = 3, y y y y y cu egalitate numai pentru y = 1. Aadar, egalitatea (1) este posibil dac s a 2 1 s daca x + + 5 = 3 = y 2 + , adica pentru x = 1 i y = 1. x y t IX.37. Pentru x [1, ), n N , demonstrai inegalitatea n n+1 + 1 (x 1) 2nxn (x 1) . x Marius Pachiariu, t Soluia I. Inegalitatea dat se transform succesiv astfel: t a a x2n+1 xn+1 + xn 1 2nxn+1 2nxn x2n+1 1 (2n + 1) xn+1 xn

Inegalitatea din enun rezulta imediat din aceasta. t 1 2 1 + q = IX.39. Sa se rezolve ecuaia q t [x] [x + 2] 2 [x]3 3 3 [x] [x + 1]3 Daniel Jing Soluie. Ecuaia are sens daca [x] > 0, adica [x] 1. Daca face t t t a [x] = y N , ecuaia dat devine: 1 1 2 . + = 2y y 3 (y + 1) 3 y y (y + 2)

Inegalitatea (1) este adevarata pentru x = 1, iar pentru x > 1 este echi 1 + x + x2 + + x2n x2n+1 1 n a (2n + 1) x sau xn , care rezult d x1 2n + 1 tatea mediilor n felul urmtor: a (2n+1)2n 1 + x + x2 + + x2n 2n+1 1 x x2 x2n = x 2(2n+1) = xn . 2n + 1 Soluia II (Irina Mustaa, elev, Iai). Prin inducie complet. t t a s t a y n+1 z n+1 xn+1 + n + n x + y + z, x, y, z > 0, IX.38. Sa se arate ca yn z x Gigel Buth, Sa Soluie. n GM - 4/2002, p. 146, L. Panaitopol enuna i demonstreaz t t s a urmtor: a Daca p 1 si ai 0, bi > 0 pentru i 1, n, atunci p Xn n ai X ap i Xni=1 p1 . bp1 i i=1 b
i=1 i

y+1 y+2 Deoarece y = y 1 (2) i 3 y = 3 y 1 1 s (3), 2 3 1 1 1 1 2 + = . Prin urma + 2y y 3 (y + 1) 3 y y (y + 1) (y + 1) (y + 2) y (y + 2) (1) are soluie dac i numai dac (2) i (3) sunt simultan egaliti, adic y t as a s at a soluia ecuaiei date este x [1, 2). t t IX.40. Fie M 6= G n planul 4ABC si D, E, F mijloacele latur [CA] si respectiv [AB]. Consideram punctele X, Y, Z astfel nct XD Y E = mY M , ZF = mZM , m 6= 1. 3 a) Daca m 6= , atunci AX, BY, CZ sunt concurente n S, cu SG = 2 3 b) Daca m = , atunci AX, BY, CZ sunt paralele cu GM . 2 Virgil Nicula, B Soluie. a) Avem: t 2m 2m 2m 3 SM SM + M G = SM SM = M G SG = 3 3 3 3 1 MS = MG MS = MA + MB + MC . 3 2m 3 2m 1 MA + MB + MC Fie punctul S 0 definit prin egalitatea M S 0 = 3 2m t verifica, prin calcul, faptul c S 0 aparine dreptelor AX, BY , CZ, deci a a 2m 0 s a a s fi concurente n S S i atunci este adevrat i egalitatea SG = 3 demonstram, de exemplu, ca S 0 AX. Pentru aceasta vom demonstra 0 0 XS i S A sunt coliniari: s M A + M B + M C 0 0 MB + MC XS = M S M X = = 3 2m 2 2m (2 2m) M A M B M C , = (3 2m) (2 2m) 0 0 M A + M B + M C S A = MA MS = MA = (2 2m) XS 0 3 2m 3 b) Pentru m = , avem: 2 3 3 XD = XM XM + M D = XM M X = 2M D = (M B + 2 2 i atunci XA = M A M X = M A + M B + M C = 3M G. Analog s Y B = ZC = 3M G, deci dreptele AX, BY , CZ sunt paralele.

Clasa a X-a

X.36. Sa se rezolve inecuaia alogb x + xlogb x a + b, unde a, b (1, t Daniela Dodan, e 2 Soluie. Din egalitatea x = blogb x , x > 0, rezult c xlogb x = blogb t a a Deci, inecuaia data este echivalenta cu t alogb x + blogb x a + b.
2 2

X.38. Fie a, b, c, d R cu a > b > c > d. Sa se arate ca a, b, c, d sunt n 3 ad aritmetica daca si numai daca (a b) (b c) (c d) = . 3 A. V. Mihai, B Soluie. Dac a, b, c, d sunt n progresie aritmetic de raie r, atunci t a a t 3 3r data este echivalenta cu r r r = , care este, evident, adevarata. 3 p Reciproc, dac are loc egalitatea din enun, atunci ad = 3 3 (a b) (b a t p sau (a b) + (b c) + (c d) = 3 3 (a b) (b c) (c d), adica media ar numerelor a b, b c i c d este egala cu media lor geometrica. De aici s a b = b c = c d, deci a, b, c, d sunt n progresie aritmetic. a

X.37. Fie a, b (0, 1) (1, ) si funcia injectiva f : (0, ) R a t funcia g : R R, g (x) = f (ax ) + f (bx ) este constanta. Sa se arate ca t ca exista funcii f care satisfac ipotezele problemei. t Dan Popescu Soluie. Fie g (x) = f(ax ) +f (bx ) = k, x R, unde k R. Atu t k = (x) + f bloga x = f alogb x + f (x), x > 0, de unde rezulta ca f f = f alogb x . Cum f este funcie injectiv, deducem c bloga x = alogb x , x t a a g (x a a log2 b = 1, adic a = b sau ab = 1. Dac a = b, atunci f (ax ) = a 2 k sau f (x) = , x > 0, ceea ce contrazice injectivitatea funciei f . Pentru t 2 t f (x) = loga x, se obine g (x) = 0, x R.

Dac facem notaia log2 x = 0 i avem n vedere obsevaiile > 1 a t s t b > a + b, 1 + b a + b, obinem ca inecuaia (1) este echi a t t log2 x 1, deci x b1 , b . b

X.39. Fie ABCDA0 B 0 C 0 D0 un paralelipiped dreptunghic cu dimensiuni AD = b, AA0 = c. Daca M Int A0 B 0 C 0 D0 , notam cu , , masurile u pe care AM le face cu AB, AD si respectiv AA0 . Sa se arate ca AM < a cos + b cos + c cos < AC 0 .

Ctlin Cali a a Soluie. Fie E, F i G proieciile punctului M t s t D pe laturile AB, AD i respectiv AA0 . Astfel avem s AE AF AG i cos = cos = , cos = s , de A B AM AM AM unde deducem ca AE cos + AF cos + AG cos = M AE 2 + AF 2 + AG2 AM 2 = = = AM . De aici, G AM AM D avnd n vedere ca AE < AB = a, AF < AD = b, 0 F i cos > 0, cos > 0, cos > 0, AG < AA = c s rezult c AM < a cos + b cos + c cos . Pentru a a A E B a doua parte a inegalitaii vom folosi inegalitatea lui t Cauchy-Buniakovski-Schwarz i identitatea cos2 + cos2 + cos2 = 1: s p p a cos + b cos + c cos < a2 + b2 + c2 cos2 + cos2 + cos2 =

b) Vom aplica inegalitatea de la punctul a) pentru x = pa, y = pb s Cu notaiile facute, avem: t

rezult c a a p x + y + x + z + y + z xy + xz + yz 3 6 xyz = 3 6x

Marian Tetiv Soluie. a) Din relaiile t t p p x + y + x + z + y + z 3 6 (x + y) (x + z) (y + z) 3 6 8xyz = 3 q p p xy + xz + yz 3 3 x2 y 2 z 2 = 3 6 x2 y 2 z 2

b) Cu notaiile uzuale, n orice triunghi are loc inegalitatea t 2 2 2 a b + ( a c) + b c R 9 . 2 2 r 4 a+ b+ c

X.40. a) Pentru x, y, z 0, demonstrai inegalitatea t p x + y + x + z + y + z xy + xz + yz 3 6xyz.

S2 = pr2 . p p 2 a+ b+ c r + 4Rr 3 Astfel, inegalitatea de la a) devine 2 2 R 27 (a + b + c) a+ b+ c r + 4Rr 54pr2 , deci 1 + 4 r a+ b+ c obinem c t a a+ b+ 9 R 27 (a + b + c) 9 3 (a + b + c) = 2 2 4 2 r 4 4 a+ b+ c a+ b+ c 2 2 2 a b + ( a c) + b c 9 . = 2 4 a+ b+ c xyz = (p a) (p b) (p c) =

x + y = c, x + z = b, y + z = a, X X xy + xz + yz = (p a) (p b) = p2 (a + b) p + ab = X = 3p2 4p2 + ab = p2 + p2 + r2 + 4Rr = r2 + 4Rr,

Clasa a XI-a

XI.36. Fie D, M doua matrice nesingulare de ordin n, D diagona triunghiulara. Daca D = t M DM , sa se arate ca M este tot o matrice avnd 1 pe diagonala principala. Adrian Cordune Soluie. t

i Mii = i mii , i = 1, n. Din d Mii 1 pentru i < j, deducem ca M este matrice diagonala i atunci = s d mii a deci avem m2 = 1, i = 1, n, adic mii = 1, i = 1, n. Cazul n care M es ii triunghiulara se trateaza n mod analog. s rezulta ca mij = 0, oricare ar fi i < j i

1 m11 m12 . . 0 ... 0 0 0 2 . . . 0 m22 . . Fie D = . . . . . . . . . . . ., cu i 6= 0, i = 1, n, M = . . . ... .. 0 0 . . . n 0 0 .. mii 6= 0, i = 1, n i Mij complementul algebric al lui mij n matricea M . s d = det M 6= 0. Relaia dat este echivalent cu DM 1 = t M D. Deoarec t a a 0 ... 0 M11 M21 . . . Mn1 1 1 0 2 . . . 0 0 M22 . . . Mn2 = DM 1 = . . . . . . . . . . . . . . . ... ... ... d 0 0 . . . n 0 0 . . . Mnn 1 M11 1 M21 . . . 1 Mn1 1 0 2 M22 . . . 2 Mn2 i s = ... ... ... ... d 0 0 . . . n Mnn 0 ... 0 0 ... 0 1 m11 m 2 . . . 0 m22 . . . 0 0 t = M D = 12 ... ... ... . . . . . . . . . . . . . . . m1n m2n . . . mnn 0 0 . . . n 0 ... 0 1 m11 1 m12 2 m22 . . . 0 = ... ... ... ... 1 m1n 2 m2n . . . n mnn

XI.37. Fie A M3 (C) astfel nct det (A + tA) = 0, unde R\ 2 ( 1)2 det A. Sa se arate ca det (A + tA) = Marian Ionescu, Piteti i Lucian Tuescu s s t t a Soluie. P (x) = det (A + x tA), x C este o funcie polinominal t mai mic sau egal cu 3. Deoarece 1 1 P (x) = det x = x3 det A + tA A + tA = x x 1t 1 = x3 det A + A = x3 P , x C , x x

rezult c P este polinom reciproc, deci P (x) = x3 + ax2 + a a a (det A) 1 Cum, prin ipotez, P () = 0, nseamn c i P a a a s = 0. De aici, 1 vedere c P (1) = 0, obinem c P (x) = (det A) (x ) x a t a (x + 1

1 2 ( 1)2 det (A + tA) = P (1) = (det A) (1 ) 1 2= det A.

i x 0, avem c fk (x) + 2fk1 (x) = 3fk2 (x), k = 2, n, de unde pr s a deducem c fn (x) + 3fn1 (x) = f1 (x) + 3x, n 3, x 0. De aici, ob a t f3 (x) + 3f2 (x) = f1 (x) + 3x, f4 (x) + 3f3 (x) = f1 (x) + 3x, f5 (x) + 3f4 (x) = f1 (x) + 3x, .................................... fn (x) + 3fn1 (x) = f1 (x) + 3x, n 3, x 0. 0 1 1 1 , a doua cu , Mai departe, nmulind prima ecuaie cu t t 3 3 2 1 etc. i apoi adunndu-le, gsim relaia s a t 3 n3 2 1 1 1 fn (x)+3f2 (x) = (f1 (x) + 3x) 1 + + + + 3 3 3

XI.38. Sa se determine funciile continue f : [0, ) [0, ) p t f (f (x)) + 2f (x) = 3x, x 0. Mihail Bencze Soluie. Fie fn (x) = f f f (x), x 0, n N . Pentru or t {z } |
n ori

sau n3 n2 ! 3 1 1 fn (x) + 9x 6f (x) = (f1 (x) + 3x) 1 , n 3 4 3 Din ipoteza rezulta ca f (x)

(x ) R are proprietatea ca xn xn+1 (1 + xn yn+1 ), n 1, arata + n n1 1 este convergent. xn n1 Gheorghe Molea, Curtea t Soluie. Deoarece xn > 0, n 1, rezulta ca inegalitatea din enun e t

n 3x 3 x, , x 0, i atunci fn (x) s 2 2 x fn (x) n , n N , x 0, adic a x 0. De aici, obinem c 0 t a 3n 2 fn (x) = 0, x 0. lim n 3n 3 Din (1) i (2), rezulta ca 9x6f (x) = (f (x) + 3x), x 0, deci f (x) = s 4 Observaie. Nu este nevoie de continuitatea funciei f . t t n P yi este converg XI.39. Fie sirul (yn )n1 astfel nct sirul
i=1 n1

lent cu a

Cum n dreapt a ultimei relaii este un ir convergent, deci mrgin partea a t s a 1 este i el mrginit. s a c irul as xn n1 n n1 P P 1 1 yn = yi yi este echi Pe de alt parte, relaia a t xn xn1 i=1 i=1 n n1 n P P P 1 1 1 yi yi , n 2, sau, cu notaia zn = t yi , z xn i=1 xn1 xn i=1 i=1 n 2. De aici, avnd n vedere ca irul (zn )n1 este marginit, fiind d s dou iruri mrginite, rezult c irul (zn )n1 este convergent. Prin urma as a a as n P 1 = zn + yi este convergent. termenul general xn i=1 XI.40. Fie x0 [1, 1]; aratai ca pentru orice n N, ecuaia 3x4x t t 1 1 o singura soluie xn+1 , . Demonstrai ca sirurile (xn )n0 si t t 2 2 sunt convergente si calculai limitele lor. t Marian Tetiv Soluie. S artm, pentru nceput, c dac a [1, 1], atunci ecuaia 3 t a aa a a t 1 1 are o singur soluie n intervalul , . Pentru aceasta, consider a t a 2 2 1 f : R R , definit prin f (x) = 3x 4x3 a. Deoarece f a 2 = (1 a) (1 a) 0 i f este continua, rezulta ca f se anuleaza s c 1 1 1 1 0 2 data n intervalul , . Cum f (x) = 3 1 4x 0, x , , 2 2 2 2 1 1 ca f este strict crescatoare pe , , deci ecuaia f (x) = 0 are o sing t 2 2 1 1 n intervalul , . 2 2 Mai putem observa c, dac = arcsin a, avem 3 sin 4 sin3 = a a 3 3 h i 1 1 i , implic sin , . Deci sin este tocmai solu a s 3 2 2 3 2 2 1 1 3 tervalul , a ecuaiei 3x 4x = a. Astfel, am demonstrat ca, pe t 2 2 1 1 a [1, 1], ecuaia 3x 4x3 = a are o singura soluie n intervalul , t t 2 2 arcsin a sin = sin . 3 3 Revenind la problema noastr, rezult, din cele artate mai sus, c a a a arcsin xn s . De aici, o orice n N, xn+1 este bine definit i xn+1 = sin 3

1 1 yn+1 , n 1, de unde deducem c a xn+1 xn n X 1 1 y2 + y3 + + yn = yi y1 , xn x1 i=1

n 1.

arcsin xn+1 =
n

arcsin xn arcsin x0 , n N. A , n N, deci arcsin xn = 3 3n s lim xn = lim sin (arcsin xn ) = 0 i


n n

lim 3n xn = lim (3n arcsin xn )


n

xn = arcsin x0 . arcsin xn

XII.36. Sa se determine n N, n 2 pentru care ecuaia x2 = x + b t 1 unica n Zn ; rezolvai ecuaia n acest caz. t t Andrei Ned Soluie. Dac b Zn este soluie a ecuaiei x2 = x + b atunci i t a a t t 1, s soluie a acestei ecuaii ((b b)2 = b 2a + b2 = b 2a + b + b = (b b) + t t 1 a 1 c a 1 c a 1 1 a \ ecuaia trebuie sa aiba soluie unica, este necesar sa avem b = b b, sau 2 t t a 1 a 2 ba2 = ba + b sau (2a b 2 = b rezulta ca c 1) Deoarece b = b + 1 implica 4b a a 4b 4, 5, aici, obinem c n = 5 i atunci ecuaia dat are soluia unic b = b t a s t a t a a 3.

Clasa a XII-a

XII.37. Fie (G, +) un subgrup al grupului (R, +). Sa se determine m crescatoare de la (G, +) la (R, +). Dan Stefan Marinescu i Viorel Cornea, H s Daca G 6= {0}, atunci exista x0 G \ {0} i atunci daca notam a s

Soluie. Daca G = {0}, atunci f : G R, f (0) = 0 este funcia caut t t

de unde, trecnd la limit pentru n , obinem a t y + x0 y + x0 ax0 f (y) + ax0 ax0 , x0 x0 deci f (y) = ay. Daca x0 < 0, se ajunge, n mod analog, la acelai rezulta s n sfrit, observm c funcia f : G R, f (y) = ay, a > 0, este u s a a t cresctor de grupuri. a

observam ca a 0. Folosind definiia morfismului de grupuri se poate t prin inducie ca f (nx) = nf (x), n Z, x G. De aici, deducem ca t = nf (x0 ) = nax0 , n Z. Fie un element oarecare y G. Dac x0 > 0, avem succesiv: a n (y + x0 ) n (y + x0 ) n (y + x0 ) < + 1, n N x0 x0 x0 n (y + x0 ) n (y + x0 ) f x0 f (n (y + x0 )) f + 1 x0 , n x0 x0 n (y + x0 ) n (y + x0 ) ax0 n (f (y) + ax0 ) + 1 ax0 , n N x0 x0 1 n (y + x0 ) 1 n (y + x0 ) 1 ax0 f (y) + ax0 + ax0 , n n x0 n x0 n

XII.38. Determinai funciile derivabile f, g : R R astfel nct f 0 (x) t t si g 0 (x) = f (x) x, x R. Gheorghe Iu

x R, C x sau (ex f (x))0 = Ce2x + xex ,x R, deci f (x) = e + k1 ex + x 1 2 C x Analog, obinem g (x) = t a s e k2 ex x + 1, x R. Se verific uor 2 funcii satisfac sistemul de ecuaii dat. t t XII.39. Fie f, g : (0, ) R astfel nct lim f (x) = lim g (x) x x Z 1 g(n) f (x) x lim = R. Sa se calculeze lim f (n) dx, unde [1, x g (x) n 0 x+ Adrian Sandovici, Piatr s Soluie. Din ipoteza rezulta ca exista n0 astfel nct f (n) > 0 i t n n0 . Pentru n n0 , avem: Z 1 g(n) Z x dx f (n) 1 g(n) 0 x In = f (n) x = dx = x+ g (n) 0 x+ 0 # " 1 Z 1 f (n) xg(n)+1 xg(n) = 2 dx . g (n) x + 0 0 (x + ) Deoarece 0 Z
1

Soluie. Adunnd cele dou relaii date, obinem (f + g) (x) = ( t a t t 0 x R, sau (ex (f + g)) (x) = 0, de unde gasim f (x) + g (x) = Cex , x C R este o constanta arbitrara. Revenind la prima ecuaie, avem t f 0 (x) = Cex + x f (x) ,

xg(n) dx

(x + ) 0 Z 1 g(n) x dx i lim g (n) = , rezulta ca lim = 0. Aadar, avem lim In s s n n 0 (x + )2 n XII.40. Fie f : [0, 1] R o funcie derivabila cu derivata continua a t f (x) 0 exista si este finita. Sa se arate xf (x) f (x), x [0, 1], iar lim x0 x x>0 Z Z 1 1 f (x) f (x) dx, f (1) min 2 dx . x 0 0 Marcel Chiria,ZB t Z
0 1

xg(n) dx =

1 g (n) + 1

f (x) exista i este finita, rezulta ca lim f (x) = 0. Astfe Deoarece lim s x&0 x x&0 Z 1 Z 1 Z 1 f (x) f (x) f 0 (x) dx = lim (f (1) f ()) dx = lim dx lim &0 &0 &0 x x 0 ceea ce, mpreun cu relaia (1), conduce la inegalitatea din enun. a t t

Soluie. Din xf 0 (x) f (x), x [0, 1] rezult c t a a xf 0 (x) dx 0 Z 1 Z 1 Z 1 1 f (x) dx f (x) dx, deci f (1) 2 f (x) dx (1). sau xf (x)|0
0 0 0

Soluiile problemelor pentru pregtirea concur t a din nr. 1 / 2003


A. Nivel gimnazial

i din ipoteza, obinem ca x | 10m 1 (10nq 1) = 10nq+r 10nq = 10nq s t Deoarece din x | 10n 1 rezult c (x, 10) = 1, deci (x, 10nq ) = 1, a a precedent deducem c x | 10r 1, ceea ce contrazice ipoteza. Prin urm a a . . 10m 1 . x, atunci m . n. . .

G36. Fie x, n N astfel nct x divide 10n 1, nsa x nu divide 10k . k < n. Sa se arate ca x divide 10m 1 daca si numai daca m . n. . N. N. H . Soluie. Daca m . n, atunci m = 0 sau exista q N astfel nct m t . . . x, iar n al doilea avem: primul caz, avem 10m 1 = 0 . . 10m 1 = (10n )q 1 = (10n 1) ((10n )q1 + + 1) . x. . . . s Sa presupunem acum ca 10m 1 . x, m 6= 0 i m = nq + r, cu 0 < r <

G37. 2n muzicieni (n > 2) participa la un festival. La fiecare conce dintre ei cnta iar ceilali asculta. Sa se determine numarul minim de conc t nct fiecare muzician sa-i asculte pe toi ceilali. t t Titu Zvonaru, B Soluie. Fie a1 , a2 , . . ., a2n cei 2n muzicieni. Dac la un concert, u t a ei asculta pe un coleg care cnta, spunem ca are loc o "audiie". p t As concert la care cnta p muzicieni, exista p (2n p) audiii. Deoarece p ( t p + 2n p a a a = n, adic p (2n p) n2 , rezult c numrul maxim de a 2 loc atunci cnd n muzicieni cnt i n ascult. as a Sa presupunem ca la primul concert cnta muzicienii a1 , a2 , . . ., an . putea fi ascultat de a2 , a3 , . . ., an , muzicianul a1 trebuie sa mai cnte la n care s nu cnte a2 , a3 , . . ., an , apoi a2 trebuie s cnte ntr-un conc a a nu cnt a1 , a3 , . . ., an i aa mai departe. Deci, numrul minim de con a s s a cel puin n + 1. Sa aratam ca acest numar este n + 1 indicnd o aranja t n + 1 concerte astfel nct sa fie ndeplinita cerina problemei. Pentru acea t notaiile: t Ak = {an+1 , an+2 , . . . , a2n } {an+k } , Bk = {a1 , a2 , . . . an } {ak } ,

i repartizm muzicienii astfel: s a Muzicieni care cnta Muzicieni care asculta 1) a1 , a2 , . . ., an an+1 , an+2 , . . ., a2n 2) a1 , A1 B1 , an+1 3) a2 , A2 B2 , an+2 ... ................ .................... n) an1 , An1 Bn1 , a2n1 n + 1) an , An Bn , a2n G38. Mulimea A Z are cinci elemente. Adunnd n toate modur t

cte trei elemente din mulime, obinem urmatoarele 10 sume: 3, 6, 8, 1 t t 15, 16, 18, 20. Determinai mulimea A. (n legatura cu o problema de c t t Iugoslavia.) Gabriel P Soluie. Fie A = {a, b, c, d, e} cu a < b < c < d < e i fie S = a + b + t s Adunnd cele 10 sume din enun obinem 120. Deoarece n aceasta su t t element al mulimii A apare exact de 6 ori, rezult c 6S = 120, dec t a a Prin urmare, adunnd n toate modurile posibile cte dou elemente din a obinem urmatoarele 10 rezultate: 17 (= 20 3), 14 (= 20 6), 12, 10, t 2, 0. Acum, din a + b = 0 i a + c = 2, gasim b = a i c = 2 a. s s e + d = 17 i e + c = 14, deducem c d = e + 17 i c = e + 14, deci e s a s Aadar, putem scrie A = {a, a, a + 2, a + 5, a + 12}. A treia sum s a crescatoare) este a + d sau b + c i este egala cu 4. Cum a + d = a a + 5 = s doar b + c = a a + 2 = 2a + 2 = 4, de unde rezulta ca a = 1. n avem A = {1, 1, 3, 6, 11}. G39. Fie xi R, i = 1, n, unde n 2003, astfel nct x1 (n + 1) x2 + nx3 n 1 ................................................ xn2 (n + 1) xn1 + nxn n 1 xn1 (n + 1) xn + nx1 n 1 n2 xn (n + 1) x1 + nx2 2n 1. Daca x1 = 1, sa se calculeze x2003 .

nmulind membru cu membru relaiile din sistemul (2), obinem t t t

de unde, prin adunare, obinem 0 0. Aadar, toate inegalitile din si t s at trebuie sa fie egalitai, adica t y1 y2 = n (y2 y3 ) , y2 y3 = n (y3 y4 ) , . . . , yn y1 = n (y1 (y1 y2 ) (y2 y3 ) . . . (yn1 yn ) (yn y1 ) = = nn (y1 y2 ) (y2 y3 ) . . . (yn1 yn ) (yn y1 ) ,

y1 (n + 1) y2 +ny3 0, y2 (n + 1) y3 +ny4 0, . . . , yn (n + 1) y1 +ny

Romeo Ce Soluie. Cu notaiile yk = xk k + 1, k = 1, n, sistemul dat devine: t t

de unde rezulta ca una din paranteze este zero i atunci conform cu (2), s = y2 = = yn . Prin urmare, 1 = x1 = x2 1 = x3 2 = = xn n x2003 = 2003. G40. Comparai numerele reale a si b, stiind ca a2 14a + b2 + 6b + t Bogdan Rducanu, a Soluia I (un grup de elevi de la Colegiul Naional din Iai i A t t s s Negrescu, elev, Botoani). Relaia din enun se scrie (a 7)2 + (b + s t t Deducem c |a 7| 5 i |b + 3| 5, de unde a [2, 12] i b [8 a s s b 2 a i cum a 6= b (prin vericare direct), rezult c a > b. s a a a Soluia II. Dac facem notaia a = b + t problema revine la comparar t a t zero. nlocuind pe a n egalitatea dat, obinem 2b2 + 2b (t 4) + t2 14t a t ecuaie care are soluie daca i numai daca = t2 8t + 16 2t2 + 2 t t s

= t2 + 20t 50 0. De aici, rezult c t 10 5 2, 10 + 5 2 , d a a Aadar, avem a > b. s G41. Daca 0 < x y z, sa se arate ca x z y2 z2 y x y z x z x2 3 + + + + + 1+ 2 + 2 + 2. z y x y z x z x y z x Ovidiu Pop, Sa Soluie. Prima inegalitate rezulta din inegalitatea mediilor. Inegalita t este echivalent cu x2 y + z 2 x + y 2 z x2 z + y 2 x+ z 2 y, sau (y x) (z x) (z a care este adevrat n virtutea ipotezei 0 < x y z. Inegalitatea a a a x y x echivalenta cu + + 1, adica (z y) (x y) 0, care este adev y z z sfrit, pentru a demonstra ultima inegalitate vom folosi din nou inegalit s 2 2 2 2 2 2 y x y z z x x + 2 , + + 3, + ilor. Avem: y z z y z x x x 2 2 2 x y z z Adunnd aceste relaii, obinem 2 t t + 2 + 2 2 +1+ , q.e 2 y z x z x G42. Determinai a, b R, daca [x] + [x + a] = [bx], x R. t Gheorghe Iu Soluie. Lund x = 0 n egalitatea data gasim [a] = 0, deci a [0, 1). D t x 1 + x + a 1 < [x] + [x + a] = [bx] bx, x R, rezult c a 2 a a x R, deci b = Aadar, avem [x] + [x + a] = [2x], R, de un 2. s x 1 1 n vedere c [x] + x + a = [2x], deducem c [x + a] = x + , x a 2 2 1 1 1 3 aici, lund x = , obinem a + t [1, 2), adic a a , [0, 1) 2 2 2 2 1 1 i avem 0 < Dac a a , 1 , atunci putem alege un x0 1 a, s 2 2 1 a < 1 < x0 + a. n acest caz ns x0 + = 0, iar [x0 + a] 1, deci rela 2 1 este valabil. Prin urmare, avem a = , b = 2, valori care verific egalitat a a 2 d G43. Fie xOy un unghi oarecare si P un punct n interiorul sau. Se punctele A, B [Ox cu A (OB) si C, D [Oy cu C (OD) astfel nct rile P AB si P CD sa fie echilaterale. Aratai ca dreptele OP , AD si t concurente daca si numai daca P se afla pe bisectoarea unghiului dat. Temistocle B Soluie. Fie OP BD = {M }, P R CD, t P Q AB (R CD, Q AB) i P Q = d1 , P R = d2 , s R OP = d. Avem: C d2 q MB OB d1 OB sin 1 = , OQ = d2 d2 , = 1 MD OD sin 2 OD d2 q 2 d OA = OQ AQ = d2 d2 d1 / 3, 1 1 q OB = OQ + QB = d2 d2 + d1 / 3, 1 O A

q q q OR = d2 d2 , OC = d2 d2 d2 / 3, OD = d2 d2 + d2 1 2 2 Cu aceste observaii, putem scrie succesiv: OP , AD i BC sunt concuren t s M B CD AO OB d1 2d2 / 3 OA = 1 OA OB = O =1 M D CO AB OD d2 OC 2d1 / 3 q q q q 2 d2 d / 3 2 d2 + d / 3 = 2 d2 d / 3 d d d d2 d2 1 1 2 1 1 2 2 d2 d2 d2 /3 = d2 d2 d2 /3 d1 = d2 1 = 2 . 1 1 2 2

G44. Fie V ABC o piramida, iar G centrul de greutate al 4ABC ce trece prin G taie dreptele V A, V B, V C n A0 , B 0 si respectiv C 0 . Sa s VB VC VA + + = 3. V A0 V B0 V C0 Constantin Co

Soluie. Fie {N } = B 0 C 0 BC i {M } = A0 C 0 AC. t s V Aplicnd teorema lui Menelaus n triunghiurile V AC A0 A C 0V MC i V BC, obinem: s t 0 = 1 i s 0V A CC MA 0 0 B B C V NC M A = 1, de unde rezult c a a B0V C 0C N B G CC 0 AM NB CC 0 A0 A B 0 B + = sau + = N B 0 0 0 0 VA VB V C MC NC VC V A0 V A V B 0 V B V C V C0 + = sau A 0 0 VA VB V C0 B VA VB VC AM BN + + = 3. (Am folosit c G M N implic a a + = V A0 V B0 V C0 MC NC G45. Fie SABC un tetraedru n care 4ABC nu este echilateral, i ile [SA] , [SB] , [SC] nu sunt toate congruente. Demonstrai ca exista sa t A1 , B1 , C1 , A2 , B2 , C2 pe dreptele SA, SB, SC, BC, AC si respectiv AB a trulaterele A1 B1 A2 B2 , B1 C1 B2 C2 si A1 C1 A2 C2 sa fie trapeze isoscele (A1 A1 C1 kA2 C2 , B1 C1 kB2 C2 ) daca si numai daca SA2 AB 2 AC 2 + SB 2 BC 2 BA2 + SC 2 CA2 CB 2 =

Daly Marciuc, Sa Soluie. S presupunem c A1 B1 A2 B2 , B1 C1 B2 C2 i A1 C1 A2 C2 su t a a s isoscele n modul indicat. Din A1 B1 k A2 B2 rezulta ca B2 A2 k AB i ap s rezulta ca B2 C2 k BC i A2 C2 k AC. De aici, deducem ca AB2 A2 C2 i s s sunt paralelograme, deci C2 este mijlocul lui AB. Analog, obinem c B2 t a mijloacele laturilor AC i BC. s Din A1 B1 k AB, A1 C1 k AC i B1 C1 k BC rezulta ca s A1 A B1 B C1 C = = = k. SA SB SC 2 2 Notnd BC = a, AC = b i AB = c, avem: A1 B2 = A2 B1 s 2 2 2 2 2 b a SA + b SC SB 2 + a2 S k 2 SA2 + k = k 2 SB 2 + k 4 2 4 2 2 2 A1 B2 = A2 B1 2k SB 2 SA2 = a2 b2 .

n mod analog, gsim echivalena: a t

n fine, din (2) i (3) rezult c s a a 2 2 SA c b2 + SB 2 a2 c2 + SC 2 b2 a2 = 0. Reciproc, relaia (4) poate fi scrisa astfel: t a2 b2 c2 b2 not = k. = 2 (SB 2 SA2 ) 2 (SB 2 SC 2 )

2 2 B1 C2 = C1 B2 2k SB 2 SC 2 = c2 b2 .

Alegem A1 , B1 , C1 pe SA, SB, SC astfel nct s avem relaia (1). n ac a t (5) rezulta ca A1 B1 A2 B2 i B1 C1 B2 C2 sunt trapeze isoscele, unde A2 , B2 s mijloacele laturilor BC, AC i AB (A1 B1 k AB k A2 B2 etc.). Daca A1 B s a a B1 C1 B2 C2 sunt trapeze isoscele nseamn c A1 A2 = B1 B2 i B1 B2 = C s A1 A2 = C1 C2 , adic i A1 C1 A2 C2 este isoscel. as

B. Nivel liceal

L36. Fie 4ABC si M triunghiul sau median. Daca P este un pun interiorul sau pe laturile lui M, iar A0 , B 0 , C 0 sunt interseciile dreptelor t 1 AP BP CP CP cu laturile BC, CA si respectiv AB, atunci < 4 AA0 BB 0 CC 0 Marian Ionesc s Soluie. Notm S1 = (P BC), S2 = (P CA), S3 = (P AB) i S = t a S2 + S3 AP i analoagele i se deduce = Se stabilesc cu uurina relaia s t t s s AA0 S AP BP CP Gergonne + + = 2. Cu inegalitatea mediilor AA0 BB 0 CC 0 r AP BP CP 2 33 , de unde deducem a doua parte a dublei ineg AA0 BB 0 CC 0 enun. Pentru prima parte, observm mai nti c, dac P se afl n interio t a a a a laturile triunghiului M, au loc inegalitile S2 +S3 S1 , S3 +S1 S2 i S1 at s 1 1 1 Notnd x = (S2 + S3 S1 ), y = (S3 + S1 S2 ), z = (S1 + 2 2 2 t = x + y + z i observnd ca x 0, y 0, z 0 (numai unul poa s t > 0, avem:

L37. Fie cercurile C1 , C2 si C astfel nct C1 si C2 sunt tangente e D, iar cercurile C1 si C2 sunt tangente interior lui C n B, respectiv C.

AP BP CP (S2 + S3 ) (S3 + S1 ) (S1 + S2 ) = = 0 BB 0 CC 0 AA S3 t3 + t2 (x + y + z) 2t3 (t + x) (t + y) (t + z) > = 3 = 3 3 8t 8t 8t Not. Soluie corect s-a primit de la Marius Pachiariu, elev, Iai a t a t s Not. Aceasta problema apare n articolul "About elementary inequali a angle" (M. Dinc, M. Bencze) din revista Octogon Math. Magazine, a no. 1B, p. 472. Aici nu se cere ca punctul P s fie n interiorul sau pe l a unghiului M, dar soluia prezentat este incorect. t a a

43 a3 4a3 i, innd seama de (2) i (3), gsim 4 2 1 a2 + cos B s t s a c b Din nou utiliznd teorema cosinusului, obinem t 2 1 2b2 c2 + 2 b2 a2 + c2 b2 c2 a2 + b2 c2 = 0 sa b2 a2 b2 c2 2 c2 a2 b2 c2 = 0, c a ultima echivalent cu = sau a2 = b2 + c2 , de unde rezult concluzia a b

comuna interioara cercurilor C1 si C2 taie cercul C n A si A1 , dreapta A 1 2 1 n K, iar AC taie C2 n L. Sa se arate ca = + . DA DA1 KL Neculai Roman, Mirce Soluie. Fie {M } = C1 BC, {N } = C2 BC i T t s un punct pe tangenta n B la cercurile C1 i C2 . s C Artm c dreapta KL este tangenta comun aa a a exterioara cercurilor C1 i C2 . ntr-adevar, avem s K \ \ [ \ m(M KB) = m(M BT ) = m(CBT ) = m(CAB), deci \ \ M K k CA. Ca urmare, M KL KLA. Cum \ CBA, deoarece 4KLA 4CBA (fapt ce \ KLA M decurge din AK AB = AL AC = AD2 ), rezulta ca \ \ \ \ M KL CBA. Deci M KL M BA, adic KL este a B C1 tangenta la cercul C1 . Analog se arata ca drepta KL T este tangenta la C2 . Aplicm teorema lui Casey pentru cercurile C1 , C2 , cercurile degener a tangente interior la C i obinem relaia AD A1 D + AD A1 D = AA1 s t t AA1 2 1 1 2 = , adic a = + . KL AD A1 D KL AD A1 D L38. Fie 4ABC si punctele D, D0 BC conjugate armonic n rapo furile B si C. Cercul circumscris 4ADD0 intersecteaza AB n M si Aratai ca, daca M N BC, atunci [AD si [AD0 sunt bisectoarele un t b (interioara si exterioara) sau m(A) = 90 . Temistocle B Soluie. Avem M N DD0 M D2 + N D02 = M D02 + N D2 (1). D t DB D0 B a = 0 = , atunci BD = , CD = DC DC 1+ a a a , BD0 = , CD0 = (2). Ex= A 1+ 1 1 primnd puterea punctelor B i C faa de cercul s t t t (ADD0 ), vom obine relaiile: c BM = BD BD0 i b CN = CD CD0 sau s B D C 2 a2 a2 i CN = BM = s . (3) c (2 1) b (2 1) s Utiliznd teorema cosinusului n 4BM D, 4CN D0 , 4BM D0 i 4CN D, ( 2 2 2 02 BM + BD 2BM BD cos B + CN + CD 2CN CD0 cos = BM 2 + BD02 2BM BD0 cos B + CN 2 + CD2 + 2CN CD c

L39. Determinai toate numerele naturale nenule n pentru care t patrat perfect, unde a, p N .

an (an p (p

Mihai Ha a2 n2 + 2an 2 Soluie. Fie t = y 2 , y N . Avem (an + 1) p (p + 1) y p (p + 1) unde, cu x = an + 1, obinem ecuaia lui Pell: x2 p (p + 1) y 2 = 1, care t t fundamentala (x0 , y0 ) = (2p + 1, 2) i soluia generala s t k k p p 1 xk = = x0 + y0 p (p + 1) + x0 y0 p (p + 1) 2 k k p p 1 = = 2p + 1 + 2 p (p + 1) + 2p + 1 2 p (p + 1) 2 1 p 2k p 2k = p+1+ p + p+1 p , 2 k k p p 1 x0 + y0 p (p + 1) x0 y0 p (p + 1) yk = p 2 p (p + 1) p 1 2k p 2k p+1+ p p+1 p . = p 2 p (p + 1) Prin urmare, avem: 1 p 2k p 2k nk = p+1+ p + p+1 p 2 2a h i 2k 2k care este soluie dac 2a | t a p+1+ p + p+1 p 2 . Not. Soluie corect s-a primit de la Marius Pachiariu, elev, Iai a t a t s 2 2 L40. Fie A, B Mn (Z) astfel nct det A B + AB este impar. S ca A + B este inversabila pentru orice Q. Marian Ursrescu a Soluie. Deoarece det A2 B + AB 2 = det A det (A + B) det B este t impar, rezult c det A, det (A + B) i det B sunt numere impare. F a a s mul p (X) = det (A + XB) = det A + a1 X + + an1 X n1 + (det B) p (1) = det (A + B) = det A + a1 + + an1 + det B este numar impar, ca i a1 + a2 + + an1 este numar impar. Sa presupunem acum ca s p p are o rdcin raional = , cu p, q Z, q 6= 0, (p, q) = 1. n a a a t a q avem p | det A i q | det B, deci p i q sunt impare. Din p () = 0, s s (det q n + a1 pq n1 + + an1 pn1 q + (det B) pn = 0, sau (det A) q n + (d A) +a1 pn1 q 1 + + an1 pn1 q 1 = (a1 + + an1 ), egalitat fals deoarece membrul din stnga este par, iar cel din dreapta este im a urmare p () = det (A + B) 6= 0, pentru orice numr raional , adic a t a A + B este inversabila oricare ar fi Q. L41. Demonstrai ca grupul simetric S32 nu are elemente de ordin 20 t Paul Georgescu i Gabriel P s Soluie. Presupunem c exist S32 un element de ordin 2002. t a a = 1 2 n descompunerea sa n produs de cicli disjunci cu ordinele k t

kn . Avem k1 + k2 + + kn = 32 i [k1 , k2 , . . . , kn ] = 2002. Cum 2002 = 2 s rezulta ca exista ki1 , ki2 , ki3 , ki4 , nu neaparat distincte, astfel nct 2 | k 11 | ki3 , 13 | ki4 . Daca ki1 , ki2 , ki3 , ki4 sunt distincte, atunci ki1 +ki2 +ki3 + ceea ce este fals. Dac dou, sau mai multe, din cele patru ordine coinc a a ordinul corespunztor se divide cu produsul factorilor ce-i corespund, fiind a sau egal dect produsul acelorai factori i deci mai mare sau egal dect s s Astfel, n acest caz obinem iarai ca suma ordinelor este mai mare sau eg t s ceea ce este fals. L42. Fie (A, +, ) un inel comutativ si finit, cu celpuin 5 elem t 1 + 1 A inversabil. Fie M = x A | x2 = 1 , I = x A | x2 = arate ca card M = card I < card A /2. Ovidiu Munteanu Soluie. Dac a A, atunci 21 (1 + a) A i avem: 21 (1 + a t a s 2 1 1 2 2 2 (1 + a) = 2 (1 + a) 2 1 + 2a + a = 21 (1 + a) 1 + 2 = 2 (1 + a) a = 1 a M , de unde rezult egalitatea card M = card a Sa demonstram acum partea a doua a relaiei date. Daca avem card I = t card A 5 > 2 card I. n continuare ne ocupam de cazul n care card aceast situaie, fie a I \ {0, 1} i atunci 1 a I \ {0, 1, a}. ntr-ad a t s 1a = a, rezult c a = 21 , adic a este inversabil i din a2 = a obinem a a a a s t ce este fals. Avem deci card I > 3. Fie J = {x A | x I} i atunci I s pentru ca x I J nseamna x = x = x2 , deci 2x = 0, adica x = 0. parte, avem I M = {1} i J M = {1}. Cum I, J, M au acelai s s elemente, rezult c are loc card A 3 card I 3 > 2 card I. a a L43. Determinai polinoamele P R [X] pentru care P (z) C\R, z t Gheorghe Iu Soluie. Polinoamele de gradul 1, P (X) = aX + b (a, b R, a 6= t ipoteza, deci sunt soluii ale problemei. Aratam ca acestea sunt singurele t Fie P R [X] cu grad P = n 2 i f : R R, f (x) = P (x) = a0 xn + s + + an (a0 6= 0) funcia polinomial asociat acestuia. Fie a0 > 0 (l t a a proceda dac a0 < 0). Observm c m R ecuaia f (x) = m are num a a a t reale (n soluii), n caz contrar ar exista z C\R i f (z) = m R. t s Daca n este par, atunci lim f (x) = . De aici i din continuitatea lu s
x

f 0 (x) > 0 pentru |x| suficient de mare. Deci f este strict cresctoare pe a (, ) i (, ) (, R convenabil alei). De aici, din continuitat s s f (deci marginirea ei pe orice interval [, ]) i din faptul ca lim f (x s
x+

c Im f = [m, ), unde m = inf {f (x) ; x R}. Pentru k < m ecuaia f ( a t are soluii reale, fals. t Dac n este impar, avem f 0 (x) = na0 xn1 + , deci lim f 0 (x) a
x

deducem ca R astfel nct f () f (x), x (, ]. Ca urmar f (x) = k, cu k > f () au soluie real unic, fals. t a a L44. Fie n 2 numar natural, iar f0 , f1 , f2 , . . . un sir de polinoa 0 prin f0 = (X + 1)n , fp+1 = X fp , p 0. Definim nca hp = fp X p1 p1 + + (1) p1 f1 , p 1, unde n = i1 i2 . . . ik , k {1 k
1i1 <<ik n

sunt sumele simetrice fundamentale ale numerelor 1, 2, . . . , n. Sa se arate hp = n (n 1) (n p + 1) X p (X + 1)np , p = 1, 2, . . . Marian Tetiv p X 0 Soluie. Sa aratam ca hp+1 = Xhp php . Avem hp+1 = t (1)k
k=0 k=0 p1 X k=0 k=0 k=1

p p1 p X X X = (1)k p1 + p p1 fp+1k = (1)k p1 fp+1k + (1)k p p k k1 k k

= X

0 (1) p1 fpk p k

n = 1 i n = 0, pentru k < 0 sau k > n). s k 0 Demonstraia se poate face prin inducie i se bazeaz pe formula stabil t t s a n1 0 din enun, deducem c h1 = f1 = Xf0 = nX (X + 1) t a . S presupunem a are loc egalitatea hp = n (n 1) (n p + 1) X p (X + 1)np . n acest c scrie: np hp+1 = Xh0 php = n (n 1) (n p + 1) pX p (X + 1) + p +n (n 1) (n p + 1) (n p) X p+1 (X + 1)np+1 = n (n 1) (n p + 1) (n p) X p+1 (X + 1)
n n1

k=1

p X

(1)

k1

p1 fp+1k = Xh0 php (am p k1

pn (n 1) (n p + 1) X p (X + 1)np =

np+1

Sa mai observam ca, deoarece hn = n! X , hn+1 = Xn! nX n n! X n = c hp = 0, pentru orice p n + 1. a L45. Fie f : [0, ) [0, ) continua. Daca funcia F : [0, ) R Z t Z x 1 = f (t) dt este marginita, sa se arate ca lim n xf (nx) dx = 0. n 0 0 Adrian Zano Z 1 Soluie. Dac n integrala In = t a nxf (nx) dx facem schimbarea d 0 Z n t nx = t, obinem In = t f (t) dt. Fie (0, 1). Avem 0 n Z n Z n Z n Z n n n f (t) dt + f (t) d f (t) dt + f (t) dt = |In | n 0 n n 0 n Deoarece F este mrginit, exist M > 0 astfel nct F (x) < M , x R+ a a a f este continua, rezulta ca F este derivabila i F 0 (x) = f (x) 0, x R s este crescatoare. De aici, avnd n vedere marginirea funciei F , deducem t Z
x x

lim F (x) i este finita. Prin urmare, lim (F (x) F (x)) = lim s
x

de unde rezult c exist n0 N astfel nct n n0 are loc: a a a Z Z n n f (t) dt = f (t) dt < . n nZ n f (t) dt + < (M + 1) Din relaiile (1), (2), i (3) obinem |In | < t s t 0 ceea ce nseamn c lim In = 0. a a
n

Probleme propuse
Clasele primare

P.64. ntr-o pies de teatru sunt 12 personaje, copii i aduli. Ci cop a s t t pies, dac la fiecare doi aduli corespunde un copil? a a t ( Clasa I ) Alexandra Radu, e P.65. Se dau jetoanele AT II CRE TII ATII RECR

REC . Care este numrul cel mai mare de jetoane cu care se poate form a "RECREATII"? ( Clasa I ) Oxana Pascal, elev, Rep. a

P.66. ntr-o livad sunt tot atia peri ct i meri. Sunt 6 rnduri c a t s rnduri cu meri. Numarul merilor de pe un rnd ntrece cu 5 numarul pe un rnd. Ci pomi sunt n acea livad? t a ( Clasa II-a) nv. Maria R

P.67. Dintr-o mulime de 5 copii, orice grupare de trei conine cel pu t t Ci biei pot fi n mulime? t a t t ( Clasa II-a) Andreea Surugiu, e

P.68. Dac Ina ar mpri numrul nucilor culese de ea la numrul nuc a at a a de sora sa, ar obine 7 rest 6. Stiind c Ina a cules cu 78 nuci mai mult t a sa, aflai cte nuci a cules fiecare. t ( Clasa III-a) nv. Doinia Sp t

P.69. ntr-o mparire cu rest, n care mparitorul este mai mare ca nou t t mparitorul cu o unitate i efectund din nou mparirea obinem ctul s t t t 0. Aflai ctul i restul mpririi iniiale. t s at t ( Clasa III-a) nv. Mariana Toma, Muncelu de S

P.70. ntr-o tabr internaional de matematic sunt elevi din patru aa t a a garia, Grecia, Republica Moldova i Romnia. Dac 21 elevi nu sunt din s a 23 nu sunt din Grecia, 22 elevi nu sunt din Republica Moldova i 21 ele s din Romnia, ci elevi sunt din fiecare ara? t t ( Clasa III-a) Georgiana Ciobanu, e

P.71. Fiecare patrat din figura alaturata se coloreaza cu o alta c cte moduri putem face acest lucru avnd la dispoziie patru culori? t ( Clasa IV-a) nv. Ctlina Raa, Coarnele Cap a a t

P.72. Aruncm dou zaruri i adunm punctele de pe cele dou fee de a a s a a t a) Cte sume diferite putem obine? b) Cte sume se pot forma n tr t diferite? ( Clasa IV-a) nv. Gheorghe Toma, Muncelu de S

P.73. n figura alaturata este pus n evidena un t drum format din ase segmente care pleaca din A i s s ajunge n B. Cte drumuri de felul acesta se pot A construi? ( Clasa IV-a) nv. Constantin Raa, Coarnele Cap t

Clasa a V-a

V.46. Aflai n N pentru care 11n + 9n i 11n 9n sunt simulta t s perfecte. Andrei - Sorin Cozma,

V.47. Sa se arate ca numarul 51a51a nu poate fi scris ca produsul a pat prime. Ctlin Bude a a 9 10 11 V.48. Se consider fraciile x1 = a t , x2 = , x3 = , . . . . Scr 14 21 28 s t a t x1000 i apoi ordonai cresctor primele 1000 de fracii. Dumitru Gherman

V.49. Determinai numarul tripletelor (a, b, c) N3 daca 3a + 2b + t a + 2b + 3c = 602. Dac n plus a < b < c, determinai a, b i c. a t s Gheorghe Iu

V.50. Cte numere de 7 cifre se pot scrie folosind cifrele 1, 2 i 3, ast s sa apara de 2 ori, 2 sa apara de 3 ori i 3 sa apara de 2 ori? Dar daca n lo s 1, 2 i 3 considerm cifrele 0, 1 i respectiv 2? s a s Petru Asa

Clasa a VI-a

VI.46. Suma dintre opusul unui numar natural i inversul altui num s este 119, 992. S se determine numerele. a Ciprian Ba

VI.47. Aflai restul mparirii numarului N = 28442844 + 41074107 + t t prin 79. Tamara C

VI.48. a) ntr-o proporie cu termeni nenuli, un extrem este suma t trei termeni daca i numai daca celalalt extrem are inversul egal cu suma s celorlali trei termeni. t b) Dac din patru numere raionale nenule distincte unul este suma celo a t iar altul are inversul egal cu suma inverselor celorlaltor trei, atunci num termeni ai unei proporii. t Claudiu - Stefan P

VI.49. Sa se arate ca orice numar natural relativ prim cu 10 admite u care se scrie folosind numai cifra 3. Lucian - Georges Ldu a

VI. 50. Fie 4ABC cu [AC] [BC], D mijlocul lui [AB], P un punct AB, iar M i L picioarele perpendicularelor din P pe AC, respectiv BC. S s c [DM ] [DL]. a Neculai Roman, Mirce

Clasa a VII-a

VII.46. Sa se rezolve n R inecuaiile: t a) x100 + x77 + x50 + x21 + x10 + x5 + 1 > 0; b) x100 x77 + x50 x21 + x10 x5 + 2 < 0. Vasile Solcanu, Bogdneti ( a s

VII.47. S se rezolve n Z2 ecuaia u2 v + uv 2 = 2u2 + 2v 2 40. a t Mihai Crciun a VII.48. Dac ai = i + i, i = 1, 2004, precizai dac numrul a t a a

N = a1 a2 a3 + a4 + a5 a6 a7 + a8 + + a2001 a2002 a2003

este negativ, pozitiv sau nul. Viorel Cornea i Dan Stefan Marinescu, H s

VII.49. Fie 4ABC echilateral i D (BC). Notm cu M1 , M2 s a segmentelor [BD], respectiv [CD]. Paralela prin M1 la AC intersecteaza iar paralela prin M2 la AB intersecteaza AC n E. Sa se arate ca dreptele i M2 F sunt concurente. s Nicolae Gross i Lucian Tuescu s t

VII. 50. Fie ABCD un trapez cu bazele [AB] i [CD]. O parale s intersecteaz AD, AC, BD i BC n punctele E, F , G i respectiv H. S a s s c EH = 3F G dac i numai dac DF , CG i AB sunt drepte concurent a as a s Adrian Zano

Clasa a VIII-a

m VIII.46. Sa se demonstreze ca nu exista m, n N pentru care + n Alexandru Negrescu, elev,

VIII.47. Pentru x (0, ), s se demonstreze inegalitatea a 5 3 2 3 2 4 3 x +x +x +1 x +x +2 + x +x +x+1 x3 +x+2 + x3 +x2 +x+1 x x6 + x5 + x4 + 2x3 + x2 + x + 1 Mircea Cobuc, s

VIII.48. Gasii numerele prime p i q pentru care p2 + q = 37q 2 + p. s t Liviu Smarandache

VIII.49. Fie 4ABC dreptunghic n A cu AB = AC = a. Considera \ (ABC), M A = a 2 i N AM astfel nct m(CN, BM ) = 60 . s lungimea segmentului [AN ]. Romana Ghia i Ioan Gh t t s b VIII.50. Fie patrulaterul convex ABCD cu AB = BC, m(A) = m( b s m(B) 90 i fie O mijlocul lui [BD]. Pe perpendiculara n O pe planul ia un punct V astfel nct OV = OB. S se arate c d (D, (V AB)) = 2 d (D a a \ = 60 . dac i numai dac m(ABC) as a Monica Ned

Clasa a IX-a
IX.46. S se rezolve ecuaia a t

x 1 + 3 x 2n x 2 = 2, unde n Dan Popescu a2 = (1) n


n1

IX.47. Sa se determine irul (an )n1 de numere strict pozitive pentru s a2 a2 + a2 + (1) 1 2 3
n1

(a1 + a2 + + an ) , n

Marian Ursrescu a

IX.48. Fie a, b, c (0, ) cu a + b + c + abc = 4. S se arate c a a 2 2 2 b 3 a c + + . 2 a + bc b + ca c + ab Cezar Lupu, elev, C IX.49. Sa se arate ca 4ABC este isoscel n fiecare din ipotezele: a) 2ma + b = 2mb + a; b) 2ma + a = 2mb + b. Marius Pachiariu, t IX.50. Fie I centrul cercului nscris n triunghiul ascuitunghic ABC. D t C sunt masurile n radiani ale unghiurilor triunghiului, iar A IA+B IB+C s se arate c 4ABC este echilateral. a a Constantin Micu, Meline

Clasa a X-a
aib soluii n Z Z. a t X.46. S se determine a R astfel nct ecuaia 2x1 + 2x a t
2

y2 + y2

Petru Rduc a s X.47. Fie z1 , z2 , z3 C distincte, cu z2 +z3 = 2 i astfel nct |z1 1| = = |z3 1|. S se arate c (z1 z2 ) (1 z3 ) este numr complex pur ima a a z a Lidia Nicola X.48. Se considera planele paralele i aflate la distana h unul de s t 4ABC echilateral inclus n planul . a) S se afle locul geometric al punctelor M pentru care M A a = M B2 + M C 2. b) Sa se determine M astfel nct suma M A2 + M B 2 + M C 2 sa f Viorel Cornea i Dan Stefan Marinescu, H s X.49. S se arate c sin3 x + sin3 y + sin3 z 3 sin x sin y sin z a a 3 [sin x (1 cos (y z)) + sin y (1 cos (z x)) + sin z (1 cos (x 4 x, y, z [0, /3]. Marian Tetiv X.50. Fie ak , bk , ck N, k 1, n; notam cu f (p) numarul tripletelor de submulimi (nu neaprat nevide) cu reuniunea M = {1, 2, . . . , n}, or t a P P P disjuncte i astfel nct numrul s a ai + bi + ci p s fi a iM\A iM\B iM\C P de 3 (convenim ca xi = 0). Artai c dac f (0) = f (1) = f (2), atun a t a a i . pentru care ai + bi + ci . 3. . Gabriel Dospinescu, student, B

Clasa a XI-a

XI.46. Determinai A, B Mn (Z) pentru care det (A + B) = 2 i det (A t s Cezar Lupu, elev, C a, dac i = j a XI.47. Fie A Mn (R) matrice cu aij = , unde b 6= 0 b, daca i 6= j Artai c A este inversabil i determinai A1 . a t a as t Gheorghe Iu

a |xn+1 2xn + xn1 | + |xn+1 3xn + 2xn1 | an , n 1. S se arate c este convergent. Paul Georgescu i Gabriel P s n XI.50. Fie n 2 N, iar f : R R o funcie cu proprietatea c f t a t f ( n+1 xn y), x, y R. Sa se arate ca funcia este descrescatoare pe crescatoare pe [0, ). (n legatura cu Problema 2819 din Crux Mathem nr. 2/2003.) Titu Zvonaru, B

XI.48. Se denete irul (xn )n0 prin xn = x2 [xn1 ], n s s n1 0, 1 + 5 /2 . S se arate c lim xn = 0. a a n a Ctlin Tigeru a a P XI.49. Fie (xn )n0 , (an )n0 iruri de numere reale astfel nct s

n=

Clasa a XII-a

XII.46. Sa se determine funcia f : R R daca (R, ) este gru t cu proprietatea c simetricul oricrui element x [1, 1] se afl n [ a a a x y = f (x) + f (y), x, y R. Ioan Scleanu a a XII.47. Fie G = (a, b), a, b R, iar "" nmulirea numerelor rea t determine a, b astfel nct R , (G, ) printr-un izomorsm de forma f = + x + f (x) = , x R+ , cu , , , R. x + Alexandru Blaga i Ovidiu Pop, Sa s XII.48. Fie (G, ) grup de element neutru e i x, y G penrtru care a s a) k N \ {1} a. . xk = e; b) p N \ {1} a. i. xy = y p x. S se arate c: a a 1) xy n xk1 = y np , n N ; 2) xy = yx y n(p1) = e, n N . Mihai Ha XII.49. Se considera numerele reale b > a 0, c 1 i funciil s t Z nb f, g : R+ R+ astfel nct lim g (x) dx = d R. S se arate c iru a as n na Z b 1 un = dx este convergent i sa se afle limita sa. s c + f (x) + g (nx) a D. M. Btineu - Giurgiu, B a t

XII.50. Fie s (n) suma cifrelor numrului natural n. S se calculeze lim a a unde k N este fixat.

Gabriel Dospinescu, student, B

Probleme pentru pregtirea concursurilo a


A. Nivel gimnazial

G56. Fie m Z, n 2 Z + 1 fixate. Sa se arate ca ecuaia nx + y = m t are o unica soluie (x0 , y0 ) cu proprietatea ca |y0 | < |n| /2. t Petru Asa G57. Un eic a lsat motenire celor doi fii ai si cinci cmile, cu c s a s a a unul sa primeasca jumatate, iar celalalt o treime. Motenitorii nu i-au put s s averea, aa ca au apelat la un nelept care trecea pe acolo, calare pe o cam s t a procedat neleptul? t Cte probleme asemntoare mai putem formula (n care motenirea a a s camile, iar fiii primesc a p-a i a q-a parte)? s Gabriel P G58. S se rezolve n N2 ecuaia 2x + 1 = 5y . a t Irina Mustaa, elev, i Valentina Blen t a s G59. Fie A = {n N | s (2000n) + s (2002n) = 2s (2001n)}, unde pri notat suma cifrelor lui x. Demonstrai ca orice numar natural nenul are u t ce aparine lui A. t Gabriel Dospinescu, student, B G60. Sa se demonstreze ca pentru orice a, b, c (0, ) are loc ab bc ca 1 4abc + + + . 4 (a + b) (b + c) (c + a) (a + b)2 (b + c)2 (c + a)2

Gabriel Dospinescu, student, B G61. S se demonstreze c pentru orice a, b, c (0, ) are loc a a p 3 (a2 +b2 )(b2 +c2 )(c2 +a2 ) a+b b+c c+a (a+b)(b+c 54 2 + + 27 c a b abc abc Marian Tetiv G62. Fie ABCD un patrulater convex n care se poate nscrie patratu de centru O (M (AB), N (BC), P (CD), Q (AD)). S s a AB + BC + CD + DA 2 (AO + BO + CO + DO). Cnd are loc egal Lucian Tuescu, Craiova i Ioan Serdean t s b b G63. n 4ABC cu m(A) = 10 i m(B) = 100 construim M s \ \ \ N (AC) astfel ca m(M CB) = 40 i m(N BC) = 75 . S se afle m(AM a s Octavian Bondo G64. Prin punctul P al laturii (AC) a 4ABC se duc paralele la med i CC 0 , care intersecteaz laturile (BC) i (AB) n E, respectiv F . F a s s = EF AA0 , {N } = EF CC 0 , iar L i Q mijloacele segmentelor [F P ] s [P E]. Sa se arate ca dreptele M L, N Q i A0 C 0 sunt concurente. s Andrei Ned G65. Fie SABCD o piramida cu baza ABCD dreptunghi, M proie pe SB i N proiecia lui C pe SA, iar {P } = AM N B. Stiind c M s t a N (SA), s se arate c N P SA M B = SM AN P B. a a Daniel Stefan Ninu,

A. Nivel liceal

L56. Fie ABCD patrulater convex i {P } = AB CD, {Q} = A s Consideram J (AQ), L (BQ), K (DP ), N (AP ) astfel nct Q QL = CB, P K = DC i P N = AB. S se arate c JL k N K. s a a Carmen Nejn L57. Fie 4ABC nscris n cercul C i punctele D (CB, D0 (BC a s \ \ ABC, BAD0 ACB. Se mai considera cercul C1 tangent la AD \ \ CAD C, cercul C2 tangent la AD0 , CD0 i la C, iar {E} = C1 [BD], {F } = C2 s se arate c cercul circumscris 4AEF i cercul nscris n 4ABC sunt conc a s Neculai Roman, Mirce L58. Pe muchiile (Ox, (Oy i (Oz ale unui triedru oarecare se consider s A, L (Ox, B, M (Oy i C, N (Oz astfel nct OA = OB = OC = s \ \ = OM = ON = b (a < b). Notm = m(Oy, Oz), = m(Oz, Ox), = m a i {P } = (AM N ) (BN L) (CLM ), {Q} = (LBC) (M CA) (N A s calculeze distana P Q n funcie de a, b, , , . t t Temistocle B L59. Care este probabilitatea ca latura i diagonalele unui romb, luate s plare, s fie laturile unui triunghi? a Petru M L60. Fie A1 A2 . . . An i B1 B2 . . . Bn (n > 2) dou poligoane nscrise a s cerc de centru O i avnd centrele de greutate tot n O. S se arate c pu s a a merota vrfurile poligonului A1 A2 . . . An pentru a obine un nou poligon Ai1 t n care Aij 6= Bj pentru j {1, 2, . . . , n}. Gabriel Dospinescu, student, B L61. Fie n 3. Sa se determine maximul expresiei E = x3 x2 + x3 1 2 2 2(n1) 3 3 x1 x2 x3 , cnd numerele nenegative x1 , x2 , . . . , xn +x3 x2 +(n1) n 1 n Gabriel Dospinescu, student, B L62. Rezolvai ecuaia 2x2 = y (y + 1); x, y N. t t Mircea B L63. Fie G Mn (R) un grup netrivial n raport cu produsul uzual al m Presupunem ca exista X G astfel nct pe fiecare linie, respectiv coloa existe cel mult un element nenul i acesta egal cu 1. Sa se demonstreze s k {1, 2, . . . n} astfel nct G este izomorf cu un subgrup al lui GLk (R) GLn (R) = {A Mn (R) | det A 6= 0}). Ovidiu Munteanu [xn+1 , xn ] L64. Fie irul (xn )n1 definit prin: x1 , x2 N , xn+2 = s ,n xn+1 t as x2003 = 2004, demonstrai c irul nu este convergent. Iuliana Georgescu i Paul Georg s L65. Fie n N i funciile f, g : R R, unde f (x) = x2n cos (1/x s t f (0) = 0, f (x) = x2n sin (1/x), x > 0, iar g (x) = x2n+1 sin (1/x), x < 0 i g (x) = x2n+1 cos (1/x), x > 0. Sa se afle cel mai nalt ordin de deriv s acestor funcii i s se studieze problema continuitii acestor derivate n t s a at Gheorghe Costo

Pagina rezolvitorilor

BOTO ANI S Colegiul Naional "A. T. Laurian". Clasa a IX-a. NEGRESCU A t VII(39,40,42,44), VIII(36,40,42), IX(36,37,39), X(39,40), G(42,54). BRA OV S Scoala gen. nr. 5. Clasa a VII-a. POSTEUCA Bogdan: V.37, VI VII.39; POSTEUCA Raluca: V.37, VI (37,38,40), VII.39. CONSTANTA Colegiul Naional "Mircea cel Batrn". Clasa a X-a. LUPU Cezar t IX(38,39,40), X(38,40), XI(37,38). CRAIOVA Scoala nr. 22 "M. Eliade". Clasa a IV-a (nv. VANTU Angela). Ioan: P(54-63). HRLAU (Iai) s Liceul "Stefan cel Mare". Clasa a VI-a. CIOFU Alexandra: P(50, VI.39, VII.41; SAVA Cristina Amelia: P(52,61,63), V(41,44); SCRIPCARIU P.61, V(41,44), VI(37,42); SPIRIDON Florin: P(50,61), V(37,41), VII.41; S Ionu: V(37-39), VI(38,39). Clasa a VIII-a. ANTOCI Bogdan: VI(39 t VII.44; BURICAN Bogdan - Alexandru: VI(37,38,42,44,45), VII(41,44); Lucian: VI(39,40,44,45), VII.44. HUNEDOARA Liceul "Iancu de Hunedoara". Clasa a VII-a. CRACIUN Maria: VI(42,44), VII.41. IA I S Colegiul Naional "C. Negruzzi". Clasa a V-a. OLARIU Tudor: P(51 t V(43,45), VI.39; TIBA Marius: P(58,61-63), V(42,45), VI.42. Colegiul Naional. Clasa a V-a. ANDRONIC Adrian: V(36,38,39,41t ANDRONIC Adrian Constantin: V(36,38,39,41-45), VI.42; BALAN Elen V(36,38,39,41-44); BARBACARIU Ioana: V(36,38,39,41,42); BERCU Tu 38,39,41-44); CAPRARU Mdlina: V(36,38,41-44); CHELSAU Andre a a 38,39,40-45); CHIDIU Alexandru: V(36,38,39,41-45); DOBROVICEANU V(36,38,39,41-43); GAFITANU Oana: V(36,39,41-43); GEORGESCU A 62), V(36,38,41,42,44); MNZATEANU Maria-Adelina: V(36,38,39,41-44 Monica: V(36,38,39,41-44); MO NEGUTU Ctlina Elena: V(36,38,39,4 S aa LAGHIA Irina: V(36,38,39,41-45); POPA Ana-Maria: V(36,38,39,41-43 TUR George: V(36,38,39,41-44); SMARANDA Sava: V(36,38,40-44); TO xandra: V(36,38,39,41,42). Clasa a IX-a. CAZACU Roxana: VII.41, G(40,47); CHIRUTA Marta: VII.41, VIII.42, IX(38,39), G.40; HAMCIU VII(41,42), VIII.42, IX(36,39); PRODAN Diana: VII.41, IX(36,38), G(4 MOFTE Diana: VII.41, IX(36,38), G(40,47). Clasa a X-a. DUMITRE xana: VIII(37,42), IX(37,39), X(36-38,42), G(40,50); PACHITARIU Mar 50,52), L(46,47,49,50). Clasa a XI-a. MUSTATA Irina: X.42, XI(41,4 G(46,47,52), L(46,47). Liceul "M. Eminescu". Clasa a V-a. BOHOTIN Alexandru: P(48,49,5

V.37; COHAL Clin: P(48,58,60,63), V(38,39,42), VI.38. Clasa a VI-a a Mihaela: V(42-44), VI(37,42); CIURARU Ionela Alexandra: V(42-44), IPATE Cristina Alexandra: V(42-44), VI(37,42). Liceul "G. Ibraileanu". Clasa a VII-a. UNGUREANU Drago: V s VI(39,42). Scoala nr. 7 "N. Tonitza". Clasa a II-a (nv. TUDOSE Elena). CR P(54-57,60); DOBRIN Diana - Maria: P(54-57,60); LEONTE Anca: P( POSTICA Simona - Alexandra: P(54-57,60); ROTARIU Larisa: P(54-57,6 Rzvan: P(54-57,60). Clasa a II-a (nv. MELINTE Rodica). BACIU Cipr a 57,60); BRZU Constantin: P(54-57,60); BOTO ANU Bianca - Miha S t 57,60); BUZDUGAN Petru - Catalin: P(54-57,60); CEUCA Danu - Vasi 57,60); CONSTANTINESCU Diana - Gabriela: P(54-57,60); CUCUTEA Ctlin: P(54-57,60); GU OVATE Diana - Stefana: P(54-57,60); LEOGA aa S Diana: P(54-57,60); MIRON Vlad - Stefan: P(54-57,60); MOTAN Geanin P(54-57,60); ROTARIU Marian: P(54-57,60); SUCIUC Raluca: P(54-57, - COSTIN Andrada - Mihaela: P(54-57,60). Clasa a IV-a (nv. MARCU BUTNARU Valentin: P(52,58-62); ONUTA Alin: P(52,58-62). Scoala nr. 13 "Alexandru cel Bun". Clasa a III-a (nv. SPNU BURLACU Ionu: P(54-57,61); DAMIAN Daniel: P(54-57,61); FURTUN t P(54-57,61); IFTENIE Ioana - Ctlina: P(54-57,61); RUSU Alexand aa 57,61); URSU Gina - Ioana: P(54-57,61). Scoala nr. 22 "B. P. Hasdeu". Clasa a II-a (nv. TRZIORU Iuliana). LITEI Victor: P(54-57,60); APOSTOL Ana - Maria: P(54-57,60,61); BA aa drei: P(54-57,60); BURUIANA Ctlina: P(54-57,60,61); CUBERSCHI Pa 57,60,61); E ANU Geogiana: P(54-57,60); GREIEROSU Claudia: P(54 S GNDU Alexandra - Livia: P(54-57,60,61); LAMATIC Ioana: P(47,54-5 BEGEA Andrada: P(54,56,57,60,61); UNGUREANU Teofana: P(54-57,60 sa a II-a. (nv. TUTU Laura). ANDRONICIUC Ana - Miruna: P(54 BUHU Vlad: P(54-57,60); BURUIANA Sebastian: P(54-57,60,61); BUZ - Andrei: P(54-57,60); CEOBANU Andrei - Nicolae: P(54-57,60); CH Alexandra - Elena: P(54-57,60,61); COSTACHESCU Ioana: P(54-57,60 HOI Ovidiu: P(54-57,60,61); GELIP Ioana: P(54-57,60); GHERAN An P(54-57,60); GRIGORE Georgiana: P(54-57,60); GURAU Raluca - Clau 57,60); HATESCU Iustina: P(54-57,60); HORBOVANU Bianca - Alexan 57,60); NASTASE Andrei Ionu: P(54-57,60,61). Clasa a II-a (nv. DO t Liliana). TURCU Andrei - Daniel: P(54-57,60,61). Scoala nr. 23 "T. Maiorescu". Clasa a IV-a (nv. CHIRILA Beatrice RACHE Alexandru - Gabriel: P(54-63). Scoala nr. 26 "G. Cobuc". Clasa a III-a (nv. RACU Maria). BA s Ioana - Mioara: P(54-57,61); BULGARU Ionela - Alexandra: P(54-57,61) CU Claudiu: P(54-57,61); CALOIAN Andrei: P(54-57,61); CALIN Georg dreea: P(54-57,61); CRACIUN Madalina: P(54-57,61); IFROSA Adria Petru - Alexandru: P(54-56,59,61); LEAGAN Crina - A 57,61); IOJA P(54-57,61); MOISA Bogdan: P(54-57,61); PINTILIE Rzvan - Florin: P( a RUSU Flavia: P(54-57,61). Clasa a III-a (nv. GALIA Paraschiva). AL

dra - Mdlina: P(54-57,61); CIOABA Oana - Ctlina: P(54-57,61); a a aa Marius - Catalin: P(54-57,61); HOMEA Liviu: P(54-57,61); HUIDE G S 57,61); MANOLIU Madalina: P(54-57,61); MIHAILESCU Laura: P(54-57 CA Alexandru: P(54-57,61); POPA Florin: P(54-57,61); SCUTARIU C P(54-57,61).

Premii acordate rezolvitorilor

Pentru apariia de trei ori la rubrica "Pagina rezolvitorilor" redac t t "Recreaii matematice" acord cte o diplom i un premiu n cri u t a as at elevi: BARABULA Ioana ( c. nr. 26 "G. Cobuc", cl. a III-a): 1/2003 (5pb S s (5pb), 1/2004 (5pb); BURLACU Ionu ( c. nr. 13 "Alexandru cel Bun", cl. a III-a): 1/2003 (6p t S (5pb), 1/2004 (5pb); BUTNARU Valentin ( c. nr. 7 "N. Tonitza", cl. a IV-a): 1/2003 (5pb S (6pb), 1/2004 (6pb); CALOIAN Andrei ( c. nr. 26 "G. Cobuc", cl. a III-a): 1/2003 (5pb), 2/2 S s 1/2004 (5pb); CALIN Georgiana ( c. nr. 26 "G. Cobuc", cl. a III-a): 1/2003 (5pb), 2/2 S s 1/2004 (5pb); CIOABA Oana - Ctlina ( c. nr. 26 "G. Cobuc", cl. a III-a): 1/20 a a S s 2/2003 (5pb), 1/2004 (5pb); CRACIUN Mdlina ( c. nr. 26 "G. Cobuc", cl. a III-a): 1/2003 (5pb a a S s (5pb), 1/2004 (5pb); DAMIAN Daniel ( c. nr. 13 "Alexandru cel Bun", cl. a III-a): 1/2003 (6p S (5pb), 1/2004 (5pb); Marta ( c. nr. 13 "Alexandru cel Bun", cl. a III-a): 1/20 FURTUNA S 2/2003 (5pb), 1/2004 (5pb); IFTENIE Ioana - Ctlina ( c. nr. 13 "Alexandru cel Bun", cl. a III-a a a S (6pb), 2/2003 (5pb), 1/2004 (5pb); LEAGAN Crina - Alexandra ( c. nr. 26 "G. Cobuc", cl. a III-a): 1/2 S s 2/2003 (5pb), 1/2004 (5pb); MIHAILESCU Laura - Ioana ( c. nr. 26 "G. Cobuc", cl. a III-a): 1/2 S s 2/2003 (5pb), 1/2004 (5pb); MOISA Bogdan ( c. nr. 26 "G. Cobuc", cl. a III-a): 1/2003 (5pb), 2/2 S s 1/2004 (5pb); NEGRESCU Alexandru (C. N. "A. T. Laurian", Botoani, cl. IX-a): 1/2 s 2/2003 (17pb), 1/2004 (14pb); ONUTA Alin ( c. nr. 7 "N. Tonitza", cl. a IV-a): 1/2003 (5pb), 2/20 S 1/2004 (6pb). PINTILIE Rzvan - Florin ( c. nr. 26 "G. Cobuc", cl. a III-a): 1/2 a S s 2/2003 (5pb), 1/2004 (5pb);

POSTEUCA Bogdan ( c. nr. 5, Braov, cl. a VII-a): 1/2002 (5pb), 1/2 S s 1/2004 (5pb). POSTEUCA Raluca ( c. nr. 5, Braov, cl. a VII-a): 1/2002 (5pb), 1/2 S s 1/2004 (5pb). RUSU Alexandru ( c. nr. 13 "Alexandru cel Bun", cl. a III-a): 1/2003 (8p S (5pb), 1/2004 (5pb); RUSU Flavia ( c. nr. 26 "G. Cobuc", cl. a III-a): 1/2003 (5pb), 2/20 S s 1/2004 (5pb); SCUTARU Constantin ( c. nr. 26 "G. Cobuc", cl. a III-a): 1/2003 (5p S s (5pb), 1/2004 (5pb); URSU Gina - Ioana ( c. nr. 13 "Alexandru cel Bun", cl. a III-a): 1/2 S 2/2003 (5pb), 1/2004 (5pb).

LISTA MEMBRILOR FILIALEI IA I a S. S S


110. 111. 112. 113. 114. 115. 116. 117. 118. 119. 120. 121. 122. 123. 124. 125.

continuare din nr. 1/2000, 1/2001, 1/2002 i 1/2003 s A Marcela MIHAIL Scoala "D.D.Patracanu", Tome s s BOBOC Romela Scoala "D.D.Ptrcanu", Tome a as s TEMNEANU Mitica Univ. Tehnica "Gh. Asachi", Ia s MIRON Mirel Liceul "C. Negruzzi", Iai s ROTUNDU Raluca Scoala gen. Gropnia, jud. Iai t s APETREI Laura NAVROTESCU Mariana Gr. c. "Al. I. Cuza", Iai s s CHIORESCU Daniela Marinela Gr. c. "D. Mangeron", Iai s s AVADANI Adela Scoala gen. nr.37, Iai s STRACHINA Monica Scoala gen. nr.37, Iai s BRGHI AN Mariana S Gr. c. "Tehnoton", Iai s s SPIRIDON Ana - Mrioara a Sc. nr. 3 "Iordache Cantacuzino TUDORACHE Nelu Liceul "V. Alecsandri", Iai s DASCALU Cristina Liceul "M. Eminescu", Iai s CORDUNEANU Adrian Univ. Tehnic "Gh. Asachi", Ia a s RO U Mrioara S a Liceul de art, Iai a s

IMPORTANT

n scopul unei legturi rapide cu redacia revistei, pot fi utilizate urmtoa a t a e-mail: tbi@math.tuiasi.ro, popagabriel@go.com . Pe aceasta cale co pot purta cu redacia un dialog privitor la materialele trimise acesteia, p t numerelor revistei etc. La problemele de tip L se primesc soluii de la orice iubitor de matematici e t (indiferent de preocupare profesionala sau vrsta ). Fiecare dintre solui t probleme - ce sunt publicate n revista dupa un an - va fi urmata de nume celor care au rezolvat-o. Adresm cu insistena rugmintea ca materialele trimise revis a t a fie (s nu fi fost) trimise i altor publicaii. a s t

Anul VII, Nr. 1

Ianuarie Iunie

RECREAII MATEMATICE

REVIST DE MATEMATIC PENTRU ELEVI I PROFES

e i = 1

Editura Crengua Gldu IAI - 2005

Semnificaia formulei de pe copert: i ntr-o form concis, formula e = 1 leag cele patru ramuri fun
tale ale matematicii: ARITMETICA GEOMETRIA ALGEBRA ANALIZA MATEMATIC reprezentat reprezentat reprezentat reprezentat de de de de 1 i e

Redacia revistei :

Petru ASAFTEI , Temistocle BRSAN, Dan BRNZEI, Ctlin - Cristian BUD Constantin CHIRIL, Eugenia COHAL, Adrian CORDUNEANU, Mihai CR (Pacani), Gabriel DOSPINESCU (student, Paris), Marius FARCA, Paraschiva G Paul GEORGESCU, Dumitru GHERMAN (Pacani), Gheorghe IUREA, Lucian G LDUNC, Mircea LUPAN, Dan tefan MARINESCU (Hunedoara), MRANU, Andrei NEDELCU, Gabriel POPA, Dan POPESCU (Suceava), POPOVICI (Braov), Maria RACU, Ioan SCLEANU (Hrlu), Ioan ER (Ortie), Dan TIBA (Bucureti), Lucian TUESCU (Craiova), Adrian ZANOSCH

Adresa redaciei: Catedra de Matematic Universitatea Tehnic Gh. Asachi Iai Bd. Carol I, nr.11, 700506, Iai Tel. 032 213737 / int. 123 E-mail: acord@math.tuiasi.ro EDITURA CRENGUA GLDU Toate drepturile rezervate ISSN 1582 - 1765 Bd. N. Iorga, Bl. K2, ap. 4, IAI Tel. / Fax: 032 230598 TIPRIT LA SL&F IMPEX IAI Bd. Carol I, nr. 3-5 Tel. 0788 498933 E-mail: simonaslf@yahoo.com

Anul VII, Nr. 1

Ianuarie Iunie

RECREAII MATEMATICE

REVIST DE MATEMATIC PENTRU ELEVI I PROFES

e i = 1
Revist cu apariie semestrial publicat de

ASOCIAIA RECREAII MATEMATICE

IAI - 2005

200 de ani de la naterea lui Dirichlet s

Johan Peter Gustav Lejeune a fost unul dintre marii matematicien lui XIX i ai tuturor timpurilor. s S-a nscut la 13 februarie 1805, a tatea Dren, ora situat la jumtate s a dintre Kln i Aachen, n Germania s n Imperiul Francez al lui Napoleon), unui funcionar potal. Numele de fa t s Dirichlet, mai precis Lejeune-Dirichl de la bunicul su care a locuit n ora a s n apropiere de Lige, n Belgia ("L Richlet"). Dirichlet a fost un elev strluc a nnd studiile secundare la vrsta d nc nainte de a intra la gimnaziul a dovedete pasiune pentru matemat s tuind banii de buzunar pe cri de m at Dup absolvirea colegiului, la Kln, se hotrte s urmeze cursurile u a a as a n Frana. Sosete la Paris n 1822, purtnd cu el ca pe o biblie,Disquisitio t s meticae, cunoscutul tratat al lui Gauss. Aici va urma cursurile Facultii at de la Collge de France. ncepnd din vara lui 1823 este bine primit n casa g Foy, unde locuiete i d lecii de limb german soiei i icei acestuia. s s a t a a t s Maximilien Sbastien Foy a fost una dintre gurile remarcabile ale rzb a Napoleon i din 1819 lider al opoziiei liberale din Camera deputailor. s t t La Paris, Dirichlet intr n contact cu mari matematicieni precum a Poisson, Laplace i Fourier. Ultimul l-a impresionat n mod deosebit, fapt c s consecina interesul su pentru seriile trigonometrice i fizica matematic. t a s a perioad redacteaz prima sa contribuie original n matematic : dem a a t a a marea teorem a lui Fermat pentru cazul n = 5. Aceast teorem afirm a a a a orice numr natural n, n > 2, nu exist numere ntregi diferite de 0 a a a xn +y n = z n . Demonstraia complet a acestei teoreme a fost dat abia n t a a ai secolului XX (v. [1], [2], [4], [5]). Ulterior, Dirichlet a fost primul ma care a observat c unele demonstraii date pentru cazuri particulare al a t lui Fermat, de ctre mari matematicieni, erau greite deoarece se bazau a s c n inele de extensiune a lui Z, descompunerea unui numr ca produs a a ireductibili (care nu se mai pot descompune) este unic, ipotez care este fa a a unele dintre aceste inele. Aceast observaie a impus diferenierea ntre a t t de numr ireductibil i cea de numr prim (un numr diferit de zero i a s a a s numete numr prim dac ori de cte ori divide un produs de numere, divid s a a unul dintre factori) i a avut implicaii profunde n dezvoltarea teoriei nu s t algebrei. La sfritul anului 1825, dup moartea generaluli Foy, Dirichlet se hot s a a ntoarc n Germania. La recomandarea lui Alexander von Humboldt a doctorat onorific (nu cunotea limba latin, condiie obligatorie n acel ti s a t

obinerea unui doctorat) ceea ce i permite s-i susin teza de docena t as t a t s titlul de profesor. Pred la Universitatea din Breslau (astzi Wroklaw, a a la Liceul militar din Berlin i, timp de 27 de ani nentrerupt, la Univers s Berlin. Dintre cei mai strlucii elevi ai si, din aceast perioad, men a t a a a L. Kronecker i B. Riemann. s n 1831 se cstorete cu Rebeca Mendelssohn, sor cu celebrul compo aa s a Contemporanii si l apreciau ca pe un excelent matematician i profes a s era lipsit de anumite defecte: se mbrca neglijent, era mereu cu o igar a t a o cafea n faa, puin preocupat de imaginea sa i mereu n ntrziere. t t s n 1843, maestrul i prietenul su Karl Jacobi este diagnosticat ca fi s a de diabet i Dirichlet l nsoete pentru o perioad de 18 luni n Italia, und s t s a Roma, Florena, Sicilia. Climatul blnd din Italia amelioreaz starea de t a lui Jacobi. Cltoria efectuat n 1844 - 1845 a fost posibil datorit une aa a a a obinute de Alexander von Humboldt de la Friedrich Wilheem al IV-lea. t Dup moartea lui Gauss, n 1855, succede acestuia la catedra de la Un a din Gttingen. Activitatea sa n acest mare centru matematic al lumii e la 5 mai 1859 Dirichlet trece n lumea umbrelor ca urmare a unei maladii Evantaiul lucrrilor lui Dirichlet ilustreaz profunzimea culturii matem a a mane din perioada de nceput a epocii de aur a acesteia, inaugurat de Kar a Gauss, cel mai mare matematician al timpurilor moderne. Lucrrile lui aco a aspectele ale matematicii; totui cele de teoria numerelor, analiz i teoria p s as lui sunt cele mai importante. Multe noiuni i rezultate i poart acum nu t s a n teoria numerelor a demonstrat c, dac a i b sunt numere ntregi i a a s s n irul (an + b)nN exist o innitate de numere prime, rezultat cunoscut s s a de teorema lui Dirichlet. Demonstraia dat de Dirichlet acestei teorem t a este considerat actul de natere a teoriei analitice a numerelor. Dirichle a s contribuie important la elaborarea instrumentelor de lucru pentru teoria t a numerelor prin introducerea seriilor ataate funciilor aritmetice, numite a s t ile lui Dirichlet, crearea teoriei unit tilor i preocuprile sale privind rep a s a numerelor ntregi prin forme p tratice aritmetice (v. [4]). a De asemenea, lui Dirichlet datorm principiul sertarelor, ce arm c, a a a n + 1 obiecte n n sertare, atunci cel puin un sertar conine cel puin dou t t t Dirichlet utilizeaz acest principiu n studiul corpului numerelor algebrice a n teoria potenialului se ocup cu problema Dirichlet privind existena t a t armonice. Tot el a dat condiia Dirichlet pentru convergena seriilor trigo t t Ideile lui Dirichlet nu au pierdut strlucirea odat cu trecerea timpul a a tarea matematicii n ultimii 200 de ani a pus n evidena profunzimea ace t Bibliografie 1. 2. 3. 4. 5.

A. Corduneanu - Despre Marea teorem a lui Fermat, RecMat - 1/1999 a P. Minu - Pierre Fermat - Patru secole de la naterea sa, RecMat - 2/2 t s P. Minu - Numere prime din progresii aritmetice, RecMat - 1/2003, 15t P. Minu - Teoria numerelor, Ed. Matrix-Rom, Buc., 2001 (pp. 35-50, 2 t M. M. Postnikov - Despre teorema lui Fermat, Ed. did. i ped., Bucure s

Prof. dr. Petru MINUT

Fractali

(II)

Stefan FRUNZ 1 , Irina FRUNZ 2 A A

Fractal este un obiect fracionat la infinit (cuvntul deriv din latin t a derivat, la rndul su, din verbul frangere - a rupe, a sparge). Terme a introdus de americanul de origine polonez Benoit Mandelbrot n 1975. a n acelai timp precis i general a unui obiect fractal este dificil. Pentru s as a a vag, fractalii sunt, dup Mandelbrot, mulimi care prezint neregularit a a t a a scrile. O structur fractal e aceeai n mic ca i n mare, aproape sau d a a a s s n natur se pot ntlni structuri prefractale (termenul i aparine de a t lui Mandelbrot). Ca exemple ar putea servi frunzele de spini, cele de fe de zpad, norii etc. Structurile fractale sunt abstraciuni matematice ob a a t regul, din structuri prefractale prin trecere la limit. Sirul de mulimi a a t care ne d la limit o structur fractal este, de regul, un ir recure a a a a a s obine dintr-o structur prefractal iniial (iniiator) aplicndu-i succe t a a t a t transformare (generator ). Exemple n acest sens se gsesc n [1]-[6]: curba a curba fulgului de z pad , praful lui Cantor, sita lui Sierpinski, carpeta lui a a buretele lui Menger etc. n [1], pe care o continum n nota de faa, au fost stabilite sau indicat a t unile n sens Richardson ale ctorva fractali: curba lui Koch are dimensiun log 4 log 2 log 3 D= , praful lui Cantor , sita lui Sierpinski , sita tridim log 3 log 3 log 2 log 4 log 20 a lui Sierpinski , buretele lui Menger etc. log 2 log 3

O alt caracteristic a unei structuri fractale, pe care ne propunem s o a a a n continuare, este autosimilaritatea. Majoritatea structurilor fractale sunt autosimilare ntr-un sens care va n continuare. S ncepem prin a considera covorul lui Sierpinski. El se po a pornind de la triunghiul echilateral iniial ABC i aplicnd succesiv tre t s omotetia de centru A i raport 1/2 i analoagele cu centrele B i C (s s s s a respectiv cu f1 , f2 , f3 ).
A f1 A f2 A f3

T0

f1 (T0 )

T0

f 2 (T0 )

T0

Astfel, dup primul pas iterativ, putem scrie: a S1 = f1 (T0 ) f2 (T0 ) f3 (T0 ) ,
1 2

Prof. dr., Facultatea de matematic, Univ. "Al. I. Cuza", Iai a s Profesor, Grupul Scolar Agricol "M. Koglniceanu", Miroslava (Iai) a s

cele trei mulimi ale reuniunii putnd avea n comun dou cte dou ce t a a punct. S notm f1 (T0 ) = T01 i s urmrim efectul aplicrii celor tre a a s a a a asupra lui.
f1 f2 f3

T01

T01

T01

Urmrind analog transformrile lui f2 (T0 ) = T02 i f3 (T0 ) = T03 cons a a s S2 = f1 (S1 ) f2 (S1 ) f3 (S1 ) . Se poate demonstra prin inducie c t a Sn+1 = f1 (Sn ) f2 (Sn ) f3 (Sn ) , n 1,

unde Sn este aproximanta de ordin n a covorului lui Sierpinski S [1]. Mu membrul drept al relaiei (1) nu pot avea n comun dou cte dou dec t a a un punct. Din relaia (1) se poate deduce (elementar) c t a S = f1 (S) f2 (S) f3 (S) .

Mulimile din membrul drept al lui (2) pot avea dou cte dou cel mult t a a 1 n comun i fiecare este similar cu S cu raportul de asemnare . s a a 2 O asemenea situaie am mai ntlnit-o doar la gurile cele mai simple d t tria elementar (segmente de dreapt sau arce de cerc). a a

Un segment de dreapt poate fi mprit n orict de multe segmente adiac a at lare fiecare cu segmentul iniial. t Rezultatul precedent are o strns legtur cu noiunile de msur i d a a a t a as S amintim rezultatul din plan c raportul ariilor a dou figuri asemene a a a cu ptratul raportului de asemnare. Un rezultat similar are loc n spaiu a a t volumelor a dou corpuri asemenea este egal cu puterea a treia a rap a asemnare. a

n [1] s-a artat c i obiectele fractale pot fi "msurate" i c msu a a s a s a a legtur cu dimensiunea fractal. Dac rezultatul de geometrie clasic am a a a a a sus s-ar extinde corespunztor la obiecte fractale, atunci din relaia (2) am a t m (S) = m (f1 (S)) + m (f2 (S)) + m (f3 (S)) = D D D 1 1 1 = m (S) + m (S) + m (S) , 2 2 2

unde m (S) este msura lui S iar D este dimensiunea fractal a lui S. Ad a a

0 < m (S) < (ceea ce e destul de rezonabil), obinem din (3) relaia t t D 1 1=3 2 log 3 . Reg a care permite determinarea direct a lui D: 2D = 3, D = a log 2 dimensiunea determinat prin metoda lui Richardson. a

O proprietate analoag de "autosimilaritate" o prezint i mulimea l a as t ale crei aproximante succesive sunt: a 2 1 C0 = [0, 1] , ,1 , C1 = 0, 3 3 2 3 6 7 8 9 1 C2 = 0, 2 2 , 3 2 , 2 2 , 2 , . . . 3 3 3 3 3 3 3 1 x 2 Se observ c 0, a a = f1 (C0 ), unde f1 (x) = i , 1 = f2 (C0 ), unde f ( s 3 3 3 Se poate demonstra c a Cn+1 = f1 (Cn ) f2 (Cn ) , n 1, de unde se deduce (elementar) c a C = f1 (C) f2 (C) . 1 ntruct f1 i f2 sunt similariti de raport , pentru determinarea d s at 2 D log 2 2 n concordana c t , de unde D = fractale se obine ecuaia 1 = 2 t t 3 log 3 determinat prin metoda lui Richardson. a

Sistemele de funcii {f1 , f2 , f3 } (implicat n covorul lui Sierpinski) i {f t s plicat n mulimea lui Cantor) se numesc sisteme iterate de funcii. Curba l t t poate obine printr-un sistem iterat de 4 similariti (implicnd omotetii t at 1 toate de raport [2]. Pentru determinarea dimensiunii fractale a curbe 3 D log 4 1 n concordana iari c t as , de unde D = se obine ecuaia 1 = 4 t t 3 log 3 determinat prin metoda lui Richardson. a Bibliografie 1. 2. 3. 4.

St. Frunz - Fractali, RecMat - 2/2002, 1-5. a St. Frunz - Fractali, curs opional, anul IV, Fac. Matematic - Informa a t a J. - F. Gornyet - Physique et structures fractales, Masson, Paris, 1992. B. B. Mandelbrot - Les objects fractales: forme, hasard et dimension, Fla Paris 1975. 5. A. Le Mhaut - Les gomtries fractales, Herms, Paris, 1990. 6. H. Takayasu - Fractales in the physical sciences, Manchester Univers Manchester and New York, 1990.

Asupra monotoniei unor iruri s


Dumitru MIHALACHE, Marian TETIVA1

n aceast not ne propunem s investigm monotonia irurilor care a a a a a s cizarea ordinului de convergena al ctorva iruri uzuale. Mai precis, d t s irului (an )n1 este a (finit), iar (zn )n1 este un ir cu limita zero astfel n s a s an a , lim n zn finit i nenul, atunci spunem c (zn )n1 d ordinul de convergena al irul as a a a t s ctre limita sa (este un fel de a msura "rapiditatea" cu care (an )n1 tinde l a a ajutorul unor iruri cunoscute i/sau mai accesibile; a se vedea i [2]). Ne s s s s cercetm dac irul cu termenul general (an a)/zn este sau nu monoto a a as s a unor iruri (an )n1 des ntlnite i importante n analiza matematic. s Primul ir de care ne ocupm este cel cu termenul general s a 1 1 1 En = 1 + + + + , n N . 1! 2! n! Aa cum se tie, acesta este convergent, are limita e, iar ordinul su de co s s a este dat de relaia t lim n n!(e En ) = 1,
n

Un calcul simplu (chiar dac, poate, neplcut!) arat c expresia de pe ul a a a a este egal cu a n+1 n3 + 10n2 + 34n + 41 > , n!(n + 1)(n + 2)(n + 3)(n + 4) n!(n2 + n + 1) ultima inegalitate fiind echivalent cu a adic evident pentru orice numr natural n. Astfel am demonstrat a a a
1

ceea ce se veric uor cu teorema lui Cesro - Stolz (cazul 0 ). Pentru a ve a s 0 a tonia irului (xn )n1 cu termenul general xn = n n!(e En ) trebuie s s xn cu xn+1 . Avem n+1 1 xn ? xn+1 n(e En ) ? (n + 1)2 e En (n + 1)! n!(n2 + n + 1) n+ 1 Este bine cunoscut i inegalitatea e En < as (v. [3]); cum n n! n!(n2 + 1 < , de aici nu putem obine nimic. Atunci vom proceda astfel: t n n! p X 1 1 lim > e En = n! p (n + 1)(n + 2) (n + k) k=1 1 1 1 1 1 + + + > n! n+1 (n+1)(n+2) (n+1)(n+2)(n+3) (n+1)(n+2)(n+3)

n5 + 11n4 + 45n3 + 85n2 + 75n + 41 > n5 + 11n4 + 45n3 + 85n2 + 74n

Profesori, Colegiul Na ional "Gheorghe Roca Codreanu", Brlad t s

Propoziia 1. Sirul (xn )n1 cu termenul general xn = n n!(e En ) t cresc tor. a O alt metod de a demonstra Propoziia 1 este prezentat n a a t a

Exerciiul 1. Ar tai c sirul (un )n1 cu termenul general t a t a n+1 un = En + , n N , n!(n2 + n + 1) este strict cresc tor cu limita e si deducei de aici inegalitatea xn < xn+1 a t Si nc o observaie: avem acum inegalitile a t at n+1 1 < e En < , n N , n!(n2 + n + 1) n n! care implic, i ele, faptul c lim n n!(e En ) = 1 (cum?). a s a
n

Urmtorul de care ne ocupm este sirul lui Euler, care definete numru a a s a considerm a n 1 , n 1. en = 1 + n Iar facem apel la o relaie cunoscut (a se vedea [4], sau [6]), anume t a e lim n (e en ) = ; n 2 deci ne intereseaz acum (din punct de vedere al monotoniei) irul (xn )n a s menul general xn = n(e en ), n 1. Dar, nainte, s demonstrm a a x 1 Lema 1. Funcia definit prin f (x) = x 1 + t a , pentru orice x > 0 x convex pe (0, ). a Demonstraie. Vom folosi prima dintre inegalitile t at 1 1 2 < ln 1 + <p 2x + 1 x x(x + 1)

(pentru care se poate vedea tot [6]; de altfel, sunt nite inegaliti destul s at late, care arat c, pentru funcia logaritm natural, punctul intermediar d a a t lui Lagrange este cuprins ntre mediile aritmetic i geometric ale extr a s a intervalului pe care se aplic, aici intervalul [x, x + 1]), desigur, dup ce vo a a derivata a doua. Cititorul este invitat s verice c a a x 1 1 1 x ln 1 + + , x > 0 i s f 0 (x) = 1 + x x x+1 x 1 1 2 1 x+3 x ln2 1 + , + ln 1 + f 00 (x) = 1 + x x x+1 x (x + 1)2 Atunci, procednd cum am anunat, vom putea scrie t x 4 2 x+3 1 2 00 f (x) > 1 + x s + x (2x + 1)2 x + 1 2x + 1 (x + 1)2 x 3x + 1 1 , x > 0, f 00 (x) > 1 + x (x + 1)2 (2x + 1)2

de unde f 00 (x) > 0, x > 0, i lema este dovedit. s a

Propoziia 2. Sirul (xn )n1 definit prin t n 1 xn = n e 1 + = n(e en ), n N , n este strict cresc tor. a Demonstraie. Inegalitatea de demonstrat este n(e en ) < (n + 1)(e t este echivalent cu (n + 1)en+1 nen < e, n N . Cum a
n

lim [(n + 1)en+1 nen ] = lim [(n + 1)(en+1 e) n(en e) + e]


n

nseamn c ar fi suficient s artm c irul ((n + 1)en+1 nen )n1 a a a aa a s cresctor. Dar monotonia acestui ir decurge din inegalitatea a s

aceast ultim form nu este nimic altceva dect inegalitatea lui Jense a a a funciei convexe f i numerelor n i n + 2. Demonstraia este ncheiat. t s s t a Exerciiul 2. Studiai monotonia sirului (xn )n1 definit prin t t " # n+1 1 e , n N . xn = n 1 + n

(n + 1)en+1 nen < (n + 2)en+2 (n + 1)en+1 f (n + 2) + f (n) > 2f

Mai departe considerm irul cu termenul general a s 1 1 n () = 1 + + + , n N , 2 n unde > 1 este un numr fixat. Se vede imediat c acest ir este strict a a s i (ceva mai greu) mrginit superior, deci are o limit finit, pe care o s a a a (). n parantez fie spus, funcia care asociaz fiecrui numr real > a t a a a se numete funcia zeta a lui Riemann i are mare importana n multe d s t s t matematicii. Se mai tie c ordinul de convergena al irului ( n ())n1 e s a t s egalitatea 1 lim n1 (() n ()) = n 1 (vezi, de exemplu, [8]). Aadar, ne intereseaz monotonia irului (xn )n1 s a s n legtur cu aceasta avem a a xn = n1 (() n ()) , n N ;

Propoziia 3. Sirul (xn )n1 , cu termenul general xn = n1 (() t n N , este strict cresc tor. a a Demonstraie. Inegalitatea xn < xn+1 devine n acest caz (dup cte t 1 innd cont de n+1 () = n () + t ) (n + 1) 1 + n () < (). (n + 1)[(n + 1)1 n1 ] Notnd 1 + n (), n N , un = (n + 1)[(n + 1)1 n1 ]

observm c este suficient s demonstrm c irul (un )n1 este strict cre a a a a as s rezulte inegalitatea pe care o avem de demonstrat (deoarece lim un = a
n

ce se stabilete uor). Avem s s 1 1 un < un+1 < + (n+1)[(n+1)1 n1 ] (n+2)[(n+2)1 (n+1)1 ] ( 1 1 1 < n1 (n + 1) (n + 1)1 (n + 2 (n + 1) (n + 1) (n + 2) n1 (n + 1)(n + 2) n1 (n + 1) < (n + 1)2 n1 [(n + 2) (n + 1) ] < (n2 + 2n + 1) (n2 + 2n) .

Pentru a demonstra aceast ultim inegalitate, aplicm teorema lui Lagran a a a s t x 7 x , pe intervalele [n + 1, n + 2] i [n2 + 2n, n2 + 2n + 1]; obinem (n + 2n + 1) (n + 2n) =
2

(n + 2) (n + 1) = c1 , n
2

d1 , n

cn [n + 1, n + 2] i s

s a Cum ncn < n(n + 2) = n2 + 2n < dn i > 1, rezult n1 c1 < n este tocmai inegalitatea de demonstrat.

dn [n2 + 2n, n2 + 2n +

n fine, ne vom ocupa de irul care d ordinul de convergena al irului c s a t s constanta lui Euler. Stim c aceasta este a 1 1 c = lim 1 + + + ln n n 2 n i c s a 1 1 1 lim n 1 + + + ln n c = n 2 n 2 (a se vedea [5]). Avem i aici nevoie, mai nti, de un mic rezultat ajutt s a Lema 2. Funcia f : (0, ) R dat de t a

conform inegalitii ln(1 + t) < t, t > 1, t 6= 0. Cum lim f (x) = 0 at


x

1 , x x+1 este strict descresc toare si strict pozitiv pe intervalul (0, ). a a Demonstraie. Funcia f este strict descresctoare pe intervalul (0, t t a 1 1 f 0 (x) = ln 1 + < 0, x > 0, (x + 1)2 (x + 1)2 f (x) = x ln x + (x + 2) ln(x + 2) 2(x + 1) ln(x + 1) rutin!) rezult c f (x) > 0, x (0, ). a a a

Exerciiul 3. Funcia f din Lema 2 este strict convex pe intervalul t t a Propoziia 4. Sirul (xn )n1 cu termenul general t 1 1 xn = n 1 + + + ln n c 2 n este strict cresc tor. a

Demonstraie. Inegalitatea xn < xn+1 se rescrie n forma t 1 1 1 + + + + 1 + n ln n (n + 1) ln(n + 1) > c 2 n i ar fi demonstrat dac am putea arta c irul (un )n1 , dat de s a a a as 1 1 un = 1 + + + + 1 + n ln n (n + 1) ln(n + 1), n N 2 n este strict descresctor (deoarece (un )n1 are limita c - demonstrai!). D a t tatea un > un+1 se reduce la 1 n ln n + (n + 2) ln(n + 2) 2(n + 1) ln(n + 1) >0 n+1 i decurge din lema anterioar, pentru orice n N ; astfel, demonst s a ncheiat. a

Si, odat cu ea, este ncheiat i aceast not. Dar calculele pot cont a as a a exemplu:

Exerciiul 4. Studiai monotonia sirului (xn )n1 , pentru t t 1 n a) xn = (n + 1) e 1 + ; n n+1 1 b) xn = (n + 1) 1 + e ; n 1 1 1 1 c) xn = n 1+ + + 2 nl , unde l = lim 1+ + + n n 2 2 Bibliografie

1. D. Andrica, V. Berinde, L. Toth, A. Vernescu -Ordinul de convergen siruri, G. M. A, 7-8/1998. 2. V. Berinde - Despre ordinul de convergena al sirurilor de numere rea t 4/1998. 3. Gh. Gussi, O. Stnil, Gh. Stoica - Analiz matematic , manual pe a as a a a a XI-a, Editura Didactic i Pedagogic, 1983. as a n 1 4. E. Pltnea - Asupra vitezei de convergena a sirului 1 + a a t , G. M. n 5. A. Vernescu - Ordinul de convergena al sirului de definiie a constantei t t G. M. 10-11/1983. 6. A. Vernescu - O demonstraie simpl a unei inegalit ti relative la num t a a M. 5-6/1988. 7. A. Vernescu - Asupra convergenei unui sir cu limita ln 2, G. M. 10-11/ t 8. A. Vernescu - Asupra seriei armonice generalizate, G. M. A, 3/1997.

O teorem uitat - inegalitatea lui Surny a a


Gabriel DOSPINESCU 1

Cu mult timp n urm, pe lista problemelor propuse pentru prestigiosu a "Miklos Schweitzer" a aprut i urmtoarea inegalitate datorat lui Surn a s a a (n 1) (an + an + + an ) + na1 a2 an 1 2 n

Teorem. Pentru orice numere reale nenegative a1 , a2 , . . . , an are loc in a

(a1 + a2 + + an ) an1 + an1 + + an n 1 2

Departe de a fi o simpl aplicaie a unor inegaliti cunoscute cel pu a t at cte cunoatem pn n prezent aceast teorem dicil reprezint o s a a a a a foarte tare, ce permite rafinri ale multor inegaliti clasice. Fiind mai pu a at cut, vom prezenta pentru nceput demonstraia ei, deloc facil, urmnd a t a s subliniem cteva consecine interesante. Aadar, a t s

Demonstraia teoremei. Vom folosi inducia matematic. Pentr t t a inegalitatea este trivial. S presupunem c am reuit s o demonstrm a a a s a a variabile i s considerm a1 , a2 , . . . , an , an+1 0. Datorit simetriei i om s a a a s acestei inegaliti, putem presupune c a1 a2 . . . an+1 i a1 +a2 + at a s S observm ns c inegalitatea pe care trebuie s o demonstrm se resc a a a a a a ! n n n n X Y Y X an+1 +nan+1 +nan+1 ai +an+1 ai (1+an+1 ) an + an n i n+1 n+1 i
i=1 i=1 i=1 i=1

Ipoteza de inducie asigur valabilitatea inegalitii t a at n n n Y X X nan+1 ai an+1 an1 (n 1)an+1 an . i i


i=1 i=1 i=1

Ca urmare, mai rmne de demonstrat inegalitatea a n ! n ! n n X X X X n+1 n1 n n n + ai ai an+1 n ai ai


i=1 i=1 i=1

+ an+1

i=1

i=1

Desigur, va fi suficient s demonstrm inegalitile a a at n ! n ! n n X X X X n+1 n1 n n n 0, ai ai an+1 n ai ai


i=1

n Y

ai + (n

1)an n+1

an1 n+1

an+1

pentru a finaliza demonstraia pasului inductiv. Din fericire, (5) i (6) nu su t s ntr-adevr, (5) rezult combinnd trei observaii simple. Prima este in a a t
1

i=1 n Y

i=1

i=1

i=1

ai + (n

1)an n+1

an1 n+1

Student, cole Normale Suprieure, Paris

i=1

relaia evident an+1 t a

n P

an i

i=1

n P

an1 0, ce rezult imediat din inegalitatea lui Cebev, a a s i

1 n a n i i = 1, n. (6) este i mai uor de demonstrat; rezult din urmtorul ir de s s a a s i identiti evidente: s at n n Y Y n1 n ai + (n 1)an+1 an+1 = (ai an+1 + an+1 ) + (n 1)an n+1 ce se poate obine adunnd toate inegalitile de forma nan+1 + t at i
i=1 i=1

1 i, n sfrit, a treia observaie este inegalitatea s s t n ! n n n n X X X X 1 n+1 n1 n n , n ai ai ai ai n n i=1 i=1 i=1 i=1

an + an1 n+1 n+1

Astfel, am reuit s dovedim (2) i demonstraia se ncheie. s a s t

n X i=1

(ai an+1 ) + (n 1)an an1 = 0. n+1 n+1

Odat demonstrat (1), s vedem ce obinem pentru valori mici ale num a a a t variabile. Pentru n = 3 obinem t adic binecunoscuta inegalitate a lui Schur. a Pentru n = 4, dup calcule de rutin, rezult inegalitatea a a a 4 ! 4 X X Y 2 a4 + 2 ai ai aj a2 + a2 , i i j
i=1 i=1 1i<j4 4 X i=1

a3 + a3 + a3 + 3a1 a2 a3 a2 (a2 + a3 ) + a2 (a3 + a1 ) + a2 (a1 + a2 1 2 3 1 2 3

care este mai tare dect inegalitatea lui Turkevici [1]: a4 + 2 i


4 Y

i=1

ai

1i<j4

a2 a2 . i j

s S observm c n ipoteza ai < a a a , i = 1, n, obinem o ntrire a clasicei t a n1 a lui D. S. Mitrinovi i D. D. Adamovi [2]: cs c a1 a2 an (s (n1)a1 )(s (n1)a2 ) (s (n1)an ), dac ai < a

Continum cu simple observaii ce decurg din (1). Astfel, lund s = a t + + an , rezult c a a n X n a1 a2 an an1 (s (n 1)ai ) . i
i=1

s = a1 +a2

s , i = 1, n. (Pentru a vedea c (7) este mai tare dect (8) a n1 dect s aplicm inegalitatea mediilor n (7).) a a

Putem obine uor din (1) i urmtoarea rafinare a inegalitii mediilo t s s a at n an + an + + an n a1 + a2 + + an 1 2 n a1 a a1 a2 an n n1 n

Din (1) i (10), dupa cteva calcule simple, obinem (9). Din pcate, pr s t a n din membr metod nu obinem cea mai bun constant n locul lui a t a a n1 n1 n ,d inegalitii (9). Am reuit s demonstrm c aceasta este at s a a a n1 straia depete cadrul acestei scurte note. t as s

ntr-adevr, s remarcm c inegalitatea lui Jensen implic imediat inega a a a a a n1 a1 + a2 + + a (a1 + a2 + + an ) a1 + an1 + + an1 n2 n 2 n

O aplicaie surprinztoare a inegalitii lui Surnyi o constituie i urm t a at s

Propoziie (Vasile Crtoaje). Pentru orice numere a1 , a2 , . . . , an > t inegalitatea an + an + + an + n(n 1)a1 a2 an 1 2 n a1 a2 an (a1 + a2 + + an )

1 1 1 + + + a1 a2 a

Demonstraie. Din nou, vom folosi inducia, dar vom vedea c pasu t t a se reduce exact la inegalitatea lui Surnyi. Pentru n = 3 , (11) coincid cu inegalitatea lui Schur. S presupunem acum c (11) este adevrat pe a a a a a variabile i fie a1 , a2 , . . . , an > 0. Aplicnd ipoteza inductiv pentru fiecar s n 1 numere dintre a1 , a2 , . . . , an , obinem t X X X 1 , an1 + (n 1)(n 2)a1 a2 an a1 a2 an ai aj i ai
i6=j i6=j i6=j

nsumnd relaiile din (12), rezult inegalitatea t a n ! n ! n X X X ai an1 an + n(n 1)(n 2)a1 a2 an i i
i=1 i=1 i=1 n X i6=j

Fie acum A =

n n X X X X 1 1 = ai (Aaj ) B = nAB AB AB +n = (n ai aj j=1 j=1


i6=j i6=j

i=1

n P

ai , B =

n P 1 . Este evident lanul de egaliti urmtor t at a i=1 ai

X X 1 a1 a2 an ai ai j=1
i6=j

care, mpreun cu (13), implic a a ! n n ! n X X X ai an1 an +n(n1)(n2)a1 a2 an a1 a2 an (n+( i i


i=1 i=1 i=1

Combinnd acum (1) cu (14), obinem t (n 2)


n X i=1

an i

+ n(n 1)(n 2)a1 a2 an (n 2)a1 a2 an

care nu este altceva dect (11) nmulit cu n 2 . Pasul inductiv fiind de t a propoziia este dovedit. t a S observm c i (11) este o rafinare a inegalitii mediilor. Scriind-o a a as at

n X
i=1

ai

ne ntrebm dac nu cumva este mai slab dect (9). Rspunsul este ne a a a a inegalitatea n n2 a1 + a2 + + an a1 a2 an n1 n 1 1 1 a1 a2 an (a1 + a2 + + an ) + + + a1 a2 an a nu este adevarat pentru orice a1 , a2 , ..., an > 0 . Aceasta se observ ime a a1 0 i a2 = a3 = ... = an = 1 . ns nici (11) nu este mai tare dect ( s a rezult iari uor. a as s

an + an + + an 1 2 n a1 a2 an n a1 a2 an 1 1 1 + + + (a1 + a2 + + an ) n a1 a2 an

n ncheiere, menionm c, folosind aceeai tehnic precum cea utiliz t a a s a a demonstra (11), se poate arta c inegalitatea a a

(a1 + a2 + + an )n n a1 a2 an 1 1 1 + + + (n 1)n1 (a1 + a2 + + an ) a1 a2 an este adevrat pentru a1 , a2 , ..., an > 0 i c este mai tare dect (11). D a a s a aceasta i multe altele, ntr-o alt poveste. . . s a

Bibliografie 1. Colecia revistei Kvant. t 2. D. S. Mitrinovi J. E. Peari A. M. Fink - Classical and New Ineq c, c c, Analysis, Kluwer Academic Publishers, 1993. 3. T. Andreescu, V. Crtoaje, G. Dospinescu, M. Lascu - Old and Ne lities, Gil Publishing House, 2004.

Rapoarte determinate de o cevian a i o secant ntr-un triunghi s a


Titu ZVONARU 1 , Bogdan IONIT 2 A

n triunghiul ABC consider m ceviana AD, cu D (BC). Dac o s a a tersecteaz laturile AB, AC si ceviana AD n punctele M, N , respectiv a sunt adev rate urm toarele dou relaii: a a a t AM AN BC AP AM BD AC P N MB N C = 1 (R1 ) , = AN AM AB DC AN P M PD + DC BD MB NC Demonstraie. (1) Fie M 0 , N 0 , B 0 , C 0 t A proieciile punctelor M, N , B, respectiv C pe t dreapta AD. Folosind triunghiuri dreptunghice N asemenea, avem: 0 0 N AM MM BB BD P = = ; ; 0 AB BB DC CC 0 M B M AC CC 0 NN0 PN = = ; , AN NN0 PM MM 0 de unde, prin nmulire, obinem relaia (R1 ). t t t D B AM AN (2) Notm a = x, = y. Dac a C MB NC AM AN AP = = i relaia (R2 ) este adevrat. s t a a M N k BC, atunci PD MB NC Dac x 6= y, fie {S} = MN BC; presupunem a A c punctul B este situat ntre S i C. Cu teoa s rema lui Menelaus aplicat la 4ABC i transvera s N SB N C M A = 1, de sala SMN obinem t SC N A MB P y ay SB M = , adic SB = a . unde rezult a SB + a x xy Aplicnd acum teorema lui Menelaus n 4ABD cu SB P D M A = 1, adic S a transversala SMP avem D B SD P A MB PA SB PA BC xy = x i obinem s t = , care este tocma PD SB + BD PD BD x + DC y

s Prezentm n continuare cteva aplicaii ale relaiilor (R1 ) i (R2 ). a t t Problema 1. Laturile (AB), (BC), (AC) ale triunghiului ABC sun cercului nscris de centru I n punctele D, E, respectiv F . Bisectoarea in [ unghiului BIC intersecteaz latura BC n punctul M. Not m {P } = F a a \ (Propus de S se demonstreze c (DP este bisectoarea unghiului F DE. a a a Olimpiada Mediteranean de Matematic n 1998.) a a
1 2

Profesor, Comneti (Bacu) a s a Profesor, Bucureti s

A Soluie. Cu notaiile obinuite ntr-un triunghi, t t s avem AE = AF = p a, BD = BF = p b, B E CD = CE = p c, DF = 2 (p b) sin , DE = 2 P C F s 2 (p c) sin . Cu teorema bisectoarei i teorema I 2 C sin BM BI 2. sinusurilor, n 4BIC obinem t = = B MC CI sin B D M 2 AF MB AC P E = 1, de unde rezult Aplicnd relaia (R1 ), avem t AB M C AE P F B B B C sin c sin c sin sin2 PE c 2 pa = 2 = 2 2 = = C C B PF pa b 2 C sin b sin b sin sin 2 2 2 2 C C c sin (p c) sin ab (p a) (p c) 2 2 = = = B B ac (p a) (p b) b sin (p b) sin 2 2 i, conform reciprocei teoremei bisectoarei, (DP este bisectoarea unghiulu s

Problema 2. Fie ABC un triunghi si M, N (BC), P, Q (AC), R, puncte astfel nct BM = CN = CP = AQ = AR = BS = x, unde min {AB, BC, CA}. Fie A1 , B1 , C1 puncte aparinnd segmentelor (SP t (MQ) astfel nct AA1 , BB1 , CC1 sunt ceviene de rang k n triunghiu BRN , respectiv CQM. Demonstrai c dreptele AA1 , BB1 , CC1 sunt co t a (Generalizarea unei probleme propuse de Constantin Cocea n RMT - 1/ se obine pentru k = 0.) t A Soluie. Notm cu A0 intersecia dreptelor t a t AA1 i BC. Avem AS = c x, AP = b x, s k A1 S cx = i cu relaia (R1 ) obinem s t t A1 P bx S 0 A1 AS A B AC A1 P 0 = 1, de unde rezult c a a AB A C AP A1 S k+1 c bx A0 B . = B A0 C b cx A s a Consideraii analoage relativ la punctele B 0 i C 0 . Concluzia rezult t reciproca teoremei lui Ceva.

Problema 3. Laturile (AB), (BC), (AC) ale triunghiului ABC sun cercului nscris de centru I n punctele C1 , A1 , respectiv B1 . Dac B2 a locul laturii (AC), demonstrai c dreptele B1 I, A1 C1 si BB2 sunt c t a (Olimpiad , Republica Moldova) a t a Soluie. Fie B 0 proiecia vrfului B pe latura AC. Notm cu M p t intersecie al medianei BB2 cu B1 I. Deoarece B1 I AC, rezult c M B t a a

obinem t
0

b B1 B B1 = p a c cos A, B1 B2 = (p a), deci 2 B2 C1 b2 + c2 a2 I pac BM 2bc = = M b M B2 (p a) 2 A1 B b2 + c2 a2 b+ca a2 c2 b (a c) ab+c b = = . = b (b + c a) b (a c) b Dac A1 C1 BB2 = {M 0 } cu relaia (R2 ) avem a t pb pb b BM 0 pb 2 (p a) (p c) a pc pa = = = 0B b pb b pb M 2 (p a) (p c) pa+pc + 2 pc 2 pa i rezult c M M 0 ; dreptele A1 C1 , B1 I, BB2 sunt concurente. s a a

BM B 0 B1 = . Dar M B2 B1 B2

A B

Problema 4. Fie ABC un triunghi cu AB < AC, I centrul cercu si M mijlocul laturii BC. Not m cu D intersecia dintre IM si AB, a t intersecia lui CI cu perpendiculara din B pe AI. S se demonstreze c t a a (Problema 2915, Crux Mathematicorum - 2/2004, propus de Toshio Sei a Soluie. Notm {B1 } = BE AC, {B 0 } = t a A s BI AC. n triunghiul ABB1 , bisectoarea AI este i nlime, deci AB1 = AB = c, B1 C = b c. Cu teoat D BE a rema bisectoarei n 4BCB1 obinem . t = B EB1 bc I BD Fie acum a a = x. Cu relaia (R2 ) rezult c t DA BM AC x E BI MC = ; B M BM IB 0 + B0A x CB 0 MC ab bc a BI BM = , deci = 1, CB 0 = , B0A = , dar 0 MC a+c a + c IB CB 0 bx a a+c bx = x= . 1= bc ab b a + cx bc + x a+c a+c BD BE i rezult c DE k AC. s a a = Deci EB1 DA Observaie. Alte soluii pentru Problema 2 (cazul k = 0) i Problem t t s gsite n [1]. a

Bibliografie 1. G. Popa, P. Georgescu - O metod de demonstrare a concurenei un a t RecMat - 1/2004, 2932.

Asupra unor ecuaii diofantice ptratice t a


Gheorghe MOLEA1

Scopul acestui material este de a prezenta cteva procedee de rezolva ecuaii diofantice de gradul al doilea cu dou sau trei necunoscute accesibil t a gimnazial i a le aplica n rezolvarea ctorva probleme ntlnite n Gazeta M s sau alte publicaii. t Teorema 1. Soluiile ntregi ale ecuaiei t t ax2 + bxy + cy + d = 0,

unde a, b, c Z si d Z, sunt p c q ap + 2ac Z Z; p, q Z, pq = ac2 + b2 d , (x, y) 2 b b

Ca urmare, soluiile ntregi ale ecuaiei (1) sunt soluiile ntregi ale sistem t t t bx + c = p, abx + b2 y ac = q; p, q Z, 2 t unde pq = ac + b2 d . Rezolvarea acestor sisteme conduce la soluiile pc q ap + 2ac x= , , y= b b2 deci soluiile ntregi ale ecuaiei (1) sunt date de (2). t t

s a Demonstraie. nmulind ecuaia cu b2 i apoi scznd ac2 din amb t t t obinem t ab2 x2 + b3 xy + cb2 y + db2 ac2 = ac2 (bx + c) abx + b2 y ac =

Aplicaie. Rezolvai n numere ntregi ecuaia 9x2 4xy y = 1 (L t t t Agore E:12418, G.M. - 10/2002). Soluie. Avem a = 9, b = 4, c = 1, d = 1, deci ac2 + b2 d = t (p, q) {(1, 7) , (7, 1) , (1, 7) , (7, 1)}. Tinnd seama de (2), obine t (x, y) {(2, 5) , (0, 1)}. Teorema 2. Soluiile ntregi ale ecuaiei t t a, b, c Z , sunt ax2 + bx ay 2 + c = 0,

p + q 2b q p 2 , Z Z; p, q Z, pq = b 4ac (x, y) 4a 4a Demonstraie. nmulind (3) cu 4a i adunnd apoi b2 la ambii membr t t s 2 2 2 2 2 2 4a x + 4abx + b 4a y = b 4ac (2ax + b 2ay) (2ax + b + 2ay) Revine la a gsi soluiile sistemelor liniare a t
2

t s t cu pq = b 4ac. Soluiile ntregi ale acestora, deci i ale ecuaiei (3), su (4).
1

2ax + b 2ay = p,

2ax + b + 2ay = q;

p, q Z,

Profesor, Scoala "Basarab I", Curtea de Arge s

Aplicaie. S se rezolve n numere ntregi ecuaia x2 + 13x = y 2 t a t Achim C:2487, G.M. - 3/2002). Soluie. Avem a = 1, b = 13, c = 26 i b2 4ac = 65. Deci (p, q) t s (65, 1) , (5, 13) , (13, 5) , (1, 65) , (65, 1) , (5, 13) , (13, 5)} i, din s ile cutate sunt (x, y) {(10, 16) , (10, 16) , (2, 2) , (2, 2) , (23, 16) , a (11, 2) , (11, 2)}. t Teorema 3. Soluiile n Z3 ale ecuaiei t 2 2 x + ay = a + b2 z 2 , a, b Z ,

Demonstraie. Ecuaia (5) se mai scrie (x + bz) (x bz) = a (z y t t este echivalent cu ansamblul sistemelor liniare a m (x + bz) = an (z y) , n (x bz) = m (z + y) ; m, n Z.

sunt date de x = t 2amnbm2 +abn2 , y = t an2 2bmnm2 , z = t an2 +m2 , t, m,

Rezolvate n raport cu x i y, acestea conduc la soluiile s t 2 2 2 z 2amn bm + abn z an 2bmn m2 x= , y= ; m, n an2 + m2 an2 + m2 2 Notnd z = t an + m2 , t Z, obinem ca posibile soluii ale ecuaiei (5 t t t (x, y, z) cu x, y, z dai de (6). Se verific uor c toate aceste triplete sunt t a s a ecuaiei date. t

Aplicaie. S se arate c exist o infinitate de numere ntregi x, y, z a t a a a x2 + y 2 = 2z 2 (Dana i Eugen Radu Probleme de matematic pentru s a si examene). Soluie. Lund n (6) a = b = 1, gsim c soluiile ecuaiei x2 + y2 = t a a t t date de x = t n2 2mn m2 , y = t n2 + 2mn m2 , z = t m2 + n2 , t, m Exerciiu. Rezolvai n Z Z ecuaia x2 y 2 = 3z 2 . t t t Teorema 4. Soluiile ecuaiei t t ax2 + bxy + c2 y 2 = z 2 , n mulimea Z sunt date de t 2 x = t bm 2cmn , y = t n2 am2 , nx + cmy = mz,
3

a, b, c Z, t, m,

Demonstraie. Scriind ecuaia dat n forma x (ax + by) = (z cy) t t a procednd ca n cazul ecuaiei (5) obinem sistemele liniare t t i n cele din urm rezultatul dorit. s a
2

z = t bmncn2 acm2 ,

amx + (bm cn) y = nz;

m, n Z,

t a t Aplicaie. Se consider a, b, c Z . Demonstrai c ecuaia t a ax + bx + c2 = z 2 are o innitate de soluii n mulimea numerelor raion t t t Buth C:2690, G.M. - 12/2003).

Aplicaie. S se arate c ecuaia x2 + xy + y 2 = 1 are o infinitate t a a t numere raionale (L. Panaitopol, D. Serbnescu Probleme de teoria t a si combinatoric pentru juniori, Problema 153). a Soluie. Procedm ca n aplicaia precedent. Lund a = b = c = 1 t a t a (7), se obine ecuaia dat. Din (8), avem t mn m2 n2 = 1 i apoi t t a s x= m2 2mn Q, mn m2 n2 y= n2 m2 Q; mn m2 n2

bmn cn2 acm2 bm2 2cmn , z= ; m, n Z cu n2 am n2 am2 n2 am2 Aadar, ecuaia dat are o infinitate de soluii (x, y) cu x, y Q. s t a t x=

Soluie. Ecuaia din enun se obine din (7) pentru y = 1. Tinnd sea t t t t urmeaz c t n2 am2 = 1 i, deci, a a s

m, n Z.

Observaie. Pentru a = c = 1 i b = 0 ecuaia (7) devine ecuaia p t s t t x2 + y 2 = z 2 iar (8) conduce la soluiile acesteia: x = 2mnt, y = m , t z = m2 + n2 t cu t, m, n Z.

Truelul

Un truel este asemntor cu un duel, dar exist trei participani n loc a a a t

Domnii X, Y i Z se hotrsc s rezolve un conict truelndu-se cu s aa a pn cnd va rmne n viaa doar unul dintre ei. X este cel mai pros a a t nimerete n medie inta doar o dat din trei. Y este un trgtor mai bun, s t a a a inta de dou ori din trei. Z este cel mai bun trgtor, nimerete inta t a a a s t dat. Pentru a face truelul mai echitabil, X trage primul, urmat de Y a este n viaa), urmat de Z (dac mai este n viaa) . a. m. d., lund ostilit t a t s nceput, pn cnd rmne n viaa numai unul singur dintre ei. a a t

ntrebarea este urmtoarea: asupra cui ar trebui s trag X primul s a a a a Not. Rspunsul se gsete la p. 70. a a a s

O caracterizare a funciilor convexe cu ajuto t derivatelor laterale


Florin POPOVICI 1

Vom presupune cunoscute proprietile elementare ale funciilor convexe at t privina pot consultate [1] sau [3]. Un rol aparte l va juca urmtoarea t a Teorem (O. Stolz; v. Teorema 1.3.3 din [1] sau Propoziia 7.1.4 din a t f : I R este o funcie convex pe intervalul deschis I, atunci f este de t a stnga si la dreapta n orice punct din I si, dac x, y I si x y, avem a 0 0 0 0 f (x) f+ (x) f (y) f+ (y) .

i, conform Corolarului 3 al Propoziiei 1.4.3 din [3], funcia f este conve s t t Corolar. Fie I R un interval deschis. O funcie f : I R este con t si numai dac f este derivabil la stnga si la dreapta pe I si are loc (1) pe a a x, y I cu x < y.

Nota de faa are strns legtur cu lucrarea [2] i o presupune cunos t a a a s liznd derivatele laterale n locul derivatei, n [2] este presentat o gen a teoremei de medie a lui Lagrange i, ca o consecina, se face urmtoarea l s t a funciile convexe: t Teorema lui Lagrange pentru funcii convexe ([2], Corolarul 1 t I R o funcie convex , I deschis. Atunci, a, b I cu a < b, c (a, b t a f (b) f (a) 0 0 f (c) f+ (c) . ba Scopul nostru este de a dovedi o reciproc a Teoremei lui Stolz i, a a s obine o caracterizare a funciilor convexe Corolarul de mai jos. t t Teorem. Dac f : I R, I interval deschis, este derivabil la st a a a dreapta pe I si x, y I, x < y, au loc inegalit tile (1), atunci f este conv a Demonstraie. Fie x, y, z I cu x < y < z. Teorema precedent t a c (x, y) i d (y, z) astfel nct s f (y) f (x) f (z) f (y) 0 0 0 0 f (c) f+ (c) i f (d) f+ ( s yx zy 0 a a Cum, datorit ipotezei, c < d implic f+ (c) f (d), rezult c a a 0 1 1 1 f (y) f (x) f (z) f (y) y z 0 x yx zy f (x) f (y) f (z)

Bibliografie 1. C. P. Niculescu - Convex Function, Basic Theory and Applications, Un Press, Craiova, 2003. 2. F. Popovici - O generalizare a teoremelor de baz ale calculului diferenial a t 2/2004, 104-105. 3. Gh. Sirechi - Calcul diferenial si integral, vol. I, Ed. St. i Encicl., Bu t t s
1

Profesor, Liceul "N. Titulescu", Braov s

Asupra problemei G67


Adrian ZAHARIUC 1
n RecMat - 2/2004, p. 157, am propus urmtoarea problem: a a

Problema 1. Spunem c un num r natural este decompozabil dac a a scrie ca suma a dou numere naturale cu aceeai sum a cifrelor. S s a s a a exist o infinitate de numere naturale care nu sunt decompozabile. a

n original, problema se referea la o baz de numeraie oarecare b. U a t ment simplu (un exemplu trivial i un argument de paritate) ne arat c s a b impar, mulimea numerelor decompozabile coincide cu mulimea nume t t deci problema este epuizat. n cazul b par, numerele nedecompozabile a mai rare. n cele ce urmeaz, ne vom referi la cazul decimal, b = 10, pent a complicaii inutile. Cititorul poate extinde folosind exact aceeai tehnic t s a obinut n cazul b = 10 pentru orice numr b par. t a Scopul acestei Note este de a da o form general simpl tuturor nume a a a compozabile. Voi da rspunsul la aceast problem nc din enun: a a a a t

Problema 2. Un num r natural n este nedecompozabil dac si numa a a una din formele: 19 . . . 99, 39 . . . 99, 59 . . . 99, 79 . . . 99, 99 . . . 99, cu un nu de cifre sau 29 . . . 99, 49 . . . 99, 69 . . . 99, 89 . . . 99, cu un num r par de cif a

Soluie. S rezolvm nti partea mai delicat: dac n nu are nic t a a a a tre formele menionate mai sus, atunci n este decompozabil. Aceasta r t urmtoarele dou leme: a a Lema 1. Pentru orice astfel de n, exist a n astfel nct a s(a) s(n a)(mod 2).

Demonstraie. Dac s(n) este par, atunci lum direct a = 0, deci t a a s(n) impar. n acest caz, n trebuie s aib o cifr, n afar de prima, a a a a 9 deoarece altfel ar avea una dintre formele interzise. Fie c valoarea ac i p poziia ei (de la dreapta la stnga). Evident, putem alege aceast s t a nct naintea ei s nu fie cifra zero. Atunci lum a = 10p1 (c + 1). La a a a + (n a) = n se face un singur transport, deci s(n a) + s(a) = s(n) + 9 0(mod 2) s(a) s(n a)(mod

Lema 2. Dac exist a n astfel nct s(a) s(n a)(mod 2), atu a a A n astfel nct s(A) = s(n A). Demonstraie. Fie k numrul de cifre ale lui n. Notm t a a a = a1 a2 . . . ak , unde cteva dintre primele cifre pot fi 0. Stim c a n a = b1 b2 . . . bk ,

0 a1 + + ak + b1 + + bk = (a1 + b1 ) + + (ak + bk )(mod


1

Elev, cl. a X-a, Colegiul Na ional "Ferdinand I", Bacu t a

i Bi = ai + bi Ai , i = 1, k. Este clar c Ai , Bi {0, 1, . . . , 9}. Avem s a A1 A2 . . . Ak +B1 B2 . . . Bk = Avem c a


k X i=1 k X i=1 k X i=1

deci numrul elementelor mulimii I = {i {1, 2, . . . , k}; 2 nu divide ai a t par. Atunci exist I = I1 I2 o partiie a lui I n dou clase cu acelai a t a s elemente. Fie iI / (ai + bi ) /2, (ai + bi + 1) /2, i I1 Ai = (ai + bi 1) /2, i I2 (Ai + Bi ) 10ki =
k X i=1

(ai + bi ) 10ki = a+
k

Ai =

dar

k X ai + bi i=1

|I1 | |I2 | X ai + bi = , 2 2 2 i=1


k X i=1

Ai +

s a deci s(A) = s(n A), unde A = A1 A2 . . . Ak i prima parte este rezolvat

k X i=1

Bi =

k X i=1

(ai + bi )

Ai =

k X i=1

Bi ,

S presupunem acum c n are una dintre formele interzise i s demo a a s a este nedecompozabil. S presupunem prin absurd c exist a, b n cu a a a astfel nct s(a) = s(b). Observaia esenial este c la adunarea a + b = n t t a a transporturi. S presupunem prin absurd c totui am avea transporturi a a s cifra cea mai nesemnificativ (cea mai din dreapta) la care se face trans a faptul c este cea mai nesemnificativ cifr cu aceast proprietate rezult a a a a a din dreapta sa nu s-a facut transport. Atunci aceast cifr este obtinua a a t adunarea unei cifre m a lui a cu o cifr n a lui b, dar m + n 18, iar restul a lui m + n la 10 trebuie s fie 9, deci m + n = 9. Rezult c la acea cifr n a a a a transport, contradicie. Atunci s(n) = s(a) + s(b) = 2s(a), deci s(n) este t nici unul dintre numerele care intr n discuie nu are suma cifrelor par a t as la o contradicie, deci n este nedecompozabil. t

1. n "egalitatea"

XXIII = II V II mutai un beior astfel nct s obinei o egalitate aproximativ ct ma t ts a t t a Roxana Cpn, e a at a

2. Adugai o cifr par la dreapta unui numr, astfel nct s obinei a t a a a a t t impar. Gabriel P Not. Rspunsurile se gsesc la p. 26. a a a

Matematic i algoritmi as
Irina MUSTAT 1 A

Majoritatea problemelor de informatic se rezolv cu ajutorul unei ser a a ritmi, care se adapteaz de la caz la caz. Din problemele de mai jos, se v a uneori anumite observaii matematice fie uureaz abordarea acestor pro t s a ntresc rezultatul. a

1. La Concursul Info-Oltenia din 2004 s-a dat urmtoarea problem a

S se afieze num rul de descompuneri ale unui num r dat n sum a s a a a naturale nenule. O descompunere si o permutare a ei nu se vor num ra de a (Enun adaptat). t

Se observ c att codul, ct i resursele consumate sunt mult mai restr a a s n cazul backtracking-ului. 2. O alt problem de acelai gen ar fi: a a s Afiai toate soluiile ecuaiei 3x + y + 4xz = 100 cu x, y, z N. s t t t
1

Un programator "contiincios" va aplica probabil backtracking (algo s genereaz toate soluiile de descompunere) i apoi le va contoriza. De a t s de abordare este corect, se tie c backtracking-ul are o complexitate a s a operaii efectuate) exponenial, determinnd un timp de execuie mai n t t a t S privim problema altfel: vom ncerca s gsim o funcie recursiv a a a t turneaz numrul cutat. Pentru a evita numrarea simultan a unei solu a a a a a t mutrii ei, vom lua numerele care apar n descompunerea lui n n ordine cr a Presupunem c putem construi o funcie recursiv f (n) (cu un singur p a t a care returneaz numrul cerut. Atunci f (n) = f (n 1) + f (n 2) + a a (Adic din n scdem k iar numrul de descompuneri posibile va fi f (n a a a facem suma dup k). Numai c, n acest caz apare o problem major: d a a a a n = 2+n2 i calculm f (n 2), la un moment dat primul termen al desc s a ar fi 1, deci am obine n = 2 + 1 + . . . , ceea ce contrazice ordonarea alea t numerele din descompunerea lui n. Se impune, aadar, necesitatea construirii unei funcii cu doi paramet s t cu urmtoarea semnificaie: f (n, k) reprezint numrul de descompuner a t a a ale lui n n sum de termeni k. Este clar c enunul problemei cere f a a t asemenea se observ c f (n, n) = 1 i f (n, k) = 0 pentru k > n. a a s Acum putem scrie definiia funciei f (n, k), creia i putem da ulterior t t a n, 1: n = k, 1, 0, n < k, f (n, k) = n1 P f (n l, l) , n > k.
l=k

Elev, cl. a XII-a, Colegiul Na ional, Iai a t s

Si n acest caz este de ateptat o aplicare de backtracking, dar putem s ecuaia: x (4z + 3) = 100 y. Acum raionm n felul urmtor. Alegem z t t a a 100 24 i, pentru z astfel luat, alegem x de la 1 la s . Apoi l obinem t 4z + 3 100 x (4z + 3) i tiprim tripletul (x, y, z). n final tiprim i soluiile (0 s a a s t z N. Prin metoda de mai sus, obinem ntr-adevr toate soluiile (z poat t a t valoare de la 0 la 24, deoarece pentru z 25 i x nenul avem 4z + 3 > 100 s 100 100 i, cum x N, x s ). 100 y 100, deci x 4z + 3 4z + 3 Observaie. Numrul 100 poate fi nlocuit cu orice n N . t a 3. Un alt exerciiu interesant este urmtorul: t a

Se dau m numere ntregi nenule b1 , . . . , bm si n numere ntregi nenule a a t t a an . S se determine o submulime a mulimi B = {b1 , . . . , bm } care s m valoarea expresiei E = a1 x1 + a2 x2 + + an xn , stiind c m n si xi {b1 , . . . , bm }, i = 1, n. a

Sortm cresctor ambele iruri (ai ) i (bi ). Evident, la ambele iruri a a s s s numerele negative n partea stng i cele pozitive n partea dreapt. as a Vom enuna inegalitatea rearanj rilor, pe care o vom aplica la aceast t a a

este maxim pentru (i) = i, i = 1, n, si minim pentru (i) = n + 1 i,

Fie a1 a2 an , b1 b2 bn siruri de nume n P S () = ai b(i) , unde este o permutare a mulimii {1, 2, . . . , n}. At t
i=1

{ak+1 , . . . , an } i {bmn+k+1 , . . . , bm }. s S remarcm c programul propriu-zis va fi astfel redus la nite pai a a a s s s a simpli: sortarea cresctoare a irurilor (ai ) i (bi ), aflarea numrului k d a s negative ale lui (ai ) i, n final, tiprirea soluiei: primele k numere din st s a t s (bi ) i ultimele n k numere din dreapta din (bi ).

Sirul (ai ) este alctuit din k numere strict negative i n k numere n a s pozitive. Pentru a maximiza suma este necesar s nmulim numerele p a t (ai ) cu cele mai mari numere din (bi ) i numerele negative cu cele mai m s din (bi ) (deoarece cele mai mici numere strict negative din (bi ) au modulul deci ai bi > 0 va putea fi maximizat, iar cele mai mici numere pozitive n numere negative vor da cele mai mici n modul numere negative, deci ce termeni). Aadar vom nmuli k numere negative ale irului (ai ) cu cele m s t s numere ale irului (bi ), iar cele n k numere pozitive ale irului (ai ) le vo s s a t cu cele mai mari n k numere ale irului (bi ). Aceste dou submulimi s suprapune, ntruct m n, iar numrtoarea ncepe din capetele opuse aa a t a (bi ) sortat. Din inegalitatea rearanjrilor obinem c permutarea lui { k P pentru care ai bi este maxim e permutarea identic i analog pentru a as
i=1

4. Un colier este format din m rgele roii si albastre. La un moment a s colierul ntr-un punct si l ntindem n linie dreapt . Apoi scoatem de pe fir a din cap tul stng pn cnd ntlnim una din cealalt culoare. Analog a a a pentru cap tul drept. Dndu-se datele despre colier sub forma unui sir rarr a indic culoarea m rgelelor succesive), s se calculeze num rul maxim de a a a a care l putem scoate, precum si locul unde trebuie t iat colierul. (Enun a t (Olimpiada Internaional de Informat t a

Considerm un vector cu n componente (adic un ir a1 , . . . , an ). Ini a a s t 0 toate elementele vectorului, ne plasm pe primul element al vectorului i a s colierului caracter cu caracter, ct vreme caracterul citit este acelai cu ce a s mrim cu 1 valoarea reinut de primul element al vectorului. Cnd ca a t a schimb ne poziionm pe urmtoarea component a vectorului i contin a t a a a s la sfritul irului cu acelai procedeu. De fapt, vectorul reine lungim s s s t secvenelor succesive de mrgele colorate la fel. t a Cnd tiem colierul, o putem face n dou feluri: fie n interiorul une a a colorate la fel, fie desprind dou secvene colorate diferit. n primul at a t ce ntindem colierul, numrul maxim de mrgele ce pot fi scoase este n a a mrgele al acelei secvene, n timp ce n al doilea caz, numrul maxim ob a t a suma numerelor de mrgele din dou secvene consecutive. Este evident a a t noi e mai convenabil s tiem colierul ntre dou secvene. a a a t Maximul care poate fi obinut este maximul dintre sumele numerelor d t a dou secvene succesive. Acesta se poate gsi uitndu-ne la vectorul a t a anterior. Fie k numrul de elemente nenule al acestui vector. Dac k a a comparm sumele v [1]+v [2], v [2]+v [3], . . . , v [k 1]+v [k]. Dac k este im a a oricare dou secvene consecutive au culori diferite, rezult c secvenele a t a a t de v [1] i v [k] au aceeai culoare, deci la "nchiderea" colierului se vor s s singur secvena de lungime v [1] + v [k]. n acest caz, trebuie aflat maxi a t v [k] + v [1] + v [2], v [1] + v [k] + v [k 1], v [i] + v [i + 1], i = 2, k 2. Locul unde trebuie tiat colierul se obine uor. Presupunem c facem a t s a i P ntre v [i] i v [i + 1]. Atunci poziia unde vom tia este s t a v [j].
j=1

Soluiile problemelor enunate la p. 23. t t

1. Mutnd un beior de la numrtor deasupra numrului din memb ts aa a obine t XXII = , V II 22 XXII = 3 care este o egalitate aproximativ cu o eroare foarte mic ( a a V II 7 iar 3, 14159 . . .).

2. Se adaug cifra 0, dar . . . la exponent, obinndu-se numrul im a t a exemplu, 70 = 1 etc.).

Asupra unei probleme de concurs


Alexandru NEGRESCU 1

La cea de-a VI-a ediie a Concursului interjudeean de matematic "Rad t t a care a avut loc la Vaslui n perioada 5 - 7 noiembrie 2004, a fost propus a clasa a X-a urmtoarea problem: a a Dac A, B, C sunt m surile unghiurile unui triunghi, atunci a a 1 1 1 + + 2 3. sin A sin B sin C Precizai cnd are loc egalitatea. t

V prezentm n continuare cinci soluii pentru aceast inegalitate, ls a a t a a dumneavostr s decidei care este cea mai frumoas. a a t a

Soluia I (prezentat n baremul de corectare). Deoarece sin A, s t a sunt strict pozitive, avem X X 1 X 1 9 sin A 9 sau P . sin A sin A sin A Conform inegalit tii Cauchy - Buniakowski - Schwarz, avem a X 2 X X 1 cos 2A 9 3X = cos 2A sin2 A = 3 sin A 3 2 2 2 Cum X cos 2A = 2 cos2 C 1 2 cos (A B) cos C = i 1h = (2 cos C cos (A B))2 + sin2 (A B) 3 2 rezult c a a X 2 9 9 X 27 3 3 sau sin A . sin A + = 2 4 4 2 P 1 2 9 = 2 3, adi Combinnd rezultatele precedente obinem t sin A 3 3 tatea dorit. Se constat uor c egalitatea are loc dac i numai dac a a s a a s a este echilateral.

Soluia a II-a (Alexandru Negrescu). Conform teoremei sinusur t a sin A = etc. Inegalitatea de demonstrat devine 2R 2R 2R 2R 1 1 1 3 + + 2 3R + + 3 1 1 1 a b c a b c a + b + c 1 1 1 a+b+c 2p 2p Cum 3/ + + = , rmne s artm c a a aa a c 3 3 3 a b 3 3 p R, ceea ce este o cunoscut inegalitate a lui Mitrinovi. a c 2
1

Elev, cl. a X-a, Colegiul Na ional "August Treboniu Laurean", Botoani t s

Soluia a III-a (Alexandru Negrescu i prof. Liliana Tomia). t s t inegalitii mediilor avem at 1 1 3 1 + + . 3 sin A sin B sin C sin A sin B sin C 3 3 ([1], pag. 123-124), cu egalitate pentru Stim c sin A sin B sin C a 8 echilateral. nlocuind mai sus, obinem t s . 3 33 . 3 1 1 1 + + 3 =3 = 2 3. sin A sin B sin C 8 2

Soluia a IV-a (Alexandru Turcanu i Serban Vatavu, elevi, C.N t s nescu", Botoani). Conform inegalitii mediilor, avem s at 1 1 1 9 + + . sin A sin B sin C sin A + sin B + sin C Dar, conform inegalit tii lui Jensen pentru funcii concave are loc a t sin

A+B+C sin A + sin B + sin C sau sin A + sin B + sin C 3 3 care, nlocuit n relaia precedent, conduce la inegalitatea dorit. a t a a

Q 3 Pe de alt parte, cu inegalitatea mediilor i innd seama c sin A a s t a i s v ! u pQ P . u 33 2 233 sin A sin A 6 3 t Q 2Q 6 = = qQ 8 sin A sin A 3 ( sin A)2 Prin adunare, obinem inegalitatea (), care este echivalent cu cea de de t a

Soluia a V-a (Alexandru Negrescu). Ridicnd relaia din enun t t t obinem t X 1 X 1 12 +2 2 sin A sin B sin A sau P X 1 sin A 12. +2 Q sin A sin2 A Q 1 Cu inegalitatea mediilor i innd seama c cos A , rezult c s t a a a 8 X 1 9 9 9 Q = 4. P 2 = 2 + 2 cos A 2+2 1 sin2 A sin A 8

Bibliografie 1. T. Cohal - Probleme de trigonometrie, Editura Moldova, Iai, 1994. s 2. M. Ganga - Manual pentru clasa a IX-a, profil M1, M2, Editura Mathpres 2003. 3. M. Ganga - Manual pentru clasa a X-a, profil M1, Editura Mathpres 2003.

Din nou asupra unei probleme de concur


Constantin APOSTOL1

n revista "Recreaii matematice" nr. 2, iulie - decembrie 2004, a apru t a profesorilor D. Mihalache i M. Tetiva, care "generalizeaz" o problem s a am propus-o la Concursul Naional de Matematic "Laureniu Duican" t a t mai 2003, cu urmtorul enun: a t

b b b b m(A) m(B) m(C) m(D) Printr-un calcul simplu, din faptul c a = = = 8 12 5 11 b b b b c m(A) = 80 , m(B) = 120 , m(C) = 50 , m(D) = 110 . a Problema am propus-o pentru clasa a VII-a, aa c o rezolvare trigonom s a era de ateptat din partea elevilor. Profesorii nominalizai mai sus, au dat s t soluie trigonometric, care depete nivelul de pregtire al elevilor, dar t a as s a soluie geometric, la nivelul programei colare, care le-a permis i o "gen t a s s n sensul elaborrii unor probleme rezolvabile pe aceeai idee. a s n rndurile care urmeaz mi propun s exprim soluiile cu care am t a a t blema comisiei de concurs, care se ncadreaz cerinelor programei de conc a t este aceeai cu a Olimpiadei Naionale pentru clasa a VII-a. s t

b b b b n patrulaterul convex ABCD, m surile unghiurilor A, B, C si D a porionale cu numerele 8, 12, 5 si 11. S se arate c dac (BD estre t a a a b b unghiului B, atunci (AC este bisectoarea unghiului A.

Soluia I. Prelungind latura (AB), obinem t t \ are msura de 60 c suplementul unghiului ABC a a b D i cum (BD este bisectoarea unghiului B, care are s a msura de 120 , deducem c (BC este bisectoarea a b exterioar a unghiului B din triunghiul ABD. a n triunghiul ABD, prin calcul, deducem c a \ = 40 i, deci, m(BDC) = 70 . Suple\ m(ADB) s \ mentul unghiului ADC, care se obine prelungind t B s latura (AD), are 70 . Aadar, (DC este bisec- A b toarea exterioar a unghiului D n triunghiul ABD. a Astfel, pentru triunghiul ABD, (BC i (DC sunt bisectoarele exte s b respectiv D; acestea sunt concurente n C. Deducem c b unghiurilor B, a b bisectoarea interioar a unghiului A. a

Soluia a II-a. Pe semidreapta (AB lum punctul E astfel nct AE = t a b arta c 4ADC 4AEC. Din triunghiul isoscel ADE, cu m(A) = 80 a a \ \ \ \ a a m(ADE) = m(AED) = 50 . Rezult m(EDC) = 60 , cci m(ADC) = \ deci m(ABD) = m(DBC) = m(CB \ \ \ (BD este bisectoarea unghiului ABC,
1

Profesor, Colegiul Na ional "Al. Vlahu a", Rmnicu Srat t t a

n continuare, voi prezenta o problem care se poate rezolva cu ideea a anterioare: b n patrulaterul convex ABCD m surile unghiurilor sunt: m(A) = 60 , m a b = 60 , m(D) = 135 . S se arate c dac (BD este o trisectoare a b m(C) a a a b b D, atunci (AC este bisectoarea unghiului A sau (CA este bisectoarea ung Soluie. Vom deosebi dou cazuri: t a b m(D) 135 \ I. cnd m(CDB) = = = 45 ; 3 3 b 135 m(D) \ = = 45 . II. cnd m(ADB) = Y 3 3 n cazul I, prelungind latura (AB), obinem t 45 \ D 45 m(CBX) = 180 105 = 75 , deci (BC este bi90 \ sectoarea unghiului DBX (1). Prelungind latura \ ) = 180 135 = 45 , deci (AD), obinem m(CDY t 60 30 \ (DC este bisectoarea unghiului BDY (2). Din (1) i (2), deducem c (AC este bisectoarea A s a b unghiului A. n cazul II, prelungind latura (CB), obinem t [ = 180 105 = 75 , deci (BA este bim(ABZ) \ sectoarea unghiului DBZ (3). Prelungind latura (CD), obinem t [ m(ADT ) = 180 135 = 45 ,

\ Avem, n patrulaterul convex DBEC, m(EDC) = \ = 60 , deci acest patrulater este = m(EBC) \ inscriptibil, de unde deducem c m(CED) = a \ = 60 . = m(CBD) \ \ s Din m(CED) = 60 i m(EDC) = 60 , deducem c triunghiul CDE este isoscel cu vrful C i a s deci, CD = CE. Rezult 4ADC 4AEC (LLL); a b aadar, (AC este bisectoarea unghiului A. s

45 45

90

\ deci (DA este bisectoarea unghiului BDT (4). Din (3) i (4), deducem c (CA este bisectoarea s a b unghiului C. Bibliografie

3 60

75 75

1. F. Diac - A XI-a ediie a Concursului Naional de Matematic "Laureniu t t a t Braov, 2003, G.M. - 11/2003, 433-438. s 2. D. Mihalache, M. Tetiva - Asupra unei probleme de concurs, RecMat 111-113. 3. C. Apostol - Preocup ri matematice, Ed. "Radical", 1966. a

Dou funcii cu aceeai derivat pe un inter a t s a nu difer neaprat printr-o constant a a a


Paul GEORGESCU 1 , Gabriel POPA2

Printre "cunotinele" dobndite de unii elevi n urma studierii Analize s t tice de liceu se numr, din pcate, i urmtoarea "teorem", menionat n aa a s a a t a [1], pag. 282: Dac f, g : (a, b) R au aceeai derivat pe (a, b), atunci ele difer a s a constant . a "Demonstraia" urmeaz linia de mai jos: t a Dac f 0 (x) = g 0 (x), x (a, b), atunci (f g)0 (x) = f 0 (x) g a x (a, b), deci f g este constant pe (a, b). a Desigur, nu avem neaprat c (f g)0 (x) = f 0 (x) g 0 (x), x (a, b) a a s ce nici mcar nu are sens pentru f 0 (x) = g 0 (x) = i deci demonstra a sus este invalid. a Observm c "f are derivat pe (a, b)" nu este acelar a a a si lucru cu "f este de a+b , care ar (a, b)"; un exemplu este dat de f : (a, b) R, f (x) = 3 x 2 a+b a+b 0 0 = fd pe (a, b) fr a fi derivabil pe (a, b), deoarece fs aa a 2 2 Dac, n plus, f i g sunt presupuse a fi derivabile pe (a, b), atunci rai a s t de mai sus este corect, conducnd la urmtorul binecunoscut rezultat: a Dac f, g : (a, b) R derivabile au aceeai derivat pe (a, b), atunci a s a printr-o constant . a Prezentm n continuare un exemplu, datorat matematicianului polone a law Ruziewicz, de un numr innit de funcii cu aceeai derivat pe u a t s a diferena oricror dou neind constant, [3]. Vezi, de asemenea, [2] pag. t a a a Fie s 1. mprim segmentul [0, s] n trei pri n aa fel nct segm at at s 1 lociu are lungimea , iar centrul su este centrul segmentului [0, s]. Acop a 3 1 segmentul mijlociu cu semicercul superior de diametru determinat de ace 3 tergem acest segment (exclusiv capetele). mprim apoi fiecare dintre s at segmente rmase n cte trei segmente n aa fel nct segmentele mijlo a s 1 at a a lungimea 2 , iar centrele segmentelor care sunt mprite s coincid cu ce 3 mentelor mijlocii. De asemenea, acoperim segmentele mijlocii cu semicercu 1 s s ametru 2 determinate de acestea i apoi tergem aceste segmente, exclusi 3 Repetnd aceast procedur, la pasul n vom avea de construit 2n1 sem a a 1 s s a s diametru n i de ters diametrele corespunztoare, exclusiv capetele, 3 Obinem deci o innitate numrabil de semicercuri i fie Ms reuniunea se t a a s rmase neacoperite i a semicercurilor. a s
1 2

Lector dr., Catedra de matematic, Univ. Tehnic "Gh. Asachi", Iai a a s Profesor, Colegiul Na ional, Iai t s

a t Este evident c Ms poate fi privit ca graficul unei funcii continue fs 1 t s t a 0, ; demonstraia acestui fapt folosete convergena uniform a unui 6 de funcii continue. t h i Rt , Fs (t) = 0 fs (x) ds, pentru 0 t Fie atunci Fs : [0, s] 0, 56 a a Fs este derivabil cu derivata continu, iar deoarece fs (x) 0, x [0, s] a a cresctoare. Mai mult, din modul de construcie a lui Ms se observ c pe a t 0 t1 < t2 s intervalul [t1 , t2 ] are un subinterval comun cu un interva de un semicerc, deoarece dup pasul n orice subinterval al lui [0, s] de lung a are un subinterval comun cu un interval acoperit de un semicerc. Cum pe aceste subintervale f ia valori strict pozitive i este continu, o s a Fs (t1 ) < Fs (t2 ), deci Fs este strict cresctoare. Mai mult, a 1 22 + 4 + . . . = , iar Fs (0) = 0. Fs (s) = 8 32 3 56 a s a as Cum Fs este strict cresctoare i surjectiv, ea este inversabil i fie s :

a s [0, s] inversa sa. Observm c, deoarece Fs este continu i strict cresc a a este hde asemenea continu i strict cresctoare. Determinm acum 0 ( a s a a s i . u 0, 56 1 = Dac u = Fs (ts ), 0 ts s, cu fs (ts ) 6= 0, atunci 0 (u) = 0 a s Fs (ts ) 0 Dac u = Fs (ts ), 0 ts s, cu fs (ts ) = 0, atunci Fs (ts ) = 0 i nu pu a s formula de derivare a funciei inverse. Totui, utiliznd definiia derivate t s t c Fs este strict cresctoare, obinem c 0 (u) = +. a a t a s t a Fie acum 1 s1 < s2 < . Utiliznd semnificaia geometric a Riemann, remarcm c are loc urmtoarea proprietate: a a a Dac 0 t1 s1 si 0 t2 s2 , iar Fs1 (t1 ) = Fs2 (t2 ), atunci fs1 (t1 ) a

(este important de notat c razele semicercurilor construite pentru cel a ale lui s nu depind de s, iar dac Fs1 (t1 ) = Fs2 (t2 ), atunci M1 (t1 , a a s at M2 (t2 , fs2 (t2 )) trebuie s se afle la aceeai nlime, pe semicercuri corespu h i . Dac u = fs1 (ts1 ), 0 ts1 s1 , cu fs1 (ts1 ) 6= 0, fie 0 a Fie u 0, 56 astfel ca u = fs2 (ts2 ). Atunci Fs1 (ts1 ) = Fs2 (ts2 ), deci fs1 (ts1 ) = fs2 (t2 1 proprietii de mai sus, i deci 0 1 (u) = 0 2 (u) = at s . Dac u = a s s fs1 (ts1 ) 0 ts1 s1 , cu fs1 (ts1 ) = 0, fie 0 ts2 s2 astfel ca u = Fs2 (ts2 Fs1 (ts1 ) = Fs2 (ts2 ), deci fs1 (ts1 ) = fs2 (ts2 ) = 0, conform proprietii de at deci 0 1 (u) = 0 2 (u) = +. s s h i . Cum Fs1 (0) = F s s a n concluzie, s1 i s2 au aceeai derivat pe 0, 56 iar Fs1 (s1 ) = Fs2 (s2 ) = , deducem c s1 (0) = s2 (0) = 0, iar s1 a 56 s2 s a a = s2 , deci s1 i s2 nu difer printr-o constant. De aici, 56 h i F = {s ; s 1} este o mulime de funcii cu aceeai derivat pe 0, t t s a 56

oricror dou nefiind constant, ceea ce ncheie construcia exemplului. a a a t

* * * Prezentm n cele ce urmeaz, pe scurt, definiia derivatelor Dini ale u a a t reale [2]. a Fie f : (a, b) R i x0 (a, b). Numim derivata Dini superioar s f (x a inferioar ) la dreapta a lui f n x0 numrul D+ f (x0 ) = lim sup a
x&x0

f (x) f (x0 ) (respectiv D+ f (x0 ) = lim inf ). Analog se pot defini der x&x0 x x0 superioar , respectiv inferioar , la stnga a lui f n x0 , notate D f (x0 ) a a D f (x0 ). Privitor la relaia dintre derivatele Dini i derivatele clasice ale unei func t s a a punct x0 , se poate observa c dac D+ f (x0 ) = D+ f (x0 ), atunci f are d s 0 dreapta n x0 n sens clasic i f+ (x0 ) = D+ f (x0 ) = D+ f (x0 ), un rezultat tor avnd loc i pentru derivata la stnga, iar dac toate cele patru deriva s a o valoare comun n x0 , atunci f are derivat n x0 . Este ns de remarcat a a a funcii f i se pot asocia cele patru derivate Dini n x0 , spre deosebire de t laterale clasice. Indicm acum extinderile unor rezultate clasice folosind derivate Dini. a Teorema 1. Fie f : (a, b) R continu pe (a, b). Dac Df = a a cu excepia unei mulimi cel mult num rabile, unde D poate fi orice deri t t a atunci f este constant pe (a, b). a Teorema 2. Fie f : (a, b) R continu pe (a, b). Dac Df a a cu excepia unei mulimi cel mult num rabile, unde D poate fi orice deri t t a atunci f este cresc toare pe (a, b). a a Teorema 3. Fie f : (a, b) R continu pe (a, b) si x0 (a, b). Dac a derivatele Dini este continu n x0 , la fel sunt si celelalte trei. n acest a a cele patru derivate Dini n x0 sunt egale, iar f este derivabil n x0 . ncheiem cu o extindere a rezultatului privitor la funciile derivabile t derivat pe un interval . a Teoema 4. Fie f, g : (a, b) R continue pe (a, b). Dac Df si Dg a egale si finite pe (a, b) cu excepia unei mulimi cel mult num rabile, und t t a fi orice derivat Dini, atunci f si g difer printr-o constant pe (a, b). a a a

Bibliografie 1. M. Ganga - Manual de matematic : clasa a XI-a, Mathpress, Ploieti, 2 a s 2. R. Kannan, C. K. Krueger - Advanced Analysis on the Real Line, Sprin New York, 1996. 3. S. Ruziewicz - Sur les fonctions qui ont la mme drive et dont la di pas constante, Fundamenta Mathematicae 1(1920), 148151 (accesibil i s electronic la adresa http://matwbn.icm.edu.pl/wyszukiwarka.php).

Probleme selectate de la Olimpiadele de Matem ale Republicii Moldova

Not. Material trimis pentru publicare Redaciei de c tre Dr. Valeriu a t a Facultatea de Matematic si Informatic , Universitatea de Stat din Chiin a a s

Enunuri t

hni hni hni + + + , n N 1. Se consider irul (an )nN , an = as 1 2 n este partea ntreag a numrului x. S se arate c a a a a an = 2 + an1 ,

dac i numai dac n este un numr prim. (O. R. M. 1997 ) as a a 2. S se demonstreze c, pentru orice numere a naturale m, n 2, ce a s as s a dintre numerele n m i m n nu depete numrul 3 3. (O. R. M. 1996 3. Polinomul P (X) de grad n 5, cu coeficieni ntregi, are n rdc t a a a a a a a distincte 1 , 2 , . . . , n , unde 1 = 0. S se gseasc toate rdcinile polinomului P (P (X)). (O. R. M. 1997 ) 4. Fie triunghiul ABC cu nlimea CD. Se tie c AB = 1999, B at s a i AC = 2000. Cercurile nscrise n triunghiurile ACD i BCD sunt t s s segmentul CD n punctele M i N respectiv. S se ae M N . (O. R. M. s a 5. Fie ABCD un ptrat de latur 1. Pe laturile AB i CD se iau a a s interioare X i Y . Fie M punctul de intersecie a dreptelor XD i Y A i s t s s de intersecie a dreptelor XC i Y B. Pentru ce poziie a punctelor X t s t patrulaterului XN Y M este maxim? (O. R. M. 1997 ) a 6. Doi frai au vndut n pui cu cte n lei fiecare. Banii i-au mp t a fratele mai mare a luat 10 lei, apoi cel mai mic 10 lei, apoi din nou cel . a. m. d. Fratelui mai mic i-a revenit la sfrit o sum mai mic dect 1 s s a a luat acest rest i nc briceagul fratelui mai mare, ambii acceptnd c au s a a final acelai ctig. Ct cost briceagul? (O. R. M. 1996 ) s s a 7. Fie n un numr natural astfel nct numrul 2n2 are 28 de divizori a a distinci, iar numrul 3n2 are 24 de divizori distinci. Ci divizori di t a t t numrul 6n2 ? (O. R. M. 1999 ) a 8. Din cubulee de latur 1 se construiete un cub de latur 45. n cub t a s a 1998 de cubulee sunt populate de bacterii. n fiecare secund bacteriile t a n orice alt cubule, care are cel puin trei fee comune cu cubuleele deja t t t t Este posibil ca bacteriile s ocupe toate cubuleele? (O. R. M. 1998 ) a t 9. ntr-o coal primar rural nvaa 20 de copii. Fiecare doi copii au s a a a t un bunic comun. S se demonstreze c unul dintre bunici are n aceast a a a puin 14 nepoi. (O. R. M. 1996 ) t t 10. n timpul unei btlii comune fiecare dintre cei 2001 de cocoi a r aa s pan de la un alt coco i fiecare coco a rmas fr o pan. Se tie c prin a ss s a aa a s a

3 cocoi se gsete unul care nu a rupt nici o pan de la ceilali doi. S se s a s a t a cel mai mic numr k cu proprietatea: tind cel mult k cocoi putem ae a a s s cocoi n dou cotee astfel nct nici un posesor de pan strin s nu n s a t a a a a n acelai cote cu stpnul penei. (O. R. M. 2001 ) s t a

Soluiile problemelor t
1. Scriem egalitatea (1) sub forma hni hni hni n1 n1 n1 + + + =2+ + + + 1 2 n 1 2 n1 Dup reducerea termenilor obinem a t hni hni n n1 n1 n1 + + + = + + + 2 3 n1 2 3 n1 hni n 1 Pentru orice 2 k n 1 avem . Deci, k k hni hni n n1 n1 n1 + + + + + + 2 3 n1 2 3 n1

Presupunem adevrat egalitatea (1), a a deci i s (2). Dac n este un numr a a hni n1 =b> = b 1. Ca urmare, inega n = ab, 2 a < n, atunci a a este strict, ceea ce contrazice (2). a Reciproc, fie n un numr prim. Atunci pentru orice 2 k n1 avem a hni n1 =t= . Prin urmare, cu t, r N i 1 r < k. n acest caz s k k (2), deci i (1), este adevrat. s a a n m n. Pentru m > n avem m n 2. a atunci n Dac m < n, m m < n < n < n m. Deci, min { n, m} n n, oricare ar fi m, n 2. Vom demo inducie c pentru orice n 2 avem n n 3 3 sau n3 3n . Pentru n = t a inegalitatea este adevrat. Presupunem c ea este adevrat pentru k a a a a a

(k + 1)3 = k3 + 3k2 + (3k + 1) k3 + k3 + k3 = 3k3 3 3k = 3k ceea ce finalizeaz demonstraia. Deci, pentru orice m, n 2 avem min{ m a t 3 3.

3. Numrul k este o rdcin a polinomului P (P (X)), dac el este a a a a a a aa a uneia din ecuaiile P (x) = i , unde i = 1, 2, . . . , n. S artm c P (k) 6= t ar fi k Z i i 2. s Presupunem c exist k Z astfel nct P (k) = 2 . Atunci k 6= 0. a a rezult c polinomul P (X) are forma P (X) = aX(X 2 )(X 3 ) (X a a t a Z . Prin urmare, ak(k 2 )(k 3 ) (k n ) = 2 . Relaia k | 2 = kt, t Z . Dar ak(1 t)(k 3 ) (k n ) = t implic (1 t) | t. a Rezult c t = 2, pentru care avem ak(k 3 )(k 4 )(k 5 ) (k a a Observm c numerele k, k 3 , k 4 i k 5 sunt distincte. Dar nu a a s nu poate fi scris ca produs de cteva numere ntregi, dintre care cel pui t

fie distincte. Deci, presupunerea fcut este fals i P (k) 6= i , oricare ar a a as i 2. Deci, rdcinile ntregi ale polinomului P (P (X)) sunt soluiile ntregi a a a t P (x) = 0, adic numerele 1 , 2 , . . . , n . a

5. Observm c 4BN X 4Y N C. Presupunem c BX CY a a a XN N C (menionm c XN = N C, dac i numai dac BX = C t a a a s a siderm pe [XN ] punctul P astfel nct N P = N C. Atunci: SBN P a SY P N = SY N C , SBP X SY XP (egalitatea are loc doar n cazul cnd B Din trapezul XBCY avem SXNY = SBCN . Prin urmare, SBN X + SY NC = SBP X + SBNP + SY N C SY XP + SBCN + SY P N = SXNY + SBCN = 2 SXNY . 1 1 a a Rezult c SXN Y SXBCY . Prin analogie, se arat c SXY M SA a a 4 4 urmare, 1 1 SXNY M = SXNY + SXY M (SXBCY + SAXY D ) = SABCD 4 4 1 1 Dac BX = CY , atunci aria este maxim: SXNY M = SABCD = . a a 4 4 6. Din enun rezult c suma ncasat este de n2 lei. Fie n = 10a t a a a a, b N, 0 b < 10. Atunci n2 = 100a2 + 20ab + b2 = 20a(5a + b) + b2 luat n total un numr impar de cte 10 lei. Rezult c numrul zecilor a a a a n2 este impar, ceea ce implic b2 = 16 sau b2 = 36. n ambele cazuri restu a cu 6. Deci, ultima dat fratelui mai mic i-a revenit 6 lei, cu 4 lei mai p a fratelui mai mare. Deoarece mprirea a fost corect, rezult c preul b at a a a t este de 2 lei.

nlocuind valorile date, obinem c AD BD = 4. t a Pe de alt parte, avem DE = DM, DF = DN i CD + AD = 2D a s CD + BD = 2DN + BC. De aici rezult c a a AD BD = 2DM + AC 2DN BC = 2(DM DN ) + 2. Prin urmare, 2(DM DN ) = AD BD 2 = 2. Rezult c MN = 1. a a

4. Fie E i F punctele de tangena a dreptei AB cu cercurile nscrise s t n triunghiurile ADC i BCD. Aplicnd teorema lui Pitagora, avem s AC 2 BC 2 = (AC 2 CD2 ) (BC 2 CD2 ) = AD2 BD2 = (AD BD)(AD + BD) = (AD BD) AB.

7. Se tie c pentru orice numr natural m cu descompunerea s a a a t a m = p1 pk , numrul divizorilor (pozitivi) distinci ai numrului m 1 k cu d(m) = (1 + 1) (k + 1) (se demonstreaz prin inducie n raport c a t k de factori primi). Scriem descompunerea canonic a numrului n = 2 3 p1 1 pk k , unde a a sunt factori primi distinci i diferii de 2 i 3 cu exponeni ntregi , t s t s t (i = 1, . . . , k). Atunci, numrul divizorilor (pozitivi) distinci ai a t 2 2 2n2 = 22+1 32 p1 1 pk k este egal cu d(2n2 ) = (2 + 2)(2 + 1)(2 1 + 1) (2 k + 1).

Analog, avem d(6n2 ) = (2 + 2)(2 + 2)(2 1 + 1) (2 k + 1). d(3n2 ) = (2 + 1)(2 + 2)(2 1 + 1) (2 k + 1),

t Notm c = (2 1 + 1) (2 k + 1). Din condiiile problemei avem a (2 + 2) (2 + 1) c = 28 . (2 + 1) (2 + 2) c = 24

Numrul c este un divizor comun al numerelor 28 i 30. n plus, c este im a s c = 1. Sistemul (4) pentru c = 1 are soluia n numere naturale = t Rezult c d(6n2 ) = (2 + 2)(2 + 2)c = 32. a a

8. Dup ocuparea de ctre bacterii a unui cubule nou numrul feelor c a a t a t de pe frontier ale volumului ocupat de bacterii nu crete. Numrul maxim a s a de acest fel este egal cu 1998 6 = 11988, iar numrul feelor cubuleelor d a t t ale cubului construit este egal cu 45 45 6 = 12150. Deci, bacteriile nu toate cubuleele. t

9. Notm cu A i B cei doi bunici ai unui elev oarecare. Fie X mulim a s t care au A i B ca bunici. n condiiile problemei, restul elevilor vor av s t dintre bunici sau pe A sau pe B. Fie Y i Z mulimile acelora pentru s t respectiv B este bunic. Notm cu C al treilea bunic al unui elev din m a (primul bunic ind A). Ca urmare, toi elevii din Z vor avea ca bunici B t acelai mod, deducem c i toi elevii din Y au pe C ca bunic. Aadar, e s as t s trei bunici: A, B, C. Dac a, b, c noteaz numrul de nepoi pe care i a a a a t clas A, B i respectiv C, rezult c a + b + c = 40. Observm c a 1 a s a a a a c 13 implic a + b + c 39 < 40. Deci, mcar unul dintre a, b, c este a a mcar un bunic are 14 nepoi. a t

10. Vom spune c n (n > 1) cocoi formeaz un n-ciclu, dac ei pot a s a a astfel nct primul coco a rupt o pan de la al doilea, al doilea a rupt o pa s a treilea etc., iar ultimul coco a rupt o pan de la primul. Astfel, mulime s a t poate fi partiionat n cicluri. t a Toi cocoii din orice n-ciclu par (n este numr par) se aaz n cot t s a s a cocoii cu un numr de ordine par ntr-un cote, iar cei cu numr de ordi s a t a cellalt. a Pentru orice n-ciclu impar (n este un numr impar) este necesar i s a s tiem un coco, de exemplu, ultimul. n acest caz ceilali cocoi se aaz a s t s s cotee dup metoda precedent. Menionm c din ipotez rezult c nu t a a t a a a a a un 3-ciclu de cocoi. s Deoarece 2001 = 5 400 + 1, rezult c nu exist mai mult de 400 ciclu a a a Dar deoarece un coco nu poate forma un ciclu, rezult c exist cel m s a a a cicluri impare (de exemplu, 399 de 5-cicluri i un 6-ciclu). s Prin urmare, se pot tia oricnd cel mult k = 399 de cocoi (cte unul a s impare), pentru a aeza ceilali cocoi conform cerinelor din enun. s t s t t

Probleme pentru clasa a VIII-a Holger STEP


Enunuri i soluii t s t

1. Patru numere adunate dou cte dou dau sumele 4, 7, 9, 14, 16, a a sunt cele patru numere? Soluie. Fie x1 , x2 , x3 i x4 cele patru numere. Vom presupune, fr a t s aa generalitatea, c x1 x2 x3 x4 . Observm c x1 + x2 x1 + x3 a a a s x2 + x3 x2 + x4 x3 + x4 . Relativ la sumele x1 + x4 i x2 + x3 putem x1 +x4 x2 +x3 ct i x2 +x3 x1 +x4 . Ca urmare, suntem condui la ur s s dou sisteme: a x1 +x2 = 4, x1 +x3 = 7, x1 +x4 = 9, x2 +x3 = 14, x2 +x4 = 16, x3 +x4 x1 +x2 = 4, x1 +x3 = 7, x1 +x4 = 14, x2 +x3 = 9, x2 +x4 = 16, x3 +x4 Rezolvm mai nti sistemul (1). Adunnd primele dou ecuaii i d a a t s scznd a patra, obinem 2x1 = 3. Prin nlocuire n primele trei ecua a t 3 11 17 21 as a t x1 = , x2 = , x3 = , x4 = , care verific i ultimele dou ecua 2 2 2 2 n mod similar, sistemul (2) conduce la soluia x1 = 1, x2 = 3, x3 = 6, t

s Dar 3107 + 7 = 73 410956 i, deci, 7x + 3y este divizibil cu 73. Deoarece c lui x i y nu se divid cu 73, rezult c x i y pot fi doar simultan divizibil s a a s

2. Demonstrai c prin "rotirea c tre dreapta" a unui num r de 8 cifr t a a a cu 73, se obine tot un num r divizibil cu 73. (Se spune c un num r n t a a a "rotit c tre dreapta", dac ultima cifr este mutat n faa primei cifre a a a a t 1234 4123.) Soluie. Fie x = x7 x6 x5 x4 x3 x2 x1 x0 un astfel de numr i y = x0 x7 x6 x t a s rotitul su ctre dreapta. Avem a a 7x + 3y = 7 107 x7 + 106 x6 + + 10x1 + x0 + 3 107 x0 + 106 x7 + + 1 = 3 107 + 7 x0 + 7 107 + 3 106 x7 + 7 106 + 3 105 x6 + + 7 102 + 3 10 x2 + (7 10 + = 3 107 + 7 x0 + 73 106 x7 + 73 105 x6 + + 73 10x2 + 73x

3. Aflai cifrele necunoscute x, y, z din egalitatea 20 058 47311! = x00y t Soluie. Egalitatea din enun se scrie 28 34 52 71120 058 473 = x00yz0 t t Observm c puterile lui 5 sau ale lui 2, cu care este divizibil numrul, dau a a a doar despre ultimele lui 8 cifre (care, ns, sunt cunoscute). Apelm la c a a divizibilitate cu 7, 9 i 11, aplicate numrului x00yz00550464, care vor antr s a x, y, z; obinem t x + y + z + 24 = 9i, x y + z + 2 = 11j,

(pentru ultima egalitate s-a folosit faptul c un numr este divizibil cu 7 a a ponderat a cifrelor sale cu ponderile 1, 3, 2, 1, 3, 2, utilizate periodi a de la uniti spre puterile mai mari ale lui 10, este divizibil cu 7). at a a a Cum 1 x 9 i 0 y, z 9, din (1) rezult c avem 25 9i 51, 7 s 4 7k 32, adic a i {3, 4, 5} , j {0, 1} , k {0, 1, 2, 3, 4} .

x y + 2z + 5 = 7

Soluia sistemului (1) n x, y, z este t 1 1 x = (9i + 33j 14k 20) , y = (9i 11j 22) , z = 7k 11j 2 2 Expresia lui y din (3) arat c i i j trebuie s aib aceeai paritate. C a a s a a s exist trei cazuri: a s 1 i = 3 i j = 1, caz ce conduce la y = 3, ceea ce este imposibil; s a s 2 i = 5 i j = 1, care d x = 29 7k, y = 6 i z = 7k 14; pentru obine soluia x = 8, y = 6, z = 7; t t s a 3 i = 4 i j = 0, care d x = 8 7k, y = 7, z = 7k 3; pentru k = soluia x = 1, y = 7, z = 4. t Aadar, sunt posibile dou numere divizibile cu 9, 7 i 11: 800 670 055 s a s 100 740 055 046 400. Pentru a decide care este numrul corect este neces a test de divizibilitate, i anume, cu 27. Puterile lui 10 mprite la 27 pot d s at 1, 10 sau 8, n mod periodic. Aplicnd aceast observaie la primul num a t deci acest numr este divizibil cu 27. Relativ la al doilea numr avem a a

1 (0 + 6 + 5 + 0 + 0) + 10 (0 + 4 + 5 + 7 + 0) 8 (4 + 0 + 0 + 6 + 8

adic acest numr nu se divide cu 27 i trebuie exclus. a a s n final, singura soluie valabil este numrul 800 670 055 046 400. t a a 4. Un num r x format din cinci cifre diferite si nenule este divizibil cu a c suma tuturor numerelor de cinci cifre distincte ce se pot forma cu a a cifre (inclusiv x) este divizibil cu 2399976. a Soluie. Fie x = x1 x2 x3 x4 x5 . Cifra x1 este exact de 4! = 24 ori pe prim t Adunarea tuturor acestor prime poziii egale cu x1 d numrul 24 1000 t a a poziia a doua, a treia etc. apare cifra x1 tot de 24 ori. Cifra x1 va contrib t total S cu a Aceasta e valabil pentru orice cifr. Deci pentru suma S avem a S = 266664 (x1 + x2 + x3 + x4 + x5 ) .

1 (0 + 6 + 5 + 0 + 0) + 10 (0 + 4 + 5 + 4 + 0) 8 (4 + 0 + 0 + 7 + 1

24 10000 + 24 1000 + 24 100 + 24 10 + 24 1 = 24 11111 = 266

Deoarece x e divizibil cu 9, atunci i x1 + x2 + x3 + x4 + x5 este divizib s consecina, S este divizibil cu 266 664 9 = 2 399 976. t a 5. G sii toate perechile de numere ntregi x si y care sunt soluii a a t t diofantice 2x2 + 7xy + 3y 2 = 228. Soluie. Scriem ecuaia n forma (x + 3y) (2x + y) = 228. Dac nu t t a este descompus ntr-un produs de doi factori ntregi p q = 228, atunci p sistemul 2x + y = p, x + 3y = q cu soluia t 3p q 2q p x= , y= . 5 5 228 are divizorii 1, 2, 3, 4, 6, 12, 19, 38, 57, 76, 114, 228 i cei corespu s semnul minus. Dac nlocuim p i q cu aceti divizori i calculm, obinem a s s s a t

p q x y

1 228 45 91

2 114 108 5

3 76 67 5

4 57 9 22

6 38 4 14

12 19 17 5

19 12 9 1

38 6 108 5

57 4 167 5

76 3 45 14

114 2

68

22

n concluzie, avem perechile de soluii (45, 91), (9, 22), (4, 14), (9, 1 t i (68, 22), mpreun cu variantele negative (45, 91), (9, 22), (4, 14), s a (45, 14) i (68, 22). s 6. G sii toate perechile de numere ntregi x si y care sunt soluii a a t t diofantice 2x2 + 3y 2 = 77. Soluie. Dac perechea (x, y) este soluie, atunci i perechile (x, y t a t s i (x, y) sunt soluii. E suficient s considerm soluiile nenegative. s t a a t x2 0 avem 0 3y 2 77 sau 0 y 5. y nu poate fi par, deoarece 2x i 77 este impar. E destul s dm lui y valorile 1, 3 i 5. Ultimele dou s a a s a valori ntregi pentru x. Obinem (x, y) = (1, 5), (5, 3) i (x, y) = (1, 5 t s (1, 5), (5, 3), (5, 3), (5, 3). 7. Consider m num rul natural n, 1000 n < 5000. Form m num a a a a sau 13 cifre) obinut scriind n ordine cifrele lui 3n, 2n si respectiv n. t acest num r este divizibil cu 28 + 1. a Soluie. Numrul 2n are patru cifre deoarece n < 5000. Deci numrul t a a ca n enun, are 12 sau 13 cifre. Atunci avem t

Deci m este divizibil cu 28 + 1 = 257. a 8. Sase numere prime 7 < p1 < p2 < p3 < p4 < p5 < p6 formeaz un numere prime", dac p2 , p3 si p4 , p5 sunt numere prime gemene (adic a a t a = p5 p4 = 2), iar p2 p1 = p4 p3 = p6 p5 = 4. Demonstrai c sum divizibil cu 630. a Soluie. Fie M numrul care satisface t a

m = 300 000 000n + 20 000n + n = 300 020 001n = 3 41 257 9491

Deoarece pi sunt numere prime mai mari ca 7, nu pot fi divizibile prin 2, Ca urmare, M este impar i divizibil cu 3, 5 i 7. M se poate reprezenta, d s s M = 210k + 105. Atunci, avem p1 = 210k + 97, p2 = 210k + 101, p3 = 210k + 103, p4 = 210k + 107, p5 = 210k + 109, p6 = 210k + 113

p1 = M 8, p2 = M 4, p3 = M 2, p4 = M + 2, p5 = M + 4, p6 =

i, n concluzie, p1 + p2 + p3 + p4 + p5 + p6 = 1260k + 630 = 630 (2k + 1). s 9. Este posibil ca suma a sapte p trate perfecte succesive s fie un p tr a a a Soluie. Suma a apte ptrate perfecte consecutive se poate scrie astf t s a (n 3)2 + (n 2)2 + (n 1)2 + n2 + (n + 1)2 + (n + 2)2 + (n + 3)2 = 7

a Pentru ca rezultatul s fie ptrat perfect este necesar ca n2 + 4 s fie div a a Dar n2 d doar resturile 0, 1 sau 3 la mprirea cu 7. Ca urmare, 7 n a at divizibil cu 7, dar nu cu 49 i nu poate s fie ptrat perfect. s a a

Concursul "Recreaii Matematice" t


Ediia a IV-a, Muncel - Iai, 28 August 2004 t s Clasa a VII-a

1. S se rezolve n Z Z ecuaia x2 + y 2 xy + 2x 2y + 1 = x2 y 2 . a t Ctlin Bude a a 999 2. S se arate c numrul 4 a a a se scrie n baza 10 cu cel puin 601 cifr t Gabriel P b b 3. Fie 4ABC cu m(B) = 60 , m(C) = 50 i punctele M (BC), s \ \ a astfel nct m(BAM) = m(BCN ) = 5 . Dac I este centrul cercului 4ABC, s se arate c punctele M, N , I sunt coliniare. a a Gheorghe Iu 4. Fie ABCD un dreptunghi de centru O. Considerm N (AO), M a lui [AD], {P } = MN CD, {E} = OP BC. S se arate c N E BC. a a Andrei Nedelcu, Iai (RecMat s

Clasa a VIII-a
1. Considerm mulimile A = a t se demonstreze c A = B. a

x [x] ,x 1 , B = y | y R, y = x

Vasile Nec t 2. S se arate c ecuaia x3 + y5 = t7 are o infinitate de soluii n (N a a t Artur Bluc, aa a 0 0 0 0 3. Fie ABCDA B C D un cub de muchie a, iar P un punct pe segmen Determinai poziiile lui P pentru care cu distanele de la P la planele t t t (B 0 BD), respectiv (C 0 BD) se poate construi un triunghi. Gabriel Popa, Paul Georg 4. Fie b N, b 2. Spunem c un numr natural este decompozab a a poate scrie ca suma a dou numere cu aceeai sum a cifrelor n baza b. S a s a c exist o infinitate de numere care nu sunt decompozabile. a a Adrian Zahariuc, Bacu (RecMat a

Clasa a IX-a

1. Fie n N fixat. Determinai x, y R pentru care xn +yn = xn+1 + t Gheorghe Iu 2. Fie 0 < ni < 144, i = 1, 2, . . . , 12, dousprezece numere naturale dis a se demonstreze c mulimea S = {n1 , n2 , . . . , n12 } conine dou submulim a t t a t ale cror elemente au aceeai sum. a s a Ctlin Bude a a 3. Fie 4ABC de laturi a, b, c; notm cu la , lb lungimile bisectoarelor a b b ale unghiurilor A, respectiv B. Dac la = a i lb = b, s se calculez a s a unghiurilor 4ABC. Vasile Nec 4. Exist funcii f : R R pentru care a t

|f (x + y + z + t) + cos x + cos y + cos z + cos t| < 4,

Lucian Tuescu, Craiova (RecMat t


2

x, y, z, t

Clasa a X-a
aib soluii n Z Z. a t 1. S se determine a R astfel nct ecuaia 2x1 + 2x a t
1

y2 + a y2

Petru Rducanu, Iai (RecMa a s 2. Artai c exist o infinitate de numere naturale n pentru care n a t a a divide n!. Lucian Tuescu t 3. Fie S1 (O1 , r1 ), S2 (O2 , r2 ) dou sfere, iar Ai S1 , Bi S2 , i = 1 a astfel nct Ai Bi , i = 1, 4 s fie tangente comune la cele dou sfere. Fie M a a a segmentului [Ai Bi ], i = 1, 4. S se determine un punct P O1 O2 pentru P M1 + P M2 + P M3 + P M4 s fie minim. a a Gabriel Popa, Paul Georg 4. Exist triunghiuri pentru care IH = 2004 i GH = 2003? (notaii c a s t Lucian Ldunc, Andrei Ned a a

Clasa a XI-a
g (x) = h (x) , (f g) (x) ag (x) = (f h) (x) ah (x)

1. a) Fie f : R R funcie derivabil pe R i a R \ Im f 0 . S s t a s a pentru orice funcii g, h : R R, ecuaiile t t au aceleai mulimi de soluii. s t t b) S se rezolve ecuaia 2x + sin (sin x) = 3 sin x. a t

Silviu Boga 2. Se d cercul (C) i dreapta (d) tangent lui. Din punctul M , mobil p a s a (d), se duce tangenta MT la cercul (C). Se cere: a) locul geometric al punctului care mparte segmentul [M T ] n raport b) s se reprezinte grafic acest loc pentru k = 1. a Gabriel Mr s 3. Fie k, n, p, q N , n, p, q impare, iar A Mn (Z). S se demo a k A2 + pA + qIn 6= On . Romeo Ilie a a t 4. Fie k N ; s se arate c ecuaia xn+k xn xn1 x 1 singur soluie pozitiv (pe care o notm cu xn ). S se arate c irul (x a t a a a as convergent i s se afle limita sa. s a Dumitru Mihalache, Marian Tetiva, Brlad (RecMat

FUNDA TIA CULTURALA "POIANA" (dir.

Da

a oferit dou premii n valoare de cte 1 000 000 lei, pentru cele mai bu a la gimnaziu i liceu, urmtorilor elevi: s a 1. Hurmuz Daniel, cl. a VIII-a, Scoala nr. 7, Botoani, s 2. Svescu Cristian, cl. a IX-a, Colegiul Naional "Unirea", Focani a t s

Concursul interjudeean "Octav Onicescu t


Ediia a VIII-a, Botoani, 30 octombrie 2004 t s

1. Pe data de 30 octombrie o veveria lacom are 20042004 alune. n t a are un numr par de alune ea mnnc jumtate din ele, iar n celela a a a a mnnc nimic i mai culege 3 alune. Artai c ncepnd cu o anumit a a s a t a a va mnca doar cte 3 alune odat la 2 zile. a

2. Broscuele stau n cerc n jurul lacului de la Ipoteti. n prima secun t s una din ele. n fiecare din secundele urmtoare orcie doar acelea ce a a a a vecin care a orcit n secunda anterioar. Dovedii c 2005 broscue a a a a t a t toate deodat de la un moment dat ncolo, dar 2004 broscue nu vor orc a t a a toate deodat. a

3. Pe o tabl de ah n n sunt aezate n2 numere egale fiecare cu a s s (nu toate egale). Prin mutare se nelege c alegem o linie i o coloan o t a s a schimbm semnele tuturor numerelor de pe linie i apoi tuturor celor de p a s Se tie c dup cteva astfel de mutri toate numerele pot fi fcute 1. s a a a a dac n este par, plecnd de la configuraia iniial putem face toate nume a t t a dac n este impar nu putem face niciodat toate numerele 1. a a

4. De aceeai parte a unei drepte se consider 2003 puncte distincte A s a a a t a A2003 astfel nct orice cerc cu centrul pe dreapt s conin cel mult d puncte. Se numete "cerc bun" un cerc cu centrul pe dreapt, ce trece p s a din puncte i las 1001 puncte n interiorul su i 1001 n exterior. Arta s a a s a a) orice punct de pe dreapt (cu excepia eventual a unui numr finit a t a este centrul unui "cerc bun"; b) dac absolut toate "cercurile bune" trec prin A1002 (deci prin acel a atunci cele 2003 puncte sunt situate pe o perpendicular pe dreapta dat a a

5. a) Se dau n 2004 sertare aezate n linie. n primele 2003 din st s cte o bil. La un pas se alege o bil oarecare ce are sertar gol n dreapta a a n acesta, astfel c bilele "migreaz" una cte una spre dreapta. Artai c a a a t 2003 (n 2003) pai nici o bil nu mai poate fi mutat. s a a b) Se dau n 2004 numere reale distincte astfel nct suma oricror 20 a s fie tot unul din cele n numere. Artai c n = 2004, c jumtate din nu a a t a a a pozitive i celelalte sunt opusele lor. s

Rezultatele obinute au fost urmtoarele: t a Premiul I Pachiariu Marius (Colegiul Naional Iai). t t s Premiul II Chiril Cezar (Colegiul Naional Tudor Vianu, Bucu a t Premiul III Turcanu Alexandru, Vatavu Serban (Colegiul Naio t Eminescu, Botoani). s Meniuni Rou Eugenia (Iai), Istrate Carmen (Focani), Berea t s s s (P. Neam), Milatinovici Bianca (Iai), Hurmuz Daniel (Botoani), t s s Ciprian (Suceava), Galea Lucian (Botoani), Cepoi Alexandru (Sucea s clu Sorin (P. Neam), Georgescu Flavian (P. Neam), Cobuc Mir a t t s Mihalcea Marcel (Vaslui), Plmad Andrei (Botoani). a a s

Soluiile problemelor propuse n nr. 1 / 20 t


Clasele primare

P.64. ntr-o pies de teatru sunt 12 personaje, copii si aduli. Ci c a t t n pies , dac la fiecare doi aduli corespunde un copil? a a t ( Clasa I ) Alexandra Radu, e Soluie. Formm grupe de forma (copil, adult, adult) pn epuizm p t a a a Putem forma patru grupe de acest fel. Rezult c n pies joac patru co a a a a

P.65. Se dau jetoanele AT II CRE TII ATII RECR EA R Care este num rul cel mai mare de jetoane cu care se poate forma cuvntul a ATII"? ( Clasa I ) Oxana Pascal, elev, Rep. a Soluie. Cuvntul "RECREATII" poate fi format astfel: RE t REC RE AT II i RECR s utilizate este patru. EA TII . Numrul cel mai mare a

CR

P.66. ntr-o livad sunt tot atia peri ct si meri. Sunt 6 rnduri cu a t rnduri cu meri. Num rul merilor de pe un rnd ntrece cu 5 num rul pe a a un rnd. Ci pomi sunt n acea livad ? t a ( Clasa a II-a) nv. Maria R Soluie. Utilizm metoda figurativ. t a a Numrul perilor din cele 6 rnduri: a 5 5 5 5 Numrul merilor din cele 4 rnduri: a Din figurarea mrimilor se deduce c un rnd de peri are 5+5 = 10 pomi a a perilor este 6 10 = 60. Numrul tuturor pomilor din livad este 60 + 60 = a a

P.67. Dintr-o mulime de 5 copii, orice grupare de trei conine cel pu t t Ci b iei pot fi n mulime? t a t t ( Clasa a II-a) Andreea Surugiu, e Soluie. n mulime nu putem avea mai mult de 2 biei, altfel am gsi t t a t a de trei n care nu avem cel puin o fat. n concluzie, putem avea 2 bie t a a sau nici unul.

P.69. ntr-o mp rire cu rest, n care mp ritorul este mai mare a t a t m rind mp ritorul cu o unitate si efectund din nou mp rirea obine a a t a t t

P.68. Dac Ina ar mp ri num rul nucilor culese de ea la num rul nuc a a t a a de sora sa, ar obine 7 rest 6. Stiind c Ina a cules cu 78 nuci mai mult t a sa, aflai cte nuci a cules fiecare. t ( Clasa a III-a) nv. Doinia Sp t 78 Soluie. Utilizm metoda figurativ. t a a Numrul nucilor culese de Ina: a Numrul nucilor culese de sora Inei: a Numrul nucilor culese de sora Inei este (78 6) : 6 = 72 : 6 = 12. a nucilor culese de Ina este 7 12 + 6 = 84 + 6 = 90.

si restul 0. Aflai ctul si restul mp ririi iniiale. t a t t ( Clasa a III-a) nv. Mariana Toma, Muncelu de S Soluie. mprirea iniial este D = I C + R, R < I. A doua mp t at t a exact i avem D = (I + 1) 9. Rezult D = I 9 + 9. Cum I > 9 obin as a t i R = 9. s

P.70. ntr-o tab r internaional de matematic sunt elevi din patru a a t a a garia, Grecia, Republica Moldova si Romnia. Dac 21 elevi nu sunt din a 23 nu sunt din Grecia, 22 elevi nu sunt din Republica Moldova si 21 ele din Romnia, ci elevi sunt din fiecare tar ? t a ( Clasa a III-a) Georgiana Ciobanu, e Soluie. Dac 21 elevi nu sunt din Bulgaria, nseamn c sunt din G t a a a publica Moldova i Romnia. s Analog, 23 elevi sunt din Bulgaria, Republica Moldova i Romnia; s 22 elevi sunt din Bulgaria, Grecia i Romnia; s 21 elevi sunt din Bulgaria, Grecia i Republica Moldova. s Triplul elevilor din cele patru ari este 21 + 23 + 22 + 21 = 87. Numr t a din cele patru ari este 87 : 3 = 29. Rezult 29 21 = 8 elevi din Bulgar t a elevi din Grecia, 29 22 = 7 elevi din Republica Moldova i 29 21 = s Romnia.

P.71. Fiecare p trat din figura al turat se coloreaz cu o alt c a a a a a cte moduri putem face acest lucru avnd la dispoziie patru culori? t ( Clasa a IV-a) nv. Ctlina Raa, Coarnele Cap a a t Soluie. Dac alegem culorile C1 , C2 , C3 din cele patru, putem s co t a a tratele n ase moduri diferite: (C1 , C2 , C3 ), (C1 , C3 , C2 ), (C2 , C1 , C3 ), (C s (C3 , C1 , C2 ), (C3 , C2 , C1 ). Cele trei culori pot fi alese n patru modu (C1 , C2 , C3 ), (C1 , C2 , C4 ), (C1 , C3 , C4 ) i (C2 , C3 , C4 ). Pentru fiecare ale s ase moduri diferite de colorare. n total avem 6 4 = 24 moduri diferite d s

P.72. Arunc m dou zaruri si adun m punctele de pe cele dou fee de a a a a t a) Cte sume diferite putem obine? b) Cte sume se pot forma n tr t diferite? ( Clasa a IV-a) nv. Gheorghe Toma, Muncelu de S Soluie. a) Suma minim care se poate forma este 1 + 1 = 2, iar ce t a este 6 + 6 = 12. Toate numerele de la 2 la 12 sunt sume posibile. b) sume care se pot forma n trei moduri diferite sunt: 6 = 1 + 5 = 2 + 4 7 = 1 + 6 = 2 + 5 = 3 + 4 i 8 = 2 + 6 = 3 + 5 = 4 + 4. n trei moduri dife s forma trei sume.

P.73. n figura al turat este pus n evidena un a a t drum format din sase segmente care pleac din A si a ajunge n B. Cte drumuri de felul acesta se pot con- A strui? ( Clasa a IV-a) nv. Constantin Raa, Coarnele Cap t Soluie. Orice drum de acest fel conine numai t t 6 7 8 dou segmente verticale. Numrul drumurilor coina a 1 2 3 cide cu numrul perechilor distincte de segmente ver- A a

ticale prin care trec drumurile cu ase segmente. Aceste perechi sunt: (1, s (1, 8), (1, 7), (1, 6), (2, 9), (2, 8), (2, 7), (2, 6), (3, 8), (3, 7), (3, 6), (4, 7), (4 n total se pot construi 15 drumuri formate din ase segmente. s

Clasa a V-a

V.46. Aflai n N pentru care 11n + 9n si 11n 9n sunt simult t perfecte. Andrei - Sorin Cozma, = Soluie. Dac n este par, U (11n + 9n ) = 1 + 1 2, deci 11n + 9n n t a ptrat perfect. Dac n este impar, U (11n 9n ) = U . . . 1 . . . 9 = 2, de a a nu poate fi ptrat perfect. Rezult c nu exist n N cu proprietile do a a a a at

V.50. Cte numere de 7 cifre se pot scrie folosind cifrele 1, 2 si 3, a 1 s apar de 2 ori, 2 s apar de 3 ori si 3 s apar de 2 ori? Dar da a a a a a a cifrelor 1, 2 si 3 consider m cifrele 0, 1 si respectiv 2? a Petru Asa Soluie. Dac prima cifr 1 se gsete pe primul loc, a doua cifr 1 po t a a a s a locurile 2, 3, . . . , 7, deci 6 poziii. Dac prima cifr 1 se gsete pe locul a t a a a s doua cifr 1 poate ocupa locurile 3, 4, . . . , 7, deci 5 poziii etc. n total, c a t 1 pot fi aezate n 6 + 5 + + 1 = 21 moduri. Pentru fiecare poziionare s t 1, rmn 5 locuri libere care pot fi ocupate n 4 + 3 + 2 + 1 = 10 moduri a

V.49. Determinai num rul tripletelor (a, b, c) N3 dac 3a + 2b + t a a a + 2b + 3c = 602. Dac n plus a < b < c, determinai a, b si c. a t Gheorghe Iu Soluie. Avem c (a + 2b + 3c) (3a + 2b + c) = 602 598, deci c t a aici, c = a + 2 i apoi b = 298 2a, unde a {0, 1, 2, . . . , 149}. Prin urm s 150 de triplete, avnd forma (a, 298 2a, a + 2). Dac n plus a < b < a a < 298 2a < a + 2, de unde 296 < 3a < 298, deci a = 99 i apoi b = 100 s

V.47. S se arate c num rul 51a51a nu poate fi scris ca produsul a pat a a a prime. Ctlin Bude a a Soluie. Avem c 51a51a = 51a 1001 = 7 11 13 51a. ns 51a e t a a compus, oricare ar fi cifra a n baza 10, de unde concluzia. 9 10 11 , x2 = , x3 = , . . . . Scr V.48. Se consider fraciile x1 = a t 14 21 28 t a t x1000 si apoi ordonai cresc tor primele 1000 de fracii. Dumitru Gherman Soluie. Numrtorii fraciilor sunt 9, 9 + 1, 9 + 2, . . . Numrtorul lu t aa t aa fi 9+999 = 1008. Numitorii fraciilor sunt 14, 14+7, 14+ 27, . . . Numitoru t 1008 1 1 . Observm c x1 = + ; x2 a a va fi 14 + 999 7 = 7007; deci x1000 = 7007 7 2 1 1 1 1 . Atunci x1000 < x999 < < x3 < x2 x3 = + ; . . . ; x1000 = + 7 4 7 1001 Observm c dac indicelui i adunm 8 obinem numrtorul. Dac a a a a t aa a 1000 + 8 adunm 1 i l nmulim cu 7 obinem numitorul, deci x1000 = a s t t (1000 + 1)

rmnnd astfel exact trei locuri libere pentru cifrele 2. Folosind regula p a obinem 21 10 1 = 210 numere de 7 cifre. t n al doilea caz, cifrele 0 pot fi aezate n 5 + 4 + 3 + 2 + 1 = 15 modur s n 4 + 3 + 2 + 1 = 10 moduri, rmnnd libere exact trei poziii pentru 1 a t 15 10 1 = 150 numere.

Clasa a VI-a

VI.48. a) ntr-o proporie cu termeni nenuli, un extrem este suma t trei termeni dac si numai dac cel lalt extrem are inversul egal cu suma a a a celorlali trei termeni. t b) Dac din patru numere raionale nenule distincte unul este suma celo a t iar altul are inversul egal cu suma inverselor celorlaltor trei, atunci num termeni ai unei proporii. t Claudiu - Stefan P Soluie. a) n condiiile a, b, c, d Q , ad = bc, avem c a = b + c + d t t a b+c 1 1 1 1 1 1 1 1 ad = = + = + + . ad=b+c ad bc d a c b d a b c 1 1 1 1 b) Dac a, b, c, d Q i a = b + c + d, = + + , atunci a d = b a s d a b c 1 1 1 1 ad b+c = + , adic a d = b + c 6= 0 i a s = , deci ad = bc, i. d a b c ad bc VI.49. S se arate c orice num r natural relativ prim cu 10 admite u a a a care se scrie folosind numai cifra 3. Lucian - Georges Ldu a Soluie. Fie n N cu (n, 10) = 1. Considerm numerele 3, 33, 333, . . t a

1 1 = , adic n = 120, k = 125. a k 125 VI.47. Aflai restul mp ririi num rului N = 28442844 + 41074107 + t a t a prin 79. Tamara C Soluie. Se tie c (a + b)n = Ma + bn ; atunci 28442844 = M79, 4 t s a = (4108 1)4107 = M79 1, iar 63986398 = (6399 1)6398 = M79 + urmare, N = M79, deci restul cerut este 0. deci

VI.46. Suma dintre opusul unui num r natural si inversul altui num a este 119, 992. S se determine numerele. a Ciprian Ba 1 a a Soluie. Fie n N i k N cele dou numere, adic n + = t s k 1 1 1 a Atunci k 6= 1 i 0, 008 = n 120, de unde s k 125 N. ns k 1 1 1 + 1 1 + 1 < 1, k 125 k 125 2 125

exist printre acestea mcar dou care dau acelai rest la mprirea prin n a a a s at principiului cutiei. Evident c n divide diferena acestor numere, iar aceast a t este de forma 33 . . . 3 10k . Deoarece n, 10k = 1, urmeaz c n | 33 . . . 3. a a

VI.50. Fie 4ABC cu [AC] [BC], D mijlocul lui [AB], P un punct AB, iar M si L picioarele perpendicularelor din P pe AC, respectiv B arate c [DM] [DL]. a Neculai Roman, Mirce Soluie. Deosebim trei cazuri, dup cum P [AB], t a P [BA \ [AB] sau P [AB \ [AB]. Vom trata numai prima situaie, celelalte rezolvndu-se asemnt a a D tor. Ne situm cu P [DA], ca n figur i a a s P fie D0 , D00 mijloacele segmentelor [P A], respectiv D [P B]; demonstrm c 4DD0 M 4LD00 D. Avem a a 1 1 D c DD0 = a (AB P A) = P B = D00 L, iar 2 2 1 1 (AB P B) = P A = D0 M . n plus, DD00 = 2 2 \ \ \ \ m(DD0 M) = 2 m(BAC) = 2 m(ABC) = m(LD00 D) B L i atunci congruena de triunghiuri anunat urmeaz conform LUL. R s t t a a [DM] [DL].

Clasa a VII-a

VII.46. S se rezolve n R inecuaiile: a t a) x100 + x77 + x50 + x21 + x10 + x5 + 1 > 0; b) x100 x77 + x50 x21 + x10 x5 + 2 < 0. Vasile Solcanu, Bogdneti ( a s Soluie. a) Fie E (x) = x100 + x77 + x50 + x21 + x10 + x5 + 1. Da t evident c E (x) > 0. Pentru x (1, 0), avem x2k > 0 i x2k+1 + 1 a s a E (x) = x100 + x50 x27 + 1 + x10 x11 + 1 + x5 + 1 > 0. Dac x = 2k+1 E (1) = 1 > 0. n sfrit, dac x a (, 1), atunci x + 1 < 0, x2k+ s 21 29 23 5 77 5 E (x) = x x + 1 + x x + 1 + x x + 1 + 1 > 0. n concluzie, x R. b) Dac F (x) = x100 x77 +x50 x21 +x10 x5 +2, atunci F (x) = E ( a x R, deci inecuaia dat nu are soluie n R. t a t VII.47. S se rezolve n Z2 ecuaia u2 v + uv 2 = 2u2 + 2v 2 40. a t Mihai Crciun a Soluie. Ecuaia se scrie echivalent uv (u + v) = 2 (u + v)2 4u t t i h uv (u + v + 4) = 2 (u + v)2 16 8 (u + v + 4) (uv 2u 2v + 8) uv 2u 2v + 8 = (u 2) (v 2) + 4. Considernd toate cazurile posibile nal soluiile (u, v) {(2, 8) , (8, 2)}. t VII.48. Dac ai = i + i, i = 1, 2004, precizai dac num rul a t a a N = a1 a2 a3 + a4 + a5 a6 a7 + a8 + + a2001 a2002 a2003 este negativ, pozitiv sau nul. Viorel Cornea i Dan Stefan Marinescu, H s Soluie. Avem c N = N1 + N2 , unde t a N1 = (1 2 3 + 4) + (5 6 7 + 8) + + (2001 2002 2003 + 200 N2 = 1 2 3+ 4 + 5 6 7+ 8 + + 2001 2002 20

Evident c N1 = 0 i cum vom arta c fiecare parantez din scrierea lui N2 a s a a a tiv, va rezulta c N < 0. Pentru a demonstra c p + p + 3 < p + 1 a a a p 1, este suficient s ridicm la ptrat n ambii membri, obinnd dup a a a t p2 + 3p < p2 + 3p + 2, fapt adevrat. a

VII.49. Fie 4ABC echilateral si D (BC). Not m cu M1 , M2 a a segmentelor [BD], respectiv [CD]. Paralela prin M1 la AC intersecteaz iar paralela prin M2 la AB intersecteaz AC n E. S se arate c dre a a a M1 E si M2 F sunt concurente. Nicolae Gross i Lucian Tuescu s t Soluie. Deoarece 4BF M1 i 4CEM2 sunt echilat s A terale, avem BM1 = BF = F M1 , CE = CM2 = EM2 , AF = CM1 i AE = BM2 . Fie {S} = AD EM1 , s {T } = AD F M2 . Aplicnd teorema lui Menelaus n 4ABC cu transversala M1 S E i n 4ABD cu s t transversala F T M2 , obinem M2 D BF AT M1 D CE AS =1= , M1 C EA SD M2 B F A T D AT AS = , adic S = T . a de unde SD TD
F

T= B M1 D

VII. 50. Fie ABCD un trapez cu bazele [AB] si [CD]. O parale intersecteaz AD, AC, BD si BC n punctele E, F , G si respectiv H. S a c EH = 3F G dac si numai dac DF , CG si AB sunt drepte concuren a a a Adrian Zano Soluie. Aplicm teorema fundamental a t a a D asemnrii n 4ADC i n 4BDC, obinem c a a s t a EF AE GH BH = i s = de unde, avnd n DC AD DC BC BH AE = , rezult c EF = GH = u. a a vedere c a AD BC E Notm nc v = F G i s presupunem c a a s a a F G EH = 3F G; atunci 2u + v = 3v, adic u = v. Asta fel, n 4DEG avem DF median i AB k EG, deci as DF intersecteaz AB n mijlocul M al segmentului A a M=P [AB]. Analog rezult c CG intersecteaz AB n a a a M , de unde urmeaz c DF , CG i AB sunt concurente. Reciproc, dac a a s a u drepte sunt concurente n P , notm AP = a, P B = b i obinem c = a s t a v b de unde u = v, adic EH = 2u + v = 3v = 3F G. a

Clasa a VIII-a

m + VIII.46. S se demonstreze c nu exist m, n N pentru care a a a n Alexandru Negrescu, elev, m Q. Soluie. Relaia dat se scrie t2 2003t + 1 = 0, unde t = t t a n condiii, discriminantul ecuaiei n t trebuie s fie ptratul unui numr rai t t a a a t a ptrat perfect. ns = 20032 4 = 2001 2005 nu este ptrat perfect. a a

n n m m + N implic + = 2. a n m n m VIII.47. Pentru x (0, ), s se demonstreze inegalitatea a 5 3 2 3 2 4 3 x +x +x +1 x +x +2 + x +x +x+1 x3 +x+2 + x3 +x2 +x+1 x x6 + x5 + x4 + 2x3 + x2 + x + 1 Mircea Cobuc, s Soluie. Cu notaiile a = x + 1, b = x2 + 1, c = x3 + 1, inegalitat t t succesiv a b a c b c ac (a + c) + bc (b + c) + ab (a + b) 6 + + + + + abc b a c a c b a b s Ultima inegalitate rezult din cunoscutele + 2 i analoagele. a b a VIII.48. G sii numerele prime p si q pentru care p2 + q = 37q 2 + p. a t Liviu Smarandache Soluie. Evident p 6= q i scriem ipoteza sub forma p (p 1) = q (37q t s (p, q) = 1. Urmeaz c 37q 1 = tp, t N , deci p (p 1)= qtp, adic p a a a nlocuind n ipotez gsim c qt2 + t = 37q 1. De aici, q 37 t2 = t + a a a a a 37 t2 0, prin urmare t {1, 2, 3, 4, 5, 6}. Dup ncercri, singurul ca rmne t = 6, cnd q = 7, p = 43. a Not. Se poate arta c relaia a a a t

VIII.49. Fie 4ABC dreptunghic n A cu AB = AC = a. Consider m M a \ M A = a 2 si N AM astfel nct m(CN, BM ) = 60 . S se afle lun a mentului [AN ]. Romana Ghia i Ioan Gh t t s Soluie. Fie N de aceeai parte a planului (ABC) ca t s N i M; notm AN = x. Construim d k M B, cu N d; s a evident c N P (AMB) i fie {P } AB d. a s = Din M x 2 x 6 4ABM 4AP N , obinem AP = t , NP = . 2 2 Cu teorema lui Pitagora n 4AP C i 4N AC, gsim s a r x2 \ P C = a2 + , N C = a2 + x2 . Deoarece m(P N C) = A 2 \ 4N = m(CN, BM ) = 60 , teorema cosinusului n P C duce a 15 la o ecuaie n x, cu soluia admisibil x = t t a . Analog B 5 se trateaz cazul cnd N se afl de cealalt parte a planului a a a (ABC), obinnd acelai rezultat. t s b VIII.50. Fie patrulaterul convex ABCD cu AB = BC, m(A) = m( b m(B) 90 si fie O mijlocul lui [BD]. Pe perpendiculara n O pe planul ia un punct V astfel nct OV = OB. S se arate c d (D, (V AB)) = 2 d (D a a \ = 60 . dac si numai dac m(ABC) a a Monica Ned Soluie. Observm c ABCD este patrulater nscris n cercul de diam t a a i fie r = OA = OB = OC = OD = OV . Notm {P } = AC BD i s a s evident c AP = P C i AP BD. Calculnd n dou moduri volumul te a s a

SABD 2r 5 . VABD, obinem c h1 = d (D, (VAB)) = V O t a = SVAB 5 2 2 s Deoarece V P AC, V P = r + x i AC = 2 x2 , calculnd n dou moduri volumul lui 2 r a V DAC, gsim a V O SDAC 2r (r x) = . SVAC r2 + x2 Atunci h1 = 2h 5 (r x) = r2 + x2 3r r 2x2 5rx+2r2 = 0 i x [0, r] x = BP = s 2 2 i s r 3 3 \ AP = tg ABD = 2 3 \ \ m(ABD) = 30 m(ABC) = 60 . h2 = d (D, (VAC)) =

P A B

x 1 + 3 x 2n x 2 = 2, unde n Dan Popescu Soluie. Din condiiile de existena a radicalilor obinem x [2, 3 t t t t t t = 2n x 2 [0, 1]. Ecuaia se scrie atunci t2n + 1 + 1 t2n = 2 a + b 2 a + b, deci 2 + t 2 t2n + 1 + 1 t2n = 2, adic a consecina, singura soluie a ecuaiei este x = 2. t t t IX.46. S se rezolve ecuaia a t

Clasa a IX-a

IX.47. S se determine sirul (an )n1 de numere strict pozitive pentru a

a2 a2 + a2 + (1)n1 a2 = (1)n1 (a1 + a2 + + an ) , n 1 2 3 n

Marian Ursrescu a a Soluie. Pentru n = 1, obinem a2 = a1 , adic a1 = 1. Pentru n = 2 t t 1 a2 a2 = a1 a2 , deci a2 a2 2 = 0 i, cum a2 > 0, gsim a2 = 2. s a 1 2 2 a t a an = n, n 1, fapt care se demonstreaz prin inducie matematic. Pr c a1 = 1, a2 = 2, . . . , ak = k, avem c a a
k P

12 22 + 32 + (1)k1 k2 + (1)k a2 = (1)k (1 + 2 + + k) + ( k+1 de unde, dup calcule, folosind faptul c a a


k P

(1)i1 i2 = (1)k1

k (k

i=1

k (k + 1) , obinem c a2 ak+1 k (k + 1) = 0. Unica soluie t a k+1 i= t 2 i=1 t acestei ecuaii de grad II este ak+1 = k + 1, ceea ce ncheie demonstraia. t a a IX.48. Fie a, b, c (0, ) cu a + b + c + abc = 4. S se arate c

c2 a2 b2 3 + . + 2 a + bc b + ca c + ab Cezar Lupu, elev, C

n ipoteza a), obinem c (a b) [2ma + 2mb + 3 (a + b)] = 0 i, cum par t a s trat ia valori strict pozitive, rmne c a = b. n ipoteza b), gsim (a a a a a a +2mb 3 (a + b)] = 0. ns 2ma < b + c, 2mb < a + c, deci 2 (ma + mb ) < n plus, c < a + b, prin urmare 2 (ma + mb ) < 3 (a + b), adic paranteza a valori strict negative i din nou a = b. s

IX.49. S se arate c 4ABC este isoscel n fiecare din ipotezele: a a a) 2ma + b = 2mb + a; b) 2ma + a = 2mb + b. Marius Pachiariu, t Soluie. Folosind torema medianei, avem c t a 4 m2 m2 = 2 b2 + c2 a2 2 a2 + c2 b2 = 3 b2 a2 a b

p 4 Soluie. Conform inegalitii mediilor, 4 = a + b + c + abc 4 t at de unde abc 1, prin urmare a + b + c 3. Folosind acum inegalitatea Schwartz i cunoscuta (a + b + c)2 3 (ab + bc + ac), avem: s h i c2 b2 a2 + + (a + bc) + (b + ca) + (c + ab) (a a+ bc b+ ca c+ ab X a2 3 (a + b + c)2 (a + b + c)2 P . P 2 (a + b + c) 2 a + bc bc a+

IX.50. Fie I centrul cercului nscris n triunghiul ascuitunghic ABC t B, C sunt m surile n radiani ale unghiurilor triunghiului, iar A IA + B a IC = 0 , s se arate c 4ABC este echilateral. a a Constantin Micu, Meline a Soluie. Deoarece a IA+b IB+c IC = 0 , relaia din ipotez arat c t t a a a A a b c sin A sin B Pe de alt parte, a = = , prin urmare = sin A sin B sin C A B sin x Considerm funcia f : 0, a t R, f (x) = ; vom demonstra c f a x 2 descresctoare. Fie x, y 0, a , x < y; avem: 2 sin y sin x x (sin y sin x) (y x) sin x f (y) f (x) = = = y x xy yx x+y 2x sin cos (y x) sin x 2 2 < = xy yx 2x cos x (y x) sin x (y x) (x tg x) cos 2 = < xy xy x+ (Am folosit faptul c 0 < sin x < x < tg x, x 0, a , apoi c a 2 2 x+y < cos x, deoarece funcia cosinus este descresctoare pe 0, t a .) cos 2 2 c f este injectiv i atunci faptul c f (A) = f (B) = f (C) conduce la A a as a deci 4ABC este echilateral.

Clasa a X-a
s aib soluii n Z Z. a a t X.46. S se determine a R astfel nct ecuaia 2x1 + 2x a t
2

y2

Petru Rduc a Soluie. Observm c pentru orice a R ecuaia dat are soluii n t a a t a t exemplu, perechea (0, 0) pentru a 6= 0 i perechile (0, y), y Z , pentru a s X.47. Fie z1 , z2 , z3 C distincte, cu z2 +z3 = 2 si astfel nct |z1 1| = a a z a = |z3 1|. S se arate c (z1 z2 ) (1 z3 ) este num r complex pur ima Lidia Nicola Soluia I. S observm nti c (z1 z2 ) (1 z3 ) 6= 0, dat fiind faptu t a a a z z3 sunt distincte. Atunci faptul c (z1 z2 ) (1 z3 ) este pur imaginar es a z echivalent cu: z z (z1 z2 ) (1 z3 ) + (1 z2 ) (z1 z3 ) = 0 z z z z (z1 z2 ) [(1 z2 ) + (2 z3 )] + [(1 z3 ) + (3 z2 )] (z1 z3 ) = |z1 z2 |2 + |z1 z3 |2 = |z2 z3 |2 .

z z (z1 z2 ) (1 z2 ) + (1 z3 ) (z1 z3 ) = z2 z2 z2 z3 z2 z3 + z3 z3

s Fie A1 , A2 , A3 , C punctele de afixe z1 , z2 , z3 i respectiv 1. Deoarece z implic (z2 1) + (z3 1) = 0 CA2 + CA3 = 0 , rezult c [A2 A3 ] este a a a i, deci 4A1 A2 A3 este dreptunghic n A1 . Atunci A1 A2 2 + A1 A3 2 = A2 s tocmai relaia de demonstrat scris sub forma (1). t a Soluia a II-a (prof. Dumitru Gleat, Iai; Diana Timofte, elev, t a a s a condiia z2 +z3 = 2 rezult c z3 = 2z2 i, deci, |z3 1| = = |2 z2 1| t a a s aadar, egalitatea a 2-a din condiie este superflu. Notnd zk = ak + ibk s t a din z2 + z3 = 2 deducem c a2 + a3 = 2 i b2 + b3 = 0, iar din |z1 1| a s obinem a2 2a1 + 1 + b2 = = a2 2a2 + 1 + b2 . Tinnd seama de aceste r t 1 1 2 2 calcul direct, se arat c Re (z1 z2 ) (1 z3 ) = 0. a a z X.48. Se consider planele paralele si aflate la a distana h unul de cel lalt si 4ABC echilateral inclus n t a A planul . a) S se afle locul geometric al punctelor M pentru a care M A2 + h2 = MB 2 + MC 2 . B b) S se determine M astfel nct suma MA2 + a +MB 2 + MC 2 s fie minim . a a Viorel Cornea i s Dan Stefan Marinescu, Hunedoara Soluie. a) Fie M 0 = Pr M; atunci MA2 = h2 + M 0 A2 t M i analoagele, deci relaia care caracterizeaz pe M devine s t a 0 2 0 2 0 2 t s M A = M B +M C . Fie D simetricul lui A faa de BC i fie a latura 4ABC. Aplicnd teorema medianei n 4M 0 AD i n 4M 0 BC s 0 2 0 2 0 2 0 2 2 4M P = 2 M A + M D AD = 2 M B + M 0 C 2 BC 2

de unde, dup calcule, M 0 D2 = a2 . Atunci locul punctului M 0 este inclu a

de centru D i raz a. Se arat uor c orice punct de pe acest cerc apar s a a s a t lui M 0 . Rezult c locul lui M este proiecia locului lui M 0 pe planul . a a t b) Dac T este punctul lui Torricelli al 4ABC, avem: a 3h2 +

X.49. S se arate c sin3 x + sin3 y + sin3 z 3 sin x sin y sin z a a 3 [sin x (1 cos (y z)) + sin y (1 cos (z x)) + sin z (1 cos (x 4 x, y, z [0, /3]. Marian Tetiv Soluie. Funcia sinus este concav pe intervalul [0, ]; aplicnd inega t t a Jensen cu argumentele 3x, 3y, 3z [0, ], obinem c t a X

(M 0 A + M 0 B + M 0 C)2 (T A + T B + T C)2 3h2 + , 3 3 s a cu egalitate dac M 0 = O - centrul cercului circumscris i, pe de alt parte a ns 4ABC este echilateral, deci O = T i atunci suma este minim dac M a s a a

MA2 + M B 2 + MC 2 = h2 + M 0 A2 + h2 + M 0 B 2 + h2 + M 0 C 2

Dup rearanjarea termenilor n membrul drept i mprirea prin 4, obin a s at t litatea dorit. Egalitate se obine pentru x = y = z. a t

sin 3x + sin 3y + sin 3z 3 sin (x + y + z) X X sin x cos y cos z 3 sin x sin y sin z 3 sin x 4 sin3 x 3 X X X sin x 3 sin x cos y cos z 9 sin x 4 sin3 x 12 sin x sin y sin z 3

a a X.50. Fie ak , bk , ck N, k 1, n; not m cu f (p) num rul tripletelor de submulimi (nu neap rat nevide) cu reuniunea M = {1, 2, . . . , n}, or t a P P P disjuncte si astfel nct num rul a ai + bi + ci p s fi a iM\A iM \B iM\C P xi = 0). Ar tai c dac f (0) = f (1) = f (2), atun a t a a de 3 (convenim ca i . . pentru care ai + bi + ci . 3. Gabriel Dospinescu, student, B Soluie. Vom folosi metoda descris n articolul Combinatoric ... t a a 2 publicat de autorul problemei n nr. 2/2003 al revistei. Fie = cos 3 avem c a n X Y (ai +bi )+ (ai +ci )+ iB i iC ai +bi + bi +ci + ci +ai =
i=1 ABC=M AB=BC=CA=

fapt care se obine desfcnd parantezele n stnga i grupnd terme t a s s t a p = p(mod 3) i atunci, utiliznd ipoteza, obinem c n Y ai +bi + bi +ci + ci +ai = f (0) + f (1) + f (2) 2 .
i=1

Dac f (0) = f (1) = f (2),produsul din stnga este zero, deci exist i a a as a care ai +bi + bi +ci + ci +ai = 0. Aceasta este posibil dac i numai dac

ai + bi (mod 3), bi + ci (mod 3) i ci + ai (mod 3) reprezint o permutare a s a 0, 1, 2 i atunci 3 | ai + bi + bi + ci + ci + ai , deci 3 | ai + bi + ci . s

Clasa a XI-a

XI.46. Determinai A, B Mn (Z) pentru care det (A+B) = 2 si det (A t Cezar Lupu, elev, C Soluie. Fie A, B ca n enun i fie P (X) = det (A + XB) Z [X], p t ts grad n. Din ipotez, P (1) = 2 i P (3) = 5. Se tie c, dac P Z [X] a s s a a . . a b; n cazul nostru, P (3) P (1) . . sunt distincte, atunci P (a) P (b) . . . . 2, absurd. n concluzie, nu exist matrice cu proprietile dorite. 3. a at a, dac i = j a , unde XI.47. Fie A Mn (R) matrice cu aij = b, dac i 6= j a a Z. Ar tai c A este inversabil si determinai A1 . / a t a a t b Gheorghe Iu

Soluie. Adunnd toate liniile la prima i apoi scznd, pe rnd, t s a din celelalte coloane, obinem c det A = [a + (n 1) b] (a b)n1 6= 0 t a a a t a + (n 1) b = 0 = 1 n Z, iar a b = 0 = 1 Z, situaii cont b b Rezult c A este inversabil. Pentru determinarea inversei, fie A = (a a a a unde I este matricea unitate, iar B matricea avnd toate elementele e Avem:

A2 = (a b)2 I + 2b (a b) B + b2 B 2 = 1 1 = (a b)2 I + 2b (a b) [A (a b) I] + b2 n [A (a b b b h i = I (a b)2 2 (a b)2 bn (a b) + A [2 (a b) + bn] = A = [2a + b (n 2)] I (a b) [a + b (n 1)] A1 1 2a + b (n 2) I A A1 = (a b) (a + bn b) (a b) (a + bn b) = [2a + b (n 2)] A (a b) [a + b (n 1)] I

XI.48. Se denete sirul (xn )n0 prin xn = x2 s n1 [xn1 ], a a x0 0, 1 + 5 /2 . S se arate c lim xn = 0. n Ctlin Tigeru a a a Soliie. Dac x0 [0, 1), atunci [x0 ] = 0 i se demonstreaz uor pri t a s a s n a a c xn = x2 ; prin urmare, lim xn = 0. Dac x0 = 1, se observ imediat a 0 n s n 1, adic lim xn = 0 i n acest caz. a n 1+ 5 . Se arat prin inducie c xn 0, a t a Presupunem c x0 1, a 2 n 1. n plus, xn poate lua valorile x2 , 0 sau x2 1, dup cum xn a n1 n1 1+ 5 xn1 = 1, respectiv xn1 1, ; n fiecare caz, xn xn1 0, 2 este descresctor. Rezult c irul este convergent i fie l limita sa. Vom a a as s

c (zn )n0 este mrginit. Apoi, fiindc an zn+1 zn an , n a a a P P P ak zn + ak , deci irul yn = zn + s ak este monoton de zn+1 + k=n+1 k=n k=n P i mrginit, adic (yn )n0 este convergent. ns irul s a a as ak este c
k=n n0

a |xn+1 2xn + xn1 | + |xn+1 3xn + 2xn1 | an , n 1. S se arate c este convergent. Paul Georgescu i Gabriel P s Soluie. Demonstrm nti c, dac (zn )n0 i (an )n0 sunt iruri t a a a s s P an < , astfel nct |zn+1 zn | an , n 0, atunci (z reale cu n=1 n1 n1 P P convergent. ntr-adevr, deoarece |zn | |z0 |+ a |zk+1 zk | |z0 |+ a
k=0 k=0

, n n0 (prin 1+ 5 de unde concluzia. S presupunem deci prin absurd c xn 1, a a 2 s a a t n acest caz, xn = x2 1, n N i, trecnd la limit n aceast relai n1 1+ 5 c l = a . Aceasta contrazice ns faptul c irul este descrescto a a s a 2 ncheie demonstraia. t P XI.49. Fie (xn )n0 , (an )n0 siruri de numere reale astfel nct

c exist n0 N astfel nct [xn0 ] = 0; atunci xn = x2 0 a a n

nn0

n=

la 0, deci (zn )n0 este ir convergent. s 1 Aplicm acest rezultat irurilor zn = xn+1 xn i zn = xn+1 2xn a s s 2 convergena lor; atunci (xn )n0 este convergent ca diferena de iruri con t t s n XI.50. Fie n 2 N, iar f : R R o funcie cu proprietatea c f t a f ( n+1 xn y), x, y R. S se arate c funcia este descresc toare pe a a t a cresc toare pe [0, ). (n legtur cu Problema 2819 din Crux Mathem a a a nr. 2/2003.) Titu Zvonaru, B Soluie. Fie a, b [0, ) astfel nct a > b. Deoarece t x + x + + x + y nx + y = n+1 xn y, x, y > 0, n+1 n+1 nx + y ncercm s gsim x, y astfel nct a = a a a , b = n+1 xn y. Prin s n+1 x trebuie s fie soluie a ecuaiei nxn+1 (n + 1) axn + bn+1 = 0. C a t t a t a g (x) = nxn+1 (n + 1) axn + bn+1 , avem c g este funcie continu de = bn+1 an+1 < 0 i lim g (x) = , deci ecuaia g (x) = 0 admite o un s t
n

a a t x0 > a > 0; corespunztor gsim pe y0 . n aceste condiii, p nx0 + y f (a) = f f n+1 xn y0 = f (b) , 0 n+1 adic f este cresctoare pe [0, ). a a

Dac a, b (, 0] cu a > b procedm asemntor, cu observaia c n a a a a t a n nx + y n+1 x y, iar radicalul are sens dat fiind faptul c n este par. a n+1

Clasa a XII-a

XII.46. S se determine funcia f : R R dac (R, ) este gr a t a cu proprietatea c simetricul oric rui element x [1, 1] se afl n [ a a a x y = f (x) + f (y), x, y R. Ioan Scleanu a a Soluie. Notm cu e elementul neutru al grupului i cu x0 simetricul t a s x = x e, x R, rezult c f (x) = x f (e), x R. Pentru x = e a a e e a f (e) = , deci f (x) = x , x R. Deoarece xx0 = e, avem c f (x)+ 2 2 0 0 prin urmare x = 2e x. Pentru x = 1, x = 2e 1 [1, 1], de unde e [ a a x = 1, x0 = 2e + 1 [1, 1], de unde e [1, 0]. Rmne c e = 0, deci funcie care verific toate condiiile din problem. t a t a

i lim f (x) = +, adic = a i = 0, > 0. Aadar, s s s a x x + a f (x) = = (1 a) x + a, x R + (am utilizat faptul c relaia + = + revine la = (1 a) . Cu a t x = y = 2 n condiia de morfism obinem f (4) = [f (2)]2 , rezult c t t a a 4 3a = (2 a)2 a2 a = 0 a {0, 1} .

XII.47. Fie G = (a, b), a, b R, iar "" nmulirea numerelor reale. S t mine a, b astfel nct R , (G, ) printr-un izomorsm de forma f : = + x + , x R , cu , , , R. f (x) = + x + Alexandru Blaga i Ovidiu Pop, Sa s Soluie. Deoarece (G, ) este grup i 1 este unitatea faa de nmulire, t s t t a < 1 < b i f (1) = 1 + = + . s Dac b R, fie x, y G, 1 < x < b, 1 < y < b. Cum (G, ) este grup a c xy < b2 b, de unde b2 b sau b (0, 1), ceea ce este n contradicie a t Rezult c b R, deci b = +. a a / Deoarece f 0 (x) = , funcia f este strict monoton pe (0, ) t a (x + )2 Cazul I. f strict cresctoare pe (0, ) > 0. Este necesar ca lim a

x0

Convine doar a = 0. n concluzie, a = 0, b = + i f (x) = x, x R . s + Cazul II. f strict descresctoare pe (0, ) < 0. Urmm p a a 1 aceeai cale i obinem a = 0, b = + i f (x) = , x R . s s t s + x XII.48. Fie (G, ) grup de element neutru e si x, y G penrtru care a) k N \ {1} a. . xk = e; b) p N \ {1} a. i. xy = yp x. S se arate c : a a 1) xy n xk1 = y np , n N ; 2) xy = yx y n(p1) = e, n N . Mihai Ha

s 2) Dac xy = yx, atunci y p x = yx, deci y p = y i prin urmare y p1 = a a avem i y n(p1) = en = e, n N . Reciproc, dac y n(p1) = e, n s particular y p1 = e, deci xy = y p x = yp1 yx = e yx = yx, ceea rezolvarea. XII.49. Se consider numerele reale b > a 0, c 1 si funciil a t Z nb g (x) dx = d R. S se arate c siru a a f, g : R+ R+ astfel nct lim n na Z b 1 un = dx este convergent si s se afle limita sa. a a c + f (x) + g (nx) D. M. Btineu - Giurgiu, B a t Soluie. Din ipotezele problemei, avem c t a 1 g (nx) 1 = 0 c + f (x) c + f (x) + g (nx) [c + f (x)] [c + f (x) + g (nx)] a pentru x [a, b] i n N . Prin integrare, deducem c s Z b Z b 1 0 un g (nx) dx. c + f (x) a a Z Z b ns a 1 bn 1 g (nx) dx = lim g (t) dt = d lim = 0, 0 lim n a n n an n n Z b 1 prin urmare exist limita irului (un )n1 , egal cu a s a dx. c + f (x) a unde k N este fixat.

Soluie. 1) Demonstrm afirmaia prin inducie dup n. Pentru n = 1, t a t t a = y p xxk1 = y p xk = y p . Presupunem concluzia adevrat pentru n i s o a a s a pentru n + 1; avem: xy n+1 xk1 = (xy) yn xk1 = y p x y n xk1 = yp y np = y (n+1)p .

XII.50. Fie s (n) suma cifrelor num rului natural n. S se calculeze li a a

cifrelor cel puin m. ntr-adevr, s presupunem c exist B cel mai mi t a a a a nenul al lui A cu s (B) < m. Cum s (iA) m, i 1, 9 se impun a B 10A > 10m ; fie B = ar 10r + + a1 10 + a0 , unde r m. Considerm C = B 10rm (10m 1). Evident c C < B i A divide C. n plus, dac arm < 9 atunci s (C) = a s a dac arm = 9, atunci s (C) < s (B), prin urmare C este un numr mai a a B i avnd proprietile acestuia, absurd. Rmne deci c s (nA) m, n s at a a Acum, deoarece 11 {z . 1 | 10[lg n] 1, iar 10[lg n] 1 | n! fiindc 10[lg n a | .. }
[lg n]

Gabriel Dospinescu, student, B Soluie. Vom arta nti c orice multiplu nenul al lui A = | {z . 1 t a a 11 . . }
m

aplicnd rezultatul demonstrat mai sus urmeaz c s (n!) [lg n]. Se a a s [lg n] s (n!) lim = , prin urmare lim k = . n lnk ln n n ln ln n

Soluiile problemelor pentru pregtirea concur t a din nr. 1 / 2004


A. Nivel gimnazial

G56. Fie m Z, n 2 Z + 1 fixate. S se arate c ecuaia nx + y = m a a t a are o unic soluie (x0 , y0 ) cu proprietatea c |y0 | < |n| /2. a t Petru Asa Soluie. Pentru existena soluiei, s observm c exist q, r Z, 0 t t t a a a a |n| m = nq + r (din teorema mpririi cu rest). Dca 0 r < at a , lum x0 = a 2 |n| Dac a s a < r < n, lum x0 = q + sgn (n) i y0 = r |n|; avem evident c a 2 nx0 + y0 = n (q + sgn (n)) + r |n| = nq + |n| + r |n| = nq + r = iar |y0 | = |n| r < |n| |n| |n| = . 2 2

atunci n (x0 x1 ) = y0 y1 , de unde

Pentru demonstrarea unicitii, fie nc (x1 , y1 ) soluie a ecuaiei cu | at a t t |n| |x0 x1 | = |y0 y1 | |y0 | + |y1 | < |n|

i cum |x0 x1 | N, n mod necesar trebuie s avem |x0 x1 | = 0, adic s a a apoi y1 = y0 .

G57. Un seic a l sat motenire celor doi fii ai s i cinci c mile, cu c a s a a unul s primeasc jum tate, iar cel lalt o treime. Motenitorii nu si-au put a a a a s averea, aa c au apelat la un nelept care trecea pe acolo, c lare pe o c m s a t a a a procedat neleptul? t Cte probleme asem n toare mai putem formula (n care motenirea a a s c mile, iar fiii primesc a p-a si a q-a parte)? a Gabriel P Soluie. Problema este clasic, dar cu 3 fii i 17 cmile. Totul se b t a s a 1 1 5 faptul c + = 6= 1. neleptul aeaz cmila sa lng cele cinci lsate a t s a a a a 2 3 6 1 1 a a a de eic. Primul fiu ia din cele 6 cmile, adic 3; al doilea ia , adic 2 s 2 3 neleptului i rmne cmila sa. t a a Pentru a formula alte asemenea probleme, trebuie s gsim trei num a a 1 n 1 . Evident c p, q, n astfel nct p i q s divid n + 1, iar + = s a a p q n+1 n 1 1 1 1 5 mcar unul cu inegalitate strict; atunci a a = + + = ,d n+1 p q 2 3 6 Considernd cazurile posibile, obinem c (n, p, q) {(5, 2, 3) ; (3, 2, 4)}, t a n ipoteza p q. Afar de problema dat, mai putem formula nc una. a a a

Generalizare (Petru Asaftei, Iai). Dac neleptul are r cmile, r s a t a formula o innitate de probleme. Mai precis, pentru p, q 2 numere natu 1 1 p + q 6= 4, vom determina n, r N , p - n, q - n, astfel nct + (n p q

1 1 n pq 1 1 + (n + r) = n + = Dac (p, q) = 1, atunci a p q p q n+r n . Deoarece (pq p q, pq) = 1, avem r = k (pq p q), n + r = p n+r a k (p + q), cu k N , k neind multipli de p sau q. Dac (p, q) = d > p1 q1 d2 p1 d q1 d p1 q1 d p1 q1 pq p q = , cu (p1 , q1 ) = 1. = pq p1 q1 d2 p1 q1 d (p1 q1 d p1 q1 , p1 q1 d) = 1, atunci r = k (p1 q1 d p1 q1 ), n + r = n = k (p1 + q1 ), cu k N , k neind multiplu de p sau q. Conditia p + q 6= cazul p = q = 2, care ar conduce la n + r = n r = 0. G58. S se rezolve n N2 ecuaia 2x + 1 = 5y . a t Irina Mustaa, elev, i Valentina Blen t a s Soluie. Dac x, y sunt ambele pare, x = 2p i y = 2q, p, q t a s 2x + 1 = 4p + 1 = (3 + 1)p + 1 = M3 + 2, iar 5y = 25q = (M3 + 1)q = M a 2x + 1 6= 5y . Dac x, y sunt ambele impare x = 2p + 1, y = 2q + 1, p, q 2x + 1 = 2 6= 5y pentru p = 0 i 2x + 1 = 2 4p + 1 = M8 + 1, 5y = 5 25q = s s a deci 2x + 1 6= 5y i pentru p 1. Dac x = 2p + 1, y = 2q, p, q 2x +1 = 24p +1 = 2 (M3 + 1)+1 = M3 iar 5y = M3+1, deci 2x +1 6= 5y . pentru x = 2p, y = 2q + 1, p, q N, avem n cazul p 2 c 2x + 1 = 4p + 1 a iar 5y = M8 + 5, adic 2x + 1 6= 5y . Dac p 1, prin verificri obinem un a a a t a ecuaiei (2, 1). t

G59. Fie A = {n N | s (2000n) + s (2002n) = 2s (2001n)}, unde pri notat suma cifrelor lui x. Demonstrai c orice num r natural nenul are u t a a ce aparine lui A. t Gabriel Dospinescu, student, B Soluie. Notm N (k, p) = 11 {z . 100 {z . 0 ; atunci pentru orice p > t a . . }| . . } |
p k p k+3 p3

s (2000 N (k, p))+s (2002 N (k, p)) = 2p+6+4 (p 3)+6 = 6p = 2s (2001

2001 N (k, p) = 22233 {z . 311100 {z . 0, deci | .. } | .. }


p3 k

k N avem c 2000 N (k, p) = 22 {z . 200 {z . 0, 2002 N (k.p) = 22244 {z . 42 a | .. } | . . }| . . }

adic N (k, p) A. Fie acum m N oarecare; considernd numerele 1, 1 a conform principiului cutiei rezult c putem gsi i < j astfel nct m a a a diferena 11 {z . 1 11 {z . 1, deci m | N (j i, i) ceea ce ncheie soluia. t | .. } | .. } t
j i

G60. S se demonstreze c pentru orice a, b, c (0, ) are loc a a ab (a + b)2 + bc (b + c)2 + ca (c + a)2

4abc 1 + . 4 (a + b) (b + c) (c + a)

Gabriel Dospinescu, student, B

Soluie. S observm c t a a a 4abc 2 [(a+b) (b+c) (c+a) ab (a+b) bc (b+c) ca = (a + b) (b + c) (c + a) (a + b) (b + c) (c + a) 2 2 a + bc (b + c) + b + ac (a + c) + c2 + ab (a + b) = =2 (a + b) (b + c) (c + a) c a a b b b a c c c a + + + + + =2 a+b b+c a+b a+c b+c a+b b+c a+c c+a b+c c+a " bc b c c a ab a b = 2 + + + + a+b b+c c+a a+b b+c c+a (a+b)2 (b+c)2 Aplicm inegalitatea mediilor numerelor a a cror sum este 3; obinem c a a t a b c c a 9 a b + + + + . a+b b+c c+a a+b b+c c+a 4

a b c b c + + i s + a+b b+c c+a a+b b+

nlocuind n identitatea demonstrat, obinem concluzia. Egalitatea e a t atunci cnd dou dintre numerele a, b, c sunt egale. a G61. S se demonstreze c pentru orice a, b, c (0, ) are loc a a p 3 (a2 +b2 )(b2 +c2 )(c2 +a2 ) a+b b+c c+a (a+b)(b+c + + 27 54 2 c a b abc abc Marian Tetiv Soluie. Prima inegalitate se scrie succesiv t p [ab (a+b) + bc (b+c) + ca (c+a)]3 54 2 a2 b2 c2 (a2 +b2 ) (b2 +c2 ) (c2 + p 3 2 2 54 2 a2 b2 c2 (a2 +b2 ) (b2 +c2 ) a b +c + b a2 +c2 + c a2 +b2

fapt care rezult prin nmulirea membru cu membru a inegalitilor a2 + a t at b2 + c2 2bc, c2 + a2 2ac. n ce privete a douap s inegalitate, ce rezult prin nmulirea membru cu a t p p inegalitilor cunoscute 2 (a2 + b2 ) a+b, 2 (b2 + c2 ) b+c, 2 (c2 + at Egalitatea se atinge pentru a = b = c. G62. Fie ABCD un patrulater convex n care se poate nscrie p tratu a de centru O (M (AB), N (BC), P (CD), Q (AD)). S s a AB + BC + CD + DA 2 (AO + BO + CO + DO). Cnd are loc egali Lucian Tuescu, Craiova i Ioan Serdean t s

Pentru a demonstra aceast inegalitate, vom intercala ntre cele dou ca a a 27abc a2 + b2 b2 + c2 c2 + a2 . Faptul c a 3 2 27abc a2 + b2 b2 + c2 c a b + c2 + b a2 + c2 + c a2 + b2 rezult din inegalitatea mediilor aplicat numerelor a b2 +c2 , b a2 +c2 i a a s Apoi, avem p 27abc a2 + b2 b2 + c2 c2 + a2 54 2 a2 b2 c2 (a2 + b2 ) (b2 + c2 ) (c2 2 a + b2 b2 + c2 c2 + a2 8a2 b2 c2 ,

Soluie. Conform inegalitii lui Ptolomeu aplicat n patrulaterul AM t at a a 2 (AQ + AM ) a AO, u AQ MO + AM OQ AO MQ, adic a 2 latura ptratului M N P Q; prin urmare, AQ + AM 2 AO. Scriin a trei inegaliti analoage i adunndu-le, urmeaz concluzia. Egalitatea ar at s a cele patru patrulatere n care s-a aplicat Ptolomeu sunt inscriptibile, fa ntmpl dac ABCD este dreptunghi. ns n ABCD trebuie s se po a a a a ptratul MNP Q, deci ABCD este el nsui ptrat. a s a A Justificare: ABCD - dreptunghi, M N P Q - ptrat a ABCD - ptrat. a ) \ \ m(M N B) + m(CN P ) = 90 M \ \ BM N CN P \ \ m(MN B) + m(NMB) = 90 ) \ \ m(BMN ) + m(AMQ) = 90 \ \ BM N AQM \ \ m(AM Q) + m(AQM ) = 90 Considerm 4N CP , 4MBN i 4QAM a s b b b (m(A) = m(B) = m(C) = 90 )

[N P ] [M N ] [MQ] IU 4N CP 4M BN 4QAM \ \ \ CN P BM N AQM [BN ] [AM] [BC] [AB] ABCD - ptrat. a [N C] [MB] b b G63. n 4ABC cu m(A) = 10 si m(B) = 100 construim M \ \ \ = 40 si m(N BC) = 75 . S se afle m(AM a N (AC) astfel ca m(MCB) Octavian Bondo \ Soluie. Fie P (AC) astfel ncat m(P BC) = 40 . Obinem atunci 4 t t cel, deci [BP ] [BC], apoi 4MBC isoscel cu [BM ] [BC], de unde [BP \ \ s ns m(M BP ) = 60 , deci 4MBP este echilateral i [M P ] [MB], iar m(B a Pe de alt parte, tot din congruene de unghiuri, 4N P B este isoscel cu [N P a t \ \ a prin urmare [N P ] [P M ] i cum m(N P M) = 50 , gsim m(NMP ) = 65 s \ m(AM N ) = 180 65 60 = 55 . G64. Prin punctul P al laturii (AC) a 4ABC A se duc paralele la medianele AA0 si CC 0 , care inter secteaz laturile (BC) si (AB) n E, respectiv F . a F Fie {M} = EF AA0 , {N } = EF CC 0 , iar L si Q L mijloacele segmentelor [F P ], respectiv [P E]. S se C a 0 0 M arate c dreptele ML, N Q si A C sunt concurente. a G N Andrei Nedelcu, Iai s S Soluie. Fie G centrul de greutate al 4ABC t i {T } = P E CC 0 . Deoarece T N k P F , avem s B A EN ET ET GA0 1 = . ns a = = , de unde 2EN = NF TP TP GA 2 = N F . Analog se arat c 2MF = ME, prin urmare F M = MN = N E. a a

PC PE FS C 0F = = 0 = , deci F S = 0 AA CA AC AA0 2 F P ES este paralelogram. n 4F SP , M se afl pe mediana din F la de a 3 M este centrul de greutate al 4F SP i atunci SM este median i va tre s as Analog, Q SN , de unde concluzia. F S k AA0 , S A0 C 0 ; atunci

G65. Fie SABCD o piramid cu baza ABCD dreptunghi, M proie a pe SB si N proiecia lui C pe SA, iar {P } = AM N B. Stiind c M t a N (SA), s se arate c N P SA M B = SM AN P B. a a Daniel Stefan Ninu, Soluie. Fie {O} = AC BD i a = OA = OB = OC = OD t s c N O = MO = a, ce mediane corespunztoare ipotenuzelor n triungh a a tunghice. Prin urmare, punctele A, B, M , N aparin sferei de centru O t ns cele patru puncte sunt coplanare, iar un plan taie o sfer dup un cer a a a trulaterul ABMN este inscriptibil. Atunci 4N P A 4MP B i 4N P M s NP NA PM MN NP P M N de unde = , respectiv = , prin urmare = MP MB PB AB MP P B M MB NP MN MN = . Pe de alt parte, 4SM N 4SAB, deci a adic a AB PB NA AB Comparnd ultimele dou relaii, rezult concluzia. a t a

B. Nivel liceal

L56. Fie ABCD patrulater convex si {P } = AB CD, {Q} = A Consider m J (AQ), L (BQ), K (DP ), N (AP ) astfel nct Q a QL = CB, P K = DC si P N = AB. S se arate c JL k N K. a a Carmen Nejn Soluie. Fie M Int ABCD, avem: t Q SMAD + SM BC = AD d (M, AD) BC d (M, BC) + = 2 2 QJ d (M, QJ) QL d (M, QL) = + = J 2 2 D = SMJQ + SMQL = SQJL + SM JL
L

i cum SQJL este constant, locul geometric al s a C punctelor M pentru care SMAD + SMBC = k este o poriune dintr-o dreapt d paralel cu JL. Analog se t a a M arat c locul geometric al punctelor M pentru care a a BN SMAB + SMCD = k0 este o poriune dintr-o dreapt A t a 1 d0 paralel cu N K. Lund k = k0 = SABCD cele dou locuri geometri a a 2 prin urmare JL k N K.

Not. Soluie corect au dat urmtorii elevi: Andrei-Codru Onofre a t a a t Rotaru, Cosmin-Alexandru Spnu.

L57. Fie 4ABC nscris n cercul C si punctele D (CB, D0 (BC a \ ABC, BAD0 ACB. Se mai consider cercul C1 tangent la AD a CAD \ \ \

C, cercul C2 tangent la AD0 , CD0 si la C, iar {E} = C1 [BD], {F } = C2 se arate c cercul circumscris 4AEF si cercul nscris n 4ABC sunt co a Neculai Roman, Mirce Soluie. Fie {N } = AD C1 , {M1 A} = AD C. t \ \ Avem c CAD ABC AMC, deci [AC] [M C]. a \ Aplicm teorema lui Casey cercurilor cele A, C, M (dea C1 t generate) i C1 , tangente interior cercului C; obinem s succesiv: AC M N + AN M C = AM CE AC MN + AN AC = AM CE AC (M N + AN ) = AM CE AC = CE.

N B E D

Rezult c 4ACE este isoscel, deci mediatoarea lui [AE] a a \ M este bisectoarea unghiului ACB. Analog se arat c a a \ de unde concluzia. mediatoarea lui [AF ] este bisectoarea lui ABC,

L58. Pe muchiile (Ox, (Oy si (Oz ale unui triedru oarecare se consider A, L (Ox, B, M (Oy si C, N (Oz astfel nct OA = OB = OC = \ \ = OM = ON = b (a < b). Not m = m(Oy, Oz), = m(Oz, Ox), = m a si {P } = (AMN ) (BN L) (CLM), {Q} = (LBC) (M CA) (N A calculeze distana P Q n funcie de a, b, , , . t t Temistocle B Soluie. t Existena punctului P (ca i t s a lui Q) se arat uor; a s ntr-adevr, a z (AMN ) (BN L) = N X, unde {X} = AM BL, N iar N X (CLM) = {P }. Notm OA = rA etc. n planul (AM N ) putem a C scrie P AP = AM + AN B O rP rA = (rM rA ) + (rN rA ) A X rP = (1 ) rA + rM + rN . (1)
L

Motive de simetrie ne sugereaz s considerm = 0 = 00 i = a a a s i s cutm , pentru care au loc egalitile precedente. Egalnd co s a a a at corespunztoare versorilor u1 , u2 , u3 , obinem sistemul a t (1 ) a = b = b, b = (1 ) a = b,

Notnd cu u1 , u2 , u3 versorii pe (Ox, (Oy i (Oz, relaiile (1) i (2) se po s t s 0 0 rP = (1 ) au1 + bu2 + bu3 = 1 au2 + 0 bu3 + 0 = 1 00 00 au3 + 00 bu1 + 00 bu2 .

Procednd similar n planele (BN L) i (CLM) x s obinem i relaiile t s t rP = 1 0 0 rB + 0 rN + 0 rL , rP = 1 00 00 rC + 00 rL +

b = b = (1

care are soluia unic = = t a

a . Ca urmare, relaiile (3) devine t 2a + b ab rP = OP = (u1 + u2 + u3 ) . 2a + b ab (u1 + u2 + u3 ) . a + 2b

n mod analog, n privina punctului Q gsim t a rQ = OQ =

n sfrit, avem urmtoarele s a ab ab (u1 + u2 + u3 ) , PQ = a + 2b 2a + b 2 ab ab 2 PQ = PQ PQ = (u1 + u2 + u3 ) (u1 + u2 + u a + 2b 2a + b = deci a2 b2 (a b)2 (3 + 2 cos + 2 cos + 2 cos ) , (a + 2b) (2a + b)

L59. Care este probabilitatea ca latura si diagonalele unui romb, luat plare, s fie laturile unui triunghi? a Petru M Soluie. Proprietatea de a putea construi un triunghi cu laturile c t cu latura i diagonalele unui romb este adevrat sau nu pentru toate s a a din plan dintr-o clas de romburi asemenea. Este suficient s rezolvm a a a pentru romburile a cror laturi au lungimea egal cu 1. Un asemenea a a determinat de unghiul pe care l formeaz diagonala cea mai lung cu a a laturile rombului; notm msura acestui unghi cu x. Mulimea cazuril a a t i este D = 0, . D 4 Din triunghiul BCD, conform teoremei cosinusului, avem d2 = BD2 = BC 2 + DC 2 2 BC CD cos 2x = 1 = 2 2 cos 2x = 4 sin2 x d1 = 2 sin x.

p ab (a b) PQ = p 3 + 2 (cos + cos + cos ). (a + 2b) (2a + b)

180 2 x Analog, din 4ABC, se obine c d2 = AC = 2 cos x. t a rezult c d1 = 2 sin x 2 i A a a s Din 0 < x 4 d2 = 2 cos x 2. Dintre numerele l = 1, d1 = 2 sin x, d2 = 2 sin x cel este d2 . Putem construi un triunghi cu laturile avnd aceste lungimi, dac x dac l + d1 > d2 1 + 2 sin x > 2 cos x. Notnd tg = t, avem a 2 2 72 4t 1t 2 1+ , > 3t + 4t 1 > 0 x 2 arctg 2 2 1+t 1+t 3 4

# 72 Domeniul valorilor posibile este D0 = 2 arctg , . Proprietatea c 3 4 72 72 8 arctg 2 arctg mes D0 4 3 3 =1 = . p= mes D 4 a L60. Fie A1 A2 . . . An si B1 B2 . . . Bn ( n > 2) dou poligoane nscrise cerc de centru O si avnd centrele de greutate tot n O. S se arate c pu a a merota vrfurile poligonului A1 A2 . . . An pentru a obine un nou poligon Ai1 t n care Aij 6= Bj pentru j {1, 2, . . . , n}. Gabriel Dospinescu, student, B Soluie. Trebuie s artm c exist o permutare Sn astfel nct A t a aa a a pentru i M = {1, 2, . . . , n}. S definim matricea T = (tij )i,jM , tij a Vom arta c sumele elementelor de pe orice linie i coloan n T sunt ega a a s a presupune c poligoanele sunt nscrise n cercul unitate (O fiind origine a complex) i fie ai , bi afixele punctelor Ai , Bi . Suma elementelor de pe s matricea T este n n n n X X X X |ai | + |bj |2 aij ai bj = 2n ai b |ai bj |2 = bj ai
j=1 j=1 j=1 j=1 j=1

Analog, suma elementelor de pe coloana j a matricei T este 2n. S presu a pentru orice permutare Sn exist i astfel nct ti(i) = 0. Vom spune a a t Ri place linia Sj " dac elementul de la intersecia liniei si coloanei est matricea T . Rezult c nu putem asocia cte o linie distinct fiecrei colo a a a a nct coloanele respective s plac liniile asociate lor. Deci, din lema m a a rezult c exist k > 0 i k coloane ce plac cel mult k 1 linii. Prin per a a a s linii i coloane, putem presupune c aceste linii i coloane sunt primele din s a s T . S facem suma elementelor dreptunghiului determinat de aceste k a k coloane. n dreptunghiul determinat de primele k coloane i ultimele s linii avem numai zerouri (cci cele k coloane nu plac nici una dintre liniil a . . . , n), deci suma elementelor din dreptunghi este egal cu suma element a primele k coloane, adic 2nk. Pe de alt parte, evident, suma este cel mul a a elementelor de pe primele k 1 linii, adic 2n (k 1). Deducem c 2n (k a a contradicie. t L61. Fie n 3. S se determine maximul expresiei a

(am folosit faptul c centrul de greutate al poligonului B1 B2 . . . Bn es a n P bj = 0).

E = x3 x2 + x3 x2 + + x3 x2 + (n 1)2(n1) x3 x3 x3 , 1 2 2 3 n 1 1 2 n cnd numerele nenegative x1 , x2 , . . . , xn au suma 1. Gabriel Dospinescu, student, B 3 2 t Soluie. Pentru x1 = , x2 = , xk = 0, k 3, n, obinem pentru E t 5 5 2 3 2 3 108 108 = 5 , deci maximul cerut este cel puin . S demonstrm c a a t 3125 5 3125

108 . Fie x1 , x2 , . . . , xn 0 cu suma 1, fixate. Permu lui E este cel mult 3125 at variabilele, putem presupune c x1 = max xk . Conform inegalitii mediil a s 3 5 x (x2 + + x1 x1 x1 x2 + + xn x2 + + xn 1 + + + + 5 1= 3 3 3 2 2 33 22 deci este suficient s demonstrm c a a a ns a

x3 (x2 + + xn )2 x3 x2 + + x3 x2 + (n 1)2(n1) x3 x3 x3 1 1 2 n 1 1 2 n

unde de fiecare dat am folosit faptul c x1 = max xk . n aceste condiii a a t demonstra (1) este suficient s artm c a aa a x3 x2 x3 (n 1)2(n1) x3 x3 x3 . 1 1 2 n

x3 (x2 + + xn )2 2 x3 x2 x3 + + x3 xn1 xn + x3 x2 + x3 x2 1 1 1 1 2 1 n 3 3 2 3 3 2 x1 x2 x3 + + x1 xn1 xn + x2 x3 + + xn1 xn + x3 x2 + 1 2 3 2 3 3 2 3 2 3 2 x1 x2 x3 + x1 x2 + x2 x3 + + xn1 xn + x1 xn x3 x2 x3 + x3 x2 + x3 x2 + + x3 x2 + x3 x2 , 1 1 2 2 3 n1 n n 1

1 x2 + x3 Pentru n = 3, aceasta se scrie x2 x3 i rezult din x2 x3 s a 4 2 pentru n > 3, (2) se scrie sub forma 1 x2 x2 x3 x3 2 3 4 n (n 1)2(n1) i rezult din s a 2(n1) x2 + x3 + + xn 1 2 2 2 3 3 x2 x3 x4 xn (x2 x3 xn ) n1 (n 1) Rezolvarea este astfel ncheiat. a L62. Rezolvai ecuaia 2x2 = y (y + 1); x, y N. t t

Mircea B s Soluia I. Cazul y = 2h, h N. Ecuaia dat devine x2 = h (2h + 1) t t a (h, 2h + 1) = 1, urmeaz c h, 2h + 1 sunt ptrate perfecte, adic m, a a a a h = n2 i 2h + 1 = m2 . De aici rezult c m i n trebuie s satisfac ecua s a a s a a iar soluiile ecuaiei iniiale sunt date de t t t m2 2n2 = 1, y = 2n2 .

x = mn,

Observm c (3, 2) este cea mai mic soluie nebanal a ecuaiei (1) i, a a a t a t s este cunoscut, soluiile ecuaiei (1) n N sunt perechile (mk , nk )kN cu t t k k k 1 1 3+2 2 3 mk = 3+2 2 + 32 2 , nk = 2 2 2

n conformitate cu (2), soluiile ecuaiei date, n cazul y par, sunt perechile ( t t unde 2k 2k 2k 1 1 xk = 3+2 2 32 2 , yk = 3+2 2 + 32 4 4 2

Cazul y = 2l 1, l N . Urmm calea din cazul precedent. Ecuaia di a t scrie x2 = l (2l 1). Deoarece (l, 2l 1) = 1, aceste numere sunt de form 2l1 = n2 . Rezult c m i n verific urmtoarea ecuaie Pell conjugat ec a a s a a t a iar pentru ecuaia dat avem t a 2m2 n2 = 1, x = m n, y = 2m2 1.

Cum (1, 1) este cea mai mic soluie nebanal a ecuaiei (5), soluiile aces a t a t t sunt perechile (mk , nk )kN cu mk = mk + nk , nk = mk + 2nk ,

unde (mk , nk )kN sunt soluiile ecuaiei (1) date de relaiile (3) (T. An t t t D. Andrica - Asupra rezolv rii n numere naturale a ecuaiei ax2 by 2 a t 4/1980, p. 146-148). Tinnd seama de (6), ecuaia din enun are, n cazu t t soluiile (k , yk )kN cu t x xk = (mk + nk ) (mk + 2nk ) , yk = 2 (mk + nk )2 1,

unde mk , nk sunt date de (3). s x n concluzie, mulimea soluiilor este format din (xk , yk )kN i (k , yk ) t t a yk , xk , yk date de (4), (7) i (3). s Soluia II. Observm c ecuaia dat admite soluiile banale x = t a a t a t x = y = 1. Cutm soluiile (x, y) cu x, y {0, 1}. a a t / s Cazul y = 2h, h N . Ca mai sus, m, n N astfel nct h = n2 i 2h deci m, n satisfac relaia 2n2 + 1 m2 . Rezult c m este impar i n t a a s t a adic m = 2k + 1, n = 2l cu k, l N . nlocuind n relaia precedent a 2l2 = k (k + 1). Aadar, dac (x, y) este o soluie nenul a ecuaiei date s a t a t atunci exist o alt soluie nenul (l, k) astfel nct a a t a x = 2l (2k + 1) ,

y = 8l2 ,

i x > l, y > k. s

Cazul y = 2h + 1, h N . Procednd asemntor, dar dup calcule a a a complicate, ajungem la concluzia c pentru orice soluie nebanal (x, y) a t a date, cu y impar, exist o soluie nenul (u, v) a acesteia astfel nct a t a x = (4u + 2v + 1) (2u + 2v + 1) , y = (4u + 2v + 1)2

i x > u, y > s

Rezult c oricare ar fi o soluie (x, y) a ecuaiei date diferit de (0, 0 a a t t a dup un numr finit de pai n care se gsesc, recursiv, soluii mai mici de a a s a t prin relaii de tipul (1) sau (2), vom obine soluia (1, 1) . Cu alte cuvinte, t t t S S a soluiilor este dat de S = {(0, 0)} t a Sn , unde S0 = {(1, 1)} n=0 n 2i (2j + 1) , 8i2 , (4i + 2j + 1) (2i + 2j + 1) , (4i + 2j + 1)2 | (i, j) S s s Mai observm c Sm i Sn sunt disjuncte pentru m 6= n i card (Sn ) = 2n a a

afl dou elemente egale cu 1, deci s = j. Totodat, pe coloana j nu se a a a elemente egale cu 1, deci s = i. Rezult i = j, adic eventualele elemente a a lui E sunt pe diagonala principal. Evident E 6= On deoarece G este netr a presupune acum c elementele nenule ale lui E sunt ei1 j1 = 1, . . . , eir jr = a Fie A G oarecare; A = (aij )i,j=1,n . Din A = AE = EA P P aij = aik ekj = eik akj . Folosind (1) va rezulta c singurele eleme a
k=1,n k=1,n

L63. Fie G Mn (R) un grup netrivial n raport cu produsul uzual al m Presupunem c exist X G astfel nct pe fiecare linie, respectiv coloa a a existe cel mult un element nenul si acesta egal cu 1. S se demonstreze a k {1, 2, . . . n} astfel nct G este izomorf cu un subgrup al lui GLk (R) GLn (R) = {A Mn (R) | det A 6= 0}). Ovidiu Munteanu Soluie. Vom spune c o matrice A are proprietatea (P ), dac pe fie t a a respectiv coloan a sa exist cel mult un element nenul, i acesta egal c a a s produsul a dou matrici cu proprietatea (P ) are proprietatea (P ) rezult a a X 3 , X 4 , . . . au proprietatea (P ). Dar, cum exist un numr finit de a a ordin n cu elemente 0 sau 1 rezult c mulimea X, X 2 , X 3 , . . . este f a a t k, h 1, k > h, astfel nct X k = X h . Acum, simplificnd n G, v a X kh = E, unde E este elementul neutru n G. Avem deci c E are pr (P ) i E 2 = E. Fie E = (eij )i,j=1,n ; s presupunem c i, j astfel ca eij = s a a P c 1 = a eik ekj , deci s astfel ca eis = esj = 1. Dar, pe linia i a lu
k=1,n

t ale lui A pot fi din mulimea {Ais jt }s,t=1,r . Fie aplicaia care asociaz t e e e A matricea A Mr (R), A = (ais jt )s,t=1,r . Evident, E = Ir i : G s este morsm injectiv i unitar de monoizi. ntruct (G, ) este grup i e s s rezult c (G) GLr (R). Prin urmare, G este izomorf cu (G), care es a a al lui GLr (R). [xn+1 , xn L64. Fie sirul (xn )n1 definit prin: x1 , x2 N , xn+2 = xn+1 Dac x2003 = 2004, demonstrai c sirul nu este convergent. a t a Iuliana Georgescu i Paul Georg s xn+1 xn Soluie. Se observ c xn N , n 1. n plus, xn+2 t a a = xn+1 (x2n )n1 i (x2n1 )n1 sunt monoton descresctoare i, cum sunt iruri s a s s naturale, sunt constante de la un loc ncolo, egale cu a, respectiv cu b. Ast [a, b] , deci (a, b) = 1. S presupunem prin a n suficient de mare, b = a (xn )n1 este convergent; atunci a = b = 1. Fie i0 indicele primului term s care este egal cu 1; avem c i0 > 1 deoarece x1 x2003 > 1 i fie xi0 1 a [1, a] [a, 1] Avem c xi0 +1 = a = a, xi0 +2 = = 1 i, prin inducie, xi0 + s t 1 a a t t s xi0 +2k = 1, k 0. Acest fapt intr n contradicie cu convergena irul ncheie rezolvarea. L65. Fie n N si funciile f, g : R R, unde f (x) = x2n cos (1/x t f (0) = 0, f (x) = x2n sin (1/x), x > 0, iar g (x) = x2n+1 sin (1/x)

g (0) = 0 si g (x) = x2n+1 cos (1/x), x > 0. S se afle cel mai nal a derivabiliate al acestor funcii si s se studieze problema continuit tii acesto t a a n origine. Gheorghe Costo Soluie. Se arat c f i g sunt derivabile de ordin n, f (n) este d t a a s a t n origine i g (n) este continu n origine. Aceste afirmaii decurg din s rezultat cunoscut (American Mathematical Monthly; 54(1947), p. 224 i s 1 k p. 97): funcia h : R R dat de h (0) = 0 si h (x) = x sin , x t a x derivabil de m ori dac k = 2m sau k = 2m + 1 si h(m) este discon a a continu n x = 0 dup cum k = 2m sau k = 2m + 1. Pentru demonstra a a t se stabilete prin inducie complet c s t a a 1 1 h(r) (x) = Pr (x) sin + Qr (x) cos , x 6= 0, 0 r m, x x t at unde Pr , Qr sunt funcii polinomiale cu proprietile: 1) sunt de grad k 2r, 2) f (r) (0) = 0, 0 r m, 3) dac k = 2m, atunci sau Pm sau Qm are termen liber nenul, a s 4) dac k = 2m + 1, atunci Pm i Qm au termenii liberi nuli. a

Truelul (rspuns la ntrebarea pus la p. 20) a a

S examinm posibilitile lui X. Prima ar ca X s trag asupra lu a a at a a Y este ucis, atunci urmtoarea lovitur revine lui Z. Dar Z nu mai are a a singur adversar i, cum Z este trgtor prefect, X va un om mort. s a a O opiune mai bun este s inteasc asupra lui Z. Dac l doboa t a a t a a urmtoarea lovitur va reveni lui Y . Cum Y nimerete inta de dou o a a s t a exist o ans ca X s trag la rndul lui asupra lui Y i eventual s cti a s a a a s a s Exista ns o a treia opiune pe care o poate adopta X, mai bun de a t a dentele: X poate s trag n aer. Va urma Y , care va inti asupra lui Z, c a a t este adversarul cel mai periculos. Dac Z supravieuete, el va trage n a t s acesta este adversarul mai periculos. Ca urmare, trgnd n aer, X i per a s-l elimine pe Z i invers. a s Aceasta este cea mai bun strategie a lui X: n cele din urm Y sau Z a a X va trage asupra supravieuitorului, oricare ar el. X a manevrat astfe t loc de prima lovitur ntr-un truel, s aib prima lovitur ntr-un duel. a a a a

Probleme propuse1
Clasele primare

P.84. Aflai numrul m tiind c 47 este mai mare dect m 14 cu 2 t a s a (Clasa I ) nv. Maria R P.85. ntr-un co sunt 6 mere, iar n altul sunt 5 pere. Cum pot pri s mere i pere astfel nct nici un co s nu rmn gol? s s a a a (Clasa I ) Veronica Corbu, e P.86. n urm cu 4 ani, cnd tatl avea 29 de ani, s-a nscut fiul. Sor a a a avea atunci 2 ani, iar acum este de trei ori mai mare. Mama este de pat mare dect aceasta. Ci ani are acum fiul? t (Clasa a II-a) nv. Oana-Maria L P.87. Un acrobat cade pe o plas elastic de la o anumit nlime a a a at s dup ce atinge plasa la jumtatea distanei dintre plas i locul de und a a t a s anterior. Stiind c atinge de 3 ori plasa i c ultima oar s-a ridicat la n a s a a 2 m, iar plasa este montat la 2 m deasupra solului, s se afle distana de a a t unde a czut prima dat pn la sol. a a a (Clasa a II-a) Andrei Stativ, a P.88. Triau odat o bab i un moneag; moul avea 100 ani, iar a a a s s s amndoi erau albi ca iarna i triti ca vremea cea rea pentru c erau s s s a spune c ar fi avut un copil pe cnd vrsta babei era jumtate din jum a a acum a vrstei moneagului i c acesta ar fi plecat n lume cnd vrsta m s s a era de dou ori ct vrsta aceea a babei. Fiul nu s-a mai ntors. Ce vrst a cnd a plecat n lume? (Clasa a III-a) nv. Ileana Ro P.89. La un concurs de biciclete, triciclete i mainue (cu patru roi s s t t Bogdan numr roile vehiculelor i observ c sunt 34. Cte vehicule aa t s a a fiecare fel? Gsii toate posibilitile, tiind c numrul vehiculelor de fie a t at s a a depete 5. as s (Clasa a III-a) nv. Doinia Sp t P.90. Lungimea laturii unui ptrat este de 17 m. O persoan pleac a a vrf al ptratului i, mergnd n acelai sens pe laturile acestuia, parcurge a s s de 637 m. Din punctul n care a ajuns se ntoarce i parcurge 773 m. A s distana se va situa n final persoana faa de punctul de plecare. t t (Clasa a III-a) Oxana Pascal, e P.91. Se mpart dou numere naturale. Dac mpritorul, ctul i r a a at s trei numere consecutive cu suma 30, s se afle dempritul. a at (Clasa a IV-a) Vasile Solcanu, Bogdneti a s P.92. Observ regula i completeaz, apoi verific rezultatele gsite: 2+ a s a a a 2 + 4 + 6 = 4 + 4 + 4; 2 + 4 + 6 + 8 = 5 + 5 + 5 + 5; 2 + 4 + 6 + 2 + 4 + 6 + + 14 = ; . . . ; 2 + 4 + 6 + + (a + a) = . (Clasa a IV-a) Valeria Gheorghia, e t
1

Se primesc solu ii pn la data de 31 decembrie 2005. t a

P.93. O foaie de hrtie dreptunghiular se ndoaie de-a lungul de 6 ori, a se 7 benzi egale i suprapuse. Dreptunghiul obinut se ndoaie de-a latu s t rezultnd n nal un ptrat cu perimetrul de 12 cm. S se afle perime a a tunghiului iniial. t (Clasa a IV-a) Petru Asa

Clasa a V-a
V.56. Se consider numrul A = 5 + 52 + 53 + + 52005 . a a a) S se arate c A nu este ptrat perfect. a a a b) S se gseasc 5 divizori mai mici dect 100 ai lui A. a a a Andrei Tofan, V.57. Aflai restul mpririi prin 47 a numrului N = 1268 99 {z . 9 . t at a | .. }
2005 cifre

Alexandru Negrescu, elev, V.58. Aflai numerele naturale x, y, z cu proprietatea c t a 24x+1 + 23y+1 + 22z+1 = 9248.

Cristian - Ctlin Bude a a a a V.59. Dac a1 a2 . . . an 2 b1 b2 . . . bn = c1 c2 . . . cn 2 , s se arate c a


2 2

a1 a2 . . . an a1 a2 . . . an 2 b1 b2 . . . bn b1 b2 . . . bn = c1 c2 . . . cn c1 c2 . . . c
99
9

Petru Asa

V.60. S se determine ultimele dou cifre ale numrului 7 . a a a Artur Bluc, aa a

Clasa a VI-a

VI.56. Determinai, n funcie de numrul ntreg x, cel mai mare divi t t a al numerelor 2005x + 2 i 2006x + 3. s Tamara C VI.57. Un vnztor de autoturisme scade procentul beneficiului su de a a 20% din valoarea vnzrilor. Datorit scderii preurilor, crete valoarea a a a t s Aflai procentul cu care a crescut valoarea vnzrilor, tiind c beneficiu t a s a cu 10%. Marius Fa VI.58. Se aeaz cifrele 2, 3, 4, 5, 6, 8, 9 ntr-o ordine oarecare ob s a numr A. Se aeaz apoi aceleai cifre n alt ordine, obinnd numrul B a s a s a t a se arate c A nu se divide cu B. a Cristian - Ctlin Bude a a b = 120 . Dac mediana [BM] este perp VI.59. Fie 4ABC cu m(B) a pe BC, artai c AB = 2BC. a t a Bogdan Posa, elev, Motr VI.60. Fie 4ABC i punctele E (AB), F (AC) i M (BC) a s s AE = EB, iar ntre 4AEF i 4EF M s existe o congruena. S se arat s a t a a) F este mijlocul lui [AC]; b) [AM] este median sau nlime. a at Ioan Scleanu a a

Clasa a VII-a

VII.56. Fie x, y R cu x2 2y = y 2 + xy = 4. S se arate c x2 2 a a Gigel Buth, Sa

VII.57. Fie x (0, 1), iar n N \ {0, 1}. Artai c nx2 + 2n > n + ( a t a Ion Vian s

VII.58. Fie a, b R astfel nct mediile aritmetic, geometric i arm a as + lor s fie laturi ale unui triunghi dreptunghic. Aflai sinusul celui mai a t unghiurile triunghiului. Romana Ghia i Ioan Gh t t s

VII.59. Fie 4ABC i A0 mijlocul lui [BC]. Dac D (AC), BD A s a 1 1 1 i paralela prin F la BC taie AC n E, s se arate c s a a = + + DE AD CE Claudiu - Stefan P

VII.60. n cercul C se consider coardele [AM] i [AN ]astfel nct AM a s 1) S se determine mulimea punctelor X C ce ndeplinesc condi a t t AX AN . 2) n ce caz mulimea gsit este un arc de cerc? t a a Temistocle B

Clasa a VIII-a

VIII.56. S se determine x, y, z R pentru care x3 y 3 + z 3 = 8, x a Andrei - Sorin Cozma, VIII.57. Fie a, b, c > 0 astfel nct a + b + c = 1. S se arate c a a 1 1 9 1 p +p +p . 2 (1 a) (1 b) (1 b) (1 c) (1 c) (1 a)

Cristian Svescu, elev a

VIII.58. Fie x, y, z (0, ) cu x + y + z 3. S se arate c xn + y n a a n N. Romeo Ilie

VIII.59. Determinai x, y, z R, tiind c x+y+z = 1, iar xy+(x + y)( t s a Gheorghe Molea, Curtea

VIII.60. Se consider prisma triunghiular regulat ABCA0 B 0 C 0 cu A a a a 0 i AA = 3 3. S se arate c pentru fiecare numr a 0, 3 3 , exist e s a a a a 0 00 0 00 puncte Ma , Ma pe dreapta CC 0 astfel nct d (B 0 , (Ma AB)) = d (B 0 , (Ma Mirela M

Clasa a IX-a

IX.56. Determinai numerele reale pozitive x, y, z, t pentru care x+y+ t iar xy + xz + xt + yz + yt + zt + 475 = xyzt. Lucian Tuescu i Liviu Smarandache t s IX.57. S se determine toate funciile f : R R cu proprietatea a t

f (f (f (x) + y) f (x f (y))) = x2 y 2 ,

Adrian Zahariuc, ele x R.

x, y R.

IX.58. Fie n N, n 2. Determinai a1 , a2 , . . . , an1 , c R pentru c t [x] + [x + a1 ] + + [x + an1 ] = [cx] ,

(n leg tur cu G.42 din RecMat 1/2003.) a a Iuliana Georgescu i Paul Georg s IX.59. Fie C (I, r) cercul nscris n 4ABC. S se arate c a a IA IB + IB IC + IC IA 12r2 .

D. M. Btineu - Giurgiu, B a t

IX.60. Fie ABC un triunghi ascuitunghic cu a < b < c. Cercul C (I t triunghiului este tangent dreptelor BC, CA i AB n punctele D, E i re s s s a Dreapta ID intersecteaz CA n D0 i AB n D00 , IE intersecteaz AB n a a s a t a n E 00 , iar IF intersecteaz BC n F 0 i CA n F 00 . Artai c E 0 E 00 = D0 D Temistocle B

Clasa a X-a

X.56. Fie tetraedrul ABCD i M un punct n spaiu. Dac G, GA s t a GD sunt centrele de greutate ale tetraedrelor ABCD, M BCD, M ACD respectiv MABC, s se arate c AGA + BGB + CGC + DGD = 0 dac a a dac M G. a Marius Olteanu, Rmnic

X.57. Dac x, y, a (1, ), s se arate c a a a x+y (x + y + loga x) xy + loga x + (x + y + loga y) xy + loga y x+y (x + xy + y) loga xy. X.58. Let n N, n 1. Prove that n X n2 k2 + n2 2n 2 ln < . k n1 n2
k=1

Mihail Bencze

Jos Luis Daz - Barrero, Barcelon

X.59. Fie f R [X] un polinom de grad n 3 ce admite n rd a a pozitive i subunitare. Dac |f (0)| = f (1), s se arate c produsul rdc s a a a a a 1 cel mult egal cu n . 2 Ioan Serdean

s X.60. Fie k N fixat. Alegem n {1, 2, . . . , k} i a1 , a2 , . . . , an num a mai mari dect 3. Dac probabilitatea ca a2 + a2 + + a2 s se divid a n a 1 2 1 cel puin egal cu t a , s se arate c 24 divide k. a a 24 Cristian Svescu, elev a

Clasa a XI-a
XI.56. Let n be a positive integer. For each positive integer k, let Fk

Fibonacci number (F1 = F2 = 1, Fk+2 = Fk+1 + Fk for all k 1). Prove 2 2 2 Fn Fn+1 2Fn Fn+1 Fn+2 2 2 2 =0 Fn+2 2Fn+1 Fn+2 Fn Fn+1 2 2 2 2Fn Fn+2 Fn+1 Fn+2 Fn

Jos Luis Daz - Barrero, Barcelon

XI.57. Fie ptratul ABCD circumscris cercului C. n ptrat se n a a gonul EF GHIJKL, circumscris cercului C, astfel nct E, F (AB), E G, H (BC), G (BH), I, J (CD), I (CJ), K, L (DA), K {M} = EL AC, {N } = F G BD, {P } = HI AC, {Q} = JK BD. S c suma a AE BF BG CH CI DJ DK AL AM BN C S= + + + + + + + + + + EB F A GC HC ID JC KA LD MC N D P

nu depinde de alegerea vrfurilor octogonului pe laturile ptratului. a Ctlin Cali a a

XI.58. Dac n este un numr natural iar p un numr prim, at a a a a (xn+1 (p) xn (p))n0 este divergent, unde xn (p) reprezint exponentu apare numrul p n descompunerea lui n!. a Sorin Pupan s a
n

XI.59. Fie irul de numere supraunitare (an )n1 , astfel ca lim an = s

studieze continuitatea funciei f : R R, f (x) = lim {xan }, unde {x} t n partea fracionar a numrului real x. t a a Dan Popescu 2 XI.60. Fie a (0, 1); s se demonstreze c pentru orice x > a a es ln a 1 x 1 inegalitatea (1 ax ) x+1 < 1 ax+1 . Angela Tigeru a

Clasa a XII-a

XII.56. Fie Sn mulimea permutrilor de ordin n, iar Sn . Se t a funcia mrginit f : R R. S se calculeze: t a a a 1 1 1 f ( (1)) + f ( (2)) + + f ( (n)) . lim n n 2 n

(O generalizare a problemei 24131, G. M. 5-6/1999.) Marius Olteanu, Rmnic 1 1 XII.57. Considerm matricea A = a i mulimea s t 1 1 1 G = Xa | Xa = I2 + aA, a , . 2

Artai c (G, ) este grup izomorf cu (R, +). Calculai X 1 X 3 X 2n1 a t a t 2 2 2 Gheorghe Iu XII.58. Let f : R [1, 1] be a continuous function. Prove that

Zdravko Starc, Vrac, Serbia and Mo XII.59. Fie f : R R, f derivabil i neconstant pe nici un interva as a Dac a f 0 (x) f 0 (sin x) cos x + f 0 (cos x) sin x, x R, 1 + f 2 (x) demonstrai c nu exist lim f (x). t a a x Paul Georgescu i Gabriel P s XII.60. Fie n > 1 i a1 , a2 , . . . , an (0, 1) astfel nct a1 + a2 + + a s f (x) = 1 i f (x) = f (a1 x) + s se determine funciile f : R R, dac lim t a x0 x + f (an x) pentru orice x R. Gabriel Dospines

Z 3

Z q 1 (f (x))2 dx +

f (x) dx 4.

IMPORTANT

n scopul unei legturi rapide cu redacia revistei, pot fi utilizat a t mtoarele adrese e-mail: tbi@math.tuiasi.ro, vpgeo@lycos.c a profgpopa@yahoo.co.uk . Pe aceast cale colaboratorii pot pur a redacia un dialog privitor la materialele trimise acesteia, procu t numerelor revistei etc.

La problemele de tip L se primesc soluii de la orice iubitor de matem t elementare (indiferent de preocupare profesional sau vrst ). F a a dintre soluiile acestor probleme - ce sunt publicate n revist du t a an - va fi urmat de numele tuturor celor care au rezolvat-o. a

Lucrrile originale ale elevilor vor fi publicate ntr-o rubric sp a a dedicat acestora: NOTA ELEVULUI . Anual, se vor acorda el a - autori dou premii n bani, pentru cele mai bune note publica a revist. a

Adresm cu insistena rugmintea ca materialele trimis a t a vistei s nu fie (s nu fi fost) trimise i altor publicaii. a a s t

Probleme pentru pregtirea concursurilor a


A. Nivel gimnazial

2 x y = y2 z = G76. Rezolvai n mulimea numerelor naturale sistemul t t 2 z x= Adrian Zano a2 b2 G77. i) Fie a, b, c R cu a > b > c; atunci + > a + 2b + c ab bc a2 b2 c2 b2 a2 c2 + + ii) Fie a, b, c R cu a b c > 0; atunci c a b Ioan Serdean b (a + c) c (b + d) d (a + c) a (b + d) + + + 4. c (a + b) d (b + c) a (d + c) b (a + d) Artur Bluc, aa a

G78. Dac a, b, c, d (0, ), s se demonstreze inegalitatea a a

G79. Dac x, y, z (0, ) sunt astfel nct x + y + z = xyz, atunci a p p p xy + yz + zx 3 + x2 + 1 + y 2 + 1 + z 2 + 1.

Florina Crlan i Marian Tetiv s

G80. Fie A mulimea tuturor sumelor de tipul 12 32 52 t n N, unde semnele pot fi alese n orice combinaie posibil. S se arate t a a (n leg tur cu teorema Erds-Surnyi.) a a Petru Asa

G81. Fie n N i k {0, 1, . . . , 2n 1}. S se arate c exist o muli s a a a t cu n elemente care are exact k submulimi cu suma elementelor strict poz t Adrian Zahariuc, ele

G82. Un cal se afl pe tabla de ah n cmpul A1 i dorim s-l ducem a s s a H8 ntr-un numr minim de srituri. Aflai care este acest numr minim, a a t a cte trasee de lungime minim exist. a a Gheorghe Crciun, Plopeni i Gabriel P a s

G83. Fie ABCD patrulater convex i punctele M, N (AB), P, R ( s BM N A P R CD nct AD BC M R N P 6= . S se arate c a a = M N DP RC AB Andrei-Sorin Cozma,

G84. Fie ABCD un trapez cu ABkCD, AB < CD. Se consider CF AE = . Dreapta EF intersectez a E (AD) i F (BC) astfel nct s ED FB DC AB MN = . n M, respectiv N . S se arate c a a EF DC + AB Andrei Ned
1

Se primesc solu ii pn la data de 31 decembrie 2005. t a

G85. Fie A0 , B 0 , C 0 picioarele bisectoarelor unghiurilor 4ABC. Pe la considerm punctele D i E astfel nct D (BE) i cevienele AD a s s s izogonale. S se demonstreze c DB 0 i EC 0 se intersectez pe AA0 . (n l a a s a Propoziia 1, p. 99, RecMat - 2/2004.) t Titu Zvonaru, C

B. Nivel liceal

L76. Fie cercurile C1 i C2 tangente interior unui cerc C n punctele di s respectiv N . Cercurile C1 i C2 sunt secante sau tangente exterior iar axa s s s s cercurilor C1 i C2 taie cercul C n A i B. Dreptele AM i AN taie din no C1 i C2 n K, respectiv L. Artai c AB 2KL. n ce caz avem egalita s a t a Neculai Roman, Mirce L77. Fie punctele P1 , P2 , . . . , P13 n plan astfel nct oricare trei sunt n i toate au coordonate ntregi. S se arate c exist cel puin un triung s a a a t astfel nct centrul su de greutate s aib coordonate ntregi. a a a Vasile Prav i Titu Zvonaru, Comneti at s a s L78. Considerm irul de puncte (Pn )nN pe cercul trigonometric a a s 5 \ \ m(Pn OPn+1 ) = arctg pentru orice n N, Pn OPn+1 fiind considera 12 orientat. S se arate c pentru a a orice punct P pe cercul trigonometric ex 1 astfel nct Pj Int C P, . 2005 Lucian - Georges Ldunc i Andrei Ned a as s s L79. Fie a1 , a2 , . . . , an R n aa fel nct a1 + a2 + + an = 0 i 1 hn 2 2 2 t a max {|ai aj | ; 1 i < j n} 1. Demonstrai c a1 +a2 + +an n 2 i precizai n ce caz are loc egalitate. s t Marius Pachiariu, t L80. Fie un alfabet cu 4 litere a, b, c, d. n acest alfabet se pot form dup urmtoarele reguli: dup a nu poate urma b, dup b nu poate urma a a a a nu poate urma d i dup d nu poate urma a. Cte cuvinte palindromice d s a n, n 2, se pot forma conform acestor reguli? (Prin cuvnt palindromic un cuvnt n care litera de pe poziia k coincide cu litera de pe poziia t t pentru orice k {1, 2, . . . , n}.) Irina Mustaa, e t L81. Fie n 1 un numr natural fixat. O tabl infinit de ah est a a a s n alb i negru n maniera obinuit. O mulime C de csue ale tablei s s s a t a t conex dac putem ajunge din fiecare csua a lui C n fiecare alt csu a a a t a a printr-o succesiune de deplasri n C dintr-o csua ntr-o csua vecin (c a a t a t a comun). Fie S o mulime conex cu 4n csue. Numim raportul cromatic a a t a a t S raportul dintre numrul de csue albe i numrul de csue negre din a a t s a a t afle cea mai mic i cea mai mare valoare posibil a raportului cromatic. as a Adrian Zahariuc, ele L82. Determinai P, Q R [X] pentru care f : R R, f (x) = {p (x) + t este periodic, unde p, q : R R sunt funciile polinomiale asociate lui a t

tiv Q. L83. S se calculeze a " # 1 2 n 1 2 1 3 1 n+1 lim 1+ + 1+ + + 1 + n . n n n n

Paul Georgescu i Gabriel P s

Determinai inf A. t

Marius Olteanu, Rmnic L84. Fie n N, n 3 i s n n A = x > 0; x = a0 + a1 n n + + an1 nn1 ; o a0 , a1 , . . . , an Z; n 1 | a0 + a1 + + an .

Paul Georgescu i Gabriel P s L85. Fie f : R R o funcie pentru care mulimea punctelor n care f t t finit la stnga este dens n R. S se arate c mulimea punctelor n c a a a a t continu este de asemenea dens n R. (O mulime D R se numete d a a t s dac orice interval deschis al axei reale conine mcar un element din D.) a t a Gabriel Dospinescu, Paris, i Marian Tetiv s

Training problems for mathematical contes


A. Junior high school level

x2 y = u2 y2 z = v 2 in the set of natural numbers G76. Solve the system 2 z x = t2 Adrian Zano a2 b2 G77. i) Prove that + > a + 2b + c for any a, b, c R, a > ab bc 2 2 2 2 a b c b a2 c2 ii) Prove that + + 3a 4b + c for any a c a b a b c > 0. Ioan Serdean G78. Prove that b (a + c) c (b + d) d (a + c) a (b + d) + + + 4 c (a + b) d (b + c) a (d + c) b (a + d) for any a, b, c, d (0, ). Artur Bluc, aa a G79. Prove that p p p xy + yz + zx 3 + x2 + 1 + y2 + 1 + z 2 + 1

for any x, y, z (0, ) such that x + y + z = xyz. Florina Crlan and Marian Tetiv

G80. Let A be the set of all sums of type 12 32 52 (2n + 1 for any combination of the signs. Prove that A = Z. (Regarding Erd theorem.) Petru Asa

G81. Let n N and k {0, 1, . . . , 2n 1}. Prove that there exists a with n elements which has precisely k subsets whose sum of elements a positive. Adrian Zahariuc, high school studen

G82. Find minimal number of moves required to transfer a knight on a c from the square A1 to the square H8. For this minimal number, find the distinct paths of minimal length. Gheorghe Crciun, Plopeni, and Gabriel P a

G83. Let ABCD be a convex quadrilateral and let M, N (AB), P, BM N A P R CD such that AD BC MR N P 6= . Prove that MN DP RC AB Andrei-Sorin Cozma, junior high school stud

G84. Let ABCD be a trapezoid with ABkCD, AB < CD. Consider AE CF and F (BC) such that = . The line EF meets BD and A ED FB DC AB MN = . respectively in N . Prove that EF DC + AB Andrei Ned

G85. Let ABC be a given triangle and let A0 , B 0 , C 0 be the legs of its Let D, E be points on the side (BC) such that D (BE) and the cevian AE are isogonal. Prove that DB 0 and EC 0 intersect each other in a poin on AA0 . (Regarding Proposition 1, p. 99, RecMat - 2/2004.) Titu Zvonaru, C

B. High school level

L76. Let C1 , C2 be circles which are internally tangent to a given circ respectively in N, M 6= N . Suppose that C1 and C2 are secant or external and that the radical axis of C1 and C2 meets C in A and B. Let us den respectively by L, the points in which the lines AM, AN meet again C1 , re C2 . Prove that AB 2KL and characterize the case of equality. Neculai Roman, Mirce

L77. Let P1 , P2 , . . . , P13 points with integer coordinates in a plane any given three are not collinear. Prove that there exists at least a triang such that its centroid has integer coordinates. Vasile Prav and Titu Zvonaru, Comneti at a s L78.

Consider (Pn )nN a sequence of points on the unit circle 5 \ \ m(Pn OPn+1 ) = arctg for all n N, Pn OPn+1 being considered as a 12 angle. Prove that for any point P on the unit circle there exists j N

Lucian Ldunc and Andrei Ned a a L79. Let a1 , a2 , . . . , an R be such that a1 + a2 + + an = 0 and 1 hn max {|ai aj | ; 1 i < j n} 1. Prove that a2 + a2 + + a2 n 1 2 n 2 and characterize the case of equality. Marius Pachiariu, high school stud t L80. Consider an alphabet with four letters a, b, c, d. Using this alp can construct words according to the following rules: b cannot succeed a succeed b, d cannot succeed c and a cannot succeed d. How many palyn length n, n 2, can one construct in this manner? (By palyndrome we me in which the k-th letter coincides with the n k + 1-th letter for all k {1, Irina Mustaa, high school stud t L81. Let n 1 be a fixed natural number. An infinite chessboard is black and white in the usual manner. A set C of squares is then called set if one can reach any square in C starting from any given square in a succession of moves in C from a square to a neighboring square (with edge). Let S be a connected set with 4n squares. One calls the chromatic i the quotient between the number of white squares and the number of blac Find the maximal and the minimal value of the chromatic index. Adrian Zahariuc, high school studen L82. Find P, Q R [X] such that f : R R, f (x) = {p (x) + si periodic where p, q : R R are the polynomial functions associated to P , re to Q. Paul Georgescu and Gabriel P L83. Find " # 1 2 n 1 2 1 3 1 n+1 1+ + 1+ + + 1 + n . lim n n n n L84. Let n N, n 3 and n n A = x > 0; x = a0 + a1 n n + + an1 nn1 ; Find inf A.

Pj Int C P,

1 . 2005

Marius Olteanu, Rmnic

o a0 , a1 , . . . , an Z; n 1 | a0 + a1 + + an .

Paul Georgescu and Gabriel P L85. Let f : R R a function such that the set of points in which f left-sided limit is dense in R. Prove that the set of continuity points of f is in R. (A subset D of R is called dense in R if any open interval of R contai an element of D.) Gabriel Dospinescu, Paris, and Marian Tetiv

Pagina rezolvitorilor

BRA OV S Liceul "M. Titulescu". Clasa a IX-a. BARDA Dan: VIII(52-54), BOI TEANU Claudia: VIII(52-54), IX(53,54); BUTNARIU Anda: V S IX(52-55); CHIRA Roxana: VIII(52-54), IX(52-55); COSTA Larisa: V S IX(52,53); ENA OAE Emanuela: VIII(52-54), IX(53,54); GIRIGAN And (52-54), IX(53,54); MZGACIU Alexandra: VIII(52-54), IX(53,54); NAN na: VIII(52-54), IX(53,54); NEGOESCU Anamaria: VIII(52,53), IX(52Iulian: VIII(52-54), IX(52,53,55); NUTU Cosmin: VIII(52-54), IX(52-55 CEA Drago: VIII(52-54), IX(52,55); POTEC Monica: VIII(52-54), s SZCS Daniel: VIII(52-54), IX(52-54); SCHIOPU Iulian: VIII(52-54), ZBARCEA Adrian: VIII(52-54), IX(52,53).

CRAIOVA Scoala nr. 22 "M. Eliade". Clasa a V-a. STANCIU Ioan: P(74-76,78 V(51,52).

HRLAU Scoala "P. Rare". Clasa a II-a (nv. BUDACEA Maria). NEICU M s 67,72,74-79). Liceul "Stefan cel Mare". Clasa a VII-a. CIOFU Alexandra: V(46-4 MANOLIE Ioan: V(46-49), VI.46; SAVA Cristina: V(46-49), VI.48; SCR Gabriela: V(46-49), VII.48; SPIRIDON Florin: V(46-49), VII.48; ZAMFIR Adela: V(46-49), VI.48. Clasa a IX-a. ONOFREI Andrei-Codru: V t VIII.47, IX(46,47,49), L.56; ROTARU Lucian: VII(48-50), VIII.46, IX(46 SPNU Cosmin-Alexandru: VII(46-48), VIII.49, IX.47, L.56.

IA I S Scoala nr. 4 "I. Teodoreanu". Clasa a III-a (nv. BUJOR Lorena) Dana: P(74-80,82,83).

Scoala nr. 7 "N. Tonitza". Clasa a III-a (nv. TUDOSE Elena). AN liana: P(74-79); LEONTE Anca: P(74-79); NECHITA Paula: P(74-7 TICA Simona-Alexandra: P(74-79); SAVIN Rzvan: P(74-79). Clas a (nv. MELINTE Rodica). BOTO ANU Bianca-Mihaela: P(74-76,78,79) S Dnu-Vasilic: P(74-76,78, 79); CONSTANTINESCU Diana-Gabriela: P a t a 79); GU OVATE Diana- tefana: P(74-76,78,79); LEOGAN Larisa-Dia S S 76,78,79); SUCIUC Raluca: P(74-76,78,79).

Scoala nr. 22 "B. P. Hasdeu". Clasa a III-a (nv. DOHOTARU Liliana Andrei-Daniel: P(74-79); CRUCEANU Evangeline Tamara: P(74-76,78,7 a III-a (nv. TRZIORU Iuliana). ADASCALITEI Victor: P(74-79,82,8 TOL Ana-Maria: P(74-78); E ANU Georgiana: P(74-78); GREIEROSU S P(74-78); GNDU Alexandra-Livia: P(74-76,78,79); LAMATIC Ioana: REBEGEA Andrada Elena: P(74-79); UNGUREANU Teofana: P(74 Clasa a III-a (nv. CHIRILA Laura). ANDRONICIUC Ana-Miruna: P 79); BOARU Adrian: P(74-76;78,79); BURUIANA Sebastian-Andrei: P(74

BUHU Vlad: P(74-76,78,79); CEOBANU Andrei-Nicolae: P(74-76,78,80) RAU Alexandra-Elena: P(74-76,78,79); COSTACHESCU Ivona: P(74-76,7 ACONESCU Matei: P(74-76,78,79); GHERAN Ana-Maria: P(74-76,78,79 Ioana: P(74-76,78,79); HATESCU Iustina: P(74-76,78,79,83); HORBOVAN Alexandra: P(74-76,78,79).

Scoala nr. 26 "G. Cobuc". Clasa a III-a (nv. BUCATARIU Rica). D s Ambra-Georgiana: P(64-68,74-76); IACOB Robert-Ionu: P(68,69,74 t IVANCIUC Dumitru-Florin: P(68,71,75,76,78); HRISCU Alexandru: P 79); MOISA Adrian-Bogdan: P(68-70,76-79); MUSTEATA Alexandru: P 74-79); SANDU Ioana-Luiza: P(68,74-78); SCUTARU Ionela-Cristina: P 76,78,79); STOICA Diana-Maria: P(64,65,67,68,76); ZALINCA Georgiana 69,74,78,79). Clasa a IV-a (nv. RACU Maria). BULGARU Ionela-A P(74-79); BURLACU Stefan-Claudiu: P(74-79); CALIN Andreea-Georgia Adriana: P(74-78); IOJA Petru-Alexandru: P(74-78 79,82,83); IFROSA Bogdan-Gabriel: P(74-79,82,83); PINTILIE Rzvan-Florin: P(74-79); a Ionu: P(74-78). Clasa a IV-a (nv. GALIA Paraschiva). ALUPEI Andra t P(74-79); CIOABA Oana-Ctlina: P(74-79); GHERCA Marius-Ctlin: aa aa HOMEA Liviu: P(74-79); HUIDE Gina: P(74-79); MANOLIU Mdlina: S a a MIHAILESCU Laura: P(74-79); PISICA Alexandru: P(74-79); SCUTA stantin: P(74-79); SEGNEANU Elena: P(74-79); ZDREVIT Maria: P(74

Liceul "M. Eminescu". Clasa a V-a. DUCA Mdlina: P(71-73,82,83) a a IACOB Ioana: P(71,73,81-83), V(46,51); POPA Alexandra: P(71,73,81-83) Clasa a VI-a. COHAL Clin: V(51,53-55), VI.51. a Liceul "G. Ibr ileanu". Clasa a VIII-a. UNGUREANU Drago: a s VII(54,55), VIII.52.

Colegiul Naional "C. Negruzzi". Clasa a VI-a. OLARIU Tudor: t V(46-50,52-55), VI(46,47), G.66; TIBA Marius: V.48, VI(46-48,50), VIII(

Colegiul Naional "E. Racovia". Clasa a V-a. TUDORACHE A t t Gabriel: P(80-83), V(51-55).

Colegiul Naional Iai. Clasa a V-a. BACUSCA Alberto: P(71,73,81 t s a CAPATNA Roxana-Maria: P(71-73,80-83), V(46-49); CEUCA Rzvan 82,83), V(46,49); CHELARU Adrian: P(71, 72,80-82); V(46,47); CUR P(71,72,81), V(46,47); MOCANU Dan: P(71-73,80-83), V(46,47,49,51); N Adina-Ioana: P(71-73,80-83), V(46,47,49,50); OROIAN Bianca: P(71,73, 47,49). Clasa a VI-a. CADAR Alexandra: V(46-48,51-53), VI(51,55). X-a. TIMOFTE Diana: VIII(46,47), IX(46,47,49,50), X(46,47).

Premii acordate rezolvitorilor

ASOCIATIA "RECREATII MATEMATICE" n colaborare c revistei RECREATII MATEMATICE acord cte o diplom i un p a as cri pentru trei apariii la rubrica "Pagina rezolvitorilor" elevilor urmt a t t a

Liceul "M. Eminescu" AVRAM Mircea (cl. a IX-a): 1/2002 (7pb), 1/2003 (10pb), 2/2004 (5p

Scoala nr. 7 "N. Tonitza" SAVIN Rzvan (cl. a III-a): 1/2004 (5pb), 2/2004 (7pb), 1/2005 (6pb) a

Scoala nr. 22 "B. P. Hasdeu" ANDRONICIUC Ana-Miruna (cl. a III-a): 1/2004 (6pb), 2/2004 (5p (5pb); APOSTOL Ana-Maria (cl. a III-a): 1/2004 (6pb), 2/2004 (7pb), 1/20 BURUIANA Sebastian-Andrei (cl. a III-a): 1/2004 (6pb), 2/2004 (6p (5pb); CEOBANU Andrei-Nicolae (cl. a III-a): 1/2004 (5pb), 2/2004 (5pb), (5pb); COSTACHESCU Ivona (cl. a III-a): 1/2004 (5pb), 2/2004 (6pb), 1/2 GNDU Alexandra-Livia (cl. a III-a): 1/2004 (6pb), 2/2004 (6pb), 1 (5pb); GHERAN Ana-Maria (cl. a III-a): 1/2004 (5pb), 2/2004 (5pb), 1/200 HORBOVANU Bianca-Alexandra (cl. a III-a): 1/2004 (5pb), 2/2004 1/2005 (5pb); TURCU Andrei-Daniel (cl. a III-a): 1/2004 (6pb), 2/2004 (5pb), 1/20 UNGUREANU Teofana (cl. a III-a): 1/2004 (6pb), 2/2004 (5pb), 1/2

Scoala nr. 26 "G. Cobuc" s MANOLIU Mdlina (cl. a IV-a): 1/2004 (5pb), 2/2004 (5pb), 1/2005 a a

Revista semestrial RECREAII MATEMATICE este edita ASOCIAIA RECREAII MATEMATICE. Apare la datele de 1 i 1 septembrie i se adreseaz elevilor, profesorilor, studenilor i tuturo pasionai de matematica elementar. n atenia tuturor colaboratorilor Materialele trimise redaciei spre publicare (note i articole, chesti metodic, probleme propuse etc.) trebuie prezentate ngrijit, clar i conc trebuie s prezinte interes pentru un cerc ct mai larg de cititori. Se reco ca textele s nu depeasc patru pagini. Evident, ele trebuie s fie ori i s nu fi aprut sau s fi fost trimise spre publicare altor reviste. Problemele destinate rubricilor: Probleme propuse i Probleme p pregtirea concursurilor vor fi redactate pe foi separate cu en demonstraie/rezolvare (cte una pe fiecare foaie) i vor fi nsoite de n autorului, coala i localitatea unde lucreaz/nva. Redacia va decide asupra oportunitii publicrii materialelor prim n atenia elevilor Numele elevilor ce vor trimite redaciei soluii corecte la probleme rubricile de Probleme propuse i Probleme pentru pregatirea concurs vor fi menionate n Pagina rezolvitorilor. Se va ine seama de regulile: 1. Pot trimite soluii la minimum cinci probleme propuse n nu prezent i cel anterior al revistei; pe o foaie va fi redactat solui singure probleme. 2. Elevii din clasele VI-XII au dreptul s trimit soluii la prob propuse pentru clasa lor, pentru orice clas mai mare, din dou clase mai m imediat anterioare. Elevii din clasa a V-a pot trimite soluii la prob propuse pentru clasele a IV-a, a V-a i orice clas mai mare, iar elevii c I-IV pot trimite soluii la problemele propuse pentru oricare din clasele p i orice clas mai mare. Orice elev poate trimite soluii la problem concurs (de tip G i L). 3. Vor fi menionate urmtoarele date personale: numele i prenu clasa, coala i localitatea. 4. Plicul cu probleme rezolvate se va trimite prin pot (sau va f direct) la adresa Redaciei: Prof. dr. Temistocle Brsan Str. Aurora, nr. 3, sc. D, ap. 6, 700 474, Iai Jud. IAI E-mail: tbi@math.tuiasi.ro

CUPRINS
ARTICOLE I NOTE

200 de ani de la naterea lui DIRICHLET........................................................

t. FRUNZ, I. FRUNZ Fractali (II)................................................................. D. MIHALACHE, M. TETIVA Asupra monotoniei unor iruri .......................... G. DOSPINESCU O teorem uitat - inegalitatea lui Surnyi............................ T. ZVONARU, B. IONI Rapoarte determinate de o cevian i o secant ntr-un triunghi............................................... Gh. MOLEA Asupra unor ecuaii diofantice ptratice........................................ Fl. POPOVICI O caracterizare a funciilor convexe cu ajutorul derivatelor pariale ................................................................

NOTA ELEVULUI

A. ZAHARIUC Asupra problemei G67................................................................ I. MUSTA Matematic i algoritmi ................................................................. A. NEGRESCU Asupra unei probleme de concurs ............................................

CHESTIUNI METODICE

C. APOSTOL Din nou asupra unei probleme de concurs .................................. P. GEORGESCU, G. POPA Dou funcii cu aceeai derivat pe un interval nu difer neaprat printr-o constant.......

CORESPONDENE

V. GUU Probleme selectate de la Olimpiadele de Matematic ale Republicii Moldova....................................................... H. STEPHAN Probleme pentru clasa a VIII-a ....................................................

CONCURSURI I EXAMENE

Concursul Recreaii Matematice ed. a IV-a, Muncel - Iai, 2004...................... Concursul Octav Onicescu, ed. a VIII-a, Botoani, 2004 ..................................

PROBLEME I SOLUII

Soluiile problemelor propuse n nr. 1/2004............................................................. Soluiile problemelor pentru pregtirea concursurilor din nr. 1/2004 .................... Probleme propuse........................................................................................................ Probleme pentru pregtirea concursurilor ................................................................. Training problems for mathematical contests ..........................................................

Pagina rezolvitorilor .................................................................................................

Anul VIII, Nr. 1

Ianuarie Iuni

RECREAII MATEMATIC

REVIST DE MATEMATIC PENTRU ELEVI I PROF

e i = 1

Editura Recreaii Matematic IAI - 2006

Semnificaia formulei de pe copert: i ntr-o form concis, formula e = 1 leag cele patru ramuri fun
ale matematicii: ARITMETICA GEOMETRIA ALGEBRA ANALIZA MATEMATIC reprezentat reprezentat reprezentat reprezentat de de de de 1 i e

Redacia revistei :

Petru ASAFTEI, Dumitru BTINEU-GIURGIU (Bucureti), Temistocle BRS BRNZEI, Ctlin - Cristian BUDEANU, Constantin CHIRIL, Eugenia COHA CORDUNEANU, Mihai CRCIUN (Pacani), Gabriel DOSPINESCU (stude Marius FARCA, Paraschiva GALIA, Paul GEORGESCU, Mihai HAIVAS, IUREA, Lucian Georges LDUNC, Mircea LUPAN, Dan tefan MA (Hunedoara), Gabriel MRANU, Andrei NEDELCU, Gabriel POPA, Dan (Suceava), Florin POPOVICI (Braov), Maria RACU, Ioan SCLEANU (Hr ERDEAN (Ortie), Dan TIBA (Bucureti), Adrian ZAHARIUC (Bacu) ZANOSCHI.

Adresa redaciei: Catedra de Matematic Universitatea Tehnic Gh. Asachi Iai Bd. Carol I, nr.11, 700506, Iai Tel. 032 213737 / int. 123 E-mail: recreatii.matematice@gmail.com http://www.recreatiimatematice.uv.ro

COPYRIGHT 2006, ASOCIAIA RECREAII MATEMATICE Toate drepturile aparin Asociaiei Recraii Matematice. Reproducerea in parial a textului sau a ilustraiilor din aceast revist este posibil numai cu acordu scris al acesteia. TIPRIT LA SL&F IMPEX IAI Bd. Carol I, nr. 3-5 Tel. 0788 498933 E-mail: simonaslf@yahoo.com

Anul VIII, Nr. 1

Ianuarie Iunie

RECREAII MATEMATICE

REVIST DE MATEMATIC PENTRU ELEVI I PROF

e i = 1
Revist cu apariie semestrial publicat de

ASOCIAIA RECREAII MATEMATIC

IAI - 2006

Elogiu adus revistei "Gazeta Matematic a la 110 ani de apariie nentrerupt t a

Cum podul de la Cernavod i ntinde braele peste apele Dunrii, aa a s t a s Matematic i ntinde existena, care are nceputul n secolul al XIX a s t ntreg secolul al XX-lea i continu s-i aduc aportul la dezvoltarea nv s a as a at i tiinelor matematice din ara noastr i n acest nou secol, al XXI-lea. s s t t as Gazeta Matematic, prin cele 110 tomuri durate n timp unul dup a a piramida Keops a publicisticii periodice romneti, punctul de maxim s acesteia. Este a doua publicaie de matematic din lume ce se adreseaz t a a i prima de acest fel n privina apariiei nentrerupte i longevitii. s t t s at A avut o existena zbuciumat i cu multe momente dramatice, existe t as legat de soarta nvamntului din colile romneti - cel matematic mai a a at s s de cea a poporului romn. A trecut prin dou rzboaie mondiale, reforme n a a ale nvamntului public, regimuri adverse poporului romn etc. S am at a un singur episod din existena Gazetei. n primul rzboi mondial, n urm t a capitalei, Iaul devine centrul politic i administrativ al arii; tot aici se r s s t Gazeta Matematic, care, datorit devotamentului i strdaniilor lui Traia a a s a Vasile Teodoreanu i altor membri din redacie, continu s fie tiprit i s t a a a as ajungnd chiar i n minele abonailor aflai n primele linii ale frontului s t t Sub deviza Entuziasm, armonie, munc dezinteresat , sacricii contin a a depite toate greutile materiale i vicisitudinile vremurilor; oameni minu as at s n jurul i ptruni de spiritul Gazetei Matematice, au asigurat prin mu s a s bia lor mersul ei nainte. n prima jumtate de veac, la crma destinel at a au fost "cei 4 stlpi ai Gazetei Matematice": Ion Ionescu, Gheorghe Tiei t Ioachimescu i Vasile Cristescu. Merit elogiat, de asemenea, contribuia s a a t Teodorescu, care a condus cu mult competena, n ultimul sfert al secolu a t activitile la Gazet i cele legate de ea, devenite ntre timp mult mai d at as Tinerii talentai din generaii succesive au "frecventat" coala Gazetei t t s tice, unde i-au format deprinderile i tehnicile de lucru, i-au lefuit rai s s s s t matematic sau i-au vzut publicate primele ncercri originale n paginile s a a Nume de viitori ilutri matematicieni romni se gsesc menionate s a t Gazetei Matematice printre rezolvitorii i propuntorii de probleme, s a unor concursuri sau ca autori de note originale interesante, iar unii dintre e du-i responsabiliti redacionale. Lista lor fiind prea lung, amintim doar s at t a me: Gh. Tieica, C. Popovici, T. Lalescu, N. Abramescu, O. Mayer, Al. P t Barbilian, Fl. Vasilescu, N. Cior nescu, T. Popoviciu, Gr. Moisil, N. Teodo a n aceti 110 ani de existena, Gazeta Matematic a devenit o caria s t a turii romneti, simbol al permanenei i continuitii, component a nv s t s at a at matematic din ara noastr, pepinier de talente matematice. t a a Ca o recunoatere a meritelor sale, Preedintele Romniei, prin decretu s s octombrie 2005, confer Gazetei Matematice Ordinul "Meritul Cultural a de Ofier, categoria H "cercetare stiinic ", pentru contribuia deoseb t t a t movarea nvamntului i cercetrii aprofundate a tiinelor matematice. at s a s t

Prof. dr. Temistocle BRS

100 de ani de la naterea matematicianulu s Grigore C. Moisil

Multe capitole ale matematicii mi-a dragi. Matematica e una (Gr. C

n Istoria matematicii n Romnia St. Andonie l prezint pe Grigore C. Mo a fericit ntruchipare a dominantelor matema a tre: dinamism, varietate, tendina spre unive t A fost unul dintre cei mai mari i talenta s t ticieni romni i, indiscutabil, cel mai pro s cnd cu succes aproape toate domeniile m pure i aplicate s-a dovedit un creator n co s noire. Jovial i optimist, cu un umor plin d s un povestitor fermector. a Opera lui Grigore C. Moisil nu este do individual a unui om deosebit de nzestr a bazeaz pe tradiia "dinastiei" Moisiletilo a t s jinul permanent al familiei care a stimulat inteligena sa sclipitoare. S-a mndrit ntot t se trage dintr-o familie de grniceri nsudeni. Familia sa, originar din M a aa a a descins n comuna San, n imediata vecintate a Nsudului. Strbunicu t a aa a ticianului, care purta numele de Grigore, a fost primul crturar care s-a a comuna San. A fost preot, profesor i primul director al celui de al pat t s romnesc ninat n Austria, la Nsud. Fiul acestuia, Constantin, a ob t aa de doctor n tiine lologice la Universitatea din Viena i a funcionat 32 s t s t profesor la Nsud. Unul dintre fii acestuia, numit tot Constantin, tat aa a ticianului, a urmat, cu sprijinul lui Al. Odobescu, coala Normal Supe s a Bucureti; a funcionat ca profesor la Focani, Tulcea i Bucureti. Ulterior s t s s s profesoratul consacrndu-se arheologiei i numismaticii, devenind un repu s list n acest domeniu i membru al Academiei Romne. La Tulcea s-a c s a institutoarea Elena Nicolescu, care a devenit ulterior directoarea Scolii Vcrescu" din Bucureti. La Tulcea s-au nscut primii trei copii ai famili a a s a (10 ianuarie 1906), Florica (cercettoare la Biblioteca Academiei; cstorit a aa Emil Condurache) (1909) i Ioan (1910); ultimul copil, Gheorghe, s-a nsc s a la Vaslui (n timpul refugiului). Ambii frai au fost ingineri, profesori un t Dotat cu o inteligena vie i un umor sntos, mama, Elena, a avut un a t s a a n formarea lui Grigore, care i-a urmat cu snenie sfaturile, n ntreaga sa t la ea a motenit deviza: "Nu crede tot ce ti se spune, judec tu singur". s a Grigore C. Moisil a urmat coala primar n Bucureti i liceul la "Mih s a s s niceanu" din Vaslui (1916 1918) i "Spiru Haret" din Bucureti (1918 s s nscris apoi la secia de matematic de la Facultatea de Stiine a Univer t a t Bucureti, unde a a vut ca profesori pe D. Pompeiu, Gh. Titeica, A. s Tr. Lalescu. Primul i-a fost mentor nu numai n matematic, ci i n anum a s de viaa. ntr-un articol ("Viaa studeneasc", nr. 11, 1967) mrturise t t t a a

marea generaiei matematice din care fac parte coincide cu nceputurile m t abstracte romneti. Generaia mea a p sit cu dreptul. Ea a protat de f s t a fi avut ca profesori oameni de stiina si ai c ror profesori si ei oameni d t a Ca student a participat i la cursuri de istorie (N. Iorga), lozoe, socio s hail Dragomirescu), istoria artelor. A urmat n paralel i secia de constr s t Institutul Politehnic Bucureti la care a renunat (n anul al IV-lea) cnd s t doctoratul i a plecat n strintate. s a a A luat doctoratul la 4 iunie 1929 n faa unei comisii prezidat de Gh. Ti t a care fceau parte Dimitrie Pompeiu i Anton Davidoglu. n teza de doctorat a s Mecanica analitic a sistemelor continue a studiat analitic mecanica sist a un numr infinit de grade de libertate folosind metoda funcional (no a t a funcional fusese introdus cu puin timp n urm de Vito Volterra). n 19 t a a t a cu o burs a ministerului la Paris, unde ia contact cu Jaques Hadamard, a Henri Villat, Paul Montel si Elie Cartan, care apreciaz elogios contribuiil a t din teza de doctorat. La 1 iulie 1931 i trece docena n specialitatea analiz matematic, s t a a sitatea din Bucureti. Se ntoarce la Paris unde urmeaz cursul lui Vito s a n toamna anului 1932 se stabilete la Iai ind numit confereniar la Un s s t "Al. I. Cuza". Matematicianul Ion Creang , fost profesor i rector al Univ a s amintete: n acel timp eram student n anul al III-lea al seciei de mate s t la Universitatea din Iai; n curnd am aflat c la secia noastr a ncep s a t a de factur modern predat de un tn r matematician, deja cu renume form a a a revoluioneaz concepia noastr despre algebr . Am fost atras de acest cu t a t a a ceput s -l audiez si n curnd am fost furat de nout tile att de atractive c a a leciile cursului. Prelegerile lui Moisil ne-au deschis porile spre fermec to t t a a structurilor algebrice, a laticelor, a mpletirii strnse dintre procesele d abstractizarea teoriei mulimilor. Perioada de 10 ani petrecui n Iai a fo t t s importana pentru creaia sa tiinic i pentru desvrirea personalit t t s t as a s a vrst de 26 de ani, a gsit la Iai o atmosfer de nalt cultur. Peste ani a a s a a a s La Iai era o extraordinar densitate de oameni detepi pe metrul p trat. A s a s t a matematicieni de mare valoare tiinic i spiritual i a rmas toat viaa s t as as a a t cei care l primiser cu simpatie la sosirea n Iai: Alexandru i Vera Myll a s s Sanielovici, Octav Mayer, Mendel Haimovici, Ilie Popa, Adolf Haimovici i s su din acea perioad Ion Creang . n Biblioteca Seminarului Matematic a a a gsit crile care aveau s fac din el un matematician modern. Proaspt a at a a a ca profesor n 1935, n introducerea primului curs de algebr abstract mod a a n Romnia, afirm: La Iai am citit multe c ri de algebr dar cartea a s a t a, a fost cea a lui B. L. Van der Warden "Moderne Algebra". Era acolo un de a concepe matematica si anume algebra, dar nu numai algebra; mate conceput nu ca o stiina a cantit tii, ci ca o stiina a structurii. Peste c a t a t ap rut alte dou c ri care evideniau acelai mod de a privi matematica a a a t t s gia" lui Kuratowski si cartea lui St. Banach asupra spaiilor care i poart n t a putea, cu aceste volume si punnd n fruntea lor "Teoria numerelor trans W. Sierpinski, organiza un curs de matematici n nelesul de studiu al st t nelegeam ncet, ncet c matematica se schimbase. Se schimb . Se va sc t a a

n Iai s-a simit n largul su, a legat numeroase prietenii, participnd s t a rana specic tinereii la viaa acestui ora pentru care a pstrat permane t a t t s a une nedesminit. S-au creat legende n legtur cu viaa boem a tn t a a a t a a fat" al Iaului. La restaurantul de lng vechea cldire a Academiei Mi at s a a pstrat ntr-un col discret, pn la demolarea localului, o mas cunoscu a t a a mele de "masa lui Moisil". Se spune c la restaurantul "Corso" din centru a i plcea s asculte orchestra interpretnd un vals a crui melodie i ve a a a s compuse chiar de Moisil. Tot la Iai s-a petrecut un eveniment care i-a marcat ntreaga viaa s t O cunoate pe Viorica Constante cu care se va cstori. Viorica Mois s aa alturi n permanena, l-a sprijinit i stimulat, i-a asigurat calmul i confortu a t s s creaiei. Dup moartea savantului, pe baza scrisorilor i altor documente t a s i-a dedicat o carte minunat scris cu talent, dragoste i discreie "Un om a a s t altul. Grigore C. Moisil", aprut n 1979 n editura Albatros. a a n anul 1942 s-a creat la Facultatea de Matematic a Universitii din a at catedra de analiz superioar i logic la care este ncadrat Grigore C. Mo a as a perioada 1946-1948 cnd a fost ambasador al Romniei n Turcia, revine sitatea Bucureti unde a predat cursuri de elasticitate, algebr i maini s as s n 1948 devine mebru activ al Academiei Romne i eful seciei de alg s s t Institutul de Matematic al Academiei, nou ninat. n 1948 este ales pr a t Societii Romne de Matematic, post pe care l va ocupa toat viaa. at a a t Dup ce Grigore C. Moisil formeaz la Bucureti o veritabil coal de a a s as a solidelor deformabile, ncepnd din 1949 ia natere n jurul su Scoala de t s a bric a mecanismelor automate. Alturi de ruii V. I. Sestacov i M. Gavril a a s s ricanul Shannon este fondatorul acestei teorii, care are la baz utilizarea a Boole n studiul automatelor. n aceast direcie public dou tratate: T a t a a bric a mecanismelor ordonate i Teoria algebric a schemelor cu contact a s a ncepnd din 1955 cltorete foarte mult, fiind invitat la congrese, aa s comunicri, cursuri sau conferine. Devine membru al Academiei din Bo a t Institutului Internaional de Filozofie din Paris. t Grigore C. Moisil are lucrri importante n analiza funcional, mec a t a retic, geometrie diferenial i algebr. Partea cea mai original din cr a t a s a a constituie preocuprile de logic matematic (ncepute n perioada de la Ia a a a inut i primele cursuri de logic matematic din Romnia), care l-au cond t s a a sideraii lozoce asupra matematicii i la teoria algebric a mecanismelor t s a Aceste preocupri i-au asigurat un loc cu totul aparte n matematica rom a S-a stins din viaa la 21 mai 1973, la Ottawa, n Canada, n timpul un t care a pus jaloanele colaborrii ntre informaticienii candieni i cei romni a s de la dispariia sa, fostul su elev, Mircea Malia l caracteriza: Moisil a fos t a t dect un savant, a fost mai muli savani ntrunii n sesiune permanent s t t t a si locul unul altuia n cicluri succesive mari, reprezentate de temele fundam care le-a abordat. A fost pn n ultimele zile deschiz tor de drumuri, in a a aceast aventur spiritual nu a admis dilentatismul superficial. a a a

Prof. dr. Petru MIN

Asupra problemei 809 din Gazeta Matemat volumul VIII (19021903)


D. M. B ATINETU - GIURGIU 1

Cu ocazia anivers rii a 110 ani a nentrerupt a Gazetei Matemat a

n istoria matematicii din ara noastr Traian Lalescu reprezint t a a de diversitate rar, un mare animator al generaiei sale de matematicie a t dotat cu o mare putere de munc i inteligena scnteietoare, un profeso as t cu deosebit talent pedagogic. Traian Lalescu s-a nscut la 12/24 iulie 1882, n Bucureti. Studiile pr a s fcut la Bucureti, primele dou clase de gimnaziu la Craiova (1892-1894), a s a a III-a i a IV-a la Roman (1894-1896). Clasele a V-a i a VI-a le-a fcu s s a Internat din Iai (actualul Colegiu Naional "C. Negruzzi") n perioada 18 s t n liceu, ca i n gimnaziu, Lalescu a fost premiantul I al clasei i a prim s s de onoare al colii (Lalescu se afl trecut pe tabela de onoare a Liceului In s a Iai). s Chiar din clasa a VI-a a liceului (februarie 1898), Lalescu ajunge cores Gazeta Matematic. Profesorul su de mai trziu, inginerul Ion Ionescu a a despre Traian Lalescu c: "Intrarea lui n rndul corespondenilor "Gazet a t atice" nu s-a fcut ca de obicei, n mod timid, lent, progresiv, ci deodat a a maximum posibil. A fost un caz unic de apariiune la "Gazeta Matem t activitate prodigioas a unui tnr licean!". a a n v. VIII (1902-1903), la pagina 244, Traian Lalescu a propus Problem urmtorul enun: a t 1 ch d2n+1 1 x . 2n S se arate c : a a x sh = 2n+2 dx2n+1 x x

La pag. 283 din Gazeta Matematic, v. IX (1903-1904), este publica a dat de Traian Lalescu acestei probleme, urmat de o not: a a a "Se tie c: s a sh x = x x3 x2n+1 + + + + 1! 3! (2n + 1)! sh i s ch x = 1 +

x2 x2n + + 2! (2n)

Vom avea deci:

i, prin urmare: s 1 1 1 1 x2n sh = P (x) + + + + x (2n + 1)!x (2n + 3)!x3 (2n + 2p + 1)!x2p+ P (x) fiind un polinom ntreg n x de gradul 2n.
1

1 1 1 1 + ... + = + 3 x x 3!x (2n + 1)!x2n+1

Profesor, Colegiul Na ional "Matei Basarab", Bucureti t s

Seria din membrul al II-lea, uniform convergent n tot planul exceptn a e derivabil termen cu termen i rezultatele gsite sunt serii convergente a s a ntindere, ale cror sume sunt date de derivatele de acelai ordin ale mem a s Observnd acum c: a d2n+1 d2n+1 1 (2p + 1) (2p + 2) (2p + (P (x)) = 0, = 2n+1 2n+1 2p+1 dx dx x x2n+2p+2 i c, prin urmare s a d2n+1 1 1 1 , = 2n+1 2p+1 2p x2n+2 dx (2p + 2n + 1)!x (2p)!x obinem t d2n+1 1 1 1 1 2n + + + = x sh = 2n+2 1 + dx2n+1 x x 2!x2 (2p)!x2p

Not. Aceast problem a fost rezolvat de D-nii: N. Abramescu, Gr a a a a G. Constantinescu, M. Radu, C. Gheorghiu i I. G. Niculescu. s n acelai mod se pot demonstra i formulele: s s 1 cos 1/x d2n+1 1 dn n1 1/x n+1 n e 2n x; e x sin x = (1) n+1 = (1) 2n+1 2n+2 n dx x x dx x

Ca un omagiu adus marelui matematician romn Traian Lalescu, vom probleme o nou soluie, accesibil elevilor actualului liceu. a t a 1 S considerm funciile fn : R R, fn (x) = x2n sh , unde n N a a t x a stat imediat c fn este indenit derivabil, oricare ar fi n N. Ne pro a a demonstrm c a a 1 1 (2n+1) (x) = 2n+2 ch , n N fn x x prin metoda induciei matematice, folosind formula lui Leibniz de deriv t dusului a dou funcii indenit derivabile, adic a t a n X n k (nk) (k) Cn u v , n N . (uv) =
0 Avem f0 (x) =

De asemenea avem: 1 1 1 0 f1 (x) = x2 sh , deci f1 (x) = 2x sh ch ; x x x 1 1 2 1 1 00 f1 (x) = 2 sh ch + 2 ch ; x x x x x 2 1 1 1 1 1 2 2 1 1 000 f1 (x) = 2 ch + 2 ch + 3 sh 4 ch = 4 ch , x x x x x x x x x x deci i pentru n = 1 formula (1) se verific. s a a Presupunem c formula (1) este adevrat pentru n N adic are loc a a a

sh

1 x

k=0

1 1 ch , deci pentru n = 0 formula (1) x2 x

1 1 ch , x2n+2 x i demonstrm c ea este adevrat i pentru n + 1, adic avem s a a a as a 1 1 (2n+3) fn+1 (x) = 2n+4 ch . x x S observm c a a a fn+1 (x) = x2 fn (x) , n N, i atunci, cu ajutorul formulei (2), avem: s
(2n+1) fn (x) =

fn+1

(2n+3)

i atunci s

Conform presupunerii relaia (3) fiind adevrat, rezult c: t a a a 0 a 0 1 1 2n + 2 1 1 1 (2n+2) (2n+1) fn (x) = fn (x) = 2n+2 ch = 2n+3 ch + 2n+4 sh x x x x x x

(2n+3) (2n+2) (2n+1) = x2 fn (x) + 2(2n + 3) xfn (x) + (2n + 3)(2n + 2) fn (x),

0 (2n+3) 1 (2n+2) 2 (x) x2 + C2n+3 fn (x) 2x + C2n+3 f (2n+1) (x) 2 = = C2n+3 fn

X (2n+3) 2n+3 k (k (2n+3k) (x) = x2 fn+1 (x) = C2n+3 fn (x) x2


k=0

0 0 2n + 2 1 1 1 (2n+3) (2n+2) (x) = fn (x) = ch + 2n+4 sh = fn x2n+3 x x x 1 2n + 2 1 2n + 4 1 (2n + 2) (2n + 3) 1 ch 2n+5 sh 2n+5 sh 2n = x2n+4 x x x x x x 1 4n + 6 1 (2n + 2) (2n + 3) x2 + 1 ch 2n+5 sh . = 2n+6 x x x x Dac inem seama de relaiile (3), (7) i (8), relaia (6) devine at t s t (2n+2)(2n+3) x2 + 1 1 4n+6 1 2(2n+2)(2n+ (2n+3) ch 2n+5 sh + fn+1 (x) = 2n+4 x x x x x2n+2 1 (2n + 2) (2n + 3) 1 2 (2n + 3) sh ch = + x2n+3 x x2n+2 x 1 1 = 2n+5 (2n+2)(2n+3) x2 + 1 ch + 2x2 (2n+2) (2n+3 x x 1 1 1 (2n + 2) (2n + 3) x2 ch = 2n+4 ch , x x x ceea ce demonstreaz c relaia (4) este adevrat. a a t a a Conform principiului induciei matematice, rezult c t a a 1 1 (2n+1) fn (x) = 2n+2 ch , n N. x x Bibliografie

1. G. St. Andonie - Istoria matematicii n Romnia, v. 1, Ed. St., Buc., 1 2. M. D. Btineu-Giurgiu, M. Btineu-Giurgiu, I. Brchi-Damian, a t a t nescu - Analiz matematic . Probleme pentru clasa a XI-a, Ed. Matrix R a a 2003. 3. Colecia "Gazeta Matematic ", 1895-2005. t a

Cteva proprieti ale subgrupurilor finit at din GLn (Z)


Gabriel DOSPINESCU 1

Cu ocazia anivers rii a 110 ani a nentrerupt a Gazetei Matemat a

1. Introducere: lema lui Serre. Ceea ce vei citi n continuare es t care timid de a expune o colecie de rezultate referitoare la subgrupurile a t GLn (Z). Se prea poate ca demonstraiile care urmeaz s fie cunoscute; a t a a gsit "aproape" singur i crede c merit s fie prezentate. Articole (ma a s a a a despre proprietile acestor subgrupuri s-au scris multe i, cu sigurana, at s t scrie, cci problemele referitoare la ele sunt dicile i multe dintre ele i a a s s ani buni rezolvrile. l invitm pe cititorul interesat de rezultate mai p a a citeasc articolele din bibliograe, mult mai tehnice i mai specializate. Se a s [3] ar fi o descriere superb a acelorai (sau chiar a mai multor) rezultate a s pcate, nu am avut acces la acest articol, aa c nu putem dect s-l rec a s a a "orbete" cititorilor interesai de asemenea aspecte. s t Iat, mai nti, ce rezultate vom demonstra (sau doar aminti). Vom ded a simpl a teoremei Jordan-Zassenhaus (cu ajutorul lemei lui Serre, de car a cunotina din [7]) relativ la finitudinea claselor de izomorfism ale subgrupu s t ale lui GLn (Z), apoi vom demonstra c orice subgrup finit din GLn (Z) ar a (2n)! elemente i c exist 9 clase de izomorfism pentru subgrupurile lui G s a a Vom ncepe cu lema lui Serre, un rezultat de o frumusee deosebit, ca t a o prim majorare a ordinului subgrupurilor finite din GLn (Z); utilitatea a a permite s o numim "teorem". Toate grupurile despre care va fi vorba n a a au cel puin dou elemente. t a Teorema 1 (Lema lui Serre). Fie G GLn (Z) un grup finit si p > 2 prim. Consider m aplicaia : GLn (Z) GLn (Zp ) care asociaz fiec r a t a a A matricea claselor de resturi modulo p ale elementelor din A. Atunci acestei aplicaii la G este injectiv . t a Demonstraie. Desigur, este bine definit i este un morsm ntre t as GLn (Z) i GLn (Zp ) (aa cum se veric imediat). S presupunem c rest s s a a a caiei la G nu este injectiv, deci exist A G, A 6= In astfel nct (A t a a Asta nseamn c putem scrie A = In + pB, unde B Mn (Z). Fie 1 , 2 , . a a orile proprii ale matricii B; se tie atunci c A are valorile proprii 1 + pi , s a Acum s privim cu atenie sumele Sk = k + k + + k (pentru k num a t a 1 2 n toate vor fi numere ntregi (cel mai simplu argument este teorema funda polinoamelor simetrice, cci toate aceste sume sunt polinoame cu coecie a n sumele simetrice fundamentale ale numerelor 1 , 2 , . . . , n , iar aceste su rice sunt - modulo un semn plus sau minus - coecienii polinomului cara t matricii B Mn (Z), deci ntregi). ns, G ind finit, putem scrie A|G| a trebuie s avem (1 + pi )n = 1, pentru fiecare 1 i n, iar de aici obine a t c |i | < 1, 1 i n. Or, aceasta nseamn c irul de numere ntregi (Sk a a as
1

Student, cole Normale Suprieure, Paris

la zero, deci trebuie ca toi termenii si s fie nuli (de la un rang ncolo). t a a aplicare a formulelor lui Newton ne va duce la concluzia c e necesar, pent a toi i s fie egali cu 0; dar atunci toate valorile proprii ale matricii A sunt t a deci (teorema Cayley-Hamilton) ea este "rdcin" a polinomului (X 1)n a a a vzut, mai este rdcin i pentru X |G| 1, deci va fi rdcin pentru cel a a a as a a a divizor comun al acestor polinoame, care este X 1: adic A = In (alt ar a fi c identitatea este singura matrice unipotent diagonalizabil, iar matr a a a aceste dou proprieti: este unipotent - cci tocmai am artat c toate v a at a a a a proprii sunt egale cu 1 - i diagonalizabil, deoarece polinomul su minim s a a dect rdcini simple, ind un divizor al lui X |G| 1) i teorema 1 este dem a a s S examinm puin consecinele acestei teoreme; obinem imediat c ( a a t t t a nea lui G prin morsmul ) este un subgrup cu |G| elemente din GLn ( GLn (Zp ) are exact (pn 1)(pn p) (pn pn1 ) elemente (lsm cit aa exerciiu demonstraia acestui rezultat clasic). Rezult atunci, din teore t t a grange, c |G| divide pe (pn 1)(pn p) (pn pn1 ), pentru orice sub a G GLn (Z) i orice p > 2 prim. n particular, exist un numr finit s a a posibile ale matricilor din GLn (Z) (participanii la olimpiade - i nu num t s buie s-i fi amintit celebra problem: orice matrice din GL2 (Z) are or as a 3, 4, sau 6; ncercai s demonstrai aceasta pentru n = 3!; mai mult, c t a t pot gndi la o variant mult mai general: mulimile ordinelor posibile ale a a t din GL2k (Z) i GL2k+1 (Z) coincid, pentru orice k 1 natural). De asem s rezult (tot ca un caz particular) c ordinul oricrei matrici din GLn (Z) a a a (3n 1)(3n 3) (3n 3n1 ) (aceast problem a fost propus de aut a a a Mat, pe vremea cnd nu cunotea lema lui Serre; de altfel, am sit s de s reu a c ordinul oricrei matrici din GLn (Z) este mai mic dect A n ln n , und a a constant pozitiv ce nu depinde de n, dar nu despre asta ne-am propus a a aici). Tot din lema lui Serre mai putem deduce i varianta simpl a te s a Jordan-Zassenhaus, cci am obinut c orice subgrup finit al lui GLn (Z) a a t a (3n 1)(3n 3) (3n 3n1 ) elemente, deci, cu sigurana, exist un n t a de clase de izomorsm n GLn (Z). Desigur, de aici i pn la demonstrare s a lui Jordan-Zassenhaus (care afirm nitudinea numrului claselor de con a a subgrupurilor nite ale lui GLn (Z)) mai e mult de munc, i, oricum, nu a s asta aici; recomandm excelentul articol [7]. a

2. Majorri pentru ordinele subgrupurilor finite ale lui GLn (Z a c ne apropiem de un punct sensibil al acestei note, anume de obinerea une a t bune pentru ordinul oricrui subgrup finit din GLn (Z); am obinut deja a t mare divizor comun al numerelor (pn 1)(pn p) (pn pn1 ), p > 2, p prim este un astfel de majorant. Minkowski a demonstrat i un rezultat asemn s a a 2 p = 2, anume c ordinul oricrui subgrup finit din GLn (Z) divide pe 2n (2n a a 2) (2n 2n1 ). Din pcate aceast majorare este oricum, dar nu uo a a s departe de a fi cea mai bun. Vom ncerca s dm un rezultat mai "simplu" a a a c formula e mai simpl) care este, i el, departe de valoarea optimal con a a s a Teorema 2. Orice subgrup din GLn (Z) are cel mult (2n)! element

ordinul oric rui subgrup din GLn (Z) divide pe (2n)!. a Menionm c o majorare bun pentru ordinul maxim al subgrupurilor d t a a a este, dup cte tim noi, o problem deschis i foarte dicil. Cititorul va a s a as a o minorare aproape evident: exist subgrupuri cu 2n n! elemente (gn a a exemplu, la matricile ce au exact un 1 sau 1 pe fiecare linie i pe fiecar s n rest zerouri!). Cel mai bun rezultat obinut pn n prezent pare s fie o t a a de forma C n (n!)1+ , unde C este o constant care depinde de , nu i a s aceasta necesit un efort considerabil, pe care nu-l vom face aici. Invitm c a a gseasc mai multe detalii n [5], unde exist chiar i o meniune referitoar a a a s t c 2n n! este valoarea maxim a ordinului unui subgrup finit din GLn ( a a toi n 6 {2, 4, 6, 7, 8, 9, 10} (armaie atribuit acolo lui W. Feit). t t a S revenim acum la Teorema 2, a crei origine nu o tim - tim doar c a a a s s a [7] fr meniuni suplimentare i fr. . . demonstraie. Demonstraia (cel aa t s aa t t pe care am gsit-o noi) cere rbdare din partea cititorului, precum i nit a a s s ajuttoare, pe care le vom numi tot teoreme, datorit frumuseii i utilit a a t s a Teorema 3. Fie G GLn (Z) un subgrup finit. Atunci, pentru oric |G| este un divizor al num rului a X (tr(g))k .
gG

Demonstraie. nainte de toate, s spunem c nici mcar nu e nev t a a a supunem c elementele matricilor sunt numere complexe; acestea pot fi din a comutativ oarecare a crui caracteristic este numr prim cu |G|. Demon a a a nti armaia pentru k = 1. S considerm matricea t a a 1 X M= g |G|
gG

pentru care, clar, avem

deoarece, pentru fiecare g G, avem (G fiind grup) {gh | h G} = G. a a M 2 = M implic faptul c toate valorile proprii ale matricii M sunt 0 s tr(M ) (care este urma matricii M , deci suma valorilor proprii) este un num or, folosind proprietile urmei, avem at 1 X tr(M ) = tr(g), |G|
gG

1 XX gh = M, M2 = |G|2
gG hG

suma valorilor proprii ale matricii M - definit ca mai sus - este 0, deci toa a proprii sunt 0; atunci M este idempotent i nilpotent, deci este matrice as a Acelai argument nu funcioneaz ns pentru k 2 (din pcate); i s t a a a s clip de graie n algebra liniar a permis introducerea noiunii de produ a t a t a dou matrici. Astfel, dac A Mn (K) i B Mp (K), produsul lor ten a a s definit prin

deci demonstraia pentru k = 1 este ncheiat (totodat am rezolvat i o t a s P a mai veche de la concursul Putnam: dac a tr(g) = 0, G fiind un grup finit gG P P g = 0; ntr-adevr, egalitatea a tr(g) = 0 implic a ptratice, atunci a
gG gG

a11 B . . . a1n B . . .. . AB = . Mnp (K). . . . an1 B . . . ann B O proprietate fundamental a produsului tensorial (uor de verificat) este a s (A B) (C D) = (AC) (BD), A, C Mn (K),
0

aceast egalitate ne permite s definim un subgrup G GLn2 (Z) prin G0 a a g G} (relaia de mai sus, precum i faptul c det(A B) = (det A)p t s a pentru A, B ca mai sus, folosesc ca s artm c G0 este subgrup al lui a aa a Acest subgrup are, evident, tot |G| elemente, deci i putem aplica rezu demonstrat pentru a deduce c a X X tr(g g) = (tr(g))2 |G||
gG gG

B, D M

(dac mai folosim i formula foarte simpl tr(A B) = tr(A) tr(B)). C a s a neles acum modul n care va demonstra afirmaia pentru orice k N t t spune doar c pentru k = 3 trebuie considerat G00 = {(g g) g | g G} a

Acum putem ncepe s demonstrm Teorema 2. S notm x1 > x2 > a a a a elementele mulimii {tr(g) | g G} i s observm c avem q 2 i x1 = t s a a a s evidente (dar interesante i n sine) vom demonstra aceste proprieti. s at rnd, am vzut c, dac A G, atunci A|G| = In , deci valorile propri a a a sunt rdcini ale unitii, n particular ele au modulul 1. E clar atunc a a at | tr(A)| n, pentru orice A G; cum In G, se cheam c x1 = n. D a a observm, dac A G {In } (i existena unei asemenea matrici e as a a s t presupunerea fcut nc de la nceput), nu putem avea tr(A) = n, cci at a a a a valorile proprii ale matricii A ar fi egale cu 1, ceea ce este imposibil (citi uitat argumentul final din demonstraia teoremei 1); deci q 2. n plus, d t cu a1 , a2 , . . . , aq numrul apariiilor numerelor x1 , x2 , . . . , xq respectiv n a t urmelor matricilor din G, teorema 3 afirm c a a |G||a1 xk + a2 xk + + aq xk , 1 2 q k 1.

s Desigur, mai avem i |G| = a1 + a2 + + aq , precum i a1 = 1 (est s s fi neles argumentele din acest paragraf pentru a ne convinge i de a a t s precum i de faptul c, dac xq = n, atunci i aq = 1; toate aceste observ s a a s dovedi eseniale n studiul subgrupurilor finite ale lui GL2 (Z)). Iar avem t un rezultat ajuttor. a m | a1 xk + a2 xk + + aq xk , 1 2 q k N .

Teorema 4. Fie a1 , a2 , . . . , aq , x1 , x2 , . . . , xq si m numere ntregi astf Atunci avem si

i s observm c s a a a

m | a1 (x1 x2 ) (x1 xq ). Demonstraie. S considerm seria formal t a a a a1 a2 aq f (z) = + + + 1 x1 z 1 x2 z 1 xq z

f (z) =

deci, folosind ipoteza, rezult existena unor numere ntregi b0 , b1 , b2 , . . . a a t P f (z) = m bj z j . Pe de alt parte, putem scrie i a s j0 P a1 (1 x2 z) (1 xq z) f (z) = . (1 x1 z)(1 x2 z) (1 xq z) Asta ne arat c seria formal (de fapt, polinomul) de la numrtor poate a a a aa formaX X a1 (1 x2 z) (1 xq z) = m(1 x1 z)(1 x2 z) (1 xq z) deci are toi coeficienii divizibili cu m, de unde obinem c m | t t t a
i=1

q X i=1

ai +

q X
i=1

ai xi

z+

q X
i=1

ai x2 i

z2 + ,

este a t-a sum simetric fundamental n x1 , . . . , xi1 , xi+1 , . . . , xq , ceea a a a i s q q q X X X (i) (i) ai xq2 ai S1 + + (1)q1 ai Sq1 m|xq1 1 1 sau
i=1 i=1 i=1 (i)

q P

j0 (i) ai St ,

m|

Cum, pentru i > 1, avem (x1 x1 ) (x1 xi1 )(x1 xi+1 ) (x1 xq ) xq1 xq2 S1 + + (1)q1 Sq1 = 0, 1 1
(i) (i)

q X i=1

ai (xq1 xq2 S1 + + (1)q1 Sq1 ). 1 1

(i)

ne rmne doar c a a (1) (1) m | a1 (xq1 xq2 S1 + + (1)q1 Sq1 ) = a1 (x1 x2 ) (x1 1 1 ceea ce trebuia demonstrat. Iar asta ncheie i demonstraia teoremei 2: din teoremele 3 i 4 i faptul s t s s rezult c |G| divide (x1 x2 ) (x1 xq ), care este produsul a q 1 numer a a diferite i cel mult egale cu 2n (deoarece urma oricrei matrici din G este s a ntreg cuprins ntre n i n), deci divide i pe (2n)!. s s Bibliografie

1. G. P. Dresden - There are only nine finite groups of fractional linear with integer coeicients, Mathematics Magazine, June 2004, 211-218. 2. R. A. Horn, Ch. R. Johnson - Analiz matricial , Fundaia Theta, a a t 2001. 3. J. Kuzmanovich, A. Pavlichenkov - Finite groups of matrices whose e integers, American Mathematical Monthly, February 2002. 4. T. J. Laey - Lectures in integer matrices. 5. D. N. Rockmore, Ki-Seng Tan - A note on the order of finite su GLn (Z), Commutative Algebra, 2/1999. 6. Ken-Ichi Tahara - On the finite subgroups of GL3 (Z), Nagoya Math. Jo 7. Nicolas Tossel - Reseaux et thormes de nitude, Revue des mathmat ciales, 1-2/2005.

Ceviene i triunghiuri triomologice s


Temistocle BRSAN 1

Cu ocazia anivers rii a 110 ani a nentrerupt a Gazetei Matemat a

n aceast Not, pornind de la un triunghi oarecare, punem n evidena a a t raie de triunghiuri triomologice cu acelai centru de greutate ca i triungh t s s Dou triunghiuri, 4ABC i 4XY Z, se numesc omologice dac dreptele a s a CZ sunt concurente; punctul de concurena se numete centru de omolo t s unghiurilor. Triunghiurile date sunt triomologice dac admit trei centre de a

1. Fie ABC un triunghi oarecare i numerele , , R \ {1} cu s dreapta BC considerm punctele A , A , A determinate de rapoartele a

A B A B = i respectiv s = (utilizm segmentele orientate pentru c a A C A C A , A i A s poat fi situate n orice poziie pe BC, exceptnd vrfur s a a t ale 4ABC). Punctele B , B , B CA i C , C , C AB se determi s similar. Condiia = 1 asigur existena punctelor X , Y etc. defin t a t {X } = AA BB CC , {X } = AA BB CC , {X } = AA BB CC , {Y } = AA CC BB , {Y } = AA CC BB , {Y } = AA CC BB . Att pe figur ct i schematic din a s A B C A B C X ( ) ( ) Y X ( ) ( ) Y X ( ) ( ) Y s se poate urmri formarea acestor puncte i a triunghiurilor X X X i Y a s

C Y

B X G

1 3 1 = 2 = 6 =

Y X

C B
1

X A

Y A

Prof. dr., Catedra de matematic, Univ. Tehnic "Gh. Asachi", Iai a a s

Propoziia 1. Triunghiurile X X X si Y Y Y sunt triomologice, c t de omologie fiind vrfurile triunghiului ABC. Demonstraie. Vom arta urmtoarele: t a a (i) 4X X X i 4Y Y Y sunt omologice cu centrul A; s (ii) 4X X X i 4Y Y Y sunt omologice cu centrul C; s (iii) 4X X X i 4Y Y Y sunt omologice cu centrul B. s Aceste trei afirmaii decurg din (1). Astfel, afirmaia (1) revine la a t t dreptele X Y , X Y i X Y sunt concurente n A. Cum din prima s egalitate din (1) rezult c X , Y AA , vom avea c A X Y . La f a a a relaiile A X Y i A X Y . Aadar (i) este adevrat. Pe acee t s s a a dovedesc (ii) i (iii). Q.e.d. s

Se observ c 4ABC i 4X X X sunt invers orientate, pe cnd a a s 4Y Y Y sunt la fel orientate.

Observaie. n consecina, conguraia conine i perechile de triun t t t t s omologice: 4ABC i 4X X X , 4ABC i 4Y Y Y ; pentru prima pere s s 4ABC, 4X X X ; Y , 4ABC, 4X X X ; Y , 4ABC, 4X X X ; Y , iar pentru a doua avem: 4ABC, 4Y Y Y ; X , 4ABC, 4Y Y Y ; X , 4ABC, 4Y Y Y ; X , (pe un rnd sunt scrise dou triunghiuri, pe baza schemei (2), i cent a s omologie).

2. n aceast seciune vom stabili o alt proprietate a conguraiei: ce a t a t unghiuri au acelai centru de greutate. Pentru aceasta, vom utiliza metoda s Avem nevoie de urmtoarea a

Lem. Fie ABC un triunghi oarecare si punctele A0 BC, B 0 C a A0 B B0C = 0 , = 0 si + 1 6= 1, atunci cevienele AA0 si BB 0 au u AC BA intersecie X si avem t 1 = r r ( A + B C ) . r rX + 1 a t t ( X noteaz vectorul de poziie al punctului X faa de o origine arbitrar r Demonstraie. Cu teorema lui Thales se arat uor c + t a s a AA0 k BB 0 . A0 B B0C A s a a Din = 0 i = 0 urmeaz c AC BA B 0= 1 , rA rB rC X 1 1 1 . 0= rC rA rB 1 1

Tinnd cont de aceste relaii, ecuaiile vectoriale ale cevienelor: (AA0 ) t t r 0 ), (BB 0 ) = + v ( 0 ) se scriu sub forma u( r A rA r rB rB rB = (1 u) + u u , (AA0 ) r rA rB rC 1 1 = (1 v) + v v . rC rA r rB (BB 0 ) 1 1 Vectorul X asociat punctului X de intersecie se obine din (4) sau (5 r t t sau v luat dintr-o soluie (u, v) a sistemului liniar de ecuaii t t v u u v 1u= , = 1 v, = . 1 1 1 1 Gsim, cu uurina, ca soluie a sistemului (6) perechea (u, v) cu a s t t 1 u= , v= . + 1 + 1 Dup nlocuirea lui u sau v din (7) n (4) sau (5), obinem pentru X rep a t r (3), q.e.d.

Tinnd seama de (9), (10) i (11), avem s 1 + r X X + X = r r ( + 1 ) A r + 1 + (1 + ) B + ( + + 1) r r + + . = rA rB rC adic are loc (8), q.e.d. a

s Propoziia 2. Triunghiurile ABC, X X X si Y Y Y au acelai t greutate. Demonstraie. Vom arta c 4X X X i 4ABC au acelai centru d t a a s s (la fel se procedeaz cu perechea format din 4Y Y Y i 4ABC). Este s a a s stabilim c a + r X X + X = A + B + C . r r r r r ntr-adevr, utiliznd Lema relativ la 4ABC i cevienele AA i BB , o a s s 1 = r X r r ( A + B C ) ; r + 1 similar obinem i relaiile: t s t 1 = r X r r (4CAB i CC , AA s ( A B + C ) r + 1 1 = r X r r (4BCA i BB , CC s ( A + B + C ) r + 1

3. S presupunem c triunghiul ABC este echilateral. Se constat u a a a s elementar i ca o consecina a relaiilor (1), c triunghiurile X X X a s t t a sunt, la rndul lor, echilaterale. Conform Propoziiei 2, aceste triunghiuri t centru ca i triunghiul ABC. Este evident, n acest caz particular, nrud s a rezultat remarcabil, teorema lui Barbilian: dou triunghiuri echilaterale a centru sunt triomologice.

Construcii aproximative cu rigla i compasu t s numrului a


Alexandru MOSCALIUC 1

Notaia pentru raportul dintre lungimea unui cerc i diametrul su s-a t s a n matematic datorit lui L. Euler, care a utilizat-o n tratatul su Intr a a a analysis innitorum (1748). Valori aproximative ale lui au fost utilizat antichitatea timpurie de multe popoare. Amintim doar c Arhimede, a 10 1 Asupra m sur rii cercului, a gsit c 3 a a a a < < 3 prin aa-numita acum s 71 7 perimetrelor (cea cu poligoanele regulate nscrise i circumscrise). s n strns legtur cu identitatea numrului este problema cuadratur a a a a construcia cu rigla i compasul a unui ptrat de arie egal cu aria unu t s a a problema revine la rectificarea cercului construcia cu aceleai instrume t s segment de lungime egal cu lungimea unui cerc dat ce se reduce la a construcia cu rigla i compasul a unui segment de lungime . t s Aceast problem celebr formulat de grecii antici i-a gsit rezolvar a a a a s a 1882, cnd F. Lindemann a dovedit c este transcendent (adic n a a algebric). Graie acestui rezultat i faptului c numerele ce se pot construi t s a compasul formeaz o parte a mulimii numerelor algebrice, rezult c este a t a a cuadratura cercului. Putem aproxima, ns, numrul cu numere constructibile cu rigla i a a s Scopul acestei lucrri este de a da o astfel de aproximare a lui i ctev a s ilustrative, ntr-o prezentare accesibil elevilor de cl. a IX-a. a Propoziie. Are loc urm toarea inegalitate: t a 2 + 3 0, 01 < < 2 + 3, i.e. 2 + 3 aproximeaz num a arul prin adaos cu o eroare mai mic de a Soluie. Fie l, L lungimile laturilor poligoanelor regut A l late cu n laturi nscris i respectiv circumscris unui cerc s de raz egal cu 1. ntre perimetrele acestor poligoane i a a s lungimea cercului avem relaia t 1 nl < 2 < nL. (2) O 360 180 0 OB 0 ) = \ \ Deoarece m(AOB) = m(A i l = 2 sin s , n n 180 1 L = 2 tg , relaia (2) se scrie t n 180 180 B L n sin < < n tg . (3) n n Lund n (3) n = 60, obinem t 60 sin 3 < < 60 tg 3 . Tinnd seama c 3 = 18 15 , vom avea a tg 18 tg 15 sin 3 = sin 18 cos 15 sin 15 cos 18 i tg 3 = s 1 + tg 18 tg 1
1

Profesor, S coala general nr. 6, Botoani a s

r q 1 1 2 sin 18 = 5 1 , cos 18 = 10 + 2 5, tg 18 = 1 4 4 5 6+ 2 1 6 2 , cos 15 = sin 15 = , tg 15 = 2 3 4 4 q inegalitile (4) se scriu: at q 1 2 5 60 5 q 51 6 + 2 10 + 2 5 6 2 < < 60 16 1+ 1 2 5 Cum
5

Printr-un calcul de rutin anevoios i neplcut se verific faptul c mem a s a a a din (6) este mai mare ca 2 + 3 0, 01, pe cnd cel drept este mai mic ca n concluzie, inegalitile (1) sunt adevrate. at a Observaie. Construcia cu rigla i compasul a unui segment de lungim t t s (n prezena unui segment unitate) este elementar. Ca urmare, Propozi t a modalitate de a construi aproximativ numrul cu rigla i compasul. a s n aplicaiile urmtoare ale Propoziiei se face cuadratura/rectificarea un t a t rigla si compasul n mod aproximativ, adic se construiete cu aceste in a s un ptrat/segment avnd aria/lungimea aproximativ aria/lungimea cercu a Aplicaia 1. Fie ABC un triunghi isoscel cu AB = AC = 3, B t si C (I, r) cercul nscris acestuia. Atunci lungimea cercului C (I, r) este a egal cu BC, iar aria lui este aproximativ egal cu aria 4BIC; n amb a a eroarea aproxim rii fiind mai mic ca 0, 01. a a Soluie. Avem: AD2 = AB 2 BD2 = t A 1, deci AD = 1 i s 2 S AD BC . r= = = I p AB + BC + AC 2+ 3 Tinnd cont de faptul c ' 2 + 3, a pentru cercul C (I, r) obinem: t B D 2 = 2 2, adic L ' BC; 2+ 3 a L = 2r ' 2 2+ 3 !2 2 2 , adic A ' A 2+ 3 = a A = r2 ' 2+ 3 2+ 3 2 1 1 2 = ). (ntr-adevr, ABIC = BC ID = 2 2 a 2 2 2+ 3 2+ 3 S dovedim c, n formulele gsite, L i A sunt aproximate cu o eroare a a a s dect 0, 01. ntr-adevr, nmulind inegalitatea 2 + 3 < 0, 01 ( a t 2 2 + 3 L < 0, 0 conform Propoziiei!) cu 2r, obinem 2 t t 2+ 3 2 2 < 1, urmeaz c BC L < 0, BC L < 0, 01 2r. Cum 2r = a a 2+ 3 dar nmulind aceeai inegalitate cu r2 , obinem ABIC A < 0, 01. t s t

Aplicaia 2. Fie cercul C (O, 1) si punctele A, B, t C si D ca n figura de mai jos: BC = 1, BD = AC.

Ar tai c lungimea semicercului AB (aria semicera t a cului) este aproximativ egal cu lungimea segmentului a 1 [AD] (respectiv aria triunghiului ABD), eroarea ind A mai mic dect 0, 01. a O 1 Soluie. Deoarece AB = 2 i BC = 1, rezult c t s a a AC = 3; la fel, din BC = 1 i BD = AC = 3, s deducem c CD = 2. Atunci, AD = AC + CD = a 1 1 1 2 + 3 ' i AABD = AD BC = s 2+ 3 ' = 12 etc 2 2 2 2 Aplicaia 3. Dat un p t atrat de latur 1, construii numai cu compas a t de lungime aproximativ egal cu perimetrul p tratului. a a Soluie. Mai nti, s observm c un cerc de lungime egal cu perime t a a a a 2 2 2 =2 t t tului dat are raza . Dar, innd cont de Propoziie, ' 2 3 + Aadar, urmeaz s construim cu compasul un cerc de raz 2 3 2 . s a a a Etapele unei posibile construcii sunt: t D 1. Construim simetricul E al punctului B faa de A: t {E} = C (A, 1) C (D, DB). 2. Construim punctul F astfel nct 4BEF s fie a E A echilateral, iar F i D s fie de o parte i de alta a s a s dreptei BE: {F } = C (B, BE) C (E, EB); evident, A, D, F sunt coliniare i AF = 3 (nlime n 4BEF de s at latur 2). a 3. Construim punctul G de partea dreptei BE n care G se afl F prin {G} = C (A, AC) C (B, AF ). Deoarece a F AB = 1, AG = 2 i BG = 3, rezult c 4AGB este s a a H dreptunghic n A i, ca urmare, s punctele A, F , G sunt coliniare, iar F G = AF AG = 3 2. 4. Construm simetricul H al lui G faa de F (construcia, t t P numai cu compasul, a simetricului M 0 al punctului M faa t de un punct O poate fi urmrit pe figura alturat); evident a a a a M O GH = 2 3 2 . 5. Construim C(H, HG), care va fi cutat: luncercul a gimea lui este 4 3 2 ' 4 2 + 3 3 2 = 4, Q cu o eroare de h i 4 3 2 4=4 3 2 3+ 2 <4 3 2 1 conform cu (1). Cum 3 2 < , vom avea 2 1 3 2 4 < 4 0, 01 = 0, 02 , 4 2 adic eroarea cu care lungimea cercului construit este aproximat de peri a a tratului este mai mic dect 0, 02. a

Inegaliti generatoare de noi inegaliti at at


I. V. MAFTEI 1
Pornind de la anumite inegaliti cunoscute ne propunem s obinem at a t liti. at

Propoziia 1. S se demonstreze c t a a n1 n1 n1 xn+k1 + xn+k1 + + xn+ x1 x2 xk x1 + x2 + + xk 1 2 k x1 , x2 , . . . , xk R , + n, k N, n, k 2.

Demonstraie. Utiliznd relaia dintre mediile aritmetic i geometric t t as t numerelor a1 , a2 , . . . , ak R , n, k N, n, k 2, obinem succesiv: + p n na1 + a2 + + ak n+k1 a1 a2 ak , n+k1 p n a1 + na2 + + ak n+k1 a1 a2 ak , n+k1 ....................................... p a1 + a2 + + nak n+k1 a1 a2 an . k n+k1 Sumnd inegalitile (2), rezult c at a a p n p n p n+k1 n+k1 a1 a2 ak + a1 a2 ak + + n+k1 a1 a2 an a1 + a2 + k n+k1 a = x , i = 1, k, ob inem Dac notm a a t i i care este tocmai inegalitatea (1).

xn x2 xk + x1 xn xk + + x1 x2 xn xn+k1 + xn+k1 + + 1 2 k 1 2 Pentru k = 2 i n = 2h, h N , inegalitatea (1) devine s x2h+1 + x2h+1 x1 x2 x2h1 + x2h1 . 1 2 1 2 (ab)k1 a2k+1 + b2k+1 + (ab)
k1

Propoziia 2. Fie a, b, c R si k N. Atunci, are loc inegalitatea t + + (bc)k1


k1

b2k+1 + c2k+1 + (bc) a2k+1 + c2k+1 + ( 1 1 1 + + ab (a + b) + 1 bc (b + c) + 1 ac (a + c) + 1 x1 , x2 R , + k N.

(ac)k1

Demonstraie. Aplicnd inegalitatea (3) de k ori, obinem t t x2k+1 + x2k+1 (x1 x2 )k (x1 + x2 ) , 1 2 Tinnd seama de (5), putem scrie de unde
1

a2k+1 + b2k+1 (ab)k (a + b) , a2k+1 + b2k+1 + (ab)


k1

(ab)

k1

[ab (a + b) + 1]

Profesor, Colegiul Na ional "Sf. Sava", Bucureti t s

sau a2k+1

1 . ab (a + b) + 1 + + (ab) Sumnd aceast inegalitate cu analoagele ei, obinem (4). a t Observaie. Dac n (4) lum k = 2 i considerm abc = 1, suntem t a a s a inegalitatea bc ca ab + + 1, a5 + b5 + ab b5 + c5 + bc c5 + a5 + ca care a fost discutat la O. I. M. din anul 1996, India. a (ab) b2k+1
k1

k1

a2n+1 b2n+1 c2n+1 + n + an b + bn c + cn a, n N. bn c an Demonstraie. nmulind inegalitatea (6), considerat pentru k = n t t a iar analoagele ei cu bn an i respectiv bn cn , vom obine relaiile s t t a3n+1 cn + b2n+1 an cn a2n+1 bn cn + a2n bn+1 cn , b3n+1 an + c2n+1 bn cn b2n+1 an cn + b2n cn+1 an , c3n+1 bn + a2n+1 cn bn c2n+1 bn an + c2n an+1 bn ,

Propoziia 3. Fie numerele a, b, c R . S se demonstreze c t a a +

din care, prin adunare, deducem c a adic (8) . a

a3n+1 cn + b3n+1 an + c3n+1 bn an bn cn (an b + bn c + cn a) ,

Procednd ca n Propoziia 3 se obine t t Propoziia 4. Pentru n N si a, b, c R avem t + a

c2n+2 a2n+2 b2n+2 + b n + c n abn+2 + bcn+2 + can+2 . cn a b

Propoziia 5. S se arate c n N avem: t a a q n (n + 1) n 2k+1 2k+1 +2 + + n2k+1 (n!)2k , a) 1 2 n (n + 1) b) 12n+1 + 22n+1 + + n2n+1 (n!)2 . 2 Demonstraie. Pentru n = 1 avem egalitate. Considerm n 2 i n t a s inegalitatea (6) succesiv a = 1 i b = n, a = 2 i b = n 1, . . . , a = n s s Sumnd inegalitile rezultate, vom obine at t h 2k+1 2 1 + 22k+1 + + n2k+1 (n + 1) 1k nk + 2k (n 1)k + + q q 2 n n Cum paranteza ptrat este n (1k 2k nk ) = n (n!)2k , avem a a q 2k+1 n 2k 2k+1 2k+1 2 1 +2 + + n n (n + 1) (n!) , adic (10). Lund n (10) k = n, obinem inegalitatea (11). a t

Asupra unei probleme dat la ONM, Bistria, a t


Claudiu-Stefan POPA1

Cele ce urmeaz au ca punct de plecare o problem dat la ONM, Bis a a a [1] aparinnd autorului acestei note i pe care o vom nota n continuare t s

(P ) Fie ABCD un trapez cu bazele AB si CD, avnd diagonalele perp n O. Pe semidreptele (OA si (OB se consider punctele M si respecti a \ si BM D s fie drepte. Not m cu E mijlocul se \ a nct unghiurile AN C a M N . S se arate c : a a a) triunghiurile OM N si OBA sunt asemenea; b) dreapta OE este perpendicular pe dreapta AB. a Rezolvarea acestei probleme poate fi gsit de asemenea n [1]. a a

Considerm configuraia geometric pus n valoare de (P ) ndeajuns de a t a a pentru a prezenta alte cteva rezultate legate de ea. Dm nti o carac a trapezului ortodiagonal, interesant i n sine. as

Propoziia 1. Fie ABCD un patrulater convex si AB k CD. Dac t a este intersecia diagonalelor sale, patrulaterul este ortodiagonal dac si n t a AB CD = AO CO + BO DO. Demonstraie. AB k CD 4AOB 4COD t AO BO DO AB CD AO CO + B BO AB AO CO = = = = = = 2 2 2 CO DO CD CO DO CD CO2 + D 2 2 2 Acum AO CO + BO DO = AB CD CD = CO + DO AC B

Adugm la ipoteza problemei (P ): punctele K, L sunt mijloacele baz a a t respectiv [CD] iar punctul D0 este simetricul punctului D faa de punc aceste condiii, pentru cele ce urmeaz presupunem cunoscute urmtoarele t a a K, O i L sunt coliniare, AAOD = ABOC , A2 s = AAOB ACOD ([2], p AOD

Propoziia 2. n ipoteza problemei (P ), au loc urm toarele: t a s i) M N = AB CD i M N < KL; ii) M N KL; iii) AOM N = AAOB ACOD ; iv) AN k M D0 ; Demonstraie. i) 4AN C i 4BM D sunt dreptunghice n N , resp t s N O AC, M O BD. Aplicnd teorema nlimii obinem N O2 = A at t 2 \ ) = 90 , avem M O2 + N O2 = M N 2 M O = BO DO. Cum m(M ON AB CD AB M N = AB CD. Deoarece KL = KO + LO = + = 2 2 AB + CD AB CD < (ABCD trapez, deci AB 6= CD), rezult c M N a a 2 ii) Fie R (OL astfel nct L (OR) i (OL) (RL). Cum (DL s urmeaz c OCRD este paralelogram. Dar CO DO, deci OCRD este dre a a
1

Profesor, S coala "Alecu Russo", Iai s

\ \ \ \ \ avem CDO CRO. Din (P ), punctul a) avem CDO N M O; deci CRO Aceasta i M O CR conduc la N M RO N M KL. s ON OM iii) C D AMON = = 2 AO CO BO DO = = O 2 AO CO BO DO N . = 2 P Cum 4AOB 4COD, avem AO DO = E AO DO A BO CO, deci AMON = = AAOD . K 2 M Dar A2 AOD = AAOB ACOD , deci AOMN = = AAOB ACOD . ON OD ON iv) La fel ca la iii), OM ON = OA OD. Deci = sau OA OM OA adic AN k M D0 , q.e.d. a

Observaie. Dac M N LK = {P }, propunem cititorului s demo t a a p OE = LO KO i OP = dist (O; AB) dist (O; CD). s Bibliografie

1. G.M. seria B, nr. 7/2005, p.298 i p. 301. s 2. D. Mihalca, I. Chiescu, M. Chiria - Geometria patrulaterului, Ed. T t t cureti, 1998. s

ERATA

Mai multi colaboratori aduc la cunotina Redaciei revistei urmtoare t s t t a n scrierea numelui marelui matematician Leonhard Euler : n loc de Leo scris Leonard att n titlul materialului din nr. 2/2004, p. 129, ct i n s 2/2005, p. 119 (prin preluarea primului pe calculator). COMENTARIU

D-l D. Pl su din Iai semnaleaz Redaciei faptul c Problema L.62 ae s a t a Brsan, publicat n nr. 1/2004 este cunoscut apare n cartea lui W. a a intitulat Ce stim si ce nu stim despre numerele prime (n l. rom. la Ed a inic, Bucureti, 1966) la p. 104. Cele dou soluii date acestei probl t a s a t 1/2005, pp. 67-68, difer de soluia prezentat n cartea menionat. a t a t a

Vizitai pe Internet revista "Recreaii Matematice" la t t

http://www.recreatiimatematice.uv.ro

Asupra criteriului de congruena LLU t


Marius TIBA1

Aa cum se arat n [1], urmtorul criteriu de congruena a triunghiuri s a a t numeroase capcane prin aplicarea sa incorect. Redm aici rezultatul di a a curge acest criteriu.

Propoziie. Dac dou laturi si unghiul opus uneia dintre ele ale unu t a a sunt respectiv congruente cu dou laturi si unghiul opus uneia dintre el a triunghi, iar si noteaz unghiurile opuse celorlalte laturi congruente, b b a a) atunci si sunt sau congruiente sau suplementare; b b b) (Criteriu LLU) dac n plus si sunt de acelai tip (adic am a b b s a ascuite, obtuze sau drepte), atunci cele dou triunghiuri sunt congruente. t a Problema 1. Pe laturile (OX si (OY ale unui unghi ascuit XO t \ punctele A si B astfel nct [OA] [OB]. n interiorul unghiului se ia un \ \ supus condiiei OM A OM B. G sii locul geometric descris de M . t a t Rezolvare. Vom arta c locul cutat este format din a a a \s bisectoarea interioar a unghiului XOY i arcul cercului cira cumscris 4AOB cuprins n interiorul unghiului, pe care-l t t notm AB. Fie M un punct ce satisface condiiile din enun. a \ i OBM \ Ca urmare 4OAM i 4OBM au unghiurile OAM s s sau congruente sau suplimentare (conform punctului a) al \ \ Propoziiei). n cazul n care OAM OBM , punctul M se t b afl pe bisectoarea unghiului O. Dac aceste unghiuri sunt a a suplimentare, atunci patrulaterul OAM B este inscriptibil i, s ca urmare, M AB. Reciproca rezult imediat. a

A X

Menionm c la faza judeean a O. M. din Vaslui, 2005, cl. a VI-a t a a t a s se arate c locul geometric este doar bisectoarea interioar a unghiu a a a Nerespectarea Propoziiei conduce la erori ca aceasta (prezent i n barem t as din problema T12, [2, pag. 10]. Problema 1 ne sugereaz a \ Problema 2. G sii locul geometric al punctelor M pentru care OAM a t unde notaiile sunt aceleai ca n Problema 1. t s Rezolvare. Judecnd analog ca la Problema 1, obinem locul cutat t a \ i segmentul [AB] (fr capete). bisectoarea interioar a unghiului XOY s a aa Ca o extindere a Problemei 1, dm urmtoarea a a

Problema 3. Fie ABCD un trapez isoscel si fie O mijlocul bazei m G sii locul geometric al punctelor M situate n interiorul liniei frnte fo a t \ \ semidreptele (DA, (CB si segmentul [CD], astfel nct OM A OM B.
1

Elev, cl. a VII-a, Colegiul Na ional "C. Negruzzi", Iai t s

Rezolvare. Aceast problem se reduce la Problema 1, a a deoarece [OA] [OB] (4OBC 4OAD (LUL)). Astfel, locul geometric cerut este format din punctele mediatoarei segmentului CD i ale arcului cercului circumscris 4OAB, s aflate n interiorul liniei frnte date. Problema 3 ne sugereaz urmtoarele dou probleme, pe a a a care le propunem cititorului spre rezolvare: Problema 4. Se modific Problema 3 lund punctul O a la intersecia laturilor neparalele ale trapezului. t Problema 5. Modific m Problema 3, lund condiia a t \ \ AM D BM C. Bibliografie

1. D. Mihe - Criteriul de congruena LLU, RMT an II (seria a 4-a), nr t t pag. 3-7. 2. I. Ptracu - Probleme de geometrie plan , Editura Cardinal, Craiova, 1 a s a

IMPORTANT

n scopul unei legturi rapide cu redacia revistei, pot fi utilizate urm a t adrese e-mail: tbirsan@math.tuiasi.ro, profgpopa@yahoo.co aceast cale colaboratorii pot purta cu redacia un dialog privitor a t rialele trimise acesteia, procurarea numerelor revistei etc. Sugerm a ratorilor care trimit probleme originale pentru publicare s le nume a s-i rein o copie xerox a lor pentru a putea purta cu uurina o as t a s t prin e-mail asupra acceptrii/neacceptrii acestora de ctre redacia a a a t

La problemele de tip L se primesc soluii de la orice iubitor de ma t elementare (indiferent de preocupare profesional sau vrst ). Fiecar a a soluiile acestor probleme - ce sunt publicate n revist dup un a t a a urmat de numele tuturor celor care au rezolvat-o. a

Adresm cu insistena rugmintea ca materialele trimise r a t a s nu fie (s nu fi fost) trimise i altor publicaii. a a s t

Rugm ca materialele tehnoredactate s fie trimise pe adresa reda a a A soite de ierele lor (de preferina n L TEX). t s t

O generalizare a identitii Botez - Catala at


Ioana OLAN 1

n 1872, N. St. Botez public o lucrare original n care apare identi a a 1 1 1 1 1 1 1 + ++ =1 + + + , n+1 n+2 2n + 1 2 13 25 n (2n + 1) care, dac inem seama de formula de descompunere at 1 1 1 = , k N , 2k (2k + 1) 2k 2k + 1 se aduce la forma 1 1 1 1 1 1 1 1 + + + = 1 + + + , n n+1 n+2 2n 2 3 4 2n 1 2n numit identitatea Botez - Catalan. Ne propunem s-i dm o generalizare a a a Amintim o demonstraie a formulei (1), generalizarea obinndu-se n t t 1 1 1 1 1 1 1 1 1 = 1 + ++ 1 + ++ 2 3 4 2n 1 2n 2 3 4 2n 1 1 1 1 1 1 1 1 1 = 1+ 2 + + 2 + + + 2 2 2 3 4 4 2n 1 2n 2n 1 1 1 1 1 1 1 1 = 1 + + + + + + 2 + + + = 2 3 4 2n 1 2n 2 4 2n 1 1 1 1 1 1 1 1 = 1 + + + + + 1 + + + = + + 2 3 2n1 2n 2 n n+1 n+2 Propoziie. Pentru n N si m N, are loc egalitatea t 2m 1 1 2m 1 1 2m 1 1 1 + m + + m m = m + + m m 2 3 4 (2n 1) (2n) (n + 1) ( (Pentru m = 1 se obine identitatea (1).) t Demonstraie. ntr-adevr, avem: t a 2m 1 1 2m 1 1 2m1 1 + m + + m m = 2m 3 4m (2n 1) (2n) 1 1 1 1 1 1 1 = 1+ m 2m m + m + m 2m m + + 2m m+ 2 2 3 4 4 (2n 1) (2n)m 1 1 1 1 1 1 1 m = 1 + m + m + m + + + m + + m + m 2 m 2 3 4 2 4 (2n 1) (2n) 1 1 1 1 1 1 = 1 + m + m + + = m + m 1 + m + + m 2 3 2 n (2n 1) (2n) 1 1 1 + + + , q.e.d. = (n + 1)m (n + 2)m (2n)m Cazuri particulare. Pentru m = 2 i m = n, formula (2) devine: s 1

3 1 3 1 3 1 1 + 2 2 + + + + 2 2 = 2 + 2 2 3 4 (2n1) (2n) (n+1) (n+2)2 2n 1 1 2n 1 1 2n 1 1 1 1 n + n n + + n n = n+ n + 2 3 4 (2n1) (2n) (n+1) (n+2)


1

Elev, cl. a VIII-a, Colegiul Na ional "C. Negruzzi", Iai a t s

Acoperiri ale planului laticial cu figuri


Marius PACHITARIU 1

Exist numeroase probleme de concurs care implic acoperiri ale unor a a ciale, de exemplu dreptunghiuri, cu un numr de copii ale unei alte guri d a nouri, trominouri) sau cu alte copii scalate ale nsui dreptunghiului. s Observnd metodele si tehnicile acestor tipuri de probleme, putem acoperiri ale ntregului plan cu diferite guri. Vom lucra n continuare doa laticial. Pentru aceasta, s definim planul laticial i s dm coordonate p a s a a care l alctuiesc. Introducem n continuare o serie de noiuni. a t Considerm dreptele de ecuaie x = r, r Z, y = q, q Z. Ptratele d a t a determinate de punctele lor de intersecie vor constitui elementele planul t Vom atribui coordonate acestor ptrate n urmtorul mod: dac un ptra a a a a intersecia benzilor determinate de x = r, x = r + 1 i respectiv, y = q, t s atunci vom spune c ptratul are coordonate r, q sau mai simplu vom num a a [r, q]. Vom numi vector ntre dou ptrate vectorul ai + bj, cu i, j versor a a a, b numrul de ptrele orizontale, respectiv verticale care separ cele do a a at a considerate, cu semnul asociat corespunztor. a Numim plantaie orice colecie de ptrate ale planului laticial, conex t t a a c din orice ptrat putem ajunge n oricare altul printr-o succesiune fin a a plasri unitare (translaii dup una din cele 4 direcii), astfel nct dup a t a t a (translaie) ne aflm nc n unul dintre ptratele coleciei. Numim gur t a a a t a maximal de plantaii cu proprietatea c fiecare plantaie poate fi obinu a t a t t care alta a gurii prin translaii, rotaii i simetrii faa de drepte paralel t t s t Cu alte cuvinte, figura reprezint o clas de echivalena. a a t Fie un set X de figuri. Numim acoperire a planului orice set Y de p elemente aparinnd figurilor din X, astfel nct fiecare punct al planulu t la cel puin o plantaie din Y . Numim m sura unei acoperiri sup n[i, j], u t t a este numrul de plantaii crora i aparine patratul [i, j]. Numim n-a a t a t acoperire n care fiecare ptrat al planului aparine aceluiai numr d a t s a modulo n. Pentru 2-acoperiri vom considera acoperire impar cea n c a ptrat e acoperit de un numr impar de ori. 2-acoperirile pare nu ne in a a ntruct considernd acceai plantaie de dou ori i considernd orict s t a s perechi, vom obine ntotdeauna o 2-acoperire par. t a Observaia 1. Reuniunea a dou acoperiri este o acoperire i msura t a s a a dou acoperiri este cel mult suma msurilor celor 2 acoperiri. a a Putem acum s ne punem o serie de ntrebri: a a ntrebarea 1. Care sunt seturile X de cardinal 1 pentru care exist a a msura 1? a Un exemplu netrivial de figuri n spaiu cu aceast proprietate l ofer u t a a Problem. Lipim cte un cub unitate pe fiecare faa a unui cub unitat a t c putem umple spaiul folosind copii ale solidului rezultat. (Austrian-Po a t
1

Elev, Colegiul Na ional, Iai t s

ntrebarea 2. Care sunt seturile X de cardinal 2 pentru care exist a a msura 1? a ntrebarea 20 . Dati exemple de doua seturi Y i Z de cardinal 1 pent s exista acoperire cu msura 1 cu setul Y i nu exist acoperire cu msura a s a a S Z, dar pentru care exist acoperire cu msura 1 cu setul Y Z. a a Vezi la pagina 74 exemplul 1. Demonstrai c exemplul este ntr-adev t a a 00 ntrebarea 2 . Dai exemple de dou seturi de figuri de cardinal 1: t a care nu exist acoperire cu msura 1 i S pentru care exist o astfel de a a a s Z, a pentru care exist acoperire cu setul Y Z cu msura 1. a a Vezi la pagina 74 exemplul 2. Demonstrai c exemplul este ntr-adev t a a ntrebarea 3. Care sunt seturile X de figuri de cardinal n, n N, p exist acoperiri cu msura 1? a a 0 ntrebarea 3 . Dai exemple de seturi de n seturi Xi , i = 1, n de t cardinale 1, cu proprietatea c nici unul dintre ele nu poate genera o ac a msura 1, dar reuniunea lor da. a Vezi la pagina 74 exemplul 3. Demonstrai c exemplul este ntr-adev t a a ntrebarea 4. Orice figur poate genera o 2-acoperire impar a p a a msura finit? Dac vom considera minimul msurii peste toate acoperiri a a a a putem gsi un maxim pentru acesta n funcie de figura folosit? a t a Nu vom rspunde aici ntrebrilor 1, 2, 3, fiind prea generale. Desigu a a X care constituie rspunsurile primelor 2 sunt particularizri ale seturilor a a ntrebare. Caracterizri ale primului tip de seturi din anumite puncte de v a cu sigurana posibile, pe cnd o caracterizare n cazul general pare impos t exemplele ntrebrilor 20 , 200 , 30 ). a Vom rspunde n schimb ultimei ntrebri. Rspunsurile sunt DA i DA a a a s ile modulo 2 ne dau ntr-adevr de ajuns libertate. Pentru demonstraiile a a t vom renuna la condiia de conexitate din definiia plantaiilor. t t t t Demonstraia 1. Vom numi cardinal al unei plantaii (guri) n t t ptrele pe care le conine. Pentru o gur de cardinal impar putem lua u a at t a acoperire: Fie o plantaie oarecare i toate translaiile ei care se pstreaz t s t a a Atunci plantaiile rezultate i cu cea iniial reprezint o 2-acoperire. n t s t a a fiecare ptrat al planului laticial este acoperit de exact (cardinalul plan a deci pentru cardinalul impar avem o 2-acoperire. Mai mult, avem msura a egal cu cardinalul gurii. a Din pcate aceast cale nu pare s furnizeze soluie pentru figurile d a a a t par. Demonstraia 2. Figura F fiind finit, o putem include ntr-un ptrat t a a pe latice, deci o putem include ntr-un ptrat cu latura putere a lui 2. Fie a celui mai mic astfel de ptrat. S considerm acoperirea de msur 1 cu a a a a a ptrate de latur 2k (chiar o omotetie a planului laticial). Plasm n fiecare a a a latur 2k al acestei acoperiri figura F corespunztoare. Asociem ptrele a a a at ale ptratului de latur 2k valoarea 1 dac ptrelul este n F i 0 altfe a a a a at s cuvinte am fcut o prim acoperire a planului cu figuri F . Vom face o ser a a

Primul pas: Considerm acoperirea obinut ca mai sus, dar nlocuin a t a cu simetrica ei faa de (Oy, astfel nct aceast acoperire s suprapun t a a a ptrate de latura 2k peste cele considerate anterior. Fie F 0 noua figur ob a a suprapunerea celor dou acoperiri (reuniunea) i considerarea modulo 2 (u a s are asociat 1 dac este acoperit de un numr impar de plantaii i 0 altfel). a a t s analog cu F 0 dar faa de axa (Ox. Obinem o nou figur F 00 , care este s t t a a faa de orizontal i faa de vertical, ncadrat n ptratul de latur 2k . S t a s t a a a a a c fiecare ptrael a fost acoperit de cel mult 4 ori. a a t Realizm acoperirile determinate de translaii ale acoperirii de mai su a t F 00 , de vectori 2k1 i, 2k1 j, 2k1 i + 2k1 j. S considerm reuniunea celor 4 acoperiri. Privind mai atent, observ a a obinut astfel o nou acoperire cu ptrate de latur 2k1 , cu o figur F (2) t a a a a ptrat, simetric orizontal i vertical. Acest proces reprezint de fapt ec a a s a primei pri, ntruct acoperim figura din ptratul de latur 2k1 cu si at a a faa de axa vertical i apoi cu simetricele faa de axa orizontal a celor d t as t a obinute. Fiecare ptrat a fost acoperit de cel mult 4 + 4 + 4 + 4 = 4 4 o t a Putem continua acum cu urmtorii pai analogi cu partea a doua a a s (partea cu translaiile) pn ajungem la acoperirea cu ptratul de latura t a a a n interiorul lui figura F (k + 1), care poate fi ori mulimea vid ori nsui t a s caz n care am obinut o 2-acoperire impar a planului cu F (i o msur d t a s a a 4 4 4 . . . 4 = 4k ). Pentru a determina numrul la care ajungem (0 sau 1) n F (k + 1), vom a aceeai serie de pai ca mai sus, urmrind n acelai timp cum evolueaz sum s s a s a valorilor asociate patrelelor interiorului ptratului de latur 2k . at a a S presupunem c am considerat ptratul de vrfuri [1, 1], [1, 2k ], [2k , a a a (ne referim aici la cele 4 ptrate unitare) i fie ai,j = 1 dac ptratul [i a s a a figura F i 0 altfel. s Vom da acum un exemplu de "pai" fcui ca mai sus: s a t Exemplul 4.
1 1 1 0 1 0 0 1 0 1 0 0 1 0 0 0 0 1 1 0 1 1 0 1 1 1 0 1 0 1 1 0 0 0 0 0 0 1 1 0 0 1 1 0 0 0 0 0

1 1 1 1

S notm bi,j , i = 1, 2k1 , j = 1, 2k1 valorile asociate acoperirii rez a a suma celor 4 acoperiri (fr a le reduce modulo 2; adunm ai,j -urile coresp aa a pentru a proba dac [i, j] este sau nu n F (2)). Acum este uor s ob a s a bi,j = ai,j + ai,2k +1j + a2k +1i,j + a2k +1i,2k +1j . Dar aceasta nseamn a X X X bi,j = ai,j + ai,2k +1j + a2k +1i,j + a2k +1i,2k +1j =
i2k1 j2k1 i2k1 j2k1

i j

Am demonstrat prin aceasta c suma valorilor asociate rmne aceea a a s cuvinte, valoarea din ultimul ptrat dup ultimul pas este egal cu sum a a a iniiale ale figurii F . Deci, dac figura F are un numr impar de ptr t a a a at

figura F (k + 1) este constituit dintr-un ptrel, adic avem o acoperire a a at a Dar nu am obinut acelai lucru n demonstraia 1 i mult mai uor? t s t s s Ba da, dar demonstraia 1 nu ne perminte o rafinare pentru a o face s f t a i pe figuri de cardinal par, pe cnd aceasta da. s Dac la sfrit nu obinem un ptrel, ci mulimea vid, rezult c la u a s t a at t a a a dat, dup o simetrizare, figura s-a anulat pe sine nsi. Dar aceasta nu s a as dect n cazul n care figura era deja simetric faa de axa faa de care s-a f a t t tria. Dar suprimnd aceast etap din pasul corespunztor putem continua a a a de simetrizare, ntruct figura e deja simetric faa de acea ax. Suprim a t a etapele n care am face o astfel de simetrie nedorit, ne pstrm pe linia ra a a a tului anterior i, mai mult, putem fi siguri c ajungem la o gur unitar s a a a considerm, de exemplu, ultimul pas, plecnd de la cele 16 posibiliti i a at s acele simetrii care nu sunt favorabile. Exemplul 5. 1 1
1 1

STOP
0 0 1 1 1 1 0 0 1 1 1 1

Exemplul 6. Exemplul 7. Exemplul 8. Exemplul 9.

1 1 1 0 1 1 0 0 1 0 1 0

1 1 1 1 1 1 1 1

STOP STOP

STOP

1 0 1 1 1 1 STOP 0 0 0 0 1 1 Nu este greu de observat c fr a suprima nici o simetrizare avem a aa plantaii peste fiecare ptrel. Acest numr reprezint toate ptrelele t a at a a a at 2k 2k , deci ne furnizeaz o msur mai mare dect cea furnizat de dem a a a a 1 pentru figuri de cardinal impar. Pentru cardinal par n schimb, putem aceast msur ca un bun majorant pentru valoarea minim posibil a ms a a a a a a mai mult, innd cont c am suprimat cel puin o etap, nseamn c n t a t a a a dublat n momentul acela msura, deci obinem cel mult 2 4k1 care pentr a t ajuns de compacte i de bine ncadrate n ptratul de latur 2k corespunz s a a fi chiar mai mic dect cardinalul plantaiei. t

Problem. Colorm planul n alb i negru ca tabla de ah. Fie o a a s s pentru care numrul de ptrele albe pe care le conine, A, este impar a a at t s de ptrele negre pe care le conine, B, este tot impar. S se gaseasc o a a at t a a planului cu figura reprezentat de aceast plantaie cu msura egal cu m a a t a a Conjenctur. Putem acoperi modulo 2 planul cu translaii ale une a t plantaii, oricare ar fi aceasta. Mai mult, msura obinut poate fi mai m t a t a cea dat de demonstraia 2 i chiar dect cardinalul plantaiei. a t s t Un nceput de demonstraie pentru conjenctur este sugerat de prob t a rioar. a

Metoda normrii a
Marian TETIVA1

Introducere. n aceast not vrem s dm cteva exemple de utilizare a a a a norm rii, pe care am preluat-o, cu tot cu acest nume, din excelenta car a pornit de la faptul c acolo nu exist prea multe aplicaii i, la nceput a a t s n glum, am demonstrat pe aceast cale cteva inegaliti (nu tocmai u a a at s timpul s-au adunat din ce n ce mai multe asemenea inegaliti (i din ce at s grele). Metoda s-a dovedit extrem de eficient pentru demonstrarea ine a omogene dar i pentru obinerea unor identiti altfel greu de gsit, iat de s t at a a o prezentm aici; totui, trebuie s-o spunem, metoda normrii nu este rec a s a celor care au "alergie" la calcule: este pentru cei rbdtori i stpni a a s a matematic (ne referim la calculul elementar). De asemenea, se aplic ine a a n care variabilele implicate sunt numere reale pozitive (sau nenegative). S ncepem cu o inegalitate foarte cunoscut (a se vedea i [3], capitol a a s a2 + b2 + c2 ab + ac + bc;

se tie c aceasta e valabil pentru orice numere reale a, b, c, dar noi o vom s a a (complicat, vei spune, dar e numai pentru a da un exemplu) doar pentru t (iat un dezavantaj; nu unul mare, pentru c majoritatea inegalitilo a a at urma prezint interes pentru cazul variabilelor nenegative i, uneori, se po a s plecnd de la acesta, la orice valori reale ale variabilelor). Datorit simetr a presupune fr a particulariza, c c = min {a, b, c}. Pentru c = 0 inegal aa a evident: a 2 a2 + b2 ab (a b) + a2 + b2 0. a b Fie c > 0 i s notm = 1 + x, = 1 + y. Conform presupunerilor fc s a a a c c x 0 i y 0, iar inegalitatea de demonstrat devine (dup mprirea c s a at noile notaii) t

i revenind la a, b, c (cu x = s

evident (ca mai sus) chiar pentru orice x, y R, nu doar pentru x, y a uor c egalitatea are loc doar pentru x = y = 0, deci numai dac a = b = s a a mult, plecnd de la (1 + x)2 + 1 + y 2 + 1 (1 + x) (1 + y) 1 x 1 y = x2 + y 2

(1 + x)2 + (1 + y)2 + 1 (1 + x) (1 + y) + 1 + x + 1 + y x2 + y 2

care permite demonstrarea inegalitii pentru orice a, b, c R i chiar obi at s t rafinri: a 1 2 2 a2 + b2 + c2 ab ac bc (a c) + (b c) , a, b, c R 2


1

ac bc ,y= ) gsim identitatea a c c 2 2 a2 + b2 + c2 ab ac bc = (a c) + (b c) (a c) (b c)

Profesor, Colegiul Na ional "Gheorghe Roca Codreanu", Brlad t s

Desigur, toate acestea se puteau face i altfel i sunt cunoscute, dar . . . n s s demonstraia obinuit a acestei inegaliti, nu-i aa? t s a at s Mai departe s considerm inegalitatea a a a4 + b4 + c4 a3 b + b3 c + c3 a, a, b, c 0,

creia i aplicm acelai tratament. Simetria (de data asta, doar circular) a a s a s presupunem, fr a restrnge generalitatea, c c = min {a, b, c}; c = a aa a inegalitatea n forma a4 + b4 a3 b, pentru orice a, b 0 (exerciiul 1: de t acest caz particular!). Mai departe fie c > 0 i s facem aceleai notaii c s a s t mprim cu c4 i inegalitatea devine at s (1 + x)4 + (1 + y)4 + 1 (1 + x)3 (1 + y) + (1 + y)3 + 1 + x,

de demonstrat pentru x, y 0. Calcule simple (exerciiul 2: verificai-le! t t form n a 3 x2 + y 2 xy + 3 x3 + y 3 x2 y + x4 + y 4 x3 y 0; aceasta este adevrat, ba chiar se poate ntri, innd cont de a a a t 1 2 x + y2 , x2 + y 2 xy 2 de 2 x3 + y 3 x2 y xy 2 ( (x y) (x + y) 0) i de s x4 + y 4 x3 y xy 3 ( (x y)2 x2 + xy + y 2 0). Astfel am obinut de fapt t

(mai conteaz cum sunt numerele a, b, c?) i s obinei i alte ntriri ale i a s a t t s a considerate.

(1 + x)4 + (1 + y)4 + 1 (1 + x)3 (1 + y) (1 + y)3 1 x 3 2 x + y 2 + 2xy 2 + xy 3 , x, y 0; 2 t aici s revenim la variabilele iniiale a, b, c i s nmulim cu c4 . Exerciiul a t s a t c ajungem la inegalitatea a a4 + b4 + c4 a3 b + b3 c + c3 a 3 2 2 2 3 c2 (a c) + (b c) + 3c (a c) (b c) + (a c) (b c) 2 pentru orice numere nenegative a, b, c, c fiind cel mai mic dintre ele (este acest lucru?). Iar exerciiul 4 v cere s demonstrai identitatea t a a t 3 3 2 2 2 a4 + b4 + c4 a b + b3 c + c3 a = (a b) + (a c) + (b c) 2 2 2 +3c (a b) (a + b 2c) + 3c (a c) (b c) + 2 2 2 + (a b) (a c) + (b c) + (a c) (b c) + (a c) (b c)

Acum, c ai cam neles n ce const metoda normrii i, n plus, a a t t a a s antrenament la calcule de acest tip, putei exersa chiar singuri i ceva ma t s

d2 (a d) + d2 (b d) + d2 (c d) + 2d (a d) (b d) (c d Deducei inegalitatea lui Turkevici t a4 + b4 + c4 + d4 + 2abcd a2 b2 + a2 c2 + a2 d2 + b2 c2 + b2 d2 + c2 (pentru orice a, b, c, d 0), identitate a crei consecina este i, eventual, a a t s ale ei.

Exerciiul 5. Artai c, pentru orice numere reale nenegative a, b t a t a d = min {a, b, c, d} are loc inegalitatea a4 + b4 + c4 + d4 + 2abcd a2 b2 a2 c2 a2 d2 b2 c2 b2 d2 c2 d
2 2 2

Metoda normrii i o demonstraie a inegalitii mediilor. a s t at s-a vzut, metoda descris n aceast not se aplic n cazul inegalitilo a a a a a at i omogene n n + 1 variabile, s le spunem a1 , a2 , . . . , an+1 . Simetria n s a s considerm, nerestrictiv, c, de exemplu, an+1 este cel mai mic di a a a numerele a1 , a2 , . . . , an+1 (chiar i simetria circular ne permite o asem s a supunere). Dup ce vericm inegalitatea pentru an+1 = 0 (dac e cazu a a a substituiile t a2 an a1 = 1 + xi , = 1 + x2 , . . . , = 1 + xn , an+1 an+1 an+1 a unde, desigur, x1 , x2 , . . . , xn sunt nenegative (ceea ce, de obicei, ajut n no inegalitii). Se poate alege i an+1 = max {a1 , a2 , . . . , an+1 }, dar e doar o at s de gust, nu schimb esenial calculele. Apoi, folosind transformrile inver a t a a1 an+1 a2 an+1 an an+1 , x2 = , . . . , xn = , x1 = an+1 an+1 an+1 ne putem ntoarce la inegalitatea noastr pentru a obine chiar ntriri a a t a de obicei, rmn n diferena dintre cei doi membri ai inegalitii transform a t at termeni nenegativi care pot fi utilizai n acest scop), sau identiti interes t at o implic). Cine a citit cu atenie descrierea metodei n [3] a observat deja a t considerat o form particular a acesteia. Asta pentru c aa am lucrat a a a s obinut destule rezultate interesante (chiar mai multe dect cele expuse a t citii [3] i vei afla i mai multe (chiar dac n capitolul dedicat normrii s t s t s a a exemple)!

n aceast seciune am ales pentru exemplificare demonstraia inegalit a t t n n n a1 + a2 + + an na1 a2 an , a1 , a2 , . . . , an 0, adic a inegalit tii mediilor, o demonstraie care s-ar putea s par com a a t a a chiar este!) faa de multe alte demonstraii cunoscute (cci, nu-i aa? t t a s foarte multe demonstraii ale inegalitii mediilor); totui s o facem. t at s a Cum verificarea n cazurile n {1, 2} (sau chiar n = 3) nu mai consti blem trecem direct la pasul de inducie; pe care o facem dup schema: pr a t a c am demonstrat c, pentru fiecare k n inegalitatea a a ak + ak + + ak ka1 a2 ak 1 2 k are loc (pentru orice a1 , a2 , . . . , ak 0) i o dovedim pentru k = n+1. n in s de demonstrat an+1 + an+1 + + an+1 + an+1 (n + 1) a1 a2 an an+1 , a1 , a2 , . . . an n 1 2 n+1

putem presupune, cum am spus, an+1 = min {a1 , a2 , . . . , an+1 }. Cum an+ la o inegalitate absolut banal, putem considera an+1 > 0, s mprim a a at s facem substituiile de mai sus; vom avea de demonstrat c a t a (1 + x1 )n+1 +(1 + x2 )n+1 + +(1 + xn )n+1 +1 (n + 1) (1 + x1 ) (1 + x2 ) pentru orice x1 , x2 , . . . , xn 0. Exerciiul 6. Artai c, dup cteva calcule, ne rmne inegalitatea t a t a a a n n n X X X X k Cn+1 xk (n + 1) x1 x2 xk + xn+1 0. j j
k=2 j=1 j=1

k Prin x1 x2 xk nelegem suma tuturor celor Cn produse de cte k fact t dici distinci) alei dintre x1 , x2 , . . . , xn (pentru k = n suma conine doar u t s t produsul x1 x2 xn ). Pentru a demonstra aceast inegalitate, s observm nti c, dac folos a a a a a de inducie, avem (pentru 2 k n) t k i chiar putem scrie Cn inegaliti de acest tip (cte una pentru fiecare s at dintre numerele x1 , x2 , . . . , xn ); adunm toate aceste inegaliti i avem a at s X X xk + xk + + xk k x1 x2 xk . 1 2 k k1 Cn1 n X j=1

xk + xk + + xk kx1 x2 xk 1 2 k

k1 Cum n membrul stng fiecare xj , 1 j n, apare de Cn1 ori, de fapt a

xk k j

pentru fiecare k {2, . . . , n}. Prin urmare, membrul stng al inegalitii at strat se minoreaz astfel a n n n X X X X k Cn+1 xk (n + 1) x1 x2 xk + xn+1 j j
k=2

x1 x2 xk

n X n + 1 k1 X k xj (n + 1) x1 x2 Cn1 n j=1

Evident, mai avem s demonstrm c expresia din membrul drept este a a a calcul simplu ne arat c a a n + 1 k2 k 1 n + 1 k1 n + 1 k1 k1 k Cn+1 Cn1 = Cn Cn1 = Cn1 = C n n k n deci, de fapt, ne-a mai rmas a n n n X k1 X X k xk + xn+1 0, Cn+1 j j n j=1 j=1
k=2

n n n X n + 1 k1 X k X n+1 k Cn+1 xj + xj . Cn1 k j=1 j=1


k=2

j=1

j=1

care este evident, datorit faptului c x1 , x2 , . . . , xn sunt nenegative; dem a a a prin inducie este ncheiat. t a

pentru orice numere (chiar numere complexe) a1 , a2 , . . . , an . (Cea de-a d k din paranteza mare cuprinde toate cele Cn1 produse de k factori alei dint s a2 an , . . . , an1 an .) De exemplu, 2 2 2 a4 +b4 + c4 + d4 4abcd = 2d2 3 (a d) + 3 (b d) + 3 (c d) 2 (a d) (b d) 2 (a d) (c d) 2 (b d) (c d) + 4d (a d) + (b d)3 + (c d)3 (a d) (b d) (c d) + (a d)4 + (b d)4 + pentru orice numere (complexe) a, b, c, d. Si atunci nu v va fi greu cu a

Exerciiul 7. Demonstrai identitatea t t n a1 + an + + an na1 a2 an = 2 n n1 n1 n1 X X X X k nk k = an (aj an ) n (a1 an ) (a2 an ) (ak an ) + Cn


k=1 j=1

j=1

Exerciiul 8. Artai c pentru orice numere reale nenegative a, b, c, t a t a cel mai mic dintre ele, are loc inegalitatea 2 2 2 a4 + b4 + c4 + d4 4abcd 2d2 (a b) + (a c) + (a d) + + (b c)2 + (b d)2 + (c d)2 . (Putei rezolva asta prin metoda normrii?) t a

n ncheiere v mai propunem cteva exerciii (grele!). Primul dint a t totui mai simplu i vine n completarea celei de-a doua inegaliti din int s s at

Exerciiul 9. Demonstrai cea de a doua inegalitate din t t 4 4 4 a + b + c a3 b + b3 c + c3 a abc (a + b + c) , a, b, c 0. Nu uitai s cutai mbuntiri ale acestei inegaliti, precum i identita t a a t a at at s consecina este! t

Exerciiul 10. Demonstrai, folosind metoda normrii, inegalitatea lui S t t a n n n (n 1) (a1 + a2 + + an ) + na1 a2 an (a1 + a2 + + an ) an1 + an1 + + 1 2 i obinei ntriri ale ei. s t t a Si, n sfrit, pentru ultimul exerciiu avei nevoie i de ceva inspirai s t t s t demonstra inegalitatea la care se ajunge dup normare (nu ntotdeauna ea a fr probleme!). aa

Exerciiul 11. Demonstrai c inegalitatea t t a a2 (a b) (a 2b) + b2 (b c) (b 2c) + c2 (c a) (c 2a) 0 are loc pentru orice numere reale a, b, c. Aceast inegalitate a fost pu a Vasile Crtoaje n Gazeta Matematic , n urm cu mai muli ani [1]. a a t Bibliografie

1. V. Crtoaje - Problema 22694, Gazeta Matematic, seria B, 7-8/1992. a 2. G. Dospinescu - O teorem uitat - inegalitatea lui Surnyi, RecMat - 1 a a 3. M. Onucu Drimbe - Inegalit ti - idei si metode, Editura Gil, Zalu, 200 a a

Asupra unei recurene de ordin doi t


Gheorghe IUREA1

n [1], la pag. 59, apare urmtoarea problem (autor Vasile Berinde a a

t Fie sirul de numere reale (xn )nN definit prin relaiile: x0 R, xn+ 2xn 1, n N. a) Aflai numerele M R cu proprietatea c , dac x0 M , atunci xn M t a a 3 1 b) Ar tai c pentru x0 , a t a t sirul (xn )nN converge si calculai 2 2 c) Determinai expresia lui xn n funcie de x0 si n. t t

ntruct n rezolvarea acestei probleme dat n [1] sunt unele scpri, ne a a a s ne ocupm mai amnunit cu acest ir interesant. a a a t s n privina punctului a), fie M R cu proprietatea cerut; e x0 = M t a t 0 i trebuie s avem x1 M . Dar x1 = 2t2 2t (1 + 2M ) + 2M 2 + 2M s a lim x1 = , deducem c pentru t suficient de mare avem x1 > M . Prin u a t exist M cu proprietatea cerut. a a 1 S observm c recurena dat este echivalent cu xn+1 + = 2 xn + a a a t a a 2 1 1 2 n N . Notnd yn = xn + , obinem yn+1 = 2yn 1, y0 = x0 + t 2 2 studiul irului (xn )nN se reduce la studiul irului (yn )nN . s s Faptul c cerina de la punctul b) este greit rezult din urmtoarea a t s a a a Propoziie. Fie (yn )nN un sir de numere reale, definit prin t

b) Pentru y0 [1, 1], (yn )nN este convergent dac si numai dac exi a a astfel nct sirul (yn )nn0 este constant. n e2 t0 t c) y = cos (2n t0 ), 0 = arccos y0 pentru y0 [1, 1] si yn = n p 2 t0 = ln |y0 | + y0 1 , pentru y0 (, 1) (1, ). d) Dac M = y0 R; (yn )nN este convergent atunci: a 1. M este nevid , finit . a a 2. M = A B, unde 2 + 2k 2k ; k = 0, 1, . . . , 2 A = cos n0 ; k = 0, 1, . . . , 2n0 1 , B = cos 3 n0 2 2 0 cu n0 , n0 numere naturale fixate. 0 e) Sirul (yn )nN este convergent dac si numai dac y0 A0 B 0 , und a a v v s s u u r u u 1 1 1 1 t1 1 1 t1 1 1 , B0 = + A0 = 2 2 2 2 2 2 2 2 2 2 | | {z } {z
n0 radicali n0 radicali 0
1

2 y0 R, yn+1 = 2yn 1, n N. a) Pentru y0 (, 1) (1, ), (yn )nN este divergent si lim yn = n

Profesor, Liceul Teoretic "Dimitrie Cantemir", Iai s

semnele ind alese n toate modurile posibile. 2 Demonstraie. a) Dac y0 (, 1) (1, ) , atunci y0 > 1 t a 2 tiv rezult c yn > 1 pentru orice n N. Cum yn+1 yn = 2yn a a (2yn + 1) (yn 1) > 0, rezult c (yn ) este strict cresctor. Prin urm a a a 1 l = lim yn ; dac l R, rezult l = 2l2 1, deci l 1, , imposibi a a n 2 s a a yn > 1, n N i (yn ) este strict cresctor. Conchidem c lim yn = .
n

b) Fie (yn ) convergent. Dac lim yn = l, rezult l = 2l 1, deci l a a


n

Dac l = 1, din relaia de recurena rezult yn+1 1 = 2 (yn 1 a t t a deci |yn+1 1| = 2 |yn 1| |yn + 1|. Presupunnd c yn 6= 1 pentru ori a |yn+1 1| s avem c lim a = lim 2 |yn + 1| = 4 i din criteriul raportu n |yn 1| n lim |yn 1| = , absurd. Aadar, exist n0 N cu yn0 = 1 i atunci yn = s a s n 1 n cazul l = scriem relaia de recurena sub forma 2yn+1 +1 = (2yn + t t 2 i continum ca mai sus. s a Presupunem acum c exist n0 N astfel nct (yn )nn0 este cons a a 1 a relaia de recurena deducem yn0 1, . Cum yn0 = 1 implic yn = t t 2 a a rezult c (yn )nN este convergent cu limita egal cu 1. La fel, dac y a a 1 1 s deducem yn = , n n0 i lim yn = . n 2 2 c) Se demonstreaz prin inducie matematic. a t a a d) 1. Cum 1 M , rezult c M 6= . Folosind a) i b), y0 M implic a a s 1 unui n0 astfel nct (yn )nn0 este constant (egal cu 1 sau ). Dac yn = a 2 2 2 2 din sistemul yn0 = 2yn0 1 1, yn0 1 = 2yn0 2 1, . . . , y1 = 2y0 1, di n aproape, gsim un numr finit de valori pentru y0 . La fel se analiz a a 1 yn = , n n0 . 2 1 2. y0 M implic faptul c yn = 1, n n0 sau yn = , n n0 (n0 , a a 0 2 naturale arbitrare, dar fixate). n si Folosind c) y0 M [1, 1], deducem cos (2 arccos y0 ) = 1, de 2k cos n0 ; k Z . Folosind periodicitatea funciei cosinus rezult c y0 t a a 2 1 a din yn = , n n0 , deducem c y0 B. n concluzie, M = A B. 0 r 2 x 1 1 e) Folosind formula cos = + cos x, x R, deducem c A a 2 2 2 mulimile au ecare cte 2n0 elemente, rezult c A = A0 . La fel, B = B 0 t a a

Bibliografie 1. Gh. Eckstein et al. - Olimpiadele si concursurile de matematic IX a tura Brchi, 2005.

Olimpiada Internaional de Matematic t a a "B. O. Zhautykov" Ediia I, Alma-Ata, 2005 t


Enunuri i Soluii juniori t s t

Prima zi 13 ianuarie 2005 1. Pe o tabl 9 9 sunt marcate 40 celule. O linie orizontal sau a a format din 9 celule se spune c este bun , dac ea are mai multe celu a a a a dect nemarcate. Care este cel mai mare numr de linii bune (orizontale a s pe care-l poate avea tabla?

2. Artai c numrul 22n+2 + 2m+2 + 1, unde m, n Z i 0 m a t a a s ptrat perfect dac i numai dac m = n. a as a

3. Fie A o mulime format din 2n puncte dintr-un plan astfel nc t a trei dintre acestea nu sunt coliniare. Artai c pentru orice dou punct a t a a a, b A exist o dreapt ce mparte A n dou submulimi coninnd n pun a a a t t i astfel nct a i b se afl de pri diferite n raport cu aceast dreapt. s s a at a a

A doua zi 14 ianuarie 2005 4. Pentru orice numere a, b, c reale i pozitive, artai inegalitatea s a t c a b + + 1. a + 2b b + 2c c + 2a 5. Cercul nscris triunghiului ABC este tangent laturii AB n punc M este mijlocul acestei laturi. Artai c M , centrul cercului nscris a t a segmentului [CD] sunt coliniare.

6. Determinai numerele prime p, q mai mici ca 2005 i astfel nct t s divide cu q, iar q 2 + 4 se divide cu p. * * * 1. Deoarece sunt marcate 40 celule i o linie are cel s puin 5 celule marcate, rezult c putem avea cel mult 8 t a a linii orizontale bune i cel mult 8 linii verticale bune. n s total, putem avea cel mult 16 linii bune. Alturat este dat a un exemplu de tabl cu 16 linii bune. Deci 16 este numrul a a maxim de linii bune. 2 2. Dac m = n, atunci 22n+2 + 2m+2 + 1 = 2n+1 + 1 . a n+1 2 Dac m < n, atunci avem 2 a < 22n+2 + 2m+2 + 1 < n+1 2 2 + 1 i, deci, numrul examinat nu poate fi un ptrat perfect. s a a Fie acum n < m 2n. S presupunem c ar exista x natural astfel a a 22n+2 + 2m+2 + 1. Observm c x este impar i x > 1. Scriem relaia pre a a s t forma (x 1) (x + 1) = 2m+2 22nm + 1 .

Deoarece (x 1, x + 1) = 2, numrul 2m+1 divide una dintre parantezele a brul stng, iar cealalt nu va fi mai mic dect 2m+1 2. Cum m 2n a a 22nm+1 2, urmeaz c a a

2m+1 2 22nm+3 2 = 4 22nm+1 2 > 22nm+1 + 2. n consecina, (x 1) (x + 1) 2m+1 2m+1 2 > 2m+2 22nm + 1 , ce t trazice (). Aadar, nici n acest caz numrul dat nu-i ptrat perfect. s a a

3. Fie dab dreapta determinat de punctele a i b. Pe segmentul de e a s a i b alegem un punct O astfel nct orice dreapt ce trece prin O i nu c s a s dab conine cel puin un punct din A; pentru c A este mulime finit, t t a t a dreapt exist. a a Notm cu d dreapta obinut rotind dab cu unghiul (n sens contra a t a ceasornic, de exemplu); avem d0 = d = dab . Dac dab mparte A \ {a, a submulimi cu n 1 elemente fiecare, atunci rotind-o cu un unghi sufici t obinem dreapt cutat. t a a a Presupunem c ntr-un semiplan determinat de d mpreun cu a se afl a a a din A, iar n cellalt mpreun cu b se afl 2nm (m 6= n). Dac unghiul de a a a a fi suficient de aproape de , atunci situaia se inverseaz: ntr-un semiplan t a cu a se afl 2n m puncte din A, n cellalt mpreun cu b sunt m punct a a a Deoarece trecerea de la perechea (m, 2n m) la (2n m, m) prin rota lui O se face printr-o compunere de transformri de tipul (x, y) (x 1, a exista o valoare 0 pentru care corespunde perechea (n, n); d0 este dreapt

4. Fie a + 2b = x, b + 2c = y, c + 2a = z. Rezolvnd n raport cu a, b, c 4 1 2 4 1 2 4 1 2 a = z + x y, b = x + y z, c = y + z x. Inegalitatea dat a 9 9 9 9 9 9 9 9 9 4 y z x 1 z x y 2 + + + + + 3 1. 9 x y z 9 x y z 9

Cum sumele din paranteze sunt 3 (inegalitatea mediilor), deducem c u a galitate este adevrat. a a

5. Dac M coincide cu D, atunci se arat uor c BC = AC, adic a a s a a este isoscel. n acest caz [CD este bisectoarea unghiului C i coliniaritatea s puncte este evident. a Dac M 6= D, fie E punctul diametral opus lui D pe cercul nscris a CE AB. Se tie c F este punctul de tangena cu latura AB a cerc s a t scris corespunztor acestei laturi i c M este mijlocul segmentului [F D] a s a demonstrai!). Atunci M I este linie mijlocie n 4DEF (I noteaz centr t a nscris). Rezult c M I k CF i, n nal, M I trece prin mijlocul segment a a s

6. Dac p = q, atunci aceste numere divid 4 i, deci, p = q = 2. n a s obinem soluia (p, q) = (2, 2). t t S determinm soluiile (p, q) cu p 6= q. Vom spune c perechea (x, y) a a t a naturale este admis , dac ndeplinete condiiile: (A) x, y sunt relati a a s t x y; (B) x2 + 4 se divide cu y i y 2 + 4 se divide cu x. Observm c s a a admis este format din numere impare. a a

Artm mai nti c, dac (x, y) este o pereche admis, atunci perechea aa a a a este de asemenea admis. n acest scop, fie z = y 2 + 4 /x. Deoarece x a y 2 + 4, rezult c y < z. Apoi, dac d divide y i z, atunci d divide y i y a a a s s d divide 4, ceea ce conduce la d = 1. Aadar, perechea (y, z) ndeplinet s s y2 y2 + 8 + 4 (A). Evident, z divide y 2 + 4; pe de alt parte, z 2 + 4 = a x2 unde numrtorul se divide cu y, care este relativ prim cu x. n concluzie aa (y, z) este admis. a S considerm irul (ai )i0 definit de a0 = a1 = 1 i ai+2 = a2 + 4 / a a s s i+1 Din ceea ce s-a stabilit mai sus, rezult c perechile (ai , ai+1 ), i N, sunt a a S artm acum c orice pereche admis este de forma (ai , ai+1 ) pentru a aa a a i 0. S presupunem c exist perechi admise ce nu-s de aceast form a a a a as perechea de acest fel cu suma x+y minim. Cum x2 +4 = ay i y 2 +b= bx a s s 2 x2 x + 8 + 4 2 relativ prime (se arat ca mai sus!), obinem y + 4 = a t a2 2 bx i a + 4 se divide cu x. Dac a x, atunci (a, x) este pereche s a datorit minimalitii, avem (a, x) = (ai , ai+1 ). Rezult c (x, y) = (ai+ a at a a contradicie cu presupunerea fcut. Dac a > x, atunci a x + 2 i cum t a a a s 2 avem i y x + 2, putem scrie x2 + 4 = ay (x + 2) = x2 + 4x + 4, s contradicie. t n sfrit, s scriem termenii irului (ai )i0 ce nu depesc 2005; ac s a s as a0 = a1 = 1, a2 = 5, a3 = 29, a4 = 169 i a5 = 985. Sunt numere prime s 29. Soluiile problemei sunt perechile (p, q) {(2, 2) , (5, 29) , (29, 5)}. t Enunuri i Soluii seniori t s t Prima zi 13 ianuarie 2005 1. Artai c ecuaia x5 + 31 = y 2 nu are soluii ntregi. a t a t t

2. Fie r un numr real astfel nct pentru orice ir (an )n1 de nu a s pozitive are loc inegalitatea oricare ar fi m N . Artai c r 4. a t a

a1 + a2 + + am+1 ram ,

3. Fie SABC o piramid triunghiular regulat, i.e. SA = SB = SC a a a BC = AC. Determinai mulimea punctelor D (D 6= S) din spaiu t t t condiia t |cos A 2 cos B 2 cos C | = 3, unde X = ]XSD pentru X {A, B, C}.

A doua zi 14 ianuarie 2005 4. Pentru orice numere a, b, c, d reale i pozitive, artai inegalitatea s a t d a b 4 c + + + . a + 2b b + 2c c + 2d d + 2a 3 5. Se spune c punctul X interior unui patrulater (convex) este obse a latura Y Z dac piciorul perpendicularei din X pe dreapta Y Z aparine se a t

[Y Z]. Un punct interior patrulaterului se spune c este k-punct dac este a a din exact k laturi ale patrulaterului (de exemplu, orice punct din inter ptrat este 4-punct). Artai c, dac n interiorul unui patrulater exist u a a t a a a atunci exist i un k-punct pentru k {2, 3, 4}. as

6. Determinai numerele prime p, q mai mici ca 2005 i astfel nct t s divide cu q, iar q 2 + 8 se divide cu p. * * * 1. Dac x este par, atunci x5 + 31 3 (mod 4) i nu poate fi ptr a s a Urmeaz c x este impar i, deci, y este par. Mai mult, x5 1 (mod a a s x 1 (mod 4). S scriem ecuaia dat n forma a t a x5 + 25 = y 2 + 1.

Partea stng se divide cu x + 2 i x + 2 3 (mod 4) va avea un divizo a s tipul 4l + 3. Dar, conform lemei de mai jos, numrul impar y 2 + 1 are div a numai de tipul 4m + 1. n concluzie, n ipoteza c ecuaia dat ar avea solu a t a ajungem la o contradicie. t

Lem. Dac y 2 +1 admite un divizor prim impar p, atunci p este de tip a a

ntr-adevr, avem y 2 1 (mod p). n conformitate cu mica teor a Fermat, avem i y p1 1 (mod p). Atunci s (p1)/2 (1)(p1)/2 1 (mod p) . y p1 y 2 Ultima congruena spune c (p 1) /2 este par, adic p 1 (mod 4). t a a

2. Notm bm = a1 + a2 + + am . Atunci, irul (bn )n1 este strict c a s verific relaia bm+1 r (bm bm1 ), m N . Pentru cm = bm / (bm1 a t relaie devine cm+1 cm + 1 cm r.Deci, pentru orice m N avem t 1 cm+1 + r. cm 1 Utiliznd inegalitatea c + 2 (c > 0), obinem t c 1 1 1 + cn + + + c2 + n r cn+1 + cn cn1 c1 1 2 (n 1) , n N . cn+1 + 2 (n 1) + c1 2 n1 n1 , n N , i rezult c r 4 (numrul s a a a Deci r 4 poate f n n aproape dorim de 1). 3. Fie eX versorul vectorului SX, X {A, B, C, D}. Atunci, innd t cos = (eD , eX ) (produs scalar), X {A, B, C}, condiia din enun se scr t t 1 (eD , eA ) 2 (eD , eB ) 2 (eD , eC ) = eD , 1 eA 2 eB 2 eC 3 3 3 3 3 3

2 2 1 a eB + eC eA . Vectorul f este unitar, cci 3 3 3 1 2 |f |2 = (f, f ) = 4eB + 4e2 + e2 + 8 (eB , eC ) 4 (eA , eB ) 4 (eA , eC C A 9 1 = (9 + 8 cos 4 cos 4 cos ) = 1, 9 a unde = ]ASB. Atunci, condiia |(eD , f )| = 1 este echivalent cu t vectorii eD i f sunt coliniari. s Fie SH nlimea piramidei i F simetricul punctului A n raport cu at s lalele urmtoare arat c vectorii SF i f sunt coliniari: a a a s 1 AB = SB SA, AC = SC SA, AH = AB + AC ; 3 2 2 AB + AC = SB + SC 2 SA ; AF = 2 AH = 3 3 1 SF = SA + AF = 2 SB + 2 SC SA . 3 Ca urmare, locul geometric cutat este dreapta dSF din care se exclude p a Notm f = a 4. Notm a A=

c d a b + + + , a + 2b b + 2c c + 2d d + 2a b + 2c c + 2d d + 2a a + 2b B= + + + , a + 2b b + 2c c + 2d d + 2a a+c b+d a+c b+d C= + + + . a + 2b b + 2c c + 2d d + 2a Constatm uor c 2B + C = 5A + 4. Conform inegalitii mediilor, ave a s a at 1 4 1 (u, v > 0), obinem t Tinnd seama de inegalitatea + u v u+v 4 8 4 + (b + d) C (a + c) a + 2b + c + 2d b + 2c + d + 2a 3 x (ultima inegalitate se obine lund x = a + c i y = b + d n t s + x + 2y y iar aceast inegalitate se stabilete astfel: a s 3xy 3xy x y 2x2 + 2xy + 2y 2 =1 2 1 + = 2 x + 2y y + 2x 2x + 5xy + 2y 2 2x + 5xy + 2y 2 9xy 8 4 Aadar, 5A + 4 8 + , deci A , q.e.d. s 3 3 5. S numim zon de observaie a laturii XY a patrulaterului convex s a a t a a notat ZXY , care este mrginit de segmentul [XY ], perpendicularele a duse n extremiti i care este situat n acelai semiplan cu patrulaterul at s a s un punct interior patrulaterului este observabil din latura XY dac i n as aparine ZXY . t S examinm diferitele cazuri ce apar: a a a) Dac patrulaterul nu are unghiuri obtuze, atunci el este dreptun a numai 4-puncte.

b) Presupunem c patrulaterul ABCD are un singur unghi obtuz, a Atunci zonele de observare ale laturilor BC i CD acoper patrulateru s a 1-puncte nu exist. a b s b c) Fie ABCD cu exact dou unghiuri obtuze i vecine, anume B i C a s caz patrulaterul este situat n ntregime n ZAD , dar i n ZAB ZBC s urmare, nu exist 1-puncte. a b s d) Fie ABCD cu exact dou unghiuri obtuze i opuse, anume B i D a s patrulaterul este situat att n ZAB ZBC ct i n ZAD ZCD i nu va avea s s b e) Fie ABCD cu trei unghiuri obtuze i fie A unghiul su ascuit. At s a t secia ZBC ZCD este situat n patrulater i formeaz paralelogramul LM t a s a n ABCD, vrfurile sunt notate n sensul acelor de ceasornic). Fie {E} = i {F } = DM AB. Se constat uor c 4ABE ZAB i 4AF D ZAD s a s a s M este 4-punct, punctele segmentului deschis (M F ) sunt 3-puncte, iar cele patrulaterului AF M E sunt 2-puncte.

6. Procedm ca i n cazul problemei J6 (problema 6 de la juniori, a s mai sus). Dac p = q, atunci numerele p i q divid 8 i, deci, p = q = 2, adic a s s a soluie a problemei. t S determinm soluiile (p, q) cu p 6= q. O pereche (x, y) de numere n a a t numete admis dac: (A) x, y sunt relativ prime i x y; (B) x2 + 8 se s a a s y, iar y 2 + 8 se divide cu x. Mai nti, observm c o pereche admis es a a a din numere impare. Apoi, ca i n problema J6 se s a a a 2 demonstreaz c, dac pereche admis, atunci i perechea y, y + 8 /x este admis. Acest re a s a urmtoarele consecine: a t 1) Dac (ai )i0 este irul dat de a0 = a1 = 1 i ai+2 = a2 + 8 /a a s s i+1 atunci orice pereche (ai , ai+1 ) este admis; a 2) Dac (bi )i0 este irul dat de b0 = 1, b1 = 3 i bi+2 = b2 + 8 / a s s i+1 atunci orice pereche (bi , bi+1 ) este admis. a S artm acum c orice pereche admis are forma (ai , ai+1 ) sau (bi , bi a aa a a un anumit indice i 0. Presupunem c ar fi adevrat situaia contrar a a a t as perechea minimal (n raport cu suma x + y) care nu-i de nici una dint a 2 precedente. Cum x2 8 = ay, y + 8 = bx i a, x sunt relativ prime + s 2 2 2 x x + 16 + 8 a + 8 y2 + 8 = = bx i a2 + 8 se divide cu x. Da s a2 atunci (a, x) este pereche admis i, datorit minimalitii, avem (a, x) = as a at sau (a, x) = (bi , bi+1 ); rezult c (x, y) = (ai+1 , ai+2 ) sau (x, y) = (bi+ a a contradicie cu presupunerea fcut. Dac a > x, atunci x2 + 8 = ay t a a a a2 + 4x + 4, de unde x = 1, a = y = 3, din nou contradicie. t S scriem acum termenii irurilor (ai )i0 i (bi )i0 ce nu depesc 2 a s s as a1 = 1, a2 = 9, a3 = 89, a4 = 881; b0 = 1, b1 = 3, b2 = 17, b3 = 99, b4 = 5 aceste numere sunt prime numai 3, 17, 89, 881 i 577. Ca urmare, soluiile s t sunt perechile (p, q) {(2, 2) , (3, 17) , (17, 3) , (89, 881) , (881, 89)}.

Soluiile problemelor propuse n nr. 1 / 20 t


Clasele primare

P.84. Aflai num rul m stiind c 47 este mai mare dect m 14 cu t a a (Clasa I ) nv. Maria R Soluie. m 14 = 47 28; m 14 = 19; m = 14 + 19; m = 33. t P.85. ntr-un co sunt 6 mere, iar n altul sunt 5 pere. Cum pot pri s mere si pere astfel nct nici un co s nu r mn gol? s a a a (Clasa I ) Veronica Corbu, e Soluie. Fiecare copil primete cte o par, unul dintre ei va primi p t s a Merele pot fi mprite astfel: fiecare copil primete cte un mr i un m at s a s n co sau unul dintre copii primete dou mere care trebuie s se afle n s s a a P.86. n urm cu 4 ani, cnd tat l avea 29 de ani, s-a n scut fiul. Sor a a a avea atunci 2 ani, iar acum este de trei ori mai mare. Mama este de pat mare dect aceasta. Ci ani are acum fiul? t (Clasa a II-a) nv. Oana-Maria L Soluie. Deoarece fiul s-a nscut n urm cu 4 ani, acesta are acum v t a a ani. P.87. Un acrobat cade pe o plas elastic de la o anumit n lime s a a a at dup ce atinge plasa la jum tatea distanei dintre plas si locul de und a a t a anterior. Stiind c atinge de 3 ori plasa si c ultima oar s-a ridicat la n a a a 2 m, iar plasa este montat la 2 m deasupra solului, s se afle distana a a t de unde a c zut prima dat pn la sol. a a a (Clasa a II-a) Andrei Stativ, a Soluie. A treia oar cade de la 2m + 2m = 4m faa de plas. A doua t a t a de la 4m + 4m = 8m faa de plas. Prima oar a czut de la 8m + 8m = t a a a de plas. Distana de la locul de unde a czut pn la sol este de 16m + 2 a t a a P.88. Tr iau odat o bab si un moneag; moul avea 100 ani, ia a a a s s amndoi erau albi ca iarna si triti ca vremea cea rea pentru c erau s s a spune c ar fi avut un copil pe cnd vrsta babei era jum tate din jum tate a a a a vrstei moneagului si c acesta ar fi plecat n lume cnd vrsta mone s a s de dou ori ct vrsta aceea a babei. Fiul nu s-a mai ntors. Ce vrst ave a a a plecat n lume? (Clasa a III-a) nv. Ileana Ro Soluie. La naterea fiului, baba avea: 100 : 2 : 2 = 25 (ani). La nat t s s moul avea: 25 + 10 = 35 (ani). Fiul a plecat n lume la vrsta de 50 35 = s P.89. La un concurs de biciclete, triciclete si mainue (cu patru roi s t t Bogdan num r roile vehiculelor si observ c sunt 34. Cte vehicule a a t a a fiecare fel? G sii toate posibilit tile, stiind c num rul vehiculelor de fie a t a a a dep sete 5. a s (Clasa a III-a) nv. Doinia Sp t Soluie. Nu putem avea un numr nepereche de triciclete deoarece num t a de roi este numr pereche. Putem avea dou sau patru triciclete. Dac a t a a a

triciclete, nseamn c numrul roilor bicicletelor i mainuelor este 34 a a a t s s t 34 6 = 28. n acest caz avem 4 biciclete i 5 mainue. Dac avem 4 s s t a atunci numrul roilor bicicletelor i mainuilor este 34 4 3 = 34 12 a t s s t acest caz putem avea: o biciclet i 5 mainue sau 3 biciclete i 4 mai as s t s s biciclete i 3 mainue. s s t

P.90. Lungimea laturii unui p trat este de 17 m. O persoan pleac a a vrf al p tratului si, mergnd n acelai sens pe laturile acestuia, parcurge a s de 637 m. Din punctul n care a ajuns se ntoarce si parcurge 773 m. A distana se va situa n final persoana faa de punctul de plecare. t t (Clasa a III-a) Oxana Pascal, e Soluie. Perimetrul ptratului este 4 17m = 68m. t a A 17 m Considerm primul sens de parcurs de la A ctre B. Avem a a 10 68m = 680m = 637m + 43m. nseamn c persoana a a a parcurs conturul n ntregime de 9 ori, iar din al zecelea contur a parcurs 68m 43m = 25m, oprindu-se n punctul M . n punctul M se ntoarce i parcurge 773m. s Avem 773m = 680m + 68m + 25m, ceea ce nseamn c a a a parcurs conturul n ntregime de 11 ori, iar din al doisprezecelea contur a parcurs 25m, adic M B + BA. n a 8m nal cltorul ajunge din nou n punctul de unde a plecat aa B iniial. Distana ceruta este de 0m. t t

P.91. Se mpart dou numere naturale. Dac mp ritorul, ctul si a a a t trei numere consecutive cu suma 30, s se afle demp ritul. a a t (Clasa a IV-a) Vasile Solcanu, Bogdneti a s Soluie. Cele trei numere consecutive sunt 9, 10 i 11. Formula m t s rest este D = I C + R, R < I. Se observ c 11 nu poate fi mprit a a at D = 10 11 + 9 = 119 sau D = 9 11 + 10 = 109.

P.92. Observ regula si completeaz , apoi verific rezultatele g site: 2+ a a a a 2 + 4 + 6 = 4 + 4 + 4; 2 + 4 + 6 + 8 = 5 + 5 + 5 + 5; 2 + 4 + 6 + 2 + 4 + 6 + + 14 = ; . . . ; 2 + 4 + 6 + + (a + a) = . (Clasa a IV-a) Valeria Gheorghia, e t Soluie. 2 + 6 = 3 + 3; 2 + 4 + 6 = (2 + 6) + 4 = 8 + 4 = 4 + 4 + t 6 + 8 = (2 + 8) + (4 + 6) = 10 + 10 = 5 + 5 + 5 + 5. Se observ c term a a se repet n membrul drept este jumtatea sumei dintre primul termen a a termen din membrul stng. Acest termen se repet de un numr de o a a numrul termenilor din membrul stng. n suma 2 + 4 + 6 + + 12 avem a iar 2+12 = 7+7. Obinem: 2+4+6+ +12 = | + 7 + + 7; 2+4+6+ t 7 {z }
6 ori 7 ori a ori

8 + 8 + + 8; 2 + 4 + 6 + + (a + a) = (a + 1) + (a + 1) + + (a + 1 | {z } {z |

P.93. O foaie de hrtie dreptunghiular se ndoaie de-a lungul de 6 ori, a se 7 benzi egale si suprapuse. Dreptunghiul obinut se ndoaie de-a latu t rezultnd n final un p trat cu perimetrul de 12 cm. S se afle perimetru a a

ghiului iniial. t (Clasa a IV-a) Petru Asa Soluie. Dup prima serie de ndoiri se obine un dreptunghi care are t a t egal cu lungimea dreptunghiului iniial, iar limea este egal cu latura a t at a obinut n final, a crui latur are lungimea 12m : 4 = 3m. Deoarece dre t a a iniial a fost ndoit de-a lungul de 6 ori, nseamn c limea lui este 7 3 t a a at Ultimele 9 ndoiri sunt echivalente cu ndoirile de-a latul ale dreptunghiu de unde rezult c lungimea dreptunghiului iniial este 10 3m = 30m. P a a t dreptunghiului iniial este 2 (30m + 21m) = 2 51m = 102m. t

Clasa a V-a

V.56. Se consider num rul A = 5 + 52 + 53 + + 52005 . a a a) S se arate c A nu este p a a atrat perfect. b) S se g seasc 5 divizori mai mici dect 100 ai lui A. a a a Andrei Tofan, . . . 5, ns A / 52 , prin urmare A nu poate fi ptr . Soluie. a) Evident c A . t a a . a

Cum 3905 = 5 11 71, numrul A admite ca divizori pe 1, 5, 11, 55, 71. a V.57. Aflai restul mp ririi prin 47 a num rului N = 1268 99 {z . 9 . t a t a | .. }
2005 cifre 2005 cifre

b) Suma A are 2005 termeni, pe care i vom grupa cte 5: A = 5 + 52 + 53 + 54 + 55 + + 52001 + 52002 + 52003 + 52004 + 5200 = 5 + 52 + 53 + 54 + 55 1 + 55 + + 52000 = 3905 1 + 55 + +

N + 1 se divide cu 47. Atunci restul mpririi lui N prin 47 este 47 1 = at V.58. Aflai numerele naturale x, y, z cu proprietatea c t a 24x+1 + 23y+1 + 22z+1 = 9248.

Alexandru Negrescu, elev, Soluie. Observm c N + 1 = 1269 00 {z . 0 = 1269 102005 = 47 27 1 t a a | .. }

Cristian - Ctlin Bude a a t Soluie. Deoarece 924810 = 100100001000002 , egalitatea din enun r t torit unicitii scrierii unui numr n baza 2, la faptul c (4x + 1, 3y + 1 a at a a {(13, 10, 5) , (13, 5, 10) , (10, 13, 5) , (10, 5, 13) , (5, 10, 13) , (5, 13, 10)}. Cerce re caz n parte, obinem soluii numai n prima i penultima situaie, anume t t s t {(1, 3, 6) , (3, 3, 2)}. V.59. Dac a1 a2 . . . an 2 b1 b2 . . . bn = c1 c2 . . . cn 2 , s se arate c a a a
2 2

Petru Asa Soluie. Notm A = a1 a2 . . . an , B = b1 b2 . . . bn , C = c1 c2 . . . cn . Obse t a i nc dou relaii similare. Astfel, egalitatea de demonstrat revine la s a a t A2 (10n + 1)2 B 2 (10n + 1)2 = C 2 (10n + 1)2 , a1 a2 . . . an a1 a2 . . . an 2 = (A 10n + A) = A2 (10n + 1)
2 2

a1 a2 . . . an a1 a2 . . . an 2 b1 b2 . . . bn b1 b2 . . . bn = c1 c2 . . . cn c1 c2 . . . c

care se obine nmulind ambii membri ai ipotezei cu (10n + 1) . t t


99

de forma M4 + 1. Din aceste dou observaii, rezult c U2 79 a t a a

V.60. S se determine ultimele dou cifre ale num rului 79 . a a a Artur Bluc, aa a Soluie. Urmrind ultimele dou cifre ale numrului 7n (pe care le t a a a U2 (7n )), observm c U2 74k = 01, U2 74k+1 = 07, U2 74k+2 = 49, U2 a a 43. Pe de alt parte, cum 9 = M4 + 1, atunci orice putere a lui 9 (deci a s
99

= 07.

Clasa a VI-a

VI.56. Determinai, n funcie de num rul ntreg x, cel mai mare divi t t a al numerelor 2005x + 2 si 2006x + 3. Tamara C Soluie. Notm d = (2005x + 2, 2006x + 3). Vom aplica algoritmul t a dac a = bq + r, a, b, q Z, r < b, atunci (a, b) = (b, r). Observm c a a a 2006x + 3 = (2005x + 2) 1 + (x + 1) ; 2005x + 2 = (x + 1) 2005 + (2003) ,

prin urmare (2005x + 2, 2006x + 3) = (2005x + 2, x + 1) = (x + 1, 2003). este numr prim, deci: a . a) Dac x + 1 . 2003, atunci d = 2003; a . . . b) Dac x + 1 / 2003, atunci d = 1. a . VI.57. Un vnz tor de autoturisme scade procentul beneficiului s u de a a 20% din valoarea vnz rilor. Datorit sc a a aderii preurilor, crete valoarea v t s Aflai procentul cu care a crescut valoarea vnz rilor, stiind c beneficiu t a a cu 10%. Marius Fa Soluie. Notm cu x valoarea iniial a vnzrilor i cu p procentul d t a t a a s a valorii vnzrilor. Iniial, beneficiul global era 25% din x, iar n final ace a t 20% din ((100 + p) % din x). Conform ipotezei, putem scrie ecuaia: t 20 100 + p 110 25 x= x p = 37, 5 . 100 100 100 100 Prin urmare, vnzrile au crescut cu 37, 5% . a VI.58. Se aeaz cifrele 2, 3, 4, 5, 6, 8, 9 ntr-o ordine oarecare ob s a num r A. Se aeaz apoi aceleai cifre n alt ordine, obinnd num rul B a s a s a t a se arate c A nu se divide cu B. a Cristian - Ctlin Bude a a Soluie. Stim c un numr natural i numrul dat de suma cifrelo t a a s a acelai rest la mprirea prin 9. Cum suma cifrelor lui A, respectiv B, es s at 4 + 5 + 6 + 8 + 9 = 37, atunci A = 9a + 1, B = 9b + 1, cu a, b N. . Presupunem prin absurd c A . B, atunci A > B i A = B c 9a + 1 = a . s

cu 1 < c < 5 (deoarece A 9865432, iar B 2345689). Obinem t 9 (a bc) = c 1, relaie imposibil deoarece c {2, 3, 4} i n mem t a s nu putem avea multiplu de 9. n concluzie A nu se poate divide cu B.

b VI.59. Fie 4ABC cu m(B) = 120 . Dac mediana [BM ] este perp a pe BC, ar tai c AB = 2BC. a t a A Bogdan Posa, elev, Motru (Gorj) Soluie. Fie T mijlocul lui [AB]. Cum [M T ] este linie t 1 mijlocie n 4ABC, rezult c M T = BC i M T k BC. a a s 2 ns M B BC, deci 4BM T va fi dreptunghic n M , cu a 1 \ m(M BT ) = 120 90 = 30 . Prin urmare, M T = BT , 2 adic BC = BT , de unde concluzia. a

M T

VI.60. Fie 4ABC si punctele E (AB), F (AC) si M (BC) a AE = EB, iar ntre 4AEF si 4EF M s existe o congruena. S se ara a t a a) F este mijlocul lui [AC]; b) [AM ] este median sau n lime. a at Ioan Scleanu a a Soluie. a) Fie {O} = AM EF , iar T , R proiecit t A ile pe EF ale punctelor A, respectiv M . Cum ntre 4AEF i 4EF M exist o congruena, cele dou tris a t a unghiuri vor fi echivalente, deci 1 1 O R F SAEF = SMEF EF AT = EF M R AT = M R. E T 2 2 ns AT k M R i rezult c AT M R este paralelogram. a s a a De aici, O este mijlocul lui [AM ], adic [EO] este linie a mijlocie n 4ABM i atunci EO k BC. Cu reciproca s teoremei liniei mijlocii n 4ABC, obinem c F este t a mijlocul lui [AC].

b) Deosebim trei situaii: t [ \ i) AEF EF M ; atunci AE k F M i, cu reciproca teoremei liniei mijlo s c M este mijlocul lui [BC], adic [AM ] este median. a a a [ \ ii) AEF F EM ; atunci 4AET 4M ER (C.U.), deci ET = ER, p T = R. Rezult n acest caz c [AM ] este nlime. a a at [ EM F ; atunci, dac AF E F EM vom obine ca la i) c \ [ \ iii) AEF a t a \ median, iar dac AF E EF M , [AM ] va fi nlime. a a [ at

Clasa a VII-a

VII.56. Fie x, y R cu x2 2y = y 2 + xy = 4. S se arate c x2 a a Gigel Buth, Sa Soluie. Prin scderea relaiilor din enun obinem c x2 y 2 = y t a t t t a 1 2 a a nlocuind y = x 4 , gsim c 2 1 3 x2 y 2 = x + 2x2 4x 8 . 2 Pe de alt parte, eliminnd direct pe y ntre relaiile date, obinem a t t x3 + 2x2 4x 8 = 4x.

Din (1) i (2) rezult c x2 y 2 = s a a

a VII.59. Fie 4ABC si A0 mijlocul lui [BC]. Dac D (AC), BD A 1 1 1 si paralela prin F la BC taie AC n E, s se arate c a a = + DE AD CE Claudiu - Stefan P Soluia 1. Fie {E 0 } = EF AB i D0 [BF ] astfel t s A nct DF = D0 F . Cum [AA0 ] median n 4ABC i a s 0 0 0 0 EE k BC, atunci EF = E F . Rezult c DED E a a este paralelogram, deci [D0 E 0 ] este paralel i congruent s D BE 0 0 0 a cu [DE]. De aici, 4BE D 4BAD, adic = F E E BA 0 0 0 0 0 BE CE ED CE ED . ns a = , deci = , de unde AD BA CA AD CA D 0 0 1 ED ED = = . Obinem succesiv: t AC AD CE AD CE B A 1 AD + DE + CE 1 AC = = AD CE ED AD CE ED 1 1 DE 1 1 1 1 1 = + + = + + . ED CE AD CE AD DE CE AC AD

de unde concluzia problemei. a a VII.58. Fie a, b R astfel nct mediile aritmetic , geometric si + ale lor s fie laturi ale unui triunghi dreptunghic. Aflai sinusul celui mai a t unghiurile triunghiului. Romana Ghia i Ioan Gh t t s Soluie. Cum ma mg mh , ipotenuza va fi ma , iar cel mai t unghiuri va fi cel ce se opune lui mh . Cum ma , mg , mh sunt laturi ale un dreptunghic, n ipoteza nerestrictiv a > b avem: a 2 2 2 2 a+b 2ab (a + b) 2 2 2 = ab + ma = mg + mh ab = 2 a+b 4 ( 2 2 2 (a b) 4a b 2 2 2 2 = 2 a b = 4ab a 4ab b = 0 a = b 2 4 (a + b) Evident, convine doar a = b 2 + 5 . Sinusul unghiului dorit este 4b2 2 + 5 4ab mh 51 = sin = . 2 = 2 = ma 2 (a + b) b2 3 + 5

1 4x, adic x2 2x = y 2 . a 2 VII.57. Fie x (0, 1), iar n N \ {0, 1}. Ar tai c nx2 + 2n > n + a t a Ion Vian s Soluie. Avem succesiv: t h n n2 2n (1 + x) = (2 1 x) 2n1 + 2n2 (1 + x) + + 2 (1 + x) + (1 + h > (1 x) (1 + x)n1 + (1 + x)n2 (1 + x) + + (1 + x)(1 + x)n2 + (1 + = (1 x) n (1 + x)n1 (1 x) n (1 + x) = n 1 x2 ,

VII.60. n cercul C se consider coardele [AM ] si [AN ] astfel nct A a 1) S se determine mulimea punctelor X C ce ndeplinesc condi a t t AX AN . 2) n ce caz mulimea g sit este un arc de cerc? t a a Temistocle B Soluie. 1) Considerm cercurile de centru A i raze [AM ] i [AN ] t a s s intersecteaz a doua oar cercul C n punctele M 0 i respectiv N 0 . Pun a a s satisfac condiia AM AX AN se afl n coroana determinat de aces t a a Punctele X C ce satisfac aceast condiie se afl pe arcele cercului C in a t a de coroan. a

AF = k, k (0, 1). Din teorema lui AC BA0 CD AF t = 1 n 4AA0 C cu transversala B F D, obinem BC DA F A0 2k AE 2k AF = . Deoarece 4AF E 4AA0 C, atunci = i AE = s F A0 1k AC 1+k 1k k (1 k) b i CE = AC AE = s Prin urmare, DE = AE AD = 1+k 1+k 1+k 1 1+k din enun este echivalent cu t a = + + 1, care se verific i a k (1 k) k 1k Soluia 2. Fie AC = b i t s

N O

N M O N M A M N

\ Dac O, centrul cercului C, este exterior unghiului M AN , atunci mul a \ tat este M N M 0 N 0 . Dac O este interior unghiului M AN , atunci aceast a a

este M N 0 M 0 N (acest caz se reduce la precedentul prin nlocuirea coard [AN 0 ]). 2) n ambele cazuri, obinem un arc de cerc atunci cnd [AN ] este dia t

Clasa a VIII-a

VIII.56. S se determine x, y, z R pentru care x3 y 3 + z 3 = 8, x a Andrei - Sorin Cozma, Soluie. Este cunoscut identitatea t a (a + b + c) = a3 + b3 + c3 + 3 (a + b) (b + c) (c + a) ,
3

a, b, c R

Pentru a = x, b = y, c = z, folosind i ipoteza, obinem c 8 = 8+3 (x s t a (x + z), deci x y = 0, z y = 0 sau x + z = 0. nlocuind pe rnd n a dou gsim soluiile sistemului: {(2, , ) , (, 2, ) , (, , 2) | , , R }. a t

Cristian Svescu, elev a Soluie. Evident c a, b, c sunt subunitare, deci au sens radicalii. t a inegalitatea mediilor M G M A, obinem t 1 1 2 1 p = = 1a+1b 2 (1 c) 1+c (1 a) (1 b) 2 2 i nc dou relaii analoage. Folosind acum inegalitatea mediilor M H s a a t n n P P 1 forma n2 xi , deducem c a i=1 xi i=1 X 1 1 1 9 1 p 2 + + 2 1+c 1+a 1+b 3 + (a + b + c (1 a) (1 b) 1 1 n2 p + + p . (n 1) k (k a1 ) (k a2 ) (k an ) (k a1 )

VIII.57. Fie a, b, c > 0 astfel nct a + b + c = 1. S se arate c a a 1 1 9 1 p +p +p . 2 (1 a) (1 b) (1 b) (1 c) (1 c) (1 a)

Not. Mai general, dac a1 , a2 , . . . , an > 0, a.. a1 + a2 + + an = k a a

VIII.58. Fie x, y, z (0, ) cu x + y + z 3. S se arate c xn + y n a a n N. Romeo Ilie Soluiile. Pentru n = 0, concluzia este evident; fie deci n 1. t a inegalitii mediilor M A M G, are loc: at n xn + (n 1) = xn + | + 1 + + 1 n xn 1 1 . . . 1 = nx 1 {z }
n1

pentru x (0, ) i n 2; pentru n = 1, inegalitatea (1) se transform n s a Aplicnd de trei ori (1), obinem c t a

de unde rezult concluzia. a

xn + y n + z n + 3 (n 1) nx + ny + nz = n (x + y + z) 3n,

VIII.59. Determinai x, y, z R, stiind c x+y+z = 1, iar xy+(x + y t a Gheorghe Molea, Curtea Soluia 1. Avem: t
2

(x + y + z) = 1 x2 + y 2 + z 2 + 2 (xy + xz + yz) = 1 4 5 2 2 2 x +y +z +2 x y = 1 x2 + y 2 + z 2 2x 2y = 3 3 1 (x 1)2 + (y 1)2 + z 2 = . 3 1 Pe de alt parte, din binecunoscuta inegaliate 2 + 2 + 2 ( + + a 3 se reduce, dup calcule, la ( )2 + ( )2 + ( )2 0), pentru a

= y 1, = z, obinem t (x 1)2 + (y 1)2 + z 2

1 1 [(x 1) + (y 1) + z]2 = (1)2 = 3 3

cu egalitate pentru x 1 = y 1 = z. Relaia () arat c se atinge ega t a a avem, ntr-adevr, x1 = y1 = z x = z +1, y = z +1. nlocuind n x+ a 2 2 1 obinem c x = , y = , z = . t a 3 3 3

Soluia 2. Eliminnd pe z ntre cele dou relaii, obinem xy+(x + y) (2 t a t t 4 a t sau nc 3x2 + 3y 2 + 3xy 6x 6y + 4 = 0. Considernd aceast relaie a 3 2 2 de gradul doi cu necunoescuta x, avem: 3x + x (3y 6) + 3y 6y + 4 2 2 a x R, atunci x 0; iar x = 3 (3y 2) 0 implic y = . Pen 3 2 1 gsim x = i z = . a s 3 3

VIII.60. Se consider prisma triunghiular regulat ABCA0 B 0 C 0 cu A a a a 0 si AA = 3 3. S se arate c pentru fiecare num r a 0, 3 3 , exist e a a a a 0 00 0 00 puncte Ma , Ma pe dreapta CC 0 astfel nct d (B 0 , (Ma AB)) = d (B 0 , (Ma Mirela M Soluie. Fie M CC 0 i vom nota cu x lungimea t s segmentului CM , considernd x 0 dac M [CC 0 i A a s N x < 0 dac M CC 0 \[CC 0 . Deoarece A0 B 0 k AB i AB a s B (M AB), atunci A0 B 0 k (M AB), deci d (B 0 , (M AB)) = 0 0 0 0 d (N , (M AB)), unde N este mijlocul lui [A B ]. Fie N mijlocul lui [AB], iar P proiecia lui N 0 pe M N . Cum t (N CC 0 ) (ABN ), atunci vom avea c N 0 P (ABM ), a P adic d (B 0 , (M AB)) = N 0 P . a A 0 Din asemnarea imediat 4N N P 4N M C, a a N NC NN0 NN0 N 0P = N 0P = = obinem c t a B NC MN MN 23 3 . (Am folosit egalitile evidente N C = 2, N N 0 = 3 3 at 2 4+x q N C 2 + M C 2 = 4 + |x|2 = 4 + x2 .) 6 3 0 t =a Pentru fiecare valoare a 0, 3 3 , ecuaia N P = a 4 + x2 2 27 a2 dou soluii, anume x = a t , de unde concluzia problemei. a

Clasa a IX-a

IX.56. Determinai numerele reale pozitive x, y, z, t pentru care x+y+ t iar xy + xz + xt + yz + yt + zt + 475 = xyzt. Lucian Tuescu i Liviu Smarandache t s

Tot conform inegalitii mediilor, obinem at t xyzt = xy + xz + xt + yz + yt + zt + 475 6 6 xy xz xt yz yt zt + = 6 xyzt + 475. a Cu notaia a = xyzt, inegalitatea precedent devine t

Soluie. Conform inegalitii mediilor, t at 4 x+y+z+t xyzt xyzt 625. 4

ns evident c a > 0, prin urmare a [25, +), i.e. xyzt 625. Rez a a atinge egalitatea n (1), fapt care are loc atunci cnd x = y = z = t = 5. IX.57. S se determine toate funciile f : R R cu proprietatea a t f (f (f (x) + y) f (x f (y))) = x2 y 2 , x, y R.

a2 6a + 475 (a 25) (a + 19) 0 a (, 19] [25, +

Adrian Zahariuc, ele Soluie. Pentru x = y = 0, obinem c f (f (f (0)) f (f (0))) = t t a a = f (f (0)) f (f (0)), deci f (a) = 0. Pentru x = y = a, gsim c a a f (f (f (a) + a) f (a f (a))) = 0,

adic f (f (a) f (a)) = 0, prin urmare f (0) = 0. a Lum acum y oarecare i x = f (y) i obinem f 2 (y) y 2 = 0, deci f a s s t x R. Considerm mulimile A = {x R | f (x) = x} i B = {x R | f a t s Dac A, B 6= R , exist x A i y B cu x 6= y i, ntorcndu-ne n ecua a a s s t

ceea ce contrazice x 6= y. a Rezult c A = R sau B = R , adic f (x) = x, x R sau f (x) = a a Aceste dou funcii verific ecuaia, deci sunt soluii. a t a t t [x] + [x + a1 ] + + [x + an1 ] = [cx] , x R.

x2 y 2 = f (f (f (x) + y) f (x f (y))) = (x + y)2 x y = (x

IX.58. Fie n N, n 2. Determinai a1 , a2 , . . . , an1 , c R pentru c t

(n legtur cu G.42 din RecMat 1/2003.) a a Iuliana Georgescu i Paul Georg s Soluie. Notm ki = [ai ], bi = {ai }, i = 0, n 1. Cum bi [0, 1) pent t a lund x = 0 n relaia din ipotez obinem k1 + k2 + + kn1 = 0, de u t a t [x] + [x + b1 ] + + [x + bn1 ] = [cx] , x R.

S presupunem, pentru fixarea ideilor, c b1 b2 bn1 ; n orice a a raioneaz analog. Lund, pe rnd, x = 1, apoi x = 1 n (1), obinem t a t respectiv [c] = n, prin urmare c = n. 1 Pentru x = b1 n (1), obinem c 1 = [nb1 ], deci b1 . Ap t a n 2 t a i, n continuare s x = b2 n (1), obinem c 2 = [nb2 ], deci b2 n i = 1, n 1.

1 1 Lum acum x = a n (1) i gsim c s a a + bn1 = 1, deci bn1 n n n1 2 n1 . Lund succesiv x = , . . . , x = n (1), o concluzie, bn1 = n n n i bi = , i = 1, n 1. n 1 n concluzie, {a1 , a2 , . . . , an1 } este o mulime de forma k1 + , k2 t n n1 kn1 + , cu k1 + k2 + + kn1 = 0, k1 , k2 , . . . , kn1 Z, iar c = n IX.59. Fie C (I, r) cercul nscris n 4ABC. S se arate c a a IA IB + IB IC + IC IA 12r2 . D. M. Btineu - Giurgiu, B a t Soluia 1. Evident c IA + r AIA ha , unde IA = PrBC I, deci: t a I IA+r ha a IA+ar aha = 2S = ar+br+cr a IA (b + c) r

i nc dou inegaliti similare. Prin urmare, s a a at IA IB b+c c+a 4c ab 4c = 2 r a b ab ab i analoagele. Prin urmare, obinem s t s X a abc IA IB IB IC IC IA 433 =1 + + 4 r2 r2 r2 bc ab bc ca

IX.60. Fie ABC un triunghi ascuitunghic cu a < b < c. Cercul C (I t triunghiului este tangent dreptelor BC, CA si AB n punctele D, E s F . Dreapta ID intersecteaz CA n D0 si AB n D00 , IE intersecteaz a a si BC n E 00 , iar IF intersecteaz BC n F 0 si CA n F 00 . Ar tai c a a t D0 D00 + F 0 F 00 . Temistocle B Soluie. Notm cu Ha , Hb , Hc picioarele nlimilor din vrfurile A, B t a at tiv C. Condiia a < b < c impune ordinile urmtoare: B DHa C, C t a i A F Hc B. De aici i din faptul c ID k AHa , deducem c pun s s a a D00 au poziiile indicate pe gur; la fel se procedeaz pentru precizarea t a a punctelor E 0 , E 00 i F 0 , F 00 . s Vom utiliza urmtoarele relaii cunoscute: AE = AF = p a, BD = B a t

IA IB + IB IC + IC IA 12r2 , cu egalitate n cazul triunghiului echilateral. r Soluia 2 (Mihai Haivas). Cum IA = t i analoagele, trebuie s sin A 2 P 1 12. Conform inegalitii mediilor i bine at s strm c a a sin A sin B 2 2 A B C 8 sin sin sin 1, avem: s 2 2 2 X 1 1 3 2 33 2 A 2 B 2 C 3 8 = 12. A B sin 2 sin 2 sin 2 sin 2 sin 2

CD = CE = p c (2p = a + b + c). Cu aceste pregtiri, avem: a D0 D00 = DD0 DD00 = (p c) tg C (p b) tg B, E 0 E 00 = EE 00 EE 0 = (p c) tg C (p a) tg A, F 0 F 00 = F F 0 F F 00 = (p b) tg B (p a) tg A

i, ca urmare, stabilirea egalitii E 0 E 00 = D0 D00 + s at F F 0 F 00 este imediat. a E Not. Prof. Titu Zvonaru observ c, dac na a a a locuim cercul nscris cu cercul A-exnscris, concluzia B D problemei rmne adevrat. Si pentru celelalte cer- E a a a curi exnscrise are loc concluzia, n sensul c cel mai mare dintre segmente a celorlaltor dou. a

Clasa a X-a

X.56. Fie tetraedrul ABCD si M un punct n spaiu. Dac G, GA , t a GD sunt centrele de greutate ale tetraedrelor ABCD, M BCD, M ACD respectiv M ABC, s se arate c AGA + BGB + CGC + DGD = 0 dac a a a dac M G. a Marius Olteanu, Rmnic Soluie (Alexandru Negrescu, elev, Botoani). Avem: t s X X X X AG + GGA = 0 AGA = 0 AG + GGA = 0 X 1 X GM + GB + GC + GD = 0 GGA = 0 4 3 X GM + GA = 0 GM = 0 G M. 4 X.57. Dac x, y, a (1, ), s se arate c a a a x+y + (x + y + loga y) xy + loga y x+y (x + y + loga x) xy + loga x (x + xy + y)2 loga xy. Mihail Bencze Soluie. Observm c t a a 1 x z z 1 1 x y y (x + y + z) + + =3+ + + + + + x y z y x z y x z r r 2 r r x y x y x y . +2 = 1+ + 3+ + +2 y x y x y x t Considernd, pe rnd, z = loga x, apoi z = loga y, obinem: 2 (x + y + loga x) xy + loga xx+y (x + xy + y) loga x; 2 (x + y + loga y) xy + loga y x+y (x + xy + y) loga y.

Adunm membru cu membru cele dou inegaliti i rezult concluzia. a a at s a X.58. Let n N, n 1. Prove that n X n2 k2 + n2 2n 2 ln < . n1 k n2 k=1 Jos Luis Daz - Barrero, Barcelon

Soluie. Este cunoscut inegalitatea ln (1 + x) < x, x > 0, de unde t a k 2 + n2 k2 c ln a < 2 , k, 1 k n. Atunci n2 n n n n n X n2 k2 + n2 X n2 k2 X k n2 X n 12 ln < = = = k k k1 n2 n2 n k
k=1 k=1 k=1 k=1

X.59. Fie f R [X] un polinom de grad n 3 ce admite n r d a a pozitive si subunitare. Dac |f (0)| = f (1), s se arate c produsul r d c a a a a a 1 cel mult egal cu n . 2 Ioan Serdean

Soluie. Putem presupune, fr a restrnge generalitatea c f are c t aa a a a dominant 1; fie f (X) = X n + an1 X n1 + + a1 X + a0 , cu rdcinile i = 1, n. Evident c x1 x2 xn (0, 1). Folosind ipoteza, ultima rela a t descompunerea polinomului n factori, avem: x1 x2 xn = |f (0)| = f (1) = (1 x1 ) (1 x2 ) (1 xn ) (x1 x2 xn ) = [x1 (1 x1 )] [x2 (1 x2 )] . . . [xn (1 xn )] conform inegalitilor 0 xi (1 xi ) at
2

1 4

1 a , i = 1, n. Rezult x1 x2 xn 4 X.60. Fie k N fixat. Alegem n {1, 2, . . . , k} si a1 , a2 , . . . , an num mai mari dect 3. Dac probabilitatea ca a2 + a2 + + a2 s se divid a a 1 2 n a 1 cel puin egal cu t a , s se arate c 24 divide k. a a 24 Cristian Svescu, elev a

Soluie. Cum ai sunt prime, atunci ai 1 (mod 3), deci a2 1 i Pt 2 adic a ai n (mod 3). Analog, ai 1 (mod 8) sau ai 3P (mod 8); P 2 cazuri, a2 1 (mod 8), deci ai n (mod 8). Rezult c a a a2 n i i P 2 i atunci probabilitatea a 24 divide s c ai este aceeai cu probabilitatea s k 1 k 1 1 divid cu 24, care este a = . Cum se atinge egalitate 24 k 24 k 24 k k ipotezei, obinem c t a = , adic 24 divide k. a 24 24

Clasa a XI-a

Jos Luis Daz - Barrero, Barcelon Soluie. Notm cu n determinantul din problem; adunnd prima t a a coloan la cea de-a treia, obinem c a t a

XI.56. Let n be a positive integer. For each positive integer k, let Fk Fibonacci number ( F1 = F2 = 1, Fk+2 = Fk+1 + Fk for all k 1). Prove 2 2 2 Fn Fn+1 2Fn Fn+1 Fn+2 2 2 2 Fn+2 2Fn+1 Fn+2 Fn Fn+1 =0 2 2 2 2Fn Fn+2 Fn+1 Fn+2 Fn

Folosind relaia de recurena, observm c toi termenii de pe a treia co t t a a t nuli, prin urmare n = 0. XI.57. Fie p atratul ABCD circumscris cercului C. n p atrat se n gonul EF GHIJKL, circumscris cercului C, astfel nct E, F (AB), E G, H (BC), G (BH), I, J (CD), I (CJ), K, L (DA), K ( {M } = EL AC, {N } = F G BD, {P } = HI AC, {Q} = JK BD. S c suma a AE BF BG CH CI DJ DK AL AM BN C S= + + + + + + + + + + EB F A GC HB ID JC KA LD M C N D P nu depinde de alegerea vrfurilor octogonului pe laturile p tratului. a Ctlin Cali a a Soluie. Vom demonstra c t a DK DQ DJ + + = 1. KA QB JC Odat justicat relaia (1), scriind nc trei egaliti similare corespunz a a t a at a furilor A, B, C i adunndu-le, obinem concluzia. s t Vom demonstra (1) analitic: fixm un reper cu originea n centru a lui, astfel nct A (1, 1), B (1, 1), C (1, 1), D (1, 1). Ecuaia cer t C : x2 + y 2 = 1 i fie T (cos t, sin t), t 0, s , punctul de contact cu 2 1 1 cos t ; K dreptei KJ : x cos t + y sin t = 1; obinem c J 1, t a sin t c 1 1 DJ sin t + cos t 1 DQ sin t Q , . Atunci = ; = sin t + cos t sin t + cos t JC sin t cos t + 1 QB sin t DK sin t + cos t 1 = i, dup calcule de rutin n care nu intervine dec s a a KA cos t sin t + 1 a a a a fundamental sin2 t + cos2 t = 1, rezult c (1) este adevrat. a Not. Dl Ctlin Calistru, autorul problemei, face cunoscut redac a a a t urmtoarea posibil generalizare a relaiei (1): a a t a a Fie A1 A2 . . . An un poligon regulat, d o dreapt tangent cercului nscris lui i P1 , P2 , . . . , Pn1 punctele determinate prin {Pi } = d A1 Ai+1 , i s Artai c a t a A1 P A1 P2 A1 Pn1 + + + = 1. P1 A2 P2 A3 Pn1 An XI.58. Dac n este un num r natural iar p un num prim, at a a ar (xn+1 (p) xn (p))n0 este divergent, unde xn (p) reprezint exponentul cu a num rul p n descompunerea lui n!. a Sorin Pupan s a Solu 1. Notm yn = xn+1 (p) xn (p), n N. Conform teoremei lu tia a n pn n n xn (p) = t a + 2 + 3 + . . . . Obinem c xpn (p) = xpn +1 (p) = p p p p

2 Fn 2 n = Fn+2 2F F 2 n n+2 Fn+1

2 Fn+1 2 2Fn+1 Fn+2 Fn 2 Fn+2

(Fn+2 Fn+1 )2 F

2 (Fn+1 + Fn )2 Fn

2 (Fn+2 Fn )2 Fn

ypn = 0 i xpn +p1 (p) = s s (ypn )nN i (ypn +p1 )nN (yn ) este divergent.

pn 1 pn 1 ; xpn +p = + 1, deci ypn +p1 = p1 p1 sunt subiruri cu limite diferite ale irului (yn ), p s s

Soluia 2. Cazul p = 2 se trateaz ca mai sus. S presupunem c ex t a a a a astfel nct (xn+1 (p) xn (p))n0 s fie convergent. Atunci, conform lem xn (p) este convergent i ultimele dou iruri au aceea s as s Stolz, irul s n n1 1 1 xn (p) Cum lim = , obinem lim (xn+1 (p) xn (p)) = t ,c n n n p1 p1 1 pentru p 3, N, iar xn+1 (p) xn (p) N. / p1 XI.59. Fie sirul de numere supraunitare (an )n1 , astfel ca lim an =
n

studieze continuitatea funciei f : R R, f (x) = lim {xan }, unde {x} t n partea fracionar a num rului real x. t a a Dan Popescu Soluie. Se stabilete cu uurina c: t s s t a a) pentru x0 Z, lim [x] = [x0 ]; /
xx0

b) pentru x0 Z, lim [x] = x0 1, lim [x] = x0 .


x%x0 x&x0 n n

1. pentru x Z, f (x) = x [x] = {x}; / 2. pentru x Z, f (x) = x x = 0 = {x}, x > 0 (deoarece an x x i s f (0) = 0, iar f (x) = x + 1 = 1, x < 0 (deoarece an x x i an x < x x s 1, x Z, x < 0 Prin urmare, f (x) = . Astfel, f este continu pe R a {x} , n rest 2 XI.60. Fie a (0, 1); s se demonstreze c pentru orice x > a a es ln a 1 x 1 inegalitatea (1 ax ) x+1 < 1 ax+1 . Angela Tigeru a Soluie. Inegalitatea dat este echivalent cu (1 ax )x < 1 ax+1 t a a 2 . Logaritmnd, aceasta este echivalent cu x ln (1 ax ) < (x + 1) ln a ln a 2 Artm c funcia f (x) = x ln (1 ax ), x > aa a t este strict cresctoare. a ln a ax ln a (1 ax ) ln (1 ax ) ax ln ax = . f 0 (x) = ln (1 ax ) x 1 ax 1 ax 2 Cum 1 ax > 0 pentru x > , a (0, 1), este suficient s demo a ln a 2 s (1 ax ) ln (1 ax )ax ln ax > 0 pentru x > . Notm t = ax i cum x a ln a 2 a (0, 1), atunci t 0, 1/e . Prin urmare, trebuie s demonstrm a a (1 t) ln (1 t) t ln t > 0 pentru t 0, 1/e2 . Avem g 0 (t) = ln (1 t) 1 a a Cum, pentru t 0, 1/e2 , (1 t) t < 2 , rezult c ln ((1 t) t) < e

Cum f (x) = lim (an x [an x]) = x lim [an x], obinem: t

ln t ln (1 t) 2 > 0, deci g 0 (t) > 0 pentru t 0, 1/e2 , adic g a cresctoare pe 0, 1/e2 . Cum lim g (t) = 0, deducem g (t) > 0, t 0, 1/e a
t&0

trebuia demonstrat.

Clasa a XII-a

XII.56. Fie Sn mulimea permut rilor de ordin n, iar Sn . Se t a funcia m rginit f : R R. S se calculeze: t a a a 1 1 1 lim f ( (1)) + f ( (2)) + + f ( (n)) . n n 2 n (O generalizare a problemei 24131, G. M. 5-6/1999.) Marius Olteanu, Rmnic 1 1 1 Soluie. Fie an = t f ( (1)) + f ( (2)) + + f ( (n)) . D n 2 n este mrginit exist m, M R astfel nct m f (x) M , x R. Prin a a a m M 1 1 1 1 1 + + + an 1 + + + n 2 n n 2 n 1 1 1 1 + + + = 0, deducem lim an = 0. i cum lim s n n n 2 n 1 1 XII.57. Consider matricea A = am si mulimea t 1 1 1 . G = Xa | Xa = I2 + aA, a , 2 Ar tai c (G, ) este grup izomorf cu (R, +). Calculai X 1 X 3 X 2n a t a t 2 2 2 Gheorghe Iu s a s Soluie. Deoarece A2 = 2A, deducem uor c Xa Xb = Xa+b+2ab i cu t 2ab 1 , pentru a, b 1 , , nmulirea matricelor este lege de c t 2 2 pe G. (G, ) este grup cu elementul neutru I2 , iar inversul fiecrui elemen a a este X 1+2a G. Funcia f : G R, f (Xa ) = ln (2a + 1) realizeaz izo t a cerut. Fie Xt = X 1 X 3 . . . X 2n1 , atunci: 2 2 2 f (Xt ) = f X 1 X 3 . . . X 2n1 = f X 1 + f X 3 + + f X 2n 2 2 2 2
2

ln (1 + 2t) = ln 2 + ln 4 + + ln 2n. 2n n! 1 Deducem c 1 + 2t = 2 4 . . . 2n = 2n n!, deci t = a . 2 XII.58. Let f : R [1, 1] be a continuous function. Prove that Z 1 Z 1q 3 1 (f (x))2 dx + f (x) dx 4.
1 1

Zdravko Starc, Vrac, Serbia and Mo p Soluie. Pentru orice a [1, 1] avem: 3 (1 a2 ) + a 2, cu egalit t p 1 a = . Prin urmare 3 (1 f 2 (x)) + f (x) 2, x R; integrnd acea 2 ntre 1 i 1 obinem inegalitatea cerut. Egalitate avem pentru f : R s t a 1 f (x) = . 2

XII.59. Fie f : R R, f derivabil si neconstant pe nici un interva a a Dac a f 0 (x) f 0 (sin x) cos x + f 0 (cos x) sin x, x R, 1 + f 2 (x) demonstrai c nu exist lim f (x). t a a x Paul Georgescu i Gabriel P s Soluie. Pentru orice x1 , x2 R, x2 > x1 , integrnd membru cu memb t tatea din enun avem t

arctg f (x2 ) arctg f (x1 ) f (sin x2 ) f (sin x1 ) f (cos x2 ) + f (co

prin urmare g (x2 ) g (x1 ), unde g (x) = arctg f (x) f (sin x) + f (cos deci g este monoton cresctoare pe R. Cum g este i mrginit, deducem a s a a lim g (x) R. Presupunem c exist lim f (x); atunci exist a a a
x x

lim (f (sin x) f (cos x)) = lim (arctg f (x) g (x)) R.


x

Deoarece h (x) = f (sin x) f (cos x) este periodic i are limit la , h as a stant; exist deci c R astfel nct f (sin x) f (cos x) = c. Pentru x = a a

XII.60. Fie n > 1 si a1 , a2 , . . . , an (0, 1) astfel nct a1 + a2 + + a f (x) se determine funciile f : R R, dac lim t a = 1 si f (x) = f (a1 x) + x0 x + f (an x) pentru orice x R. Gabriel Dospines Soluie. Vom demonstra prin inducie c t t a X p! f (x) = f ak1 ak2 akn x n 1 2 k1 !k2 ! kn !
k1 ,...,kn 0 k1 +k2 ++kn =p

gsim c = f (0) f (1) = f (1) f (0), deci c = 0, adic f (sin x) f (c a a x R. Prin urmare g (x) = arctg f (x) i cum g este monoton, rezult c f este s a a a dar f (0) = f (1) i atunci f (x) = f (0), x [0, 1], contradicie. s t

pentru orice x i orice p. s Pentru p = 1 este chiar relaia din enun. Presupunem c relaia este t t a t pentru p i o demonstrm pentru p + 1.nlocuim n relaia () pe rnd pe s a t . . . , an x i nsumm relaiile obinute, innd cont de relaia din enun; r s a t t t t t X p! f (x) = f ak1 +1 akn x + + n 1 k1 ! kn !
k1 ,...,kn 0 k1 +k2 ++kn =p

k1 ,...,kn 0 k1 +k2 ++kn =p

p! f ak1 akn +1 x = n 1 k1 ! kn !

k1 ,...,kn 0 k1 +k2 ++kn =p

p! (k1 + 1) f ak1 +1 akn x + + n 1 (k1 + 1)! kn !

+ Notnd k1 + 1

k1 ,...,kn 0 k1 +k2 ++kn =p 0 = k1 , . . . , kn +

p! (kn + 1) f ak1 akn +1 x . n 1 k1 ! kn1 ! (kn + 1)!

f (x) = +

0 k1 ++kn =p+1

0 k1 ++kn =p+1

Renotnd indicii de sumare obinem: t X (k1 + k2 + + kn ) p! k1 k2 f a1 a2 akn x f (x) = n k1 !k2 ! kn !


k1 ,...,kn 0 k1 +k2 ++kn =p+1

0 1 = kn , obinem: t 0 0 X p!k1 k1 kn 0 ! k ! f a1 an x + + k1 n

0 p!kn k0 f ak1 ann x . 1 0 k1 ! kn !

k1 ,...,kn 0 k1 +k2 ++kn =p+1

(p + 1)! f ak1 ak2 akn x , n 1 2 k1 !k2 ! kn !

f (x) = x0 x f (x) > 0 astfel nct dac x (0, ), atunci 1 < a < 1 + . x k1 k2 kn k1 + k2 + + kn = p, atunci a1 a2 an < (max (a1 , a2 , . . . , an ))p p max (a1 , a2 , . . . , an ) < 1, avem lim (max (a1 , a2 , . . . , an )) = 0, deci exist Fie acum un > 0, arbitrar i un a > 0 fixat. Cum lim s
p

ceea ce trebuia demonstrat.

nct (max (a1 , a2 , . . . , an ))

. a Deci pentru orice x (0, a) i orice k1 , . . . , kn cu suma p0 , avem s f ak1 akn x n 1 < 1 + . 1< k1 kn a1 an x Atunci X X p0 ! p0 ! (1 ) xak1 akn < f ak1 n 1 k1 ! kn ! 1 k1 ! kn !
p0

<

k1 ++kn =p0

k1 ++kn =p0

< (1 + ) P

k1 ++kn =p0

p0 ! s ak1 akn = (a1 + + an )p0 = 1 i atunci ( ns a n 1 k1 ++kn =p0 k1 ! kn ! f (x) < (1 + ) x pentru orice x (0, a) i orice > 0, ceea ce atrage s pentru x (0, a). Cum a a fost ales arbitrar rezult f (x) = x pentru x 0. S ob a a acelai raionament funcioneaz i pe (, 0), cu meniunea c n acest s t t as t a inegalitile i schimb sensul, prin urmare f (x) = x pentru x < 0. Pr at s a f (x) = x pentru x R.

p0 ! xak1 akn . n k1 ! kn ! 1

Soluiile problemelor pentru pregtirea concur t a din nr. 1 / 2005


A. Nivel gimnazial

2 x y = y2 z = G76. Rezolvai n mulimea numerelor naturale sistemul t t 2 z x= Adrian Zano s Soluia 1. Dac x = 0, atunci y = u2 i apoi y = 0, u = 0, z = t a t = 0. Obinem astfel soluia (x, y, z, u, v, t) = (0, 0, 0, 0, 0, 0). Dac x 6= t t a y 6= 0 i z 6= 0. Cum x2 y este ptrat perfect mai mic dect x2 , s a x2 y (x 1)2 y 2x 1. La fel z 2y 1 i x 2z 1. Adunn s trei relaii, gsim x + y + z 3 i cum x, y, z N , rezult x = y = z = t a s a soluia (x, y, z, u, v, t) = (1, 1, 1, 0, 0, 0). t Soluia 2. Fie x 6= 0 (deci y 6= 0 i z 6= 0); atunci t s 2 2 2 x u y v 2 z 2 t2 = xyz,

deci

(x u) (y v) (z t) (x + u) (y + v) (z + t) = xyz. Prin urmare (x u) (y v) (z t) este numr natural nenul, deci (x a (z t) 1 i cum (x + u) (y + v) (z + t) xyz, egalitate n () obin s t u = v = t = 0 i x u = y v = z t = 1, deci x = y = z = 1. Ave s (x, y, z, u, v, t) {(0, 0, 0, 0, 0, 0) , (1, 1, 1, 0, 0, 0)}. G77. i) Fie a, b, c R cu a > b > c; atunci

b2 a2 + > a + 2b + ab bc a2 b2 c2 b2 ii) Fie a, b, c R cu a b c > 0; atunci + + c a 3a 4b + c. Ioan Serdean a2 b2 b2 b2 a2 = a + b, iar Soluie. i) Avem evident c t a ab ab bc b (egalitate avem dac b = 0, respectiv c = 0), de unde, prin adunare, o a b2 a2 + > a+2b+c. Inegalitatea este strict deoarece nu putem avea a ab bc ii) Majorm pe rnd cei trei termeni ai membrului stng: a

(a b) (a + b) c+c a2 b2 = (a b) = 2 (a b) ; c c c c2 b2 (c b) (c + b) a+a = (c b) = 2 (c b) ; a a a (am folosit faptul c c b 0, schimbnd sensul inegalitii!) a at a c 2 2 a c (a c) (a + c) = = + (a c) (1 + 0) (a c) = a b b b b Adunnd membru cu membru relaiile (1), (2) i (3), obinem concluzia. t s t

G78. Dac a, b, c, d (0, ), s se demonstreze inegalitatea a a b (a + c) c (b + d) d (a + c) a (b + d) + + + 4. c (a + b) d (b + c) a (d + c) b (a + d) Artur Bluc, aa a Soluia 1. Prelucrm fiecare raport din stnga astfel: t a a+c 1 1 a+c + b (a + c) = c = ac = a c . 1 1 a+b a+b c (a + b) + a b b ab 1 1 1 1 n aceste condiii, cu substituiile x = , y = , z = , d = , inega t t a b c t demonstrat se scrie succesiv: y+t z+x t+y x+z + + + 4 x+y y+z z+t t+x 1 1 1 1 + + (y + t) + 4 (x + z) x+y z+t y+z t+x (x + z) (x + y + z + t) (y + t) (x + y + z + t) + 4. (x + y) (z + t) (y + z) (t + x) ns, conform inegalitii mediilor, a at
2

(x + y) (z + t)

1 4 (x + y + z + t) 4 (x + y) (z + t) (x + y + z + t (x + z) (x + y + z + t) 4 (x + z) 4 (x + z) (x + y + z + t) = 2 (x + y) (z + t) x+y+z+ (x + y + z + t)

Scriind nc o inegalitate similar i adunndu-le, obinem (). a as t 1 1 1 1 Soluia 2. Cu substituiile x = ; y = ; z = ; d = t t inegal a b c t echivalent cu: a y+t z+x t+y x+z + + + 4, x, y, z, t (0, ) . x+y y+z z+t t+x Dar a2 (a1 + a2 + + an )2 a2 a2 1 + 2 + + n , ai , bi (0, ) b1 b2 bn b1 + b2 + + bn i atunci s x+z y+t z+x t+y + + + = x+y y+z z+t t+x
2 2 2 2

(x + z) (y + t) (z + x) (t + y) + + + (x + y) (x + z) (y + t) (y + z) (z + t) (z + x) (t + y) (t + x

(z + x + y + t + z + x + t + y)2 (x + y) (x + z) + (y + t) (y + z) + (z + t) (z + x) + (t + y) (t + x) G79. Dac x, y, z (0, ) sunt astfel nct x + y + z = xyz, atunci a p p p xy + yz + zx 3 + x2 + 1 + y 2 + 1 + z 2 + 1.

Florina Crlan i Marian Tetiv s

Soluia 1 (a autorilor). Plecnd de la cunoscuta a2 + b2 + c2 ab t avem: 1 1 1 1 1 1 + 2+ 2 + + = 1 x2 y 2 + x2 z 2 + y 2 z 2 x2 y 2 z 2 x y z xy xz yz (xy + xz + yz) 2xyz (x + y + z) + x2 y 2 z 2 = 3 (x + y + z) . Mai departe,
2 2 2 2

i s

(xy + xz + yz 3) = (xy + xz + yz) 6 (xy + xz + yz) + 9 2 3 (x + y + z) 6 (xy + xz + yz) + 9 = 3 x2 + y 2 + z 2 + 9

(xy + xz + yz) (x + y + z) 9xyz xy + xz + yz 9 > 3, p prin urmare xy + xz + yz 3 + 3 (x2 + y 2 + z 2 ) + 9. Se arat ns uor c a a s a p p p p 3 (x2 + y 2 + z 2 ) + 9 x2 + 1 + y 2 + 1 + z 2 + 1, de unde concluzia rezult prin intercalare. a

Soluia 3 (Gheorghe Iurea, Iai). Din ipotez xy 1 = t s a xy > 1. Analog yz > 1, zx > 1. De asemeni,

Soluia 2 (a autorilor). Avem t 2 xyz = x + y + z 2 xy + z z ( xy) 2 xy z 0. 1 + 1 + z2 2 , obinem d t Rdcina pozitiv a trinomului zt 2t z fiind a a a z p 1 + 1 + z2 xy z xy 1 + 1 + z 2 . z Scriind nc dou inegaliti analoage, prin sumare gsim: a a at a p p p xy + xz + yz x yz + y xz + z xy 3 + x2 + 1 + y 2 + 1 + x+y z

(xy 1) (yz 1) = xy 2 z xy yz + 1 = y (x + y + z) xy xz + 1 =

i nc dou relaii analoage. s a a t n inegalitatea 2 + b2 + c2 ab + ac + bc, adevrat pentru orice a, b a a a reale, punem a = xy 1, b = yz 1, c = zx 1 i folosind relaiile ( s t inegalitatea cerut. a Egalitate pentru a = b = c, deci xy = yz = zx sau nc x = y = a ipoteza, gsim x = y = z = 3. a

Soluia 4 (Paul Georgescu, Iai). Deoarece x, y, z (0, ) exist t s (0, /2) astfel nct x = tg a, y = tg b, z = tg c. Folosind relaia din ipot t tg (a + b + c) = 0 i apoi a + b + c = . s Inegalitatea de demonstrat este echivalent cu: a 1 1 1 tg a tg b + tg b tg c + tg c tg a 3 + + + . cos a cos b cos c

Cum tg a tg b 1 =

cos (a + b) cos c = , () este echivalent cu a coa cos b cos a cos b cos b cos a 1 1 1 cos c + + + + cos a cos b cos a cos c cos b cos c cos a cos b cos c

sau nc: a cos2 a + cos2 b + cos2 c cos a cos b + cos b cos c + cos a cos c,

adevrat pe baza inegalitii x2 + y 2 + z 2 xy + yz + zx, x, y, z R. a a at Egalitate dac cos a = cos b = cos c, deci a = b = c = i apoi x = y = a s 3 G80. Fie A mulimea tuturor sumelor de tipul 12 32 52 t n N, unde semnele pot fi alese n orice combinaie posibil . S s t a a A = Z. (n legtur cu teorema Erds-Surnyi.) a a Petru Asa 2 2 2 Soluie. S observm c (2k + 1) (2k + 3) (2k + 5) + (2k + t a a a k N. Prin urmare, dac un numr ntreg n se scrie sub forma n = a a (2k 1)2 pentru o anumit alegere a semnelor +, , numrul n + 16 a a el o sciere de aceeai form: s a
2 2 2 2

De asemeni, dac n are o exprimare de forma dat, atunci i n au o a a s de aceeai form (obinut din exprimarea pe care o are n, prin schimbar s a t a semnelor). Notm cu P (n), n N, afirmaia: "n are o exprimare de forma n a t 32 (2k 1)2 , k N ". Cum P (n) P (n + 16), conform unei v inducie matematic este suficient s verificm P (0), P (1), . . . , P (15): t a a a 2 2 2 2 2 2 2 0 = 1 3 5 + 7 9 11 13 + 152 , 1 = 12 , 2 = 12 + 32 + 52 72 + 92 112 132 + 152 , 3 = 12 + 32 + 52 + 72 92 , X 4 = 12 + 32 + 52 + 72 h 2 2 2 (2k + 1) (2k + 3) (2k + 5) + (2k

n + 16 = 12 32 (2k 1) + (2k + 1) (2k + 3) (2k + 5) +

k{4,8,...,20}

5 = 12 + 32 + 52 + 72 + 92
2 2 2 2

k{5,9,...,41}

6 = 1 3 + 5 7 92 + 112 , X h 2 2 2 7 = 12 + 32 + 52 + + 132 (2k + 1) (2k + 3) (2k + 5) + (


k{8,12,...,116}

h (2k + 1)2 (2k + 3)2 (2k + 5)2 + (2k

8 = 12 32 + 52 72 92 + 112 ,

9 = 7 + 16 = 12 32 132 + h i X (2k + 1)2 (2k + 3)2 (2k + 5)2 + (2k + 7)2 + +


k{8,12,...,116}

k=120

La fel procedm i cu numerele 10, 11, . . . , 15. a s Prin urmare, orice numr natural are o exprimare de forma cerut i a a as numr ntreg are o exprimare de forma cerut. Rezult A = Z. a a a G81. Fie n N si k {0, 1, . . . , 2n 1}. S se arate c exist o muli a a a t cu n elemente care are exact k submulimi cu suma elementelor strict poz t Adrian Zahariuc, ele Soluie. Fie k = an an1 . . . a1 , ai {0, 1}, i = 1, n, scrierea n ba t k (admitem ca primele cifre s poat fi 0, astfel nct scrierea s se fac a a a a cifre). Pentru fiecare i {0, 1, . . . , n 1}, fie i = 1 dac ai+1 = 0 i a s t s ai+1 = 1; construim mulimea A = i 2i | 0 i n 1 i vom demon verific proprietatea dorit. a a Pentru fiecare submulime B A, numim indicele dominant al lui B, ce t i {0, 1, . . . , n 1} pentru care i 2i B. Dac i = 1, atunci suma elem a B este cel puin 2i 2i1 20 = 1 > 0. Dac i = 1, atunci suma e t a lui B este cel mult 2i + 2i1 + + 20 = 1 < 0. n concluzie, sem elementelor lui B depinde numai de indicele dominat. Cum exist 2i subm a indicele dominant i, numrul submulimilor cu suma elementelor strict po a t 20 a1 + 21 a2 + + 2n1 an = k i astfel problema este rezolvat. s a G82. Un cal se afl pe tabla de sah n cmpul A1 si dorim s -l ducem a a H8 ntr-un num r minim de s rituri. Aflai care este acest num r mini a a t a si cte trasee de lungime minim exist . a a Gheorghe Crciun, Plopeni i Gabriel P a s Soluie (Gheorghe Iurea, Iai). Pentru a ajunge din A1 n H8, ca t s s ctige 7 coloane i 7 linii, n total 14 poziii. La fiecare sritur, el ct a s s t a a s 3 poziii (dou linii i o coloan sau o linie i dou coloane); rezult c sun t a s a s a a a cel puin 5 srituri. ns calul nu poate ajunge n cinci srituri din A1 n H t a a a de pe care pleac i cel pe care sosete au culori diferite, ceea ce implic n as s a efecturii unui numr par de srituri (la o sritur, calul merge de pe alb a a a a a sau de pe negru pe alb). Exist trasee de 6 srituri (de exemplu, A1 B3 C5 E6 G5 a a prin urmare numrul minim de srituri necesare este 6. a a

X h

(2k + 1) (2k + 3) (2k + 5) + (2k + 7)

8 7 6 5 4 3 2 1 A B C D E F G H

94 8 44 3 3 2 1 9 4 545 7 3 4 144 33 545 6 3 3 5 12 3 12 2 184 33 9 4 2 12 43 2 23 144 13 4 2 3 3 12 11 43 1 4 23 11 12 33 2 12 12 1 A B C D E F G

Numim drum un traseu format din 6 srituri care pleac din A1 i a a s H8. Dup prima sritur putem ajunge n B3 sau n C2, iar la ultim a a a trebuie s ajung n F 7 sau G6. La a dou sritur drumul ajunge n u a a a a a ptrelele haurate pe figura 1, iar la a cincea drumul trebuie s treac a at s a a dintre ptrelele marcate cu . Sriturile 3 i 4 ne duc dintr-un ptrel a at a s a at unul marcat cu . n figura 2, am notat numrul sriturii cu cifr mic din dreapta sus. a a a a csuelor am notat numrul de drumuri pariale care duc din A1 pn n a a t a t a Se observ c numrul de drumuri pn ntr-un ptrat la sritura k este eg a a a a a a numerelor ptratelor de la sritura k 1, aflate pe un ptrat de latur 5 a a a a n acel ptrat. Recurent, se obin 108 drumuri de lungime minim. a t a

Not. Faptul c numrul minim de srituri este 6 este un rezultat c a a a a de exemplu, L. Panaitopol, D. Serbnescu - Probleme de teoria numerelor a natoric pentru juniori, GIL, Zalu, 2003. Problema numrului traseelor a a a nu ne este ns cunoscut. a a

G83. Fie ABCD patrulater convex si punctele M, N (AB), P, R BM N A P R astfel nct AD BC M R N P = {O}. S se arate c a a M N DP RC Andrei-Sorin Cozma, Soluie. Se demonstreaz cu uurina urmtoarele dou proprieti: t a s t a a at BD BD SABC i s 1. Fiind dat triunghiul ABC i D (BC), atunci s = DC SACD BC AD AE . AB AC Folosind aceste rezultate avem BM N A P R CD = M N DP RC AB BM N A P R CD = = M N AB RC DP SOMB SOAN SOP R SOCD = = SON M SOAB SORC SODP =

2. Fiind dat triunghiul ABC i D (AB), E (AC), avem c s a

O C R

OM OB OA ON OP OR OC SOMB SOAN SOP R SOCD = SORC SODP SON M SOAB OR OC OD OP ON OM OA

G84. Fie ABCD un trapez cu ABkCD, AB < CD. Se consider CF AE = . Dreapta EF intersect E (AD) si F (BC) astfel nct ED FB MN DC AB AC n M , respectiv N . S se arate c a a = . EF DC + AB Andrei Ned AB AO BD Soluie. Notm k = t a = = , k < 1, iar cu P , T intersecii t CD OC OD

paralelei prin E la AC cu CD, respectiv BD. A B AE CP AE Cum = (teorema lui Thales) i s = O ED PD ED CP CF CF N , urmeaz c a a = , adic P F k a M FB PD FB BD. Aplicnd Thales n 4EP F cu T M k P F , E ET EM ET AO T obinem c t a = ; ns a = = k, TP MF TP OC P D EM k deci = k, i.e. EM = EF . Analog se MF k+1 k arat c N F = a a EF . Astfel, k+1 2k MN 1k CD M N = EF (EM + N F ) = EF EF = = k+1 EF 1+k CD G85. Fie A0 , B 0 , C 0 picioarele bisectoarelor unghiurilor 4ABC. Pe la consider m punctele D si E astfel nct D (BE) si cevienele AD si a izogonale. S se demonstreze c DB 0 si EC 0 se intersectez pe AA0 . ( a a a cu Propoziia 1, p. 99, RecMat - 2/2004.) t Titu Zvonaru, C \ = m(EAC) = i m(DAA0 ) = m(EAA0 ) [ \ \ Soluie. Notm m(BAD) t a s 0 0 {X} = C E AD i {Y } = DB AE. Folosind teorema lui Menelaus n s transversala C 0 XE obinem: t ED BC 0 AX = 1. EB C 0 A XD Dar AD AE sin 2 ED SDEA AD sin 2 = = = EB SBEA AB AE sin (2 + ) AB sin (2 + ) BC 0 BC i 0 = s (din teorema bisectoarei). CA AC Prin urmare: AX AB AC sin (2 + ) = . XD AD BC sin 2 La fel AB AC sin (2 + ) AY = . YE AE BC sin 2 Din teorema bisectoarei n triunghiul ADE, AD DA0 = . 0E A AE AX DA0 Y E = 1 i din teorema lui Ce s Folosind (1), (2), (3) deducem XD A0 E AY 0 c dreptele EX, DY , AA sunt concurente. a Not. Soluia autorului, n esena aceeai, folosete explicit relaia l a t t s s t Se poate arta n aceeai manier c rezultatul rmne valabil i dac D a s a a a s a

B. Nivel liceal

L76. Fie cercurile C1 si C2 tangente interior unui cerc C n punctele di respectiv N . Cercurile C1 si C2 sunt secante sau tangente exterior iar axa

cercurilor C1 si C2 taie cercul C n A si B. Dreptele AM si AN taie din no C1 si C2 n K, respectiv L. Ar tai c AB 2KL. n ce caz avem egalit a t a Neculai Roman, Mirce Soluie. Fie C1 = C (O1 , r1 ), C2 = C (O2 , r2 ), C = C (O, r) i {P, Q} t s (fig. 1). Dac cercurile C1 i C2 sunt tangente exterior (fig. 2) atunci P = a s

K O O1 M B Fig. 1 Q P L O2 N

K O O1 M B Fig. 2 P L O2 N

Din AB ax radical a cercurilor C1 i C2 rezult c AK AM = AL a a s a a \ \ \ 4AKL 4AN M , de unde obinem AKL AN M , adic m(AKL) = m t a 1 m(AM ). 2 Pe de alt parte, din a 1 1 \ \ \ \ m(O1 KM ) = m(O1 M K) = m(OM A) = 90 m(M OA) = 90 m 2 2 \ obinem m(O1 KL) = 90 , deci KL este tangent la C1 . Din O1 K k O t a rezult KL O2 L, de unde KL este o tangent comun exterioar ce a a a a i C2 . Aplicm teorema lui Casey pentru cercurile A, C1 , B, C2 (A, B d s a tangente interior cercului C i obinem: s t dAO1 dBO2 + dAO2 dBO1 = dAB dO1 O2

s (am notat dO1 O2 lungimea tangentei comune exterioare cercurilor C1 i C2 p p p p AP AQ BP BQ + AP AQ BP BQ = AB KL p AP + BP AQ + AB KL = 2 AP BP AQ BQ AB KL 2 2 AB 2 AB 2KL. AB KL 2 s Dac cercurile C1 i C2 sunt tangente exterior (fig. 2), atunci avem: a 2 AP + BP AB 2 ABKL ABKL = 2AP BP ABKL 2 A 2 2

(egalitate dac AP = BP ). a s Egaliate avem dac cercurile C1 i C2 sunt tangente exterior n mijlocu a tului [AB].

L77. Fie punctele P1 , P2 , . . . , P13 n plan astfel nct oricare trei sunt n

si toate au coordonate ntregi. S se arate c exist cel puin un triunghi Pi P a a a t nct centrul s u de greutate s aib coordonate ntregi. a a a Vasile Prav i Titu Zvonaru, Comneti at s a s Soluie (Daniel Vcaru). Exist trei resturi modulo 3, aa c cel pui t a a s a t xi dau acelai rest la mprirea prin 3 (principiul cutiei!), ceea ce impune s at centrului de greutate este numr ntreg, oricum am alege trei indici din a A a celor 5 determinai mai sus. Considerm M mulimea resturilor mo t a t numerelor {yi | i A}. Dac M are trei elemente, alegem i, j, k A a a yi + yj + yk 0 + 1 + 2 (mod 3) i problema este rezolvat. Dac M s a a elemente, cel puin 3 ordonate dau acelai rest la mprirea prin 3 i le t s at s ele. n sfrit, dac M are un singur element, concluzia este imediat. s a a Not. Principial aceeai soluie a dat Vlad Emanuel, elev, Sibiu. a s t L78. Consider m sirul de puncte (Pn )nN pe cercul trigonometric a a \ ) = arctg 5 pentru orice n N, Pn OPn+1 fiind considera \ m(Pn OPn+1 12 orientat. S se arate c pentru a a orice punct P pe cercul trigonometric ex 1 astfel nct Pj Int C P, . 2005 Lucian - Georges Ldunc i Andrei Ned a as Soluie. Pentru ca mulimea {Pn , n N} s fie dens pe cercul trig t t a a de unde va rezulta imediat concluzia problemei folosind lema lui Kroneck 5 arctg 12 5 demonstrat c a R \ Q. Observm mai nti c arctg a a = 12 5 p Presupunem prin reducere la absurd c arcsin a = , (p, q) = 1, p, q 13 q 5 p Fie = arcsin . Atunci sin q = 0 i cos q = (1) . Cum (cos + s 13 p (1) , obinem c t a
2 4 sinq Cq sinq2 cos2 + Cq sinq4 cos4 = 0.

2 Dac q este par, obinem c polinomul cu coeficieni ntregi xq Cq xq2 a t a t q q q [ ] 5 2[ ] a a t a , ceea ce es . . . (1)[ 2 ] Cq 2 1 x2 2 are rdcina raional sin = 13 deoarece coeficientul termenului dominant este 2q1 , iar termenul liber est 1 deci polinomul respectiv poate avea numai rdcini de forma n , n N a a 2 impar se trateaz analog. a arc Nota. Vlad Emanuel demonstreaz c, dac a [1, 1] Q i a a a s 1 atunci a 0, 1, . 2 s L79. Fie a1 , a2 , . . . , an R n aa fel nct a1 + a2 + + an = 0 si 1h 2 2 2 max {|ai aj | ; 1 i < j n} 1. Demonstrai c a1 +a2 + +an t a n si precizai n ce caz are loc egalitate. t Marius Pachiariu, t Soluie. Deoarece inegalitatea este simetric n a1 , a2 , . . . , an , putem t a

c a1 a2 an . Conform identitii lui Lagrange, obinem innd a at t t ipotezele problemei c a n X 1 X 1 X 2 a2 = (aj ai ) (aj ai ) , i n n i=1
1i<jn 1i<jn

1i<jn

t a deoarece aj ai este pozitiv subunitar pentru toi 1 i < j n. Rm hni hn + 1i P demonstrm c a a (aj ai ) . 2 2 1i<jn Dac n este par, n = 2k, k N , atunci a X (aj ai ) = (2k 1) (a2k a1 ) + (2k 3) (a2k1 a2 ) + + (ak

(2k 1 + 2k 3 + + 1) (a2k a1 ) = k 2 (a2k a1 ) . hni hn + 1i P (aj ai ) k 2 = n concluzie, . 2 2 1i<jn Dac n este impar, n = 2k + 1, k N , atunci a X (aj ai ) = 2k (a2k+1 a1 ) + (2k 2) (a2k a2 ) + + 2 (an+2
1i<jn

hni hn + 1i . 2 2 1i<jn t Dac n este par, n = 2k, k N , egalitatea din enun are loc pentru a = ak = < 0, ak+1 = ak+2 = = a2k = > 0 i = 1, + s 1 1 = ,= . 2 2 t Dac n este impar, n = 2k + 1, k N , egalitatea din enun are a a1 = a2 = = ak+1 = < 0, ak+2 = ak+3 = = a2k+1 = a1 = a2 = = ak = < 0, ak+1 = ak+2 = = a2k+1 = > 0 i s (k + 1) + k = 0, respectiv k + (k + 1) = 0. Rezult c a1 = a2 = a a k+1 k , ak+2 = = a2k+1 = , respectiv a1 = a2 = = ak = 2k + 1 2k + 1 k ak+1 = ak+2 = = a2k+1 = . 2k + 1 Not. Soluie corect s-a primit de la Vlad Emanuel, elev, Sibiu. a t a L80. Fie un alfabet cu 4 litere a, b, c, d. n acest alfabet se pot form dup urm toarele reguli: dup a nu poate urma b, dup b nu poate urma a a a a nu poate urma d si dup d nu poate urma a. Cte cuvinte palindromice d a n, n 2, se pot forma conform acestor reguli? (Prin cuvnt palindromic un cuvnt n care litera de pe poziia k coincide cu litera de pe poziia t t pentru orice k {1, 2, . . . , n}.) Irina Mustaa, e t Soluie. S observm c pentru a construi un cuvnt palindromic i t a a a h n (p de lungime n este suficient s construim un cuvnt C de lungime a , 2 inversm i l adugm la sfritul lui C, ntre acestea insernd nc o lit a s a a s a este impar. n concluzie, P (aj ai ) k (k + 1) =

k (k + 1) (a2k+1 a1 ) .

Tinem seama c ntr-un palindrom, odat cu gruparea de dou litere c a a a apare i inversa sa. Atunci gruprile interzise n C sunt ab, bc, cd, da, ad, s a Notm cu an , bn , cn , dn numrul cuvintelor corecte de lungime n care a a n a, respectiv n b, c, d. Deoarece orice cuvnt corect de lungime n care n a este format dintr-un cuvnt corect de lungime n 1 care se termin n a care se adaug un a final, deducem c an = an1 + cn1 i analog bn = bn a a s cn = cn1 + an1 , dn = dn1 + bn1 . Notnd cu Sn numrul cuvintel a de lungime n, avem din cele de mai sus c Sn = 2Sn1 , iar cum S1 = 4 a Sn = 2n+1 . Notm acum cu Pn numrul palindroamelor de lungime n. Pe a a enun, distingem dou cazuri. t a n 1. n este par. Atunci Pn = S[ n ] = 2[ 2 ]+1 . 2 hni 2. n este impar. ntre cuvntul de lungime i inversul su se mai i s a 2 n liter, care poate fi aleas n dou moduri. Atunci Pn = 2S[ n ] = 2[ 2 ]+2 . a a a
2

Not. Soluie corect s-a primit de la Vlad Emanuel, elev, Sibiu. a t a

L81. Fie n 1 un num r natural fixat. O tabl infinit de sah es a a a n alb si negru n maniera obinuit . O mulime C de c sue ale tablei s s a t a t conex dac putem ajunge din fiecare c sua a lui C n fiecare alt c a a a t a a C printr-o succesiune de deplas n C dintr-o c sua ntr-o c sua ve ari a t a t latur comun ). Fie S o mulime conex cu 4n c sue. Numim raportu a a t a a t al mulimii S raportul dintre num rul de c sue albe si num rul de c sue t a a t a a t S. S se afle cea mai mic si cea mai mare valoare posibil a raportului c a a a Adrian Zahariuc, ele Soluie. Fie w numrul de csue albe, b numrul de csue negre, iar N t a a t a a t de perechi (ordonate) de csue din S. Cum fiecare csua alb are cel mu a t a t a negri n S, rezult c fiecare csua alb apare n cel mult 4 perechi, dec a a a t a Analog, N 4b. Demonstrm acum c N 4n 1. n acest scop, mulimii S i se asocia a a t n felul urmtor: fiecrei csue i corespunde un nod, iar nodurile acestu a a a t unite prin muchii dac acele csue care le corespund sunt adiacente. Gr a a t obinut este i el conex, N reprezint numrul su de muchii, iar 4n num t s a a a a vrfuri. Observm c un graf conex cu v vrfuri are cel puin v 1 muchii, a a t se poate demonstra astfel: eliminm muchiile care aparin unor cicluri a t obinem un arbore, despre care tim c are exact v 1 muchii. Atunci con t s a de mai sus, N 4n 1. De aici, 4w 4n 1, deci w n i analog b n. Cum b + w = 4n, o s 1 w b 3n i w 3n. De aici, s 3. Rmne acum s construim exem a a 3 b raportul cromatic atinge valorile extreme. Numim o figur format dintr-o csua i vecinii si de sus, stnga a a a t s a un T -dreptunghi; acesta se va numi T -dreptunghi alb sau negru dup c a 1 din centru este alb sau neagr. Pentru a atinge valoarea , construim a a 3 format din n T -dreptunghiuri negre suprapuse, n timp ce pentru a ating a

3 construim o figur format din n T -dreptunghiuri albe suprapuse. a a Not. Soluie corect s-a primit de la Vlad Emanuel, elev, Sibiu. a t a

L82. Determinai P, Q R [X] pentru care f : R R, f (x) = {p (x) + t este periodic , unde p, q : R R sunt funciile polinomiale asociate lui a t tiv Q. Paul Georgescu i Gabriel P s Soluie. Fie T R o perioad a lui f . Atunci {p (x + T ) + sin q ( t a + {p (x) + sin q (x)}, x R, deci p (x + T ) + sin q (x + T ) p (x) sin q x R. Deoarece g : R R, g (x) = p (x + T ) + sin q (x + T ) p (x) sin continu, rmne c ea este identic constant, deci exist k Z astfel ca p a a a a a sin q (x + T ) p (x) sin q (x) = k, x R. De aici, p (x + T ) p (x) k = sin q (x) sin q (x + T ), x R, de und c p (x + nT ) p (x) nk = sin q (x) sin q (x + nT ), x R i n N a s pentru x = 0 conduce la |p (nT ) p (0) nk| 2, n N . De aici rez este un polinom de grad maxim 1, deci P = aX + b, iar k = aT . n concluzie, a (x + T ) + b + sin q (x + T ) ax b sin q (x) = aT , x sin q (x + T ) sin q (x) = 0, x R. Aceasta implic faptul c a a 2 sin q (x + T ) q (x) q (x + T ) + q (x) cos = 0, 2 2 x R.

q (x + T ) q (x) q (x + T ) + q (x) i s nu sunt identic constante, atunc Dac a 2 2 stng poate avea cel mult o mulime numrabil de zerouri, contradicie. R t a a t aici c q (x + T ) q (x) = 2k1 , x R, cu k1 Z, sau q (x + T ) + q (x a x R, cu k2 Z. n primul caz obinem c Q este un polinom de grad cel mult 1, Q = t a a1 , b1 R, cu a1 T = 2k1 . n al doilea caz rezult imediat c Q c, c a a inclus n primul. n concluzie, P = aX + b, Q = a1 X + b1 , unde a i a1 au proprietatea s a1 T T R+ astfel ca aT, Z. Se observ imediat c n acest caz fu a a 2 periodic. a Not. Soluie corect s-a primit de la Vlad Emanuel, elev, Sibiu. a t a L83. S se calculeze a " # 1 2 n 1 2 1 3 1 n+1 1+ lim + 1+ + + 1 + n . n n n n

Marius Olteanu, Rmnic Soluie. Mai nti, se observ c t a a k n n X 1 n+1 1 k+1 nn 1+ n. 1+ n n


k=1

De asemenea,

1 1 k 1+ 1+ 1 k+1 n n = 1+ , 1 1 1 n 1 k+1 1+ 1+ k+1 n n conform inegalitii lui Bernoulli. Rezult de aici c at a a k k+1 X n n n X X (n + 1) (k + 1) 1 k n 1+ 1 = n n (k + 1) + 1 n (k + 1) +
k=1 k=1 k=1

n concluzie,

1 X n n X k+1+ 1 n 1+ 1 = n (k + 1) + 1 n n (k + 1) +
k=1 k=1
k

k=1

Avem n plus c a

! 11 n 1 1 + n 1+ n 1 1 n+1 lim n 1 + n = lim = 1, 1 n n n n

n n X 1 1X 1 1 k+1 1 1+ 1+ . n n n k+1
k=1

limita din urm putndu-se calcula cu ajutorul regulii lui lHospital, iar a n 1X 1 lim =0 n n k+1
k=1

Determinai inf A. t

conform lemei Cesar-Stolz. Din (1) i (2) rezult conform criteriului c s a limita din enun este egal cu 1. t a Not. Soluie corect s-a primit de la Vlad Emanuel, elev, Sibiu. a t a L84. Fie n N, n 3 si n n A = x > 0; x = a0 + a1 n n + + an1 nn1 ; o a0 , a1 , . . . , an Z; n 1 | a0 + a1 + + an .

. i Ak+1 + Ak+1 + + Ak+1 = (n 1) Ak . n 1 pentru k 1. Cum s 0 n1 . 1 n1 pentru k obinem c inf A = 0. t a Not. Soluie corect s-a primit de la Daniel Vcaru. a t a a L85. Fie f : R R o funcie pentru care mulimea punctelor n care f t t finit la stnga este dens n R. S se arate c mulimea punctelor n c a a a a t

Paul Georgescu i Gabriel P s k Soluie. Fie (xk )k1 definit de xk = ( n n 1) ; evident, x1 A. t n n1 A Fie k 1. Presupunem c xk = Ak + Ak n n + + Ak a 0 1 n1 n strm c xk+1 A. Avem c a a a k n xk+1 = n1 xk = nAn1 Ak + Ak Ak n n + + Ak Ak 0 0 1 n2 n1 k+1 k+1 k+1 n n1 n n + + An1 n = A0 + A1

continu este de asemenea dens n R. (O mulime D R se numete d a a t s dac orice interval deschis al axei reale conine m car un element din D. a t a Gabriel Dospinescu, Paris, i Marian Tetiv s Soluie. Vom folosi urmtoarea lem, demonstrabil uor cu ajutorul t a a a s zrii cu a limitei (finite) a unei funcii ntr-un punct t0 R: a t Dac funcia h : R R are limit finit la stnga n t0 R, atunci pe a t a a > 0 exist un interval compact nedegenerat J, situat la stnga lui t0 , a a |h (x) h (y)| < pentru orice x, y J. Fie acum I un interval deschis; trebuie s artm c acesta conine ce a aa a t punct n care f este continu. Conform ipotezei, exist x0 I n care f a a finit la stnga i, aplicnd lema de mai sus, exist I0 interval compact ne a s a situat la stnga lui x0 astfel ca |f (x) f (y)| < 1 pentru orice x, y I0 ; presupus suficient de mic, astfel ca I0 I. Fie I0 = [a0 , b0 ]; exist atunci x1 (a0 , b0 ) n care f are limit finit a a a n x1 i de asemenea exist un interval compact nedegenerat I1 (a0 , b0 s a 1 stnga lui x1 astfel ca |f (x) f (y)| < pentru orice x, y I1 . 2 Continund iterativ, obinem irul de intervale compacte (I )) I0 ) I1 t s 1 astfel ca |f (x) f (y)| < i , x, y Ii . Conform lemei intervalelor nchi 2 T 1 exist a a In ( I). Atunci |f (a) f (x)| < i , x Ii , i N, de un 2 n=0 c f este continu n a I, ceea ce trebuia demonstrat. a a

Exemplele anunate la pag. 27. t

Ex. 1

Ex. 2

Ex. 3

Premiu special acordat de FUNDA TIA CULTURALA "POI

Fundaia Cultural "Poiana" (director d-l Dan Tiba) acord elevul t a a EMANUEL, cl. a XI-a, Colegiul Naional "Gh. Laz r", Sibiu, un premiu t a de 100 lei (1 000 000 lei vechi). Premiul este oferit la recomandarea Redaciei revistei pentru abilitat t niozitatea dovedite n rezolvarea unui numr mare de probleme din rubrica a pentru pregtirea concursurilor": L(67-69,77-83). a

Probleme propuse1
Clasele primare

P.104. Suma dintre predecesorul unui numr i succesorul numrulu a s a lui este 29. Care este acest numr? a (Clasa I ) Irina Luca, e P.105. Alturat se afl roboelul "MATE". a a t a) Completai casetele goale; t b) Aflai suma numerelor pe care le ine n mini; t t c) Aflai diferena numerelor scrise n tlpile picioarelor. t t a (Clasa I ) Andrei Stativ, elev, Iai a s

P.106. Pentru desemnarea campioanei, echipele de hochei pe gheaa A i B disput un numr de partide pn ce una dintre ele c t s a a a ori. Care este numrul maxim de partide care se pot juca, tiind c nu a a s a rezultate de egalitate? (Clasa a II-a) nv. Constana Cristea i Inst. Iulian Cri t s P107. Un grup de turiti a consumat 17 prjituri i 31 ngheate. S s a s t turiti au consumat cte o ngheat i cte o prjitur, 5 turiti au consum s t as a a s cte dou ngheate, iar 4 turiti nu au consumat nimic, s se afle ci tur a t s a t grup. (Clasa a II-a) Aliona Loghin, e P108. Prin mprirea a dou numere naturale rezult ctul 3 i restu at a a s c mpritorul este un numr mai mic dect 10, aflai cele dou numere. a at a t a (Clasa a III-a) nv. Rica Buct a P.109. Figura alturat este format din beioare. a a a ts a) ndeprteaz un singur beior pentru a obine tot attea tria a ts t H unghiuri ca i ptrate; s a b) Mut dou beioare pentru a obine de dou ori mai multe a a ts t a dreptunghiuri dect ptrate. a (Clasa a III-a) Adina Voinescu, elev, Iai a s P.110. Ce or indic primul ceas, tiind c acesta respect regula i a a s a a celelalte trei?
12 9 6 3 9 6 12 3 9 6 12 3 9 6 12 3

(Clasa a III-a) Veronica Corbu, e P.111. Fie numrul N = abc + acb + bac + bca + cab + cba. a a) Care este cea mai mic i cea mai mare valoare a lui N ? as b) Cte valori diferite poate avea numrul N ? a (Clasa a IV-a) Oxana Pascal, e P.112. n urma desfurrii unui joc didactic matematic, nvator as a at ca recompens 44 baloane. Cte 4 baloane au primit un numr de part a a
1

Se primesc solu ii pn la data de 31 decembrie 2006. t a

+13

reprezint a asea parte din totalul lor, cte dou au primit a treia parte a s a participanilor au primit cte un balon. Aflai numrul participanilor la j t t a t aritmetic!). a (Clasa a IV-a) Alexandra Nistor, e

P.113. Dan i-a pus timbrele n clasor, cte 10 pe unele pagini, cte s pagini i au rmas de 4 ori mai multe pagini goale dect folosite. Dac ar p s a a timbre pe fiecare pagin, toate paginile ar fi folosite. Cte pagini poate ave a tiind c nu depete 60 (soluie aritmetic!)? s a as s t a (Clasa a IV-a) Petru Asa

Clasa a V-a

V.66. S se arate c, oricare ar fi cifra nenul a, numrul x = 2131a +32 a a a a se divide cu 10. Otilia Neme, Ocna Mur s

V.67. a) S se arate c, scznd din suma a 2006 numere pare consecu a a a numerelor situate ntre acestea, nu se poate obine rezultatul 20062 . t b) S se afle 2006 numere pare consecutive astfel nct, scznd din sum a a numerelor situate ntre ele, s se obin 20052 . a t a Marian Pan t

a a V.68. Artai c nu exist n N pentru care An = 5n + 89 s fie ptr a t a a Iulia Pleca, e s t V.69. S se rezolve n N2 ecuaia 8n + 15m = 6 + 62 + + 62006 . a Alexandru Gabriel Tudorache,

V.70. Determinai a N pentru care numerele a, a + 2, a + 6, a + 1 t a + 20, a + 26, a + 30, a + 32, a + 36, a + 60 sunt simultan prime. Lucian Tuescu t

Clasa a VI-a
VI.66. Alturat este desenat o grdin avnd forma a a a a unui poligon cu 7 laturi. n fiecare vrf se afl cte o a poart mobil astfel nct, n oricare dou vrfuri vecine, a a a porile s nchid perfect latura pe care acestea o detert a a min. S se afle lungimile porilor. a a t Roxana Cpn, elev, Iai a at a a s
9m

A G
11 m

12 m

F 10 m VI.67. n patrulaterul ABCD construim AP BD, CQ BD, P i fie M mijlocul lui (AC). Dac punctele M , P , Q sunt distincte dou s a a demonstrai c 4M P Q este isoscel. t a Marius Fa \ VI.68. Fie punctele A, C, M cu m(AM C) 6= 90 i AC = 2 AM . S s s a \ \ M este mijlocul lui [AC] dac i numai dac 2m(ACM ) = m(M AC). as a Ioan Scleanu a a

VI.69. S se arate c pentru orice alegere a semnelor n expresia 12 a a 20062 , rezultatul nu se divide cu 2006. Mihail Bencze

VI.70. Determinai m, n Z pentru care a = t

3m + 1 n+2 + Z. 2m + 1 3n + 5 Gheorghe Iu

Clasa a VII-a

VII.66. S se rezolve n R4 ecuaia a t p p 30 x y + 901 + 25 y z + 626 + 20 z x + 401 + 9 t x + 78 = . . VII.67. Aflai a, b N dac a + b = 18 i 10a+1 9b + 71 . 81. t a s Andrei-Sorin Cozma,

Ioana Olan, e

VII.68. Fie 4ABC dreptunghic, cu ipotenuza de lungime a, catetele 2 x2 + y 2 a2 = dac i numa as aria S. Dac x, y (0, ), s se arate c a a a S xy c sunt direct sau invers proporionale cu x i y. t s Veronica Pleu i Dan Pl a s s

\ a VII.69. Fie 4M N P cu m(N M P ) = 90 ; se consider punctele S, T, M M (P T ), astfel nct N S = 3 M S, P T = 3 M T . Dac {Q} = P S N T a a) QM = N P ; b) QN 2 + QP 2 = 5 N P 2 . Dorel Luc VII.70. Triunghiul alturat este considerat fix. n cte moa duri putem aeza numerele 1, 2, 3, 4, 5, 6 n cerculee, astfel nct s t suma numerelor de pe fiecare latur a triunghiului s fie aceeai? a a s Petru Asaftei, Iai s

Clasa a VIII-a

VIII.66. S se demonstreze c a a 1 1 n1 1 + 4 + + 4 < . 4 + 22 + 1 2+1 2+1 2 3 +3 n +n 3n Carmen Daniela Tama

VIII.67. Fie 0 < a < b < c < d < e i propoziiile: s t 2ac b+d 2ce ; p2 : c = ; p3 : c = ae; p4 : d = p1 : b = a+c 2 c+ S se arate c dac oricare trei dintre propoziii sunt adevrate, atunci este a a a t a i cea de-a patra. s Claudiu- tefan P S

VIII.68. Fie An = 2006n + 2005n 1992n 1991n , n N. S se de a . . 28. pentru care An . Ionel Nech

VIII.69. Fie x1 , x2 , x3 R astfel nct x2 + x2 + x2 = 1. Determina 1 2 3 mic i cea mai mare valoare a expresiei as E (x1 , x2 , x3 ) = x1 + x2 + x3 + x1 x2 + x2 x3 + x3 x1 .

Ion Vian i Lucian Tuescu s s t

VIII.70. Se consider cubul ABCDA0 B 0 C 0 D0 i fie M , N mijloacele a s [AB], respectiv [BC], iar {S} = AN CD, {T } = DM BC. S se a a unghiului format de D0 N i ST . s Gabriel P

Clasa a IX-a

Adrian Zahariuc, ele IX.68. n 4ABC se consider cevienele [AM ], [BN ], [CP ] concurent a TA TB TC se arate c a = = dac i numai dac T este centrul de g a s a TM TN TP 4ABC. Ovidiu Pop, Sa IX.69. Fie 4ABC nedreptunghic. Paralela prin B la AC i simetri s t s AC n raport cu BC se intersecteaz n A1 ; analog se obin punctele B1 i a AA1 , BB1 , CC1 sunt concurente, s se arate c 4ABC este echilateral. a a Temistocle B IX.70. S se arate c tg 15 + tg 25 + tg 35 + tg 85 > 4. a a D. M. Btineu-Giurgiu, B a t

(a2 a3 an ) + (a1 a3 a4 an ) + + (a1 a2 an1 ) = 1. S se arate c a a a1 + a2 + + an + a2 a3 an + a1 a3 an + + a1 a2 an1

IX.66. Pentru x, y R, e a = y + xy x, b = x2 + x xy. a) Dac a, b (, 0), s se compare numerele x i y. a a s b) Artai c exist o innitate de numere raionale x, y pentru care a, b a t a a t Ionu Onofrei, elev t IX.67. Fie n 2 i a1 , a2 , . . . , an 0 astfel nct s
2 2 2

Clasa a X-a

Adrian Cordune a t X.67. Fie Q 2 = a + b 2 | a, b Q . S se determine funciile c f : Q 2 (0, ) pentru care f (x + y) = f (x) f (y), x, y Q 2 . Dan- tefan Marinescu i Viorel Cornea, H S s X.68. Pe cercul trigonometric se consider punctele A, B, C de ax a 2 2 unde = cos + i sin . Fie M (z) un punct al cercului situat pe arcul 3 3 z2 + z + 1 conine A. S se arate c z 2 + z + 1 = t a a . z Marian Tetiv X.69. Dac a, b, c > 1, s se demonstreze inegalitatea a a (a + b + c 3 log b+ 3 log c 3 3 3 3 a a + b logb a+ logb c + c logc a+ logc b a 3 Titu Zvonaru, C

X.66. Notm cu D mulimea punctelor P (x, y) din planul xOy situa a t riorul sau pe laturile 4ABC. Fie a, b R, a2 + b2 6= 0; denim funcia f t f (P ) = ax + by + c. S se arate c pentru orice P D, avem a a min {f (A) , f (B) , f (C)} f (P ) max {f (A) , f (B) , f (C)} .

X.70. Fie ptratul ABCD. S se determine mulimea a a t = P Int ABCD | P A2 , 2P B P D, P C 2 sunt laturile unui triun

Ctlin Cali a a

Clasa a XI-a

XI.66. Fie xn , n N , cel mai mic numr natural cu proprietatea c e a a an an1 . . . a1 a0 (10) cu toate cifrele nenule, astfel nct M = (n + 1) n+1 an a xn a . 9xn . S se calculeze lim n 10n Valeriu Braoveanu s XI.67. Fie irul (xn )n1 denit prin x1 , 0 , xn+1 = 2xn tg x s 4 S se studieze existena limitelor lim xn i lim n xn . a t s n n Dan Popescu

XI.68. Fie f : I R, I R interval, o funcie de dou ori der t a f 00 (x) f 0 (x), x I. S se arate c f (x) f (a) (exa 1) f 0 (a), a a Pentru f (x) = ex , (, 0] [1, +), s se deduc inegalitatea lui B a a Dumitru Mihalach

XI.69. Fie A, B M3 (R) astfel nct det (AX + B) 0, X M3 arate c exist C M3 (R) pentru care A = BC. a a Gheorghe Iu

XI.70. Fie a, b, c laturile unui triunghi ale crui unghiuri au msurile a a A, B, C i care are raza cercului nscris r. S se arate c distana s a a t de 3 M (A, B, C) la planul P : ax + by + cz + r = 0 este mai mare dect . 3 Sorin Pupan s a

Clasa a XII-a

s se arate c exist (a, b) astfel nct f 00 () = 0. a a a

XII.66. Fie a, b R cu 0 a < b i e f : [a, b] R o funcie d s t derivabil pe [a, b], cu f 00 continu. Dac a a a Z b a2 b2 f (x) dx = f 0 (a) f 0 (b) + bf (b) af (a) , 2 2 a

Mihai Ha

XII.67. Fie f : [0, 1] R o funcie cu proprietatea c exist L 0 a t a a |f (x) f (y)| L |x y|, x, y [0, 1]. S se arate c pentru orice prim a a lui f i pentru orice x1 , x2 , . . . xn [0, 1], n N, n 2, are loc s X F x1 + x2 + . . . xn F (x1 ) + F (x2 ) + + F (xn ) L ( 2n2 n n
1i<jn

Dan- tefan Marinescu, H S

, g (x) = eQ(x) , unde P , Q sunt XII.68. Fie f, g : R R, f (x) = e de grad m 1, avnd coecieniiRdominania, t respectiv b, a, b (0, ). t n Rn a) S se calculeze lim f (n) 0 g (x) dx a g (n) 0 f (x) dx .
n

P (x)

b) S se studieze buna denire a irurilor (an )n1 i (bn )n1 , unde a s s 1 Rn an 1 Rn f (x) dx, g (bn ) = g (x) dx i apoi s se calculeze lim s a . 0 0 n bn n n Marius Ap

XII.69. Fie f R [X] polinom reciproc de grad 4n + 2, n N , avnd distincte, complexe i nereale. S se arate c f are cel puin o rdcin d s a a t a a a Ctlin Tigeru a a a

XII.70. Fie G un grup de ordin n 4 cu proprietatea c exist m a a m1 m < n, astfel nct G conine exact Cn1 subgrupuri de ordin m. Arta t a t abelian. Marius Trnuce a a

Probleme pentru pregtirea concursurilo a


A. Nivel gimnazial

G96. Fie a = x12m + x12n , unde m, n N . S se arate c num a a a divizibil cu 13, dac i numai dac x este divizibil cu 13. as a Artur Bluc, aa a G97. Determinai a, b {0, 1, 2, . . . , 9}, a 6= 0, astfel nct numrul A t a n > 2, s e ptrat perfect. a a

Gheorghe Iu m n+1 G98. S se determine m, n N astfel nct a N . + n m2 Gabriel Dospinescu, stude

G99. Fie m, n dou numere naturale nenule astfel nct m divide n a numerele naturale ntre 1 i n se aeaz la ntmplare pe un cerc. Se calcul s s a oricrui grup de m numere vecine. S se demonstreze c printre aceste su a a a dou pentru care diferena dintre ele este strict mai mare dect m 1. a t Titu Zvonaru, C

G100. n cte moduri putem colora cu 5 culori un ptrat 3 3, astf a ecare ptrat 2 2 s existe patru culori diferite? a a Gabriel P G101. S se demonstreze inegalitatea a ! 1 1 1 16 + + 4 1+ (1 + bc) (1 + ca) (1 a (1 + bc)2 b (1 + ca)2 c (1 + ab)2

a, b, c (0, ) n condiia abc = 1. Cnd are loc egalitatea? t Gabriel Mranu i Andrei Ned s s

G102. S se determine valoarea maxim a parametrului m R astfe a a + 2 2 2 2 2 2 p b +c c +a a +b + + m 3 (a2 + b2 + c2 ), a, b, c R a b c Dorel Bian i I. V. Maftei, B at s

G103. Pentru a, b, c (0, 1) cu a + b + c = 2, s se arate c a a abc 8 (1 a) (1 b) (1 c) .

Alexandru Negrescu, elev, \ = 120 . Fie O (BC) astfel G104. Triunghiul ABC are m(BAC) \ este bisectoarea unghiului BAC. Pe [AO se ia punctul D astfel nct \ a bisectoarea interioar a unghiului ABD. S se arate c AD + BD = AB a a AB + AC 4 AO. Petru Rduc a G105. Se consider trapezul ABCD cu bazele AB, CD (AB > CD a intersecia diagonalelor trapezului. Se duce linia mijlocie M N a trapezului t P Q prin O la bazele trapezului (M, P (AB), N, Q (BC) ). S se de a c trapezele ABM N i P QCD au diagonalele respectiv paralele. a s Claudiu- tefan P S

B. Nivel liceal

L96. Fie cercurile C1 , C2 , C astfel nct C1 i C2 sunt tangente ext s i ecare dintre ele este tangent interior lui C n B, respectiv C. Tangen s interioar cercurilor C1 i C2 taie cercul C n A i A1 . Dreapta AB taie cerc a s s iar dreapta AC taie cercul C2 n L. Din punctul M de pe cercul C se duc M T1 i M T2 la cercurile C1 , respectiv C2 (T1 C1 , T2 C2 ). Dac M A s a A1 M AM artai c M T1 + M T2 = a t a KL i |M T1 M T2 | = s KL. A1 D AD Neculai Roman, Mirce L97. S se demonstreze c n orice triunghi are loc inegalitatea a a 1 1 1 + + m2 (mb + mc ma ) m2 (mc + ma mb )2 m2 (ma + mb mc )2 a c b

I. V. Maftei i Dorel Bian, B s at L98. Se consider un triunghi oarecare ABC. Demonstrai c a t a 27 r 3 4 4 4 ; 1) sin A + sin B + sin C 2 R 4 3 r ra rb rc 2) cos4 A + cos4 B + cos4 C 5 5 5 , 8 R r r r unde R este raza cercului circumscris, r este raza cercului nscris, iar ra , razele cercurilor exnscrise. Oleg Faynshteyn, Leipzig, G L99. a) Care este numrul minim de puncte din plan de coordonate n a nct, oricum ar alese, s existe trei puncte cu centrul de greutate de c a ntregi. b) S se arate c ntr-un spaiu n-dimensional exist 2n+1 puncte de c a a t a ntregi astfel nct oricare trei dintre acestea au centrul de greutate cu c coodonat care nu este un ntreg. a Irina Mustaa, student, Bremen, G t a 1 L100. Fie x (0, 1); artai c exist n N astfel nct {nx} a t a a , 3 Ciprian Baghiu i Gheorghe Iu s

L101. Fie a, n 2 dou numere ntregi. S se arate c a a a

an ak k=0 n k Adrian Zahariuc, ele


n1 Q

Marius Pachiariu, t 2 2 2 L103. Fie a, b, c, d reale astfel nct 1 + a 1+b 1+c 1+ Artai c a t a 3 ab + bc + cd + da + ac + bd abcd 5. Mai mult, avem egalitate n cel puin una din inegalitile de mai sus dac t at dac a + b + c + d = abc + bcd + cda + dab. a Gabriel Dospinescu, stude n L104. Fie x0 > 0 i xn = x[ n ] + x[ n ] + , pentru orice n > 0. s 3 6 x 2 n este convergent la 1. a) S se arate c irul a as n n1 5 xn n b) S se arate c dac > log3 , atunci lim a a a = 0. n 2 n Gabriel Dospinescu, stude

L102. Fie p = 2k + 1 un numr prim. Atunci a 2k k X X i i Cp+i1 2p 2 mod p2 , S2 = Cp+i1 2 2p mo S1 =


i=k+1 i=1

L105. S se determine toate funciile continue f : (0, ) R, ca a t ecuaia funcional t t a nxn1 f (xn ) = (x + 1) f (x) , unde n N , n xat. x (0, ) ,

Marian Tetiva i Dumitru Mihalach s

Training problems for mathematical contes


A. Junior highschool level

G96. Let a = x12m + x12n , with m, n N . Prove that a is divisible b only if x is divisible by 13. Artur Bluc, aa a

G97. Find a, b {0, 1, 2, . . . , 9}, a 6= 0, such that A = abb {z . }, n > 2 i | .. b n times square. Gheorghe Iu m n+1 G98. Find m, n N such that N . + n m2 Gabriel Dospinescu, stude

G99. Let m, n two positive integer such that m divides n 1. A integers between 1 and n are put on a circle in an arbitrary way. One co sum of any set of m neighbors numbers. Prove that among all these sums two of them for which their dierence is strictly grater than m 1. Titu Zvonaru, C

G100. In how many ways could one colour a square 3 3 such that in square 2 2 to be four dierent colours? Gabriel P G101. Prove the following inequality ! 1 1 1 16 4 + + 1+ (1 + bc) (1 + ca) (1 a (1 + bc)2 b (1 + ca)2 c (1 + ab)2

a, b, c (0, ) under the condition abc = 1. When does the equality hold Gabriel Mranu and Andrei Ned s

G102. Find the maximal value of the parameter m R such that + 2 2 2 2 2 2 p b +c c +a a +b + + m 3 (a2 + b2 + c2 ), a, b, c R a b c Dorel Bian and I. V. Maftei, B at G103. For a, b, c (0, 1) and a + b + c = 2, show that abc 8 (1 a) (1 b) (1 c) .

Alexandru Negrescu, highschool student, \ G104. The triangle ABC has m(BAC) = 120 . Let O (BC) such t \ Let D be a point on [AO such that [BC to the angle bisector of BAC. \ angle bisector of ABD. Prove that AD + BD = AB + AC and AB + AC Petru Rduc a

G105. Let ABCD be a trapezium with AB, CD (AB > CD) as the consider that the diagonals of the trapezium intersect in O. We construct the mean line of the trapezium and the parallel P Q, through O, to the b trapezium (M, P (AB), N, Q (BC) ). Prove that the trapeziums AB P QCD have the diagonal respectively parallel. Claudiu- tefan P S

B. Highschool level

L96. Let C1 , C2 , C such that C1 and C2 touch each other externally each of them is interior tangent to C in B and C, respectively. The comm tangent to the circles C1 and C2 cuts the circle C in A and A1 . The line A circle C1 in K and the line AC cuts the circle C2 in L. From the point M the circle C we construct the tangent lines M T1 and M T2 to the circles

respectively (T1 C1 , T2 C2 ). If M A < M A1 prove that M T1 + M T2 = AM and |M T1 M T2 | = KL. AD Neculai Roman, Mirce L97. Prove that, in any triangle, the following inequality holds 1 1 1 + + m2 (mb + mc ma ) m2 (mc + ma mb )2 m2 (ma + mb mc )2 a c b L98. Let ABC be an arbitrary triangle. Prove that

I. V. Maftei and Dorel Bian, B at

27 r 3 ; 2 R 3 r 4 ra rb rc 5 5 5 , 2) cos4 A + cos4 B + cos4 C 8 R r r r where R is the radius of the circumcircle circle, r is the radius of the inscr and ra , rb , rc are the radius of the exinscribed circles. Oleg Faynshteyn, Leipzig, G L99. a) Which is the minimal number of points of integer coordinates in a plane, such that no mater how they are chosen, there exist three of their centre of gravity expressed by integer coordinates. b) Prove that, in an n-dimensional space exist 2n+1 points of integer c such that any 3 of them have the center of gravity with at least one coor an integer. Irina Mustaa, student, Bremen, t 1) sin4 A + sin4 B + sin4 C L100. Let x (0, 1); prove that there exist n N such that {nx}

(By {} we denoted the fractional part.) Ciprian Baghiu and Gheorghe Iu n1 an ak Q L101. Let a, n 2 two integers. Prove that Z. k=0 n k Adrian Zahariuc, highschool studen L102. Let p = 2k + 1 be a prime number. Then S1 =
2k X

i=k+1

Prove that

3 ab + bc + cd + da + ac + bd abcd 5. Moreover, at least one from the above inequalities becomes an equality if a a + b + c + d = abc + bcd + cda + dab. Gabriel Dospinescu, stude n L104. Let x0 > 0 and xn = x[ n ] + x[ n ] + , for any n > 0. 2 3 6 x n a) Prove that the sequence is convergent to 1. n n1 5 xn n = 0. b) Prove that if > log3 , then lim n 2 n Gabriel Dospinescu, stude L105. Find all continuous functions f : (0, ) R, which verify the equation nxn1 f (xn ) = (x + 1) f (x) , x (0, ) , where n N , n is xed. Marian Tetiva and Dumitru Mihalach

Marius Pachiariu, highschool stud t L103. Let a, b, c, d some real numbers such that 1 + a2 1 + b2 1 + c2 1 + d2 = 16.

i Cp+i1 2p 2 mod p2 ,

S2 =

k X i=1

i Cp+i1 2 2p mo

Pagina rezolvitorilor

BRA OV S Liceul "N. Titulescu". Clasa a X-a. CHIRA Roxana: VIII(56,58,61-6 61); COSTA Larisa: VIII(57,58,61-63), IX(56,61); LUCACI Adina: V IX(61,63); MOCANU Vlad: VIII(61-63), IX(61,63,65); NEGOESCU Anam (56,57,61,62), IX(56,61,63,65); NILA Iulian: VIII (61-63), IX(61,63,65); N min: VIII(61-63), IX(61,63,65); OBANCEA Drago: VIII(61-63), IX s SCHIOPU Iulian: VIII(56,61-63), IX(56,61); ZBARCEA Adrian: VIII(56 IX(56,61). Clasa a XI-a. BOTH Alexandru: IX(61,63,65), X.62, XI.61; B Andreea: IX(61,63,65), X.62, XI.61; CAIA Claudiu: IX(61,63,65), X. DELAST-VOINEA Alexandru: IX(61,63,65), X(58,62), XI.61; DIACONE an: IX(61,63,65), X.62, XI.61; DUCA Rzvan: IX(61,63, 65), X.62, X a MITRU Silviu: IX(61,63,65), X.62, XI.61; ENACHE Florin: IX(61,63, XI.61; FLOREA Luminia: IX(57,61,63,65), X.62, XI.61; GIURGIU Beniam t 63,65), X.62, XI.61; GONTEA Paul: IX(61,63,65), X.62, XI.61; GUL IX(61,63,65), X.62, XI.61; IVA CU Andreea: IX(61,63,65), X.62, XI.61; S Raluca: IX(61,63,65), X.62, XI.61; MARDALE Mihai: IX(61,63,65), X. MATI Gheorghe: IX(61,63,65), X.62, XI.61; MRZESCU Andreea: IX S X.62, XI.61; NAFRADI Jen: IX(61,63,65), X.62, XI.61; NAN Maria: IX X.62, XI.61; PANAIT Mihail: IX(61,63,65), X.62, XI.61; POPA Ctlina: aa 65), X.62, XI.61; RACEAN Bogdan: IX(61,63,65), X.62, XI.61; SAVA IX(61,63,65), X.62, XI.61; TABU CA Florin: IX(61,63,65), X.62, XI.61; S Ana-Maria: IX(61,63,65), X.62, XI.61.

CRAIOVA Colegiul Naional "Carol I". Clasa a VI-a. STANCIU Ioan: V(61-6 t 63), VIII.61, G.90. HRLAU Scoala "Petru Rare". Clasa a III-a (nv. BUDACEA Maria). NEI s P(89,95-97,99). Clasa a IV-a. (nv. CRETU Maria). BOUTIUC Mdlin a a 101); BUZILA Andreea: P(94-101); PINTILII Alina: P(94-101). Liceul "Stefan cel Mare". Clasa a VII-a. APACHITEI Ana-Maria: VI(59,60); ATRGOVITOAE Anca-Elena: V(56-58), VI(59,60); COJOCA V(56-58), VI(59,60); CURCA Ioana: V(56-58), VI(59,60); LENTER Son 58), VI(59,60); PINTILII Anda: V(56-58), VI(59,60).

IA I S Scoala nr. 3 "Al. Vlahua". Clasa a III-a (inst. MAXIM Gabriel t MARE Raluca-Iuliana: P(94-96,98,99); NEAGU Ramona-Mihaela: P(94 POPOVICI Ionu: P(94-96,98,99); RUSU Ioana-Andreea: P(94-96,98,9 t Mdlina: P(94-96,98,99); SAVA Vlad: P(94-96,98,99). Clasa a III-a (n a a UTA Valentina). CULEA Alina: P(94-96,98,99); POPA Iulian: P(94 PROCA Ancua-Ioana: P(94-96,98,99). Clasa a VII-a. DODU Cori t 63), VI.61, VII.63; IRIMIA Andreea: V(61-63), VI.61, VII.63; RUSU La V(61-63), VI.61, VII.63; UNGURU Claudiu: V(61-63), VI.61, VII.63.

Scoala nr. 4 "I. Teodoreanu". Clasa a IV-a (nv. BUJOR Lorena) Dana: P(98-100,102,103). Scoala nr. 13 "Alexandru cel Bun". Clasa II-a (inst. COJOCARIU An FITEI Elena-Roxana: P(94-97,99); CARAMALAU Andra: P(94-97,99 Andreea-Claudia: P(94-97,99); COJOCARIU Andreea: P(94-97,99); DU Luisa- tefania: P(94-97,99); LELEU Alexandrina- tefana: P(94-97,99); L S S Diana-Maria: P(94-97,99); MANOLACHE Mdlina-Andreea: P(94-97 a a HAILA Narcisa-Lorena: P(94-97,99); PASCU Gabriela: P(94-97,99); PA Tiberiu- tefan: P(94-97,99); RADUCEA Marin-Andrei: P(94-97,99); SAV S na-Simona: P(94-97,99); STEFAN Bogdan-Vasile: P(94-97,99); STIUBE Ionu: P(94-97,99). Clasa a III-a (nv. OBREJA Rodica): APETRII A t P(94-97,99); E ANU Mihai: P(94-97,99); VATAVU Iulian: P(94-97,99). S Scoala nr. 22 "B. P. Hasdeu". Clasa a IV-a (nv. CHIRILA Laura Alexandra-Elena: P(94-100). Cla Vlad: P(94-100,102,103); CHICHIRAU a (nv. TRZIORU Iuliana). APOSTOL Ana-Maria: P(94-97,99,100) Andrei: P(94-99); GNDU Alexandra: P(94-100); GRIEROSU Claudiu: LAMATIC Ioana: P(94-102); REBEGEA Andrada Elena: P(94-100); UN NU Teofana: P(94-100). Scoala nr. 26 "G. Cobuc". Clasa a IV-a (nv. BUCATARIU Rica). s George-Ciprian: P(94-97,100); DUMITRU Ambra-Georgiana: P(94-97,99, DAN Alexandru-Iulian: P(94-100); HRISCU Alexandra: P(94-100); IACO Ionu: P(94-100); MOISA Adrian-Bogdan: P(94-100); MUSTEATA A t P(94-98); SCUTARU Ionela-Cristina: P(94-100); TUDOSA Mdlina: a a ZALINCA Georgiana: P(94-98). Colegiul Naional. Clasa a VI-a. BACU CA Albert: V(56-60); BAR t S a dan: V(56-60); BOTU Alexandru: P.101, V(61-63), VI.61; CEUCA Rzv 60); MOCANU Dan Mihai: P.103, V(61,63,65), VI.61; PRISTOPAN Cod 60); TIBA Bianca Mdlina: V(59-63,65), VI.63; Clasa a VII-a. CADA a a dra: V(61-65), VI(61,62); HUMELNICU Roxana: V(61-64), VI(61-65). Colegiul Naional "Emil Racovia". Clasa a VI-a. TUDORACHE A t t Gabriel: P(101-103), V(61-65), VI(61,63,64). Colegiul Naional "C. Negruzzi". Clasa a VII-a. TIBA Marius: VII t VIII(61,62), G(89,95); Clasa a VIII-a. OLAN Ioana: VI(61,64), VII(63, 95).

SIBIU Colegiul Naional "Gh. Laz r". Clasa a XI-a. VLAD Emanuel: L(77 t a

SUCEAVA Scoala nr. 3. Clasa a II-a (inst. NECHITA Daniela). FECHET Mircea 87-89,94-97,99).

Premii acordate rezolvitorilor

ASOCIATIA "RECREATII MATEMATICE" n colaborare c revistei RECREATII MATEMATICE acord cte o diplom i un p a as cri pentru trei apariii la rubrica "Pagina rezolvitorilor" elevilor urmt a t t a

Liceul "N. Titulescu", Botoani s Iulian (cl. a X-a): 1/2005(6pb); 2/2005(8pb); 1/2006(6pb); NILA NUTU Cosmin (cl. a X-a): 1/2005(7pb); 2/2005(6pb); 1/2006(6pb); OBANCEA Drago (cl. a X-a): 1/2005(5pb); 2/2005(8pb); 1/2006(6pb s

Colegiul Nional "Carol I", Craiova a STANCIU Ioan (cl. a VI-a): 1/2005(10pb); 2/2005(7pb); 1/2006(10pb) Scoala "P. Rare", Hrlu s a NEICU Mara (cl. a III-a): 1/2005(11pb); 2/2005(5pb); 1/2006(5pb).

Scoala nr. 3 "Al. Vlahua", Iai t s DODU Corina (cl. a VII-a): 2/2004(5pb); 2/2005(6pb); 1/2006(5pb); IRIMIA Andreea (cl. a VII-a): 2/2004(5pb); 2/2005(5pb); 1/2006(5pb UNGURU Claudiu (cl. a VII-a): 2/2004(5pb); 2/2005(5pb); 1/2006(5p Scoala nr. 4 "I. Teodoreanu", Iai s LUPAN Dana (cl. a IV-a): 1/2005(9pb); 2/2005(6pb); 1/2006(5pb).

Scoala nr. 22 "B. P. Hasdeu", Iai s BUHU Vlad (cl. a IV-a): 1/2005(5pb); 2/2005(7pb); 1/2006(9pb); CHICHIRAU Alexandra-Elena (cl. a IV-a): 1/2005(5pb); 2/2005(5p 1/2006(7pb); LAMATIC Ioana (cl. a IV-a): 1/2005(8pb); 2/2005(10pb); 1/2006(9pb REBEGEA Andrada Elena (cl. a IV-a): 1/2005(6pb); 2/2005(7pb); 1/2

Scoala nr. 26 "G. Cobuc", Iai s s DUMITRU Ambra-Georgiana (cl. a IV-a): 1/2005(8pb); 2/2005(5pb 1/2006(6pb); HRISCU Alexandra (cl. a IV-a): 1/2005(6pb); 2/2005(7pb); 1/2006(7 MUSTEATA Alexandra (cl. a IV-a): 1/2005(6pb); 2/2005(6pb); 1/20 ZALINCA Georgiana (cl. a IV-a): 1/2005(6pb); 2/2005(5pb); 1/2006(

Colegiul Naional, Iai t s CADAR Alexandra (cl. a VII-a): 1/2005(8pb); 2/2005(9pb); 1/2006(7

Colegiul Naional "Emil Racovia", Iai t t s TUDORACHE Alexandru-Gabriel (cl. a VI-a): 1/2005(9pb); 2/2005 1/2006(12pb).

ASOCIA TIA RECREA MATEMATIC TII

La data de 14.02.2005 a luat ina ASOCIATIA RECREATII t MATICE, cu sediul n Iai (str. Aurora, nr. 3, sc. D, ap. 6), avnd ca s jinirea activit tilor de matematic specice nv t mntului preuniversita a a aa zarea si desf surarea de activit ti care s contribuie la dezvoltarea gustu a a a matematic n rndurile elevilor, profesorilor si iubitorilor de matematic a a larea preocup rilor si cercet torilor originale. a a

Obiectivele majore pentru atingerea scopului propus sunt: 1. editarea unei reviste destinat elevilor i profesorilor revista " a s Matematice"; 2. fondarea unei biblioteci de matematic elementar biblioteca " a a Matematice"; 3. alctuirea unei colecii de cri de matematic elementar, cri d a t at a a at i aate la prima apariie Colecia "Recreaii Matematice". s t t t

Poate deveni membru al Asociaiei, printr-o simpl completare a une t a orice perosan care ader la obiectivele acesteia si sprijin realizarea lor. a a a

Membri de onoare, academicienii:

Constantin Corduneanu Radu Miron

Continuarea listei membrilor din RecMat - 2/2005: 21 22 23 24 25 26 27 28 29 30 31 32 33 34 35 36 37 38 39 Numele si prenumele Dumitru Neagu Tudorache Rodica Tiba I. Dan Popovici Mihaela Popovici Florin Negrescu Alexandru Serdean Ioan Marinescu Dan- tefan S Popa Vasile Marin Mirela Maxim Gabriela Bejan Cornelia - Livia Brnzei Dan Teodoru Georgeta Roman Neculai Bor Dan Mircea s Haivas Mihai Popa Claudiu - Stefan Minu Petru t

Locul de munc a Data Lic. "G. Ibrileanu", Iai a s 10 Univ. Tehn. "Gh. Asachi", Iai s 14 Inst. de Mat. al Acad. Rom., Bucureti 20 s S. C. Selgros Cash&Carry, Braov s 20 Lic. "N. Titulescu", Braov s 20 C. N. "A.T. Laurian", Botoani (elev) 25 s C. N. "Aurel Vlaicu", Ortie as 28 C. N. "Iancu de Hunedoara", Huned. 28 C. N. "V. Alecsandri", Galai t 29 Sc. "Al. Vlahua", Iai t s 12 Sc. "Al. Vlahua", Iai t s 14 Univ. Tehn. "Gh. Asachi", Iai s 25 Univ. "Al. I. Cuza", Iai s 29 Univ. Tehn. "Gh. Asachi", Iai s 09 Sc. "V. Alecsandri", Mirceti (Iai) s s 11 Univ. Tehn. "Gh. Asachi", Iai s 13 Inst. Cerc. Ec. "Gh. Zane", Iai s 18 Sc. "Alecu Russo", Iai s 27 Univ. "D. Cantemir", Tg. Mure s 30

Revista semestrial RECREAII MATEMATICE este ed ASOCIAIA RECREAII MATEMATICE. Apare la datele de 1 1 septembrie i se adreseaz elevilor, profesorilor, studenilor i tutu pasionai de matematica elementar. n atenia tuturor colaboratorilor Materialele trimise redaciei spre publicare (note i articole, che metodic, probleme propuse etc.) trebuie prezentate ngrijit, clar i co trebuie s prezinte interes pentru un cerc ct mai larg de cititori. Se reco textele s nu depeasc patru pagini. Evident, ele trebuie s fie origin nu fi aprut sau s fi fost trimise spre publicare altor reviste. Rugm rialele tehnoredactate s fie nsoite de fiierele lor. Problemele destinate rubricilor: Probleme propuse i Problem pregtirea concursurilor vor fi redactate pe foi separate cu enun i de ie/rezolvare (cte una pe fiecare foaie) i vor fi nsoite de numele autoru la i localitatea unde lucreaz/nva. Redacia va decide asupra oportunitii publicrii materialelor prim n atenia elevilor Numele elevilor ce vor trimite redaciei soluii corecte la proble rubricile de Probleme propuse i Probleme pentru pregatirea conc vor fi menionate n Pagina rezolvitorilor. Se va ine seama de regulile: 1. Pot trimite soluii la minimum cinci probleme propuse n prezent i cel anterior al revistei; pe o foaie va fi redactat soluia une probleme. 2. Elevii din clasele VI-XII au dreptul s trimit soluii la pr propuse pentru clasa lor, pentru orice clas mai mare, din dou clase m imediat anterioare. Elevii din clasa a V-a pot trimite soluii la problemel pentru clasele a IV-a, a V-a i orice clas mai mare, iar elevii claselor trimite soluii la problemele propuse pentru oricare din clasele primare i s mai mare. Orice elev poate trimite soluii la problemele de concurs (tip 3. Vor fi menionate urmtoarele date personale: numele i pr clasa, coala i localitatea. 4. Plicul cu probleme rezolvate se va trimite prin pot (sau v direct) la adresa Redaciei: Prof. dr. Temistocle Brsan Str. Aurora, nr. 3, sc. D, ap. 6, 700 474, Iai Jud. IAI E-mail: tbirsan@math.tuiasi.ro

CUPRINS

Elogiu adus revistei Gazeta Matematic la 110 ani de apariie nentrerupt ... 100 de ani de la naterea matematicianului Grigore C. Moisil ..............................

ARTICOLE I NOTE

D. M. BTINEU-GIURGIU Asupra problemei 809 din Gazeta Matematic, volumul VIII (19021903).................................................. G. DOSPINESCU Cteva proprieti ale subgrupurilor finite din GLn ( ) ....

T. BRSAN Ceviene i triunghiuri triomologice.................................................. A. MOSCALIUC Construcii aproximative cu rigla i compasul ale numrulu I. V. MAFTEI Inegaliti generatoare de noi inegaliti ..................................... C.-t. POPA Asupra unei probleme dat la ONM, Bistria, 2005....................

NOTA ELEVULUI

M. TIBA Asupra criteriului de congruen LLU................................................. I. OLAN O generalizare a identitii Botez - Catalan ......................................... M. PACHIARIU Acoperiri ale planului laticial cu figuri................................

CHESTIUNI METODICE

M. TETIVA Metoda normrii ................................................................................ Gh. IUREA Asupra unei recurene de ordin doi ................................................

CORESPONDENE PROBLEME I SOLUII

Olimpiada Internaional de Matematic B. O. Zhautykov ..................................

Soluiile problemelor propuse n nr. 1/2005............................................................. Soluiile problemelor pentru pregtirea concursurilor din nr. 1/2005 .................... Probleme propuse........................................................................................................ Probleme pentru pregtirea concursurilor ................................................................. Training problems for mathematical contests ..........................................................

Pagina rezolvitorilor .................................................................................................

ASOCIAIA RECREAII MATEMATICE .........................................................

Anul IX, Nr. 1

Ianuarie Iuni

RECREAII MATEMATIC

REVIST DE MATEMATIC PENTRU ELEVI I PROF

e i = 1

Asociaia Recreaii Matemati IAI - 2007

Semnificaia formulei de pe copert: i ntr-o form concis, formula e = 1 leag cele patru ramuri fun
ale matematicii: ARITMETICA GEOMETRIA ALGEBRA ANALIZA MATEMATIC reprezentat reprezentat reprezentat reprezentat de de de de 1 i e

Redacia revistei :

Petru ASAFTEI, Dumitru BTINEU-GIURGIU (Bucureti), Temistocle BRS BRNZEI, Ctlin - Cristian BUDEANU, Constantin CHIRIL, Eugenia COHA CORDUNEANU, Mihai CRCIUN (Pacani), Gabriel DOSPINESCU (stude Paraschiva GALIA, Paul GEORGESCU, Mihai HAIVAS, Gheorghe IUREA, Georges LDUNC, Mircea LUPAN, Gabriel MRANU, Andrei NEDELCU POPA, Dan POPESCU (Suceava), Florin POPOVICI (Braov), Maria RACU ROMAN (Mirceti), Ioan SCLEANU (Hrlu), Ioan ERDEAN (Ortie), (Bucureti), Marian TETIVA (Brlad), Lucian TUESCU (Craiova), Adrian ZA (Bacu), Adrian ZANOSCHI, Titu ZVONARU (Comneti).

Adresa redaciei: Catedra de Matematic Universitatea Tehnic Gh. Asachi Iai Bd. Carol I, nr.11, 700506, Iai Tel. 032 213737 / int. 123 E-mail: recreatii.matematice@gmail.com http://www.recreatiimatematice.uv.ro

COPYRIGHT 2007, ASOCIAIA RECREAII MATEMATICE Toate drepturile aparin Asociaiei Recraii Matematice. Reproducerea in parial a textului sau a ilustraiilor din aceast revist este posibil numai cu acordu scris al acesteia. TIPRIT LA SL&F IMPEX IAI Bd. Carol I, nr. 3-5 Tel. 0788 498933 E-mail: simonaslf@yahoo.com

Anul IX, Nr. 1

Ianuarie Iunie

RECREAII MATEMATICE

REVIST DE MATEMATIC PENTRU ELEVI I PROF

e i = 1
Revist cu apariie semestrial publicat de

ASOCIAIA RECREAII MATEMATIC

IAI - 2007

300 de ani de la naterea lui s Leonhard Euler (1707 1783)

Citii pe Euler! Citii pe t t este Maestrul nostru, al t P. S. La

El este geniul care a pus geniile succesorilor sai. J. Bert

Matematician, astronom, fizician Leonhard Euler a fost fara ndoia prolic i unul dintre cei mai mari s cieni ai tuturor timpurilor, dominnd tate matematica secolului XVIII (apr ca valoare de Lagrange). Euler, ca savant, a reunit formidabil cu o imaginaie creatoare a t al, o memorie inegalabil, abiliti de a a at traordinare i o putere de munca fant s A debutat n activitatea de ce vrsta de 16 ani cu o expunere privin lui Newton i Descartes. n pofida f s la 28 de ani pierde vederea la ochiu la 54 de ani ramne complet orb, a co asiduitate aceast munc pn la vrs a a a ani, cnd, aa cum afirm Condorcet s a de odata cu a calcula i de a trai". A redactat anual, n medie, n jur de s text tiinic publicnd peste 900 de articole i 90 de volume (din care 62 d s t s tic, mecanic i astronomie). Dup ce a orbit complet, a dictat secretari a as a de memorii (dintre care multe sunt volume ntregi). Leonhard Euler s-a nascut la 15 aprilie 1707 la Basel, n Elveia, t unui preot sarac. Tatal sau, Paul Euler, era pasionat de matematica i s s tineree cu Jean i Iacob Bernoulli. Paul i iniiaz fiul n matematic t s s t a a ca acesta s-i continue cariera de preot l trimite s studieze filozofia i a a s Universitatea din Basel. Aici are ca profesor pe Jean Bernoulli care remar sau matematic de excepie. Leonhard Euler devine prieten i colabora t s profesorului, matematicienii Nicolas (16871759) i Daniel Bernoulli (17 s La propunerea acestora n 1727 devine membru al Academiei de Stiine t Petersburg, nfiinata de arina Ecaterina I-a a Rusiei. n 1730 obine c t t t matematica la aceasta academie. n acelai an se casatorete cu fiica unui s s cu care a avut 13 copii, dintre care numai 5 i-au supravieuit. n 1740 rege t al II-lea (cel mare) al Prusiei reorganizeaz Academia din Berlin, unde a numit director al seciei de matematica. Postul de preedinte al acestei a t s

fost atribuit lui Voltaire, ceea ce l nemulumete pe Euler. n 1866, la t s arinei Ecaterina a II-a i n urma unui conflict cu regele Frederic, se t s Sankt Petersburg mpreuna cu fiii sai Johan Albrecht (17341800), mat Carol (17401790), medic i Cristoph (17431812), ofier n armata prusa s t apoi a murit ca general n armata rus. Euler este numit director al Aca a Stiine din Sankt Petersburg i i se ofera o locuina confortabila. Din ne s t t scurt timp orbete complet, iar casa i este distrusa de un incendiu n car s Euler a fost salvat cu dificultate. Cu aceast ocazie i-a pierdut i o m a a s s dintre manuscrise. La vrsta de 64 de ani i moare soia i se recstorete t s aa s vitrega a primei sale soii. La 18 septembrie 1783, dupa ce rezolvase o t dificila privind micarea baloanelor (inventate de curnd de Mongolfier ) s ceai mpreun cu nepoii si le spune acestora "eu mor". Acestea au fos a t a cuvinte ale acestui savant ntre savani, supranumit de contemporanii si t a matematicii sau Analiza ncarnata. Apropiaii sai au fost unanimi n al t un om blnd i binevoitor, de o modestie copleitoare. s s Extraordinara vitalitate a lui L. Euler, pn n ultima clip a vieii, a a t prin constituia sa robust i sntoas, viaa organizat, munca sa con t a s a a a t a siunea pentru matematica, precum i prin credina sa. A fost de o re s t profund (o justicare a existenei lui Dumnezeu, bazat pe noiunea de a t a t pstrat n unele manuale de teologie pn n zilele noastre). a a Opera lui L. Euler a impulsionat profund ntreaga dezvoltare a matem asimilat n mod critic matematica marilor sai predecesori (Fermat, Pascal, Newton, Leibniz .a.), a mbogit coninutul matematicii, a introdus met s at t cercetare, a introdus simboluri adecvate, a promovat utilizarea aparatului n domenii nematematice i a insistat pentru expunerea clar, logic i ntr s a as accesibil unui cerc ct mai larg de cititori a problemelor de matematica. E de dificil s se prezinte n cteva rnduri uriaa contribuie a lui Euler n m a s t n teoria numerelor L. Euler este primul care a studiat sistematic pro tribuiei numerelor prime introducnd pentru prima oar aa numita fun t a s lui Riemann (n cazul argumentului real), a dat noi demonstraii teoremei t (care afirm c exist o innitate de numere prime), a artat necesitate a a a a suciente date de Euclid pentru ca un numr natural par s fie perfect. A a a funcia indicatoare care i poart numele i cu ajutorul creia a generalizat t a s a strat mica teorema a lui Fermat, a dat primul enun al teoremei lui Dirich t numerele prime din progresii aritmetice, a generalizat algoritmul lui Euc ducnd parantezele lui Euler ), a perfecionat aparatul fraciilor continu t t teoria resturilor patratice i a demonstrat teorema privind reprezentarea s ntregi prin forme patratice. Abilitaile sale de calcul l-au ajutat sa gaseasc t care ne uimesc i astzi: a dat exemple de numere perfecte mergnd pn s a a lui 1010 , a dat 65 de exemple de perechi de numere prietene (amiabile), e numere prime gemene i a construit contraexemple care dovedeau c unele s a obinute de predecesori ai sai, prin inducie incompleta, nu sunt adevarat t t n Geometrie, L. Euler a studiat transformrile de coordonate n spa a t ian, a stabilit reprezentrile analitice ale unor guri din acest spaiu (cilind a t suprafee de rotaie), a facut un studiu analitic al conicelor, a studiat c t t

spaiul euclidian, a introdus noiunile de linie geodezica i curbura norm t t s suprafee, a studiat suprafeele de arie minima care includ o curba nchis t t n geometria elementara numele sau este legat de cercul celor noua punct pe care se afl centrul de greutate, ortocentrul i centrul cercului circum a s triunghi (dreapta lui Euler ) etc. El este iniiatorul cercetarilor de topologie algebrica, stabilind n 17 t v m + f = 2 (v = numarul vrfurilor, m = numarul muchiilor i f s feelor unui poliedru convex), rezolvnd problema celor apte poduri din K t s etc. n Algebra a ncercat sa demonstreze teorema fundamentala a algebrei, teorema funciilor simetrice i a dezvoltat teoria determinanilor. t s t A obinut rezultate numeroase i fundamentale n diverse domenii: ca t s renial si integral (a publicat dou tratate renumite de analiz matemati t a a ecuaiilor difereniale ordinare si cu derivate pariale, teoria integralelor e t t t A introdus integrala multipla, funciile gamma si beta, a definit num t logaritmul natural, a definit funciile trigonometrice ca funcii circulare, a t t ei = 1 i, mai general, eix = cos x + i sin x etc. s De la el motenim multe din notaiile folosite astazi: f (x), e, , e s t (constanta lui Euler ) .a. s Euler a adus contribuii n mecanica punctului i mecanica corpului soli t s noiuni ca: centru de masa, centru de inerie, moment de inerie. n 1755 t t t un tratat de hidrodinamica cu numeroase aplicaii practice. A fost preoc t optica i tehnica (construcia navelor). t s Se spune c n ultimii ani de viaa L. Euler avea n memorie fiecare am a t creaia sa uriaa. Dar Euler nu a fost numai un mare matematician. El era t s vasta cultura generala avnd cunotine bogate de teologie, filozofie, botani s t istorie, medicin, literatur i muzic (a scris i o teorie matematic a muzic a as a s a din memorie n ntregime "Eneida" i tia cu ce vers ncepe i se termin fiec s s s a din cartea lui Virgiliu. Aa cum afirma J. Bertrand, nici un alt mare savant nu a lucrat cu s zel, mai multa rvna si cu mult folos pentru progresul unei stiine, ca Eu t matematica.

Prof. dr. Petru M

Profesorul Dumitru Ion Mangeron (1906 1 In Memoriam

Printre profesorii de seam, nscui n ur a a t secol, se numara i regretatul D. I. Mangero s cuta personalitate a vieii universitare i t s ieene, D. I. Mangeron s-a nscut la 15/28 s a 1906 n oraul Chiinu. Dup absolvirea s s a a oraul sau natal s-a nscris n anul 1926 la s matematica a Facultaii de tiine din Iai, s t s t atunci existau secii pentru toate tiinele exa t s t A absolvit n mod strlucit studiile univ a anul 1930, avnd profesori de mare valoare, da sa-l amintim dect pe Alexandru Myller, care toratul la vestitul matematician David Hilbe cu ninarea n 1910 de ctre A. Myller a Se t a matematic, care astazi i poarta numele, la Ia cu adevarat o activitate propriu-zisa de cerceta i-a adus aportul i fostul student D. I. Mange s s de doi ani (1930 - 1932), pregtete sub ndrumarea renumitului profe a s Picone teza de doctorat, intitulata Sopra un problema al contorno per un dierenziale alle derivati parziali di quatro ordine con le caratteristiche re ntors n ar, a funcionat la Universitatea ieean mai nti ca asistent, t a t s a n 1936 confereniar la disciplina de analiz matematic. ncepnd cu anu t a a tar 1938/1939, a fost numit confereniar la disciplina de matematici gener t ninata Scoala Politehnica din Iai, unde, n anul 1941, devine profesor t s de mecanic. ncepnd cu acest an, a predat cursuri de matematic i de a as teoretic la facultile de mecanic i de electrotehnic. Preocuprile s a at a s a a ce au fost strns legate i de cerinele unui nvaamnt tehnic superior s t t inuta, publicnd aum, pe lnga lucrarile din teoria ecuaiilor cu derivat t t i numeroase lucrri de mecanica analitica, teoria mecanismelor, teoria o s a teoria acceleraiilor reduse si a acceleraiilor de ordin superior. A publ t t crari de istoria matematicii. A fost membru activ la mai multe societai t de matematica, mecanica teoretica i aplicata, astronomie, om de tiina s t s Romniei, redactor responsabil la Buletinul Institutului Politehnic din I bru n consiliul de conducere al revistelor Revue Roumaine des Sciences T (Srie de Mcanique Applique) i Studii si Cercetari de Mecanica Aplicata s tor de doctorate n mecanica tehnica (30 de ingineri au obinut titlul de d t ndrumarea sa). Studiul ecuaiilor cu derivate pariale a devenit principalul su domeni t t a tare n care, singur sau cu unii dintre elevii i colaboratorii sai, a public s lucrari n diverse reviste de specialitate din ara noastra sau din ari cu mar t t tiinific, ceea ce face ca acum n matematic s se vorbeasc despre ecuaii s t a a a t Mangeron, operatori interpolani Mangeron, teoreme Mangeron, ecuaiile t t Profesorul D. I. Mangeron a fost un om plin de entuziasm, energie

iniiativ, colegialitate desvrit, un om care ncuraja tineretul studios t a a s a capabil de prietenia cea mai sincera. Dovada stau scrisorile pline de aten catee catre bunul sau prieten de o viaa, Al. C. Climescu, trimise de la Un t t din Edmonton (Canada), unde n mai multe rnduri a fost visiting profe ieirea sa la pensie. Cu alt ocazie, despre Al. C. Climescu a scris: A f s a atunci (la nceputul razboiului) mobilizat n arma antiaeriana si astfel au de zile, pna n 1945. ntre timp, am reuit sa se formeze comisia de conc s ncadrarea sa ca profesor. Comunicndu-i ca el se va ntoarce pe noua sa profesor titular, am primit cu emoie telegrama n care El afirma ca nu va t data acest rezultat al eforturilor facute n lipsa lui. Al. C. Climescu a avu se revaneze atunci cnd, aa cum scrie Profesorul, n acele vremuri s-a fac s s rasa din tot ce am lucrat si realizat eu. n 1954, Al. C. Climescu descop vista Memorial des Sciences Mathmatiques un articol intitulat Calcul S n care exista fraza: On leur doit Mauro Picone et D. Mangeron de principaux. Aceasta recunoatere internaionala a facut ca, n cele din urm s t la care era supus Profesorul s nceteze, s i se recunoasc meritele de cer a a a valoare i, partea oarecum comic, s i se acorde o nou diplom de docto s a a a a fizico-matematice pentru ca, chipurile, cea din Italia nu ar fi fost buna. cu aceasta, Profesorul a spus: Am trait astfel, n cadrul greutatilor pe care ani de zile, o reuita faptica si morala, graie ateniei si preocuparilor d s t t cercetator al iubitului nostru Prieten disparut. Un aspect foarte important al activitaii Profesorului D. I. Mangeron t de nfiinarea revistei Buletinul Institutului Politehnic din Iai (1946), p t s a trebuit s nving greutile ivite dup terminarea rzboiului i obtuzita a a at a a s factori de decizie. A contribuit din plin la afirmarea pe plan internai t numai naional a acestei reviste, mpreuna cu ali profesori ai Politehnicii t t de tiina din alte ari, printre care i civa laureai ai premiului Nobel. s t t s t t Profesorul D. I. Mangeron a rmas activ i optimist pn la inevitab a s a dei a locuit ntr-un apartament modest, lipsit de confort. Cnd, n cele din s rector al Politehnicii, devenita acum Universitatea Tehnica "Gh. Asachi", obin pentru Profesor o repartiie ntr-o nou locuina, acesta a spus: "D t a t a t este prea trziu". Prea trziu, a sosit i tirea alegerii sale ca membru co s s al Academiei Romne. Se afla n spital, unde a decedat la 26 februarie 19 Ne-am desparit de stimatul nostru Profesor n ziua de 1 martie 1991, d t la care au asistat muli dintre colaboratorii i elevii si, colegi, foti d t s a s ingineri i profesori, inut la biserica Sfntul Nicolae din dealul Copoului. s t a ctre cimitirul Podgoria, o ninsoare linitit a adus mpcarea n sufletele a s a a ne-am gndit ca, cel care a iubit att de mult coala, oamenii i natura a pl s s noi la fel de frumos cum a trit, lsnd o amintire i o oper pe msur, a a s a a a spiritele cu adevrat alese. a Astzi, la mormntul Profesorului D. I. Mangeron se afl un monume a a de fotii sai admiratori, iar Bulevardul pe care se afla facultaile i celela s t s ale Universitii Tehnice poart numele "Bulevardul D. Mangeron". at a

Prof. dr. Adrian Cordune

Grigore Moisil concentrat n 1939, Sineti, cu grad de subloco s


(persoana culcat) a

Fotografia-document (recto-verso) a fost oferit pentru publicare red a de catre conf. dr. Gh. Costovici.

Similitudini n plan i puncte Torricelli asoc s


Ctlin TIGAERU 1 aa

Subiectul lucrarii l reprezinta operaia de compunere a similitudinilor ap t conguraii geometrice: un triunghi ABC i doua puncte arbitrare M, N t s / la care vom ataa punctul P , care este centrul similitudinii S3 = S2 S1 , un s similitudinea centrat n C, care l transport pe M n A, iar S2 este cent a a transportndu-l pe A n N . Doua aspecte vom lamuri, legate de subiectu n primul rnd, vom arata cum rolul pe care l joaca punctul P poate fi pr punctele M i N . Mai mult, vom demonstra c putem inversa rolurile triu s a ABC i M N P . Al doilea aspect al lucrrii se refer la identicarea a do s a a Torricelli generalizate, pe care le numim asociate compunerii celor doua si n nal vom lamuri i o situaie interesanta, credem, cu caracter de noutate s t coincidena acestor puncte. n expunere se folosete formalismul complex t s permite atacarea unor probleme grele, pentru care soluia sintetic se dov t a n prima faza, greu de vazut. Interpretarile geometrice nsoesc, n limita t rezultatele teoretice. Mai precizam ca unele rezultate sunt demonstrate n

1. Compunerea similitudinilor i teorema fundamental. Mulim s a t lor planului P se identic, prin fixarea unui reper, cu mulimea numerelor a t

Definiie. Similitudinea de centru M0 P , de raport k [0, ) t s (, ] este funcia SM0 (k, ) : P P , definita astfel: daca M P t SM0 (k, ) (M ), avem |M0 M 0 | \ (a) dac M 6= M0 , atunci k = a , = m(M M0 M 0 ); |M0 M | (b) daca M = M0 , atunci M 0 = M0 .
1

Sa notam ca, daca z 0 este afixul lui M 0 , atunci z 0 z0 kei = . z z0 Propoziia 1. Consideram similitudinile SM1 (k1 , 1 ), SM2 (k2 , 2 ), a t lor M1 si M2 ind respectiv z1 si z2 . Daca k1 k2 ei(1 +2 ) 6= 1, atunci exis X, de afix x, astfel nct SM2 (k2 , 2 ) SM1 (k1 , 1 ) = SX (k1 k2 , 1 + afixul lui X este determinat de relaia t 1 k2 ei2 1 k2 ei2 = z1 + (z2 z1 ) x = z1 + (z2 z1 ) i1 k ei2 1 k1 e 1 k1 k2 ei(1 + 2 Demonstraia acestui rezultat clasic se gsete n [4]. S notm c rela t a s a a a a t s k1 k2 ei(1 +2 ) = 1 este echivalent cu relaiile k1 k2 = 1 i 1 + 2 = 0 (m
1

Reamintim c similitudinile sunt bijecii, anume (SM0 (k, )) = SM0 a t Dac z0 este afixul lui M0 i z afixul lui M , atunci expresia analitic a si a s a este descrisa de funcia t sz0 (k, ) (z) = z0 + (z z0 ) kei .

Lect. dr., Univ." tefan cel Mare", Suceava S

n cele ce urmeaz, fixm cadrul n care se va desfura analiza noastr a a as se considera triunghiul ABC, cu afixele respectiv a, b, c i fie M , N doua s plan, fixate, diferite de A, B i C, de afixe m i n. Luam n considerare sim s s de centre C i B, care transfer pe M n A, respectiv pe A n N i vom s a s punctul P , care va fi centrul compunerii celor dou similitudini. Tinnd c a ac nb i de (2), dac punem k1 ei1 = s a , k2 ei2 = , atunci punctul P mc ab a SB (k2 , 2 )SC (k1 , 1 ) = SP (k1 k2 , 1 + 2 ), afixul su, notat cu p, fiind d p = c + (b c) 1
na ba ac nb mc ab

Pentru nceput cteva observaii, legate de aspectele geometrice foarte p t ale formulei de mai sus, pe care cititorul le poate verifica prin calcul direc cazul particular k1 k2 6= 1 i 1 = 2 = 0 (mod ) este echivalent c s lui Menelaos; dac 1 + 2 = 0 (mod ), atunci punctele M , N , P sunt colinia a k1 k2 = 1 i 1 + 2 = , punctul P fiind mijlocul segmentului M N ; s o analiz a formulei arat c, dac M, N {A, B, C}, atunci i P a a a a / s exista situaii pentru care P A, cum se verifica n: a = 0, b = 1, c = i t n = i; conform (3), p = a = 0. Urmatorul rezultat lamurete prima problema asociata tripletelor{A s {P, M, N }, descrisa n introducere.

Teorema 1. Daca P are afixul p, determinat de formula (3), atunci cb ma ba (a) daca punem k3 ei3 = , k4 ei4 = , k5 ei5 = , k6 ei pb ca na va rezulta SA (k4 , 4 ) SB (k3 , 3 ) = SN (k3 k4 , 3 + 4 ), SC (k6 , 6 ) SA SM (k5 k6 , 5 + 6 ); nm bp pn (b) daca punem q1 ei1 = , q2 ei2 = , q3 ei3 = , q4 ei am np bn mp an q5 ei5 = , q6 ei6 = , atunci SP (q2 , 2 )SM (q1 , 1 ) = SC (q1 q2 cp mn SM (q4 , 4 )SN (q3 , 3 ) = SA (q3 q4 , 3 + 4 ), SN (q6 , 6 )SP (q5 , 5 ) = SB (q5 Demonstraie. ncepem prin a demonstra o forma echivalenta a form t

Lem. Afixul punctului P , notat cu p, care este centrul compuneri a dinilor SB (k2 , 2 ) SC (k1 , 1 ), verica relaia t np ac nb = mp mc ab si reciproc, daca afixul p verifica (#), atunci verifica si (3). Demonstraie. Din ipoteza rezulta ca SP (k1 k2 , 1 + 2 ) (M ) = N t conform formulei (10 ), rezult (#). S presupunem c afixul punctului P ve a a a ac nb i(1 +2 ) = Daca notam cu w = k1 k2 e , atunci putem scrie ( mc ab ac n wm, de unde (p c) (1 w) = n wm (1 w) c = n c ab

Revenim la demonstraia teoremei. Pentru punctul (a), trebuie demo t dac p satisface (3), atunci m i n verific formulele analoge, ceea ce, a s a lemei, revine la demonstrarea formulelor mn ma cb i = s pn ca pb pm ba pc = . nm na bc nb ma pb mn 1 Din (3) se obine t = ab mc i din (#) deducem s = cb 1w pn nb ma mc ab ma pb mn = = , de un departe avem cb pn ab mc ca nb ca 0 00 (# ). Analog se demonstreaza i (# ). Pentru punctul (b), trebuie demo s dac p satisface (3), atunci a, b, i c satisfac relaiile a = n + (m n) a s t

na bn ac + ab + cn = (b c) , de unde (3). ab ba

1 ca pn i celelalte, ceea ce este echivalent cu a demonstra relaiile s t = ba bn celelalte, care nu sunt altceva dect rescrieri ale relaiilor (#), (#0 ) i (#00 t s

2. Punctele Torricelli asociate compunerii a dou similitu a continuare, procedm dup cum urmeaz: considerm punctele A0 , B 0 a a a a afixe a0 , b0 i respectiv c0 , denite de A0 = SC (k1 , 1 ) (P ), B 0 = SA (k5 s C 0 = SB (k3 , 3 ) (N ). Din Teorema 1 deducem i ca A0 = SB (1/k2 , 2 ) (P ), B 0 = SC (1/k6 , s C 0 = SA (1/k4 , 4 ) (P ). Trecnd la nivelul afixelor, relaiile de mai sus t n formulele ac ab a0 = c + (p c) = b + (p b) , mc nb ba bc b0 = a + (m a) = c + (m c) , na pc 0 c = b + (n b) c b = a + (n a) c a . pb ma Dac inversm rolurile tripletelor {A, B, C} i {P, M, N } (Teorema 1 n a a s s acest lucru), putem considera analogele punctelor A0 , B 0 i C 0 , anume N 0 , de afixe p0 , m0 i respectiv n0 , unde P 0 = SN (q3 , 3 ) (A), N 0 = SM (q s M 0 = SP (q5 , 5 ) (B), analogele formulelor (4) fiind pn pm = m + (a m) , p0 = n + (a n) bn cm mp mn = n + (b n) , m0 = p + (b p) cp an n0 = m + (c m) n m = p + (c p) n p . am bp 0 0 0 0 0 0 Propoziia 2. P P AA , M M BB , N N CC sunt paralelograme, ev t generate. Demonstraie. Demonstram relaiile importante: t t

Raionamentul se urmarete uor n cele ce urmeaza: din (5) se obin t s t s (b a) (n p) (n p) (b a) i din (4) se obine a0 a = s t ; de aici bn nb 0 0 t s a a = (p p ); celelalte relaii se deduc n acelai fel. Conchidem perechile de segmente {[AA0 ] , [P P 0 ]}, {[BB 0 ] , [M M 0 ]} i {[CC 0 ] , [N N 0 ]} s pectiv congruente, paralele sau confundate, ceea ce ncheie demonstraia. t

a a0 = (p p0 ) ,

b b0 = (m m0 ) ,

c c0 = (n n0 )

Propoziia 3. Sunt adevarate urmatoarele relaii: t t c0 b cb c b0 , = 0 = ab a b a b0 pn p n0 p0 n = 0 = ; mn m n0 m n 0 0 0 0 ca mn bc p m0 a b0 np = , = , = . m p0 b a0 p n0 a c0 n m0 c b0 Demonstraie. Relaiile (4) se mai scriu i t t s 0 0 c a na b a a b pb c b b0 c mc a = = 0 ; = = 0 ; = = 0 ca ma b a a b nb c b b c pc a 0 c b cb Din a doua relaie se obine t t = 0 . Analog se obine i egalitatea t s ab a b raport din (7). Inversnd rolurile triunghiurilor ABC i M N P , rezulta s s marcam i relaiile s t p0 n an m n m0 p bp n p n0 m cm p = = 0 , = = 0 , = = pn bn m n m p cp n p n m am p 0 t care sunt echivalentele relaiilor (4 ). Relaiile (9) se deduc astfel: din t (p c) (a c) (n b) a0 c = , respectiv (5), coroborat cu (#), obinem 0 t a b (p b) (a b) (m c) (p c) (n b) (a c) (a n) s a a . nmulind (#), (#0 ) i (#00 ), rezult c t (a b) (a m) (p b) (m c) ceea ce, dup nlocuire, ncheie demonstraia. a t

Teorema 2. Consideram punctele A0 , B 0 , C 0 , de afixe a0 , b0 si re definite de relaiile (4) si punctele P 0 , M 0 , N 0 , de afixe p0 , m0 si respect t satisfac relaiile (5). t b a ma (a) Daca R, atunci tripletele {AA0 , BB 0 , CC 0 } si {M M 0 , na ca sunt formate din drepte paralele. b a ma (b) Daca R, atunci tripletele {AA0 , BB 0 , CC 0 } si {M M 0 , / na ca sunt concurente. Demonstraie. n virtutea Teoremei 1, sunt suciente demonstraiil t t ilor referitoare la tripletul {AA0 , BB 0 , CC 0 }. Demonstram ca AA0 k BB 0 k c0 a na ca (4) se obine t = R, adica C 0 AB. Din (7) deducem ba b a ma c0 a ca s a = R, adic A0 BC i B 0 CA. Pe de alt parte, din ( a b0 a ba

b0 ba b b0 a b0 b c c0 = = R, deci = , ceea ce nseamn c a 0 0 0 cc ca ac bb c c0 0 0 0 0 a BB i CC sau sunt paralele sau confundate. Dac BB CC , atun s AC CC 0 = {C}, ceea ce, n virtutea lui (4), ar conduce la M C sau B ce este fals. Deci BB 0 k CC 0 , la fel demonstrdu-se i celalalt paralelism. s c a

B A M N T T B P C N

Figura 1 ba ma Demonstrm punctul (b); dac a a C\R, rezult c dreptele a a na ca 0 i CC se intersecteaz cel puin dou cte dou. Fie {T } = BB 0 CC 0 i a t a a s s su. Va rezulta c exist , R astfel nct b b0 = (b0 t), c c0 = a a a b0 a b0 b0 a b b0 innd cont i de t s = 0 , dedusa din (40 ), ajungem la : ca c c ca c a a R; cum {C 0 , A, B} nu sunt coliniare, rezult c patrulaterul B 0 CT A este in a0 c 0 Analog dovedim ca BC AT este inscriptibil. Din (7) rezulta ca 0 a b b0 a c 0 0 s a a deoarece B CT A este inscriptibil i deoarece T BB , rezult c c a b0 0 0 a c tc b t R, deci 0 : R, adica i BA0 CT este inscriptibil. Demo s tb a b tb t T AA0 ; condiiile de inscriptibilitate ale patrulaterelor se pot scrie i t s t

t c b0 a b0 c bc = 0 0 R i innd cont de s t , dedus din (7), a t a b c ba0 b a 0 0 ta ta b c b a t c R. Rezulta ca AA0 , BB 0 i s R, adica t c ba0 b0 c t a ta0 0 0 0 0 s concurente. Inversnd rolurile tripletelor {AA , BB , CC } i {M M , N obinem i concurena dreptelor P P 0 , M M 0 i CC 0 . Demonstraia este n t s t t s R,

Observaia 1. La punctul (a) nu obinem AA0 k BB 0 k CC 0 k P P 0 k M t t cum s-ar parea ca rezulta din Propoziia 2, deoarece n exemplul a = 0, b t 1 m = 1, n = i, p = 0, unde a0 = (1 + i), b0 = n = i, c0 = m = 1, p0 = 2 0 avem AA0 P P 0 , BB 0 M M 0 , CC N N 0 . Observaia 2. Geometric, condiia t t

ba ma R, care se refer a na ca \ \ poziia punctelor din ipotez, se traduce prin m(N AB) + m(CAM ) {0 t a \s \ unghiurile N AB i CAM sunt sau opuse ca orientare i egale n valoare ab s suplementare.

Observaia 3. Cele spuse se urmresc uor pe figura 1, corespunztoa t a s a 1 > 0, 2 > 0. Se remarca paralelogramele din Propoziia 2 i urmatoare t s triunghiuri direct asemenea, n ordinea n care sunt scrise, care se deduc pretrile geometrice ale formulelor (7), (8) i (9): 4C 0 BC 4N BP a s 4N AP 0 , 4A0 CA 4P CM 4BCB 0 4P 0 AM , 4B 0 AB 4M AN 4M CN 0 i 4C 0 BA 4CBA0 4CB 0 A 4N 0 M P 4N M P 0 s Tot de aici se deduce faptul c, dac, de exemplu, M este un punct im a a 4ACB 0 , atunci i N i P sunt acelai tip de punct n triunghiurile analog s s s

Observaia 4. Punctele T i T 0 , obinute la punctul (b), sunt puncte t s t generalizate asociate tripletelor {A, B, C} i {P, M, N }. n adevar, daca s exemplu, = |b c|, = |c a0 |, = |a0 b| (sau orice numere pro cu ele), atunci punctul T este punctul n care suma |XA| + |XB| + atinge minimul, unde X este un punct oarecare din plan. Analog, punct punctul n care suma |XP | + |XM | + |XN | i atinge minimul. Pent s recomandm cititorului paragraful 1.49 din [3], unde demonstraiile sunt a t pe larg i unde sunt expuse i alte proprieti ale punctelor Torricelli. s s at n continuare, ne plasam n condiiile punctului (b) din Teorema 2. t

Teorema 3. T T 0 daca si numai daca punctele A, B, C, M , N conciclice. Demonstraie. Daca T T 0 , atunci relaia (6) asigura coincidena t t t AA0 P P 0 , respectiv BB 0 M M 0 i CC 0 N N 0 , adic {C 0 , N, a s {P 0 , A, T, P, A0 }, {M 0 , B, T, M, B 0 } sunt puncte coliniare, deci exist , a ca c0 t = (c0 n), a t = (a p). a t c0 Deoarece patrulaterul AC 0 BT este inscriptibil, rezulta ca a b c0 a p c0 b unde, dup nlocuire, se obine a t R. Tinnd cont de relaia t a b c0 n

P C N A M C A N B

T B M P

Figura 2 c0 b cb ap cb din (4), putem scrie 0 = , care conduce la R, d c n cp ab cp laterul ABP C este inscriptibil. Analog se demonstreaza i ca M , N apar t s circumscris triunghiului ABC. Reciproc, s presupunem c punctele A, a a s a t N , sunt conciclice i demonstrm c T T 0 i c punctul P aparine ce s a a ab b0 b = ; dup a cumscris triunghiului ABC. Din (4) deducem c a nm na ab mn cu R, asigurat de patrulaterul inscriptibil AN BM , a na mb 0 0 b b b b mn R, deci R, adica M BB 0 ; folosind celalalt pat nm mb mb 0 0 0 0 scriptibil, se arat i c N CC ; de aici BB M M , CC N N 0 , de un as a a0 a a0 a p m 0 a a c T T . Deoarece P AA0 , rezult c a R, adic a pa pm pa 0 a a ca ca p ce, coroborat cu = , dedus din (4), conduce la a pm cm cm p deci patrulaterul AM CP este inscriptibil. Q. e. d.

De interes credem c este o interpretare fizic a teoremelor 2 i 3 (v. [3], a a s Bibliografie

1. C.Ionescu-Bujor - Elemente de transformari geometrice, vol. I-IV Bibl Matematice a R.S.R., Ed. Tehnica, Bucureti, 1958. s 2. N. Mihileanu - Utilizarea numerelor complexe n geometrie, Bibl. Soc. S a atice a R.S.R., Ed. Tehnic, Bucureti, 1968. a s 3. L. Nicolaescu, V. Boskoff - Probleme practice de geometrie, Seria "C matematic i fizic", Ed. Tehnic, Bucureti, 1990. as a a s 4. D. Smaranda, N. Soare - Transformari geometrice, Bibl. profesorului d atica, Ed. Acad. R.S.R., 1988. a 5. C. Tigeru - Asupra unei clase de transformari geometrice, Matematica suceveana, nr. 8/1990, 1-7.

Ordinul elementelor grupului GLn (Z)


Adrian REISNER1
n RecMat 2/2005, Gabriel Dospinescu a propus problema

L94. Fie A Mn (C) o matrice cu elemente ntregi, inversabila si a mulimea Ak | k N este finita. Sa se demonstreze ca aceasta mulime a t t 2 t 3n elemente. Ramne rezultatul adevarat daca suprimam condiia ca matricei sa fie ntregi?

Soluia autorului a fost publicat n numrul 2/2006 al revistei. Pro t a a continuare o abordare oarecum diferit a problemei, care permite o mai bu a asupra laticei subgrupurilor lui GLn (Z).

t Notm cu GLn (Z) mulimea matricelor inversabile cu elemente ntre a care inversa are tot elemente ntregi. Evident ca M GLn (Z) daca i n s M Mn (Z) i det M = 1. Daca G este un subgrup finit al grupului G s p 3 este prim, considerm aplicaia : G GLn (Z), care asociaz un a t a M G acea matrice care are ca elemente redusele modulo p ale elemente

Propoziie. Aplicaia este un monomorfism de grupuri. t t Demonstraie. Se observa uor ca este bine definita, n sensul ca t s matrice inversabila n Mn (Zp ), precum i faptul ca este morfism de gru s arta c este injectiv demonstrnd c nucleul su Ker este {In }. Fie M a a a a a deci M In (mod p); exist atunci o matrice N Mn (Z) astfel nct M = a Grupul G fiind finit, rezulta ca M este de ordin finit m 1: M m = In . D egalitate deducem ca polinomul P = (1 + pX)m 1 este polinom anula matricea N . Dac 0 , 1 , . . . , m1 sunt rdcinile de ordin m ale unitii, a a a at 1 lui P vor fi j = s 1 , j = 0, m 1 i acestea sunt numere distinc p j c N este diagonalizabil. n plus, cum p 3, avem c |j | < 1, pr a a a
k

a lim N k = On . Matricea N fiind cu elemente ntregi, exist k suficien

pentru care N k = On , deci N este nilpotenta. O matrice diagonalizabila i s este nul i deducem c M = In + pOn = In , ceea ce ncheie demonstraia as a t

Consecina 1. Ordinul oricarui subgrup finit al grupului GLn (Z) es t n2 de 3 , iar GLn (Z) conine un numar finit de subgrupuri finite neizomor t Demonstraie. Cardinalul unui subgrup G este majorat, conform t 2 precedente, de cardinalul lui GLn (Z3 ), care este 3n (deoarece aplicaia t jectiva). A doua armaie rezulta imediat, deoarece GLn (Z3 ) are un num t subgrupuri neizomorfe, ind el nsui grup finit. s

Consecina 2. Ordinul oricarui element al grupului GLn (Z) este s t 2 sau majorat de 3n .
1

Cercettor, Centrul de Calcul E.N.S.T., Paris a

Demonstraie. Concluzia rezult din faptul c ordinul oricrui eleme t a a a grupului divide ordinul grupului.

Din Consecina 2 urmeaza imediat prima parte a problemei L94. t Proprietatea nu mai are loc daca A nu are elemente ntregi. De ex subgrup G GL2 (R) care este de torsiune (x G, q N cu xq = I2 fie infinit; acest fapt aduce o mbuntire soluiei problemei L94 dat n n a at t a unde se considera un contraexemplu bazat pe o matrice cu elemente comple Fie G subgrupul lui GL2 (R) format din matricele de forma cos sin R () = , {2r | r Q} . sin cos p Daca = , cu p, q Z, q 1, evident ca q cos 2p sin 2p q = I2 , [R ()] = sin 2p cos 2p deci G este grup de torsiune. Pe de alta parte, G este de ordin infinit, grupul (Q/Z, +); izomorfismul asociaz matricei R (), cu = 2r, clasa a prii fracionare a lui r (verificrile se fac uor). at t a s

Puncte coliniare

Ora de matematic la o clas cu prol sportiv. Un elev de cl. a IX a a rezolvat la tabla o problema simpla de coliniaritate, dar . . . Ce sunt punctele coliniare? intervine profesorul, decis sa-l ajute. !?!?? Bine! S-o lum altfel . . . Tu eti fotbalist. Ce nseamn coechiper? a s a Din aceeai echipa, de ce rdei de mine d-le profesor? s t Atunci, ce-ar putea sa nsemne coliniar? Din echipa advers!. . . a

Vrei s v ghicesc numrul ales? t a a a

Alegei un numar de doua cifre pe care-l dorii. nmulii prima cifra cu t t tt 3, dublai rezultatul, adaugai a doua cifra i spunei-mi rezultatul. Va vo t t t s loc numrul ales. a (Explicaii gasii la pagina 28.) t t

Variaiuni pe tema dreptei lui Euler t i cercului celor nou puncte s a


Temistocle BRSAN 1

Dou dintre cele mai cunoscute "vedete" ale geometrie triunghiului su a lui Euler i cercul celor noua puncte (cercul lui Euler ) (fig. 1). Vom adopt s uzuale. Fie 4ABC, dreapta lui Euler (determinata de H i O) i E c s s nou puncte (determinat de mijloacele laturilor A0 , B 0 , C 0 ). Sunt bine a urmtoarele proprieti ale acestei configuraii: a at t 1 G, O9 (centrul cercului E) sunt pe 4; 2 E conine picioarele nlimilor D, E, F i mijt at s loacele segmentelor [AH], [BH], [CH], adica punctele A00 , B 00 , C 00 ; 3 HO9 = OO9 , HG = 2 OG; s s 4 punctele A0 i A00 sunt diametrul opuse n E i A O = AA00 = A00 H;
0

A A E
C F G H O9 O

s 6 cercurile C (circumscris 4ABC) i E sunt omotetice prin omotetiile hH i hG s


1/2 1/2

5 AO k A0 A00 i AO = A0 A00 ; s

Fig. 1

[1]; daca C are raza R, atunci E are raza

Urmatorul rezultat, sugerat de configuraia de mai sus, reprezinta o ge t a a proprietii 4 (n loc de H considerm un punct oarecare): at

Propoziia 1. Fie 4ABC si P un punct oarecare. Aratai ca dreptel t t mijloacele A00 , B 00 si C 00 ale segmentelor ceviene [AP ], [BP ] si respecti mijloacele A0 , B 0 , C 0 ale laturilor opuse sunt concurente (fig. 2). Demonstraie. Patrulaterul B 0 C 0 B 00 C 00 este parat lelogram, cci B 0 C 0 i B 00 C 00 sunt paralele cu BC i egale a s s A BC cu s . Ca urmare, [B 0 B 00 ] i [C 0 C 00 ] se intersecteaza n 2 A P 0 aflat la jumtatea fiecruia. La fel se arat c A0 A00 a a a a C 0 00 0 00 0 i una dintre B B , C C se intersecteaz n P . a s P

Observaii. 1) Pentru a obine punctul P 0 este sut t cient s lum o singur cevian; dac aceasta este AP , a a a a a atunci P 0 este mijlocul segmentului [A0 A00 ]. 2) Distingem trei cercuri cu centrul n punctul P 0 i s de raze P 0 A0 , P 0 B 0 , P 0 C 0 ce apar n locul cercului celor nou puncte. a
1

C A

Fig. 2

Prof. dr., Catedra de matematic, Univ. Tehnic "Gh. Asachi", Iai a a s

Propoziia 2. Daca P este pe , atunci P 0 , obinut din P ca n Pr t t este de asemenea pe (fig. 3). Demonstraie. Fie A00 mijlocul segmentului eulet rian [AH] i A000 mijlocul segmentului cevian [AP ]; deci s A00 A000 k . Notm cu Q intersecia paralelei prin A0 la a t ceviana AP . Din acest fapt i din proprietaile A0 O k s t AA00 i A0 O = AA00 , rezulta ca 4A0 OQ, 4AA00 A000 sunt s congruente (ULU), deci A0 Q = AA000 = A000 P . Patrulaterul P A0 QA000 este paralelogram i P 0 este mijlocul s segmentului [P Q]. Cum P, Q , urmeaza ca P 0 . Revenind la Propoziia 1, vom impune punctului P t condiii suplimentare, care s-l apropie de H. t a

A A
A

P Q H P O B A

Fig. 3

Demonstraie. Am observat deja ca B 0 C 0 B 00 C 00 este paralelogram. t C , B 00 , C 00 sunt conciclice, atunci B 0 C 0 B 00 C 00 va fi dreptunghi, deci B 00 Dar B 00 C 0 k AP (n 4P AB). Deci AP BC, adic P AH. Implicaia a t se dovedete pe cale invers. s a
0

Propoziia 3. Fie P un punct n planul 4ABC. Punctele B 0 , C t (fig.2) sunt conciclice daca si numai daca P AH.

Propoziia 4. Fie P n planul 4ABC. Daca punctele B 0 , C 0 , B 00 , t (sau A00 ) (fig.2) sunt conciclice, atunci P coincide cu H.

Demonstraie. Cercul pe care se afl punctele are centrul n P 0 t a punctele A0 i A00 vor fi pe cerc, cci P 0 A0 = P 0 A00 i unul din ele, pri a s s este pe cerc. Conform Propoziiei 3, aplicat de trei ori, avem P AH, P t a P CH, adica P i H coincid. s

Demonstraie. Punctul A1 definit prin {A1 } = AP BC este picioru t duse din A. Ca urmare, A1 E i 4A1 A0 A00 dreptunghic n A1 este nscris s A0 A00 este un diametru n E. n consecina, mijlocul lui A0 A00 , care este es t centrul cercului E. Din faptul c B 0 , C 0 E, rezult c punctele diametra a a a vor fi pe acest cerc, adic B 00 , C 00 E. Se poate aplica Propoziia 4, conf a t P este punctul H.

Propoziia 5. Daca punctul P verifica condiia AP BC si puncte t t C 0 , A00 (fig.2) sunt conciclice, atunci P este ortocentrul H al 4ABC.

Propoziia 6. Daca punctul P verifica condiia AP BC si punctel t t C 00 , A0 (fig.2) sunt conciclice, atunci P este ortocentrul H. t Demonstraie. Simetricele punctelor A00 , B 00 , C 00 i A0 faa de P t s asemenea conciclice; aadar, A0 , B 0 , C 0 i A00 sunt conciclice. Ipotezele Pr s s fiind ndeplinite, rezulta ca P coincide cu H.

Observaii. 1) Propoziiile 4, 5 i 6 pot fi privite ca reciproce ale bine t t s afirmaii: daca H este ortocentrul unui triunghi, atunci mijloacele latu t mijloacele segmentelor euleriene si picioarele nalimilor sunt conciclice t Euler).

2) Remarcm c n enunul Propoziiei 4 este absent condiia AP B a a t t a t este doar aparent, caci n ipotezele acesteia, rezulta ca menionata condi t (conform Propoziiei 3). t

Revenind din nou la Propoziia 1, s examinm rezultatul acesteia din t a a de vedere. Notam cu transformarea geometrica (a planului 4ABC) punctul P n corespondena cu P 0 , punct construit ca n Propoziia 1 (a t t 0 0 Observaia ce-i urmeaza, punct 1) ); deci P P sau (P ) = P . t Vom indica cteva proprieti ale transformatei i o vom compara cu at s 1/2 1/2 hH i hG . s Iata cteva proprietai ale lui , care decurg direct din definiiile lui s t t stabilite mai sus: 2 (A) = mijlocul medianei [AA0 ] (analog, (B), (C)); 1 G G, H O9 ;

3 (4) = 4, adic dreapta lui Euler este transformat n ea ns a a as Propoziiei 2). t Propoziia 7. Sunt adevarate afirmaiile: t t a) (O) = U , unde U este mijlocul segmentului [O9 O]; Demonstraie. a) Afirmaia rezulta din faptul ca t t patrulaterul OA0 O9 L (fig.4) este paralelogram (O9 L ca linie mijlocie n 4AHO este paralel cu AH i egal cu a s a AH 0 a , iar OA , dup cum am amintit la nceput, are de 2 asemenea aceste dou prprieti). a at b) Argument similar: O9 A0 U K este paralelogram OA (A0 O9 i U K sunt paralele cu OA i egale cu s s ). 2

b) (O9 ) = V , unde V este mijlocul segmentului [O9 U ], adica O9 V =

A AK L V H O9 U O B A

Fig. 4 1/4 Propoziia 8. Are loc egalitatea = hG , adica este omotetia de c t 1 raport . 4 Demonstraie. Faptul c imaginea P 0 a punctului P se obine const t a t nti mijlocul A00 al segmentului [AP ] i apoi P 0 ca mijloc al segmentului s 1/2 1/2 1/2 1/2 1/2 1/2 scrie: = hA0 hA (ntr-adevar, hA0 hA (P ) = hA0 hA (P ) = h a P 0 ). Ca produs de dou omotetii, va fi tot o omotetie, cu centrul c 1 1 centrele omotetiilor factor i de raport ([1, p.81], [4, p.85]). Sa no s 2 2 0 0 centrul omotetiei , T AA . Poziia punctului T pe AA poate fi aflata c t unor formule prezente n locurile citate mai nainte; preferm s o determin a a Avem: 1/2 1/2 1/2 1/2 (T ) = T hA0 hA (T ) = T hA0 (S) = T , unde S = hA ( 0 1 1 s AT = A0 S i AS = AT AT = 2AS = 2(AA0 + A0 S) = 2 2

= 2(AA0 + 2A0 T ) = 2(AT + A0 T ) AT = 2A0 T T coincide cu ceea ce ncheie demonstraia. t

Speculnd faptul c omotetiile transform dreptele ce trec prin centru a a i cercurile n cercuri ([1], [4]), putem completa lista proprietailor lui d s t 4 dreptele suport ale medianelor 4ABC sunt invariante la ;

5 dreapta lui Nagel IN (N este punctul lui Nagel ) este invariant, cc a a avem (I) = I 0 , cu I 0 IN , 4 GI 0 = GI i (N ) = S, unde S este p s Spiecker centrul cercului nscris n triunghiul median 4A0 B 0 C 0 [2, pp.90

Observaie. Propoziia 2, creia i-am dat o demonstraie direct, dec t t a t a faptul ca 4 trece prin G i este omotetie. s

ncheiem cu o problem care poate prea dificil, dar care este uor d a a a s n contextul nostru.

Problem. Cercul determinat de mijloacele medianelor 4ABC are a R dreapta lui Euler n mijlocul segmentului [OO9 ] si raza . 4 Soluie. Observm c acest cerc este imaginea prin a cercului c t a a 4ABC (proprietatea 2) ) i apoi utilizm Propoziia 7, a). s a t Bibliografie

1. D. Brnzei, S. Ania, C. Cocea - Planul si spaiul euclidian, Bibliot t t sorului de matematica, Ed. Academiei, Bucureti, 1986. s 2. D. Brnzei, S. Ania, M. Chirciu - Geometrie. Clasa a IX-a, Colecia M t t ed. a III-a, Paralela 45, Piteti, 1998. s 3. T. Lalescu - Geometria triunghiului, Ed. Tineretului, Bucureti, 1958. s 4. D. Smaranda, N. Soare - Transformari geometrice, Biblioteca profe matematic, Ed. Academiei, Bucureti, 1988. a s

Vizitai pe Internet revista "Recreaii Matematice" la t t

http://www.recreatiimatematice.uv.ro

New Proof for an Old Inequality


Marian TETIVA1
The inequality

a2 + b2 + c2 8R2 + 4r2 is well-known to be the sharpest from all the inequalities of the form (see

a2 + b2 + c2 kR2 + hr2 ; also its a known fact (and it is easy to obtain) that it can be stated in the form (1 cos A)(1 cos B)(1 cos C) cos A cos B cos C. Many beautiful proofs for this remarkable inequality are available ([1], intention is to present in the sequel a new proof of this trigonometric fo inequality ("new" as far as we know it) and also to derive some related in Of course, the case of the right-angled or of the obtuse-angled triangl diately obtained, as long as the left-hand side is always positive and the side is less than (or equal to) zero in those cases. Therefore we may assu out mistaking, the triangle to be acute-angled, hence the cosines of its an positive; then we can also put the inequality as 1 1 1 1 1 1 1. cos A cos B cos C Let us denote x = cos A, y = cos B, z = cos C and the above parentheses w 1 1 1 1 1 , y= , z= u = 1, v = 1, w = 1 x = x y z u+1 v+1 Now observe that cos A = cos(B + C) = cos B cos C + sin B sin C p x + yz = (1 y 2 )(1 z 2 ),

whence squaring yields

x2 + y 2 + z 2 + 2xyz = 1, or, with the u, v, w - notations 1 1 2 1 + + + = 1. (u + 1)2 (v + 1)2 (w + 1)2 (u + 1)(v + 1)(w + 1) Thus we are left with the algebraic inequality uvw 1, for positive u, v, w the previous condition, or Y Y X (u + 1)2 (u + 1)2 (v + 1)2 + 2 (u + 1) = S = u + v + w, Q = uv + uw + vw, P = uvw; some simple (but boring) calculations transform the above condition in S 2 + 4S + 4 = P 2 + 2P Q + 4P S + 6P and we want to get from here the inequality P 1.
1

(the sum and the products are cyclic). Further we introduce the notation

Professor, National College "Gheorghe Roca Codreanu", Brlad s

Indeed, suppose that it is not so, hence P < 1; one gets S 2 + 4S + 4 < 1 + 2Q + 4S + 6 S 2 < 2Q + 3.

But Q S 2 /3 is a classic inequality which, together with the previous on S < 3 and this is not possible, because using () and the inequalities P Q S 2 /3, we infer for t = S/3. This last inequality may be rewritten as t6 + 6t5 + 12t4 + 6t3 9t2 12t 4 0,

and it shows that t 1 S 3. Thus the inequality S < 3 obtained supposition that P < 1 is contradictory, hence P 1 and the proof is don

(t 1)(t5 + 7t4 + 19t3 + 25t2 + 16t + 4) 0,

How to obtain new inequalities? For example, with the AM-GM ine 3 have S 3 3 P and Q 3 P 2 ; by P 1 these yield S 3 and Q 3, th Again by P 1 one gets S 2 + 4S + 4 25P (S + 2)2 25P S 5 P 2;

S 2 + 4S + 4 = P 2 + 2P Q + 4P S + 6P P 2 + 6P + 12P + 6P = P 2 +

now remember who S and P are and transform this into r X 1 Y 1 5 1 + 1, cos A cos A which we think that is not an easy to get inequality.

A slight modication of the above calculations leads us to another between S and P , namely Y 1 X 1 S P +2 1 1 + 2; cos A cos A a short computation transforms this into another (known, we believe) involving the cosines of an acute-angled triangle: X Y cos A 1 + 4 cos A.

respectively

We suggest to the reader to proceed for improving this last inequality get S 2P + 1 and S 3P , inequalities that can also be put in the form X X Y 2 cos A 2 + cos A cos B + 2 cos A, 3 X cos A 3 + 2 X cos A cos B.

Or, and we are very sure of it, the reader may nd his/hers own way to (beautiful and hard enough, we think) inequalities with this method. Bibliografie

1. D. Grinberg, M. Lascu, M. Pachiariu, M. Tetiva - Din nou despre t geometrice, G. M. 6/2006. 2. L. Panaitopol - O inegalitate geometrica, G. M. 4/1982.

Asupra calculrii unor limite de iruri a s


D. M. BATINETU-GIURGIU 1

Oricarui ir (an )n1 de numere reale strict pozitive i vom asocia iruril s s i n an ! n1 , unde a1 ! = a1 , 1 a1 ! = a1 , an+1 ! = an ! an+1 , n N . Pe s ir (xn )n1 de numere reale, vom nota xn = xn+1 xn , n N . s Vom considera mulimile de iruri: t s o n S R+ = (xn )n1 | xn R , n N , + o n D R+ = (xn )n1 S R+ | lim xn = x R , + n o n xn B R+ = (xn )n1 S R+ | lim = x R , + n n pentru care evideniem cteva proprieti. t at P1 . Oricare ar fi (xn )n1 D R , atunci (xn )n1 B R . + + Demonstraie. Fie (xn )n1 D R+ , deci lim xn = x R . C t + n xn xn+1 xn lim = lim = x. Deci D R+ B R+ . n n n (n + 1) n (1 Aceasta incluziune este stricta: irul (xn )n1 dat de xn = n + s 3 B R+ , dar nu i n D R+ . s xn+1 P2 . Oricare ar fi (xn )n1 B R , atunci lim = 1. + n xn xn+1 n n + 1 xn+1 = lim = Demonstraie. ntr-adevar, lim t n xn n n + 1 xn n P3 . Oricare ar fi (xn )n1 B R , atunci n xn ! n1 B R . + + Demonstraie. Conform criteriului Cauchy-d Alembert, avem: t ! r n xn ! nn xn+1 ! n xn ! = lim lim = lim = n+1 x ! n n n n nn n (n + 1) n xn+1 x n = R . = lim + n n + 1 n+1 e P4 . Oricare ar fi (xn )n1 B R , atunci n xn ! n1 D R . + + Demonstraie. n P3 am aratat ca, daca (xn )n1 B R+ , atunci lim t n ca urmare, p ! p n+1 n+1 xn+1 ! xn+1 ! n+1 n x e lim un = lim = lim = n n n n n n+1 n e x xn ! xn !

Profesor, Colegiul Na ional "Matei Basarab", Bucureti t s

lim (un ) = lim

n !n p n+1 xn+1 !

lim

n xn !

un 1 = 1, ln un xn+1 = lim n n+1

q.e.d.

Rezult atunci c a a p p n p xn ! un 1 n n n+1 xn+1 ! xn ! = lim ln (un lim xn ! = lim n n n n ln un

n+1 p n+1 xn+1 !

=x

Aplicaii (ale proprietaii P4 ). t t xn 1. Daca xn = n, n N , atunci (xn )n1 B R , cu lim =1 + n n n n! D R+ i s cu P4 , rezulta ca n1 p 1 n lim n+1 (n + 1)! n! = lim Ln = , n n e unde (Ln )ns>1 este sirul lui Lalescu (Problema 579, G.M., vol VI (1900-190 xn 2. Dac xn = 2n1, n N , atunci (xn )n1 B R cu lim a = + n n p D R+ i s ca n (2n 1)!! n1 2 p p lim n+1 (2n + 1)!! n (2n 1)!! = ; n e am obinut limita Problemei C:904, G.M. - 5/1989, p.187. t Propoziia 1. Fie (xn )n1 B R ; atunci (xn )n1 D R daca t + + n xn+1 = a R . daca exista lim + n xn xn Demonstraie. Conform ipotezei lim t = x R i deci (din P2 ) li +s n n n Avem relaia urmtoare: t a n " xn # xn xn xn xn n xn+1 = 1+ , n N . xn xn n xn+1 R , atunci trecnd la limit n aceast rela a a Dac a = lim a + n xn
1/x lim xn

n , obinem a = e n t , de unde lim xn = x ln a. n Reciproc, dac b = lim xn R , trecnd la limit n aceeai rela a a s + n n xn+1 c lim a = eb/x , adic a = eb/x . a n xn Propoziia 2. Daca sirurile (xn )n1 , (yn )n1 , (zn )n1 D R , a t + xn yn xy (un )n1 D R+ , unde un = , n N , si b) lim un = . n zn z

Demonstraie. Conform enunului, lim xn = x, lim yn = y i t t s n n cu x, y, z R+ . Cum, pentru orice n N , avem xn+1 yn+1 xn yn xn+1 yn+1 xn yn+1 xn yn+1 xn y un = = + zn+1 zn zn+1 zn+1 zn+1 zn xn yn+1 xn + (yn+1 zn yn zn+1 ) = = zn+1 zn zn+1 xn yn+1 xn + (yn+1 zn yn zn + yn zn yn zn+1 ) = = zn+1 zn zn+1 xn xn yn yn+1 xn + yn zn , = zn+1 zn+1 zn zn+1 y x xy trecnd la limit cu n rezult c lim un = x + y 2 z a a a n z z z demonstraia se ncheie. t

Aplicaii (ale propoziiei 2). t t (n + 1)2 n p 1. Daca xn = yn = n, zn = n!, n N , atunci lim n+1 n (n + 1)! (Problema C:890, G.M. - 4/1988). p 2. Dac xn = n n!, yn = n (2n 1)!!, zn = n, atunci a p ! p n+1 n (n + 1)! (2n + 1)!! n! (2n 1)!! 2 lim = 2, n n+1 n e iar dac xn = yn = n n!, zn = n, n N , atunci a p 2 p 2 n+1 n (2n + 1)!! (2n 1)!! 4 p lim = , n n+1 n e n (n + 1)!

etc. 3. Tot ca aplicaie a Propoziiei 2 se obin limitele problemelor u t t t C:2869, C:2878, C:2987, C:3010 din G.M., L.192 din Revista matematica stana i PP.2312, PP.2759, PP.3680, PP.5219, PP.5220, PP.5224, PP t s Octogon Mathematical Magazine. Bibliografie

1. D. M. Btineu - Siruri, Editura Albatros, Bucureti, 1979. a t s 2. D. M. Btineu-Giurgiu - O abordare a unor limite, G.M. - 5/2006, 22 a t 3. M. Tena - O alta soluie a problemei 579 (G.M.), Revista "Licriri" t a "Nicolae Balcescu", Craiova, 1978, 13-14.

O generalizare a teoremei lui Van Aubel


Silviu BOGA1

Considernd o cevian oarecare n locul bisectoarei unui a A triunghi se obine urmatoarea generalizare a teoremei bisect toarei : \ MB AB sin BAM (1) = MC AC sin CAM \ (se stabilete aplicnd teorema sinusurilor n 4ABM i B s s 4ACM pentru a exprima M B i M C). s Acest rezultat cunoscut a fost utilizat ca instrument de lucru n [1], [3 La rndul ei, relaia (1) poate fi generalizata la cazul n care ceviana t nlocuit cu o transversal B1 C1 M ce nu-i paralel cu AB, AC (cteva a a a acesteia sunt prezente n figurile de mai jos):
C1 A B1 C1 A B1 B1 A C1

c BC1 sin C1 MB ; = c MC CB1 sin B1 evident, pentru A B1 C1 relaia (2) devine (1). t Pentru a dovedi aceast relaie, procedm ca i n cazul relaiei (1): c a t a s t BC1 sinusurilor aplicat n 4C1 BM i 4B1 CM obinem M B = a s t sin C1 c sin M CB1 sin B1 , care, prin mparire, dau (2). t c sin M n unele aplicaii este utila o consecina directa a rezultatului dat de t t secina prin care sunt eliminate n fapt unghiurile. t C1 Fie trei drepte a, b, c concurente dou cte dou i a a s 0 transversalele t i t . Adoptm notaiile prezente pe figura s a t A alaturata i convenim ca (a; b) sa nsemne masura unuia s dintre unghiurile determinate de dreptele a i b. s

Propoziie. n condiiile specificate mai nainte, are t t B M loc formula 0 0 0 B M B C B1 M B CB1 M = 0 0 0 . (3) a M C BC1 M C B C1 Demonstraie. ntr-adevar, conform cu (2), aplicata triunghiurilor t 4AB 0 C 0 i transversalei B1 C1 , avem: s B 0 C1 sin (a; c) BC1 sin (a; c) M 0B0 MB = 0 = , . 0C 0 MC CB1 sin (b; c) M C B1 sin (b; c)
1

Profesor, Colegiul Na ional, Iai t s

De aici, rezult c a a

M B CB1 M 0 B 0 C 0 B1 sin (a; c) = 0 0 0 = , q.e.d. M C BC1 M C B C1 sin (b; c)

Observaii 1. Egalitatea (3) i demonstraia ei nu sufer modificri p t s t a a poziii ale dreptelor a, b, c (concurente dou cte dou) i transversalelor t a a s 2. Daca dreptele a, b, c sunt concurente n A, adica B1 i C1 coincid cu s (3) se scrie n forma M B AC M 0 B 0 AC 0 . = 0 0 M C AB M C AB 0 0 Menionam ca (3 ) poate fi stabilita uor cu ajutorul relaiei (1). t t s

Teorema 1. (generalizarea teoremei Van Aubel ). Fie 4ABC si pun (AC), C 0 (AB) cu BB 0 CC 0 = {O}. Daca prin O trece o dreapta care n A1 , (BA n C1 si (CA n B1 , atunci are loc relaia t B 0 A OA1 C 0 A OA1 + = 1. C 0 B OB1 B 0 C OC1 C Demonstraie. Aplicm (3) mai nti la dreptele CA, t a CB, CC 0 i transversalele AB i A1 B1 i apoi la dreptele s s s A B BA, BC, BB 0 i transversalele AC i A1 C1 : s s C B 0 A BC C 0 A CB OB1 CA1 OC1 BA1 , . = = O 0 B CA C OA1 CB1 B 0 C BA OA1 BC1 D A1 De aici obinem t B B 0 A OA1 CA CA1 BA BA1 C 0 A OA1 + = + . C 0 B OB1 B 0 C OC1 CB CB1 BC BC1 CD BD CA1 BA1 i Construim AD k B1 A1 i observam ca = = s s . Atun CB1 CA BC1 BA membrul drept al relaiei (5) revine la t CA CD BA BD + =1 CB CA BC BA i, n consecina, (4) este dovedita. s t

t t Observaii. 1. Dac A B1 C1 , se obine relaia Van Aubel. t a 2. Particulariznd O G (centrul de greutate al triunghiului), relaia t 1 1 1 + = (S. Boga, [1, p.43]). GB1 GC1 GA1 3. Particulariznd O I (centrul cercului nscris triunghiului), obine t AB BC AC + = . IB1 IC1 IA1 Bibliografie

1. S. Boga - O proprietate remarcabila de fascicul. Matematica n coala s 6/1989, 3-8. 2. D. Brnzei, S. Ania, M. Chirchiu - Geometrie. Clasa a IX-a (Colec t t 2000"), ed. a III-a, Editura Paralela 45, Piteti, 1998. s 3. C. Artenie, C. Constanda - Generalizarea problemei bisectoarei glisate Matematice, 1/2001, 32-33.

O propoziie echivalent cu conjectura lui Gol t a


Bogdan CIACOI 1

Punctul de plecare al acestei Note este celebra i aparent simpla co s lui Goldbach. Aceasta problema, care a impulsionat considerabil dezvolta numerelor, se afla pe lista problemelor de matematica nerezolvate nca. originea n corespondena dintre Christian Goldbach i Leonhard Euler t s i se formuleaz astfel: s a (CG) Orice numar natural par, mai mare ca 2, este suma a doua num O form mai tare a acesteia este: a 0 (CG ) Orice numar natural par, mai mare ca 6, este suma a doua num diferite. t Dupa cum se poate vedea n [2], (CG0 ) este echivalenta cu afirmaia u Orice numar natural mai mare ca 17 este suma a trei numere prime d t Sa notam cu P[n] mulimea tuturor perechilor de numere prime egal de 0 n i cu P[n] mulimea tuturor perechilor de numere prime diferite i egal s t s de n. Exemple: 0 P[2] = , P[2] = {(2, 2)}, 0 P[3] = {(3, 3)}, P[3] = , 0 P[4] = {(3, 5)}, P[4] = {(3, 5)}, 0 P[5] = {(3, 7)}, P[5] = {(3, 7)}, 0 P[6] = {(5, 7)}, P[6] = {(5, 7)}, 0 P[7] = {(3, 11), (7, 7)}, P[7] = {(3, 11)}, 0 P[8] = {(3, 13), (5, 11)}, P[8] = {(3, 13), (5, 11)}, 0 P[9] = {(5, 13), (7, 11)}, P[9] = {(5, 13), (7, 11)}, 0 P[10] = {(3, 17), (7, 13)}, P[9] = {(5, 13), (7, 11)}, 0 P[11] = {(3, 19), (5, 17), (11, 11)} etc. P[11] = {(3, 19), (5, 17)}

Observaii. 1. Postulatul lui Bertrand (pentru orice n 2 exist t a un numar prim ntre n i 2n) da o ansa existenei unei perechi de num s t s echidistante faa de n, dar nu o garanteaza. t 2. Daca n 3, atunci, din considerente de paritate, se constata ca num s 0 2 nu poate intra n perechile din P[n] i P[n] .

0 0 Evident, au loc: P[n] P[n] , n 2 si P[n] = P[n] daca n nu este numar

s 0 3. Din modul cum au fost introduse mulimile P[n] i P[n] rezulta ca a t (i) (p, q) P[n] , n 2 k {0, 1, . . . , n 2} astfel nct numerele p q = n + k sunt prime. 0 (ii) (p, q) P[n] , n 4 k {1, 2, . . . , n 3} astfel nct numerele p q = n + k sunt prime.
1

Elev, cl. a XII-a, Liceul Teoretic Ana Iptescu, Gherla a

4. Dac fixm n + k ca i numr prim ntre n i 2n, se poate arta a a s a s a calcul ca n k este de forma 6m 1, forma necesara a unui numar prim; aceasta nu garanteaza ca n k este prim. Formulm urmtoarele dou conjecturi: a a a (CCE) Pentru orice numar natural n 2 exista cel puin o pereche t prime egal departate de el. (CCE 0 ) Pentru orice numar natural n 4 exista cel puin o pereche t prime diferite si egal departate de el. Aceste pregatiri permit sa enunam urmatoarea t Propoziie. Sunt adevarate urmatoarele afirmaii: t t a) (CCE) este echivalenta cu (CG); b) (CCE 0 ) este echivalenta cu (CG0 ). Demonstraie. ntruct a) i b) se dovedesc n mod similar, vom t s numai punctul b). n n (CCE 0 ) (CG0 ) Fie n par i mai mare ca 6. Atunci N, s 4 s 2 2 n n ipotezei, k {1, 2, . . . , n 3} astfel nct k i s + k sunt prime 2 2 n n n n n = + = ( k) + ( + k), adic n este suma a dou numere prime a a 2 0 2 2 0 2 (CG ) (CCE ) Fie n N, n 4. Atunci 2n este mai mare ca 6 fiind presupus adevarat, exist dou numere prime distincte p i q a a a a a s 2n = p + q. Dac p < q (analog procedm n cazul q < p), din egalitatea p a a rezulta ca n p = q n N ; notam k = n p = q n i constatam s k {1, 2, . . . , n 3}. Cum (p, q) (n k, n + k) este o pereche de num echidistante de n, implicaia este dovedit. t a Este posibil ca aceast form echivalent cu conjectura lui Goldbach s a a a un anumit interes i n privina distribuiei numerelor prime. s t t Bibliografie

1. I. Creang, C. Cazacu, P. Minu, Gh. Opai, C. Reicher - Intr a t t teoria numerelor, Editura Didactica i Pedagogica, Bucureti, 1965. s s 2. P. Minu - Asupra ipotezei lui Goldbach, Recreaii Matematice, 4(2002), t t 3. W. Sierpi ski - Ce stim si ce nu stim despre numerele prime, Editura n Bucureti, 1966. s

Soluie (procedeul gsirii numrului ales - v. pag. 15). Scad 6 din n t a a mi l-ai comunicat i obin tocmai numrul ales. Justicarea este dat de t s t a a (a 5 + 3) 2 + b = ab + 6. Observaie. Dar daca numarul ales ar avea trei cifre? Am proceda t baza egalitaii t (a 5 + 3) 20 + (b 5 + 3) 2 + c = abc + 66.

Cum se poate obine o inegalitate t


Lucian TUTESCU 1

Prezentam n cele ce urmeaza modul n care s-a obinut inegalitatea t jos, pe care o consideram a fi interesanta i susceptibila de generalizari. s socotim c maniera de lucru urmat poate pentru elevi un model n ob a a rezultate proprii sugerate de probleme ce apar n paginile diverselor revist Daca x, y sunt numere reale cu acelai semn, atunci s 2 n 2n 2n n n x + y x y x + y 2 xy , n N.

Dei inegalitatea (1) este una algebric, povestea ei ncepe odat cu o s a a de geometrie propus la Concursul de matematica de la Kazanlk, Bulgar a 2003. \ \ Fie G centrul de greutate al 4ABC avnd m(AGB) = 2m(ACB). A 4 4 4 2 2 \ 60 . b) m(ACB) a) AB = AC + BC AC BC ; \ Soluie. a) Notm = m(ACB). Folosind teorema cosinusului, teor t a anei i relaii uzuale privind aria triunghiului, obinem: s t t cos 2 =

cu egalitate pentru a = b. Un prim pas n trecerea de la (2) la (1) este dat de:

care este tocmai (1) pentru n = 2. Relaia (2) se justific uor: t a s 2 2 (2) a4 + b4 a2 b2 a + b2 + a2 b2 2ab a2 + b2 2 2ab a2 + b2 4a2 b2 2ab (a b) 0, Dac x, y sunt numere reale nenule de acelai semn, atunci a s 2 2k k+1 k+1 k k x2 + y2 x2 y 2 x + y 2 xy , k N.
Profesor, Colegiul Na ional "Fra ii Buzeti", Craiova t t s

1 b) Este suficient sa aratam ca cos , adica c2 = a2 +b2 2ab cos 2 2 2 4 2 astfel spus c a + b ab . Apare deci firesc s dovedim inegalitatea a 2 2 a4 + b4 a2 b2 a + b2 ab ,

AG2 + BG2 AB 2 a2 + b2 5c2 ctg 2 = 2 AG BG 9 4SABG 2 cos2 1 a2 + b2 5c2 a2 + b2 5c2 1 = = 2 cos 2 sin cos 12SABC 3ab cos a2 + b2 + c2 a2 + b2 c2 2ab ab a2 + b2 5c2 = 2 = 2 2 c2 3ab ab a +b 3ab a + b2 2 2 a + b2 c4 = 3a2 b2 c4 = a4 + b4 a2 b2 .

Justificarea acestui rezultat se obine cu uurina din (2), prin indu t s t matica. Tot inducia ne ajuta sa facem trecerea la cazul general (1); pre t continuare calculele complete. Verificarea lui (1) pentru n = 0, 1, 2 este imediat. Presupunem c a adevrat pentru n = k i vom demonstra c este adevrat pentru n = k a a s a a a suficient sa aratam ca 2k+2 2k+2 x +y xk+1 y k+1 x2 + y 2 xy x2k + y 2k xk y k xk y k+2 x2k+1 y xy 2k+1 + xk+1 y k+1

de unde va urma uor concluzia inductiv. Inegalitatea () devine succesi s a

x2k+2 + y 2k+2 xk+1 y k+1 x2k+2 + x2 y 2k xk+2 y k + y 2 x2k + y 2k

relaie evident adevrat. Egaliate se obine cnd x = y. t a a t

xk+2 y k xk y k+2 + x2k+1 y + xy 2k+1 x2 y 2k y 2 x2k 2xk+1 y k+1 2 xy xk y k + x2 y k xk y k xk y 2 xk y k 0 xy xk y k xk y k + xy k1 yxk1 0 xy xk y k (x y) xk1 + y k1 0 xy (x y)2 xk1 + xk2 y + + y k1 xk1 + y k1 0,

S notm c (1) nu este, n general, adevrat atunci cnd x i y a a a a a s contrare. De exemplu, pentru n = 4, x = 1, y = 1 obinem contradicia t t

unde n N, iar x, y, z sunt numere reale de acelai semn. s Nu cunoatem pn la data publicrii acestui articol demonstraii pe s a a t (5) n cazul general, nsa am justificat cele doua inegalitai n cazul n = 2. P t calculele numai pentru (4): 2 x8 x6 y 2 + x4 y 4 x2 y 6 + y 8 x4 x3 y + x2 y 2 xy 3 + y 4

Dou generalizri naturale ale lui (1) ar putea fi inegalitile: a a at x4n x3n y n + x2n y 2n xn y 3n + y 4n x4 x3 y + x2 y 2 xy 3 + y 4 x2n + y 2n + z 2n xn y n y n z n z n xn x2 + y 2 + y 2 xy xz yz

x8 x6 y 2 + x4 y 4 x2 y 6 + y 8 x8 + x6 y 2 + x4 y 4 + x2 y 6 + y 8 2x7 y +

2x5 y 3 + 2x4 y 4 2x5 y 3 + 2x4 y 4 2x3 y 5 2x3 y 5 + 2x2 y 6 2xy 7

ceea ce este adevarat. Lsm deschis problema demonstrrii inegalitilor (4) i (5). aa a a at s

4x2 y 6 4x6 y 2 4x4 y 4 + 2x7 y + 4x5 y 3 + 4x3 y 5 + 2xy 7 0 h 2 i 2xy x3 y 3 + 2xy 2 x3 y 3 2x2 y x3 y 3 0 2xy x3 y 3 (x y) x2 + xy + y 2 2xy 0 2 2 2xy (x y) x2 xy + y 2 0,

Soluiile problemelor propuse n nr. 1 / 20 t


Clasele primare

P.104. Suma dintre predecesorul unui numar si succesorul numarulu lui este 29. Care este acest numar? (Clasa I ) Irina Luca, e Soluie. Suma dintre numarul cautat i succesorul lui este 29 = 14 + t s Deducem c numrul cutat este 14. a a a

P.105. Alaturat se afla roboelul "MATE". t a) Completai casetele goale; t b) Aflai suma numerelor pe care le tine n mini; t c) Aflai diferena numerelor scrise n talpile picioarelor. t t (Clasa I ) Andrei Stativ, elev, Iai a s Soluie. a) Pe umeri se afl numrul a astfel nct a 13 = 32, t a a de unde a = 32 + 13 = 45. b) n mna stnga are 45 3 = 42, iar n dreapta are numarul cu 1 dect 45, adic 32. Suma numerelor pe care le ine n mini este 42 + 32 = a t c) La baza bustului are numrul 67 + 2 = 69, iar pe talpa dreapt in a at 69 + 1 = 70. Diferena este 70 67 = 3. t
+13

P.106. Pentru desemnarea campioanei, echipele de hochei pe ghea t disputa un numar de partide pna ce una dintre ele ctiga de 4 ori. s numarul maxim de partide care se pot juca, stiind ca nu au fost rezultate de (Clasa a II-a) nv. Constana Cristea i Inst. Iulian Cri t s Soluie. Numarul maxim de partide care se pot juca este 7. ntr-ad t posibil sa se realizeze scorul 4 3.

P107. Un grup de turiti a consumat 17 prajituri si 31 ngheate. S s t turiti au consumat cte o ngheata si cte o prajitura, 5 turiti au consum s t s cte doua ngheate, iar 4 turiti nu au consumat nimic, iar restul cte t s produs (ngheata sau prajitura), sa se afle ci turiti sunt n grup. t t s (Clasa a II-a) Aliona Loghin, e Soluie. Numrul turitilor care au consumat cte o prjitur este 17 t a s a a Numrul turitilor care au consumat cte o ngheat este 31 7 5 5 a s t a cofetarie au intrat 7 + 10 + 5 + 14 + 4 = 40 turiti. s

P108. Prin mparirea a doua numere naturale rezulta ctul 3 si restu t ca mparitorul este un numar mai mic dect 10, aflai cele doua numere. t t (Clasa a III-a) nv. Rica Buct a Soluie. Punnd condiia r < , putem avea: 7 3 + 6 = 27, 8 3 t t 9 3 + 6 = 33.

P.109. Figura alaturata este formata din beioare. ts a) ndeparteaza un singur beior pentru a obine tot attea triunghiur ts t trate;

b) Muta doua beioare pentru a obine de doua ori mai multe drept ts tunghiuri dect patrate. (Clasa a III-a) Adina Voinescu, elev, Iai H a s Soluie. a) ndeprtnd unul din beioarele DE sau EG sau GI t a ts obinem dou triunghiuri i dou ptrate. t a s a a b) Mutam beioarele ID i JC astfel nct sa formam un drepts s tunghi de laime AB i lungime BF . n acest caz vom avea un patr s t dreptunghiuri. P.110. Ce ora indica primul ceas, stiind ca acesta respecta regula i celelalte trei?
12 9 6 3 9 6 12 3 9 6 12 3 9 6 12 3

(Clasa a III-a) Veronica Corbu, e Soluie. De la al doilea ceas la al treilea ceas avem o cretere de 1h t s la al treilea ceas la al patrulea ceas avem o cretere de 2h 400 . Regula s dublarea creterii. Cum 1h 200 este dublul lui 400 , primul ceas arata 6h 2 s P.111. Fie numarul N = abc + acb + bac + bca + cab + cba. a) Care este cea mai mica si cea mai mare valoare a lui N ? b) Cte valori diferite poate avea numarul N ? (Clasa a IV-a) Oxana Pascal, e Soluie. Avem N = 222 (a + b + c). t a) Cea mai mic valoare a ui N este 222 3 = 666. a Cea mai mare valoare a lui N este 222 27 = 5994. b) Valorile sumei a + b + c sunt de la 3 la 27. Numarul N poate lua 27 valori diferite. P.112. n urma desfasurarii unui joc didactic matematic, nvatator ca recompensa 44 baloane. Cte 4 baloane au primit un numar de part reprezinta a sasea parte din totalul lor, cte doua au primit a treia parte, participanilor au primit cte un balon. Aflai numarul participanilor la j t t t aritmetica!). (Clasa a IV-a) Alexandra Nistor, e Soluie. Presupunem ca mai adau t 4 elevi, n mod convenabil, astfel nct fiec 6
4 6 3 6

44 primeasc cte un singur balon din cele 44 a


se transform n a 4 1 4 , se transform n a 6 3 6

3 1 se transforma n Sa guram noua situaie. t 2 6 O esime din numarul elevilor participani la concurs primete 44 : 11 = s t s Numarul elevilor participani la concurs este 4 6 = 24. t P.113. Dan si-a pus timbrele n clasor, cte 10 pe unele pagini, cte pagini si au ramas de 4 ori mai multe pagini goale dect folosite. Dac cte 5 timbre pe fiecare pagina, toate paginile ar fi folosite. Cte pagini

clasorul, stiind ca nu depasete 60 (soluie aritmetica!)? t s (Clasa a IV-a) Petru Asa Soluie. Notam cu a numarul de pagini cu cte 10 timbre i cu t s de pagini cu cte 30 timbre. Din prima informaie deducem c numru t a a clasorului este 5 (a + b). Din a doua informaie rezult c numrul paginilor t a a a este 2a + 6b. Sa figuram aceasta situaie. t

Constatam ca b = 3a Distingem cazurile: a = 1 b = 3 5 (1 + 3) = 5 4 = 20 (pagini); a = 2 b = 3 2 b = 6 5 (2 + 6) = 5 8 = 40 (pagini); a = 3 b = 3 3 b = 9 5 (3 + 9) = 60 (pagini).

Clasa a V-a
a31

V.66. Sa se arate ca, oricare ar fi cifra nenula a, numarul x = 2131a 43 se divide cu 10. Otilia Neme, Ocna Mur s Soluie. Deoarece a13 = M 4 + 1, iar a31 = M 4 + 3, atunci U (21 t . U (32a13 ) = 2, iar U (43a31 ) = 7. Deducem c U (x) = 0, deci x . 10. a .

V.67. a) Sa se arate ca, scaznd din suma a 2006 numere pare consecu numerelor situate ntre acestea, nu se poate obine rezultatul 20062 . t b) Sa se afle 2006 numere pare consecutive astfel nct, scaznd din suma numerelor situate ntre ele, sa se obina 20052 . t Marian Pan t Soluie. a) ntre 2006 numere pare consecutive se afl 2005 numere t a suma numr impar; deducem c i diferena este tot impar. a as t a b) Fie a numrul cel mai mic; atunci a [a + (a + 2) + + (a + 4010)] [(a + 1) + (a + 3) + + (a + 4009)] =

a + 2005 = 20052 a = 2004 2005. V.68. Aratai ca nu exista n N pentru care An = 5n + 89 sa fie patr t Iulia Pleca, e s Soluie. Avem c A0 = 90, A1 = 94 nu sunt ptrate perfecte. Pent t a a

ultimele dou cifre ale lui 5n sunt 25, deci An se termin n 14. Deducem a a . dar An ./ prin urmare An nu poate fi patrat perfect. . 4, V.69. Sa se rezolve n N2 ecuaia 8n + 15m = 6 + 62 + + 62006 . t Alexandru Gabriel Tudorache, Soluie (Cezara Maria Enea, elev, Iai). Din 15m - numr im t a s a 62 + + 62006 - numr par, rezult c 8n este numr impar, deci n = a a a a 1 + 15n 6= M 3, iar 6 + 62 + + 62006 = M 3, contradicie, deci ecuaia nu t t V.70. Determinai a N pentru care numerele a, a + 2, a + 6, a + 1 t a + 20, a + 26, a + 30, a + 32, a + 36, a + 60 sunt simultan prime. Lucian Tuescu t

Soluie. Cum numerele 0, 2, 6, 12, 18, 20, 26, 30, 32, 36, 60 gener t resturile posibile la mparirea prin 11, acelai lucru se ntmpla i pent s t s numere date. Deducem ca cel puin unul din ele se divide cu 11 i, cum t s sunt prime, mcar unul este egal cu 11. Avem de studiat trei cazuri: a i) Dac a = 11, numere sunt 11, 13, 17, 23, 29, 31, 37, 41, 43, 47, 71, to a ii) Daca a + 2 = 11, atunci a = 9 nu este prim. iii) Daca a + 6 = 11, atunci a = 5, nsa a + 20 = 25 nu este prim. Valoarea cutat a lui a este 11. a a

A 12 m VI.66. Alaturat este desenata o gradina avnd forma 9 m unui poligon cu 7 laturi. n fiecare vrf se afla cte o poarta mobila astfel nct, n oricare doua vrfuri vecine, G porile sa nchida perfect latura pe care acestea o detert 11 m mina. Sa se afle lungimile porilor. t Roxana Cpn, elev, Iai a at a a s F 10 m Soluie. Fie x lungimea porii din A. Atunci porile t t t din B, C, D, E, F , G au respectiv lungimele:12x; 6(12 x) = x6; 7 13 x; 6 (13 x) = x 7, 10 (x 7) = 17 x; 11 (17 x) = x condiia de nchidere a porilor pe latura [AG] : x+(x 6) = 9 x = 7, 5. t t celor 7 pori vor fi 7, 5m; 4, 5m; 1, 5m; 5, 5m; 0, 5m; 9, 5m, respectiv 1, 5m t

Clasa a VI-a

VI.67. n patrulaterul ABCD construim AP BD, CQ BD, P si fie M mijlocullui (AC). Daca punctele M , P , Q sunt distincte doua demonstrai ca 4M P Q este isoscel. t Marius Fa Soluia 1 (a autorului). Fie {R} = P M CQ. t \ \ Avem c P AM RCM (alterne interne), AM P D a \ \ (opuse la vrf) i AM = M C, prin urmare CM R s P M R 4AM P 4CM R (U.L.U). Deducem c P M = M R i a s O Q atunci QM este median n 4P QR dreptunghic n Q, a S 1 A de unde rezulta ca QM = P R = P M , adica 4M P Q 2 este isoscel. Soluia 2 (Gabriel Popa). Fie S mijlocul lui [AQ], iar {O} = M SB t M S este linie mijlocie n 4ACQ, prin urmare M S k CQ. Rezult c OS a a cum S este mijlocul lui [AQ], avem ca OS este linie mijlocie n 4OAP . este mijlocul lui [P Q] i M O P Q, deci 4M P Q este isoscel. s Soluia 3 (Alexandra Cadar, eleva, Iai). Aplicam teorema media t s rema lui Pitagora: 2 P A2 + P C 2 AC 2 2 P A2 + P Q2 + QC 2 AC 2 2 PM = = 4 4 2 AQ2 + QC 2 AC 2 2 = MQ . = 4 \ VI.68. Fie punctele A, C, M cu m(AM C) 6= 90 si AC = 2 AM . S

\ \ ca M este mijlocul lui [AC] daca si numai daca 2m(ACM ) = m(M AC). Ioan Scleanu a a Soluie. Daca M este mijlocul lui [AC], atunci t M \ ) = m(M AC) = 0 , de unde 2m(ACM ) = \ \ m(ACM T \ \ m(M AC). Reciproc, s presupunem c 2m(ACM ) = a a \ m(M AC). Dac A, M , C nu ar fi coliniare, fie N mijlocul a \ N lui [AC] i [AT bisectoarea lui M AC, T M C. Atunci A s 4T AC este isoscel i T N este mediana, prin urmare T N AC. Pe de s \ \ 4M AT 4N AT (L.U.L.), deci m(AM C) = m(AN T ) = 90 , ceea ce ipotezei. Ramne ca punctele A, M , C sunt coliniare. n plus, nu putem a \ tul M pe prelungirile segmentului [AC], altfel m(M AC) = 180 6= 0 = 2 Deducem c M este mijlocul lui [AC], ceea ce ncheie rezolvarea. a VI.69. Sa se arate ca pentru orice alegere a semnelor n expresia 20062 , rezultatul nu se divide cu 2006. Mihail Bencze Soluie. Dac ar exista o alegere a semnelor pentru care rezultatul s t a a cu 2006, n mod necesar ar trebui ca 1 2 2006 i 2006 sa ai s paritate (o putere are aceeai paritate cu baza sa). Cum suma i diferena s s t paritate, ar rezulta c 1 + 2 + + 2006 este numr par, deci 1003 200 a a absurd. Astfel, rmne adevrat concluzia. a a a 3m + 1 n+2 VI.70. Determinai m, n Z pentru care a = t + Z. 2m + 1 3n + 5 Gheorghe Iu Soluie. Cele doua fracii din expresia lui a sunt ireductibile. ntr-ade t t d | 3m + 1, d | 2m + 1 d | 3 (2m + 1) 2 (3m + 1) d | 1

i analog pentru a doua. Cum (3m + 1, 2m + 1) = 1, (3n + 5, n + 2) = 1, s 3m + 1 (3n + 5) a = (3n + 5) + n + 2 Z 2m + 1 | 3n + 5; 2m + 1 n+2 Z 3n + 5 | 2m + 1, (2m + 1) a = 3m + 1 + (2m + 1) 3n + 5 prin urmare |3n + 5| = |2m + 1|. i) Daca 3n + 5 = 2m + 1, atunci 3n = 2 (m 2), deci m 2 = 3k 11k + 9 k Z, iar a = Z. Obinem c 6k + 5 | 11k + 9, de unde t a 6k + 5 6 (11k + 9) 11 (6k + 5), adic 6k + 5 | 1. Deducem c 6k + 5 { a a k = 1 i astfel (m, n) = (1, 2). s ii) Daca 3n+5 = 2m1, atunci 3n = 2 (m + 3), de unde m+3 = 3k 11k 10 Z. Ca mai sus, gsim soluiile (m, n) {(3, 0) a t k Z, iar a = 6k 5

Clasa a VII-a

VII.66. Sa se rezolve n R4 ecuaia t p p 30 x y + 901 + 25 y z + 626 + 20 z x + 401 + 9 t x + 78 =

Ioana Olan, e

Soluia 1. n condiiile de existena a radicalilor, folosind inegalitatea t t t avem: p p 30 x y + 901 + 25 y z + 626 + 20 z t + 401 + 9 t x + 7 p p p p = 900 (xy+901) + 625 (yz+626) + 400 (zt+401) + 81 (t 900+xy+901 625+yz+626 400+zt+401 81+tx+78 + + + 2 2 2 2 Cum se atinge egalitatea, n mod necesar vom avea c x y + 901, y z + a z t + 401 = 400 i t x + 78 = 81. Sistemul obinut este nedeterminat, c s t {(, + 1, + 2, + 3) | R}. Soluia 2. Egalitatea din ipotez se poate scrie sub forma t a 2 2 p p 30 x y + 901 + 25 y z + 626 + 2 2 + 20 z t + 401 + 9 t x + 78 = 0.

Fiecare termen al sumei trebuie s se anuleze etc. a

. . VII.67. Aflai a, b N daca a + b = 18 si 10a+1 9b + 71 . 81. t Andrei-Sorin Cozma, Soluie. Avem: t 10a+1 9b + 71 = 10a+1 1 9 (b + 1) + 81 = 99 {z . 9 9 (b + 1) + | .. } . . . . deci 10a+1 9b + 71 . 81 11 .{z. 10 b . 9. Restul mparirii lui 11 .{z. 10 t | . } | . }
a de 1 a de 1 a de 1 a de 1

= 9 11 {z . 1 b 1 + 81 = 9 11 .{z. 10 b + 81, | .. } | . }
a+1 cifre a de 1

a+1 cifre

VII.68. Fie 4ABC dreptunghic, cu ipotenuza de lungime a, catetele 2 x2 + y 2 a2 aria S. Daca x, y (0, ), sa se arate ca = daca si num S xy si c sunt direct sau invers proporionale cu x si y. t Veronica Pleu i Dan Pl a s s Soluie. Avem succesiv: t 2 x2 + y 2 2 b2 + c2 2 x2 + y 2 a2 = = xy b2 + c2 = bc x2 S xy bc xy bx (by cx) cy (by cx) = 0 (by cx) (bx cy) = 0 c c b b by cx = 0 sau bx cy = 0 = sau 1 = 1 . x y x y

. acelai cu restul mpririi lui | + 1 + {z + 1 + 0 prin 9, deci 11 .{z. 10b . 9 s at 1 | . } . } Cum a + b = 18, obinem soluiile (a, b) {(0, 18) ; (9, 9) ; (18, 0)}. t t

\ VII.69. Fie 4M N P cu m(N M P ) = 90 ; se considera punctele (N S), M (P T ), astfel nct N S = 3 M S, P T = 3 M T . Daca {Q} = atunci: a) QM = N P ; b) QN 2 + QP 2 = 5 N P 2 . Dorel Luc MS MT Soluie. a) Fie {R} = QM N P . Cum t = = Q MN MP 1 , din reciproca teoremei lui Thales rezult c T S k N P . a a 2 TS 1 Atunci 4M T S 4M P N , de unde = , iar 4QT S NP 2 QS QT TS 1 T 4QN P , prin urmare = = = . Astfel, N S QP QN NP 2 M i P T sunt mediane n 4QN P , deci M va fi centrul de greus tate al 4QN P . Rezult c R este mijlocul lui [N P ], iar a a QM = 2M R. Pe de alta parte, M R este mediana n 4M N P dreptunghic, deci N P = 2M R. Deducem c QM = N P . a N R b) Aplicm n mod repetat teorema lui Pitagora: a 2 2 QN 2 + QP 2 = (2T N ) + (2SP ) = 4 M T 2 + M N 2 + 4 M S 2 + M P 2 = 4 M N 2 + M P 2 + 4 M T 2 + M S 2 = 4N P 2 + 4T S 2 =

VII.70. Triunghiul alaturat este considerat fix. n cte mo duri putem aeza numerele 1, 2, 3, 4, 5, 6 n cerculee, astfel nct s t suma numerelor de pe fiecare latura a triunghiului sa fie aceeai? s Petru Asaftei, Iai s Soluie. Fie i, j, k cele trei numere din vrfuri; cum fiecare t vrf aparine la cte dou laturi, suma numerelor de pe fiecare latur va f t a a 1 i+j+k S = [(1 + 2 + + 6) + (i + j + k)] = 7 + . 3 3 . Acest numar trebuie sa fie natural, deci i + j + k . 3. Valoarea minima pent . este 1 + 2 + 3 = 6, iar cea maxim este 4 + 5 + 6 = 15, prin urmare i a {6, 9, 12, 15}. Deducem c a {i, j, k} {{1, 2, 3} ; {1, 2, 6} ; {1, 3, 5} ; {2, 3, 4} ; {1, 5, 6} ; {2, 4, 6} ; {3, 4, 5} Odat fixat mulimea {i, j, k}, cele trei numere pot fi permutate pe cele tre a a t din vrfuri n 6 moduri, iar apoi numerele din mijloacele laturilor, dac e a bine determinate. Spre exemplu, daca {i, j, k} = {1, 2, 3}, avem ca S = 9 completarile:
1 6 2 4 5 3 3 5 4 1 6 2 1 6 5 2 4 3 3 4 5 2 6 1 1 5 6 3 4 2 2 4 6 3 5

Se constata uor ca nu avem posibilitatea completarii triunghiului daca s {{1, 2, 6} ; {2, 3, 4} ; {1, 5, 6} , {3, 4, 5}}. Obinem astfel c numrul de t a a posibile este 4 6 = 24.

Clasa a VIII-a

VIII.66. Sa se demonstreze ca

1 1 n1 1 + + + 4 < . 24 + 22 + 1 34 + 32 + 1 n + n2 + 1 3n Carmen Daniela Tama 1 1 , k N , deoare Soluie. Are loc inegalitatea 4 t 2+1 k +k 3k 2 2 revine la k2 1 0. Egalitatea se atinge numai pentru k = 1. Atunci 1 1 1 + 4 + + 4 < 4 + 22 + 1 2+1 2 3 +3 n + n2 + 1 1 1 1 1 1 1 1 + + + < + + + < 3 22 3 32 3 n2 3 12 23 (n 1) n 1 1 1 1 1 1 1 1 n = 1 + + + = 1 = 3 2 2 3 n1 n 3 n Not. Elevul Florian Plia, Petroani, demonstreaz inegalitatea pr a a t s a matematica. VIII.67. Fie 0 < a < b < c < d < e si propoziiile: t 2ac b+d 2ce p1 : b = ; p2 : c = ; p3 : c = ae; p4 : d = a+c 2 c+ Sa se arate ca daca oricare trei dintre propoziii sunt adevarate, atunci este t si cea de-a patra. Claudiu- tefan P S Soluie. Consideram nti ca p1 , p4 sunt adevarate i sa aratam ca t s Avem: b+d 2ac 2ce a e c= 2c = + 1= + 2 a+c c+e a+c c+e a (c + e) + e (a + c) = (a + c) (c + e) c2 = ae c = ae a c a Demonstrm acum c p1 p2 p3 p4 . Din p3 obinem c = , deci a a t a c e a+ s Atunci, folosind p2 i apoi p1 , avem: 2ac a 2c2 2ae 2ce d = 2c b = 2c = 2c 1 = = = a+c a+c a+c a+c c+ deci p4 este adevarata. La fel se arata ca p2 p3 p4 p1 . n VIII.68. Fie An = 2006 + 2005n 1992n 1991n , n N. Sa se de . . 28. pentru care An . Ionel Nech . n n. Soluie. Folosind faptul c a b . a b, n N, deducem c 2006n t a a . . . . 2005n 1991n . 7, deci An . 7. Sa vedem cnd An . 4. Avem ca A0 = 0 . 4, A . . . . . n n n. s Daca n 2, atunci 2006 = (M 4 + 2) = M 4 + 2 . 4 i este evident c . . . (M 4)n . 4. Astfel, An . 4 2005n 1991n . 4 (M 4 + 1)n (M 4 . . . n . . 1 (1)n = 0 n par. n concluzie, A . 28 pe . M 4 + 1 (1) . .
n

{1} {2k | k N}. VIII.69. Fie x1 , x2 , x3 R astfel nct x2 + x2 + x2 = 1. Determina 1 2 3 mica si cea mai mare valoare a expresiei

E (x1 , x2 , x3 ) = x1 + x2 + x3 + x1 x2 + x2 x3 + x3 x1 .

Ion Vian i Lucian Tuescu s s t (x1 + x2 + x3 + 1)2 2 Soluie. Expresia se scrie sub forma E = t 2 Emin = 1, iar aceasta valoare se atinge, de exemplu, pentru x1 = x2 = 0 r x1 + x2 + x3 x2 + x2 + x2 1 2 3 Din inegalitatea M A M P , avem c a 3 3 2 3 1/ 3 + 1 2 1 egalitate cnd x1 = x2 = x3 = . Astfel, Emax = 2 3 1 iar aceasta valoare se atinge pentru x1 = x2 = x3 = . 3 VIII.70. Se considera cubul ABCDA0 B 0 C 0 D0 si fie M , N mijloacele [AB], respectiv [BC], iar {S} = AN CD, {T } = DM BC. Sa se a unghiului format de D0 N si ST . Gabriel P Soluie. t Fie {P } = AN DM . Din D C \ 4AM D 4BN A (C.C) rezulta ca ADP P AM . A \ B \ \ \ nsa m(P AM ) + m(DAP ) = 90 ,deci m(ADP ) + \ \ m(DAP ) = 90 i atunci m(AP D) = 90 , adic a s D C AN DM . Deducem c SP i T C sunt nlimi a s at P n 4DST , prin urmare N va fi ortocentrul acesN B tui triunghi, iar DN este tot nlime: DN T S. A M at T Din DD0 (ABC) urmeaz c DD0 T S, deci a a 0 0 T S (DD N ), de unde D N ST .

Clasa a IX-a

IX.66. Pentru x, y R, fie a = y + xy x, b = x2 + x xy. a) Daca a, b (, 0), sa se compare numerele x si y. b) Aratai ca exista o innitate de numere raionale x, y pentru care a, b t t Ionu Onofrei, elev t Soluie. a) Din a + b < 0, obinem c x2 + y < 0, deci y < x2 t t a presupunem prin absurd c x 0; din b = x(x + 1 y) < 0 rezult c x + a a a deci x < 1 + y. Dar 1 + y < 0 i am ajuns la o contradicie. Cum s t deducem ca xy > 0, prin urmare y x = a xy < 0. n concluzie, y < x. 1 n+1 b) De exemplu, putem considera x = , y = , cu n N n n 2 2 n + n + 1 i b = sunt ambele negative. s a= n2 n IX.67. Fie n 2 si a1 , a2 , . . . , an 0 astfel nct (a2 a3 an ) + (a1 a3 a4 an ) + + (a1 a2 an1 ) = 1.
2 2 2

Sa se arate ca a1 + a2 + + an + a2 a3 an + a1 a3 an + + a1 a2 Adrian Zahariuc, ele n Q Q Soluie. n dezvoltarea produsului t aj apar toi termen ai + t
i=1 j6=i

Egalitatea se atinge cnd n 1 numere sunt egale cu 1, iar cel rmas este a IX.68. n 4ABC se considera cevienele [AM ], [BN ], [CP ] concur TA TB TC Sa se arate ca = = daca si numai daca T este centrul de TM TN TP 4ABC. Ovidiu Pop, Sa TC TB [ N T P (opuse \ i BT C = Soluie. Cum t s A TN TP [ la vrf), rezult c 4BT C 4N T P , de unde T BC a a \ T N P , prin urmare P N k BC. Analog se arata ca PB AP P ; atunci = 1 k, M N k AB, P M k AC. Fie k = T AB AB AP BM AN BM BP AN = = k, = = k, = = 1 k. AC AB BC AC BC AB 1 Din ultimele doua relaii obinem k = , deci P este mij- B t t M 2 locul lui [AB], apoi M , N sunt mijloace pentru [BC], respectiv [AC]. n T este centrul de greutate al 4ABC. Reciproca este imediata. IX.69. Fie 4ABC nedreptunghic. Paralela prin B la AC si simetr AC n raport cu BC se intersecteaza n A1 ; analog se obin punctele B1 si t AA1 , BB1 , CC1 sunt concurente, sa se arate ca 4ABC este echilateral. Temistocle B A Soluie. Pentru nceput, fie 4ABC ascuitunghic. t t Se vede uor c 4BCA1 este isoscel, deci A1 se afl pe s a a mediatoarea lui [BC]. Fie {X} = AA1 BC; evident ca X (BC). Din 4BXA1 4CXA obinem t BA1 XB a XB A = = B XC CA XC 2b cos C X BA0 (caci n 4A0 BA1 dreptunghic avem cos C = = BA1 A1 YC a ). Analog caracterizam Y (CA) i Z (AB) prin s = 2BA1 YA ZA c = . Concurena dreptelor AA1 , BB1 , CC1 conduce la t ZB 2a cos B a b c 1 = 1 cos A cos B cos C = . 2b cos C 2c cos A 2a cos B 8 1 n binecunoscuta inegalitate cos A cos B cos C , egalitatea se atin 8 A = B = C, deci 4ABC este echilateral. Dac 4ABC ar fi obtuzunghic, dou dintre punctele X, Y , Z sunt a a laturi, iar al treilea pe prelungirea laturii corespunztoare. n acest caz a AA1 , BB1 , CC1 nu vor putea fi concurente.

n n Q Q P Q 2 , i = 1, n, deci aj aj = 1. Din inegalit ai + i=1 j6=i j6=i j6=i s i=1 n n Q P Q Q aj aj , de unde concluzia ai + ai + ilor avem c n n a

aj

i=1

j6=i

i=1

j6=i

IX.70. Sa se arate ca tg 15 + tg 25 + tg 35 + tg 85 > 4. D. M. Btineu-Giurgiu, B a t Not. Cu ajutorul unui tabel care da valorile tangentei, se constata c a 11, 43, valoare deja mai mare dect 4.

Clasa a X-a

X.66. Notam cu D mulimea punctelor P (x, y) din planul xOy situa t riorul sau pe laturile 4ABC. Fie a, b R, a2 + b2 6= 0; definim funcia f t f (P ) = ax + by + c. Sa se arate ca pentru orice P D, avem

Adrian Cordune aa a as Soluie. Fie M (1 , 1 ), N (1 , 1 ); artm c valorile minim i max t f (P ), cnd P parcurge [M N ], se ating n capetele segmentului. Daca P exista t [0, 1] astfel nct xP = 1 + t (2 1 ), yP = 1 + t ( 2 1 ).

min {f (A) , f (B) , f (C)} f (P ) max {f (A) , f (B) , f (C)} .

Daca f (N ) f (M ) > 0, atunci f (M ) f (P ) f (N ), P [M f (N ) f (M ) < 0, atunci f (N ) f (P ) f (M ), P [M N ]. n sf f (N ) f (M ) = 0, atunci f (P ) = f (M ), P [M N ]. Revenim la problema iniial. Fr a micora generalitatea, presu t a aa s f (A) f (B) f (C). Fie P D oarecare, iar {Q} = AP BC. f (B) f (Q) f (C), avem f (A) f (P ) f (C) i demonstraia este nc s t X.67. Fie Q 2 = a + b 2 | a, b Q . Sa se determine funciile c t f : Q 2 (0, ) pentru care f (x + y) = f (x) f (y), x, y Q 2 . Dan- s Stefan Marinescu i Viorel Cornea, H Soluie. Fie g : Q 2 R, g (x) = lg f (x); atunci g (x + y) = g ( t s a x, y Q 2 . Fie k = g (1); cu metoda obinuit de rezolvare a funcionale de tip Cauchy, se arat c g (x) = kx, x Q i g x 2 = t a a s x Q. Pentru n N arbitrar, avem: h i h i h i h i 2 g n 2 + n 2 n 2 < n 2 + 1 g n 2 ng h i h i n2 kh i k n 2 n g 2 n 2 +1 k n 2 g 2 n n g 2 k 2 , > 0. Astfel, rezulta ca g 2 = k 2, de un deci g m + n 2 = g (m) + g n 2 = km + kn 2 = k m + n 2 , adica g (x) = kx, x Q 2 , unde k (0, +). Notnd a = 10k 1, o x f (x) = 10g(x) = 10k = ax , cu a 1. Pentru orice a [1, +), funcia t verifica ipoteza problemei, deci funciile cautate sunt cele exponeniale cu t t X.68. Pe cercul trigonometric se considera punctele A, B, C de afix 2 2 unde = cos + i sin . Fie M (z) un punct al cercului situat pe arcul 3 3 z2 + z + 1 conine A. Sa se arate ca z 2 + z + 1 = t . z Marian Tetiv

f (P ) = (a1 +b 1 +c) + t [a (2 1 ) + b ( 2 1 )] = f (M ) + t [f (N )

Soluie. Fie z = cos + i sin , cu [2/3, 4/3]. Avem: t 2 z + z + 12 = (cos 2 + cos + 1)2 + (sin 2 + sin )2 = = cos2 2 + sin2 2 + cos2 + sin2 + 1+ +2 (cos 2 cos + sin 2 sin ) + 2 cos 2 + 2 cos = = 3 + 2 cos + 2 2 cos2 1 + 2 cos = 4 cos2 + 4 cos + 1 = (2 cos Cum [2/3, 4/3], atunci 2 cos + 1 0, deci z 2 + z + 1 = |2 co z2 + z + 1 2 cos 1. Evident ca = 2 cos + 1, de unde concluzia. z X.69. Daca a, b, c > 1, sa se demonstreze inegalitatea (a + b + c 3 3 3 3 3 3 a loga b+ loga c + b logb a+ logb c + c logc a+ logc b 3 Titu Zvonaru, C Soluie. Deoarece a, b, c > 1, atunci loga b > 0, loga c > 0 etc. Folosi t tatea mediilor, obinem: t p p p p 3 loga b + 3 loga c = 3 1 1 loga b + 3 1 1 loga c 1 1 1 (1 + 1 + loga b) + (1 + 1 + loga c) = loga a2 b + loga a2 c = loga 3 3 3 Cum a > 1, b > 1, c > 1, avem: 3 3 3 3 3 3 a loga b+ loga c + b logb a+ logb c + c logc a+ logc b
3
logb b 3 abc logc c 3 abc

aloga a abc + b +c = (a + b + c)2 3 3 3 3 = a abc + b abc + c abc = (a + b + c) abc . 3 Egalitatea se obine pentru a = b = c. t

X.70. Fie patratul ABCD. Sa se determine mulimea t 2 2 = P Int ABCD | P A , 2P B P D, P C sunt laturile unui triun Soluie. Notm P A = x, P B = y, P C = z, t a P D = t; atunci 4 = P Int ABCD | x2 + 2yt > z 2 , x2 + z 2 > 2yt, z 2 + 2yt > x2 .
B x P A x
x
2

Ctlin Cali a a

y Aplicnd teorema medianei n 4P AC i 4P BD, s t cum AC = BD, obinem c x2 +z 2 = y 2 +t2 . Astt a fel, x2 +z 2 > 2yt y 2 + t2 > 2yt (y t)2 > 0 C D=B y 6= t P AC. Celelalte dou inegalitti care intervin n definiia / a a t satisfcute de ctre orice punct P Int ABCD, prin urmare = Int ABC a a S demonstrm c x2 + 2yt > x2 , P Int ABCD, pentru prima a a a procedndu-se analog. Aplicm ptratului o rotaie de unghi a a t n juru 2

conserv astfel lungimile segmentelor x, y, z, t,deci 4AP P este dreptung a 0 0 cu P P = x 2. Din 4P P B, obinem: t 2 2 y + t > x 2 y + t + 2yt > 2x2 x2 + z 2 + 2yt > 2x2 z 2 + 2yt

Clasa a XI-a

10n+1 Din aceeai condiie de minimalitate deducem k = 1, deci xn = s t xn 10 = Atunci lim . n 10n 81 XI.67. Fie sirul (xn )n1 definit prin x1 , 0 , xn+1 = 2xn tg x 4 Sa se studieze existena limitelor lim xn si lim n xn . t n n Dan Popescu Soluie. Daca presupunem ca xn , 0 , atunci xn+1 = f (xn ) t 4 h i , 0 R, f (t) = 2t tg t este funcie strict cresctoare. Obinem c t a t a 4 s + 1 < xn+1 = f (xn ) < f (0) = 0 i deoarece + 1, 0 , 0 , 2 2 4 irul (xn )n>1 este corect definit, strict cresctor i mrginit. Pentru x a s a s avem c tg x = x x = 0; folosind acest fapt, prin trecere la limit n a a recurena rezult c lim xn = 0. t a a
n

XI.66. Fie xn , n N , cel mai mic numar natural cu proprietatea ca e an an1 . . . a1 a0 (10) cu toate cifrele nenule, astfel nct M = (n+1) n+1 an an xn . 9xn . Sa se calculeze lim n 10n Valeriu Braoveanu s 1 1 n+1 a a Soluie. xn = M (n + 1) t n ni a0 [M (an + an1 + 9 9 t cu egalitate pentru an = an1 = an1 = = a0 = k. Condiia de mi impusa lui xn conduce la i 1 kh xn = kk {z . k k + k + + k = .. } 11 . . . 1 (n + 1) . {z } | 9 | 9
n+1 cifre n+1 termeni

Apoi, (xn ) n = e ln

ln(xn ) n

2 tg xn xn 2 tg xn xn , n 1. Cum lim = 1, cu criteriul lui Stolz n xn xn 0 deduce c lim n xn = e = 1. a


n

, n 1, iar ln (xn+1 ) ln (xn ) = l

XI.68. Fie f : I R, I R interval, o funcie de doua ori der t f 00 (x) f 0 (x), x I. Sa se arate ca f (x) f (a) (exa 1) f 0 (a), Pentru f (x) = ex , (, 0] [1, +), sa se deduca inegalitatea lui B Dumitru Mihalach Soluie. Consideram funcia : I R, g (x) = f (x) f (a) (exa t t g f 0 (x) f 0 (a) 0 Avem ca g(a) = 0, iar g (x) = ex a . Funcia h : I R, h (x t ex e f 00 (x) ex f 0 (x) ex f 00 x f are derivata pozitiv pe I: h0 (x) = a = 2x e ex

x I, deci h este monoton cresctoare. Deducem c g 0 (x) 0 pentru a a g 0 (x) 0 pentru x < a, adica a este punct de minim pentru g. Cum obinem ca g (x) 0, x I, de unde inegalitatea dorita. t Pentru f (x) = ex , condiia f 00 (x) f 0 (x) revine la 2 , deci este t pentru (, 0][1, ). Deci are loc concluzia problemei, ce se mai scrie ex e(1)x e(1)a (1 ) ea . Trecem x ln x, a ln a i obinem s t 1 1 x x a (1 ) a x > (1 ) a + xa1 ,

pentru orice x, a > 0 i orice (, 0] [1, ). Pentru a = 1, s x 1 + (x 1), (, 0] [1, ), ceea ce constituie o uoar m s a a inegalitii lui Bernoulli. at XI.69. Fie A, B M3 (R) astfel nct det (AX + B) 0, X M3 arate ca exista C M3 (R) pentru care A = BC. Gheorghe Iu Soluie. Pentru X = O3 , obinem c det B 0. Dac det B 6= t t a a exist B 1 ; fie D = B 1 A. Avem c det B (DX + I3 ) 0, X M3 ( a a a det B > 0, deducem c det (DX + I3 ) 0, X M3 (R). Dac D = a a u x 0 0 ax + 1 0 0 1 0, cu x R pentru X = 0 0 0 obinem c DX +I3 = x t a 0 0 0 ux 0 1 Atunci det (DX I3 ) = ax + 1 0, x R, de unde a = 0. Consider + 0 0 0 X = x 0 0, x R, gasim ca bx + 1 0, x R, deci b = 0 etc. R 0 0 0 D = O3 , adica B 1 A = O3 , de unde A = O3 . Luand C = O3 , are loc conc Daca detB = 0, sa presupunem prin absurd ca exista X0 cu det(AX Cum det (AX + (AX0 + B)) = det (A (X + X0 ) + B) 0, X M3 (R cele demonstrate anterior rezult c A = O3 i atunci det B > 0, fals. R a a s det (AX + B) = 0, X M3 (R). Pe de alta parte, din det B = 0 rez 1 inversabile astfel nct B = P SQ, unde S este una dintre matricele 0 0 1 0 0 0 0 0 0 0 0 sau 0 0 0. Obinem t 0 0 0 0 0 0 det (AX + P SQ) = 0, X M3 (R) det P P 1 AX + S Q = 0, X M3 (R)

det (A1 X + S) = 0, X M3 (R) , 1 0 0 0 n cazul n care S = 0 1 0, lund X = 0 0 0 0 0

unde A1 = P 1 A. 0 1 0 0, apoi X = 0 0

a b c a b c gasim ca A1 = x y z = SM , unde M = x y z . Atunci A 0 0 0 0 0 0 P SM = SQQ1 M= BC, C = Q1 M . La fel se procedeaza P cu 1 0 0 0 0 0 0 0 0 sau S = 0 0 0. 0 0 0 0 0 0 XI.70. Fie a, b, c laturile unui triunghi ale carui unghiuri au masurile A, B, C si care are raza cercului nscris r. Sa se arate ca distana t de 3 M (A, B, C) la planul P : ax + by + cz + r = 0 este mai mare dect 3 Sorin Pupan s a Soluie. S artm nti c n 4ABC are loc inegalitatea t a aa a A B C A 2 2 S > (p a) + (p b) + . () 2 2 2 Pentru demonstraie, observm c exist trei cercuri tangente t a a a doua cte doua, cu centrele n vrfurile triunghiului; razele lor B sunt p a, p b, respectiv p c. Evalund aria zonei haurate, s obinem imediat (). t Cum A + B + C = i S = rp, relaia () se scrie echivalent s t 2 p 2 (aA + bB + cC + r) p + a2 A + b2 B + c2 C < 0. Daca 0, expresia de gradul II n p din stnga pastreaza semn constant, este +. Rezult c > 0, deci a a p aA + bB + cC + r > (a2 A + b2 B + c2 C) s 2 2 2 aA + bB + cC + r > a A + b B + c C 3, a2 + b2 + c2 3 a2 + b2 + c2 la ultima inegalitate folosind inegalitatea lui Cebev. s

Clasa a XII-a

XII.66. Fie a, b R cu 0 a < b si fie f : [a, b] R o funcie d t derivabila pe [a, b], cu f 00 continua. Daca Z b a2 b2 f (x) dx = f 0 (a) f 0 (b) + bf (b) af (a) , 2 2 a sa se arate ca exista (a, b) astfel nct f 00 () = 0. Mihai Ha Soluie. Avem: t Z b b Z b f (x) dx = xf (x) xf 0 (x) dx = a a a " # b Z b x2 x2 0 = bf (b) af (a) f (x) f 00 (x) dx = 2 a a 2 Z a2 0 b2 0 1 b 2 00 = bf (b) af (a) + f (a) f (b) + x f (x) 2 2 2 a

Rb Din ipotez rezult c a x2 f 00 (x) dx = 0. Conform teoremei de medie, a a a Rb (a, b) astfel ncat a x2 f 00 (x) dx = 2 f 00 () (b a) i cum 6= 0, atunci f 0 s XII.67. Fie f : [0, 1] R o funcie cu proprietatea ca exista L 0 a t |f (x) f (y)| L |x y|, x, y [0, 1]. Sa se arate ca pentru orice prim lui f si pentru orice x1 , x2 , . . . xn [0, 1], n N, n 2, are loc X x1 + x2 + + xn F (x1 ) + F (x2 ) + + F (xn ) F L ( 2n2 n n

1i<jn

obinem concluzia. t XII.68. Fie f, g : R R, f (x) = eP (x) , g (x) = eQ(x) , unde P , Q sunt de grad m 1, avnd coeficieniiRdominani respectiv b, a, b (0, ). t a, Rn t n a) Sa se calculeze lim f (n) 0 g (x) dx g (n) 0 f (x) dx . n b) Sa se studieze buna definire a sirurilor (an )n1 si (bn )n1 , unde 1 Rn an 1 Rn f (x) dx, g (bn ) = g (x) dx si apoi sa se calculeze lim . 0 0 n bn n n Marius Ap Soluie. a) Evident ca f i g sunt strict crescatoare. n ipoteza ne t s a b, folosind regula lui lHospital pentru nedeterminari de tipul , ob t R Rx P (x)Q(x) x Q(t) f (x) 0 g (t) dt e e dt 0 Rx Rx = lim = lim P (t) dt x g (x) f (t) dt x e 0 0 Rx Rx [P 0 (x) Q0 (x)] 0 eQ(t) dt P 0 (x) Q0 (x) xm1 0 = 1 + lim = 1 + lim x x xm1 eQ(x) eQ( Q(x) e = = 1 + m (a b) lim x (1 m) xm eQ(x) + x1m Q0 (x) eQ(x) 1 a = 1 + m (a b) = . mb b

Dan- tefan Marinescu, H S Soluie. Cum f este lipschitzian, ea este continu i n consecina adm t a as t tive. Fie x, y [0, 1], x > y; atunci L (x y) f (x) f (y) L (x funcia g : [0, 1] R, g (x) = f (x)Lx este descresctoare, iar funcia h : t a t h (x) = f (x) + Lx este cresctoare. Deducem c funcia G : [0, 1] R a a t L L F (x) x2 este concav, iar H : [0, 1] R, H (x) = F (x) + x2 est a 2 2 Aplicnd inegalitatea lui Jensen, obinem: t 2 F (x1 ) + + F (xn ) L x2 + x1 + + xn L x1 + + xn 1 F n 2 n n 2 2 F (x1 ) + + F (xn ) L x2 + x1 + + xn L x1 + + xn 1 F + + n 2 n n 2 Folosind aceste relaii i identitatea evident t s a 2 2 2 X 1 x1 + + xn x1 + + xn = 2 (xi xj )2 , n n n
1i<jn

b) Deoarece f i g sunt strict monotone, din teorema de medie re s 1 Rx f (t) dt, tena i unicitatea funciilor i denite prin f ( (x)) = t s t s 0 x 1 Rx 1 Rx 1 Rx g (t) dt, deci (x) = f 1 f (t) dt , (x) = g 1 g (t) 0 0 x x x 0 teorema de derivare a inversei, funciile i sunt derivabile, iar lim t s
x

lim (x) = +. Avem:

de unde lim

1 Rx f (x) 2 0 f (t) dt (x) 0 (x) g 0 ( (x)) x lim = = lim 0 = lim 0 x x (x) x (x) x f ( (x)) 1 Rx g (x) 2 0 g (t) dt x x 1 Rx 1 f (t) d 1 0 Q ( (x)) g ( (x)) f (x) f (x) x 0 = lim 0 x P ( (x)) f ( (x)) g (x) 1 1 Rx 1 g (t) dt g (x) x 0 Rx m1 f (x) 0 g (t) dt (x) ba (x) b Rx = lim lim = a x (x) a b x (x) g (x) 0 f (t) dt
x

(x) = 1. Am folosit pe parcurs faptul c a (x)

1 Rx Rx f (t) dt f (t) dt f (x) x 0 lim = lim 0 = lim = lim x x xf (x) x f (x)+xP 0 (x) f (x) x 1+x f (x)

XII.69. Fie f R [X] polinom reciproc de grad 4n + 2, n N , avnd distincte, complexe si nereale. Sa se arate ca f are cel puin o radacina d t a Ctlin Tigeru a a Soluie. Cum f are coeficieni reali i este reciproc, daca a C \ R est t t s 1 1 a lui f cu modulul diferit de 1, atunci a, , a, sunt patru rdcini di a a a a module diferite de 1, ale lui f . Cum polinomul are 4n + 2 rdcini, con a a imediat. a XII.70. Fie G un grup de ordin n 4 cu proprietatea ca exista m N, 1 m1 astfel nct G conine exact Cn1 subgrupuri de ordin m. Aratai ca G es t t Marius Trnuce a a Soluie. Considerm submulimile lui G care conin elementul neutru t a t t m1 i nca m1 elemente din G\{e}. Numarul acestor submulimi este Cn1 , s t aceste submulimi sunt subgrupuri ale lui G. Daca m > 2, alegem x, y t x 6= y. Cum n3 m2 1, putem alege m2 elemente din G\{e, x, y}, a1 , a2 , . . . , am2 . Notm H1 = {e, x, a1 , . . . , am2 }, H2 = {e, y, a1 , . . . , am a H1 i H2 sunt subgrupuri ale lui G, xa1 H1 (deoarece H1 subgrup), xa1 s x = a1 H2 ), xa1 6= x (n caz contrar a1 = e) i xa1 6= ai , i = 1, m s 1 x = ai a1 H2 ). Contradicia la care am ajuns arat c m = 2 i a2 = t a a s 1 deci G este grup abelian.

Soluiile problemelor pentru pregtirea concur t a din nr. 1 / 2006


A. Nivel gimnazial

G96. Fie a = x12m + x12n , unde m, n N . Sa se arate ca numa divizibil cu 13, daca si numai daca x este divizibil cu 13. Artur Bluc, aa a Soluie. Daca 13 divide x atunci 13 divide x12m + x12n = a. Reciproc t ca 13 divide a. Presupunem ca n m, adica m n p, p Daca 13 = N. x, din faptul c 13 divide a = x12m + x12n = x12n x12p + 1 rezult c a a a numrul natural x12p + 1. Conform teoremei lui Fermat, orice x N a divide cu 13 are proprietatea ca x12 da restul 1 la mparirea cu 13 i atun s t da la mparirea cu 13 restul 2, ceea ce contrazice faptul ca 13 divide a. t divide cu 13.

G97. Determinai a, b {0, 1, 2, . . . , 9}, a 6= 0, astfel nct num t abb {z . }, n 2, sa fie patrat perfect. .. b | Gheorghe Iu n ori Soluie. Dac b = 0, atunci A este ptrat perfect dac i numai dac a t a a as a iar n este par. Fie acum b 6= 0; cum b este ultima cifra a unui patrat per c b {1, 4, 5, 6, 9}. Pentru b {1, 9}, rezult c A = M4 + 3, care nu e a a a perfect. Dac b = 5, atunci A = . . . 55 = M25 + 5, care iari nu este ptr a as a Cnd b = 6, obinem situaia nefavorabil A = M4 + 2. t t a Ramne de studiat cazul b = 4. Avem:

Pentru n 3, numarul t2 este impar, deci t = 2k + 1; deducem ca a 10 11 {z . 1 0 = 4k (k + 1). Nu putem avea n 4, pentru ca ar rezulta ca 11 .. } |
n1 de 1

A = a44 . . . 4 = a 10n + 4 11 . . . 1 = 4t2 , cu t2 = a 10n2 25 + 11

i reinem A = 122 = 144. s t

Dup calcule, reinem A = 382 = 1444. n sfrit, pentru n = 2, avem a t s 2 2 2 2 A = a44 12 , 18 , 22 , 28 G98. Sa se determine m, n N astfel nct

multiplu de 4. Dac n = 3, atunci a 2 2 2 2 2 2 2 2 2 2 2 2 2 A = a444 32 , 38 , 42 , 48 , 52 , 58 , 62 , 68 , 72 , 78 , 82 , 88 , 92 ,

m n+1 N . + n m2 Gabriel Dospinescu, stude Soluie. Dac m, n verific enunul, atunci exist p N astfel nct t a a t a m3 + n2 + n = pm2 n.

Este evident c n divide 3 , adic exist k N astfel nct m3 = kn a m a a 3 deducem ca n +1+k = p k 2 n2 (k + n + 1)3 = p3 n2 k 2 . Este evident ca i deoarece n k este cub perfect rezult c n i k sunt cuburi. Fie u, v s a a s u3 = k, v 3 = n. Atunci m = uv i u3 + u3 + 1 = pu2 v 2 . Dac u = s a u = v = 1, deci m = n = 1. Fie acum u > v. Este evident ca (u,

v4 . De aici rezulta ca v 4. D 4 2 se divide cu u , deci nu este liber de ptrate. Rezult c v = 2 i u = 3 a a a s simetriei rezulta i soluia v = 3, u = 2. Corespunzator acestor doua solu t t s (m, n) {(1; 1) , (6; 8) , (6; 27)}. divide v 3 + 1 i atunci v 3 + 1 u2 s

u2 v 2 pu2 v 2 = u3 + v 3 + 1 < 2u3 + 1. Deci, u

v2 . Pe de alta part 2

G99. Fie m, n doua numere naturale nenule astfel nct m divide n numerele naturale ntre 1 si n se aeaza la ntmplare pe un cerc. Se calcul s oricarui grup de m numere vecine. Sa se demonstreze ca printre aceste su doua pentru care diferena dintre ele este strict mai mare dect m 1. t Titu Zvonaru, C Soluie. Deoarece n 1 se divide cu m rezult c n = km + 1. D t a a n P (n 1) (n + 2) inm numrul 1, suma numerelor rmase este a a a i= . Co 2 i=2 sumele din cele k grupuri de cte m numere vecine, rezulta ca media ace (n + 2) m (n + 2) km i deci cel puin una din sumele calculate, s = s t a 2k 2 (n + 2) m aceasta este a, este mai mare sau egala ca media lor, adica a. 2 n1 P (n 1) n i= . Ra acum numarul n obinem suma numerelor ramase t 2 i=1 nm mai sus rezulta existena unei sume, sa zicem b, astfel nct t b. 2 (n 2) m nm ab = m, deci exist dou sume (de exemplu a i b) p a a s 2 2 diferena lor este strict mai mare dect m 1. t

G100. n cte moduri putem colora cu 5 culori un patrat 3 3, astf fiecare patrat 2 2 sa existe patru culori diferite? Gabriel P Soluie. Patratul 2 2 din stnga poate fi colorat n 5 4 3 2 t moduri; fie A, B, C, D culorile folosite ntr-un anumit caz, ca n figur. a Dac n ptrelul (1, 3) (adic linia 1, coloana 3, i.e. ptrelul din a a at a a at dreapta sus) folosim culoarea C, n patraelul (2, 3) poate fi folosita una t dintre culorile A sau E. Daca n patraelul (3, 1) folosim culoarea din (2, t att (3, 2) ct i (3, 3) pot fi colorate n cte dou moduri; dac nu, n (3 s a a folosi 2 culori, n (3, 2) culoarea este xat, iar n (3, 3) putem folosi 2 culori a 1 2 (1 2 2 + 2 1 2) = 24 modalitai de colorare ale conturului exterior t Daca n (1, 3) nu folosim culoarea C, putem folosi una dintre culorile (neavnd importana, s zicem c aceasta este A). t a a dac n (2, 3) folosim C, n (3, 1) putem utiliza A, B sau E, pentru (3 a 2 culori i la fel pentru (3, 3). s daca n (2, 3) nu folosim C, atunci acest patrael va fi colorat cu E t (3, 1) utilizm E, avem cte dou modaliti de colorare pentru (3, 2) i (3 a a at s n (3, 1) folosim A sau B, culoarea din (3, 2) este fixat, iar pentru (3, 3) a a posibilitai. t

Obinem astfel 2 (1 3 2 2 + 1 1 2 2 + 1 2 1 2) = 23 5 modalit t lorare ale conturului exterior. n concluzie, patratul 33 se poate colora n 5432 24 + 23 5 = 26 3 moduri. G101. Sa se demonstreze inegalitatea ! 1 1 1 16 4 1+ 2 + 2 + 2 (1 + bc) (1 + ca) (1 a (1 + bc) b (1 + ca) c (1 + ab) a, b, c (0, ) astfel nct abc = 1. 1

Gabriel Mranu i Andrei Ned s s 1 a i analoagele. = s Soluie. Avem: t = 1 2 a (1 + bc)2 (1 + a)2 a 1+ a nea, 1 1 1 = 1 1 1 = (1 + a) (1 + b) ( (1 + bc) (1 + ca) (1 + ab) 1+ a 1+ b 1+ c i atunci inegalitatea enunului devine s t ! b c 16 a 1+ 4 2 + 2 + 2 (1 + a) (1 + b) (1 + c) (1 + a) (1 + b) (1 + c) 1 1 Adunnd n ambii termeni ai relaiei (1) pe 4 t 2 + 2 + (1 + a) (1 + b) obinem inegalitatea echivalenta t 1 1 1 1 16 1 4 + + 1+ +4 2 + 2 + 1+a 1+b 1+c (1+a) (1+b) (1+c) (1+a) (1+b) 2 2 2 Dac vom considera x = a ,y= ,z= inegalitatea (2) dev 1+a 1+b 1+c 2 (x + y + z) 1 + 2xyz + x2 + y 2 + z 2 . Condiia abc = 1 devine: t 2 2 2 1 1 1 = 1 8+2 (xy + yz + zx)4 (x + y + z) = x y z Conform cu (4), inegalitatea (3) devine: 2 (x + y + z) 1 + 8 + 2 (xy + yz + zx) 4 (x + y + z) + x2 + y 2 + z 2

(x + y + z)2 6 (x + y + z) + 9 0 (x + y + z 3) ceea ce este evident. G102. Sa se determine valoarea maxima a parametrului m R astf + p a2 + b2 b2 + c2 c2 + a2 + + m 3 (a2 + b2 + c2 ), a, b, c R a b c Dorel Bian i I. V. Maftei, B at s pP P b2 + c2 Soluie. Vom demonstra c t a a2 , egalitatea 2 3 a P b2 + c2 P bc 2 P 2 2 pentru a = b = c. Este evident c a 2 = b c . a a abc

demonstra c a 1 P 2 2 pP 2 a , atunci problema este rezolvat. Avem: a b c 3 abc qX X 2 X X b2 c2 3abc a2 a2 b2 c2 3a2 b2 c2 X X X X X b4 c4 + 2a2 b2 c2 a2 3a2 b2 c2 a2 b4 c4 a2 b2 c2 P P 2 xy, x, y, z R nlocuim x = b2 c2 , Dac n inegalitatea a x 2 2 z = a b , obinem inegalitatea (1). Rspunsul la cerina problemei este d t a t G103. Pentru a, b, c (0, 1) cu a + b + c = 2, sa se arate ca abc 8 (1 a) (1 b) (1 c) .

Alexandru Negrescu, elev, Soluia 1 (a autorului). Este evident ca a < 1 a + b + c < 1 + b t 1+b+c 1 < b+c i deci a < 1 < b+c, adic a < b+c i analoagele. Rezu s a s c pot fi lungimile laturilor unui triunghi ABC de semiperimetru p = 1. In enunului este echivalenta cu: t 4RSp 8S 2 Rp 2S Rp 2rp R 2r

abc 8 (p a) (p b) (p c) abcp 8p (p a) (p b) (p c)

ceea ce este evident. Soluia 2 (Florian Plia, elev, Petroani). Cum 1 a > 0, 1 t a t ps 1 c > 0, se aplica inegalitatea mediilor 2 (1 a) (1 b) 2 a b = doua analoage. Prin nmulire membru cu membru, rezulta concluzia. t Soluia 3 (Marius Tiba, elev, Iai). Notnd x = 1 a, y = 1 b, t s obinem x + y + z = 1. nlocuind, inegalitatea devine (1 x) (1 y) (1 t apoi

1 xyz + xy + xz + yz x y z 8xyz 1 + xy + xz + yz 1 1 1 1 1 1 1 + + 9 (x + y + z) + + 9, x y z x y z inegalitate binecunoscuta. \ G104. Triunghiul ABC are m(BAC) = 120 . Fie O (BC) astfel \ este bisectoarea unghiului BAC. Pe [AO se ia punctul D astfel nct \ bisectoarea interioara a unghiului ABD. Sa se arate ca AD + BD = AB AB + AC 4 AO. Petru Rduc a Soluie. Fie E AD astfel nct BE este paralel cu t a [ \ s [ \ AC. Deoarece EAC AEB i m(EAC) m(EAB) = 60 rezulta ca triunghiul ABE este echilateral, adica AB = A O BE = AE. Prelungim [AE] cu EF = AC. Deoarece (BE) (AB), D \ \ (EF ) (AC) i m(BEF ) = m(BAC) = 120 rezult c s a a C tringhiurile BEF i BAC sunt congruente de unde obinem s t \ s \ \ \ \ s ca BCA BF E i EBF ABC, deci CBD F BE i \

\ BF = BC. Din paralelismul dreptelor AC i BE rezult ACB CB s a \ \ ABC i atunci CBD EBF , deci m(CBD) + m \ s \ \ \ construcie CBD t \ \ \ \ m(EF B) + m(DBE) = m(BCA) = m(BF E). Prin urmare triunghiul isoscel cu BD = DF . Rezulta ca AD + BD = AD + DF = AF = AE AB + AC. \ Observm c m(CBF ) = 60 i din BF = BC rezult c triunghiul a a s a a \ i deci m(BCF ) = 60 . Triunghiurile AOB i ACF fiind asemen s echilateral s AO AB AB AC ca = , deci AO = , de unde AC AF AB + AC AB + AC 2AB AC , 2 AO = AB + AC 2

adica 4AO AB + AC. n (1) avem egalitate ntre media aritmetica armonic dac i numai dac AB = AC. a as a

G105. Se considera trapezul ABCD cu bazele AB, CD ( AB > CD intersecia diagonalelor trapezului. Se duce linia mijlocie M N a trapezului t P Q prin O la bazele trapezului ( M, P (AB), N, Q (BC) ). Sa se de ca trapezele ABM N si P QCD au diagonalele respectiv paralele. Claudiu- tefan P S Soluia 1 (a autorului). Fie E intert D C secia dreptelor DQ i AB. Din asemnri t s a a CD CQ CQ Q imediate obinem c t a = , = P BE QB QB O M N CD CD CD DO OD , = . Rezulta ca = , OB OB AB BE AB de unde deducem c BE = AB. a Deoarece M este mijlocul segmentului B [AD] iar B este mijlocul segmentului [AE], A rezult c [M B] este linie mijlocie n triunghiul ADE, atunci M B este p a a DQ. Analog demonstrm c AN este paralel cu P C. a a a

Soluia 2 (Marius Tiba, elev, Iai). Fie x = AB, y = CD; atunci P Q t s MN =

x+y DP 2DP 2DP 2DO 2y , iar = = = = . 2 MA AD DP + P A DO + OB x+y DC CQ PQ 2y DP i, cum congruena unghi = = = = s t ca MA MN NB AB x+y imediata, patrulaterele DCQP i M N BA vor asemenea. Urmeaza c s \ s \ \ AM B i P CQ AN B, deci DQ k M B i P C k AN . s

B. Nivel liceal

L96. Fie cercurile C1 , C2 , C astfel nct C1 si C2 sunt tangente exteri fiecare dintre ele este tangent interior lui C n B, respectiv C. Tangen interioara cercurilor C1 si C2 taie cercul C n A si A1 . Dreapta AB taie n K, iar dreapta AC taie cercul C2 n L. Din punctul M de pe cercu tangentele M T1 si M T2 la cercurile C1 , respectiv C2 ( T1 C1 , T2 C

A1 M AM KL si |M T1 M T2 | = A1 D AD Neculai Roman, Mirce Soluie. Domnul Titu Zvonaru, Comneti, remarc faptul c ace t a s a a lem este ndeaproape nrudit cu problema L76, publicat de acelai aut a a a s Mat 1/2005. Consideram totui utila elevilor includerea unei soluii detal s t n rezolvarea acestei probleme vom folosi teorema lui Casey, pe care o n continuare (fara demonstraie). t Teorema lui Casey. Daca cercurile C1 , C2 , C3 , C4 sunt tangente terior sau toate exterior) la cercul C, ordinea punctelor de tangena in t numerotarea acestor cercuri, atunci are loc relaia t M BAC, aratai ca M T1 + M T2 = t d12 d34 + d23 d41 = d13 d24 ,

unde dij este lungimea tangentei comune exterioare a cercurilor Ci si numete distana tangeniala a celor doua cercuri). Rezultatul ramne s t t daca cercurile Ci (toate sau o parte dintre ele) degenereaza n puncte sau d C devine o dreapta. Sa revenim acum la problema considerata. Fie {E} = BC C1 , {F } = BC C2 , d tangenta comun a cercurilor C, a \ [ [ = m(BA1 C)/2. Deoarece m(EKB) = m(EBT ) = m astfel nct m(T BC) \ m(BAC), rezulta ca AC k KE. Analog, obinem AB k LF . t Cum AD este axa radical a cercurilor C1 i C2 , rezult c AK AB = a a a s \ ceea ce nseamna ca patrulaterul BCLK este inscriptibil, deci ABC \ LKE (pentru c AC k KE) i ABC \ aici, innd seama de relaiile ALK t t a s \ \ rezulta ca LKE KBE, deci LK este tangenta cercului C1 . Analog de \ c LK este tangent i cercului C2 . a as Aplicm teorema lui Casey pentru cercurile M , C1 (O1 ), A1 , C2 (O2 ) a degenerate) tangente interior cercului C i obinem s t dMO1 dO2 A1 + dMO2 dA1 O1 = dM A1 dO1 O2 M T1 A1 D + M T2 A1 D = A1 M KL M T1 + M T2 =

A1 M K A1 D Pentru a demonstra a doua relaie, aplic teorema lui Casey cercurilor M , t a i C1 (O1 ): s de unde concluzia. L97. Sa se demonstreze ca n orice triunghi are loc inegalitatea 1 1 1 + 2 2 + 2 2 (m + m m ) ma b c a mb (mc + ma mb ) m2 (ma + mb mc ) c

dMA dO1 O2 + dM O1 dAO2 = dMO2 dAO1 M A KL + M T1 AD = M

I. V. Maftei i Dorel Bian, B s at Not. Concomitent cu publicarea n revista noastra, problema a a numarul 25449 n G.M. nr. 12/2005. Soluia sa poate fi gasita n G.M t P S2 9 n fapt, inegalitatea se reduce la . 4 a2 (p a)2

Vlad Emanuel, elev, Sibiu, noteaz x = p a, y = p b, z = a inegalitatea precedenta (demonstrata trigonometric de catre autorii problem P (x + y + z) xyz 9 . Daca m = xy, n = xz, p = yz, avem de la 4 x2 (y + z)2 P mn + mp + np 9 , m, n, p > 0, care este chiar inegalitatea 114 din 2 4 (m + n) Old and New Inequalities, autori T. Andreescu, G. Dospinescu, V. Crtoaje, aprut n 2004 la Editura GIL. a a

L98. Se considera un triunghi oarecare ABC. Demonstrai ca t 27 r 3 4 4 4 ; 1) sin A + sin B + sin C 2 R 3 r 4 ra rb rc 2) cos4 A + cos4 B + cos4 C 5 5 5 , 8 R r r r unde R este raza cercului circumscris, r este raza cercului nscris, iar ra , razele cercurilor exnscrise. Oleg Faynshteyn, Leipzig, G Soluie. 1) Avnd n vedere inegalitatea x2 + y 2 + z 2 xy + yz + zx (x t a b z abc S i relaiile s t = = = 2R, R = , r = i p 3 3r, ded s sin A sin B sin C 4S p

sin4 A + sin4 B + sin4 C (sin A sin B)2 + (sin B sin C)2 + (sin C sin A abc a + b + c = sin A sin B sin C (sin A + sin B + sin C) = 8R3 2R p r 2 27 r 3 rp2 S . 3 3r = = = 2R2 R 2R3 2R3 2 R t 2) Utiliznd iarai inegalitatea x2 + y 2 + z 2 xy + yz + zx, obinem s

cos4 A + cos4 B + cos4 C cos A cos B cos C (cos A + cos B + cos C

Vom exprima acum termenul din dreapta al inegalitii (1) n funcie d at t rb , rc . n acest scop utilizm faptul c n orice triunghi are loc relaia cos A a a t R+r 2R + r ra 2R + r rb cos C = , precum i cos A = s , cos B = R 2R 2R 2R + r ra ra r 2R + r rc (ntr-adevr, cos A = a 1 cos A = 2R 2R 2R S S (p b) (p c) aS 1 2 = p (p a) (p b 2R p a p bc 2Rp (p a) abc S S 2 = S S). 4R Tinnd seama de acestea i de inegalitatea R 2r, rezult c: s a a cos4 A + cos4 B + cos4 C

R + r 2R + r ra 2R + r rb 2R + r R 2R 2R 2R r 4 3r 5r ra 5r rb 5r rc ra 3 rb 5 = 5 5 R 2R 2R 2R 8 R r r adic tocmai ceea ce trebuia demonstrat. a

Not. Am primit de la Neculai Roman, Mirceti (Iai), o interesan a s s

a inegalitii de la b), anume at cos4 A + cos4 B + cos4 C

Pentru demonstraia primei pari, folosim cunoscuta sin2 A + sin2 B + s t t i inegalitatea CBS; obinem s t 2 1 2 cos4 A + cos4 B + cos4 C cos A + cos2 B + cos2 C = 3 2 2 1 2 3 9 1 2 2 = 3 sin A + sin B + sin C 3 . = 3 3 4 16 Pentru partea a doua, folosim ra + rb + rc = 4R + r; ra rb + ra rc + ra rb rc = p2 r; a2 + b2 + c2 = 2p2 2r2 8Rr. Avem ca a2 + b2 + c2 9R 2p2 2r2 + 8Rr + 9R2 , apoi ra 3 r 4 rb rc 3 r 4 3 ra + rb + rc 5 [5 5 5 = 8 R r r r 8 R r ra rb rc 3 r 4 R ra rb + ra rc + rb rc p2 5 ]= 100 100 + 4 2 + 2 3 r r 8 R r r r 4 63 r 3 r 4 2 3 R 2 + 100 100 + 2 2r + 8Rr + 9R2 = 39 8 R r r R 2 R 63 27 r 1 Daca x = 0, , funcia f (x) = 39x4 x3 + x2 are un max t R 2 2 4 3 1 , adica pentru x = , de unde concluzia anunata. t 16 2 L99. a) Care este numarul minim de puncte din plan de coordonate n nct, oricum ar fi alese, sa existe trei puncte cu centrul de greutate de ntregi. b) Sa se arate ca ntr-un spaiu n-dimensional exista 2n+1 puncte de t ntregi astfel nct oricare trei dintre acestea au centrul de greutate cu c coodonata care nu este un ntreg. Irina Mustaa, student, Bremen, G t a Soluie (Eugenia Rou, eleva, i Adrian Zanoschi, profesor, Iai). t s s s de greutate al triunghiului cu vrfurile n (x1 , y1 ), (x2 , y2 ), (x3 , y3 ) are co x1 + x2 + x3 y1 + y2 + y3 , ; aceste coordonate sunt ntregi daca i n s 3 3 x1 + x2 + x3 i y1 + y2 + y3 sunt multipli de 3. Pentru a simplica rezolv s sideram, mai departe, n locul coordonatelor punctelor, resturile acestora Pentru 8 numere, rezultatul nu este valabil. ntr-adevar daca alegem s x y 1 0 1 0 1 1 1 1 0 0 0 0 0 1 0 1

3 ra 3 r 4 rb r 5 5 5 16 8 R r r

observam ca suma x1 + x2 + x3 este multiplu de 3 daca i numai daca cele t s sunt egale. Dar, n acest caz, cea de-a doua sum y1 + y2 + y3 este 1 s a ce nseamn c, pentru orice trei dintre aceste puncte, centrul de greut a a ambele coordonate ntregi.

S analizm, n continuare, cazul a 9 numere. Reamintim c dou c a a a a congruente modulo 3 sunt considerate egale. Daca exista 5 punte care au una dintre coordonate identica, atunci p de-a doua coordonat avem unul din urmtoarele cazuri: apar toate cele t a a posibile la mprirea cu 3 sau, conform principiului cutiei, unul dintre res at de trei ori. n ambele situaii putem gasi trei coordonate cu suma multiplu t exista trei puncte cu centrul de greutate de coordonate ntregi. Presupumen acum c nu este ndeplinit condiia precedent. Atunci, a a t a c fiecare din numerele 0, 1, 2 apare cel puin o dat i cel mult de patr a t as printre abscise ct i printre ordonate. Daca doua dintre aceste numere ar s cel mult doua ori, atunci cel puin cinci coordonate ar fi egale cu al treil t ceea ce contrazice presupunerea fcut. Deci, dou dintre numerele 0, 1, a a a cel puin 3 ori la o coordonat. Notm aceste numere cu m i n. t a a s Consideram, pentru fiecare dintre cele doua grupuri cu prima coord respectiv n, ca cea de-a doua coordonata ia cel puin doua valori distin t exist trei puncte identice i centrul lor de greutate are coordonatele ntr a s avem toate cele trei valori (0, 1, 2) la ordonate, atunci suma lor este 3 i s greutate respectiv are coordonatele ntregi. Cazul cel mai nefavorabil ar cel n care n ambele grupuri, ordonatele iau dou valori a i b pentru prim a s c i d pentru cellalt grup (a 6= b, c 6= d). s a ntruct a, b, c, d {0, 1, 2}, conform principiului cutiei, rezult c e a a numere egale. Cum a 6= b i c 6= d, nseamna ca unul dintre numerele a, s cu c sau d. Putem presupune, fr a restrnge generalitatea, c a = c. aa a S artm c sumele a + c, a + d, b + c, b + d iau cele trei valori posib a aa a Avem relaiile a 6= b a + c 6= b + c, c 6= d b + c 6= b + d, b 6= a b + t Astfel, daca cele patru sume ar lua doar doua valori din cele trei, atunci a + i b + c = a + d, ceea ce implic c = d. Contradicia la care am ajuns do s a t fiecare dintre numerele 0, 1, 2 se afl printre cele patru sume. a Acum, s revenim la cel de-al treilea grup - al punctelor care au ca prim a nata un numar diferit de m i n (exista cel puin un punct de acest fel). s t unui astfel de punct adunat cu una din sumele a + c, a + d, b + c, b + d a multiplu de 3, pentru c printre aceste sume se afl toate numerele 0, 1, 2 a a Evident, punctele din primele dou grupuri cu ordonatele din suma a sus i cu punctul ales din grupa a treia formeaza un triplet a carui centru d s are coordonate ntregi. Cu aceasta, demonstraia este ncheiat. t a

b) Vom demonstra propoziia prin inducie dup n. n cazul n = 2, a t t a punctul a) un exemplu de 23 puncte care satisfac condiia din enun. t t Presupunem propoziia adevarata pentru un numar natural n 2. t c propoziia este adevrat pentru n + 1. Conform ipotezei de induc a t a a 2n+1 puncte n spaiul n - dimensional astfel nct centrul de greutate t trei dintre ele sa nu aiba toate coordonatele ntregi. Celor 2n+1 punc sus le mai adaugam la sfrit nca o coordonata egala cu 0 i apoi, din n s s punctelor, nc o coordonat egal cu 1. Astfel obinem 2n+2 puncte din sp a a a t dimensional (jumtate dintre ele se termin cu 0, iar cealalt jumtate a a a a cu 1). Se observa uor ca, oricum am alege trei dintre aceste 2n+2 punc s

lor de greutate nu are toate coordonatele ntregi. Not. Soluie corecta s-a primit de la Vlad Emanuel, elev, Sibiu. a t 1 L100. Fie x (0, 1); aratai ca exista n N astfel nct {nx} , t 3 Ciprian Baghiu i Gheorghe Iu s 1 2 1 1 Soluia 1(a autorilor). Daca x 0, t , cum = > 3 3x 3x 3x 1 2 1 2 1 2 n N cu n , , deci nx , , adica {nx} , 3x 3x 3 3 3 3 1 2 2 x , putem alege n = 1, iar pentru x = lum n = 2. a 3 3 3 2 2 2 3 3 Rmne de analizat situaia x a t , 1 . Cum ,1 = , , 3 3 3 4 4 k k+1 k k+1 , . . . , exist k 2 astfel nct x a , . Cn k+1 k+2 k + k + 2 1 3p 3p + 1 2p t a p N , obinem c x , , deci (p + 1) x p + ,p + 3p + 1 3p 2 + 3p + 1 2p 2p + 1 1 2 adic {(p + 1) x} a , , . Cnd k = 3p + 1, p N 3p + 1 3p + 2 3 3 2p + 1 2 2p + 1 2 ca (p + 1) x p + , p+ , deci {(p + 1) x} , 3p + 2 3 3p + 2 3 2p + 1 sfrit, daca k = 3p + 2, p N, atunci (p + 2) x p + 1 + s ,p + 1 + 3p + 3 2p + 1 2p + 2 1 2 deci {(p + 2) x} , , . 3p + 3 3p + 4 3 3 t Soluia 2 (Vlad Emanuel, elev, Sibiu). Daca x R \Q, mulimea {{n t este densa n [0, 1] (Lema lui Kronecker) i atunci concluzia este imedia s p x Q , fie x = , cu p, q N , (p, q) = 1; observam ca ip, i = 1, q, parc q ip ri = , deci expre resturile modulo q. Notm ri = ip (mod q); atunci a q q 1 2 qq t i = 1, q, va lua toate valorile din mulimea 0, , , . . . , . Pentru q q q 1 problema, ar fi sucient s gsim m {1, 2, . . . , q 1} astfel nct a a 3 q 2q adic m < a . Cum x (0, 1), atunci q 2. Pentru q {2, 3}, lu a 3 3 q 2q 2q q > 1 i astfel exist cel puin un ntreg n s a t , . A Pentru q 4, 3 3 3 3 2q 2q ; dac q {4, 5}, atunci a N, iar dac q 6, exist ce / a a nu este chiar 3 3 ntregi n intervalul respectiv. n1 an ak Q L101. Fie a, n 2 doua numere ntregi. Sa se arate ca k=0 n k Adrian Zahariuc, ele

Soluie. S facem mai nti cteva notaii. Fie A = t a t


n Q

al numrului m N , atunci notm cu expp m exponentul lui p din desco a a n factori primi a lui m. n1 an ak Q A Pentru a demonstra ca = Z este suficient sa aratam n! k=0 n k orice numr prim p, p n, are loc inegalitatea a Se verific uor c a s a expp n! expp A.
X n , oricare ar fi numrul prim p. a expp n! = pk k=1

k=1

ak 1 . n acest caz, avem A = a a2 . . . an1 E. Daca p este un di

n1 Q k=0

an ak

deci, relaia (1) este adevarata. t k II. Presupunem, n continuare, ca (a, p) = 1. Deoarece a(p ) 1 k k k1 1 (teorema lui Euler) i p = pk pk1 , rezulta ca pk | ap p s k k1 m(p p ) 1, oricare ar fi m N . a S demonstrm c are loc inegalitatea a a a X expp E
k=1

Fie p un numr prim mai mic sau egal ca n. a n(n1) s I. Daca p | a, atunci p2 | a2 , p3 | a3 , pn | an i p 2 | a a2 . . . an Notnd cu l cel mai mare numr natural cu proprietatea p2 n, avem a l Xn n n (n 1) 1 expp n! = 1 l expp A, pl p1 p 2
k=1

a t Pentru fiecare k N , considerm mulimile Ak = m pk pk1 i Bk = Ak {1, 2, . . . , n}. Se observ c B1 B2 Bk s a a k k1 n k . Dup cum am vzut, p | am(p p ) 1, deci fiecare elem a a pk pk1 Bk contribuie cu cel puin k la expp E. Fie Ck = Bk Bk+1 ntruct mu t sunt disjuncte doua cte doua, putem scrie: X X X X n k |Ck | = k (|Bk | |Bk+1 |) = |Bk | = expp E pk pk
k=1 k=1 k=1 k=1

n . pk pk1

ceea ce nseamn c inegalitatea (3) este adevrat. a a a a n sfrit, din relaiile (2) i (3), rezult c s t s a a X n X n expp E expp A, expp n! = pk pk pk1
k=1 k=1

deci inegalitatea (1) este adevarata.

Not. Aceeai soluie a dat Vlad Emanuel, elev, Sibiu. a s t

Marius Pachiariu, t a Soluie. Considerm inelul Zp2 , +, . Dac i {1, 2, . . . , p 1}, at t a p un element inversabil al inelului. Notam cu reprezentantul canonic al cl i p p Cu acest sens dat scrierii , avem N. Astfel, putem scrie i i
i1 i Cp =

L102. Fie p = 2k + 1 un numar prim. Atunci 2k k X X i i Cp+i1 2p 2 mod p2 , S2 = Cp+i1 2 2p mo S1 =


i=k+1 i=1

p (i 1)! Mp2 + (1) p (p 1) . . . (p i + 1) = = i! i! Mp2 p p + (1)i1 (1)i1 mod p2 , i! i i p i unde i {1, 2, . . . , p 1}. Analog, obinem relaia Cp+i1 t t mod p i orice i {1, 2, . . . , 2k}. Avem p1 X 1 1 1 i Cp p 1 + 2 3 p1 i=1 1 2 2 2 1 1 p 1 + + + + 2 3 p1 2 4 p1 2k X 1 1 1 i Cp+i1 mod p2 + + + p k+1 k+2 2k
i=k+1

Deoarece

Not. Soluie asemanatoare a dat Vlad Emanuel, elev, Sibiu. a t L103. Fie a, b, c, d reale astfel nct 1 + a2 1 + b2 1 + c2 1 + Aratai ca t 3 ab + bc + cd + da + ac + bd abcd 5.

1 1 1 Pentru a demonstra a doua cerina, observam ca 1+ + + + t 2 3 p1 ntruct termenii sumei dau resturi distincte la mparirea cu p i suma aces s t p (p 1) 0 (mod p). Prin urmare este 1 + 2 + + (p 1) = 2 p1 X 1 1 1 2 2 2 i p Cp = 2 2 p 1 + + + + 2 3 p1 2 4 p1 i=1 k X 1 1 i Cp+i1 mod p2 , p 1 = 2 k i=1 ceea ce nseamn c S2 2 2p mod p2 . a a

p1 P i=1

i Cp = 2p 2, rezulta ca

i=k+1

2k P

i Cp+i1 2p 2 mod p2 .

Mai mult, avem egalitate n cel puin una din inegalitatile de mai sus daca t daca a + b + c + d = abc + bcd + cda + dab. Gabriel Dospinescu, stude Soluie. Egalitatea din ipotez este echivalent cu t a a Y Y (i + a) (i a) = 16 X X X X X X 1i a ab + i abc + abcd 1 + i a ab i abc + Deoarece ultima egalitate se poate scrie n forma 2 X 2 X X 1 ab + abcd + a abc = 16, P rezult c |1 ab + abcd| 4, de unde obinem 3 ab + bc + cd + a a t bd + abcd 5. Se observa ca, n relaia precedenta, putem avea o egalita t P numai dacP|1 ab + abcd| = 4, ceea ce, n virtutea identitaii (), este a t P cu a = abc. Not. Vlad Emanuel, elev, Sibiu, remarc faptul c problema apar a a a and New Inequalities (citat la soluia problemei L97), semnat de acela a t a s n L104. Fie x0 > 0 si xn = x[ n ] + x[ n ] + , pentru orice n > 0. 3 6 x 2 n este convergent la 1. a) Sa se arate ca sirul n n1 5 xn n = 0. b) Sa se arate ca daca > log3 , atunci lim n 2 n Gabriel Dospinescu, stude n x2 o Soluie. a) Fie M = max 1, x1 , t . Vom demonstra prin inducie ca t 2 a a oricare ar fi n N . Datorit modului n care a fost ales numrul M , propoziia este adevarata pentru n = 1 i n = 2. hDaca inegalitatea este t s hni ni pentru orice k n 1 (n 3), atunci din x[ n ] M i x[ n ] s M 2 3 2 3 hni hni n n n n xn M+ M+ + + M = nM . Drept urmare, avem 2 3 6 2 3 6 x n a as este mrginit. a oricare ar fi n N , ceea ce nseamn c irul n n1 din enun poate fi scris n forma t a n n x[ n ] x[ n ] 1 x = n2 2 + n3 3 + . n n n 6 2 3 n n 1 1 2 De aici, innd seama de relaiile lim t t = , lim 3 = i lim (a s n n 2 n n 3 n lim an + lim bn , deducem ca
n n

xn xn lim lim xn 1 lim n n + n n + , n n 2 3 6 xn 1. Analog, demonstram ca lim xn 1. D de unde obinem lim t n n n xn xn xn xn i lim lim lim s 1 lim rezulta ca cele doua limit n n n n n n n n

sunt egale cu 1, deci lim

n x2 o 5 i ak = 1 + (M 1) s b) Consideram, din nou, M = max 1, x1 , 2 6 S demonstrm, prin inducie dup k, inegalitatea xn nak , oricare ar fi a a t a Pentru k = 0, am dovedit c propoziia este adevrat la punctul a). Presu a t a a h i hn n k+1 relaia este adevrat pentru k. Fie n > 2 3 . Atunci, t a a s 2 3k i 2 3 Conform ipotezei de inducie, rezult c t a a hni hni n 5ak + 1 xn ak + ak + n = nak+1 , 2 3 6 6 ceea ce ncheie inducia. i ht n Fie n > 1 i k = log3 s . Cum 2 log3 n 1 k 2 5 5 xn 1 + (M 1) = 1 + (M 1) n 6 6 log3 n 6 (M 1) 5 6 (M 1) n log3 5 6 2 =1+ , =1+ 5 6 5 2 nseamn c exist o constant a 0 astfel nct a a a a 5 xn 1 + anlog3 6 , oricare ar fi n > 1. n k+1 5 i dk = 2k+1 2. Vom arta, pri Considerm irurile ck = 1 a s a s 6 c xn > nck dk , oricare ar fi n 2 3k . Pentru k = 0, inegalitatea devin a n 2, care este adevrat. Presupunem propoziia adevrat pentru k. A a a t a a s t numr n 2 3k+1 i atunci, conform ipotezei de inducie, avem a hni hni n n 5n ck dk + ck dk + > 2 ck 2dk + = xn > 2 3 6 6 6 5ck + 1 n 2ck 2dk > ck+1 n 2 2dk = ck+1 n dk+1 . = 6 Cu aceasta, demonstraia prin inducie s-a terminat. t t h ni Acum, luam din nou k = log3 . Deoarece 2 k+1 log3 n 2 dk 2 2k 2 2lo 5 2 5 xn > ck =1 + >1 n n 6 n n 6 n rezulta ca log3 5 log3 5 x 6 6 1 2 n log3 2 log3 6 1 n log3 6 5 > 5 = n 21log3 1 n n 2 n 2 2 x 6 n De aici, avnd n vedere i relaia (1), deducem c irul s t as 1 nlog3 5 este n xn n xn n este mrginit. Prin urmare, lim a = 0. deci irul s 5 n nx nlog3 2 n1

xn = 1. n

L105. Sa se determine toate funciile continue f : (0, ) R, ca t ecuaia funcionala t t nxn1 f (xn ) = (x + 1) f (x) , unde n N , n fixat.

x (0, ) ,

Marian Tetiva i Dumitru Mihalach s Soluie. Pentru n = 1, ecuaia considerata devine f (x) = (x + 1) f ( t t Soluia ei, n acest caz, este f (x) = 0, x > 0. t Presupunem, n continuare, ca n 2. Ecuaia data este echivalenta cu t x+1 f (xn ) = f (x) , x > 0, nxn1 de unde, nlocuind succesiv pe x cu n x, obinem: t n n2 x+1 x + 1 n 2 f (x) = f nx , f nx = x ,..., n1 n1 f 2 n nx nx n nk x + 1 n k nn1 f x = x . n1 f nx nk De aici, deducem c a 2 k ( n x + 1) n x + 1 n x + 1 n k f x , k N . f (xn ) = n1 n1 n1 k x n + n2 ++ nk n n1 n1 n1 1 1 n1 + + = Deoarece + 1 k : 1 n n2 nk n n n rezulta ca 2 k ( n x + 1) n x + 1 n x + 1 n k f x , k N . f (x) = 1 nk x1 nk 1 k k s Avem lim x1 nk = x i lim n x = f lim x1/n = f (1). Sa calcula k k k 2 k ( n x + 1) n x + 1 n x + 1 . L = lim k nk 2 a) Daca n = 2, atunci putem restrnge produsul P = ( x +1) 2 x +1 k 2 k k x + 1 2 x+ 1 2 x + 1 2 x 1 = P 2 x1 = 2 k1 k1 = x + 1 2 x+ 1 2 x+1 2 x1 = = x+1 x 1 = x 1, x1 , pentru x 6= 1. Drept urmare, deci P = 2 k x1 2 k ( x + 1) 2 x + 1 . . . 2 x + 1 x1 x1 L = lim = lim 1/2k = , 1 k k x 2k ln x k
1/2

n acest caz, trecnd la limit n relaia (1), pentru k , obinem: a t t x1 f (1) , x 6= 1. f (x) = x ln x

Notam pe f (1) cu c (c R). Cum lim f (x) = lim


x1

x1

soluia ecuaiei considerate este: t t

x1 c = c, n x ln x

b) S vedem ce se ntmpl dac n > 2. Exist l N , astfel nct n a a a a 1 1 < li , i = 1, k. Daca x > 1, atunci ni 2 n 2 k l 2l lk n x+1 x + 1 n x + 1 < 2 x + 1 2 x + 1 2 x + 2 3 lk < 2 x + 1 2 x + 1 2 x + 1 2 x + 1 . Astfel, avem 2 k ( n x + 1) n x + 1 n x + 1 0 < k 2 n 2 2lk 2 3 x+1 x+ 1 x+1 ( x + 1) < = nk 1 x1 x1 k = 1/2kl k . = 2kl x 1 n x1 n kl l
1/2 2

( x1 c, x 6= 1 f : (0, ) R, f (x) = , unde c R . x ln x c, u 6= 1

= , deducem k 2l 1/2kl L = 0, unde x > 1, deci f (x) = 0, x > 1. 1 Dac x (0, 1), notm pe cu y i atunci y > 1. Se observ c a a s a a x 1 2 1 1 k n x+ 1 n x+ 1 n x + 1 = + 1 + 1 n k n y n2 y y n y + 1 n2 y + 1 nk y + 1 n y + 1 n2 y + 1 nk y + = = 1 1 1 1 11/nk y n y n2 y nk y n 11/n n y + 1 n2 y + 1 nk y + 1 = . 11/nk y n1 Aadar, avem s n y + 1 n2 y + 1 nk y + 1 1 = 0 1 = 0, L = lim 11/nk k k y n1 y n1 n de unde, rezulta ca f (x) = 0, oricare ar fi x (0, 1). Deoarece f este continu n x = 1, conchidem c soluia problemei, n a a t este f : (0, ) R, f (x) = 0.
k

De aici, innd cont c lim t a

1/2kl

= ln x i lim s

n k

Probleme propuse1
Clasele primare

P.124. Schimba locul unui singur beior pentru a obine o egalitate. ts t (Clasa I ) Mariana Nastasia, e

P.125. ntr-o clasa cu 24 elevi sunt 3 perechi de gemeni. La edina s t este prezent cte un singur printe din ecare familie. Ci prini p a t a t edina? s t (Clasa I ) Mihaela Glc, e a

P.126. Pentru a erbe un ou sunt necesare 4 minute. Mama vrea s a oua n trei trane. Cte minute sunt necesare? s (Clasa a II-a) Ionela Brgan, e a a

P.127. Mircea are cu 35 timbre mai mult dect fratele sau, Marius. C diferena, dac Mircea ar mai primi 10 timbre, iar Marius ar da unui prieten t a (Clasa a II-a) Inst. Maria R

t P.128. Cte numere de forma RM AT ndeplinesc condiia RAM = M (Clasa a III-a) Drago Covrig, s
MARI + ARI RI . I 7676

P.129. Scriei toate adunarile de forma t

(Clasa a III-a)

Dana Brsan, e

P.130. Dac a, b, c sunt cifre, cte egaliti de tipul a c = b : c se a at Justicai raspunsul. t (Clasa a III-a) Adina Voinescu, e

P.131. Vericai daca armaia "A se mparte exact la 5, unde A t t 2 (1 + 2 + 3 + + 1999) + 1999 + 1997" este adevrat sau fals. a a a (Clasa a IV-a) Prof. Nicolae Ivchescu as

P.132. Mama Oanei a mplinit 17532 zile pe data de 1 ianuarie 2007 lun i zi a avut o vrst de 3 ori mai mic? as a a (Clasa a IV-a) nv. Geta Cre t

P.133. Doi elevi spun pe rnd cte un numar natural, cel puin egal t mult egal cu 7. Fiecare nou numar spus se aduna la celelalte. Sa se arate elev poate s indice n aa fel numerele nct s ajung primul la suma 99 a s a a (Clasa a IV-a) Prof. Petru Asa

Clasa a V-a

V.76. Dac a, b, x sunt cifre n baza 10, s se rezolve ecuaia cu necu a a t bxa + baa + xb + ab = abb + aab. Marius Fa
1

Se primesc solu ii pn la data de 31 decembrie 2007. t a

S V.77. a se determine cte numere de trei cifre distincte abc au prop abc cba : 11 este patrat perfect. Otilia Neme, Ocna Mur s

V.78. Artai c nu exist trei numere prime a, b, c astfel nct a (b + a t a a Nicolae Ivchescu as

V.79. Artai c numrul 131000 91000 se divide cu 1000. a t a a Damian Marinescu, T

V.80. Dac restul mpririi unui numr natural la 10 este mai mar a at a spunem ca acel numar este favorabil. Aflai numerele favorabile ab cu pr t ca nici ab, nici ba nu pot fi scrise ca suma de doua numere favorabile. Ioan Scleanu a a

Clasa a VI-a
VI.76. Determinai a, b, c Z dac t a a b 3c + 5 = = . 3 4 2c + 1

Gheorghe Iu

VI.77. S se arate c ntre oricare dou puteri naturale consecutive a a a a afl cel puin o putere a lui 2. Exist dou puteri consecutive ale lui 3 n a t a a putem gasi trei puteri ale lui 2? Marius Damia

VI. 78. Fie a, b Z astfel nct mulimile {a + b, a + 2b, . . . , a + t {1, 2, . . . , 2007} coincid. S se arate c exist k N pentru care a + kb = a a a Dan Nedeianu, Drobeta-Tr

VI.79. Se consider 4ABC ascuitunghic, iar M un punct n planul a t lela prin M la AB taie AC i BC n P , respectiv N . Demonstrai ca s t dintre urmatoarele afirmaii sunt adevarate, atunci este adevarata i a tre t s \ (i) BM bisectoare pentru ABC; (ii) M C M B; (iii) [N P ] linie m 4ABC. Carmen-Daniela Tama VI.80. Sa se demonstreze ca poriunea haurata din figura t s alturat poate fi scris ca reuniune de segmente nchise, dou a a a a cte dou disjuncte. a Marius Tiba, elev, Iai s

Clasa a VII-a

VII.76. Aflai numerele naturale a, b, c pentru care 11 (a b 9) > t 11 (b c 9) > a (a 20) i 11 (c a 9) > b (b 20). s Veronica Pleu i Dan Pl a s s

VII.77. Fie x, y numere reale pozitive, ambele subunitare sau ambe 1 1 1 + 2 x + + y + . Dac unul dintre nu a nitare; s se arate c xy + a a xy x y mai mic, iar cellalt mai mare ca 1, inegalitatea i schimb sensul. a s a Marian Tetiv

VII.78. S se rezolve n numere naturale ecuaia 6a 5b = 1. a t Tudor Pdurariu, ele a

VII.79. Fie ABCD un patrulater convex, iar E i F interseciile bis s t b b unghiurilor D, respectiv B, cu diagonala [AC]. Sa se arate ca punctele E s daca i numai daca AB CD = AD BC. s Claudiu- tfean P S VII.80. Fie ABCDEF un hexagon regulat nscris ntr-un cerc, iar P

pe arcul mic BC. S se arate c P E + P F = P A + P B + P C + P D. a a Dan Radu, B

Clasa a VIII-a

VIII.76. Fie ABCD un trapez cu AB k CD, M (AD) i N s M N k AB, iar E (AB), F (CD) oarecare. Fie {O} = EF M N [N T ] perpendiculare de aceeai parte pe planul trapezului, G centrul de g s 4P EF , {Q} = M G (T BC). S se arate c M N este linie mijlocie n tr a a i numai dac Q T N . s a Bogdan Raia, t VIII.77. Pentru x, y R , sa se demonstreze inegalitatea + 2 2 6 6 2 2 x + y x y x + y 2 x3 + y 3 xy (x + y) .

Claudiu- tfean P S VIII.79. S se rezolve n numere naturale ecuaia x (x + 1) = y 2007 . a t Alexandru Negrescu, elev, VIII.80. Stiind ca 1 ianuarie 2007 este ntr-o zi de luni, sa se arate anul 2100 exist trei ani biseci n care luna februarie are trei duminici c a t zile impare. Petru Asa

Lucian Tuescu, Craiova i Gheorghe Nedele t s VIII.78. Pentru a, b, c R, s se demonstreze inegalitatea a p p p 2 + b2 ab + 2 + c2 bc + a b c2 + a2 ca a + b + c.

Clasa a IX-a

IX.76. Fie d1 , d2 , . . . , dk divizorii arului 53 72 , iar Sn = dn + dn num 1 2 4n 5 + 1 73n + 1 a a Sn , n N . Generali n N . S se arate c S2n = (5n + 1) (7n + 1) Petru Asa p IX.77. Sa se arate ca a3 + b3 ab 2 (a2 + b2 ), a, b 0. Ovidiu Pop, Sa IX.78. Fie a, b, c laturile 4ABC, iar G centrul su de greutate. No a E, F punctele de contact ale cercului nscris cu laturile BC, CA, respect se arate ca aGD + bGE + cGF = 0 daca i numai daca 4ABC este echi s Marian Ursrescu a IX.79. Fie 4ABC echilateral i P un punct n interiorul sau. C s A1 AB, B1 BC, C1 CA astfel nct P A = P A1 , P B = P B1 i P s S se arate c P este centrul de greutate al 4A1 B1 C1 . a a Iulia Pleca, e s

IX.80. Fie , , , patru numere pozitive cu suma . S se afle maxi a S = sin sin + sin sin i sa se determine situaia n care acest maxim s t Adrian Cordune

Clasa a X-a

X.76. Fie C = {z C | |z| = 1}. S se rezolve n C 2 ecuaia z1 z2 = z1 a t Gabriel Popa i Paul Georg s

X.77. Fie a, b C i z1 , z2 soluiile ecuaiei z 2 az + b = 0. S s s t t a urmatoarele armaii sunt echivalente: t s (ii) |a|2 + a2 4b < 2 |b|2 + 1 < 4. (i) |z1 | < 1 i |z2 | < 1; Marian Tetiv

X.78. Determinai triunghiurile n care tangentele unghiurilor se exp t numere naturale, exact doua dintre ele avnd aceeai paritate. s Ctlin Cali a a

X.79. Sa se arate ca n orice triunghi are loc inegalitatea (p r 2R (p rb ) (p rc ) 0. Cnd se atinge egalitatea? I. V. Maftei i Dorel Bian, B s at

X.80. Aratai ca exista o infinitate de valori n N pentru care numer t i 3n + 1 sunt ptrate perfecte. s a Gheorghe Iu

Clasa a XI-a

XI.76. Dac A M4 (R), s se arate c det (A + t A i) = det (A a a a Generalizare. Dan Popescu

XI.77. Fie f : [a, b] R o funcie de dou ori derivabil pe [a, t a a bele derivate strict pozitive. Pentru [a, b], consideram punctele A B (b, f (b)), (, yC ) Gf i D (, yD ) AB. Demonstrai ca exista C t s s 0 a+b , b astfel nct f (b) yD = yC f (a). 2 a Ctlin Tigeru a a

XI.78. Pentru x R , sa se demonstreze inegalitaile: t + 1 1 a) ln x + a (1 + ln a), unde a > 0; x a k b) ax > (1 + x) , unde a > ek , k N , iar {1}. Gheorghe Costo

XI.79. Fie (xn )n1 un ir convergent, a crui limit o notm L (xn ). De s a a a n xn = a R dac i numai dac exist lim (1 + xn as a a c exist lim a a + n L (xn ) n b R+ . Ce legatura este ntre a i b? s D. M. Btineu-Giurgiu, B a t

XI.80. Fie f : [0, 1] R o funcie continua pe [0, 1], derivabila pe t f (0) = 0. Presupunem c exist M > 0 astfel nct f 0 (x) 1x f (x) a a x x (0, 1). S se arate c f este derivabil n origine. a a a Mihai Crciun a

Clasa a XII-a

S se arate c un = 0. a a

XII.76. Fie funcia f : [a, b] R i n N . Aratai ca exista c ( t s t Z b nct f (c) + f (a) + f (b) (b a)n (f (x) + f (a) + f (b)) dx = . n1 n (c a) a Dumitru Mihalach R 1/nk a s XII.77. Fie k N fixat. Considerm irurile an = 1/(n+1)k arcsin R 1/nk an n 1 i bn = 1/(n+1)k arctg nk x dx, n 1. S se calculeze lim s a . n bn Liviu Smarandache i Lucian Tuescu s t XII.78. Daca f : R R este o funcie continua i fara puncte fixe, s t s ca nici funciile f f f , n N , nu au puncte fixe. t | {z } n ori Dorin Mrghidanu, a XII.79. Fie V spaiu vectorial de dimensiune n peste corpul K, iar u t morsm nilpotent al lui V (i.e., exist p N astfel nct up = u u a {z |
n ori

Adrian Reisn XII.80. Fie A un inel n care x4 y 4 = (x y) (x + y) x2 + y 2 , x, a) Dac inelul are unitate, s se arate c A este comutativ. a a a b) Ramne valabil rezultatul de la a) daca inelul A nu este unitar? Gabriel Dospinescu, Paris i Marian Tetiv s

Probleme pentru pregtirea concursurilo a


A. Nivel gimnazial

G116. Aflai toate numerele naturale N de patru cifre nenule distinc t prietatea ca diferena dintre cel mai mare numar obinut prin permutarea t t N i cel mai mic asemenea numr este tocmai N . s a Maria Mihe, T t G117. Fie mulimea A = {1, 2, 3, . . . , 98}. Aratai ca oricum am a t t elemente ale lui A, exist dou printre ele avnd suma cub perfect. a a Titu Zvonaru, C G118. n interiorul unui paralelogram avnd unghiul ascuit de 30 t s laturilor 17 cm i 59 cm, se considera 2007 puncte. Sa se arate ca putem s dintre aceste puncte astfel nct aria triunghiului determinat de ele s fi a egal cu 1 cm2 . a 4 Mihai Ha 0 n G119. Fie n N i A = 0 + + n 2 | 0 , 1 , . . . , n { 2 s B = m | m 2 Z + 1, |m| 2n+1 1 . Sa se arate ca A = B. Dorel Mihe, T t 2005 2007 G120. Rezolvai n N ecuaia x! (y!) t t = (z!) . Anca Stefania Tuescu, elev t a

G121. Dac a, b (0, 3/2), s se demonstreze inegalitatea a a r 1 1 1 1 1 1 + . + + + a b a+b+3 a a+ b b

Andrei Laureniu Ciupan, elev, B t

G123. Fie ABC un triunghi echilateral. Sa se arate ca orice punct M cu proprietatea ca M B = M A + M C poate fi determinat folosind doar ec echer poate fi folosit pentru a trasa drepte i unghiuri drepte.) s Nicolae Ivchescu as

G122. Fie G centrul de greutate al 4ABC i G0 proiecia sa pe dreap s t se arate c G0 [BC] dac i numai dac 3a2 < b2 c2 . a / as a Temistocle B

G124. Fie 4ABC, A0 mijlocul lui [BC], iar P i Q proieciile lui A s t respectiv AC. Sa se arate ca 4P Q AB + BC + CA. Adrian Zahariuc, ele

G125. Fie ABCD un patrat, M (AB), {O} = AC BD, {S} = C iar {E} = SO M D. Considerm AA0 (ABC), AA0 = AB, I mijlocul a iar {H} = M I A0 E. S se arate c: a a 2 VA0 ADH AB 0 a) M D (A AE); b) = . VMADH AM Petru Rduc a

B. Nivel liceal

L116. Cercul nscris n 4ABC este tangent laturii BC n punctul D1 A-exnscris este tangent aceleeai laturi n punctul D2 . Dreapta AD2 int s cercul nscris n punctele S i T . S se arate c 4ST D1 este dreptunghic s a a Titu Zvonaru, C

L117. Fie 4ABC, D (BC), iar C1 , C2 cercurile exnscrise triunghiur i ADC, tangente la BC. Aratai ca o tangenta comuna interioara cercu s t C2 trece prin punctul de contact cu BC al cercului A-exnscris. Neculai Roman, Mirc

L118. Fie M un punct al elipsei E, de focare F i F 0 . Dreptele M s intersecteaz elipsa n A, respectiv A0 . S se arate c, atunci cnd M pa a a a dreapta AA0 este mereu tangenta unei curbe fixe, care se cere a fi determ Adrian Reisn
n+3

L119. Fie n N i a.b, c R cu ab + bc + ca = 3. Sa se arate ca an+ s + c + 2abc (an + bn + cn ) 9. Titu Zvonaru, Comneti i Bogdan Ionia, B a s s t

L120. Pentru a1 , a2 , . . . , an reale pozitive, s se demonstreze inegalit a (n1)2 (n1)2 (n1)2 a2 an a1 a1 a2n1 + a2 a2n1 + + 2 3 + + + a2 a3 a1 a2 a2 a2 n 1 2

Marian Tetiv

1 L121. Fie n N dat. Sa se arate ca exista civa termeni ai irului s t m 1 a cror sum este mai mare dect a a . (n + 1) (2n + 1) Dumitru Mihalache i Marian Tetiv s

L122. La un campionat de fotbal particip 2n echipe, astfel nct a care dou se poate dinainte indica echipa mai bun. n prima etap, e a a a mpart aleator n perechi i disputa cte un meci, echipa mai buna trecn s urmatoare. Procedeul se repeta pna la nala. a) Care este probabilitatea ca a doua echip ca valoare s ias vicecam a a a b) Dac se disput i o final mic, ce probabilitate este ca, n plus, cea a as a a echipa ca valoare sa se claseze pe locul 3? Irina Mustaa, student t a L123. Pe o tabl 8 9 se aeaz dreptunghiuri 3 1 i "figuri" a s a s de forma unui dreptunghi 1 3 cruia i lipsete ptratul median (ca a s a n desenul alaturat). "Figurile" i dreptunghiurile nu se pot roti i nu s s au puncte interioare comune. Sa se arate ca exista o mulime S de 18 t ptrate 1 1 astfel nct, dac pe tabl rmn 2 ptrate neacoperite de a a a a a dreptunghiuri sau "figuri", atunci cele dou ptrate sunt obligatoriu din S a a Gabriel Dospinescu, stude

L124. Fie n N fixat.Determinai matricele A Mn (C) pentru care t In , iar A2007 + A + In = On (cu am notat operaia de conjugare). t Vlad Emanuel, el L125. Fie f : R R o funcie periodic i lipschitzian (exist L > t as a a care |f (x) f (y)| L |x y|, x, y R), iae (xn )n1 un ir strict cre s lim xn = + i lim (xn+1 xn ) = 0. Sa se arate ca mulimea punct s t n n ale irului (f (xn ))n1 coincide cu Im f . s Paul Georgescu i Gabriel P s

Training problems for mathematical contes


A. Junior highschool level

G116. Find all the natural numbers N of four distinct nonzero digit property that the diferrence between the largest number obtained by perm four digitis of N and the smallest number obtained in the same manner equ Maria Mihe, T t

G117. Let us consider the set A = {1, 2, 3, . . . , 98}. Show that two elem among any 50 elements arbitrarily chosen from A such that their sum is cube. Titu Zvonaru, C

G118. 2007 points are considered in the interior of a parallelogram wit angle equal to 30 and the lengths of its sides of 17 cm and 59 cm. Show among these points can be selected so that the area of the triangle dete

them be at most equal to

Mihai Ha G119. Let n N and A = {0 2 + + n 2 | 0 , 1 , . . . , n {1 B = {m | m 2Z + 1, |m| 2n+1 1}. Show that A = B. Dorel Mihe, T t
0 n

1 4

cm2 .

G120. Solve in N the equation x!(y!)2005 = (z!)2007 . Anca Stefania Tuescu, high-school student t G121. For 0 < a, b < 3/2, prove that the inequality r 1 1 1 1 1 1 + . + + + a b a+b+3 a a+ b b holds. Andrei Laureniu Ciupan, high-school student, B t G122. Let G be the gravity center of ABC and G0 its projection o BC. Show that G0 [BC] if and only if 3a2 < b2 c2 . / Temistocle B G123. Let ABC be an equilateral triangle. Show that any point plane with the property that M B = M A + M C can be determined using square only. (A tracing square can be used for tracing straight lines and rig Nicolae Ivchescu as G124. Let ABC be a triangle with the midpoint of [BC] denoted A0 the projections of A0 on AB and respectively AC. Prove that 4P Q AB + Adrian Zahariuc, high-school studen G125. Let ABCD be a square, M (AB), {O} = AC BD, {S} = C and {E} = SO M D. We consider AA0 (ABC), AA0 = AB, I the m [A0 D] and {H} = M I A0 E. Show that: 2 AB VA0 ADH 0 a) M D (A AE ); b) = . VMADH AM Petru Rduc a

B. Highschool level

L116. The circle inscribed in ABC is tangent to the side BC at the and the A exinscribed circle is tangent to the same side at point D2 . Th line AD2 intersects the inscribed circle at the points S and T . Show that a right-angled triangle. Titu Zvonaru, C L117. Let us consider ABC, D (BC), and C1 , C2 the exinscribed the triangles ADB and ADC that are tangent to BC. Show that a commo that passes between the circles C1 and C2 also passes through the contact BC of the A exinscribed circle to ABC. Neculai Roman, Mirc L118. Let M be a point of the ellipse E whose foci are F , F 0 . The str M F and M F 0 intersect the ellipse at points A, respectively A0 . Show t the point M runs over E, the line AA0 is ceaselessly tangent to a xed cu required to be determined. Adrian Reisn

L119. Let n N and a, b, c R with ab + bc + ca = 3. Show that + an+3 + bn+3 + cn+3 + 2abc(an + bn + cn ) 9.

Titu Zvonaru, Comneti and Bogdan Ionia, B a s t L120. Let a1 , a2 , . . . , an be positive real numbers. Prove the inequal (n1)2 (n1)2 (n1)2 a2 an a1 a2 n1 + a2 a2 n1 + + a1 2 3 + + + a2 a3 a1 a2 a2 a2 n 1 2 Marian Tetiv L121. Let n N be given. Show that a couple of terms of the 1 1 exist whose sum is greater than . m3 mn1 (n + 1)(2 n + 1) Dumitru Mihalache and Marian Tetiv L122. A football championship is attended by 2n teams such that the among each pair of teams can be predicted. In the rst stage, the teams are grouped in pairs and play by one match each pair so that the better team the next stage. The procedure is repeated until the nal match has to be a) Which is the probability for the second team as to its value to be c vice-champion? b) If a small nal match is also disputed, which is the probability for team as to its value to get classied on the third position? Irina Mustaa, student t L123. On a table of size 8 9, rectangles of size 3 1 are placed together with "gures" with the shape of a 1 3 rectangle with the middle square missing (as in the drawing aside). The "gures" and the rectagles cannot be rotated and they have no common interior points. Show that there exists a set S of 18 squares 1 1 such that, if only two squares remain on the table that are not covered by rectangles or "gures" two squares are necessarily in S. Gabriel Dospinescu, stude L124. Let n N be xed. Determine the matrices A Mn (C) such A = In , and A2007 + A + In = On ( denotes the conjugate of the element bar). Vlad Emanuel, high-school stude L125. Let f : R R be a periodical and lipschitzian function (i.e., t L > 0 such that |f (x) f (y)| L |x y|, x, y R), and let (xn )n1 be increasing sequence with lim xn = and lim (xn+1 xn ) = 0. Show th
n n

of limit points of the sequence (f (xn ))n1 is Im f . Paul Georgescu and Gabriel P

Pagina rezolvitorilor

BRA OV S Colegiul Naional de Informatica "Gr. Moisil". Clasa a IX-a. ALDE t VII(71,73), VIII(71-74); ANGHEL Ctlin: VII(71,73), VIII(71-74); A aa Sebastian: VII(71,72), VIII(72-74), IX.73; BAUM Bianca: VII(71-73), V BERCEA Laura: VII(71-73), VIII(71-74); BONTA Marcel: VII(71-73) S 74); COROESCU Alexandra: VII(71,73), VIII(72-74); DIEA Ionu: VII(71 t (71-74); DINU Cristian: VII(71-73), VIII(73,74); DRAGOMIR Drago: V s VIII(72-74); DRAGU Bianca: VII(71,73), VIII(72-74), IX.73; FARSCH VII(71-73), VIII(71-74); FOTIN Alexandra: VII(71,73), VIII(71,73-75); SAN Andreea: VII(71,73), VIII(72-74); HERMENEANU Horia: VII(71-73 75); HNCU Ramona: VII(71-73), VIII(71-74); IONESCU Marius: VII(71 (72-74), IX.73; IVAN Iulia: VII(71,73), VIII(71-74); LEVITCHI Alexandr 73), VIII(71-73); MAN Andrada: VII(71,73), VIII(71-74); MARIN Ionu: V t VIII(72-74); MATHE Emil: VII(71,73), VIII(72-74); MIEREA Ciprian: V VIII(71-74); MOLDOVAI Adelina: VII(71-73), VIII(71-75), IX(72,73); MU Oana: VII(71,73), VIII(72-74); NEGUT Bogdan: VII(71,73), VIII(72-7 Alexandru: VII(71,73), VIII(71-74); PAGU Roxana: VII(71,73), V PETRU CA Alin: VII(71,73), VIII(72-74); POPA Alexandra: VII(71,73 S 74); POPESCU Dnu: VII(71,73), VIII(71-74); POTEC Ionela: VII(71-73 a t 75); PUZDREA George: VII(71,73), VIII(72-74); RADUCU Gabriela: V VIII(72,75), IX.73; ROMAN Radu: VII(71,73), VIII(71-74), IX.72; SAT Livia: VII(71,73), VIII(71-74); SCORTEA Liliana: VII(71-73), V SERBAN Ctlina: VII(71-73), VIII(72-75); TAUS Alina: VII(71-73), V aa TENEA Codrua: VII(71,73), VIII(71-74); TICA Doris: VII(71-73), V t IX.73; VASII Cristian: VII(71,73), VIII(72-74); VLADU Ioana: VII(71,73 74); ZSILINSZKY Laura: VII(71-73), VIII(72-75).

CRAIOVA Colegiul Naional "Fraii Buzeti". Clasa a VII-a. DAN Ana-Veron t t s 75). Clasa a IX-a. TUTESCU Anca: IX(72,73), IX(71-73). Colegiul Naional "Carol I". Clasa a VII-a. STANCIU Ioan: VI(71-7 t 73,75), G(106-108,112).

HRLAU Scoala "Petru Rare". Clasa a III-a (nv. NICULESCU Carmen). SAC s Olga) Emilian: P(104-106,108-110). Clasa a IV-a (nv. PRIALA Mara: P(111,112,118,119,123). Clasa a IV-a (nv. CRETU Maria). Alina: P(104-106,108,110,112,113). Liceul Teoretic "Stefan cel Mare". Clasa a V-a. BUZILA Andreea: P Andreea: P(111,112), V(66,67,70 108,110-113), V(66,67,69); IVANUTA Madalina: P.113, V(66-70). Clasa a XI-a. BURICAN Bogdan Alexand 68,70), X.68.

IA I S Scoala nr. 3 "Al. Vlahua". Clasa a IV-a (nv. MARIUTA Valenti t

LADEANU Andrei: P(114,118,119,121,123); CULEA Alina: P(114,118,119 DIACONITA Theodor: P(104,114,118,119,121,123); HADARAG Ana-Mar 118,119,121,123); NASTASE Cosmin: P(114,118,119,121,123); POPA Iuli 118,119,121,123); PROCA Ancua-Ioana: P(114,118,119,121,123); TNC t dra-Ioana: P(114,118,119,121,123). Clasa a IV-a (inst. MAXIM Gabriela Ionela-Lavinia: P(104,105,108,114,118,121); CELMARE Raluca-Iuliana: 108,114,118,121); GHEMU Laura: P(104,105,108,114,118,121); HURCH P(104,105,108,114,118,121); NEAGU Ramona-Mihaela: P(104,105,108,114 POPOVICI Ionu: P(104,105,108,114,118,121); RUSU Ioana-Andreea: t 108,114,118,121); RUSU Madalina-Andreea: P(104,105,108,114,118,121); S P(104,105,108,114,118,121); VECHIU Madalina: P(104,105,108,114,118,1 CHI Georgiana-Alexandra: P(104,105,108,114,118,121). Clasa a VI-a. M Magdalena: V(71,73,74), VI(71,74); DASCALITA Georgiana: V(71,73,7 74); TEODORESCU Oana: V(71,73,74), VI(71,74). Scoala nr. 13 "Alexandru cel Bun". Clasa a III-a (inst. COJOCAR AGAFITEI Elena-Roxana: P(114-117,119); CARAMALAU Andra: P(114 CALIN Andreea-Claudia: P(114-117,119); COJOCARIU Andreea: P(114 DUDUMAN Luisa- tefania: P(114-117,119); LELEU Alexandrina- tefan S S 117,119); LUPA CU Diana-Maria: P(114-117,119); MANOLACHE Madal S ea: P(114-117,119); MIHAILA Narcisa-Lorena: P(114-117,119); PASCU P(114-117,119); PADURARU Tiberiu- tefan: P(114-117,119); RADUCE S Andrei: P(114-117,119); SAVIN Cristina-Simona: P(114-117,119); STEFA Vasile: P(114-117,119); STIUBEI Cosmin-Ionu: P(114-117,119). t Scoala nr. 26 "Gh. Cobuc". Clasa a II-a (inst. RACU Maria). Apach s Georgiana: P(104,114-117); BURA Emma-Andreea: P(104,114-117); Alexandra: P(104,114-117); CRACIUN Ioana-Daniela: P(104,114-117); FI Stefania: P(104,114-117); GRADINARIU Georgiana: P(104,114-117); Ovidiu Constantin: P(104,114-117); HUZA Madalina: P(104,114-117); M Drago-Claudiu: P(104,114-117); MAXIM Alexandra-Camelia: P(104,114 s CA Cosmin: P(104,114-117); VASILE Bogdan-Andrei: P(104,114-117). Scoala Normala "V. Lupu". Clasa a VI-a. NASTASIA Mariana: P V(66-75). Lieul Teoretic "M. Eminescu". Clasa a V-a. BRNOSCHI Letii t 123), V.71, VI.71; ETCU Magda: P.123, V(71,73,74), VI.71; FILIMON L P(121-123), V(71-73). Colegiul Naional "Emil Racovia", locaia "Gh. Asachi". Clasa a III-a t t t LINESCU Rodica). BAJENARU Brdia: P(114-120); CHIVULESCU A a t P(114,116-120); PETREA Madalina: P(114-120); UNGUREANU Georg 120). Clasa a VII-a. TUDORACHE Alexandru Gabriel: VI(71-75), V G(106,108,110). Colegiul Naional "C. Negruzzi". Clasa a VI-a. PAVALOI Alexand t V(66,67,69,70). Clasa a VIII-a. TIBA Marius: G(107,108,110-112). Colegiul Naional. Clasa a VII-a. MOCANU Dan Mihai: V(71,72,75), t VII.75; OROIAN Bianca: V(66,67,69,70), VI.69, VII.70; PETRESCU E

V(71,73,75), VI.71, VII.71.

ONE TI (Bacu) S a Scoala "Ghia Mocanu". Clasa a VII-a. PADURARIU Tudor: V t VIII(66,67,69), IX(68,70), G(96-98,101-105).

SIBIU Colegiul Naional "Gh. Lazar". Clasa a XII-a. VLAD Emanuel: L(9 t SUCEAVA Scoala generala nr. 3. Clasa a II-a (nv. TABARCEA Silvestru). Stefan: P(104,105,107-109). Clasa a III-a (inst. NECHITA Daniela). Mircea: P(104-106,108-110).

Premii acordate rezolvitorilor

ASOCIA TIA "RECREATII MATEMATICE" n colaborare c revistei RECREATII MATEMATICE acorda cte o diplom i un p as cri, pentru trei apariii la rubrica Pagina rezolvitorilor, elevilor urmato a t t Scoala "Petru Rare", Hrlu s a PINTILII Alina (cl. a IV-a): 2/2005(8pb), 1/2006(8pb), 1/2007(7pb).

Liceul Teoretic "Stefan cel Mare", Hrlu a BUZILA Andreea (cl. a V-a): 2/2005(8pb), 1/2006(8pb), 1/2007(11pb) BURICAN Bogdan Alexandru (cl. a XI-a): 1/2003(5pb), 1/2004(7pb),

Scoala nr. 3 "Al. Vlahua", Iai s t CELMARE Raluca-Iuliana (cl. a IV-a): 1/2006(5pb), 2/2006(8pb), 1/2 CULEA Alina (cl. a IV-a): 1/2006(5pb), 2/2006(6pb), 1/2007(5pb); NEAGU Ramona-Mihaela (cl. a IV-a): 1/2006(5pb), 2/2006(8pb), 1/2 POPOVICI Ionu (cl. a IV-a): 1/2006(5pb), 2/2006(8pb), 1/2007(6pb) t POPA Iulian (cl. a IV-a): 1/2006(5pb), 2/2006(6pb), 1/2007(5pb); PROCA Ancua-Ioana (cl. a IV-a): 1/2006(5pb), 2/2006(6pb), 1/2007 t RUSU Ioana-Andreea (cl. a IV-a): 1/2006(5pb), 2/2006(8pb), 1/2007( VECHIU Mdlina (cl. a IV-a): 1/2006(6pb), 2/2006(8pb), 1/2007(6pb a a

Scoala nr. 13 "Alexandru cel Bun", Iai s MIHAILA Narcisa-Lorena (cl. a III-a): 1/2006(5pb), 2/2006(5pb), 1/2 Colegiul Naional "C. Negruzzi", Iai t s TIBA Marius (cl. a VIII-a): 1/2006(7pb), 2/2006(8pb), 1/2007(5pb).

Colegiul Naional, Iai t s MOCANU Dan Mihai (cl. a VII-a): 1/2005(11pb), 1/2006(5pb), 1/20

IMPORTANT

n scopul unei legturi rapide cu redacia revistei, pot fi utilizate urm a t adrese e-mail: tbirsan@math.tuiasi.ro sau t_birsan@yahoo profgpopa@yahoo.co.uk . Pe aceasta cale colaboratorii pot purta c ia un dialog privitor la materialele trimise acesteia, procurarea nu t revistei etc. Sugerm colaboratorilor care trimit probleme originale a publicare sa le numeroteze i sa-i reina o copie xerox a lor pentru s s t purta cu uurina o discuie prin e-mail asupra acceptarii/neaccepta s t t tora de ctre redacia revistei. a t

La problemele de tip L se primesc soluii de la orice iubitor de ma t elementare (indiferent de preocupare profesionala sau vrsta ). Fiecar soluiile acestor probleme - ce sunt publicate n revista dupa un a t urmat de numele tuturor celor care au rezolvat-o. a

Adresm cu insistena rugmintea ca materialele trimise r a t a s nu fie (s nu fi fost) trimise i altor publicaii. a a s t

Rugm ca materialele tehnoredactate s fie trimise pe adresa reda a a A soite de ierele lor (de preferina n L TEX). t s t

ASOCIA TIA RECREA MATEMATIC TII

La data de 14.02.2005 a luat ina ASOCIATIA RECREATII t MATICE, cu sediul n Iai (str. Aurora, nr. 3, sc. D, ap. 6), avnd ca s jinirea activitatilor de matematica specice nvatamntului preuniversita zarea si desfasurarea de activitati care sa contribuie la dezvoltarea gustu matematica n rndurile elevilor, profesorilor si iubitorilor de matematica larea preocuparilor si cercetarilor originale. Obiectivele majore pentru atingerea scopului propus sunt: 1. editarea unei reviste destinata elevilor i profesorilor revista " s Matematice"; 2. fondarea unei biblioteci de matematic elementar biblioteca " a a Matematice"; 3. alcatuirea unei colecii de cari de matematica elementara, cari d t t t i aate la prima apariie Colecia "Recreaii Matematice". s t t t Poate deveni membru al Asociaiei, printr-o simpla completare a une t orice perosana care adera la obiectivele acesteia si sprijina realizarea lor. Membri de onoare ai Asociaiei, academicienii: t

Constantin Cord Radu Miron

Revista semestrial RECREAII MATEMATICE este ed ASOCIAIA RECREAII MATEMATICE. Apare la datele de 1 1 septembrie i se adreseaz elevilor, profesorilor, studenilor i tutu pasionai de matematica elementar. n atenia tuturor colaboratorilor Materialele trimise redaciei spre publicare (note i articole, che metodic, probleme propuse etc.) trebuie prezentate ngrijit, clar i co trebuie s prezinte interes pentru un cerc ct mai larg de cititori. Se reco textele s nu depeasc patru pagini. Evident, ele trebuie s fie origin nu fi aprut sau s fi fost trimise spre publicare altor reviste. Rugm rialele tehnoredactate s fie nsoite de fiierele lor. Problemele destinate rubricilor: Probleme propuse i Problem pregtirea concursurilor vor fi redactate pe foi separate cu enun i de ie/rezolvare (cte una pe fiecare foaie) i vor fi nsoite de numele autoru la i localitatea unde lucreaz/nva. Redacia va decide asupra oportunitii publicrii materialelor prim n atenia elevilor Numele elevilor ce vor trimite redaciei soluii corecte la proble rubricile de Probleme propuse i Probleme pentru pregatirea conc vor fi menionate n Pagina rezolvitorilor. Se va ine seama de regulile: 1. Pot trimite soluii la minimum cinci probleme propuse n prezent i cel anterior al revistei; pe o foaie va fi redactat soluia une probleme. 2. Elevii din clasele VI-XII au dreptul s trimit soluii la pr propuse pentru clasa lor, pentru orice clas mai mare, din dou clase m imediat anterioare. Elevii din clasa a V-a pot trimite soluii la problemel pentru clasele a IV-a, a V-a i orice clas mai mare, iar elevii claselor trimite soluii la problemele propuse pentru oricare din clasele primare i s mai mare. Orice elev poate trimite soluii la problemele de concurs (tip 3. Vor fi menionate urmtoarele date personale: numele i pr clasa, coala i localitatea. 4. Plicul cu probleme rezolvate se va trimite prin pot (sau v direct) la adresa Redaciei:

Prof. dr. Temistocle Brsan Str. Aurora, nr. 3, sc. D, ap. 6, 700 474, Iai Jud. IAI E-mail: tbirsan@math.tuiasi.ro sau t_birsan@yahoo.

CUPRINS

300 de ani de la naterea lui Leonhard Euler (1707 1783).......................... Profesorul Dumitru Ion Mangeron (1906 1991) In Memoriam ................ Grigore Moisil fotografie-document inedit .....................................................

ARTICOLE I NOTE

C. IGERU Similitudini n plan i puncte Torricelli asociate.......................

A. REISNER Ordinul elementelor grupului GLn ( Z ) .........................................

T. BRSAN Variaiuni pe tema dreptei lui Euler i cercului celor nou punc M. TETIVA New Proof for an Old Inequality ................................................... D. M. BTINEU-GIURGIU Asupra calculrii unor limite de iruri............... S. BOGA O generalizare a teoremei lui Van Aubel..........................................

NOTA ELEVULUI

B. CIACOI O propoziie echivalent cu conjectura lui Goldbach ....................

CHESTIUNI METODICE

L. TUESCU Cum se poate obine o inegalitate ...............................................

PROBLEME I SOLUII

Soluiile problemelor propuse n nr. 1/2006............................................................. Soluiile problemelor pentru pregtirea concursurilor din nr. 1/2006 .................... Probleme propuse........................................................................................................ Probleme pentru pregtirea concursurilor ................................................................. Training problems for mathematical contests ..........................................................

Pagina rezolvitorilor .................................................................................................

Anul X, Nr. 1

Ianuarie Iuni

RECREAII MATEMATIC

REVIST DE MATEMATIC PENTRU ELEVI I PROF

125 de ani de la apariia revistei Recreaii tiinifice (1883 1888)

e i = 1

Asociaia Recreaii Matemati IAI - 2008

Semnificaia formulei de pe copert: i ntr-o form concis, formula e = 1 leag cele patru ramuri fun
ale matematicii: ARITMETICA GEOMETRIA ALGEBRA ANALIZA MATEMATIC reprezentat reprezentat reprezentat reprezentat de de de de 1 i e

Redacia revistei :

Petru ASAFTEI, Dumitru BTINEU-GIURGIU (Bucureti), Temistocle BRS BRNZEI, Ctlin - Cristian BUDEANU, Constantin CHIRIL, Eugenia COHA CORDUNEANU, Mihai CRCIUN (Pacani), Paraschiva GALIA, Paul GEO Mihai HAIVAS, Gheorghe IUREA, Lucian -Georges LDUNC, Mircea LUPAN MRANU, Andrei NEDELCU, Alexandru NEGRESCU (student, Iai), Gabr Dan POPESCU (Suceava), Florin POPOVICI (Braov), Maria RACU, Necula (Mirceti), Ioan SCLEANU (Hrlu), Ioan ERDEAN (Ortie), Dan TIBA ( Marian TETIVA (Brlad), Lucian TUESCU (Craiova), Adrian ZANOSC ZVONARU (Comneti). Adresa redaciei: Catedra de Matematic Universitatea Tehnic Gh. Asachi Iai Bd. Carol I, nr.11, 700506, Iai Tel. 032 213737 / int. 123 E-mail: recreatii.matematice@gmail.com http://www.recreatiimatematice.uv.ro

COPYRIGHT 2008, ASOCIAIA RECREAII MATEMATICE Toate drepturile aparin Asociaiei Recreaii Matematice. Reproducerea in parial a textului sau a ilustraiilor din aceast revist este posibil numai cu acordu scris al acesteia. TIPRIT LA SL&F IMPEX IAI Bd. Carol I, nr. 3-5 Tel. 0788 498933 E-mail: simonaslf@yahoo.com ISSN 1582 - 1765

Anul X, Nr. 1

Ianuarie Iunie

RECREAII MATEMATICE

REVIST DE MATEMATIC PENTRU ELEVI I PROF

e i = 1
Revist cu apariie semestrial

EDITURA RECREAII MATEMATICE

IAI - 2008

Recreaii Stiinifice 125 de ani de la apar t t

La 15 ianuarie 1883, la Iai (n tipo-litografia "Buciumului Romn s primul numar al revistei Recreaii Stiinice, publicaie tiinica adresata t t t s t audiene, o premier n peisajul cultural romnesc de la acea dat. Colect t a a tuziati care au pus piatra de temelie a acestei ntreprinderi cu profunde r s n viaa academica romna era format din N. Culianu, C. Climescu, I. t la Facultatea de Stiine din Iai), G.I. Lucescu, V. Paladi, G.I. Roiu, I t s s G. Zarifopol, I.V. Praja i I.M. Dospinescu (profesori la diferite licee s s Iai). Colaboratori regulai au fost i M. Tzony, V.C. Buureanu, A. Scr s t s t Universitatea din Iai. s Noua publicaie a avut ca model reviste de prestigiu ce apareau n arile t t ndelungat din Europa, din care s-au preluat, n timp, articole i problem a s lui Gergonne (Frana), Mathesis (Gand, Belgia - a nu se confunda cu t italiana cu acelai nume fondata la Torino n 1895), Journal de mathma s mentaires (Paris), Revue Scientique etc. Revista a aprut lunar, fr ntrerupere, timp de ase ani. Iniial, fiec a aa s t avea 32 de pagini, cuprinznd articole cu subiecte variate: aritmetica, algeb trie, geometrie analitica, trigonometrie, calcul diferenial si integral, istori t ticii, mecanica, topografie, cosmograe, astronomie, chimie, geograe sa Ulterior, s-au publicat i numere de 24 de pagini sau numere comasate ( s mere de vacana) de 48 sau 40 de pagini. t Revista a cunoscut o buna raspndire n Regatul Romniei (n ntind atunci), primind colaborari sub forma de note, scrisori, etc. din Iai, s Bacu, Botoani, dar i din Nsud, Paris i a constituit un model pen a s s aa s prinderi similare ulterioare, cum ar fi cunoscuta Gazeta Matematic. a Ca "profesie de credina" a inimosului colectiv de editori, apare scris t adresare Catra Cetitori n prima pagina a nr.1 din 1883, n care se afirma noua publicaie s remedieze din lacunele existente n nvamntul nostr t a at ospitalitate profesorilor sau institutorilor care "trateaza o chestiune de stii metoada proprie lui", sa ncurajeze tinerimea studioasa. Cu modestie i ob s se adauga: "Nu pretindem ca vom produce lucrari originale. n starea n care s noastra, lucrari originale, pe terenul stiinific, snt foarte greu de intreprin t nu este vinovat pentru aceasta; trebue sa ne facem stagiul cuvenit." ntr-adevar, cteva date sunt edicatoare n privina situaiei existente t t ara noastr. Imediat dup Unirea Principatelor s-a nlturat, ocial, scri t a a a fabetul chirilic (n anul 1860 n Tara Romneasc i 1863 n Moldova), iar as cu caractere latine s-a adoptat sistemul etimologic cu utilizarea semnelor n perioada de pna la 1880, limba romna literara (moderna) i-a ncheia s de unicare i de stabilizare n forma pe care o are astzi. nvamntul t s a at mari prefaceri i frmntri: ninarea universitilor din Iai i Bucur s a a t at s s nvaamntului din 1864, numeroasele regulamente menite sa organizeze t

coli i sa stabileasca programele acestora, lipsa de manuale i cursuri n lim s s s etc. Aceste diculti, greutile materiale, ct i lipsa unui public form at at s receptarea unei reviste tiinice de acest gen fac ca actul publicrii R s t a Stiinice sa fie unul temerar i pornit dintr-o nalta responsabilitate, t s deplinirea caruia au fost necesare multa daruire i multe sacricii. In vo s se subliniaz concepia umanist a colectivului de redacie, abordarea co a t a t actului tiinic i a procesului de nvamnt de la noi: s t s at "Credem ca noi am tras cea ntai brazda care conduce catra lucrari Brazdai mica si ngusta, dar exista ! [. . . ] Noi de la nceput am avut ca o sa trebuiasca sa facem sacrificiu de timp si bani. Am facut si vom sacrificiu." In numarul 7 din anul IV este amintit rolul deosebit jucat de Ioan Po care "venit din Transilvania n anul 1858 a predat cel ntaiu n mod sis complect stiinele matematice n cursul superior al Liceului din Iai. [. . . ] s t excelent profesor. A stiut sa inspire gustul stiinilor matematice la o m t elevii lui [. . . ]."; printre acetia sunt amintii Gh. Roiu, M. Tzony, C. s t s V. Palade .a. s Rsfoind numerele Recreaiilor Stiinifice, cititorul de azi va aprecia a t t limb folosit acum peste o sut de ani, calitatea nalt a materialelor in a a a a va constata i existena multor lacune n terminologia de specialitate, ma s t n matematic. a Aproape n fiecare numr erau prezente rubricile de Probleme Propuse a s Rezolvate (soluii primite de la cititori). Terminologia vremii era Probleme t iar rezolvitorii erau de la diferite licee i coli din Iai (n primele luni) ca, m s s s apar rezolvitori din toat Moldova (Bacu, Brlad, Dorohoi, Focani, G a a a s de la Bucureti (Liceul Sf. Sava, Scoala de Poduri i Sosele, Scoala Milita s s Normal Superioar), Craiova, Alexandria sau chiar de la Paris, unde i a a s studiile muli tineri romni (Institutul Duvignau, Institutul Jauret, Coll t Barbe, Liceul St. Louis). Cel mai activ rezolvitor a fost Vasile Cristescu, la Liceul din Iai, apoi student la Scoala de Poduri i Sosele din Bucureti, s s s care a publicat i cteva Note; ulterior, a fost membru fondator al renumi s Matematica - nfiinata n anul 1895, cu apariie nentrerupta pna astazi t t cei patru "stlpi" ai acesteia. n ultimul an de apariie a Recreaiilor Stiin t t t printre rezolvitori este de remarcat prezena lui Dimitrie Pompeiu (1 t marele matematician de mai trziu, pe atunci n vrsta de 15 ani, elev n la gimnaziul din Dorohoi. Sunt numeroi rezovitorii care au devenit apo s prestigiu: Ermil Pangrati (profesor de geometrie descriptiv la universit a at i Bucureti, rector al celei din urm, organizatorul Scolii de arhitectur s s a a ministru, senator), Anastasie Obregia (profesor de chimie organica, Univer Iai), Grigore G. Stratilescu, Petru N. Culianu, Vasile Teodoreanu . a. s s Unele materiale publicate sunt menite s suplineasc lipsa de cri i a a at s ndemna studenilor i profesorilor. Constantin Climescu expune, ntr-u t s

noua articole intitulat Cteva curbe celebre si importante (opt aparute n i unul n nr.1 din 1885), principalele curbe plane clasice: cisoida lui Di s coide, cicloide, spirale etc. ncepnd cu nr.4 din 1885 i continund numr s a (cu puine excepii) pna la dispariia revistei, Miltiade Tzony publica U t t t probleme, ce cuprinde 98 de probleme de mecanic raional (anume, sta a t a sunt complet rezolvate i nsoite de guri (v. [8], pp. 138-140, pentru un st s t nunit). O expunere a determinanilor i utilizrii lor a fost fcut de Ion t t s a a a n nr.11 i 12 din 1883 i nr.1 din 1884; o alta prezentare a acestora (dupa s s ce apare n vol.V (pp. 150 i 190), este semnat cu pseudonimul Candi s a notele de subsol de la pp. 239 i 256, se aduc argumente pentru identicar s pseudonim cu Victor Costin, pe atunci student, mai trziu profesor la un din Iai. Un membru fondator i constant colaborator, publicnd chestiu s s de matematic elementar, este I.V. Praja. a a Din bogatul coninut al Recreaiilor Stiinice, punem n evidena a t t t t direcii dezvoltate n paginile sale. G.I. Lucescu public un studiu ampl t a mentat despre calendar. Vasile C. Buureanu semneaza doua lungi cicluri t n domeniul mineralogiei. August Scriban public o serie de articole de a Asiei centrale. Iacob Solomon trateaz, ntr-o serie de ase note din vol.VI a s de istoria matematicii n antichitate utiliznd surse la zi (de exemplu, Gesc Mathematik (Istoria matematicii) a lui Moritz Cantor, 1880). Revista Recreaii Stiinifice a gzduit i traduceri (pariale) ale uno t t a s t referina. G.I. Roiu traduce n romnete (dup o ediie italian a lui t s s a t a F. Brioschi, Florena,1868) i public n vol. II i III ale revistei prima t s a s Elementele lui Euclid. (Precizam ca traducerea completa a Elementelor a mult mai trziu de Victor Marian i publicat n Biblioteca Gazetei Mat s a trei volume, 1939-1941.) Sub acelai pseudonim Candide sunt prezentate s (nr.4-6 i 8-11) i vol. V (nr. 2 i 3) traduceri din Geometrie der Lage (Ge s s s poziie) a lui Staudt. t Rubrica Diverse, cu scop de informare, are un coninut bogat i variat t s toate domeniile tiinei. n nr.1 din 15 ianuarie 1883, la p.21, se preia art s t tograa Micarii" (ce anticipeaza cinematograa) dupa Revue Scientiq s dec. 1882 - o adevarata dovada de promptitudine i de capacitate de selec s t din 1883, la p.47, apare o scurt notia despre ninarea la Stockholm, a t t faimoasei Acta Mathematica, sub auspiciile regelui Suedo-Norvegiei i sub c s lui G. Mittag-Lefler; printre colaboratori se numarau i Appell, Goursat s (din Frana) care, mai trziu (n 1905), vor alcatui comisia care a examin t tez de doctorat a lui Dimitrie Pompeiu, la Sorbona [5], [6]. In nr.9 din a articol ntreg este dedicat erupiei vulcanului Krakatoa, catastrof care a t a globul (relatare oculara). n vol.III (1885), nr.5 i 6, doua articole sun s marelui matematician belgian Eugne-Charles Catalan (1814 -1894), men se inclusiv celebra sa conjectur din 1844 (numerele 8 i 9 sunt singure a s naturale consecutive care sunt puteri exacte) rezolvat mult mai trziu, n a

de matematicianul romn Preda Mihailescu [12]. Vol. IV din 1885, p.20 toarele, prezint fenomenul natural numit mascaret sau pororoca, iar la p a cititorul este informat asupra unor date tehnice privind proiectul turnului i asupra unor controverse contemporane generate de aceasta construcie. s t Sunt cultivate dezbateri n jurul unor probleme din actualitatea tii s din realitatea nvamntului romnesc. Astfel, articolul "Sf. Gheorghe at din nr. 5, anul VI, semnat de Paul Tanco (din Nsud), primul romn aa matematici, este comentat de Constantin Gogu, ilustru profesor de la Un din Bucureti, care mai public, apoi, cinci scrisori asupra regulelor nt s a pentru gsirea zilei Patilor. Menionm polemicile susinute cu publi a s t a t Contemporanul pe teme de chimie sau astronomie (p. 139, p. 142, p. 180 nr. 6 din 1883 etc.). Cu deosebita putere de patrundere, G. Zarifopol scr adeseori ipoteze gratuite, emise de un nvatat strain sunt luate de redac t Contemporanul ca ultimele adevaruri ale stiinei". Este interesant anal a t i riguroas fcut de G. Lucescu n vol.VI unui manual manuscris de A s a a a destinata cl. I i a II-a primara i urmata de avizul negativ dat acestuia. Sem s s pentru corectitudinea redaciei este faptul c sunt publicate alturat a t a a adus de un referent lucrrii lui I.V. Praja "Curs de aritmetic raion a a a t 1885, 359 pagini, ct i replica acestui autor (vol.VI, pp.238-247). s Sa remarcam i faptul ca, dintre membrii fondatori, numai N. Culianu s s nu apar n paginile revistei cu nici un fel de contribuie, iar I.M. Zamfires t doar un rezumat al unui articol din Revue Scientifique asupra filoxerei. Alte trsturi foarte interesante sunt rigurozitatea deosebit a activit aa a at tice, bazata pe standarde care sunt valabile i astazi ct i conectarea ex s s viaa tiinic mondial. Se manifest o bun nelegere a fenomenului t s t a a a a t tiinice i a evenimentelor care frmntau lumea n penultimul deceniu a s t s a al XIX-lea. Redacia acorda maxima importana relaiilor directe cu cititorii, do t t t publicarea de scrisori, note, a unor soluii diferite pentru aceeai problema t s statistici amnunite referitoare la rezolvitori. a t Pentru toate articolele i notele publicate se indic sursele folosite. Cu s a traordinar este aparatul bibliograc folosit de catre autori, la toate discipli obiectul publicaiei. Se citeaz cri i reviste de specialitate strine con t a at s a (an de apariie chiar i 1888), dar i "Aritmetica" lui Amlohie Hotiniul t s s Iai, sau Memoriile Academiei din Paris, ncepnd cu 1699. s Semnalam cititorilor i prezena unor erori specifice epocii de pionerat s t naie, de numerotare a problemelor, de tiprire etc.), corectate n parte t a erate de ctre redacia revistei i care nu afecteaz n mod esenial lectura a t s a t Fara nici o ndoiala, revista Recreaii Stiinice a fost dedicata n pr t t chestiunilor de matematica. O statistica care ia n considerare numarul arat c n aproximativ 90% din spaiul revistei sunt tratate subiecte de a a t tic, mecanic i astronomie. Numrul total al problemelor propuse, a a s a

n cei ase ani de apariie, este de 298 (cu 284 ind numerotate doua pro s t dicionarele de periodice romneti revista este menionat ca o publicaie t s t a t tribuie important la educaia matematic a tineretului [3, 7]. t a t a n istoria matematicii romneti, perioada 1860-1898 este marcata de s fcute n scopul organizrii i modernizrii nvamntului i punerii bazelo a a s a at s tiinifice originale. Contribuia Recreaiilor Stiinifice la realizarea acestu s t t t t a fost recunoscut i apreciat de generaiile care i-au urmat. n Introducere as a t an I (1895), redactorii Gazetei Matematice spun: "Mai muli dintre noi dat t gust [pentru matematic - n. n.] revistei "Recreaii Stiinifice" ce a apar a t t de 6 ani la Iai si pe care noi ncercam a o continua" [17]. Mai trziu, n s aceast revist srbtorea 40 ani de existena, I. Ionescu, Gh. Tieica i a a a a t t s aminteau rolul avut de Recreaiile Stiinifice [9, 16]. Gh. Tieica spunea: " t t t ncercare de a iei din acest impas, de a rupe cu ineria, de a determina un s t preocupare stiinifica si de a crea astfel un nceput de atmosfera prielnica t stiinei matematice, a fost facuta la Iai prin publicarea "Recreaiilor S s t t " [16, p. 69]. Alte aprecieri ale unor distini matematicieni romni po s n [18]. I. Popa face n 1955 [14], un studiu aprofundat asupra con t aportului Recreaiilor Stiinifice - numind-o precursoare a Gazetei Matem t t prilejul srbtoririi a 60 de ani de apariie a Gazetei Matematice; studiul a a t i n volumul omagial dedicat centenarului Universitaii din Iai [15]. M s s t [1], [11], dar i articolele [2], [4], [13] dau cititorului noi surse de informa s Constantin Climescu a fost, prin bogia i varietatea subiectelor p at s sacrificiile materiale fcute, susintorul principal i sufletul Recreaiilor a t a s t Pe coperta interioara a revistei din al VI-lea an de apariie este scris: "R t Administraia la Dl. C. Climescu, Profesor la Facultatea de Stiine, Strada t t Aceeai adres apare i n casetele ce urmeaz titlul n fiecare numr din a s a s a a an. Apariia revistei Recreaii Stiinifice, nvingnd dificultai de tot felul, t t t t un act de curaj, druire, nelepciune i clarviziune. Revista a reuit ca, a t s s ani de existena, s contribuie la ridicarea nivelului nvamntului din ar t a at t n special al celui matematic.

Bibliografie 1. G. St. Andonie - Istoria matematicii n Romnia, vol. I, Ed. tiinifica, s t 1965 (pp. 236-240).

2. Gh. Banta - O pagina din istoria matematicii romneti: centenar s s "Recreaii Stiinice", Probleme de istoria i lozoa tiinei, vol. X, 19 t t s s t Iai a Academiei Romne, 15-30. s

3. St. Brsnescu, F. Brsnescu - Educaia, nvatamntul, gndirea peda a a t Romnia. Dicionar cronologic, Ed. tiinifica i enciclopedica, Bucureti, t s t s s

4. T. Brsan - Recreaii Stiinifice - "cea ntai brazda", Recreaii Matematic t t t nr. 1, 1-5. 5. T. Brsan, D. Tiba - O suta de ani de la publicarea tezei de doctorat a lu Pompeiu, Recreaii Matematice VII(2005), nr.2, 85-89. t 6. T. Brsan, D. Tiba - One hundred years since the introduction of the se by Dimitrie Pompeiu, in IFIP, vol.199, "System modeling and optimization agioli, A. Dontchev, H. Furuta, K. Marti, L. Pandol eds., Springer, Bos 35-39. 7. M. Bordeianu, P. Vladcovschi - nvatamntul romnesc n date, Jun 1979. 8. Gh. Gheorghiev, D. Iean - Miltiade Tzony primul profesor de m s Universitatea din Iai, Probleme de istoria i filozofia tiinei, vol. X, 19 s s s t Iai a Academiei Romne, 125-146. Aprut, ntr-o form prescurtat, i n s a a a a s Miltiade Tzony the first professor in Mechanics at the University of Ia Travaux du Comit Roumain dHistoire et de Philosophie des Sciences, 55-60. 9. I. Ionescu - Constituirea, administrarea si redactarea "Gazetei Matematic n volumul jubiliar Gazeta Matematica, 1895-1935. Istoric-nvataminte, "Gazetei Matematice", vol. XI, Bucureti, 1935 . s 10. G. Ivnescu - Istoria limbii romne, Junimea, Iai, 2000. a s 11. N. Mihileanu - Reviste de matematici elementare din Romnia (pna a Ed. Gil, Zalau, 1995. 12. P. Mihilescu - Primary cyclotomic units and a proof of Catalans a J.Reine Angew. Math. 572 (2004), 167-196. 13. R. Miron - Centenarul revistei "Recreaii Stiinice", Probleme de isto t t zoa tiinei, vol. X, 1984, Filiala Iai a Academiei Romne, 17-19. s t s 14. I. Popa - "Recreaii Stiinifice"- precursoare a "Gazetei Matematice t t Matematica i Fizica, seria A, nr. 9, 1955, 492-493. s 15. I. Popa - Dezvoltatrea matematicii, aprut n Contribuii la istoria a t Universitatii din Iai, vol. II, pp. 7-39, Bucureti, 1960. s s t 16. G. Tieica - Rolul "Gazetei Matematice" n dezvoltarea stiinei mat t Romnia, aparut n volumul jubiliar Gazeta Matematica, 1895-1935. Isto minte, Biblioteca "Gazetei Matematice", vol. XI, Bucureti, 1935, 67-75. s 17. *** - Introducere, Gazeta Matematic, an I, nr. 1, septembrie 1895. a 18. *** - "Recreaii Stiinifice"- prezena n contiina posteritatii, Recreai t t t s t t tice, 5(2003), nr.1, p.5.

Temistocle BRSAN, Iai s

Dan TIBA, Bucure

Proiect de reeditare

n anul 1999, la Iai, a aparut revista Recreaii Matematice, iar ulterio s t a fost nfiinata o asociaie cu acelai nume, Asociaia "Recreaii Matem t t s t t membri n toat ara. Continuarea tradiiei revistei Recreaii Stiinific a t t t t revist tiinic (predominant matematic) adresat tineretului este u as t a a a important al noii asociaii: nca mai sunt valabile unele din observaiile co t t cuvntul "Catra Cetitori" din ianuarie 1883. Dar peisajul publicaiilor matematice romneti adresate tineretului st t s mult mai bogat acum ca n epoca de nceput: Gazeta Matematica apar serii, A i B, apar Revista Matematica din Timioara, Arhimede (Bucureti s s s (Braov), Foaie Matematica (Chiinau) i multe alte publicaii locale. Se s s s t meroase concursuri de matematic, la nivel local, regional, naional i inte a t s Chiar i astzi, n aceste condiii de exigena sporit, vechea revist s a t t a a Stiinifice prezinta interes, prospeime i utilitate. t s t Asociaia "Recreaii Matematice" a considerat o datorie de onoare ree t t tegral a coleciei revistei Recreaii Stiinice n forma originar, nemodi a t t t a utilizarea tehnicilor moderne de reproducere a textelor i i-a xat ca term s s de realizare a acestui proiect data de 15 ianuarie 2008. La ora actual, exist doar puine exemplare complete ale coleciei revis a a t t tii Stiinifice i care se afl ntr-o stare destul de precar. La Seminarul M s a a t "Al. Myller" din Iai exist dou colecii complete ale Recreaiilor Stiin s a a t t donata acestei biblioteci de C. Climescu, fondator i principalul animator s i o a doua donat de V. I. Praja, redactor fondator i colaborator al ei. s a s Academiei Romne are, de asemenea, colecia complet a Recreaiilor Sti t a t Redarea n circuitul public a acestui monument de nceput al culturii romneti, este de un real folos tinerimii studioase i unor cercuri largi de s s Realizarea acestui proiect n-ar fi fost posibil fr sprijinul entuziast i a aa s onat al doamnei Marinela Ghigea i al firmelor Kepler Systmes dInfo s Dazoot din Bucureti. Exprimam pe aceasta cale mulumirile noastre cele m s t pentru nelegerea i efortul depus pe parcursul a aproape doi ani de mun t s Reeditarea cuprinde cele ase volume originale ale revistei i o brour ca s s s a o introducere, o not asupra ediiei, erat, index de autori, de rezolvitori, d a t a i de surse (parial) i o galerie de portrete. n varianta electronica, aceste in s t s sunt interactive. Republicarea actual, inclusiv n format electronic, are ca scop prent a riscului dispariiei acestei opere i, pe de alt parte, s o fac accesibil t s a a a a CD (sau on-line la adresa http://www.recreatiistiintifice.ro , de putea prelua gratuit). Speram ca aceasta aciune, dedicat aniversrii a a t ani de la apariia revistei, va stimula specialitii s reanalizeze fenomenu t s a n fascinantul secol XIX, veac de ctitorie n tiina romneasc modern. s t a a

Asociaia "Recreaii Matema t t

Polinoame Fibonacci, polinoame ciclotomi

Loredana STRUGARIU, Ciprian STRUGARIU

Deoarece irul lui Fibonacci este cunoscut elevilor nca din cl. a IX-a, iar s de ordinul n ale unitii i polinoamele ciclotomice sunt n materia pr at s cl. a X-a pentru olimpiada de matematic, considerm c abordarea unui a a a subiect este util att elevilor ct i profesorilor. Vom prezenta cteva a s privind polinoamele Fibonacci i cele ciclotomice i legatura dintre ele. s s

1. Polinoame Fibonacci - denire, legtura cu triunghiul lu a Polinoamele Fibonacci sunt denite prin relaia de recurena t t Fn+1 (x) = xFn (x) + Fn1 (x) , sau prin urmatoarea formula explicita
[(n1)/2]

cu F1 (x) = 1 i F2 (x) = x s

n j 1 n2j1 , Fn (x) = x j j=0 nj1 j unde [x] este partea ntreag a lui x, iar a Cnj1 . Se c j F0 = 0. Observaie. Polinoamele Fibonacci pot denite prin relaia de recu t t X Fn (x) = xFn1 (x) + Fn2 (x) ,
2

cu F0 (x) = 0 i F1 (x) = 1 s

Exemple: F1 (x) = 1, F2 (x) = x, F3 (x) = x +1, F4 (x) = x3 +2x, F5 (x) = x etc. Lund x = 1 n (2), obinem Fn (1) = Fn , unde Fn este sirul lui Fibon t

Lema 1 (proprietatea de divizibilitate). Daca m este divizor al lui Fm este divizor al lui Fn . Daca p este un numar prim, atunci Fp (x) es ireductibil.

Teorema 1. Fie F0 , F1 , F2 , . . . polinoamele Fibonacci peste cmpul dimensiune 2. Atunci, avem: 1) F2n+1 sunt singurii termeni de grad par si nu sunt divizibili cu x; singurii termeni de grad impar, n 0; 2) Fnt + Fn+t = xFn Ft , pentru 0 t n; 2 3) F2n = xFn , pentru n 0; 2 2 4) F2n+1 = Fn + Fn+1 , pentru n 0; 5) Fmn (x) = Fm (x) Fn (xFm (x)), pentru pentru m, n 0; 6) F2mnp = xFmn Fmnp + Fp , pentru 0 p mn; 7) F2mn+p = xFmn Fmn+p + Fp . k Radacinile polinomului Fn (x) sunt de forma xk = 2i cos , pe n 1, . . . , n 1. Pentru p numr prim, aceste rdcini sunt de 2i ori part a a a
1

Profesori, Colegiul Na ional "Eudoxiu Hurmuzachi", Rdu i (Suceava) t a a t

rdcinilor polinomului ciclotomic de ordinul p. Analiznd coecienii prim a a t noame Fibonacci se observa legatura dintre triunghiul lui Pascal i aceste p s F1 (x) = 1x0 F2 (x) = 1x1 F3 (x) = 1x2 + 1x0 F4 (x) = 1x3 + 2x F5 (x) = 1x4 + 3x2 + 1x0 F6 (x) = 1x5 + 4x3 + 3x1 F7 (x) = 1x6 + 5x4 + 6x2 + 1x0 F8 (x) = 1x7 + 6x5 + 10x3 + 4x1 F9 (x) = 1x8 + 7x6 + 15x4 + 10x2 + 1x0 F10 (x) = 1x9 + 8x7 + 21x5 + 20x2 + 5x1 . A. N. Philippou i asociaii sai [4] au studiat polinoamele Fibonacci s t k, k 2, pe care le-au denite astfel: F0 (x) = 0, F1 (x) = 1 n P kj (k) (k) Fn (x) = x Fnj (x) , Fn (x) =
(k) j=1 k P j=1 (k) (k)

n = 2, 3, . . . , k; n = k + 1, k + 2, . . .

xkj Fnj (x) ,

(k)

Observaie. Pentru k = 2 acestea se reduc la Fn (x), iar pentru k = t (k) la irul lui Fibonacci Fn de ordinul k. s

2. Polinoame ciclotomice - denire, proprieti. Pentru ec at natural n 1 radacinile complexe ale ecuaiei xn = 1 se numesc radacinil t n ale unitatii. Acestea sunt numere complexe de forma 2k 2k xk = cos + i sin , k = 1, 2, . . . , n 1. n n Mulimea acestor radacini se noteaza cu t Un = {x C | xn = 1} .

t t Teorema 2. Mulimea Un este un grup ciclic faa de nmulirea nume t plexe, numit grupul radacinilor de ordinul n ale unitatii. Propoziie. Fie Un = xk = cos 2k + i sin 2k ; k = 1, 2, . . . , n 1. . t n n hxk i = Un (k, n) = 1.

Grupul ciclic Un are (n) generatori, unde (n) este numarul numerelo mai mici ca n, relativ prime cu n (indicatorul lui Euler ). Cele (n) r ordinul n ale unitii care genereaz grupul Un , adic numerele complexe at a a 2k 2k xk = cos + i sin , k = 1, 2, . . . , n 1, (k, n) = 1. n n se numesc radacinile primitive de ordinul n ale unitatii. Notm cu Pn a rdcinilor primitive de ordinul n ale unitii i cu o rdcin primitiv a a at s a a a a de ordinul n a unitaii. t

Teorema 3.

Polinomul monic ale crui rdcini sunt rdcinile primitive de ord a a a a a unitii, se numeste al n-lea polinom ciclotomic i are forma at s Y n (X) = (X ) , n N .
Pn

Observaie. Dac n = p = numr prim, atunci Pp = Up \ {1}. t a a S Pd = Un , Teorema 4. 1) d|n T 2) Pd1 Pd2 = , d1 , d2 divizori naturali ai lui n, d1 6= d2 .

n o Pn = k | 0 k n 1, (k, n) = 1 .

Gradul polinomului ciclotomic n (X) este egal cu cardinalul mulimii t (n). Teorema 5 (relaia lui Dedekind ). t Xn 1 = Y
d|n

d (X)

unde produsul se face dupa toi divizorii naturali ai lui n. t Demonstraie. Folosind descompunerea n factori a polinomului X n t rema 4, avem Y Y Y Y Y Xn 1 = (X ) = (X ) = (X ) =
Un
d|n

Teorema 6. Pentru p > 0, numar prim, avem: p (X) = X p1 + X p2 + + X + 1. k1 Observaie. pk (X) = p X p t a , k N , p numr prim, p > 0. Exemple. Primele 10 polinoame ciclotomice: 1 (X) = X 1 2 (X) = X + 1 3 (X) = X 2 + X + 1 4 (X) = X 2 + 1 5 (X) = X 4 + X 3 + X 2 + X + 1 6 (X) = X 2 X + 1 7 (X) = X 6 + X 5 + X 4 + X 3 + X 2 + X + 1 8 (X) = X 4 + 1 9 (X) = X 6 + X 3 + 1 10 (X) = X 4 X 3 + X 2 X + 1.

Pd

d|n

Pd

d|n

Teorema 7 (relaia lui Mbius-Dedekind ). Pentru orice n N exista t Y ( n ) n (X) = Xd 1 d ,


d|n

unde : N {1, 0, 1} este funcia lui Mbius, dat prin t a a 1, dac n = 1, k (n) = (1) , dac n = p1 p2 pk (p1 ,p2 ,. . . ,pk prime distinct a 0, daca n se divide prin patratul unui numar prim. Alte proprieti ale polinoamelor ciclotomice: at 1) n (X) Z [X], n N , 2) n (X) este ireductibil n inelul Z [X], n N , 3) n (X) este un polinom reciproc, n 2, 4) Pentru n N i p > 0 un numar prim avem: s i) daca p divide n, np (X) = n (X p ), n (X p ) ii) daca p nu divide n, np (X) = , n (X) 5) 2n (X) = n (X), pentru n > 1, numr natural impar. a

3. Legtura dintre polinoamele Fibonacci i polinoamele cic a s Aceast legtur a fost expus de K. Kuwano n The Design of Mathema a a a a shop (n japoneza), Scientist, 2004 i apoi preluata de K. Motose n [3]. s Considernd doua variabile x i y i notnd cu X = x + y i Y = xy, s s s polinoamele simetrice Fn (X, Y ) prin Fn (X, Y ) =

xn y n , xy numite polinoame Fibonacci de doua variabile. Exemple. F0 = 0, F1 = 1, F2 = x + y = X, F3 = x2 + y 2 + xy = X 2 Fn+2 = XFn+1 Y Fn . ii) Daca m este divizor al lui n, atunci Fm este divizor al lui Fn . Demonstraie. i) direct prin nlocuire n formula (12). t ii) Deoarece xm y m este divizor al lui xn y n , rezult armaia. a t

Lema 2. i) Fm+n = Fm Fn+1 Y Fm1 Fn , pentru n 0 si m 1; n p

Observaie. Dac considerm n formula (13) X = 1 i Y = 1 t a a s irul lui Fibonacci, fapt pentru care polinoamele Fn (X, Y ) au fost numite s Fibonacci. O alt form a polinoamelor Fibonacci de dou variabile este: a a a [(n1)/2] X n j 1 n2j1 j Fn (x, y) = y , n 1. x j j=0

( n ) Q d d . Lema 3. i) n (x, y) = x yd
d|n

n mod inductiv, vom deni polinoamele ciclotomice de doua variabile Y n n 1 (x, y) = x y, x y = d (x, y) .
d|n

ii) n (x) = n (x, 1). iii) n (x, y) = n (y, x), pentru n 2.

Demonstraie. i) rezult din formula de inversiune aP Mbius, iar i t a lui deniia lui n (x). iii) decurge din punctul i) i formula t s n = 0 pen d
d|n

Deoarece polinoamele n (x, y) sunt simetrice pentru n 2, putem d noamele Pn (X, Y ) unde X = x + y, Y = xy astfel nct n (x, y) = Pn ( exemplu, P6 = x2 + y 2 xy = X 2 3Y . Teorema 8. Vom conveni ca P1 = 1. Atunci, avem: 1) Pn este ireductibil n Z [X, Y ]; Q 2) Fn = Pd ;
d|n

3) Pn =

d|n

3) Rezult din formula de inversiune a lui Mbius. a 4) Dac mai nti considerm Pd = Pd0 atunci avem: a a d (x) = d (x, 1) = Pd (x + 1, x) = Pd0 (x + 1, x) = d0 (x) i astfel d = d0 . Din Lema 2, ii), tim ca F(m,n) este divizor comun al lui s s Daca Pd este divizor comun al lui Fm i Fn , atunci d este divizor comu s i n i deci d este divizor al lui (m, n). Astfel Pd este divizor al lui F(m,n) . s s D este divizor comun al lui Fm i Fn , atunci D este divizor al lui F(m,n) , d s este un produs al polinoamelor ireductibile distincte Pd , ceea ce implica a Bibliograe

4) (Fm , Fn ) = F(m,n) , unde ( , ) reprezinta c.m.m.d.c.; n particular (Fn , Demonstraie. 1) Pn Z [X, Y ] din deniie. Daca Pn = ST , cu S, T t t polinoame neconstante, atunci (x) = (x, 1) = P (x+1, x) = S (x+1, x) pentru polinoamele neconstante S (x + 1, x) , T (x + 1, x) Q [x], contrar litaii peste Q. t 2) Rezulta din urmatoarea ecuaie: t Y Y xn y n Fn (X, Y ) = d (x, y) = Pd (x, y) . = xy
1d|n d|n

Fd

( n ) d

1. T. M. Apostol - Resultants of Cyclotomic Polynomials. Proc. Amer. M 24(1970), 457-462. 2. T. Koshy - Fibonacci and Lucas Numbers with Applications. New Yo 2001. 3. K. Motose - On values of cyclotomic polynomials. VII, Math J. Okaya 2004. 4. A. N. Philippou, C. Georghiou, G. N. Philippou - Fibonacci poly order k, multinomial expansions and probability, Internat. J. Math. M 6(1983), 545-550. 5. M. Tena - Radacinile unitatii, Soc. St. Mat., Bucureti, 2005. s 6. W. A. Webb, E. A. Parberry - Divizibility Properties of Fibonacci Po Fibonacci Quarterly 7.5 (1969), 457-463. 7. http://mathworld.wolfram.com

Submulimi ale unei mulimi finite i matrici b t t s


Adrian REISNER1

I Familii cu proprietatea P. Fie F o familie de submulimi X1 , X t ale mulimii {1, 2, . . . , n}. Spunem c familia F verific proprietatea P t a a ndeplinete condiiile urmtoare: s t a a) |Xi | = + , unde , N , i {1, 2, . . . , n}; b) |Xi Xj | = , i, j {1, 2, . . . , n}, i 6= j. Spunem c familia F veric proprietatea duala P 0 a proprietii P a a at deplinete condiiile: s t a0 ) j {1, 2, . . . , n} aparine la exact + submulimi Xi ale lui F; t t b0 ) i, j {1, 2, . . . , n} i distincte aparin la exact submulimi ale lu s t t Numim matrice asociata familiei F, matricea binara A = (aij ) Mn (R prin aij = 1 dac i Xj i aij = 0 n caz contrar. De asemenea, dat a a s binar A, se poate face trecerea "invers" la o familie de submulimi ale lui a a t mod evident.

Fie data o mulime X de cardinal |X| finit. Consideram familia F de s t ale lui X avnd anumite proprieti. Utiliznd matricele binare (adic ac at a ca elemente 0 sau 1), vom demonstra cteva rezultate privind familia F.

Pentru familiile cu proprietatea P, ne propunem s gsim o relaie nt a a t i s demonstrm proprietatea dual P 0 . s a a a

Propoziia 1. Sunt adevarate afirmaiile: t t 1. F are proprietatea P t AA = I + J (1); 2. F are proprietatea P 0 A t A = I + J (10 ), unde I este matricea unitate si J este matricea cu toate elementele 1. Demonstraie. Calculnd produsul t AA, innd seama de condiiil t t t obinem matricea avnd elementele de pe diagonala principal egale cu t a celelalte egale cu , adica matricea I + J; formula (1) este astfel stabil n n P P Invers, dac (1) are loc, avem |Xi | = a aki = aki aki = elemetul
(1) k=1 k=1 k=1

p Propoziia 2. Daca F are proprietatea P, atunci A GLn (R) si ( t Zn . Demonstraie. Sa dovedim ca A este inversabila, i.e. A GLn (R). t 1 + 1 .. ... + .. + ... = ( + n) det (I + J) = . . . ... ... .. ... ... ... .. . . . + = ( + n) n1 .
1

a de pe diagonala matricei A t A = + , adic proprietatea a). La fel, |X n P aki akj = , adica proprietatea b). Afirmaia 10 se dovedete cu argumente similare. t s

Cercettor, Centrul de calcul E.N.S.T., Paris a

(ultima egalitate se obine n urma scderii primei coloane din celelalt t a p (detA)2 = det (t AA) = det (I +J) = (+n) n1 i detA = (+n s unde = 1. Demonstraia se ncheie, deoarece pentru orice matrice bina t det A Z.

Propoziia 3. Daca familia F are proprietatea P, atunci + n = ( t Demonstraie. Prin calcul direct, obinem c JA = ( + ) J (3) t t a 1 1 1 a J = ( + ) JA (A fiind inversabil) sau JA = J (4). + t Din (1) deducem c J t AA = J + J 2 = ( + n) J, de unde, innd a + n + t 1 t t (4), J A = ( + n) JA = J. Ca urmare, AJ = (J A) = + + + n + n 2 i JAJ = J =n J. Pe de alta parte, datorita relaiei (3 t s + + + n t ( + ) J 2 = n ( + ) J. n consecina, n = n ( + ) i rezult s a + adevrat. a a

Propoziia 5. Daca familia de submulimi F are proprietatea P, atu t t si proprietatea duala P 0 . Demonstraie. Conform Propoziiilor 1 i 4, avem t AA = I + J t t s t A A. Deci A t A = I + J i, conform punctului 10 al Propoziiei 1, dedu s t are proprietatea P 0 .

Propoziia 4. F avnd proprietatea P, au loc relaiile: t t 1 AJ = JA, 2 t AA = A t A (i.e. A este matrice normala). (5) + n (2) (3) Demonstraie. 1 AJ = t J = ( + ) J = JA. + (1) 1 2 t AA = I + J = I + t A1 t A J = I + t A1 (t AJ) = (1) = I + t A1 J t A = t A1 (I + J) t A = t A1 t AA t A =

Caz particular i exemplu. Pentru = 1, relaiile (1) i (2) se sc s t s I + J i n = 2 + + 1 = ( + 1) + 1 (deci n este impar!). Atunc s p (+1) ( + n) n1 = ( + 1) 2 Z . De exemplu, matricea binar a 1 1 1 0 0 0 0 1 0 0 1 1 0 0 1 0 0 0 0 1 1 A = 0 1 0 1 0 1 0 0 1 0 0 1 0 1 0 0 1 1 0 0 1 0 0 1 0 1 1 0 verific t AA = A t A = 2I + J. Avem = 2 i det A = 24. a s II Familii cu proprietatea R. Fie X o mulime de cardinal t t {X1 , X2 , . . . , Xm } o familie de submulimi strict incluse n X. Spunem

familie veric proprietatea R dac pentru orice pereche (i, j) X 2 exi a a unica o mulime Xk din familie astfel nct {i, j} Xk . t Asociem matricea binara B = (bij ) Mn,m (R) lund bij = 1 daca s t bij = 0 n caz contrar. Tinnd seama de proprietatea R i utiliznd nota pentru elementele unei matrice relativ la matricea B t B Mn (R) avem: (i) (B t B)ij = 1 daca i 6= j, caci elementul (B t B)ij corespunde l submulimilor coninnd {i, j}; t t (ii) (B t B)ii = di , unde di este numrul submulimilor ce conin eleme a t t Aadar, s d1 1 ... 1 1 d2 . . . 1 B tB = . . . . . . . . . . . . . 1 1 . . . dn Fie Xk o submulime ce conine elementul i. Deoarece Xk 6= X, exist t t j Xk . Ca urmare, exist cel puin o submulime Xl diferit de Xk coni / a t t a t Rezulta ca di > 1, adica di = 1 + ai , iar ai > 0.

Observm c f este o funcie polinomial i f (1) = det (B t B). a a t a s o sum de n determinani avnd o coloan (coloana derivat!) cu a t a a egale cu 1: x + a1 1 x ... x x x 1 x + a2 . . . x + a2 x + + f 0 (x) = ... . . . ... ... ... ... x 1 x . . . x + an x

Propoziia 6. Daca familia {X1 , X2 , . . . , Xm } are proprietatea R, a t tricea B t B Mn (R) este inversabila, i.e. B t B GLn (R). Demonstraie. Considerm t a x + a1 x ... x x x + a2 . . . x , x R. f (x) = ... ... ... ... x x . . . x + an ... ... ... ...

Deriva toate

Derivata f 00 va fi o sum de determinani avnd dou coloane cu elemente a t a 1; deci f 00 (x) = 0, x R. Ca urmare, f (x) = ax + b i, din f (0) = b i f 0 (0) = a, gasim s s n Y Y Y ai , a = ai + + ai . b=
i=1 i6=1 i6=n

n sfrit, s
n Y Y Y ai + ai + + ai 6= 0, det B t B = f (1) = a + b = i=1 i6=1 i6=n

adica B B GLn (R).

Observaie. n ipotezele Propoziiei 6, avem rang (B t B) = n m t t rang (B t B) = n rang B m).

O generalizare a teoremelor Stolz-Cesaro


Sorin PU SPANA1

1. Rezultate clasice. Vom prezenta n aceast prim parte bine-c a a teoreme Stolz-Cesaro i o reciproca a lor, omind demonstraiile (pentru s t t consultate [2] i [3]). s

Teorema 1. Daca (an )n1 si (bn )n1 sunt doua siruri de numere r an nct: i) sirul (bn )n1 este strict crescator si nemarginit, ii) lim n bn an = l. l R, atunci lim n bn Teorema 2. Daca (an )n1 si (bn )n1 sunt doua siruri de numere r an+1 an nct: i) lim an = lim bn = 0, ii) lim = l R, iii) sir n n n bn+1 bn an este strict descrescator, atunci lim = l. n bn Teorema 3. Daca (an )n1 si (bn )n1 sunt doua siruri de numere r bn+1 an an+1 nct: i) lim R\{1}, ii) lim = l R, atunci lim n bn n bn n bn+1

2. Generalizri. Teorema 1 admite urmtoarea generalizarea (cu a a cazului limitei infinite):

an+1 an an = l R, atunci lim = l. lim n bn+1 bn n bn Demonstraie. Daca > 0 i M este un majorant pentru irul de la t s s exist m N astfel nct n m s avem a a an+1 an l < bn+1 bn 2M |an+1 an l (bn+1 bn )| < 2M |bn+1 |an am l (bn bm )| < Obinem astfel t
n1 X |bi+1 bi | |bn | , 2M i=m 2

Teorema 4. Daca (an )n1 si (bn )n1 sunt doua siruri de numere n1 P 1 |bi+1 bi | este marg fel nct: i) lim |bn | = , ii) sirul |bn |
n i=1 n1

n m

Dar

am lbm an an am bn bm < = + l bn l bn bn bm bn |bn bm | |am lbm | an am |am lbm | < l + < + < + = , |bn | |bn | bn bm |bn | 2 2 2
1

|bn | an am bn bm l < 2 |bn bm | ,

n > m.

Profesor, Craiova

Corolarul 1. Daca (an )n1 si (bn )n1 sunt doua siruri de numere r |bn nct: i) sirul (|bn |)n1 este strict crescator si nemarginit, ii) sirul |bn an+1 an an este marginit, iii) lim = l R, atunci lim = l. n bn+1 bn n bn

ntr-adevar, ipotezele i) i ii) implica primele doua ipoteze ale Teorem s

Teorema 2 admite urmatoarea generalizare (cu pierderea cazului limite

Teorema 5. Daca (an )n1 si (bn )n1 sunt doua siruri de numere r P 1 n1 |bi+1 bi | este nct: i) lim an = lim bn = 0, ii) sirul n n |bn | i=1 n1 an+1 an an = l R, atunci lim = l. iii) lim n bn+1 bn n bn Demonstraie. Daca > 0 i M este un majorant pentru irul de la t s s exist m N astfel nct n m s avem a a an+1 an bn+1 bn l < M |an+1 an l (bn+1 bn )| < M |bn+1 bn
n+p1 X |an+p an l (bn+p bn )| < |bi+1 bi | < |bn+p | , M i=1

n m

ns, pentru orice n N, exist un ir cresctor de numere naturale (p (k) a a s a a nct bn+p(k) |bn |, k 1, deci putem presupune c |bn+p | |bn |, p 1. Obinem astfel |an+p an l (bn+p bn )| < |bn |, n m, p 1 t an la limita, dupa p , obinem |an lbn | < |bn | t bn l < , n an lim = l. n bn Corolarul 2. Daca (an )n1 si (bn )n1 sunt siruri de numere reale a i) lim an = lim bn = 0, ii) sirul (|bn |)n1 este strict descrescator, n n |bn+1 bn | an+1 an este marginit, iv) lim = l R, atunci l n bn+1 bn n |bn+1 ||bn | n1

Observaii. i) Daca n cele doua teoreme i corolare nlocuim (an )n1 t s n n cu ((1) an )n1 i, respectiv, ((1) bn )n1 obinem c, pe lng celelal t a a s P |bn+1 + bn | 1 n1 |bi+1 + bi | respectiv din marginirea irului s |bn | i=1 |bn+1 | |bn | n1 an+1 + an an = l R avem lim = l. lim n bn+1 + bn n bn t ii) Teorema reciproca 3 poate fi mbunataita cernd n locul ipotezei an+1 |bn+1 | + |bn | sa fie marginit. ntr-adevar pentru > 0 avem bn+1 |bn+1 bn | n1 |bn+1 | + |bn | , n m. |bn+1 bn |

iii) Din demonstraiile date, i din enunurile teoremelor i corolarelo t s t s dent ca ele ramn valabile i n ipoteza ca (an )n1 i (bn )n1 sunt iruri s s s complexe. De fapt rezultatele pot fi extinse i n cadru mult mai larg al s normate cu unitate; enunurile rezultatelor de mai sus se adapteaz cu u t a s demonstraiile se fac cu aceleai argumente. t s 3. Aplicaii. t

Problema 1. Fie (xn )n1 si (un )n1 doua siruri de numere reale a lim un = u (1, ). Atunci sirul (xn )n1 are limita daca si numai n 1 (un xn+1 xn )n1 are limita, caz n care avem lim xn = lim (un x n u 1 n Indicaie. Se aplic Teoremele 1 i 3 irurilor an = u1 u2 un1 x t a s s u1 u2 un1 .

Folosind Corolarul 1 putem extinde acest rezultat i pentru valori nega s u, pierznd nsa cazul limitelor infinite, ca mai jos:

Problema 2. Fie (xn )n1 si (un )n1 doua siruri de numere reale a lim un = u, |u| > 1. Atunci sirul (xn )n1 este convergnt daca si numai n 1 (un xn+1 xn )n1 este convergent, caz n care avem lim xn = lim (un n u1 n Observaie. Rezultatul ramne valabil dac irurile ce intervin n e t a s iruri de numere complexe. Urmtoarea problem ne prezint n ce condiii s a a a t anterior ramne valabil n cazul |u| < 1.
n

Problema 3. Fie (xn )n1 si (un )n1 doua siruri de numere reale a (xn )n1 sa fie marginit iar lim un = u, |u| < 1. Atunci sirul (xn )n1 es

gent daca si numai daca sirul (un xn+1 xn )n1 este convergent, caz n 1 lim xn = lim (un xn+1 xn ). n u 1 n Indicaie. Se aplic Corolarul 2 irurilor an = u1 u2 un1 xn i bn = u1 t a s s

Din nou facem observaia ca rezultatul ramne valabil i pentru iruri t s s complexe. Este evident c innd seama de observaia i) rezultatele anterio at t valabile dac nlocuim un xn+1 xn cu un xn+1 + xn i u 1 cu u + 1. a s Toate acestea pot fi restrnse n

Problema 4. Fie (xn )n1 un sir marginit de numere complexe, ia si (vn )n1 doua siruri convergente de numere complexe astfel nct l n Atunci sirul (xn )n1 este convergent daca si numai d lim vn .
n

(un xn+1 + vn xn )n1 este convergent, caz n care avem


n

lim xn =

lim (un xn+1 + vn xn )


n

lim un + lim vn
n

Problema urmatoare este demonstrata n [4] i generalizeaza Problema s

Problema 5. Fie a0 , a1 , ..., ak R, a0 6= 0 si (xn )nk un sir de nume proprietatea ca sirul (yn )nk , yn = a0 xn + a1 xn1 + + ak xnk , este c Daca polinomul a0 xk + a1 xk1 + + ak are toate radacinile de modul atunci sirul (xn )nk este convergent.

Indicaie. Demonstraia se face prin inducie dupa k, primul pas a t t t reducndu-se la: || < 1 i (xn+1 xn )nk convergent (xn )nk c s adic un caz particular al Problemei 2. a

Am vzut ns c dac cerem mrginirea lui (xn )nk , atunci afirmaia a a a a a t rmne adevrat pentru || 6= 1 i, prin urmare, teorema rmne val a a a s a radacinile polinomului sunt n modul diferite de unitate. Obinem astfel u t generalizare: (0) (1) (k) Problema 6. Fie an n1 , an n1 ,. . . , an n1 , k + 1 siruri

complexe, convergente respectiv catre a(0) , a(1) ,. . . , a(k) , si astfel nct polinomului a(0) xk + a(1) xk1 + + a(k) sa fie n modul diferite de unitat un sir marginit (xn )n1 este convergent daca si numai daca sirul (0) (1) (k) este convergent, caz n care a an xn + an xn1 + + an xnk nk+1 (0) (1) (k) lim an xn + an xn1 + + an xnk . lim xn = n n a(0) + a(1) + + a(k)
(0) (1) (k) (1) n

Cum nsa (yn )n1 este convergent rezulta ca (zn )n1 este convergent i con s blemei precedente (care este valabil i pentru iruri de numere complexe as s vaiei fcute, rezult c irul (xn )n1 este convergent iar relaia nal este t a a as t a

s Demonstraie. Dac yn = an xn + an xn1 + + an xnk i zn = t a a xn1 + +a(k) xnk atunci, cum (xn )n1 este mrginit, rezult c lim (y a a a

Problema 7. (Jensen) Fie sirurile (xn )n1 si (yn )n1 astfel nct: se |y1 | + + |yn | xn este convergenta, sirul este marginit si lim = n yn |y1 + + yn | n1 x1 + + xn lim = l. n y1 + + yn Bibliografie

1. R. Cristescu - Analiza functionala, Ed. Didactic i Pedagogic, Bucure as a 2. D.M. Batineu - Siruri, Ed. Albatros, Bucureti, 1979. t s 3. D.M. Btineu, I.V. Maftei, I.M. Stancu-Minasian - Exerciii si pr a t t analiza matematica, Ed. Didactic i Pedagogic, Bucuresti, 1981. as a 4. O. Mayer - Teoria funciilor de o variabila complexa, Ed. Academiei, 19 t 5. D. Mihe - Despre calculul unor limite de siruri, R.M.T. nr. 2/1990. t 6. D.- t. Marinescu , V. Cornea - In legatura cu o problema din G.M.A S nr. 3/2005. 7. D. Mihe, M. Piticari - O problema de convergena, G.M.-A nr. 1/1990 t t

O problem de combinatoric destul de gr a a


Marian TETIVA1
n aceast not ne ocupm de urmtoarea a a a a

Problem. Din oricare 2n + 1 numere ntregi (distincte) ale caror a depasesc pe 2n 1, se pot alege trei (tot distincte) care au suma zero. Este o problema de olimpiada (cu regret, nu tiu unde am ntlnit-o s dect pare la prima vedere. Pentru rezolvare, ns, nu avem nevoie dec a numitul "principiu al cutiei" ("principiul lui Dirichlet", mai este numit n matematica romneasca; "the pigeonhole principle", n cea engleza i a s daca vrem sa repartizam n + 1 obiecte n n cutii, atunci trebuie ca ntr-o c mai mult de un obiect. De asemenea, nu e nevoie, n esena, de alt tip de ra t dect cel din problema lui Erds [1] (una din rarele probleme "foarte simpl de Erds). Aceasta problema cere

sa se arate ca pentru k > [(n + 1)/2] si a1 , . . . , ak numere ntregi a 1 a1 < < ak n exista doua printre ele a caror suma este tot unul numere.

Soluia [2] este destul de simpl: s observm c a1 , . . . , ak pe de o parte t a a a a . . . , ak a1 pe de alta, sunt 2k 1 numere din mulimea {1, . . . , n}. Cum t doua dintre acestea sunt egale (conform principiului cutiei!) i, pentru ca s din prima grup sunt diferite dou cte dou (ceea ce e valabil i pentru a a a s din a doua grup), ajungem la o egalitate de forma ai = aj a1 ai + a1 a rezolva problema. Mai mult, pentru k = [(n + 1)/2], se pot gasi k num primele n numere ntregi pozitive astfel nct suma oricaror doua nu este e al treilea dintre ele; de exemplu, numerele [n/2] + 1, [n/2] + 2, . . . , n (tot citat). a

n cele ce urmeaz vom arta cum se rezolv problema de la care a a a nit folosind aceeai idee (de mai multe ori vom utiliza acest tip de rai s t considernd-o folositoare pentru cei care se pregtesc pentru olimpiade, sa a ral, pentru o activitate matematic susinut. a t a

Soluia problemei. Avem de considerat cteva cazuri. t Primul (i cel mai simplu) dintre ele este acela n care 0 este unul dintr s a1 , . . . , a2n+1 . ntr-adevar, 2n numere din cele 2n + 1 sunt nenule, deci m aparin mulimii {1, 2, . . . , 2n 1}; atunci (conform principiului cutiei) d t t module trebuie s fie egale, iar o asemenea egalitate furnizeaz dou nume a a a suma 0. Aceste doua numere i 0 sunt, desigur, cele trei numere cautat s suma 0). Considerm acum c 0 nu este printre cele 2n + 1 numere, pe care le n a a (2n 1) a1 < < ai < 0 < ai+1 < < a2n+1 2n 1.

Se poate uor observa ca exista cel puin doua numere negative i cel p s t s numere pozitive.
1

Profesor, Colegiul Na ional "Gheorghe Roca Codreanu", Brlad t s

Al doilea caz este acela n care |ai | 6= ai+1 ; s presupunem, de ex a |ai | < ai+1 . Avem i s 1 ai + ai+1 < < ai + a2n+1 2n 1

1 ai1 < < a1 2n 1, adic din nou dou grupe coninnd un total de 2n numere din mulimea {1, a a t t n fiecare grup numerele ind distincte; prin urmare exist un j {i + 1, . . a a i un k {1, . . . , i 1} astfel nct ai + aj = ak ai + aj + ak = 0, c s de demonstrat (evident, i, j, k sunt diferite doua cte doua). Cazul |ai | > absolut analog, suntem deci siguri c nu va reprezenta o problem pentr a a interesat. Cazul al treilea este i cel mai greu. Ne aflam acum n situaia n care s t i observm (aa cum cititorul trebuie s fi observat deja) c demonstra s a s a a a a a sus nu mai este valabil pentru c ai + ai+1 = 0 face s creasc numrul "c a a care trebuie aezate obiectele (facnd principiul inaplicabil!). Din fericire, s nu este insurmontabil. Observam mai nti ca acum avem, de fapt, i, pentru a1 > (2n 1), s 1 ai + ai+2 < < ai + a2n+1 2n 2 1 ai1 < < a1 2n 2;

astfel ca demonstraia din al doilea caz se poate utiliza i acum. Similar t s cazul a2n+1 < 2n 1, deci putem presupune n continuare ca a1 = ( a2n+1 = 2n 1. Si iar considerm dou posibiliti: i n sau i n + 1. a a at Dac i n avem numerele ai+1 , . . . , a2n i 2n 1 ai+1 , . . . , 2n 1 a s sunt toate din mulimea {1, . . . , 2n 1} i sunt n numar de 2(2n i) 2 t s ca exista j, k {i + 1, . . . , 2n} aa nct aj = 2n 1 ak . Avem j 6= k s 2n 1 este impar i egalitatea se mai scrie a1 + aj + ak = 0. s n cazul i n + 1 procedm la fel cu numerele a2 , . . . , ai i 2n 1 a s 2n 1 + ai , ceea ce ncheie demonstraia. t Observaii. 1) Cititorul atent trebuie s fi observat deja c 0, 1, . t a a sunt 2n numere din intervalul [(2n 1), 2n 1] printre care nu exis suma 0; sau, analog, se pot alege 0, 1, . . . , (2n 1) cu aceeai propri s 2n + 1 este cel mai mic numar k astfel nct oricum am alege k numere din {(2n 1), . . . , 2n 1} exist printre ele trei cu suma 0. a 2) O ntrebare se pune n mod natural n legtur cu aceast probl a a a observaia anterioara: care sunt toate posibilitaile de a alege 2n numere din t t {(2n 1), . . . , 2n 1} astfel nct printre ele s nu existe trei a cror s a a 0? Lsm n seama cititorului rezolvarea acestei probleme. aa Bibliografie

1. P. Erds - Problema E736, The American Mathematical Monthly, 53(194 2. L. Moser - Soluia problemei E736, The American Mathematical Monthly t 229-230.

Tehnici de stabilire a unor inegaliti geomet at


I. M. MAFTEI 1 , Mihai HAIVAS 2

Fie a, b, c lungimile laturilor unui triunghi T . n diverse moduri pute la T la un triunghi derivat T 0 , lungimile laturilor acestuia din urma fiin n a, b, c (rezultate ntr-un mod indicat). Dac aplicm triunghiului T 0 a a geometrice valabile n orice triunghi (de exemplu, R 2r, 3 3r p 3 a2 + b2 + c2 4 3S, 1 < cos A + cos B + cos C etc.), vom obine, c t 2 ansa, noi inegalitai interesante. Ne propunem sa ilustram mai jos acest s t Cteva triunghiuri derivate ale lui T sunt date de urmtoarea a

Lem. Daca a, b, c sunt lungimile laturilor unui triunghi, atunci a 1) pentru orice x 0, numerele l1 = ax + b, l2 = bx + c, l3 = cx + asemenea, lungimile laturilor unui triunghi; 2) pentru orice [0, 1], numerele L1 = a , L2 = b , L3 = c sunt laturilor unui triunghi; c) pentru orice x 0 si [0, 1], (ax + b) , (bx + c) si (cx + a) sun laturilor unui trinughi. Demonstraie. 1) S artm, de exemplu, c l1 + l2 > l3 . Avem t a aa a (bx + c) > (cx + a) (a + b c) x + (b + c a) > 0, ceea ce are loc x 2) Considernd c = max {a, b, c}, avem de aratat ca L1 + L2 > L3 , a a b t + este descresct a b > c . Funcia f : [0, 1] R, f () = c a+b c f () f (1), [0, 1]. Dar f (1) = > 1. Deci f () > 1, [0 c a + b > c , [0, 1]. 3) Rezulta combinnd punctele precedente.

Propoziie. Fie a, b, c lungimile laturilor unui triunghi. Sa se ara t avem urmatoarele inegalitati:

Demonstraie. 1) Fie T1 triunghiul cu lungimile laturilor l1 = ax+b, t i l3 = cx + a (x 0). Consideram subnelese notaiile: p1 , r1 , R1 , S1 rel s t t
1 2

(a+bc) x + b+ca (b+ca) x + c+ab (c+ab) x + a+bc p + p + p (ax + b) (bx + c) (bx + c) (cx + a) (cx + a) (ax + b) (b + c) x + a + c (c + a) x + b + a (a + b) x + c + b + + cx + a ax + b bx + c 2 ax + b + bx + c + cx + a ,

(a + b + c)3 (x + 1)3 27 [(ab) (1x) + c (1+x)] [(bc) (1x) + a (1 [(c a) (1 x) + b (1 + x)] ,

Profesor, Colegiul Na ional "Sf. Sava", Bucureti t s Cercettor, Academia Romn, Inst. Cerc. Economice "Gh. Zane", Iai a a s

Inegalitatea (1) se obine aplicnd triunghiului T1 inegalitatea p 3 t 1 1 adevar, p1 = (l1 + l2 + l3 ) = (a + b + c) (x + 1), iar, cu formula lui He 2 2 arie, 2 S1 (a + b + c) (x + 1) 2 r1 = 2 = 2 2 p1 4 (a + b + c) (x + 1) [(b + c a) x + c + a b] [(c + a b) x + a + b c] [(a + b c) x + b +

[(a b) (1 x) + c (1 + x)] [(b c) (1 x) + a (1 + x)] [(c a) (1 x) + 4 (a + b + c) (x + 1) 2 t Introducnd n p2 27r1 , vom obine inegalitatea (1). 1 2) Consideram triunghiul T2 avnd lungimile laturilor u = ax + b, v = 1 w = cx + a, x 0 (Lema, punctul 3) cu = ). Atunci 2 v 2 + w2 u2 (b + c a) x + c + a b p , cos A2 = = 2vw 2 (bx + c) (cx + a) (c + a b) x + a + b c (a + b c) x + b + c p p cos B2 = , cos C2 = 2 (cx + a) (ax + b) 2 (ax + b) (bx + c Cum cos A2 + cos B2 + cos C2 3, obinem imediat (2). t 3) Pentru medianele mu , mv , mw ale triunghiului T2 avem: 2 v 2 + w2 u2 (2b + 2c a) x + 2c + 2a b 2 mu = = , 4 4 (2c + 2a b) x + 2a + 2b c (2a + 2b c) x + 2b + 2 , m2 = m2 = v w 4 4 P m2 3 a (a + Tinnd seama de acestea i utiliznd cunoscuta inegaliate s a 4 triunghiul T2 , obinem (3). t Observaie. Avem egalitate n (1), (2), (3) doar pentru triunghiul ec t Cazuri particulare. Daca luam x = 0 sau x = 1 n (1), (2), (3) inegalitile (mereu n ipoteza c a, b, c sunt lungimile laturilor unui triun at a (a + b + c) 27 (a b + c) (b c + a) (c a + b) ,
3

(a + b + c)3 27abc, a (b + c a) + b (c + a b) + c (a + b c) 3 abc, 3p (a + b) (b + c) (c + a), a b+c+b c+a+c a+b 2 b+c c+a a+b + + 2 a+ b+ c , a c b c b 1 a + a+b+ b+c+ c+a . + 2 c+a b+c a+b Observaie. Se pot obine cu acest procedeu i alte inegaliti interesa t t s at ram cititorului sa aplice procedeul triunghiului cu laturile ax2 + 2 bx + cx + a , cx2 + ax + b , x 0 i [0, 1]. s

Asupra unei probleme de concurs


Angela TIGAERU 1

n nota de faa ne propunem determinarea unei condiii suficiente care t t convergena integralelor de forma t Z f (x) dx, a > 0. x a Vom demonstra c uniforma mrginire a expresiei xf (x) este sucient p a a a vergena acestui tip de integrale, extinznd astfel rezultatul cuprins n P t cl. a XII-a, data la O.N.M. din anul 2000, propusa de Mihai Piticari i S s dulescu, condiia folosit acolo ind doar existena i nitudinea limitei t a t s

(problema citat, cu rezolvarea autorilor, poate fi consultat n [1]). a a Pentru nceput, vom prezenta cteva definiii i rezultate de baza. Fie f t s R o funcie integrabila Riemann pe orice interval compact inclus n [a, ) t

Definiie. Spunem ca funcia f este integrabila pe intervalul [a, ) d t t x R i este finita limita lim f (t) dt. Daca f este integrabla pe [a, ), vom s
x a

f (t) dt = lim

f (t) dt.

Daca condiia (1) este ndeplinita, se mai folosete formularea: integrala t s

este covergenta. O condiie necesar, nu i sucient, ndeplinit de o fu t a s a a grabila pe intervalul [a, ) este ca lim f (x) = 0.
x

Un rezultat teoretic fundamental, pe care l folosim n demonstraie es t

Exist o literatur vast care are ca subiect determinarea unor criter a a a de convergena a integralelor de tipul (1) sau de calcul efectiv, cititorul in t mai multe detalii putnd consulta [3].

Criteriul lui Cauchy. Funcia f : [a, ) R este integrabila pe [a, t numai daca, pentru orice > 0, exista numarul real A () > 0, astfel nc orice x > A () si pentru orice y > 0, este valabila relaia t Z x+y f (t) dt < .
x

Rezultatul notei de faa este t Propoziia 1. Se considera funcia f : [a, ) R, a > 0, integrabi t t compact inclus n [a, ). Daca exista M > 0 astfel nct atunci
1

R a

|xf (x)| M,

x [a, ) ,

f (x) x dx

este convergenta.

Profesor, Colegiul Na ional "Petru Rare", Suceava t s

Urmatorul exemplu poate forma o imagine asupra ariei de aplicabilita tatelor de mai sus. a R {xn } dx = 1c, unde {x} reprezi Exemplul 1. Sa se arate ca lim n n 1 xn fracionar a lui x, a > 1 i c este constanta lui Euler. t a s {x} Rezolvare. Considerm funcia f : [1, ) R, f (x) = a t , car x a a R {xn } R {tx } dx = lim x dt = condiiile din propoziia 2, deci lim n t t n 1 xn x 1 tx Pentru calculul ultimei integrale procedm astfel: a n k+1 Z Z n1 k+1 n1 Z X Z {x} X {x} {x} x dx = lim dx = lim dx = lim 2 2 2 n n n x x x x2
1 1 k=1 k k=1 k

O consecina a rezultatului demonstrat mai sus este t Propoziia 2. Se considera funcia f : [1, ) R, care satisface t t Propoziiei 1. Atunci, pentru orice a > 1, este valabila relaia t t Z a Z f (t) lim x f (tx ) dt = dt. x t 1 1 t Demonstraie. Cu schimbarea de variabil u = tx se obine egalitate t a Z a Z ax 1 f (u) x x f (tx ) dt = u du. u 1 1 Avem succesiv: Z x Z x a Z ax Zx 1 a f (u) 1 a 1 ux 1 f (u) |uf (u)| du M u x 2 u x du du 1 u u u2 1 1 1 x 1x 1 + ax 1 ; =M a 1x x a f (u) ax f ( R 1 R u x du trecnd la limit i innd cont de a > 1, deducem lim as t x 1 u 1 ax f (u) ax f (u) R R R f (t) 1 u x du = lim du = dt, q.e.d. deci lim x 1 x 1 u u t 1

M M y y < 1 i deoarece x > A () = s , rezult c a a Cum x + y x x+y x+y f (t) R R f (t) dt < , adic, urmare a criteriului Cauchy, integrala a dt x t t a vergenta.

Demonstraie. n adevar, pentru un > 0, consideram A () = t x > A (), atunci, pentru orice y > 0, avem Z x+y Z x+y Z x+y 1 M y f (t) 1 |tf (t)| 2 dt M dt = dt t t t2 x x+y
x x x

n1 k+1 X 1 k k+1 1 = lim + ln x = lim k + (ln (k + 1) n n x k k+1 k k k=1 k=1 n1 X 1 1 1 = lim + (ln (k + 1) ln k) = lim 1 1 + + ln n n k+1 2 n k=1 a R {xn } dx = 1 c. deci lim n n 1 xn

n1 X

Propoziia 1 st i la baza urmtorului rezultat t as a Propoziia 3. Se considera funcia f : [a, ) R, a > 0, continua t t R F (x) care satisface condiia (3). Atunci t dx este convergenta, unde F : [ x a este o primitiva a funciei f si, mai mult, este adevarata relaia t t Z Z F (a) F (x) f (x) dt = + dx. 2 x a x
a a

F (x) [2], se deduce c lim a = 0, demonstraia nefiind imediat, deoarec t a x x (3) nu asigura aplicarea directa a lemei lui lHospital. Trecnd la limita (), se obine (5) i demonstraia se ncheie. t s t |sin x| Exemplul 2. Se considera f : [0, ) R, f (x) = , prelu x R F( continuitate n x = 0. Dac F este o primitiv a lui f , atunci a a x a R F (x) F (a) R |sin x| dt = dx. convergenta pentru orice a > 0 i s + x2 a x2 a a

Demonstraie. Pentru x > a avem t Z x Z x Z x 0 Z x F (x) F (a) F (t) x f (t) F (t) F (t) F dt = dt = dt = + + 2 t t t a t x a a a a a Tot din condiia (3), se obine ca lim f (x) = 0, de unde, conform L t t
x

ncheiem, propunndu-i cititorului exerciiul urmator: t Sa se arate ca Za 32 52 (2n 1)2 1 d (xn ) lim n dx = lim ln 0, , n n xn 42n n! (n 1)! 4
1

unde d (x) = inf {x n | n N}. Bibliografie

1. Gazeta Matematica, 7-8/2000, p.265 i p.275. s 2. D. T. Onofrei - Asupra comportarii la limita a unor primitive, Recreaii m t II(2000), nr. 2, 28-29. 3. A. Precupanu - Bazele analizei matematice, Editura Polirom, Iai, 1998 s

O clas de inegaliti a at
Mihai DICU, Lucian TUTESCU 1
Propoziia 1. Fie numerele k, m N . t 1a) Daca x, y, z R, atunci are loc inegalitatea

1c) Daca x, y, z R+ , atunci are loc inegalitatea x + y 2m+2k + z 2m+2k xk y k z k (x2mk1 yz + y 2mk1 xz + z 2m
2m+2k

x2m+2k+1 + y 2m+2k+1 + z 2m+2k+1 xk y k z k (x2mk+1 + y 2mk+1 + z 2m

1b) Daca x, y, z R+ , atunci are loc inegalitatea

x2m+2k + y 2m+2k + z 2m+2k xk y k z k (x2mk + y 2mk + z 2mk )

Demonstraie. 1a) Avem t (z

(x2m y 2m )(x2k y 2k ) 0,
2m

i apoi s

2m

)(z

2k

(y 2m z 2m )(y 2k z 2k ) 0, x2k ) 0

x2m+2k + y 2m+2k x2k y 2m + x2m y 2k z 2m+2k + x2m+2k x2k z 2m + x2m z 2k y 2m+2k + z 2m+2k y 2k z 2m + y 2m z 2k

Adunnd (1), (2), (3), punem sub forma

2(x2m+2k+y 2m+2k+z 2m+2k ) (x2my 2k+x2mz 2k )+(x2ky 2m+y 2mz 2k )+(x2kz 2 2 x2m y k z k + 2 xk y 2m z k + 2 xk y k z 2m 2xk y k z k (x2mk + y 2mk +

dup aplicarea inegalitii mediilor pentru fiecare parantez n parte. a at a 1b) La fel ca la punctul a) pentru x, y, z 0 se pleac de la inegaliti a at

Dup adunare, grupm sub forma a a

(x2m+1 y 2m+1 )(x2k y 2k ) 0, (y 2m+1 z 2m+1 )(y 2k z 2k ) 0 (z 2m+1 x2m+1 )(z 2k x2k ) 0. 2(x2m+2k+1 + y 2m+2k+1 + z 2m+2k+1 )

Dup aplicarea inegalitii mediilor, pentru fiecare parantez, se gse a at a a s tatea 1b). 1c) Pentru x, y, z 0 se folosesc inegalitaile: t

(x2m+1 y 2k + x2m+1 z 2k ) + (x2k y 2m+1 + y 2m+1 z 2k ) + (x2k z 2m+1 + y 2k

i se parcurg aceleai etape ca la punctul a) i b), grupnd corespunztor. s s s a Observaia 1. Se vede uor c fiecare dintre inegalitile (1a), (1b) t s a at mai tare dect inegalitatea mediilor.
1

(x2m1 y 2m1 )(x2k+1 y 2k+1 ) 0, (y 2m1 z 2m1 )(y 2k+1 z 2k+1 ) (z 2m1 x2m1 )(z 2k+1 x2k+1 ) 0

Profesori, C.N. "Fra ii Buzeti" Craiova t s

Cazuri particulare 1a1 ) Daca luam k N i notam 2mk = n N, atunci cu k +n par i s s avem x3k+n + y 3k+n + z 3k+n xk y k z k (xn + y n + z n ). 1a2 ) Dac n aceasta particularizm k = 1, se obine a a t xn+3 + y n+3 + z n+3 xyz(xn + y n + z n ),

cu n numar natural impar i x, y, z numere reale oarecare. s Concursul "Gheorghe Dumitre a a at 1a3 ) S remarcm cteva cazuri particulare ale ultimei inegaliti: 4 4 4 1a31 ) x + y + z xyz(x + y + z), x, y, z R; 1a32 ) x6 + y 6 + z 6 xyz(x3 + y 3 + z 3 ), x, y, z R. Concursul revistei "Arhim 1b1 ) Daca luam k N si notam 2m k + 1 = n N, atunci cu k + x, y, z R+ avem a a t 1b2 ) Dac n aceasta particularizm k = 1, se obine x3k+n + y 3k+n + z 3k+n xk y k z k (xn + y n + z n ). xn+3 + y n+3 + z n+3 xyz(xn + y n + z n ),

cu n numr natural par i x, y, z numere reale pozitive oarecare. a s Concursul "Gheorghe Dumitre 1b21 ) pentru n = 0 i x, y, z numere reale pozitive oarecare se obin s t adic inegalitatea mediilor pentru trei numere. a 1b22 ) pentru n = 2 i x, y, z numere reale pozitive, se obine s t x5 + y 5 + z 5 xyz(x2 + y 2 + z 2 ). x3 + y 3 + z 3 3xyz,

1c1 ) Daca luam k N i notam 2m k 1 = n N, atunci, cu k + s x, y, z R+ , avem x3k+n+1 + y 3k+n+1 + z 3k+n+1 xk y k z k (xn yz + y n xz + z n x Propoziia 2. Fie numerele k, m N . t 2a) Daca x, y, z R, atunci are loc inegalitatea

2b) Daca x, y, z R+ , atunci are loc inegalitatea +y 2m+2k+1 +z 2m+2k+1 xk y k z k x2mk+1 + y m z mk yz + y m x


2m+2k+1

x2m+2k + y 2m+2k + z 2m+2k xk y k z k (x2mk + y m z mk + z m y m

2c) Daca x, y, z R+ , atunci are loc inegalitatea r x 2m+2k 2m+2k 2m+2k k k k 2mk1 m mk x +y +z x y z x yz + y z + y mk z

Demonstraie. 2a) Adunm (1), (2), (3) i grupm termenii din mem t a s a sub forma

2(x2m+2k +y 2m+2k +z 2m+2k ) (x2my 2k +x2mz 2k )+(x2ky 2m +y 2kz 2m )+(x2kz 2m

Aplicnd inegalitatea mediilor pentru fiecare parantez, obinem: a t 2m k k 2m+2k 2m+2k 2m+2k x y z + 2 xk y m+k z m + 2 xk y m 2(x +y +z )2 2 x2m y k z k + 2 xk y m+k z m + 2 xk y m z m+k 2xk y k z k (x2mk + y m z mk + z m y mk ). 2b) Plecnd de la (1b), dupa adunare, grupam astfel: 2(x2m+2k+1 + y 2m+2k+1 + z 2m+2k+1 )

(x2m1y 2k+1+x2m1z 2k+1 )+(x2k+1y 2m1+y 2k+1z 2m1 )+(x2k+1z 2m1+y 2m r r x x 2m1 k k k m+k m k m m+k 2 x y z yz + x y z +x y z = z y r r x x + y mk z m . = 2xk y k z k x2mk1 yz + y m z mk z y Observaia 2. Se vede uor c fiecare dintre inegalitile 2a), 2b), 2c t s a at tare dect inegalitatea mediilor. Observaia 3. n cazul n care x, y, z R+ inegalitatea 1a) este mai t 2a) pentru m > k, respectiv, mai slab n caz contrar, pentru c a a x2mk +y 2mk +z 2mk x2mk +y m z mk +z m y mk (y m z m )(y mk z

(x2m+1 y 2k + x2m+1 z 2k ) + (x2k y 2m+1 + y 2k z 2m+1 ) + (x2k z 2m+1 + z 2k y 2(x2m+1 y k z k + xk y m+k z m yz + xk y m z m+k yz) = = 2xk y k z k (x2mk+1 + y m z mk yz + y mk z m yz). 2c) Folosind (1c), dupa adunare, termenii se grupeaza astfel: 2m+2k 2m+2k 2m+2k 2(x +y +z )

Cazuri particulare 2a1 ) Pentru m = 2k i x, y, z R, avem s 6k x + y 6k + z 6k xk y k z k (x3k + y 2k z k + z k y 2k ). 2a2 ) Pentru k = 2m i x, y, z R, avem s y m z m 6m 6m 6m 2m 2m 2m x +y +z x y z + 1+ . z y 2b1 ) Pentru m = 2k i x, y, z R+ , avem s x6k+1 + y 6k+1 + z 6k+1 xk y k z k (x3k+1 + y 2k z k yz + y k z k yz) 2b2 ) Pentru k = 2m i x, y, z R+ , avem s x6m+1 + y 6m+1 + z 6m+1 x2m y 2m z 2m (x + y m z m yz + y m z m 2c1 ) Pentru m = 2k i x, y, z R+ , avem s r r x x x6k + y 6k + z 6k xk y k z k x3k1 yz + y 2k z k + y k z 2k z y s 2c2 ) Pentru k = 2m i x, y, z R+ , avem r r x 6m 6m 6m 2m 2m 2m 1 m m x +y +z x y z yz + y z x + y m z m z

Inegaliti omogene i puin analiz... at s t a a


Titu ZVONARU 1

Vom stabili printr-o metoda unitara cteva inegalitai n trei variabile t ndeplinesc condiia a + b + c = 1. Sa remarcam ca, pentru inegalitai t t aceast condiie poate fi oricnd adugat, fr pierderea generalitii. a t a a aa at b c 3 a + + , a, b Exemplul 1 (Inegalitea lui Nesbitt). b+c c+a a+b 2 Soluie. Inegalitatea fiind omogena, putem presupune a + b + c = t x 9x 1 , x (0, 1) (inegalitate echivalenta cu (3x 1)2 0). Ca 1x 4 a b c a b c 9a 1 9b 1 9c + + = + + + + b+c c+a a+b 1a 1b 1c 4 4 (egalitile care apar sunt justificate de faptul c a + b + c = 1) i inegal at a s demonstrat. a

Aceasta soluie este de tip "iepuraul scos din joben" i cititorul se n t s s unde a aprut inegalitatea ajuttoare. Rspundem acestei ntrebri, ar a a a a a dat i ideea cluzitoare a notei noastre. as aa x Considerm funcia f : (0, 1) R dat prin f (x) = a t a . Tangenta 1x 1 t ei n punctul de abscisa are ecuaia 3 1 1 1 9x 1 yf = f0 x y= . 3 3 3 4 Deoarece funcia este convexa, pe tot domeniul sau de definiie, grac t t situat "deasupra" tangentei, fapt care se traduce prin inegalitatea uti 1 este, desigur, motivat de cazul d a sus. Alegerea punctului de abscis a 3 1 a=b=c= . 3 3 Exemplul 2 (Polonia, 1996 ). Pentru orice a, b, c cu a + b + c = 4 a b c 9 + + . a2 + 1 b2 + 1 c2 + 1 10 x ; tangenta la graficul funciei t Soluie. Fie f : R R, f (x) = 2 t x +1 36x + 3 1 are ecuaia y = t , iar acest punct este situat ntr-u de abscisa 3 50 x 36x + 3 de concavitate a funciei f . Inegalitatea 2 t (de altfel e x +1 50 3 cu (3x 1)2 (4x + 3) 0), este valabil pentru orice x i se po a s 4 numerelor a, b, c. Inegalitatea cerut rezult prin sumarea inegalitilor d a a at scrise pentru a, b, c i din faptul c a + b + c = 1: s a a b c 36a + 3 36b + 3 36c + 3 9 + 2 + 2 + + = 2+1 a b +1 c +1 50 50 50 10
1

Comneti, e-mail: tzvonaru@hotmail.com a s

(Pn la urm, metoda aceasta este o metod de spargere a inegalitilor a a a at

Exemplul 3 (USAMO, 2003 ). Pentru orice a, b, c pozitive are loc (2b + c + a)2 (2c + a + b)2 (2a + b + c)2 + 2 + 2 8. 2a2 + (b + c)2 2b + (c + a)2 2c + (a + b)2 Indicaie. Putem presupune fr a restrnge generalitatea c a + b t aa a 4x + 1 12x + 3 Are loc inegalitatea , x > 0, (care este echi 3x2 2x + 1 2 2 s (4x + 1)(3x 1) 0) i apoi o folosim ca n exemplele anterioare.

Exemplul 4 (OIM, 1995 si C:1952 din GM 7-8/1997 ). Pentru a, b, c a2 b2 c2 a+b+c + + . b+c c+a a+b 2 x2 Indicaie. Inegalitatea care trebuie folosita n acest exemplu este t 1x x (0, 1).

Exemplul 5 (E:10888 din GM 2/1995 ). Fie a, b, c lungimile latu triunghi. Atunci b c a + + 3. b+ca c+ab a+bc 1 x Indicaie. Are loc inegalitatea t 9x 2, pentru orice x 0, 1 2x 2 Exemplul 6 (Japonia, 1997 ). Pentru orice a, b, c > 0 are loc inegalita (c + a b)2 (a + b c)2 3 (b + c a)2 + 2 + 2 . 2 + (b + c)2 2 a b + (c + a) c + (a + b)2 5 54x + 27 1 , x > 0. Indicaie. Se va utiliza t 2 2x + 1 2x 25 a b c 3 Exemplul 7. + + , a, b, c R ma+b+c a+mb+c a+b+mc m+2 (o generalizare a problemei C:1079, GM 1/1991 ). 9x + m 1 x ! ncercai t Indicaie. Folosii inegalitatea t t 1 + (m 1)x (m + 2)2 stituiile ma + b + c = u, a + mb + c = v, a + b + mc = w ! t

Exemplul 8 (USAMO, SummerProgram, 2002 ). Pentru orice a, b, c > 2 2 2 3 3 2b 2c 2a 3 + + 3. b+c c+a a+b 2x 2 3 3x (3x 1)2 (3x 4) 0 pentru orice Indicaie. Avem t 1x Propunem cititorului urmtoarele exerciii: a t

Exerciiul 1 (Gigel Buth i Liviu Vlaicu, RMT 2/1998 ). Pentru a, t s n m > 0 are loc a b c 3 + + . ma + nb + nc mb + nc + na mc + na + nb m + 2n

Exerciiul 2 (Ungaria, 1996 ). Pentru a si b mai mari ca 1 cu a + b t b2 1 a2 + . a+1 b+1 3

2n 1 si a1 , . . . , an 2 a n2 n 1 a1 + + an = 1. Sa se demonstreze inegalitatea a1 an n2 + + 2 2 . a2 + an + n +1 1 Exerciiul 3. Fie n N, n 2, > t

Exerciiul 4 (India,1995 ). Pentru orice a1 , . . . , an pozitive si cu suma t inegalitatea a1 an n + + . 1 a1 1 an n1 Exerciiul 5. Pentru a, b, c > 0 cu a + b + c = 1 avem t b c 27 a + + . a3 + 1 b3 + 1 c3 + 1 28 Exerciiul 6 (Sefket Arslanagic, 2787, Crux Mathematicorum). Fie t astfel nct a + b + c = 1. Atunci 1 1 1 27 . c+a 2 a+b 2 + b+c 2 + 8 1 2 1 2 1 2

Exerciiul 7 (Sefket Arslanagic, 2739, Crux Mathematicorum). Pe t a, b, c > 0 are loc inegalitatea 9+3 a+b+c+ a a+b+c+ b a+b+c+ c + + b+c c+a a+b 2 a+b

Exerciiul 8. Pentru a, b, c > 0 si avnd suma 1, este adevarata ineg t 2 2 2 1 1 100 1 + b+ + c+ . a+ a b c 9 Exerciiul 9 (Sefket Arslanagic, 2738, Crux Mathematicorum). Fie t astfel nct a2 + b2 + c2 = 1. Atunci avem b c 3 3 a + + . 1 a2 1 b2 1 c2 2

Exerciiul 10 (Panos E. Tsaoussoglou, 2946, Crux Mathematicoru t a, b, c > 0 si a2 + b2 + c2 = 1. Avem 1 1 1 1 1 1 s + + a b c 2 3 i + + +a+b+c4 3 a b c a b c Exerciiul 11. Pentru a, b, c 1 cu a + b + c = 1 are loc t 1 1 1 27 + + . 1 + a2 1 + b2 1 + c2 10

Despre numerele reale algebrice


Silviu BOGA1

n cele ce urmeaz, cadrul de studiu este inelul polinoamelor cu coeficien a n care s-au definit conceptul de divizibilitate, cel mai mare divizor com polinoame, numr real algebric i polinom minimal asociat unui numr rea a s a Rezultata imediat din algoritmul lui Euclid i deosebit de utila n raiona s t vor urma este proprietatea: f, g Q[X] u, v Q[X] nct u f + v g = Propoziia 1. Daca doua polinoame f, g Q[X] au o radacina comu t atunci (f ; g) 6= 1. Demonstraie. Presupunnd prin absurd c (f ; g) = 1, conform p t a anterior enunat, u, v Q[X] nct u f + v g = 1. n acest caz, cu t a g() = 0, vom obine ca (u f + v g) () = 1, deci 0 = 1, contradicie. t t Propoziia 2. Daca doua polinoame f, g Q[X] au o radacina comu t si h = (f ; g), atunci h() = 0. Demonstraie. Fie f = f h, g = g h i atunci, cum (f ; g ) = 1, u t s +v g = 1 i astfel (u f +v g )h = h uf +v g = h (uf + nct u f s h() h() = 0. Propoziia 3. Fie f Q[X], f = X p + a1 X p1 + a2 X p2 + + ap t pentru care (i) f este de grad impar; (ii) f este ireductibil n Q[X]; (iii) f are o radacina R de semn contrar cu termenul liber ap 6= 0 (iv) a2 + a2 + + a2 6= 0. 1 2 p1 n condiiile (i) (iv), pentru orice n N , n este numar real iraional t t Demonstraie. Vom presupune, fr a restrnge generalitatea, c term t aa a este pozitiv. Din cele menionate, evident ca R \ Q. n acest caz, da t un anume n N ar avea loc n = a Q, polinomul g = X n a Q o rdcin comun cu polinomul f nu va fi prim cu f i, cum f este ired a a a a s . . f . Dar rdcinile x , x , . . . , x C ale polinomului g au |x rezulta g . a a 1 2 n 1 p . . f , ar rezulta ca i radacinile polinomu n = |xn | = |a| i astfel, din g . s s p s t toate de acelai modul, implicit || = p |ap | i, din condiia (iii), = s aceast situaie, din exprimarea f = (X p + ap )+X a1 X p2 + a2 X p3 + a t se va deduce ca polinoamele v = X p + ap i w = a1 X p2 + a2 X p3 + s radacina comuna . Dar w = a1 X p2 + a2 X p3 + + ap1 este neconstan cu (i) i (iv) i, datorit rdcinii comune, v i w nu ar fi prime ntre ele, s s a a a s face ca f s nu fie ireductibil n Q[X], n contradicie cu (ii). Rmne c a t a a 3 Observaie. Chestiuni de genul: Dac x R verific x + 2x + 2 t a a arate c x2006 R \ Q, imediat justificate de Propoziia 3, apar n mai mu a t printre subiectele de bacalaureat, sesiunile 2006 i 2007 (a se vedea [1]). s Urmnd pas cu pas demonstraia Propoziiei 3 (adaptrile necesare sunt t t a vom obine imediat t
1

Profesor, Liceul "V. Alecsandri", Iai s

Propoziia 4. Fie f Q[X], f = X p + a1 X p1 + a2 X p2 + + ap t pentru care (i) f este de grad par; (ii) f este ireductibil n Q[X]; (iii) f are o radacina R si ap < 0; (iv) a2 + a2 + + a2 6= 0. 1 2 p1 t n condiiile (i) (iv), pentru orice n N , n este numar real iraional t Observaie. Analog cu prima observaie, prin Propoziia 4 se dem t t t imediat afirmaii de genul: Dac x R verific x4 + x3 + x2 + x = 2007, t a a ca xn R \ Q, n N . Propoziia 5. Daca R este un numar real algebric si exista n t . . p, unde p = grad f , f fiind polinomul minimal aso n Q atunci n . (adic polinomul din Q[X] de grad minim i care admite ca rdcin ). a s a a a Demonstraie. Fie n = a Q; atunci g = X n a are rdcin com t a a a deci (g; f ) = h 6= 1. Dar f fiind polinom minimal, este ireductibil i a s . de unde n . grad f . .

polinomul minimal al lui . Cum n Q, conform propoziiei anterio t totodat , 2 , . . . , p1 R\Q, deoarece n caz contrar f nu ar mai fi min a n acest caz n p i analog cu raionamentul din propoziiile anterioare | s t t deci ||p = |ap | Q. Astfel p Q i considernd g = a1 X p1 + a2 X p s ap1 X + (ap + p ), deducem ca g() = f () = 0 i cum f este minim s g = 0, prin urmare f = X p p . Observaie. Conform ultimei propoziii, orice rdcin real iraional t t a a a a t polinom f Q[X] ireductibil i care nu este de forma f = X p a, are pr s n R \ Q, n N . Concluzie. Oricare ar fi R\Q radacina a unui polinom ireductibil f = X p + a1 X p1 + a2 X p2 + + ap1 X + ap , are loc exact una din si a) a2 + a2 + + a2 6= 0 i n acest caz n R \ Q, n N ; s 1 2 p1 b) a2 + a2 + + a2 = 0 i n acest caz n R \ Q n N este s 1 2 p1 prin p. Bibliografie

Propoziia 6. Daca R este un numar real algebric si exista n t n Q, atunci polinomul minimal asociat lui este de forma f = X p p = min {n | n Q}. Demonstraie. n cazul Q se observ c f = X . n cazul t a a evident {n | n Q} 6= i fie f = X p + a1 X p1 + a2 X p2 + + ap s

1. http://www.subiecte2007.edu.ro bacalaureat, subiecte M11, variantele 4 2. C. Nstsescu, C. Nia - Teoria calitativa a ecuaiilor algebrice, Ed a a t t Bucureti, 1982. s

Asupra unei note din revista "Recreaii matem t


Maria BATINETU-GIURGIU 1 , D. M. BATINETU-GIURGIU 2

xn+1 xn+1 n n = lim n n + 1 xn xn xn Daca, nsa, (xn )n1 are proprietatea ca exista lim = t R , nu + n n n exist lim (xn+1 xn ), dup cum arat irul xn = n + (1) , n N . a a as
n

n "Recreaii Matematice", an. V, nr. 2/2003 este publicat o Not m t a a interesant [5], pentru care eleva Oana Crj a primit premiul Fundaiei a a t pe anul 2003. Ne propunem sa ntarim i sa extindem rezultatele acestei N s Fie (xn )n1 un ir de numere reale strict pozitive astfel nct lim (xn+ s n = x R . Atunci, exist i limitele: as + lim xn xn+1 xn = lim = x i s n (n + 1) n x
n

lim

Teorema 1. Fie sirul de numere reale strict pozitive (xn )n1 a n xn xn+1 lim si = x R+ . Atunci, lim (xn1 xn ) lim n x n n x n n xn+1 lim (xn+1 xn ) = x ln lim . n n xn xn+1 Demonstraia I (D. M. Btineu-Giurgiu). Notnd un = t a t xn avem xn un 1 ln (un )n , n N . xn+1 xn = xn (un 1) = n ln un un 1 s = 1. Dac y = lim (xn+1 xn ), a Dar lim un = 1 i, deci, lim n n ln un n n n limit n (2), obinem: y = xln lim (un ) , de unde lim (un ) = ey/x . a t n n n xn+1 dac z = lim a , utiliznd din nou (2), obinem: lim (xn+ t n n xn x ln z.

Demonstraia a II-a (M. Tena). Se procedeaza la fel, n locul r t folosind formula n " xn # xn (xn+1 xn ) xn+1 xn+1 xn xn+1 xn n = 1+ , n N xn xn

Fie irurile de numere reale strict pozitive (an )n1 , (bn )n1 , (xn )n1 s an+1 bn+1 an cu proprietile: lim at = lim = a R , lim = b R + + n an n bn n bn xn yn+1 lim = x R+ , lim = y R+ . Vom numi sir Lalescu definit d n n n n
1 2

Prof., dr., Academia Tehnic Militar, Bucureti a a s Prof., Colegiul Na ional "Matei Basarab", Bucureti t s

i ponderat cu (an )n1 , (bn )n1 , (yn )n1 irul (Ln )n2 dat de s s p Ln = xn+1 n+1 an+1 yn+1 xn n bn yn , n 2.

Teorema 2. Sirul Lalescu (Ln )n2 definit prin (4) este convergent dac n xn+1 daca sirul este convergent. xn n1 xn+1 n+1 an+1 yn+1 Demonstraie. Notnd vn = t , n 2, vom avea: n xn bn yn p xn p vn 1 n bn yn ln (vn )n , n 2. Ln = xn n bn yn (vn 1) = n ln vn n ipotezele noastre, rezult c lim n an = lim n bn = a. Cum lim a a n n n avem i lim n yn = 1. Ca urmare, lim vn = 1. Din faptul ca s n n n xn+1 an+1 an yn+1 1 , n 2, (vn )n = xn an bn yn n+1 an+1 n+1 yn+1 n xn+1 n . rezulta ca lim (vn ) = b lim n n xn n xn+1 Daca z = lim , trecnd la limita n (5), obinem lim Ln = t n n xn Reciproc, dac c = lim Ln , din (5) rezult c avem c = ax ln lim (vn a a a n n n 1 c xn+1 c n lim (vn ) = e ax i, n final, lim = e ax . s n n xn b

Aplicaii ale Teoremei 1. 1) Luam xn = n n!, n 2 .Cum lim t n n 1 (n + 1)! n+1 xn+1 p p i lim = lim n+1 = = lim s n+1 n n n xn n! (n + 1)! (n + 1)! p 1 conform Teoremei 1, ca lim n+1 (n + 1)! n n! = (limita sirului n e Lalescu). 2) Lund xn = n n n, n 2, avem ! n xn 1 (n + 1)n+1 xn+1 = lim n+1 lim = 1 i s lim n n n n xn nn+1 n+1 Deci lim (n + 1) n+1 n + 1 n n n = 1 (limita sirului lui Romeo T. Ian n p 3) Daca xn = n (2n 1)!!, n 2, atunci, dupa calcule de rutina, avem n p p xn+1 2 i lim = e. Ca urmare, lim n+1 (2n + 1)!! n (2n s n n e xn (D.M. Batineu-Giurgiu, C:905, G.M.-5/1989). t p 4) Fie xn = n d1 d3 d2n1 , n 2, unde dn R i lim (dn+1 + s n R+ . Atunci

- 2-3/1992).

Ca urmare, lim (xn+1 xn ) =


n

d1 d3 d2n1 d1 d2n+1 nn = lim n+1 d d n n (n + 1) n 1 2n n 2d n d2n+1 2n + 1 = lim = , n 2n + 1 n + 1 n+1 e n d1 d3 d2n+1 1 xn+1 p = lim = lim n n d1 d3 d2n1 n+1 d d d xn 1 3 2n+1 e n+1 d2n+1 p = lim = 2d = e. n n + 1 n+1 d d d 2d 1 3 2n+1 xn lim = lim n n n
n

2d 2d ln e = (D.M. Batineu-Giurgiu, C:1 t e e

n 3) Pentru n N , sa consideram xn = n, yn = 1, an = bn = n xn+ an+1 bn+1 an xn lim = lim = e, lim = 1, lim = 1, lim n an n bn n bn n n n xn ! 2 2 n (n + 1) p i, deci, obinem lim = e (D.M. Batineu-Giurg t s t n n+1 n (n + 1)! n! G.M.-4/1989). Bibliografie 1. 2. 3. 4.

Aplicaii ale Teoremei 2. 1) Date cn R , n 1, sa notam t + a s cn+1 ! = cn+1 cn !, n 1. n Teorema 2 considerm yn = 1, n 1 i x x n 2, unde lim (cn+1 cn ) = c R . Ca mai sus, avem lim + n n n n p xn+1 = e. n consecina, lim n+1 cn+1 !an+1 n cn !bn = t lim n n xn (Teorema 3 din [6]). 2) Fie cn R , n 1, astfel nct lim (cn+1 cn ) = c R . + + n t a rnd xn = yn = cn , n 1, obinem c lim xn+1 n+1 an+1 xn+1 xn
n

= ac ln (be) (Teorema 4 din [6]).

D. M. Btineu - Siruri, Editura Albatros, 1979. a t D. M. Btineu-Giurgiu - Siruri Lalescu, R.M.T., 1-2/1989, 33-36. a t D. M. Btineu-Giurgiu - Ponderarea unor siruri, G.M., 2-3/1992, 46a t D. M. Btineu-Giurgiu - Asupra calcularii unor limite de siruri, RecM a t 22-24. 5. O. Crj - Un procedeu de calcul al limitelor unor sirui de forma (an+1 a RecMat-2/2003, 23-24. 6. A. Stroe - Asupra unei clase de siruri, G.M., seria A, 3/2007, 217-227. 7. M. Tena - O alta soluie a Problemei 579 din G.M., Revista "Licriri" t a "N. Blcescu" din Craiova, 1978, 13-14. a

Limita unor iruri de numere reale s


Gheorghe COSTOVICI 1

Prezentm mai jos o generalizare direct a Propoziiei din [1] i cteva a a t s


n

primitiva a lui f pe (0, ) si numerele p, q N , p < q. Daca lim [F (qn)


n

Propoziie. Fie f : (0, ) R continua, descrescatoare si cu lim f (x t

= l R, atunci avem si lim [f (pn + 1) + f (pn + 2) + + f (qn)] = l.


n

Demonstraie. Sa consideram irul (xn )n1 dat de xn = f (1) + f ( t s +f (n) F (n). Procednd ca n [1], aratam ca (xn )n1 este descrescator s ntr-adevar, cu teorema de medie a lui Lagrange, avem xn+1 xn = f (n + 1)[F (n + 1) F (n)] = f (n + 1)F 0 (cn ) = f (n + 1) (unde n < cn < n + 1). Deci, xn+1 xn , n 1. Pe de alta parte, cu aceeai teorema de medie, avem F (k + 1) F (k) s f (k) (k < ck < k + 1) i, sumnd pentru k = 1, n, obinem s t F (n + 1)F (1) f (1)+f (2)+ +f (n) , n 2 xn+1 f (n + 1)F ( De aici i din faptul c irul (f (n))n1 este mrginit (consecina a condiiei s as a t t

= 0), rezulta ca (xn )n1 este marginit inferior. Aadar, irul (xn )n1 este c s s Se constat uor c a s a f (pn + 1) + f (pn + 2) + + f (qn) = xqn xpn + [F (qn) F (pn)] , care, pentru n , conduce la (1). Observaie. Propoziia din [1] se obine pentru p = 1 i q = 2. t t t s

Calculul limitelor urmatoare devine simplu prin aplicarea acestei Propoz n seama cititorului vericarea condiiilor de aplicare). t 1 1 1 q 1 Exemple. 1) lim + + + = ln (f (x) = n pn + 1 pn + 2 qn p x (qp)n P 1 1 = 0 (f (x) = ). 2) lim n k=1 1 + (pn + k)2 1 + x2 (qp)n P 1 = . 3) lim n k=1 pn + k + 1 (qp)n P q 4) arctg (pn + k) = ln ( f (x) = arctg x). p k=1 (qp)n P 1 1 q = 0 (f (x) = ). 5) 2 x x2 + 1 k=1 (pn + k) (pn + k) + 1 Bibliografie

1. C. Chiser - O metoda eleganta de calcul al unor limite de siruri, R.M. 9-10.


1

Conf. dr., Catedra de matematic, Univ. Tehnic "Gh. Asachi" a a

O rafinare a inegalitii dintre media aritme at i cea logaritmic s a


Mihail BENCZE 1
Fie 0 < a < b. Vom nota cu a+b ba i A (a, b) = s L (a, b) = 2 ln b ln a media aritmetica i, respectiv, media logaritmica. Se tie [1] ca s s L (a, b) < A (a, b) . n aceast not vom da o rafinare a acestei inegaliti. a a at

Teorem. Daca 0 < a < b atunci avem inegalitatile a a + b h i2 a, b < A (a, b) . L (a, b) < L , ab < A 2 Demonstraie. Inegalitatea din dreapta rezult imediat: t a h i2 a + b 2 2 a+b a, b a b A (a, b) > A > 2 2 h i2 a + b Pe de alt parte, A a a, b = A s , ab i aplicnd (2 2 a + b a + b A a , ab > L , ab , adic inegalitatea din mijloc. 2 2 Mai rmne de demonstrat inegalitatea din stnga, care, cu notaia t a t se scrie t+1 t1 t+1 t1 2 t t+1 t+1 > t+ t ln t > > ln t ln t 2 ln 2 ln t ln 2 ln t

deci f 0 (x) > f 0 (1) = 0 i f (x) > f (1) = 0 i astfel am demonstrat in s s a + b t+1 t ln t > t + 1 ln , adic L (a, b) < L a , ab . 2 2 Bibliografie

Pentru a demonstra aceasta noua inegalitate folosim notaiile t = x2 t x2 + 1 2x ln x (x + 1) ln . Atunci 2 x2 + 1 2 (x 1) 2 (x 1)2 (x + 1 i f 00 (x) = f 0 (x) = 2 ln x 2 s 2 x +1 x (x2 + 1)2

1. F. Burk - The Geometric, Logaritmic and Aritmetic Mean Inequality, Am Monthly 94(1987), 527-528. 2. M. Bencze - New Means, new Inequalities and Refinement, Octogon Ma Magazine, Vol. 9, Nr. 1/2001, 46-105.
1

Profesor, Braov s

Estimri de sume a
Tudor PADURARIU 1

n ultimii ani, la diverse concursuri, naionale sau internaionale, au fo t t probleme n care se cere s se gseasc sau s se demonstreze o anumit a a a a a a pentru o suma (numerica, vectoriala etc). Metodele de rezolvare a unor probleme sunt variate i presupun raionamente care folosesc reducerea s t principiul cutiei, principiul extremal, inducia matematic . a. t as Prezentm n continuare cteva exemple; cititorul interesat poate exe a cile dobndite rezolvnd problemele propuse n nalul notei, precum i s probleme din articolul lui Gabriel Carroll - Estimating Sums, care poa pe internet la adresa [1] i care a constituit punctul de plecare al demersu s x2 n

(I.M.O., 1987) Soluie. Folosind inegalitatea Cauchy-Schwarz, obinem ca |x1 | + |x t t p |xn | n(x2 + + x2 ) = n. n total exist k n sume de tipul e1 x1 + a n 1 cu 0 xi k 1, iar toate aceste sume se afl n interiorul unui interval d a (k 1) n. Aceste interval poate fi acoperit cu k n 1 subintervale d (k 1) n (concluzia problemei sugereaza acest lucru!). Din principiul cu kn 1 dou sume care se gsesc n acelai subinterval, iar diferena lor satisface a a s t n+1 , Problema 2. Fie x1 , x2 ,. . . , xn numere reale cu |xi | 2 iar |x1 + x2 + + xn | = 1. Demonstrai ca exista o permutare y1 , y2 t n+1 numerelor x1 , x2 ,. . . , xn astfel nct |y1 + 2y2 + + nyn | . (I.M 2 Soluie. Presupunem prin absurd c pentru orice permutare y1 , y2 t a n+1 numerelor x1 , x2 ,. . . , xn am avea |y1 + 2y2 + + nyn | > . Ob 2 |(x1 + + nxn ) + (xn + 2xn1 + + nx1 )| = (n + 1) |x1 + x2 + + x i, daca numerele S1 = x1 + 2x2 + + nxn i S2 = xn + 2xn1 + + n s s acelai semn, modulul sumei ar fi egal cu suma modulelor, contradicie. R s t S1 i S2 au semne contrare; s zicem c S1 > 0, S2 < 0. a a s Putem realiza trecerea de la S1 la S2 schimbnd n mod repetat nt doua numere consecutive xk i xk+1 . La un anumit pas, de exemplu c s de la suma S3 = z1 + + jzj + (j + 1) zj+1 + + nzn la suma S4 = jzj+1 + (j + 1) zj + + nzn , semnul sumei S se va schimba: S3 > 0, i n+1 n+1 n+1 n Cum |S3 | > , |S4 | > , vom avea ca S3 > , iar S4 < 2 2 2 S3 S4 > n + 1. Pe de alta parte, S3 S4 = j (zj zj+1 ) + (j + 1) (zj+
1

Problema 1. Fie x1 , x2 ,. . . , xn numere reale cu proprietatea ca x2 + 1 = 1. Demonstrai ca pentru orice k N, k 2, exista ntregii e1 , e2 ,. t ( toi nuli, cu |ei | < k 1, i = 1, k, pentru care |e1 x1 + + en xn | t

Elev, Colegiul Na ional "Gr. Moisil", Oneti t s

n+1 zj+1 zj i S3 S4 = |S3 S4 | = |zj+1 zj | |zj+1 | + |zj | 2 s 2 Contradicia la care am ajuns arat c presupunerea iniial este fals. t a a t a a

Problema 3. Fie n 2 un numar ntreg. Coloram fiecare dintr numere ntregi consecutive n rou sau albastru. Demonstrai ca exista o s s t monocolora {a1 , a2 , . . . , an } astfel nct 1 a2 a1 a3 a2 a (Concursul "Al. Myller", 2007) n3 + 5n Soluie. Putem considera ca numerele sunt 1, 2,. . . , t , culorile s 6 n (n 1) i fie dn = + 1. Vom demonstra prin inducie c pentru orice n t a s 2 submulimea monocolor {a1 , a2 , . . . , an } pentru care 1 a2 a1 a t a dn . Proprietatea este imediat pentru n = 2; o presupunem adevrat p a a a

sa o verificam pentru n + 1. Conform ipotezei inductive, printre primel

numere exista submulimea monocolora {a1 , a2 , . . . , an }, colorata cu A, a t 1 a2 a1 an an1 dn . Considerm numerele an + dn , an + a an + dn + n, n numr de n + 1. Dac toate au culoarea R, proprietatea este a a pentru n+1. Daca unul dintre ele, fie acesta an+1 , are culoarea A, atunci a 3 n3 + 5n (n + 1) + 5 (n + 1) dn + n = dn+1 , iar an+1 + dn + n = , deci 6 6 {a1 , . . . , an , an+1 } ndeplinete condiia dorita. s t

Problema 4. Fiind date n numere reale, demonstrai ca exista ce t

perechi (ai , aj ) cu proprietatea ca 1 < |ai aj | < 2. Soluie. Cerina problemei sugereaza metoda de rezolvare: un graf cu t t n2 ale crui muchii nu formeaz triunghiuri, are cel mult a a muchii (rez 4 Mantel, caz particular al teoremei lui Turan). Construim un graf astfel: vr numerele date, iar dou vrfuri ai , aj se unesc printr-o muchie cnd 1 < | a 2. Dac, prin absurd, ar exista trei vrfuri unite prin muchii, am avea a 1 < |ai aj | < 2, 1 < |aj ak | < 2 i 1 < |ai ak | < 2. Putem pre s ai > aj > ak i relaiile precedente devin 1 + aj < ai < 2 + aj i 1 + ak < a s t s de unde 1 + aj < 2 + ak , adic aj ak < 1, imposibil. Astfel, am dem a graful nu are triunghiuri i aplicarea rezultatului menionat iniial ncheie s t t ncheiem prin a propune spre rezolvare celor interesai cteva problem t Problema 5. Fie O un punct pe o dreapta d, iar OP1 , OP2 ,. . . , O cu extremitatile ntr-un acelai semiplan determinat de d. Daca n este im s arate ca OP1 + OP2 + + OPn 1 (I.M.O., 1973). Bibliografie 1. http://web.mit.edu/rwbarton/public/mop 2. http://www.mathlinks.ro

Problema 6. ntr-o secvena de numere reale, suma oricaror 7 ter t secutivi este negativa, iar suma oricaror 11 termeni consecutivi este pozit determine numarul maxim de termeni ai unei asemenea secvene. (I.M.O t

Acurateea limbajului matematic n combinat t


Laureniu MODAN 1 t

Combinatorica, ramura distincta a Matematicii, a aparut ca o consecin crii de rezolvare a problemelor de numrare. Mai puin analitice dect n a a t pline matematice, raionamentele sale, fcute aproape n mod primordial, t a de aceea ce astazi se numete metalogica. s Cum aratam i n [2], o gndire matematica riguroasa i educata ar s s nceap cu nvarea Combinatoricii enumerative i s continue cu Pro a at s a combinatoriale, strict legate de cotidian, aa cum actualmente se ntm s numar important de naiuni cultivate tiinic. Nu trebuie sa uitam ca as t s t binatorica, prin noile sale ramuri: teoria grafurilor, teoria matroizilor, teori etc., este cel mai dinamic domeniu al matematicii, avnd numrul cel mai a de conjecturi enunate i rezolvate anual. t s n Romnia zilelor noastre, Matematica discreta, prin urmare i Comb s sunt efectiv neglijate! Iar educaia precara a tinerilor, n acest domeniu, t doar la mnuirea unor simple relaii algebrice, deseori numite "formule", f t ei s poat discerne cnd trebuie s foloseasc permutari, aranjamente sau a a a a simple sau cu repetiie, respectiv cnd trebuie sa foloseasca numarul fu t acioneaza de la o mulime de cardinal n, la o alta de cardinal m. t t Vom reaminti (v. [3]) c, pentru mulimea M = {x1 , x2 , . . . , xn } de ca a t i) permutarile sale difer unele de altele doar prin ordinea elementelor a numar de n!; ii) aranjamentele grupelor de cte m elemente difera prin ordinea i tipu s i sunt Am ; s n iii) combinarile ca numr al submulimilor cu m elemente difer doar a t a n m obiectelor i sunt s = Cn . m Numarul funciilor f : M {y1 , y2 , . . . , ym }, egal cu mn , a fost detali t Menionm c este regretabil faptul c tinerii elevi i studeni nu sunt t a a a s t neleaga rolul esenial al cuvintelor i exprimarilor folosite n Combinatori t t s constatm cu surprindere, c n culegerea Exerciii si probleme de algebra [1 a a t n numeroase reeditri, timp de peste un sfert de veac, apar erori perpet a elementele de Combinatoric. n acest sens, la Problema 25 din cap. X, a aceeai eroare ca i n ediia din 1981, tiparita de Editura Didactica i Pe s s t s (1) Pentru un joc, 3 baiei si 5 fete trebuie sa formeze doua echipe de t cte moduri se pot forma echipele? Dar daca n fiecare echipa trebuie sa fi un singur baiat? Observm c alctuirea celor dou echipe, de cte 4 persoane, revine a a a a submulimilor de cte 4 elemente, prin folosirea combinarilor. Prima ec t
1

Prof. dr., Departamentul de matematic, Facultatea de informatic, A.S.E., Bucur a a

4 forma n C8 moduri, n timp ce a doua se realizeaz din cele 4 persoane a 4 4 4 C4 moduri. Prin urmare, alcatuirea celor doua echipe se face n C8 C4 = 7 Vom constata apoi, ca partea a doua a problemei este fara sens! ntr-ad n fiecare echip s-ar afla un singur biat, atunci pentru prima ar trebui a a biat i 3 fete, iar pentru a doua ar mai trebui s existe o fat n plus. a s a a ntr-un limbaj combinatoric corect, partea a doua a problemei (1) ar formulata n maniera urmatoare: (2) n cte moduri se pot alcatui cele doua echipe de cte 4 persoan fiecare intra cel puin un baiat? t Pentru rezolvare, prima echip, constituit dintr-un biat i 3 fete, poa a a a s 1 3 2 n C3 C5 = 3C5 = 30 moduri, n timp ce a doua echipa, formata din 2 2 2 fete, se alege n C2 C2 = 1 mod. Cumulnd situaiile anterioare, decidem c t de 4 persoane, ce conin cel puin un biat fiecare, se constituie n 30 1 = 3 t t a Problema anterioar admite urmtoarea generalizare: a a (3) n cte moduri 2n 1 baiei si 2n + 1 fete pot forma doua echipe t persoane? Dar daca n fiecare din cele doua grupe trebuie sa intre cel puin t Lasam cititorului placerea de a constata ca, n cazul general, cele do (4n)! se constituie n moduri, n timp ce pentru situaia suplimentar, t a [(2n)!]2 formeaza n 4n3 n + 1 moduri. Cnd n = 2, regasim situaia discutata t punct de plecare al prezentei note. Pentru exersarea noiunilor din Combinatoric, propunem cititorului t a rezolvarea urmatoarei probleme: (4) Fie A = 3n | n = 0, 33 i B submulimea numerelor din A divizi s t i) n cte moduri se pot scrie pe 17 cartoane de culori diferite element ii) Cte numere de 4 cifre se pot forma cu elementele de cte 2 cifre, con iii) Cte numere de 4 cifre, ordonate crescator, se pot forma cu elem cte doua cifre, coninute n B? t Ca mijloc de verificare pentru cititor, dm rspunsurile la (4), ce au ca a a plecare mulimea B = {0, 6, 12, 18, 24, 30, 36, 42, 48, 54, 60, 66, 72, 78, 84, 90, t i) 17! moduri de scriere; ii) 15+210 = 225 numere de 4 cifre, formate cu elementele de cte doua c iii) 105 numere de 4 cifre, ordonate cresctor i formate cu elemente a s dou cifre, din B. a

Bibliografie

1. M. Brandiburu, D. Joia, C. Nstsescu, I. Nia - Exerciii si pr t a a t t Algebra, Ed. Rotech Pro, 2004, Bucureti. s 2. L. Modan - Some Remarks Counting Functions, Octogon Mathematica Braov, 15(2007), no.1, 262-265. s 3. E. Rogai - Tabele si formule matematice, Ed. Tehnic, 1984, Bucureti. a s

Asupra unei identiti clasice privind partea n at


Florin POPOVICI 1

Prezentm cteva consideraiuni de ordin metodic asupra unei identit a t privind partea ntreag a numerelor reale, pentru care dm dou demostr a a a cunoscute precum i o a treia, simpla i eleganta, despre care credem ca e s s Problema 1 ([1], p. 20). S se arate c a a h i [x] x = , x R, n N . n n

Soluie. Metoda I ([2], p. 122). Conform teoremei mparirii cu rest n t t hxi q, r Z, unice, astfel nct [x] = nq + r, 0 r n 1. Urmeaza ca n Rezult c a a [x] = q. n x [x] + {x} r + {x} r + {x} n1+1 i 0 Deoarece = =q+ s < = 1 re n n n n n hxi = q. n Din (2) i (3) rezulta ca are loc (1). s Metoda II ([1], pb. 11, p. 21 i 206). Evident, avem s h i x [x] . n n

Din deniia parii ntregi a unui numar real rezulta ca [x] + [y] [x + t t R. Prin inducie deducem c n [x] [nx], x R , n N . Renotnd v t a [x] h x i a a obinem t , x R, n N . Urmeaz c n n h i [x] x . n n Din (4) i (5) rezult (1). s a

a t a Metoda i Fie n N . Considerm funcia f : R Z denit pr h III. [x] x , x R. Avem n n hxi [x + n] x+n [x] + n x+n [x] f (x+n) = = = +1 n n n n n n

deci funcia f este periodic de perioad n. Pentru orice x [0, n] avem 0 t a a x i 0 < 1. Urmeaza ca f (x) = 0, x [0, n]. Rezulta ca f (x) = 0, x s n are loc (1).
1

Prof. dr., Colegiul Na ional "Gr. Moisil", Braov t s

Observaia 1. Metoda III este inspirat de una dintre metodele clasice t a demonstrarea celebrei identitai a lui Hermite: t 1 n1 [x] + x + + + x + = [nx] , x R, n N , n n n

Bibliografie 1. L. Niculescu, I. Ptracu, A. Seclman, M. Gleanu - Exerciii s a s a a at t de matematica. Clasa a IX-a, Editura Cardinal, Craiova, 2004. 2. I. M. Vinogradov - Bazele teoriei numerelor, Editura Academiei, Bucur

Profesorul (ctre elevul de la tabl): Preul unui produs se maret a a t s dupa care se micoreaza cu 10%. Compara preul iniial cu cel final! s t t Elevul: xy + 10% 10% = xy. Profesorul: Bine, greeala cu 10% din "nu se tie ce" e veche, dar de s s Elevul: Pi, nu ai spus produs? a t (Bogdan

Scaderea de mai jos scrisa cu cifre romane este corecta:

Rotii pagina cu 180 si vei constata ca relaia ramne adevarata (veric t t t gasii si o alta scadere de acest fel cu aceeai proprietate! s t (Titu Rspuns. Deoarece pot folosite doar numerele a I, II, III, IX, X, XI, XIX, XX, XXX, prin ncercri se obin toate scderile de acest fel: a t a X IX = I XI X = I XI IX = II.

XI X = I.

O abordare analitic a unor probleme de geom a


Gabriel POPA1 , Ioan SERDEAN 2

Cu prilejul elaborrii lucrrii [1], am constatat c o serie de probleme a a a trie propuse juniorilor la O.B.M.J. admit rezolvri analitico-trigonometric a simple dect cele "ociale". ntruct o parte dintre elevii din cl. a IX-a eligibili pentru lotul juniorilor, iar elevii buni i pasionai de matematic s t materia n avans, considerm util prezentarea n aceast manier a cto a a a a ale unor probleme care, abordate sintetic (vezi [1]), sunt dicile.

Problema 1. Fie ABCD un trapez cu AB k CD, AB > CD s b t m(B) = 90 . Sa se arate ca distana dintre mijloacele laturilor paralele es semidiferena bazelor. t (Problema 132, Lista scurt O.B.M a Soluie. Raportm planul la un reper cartezian t a cu originea n A, ca n gura. Fie D0 , C 0 proieciile t \ punctelor D, respectiv C pe AB; notm t = m(DAB), a \ a = AD0 , b = D0 C 0 , c = C 0 B. Avem ca m(CBA) = 90 t i atunci DD0 = AD0 tg t = a tg t, iar CC 0 = s c c C 0 B tg (90 t) = . Deci, a tg t = , prin urmare tg t tg t c = a tg2 t. Vrfurile trapezului vor avea coordonatele A (0, 0); B a 1 + tg2 t + b, 0 ; C (a + b, a tg D (a, a tg t), iar mijloace t); a 1 + tg2 t + b 2a + [AB] i [CD] au coordonatele M s , 0 , respectiv N 2 2 Lungimea segmentului M N este s s 2 2 2 2 a tg t 1 a2 tg2 t + 1 a tg2 t + + a2 tg2 t = = MN = 4 4 2 2 a tg t + 1 AB CD a+c Pe de alta parte, = = , de unde concluzia. 2 2 2 Problema 2. Fie ABCD patrat, E mijlocul lui [CD], iar M un pun \ = m(M BC) = m(BM E) = x. Sa se afle \ \ patratului astfel nct m(M AB) (Problema 203, Baraj O.B.M Soluie. Raportm planul la un reper cu originea t a n A, ca n gura; consideram unitatea egala cu latura patratului i atunci A (0, 0); B (1, 0); C (1, 1); D (0, 1); s E (1/2, 1). Notm m = tg x, m (0, 1) (1, ); a panta dreptei AM este m, iar panta dreptei BM este 1 1 tg (90 + x) = = . Astfel, AM : y = mx i s tg x m 1 BM : y = (x 1), iar prin intersectarea celor doua m
1 2

Profesor, Colegiul Na ional, Iai t s Profesor, Liceul Teoretic "Aurel Vlaicu", Ortie as

drepte obinem coordonatele lui M : xM = t M E este

yE yM 2m2 2m + 2 = , prin urmare xE xM m2 1 mBM mME 2m3 m2 + 2m 1 2m 1 = \ tg BM E = 1 + mBM mME m3 2m2 + m 2 = m 2 2m 1 = m, cu solu \ Cum tg BM E = tg x = m, obinem ecuaia t t m2 1, 2 3, 2 + 3 . Am vazut ca m 6= 1 i atunci ramne ca m 2 3 s \ \ adic x {15 , 75 }. Prima soluie nu convine: dac m(M AB) = m(M B a t a \ atunci BM E este unghi obtuz. n concluzie, x = 75 .

1 m , yM = . Pan 1 + m2 1 + m2

sin2 t 1 , carora le corespund punctele E, re sin2 t + 1 2 sin t 1 2 sin t 1 2 sin t Cum y2 = , avem P , . (x2 + 1) = sin t sin2 t + 1 sin2 t + 1 sin2 t + 1 Daca R este mijlocul segmentului [M N ], atunci R (0, sin t); scriem imed dreptei RF : y = (1 x) sin t. Coordonatele (x2 , y2 ) ale punctului P verifi ecuaie i de aici rezult concluzia problemei. t s a Ca urmare, x1 = 1 i x2 = s Problema 4. Un semicerc avnd diametrul [EF ] inclus n latura [BC] a triunghiului ABC este tangent laturilor AB si AC n Q, repsectiv P . Notam {K} = EP F Q. Sa se arate ca AK este nalime t n triunghiul ABC. (Problema 15, O.B.M.J., 2000) Soluie. Raportm planul la un reper cartezian t a cu originea n O mijlocul segmentului [EF ], avnd

Problema 3. Se considera triunghiul ABC cu AB = AC. Un se diametru [EF ], cu E, F [BC], este tangent laturilor AB si AC n M N , iar AE retaie semicercul n P . Sa se arate ca dreapta P F trece pri coardei [M N ]. (Problema 94, Lista scurta O.B.M Soluie. Raportam planul la un reper cartezian cu t originea n mijlocul O al segmentului [BC], avnd dreapta BC drept ax a absciselor i nlimea din A drept ax a s at a a ordonatelor. Consideram ca F (1, 0), E (1, 0), C (b, 0), \ B (b, 0) i fie t = m(CON ); atunci N (cos t, sin t), s [ M ( cos t, sin t). Cum m(ACO) = 90 t, avem c a 1 AO [ = tg(ACO) = ctg t, deci AO = b ctg t = , cci a OC sin t b cos t = ON = 1 (din triunghiul dreptunghic ON C) i s 1 1 astfel A 0, . Ecuaia dreptei AE va fi y = t (x + 1) i, intersect s sin t sin t 2 2 t t dreapt cu cercul x + y = 1, obinem ecuaia n x: a 2 2 2 1 + sin t x + 2x + 1 sin t = 0 (x + 1) 1 + sin2 t x sin2 t

pe BC ca ax Ox i perpendiculara n O pe BC ca ax Oy. Considerm c a s a a E (1, 0), P (cos a, sin a), Q (cos b, sin b). Panta lui OP este m = tg a i at s 1 lui AC va fi = ctg a; obinem ecuaia lui AC : x cos a + y sin a = 1 t t m AB : x cos b + y sin b = 1 i, intersectnd cele doua drepte, vom obine pent s t punctului A cos a+b 2 cos a+b sin ab sin a sin b 2 2 2 = . = xA = sin (a b) 2 sin ab cos ab cos ab 2 2 2 c a nlimea din A fiind paralel cu Oy, va avea ecuaia x = xA , adic x = at a t c Pentru a afla coordonatele punctului K, vom intersecta dreptele EP : x+1 y x1 y i F Q : = s = . Eliminnd pe y, gsim c a a cos a + 1 sin a cos b 1 sin b sin a cos b + cos a sin b sin a + sin b sin (a + b) (sin a sin xK = = sin b cos a sin a cos b + sin a + sin b sin (a b) + (sin a + si 2 cos a+b sin a+b sin ab cos a+b 2 2 2 2 = . = 2 cos ab sin a+b sin ab cos ab 2 2 2 2

Rezult astfel c punctul K aparine nlimii din A, de unde concluzia p a a t at

Rezolvnd problemele de geometrie din [1], am remarcat ca un procent tiv dintre ele (aproape 20%) admit soluii calculatorii, n maniera celor pre t aceast not. ncheiem prin a propune ca tem trei astfel de probleme. a a a

Problema 5. Fie ABC un triunghi echilateral de centru O, iar M K, L proieciile lui M pe AB, respectiv AC. Sa se arate ca OM trece pr t segmentului [KL]. (Problema 135, Lista scurt O.B.M a

Problema 6. Punctele M si N se gasesc pe laturile (AD) si (BC bului ABCD. Dreapta M C intersecteaza segmentul [BD] n T , iar dre intersecteaza [BD] n U . Dreapta CU intersecteaza dreapta AB n Q, i tersecteaza latura [CD] n P . Aratai ca triunghiurile QCP si M CN au ac t (Problema 232, Baraj O.B.M

Problema 7. Fie ABC un triunghi dreptunghic n C si punctele BD AE laturile [BC], respectiv [CA], astfel nct = = k. Dreptele BE AC CD \ intersecteaza n O. Sa se arate ca m(BOD) = 60 daca si numai daca k (Problema 246, Baraj O.B.M Bibliografie

1. D. Brnzei, D. Serbnescu, G. Popa, I. Serdean - 10 ani de Olim a canice ale Juniorilor, Paralela 45, Piteti, 2007. s

A study of a new geometric inequality


Chang Jian ZHAO 1

Abstract. In this paper, we find a new geometric inequality. Then we and strengthen the inequality and get several quite wider results.

1. Introduction. Recently, we have found a new inequality in a tria result can be stated as follows

Proposition. In the triangle ABC, point P is on side AC or its exte and Q is on the side AB or its extension line. Points D and E are both o BC and BD = DE = EC. Line AD intersects line P Q at F and AE line P Q at G. Then 1 4AF G 4AQP 3 and equality holds if and only if the line P Q and BC are parallel to each

In this paper, we shall generalize and strengthen this inequality. To si notation, let 4AF G denote the directed area of the triangle AF G, etc.

2. Extension of the inequality. Our main result is given by the theorem.

Theorem 2.1. In the triangle ABC, point Q is on the side AB or its line and P is on the side AC or its extension line. Points Di are on th and lines ADi intersect line QP at Ei , where i = 1, 2, . . . , n 1, and B D1 D2 = m2 , . . . , Dn2 Dn1 = mn1 , Dn1 C = mn . Then
n X i=1

mi

and equality holds if and only if QE1 = E1 E2 = = En2 En1 = En1 Proof. Let B = D0 , C = Dn , Q = E0 and P = En , then n X 4AEi1 Ei 4AQP = = 4AD0 D1 4AD0 D1 i=1 =
n n X mi 4AEi1 Ei X mi AEi1 AEi = m1 4ADi1 Di m1 ADi1 ADi i=1 i=1 v uQ n Q AEi1 AEi u n mi u n t i=1 ADi1 ADi n i=1 (2.2) mn 1

n1 X j=2

mj 4AQP n1 m1 mn nn 4AE1 En1

n1 Y i=2

4AEi1 Ei

Department of Mathematics, Binzhou Teachers College, Shandong 256604, China

Then

and equality holds if and only if all 4AEi1 Ei (i = 1, 2, . . . , n) are equa suppose that n Y AEi1 AEi M= . ADi1 ADi i=1 M=
n1 AE0 AEn AE1 AEn1 Y AEi1 AEi = AD0 ADn AD1 ADn1 i=2 ADi1 ADi

On the other hand,

n1 4AQP 4AE1 En1 Y 4AEi1 Ei . = 4ABC 4AD1 Dn1 i=2 4ADi1 Di

Qn1 n1 Y 4AEi1 Ei 4AEi1 Ei mn2 1 i=2 4AEi1 Ei = = Qn1 . mi n2 4ADi1 Di mi 4AD0 D1 4AD0 D1 i=2 i=2 i=2 m1 Therefore, by (2.3), (2.4), (2.5), and (2.6) we have
n1 Y

and

m1 4AE1 En1 4AE1 En1 = Pn1 , 4AD1 Dn1 4AD0 D1 j=2 mj

m1 4AQP 4AQP , = Pn 4ABC mi 4AD0 D1 i=1

Consequently,

Moreover, (2.2) and (2.7) yield that v u u m1 mn 4AQP 4AE1 En1 n1 Y 4AQP n n t Pn 4AEi1 Ei . Pn1 n 4AD0 D1 i=1 mi j=2 mj 4AD0 D1 i=2

M = Pn

i=1

mi

mn 1 Pn1
j=2

mj

Qn1
i=2

n1 4AQP 4AE1 En1 Y 4AEi1 E n 4AD0 D1 mi i=2

q.e.d.

n1 4AQP 4AE1 En1 Y 4AQP n m1 mn nn Pn 4AEi1 Pn1 n n 4AD0 D1 4AD0 D1 i=1 mi j=2 mj i=2

According to Theorem 1, we shall get the following Corollary 2.1. If the hypotheses of Theorem 1 are fulfilled, then PQ
n1 n X i=1 n1 X j=2 n1 Y i=2

mi

mj m1 mn n E1 En1

Ei1 Ei

and equality holds if and only if the QE1 = E1 E2 = = En2 En1 = E

Moreover, if we take m1 = m2 = = mn in Theorem 1, then we have

Corollary 2.2. Under the hypotheses of Theorem 1 and m1 = m2 = we have n1 Y (n 2) 4AQP n1 nn1 4AE1 En1 4AEi1 Ei
i=2

and equality holds if and only if the line P Q and BC are parallel to each Similarly, we can find easily also the following Corollary 2.3. If the hypotheses of Corollary 2.2 are fulfilled, then (n 2) P Qn1 nn1 E1 En1
n1 Y i=2

Ei1 Ei

and equality holds if and only if the line P Q and BC are parallel to each Corollary 2.4. Under the hypotheses of Theorem 1, if the point Q coincident, then
n X i=1

mi

n1 X j=2

mi 4ABP n1 m1 mn nn 4AE1 En1

n1 Y i=2

4AEi1 E

3. Strengthener of the inequality. Our first result is the following Theorem 3.1. If the hypotheses of Theorem 1 are fulfilled, then n1 X n n1 n1 n X X X 3 3 mi mj R2(n1) mi mj 4AQP n1 4 i=1 j=2 i=1 j=2 m1 mn nn 4AE1 En1
n1 Y i=2

4AEi1 Ei

and the first inequality in (3.1) becomes an equality if and only if AQP i triangle. 3 In fact, it is well known that 4 (abc)2/3 where a, b, c are the s 4 arbitrary triangle and 4 is the area. Hence the left side hand side of (3.1 proved.

Finally, we can again give a very well result. Theorem 3.2. Under the hypotheses of Theorem 1 and letting 4AEi ( i = 1, 2, . . . , n), then 4AQP where Hi = and X s X HS +
n2 X i=3

(Hi1 + Hi + Hi+1 ) Si ,

mi (mi1 + mi + mi+1 ) , 27mi1 mi+1

i = 2, 3, . . . , n 1,

HS = (H2 + 1) S1 + (Hn1 + 1) Sn + (H2 + H3 ) S2 + (Hn2 + Hn

and equality holds if and only if QE1 = E1 E2 = = En2 En1 = En1 Proof. If we take n = 3 in Theorem 2.1, then the inequality (2.1) redu following inequality s m2 (m1 + m2 + m3 ) 4AE1 E2 4AE0 E3 , 27m1 m3 4AE1 E2 H2 (4AE0 E1 + 4AE1 E2 + 4AE2 E3 ) ,

and equality holds if and only if the line P Q and BC are parallel to each

that is, S2 H2 (S1 + S2 + S3 ). Consequently, we have Si Hi (Si1 + S where i = 2, 3, . . . , n 1. Hence, by summation


n1 X i=2

Si

On the other hand,


n1 X i=2

n1 X i=2

Hi (Si1 + Si + Si+1 ) .

Hi (Si1 + Si + Si+1 ) =

Therefore, by (3.4) and (3.5), we get the inequality (3.2), q.e.d.

n2 X i=3

(Hi1 + Hi + Hi+1 ) Si +

HS S1

Acknowledgment. I wish to express my gratitude to professor Li W his valuable help in writing this paper. References

1. D. S. Mitrinovic - Analytic inequalities, Springer-Verlag, 1970. 2. B. G. Pachpate - On Some New Inequality Similar to Hilberts Inequality Anal. Appl. 226(1998), 166-179. 3. Ch. J. Zhao - The further research of Yang Le inequality, Journal o Teachers College, 12(1996), 32-34. 4. L. Carlitz, F. Leuenberger - Problem E1454, Amer. Math. Monthly 177 and 68 (1961), 805-806. 5. P. Finsler and H. Hadwiger - Einige Relationen in Dreieck, Comment. M 10(1937/38), 316-326.

Concursul "Recreaii Matematice" t Ediia a V-a, Muncel (Iai), 31 august 200 t s


Clasa a V-a

1. Fie a, b N. S se arate c dac ultima cifr a numrului a2 + b2 est a a a a a ultima cifra a numarului (a + b)2 este tot 9. Reciproca este adevarata? 2. Sa se determine numerele naturale nenule n, a, b, c, d, e, f tiind c s 1 2 3 n = 2a 3b 5c 7d 11e 13f . 3. S se determine numerele naturale distincte m, n i p astfel nct s a s relaia 653 < 5m + 5n + 5p < 809. t

Clasa a VI-a

1. Fie A = 3m 5n , unde m, n N. Notm cu a, b, c numrul divizorilor a a A, 3A i, respectiv, 5A. Stiind c numerele a i b sunt direct proporional s a s t iar numerele b i c sunt invers proporionale cu 15 i 16, sa se determine n s t s 2. Sa se afle numerele prime p, q i r tiind ca numarul pq +pr este patr s s 3. Fie paralelogramul ABCD n care AB = 2AD, iar punctele M mijloacele laturilor [AB] i, respectiv, [CD]. Dac [EM ] [AB], A s a DE N B = {F }, s se arate c dreptele AN i DM se intersecteaz n a a s a greutate al triunghiului F BE.

Clasa a VII-a

mediatoarea segmentului [BC], D BC, E AB, [CF bisectoarea unghi F AB, iar {I} = CF DE, {G} = CE AI. S se arate c a a a) triunghiul DF G este echilateral; b) AS SB, unde {S} = BI EC; 1 c) d (S, AB) = AB. 4

1. S se rezolve n mulimea numerelor naturale ecuaia 6a 5b = 1. a t t \ 2. Aflai aria triunghiului ABC tiind c AC = 4 cm i m(ACB) = m t s a s 45 , unde punctul M este mijlocul laturii (AC). b b s 3. Fie triunghiul ABC cu m(A) = 105 i M (B) = 30 . Se cons

Vizitai pe Internet revista "Recreaii Matematice" la t t

http://www.recreatiimatematice.uv.ro

South Eastern European Mathematical Olym for University Students (SEEMOUS) Agros, Cyprus, 7-12 March, 2007
Problems

P (x) Prove that fa is a rational function of the form fa (x) = , where P Q (x) polynomials with integer coecients. P Conversely, if ak {0, 1, 2, . . . , 9} for all k N, and fa (x) = n= P (x) x (0, 1) is a rational function of the form fa (x) = , where P and Q Q (x) nomials with integer coecients, prove that the number a = 0, a1 a2 a3 . . . i
i

1. Given a (0, 1) Q let a = 0, a1 a2 a3 . . . be its decimal representatio X fa (x) = an xn , x (0, 1) .


n=1

matrix such that f (Ax) = f (x) for all x Rn . Prove that exists a posit m such that Am is the identity matrix In .

2. Let f (x) = max |xi | for x = (x1 , x2 , . . . , xn )T Rn and let A be

3. Let F be a eld and let P : F F F be a function such that x0 F the function P (x0 , y) is a polynomial in y and for every y0 F th P (x, y0 ) is a polynomial in x. It is true that P is necessarily a polynomial in x and y, when a) F = Q, the eld of rational numbers? b) F is a nite eld? Prove your claims.

integer part of y.) b) Prove that for every > 0 there exists > 0 such that for eve 0 < y < and every x R we have w (x, y) < . c) Prove that the function w : R [0, +) [0, +) dened in a) is c

4. For x R, y 0 and n Z denote by wn (x, y) [0, ) the angle with which the segment joining the point (n, 0) to the point (n + y, 0) is the point (x, 1) R2 . P a) Show that for every x R and y 0, the series wn (x, y) con n= P wn (x, y), show that w (x, y) ([y] + 1) . we now set w (x, y) =
n=

Solutions

= a1 x+a2 x2 + +an0 xn0 +xn0 +1 R(x)+xn0 +p+1R(x)+ +xn0 +(m1)p+

1. As the expansion of a is periodic, there is a n0 such that an = an+p f Now, for n = n0 + mp, we have a1 x + a2 x2 + + an xn + =

where R (x) = an0 +1 + an0 +2 x + + an0 +p xp1 . Hence this series is a s polynomial S (x) = a0 + a1 x + a2 x2 + + an xn = a0 + a1 x + a2 x2 + + a an innite geometric series with rst element xn0 +1 R (x) and a factor xp . xn0 +1 R (x) the above series is equal to S (x) + . So the conclusion easily f 1 xp 1 For the converse, evaluate the expression for x = . 10
T i

x = (0, 1, . . . , 0)T , . . . , x = (0, 0, . . . , 1)T we get max |aij | = 1 for j = 1, 2


i T

2. Let A = (aij ). Taking x = (1, 0, . . . , 0) we get max |ai1 | = 1. Simila

Take x = (ai1 , ai2 , . . . , ain ) ; note that f (x) 1. We have Ax = (y1 n P 2 where yi = aij 1.
i=1

Calculating S =

i=1 j=1

a2 equals to n. This means tha ij column of A there exists exactly one nonzero element ai(i) = 1, and a permutation of {1, 2, . . . , n}. Therefore we have An! = diag (b1 , b2 , . . diagonal matrix with |b1 | = |b2 | = = |bn | = 1. Finally, A2n! = In . S n. Hence S = n and each sum
j=1

n n P P

a2 by columns and by rows leads us to inequ ij


n P

3. a) If F = Q or any other countable eld, the assertion is false. Let (qn ) be an enumeration of F . For consider the function P : F F as follows: if y Q, so that y = qn for some n N, set n X P (x, y) = (x q1 ) (x q2 ) (x qk ) (qn q1 ) (qn q2 ) (qn
k=1

The two expression give the same values for x = qm , y = qn . It is clear that it cannot be a polynomial in x and y. For instance, if degree N in x then the coecient of xN +1 is zero, yet the above has coec values at y = qN +2 is nonzero number (qN +2 q1 ) (qN+2 q2 ) (qN+2 b) If F is a nite eld then thee answer is yes, because every functio F F is a polynomial. Namely a linear combination of polynomials o Q x ak Q y ak which take the value 1 for x = am , y = an an k6=m am ak k6=n an ak other cases.

similarly, for x Q, x = qm for some m N, set m X P (qm , y) = (qm q1 ) (qm q2 ) (qm qk ) (y q1 ) (y q2 ) (y


k=1

4. a) If y = 0 the result is clear. If 0 < y < 1 the angle are disjoint so th less than the angle with which R is seen from (x, 1), that is . This is geo clear, but can also be seen analytically by observing that Z nx+y Z ds wn (x, y) = arctan (n x + y)arctan (n x) = 1 + s2 nx 1

In general, y = [y] + r with 0 r < 1 and


n= X [y]1

wn (x, [y] + r) =

X X

wn+j (x, 1) +

= [y] +

For n in {A, A + 1, . . . , A 1, A} (a nite set), using the fact that Z nx+y ds lim wn (x, y) = lim = 0, y0+ y0+ nx 1 + s2 P it follows that there is a > 0 such that wn (x, y) < for every y with
nZ\S

b) Let x R be xed and let positive < 1 be given. Dene A to b positive integer such that the segment [A, A] is seen from (x, 1) by an ang and set S = Z\ {A, A + 1, . . . , A 1, A}. For 0 < y < 1 we have X wn (x, y) < .
nS

j=0 n= X

n=

wn+[y] (x, r)

n=

wn (x [y] , r) < ([y] + 1) .

Combining with (1), the required result follows. To show that the limit is uniform note that, by the rst part of b), X X X y0+ wn (x, y) wn ([x] + 1, y) wn (0, y) 0.
n[x]+1 n[x]+1 n=0

Also (for 0 < y < 1), again by the rst part of b), X X X y0+ wn (x, y) = wn (0, y) wn (0, y) 0.
n[x]1 n1 n=0

Hence 0 w (x, y) 2

n=0

and converges to 0 as y 0+. The conclusion now follows. c) Let x1 , x2 R and y1 , y2 (0, ). Z Z nx2 +y2 X nx1 +y1 ds ds |w (x1 , y1 ) w (x2 , y2 )| = n= nx1 1 + s2 1+s nx2 X Z nx2 ds X Z nx1 +y1 ds + 1 + s2 n= nx2 +y2 1 + s2 nx1 n=
X |x2 x1 | X |x2 x1 | + |t2 t1 | + 1 + s2 1 + u2 n n n= n=

wn (0, y) + w0 (0, y), which does not depend

where sn is between n x1 and n x2 , and un is between n x1 + y1 and n Since the two series are convergent, the result follows on R [0, +). the uniform convergence of part b) completes the proof.

Soluiile problemelor propuse n nr. 1/200 t


Clasele primare

P.124. Schimba locul unui singur beior pentru a obine o egalitate. ts t (Clasa I ) Mariana Nastasia, e Soluie. t

P.126. Pentru a fierbe un ou sunt necesare 4 minute. Mama vrea sa oua n trei trane. Cte minute sunt necesare? s (Clasa a II-a) Ionela Brgan, e a a Soluie. 4 min +4 min +4 min = 12 min. t

P.125. ntr-o clasa cu 24 elevi sunt 3 perechi de gemeni. La sedina t este prezent cte un singur parinte din fiecare familie. Ci parini p t t sedina? t (Clasa I ) Mihaela Glc, e a Soluie. La edina participa 24 3 = 21 parini. t s t t

P.127. Mircea are cu 35 timbre mai mult dect fratele sau, Marius. C diferena, daca Mircea ar mai primi 10 timbre, iar Marius ar da unui t timbre? (Clasa a II-a) Inst. Maria R Soluie. Diferena se marete atunci cnd descazutul se marete sau t t s s se micoreaz. Diferena devine 35 + 10 + 5 = 50. s a t

P.128. Cte numere de forma RM AT ndeplinesc condiia RAM = M t (Clasa a III-a) Drago Covrig, s Soluie. R = M = T (9 valori); A (10 valori); 9 10 = 90 (numere). t P.129. Scriei toate adunarile de forma t
MARI + ARI RI . I 7676 6789 + 789 89 9 7676

(Clasa a III-a)
7324 + 324 24 4 7676 7289 + 289 89 9 7676

Dana Brsan, e

Soluie. t

P.130. Daca a, b, c sunt cifre, cte egalitati de tipul a c = b : c se Justificai raspunsul. t (Clasa a III-a) Adina Voinescu, e

c = 1 a 1 = b : 1 a = b = 0, 1, . . . , 9 (10 ca c=2 a = 1 i b = 4, a = 2 i b = 8 (2 caz s s c=3 a = 1 i b = 9 s (1 caz 10 + 2 + 1 = 13 egalitai. t P.131. Verificai daca afirmaia "A se mparte exact la 5, unde A t t 2 (1 + 2 + 3 + + 1999) + 1999 + 1997" este adevarata sau falsa. (Clasa a IV-a) Prof. Nicolae Ivchescu as Soluie. A = 2000 + 2000 1999 + 3996. Deoarece 3996 nu se mparte t aceeai proprietate o are i A. Deci, afirmaia din enun este fals. s s t t a P.132. Mama Oanei a mplinit 17532 zile pe data de 1 ianuarie 2007. luna si zi a avut o vrsta de 3 ori mai mica? (Clasa a IV-a) nv. Geta Cre t Soluie. Din 17532 = 48 365 + 12 se deduce ca anul naterii este 20 t s = 1959, pe 1 ianuarie. Restul 12 justific c de la 1.01.1959 pn la 1.01 a a a 12 ani biseci. Din 17532 = 2 5844 i 5844 = 16 365 + 4, rezult c v t s a a ori mai mica a avut-o pe 1 ianuarie 1975. P.133. Doi elevi spun pe rnd cte un numar natural, cel puin egal t mult egal cu 7. Fiecare nou numar spus se aduna la celelalte. Sa se arate elev poate sa indice n aa fel numerele nct sa ajunga primul la suma 99 s (Clasa a IV-a) Prof. Petru Asa Soluie. Avem 99 = 8 12 + 3. Primul elev va spune, prima dat, n t a Al doilea elev trebuie sa spuna un numar n, 1 n 7. n continuare, data cnd i vine rndul, primul elev va spune un numar de forma 8 n fel primul elev va completa o sum de tipul 8 k + 3, k fiind numr natu a a Pentru k = 12 se obine suma 99. t Soluie. t

Clasa a V-a

Dupa nlocuire, gasim ca 2x + 19b = 94, deci b va fi o cifra para. ncerca b {2, 4, 6, 8} i obinem valoare naturala pentru x doar cnd b = 4. n s t ecuaia nu are soluie dac (a, b) 6= (5, 4), iar n cazul n care (a, b) = ( t t a soluia x = 9. t Observaie. ncercrile pot fi evitate observnd c 76 19b 94 t a a b = 4. S V.77. a se determine cte numere de trei cifre distincte abc au prop abc cba : 11 este patrat perfect. Otilia Neme, Ocna Mur s Soluie. Din ipoteza deducem ca 9 (a c) este patrat perfect i, cum t s cifre distincte, atunci a c {1, 4}. Dac a c = 1, cifrele a, c fiind nenul a c (a, c) {(9, 8) ; (8, 7) ; . . . ; (2, 1)}. n fiecare caz, exist cte 8 valor a a diferite de a i c i astfel gasim 64 de numere abc. Daca a c = 4, vom s s

V.76. Daca a, b, x sunt cifre n baza 10, sa se rezolve ecuaia cu ne t x: bxa + baa + xb + ab = abb + aab. Marius Fa Soluie. Efectund descompunerea n baza 10, dupa reduceri de termen t . egalitatea 20x + 190b = 188a. Atunci 188a . 10 i, cum a 6= 0, trebuie s av . s a

V.80. Daca restul mparirii unui numar natural la 10 este mai mar t spunem ca acel numar este favorabil. Aflai numerele favorabile ab cu pr t ca nici ab, nici ba nu pot fi scrise ca suma de doua numere favorabile. Ioan Scleanu a a Soluie. Numerele favorabile sunt cele care se temin n 6, 7, 8 sau 9. t a c un numr natural se scrie ca sum de dou numere favorabile dac el a a a a a n 2, 3, 4, 5, 6, 7 sau 8, iar aceast scriere nu este posibil atunci cnd el a a n 1 sau 9. Cum ab i ba sunt ambele favorabile, obinem ca a = b = 9. s t

perechi (a, c), deci 5 8 = 40 de numere abc. n total exist 64 + 40 = 104 a proprietaile dorite. t V.78. Aratai ca nu exista trei numere prime a, b, c astfel nct a (b + t Nicolae Ivchescu as Soluie. Sa presupunem prin absurd ca ar exista trei numere prime t etatea dat. Distingem trei cazuri: a i) b, c impare; atunci a (b + c) este par, bc este impar, imposibil. ii) b, c pare; atunci b = c = 2, deci a (2 + 2) = 2 2, adica a = 1, fals. iii) b par, c impar (sau invers); atunci b = 2, deci a (2 + c) = 2c, cu 2 Deducem c a este par, adic a = 2 i se ajunge la contradicia 2 + c = c. a a s t V.79. Aratai ca numarul 131000 91000 se divide cu 1000. t Damian Marinescu, T Soluie. Deoarece 134 = 28561, iar 94 = 6561, avem: t 250 4 250 250 250 13100 91000 = 134 9 = (M1000 + 561) (M1000 + 561)

Clasa a VI-a
VI.76. Determinai a, b, c Z daca t a b 3c + 5 = = . 3 4 2c + 1

Gheorghe Iu . b 3c + 5 Soluie. Cum a = 3 i a, b Z, atunci b . 4. Rezult c t s . a a 4 2c + 1 2c + 1 | 2 (3c + 5) 3 (2c + 1), adic 2c + 1 | 7. Obinem c c {4, 1 a t a soluiile problemei vor fi: (3, 4, 4), (6, 8, 1), (15, 20, 0), (6, 8, 3). t VI.77. Sa se arate ca ntre oricare doua puteri naturale consecutive a afla cel puin o putere a lui 2. Exista doua puteri consecutive ale lui 3 n t putem gasi trei puteri ale lui 2? Marius Damia Not. Anterior publicrii n revista noastr, problema a aprut a a a a 2/2006, cu numarul O.VI.146, semnata de acelai autor. Soluia poate s t R.M.T. 3/2006. VI. 78. Fie a, b Z astfel nct mulimile {a + b, a + 2b, . . . , a + t {1, 2, . . . , 2007} coincid. Sa se arate ca exista k N pentru care a + kb = Dan Nedeianu, Drobeta-Tr Soluie (Mdlina-Vasilica Solcanu). Din ipotez deducem c t a a a a a + b + a + 2b + + a + 2007b = 1 + 2 + + 2007, prin urmare 2007a + 2007 1004b = 2007 1004, deci a + 1004b = 1004. As k = 1004 N pentru care a + kb = k.

VI.79. Se considera 4ABC ascuitunghic, iar M un punct n planul t lela prin M la AB taie AC si BC n P , respectiv N . Demonstrai ca t dintre urmatoarele afirmaii sunt adevarate, atunci este adevarata si a tre t \ (i) BM bisectoare pentru ABC; (ii) M C M B; (iii) [N P ] linie m 4ABC. Carmen-Daniela Tama \ \ Soluie. (i) + (ii) (iii) Cum N BM M BA t \ BM N , rezult c 4N M B este isoscel cu N B = N M . a a \ \ Apoi, N M C N CM , ambele avnd acelai comples ment, deci 4N M C este isoscel cu N M = N C. Deducem c N este mijlocul lui [BC] i cum P N k AB, a s atunci [N P ] va fi linie mijlocie n 4ABC. (ii) + (iii) (i) Mediana [M N ] din 4M BC dreptunghic este egal cu [BN ], deci 4N BM este isosa \ N M B. nsa N M B M BA (alterne interne), pr \ \ cel, cu N BM \ \ \ ABM M BC. t t (iii) + (i) (ii) Ca la prima implicaie obinem ca BN = N M . Cu mijlocul lui [BC], [M N ] va fi median n 4M BC, egal cu jumtate din l a a a \ = 90 . Deducem ca 4BM C este dreptunghic, cu m(BM C) VI.80. Sa se demonstreze ca poriunea haurata din figura t s alaturata poate fi scrisa ca reuniune de segmente nchise, doua cte doua disjuncte. Marius Tiba, elev, Iai s Soluie. Notm cu C1 conturul exterior i cu C2 pe cel interior. Dac t a s a punct fixatn interiorul lui C2 , M este un punct care parcurge C1 , iar {N } = atunci mulimea tuturor segmentelor de forma [M N ], M C1 , verific t problemei.

Clasa a VII-a

VII.76. Aflai numerele naturale a, b, c pentru care 11 (a b 9) > t 11 (b c 9) > a (a 20) si 11 (c a 9) > b (b 20). Veronica Pleu i Dan Pl a s s Soluie. Dac m, n Z i m < n, atunci m + 1 n. Astfel, t a s c (c 20) < 11 (a b 9) c2 20c 11a 11b 100. Analog obinem nc dou relaii i, prin adunare, deducem c t a a t s a a2 + b2 + c2 20 (a + b + c) 300 (a 10)2 + (b 10)2 + (c 10 prin urmare a = b = c = 10. VII.77. Fie x, y numere reale pozitive, ambele subunitare sau ambe 1 1 1 nitare; sa se arate ca xy + + 2 x + + y + . Daca unul dintre nu xy x y mai mic, iar celalalt mai mare ca 1, inegalitatea i schimba sensul. s Marian Tetiv Soluie. Avem voie s eliminm numitorii i obinem succesiv t a a s t 2 2 2 2 x y + 1 + 2xy x y + y + xy + x (xy + 1)2 (x + y) (xy + 1)

adic (xy + 1) (x 1) (y 1) 0, evident adevrat. Cu aceleai calcule a a s streaza i partea a doua. Egalitatea se realizeza daca i numai daca macar u s s numere este 1. VII.78. Sa se rezolve n numere naturale ecuaia 6a 5b = 1. t Tudor Pdurariu, ele a Soluie. Daca a, b 2, atunci 5b se termina n 25, iar 6a se termina t x cifr impar; evident c diferena lor nu va fi niciodat 1. Cercetnd a a a t a rmase, obinem c (1, 1) este unica soluie a ecuaiei. a t a t t VII.79. Fie ABCD un patrulater convex, iar E si F interseciile bi t b b unghiurilor D, respectiv B, cu diagonala [AC]. Sa se arate ca puncte coincid daca si numai daca AB CD = AD BC. Claudiu- tefan P S Soluie. t Din teorema bisectoarei obinem ca t AE AB AF AD AC AD i = s = , de unde AE = , CD CE BC CF AD + CD AB AC . Cum punctele E i F sunt interioare s iar AF = AB + BC patrulaterului, atunci AD AC AB AC E = F AE = AF = AD + CD AB + BC AD (AB + BC) = AB (AD + CD) ADBC = ABCD.

VII.80. Fie ABCDEF un hexagon regulat nscris ntr-un cerc, iar P

pe arcul mic BC. Sa se arate ca P E + P F = P A + P B + P C + P D. Dan Radu, B Soluie. Teorema lui Ptolemeu aplicata n patrulaterele inscriptibile t P BF D d P A CE + P C AE = AC P E i P B F D + P D BF = P F a s CE = AE = AC i F D = BF = BD, rezult c P A + P C = P E i P B + s a a s () de unde, prin sumare obinem concluzia. t Observaii. 1) Aplicnd teorema lui Van Schooten (care poate fi sta t teorema lui Ptolemeu!) punctului P i 4ACE, 4BDF echilaterale, ob s t litaile () i continuam ca mai sus. t s 2) Alexandru Tudorache, elev, Iai, propune o soluie trigonometr s t

pe teorema sinusurilor. Daca = m(P C) i R este raza cercului ci s obinem: P E = 2R sin 60 + t , P F = 2R cos , P A = 2R sin 60 2 2 2R sin 30 , P C = 2R sin i P D = 2R sin 30 + s . Relaia este t 2 2 2 astfel cu uurina. s t

Clasa a VIII-a

VIII.76. Fie ABCD un trapez cu AB k CD, M (AD) si N M N k AB, iar E (AB), F (CD) oarecare. Fie {O} = EF M N , [OP perpendiculare de aceeai parte pe planul trapezului, G centrul de greutate a s {Q} = M G (T BC). Sa se arate ca M N este linie mijlocie n trapez dac daca Q T N . Bogdan Raia, t

Soluie. Avem: Q T N QN (ABC) t (M QN ) (ABC) (M GO) (ABC) GO (ABC) G P O P O este mediana EO AM n 4P EF =1 = 1 M N este OF MD linie mijlocie a trapezului. VIII.77. Pentru x, y R , sa se demon + streze inegalitatea 2 x6 +y 6 x2 y 2 x2 + y 2 x3 + y 3 xy (x + y) .

evident adevarat. Egalitatea se atinge cnd x = y. VIII.78. Pentru a, b, c R, sa se demonstreze inegalitatea p p p 2 + b2 ab + 2 + c2 bc + a b c2 + a2 ca a + b + c.

Lucian Tuescu, Craiova, i Gheorghe Nedele t s Soluie. Inegalitatea devine succesiv: t 2 x6 + y 6 x4 y 2 x2 y 4 x3 + y 3 2xy (x + y) x3 + y 3 + x2 y 2 (x + x4 y xy 4 2x3 y 3 2xy x4 + xy 3 + x3 y + y 4 + x4 y 2 + 2x3 y 3 + x 2 4x3 y 3 + 2x5 y + 2xy 5 0 2xy x2 y 2 0,

Claudiu- tefan P S a+b 2 + b2 ab Soluie. Are loc inegalitatea a t ; acest fapt este 2 a + b < 0, iar pentru a + b 0 ea revine, dupa calcule, la (a b)2 0. S dou relaii analoage i sumndu-le, obinem concluzia. Egalitatea se atin a t s t a = b = c 0. VIII.79. Sa se rezolve n numere naturale ecuaia x (x + 1) = y 2007 . t Alexandru Negrescu, elev, Soluie. Pentru x = 0, obinem y = 0. Fie acum x 6= 0; cum (x, x t t numerele x i x + 1 trebuie sa fie puteri de exponent 2007: x = a2007 , x + s cu b > a > 0. Atunci x + 1 = b2007 (a + 1)
2007

contradicie. Rezulta ca unica soluie a ecuaiei este (0, 0). t t t VIII.80. Stiind ca 1 ianuarie 2007 este ntr-o zi de luni, sa se arate anul 2100 exista trei ani biseci n care luna februarie are trei duminici c t zile impare. Petru Asa Soluie. Trei duminici ale unei luni februarie dintr-un an bisect cad n z t atunci cnd 1 februarie este ntr-o duminica. Considernd o axa a timpu ziua nti este 1 ianuarie 2007, fiecare duminic va avea ca numr de a a multiplu de 7. Ziua de 31 ianuarie 2008 are numrul de ordine 396, iar u a patru ani de tipul 1 februarie 2008 31 ianuarie 2012 are 1461 de zile. P

> (a + 1) a2006 = a2007 + a2006 a2007 + 1 =

zi de 1 februarie a unui an bisect s cad duminic, trebuie ca 396 + 146 a a a deci 1461k = M7 + 2. Deducem ca (7 208 + 5) k = M7 + 2, adica 5k = M

mai mici valori ale lui k avnd aceast proprietate sunt 6, 13, 20, iar lor le a anii biseci 2032, 2060, 2088. t

Clasa a IX-a

IX.76. Fie d1 , d2 , . . . , dk divizorii arului 53 72 , iar Sn = dn +dn num 1 2 4n 5 + 1 73n + 1 n N . Sa se arate ca S2n = Sn , n N . General (5n + 1) (7n + 1) Petru Asa Soluie. Numrul 53 72 are 4 3 = 12 divizori, anume 1, 5, 52 , 53 , t a 7 52 , 7 53 , 72 1, 72 5, 72 52 , 72 53 . Se constata uor ca s

Generalizarea este imediat: dac p1 , p2 , . . . , pm sunt prime, iar d1 , a a sunt divizorii lui p1 p2 pm , atunci m 1 2 ( +1)n ( +1)n + 1 pm m +1 p1 1 S2n = Sn , n N . (pn + 1) (pn + 1) m 1 p IX.77. Sa se arate ca a3 + b3 ab 2 (a2 + b2 ), a, b 0. Ovidiu Pop, Sa Soluie. Dac a = 0 sau b = 0, inegalitatea este evident. Fie a, t a a a b notaiile x = + , inegalitatea se scrie succesiv t b a b3 a a3 b + a6 + 2a3 b3 + b6 2a2 b2 a2 + b2 3 + 2 + 3 2 b a b a x3 5x + 2 0 (x 2) x2 + 2x 1 0.
2

54n 1 73n 1 Sn = 1 + 5n + 52n + 53n 1 + 7n + 72n = n ; 5 1 7n 1 58n 1 76n 1 S2n = 1 + 52n + 54n + 56n 1 + 72n + 74n = 2n = 5 1 72n 1 4n 3n 3n 3n 4n 4n 5 1 5 +1 7 1 7 +1 5 +1 7 +1 = = (5n 1) (5n + 1) (7n 1) (7n + 1) (5n + 1) (7n + 1)

Cum x 2, iar x2 + 2x 1 = (x + 1) 2, aceast din urm inega a a adevarata i de aici rezulta cerina problemei. Egalitatea se atinge cnd x t s pentru a = b.

IX.78. Fie a, b, c laturile 4ABC, iar G centrul sau de greutate. No E, F punctele de contact ale cercului nscris cu laturile BC, CA, respect se arate ca aGD + bGE + cGF = 0 daca si numai daca 4ABC este ech Marian Ursrescu a Soluie. Implicaia invers este imediat. Fie deci aGD + bGE + cGF t t a a DB pb aratam ca a = b = c. Pentru k = = , obinem ca t DC pc GB + k GC (p c) GB + (p b) GC GD = = , 1+k a

IX.79. Fie 4ABC echilateral si P un punct n interiorul sau. C A1 AB, B1 BC, C1 CA astfel nct P A = P A1 , P B = P B1 si P Sa se arate ca P este centrul de greutate al 4A1 B1 C1 . Iulia Pleca, e s Soluie. Fie P1 , P2 , P3 proieciile lui P pe AB, BC, t t respectiv CA. Daca O este centrul 4ABC, atunci 1 1 P P1 + P P2 + P P3 = P A + P A1 + P B + P B1 + 2 2 1 + P C + P C1 = 2 1 3 P A1 + P B1 + P C1 . = PO + 2 2 3 s Pe de alta parte, P P1 + P P2 + P P3 = P O () i astfel 2 deducem ca P A1 + P B1 + P C1 = 0 , prin urmare P este centrul de greutate al 4A1 B1 C1 . Relaia () este relativ uzual i se justic astfel: ducem prin P s t a s a U V k AB, XY k BC, ST k AC, cu X, S AB, U, T BC, V, Y AC. 1 urile P U T , P Y V i P XS sunt echilaterale i obinem ca P P1 = s s t P 2 1 1 P U + P T , P P3 = P V + P Y . Din regula paralelogramul P P2 = 2 2 P X + P U = P B, P T + P Y = P C i P S + P V = P A. Atunci s 1 3 P P1 + P P2 + P P3 = P A + P B + P C = P O. 2 2

deci aGD = (p c) GB + (p b) GC. Scriem nc dou relaii similare i a a t s rezulta ca aGA + bGB + cGC = 0 . nsa GA + GB + GC = 0 , deci aGA+bGB c GA + GB = 0 (a c) GA+(b c) GB = 0 ac = n o (am folosit faptul ca GA, GB constituie o baza). Deducem ca a = b =

IX.80. Fie , , , patru numere pozitive cu suma . Sa se afl sumei S = sin sin + sin sin si sa se determine situaia n care acest m t atins. Adrian Cordune Soluie. Considerm un cerc de centru O i raz R, n care nscriem pa t a s a \ \ \ \ ABCD cu m(ADB) = 2, m(BOC) = 2, m(COD) = 2 i m(DOA) = s AB = 2R sin , BC = 2R sin , CD = 2R sin , AD = 2R sin , iar A BD 2R. Tinnd seama de teorema lui Ptolemeu,

deci S 1. Aceast valoare maxim este atins cnd AC = BD = 2R, a a a ABCD este dreptunghi; rezulta ca S este maxima cnd = , = = 0<< . 2

AB CD + BC AD = AC BD 4R2 sin sin + 4R2 sin sin

Clasa a X-a

X.76. Fie C = {z C | |z| = 1}. Sa se rezolve n C 2 ecuaia z1 z2 = z1 t Gabriel Popa i Paul Georg s Soluia 1 (a autorilor). Dac z1 , z2 C, atunci t a z1 z2 C, iar z1 + z2 + 3 este un numr complex a cu imaginea pe discul D de centru B (3, 0) i raza 2. s Evident ca C D = {A}, cu A (1, 0), prin urmare z1 z2 = z1 + z2 + 3 = 1. Atunci z2 = z1 2 i obinem s t 2 c z1 (z1 2) = 1 z1 + 2z1 + 1 = 0 z1 = 1, a apoi z2 = 1. Soluia 2 (Mihai Haivas). Fie z1 = cos + i sin , z2 = cos + i sin t data devine

s t Cum z1 + z2 = a i z1 z2 = b, avem prima condiie din (ii). n plus, |b| = de unde a doua parte a lui (ii). (ii) (i) Facem un raionament invers celui de mai sus. t X.78. Determinai triunghiurile n care tangentele unghiurilor se exp t numere naturale, exact doua dintre ele avnd aceeai paritate. s Ctlin Cali a a Soluie. Se tie c tg A + tg B + tg C = tg A tg B tg C. Distingem s t s a a) tg A = 2p, tg B = 2q, tg C = 2r+1, cu p, q N , r N. Atunci 2p+2q 4pq (2r + 1), egalitate imposibil ntruct membrul stng este impar, iar cel a par. b) tg A = 2p + 1, tg B = 2q + 1, tg C = 2r, cu p, q N, r N ; avem

cos ( + ) + i sin ( + ) = (cos + cos + 3) + i (sin + sin ) + + + Egalnd prile imaginare, gsim c sin at a a cos = sin cos 2 2 2 + + = 2k (deci sin = 0), obinem c 1 = cos ( + ) = cos + t a 2 de unde cos + cos = 2, deci cos = cos = 1 i atunci z1 = z2 = s + + + 6= 2k, deducem ca cos = cos , de unde = 2 2 2 2 Ambele situaii conduc imediat la contradicii. t t t t X.77. Fie a, b C si z1 , z2 soluiile ecuaiei z 2 az + b = 0. Sa s urmatoarele afirmaii sunt echivalente: t (i) |z1 | < 1 si |z2 | < 1; (ii) |a|2 + a2 4b < 2 |b|2 + 1 < 4. Marian Tetiv Soluie. (i) (ii) Fie r1 = |z1 |, r2 = |z2 |; condiia r1 < 1, r2 t t echivalent cu (r1 1) + (r2 1) < 0 i (r1 1) (r2 1) > 0, deci cu r1 + a s r1 + r2 < r1 r2 + 1. Din a doua relaie, prin ridicare la ptrat, obinem t a t 2 2 2 2 r1 + r2 < r1 r2 + 1 |z1 + z2 |2 + |z1 z2 |2 < 2 |z1 z2 |2 + 1 .

i dac p + q > 1, atunci r (4pq + 2p + 2q) 4pq + 2p + 2q > p + q + 1 s a ca p + q 1 i, cum nu putem avea p = q = 0, deducem ca p + q = s

2 (p + q + 1) + 2r = (2p + 1) (2q + 1) 2r p + q + 1 = (4pq + 2p +

r = 1. Triunghiurile care satisfac cerina sunt asemenea, avnd unghiurile t i arctg 3. s X.79. Sa se arate ca n orice triunghi are loc inegalitatea (p r 2R) (p ra ) (p rb ) (p rc ) 0. Cnd se atinge egalitatea? I. V. Maftei i Dorel aian, B s B t Q Q A Soluie. Are loc identitatea t (p ra ) = p3 1 tg , prin ur 2 Q a dusul (p ra ) este pozitiv, nul sau negativ dup cum 4ABC este ascu dreptunghic sau obtuzunghic. Pe de alta parte, avem: 1 X Y X 1 1 2 cos A = a + b2 + c2 1 cos2 A = sin2 A 2 = 2 2 8R2 i 1 2 1 h 2 = p (2R + r)2 2 p r2 4Rr 8R2 = 8R2 4R2 Y 4R2 p r 2R = cos A. p + r + 2R Deducem de aici c p r 2R este pozitiv, nul sau negativ, dup cum 4 a a ascuitunghic, dreptunghic sau obtuzunghic, ceea ce ncheie justificarea in t Egalitatea se atinge n cazul triunghiului dreptunghic. X.80. Aratai ca exista o infinitate de valori n N pentru care numer t si 3n + 1 sunt patrate perfecte. Gheorghe Iu Soluie. Observam ca t 2k+1i i X 2k+1 2k+1 i 3+ 2 = C2k+1 3 2 = Ak 3 + Bk 2, cu Ak
i=0

2 Considernd nk = Bk A2 obinem ca 2nk + 1 = A2 , iar 3nk + 1 = t k k urmare numerele 2n + 1 i 3n + 1 sunt patrate perfecte pentru o infinitat s ale lui n.

2k+1 3 2 = Ak 3 deoarece 2k + 1 i i i au paritai diferite, iar s t Rezulta ca h 2 2 3+ 2 3 2 3Ak 2Bk = Ak 3 + Bk 2 Ak 3 Bk 2 =

Clasa a XI-a

prin urmare det (A + t A i) = det (A t A i) dac i numai dac n este par as a cular, egalitatea are loc pentru n = 4.

XI.76. Daca A M4 (R), sa se arate ca det (A + t A i) = det (A Generalizare. Dan Popescu Soluie. Dac A Mn (R), atunci t a 1 n n det A t A i = (i) det t A A = (i) det t A + i A = i t t n = (i) det A + i A = (i)n det A + t A i

XI.77. Fie f : [a, b] R o funcie de doua ori derivabila pe [a, b t bele derivate strict pozitive. Pentru [a, b], consideram punctele A B (b, f (b)), (, yC ) Gf si D (, yD ) AB. Demonstrai ca exista si C t 0 a+b , b astfel nct f (b) yD = yC f (a). 2 a Ctlin Tigeru a a b Soluie. Obinem imediat ca yC = f (), iar yD = t t f (a) + ba b Considerm g : [a, b] R, a b a g () = f (b) g (b) yD (yC f (a)) = f (b) + f (a) ba ba f (b) Funcia g este de dou ori derivabil pe [a, b]. Cum g 0 () = t a a b a a a f 0 () < 0, [a, b], rezult c g este strict descresctoare; n plus, g( (f (b) f (a))2 < 0. Urmeaza ca exista i este unic 0 (a, b) pentru car s a + b a + b 1 0. Faptul ca 0 , b rezulta din aceea ca g = [f (a) 2 2 2 a + b f > 0, ntruct f este strict convexa, iar g (b) = f (a) f (b) < 0, 2 este strict cresctoare. a XI.78. Pentru x R , sa se demonstreze inegalitatile: + 1 1 a) ln x + a (1 + ln a), unde a > 0; x a k b) ax > (1 + x) , unde a > ek , k N , iar {1}. Gheorghe Costo 1 1 Soluie. a) Fie f : (0, ) R, f (x) = ln x + a (1 + ln a); atun t x a 1 1 a s a+1 = a+1 (xa a). Avem f 0 (x) < 0 pentru x 0, a1/a i x x x 1/a 1/a pentru x a , + , prin urmare f are un minim n x0 = a , egal cu f Deducem ca f (x) 0, cu egalitate cnd x = a1/a . k b) Fie g : (0, ) R, g (x) = ax (1 + x) . Dezvoltm funcia g n se a t ntr-o vecintate a originii: a x x2 xk (k) xk+1 (k+1) g (x) = g (0) + g 0 (0) + g 00 (0) + + g (0) + g 1! 2! k! (k + 1)!
k+1

unde 0 < k < 1. Avem g (0) = 0, g (k+1) (xk ) = axk (ln a) > 0, cc a iar g (p) (0) = (ln a)p p k (k 1) (k 2) (k (k p + 1)), p = 1, k. Cu obinem ca (ln a)p > k p , prin urmare g (p) (0) > 0, p = 1, k i astfel se obine t s t x (0, ). XI.79. Fie (xn )n1 un sir convergent, a carui limita o notam L(xn ). D x n n = a R+ daca si numai daca exista lim (1 + xn ca exista lim n L (xn ) n b R+ . Ce legatura este ntre a si b? D. M. Btineu-Giurgiu, B a t Soluie. Sunt imediate egalitile: t at x n 1 lim n(xn L(xn )) lim n lim = e L(xn ) n ; lim (1 + xn L(xn ))n = en n L (xn ) n

x n n lim n (xn L (xn )) lim (1 + xn n L (xn ) n n Se deduce uor ca b = aL(xn ) . s Atunci exist lim a

ex f (x) . Din teorema lui x obinem c |h (x) h (y)| < M |x y|, x, y (0, 1), x 6= y. Fie (xn )n0 t a ir convergent la zero; avem c |h (xn ) h (xm )| M |xn xm |, m, n N s a (h (xn ))n0 este ir Cauchy. Deducem ca (h (xn ))n0 este convergent i fie s s Dac (yn )n0 este un alt ir convergent la zero, cum |h (xn ) h (yn )| M a s n N, rezult c lim h (yn ) = lim h (xn ) = l, prin urmare exist lim a a a unde h : (0, 1) R este definit prin h (x) = a
n n 0

XI.80. Fie f : [0, 1] R o funcie continua pe [0, 1], derivabila pe t f (0) = 0. Presupunem ca exista M > 0 astfel nct f 0 (x) 1x f (x) < x x (0, 1). Sa se arate ca f este derivabila n origine. Mihai Crciun a Soluie. Condiia din enun se scrie echivalent t t t x xe f (x) + xex f 0 (x) ex f (x) < M |h0 (x)| < M, x (0, 1 x2

x0

f (x) Urmarind definiia lui h, obinem existena lui f (0) = lim t t t = l, d x0 x derivabil n origine. a

Clasa a XII-a

XII.76. Fie funcia continua f : [a, b] R si n N . Aratai ca exista t t astfel nct Z b f (c) + f (a) + f (b) (b a)n (f (x) + f (a) + f (b)) dx = . n1 n (c a) a

unde k este o constanta ce urmeaza a fi determinata din condiia (a t Rb Cum (a) = 0 rezulta (b) = 0, adica k = a (f (t) + f (a) + f (b)) dt

Dumitru Mihalach Soluie. Vom considera funcia : [a, b] R, t t Z x (x a)n (f (t) + f (a) + f (b)) dt k (x) = , n a

Aplicnd teorema lui Rolle funciei , exist c (a, b) astfel nct 0 ( t a f (c) + f (a) + f (b) n1 = 0. Obinem k = t f (c) + f (a) + f (b) k (c a) n1 (c a) cele doua expresii ale lui k, rezulta concluzia . Rb Not. Pentru n = 1 obinem c a f (x) dx = f (c) (b a), prin urmare a t a constituie o generalizare pentru teorema de medie. R 1/nk XII.77. Fie k N , fixat. Consideram sirurile an = 1/(n+1)k arcsin R 1/nk an . n 1 si bn = 1/(n+1)k arctg nk x dx, n 1. Sa se calculeze lim n bn Liviu Smarandache i Lucian Tuescu s t

Soluie. Conform teoremei de medie, pentru orice n 1, exist t a 1 1 , astfel nct (n + 1)k nk 1 1 1 1 i bn = an = arcsin nk cn arctg s k k nk nk (n + 1) (n + 1) arcsin nk cn an = lim Astfel, lim . Cu teorema cletelui obinem c lim s t a n bn n arctg (nk dn ) n k arcsin 1 an = = 2. lim n dn = 1. Prin urmare, lim n n bn arctg 1 XII.78. Daca f : R R este o funcie continua si fara puncte fixe, s t ca nici funciile f f f , n N , nu au puncte fixe. t {z } | n ori Dorin Mrghidanu, a Soluie. Considerm funcia g : R R, g (x) = f (x) x, care t a t tinua i nu se anuleaza pe R; deducem ca g pastreaza semn constant pe R s f n = f f f , rezult c pentru orice x R, numerele g (x), g (f a a {z } | n n1 ori g f (n) sunt simultan sau pozitive, sau negative. Cum f n (x) x = g f n1 (x) + g f n2 (x) + + g (f (x)) + g (x) , x urmeaz afirmaia problemei. a t XII.79. Fie V spaiu vectorial de dimensiune n peste corpul K, iar u t morsm nilpotent al lui V (i.e., exista p N astfel nct up = u u {z | Sa se arate ca un = 0.

n ori

Adrian Reisn Soluie. Presupunem prin absurd c un 6= 0; atunci polinomul mini t a domorsmului u este X p , cu p > n. Acest fapt este imposibil, ntruct minimal al lui u divide polinomul caracteristic al lui u (teorema Hamilto iar polinomul caracteristic este de grad n. Not. n fapt, autorul a propus spre publicare urmtorul rezultat, m a a a crui demonstrare depete ns nivelul unui absolvent de liceu: a as s a Fie V spaiu vectorial de dimensiune n peste corpul K, iar u1 , u t endomorfisme nilpotente, care comuta doua cte doua; atunci u1 u2 4 4 XII.80. Fie A un inel n care x y = (x y) (x + y) x2 + y 2 , x a) Daca inelul are unitate, sa se arate ca A este comutativ. b) Ramne valabil rezultatul de la a) daca inelul A nu este unitar? Gabriel Dospinescu, Paris i Marian Tetiv s Soluie. Conform ipotezei, avem c t a f (x, y) = (x y) (x + y) x2 + y 2 x4 y 4 =

= x2 y 2 + xyx2 + xy 3 yx3 yxy 2 y 2 x2 = 0, x, y A Dupa calcule simple, obinem apoi: t g (x, y) = f (x + 1, y) f (x, y) = 2xy 2 + 2xyx + xy 2y 2 x 2yx2 f (x + 2, y) 2f (x + 1, y) + f (x, y) = g (x + 1, y) g (x, y) = 2 (xy

pentru orice x, y A (unde 2 = 1 + 1, iar 1 este unitatea inelului). Cum calculate sunt identic nule datorita ipotezei, obinem ca g (x, y) = 0 i 2 (x t s 0, x, y A. Atunci g (x, y) = 2 xy 2 y 2 x + 2 (xy yx) x + xy yx = 0, x, y A 0 + 0 + xy yx = 0, x, y A xy = yx, x, y A.

Absena elementului unitate anuleaz valabilitatea concluziei, dup t a a 0 a b exemplul inelului matricelor de forma 0 0 c; a, b, c R. Cum 0 0 0 oricaror trei matrice de acest tip este matricea nula, e clar ca egalitatea este valabil n acest inel, ns inelul nu este comutativ. a a

IMPORTANT

n scopul unei legturi rapide cu redacia revistei, pot fi utilizate urm a t adrese e-mail: t_birsan@yahoo.com i profgpopa@yahoo.co s aceasta cale colaboratorii pot purta cu redacia un dialog privitor t rialele trimise acesteia, procurarea numerelor revistei etc. Sugeram ratorilor care trimit probleme originale pentru publicare s le nume a s-i rein o copie xerox a lor pentru a putea purta cu uurina o as t a s t prin e-mail asupra acceptarii/neacceptarii acestora de catre redacia t

La problemele de tip L se primesc soluii de la orice iubitor de ma t elementare (indiferent de preocupare profesionala sau vrsta ). Fiecar soluiile acestor probleme - ce sunt publicate n revist dup un a t a a urmata de numele tuturor celor care au rezolvat-o.

Adresm cu insistena rugmintea ca materialele trimise r a t a s nu fie (s nu fi fost) trimise i altor publicaii. a a s t

Rugm ca materialele tehnoredactate s fie trimise pe adresa reda a a A soite de ierele lor (de preferina n L TEX). t s t

Pentru a facilita comunicarea redaciei cu colaboratorii ei, autorii t alelor sunt rugai s indice adresa e-mail. t a

Soluiile problemelor pentru pregtirea concur t a propuse n nr. 1/2007


A. Nivel gimnazial

G116. Aflai toate numerele naturale N de patru cifre nenule distinc t prietatea ca diferena dintre cel mai mare numar obinut prin permutarea t t N si cel mai mic asemenea numar este tocmai N . Maria Mihe, T t Not. A se vedea articolul d-lui Titu Zvonaru O problema cu c a numar, aprut n RecMat 1/2007. a

G117. Fie mulimea A = {1, 2, 3, . . . , 98}. Aratai ca oricum am a t t elemente ale lui A, exista doua printre ele avnd suma cub perfect. Titu Zvonaru, C Soluie. Consideram mulimile Ai = {i, 27 i}, i = 1, 2, . . . , 13, Aj t t 112 j}, j = 14, 15, . . . , 49, cu proprietatea c suma elementelor din fiecar a este cub perfect, iar A = A1 A2 A49 . La alegerea a 50 de elemente, dou din aceeai mulime Ai , conform principiului cutiei, i de aici rezult a s t s a

G118. n interiorul unui paralelogram avnd unghiul ascuit de 30 s t laturilor 17 cm si 59 cm, se considera 2007 puncte. Sa se arate ca putem dintre aceste puncte astfel nct aria triunghiului determinat de ele sa fi egala cu 1 cm 2 . 4 Mihai Ha Soluie. Descompunem paralelograt mul n 17 59 = 1003 romburi de latura 1 1, aria fiecaruia ind 1 1 sin 30 = 2 2 a cm . Conform principiului cutiei, exist un romb care sa conina (n interior sau t pe laturi) mcar 3 dintre cele 2007 puncte. Dac E, F , G sunt aces a a aparinnd rombului M N P Q, fie E 0 i F 0 interseciile lui EF cu dou d t t a s rombului, iar {G} = E 0 G N P (notaiile sunt cele din gur). Evident t a 1 1 AEF G AE 0 F 0 G0 AM NP Q = cm2 . 2 4 2 G119. Fie n N si A = 0 0 + + n 2n | 0 , 1 , . . . , n { B = m | m 2 Z + 1, |m| 2n+1 1 . Sa se arate ca A = B. Dorel Mihe, T t Soluie. Mulimea B are 2 2n = 2n+1 elemente. Sa remarcam ca toat t t din A sunt impare, A are cel mult 2n+1 elemente (sunt 2n+1 posibilitai t ale numerelor i ), mai mare element din A este 1 + 2 + + 2n = 2n cel cel mai mic este 2n+1 1 . Dac artm c la combinaii diferite de i a aa a t numere diferite, va rezulta ca A are exact 2n+1 elemente, deci A = B. Fie 0 20 + +n 2n = 0 20 + + n 2n , cu i , i {1, 1}; ramne c i = i , i = 1, n. Observm c semnul lui 0 20 + + n 2n este dat a a a lui n (se demonstreaz imediat, innd seama de faptul c 2n > 2n1 + a t a aceste condiii, din 0 20 + + n 2n = 0 20 + + n 2n , rezulta c t

Reducnd termenii cu 2n , obinem o egalitate similar cu n1 i n1 n t a s i n , deci n1 = n1 etc. s O alta soluie se poate da folosind unicitatea scrierii unui numar n ba t 2005 2007 G120. Rezolvai n N ecuaia x! (y!) t t = (z!) . Anca Stefania Tuescu, elev t a Soluie. Vom arta c, dac (x, y, z) este soluie, atunci x, y, z {0 t a a a t supunem prin absurd c z 2 i fie p cel mai mare numr prim a s a p2007 | (z!)2007 , de unde p2007 | x! (y!)2005 . Daca p2 nu divide x!, 2005 p2006 | (y!) , de unde p2 | y!; prin urmare, cel puin unul dintre nu t sau y! se divide cu p2 i, s zicem c x! are aceast proprietate. Deducem s a a a dusul x! apar factorii p i 2p i este cunoscut faptul ca ntre p i 2p mai p s s s macar un numar prim q (Postulatul lui Bertrand). Evident ca q!z! i deci s ceea ce contrazice maximalitatea lui p. Ecuaia admite 8 soluii (x, y, z), cu x, y, z {0, 1}. t t G121. Daca a, b (0, 3/2), sa se demonstreze inegalitatea r 1 1 1 1 1 1 + . + + + a b a+b+3 a a+ b b Andrei Laureniu Ciupan, elev, B t 3 3 Soluie. Din a, b 0, t , rezult c ab i atunci a + b + 3 a a s 2 2 2 a + b . Este cunoscut (sau se poate demonstra uor) i a s 2 ab = 2 1 1 1 1 1 1 + + = ; obinem: t + x2 y 2 (x + y)2 x y x+y 1 1 1 1 1 1 1 1 1 + + + + 2 = a + b a + a b a+b+3 a b a+ b

De aici, inegalitatea din enun este imediat. t a G122. Fie G centrul de greutate al 4ABC si G0 proiecia sa pe dre t Sa se arate ca G0 [BC] daca si numai daca 3a2 < b2 c2 . / Temistocle B Soluie. Condiia G0 [BC] este echivalenta cu faptul ca 4GBC t t / \ \ tuzunghic, avnd unghiul obtuz GBC sau GCB, dup cum c < b sau b < a

BC 2 +GB 2 GC 2 \ \ GBC obtuz cos GBC < 0 <0 2 BC GB 2 2 2 2 a2 + mb mc < 0 9a2 + 4m2 4m2 < 0 b c 3 3 9a2 + 2 a2 + c2 b2 2 a2 + b2 + c2 < 0 3a2 < b2 c2 \ Analog, GCB este obtuz 3a2 < c2 b2 i urmeaza concluzia. s G123. Fie ABC un triunghi echilateral. Sa se arate ca orice punct M cu proprietatea ca M B = M A + M C poate fi determinat folosind doar ech

echer poate fi folosit pentru a trasa drepte si unghiuri drepte.) Nicolae Ivchescu as Soluie. Folosind teorema lui Ptolemeu, se arata ca t punctele M cu proprietatea din enun sunt cele de pe arcul t s mic AC al cercului circumscris triunghiului (a se vedea i soluia problemei V II.80). Mijlocul acestui arc se afl int a tersectnd perpendiculara n A pe AB cu perpendiculara n C pe BC. Celelalte puncte ale arcului se construiesc proiectnd mijlocul arcului AC pe semidrepte cu originea \ n B, interioare unghiului ABC.

G124. Fie 4ABC, A0 mijlocul lui [BC], iar P si Q proieciile lui A t respectiv AC. Sa se arate ca 4P Q AB + BC + CA. Adrian Zahariuc, ele Soluie. Notam cu B 0 i C 0 mijloacele laturilor [AC] i t s s [AB], iar cu T i S mijloacele segmentelor [A0 C 0 ], respectiv s [A0 B 0 ]. Atunci A0 C 0 B0C 0 A0 B 0 AC BC AB + + = + + , 2 2 2 4 4 4 de unde 4P Q AB + BC + CA. = P Q P T + T S + SQ =

Nota autorului. Dup calcule laborioase, se poate demonstra c pro a a R ma . echivalent cu inegalitatea a ha 2r G125. Fie ABCD un patrat, M (AB), {O} = AC BD, {S} = C iar {E} = SO M D. Consideram AA0 (ABC), AA0 = AB, I mijlocul iar {H} = M I A0 E. Sa se arate ca: 2 VA0 ADH AB 0 a) M D (A AE); b) = . VMADH AM Petru Rducanu, Iai a s Soluie. a) Fie {L} = SO AB. Aplicnd teorema lui t Menelaus n 4AM D cu transversala E L S i n 4AM C s cu transversala O L S, obinem t EM SD LA LA SM OC = 1; = 1. ED SA LM LM SC OA SC SD CD EM i deducem c ns a = = s a = SM SA MA ED 2 AM , prin urmare AE M D. Avem i c AA0 s a AD2 0 M D i astfel M D (A AE). s b) De la a) obinem ca A0 E M D. n plus, se t observ c M D = M A0 i, cum M I este median n a a s a a a 4M DA0 isoscel, atunci M I A0 D. Rezult c H este ortocentru n 4M DA0 , deci DH A0 M . Avem i s

A0 M DA (deoarece DA (AM A0 )), prin urmare A0 M (ADH). Ded 0 2 2 1 0 AB AA A0 T VA0 ADH 3 AADH A T = 1 = . = = VMADH MT AM AM 3 AADH M T

B. Nivel liceal

L116. Cercul nscris n 4ABC este tangent laturii BC n punctul D1 A-exnscris este tangent aceleeai laturi n punctul D2 . Dreapta AD2 int s cercul nscris n punctele S si T . Sa se arate ca 4ST D1 este dreptunghi Titu Zvonaru, C Soluie. Fie E1 , E2 punctele de tangena ale cert t cului nscris, respectiv A-exnscris cu latura AC. Cu notaiile uzuale, avem: BD2 = p c, CD2 = p b, t AE1 = p a, AE2 = p, D1 D2 = |b c|. Dac a = AD2 , cu relaia lui Stewart obinem: t t a2 = c2 (p b) + b2 (p c) a (p b) (p c) = Mai notam x = AS, y = T D2 , z = ST . Folosind 2 puterea punctului faa de cerc avem AS AT = AE1 i t s 2 D2 S D2 T = D1 D2 , prin urmare nlocuim z = x y n primele dou ecuaii: a t x xy = (p a)2 ,
2

= p (b c) + ap (p a) .

x2 + xz = (p a) ,

y 2 + yz = (b c) ,

x + y + z = .

y xy = (p c)2 .

(b c) i prima ecuaie devine s t x h i (b c)2 x 2 2 x + = (pa) ax2 x (p a) (b c) + a (b + c) + a (p x (p a) p (p a) i x2 = i core Soluiile acestei ecuaii sunt x1 = t t s s p 2 p (b c) a obinem y1 = t , y2 = . Observm c a a a p Acum substituim y =

ap2 > p (b c)2 + ap (p a) (b c)2 < a (p p + ) (b c)2 s adevarat. Atunci valorile x2 , y2 nu convin i ramne ca AS pa = . AD2 p

x2 + y2 > p2 (p ) + a2 > 2 p p2 (p a) > 2 (p a)

Dac I, Ia sunt centrele cercurilor nscris, respectiv A-exnscris, atun a

pa AS AI AE1 = = , de unde IS k Ia D2 . ns Ia a , prin urmare AE2 p AIa AD2 i deducem c IS BC. Avem c ID1 BC; rezult c punctele S, I a a s a a coliniare, adica [SD1 ] este diametru i astfel 4ST D1 este dreptunghic. s

L117. Fie 4ABC, D (BC), iar C1 , C2 cercurile exnscrise triu ADB si ADC, tangente la BC. Aratai ca o tangenta comuna interioara t C1 si C2 trece prin punctul de contact cu BC al cercului A-exnscris. Neculai Roman, Mirc Soluie. Dac D este punctul de contact cu BC al t a cercului A-exnscris 4ABC, atunci AD este tangenta comuna interioara cautata. Mai mult, n acest caz se poate arta c C1 i C2 sunt tangente exterior. a a s Considerm c D nu este punctul de contact cu a a BC al cercului A-exnscris 4ABC i notam {M1 } = s C1 BC, {M2 } = C2 BC, {N1 } = AD C1 , {N2 } = AD C2 , P1 i P2 sunt punctele de cons tact cu C1 , respectiv C2 , ale tangentei comune interioare cercurilor C1 i C2 (diferita de AD), iar {T } = s 1 a P1 P2 BC. Avem c DM1 = (AB + BD AD), 2 1 DM2 = (AC + CD AD). Distingem dou cazuri: a 2 I. T (CD); avem c T B = BD + T D, T D = T M1 DM1 , T M1 a T P2 + P1 P2 = T M2 + N1 N2 = T M2 + DN2 DN1 = T M2 + DM2 DM alta parte, T M1 = DM1 + DM2 T M2 , prin urmare DM2 = T M1 . Apoi AC AB + DC DB T D = T M1 DM1 = DM2 DM1 = 2 AC AB + DC DB AC + BC AB T B = BD + = , 2 2 ceea ce trebuie demonstrat. II. T (BD); se efecteaz un calcul asemntor. a a a

Not. Vlad Emanuel, student, Bucureti, a soluionat problema c a s t unor calcule elaborate, etalnd tehnici diverse i de mare finee. s t

L118. Fie M un punct al elipsei E, de focare F si F 0 . Dreptele M intersecteaza elipsa n A, respectiv A0 . Sa se arate ca, atunci cnd M p dreapta AA0 este mereu tangenta unei curbe fixe, care se cere a fi determ Adrian Reisn Soluie. Consideram elipsa E raportata la axele sale de simetrie; ecua t a 1 t2 2bt ,y= , t R. Fie M corespun metrice ale elipsei vor fi x = 2 1+t 1 + t2 0 s rii t0 a parametrului; vom calcula valorile t1 i t1 ce corespund punctelor A x y 0 t s A . Ecuaia dreptei M A este (1 t0 t1 ) + (t0 + t1 ) (1 + t0 t1 ) = 0 i, a b c faptul ca M A trece prin punctul F (c, 0), obinem ca (1 t0 t1 ) (1 + t0 t a ca c+a 0 unde se deduce c t1 = a . Analog gsim t1 = a , deci ecua (c + a) t0 (c a) t0 0 AA este 2 a2 + c2 t0 2 0 x y t2 + 1 = 0. t0 1 AA : 0 3 a b

x 2 a2 + c2 y 2 Scriem aceast ecuaie sub forma a t t0 1 t0 + a b3 a impunem ca ea sa admita soluie dubla n t0 , adica = 0. Dupa calcule t 2 2 a + c2 y 2 x2 elipsa E 0 : 2 + 1 = 0 i aceasta va fi curba fixa cautata; s a b6 0 numele de curba nfasuratoare a familiei de drepte AA . L119. Fie n N si a.b, c R cu ab + bc + ca = 3. Sa se arate ca an+ + n+3 c + 2abc (an + bn + cn ) 9. Titu Zvonaru, Comneti i Bogdan Ionia, B a s s t Soluie. Inegalitatea lui Schur t x

an+1 (a b) (a c) + bn+1 (b c) (b a) + cn+1 (c a) (c b) devine, innd seama i de ipotez, t s a n+3 n+3 n+3 n +b +c + 2abc (a + bn + cn ) (ab + bc + ca) an+1 + bn+1 a an+3 + bn+3 + cn+3 + 2abc (an + bn + cn ) 3 an+1 + bn+1 + cn+1 . ap + bp + 1 + + 1 pab, bp + cp + 1 + + 1 pbc, cp + ap + 1 + | {z } | {z } | {z
p2 p2

Problema este rezolvata daca dovedim ca, n ipoteza ab + bc + ca = 3, avem cp 3, p N. Cu inegalitatea M A M G, obinem t
p2

Scriem nc n 1 inegaliti de acelai tip (obinute prin permutri cir a at s t a summ i rezult inegalitatea dorit. a s a a L121. Fie n N dat. Sa se arate ca exista civa termeni ai sirului t m 1 a caror suma este mai mare dect . (n + 1) (2n + 1) Dumitru Mihalache i Marian Tetiv s 1 1 t a Soluie. Notm Sn = 1+ 3 + + 3 suma parial a seriei armonice d t a 2 n 3, deci supraunitar. Rezult c irul (Sn )n1 este convergent; fie S limita a as a rezolva problema, ar trebui sa dovedim ca lim (Sn+k Sn ) >
k

Prin adunare, rezulta ca 2 (ap + bp + cp ) + 3p 6 p (ab + bc + ca), adica cp 3. L120. Pentru a1 , a2 , . . . , an reale pozitive, sa se demonstreze inegali (n1)2 (n1)2 (n1)2 a2 an a1 a2n1 + a2 a2n1 + + a1 2 3 + + + a2 a3 a1 a2 a2 a2 n 1 2 Marian Tetiv Soluie. Folosind inegalitatea M A M G pentru (n 1)2 numere, ob t (n1)2 (n1)2 (n1)2 a1 a2 an2 an1 (2n3) + (2n5) + + 3 + a2 a3 an1 an v 2 u" #(n1) u a 2n3 a 2n5 a a 1 2 n1 t 2 (n1)2 (n 1) = (n 1)2 2 a2 a3 an a1

(n + 1

1 , n 1. Evident, pentru asta ar fi s (n + 1) (2n + 1) 1 dovedim ca irul (xn )n1 , xn = Sn + , n 1 (a carui s (n + 1) (2n + 1) tot S) este strict cresctor, fapt care rezult dup un calcul simplu: a a a deci c S > Sn + a xn+1 xn = = 1 (n + 1)
3

1 1 = (n + 2) (2n + 3) (n + 1) (2n + 1) , n N .

3n2 + 5n + 1 (n + 1) (n + 2) (2n + 1) (2n + 3)


3

Not. Principial, aceeai soluie a dat Vlad Emanuel, student, Bucu a s t

L122. La un campionat de fotbal participa 2n echipe, astfel nct din doua se poate dinainte indica echipa mai buna. n prima etapa, echipele aleator n perechi si disputa cte un meci, echipa mai buna trecnd n etapa u Procedeul se repeta pna la finala. a) Care este probabilitatea ca a doua echipa ca valoare sa iasa vicecam b) Daca se disputa si o finala mica, ce probabilitate este ca, n plus, cea echipa ca valoare sa se claseze pe locul 3? Irina Mustaa, student t a Soluie. Numerotm echipele de la 1 la 2n , 1 fiind cea mai bun t a a numrul modalitilor distincte (ignornd permutrile) de a mpri 2n e a at a at 2 perechi. Din 2n numere, o pereche se poate alege n C2n moduri, urmtoar a 2 n C2n2 moduri, etc. gnornd permutarile implicate, obinem ca t P2n = 1 2 2 (2n)! 2 C2 = n = (2n 1)!! C C n! 2n 2n2 2 n!

b) n seminale avem permise doar mperecherile (1, 3), (2, 4) sau (1 iar ntr-o etap anterioar sunt interzise meciurile (1, 2), (1, 3) i (2, 3). N a a s repartizri convenabile ntr-o etap cu 2i echipe este P2i 3P2i 2 i atunc a a s itatea ceruta va fi n n Q Q i 2 2 (P2i 3P2i 2 ) 2 3 !! 4 2i2 1 22n = i=3 P 0 = i=3 Q = n n n Q i (2 1) (2n P2i (2 1)!!
i=2 i=2

a) n prima etap, numrul cazurilor posibile este P2n . Cazurile ne a a sunt cele n care este selectat perechea (1, 2), n numr de P2n 2 ; numru a a a favorabile va fi P2n P2n 2 . Continund raionamentul, probabilitatea even t cerut este n n Q Q i (P2i P2i 2 ) 2 3 !! 2 2i1 1 2n1 P = i=2 Q = i=2 = n . n n Q i 2 1 P2i (2 1)!!
i=1 i=2

Not. Principial, aceeai soluie a dat Vlad Emanuel, student, Bucu a s t

L123. Pe o tabla 8 9 se aeaza dreptunghiuri 3 1 si "figuri" s de forma unui dreptunghi 1 3 caruia i lipsete patratul median (ca s n desenul alaturat). "Figurile" si dreptunghiurile nu se pot roti si nu au puncte interioare comune. Sa se arate ca exista o mulime S de 18 t patrate 1 1 astfel nct, daca pe tabla ramn 2 patrate neacoperite de dreptunghiuri sau "figuri", atunci cele doua patrate sunt obligatoriu din S Gabriel Dospinescu, stude Soluie. Numerotm ptratele tablei cu (1, 1), (1, 2), . . . , (8, 9) i t a a s ptratul (k, j) numrul k ij , unde i i sunt rdcinile primitive de ordin 3 a a s a a 4 ale unitaii. Cum 2 + + 1 = 0 i i2 = 1, suma numerelor din orice d s t P sau "figura" va fi 0. Suma tuturor numerelor de pe tabla este P 8
k=1

Daca A Mn (C), avem n n n n n XX X X X hAX, Y i = aij xj yi = aij xj yi = xj aij yi = X, t


i=1 j=1 i,j=1 j=1 i=1

scriind n doua moduri suma numerelor de pe tabla, obinem ca a1 ib1 + a t Notm z1 = a1 ib1 1 , z2 = a2 ib2 1 i vom avea c |z1 | = |z2 | = 1, iar z1 + a s a 1 1 3 3 + = 1 i de aici z1 = z2 = 1. Astfel, i3(b1 1) = i3( s Deducem c a z1 z2 adic b1 1 (mod 4) i b2 1 (mod 4). Cum a1 + a2 = 1, rezult c a1 a s a a resturile 1 i 2 la mparirea cu 3. Vom considera S ca mulimea patra s t t interseciile liniilor 1, 2, 4, 5, 7, 8 cu coloanele 1, 5, 9 i rezolvarea este n t s N t L124. Fie n 2007 fixat. Determinai matricele A Mn (C) pe t A A = In , iar A + A + In = On (cu am notat operaia de conju t Vlad Emanuel, el Soluie. Demonstram nti doua rezultate ajutatoare: t t Lema 1. Daca A A = In , atunci toate valorile proprii ale matric de modul 1. Demonstraie. Pentru X = (xi )n1 , Y = (yi )n1 definim un pro t n P xi yi (proprietaile produsului scalar se verifica complex prin hX, Y i = t
i=1

P 8 k

ij

j=1

1k8,1

= i. Sa presupunem ca au ramas patratele (a1 , b1 )

Fie valoare proprie a lui A, deci AX = X, cu X 6= On,1 . Atunci hAX, AXi = X, t A AX = hX, Xi hX, Xi = hX, Xi . . Lema 2. Fie x C cu |x| = 1 si xn + x + 1 = 0; atunci n 2 . 3. Demonstraie. Fie x = a + bi, cu a, b R, a2 + b2 = 1. Avem: t hX, Xi = hX, Xi = 1 || = 1.

|1 + x| = |xn | = |xn | = 1 (a + 1)2 + b2 = 1 a2 = (a + 1)2 a 3 i apoi gsim c b = . n ambele cazuri x3 = 1, iar 1 + x + x2 = 0, pr s a a 2

L125. Fie f : R R o funcie periodica si lipschitziana (exista L > t care |f (x) f (y)| L |x y|, x, y R), iar (xn )n1 un sir strict cre lim xn = + si lim (xn+1 xn ) = 0. Sa se arate ca mulimea punct t
n n

. xn x2 = 0, de unde xn2 = 1, adic n 2 . 3. a . Revenim la problem. Fie o valoare proprie pentru A; atunci matricea a va avea ca valoare proprie pe 2007 +. nsa det A2007 + A (1) In = d deci 1 este valoare proprie pentru A2007 + A, prin urmare exist C a a 2007 + = 1. Din Lema 1 deducem c || = 1, iar din Lema 2 rezul a . 2007 2 = 2005 . 3, fals. n concluzie, nu exista matrice A cu proprietail . t

ale sirului (f (xn ))n1 coincide cu Im f .

Paul Georgescu i Gabriel P s Soluie. Fie Im f ; din periodicitatea funciei, exista > 0 a t t = f (). Fie T > 0 o perioad a lui f . Notm n = xn xn1 , n a a (n )n2 are limita 0. Pentru k N oarecare, notm cu nk primul indice a care xn + kT ; existena i unicitatea lui nk sunt asigurate de faptul c t s este strict crescator, cu limita +. Cum nk este primul indice cu proprieta atunci xnk + kT + nk . Astfel, xnk Ik , unde Ik = [ + kT, + kT + interval de lungime nk . Din lipschitzianitatea funciei, deducem ca f (xnk ) aparine unui inte t t [uk , vk ]: f este continua, deci transporta compactul Ik n compactul Jk , inf {f (x) | x Ik }, iar vk = sup {f (x) | x Ik }. n plus, lungimea lui pete L nk , iar Jk conine = f (). Pentru k , aplicnd criteriu as s t obinem ca f (xnk ) , ceea ce trebuia demonstrat. t Not. Soluie corect a dat Vlad Emanuel, student, Bucureti. a t a s

Diplom de excelena a t
MARIUS TIBA

acordat de ctre Ministerul Educaiei, Cercetrii i Tineretulu a a t a s

Pentru rezultatele deosebite obinute la olimpiadele internaionale des t t anul colar 2006 - 2007, elevul Marius Tiba, acum n cl. a IX-a, C. N. "C. s din Iai, a fost distins cu o diplom de excelena de ctre M. E. C. T. s a t a Principalele rezultate obinute n perioada menionata sunt: medalia t t la Balcaniada de Matematica - juniori (Soa, iunie 2007), medalia d Olimpiada Naionala de Matematica (aprilie 2007), medalia de aur la t tip O.I.M. (Vlcea, aprilie 2007), diploma "Marele merit" pentru punc obinut la Concursul "Cristian Calude" (Galai, noiembrie 2006), premiu t t cursul "Fl. T. Cmpan", etapa judeeana (Iai, februarie 2007), diploma pen t s deosebite n colaborarea cu revista "Recreaii Matematice". t S mai spunem c, la sfritul anului colar 2006-2007, ca elev n cl. a a a s s primit premiul I pentru rezultate la nvaatura i purtare. t s Redacia revistei "Recreaii Matematice" felicita calduros pe talen t t Marius Tiba si i adreseaza pe acesta cale urari de sanatate deplina si d n egala masura, n anii ce urmeaza.

Probleme propuse1
Clasele primare

P.144. Elevii clasei I intra n clasa n rnd cte unul. Ci elevi sunt n c t Matei este al 12-lea cnd se numara ncepnd din fata i al 16-lea cnd s ncepnd din spate. (Clasa I-a) nv. Eleva Po P.145. Un fluture zboar din floarea 1 n floarea 3, a apoi din aceasta n floarea 5 i aa mai departe (figura 1). s s Dup cte zboruri ajunge n floarea de pe care a plecat? a (Clasa I-a) Evelina Zaporojanu, elev, Iai a s P.146. Dup ce fratele meu mi-a dat un sfert din a merele sale, le-am amestecat cu cele 6 ale mele i pe acess tea le-am aezat pe doua farfurii cu cte 5 mere fiecare. s Cte mere are fratele meu? (Clasa a II-a) Inst. Elena Nua, Iai t s

Figura 1

P.147. Dac numerele ar fi puse corect n cele trei a cercuri, atunci am avea aceeai suma a celor aflate n s fiecare dintre cercuri (figura 2). n cte moduri pot fi aezate corect aceste numere? s (Clasa a II-a) Ctlina Istrate, elev, Iai a a a s Figura 2 P.148. Aflai valoarea a tiind c 10099 : 9998 : 9897 : 97 t s a (Clasa a III-a) Mariana Nastasia, e

P.149. Irina i spune Mioarei: D-mi 2 lei ca s am i eu ct tine! a a s Mioara i rspunde: a Da-mi tu 2 lei ca eu sa am o suma de 2 ori mai mare dect suma ce ie! t Ce sum a avut la nceput fiecare fat? a a (Clasa a III-a)

Inst. Maria R

P.150. Romanul Harry Potter are 7 volume. Stiind c fiecare volum, n a al doilea, are cu 144 pagini mai puin dect dublul numarului de pagini al t precedent, iar al treilea volum are 176 de pagini, aflai cte pagini are ntre t (Clasa a III-a) Robert Vicol, P.151. Descoperii regula de formare a irului 1, 3, 6, 10, 15, 21, . . t s numarul de pe locul 2008. (Clasa a IV-a) Petru Asa P.152. n irul de patrate egale, fiecare pas trat este mprit n ptrate mai mici, dup o at a a anumit regul. a a a) Aratai ca nu exista n acest ir un patrat s t mparit n 23 patrate mai mici; t
1

Se primesc solu ii pn la data de 31 decembrie 2008. t a

b) Artai c exist n ir un ptrat mprit n 2008 ptrate mai mici a t a a s a at a (Clasa a IV-a) Ana Tbcaru, e a a

P.153. O veveria transport nite alune la scorbura sa n 6 ore, ia t a s acelai lucru n 3 ore. n cte ore cele dou veverie ar transporta alunele s a t (Clasa a IV-a) Alexandru-Dumitru Chiriac,

Clasa a V-a

V.88. O secvena de numere este formata din multipli consecutivi ai l t nct suma dintre primul i ultimul numr este 280, iar suma ultimelor do s a este 508. Artai c media aritmetic a tuturor numerelor este termen a a t a a considerate. Mirela M

V.89. Determinai cifrele x, y, z pentru care xy 2 + xz 2 = 168x. t Ioan Scleanu a a

V.90. Fie E (n) = 3n + 5n , n N. Aflai ultimele dou cifre ale numer t a i E (2008). s Mihaela Buc a

V.91. S se arate c 61n , n N , se poate scrie att ca sum, ct i ca a a a s de doua patrate perfecte nenule. Alexandru Negrescu, stud 2 2 2 2 3 4 172 V.92. Demonstrai ca t + + + + > 171. 1 2 3 16 Petru Asa

V.93. Fie A = {2, 3, 4, . . . , 50, 52, 53, 100}. Folosind fiecare element al o singura data, fie ca numarator, fie ca numitor, se scriu 49 de fracii. De t ca macar una dintre aceste fracii este reductibila. t Gabriel P

V.94. Fie A mulimea acelor numere naturale cel mult egale cu 2008, ca t cu 2, dar nu se divid cu 6. Dac scriem elementele lui A n ordine descr a care este al 322-lea numr? a Enache Ptracu a s

Clasa a VI-a
VI.88. Fie a, b, c, d numere raionale pozitive astfel nct t c+1 a+x c+x . S se arate c a a = , x Q+ . d+1 b+x d+x VI.89. Artai c numrul N = 1 a t a a
2007

a c = s b d

Claudiu- tefan P S +2
2007

+ + 20082007 se divid Ctlin Cali a a

VI.90. Sa se determine numerele naturale cu proprietatea ca att rsturnatele lor se scriu ca produs de doi factori primi, fiecare factor a a cifre i fiind rsturnatul celuilalt. s a Temistocle B

VI.91. Considerm numerele scrise n baza 8: a1 = 0, 0 (4)(8) ; a2 = 0 a

divizibil cu 14(10) .

. . . ; an = 0, 0(00 {z . 0 4)(8) . S se arate c numrul N = a a a | .. }


n1

1 1 + + a1 a2

Vasile Chiria

VI.94. Un joc are trei beculee. Primul se aprinde la fiecare dou se t a doilea se aprinde prima dat la o secund dup aprinderea primului, apo a a a trei secunde. Al treilea se aprinde prima dat la a dou aprindere a primu a a fiecare 5 secunde. n primele 10 minute de funcionare, de cte ori cele tr t sunt aprinse simultan? Gabriel P

VI.92. De o parte i de alta a unei drepte AB se considera puncte s \ \ astfel nct 4ABM 4ABN , m(M AN ) + m(M BN ) = 180 , iar [AB] [ S se arate c B este ortocentrul 4AM N . a a Petru Asa b b VI.93. Fie ABCD un patrulater convex cu m(A) = m(B) = 80 i AB s \) = 20 . DA i astfel nct exista F (BC) pentru care m(BAF s a) Demonstrai c 4AF D este echilateral. t a bs b b) Determinai msurile unghiurilor C i D. t a Cristian L

Clasa a VII-a

VII.88. Fie x, y, z numere reale distincte, iar a = (xy) (yz), b = (y c = (zx) (xy). Sa se arate ca exact doua dintre numerele a, b, c sunt iar al treilea este pozitiv. Ovidiu Pop, Sa p VII.89. Determinai cifrele x, y, z pentru care 14xyzx5 Q. t Damian Marinescu, T VII.90. Rezolvai n numere ntregi ecuaia 4x = 5y + 4. t t

Ion Vian s VII.91. Fie a N , a 98, iar n =

1 1 + + a (a + 1) (a + 1) (a + 2) Demonstrai ca n nu poate fi patratul unui numar raional. t t Gheorghe Iu

VII.92. n trapezul ABCD cu baza mare [CD], diagonala BD este b \ unghiului ABC. Perpendiculara n B pe diagonala BD intersecteaz drea a E. S se arate c dreapta CE trece prin mijlocul laturii [AB]. a a Dan Nedeianu, Dr. Tr

VII.93. Pe latura [AB] a triunghiului ABC se considera punctul M m = AM , n = BM . Paralela prin M la AC taie BC n N , iar paralela pri m taie AC n P . Fie S1 = ABMN , S2 = ACN P , S = AABC , iar x = . Sa n S1 + S2 raportul n funcie de x i s se afle x pentru care acest raport e t s a S Adrian Cordune

VII.94. Determinai poligoanele regulate care au proprietatea ca o t

vrfuri ale lor determin un triunghi isoscel. a

Gheorghe Iu

Clasa a VIII-a

VIII.88. Fie A = {1, 3, 5, . . . , 2n 1}, iar S1 i S2 reprezinta suma e s lui A, respectiv suma patratelor elementelor lui A. Sa se determine n care S2 3 |A| S1 . Laureniu Modan, B t 4n2 VIII.89. Demonstrai ca n2 + 1 + n2 + 2 + + n2 + 2n < t n N . Lucian Tuescu t VIII.90. Demonstrai ca mulimea A = x | x = 276n+2 + 312n+5 + t t nu conine numere prime. t Damian Marinescu, T

VIII.91. Se considera funcia f : {1, 2, . . . , 2008} N care asociaza un t n al domeniului, numarul divizorilor sai naturali. a) Determinai n pentru care f (n) = 7. t b) Aflai valoarea maxim a funciei. t a t c) Daca f (n) + f (m) + f (p) = 33, aratai ca macar unul dintre num t sau p este patrat perfect. Monica Ned

VIII.92. S se arate c pentru orice numr ntreg impar n, exist a a a a naturale a i b astfel nct a (a + 2n) = b (b + 2n). s Constantin Apostol, R

VIII.93. Fie ABCDA0 B 0 C 0 D0 un paralelipiped oarecare i O, O0 p s intersecie a diagonalelor bazelor. Se noteaz cu GA i GA0 centrele de gr t a s 4BCD i, respectiv, 4B 0 C 0 D0 i cu A1 mijlocul segmentului [GA GA0 ]. s s GB , GB 0 i B1 etc. se introduc n mod similar. Aratai ca dreptele AA1 , B s t DD1 sunt concurente ntr-un punct P situat pe OO0 i precizai poziia s t t [OO0 ]. Temistocle B

a a a VIII.94. Fie V A1 A2 . . . An o piramid regulat; notm cu P poligonul o n i fie U = m(V M, \ 2 A3 )) | M P . Demonstrai c max U < 2 min t a s (A1 A

Claudiu- tefan P S

Clasa a IX-a

IX.86. Fie O mijlocul ipotenuzei [BC] a triunghiului dreptunghic AB cercului nscris, iar R1 i R2 razele cercurilor cirscumscise triunghiurilor s a2 . spectiv AOC. S se demonstreze c R1 R2 a a 2a + 4r D. M. Btineu-Giurgiu, B a t

IX.87. Demonstrai c ntr-un triunghi ascuitunghic, cu notaiile u t a t t loc inegalitatea

Titu Zvonaru, C IX.90. S se determine toate irurile de numere reale (an )n0 cu prop a s an+m + anm = a3n + n, n, m N. I. V. Maftei, Bucureti i Mihai Ha s s

b c 3 a + + < . b4 + c4 c4 + a4 a4 + b4 4Rrp Gheorghe Molea, Curtea . . IX.88. Demonstrai ca 15 25n + 32 n2 + 120n 15 . 128, n N. t Lucian Tuescu t IX.89. S se arate c pentru orice n N are loc inegalitatea a a ! 1 1 1 1 1 1 1+ + + + > + + + 2 22 52 n+1 n+2 3n 2 2 (3n 1)

Clasa a X-a

X.86. Aflai x, y R pentru care lg2 t +

Determinai f , tiind c exist p, q N numere prime ntre ele astfel nc t s a a fq (x) = 2x, x A. Romeo Ilie X.88. Fie ABCD un paralelogram, iar M i N mijloacele laturilor (BC s tiv (CD). Daca AM = BN i AM BN , aratai ca ABCD este patrat. s t Gheorghe Iu X.89. n planul complex se consider punctele A (3i), B (4), iar M est a variabil de modul 1. a) Determinai locul geometric al punctului N cu proprietatea ca tri t AOB i AM N sunt asemenea i la fel orientate. s s b) Gsii punctele N1 , N2 ale locului ce se plaseaz pe segmentele [BA] a t a [BO], precum i punctelor M1 , M2 din care provin. s Dan Br X.90. Fie X1 , X2 , . . . , Xn variabile aleatoare independente, fiecare lund valorile 1 i 1 cu probabilitile p, respectiv q. Considerm Y = X s at a + Xn . a) Sa se calculeze media i dispersia lui Y . s b) S se precizeze care este valoarea luat de Y cu probabilitate maxim a a Petru M

x x 3 = 3 lg lg . y 3 y A. V. Mihai, B i f : A A o funcie injectiva; notam fn = f X.87. Fie A N s t |

XI.86. Fie n 2N i A Mm (R); artai c numerele det An+ a t a s det (A Im ) au acelai semn. s Romana Ghia i Ioan Gh t t s

Clasa a XI-a

XI.87. Studiai convergena irurilor (an )n1 i (bn )n1 , unde t t s s 2008 + cos n 2009 + cos n , bn = , n N . an = 2008 + cos n + 1 2008 + cos n + 1 Liviu Smarandache XI.88. Fie irul (xn )n1 definit prin: x1 0, s , xn+1 = 2 tg x 2 t t s n N . Studiai existena limitelor lim xn i lim nxn . n n Dan Popescu 1 1 1 + + 2 XI.89. Calculai lim t + 2 1 . n 12 sin 1 2 sin 1 n sin n 2 Silviu B

XI.90. Exista funcii polinomiale p : R R care sa aiba exact n p t distincte a1 , a2 , . . . , an R i astfel nct pentru fiecare 1 j n, ecuaia s t s aib soluie real unic? a a t a a Marian Tetiv

Clasa a XII-a

XII.86. Fie c R , iar f, g : [a, b] R funcii continue astfel nct f (a t + g (x), x [a, b]. S se determine y [a, b] pentru care a Z a+by Z b g(x) f (x) [f (x)] [g (x)] dx = c d g(x) a [f (x)] y + [g (x)]f (x) [f (x)]g(x) + [g (x)]f (x)

D. M. Btineu-Giurgiu, B a t

XII.87. a) Fie f : [0, a] R derivabila cu derivata continua, astfel nc a 2 a R a3 R 0 2 0. S se arate c a a f (t) dt [f (t)] dt. Pentru ce funcii se t 3 0 0 egalitatea? b) Fie f : [0, a] R de dou ori derivabil cu f 00 continu, astfel nc a a a a 2 5 R a R a 2 f 0 (a) = 0. S se arate c a a f (t) dt (f 00 (t)) dt. Pentru ce 20 0 0 realizeaz egalitatea? a Adrian Cordune

XII.89. Sa se arate ca polinomul f = 200X 5 + 196X 4 49X 3 + 35X 2 + este reductibil peste Q. Mihai Ha
2

XII.88. S se arate c irul (an )n1 este convergent i s se afle limit a as s a n 1 P k 2 n2 k + n ln an = , n N . n k=1 n2 k Laureniu Modan, B t

XII.90. Fie H corpul cuaternionilor, iar i, j, k unitile cuaternionice at k = 1, ij = ji = k, jk = kj = i, ki = ik = j). Definim ( b+c 3 b 3 c + 2d 3 3b c 3 2d K= a+ i+ j+ k | a, b, c, d 2 4 4

pe care considerm operaiile uzuale ntre cuaternioni. S se arate c n a t a a obinem un corp necomutativ, izomorf cu corpul cuaternionilor. t Dumitru Mihalach

Probleme pentru pregtirea concursurilo a


A. Nivel gimnazial
G136. Determinai numerele reale x, y, z, pentru care t 2x + 3 2y + 2z = 2x + 3 2y+2 + 2z+2 = 9.

Andrei Ned 2 a+ b c G137. Fie a, b, c Q i = . Sa se exprime n func + s a+ b+ c a b+ c c i numarul real = s . 2 a+ b+ c I. V. Maftei, Bucureti i Mihai Ha s s

G138. a) Numerele reale pozitive a, b, c sunt astfel nct 4abc = a + b2 + c2 c2 + a2 b2 + a2 Sa se arate ca + + 2 (ab + bc + ca). a b c b2 + c2 c2 + a2 + + b) Numerele reale pozitive a, b, c sunt astfel nct a b 2 (ab + bc + ca). Sa se arate ca a + b + c + 1 4abc. Andrei Laureniu Ciupan, elev, B t

G139. Denisa sccrie pe tabla numerele 1, 2, 3, . . . , 2008. Ea alege dou le terge de pe tabl i scrie n loc modulul diferenei lor, repetnd aceast s as t pn cnd pe tabl rmne un singur numr. Poate proceda Denisa n a a a a a s numarul ramas sa fie 2007? Dar 2008? Julieta Grig

G140. Un poligon cu n laturi este mparit n n2 triunghiuri cu ajuto t diagonale ale sale care nu se intersecteaz n puncte interioare (o astfel de a se numete triangulaie a poligonului). Notm cu T0 numrul triunghiurilo s t a a laturi sunt toate diagonale ale poligonului i cu T2 numarul triunghiuril s cte doua laturi care sunt laturi i pentru poligon, iar a treia latura d s poligonului. S se arate c T2 = T0 + 2. a a Marian Tetiv

G141. Se consider o reea de drepte care formeaz prin intersec a t a congruente. Marcam 2n + 1 vrfuri ale unor astfel de patrate, n 2, a orice dreapta din reea sa conina cel mult un punct marcat. Sa se arate t t mcar dou puncte marcate care sunt separate att pe orizontal, ct i pe a a a s de cte un numr impar de drepte ale reelei. a t Petru Asa

G142. Spunem c vrful A al triunghiului ABC are proprietatea (P ) d a BC, X (BC). Sa se arate ca daca fiecare vrf al 4ABC are proprie

atunci triunghiul este echilateral.


0

Doru Bu

G143. Considerm triunghiul ABC, iar D, D sunt puncte pe dreapta a \ ABC, iar BAD0 ACB. Bisectoarele interioare ale unghiu \ \ \ nct CAD \0 taie dreapta BC n E, respectiv F . S se arate c cercul circumsc i CAD a a s i cercul nscris n 4ABC sunt concentrice. s Neculai Roman, Mirce

G144. Fie ABCD un patrulater cu AB = BC. S se arate c m a a \ m(BCD) = 90 daca i numai daca AB 2 CD2 + AD2 BC 2 = AC 2 BD s Ioan Scleanu a a

G145. Se considera triunghiul isoscel ABC cu AB = AC, iar pe arc

BC care nu-l conine pe A al cercului circumscris triunghiului se ia un pu t se arate ca AB AC . MB MC < MA < MB MC + MB MC Gheorghe Costo

B. Nivel liceal

L136. Fie A, B, C trei puncte pe sfera S de centru O, iar M1 i s puncte exterioare sferei astfel nct OM1 i OM2 sa intersecteze planul s dou puncte interioare 4ABC. Dac M1 A M2 A, M1 B M2 B i M1 C a a s s se arate c M1 O M2 O. a a a Ctlin Tigeru a a

L137. Considerm 4ABC nscris n cercul C i fie C1 cercul de centru O a s la AB, BC i la cercul C n M , K, respectiv L. Paralela prin B la M K int s \ dreptele LM i LK n R, respectiv S. S se arate c unghiul RO1 S este a s a a Neculai Roman, Mirce

b b L138. Fie ABC un triunghi cu AB 6= AC, m(A) < 90 , unghiul A fii mare al triunghiului. Notm cu M mijlocul lui [BC] i T punctul de int a s simedianei din A cu mediatoarea lui [BC]. S se arate c 2AM < AT . a a Titu Zvonaru, Comneti i Cristian Pr a s s

L139. Fie A1 A2 An un poligon regulat, iar M un punct variabil n sau pe laturile poligonului. S se determine maximul produsului f (M ) a M A2 M An , precum i punctele M care realizeaza acest maxim, n s cazurile: a) n = 3; b) n = 6. Dumitru Mihalache i Marian Tetiv s

L140. Fie a, b, c R astfel nct (a+b)2+(b+c)2+(c+a)2+(a+b)(b+c) + a b c a+b b+c c+a Sa se arate ca + + + + . bc ca ab c a b Andrei Vrjitoarea, elev a

L141. Daca x, y, z sunt numere reale pozitive cu x3 + y 3 + z 3 = 3, at

y+2 z+2 x+2 + + 3. 2x2 + 1 2y 2 + 1 2z 2 + 1

Titu Zvonaru, Comneti i Nela Cice a s s

L142. Consideram n N , numerele reale strict pozitive a1 < a2 < i A mulimea tuturor sumelor a1 a2 an , unde semnele se ale s t n2 + n + 2 i determinai numerele s t modurile posibile. Artai c |A| > a t a 2 care se atinge egalitatea. Gheorghe Iu

L143. Sa arate a pentru p ar natural prim i m, n {0, 1, . c num s se 2p + m p+m m m > n, avem 2 mod p2 . 2p + n p+n n Marian Tetiv p L144. Fie p N, p 2; definim irurile (xn )n1 i (yn )n1 prin: x1 = s s p xn+1 = p (p 1) + xn , yn = 2n pn1 xn , n N , unde {} desemne fracionar. S se arate c irul (yn ) este strict monoton. t a a as Sorin Pupan s a

convergente i aflai limitele lor. s t

L145. Fie 0 < < ; definim irurile (xn )n0 , (yn )n0 prin x0 = s yn xn R 2 R 2 e t2 dt, yn+1 = e t2 dt, n N. Artai c cele dou a t a a xn+1 =
xn yn

Marius Ape

Training problems for mathematical contes


A. Junior highschool level
G136. Determine the real numbers x, y, z satisfying the equation 2x + 3 2y + 2z = 2x + 3 2y+2 + 2z+2 = 9.

Andrei Ned 2 a+ b c G137. Let us consider a, b, c Q and = . It is r + a + b + c a b+ c express, in terms of a, b, c and , the real number = . 2 a+ b+ c I. V. Maftei, Bucureti and Mihai Ha s

G138. a) The positive real numbers a, b, c satisfy the equation 4abc = a b2 + c2 c2 + a2 b2 + a2 Prove that + + 2 (ab + bc + ca). a b c b2 + c2 + b) The positive real numbers a, b, c satisfy the inequality a 2 2 b +a 2 (ab + bc + ca). Show that a + b + c + 1 4 abc. c Andrei Laureniu Ciupan, highschool student, B t

G139. Denise writes down on the blackboard the numbers 1, 2, 3, . . . , chooses two numbers, deletes them from the blackboard and replaces th modulus of their dierence, repeating this operation until a single numb on the blackboard. Can Denise proceed in such a way that the remaining 2007? What about 2008? Julieta Grig G140. A polygon with n sides is divided into n 2 triangles by means its diagonals that do not cut each other(s) at interior points (such a parti called a triangulation of the polygon). Let us denote by T0 the number o whose sides are all diagonals of the polygon, and by T2 the number of tria two of their sides being sides of the polygon as well, the third side being of the polygon. Prove that T2 = T0 + 2. Marian Tetiv G141. It is considered a network of straight lines that form congrue by intersecting themselves. We mark 2 n + 1 corners of such squares, n any line in the network pass through at most one marked vertex point. at least two marked points exist such that they are separated, both along and vertical directions, by an odd number of lines in the network. Petru Asa G142. We say that the vertex A of the triangle ABC has the prope AX < BC, X (BC). Show that if each vertex of 4ABC enjoys the pro then the triangle is equilateral. Doru Bu G143. We consider the triangle ABC with D, D0 two points on the lin \ \ \ \ that CAD ABC, iar BAD0 ACB. The interior bisectrices of the an 0 cut the line BC at E, respectively F . Show that the circle circ \ and CAD to 4AEF and the circle inscribed in 4ABC are concentric. Neculai Roman, Mirce

G144. Let ABCD be a quadrilateral with AB = BC. Show that m \ = 90 if and only if AB 2 CD2 + AD2 BC 2 = AC 2 BD2 . m(BCD) Ioan Scleanu a a G145. It is considered the isosceles triangle ABC with AB = AC a

M is taken on the open arc BC, which does not contain the corner A, of circumscribed to the triangle. Show that AB AC . MB MC < MA < MB MC + MB MC Gheorghe Costo

B. Highschool level

L136. Let A, B, C be three points on the sphere S of center O, whi M2 are two exterior points with respect to the sphere S such that OM1 intersect the plane (ABC) at two points that are interior to 4ABC. If M1 M1 B M2 B and M1 C M2 C, show that M1 O M2 O. a Ctlin Tigeru a a

L137. We consider 4ABC inscribed in the circle C and let C1 be th center O1 , which is tangent to AB, BC and to the circle C at M , K, and re L. The parallel line through B to M K intersects the lines LM and \ respectively S. Show that the angle RO1 S is acute. Neculai Roman, Mirce b L138. Let ABC be a triangle with AB 6= AC, m(A) < 90 , the ang the largest angle of this triangle. We denote by M the midpoint of [BC] the intersection point of the simedian from A with the mid-perpendicula Show that 2 AM < AT . Titu Zvonaru, Comneti and Cristian Pr a s L139. Fie A1 A2 An be a regular polygon, and M a variable po the polygon or on its sides. Determine the highest value of the product M A1 M A2 M An , as well as the points M at which this maximum in each of the cases: a) n = 3; b) n = 6. Dumitru Mihalache and Marian Tetiv L140. Let a, b, c R+ be numbers such that (a + b)2 + (b + c)2 + a b c a+b b+c c+ (a + b) (b + c) (c + a) = 4. Prove that + + + + bc ca ab c a Andrei Vrjitoarea, highschool student a L141. If x, y, z are real reale positive numbers with x3 + y 3 + z 3 = 3, x+2 y+2 z+2 + 2 + 2 3. 2+1 2x 2y + 1 2z + 1 Titu Zvonaru, Comneti and Nela Cice a s L142. We consider n N , the strictly positive real numbers a1 < a2 and A = the set of all the sums a1 a2 an , where the signs are ch n2 + n + 2 possible ways. Show that |A| > and determine the numbers an 2 the equality is achieved. Gheorghe Iu L143. Show that, for a prime natural number p and m, n {0, 1, . 2p + m p+m m m > n, we have 2 mod p2 . 2p + n p+n n Marian Tetiv L144. Let p N, p 2; we define the sequences (xn )n1 i (yn )n1 s p p n n1 p (p 1), xn+1 = p (p 1) + xn , yn = 2 p xn , n N , where { the decimal (or fractional) part. Show that the sequence (yn ) is strictly m Sorin Pupan s a L145. Let 0 < < ; we define the sequences (xn )n0 , (yn )n0 b yn xn R R 2 2 y0 = 0, xn+1 = e t2 dt, yn+1 = e t2 dt, n N. Show that the two
xn yn

are convergent and find their limits.

Marius Ape

Pagina rezolvitorilor

CRAIOVA Colegiul Naional "Carol I". Clasa a VII-a (prof. STANCA Monica). t Ioan: VII(76-80), VIII(77-80), G(116-123). Not. n nr. 2/2007 a fost omisa localitatea Craiova la care fig a STANCIU Ioan cu 11 probleme rezolvate corect. Ne cerem scuze pentru dent.

HRLAU Liceul "Stefan cel Mare". Clasa a V-a (prof. SACALEANU Ioan). Carla: P(131,132), V(78,80), VI.78; CALINESCU Ana Maria: P(131,132), VI.78; HUTANU Mdlina-Georgiana: P(131,132), V(76,78,80), VI.78; a a Bianca: P.131, V(76,78,80), VI.78; MITITELU Melisa: P(131,132), V(77, NEICU Mara: P.131, V(76-78), VI.78; SAVA Diana Alexandra: P(131,1 78), VI.78. Clasa a VI-a (prof. SACALEANU Ioan). BARAU Larisa-Ion V(76-78), VI.76; BUZILA Andreea: P.131, V(76,77,80), VI.76; CEUCA P.131, V(76,77), VI(76,78); IVANUTA Andreea Simona: P.131, V(76,78), JITARIU Adina-Diana: P.131, V(76,77), VI(76,78); PINTILII Alina: P.131 VI(76,78); PLETAN Denisa Elena: P.131, V(76-78), VI.78.

IA I S Scoala nr. 11 "Otilia Cazimir". Clasa a III-a (inst. HUZUM Lina). Laura: P(134,136-139); HUZUM Andrei: P(135-139); MARIU Simina: TA 139); STOIAN Ioana: P(135-139). Scoala nr. 13 "Alexandru cel Bun". Clasa a IV-a (inst. COJOCAR AGAFITEI Elena-Roxana: P(134-136,138,139); CARAMALAU Andra: P 138,139); CALIN Andreea-Claudia: P(134-136,138,139); COJOCARIU P(134-136,138,139); DUDUMAN Luisa- tefania: P(134-136,138,139); LE S xandrina- tefana: P(134-136,138,139); LUPA CU Diana-Maria: P(134-13 S S MANOLACHE Madalina-Andreea: P(134-136,138,139); MIHAILA Narci P(134-136,138,139); PASCU Gabriela: P(134-136,138,139); PADURARU Stefan: P(134-136,138,139); RADUCEA Marin-Andrei: P(134-136,138,13 Cristina-Simona: P(134-136,138,139); STEFAN Bogdan-Vasile: P(134-136 STIUBEI Cosmin-Ionu: P(134-136,138,139). t Scoala nr. 14 "Gh. Mrzescu". Clasa a III-a (inst. NUTA Elena). B dor: P(134-139); CHIRILUTA George- tefan: P(134-139); POSTUDOR G S Madalina: P(134-139); STOICA Adriana: P(134-139). Scoala nr. 26 "G. Cobuc". Clasa a III-a (inst. RACU Maria). AP s Aura Georgiana: P(134-139); BURA Emma-Andreea: P(134-139); CIOR xandra: P(134-139); CRACIUN Ioana-Daniela: P(134-139); FILIP Ingrid P(134-139); GHEORGHITA Narcis-Eugen: P(134-139); HRISCU Ovidiu-C P(134-139); HUZA Madalina: P(134-139); MARICIUC Drago-Claudi s 139); MAXIM Alexandra-Camelia: P(134-139); TUDOSE Ema-Alina: P TUCA Cosmin: P(134-139); VASILE Bogdan-Andrei: P(134-139). Clas (nv. HRIMIUC Valeria). BRUMA Andrei-Alexandru: P(134-139); DU

Bianca: P(134-139); HARAPCIUC Eduard-Gabriel: P(134-139); MANTA Adrian: P(134-139); NEAMTU Alexandru: P(134-139); OLARU Alexand 139); SANDU Beatrice-Gabriela: P(134-139); VORNICU Sorin: P(134-13 Colegiul Naional - locaia Scoala "Gh. Asachi". Clasa a IV-a (inst. CA t t Rodica). BAJENARU Brdia: P(134-141,143); BERECHET Alexandr a t 141,143); CHIVULESCU Alexandru: P(134-141,143); PETREA Mdlin a a 136,138,139); UNGUREANU George: P(134-141,143).

Colegiul Naional. Clasa a V-a (prof. POPA Gabriel). AMARAN t P.141, V(81-84); CERNAT Radu: P.143, V(81-84); MANGALAGIU Io V(81-84); PURICE Ioana: P(141,142), V(81,82,84); STOLERU Georgi P(141-143), V(81-84); STREANGA Iulia Mdlina: P(141,143), V(81-84) a a VII-a (prof. POPA Gabriel). IONITA Norbert-Traian: V(81-86), VI(82-8

Colegiul Naional "C. Negruzzi". Clasa a VII-a (prof. IONESEI t PAVALOI Alexandru: VI(76,79), VII(76,78,79); Clasa a X-a (prof. ZA Adrian). VLAD Ilinca: VIII(76,77,80), IX(77,78,80), X(76,77). Colegiul Naional "E. Racovia". Clasa a VIII-a (prof. PITU Leon) t t RACHE Alexandru: VIII(81-84,86,87), G131.

SFNTU GHEORGHE Scoala "Avram Iancu". Clasa a VII-a (prof. LZR Emese). FURTU V(76-78,80), VI(77-80), VII(76,78,79).

SIBIU Colegiul Naional "Gh. Lazar". Clasa a XII-a. VLAD Emanuel: X t L(116-122,125).

SUCEAVA Scoala cu clasele I-VIII, nr. 3. Clasa a III-a (nv. TABARCEA FECHET Stefan: P(127-130,132); Clasa a IV-a (inst. NECHITA Dan CHET Mircea: P(125-131).

TRGU NEAMT Colegiul Naional "Stefan cel Mare". Clasa a III-a (nv. ). CEBER t P(125-127,129,134-136,139).

Premii acordate rezolvitorilor


Scoala nr. 13 "Alexandru cel Bun", Iai s

AGAFITEI Elena-Roxana (cl. a IV-a): 1/2007(5pb), 2/2007(5pb), 1/2

CARAMALAU Andra (cl. a IV-a): 1/2007(5pb), 2/2007(5pb), 1/2008

CALIN Andreea-Claudia (cl. a IV-a): 1/2007(5pb), 2/2007(5pb), 1/20

COJOCARIU Andreea (cl. a IV-a): 1/2007(5pb), 2/2007(5pb), 1/2008

DUDUMAN Luisa- tefania (cl. a IV-a): 1/2007(5pb), 2/2007(5pb), 1/ S

LELEU Alexandrina- tefana (cl. a IV-a): 1/2007(5pb), 2/2007(5pb), 1 S

LUPA CU Diana-Maria (cl. a IV-a): 1/2007(5pb), 2/2007(5pb), 1/200 S

MANOLACHE Mdlina-Andreea (cl. a IV-a): 1/2007(5pb), 2/2007 a a 1/2

PASCU Gabriela (cl. a IV-a): 1/2007(5pb), 2/2007(5pb), 1/2008(5pb);

PADURARU Tiberiu- tefan (cl. a IV-a): 1/2007(5pb), 2/2007(5pb), 1 S

RADUCEA Marin-Andrei (cl. a IV-a): 1/2007(5pb), 2/2007(5pb), 1/2

SAVIN Cristina-Simona (cl. a IV-a): 1/2007(5pb), 2/2007(5pb), 1/200

STEFAN Bogdan-Vasile (cl. a IV-a): 1/2007(5pb), 2/2007(5pb), 1/200

STIUBEI Cosmin-Ionu (cl. a IV-a): 1/2007(5pb), 2/2007(5pb), 1/200 t Colegiul Naional - locaia Scoala "Gh. Asachi", Iai t t s

BAJENARU Brdia (cl. a IV-a): 1/2007(7pb), 2/2007(5pb), 1/2008(9 a t

CHIVULESCU Alexandru (cl. a IV-a): 1/2007(6pb), 2/2007(5pb), 1/2 Scoala nr. 26 "G. Cobuc", Iai s s

CIORNEI Alexandra (cl. a III-a): 1/2007(5pb), 2/2007(5pb), 1/2008(6

CRACIUN Ioana-Daniela (cl. a III-a): 1/2007(5pb), 2/2007(5pb), 1/2 TUCA Cosmin (cl. a III-a): 1/2007(5pb), 2/2007(5pb), 1/2008(6pb); Colegiul Naional "Carol I", Craiova t Liceul "Stefan cel Mare", HRLAU NEICU Mara (cl. a V-a): 2/2006(5pb), 1/2007(5pb), 1/2008(5pb);

MARICIUC Drago-Claudiu (cl. a III-a): 1/2007(5pb), 2/2007(5pb), 1 s

STANCIU Ioan (cl. a VII-a): 2/2006(13pb), 1/2007(12pb), 2/2007(14pb

IVANUTA Andreea Simona (cl. a VI-a): 1/2007(5pb), 2/2007(5pb), 1/

Revista semestrial RECREAII MATEMATICE este ed ASOCIAIA RECREAII MATEMATICE. Apare la datele de 1 1 septembrie i se adreseaz elevilor, profesorilor, studenilor i tutu pasionai de matematica elementar. n atenia tuturor colaboratorilor Materialele trimise redaciei spre publicare (note i articole, che metodic, probleme propuse etc.) trebuie prezentate ngrijit, clar i co trebuie s prezinte interes pentru un cerc ct mai larg de cititori. Se reco textele s nu depeasc patru pagini. Evident, ele trebuie s fie origin nu fi aprut sau s fi fost trimise spre publicare altor reviste. Rugm rialele tehnoredactate s fie nsoite de fiierele lor. Problemele destinate rubricilor: Probleme propuse i Problem pregtirea concursurilor vor fi redactate pe foi separate cu enun i de ie/rezolvare (cte una pe fiecare foaie) i vor fi nsoite de numele autoru la i localitatea unde lucreaz/nva. Redacia va decide asupra oportunitii publicrii materialelor prim n atenia elevilor Numele elevilor ce vor trimite redaciei soluii corecte la proble rubricile de Probleme propuse i Probleme pentru pregatirea conc vor fi menionate n Pagina rezolvitorilor. Se va ine seama de regulile: 1. Pot trimite soluii la minimum cinci probleme propuse n prezent i cel anterior al revistei; pe o foaie va fi redactat soluia une probleme. 2. Elevii din clasele VI-XII au dreptul s trimit soluii la pr propuse pentru clasa lor, pentru orice clas mai mare, din dou clase m imediat anterioare. Elevii din clasa a V-a pot trimite soluii la problemel pentru clasele a IV-a, a V-a i orice clas mai mare, iar elevii claselor trimite soluii la problemele propuse pentru oricare din clasele primare i s mai mare. Orice elev poate trimite soluii la problemele de concurs (tip 3. Vor fi menionate urmtoarele date personale: numele i pr clasa, coala i localitatea. 4. Plicul cu probleme rezolvate se va trimite prin pot (sau v direct) la adresa Redaciei: Prof. dr. Temistocle Brsan Str. Aurora, nr. 3, sc. D, ap. 6, 700 474, Iai Jud. IAI E-mail: t_birsan@yahoo.com

CUPRINS

Recreaii tiinifice 125 de ani de la apariie ................................................................. Proiect de reeditare ................................................................................................................

AR TIC O LE I N OTE

L. STRUGARIU, C. STRUGARIU Polinoame Fibonacci, polinoame ciclotomice ........... A. REISNER Submulimi ale unei mulimi finite i matrici binare ................................ S. PUPAN O generalizare a teoremelor Stolz-Cesaro .................................................. M. TETIVA O problem de combinatoric destul de grea .............................................. I. M. MAFTEI, M. HAIVAS Tehnici de stabilire a unor inegaliti geometrice ............ A. IGERU Asupra unei probleme de concurs............................................................... M. DICU, L. TUESCU O clas de inegaliti .................................................................. T. ZVONARU Inegaliti omogene i puin analiz... ................................................. S. BOGA Despre numerele reale algebrice......................................................................... M. BTINEU-GIURGIU, D. M. BTINEU-GIURGIU Asupra unei note din revista "Recreaii matematice"................................ Gh. COSTOVICI Limita unor iruri de numere reale....................................................... M. BENCZE O rafinare a inegalitii dintre media aritmetic i cea logaritmic .........

NO TA ELEVU LUI

T. PDURARIU Estimri de sume .....................................................................................

CHESTIUNI M ETOD IC E

L. MODAN Acurateea limbajului matematic n combinatoric ....................................... F. POPOVICI Asupra unei identiti clasice privind partea ntreag ............................... G. POPA, I. ERDEAN O abordare analitic a unor probleme de geometrie ...............

COR E SPOND EN E

C. J. ZHAO A study of a new geometric inequality ........................................................

CONCUR SUR I I EX AMEN E

Concursul "Recreaii Matematice", Ediia a V-a, Muncel (Iai), ............................................. South Eastern European Mathematical Olympiad for University Students, Cyprus, 2007

PR O BLEME I S O LU I I

Soluiile problemelor propuse n nr. 1/2007 ........................................................................... Soluiile problemelor pentru pregtirea concursurilor din nr. 1/2007 ................................... Probleme propuse ...................................................................................................................... Probleme pentru pregtirea concursurilor................................................................................ Training problems for mathematical contests .........................................................................

Pagina rezolvitorilor................................................................................................................

ISSN 1582 1765

Prof. Constantin Corduneanu la a 75-a aniversa

La 26 iulie a. c., prof. C. Corduneanu mplinete vrsta de 75 ani. N s a 26 iulie 1928 la Iai, domnia sa a urmat coala primar n satul Potngen s s a Movileni, jud. Iai, unde parinii sai erau nvaatori. Studiile liceale le-a f s t t Liceul Militar (Iai i Predeal) n perioada 19401947. Citam dintr-un inte s s d-sale, acordat ziarului "Curierul Romnesc": "Am avut sansa de a studia scoala de elita, unde profesorul de istorie era doctor la Oxford, cel de chimie asemenea doctor n chimie si inginer chimist, cel de geografie era simultan si a universitar, cel de franceza era doctor n filologie, cu stagiu de specializare n F devenind apoi seful catedrei de limba franceza la Universitatea din Iai. Pro s de matematica fusese si cadru universitar (confereniar suplinitor). Cnd am t pragul universitatii ieene, opiunile mele de viitor erau deja clare." n clasa a s t de liceu (1947), a luat premiul I pe ara la concursul anual, pe care l organiza t Gazeta matematica. Dup trecerea bacalaureatului n 1947, se nscrie la secia de matematic a t a de Stiine din Iai, promovnd primii doi ani de studii ntr-un singur an, astfe t s 1950 este declarat absolvent. Fiind un student eminent, n ultimul an de facu fost numit preparator, regulamentul n vigoare la acea vreme permind acest t Despre perioada studeniei i despre profesorii si, precum i despre acti t s a s din cadrul Seminarului Matematic "Al. Myller", spunea: "A fost o alta s vieii mele sa am posibilitatea sa ma formez ca specialist ntr-o atmosfera de t tinuta stiinifica si sa fiu sprijinit, n momente critice ale vieii, de catre pro t t mei" (citat din acelai interviu). Dintr-o scrisoare redactat n urma plec s a a din ara, citam: "Ramnem si noi, cei dinafara, datori fotilor profesori, care t s pregatit att de solid pentru viaa. Toata activitatea noastra este patrunsa de t respectabilitatii faa de bunul nume pe care matematica romneasca l-a dob t continua, credem, sa-l pastreze. Este tributul pe care-l datoram memoriei profe notri, singurii cu adevarat fauritori ai viitorului nostru, n condiii care nu s t tocmai prielnice". A urcat cu rapiditate treptele ierarhiei universitare, n 1965 fiind profesor ntre anii 19541968, a fost, n paralel, cercettor principal i apoi ef de se a s s Institutul de Matematica al Academiei, filiala Iai. n 1961 este distins cu p s Ministerului Educaiei si nvatamntului, iar n 1964 primete premiul "Gh. t s al Academiei. A fost rector al Institutului Pedagogic din Suceava (19641967), decan al Matematica din Iai (19681972) i prorector al Univ. "Al. I. Cuza" (197219 s s 1974, a fost ales membru corespondent al Academiei Romne (secia matema t Desfiinarea Institutului de Matematic i deteriorarea continu a climatul t as a tic, social i cultural din ara noastr l-au determinat s emigreze n str s t a a a (1977). Dupa un an petrecut la Univ. din Kingston (Rhode Island) i altul la s din Knoxville (Tennessee), se stabilete definitiv la Univ. of Texas at Arlington s a funcionat pn la pensionare. i place s spun c a avut 4000 studeni n Ro t a a a a t i aproximativ 3000 n America, perioada activitii sale universitare fiind de s at Activitatea tiinifica a prof. C. Corduneanu s-a desfaurat fara ntre s t s 1

fiind continuat i n prezent. Este autorul a peste 150 de articole, publi a s reviste de mare prestigiu din ara i din strainatate. La Iai, a nfiinat Sem t s s t special de teoria calitativa a ecuaiilor difereniale i integrale, d-sa fiind ini t t s t acestei direcii de cercetare la Iai. Muli tineri s-au format n ambiana t s t t Seminar, fiindu-i recunosctori profesorului lor. a Domeniul de cercetare a prof. C. Corduneanu este teoria ecuaiilor difere t integrale i integro-difereniale, n care a abordat probleme de existena, comp s t t asimptotic, periodicitate, aproape-periodicitate i stabilitate a soluiilor un a s t fel de ecuaii. Merit subliniat introducerea "metodei comparaiei" n teor t a a t bilitaii, studiul sistematic al teoriei admisibilitaii operatorilor integrali, m t t frecveniala n studiul stabilitaii ecuaiilor integrale Volterra etc. Pentru a t t t o idee despre opera matematic a d-sale, amintim doar monograile pe ca a publicat pn n prezent: a 1. Funcii aproape periodice, Ed. Academiei, Bucureti, 1961; t s 2. Almost Periodic Functions, Interscience Publ., New-York, 1968; Chelsea Company, New-York, 1989; 3. Principles of Dierential Equations and Integral Equations, Allyn and Boston, 1971; Chelsea Publ. Company, New-York, 1977; 1988; 4. Integral Equations and Stability of Feedback Systems, Academic Press York, 1973; 5. Integral Equations and Applications, Cambridge Univ. Press, 1991; 6. Functional Equations with Causal Operators, Taylor and Francis, Londo Monografia "Funcii aproape periodice" a fost prima pe plan mondial cons t acestui important subiect. Cele dou ediii n limba englez ale ei au fost com a t a i dezvoltate cu noi capitole; la cea de a doua ediie englez i-au adus cont s t as i renumiii profesori ieeni N. Gheorghiu i V. Barbu. Se cuvine a adauga c s t s s C. Corduneanu a fost primul matematician din Iai, ale crui cri au fost pu s a at la edituri din strintate dac avem n vedere perioada postbelic. Muli mat a a a a t cieni din ar i de peste hotare citeaz i folosesc contribuiile originale ale d t as as t lucrarile lor de specialitate. Dintre cei din ultima categorie amintim pe L. C R. Conti, J. Massera, J. J. Scher, N. Rouche, W. Coppel i Ph. Hartman. s Prof. C. Corduneanu a fost i este membru n comitetul de redacie a s t importante reviste de matematic din ara noastr i din alte ari. Este ed a t a s t revistei Libertas Mathematica fondata n anul 1981 i ajunsa anul acesta la al s lea numr. Este membru al Academiei Romno-Americane de Stiine i Arte a t s fiind chiar preedintele acestei instituii, n perioada 19951998. Este Doctor H s t Causa al universitatilor din Iai, Constana si Braov. Are o prezena activ la s t s t a manifestari tiinifice din ara i din America. Este invitat de numeroase univ t s s t pentru a ine conferine i a prezenta comunicri. t t s a A ncurajat revista Recreaii matematice, susinnd necesitatea unei as t t publicaii, ori de cte ori venea la Iai. t s La a 75-a aniversare, revista Recreaii matematice ureaza acad. prof. C t duneanu "La muli ani si multe succese n activitatea sa viitoare!". t Redacia revistei t 2

A. N. Kolmogorov - 100 de ani de la natere s

Andrei Nicolaevici Kolmogorov este unul dintre cei mai mari matema ai secolului al XX-lea i unul dintre cei mai fertili matematicieni ai tuturo s purilor. n peste 300 de lucrari tiinice, manuale i monograi, Kolmogorov a s s t aproape toate domeniile matematicii, exceptnd teoria numerelor. n lucrar atac numai subiecte fundamentale i care deschid noi domenii de investigaie. a s t neexhaustiv a direciilor cercetrilor sale cuprinde: teoria seriilor trigonom a t a teoria mulimilor, teoria masurii, teoria integrarii, logica constructiva (intuiio t t topologia, teoria aproximarii, teoria probabilitatilor, teoria proceselor stochasti ria informaiei, statistica matematica, sisteme dinamice, teoria automatelor, t algoritmilor, lingvistica matematica, mecanica cereasca, teoria turbulenelor, t difereniale, problema a 13-a a lui Hilbert, balistica, aplicaii ale matematici t t ologie, geologie si cristalizarea metalelor. S-a nscut la 25 aprilie 1903 n oraul Tambov (Rusia). Rmne orfan de a s a nc de la natere. Era de origine aristocrat prin mama sa; tatl su, a s a a a agronom, avea o vasta cultura generala. n 1920 a intrat la Universitatea din M urmnd i absolvind cursurile facultaii de matematica - mecanica. n forma s t A. N. Kolmogorov un rol important a avut seminarul condus de V. V. St consacrat seriilor trigonometrice i coala de analiz matematic creat la M s s a a a de N. N. Luzin. n 1931 devine profesor al acestei universitai. S-a stins din t la Moscova la 20 octombie 1982 dupa o bogata activitate tiinica i didac s t s aproape apte decenii. s n lucrrile sale, A. N. Kolmogorov prezint ntr-o unitate surprinz a a chestiuni din domenii aparent deosebite ale matematicii. nca din studenie a o t rezultatele care au produs o puternica impresie n lumea matematicienilor. n construit un exemplu de serie Fourier - Lebesgue divergent aproape peste to a altul de serie divergent n fiecare punct. Sub inuena lucrrilor lui M. S. Su a t a N. N. Luzin, n acelai an, face un studiu asupra operaiilor cu mulimi introd s t t o clasa foarte larga de operaii. t ncepnd din 1924 interesul lui A. N. Kolmogorov se ndreapt spre a probabilitilor, domeniu n care va deveni o autoritate de necontestat. F at metode noi, n particular aa numita inegalitate a lui Kolmogorov, stabilete c s s necesare i suciente pentru legea numerelor mari i demonstreaza legea s s mului iterat. nc din 1909, E. Borel a neles importana teoriei msurii a t t a construcia fundamentelor teoriei probabilitilor. Ideile sale au fost dezvolt t at A. Lomnicki ntr-un articol din 1923 i au devenit obiectul cercetarilor lu s mogorov n 1929 care, n lucrarea "Teoria generala a masurii si calculul p litatilor", propune primul sistem axiomatic al teoriei probabilitilor funda at pe teoria msurii i teoria funciilor de variabil real. Aceast axiomatizare a s t a a a forma nala, acceptata astazi unanim, n monograa "Noiunile fundament t teoriei probabilitatilor" aparuta n 1933, editura Springer. Referindu-se la lu "Metode analitice n teoria probabilitialor" (1931), unul dintre elevii strluci t a t Kolmogorov, B. V. Gnedenko, afirm: "Teoria actuala a proceselor aleatoare M a sau, aa cum le numte Kolmogorov, proceselor fara postaciune, a fost fundam s s t 3

de Kolmogorov n aceasta lucrare. A aparut un domeniu al matematicii cu apli zica, biologie, chimie, activitatea inginereasca. Gndirea lui Kolmogorov a a n diverse domenii n care a pus probleme noi si a dat raspunsuri unor ch principiale". Pentru preocuprile sale de teoria informaiei unii matematicieni l-au nu a t A. N. Kolmogorov "Newton al secolului XX". n teoria algoritmica a info a introdus noiunea de -entropie a unei mulimi dintr-un spaiu metric, c t t t aplicaii n probleme privind superpoziiile (compunerile) de funcii i i-a per t t t s revin asupra problemei a 13-a a lui Hilbert: "O funcie continua de trei va a t poate fi reprezentata ca o superpoziie de funcii continue de doua variabile t t 1956 Kolmogorov a demonstrat ca orice funcie continua de mai multe varia t reprezint ca o superpoziie de funcii continue de trei variabile. n 1957, V. I. A a t t probabil cel mai renumit dintre elevii lui Kolmogorov, a demonstrat c orice a continua de trei variabile se reprezinta ca o superpoziie de funii continue d t t variabile. n acelai an, Kolmogorov arata ca o funcie continua de un numar o s t de variabile se reprezint ca o superpoziie de funcii continue de o variabil r a t t a care se adaug operaia de adunare (funcie continu de dou variabile). Cre a t t a a teoriei algoritmice a informaiei, Kolmogorov a introdus noiunea centrala a t t teorii, aceea de complexitate a unui obiect matematic (numit astzi compl a a Kolmogorov ). n 1925 A. N. Kolmogorov a publicat un articol despre legea terului t Acest articol a intrat n fondul de aur al logicii matematice fiind prima lucr plan mondial a logicii matematice. n 1932 dezvolt semantica logicii intuiio a t lui A. Heyting dnd acesteia aspectul de logica constructiva. n 1952 a dat de cele mai generale ale noiunilor de obiect constructiv i algoritm. n anii 60 a t s sa atraga un numar mare de cercetatorii n studiul limbii i literaturii prin m s matematice. A. N. Kolmogorov a creat puternice coli de cercetare matematic d s a s-au ridicat un numr impresionant de matematicieni de mare valoare, dar n a timp a acordat o atenie deosebita i nvaamntului preuniversitar. Din in t s t sa a fost nfiinat la Moscova coala internat numrul 18 (cunoscut sub t a s a a de "coala Kolmogorov"), un liceu de matematic unde erau selectai copiii ta s a t la matematic, care se remarcau la olimpiade, de pe teritoriul ntregii foste a Sovietice. A fost membru al Academiei de Stiine din tara sa (1939), al academii t Olanda, Anglia, Frana, SUA, Germania, Polonia, India, Romnia (1956 t doctor honoris causa al multor universiti din ntreaga lume, membru de on at unor societai prestigioase. A fost laureat al premiului de stat i de apte ori s s t al premiului Lenin, cel mai prestigios premiu din fosta Uniune Sovietic. a n 1963 a primit cea mai nalt distincie internaional care se acord m a t t a a ticienilor, premiul Bolzano.

Prof. dr. Petru MINU

Inegaliti pentru mediane, bimediane, bisectoa at


Dan Stefan MARINESCU si Viorel CORNEA1
Vom demonstra pentru nceput urmatoarea Lem. Fie ABC un triunghi si M (BC) astfel nct a BM = k (0, 1). Atunci AM < k AC + (1 k) AB. BC Demonstraie. Fie N (AB) astfel nct M N k AC t (fig. 1). Din teorema fundamentala a asemanarii obinem t M N = k AC i AN = (1 k) AB. Aplicnd n triunghiul s AM N inegalitatea triunghiului avem AM < M N + AN sau, conform celor de mai sus, AM < k AC + (1 k) AB.
A

M Fig. 1

Propoziia 1. Fie ABCD un tetraedru, M int (BCD), N (CM t BP BN = u si = v, atunci P (DM BC. Daca ND PC v u 1 AB + AC + AD. AM < u+v+1 u+v+1 u+v+1

Demonstraie. Fie Q (BM CD (fig. 2). n baza teoremei lui van Au t BM BN BP BM u+v = + = u + v, de unde = . MQ ND PC BQ u+v+1 Tinnd seama de (1) i de Lem, n 4ABQ avem s a 1 u+v AM < AB + AQ. u+v+1 u+v+1 Din teorema lui Ceva, aplicata n 4BCD, obinem t CQ u CQ u = , adica = . Aplicnd iarai Lema s QD v CD u+v n 4ACD vom avea v u AQ < AC + AD. (3) u+v u+v Relaiile (2) i (3) conduc la t s 1 v u AM < AB + AC + AD, B u+v+1 u+v+1 u+v+1 ceea ce ncheie demonstraia acestei propoziii. t t Cteva cazuri particulare ale acestui rezultat prezint interes n sine. a

N M P C Fig. 2

Corolarul 1 (Inegalitatea medianei). Fie ABCD un tetraedru si GA cen 1 greutate al feei BCD; atunci AGA < (AB + AC + AD). t 3
1

Profesori, Liceul Teoretic "Iancu de Hunedoara", Hunedoara

Demonstraie. Evident, n acest caz u = v = 1 i aplicnd Propoziia 1 o t s t inegalitatea ceruta.

Corolarul 2. Medianele unui tetraedru pot fi lungimile laturilor unui patr Demonstraie. Fie GA , GB , GC , GD centrele de greutate ale feelor t t ACD, ABD, ABC n tetraedrul ABCD i G centrul su de greutate. Aplic s a 1 tetraedrul GBCD inegalitatea din Corolarul 1, avem GGA < (GB + GC + 3 Conform unui rezultat cunoscut 3 3 3 BG = BGB , CG = CGC , DG = DGD . 4 4 4 1 1 Ca urmare, GGA < (BGB + CGC + DGD ). Cum GGA = AGA , ga 4 4 AGA < BGB + CGC + DGD . Procednd la fel gsim i inegaliti similare, ceea ce asigur faptul c me a s at a a tetraedrului pot fi laturile unui patrulater.

Corolarul 3 (Inegalitatea bisectoarei). Fie ABCD un tetraedru, AE bise triedrului cu vrful n A, E int (BCD); atunci SC SD SB AB + AC + AD. AE < SB + SC + SD SB + SC + SD SB + SC + SD Demonstraie. Din teorema planului bisector (AE este intersecia plane t t sectoare ale diedrelor ce compun triedrul cu vrful n A), avem (fig. 2, cu M BN BP SD SC i v = u= . Aplicnd Propoziia 1, vom obine: t t s = = ND SB PC SB SB SC SD AE < AB + AC + AD, SB + SC + SD SB + SC + SD SB + SC + SD adic inegalitatea din enun. a t

Corolarul 4. Fie ABCD un tetraedru si IA centrul cercului nscris n triu BCD; atunci: CD BD BC AIA < AB + AC + A BC + CD + BD BC + CD + BD BC + CD + BD Demonstraie. n fig. 2 consideram M IA ; din teorema bisectoarei t BN BC BP BD i v = u= = s = . Se aplic Propoziia 1. a t ND CD PC CD Observaie. Propoziia 1 se poate demonstra i cu ajutorul relaiei lui S t t s t care permite determinarea lui AM n funcie de AB, AC, AD i u, v. t s

AM Propoziia 2. Fie ABCD un tetraedru, M (AB) astfel nct t AB CN N (CD) astfel nct = 1 u; atunci au loc CD |u BC (1 u) AD| < M N < u BC + (1 u) AD, |u BD (1 u) AC| < M N < u BD + (1 u) AC. Demonstraie. Fie P (AC) astfel nct M P k BC (fig. 3). Din te t CN PC = 1 u. Cum = fundamentala a asemanarii avem M P = u BC i s AC CD 6

vom obine c P N k AD i n consecina, n baza teot a s t remei pomenite mai sus, vom avea P N = (1 u) AD. Deoarece punctele M , P , N nu pot fi coliniare, din inegalitile triunghiului obinem at t sau, dup nlocuiri, avem a |M P P N | < M N < M P + P N
B

A M

Procednd la fel obinem i al doilea grup de inegaliti. t s at

|u BC (1 u) AD| < M N < u BC + (1 u) AD.

N C Fig. 3

Corolarul 5 (Inegalitile bimedianei). Fie ABCD un tetraedru, M mijlo at [AB] si N mijlocul lui [CD]; atunci

|BC AD| < 2M N < BC + AD, |BD AC| < 2M N < BD + AC. 1 t t Demonstraie. Lund u = n Propoziia 2, gasim inegalitaile enunat t t 2 Pentru inegalitaile care urmeaza avem nevoie de urmatoarea t

Lem. Fie ABCD un tetraedru. Cu conveniile din Propoziia 1 (fig. 2), a t t egalitatea: 1 v u AB 2 + AC 2 + AD2 AM 2 = 1+u+v 1+u+v 1+u+v u uv v 2 2 2 2 BC 2 BD 2 CD . (1 + u + v) (1 + u + v) (1 + u + v) BM u+v CQ i Demonstraie. Ca n Propoziia 1, obinem t t t = s = BQ 1+u+v CD Aplicnd relaia lui Stewart n triunghiul ABQ, avem: t 1 u+v u+v 2 AB 2 + AQ2 AM 2 = 2 BQ . 1+u+v 1+u+v (1 + u + v) Aceeai relaie aplicat n triunghiurile BCD i ACD conduce la s t a s v u uv BQ2 = CD2 , BC 2 + BD2 u+v u+v (u + v)2 v u uv 2 AQ2 = AC 2 + AD2 2 CD . u+v u+v (u + v) Din (1), (2) i (3) obinem egalitatea din enun. s t t Propoziia 3. Fie ABCD un tetraedru. Are loc inegalitatea t 1 v u 1 2 2 2 AM AB + AC + AD , 2R 1 + u + v 1+u+v 1+u+v

unde R este raza sferei circumscrise tetraedrului. Demonstraie. Fie A0 punctul n care (AM intersecteaz a doua oar t a circumscrisa tetraedrului. Avem evident 2R AM AA0 AM = (AM + A0 M ) AM = AM 2 + A0 M AM. 7

Puterea punctului M faa de sfer este egal cu t a a 0 A M AM = R2 OM 2 , unde O este centrul sferei. Din lema, vom avea: 1 v u 2 2 AM = AB + AC 2 + AD2 1+u+v 1+u+v 1+u+v u uv v 2 2 CD2 , 2 BC 2 BD (1 + u + v) (1 + u + v) (1 + u + v)2 1 v u OM 2 = OB 2 + OC 2 + OD2 1+u+v 1+u+v 1+u+v u uv v 2 2 CD2 . 2 BC 2 BD (1 + u + v) (1 + u + v) (1 + u + v)2 Cum OB = OC = OD = R, ultima egalitate devine: v u uv 2 2 2 OM 2 = R2 2 BC 2 BD 2 CD . (1 + u + v) (1 + u + v) (1 + u + v) Din (1), (2), (3) i (4) obinem inegalitatea din enun. s t t Corolarul 6. Fie ABCD un tetraedru si GA centrul de greutate al feei t atunci 1 AB 2 + AC 2 + AD2 . AGA 6R Demonstraie. n Propoziia 3 considerm M GA (u = v = 1). t t a

n cele ce urmeaza, daca ABCD este un tetraedru, vom nota cu mX , h mediana, nlimea, bisectoarea din vrful X (X {A, B, C, D}). at Corolarul 7. n orice tetraedru ABCD avem 3R (mA + mB + mC + mD ) a2 + b2 + c2 + l2 + m2 + n2 , unde a = BC, b = CA, c = AB, l = DA, m = DB, n = DC, iar mA = AGA Demonstraie. Din Corolarul 6 avem 6R mA c2 +b2 +l2 , 6R mB c2 +a t 6R mC a2 + b2 + n2 , 6R mD l2 + m2 + n2 , de unde, prin sumare, o inegalitatea ceruta. Corolarul 8. Fie ABCD un tetraedru si IA punctul n care bisectoarea d tetraedrului intersecteaza faa BCD; atunci t SC SD SB AB 2 + AC 2 + A 2R AIA SB + SC + SD SB + SC + SD SB + SC + SD SC SD i v = . s Demonstraie. n Propoziia 3 se pune u = t t SB SB Observaie. Ca i n Corolarul 7 se pot obine inegaliti pentru suma t s t at toarelor i suma nalimilor. s t

Bibliografie 1. D. Brnzei, S. Ania, C. Cocea - Planul si spaiul euclidian, Editura Aca t t Bucureti, 1986. s 2. D. St. Marinescu - Inegalitati pentru ceviene, R.M.C. 12, 19951996, 57. 3. N. Pavelescu, M. Lascu - Inegalitati n triunghiuri si tetraedre, G.M. 10 362366. 8

Proprieti de coliniaritate n patrulatere at


Temistocle BRSAN 1

n scopul stabilirii proprietailor de mai jos vom utiliza urmatoarele trei l t A Lema 1. Fie ABC un triunghi, X (BC), Y (CA) si {Z} = AX BY . Are loc relaia t Y BX AC BZ = . (1) ZY XC AY Z Demonstraie. Conform Teoremei lui Menelaus, t aplicata triunghiului BCY i transversalei XZ, avem s B X BX AC ZY = 1, de unde deducem (1). Fig. 1 XC AY BZ Lema 2 [1, p. 65]. Fie ABCD un patrulater convex si {I} = AC BD sideram punctele E (AB) si F (CD) cu poziiile date de rapoartele p = t FD respectiv q = . Atunci FC I EF IA ID = pq IB IC. Demonstraie. Presupunem c AB i CD t a s F se intersecteaza n S. Notnd {I 0 } = EF AC D S 00 i {I } = EF BD, obinem relaiile: s t t I I' ES I 0 A F C I'' 0 = 1 (4SAC i transversala EF ), s EA I C F S A 00 ES I B F D = 1 (4SBD i transversala EF ), s E EB I 00 D F S de unde, prin egalare, Fig. 2 () I 0 A I 00 D = pq I 00 B I 0 C.
()

Atunci, avem I EF I 0 = I 00 = I IA ID = pq IB IC. Reciproc, I 0 A I 00 D () IA ID IA ID = pq IB IC = 0 00 I 0 = I 00 = I IC IB IC I B (ntr-adevr, dac I 0 6= I, atunci i I 00 6= I i am avea, n poziia din figur a a s s t a I 0 A ID I 00 D IA < 0 , < 00 ). EF , c a IC I C IB I B Menionm c egalitatea () se obine prin asemnare de triunghiuri, dac t a a t a a CD, restul demonstraiei ramnnd acelai. t s Lema 3. n condiiile Lemei 2 au loc relaiile t t q + 1 IB 1 q + 1 IA IE =p = . (3) IF p + 1 ID q p + 1 IC Demonstraie. n 4ECD scriem relaia lui van t t Aubel: IE JE KE = + , IF JD KC
1

D I J A E

K
Fig. 3

Prof. dr., Catedra de Matematic, Univ. Tehnic "Gh. Asachi", Iai a a s

V Fie ABCD un patrulater convex i s {I} = AC BD. Considerm un punct M D a L T H (AI) i adoptm notaiile:{X} = BM AD, s a t P H' Q {Y } = DM AB, apoi {N } = CY BD, I T' {Q} = CX BD i, n sfrit {U } = AN BC, s s K' S' M {V } = AQ CD (fig. 4). L' X N G' Consideram poziia punctului M pe diagot S K G nala AC data de raportul Y A AM . (4) = Fig. 4 MI Cu uurina putem preciza poziiile punctelor X, Y , N , Q, U i V func s t t s t i elementele patrulaterului. ntr-adevr, aplicm Lema 1 de dou ori n triu s a a a AX BD AM AY DB AM i = s = , adica 4ABD i obinem relaiile: s t t MI XD BI MI Y B DI IB AY ID AX i = s = . XD BD YB BD Pentru determinarea poziiilor punctelor N i Q aplicam Lema 1 n 4CAB i 4 t s s BN BY CA DQ DX CA i i, innd seama de (5), vom obinem t = s = s t NI Y A CI QI XA CI BN 1 AC BD DQ 1 AC BD = , = . NI IC ID QI IB IC BU BN = Pentru punctele U i V aplicm Lema 1 n 4ABC i 4ADC: s a s NI UC DV AC DQ = . Din aceste relaii i (6) rezult c t s a a QI V C AI 1 BD IA DV 1 BD IA BU = , = . UC IC ID VC IB IC Punem acum n evidena cteva proprietai de coliniaritate ale conguraiei t t t

unde {J} = ED AC i {K} = EC BD. Conform Lemei 1, avem s ID AB CK IC AB DJ = (4ABD) i s = (4BCA) . JE IB AE KE IA BE Ca urmare, obinem: t IB AE IA BE p IB 1 IA IE = + = + = IF ID AB IC AB p + 1 ID p + 1 IC 1 p IB IC + IA ID = . p+1 IC ID p (q + 1) IB IE = . A Deoarece IA ID = pq IB IC (Lema 2), rezult c a a IF p+1 ID egalitate rezult din prima i relaia IA ID = pq IB IC. a s t

Propoziia 1. Punctul P definit de {P } = BV DU se afla pe diagonal t Demonstraie. A arata ca P AC revine la a arata ca n 4BCD ce t DU , BV i CI sunt concurente. Datorita relaiilor (7), avem s t IB IC ID BU CV DI 1 BD IA = =1 U C V D IB IC ID BD IA IB 10

i aplicm reciproca teoremei lui Ceva. s a

Observaie. Octogonul stelat AY BU CV DX, generat pornind de la p t M , este nscris n patrulaterul convex dat avnd patru din vrfurile sale vrfurile patrulaterului, iar celelalte patru vrfuri, ce alterneaza cu acestea situate pe laturile patrulaterului. Propoziia precedenta i cele care vor urma t s evidena faptul c punctul I (de intersecie a diagonalelor patrulaterului) jo t a t rol important pentru acest octogon stelat.

Propoziia 2. Punctele din fiecare dintre tripletele X, I, U si Y , I, t coliniare. Demonstraie. Conform Lemei 2, pentru ca punctele X, I, U sa fie co t este suficient ca egalitatea urmatoare sa fie ndeplinita: XD U C ID IC = IA IB. XA U B Dar, innd sema de (5) i (7), egalitatea devine t s IC ID 1 BD IA IB 1 = 1. ID IC = IB BD IA La fel se arat c Y , I, V sunt coliniare. a a

Observaie. Triunghiurile XBC i U DA sunt omologice i punctul I este t s s lor de omologie. Conform teoremei lui Desargue, perechile de drepte (BC (XB, U D) i (XC, U A) au puncte de intersecie coliniare. Observaie similara s t t la triunghiurile Y CD i V AB. s

Introducerea punctelor G, G0 , H, H 0 , K, K 0 , L, L0 , S, S 0 , T , T 0 rezulta din

Propoziia 3. Punctele G, I, H sunt coliniare. Aceeai proprietate o t s punctele G0 , I, H 0 . Demonstraie. Conform Lemei 2, aplicat patrulaterului Y BV D i pu t a s G i H, pentru coliniaritatea punctelor G, I, H este suficient s artm c s a aa a GD HV ID IV = IY IB. GY HB Cu Lema 3, aplicat relativ la patrulaterul ABCD i punctele Y i V , avem a s s q + 1 IB AY DV IY i q = =p , unde p = s . IV p + 1 ID YB VC 1 BD IA ID i, dup calcule, urmeaz ,q= s a Din (5) i (7), avem c p = s a BD IB IC IB IC + IA BD IY = . IV IC ( ID + BD) Pe de alt parte, utiliznd Lema 1 n 4ADB i 4CBD, obinem a s t N D AB HB QB CD GD i = s = . GY N B AY HV QD CV Din nou apelnd la relaiile (5) i (7), gsim t s a AB ID + BD CD IB IC + IA BD = , = . AY ID CV IB IC 11

nlocuind factorii prezeni n (8) cu expresiile lor date de (9) (12) se aju t egalitatea 1 = 1. n concluzie, punctele G, I, H sunt coliniare. Similar se ob i punctele G0 , I, H 0 sunt coliniare. s

N D QB i s prin elementele patrulaterului dat, le N B QD ND NI ID QB QI IB i apoi folosim r mai nti sub forma = + , = + s NB NB N B QD QD QD (6); se obine t ND ID (IC + AC) QB IB (IC + AC) i = s = . NB IB AC QD ID AC Pentru a exprima rapoartele

n aceeai maniera, adica folosind lemele i formulele (4) (7), se dove s s rezultatele urmtoare. a

Propoziia 4. Punctele K, I, L sunt coliniare. Sunt coliniare de asem t punctele K 0 , I, L0 . Propoziia 5. Punctele S, I, T si punctele S 0 , I, T 0 sunt triplete de t coliniare. Observaie. ntruct proprietile precedente sunt de natur proiectiv t at a a lipsit de interes stabilirea acestora cu mijloacele geometriei proiective. a

Pentru configuraia n discuie pot fi puse n evidena i proprieti de alt n t t t s at a

Propoziia 6. Daca patrulaterul ABCD dat este paralelogram, atunci t laterul XY U V este un paralelogram cu laturile paralele cu diagonalele cel Armaia reciproca este de asemenea adevarata (fig. 4). t Demonstraie. Faptul c ABCD este paralelogram este echivalent cu t a

IA = IC i IB = ID. s AX AY Din (13) i (5) deducem c s a = , adic XY k BD; din (13) i (7) a s XD YB BU DV AX CV ca = , adica U V k BD. La fel deducem ca = ((13), (5) UC VC XD VD AY CU i s = ((13), (5) i (7)); ca urmare, XV k AC i Y U k AC. n con s s YB UB XY U V este paralelogram. Afirmaia reciproca se dovedete pe cale inversa. t s

Corolar. Dndu-se un paralelogram ABCD si un punct M (AD), construiasca numai cu rigla (negradata) un paralelogram nscris n acesta si punctul X ca unul dintre vrfurile sale. Soluie. Cu rigla construim punctul M ca intersecie a dreptelor BX t t Pornind de la M construim cu rigla punctele Y , U , V aa cum s-a proce s nceputul acestei note. Conform Propoziiei 6, XY U V ndeplinete condiiile t s t

Bibliografie 1. D. Mihalca, I. Chiescu, M. Chiria - Geometria patrulaterului, Teora, 1 t t

12

O construcie geometric a unor medii t a


Claudiu Stefan POPA1

Se tie ca n orice trapez, lungimea liniei mijlocii este media aritmetica a lun s bazelor, iar lungimea segmentului care se sprijina pe laturile neparalele, tre intersecia diagonalelor i este paralel cu bazele, este media armonic a lun t s a bazelor. De asemenea, o paralel la baze ce mparte o latur neparalel n ra a a a m a a , m, n N , determin n interiorul trapezului un segment a crui lungim n media ponderat a bazelor cu ponderile m i n (toate aceste rezultate pot fi a s spre exemplu, n [2]). n lucrarea [1] se demonstreaza ca un segment paralel cu bazele i av s lungime media geometrica a acestora este situat "ntre" linia mijlocie i seg s paralel cu bazele ce trece prin intersecia diagonalelor. Ne propunem n con t s dm o construcie efectiv a segmentului paralel cu bazele, de lungime me a a t a ometrica a acestora, bazndu-ne pe un rezultat interesant n sine i pe care a s nu l-a mai ntlnit n literatura de specialitate.

Propoziie. Fie ABCD un trapez, AB k CD, {O} = AC BD si fie S t R (CD) astfel nct RS k AD. Atunci O RS daca si numai daca AS este geometrica a lungimilor segmentelor [CR] si [BS]. Demonstraie. S presupunem c O RS i fie t a a s D R C P (AD), Q (BC) astfel ca P Q k AB, O (P Q). Se tie ca P O = OQ i atunci OQRD i OQSA sunt s s s O paralelograme (OQ k DR k AS, OQ = DR = AS). P Urmeaz c RQ k BD, SQ k AC. Aplicnd teoa a rema lui Thales n 4CDB i 4BAC, obinem c s t a BQ BS DR i, cum DR = SA, rezult c = = s a a RC QC SA SA2 = CR BS. Reciproc, presupunem c RD = SA = SB a RC. S A Notm SB = a, RC = b i atunci AB = a + ab, a s Fig. 1 CD = b+ ab. Deoarece lungimea segmentului paralelel cu bazele ce trece prin intersecia diagonalelor este media armonic a lun t a bazelor, avem c a 2 a + ab b + ab a b a+ b AB CD PQ = 2 =2 =2 =2 2 AB + CD a + b + 2 ab a+ b

prin urmare P O = OQ = AS = DR, adica P ORD i P OSA sunt paralelo s De aici, OR k AD, OS k AD, deci O RS.

Drept consecina a acestui rezultat obinem un procedeu de construcie a t t t geometrice si a mediei patratice ale bazelor unui trapez, ca segmente ce se spri laturile neparalele si sunt paralele cu bazele.
1

Profesor, S coala "Alecu Russo", Iai s

13

Fie, ca n gur, [P Q] i [M N ] segmentele ce a s reprezinta media armonica, respectiv media arit metica ale bazelor trapezului ABCD. Obinem t un segment avnd ca lungime media geometric a a bazelor, folosind unul dintre procedeele cunoscute ([2]). Aezam acest segment n prelungirea uneia s dintre bazele trapezului; punctul D0 astfel obinut t l unim cu B i fie {E} = BD0 AD. Paralela s prin E la baze determin segmentul [EF ] cutat. a a

D'

D P E O

M'

M K

X Fie XY simetrica dreptei P Q faa de M N , cu t X AD, Y BC. Atunci 2 AB CD XY = 2 M N P Q = AB + CD = A AB + CD 2 2 2 Fig. 2 (AB + CD) 2 AB CD AB + CD = = AB + CD AB + CD s r AB 2 + CD2 AB 2 + CD2 AB + CD = = XY M N . 2 AB + CD 2

Prin urmare, construcia cu rigorile impuse iniial a segmentului de lungime t t patratica a bazelor trapezului ABCD se reduce la construcia segmentulu t avnd ca lungime media geometrica a bazelor trapezului XY M N (fig. 2).

Propunem spre rezolvare urmatoarele probleme: 1. Fie ABCD trapez cu [AB] baza mare i A0 (AB), D0 (DC astfe s 0 0 0 s a a C (DD ) i AA = DD = AB CD. S se demonstreze c: i) A0 C, BD0 i AD sunt drepte concurente; s ii) a E este punctul de concurena de la i) i EF k AB, F BC, dac t s EF = AB CD. 2. Fie ABCD un trapez, AB k CD i punctele E (AD) i F BC s s nct EF k AB. Trapezele ABF E i EF CD au diagonalele respectiv parale s i numai dac EF = AB CD. s a 3. Fie trapezul ABCD cu baza mare AB, AC BD = {O}, {M, E, P } {N, F, Q} (BC) astfel nct [M N ] este linia mijlocie a trapezului, EF k AABY X , EF 2 = AB CD, iar P Q k AB, O P Q. Daca notam RXY = ACDXY 2 REF = RMN RP Q . Bibliografie.

1. L. Constantinescu - O interpretare geometrica a inegalitatii mediilor, R. M. 1/1982. 2. J. Hadamard - Geometrie plana, Ed. Tehnica, Bucureti, 1960. s

14

Irelevana primitivabilitii pentru ecuaii t at t funcionale de forma f (x + y) = g (f (x) , f (y)) t


Dan Stefan MARINESCU si Viorel CORNEA1
n [1] este demonstrat urmtorul rezultat: a

"Daca f : R R admite primitive si f (x + y) = f (x) + f (y) pentru x, y R, atunci f este continua". f. n cele ce urmeaz vom dovedi urmtoarea a a

De remarcat c demonstraia se bazeaz pregnant pe primitivabilitatea f a t a

Propoziie. Fie g : R R o funcie arbitrara si f : R R o func t t t admite primitive. Daca f (x + y) = g (f (x) , f (x)) , x, y R,

atunci f este continua. Demonstraie. Daca f este injectiva, mpreuna cu faptul ca f are propr t lui Darboux, suntem condui la monotonia funciei f pe R. Se deduce imedi s t tinuitatea funciei f , ceea ce ncheie demonstraia. t t Dac f nu este injectiv, vom dovedi c f este constant pe R i, n cons a a a a s f este continua. Din faptul ca f nu este injectiva exista a, b R cu a f (a) = f (b). Artm c aa a

> 0, t1 , t2 R cu 0 < t2 t1 < i f (t1 ) = f (t2 ) . s ba Pentru aceasta fie n N nct < 1 i funcia s t n ba f (x) , x R. h : R R, h (x) = f x + n Evident, h are proprietatea lui Darboux: h = G0 , unde G : R R, G ba F x+ F (x), x R, cu F o primitiva a lui f . n Dac h nu se anuleaz pe R, cum h are proprietatea lui Darboux, avem h ( a a sau h (x) > 0, x R. Fie h (x) > 0, x R; atunci ba ba ba > 0, h a + 2 > 0, . . . , h a + (n 1) h (a) > 0, h a + n n n de unde ba f a+ f (a) > 0, n ba ba > 0, f a+2 f a+ n n ......................................................
1

Profesori, Liceul Teoretic "Iancu de Hunedoara", Hunedoara

15

ba ba > 0, a+n f a + (n 1) n n relaii care adunate conduc la f (b) > f (a), ceea ce este fals. t ba n consecina, exista c R astfel ca h (0) = 0, adica f c + t = f (c) n ba ba t1 = c, t2 = c + avem f (t1 ) = f (t2 ) cu 0 < t2 t1 = < , adica ( n n dovedit. a t Aratam n continuare ca t2 t1 este perioada a funciei f . n adevar, f f (x + t2 t1 ) = f (x t1 + t2 ) = g (f (x t1 ) , f (t2 )) = = g (f (x t1 ) , f (t1 )) = f (x t1 + t1 ) = f (x) .

Din aceasta i (1) deducem c exist un ir (an )nN de numere reale cu a s a a s an 0 i s f (x + an ) = f (x) , x R, n N. Fie F o primitiva a funciei f i n N. Definim funcia t s t F (x + an ) F (x) , x R. Fn : R R, Fn (x) = an 0 Evident Fn este derivabila i Fn (x) = 0 (n conformitate cu (2)), adica F s constanta pe R; deci Fn (x) = Fn (0), x R, adica F (an ) F (0) F (x + an ) F (x) = , x R, an an de unde, prin trecere la limita pentru n tinznd la +, gasim f (x) = f (0), Aadar, f este constant pe R. Cu aceasta demonstraia este ncheiat. s a t a

Corolar. Fie g : RR R o funcie, P (R) = {f : R R | f admite prim t t t C (R) = {f : R R |f continua} si ecuaia funcionala f (x + y) = g (f (x) , f (y)) , x, y R.

Atunci ecuaia (3) are soluie n P (R) daca si numai daca are soluie n C (R t t t Demonstraie. Dac (3) are soluie n C (R), atunci, evident, are sol t a t P (R). Reciproc, daca (3) are soluie n P (R), atunci conform Propoziiei are t t n C (R). Bibliografie

1. S. Rdulescu, P. Alexandrescu, M. Chiria - Olimpiada locala, Bucureti a t s

16

Asupra problemei XII.32 din RecMat - 2/200


Marian TETIVA1

n numrul 2/2002 al Recreaiilor Matematice a fost publicat problem a t a

0). Pentru aceasta, se definete funcia F : A B prin F (f ) = f 1 (b Av s t f 1 (b = Ker f este subgrup n G, a f 1 (b iar f x3 = 3f (x) = 0) / 0), x3 f 1 (b x G; prin urmare, F este bine definita. Daca F (f ) = F (g), 0), f 1 (b = g 1 (b n plus, f 1 (b = g 1 (b iar f 1 (b = g 1 (b din surjecti 0) 0). 2) 2), 1) 1) funciilor f i g; n concluzie, F este injectiv. t s a Ramne sa dovedim ca F este surjectiva. Fie H B; este normal sa cons fH : G Z3 data prin fH (x) = b x H, fH (a) = b iar fH (x) = b 0, 2, 1 G \ (H {a}). Avem c F (fH ) = H, fH este surjectiv, iar fH (x) = b a a 2 Este ns aplicaia fH astfel definit morfism de grupuri? Dovedirea acest a t a impune studierea modului n care condiia din ipoteza influeneaza structura gr t t G. Acest studiu va releva ca ipoteza poate fi slabita, concluzia poate fi mbun i va permite obinerea unei generalizri interesante a problemei n discuie. s t a t

XII.32. Fie (G, ) grup, iar a G\{e} fixat. Aratai ca numarul morfi t 2 surjective de la G la (Z3 , +) cu proprietatea ca f (x) = b x = a este numarul subgrupurilor H ale lui G care nu-l conin pe a si care au propriet t x3 H, x G. Dana Stan, elev Ideea autoarei era stabilirea unei corespondene bijective ntre mulimile t t o n 2 A = f : G Z3 | f morfism surjectiv cu f (x) = b x = a ; B = H G | a H i x3 H, x G . / s

Fie deci f : G Z3 morfism de grupuri astfel nct exist a G \ a proprietatea f (x) = b x = a 2 2 2 2 1s 0; Avem atunci f (a) = b apoi f a = b + b = b i f a3 = b prin urmare, c 2, 2 s de surjectivitate impus lui f este superflu. n continuare, f a4 = b i a a obinem c a4 = a, adic a3 = e. Pe de alt parte, fie b G astfel nct f ( t a a a avem succesiv: f (ab) = f (a) + f (b) = b + b = b ab = a b = e. 2 0 2

Iar dac se consider c G cu f (b) = b atunci a a c 1, 2 5 2 5 f c = f c = b c = c = a c2 = a i c3 = e ac = e c = a 2 s a n concluzie, G = e, a, a2 , cu a element de ordin 3, deci exist un singur m ca n enun (f (e) = b f (a) = b f a2 = b i un singur subgrup H cu propr t 0, 2, 1) s cerute, H = {e} (oricum, i morfisme i subgrupuri sunt cte doua de toate!) s s Iata acum generalizarea anunata: t
1

Profesor, Colegiul Na ional "Gh. Roca Codreanu", Brlad t s

17

Propoziie. Fie (C, ) un grup ciclic si g C un generator al sau. Fie (G t alt grup (notam la fel operaiile celor doua grupuri, pentru simplitate). Presu t ca exista un morfism f : G C si un element a G, a 6= e, astfel nct f (x) x = a. Atunci G este izomorf cu C. Demonstraie. n primul rnd, f (a) = g f ak = g k , k Z. t fie x G un element oarecare. Trebuie sa avem f (x) = g k , pentru un 1k 1k k Z. Rezult (f fiind morfism) c f a a a x = f (a) f (x) = g 1k g Dar singurul element din G pentru care f ia valoarea g este a, deci n mod n avema1k x = a x = ak . Aadar orice element al lui G este o putere a s G ak | k Z . Din f ak = g k , k Z, rezult c f este o aplicaie surjectiv. n cazul n a a a t este finit i are n elemente avem g n = e i f an+1 = g n+1 = g an+1 = a s s (e fiind elementul neutru din C), deci G ak | k = 0, 1, . . . , n 1 i cum s o surjecie de la G la C, obligatoriu G are tot n elemente i, evident, este i t s cu C. k t Daca C este infinit nu putem avea a =am pentru exponenii ntregi distin m, deoarece asta ar nsemna i g k = f ak = f (am ) = g m , ceea ce este impo s fiind acum de ordin infinit. Deci i n acest caz G este izomorf cu C, fiind acum s ciclic infinit (generat tot de a, dar acesta nu mai are acum ordin finit). Demon este ncheiata.

Recreaii matematice

1. Patru oameni a, b, c i d vor sa traverseze ntr-o noapte un pod, veni s din aceeai direcie. Pentru a ajunge toi de cealalt parte, au la dispoziie 17 s t t a t i doar o lantern. Podul susine cel mult 2 oameni odat i orice echip car s a t as a podul, de unul sau doi oameni, trebuie sa aiba lanterna cu ei. Lanterna sa fie transportata nainte i napoi, deci, ea nu poate fi aruncata etc. Se s a traverseaz podul ntr-un minut, b n dou minute, c n cinci minute, d a a minute, iar o echip traverseaz podul cu viteza celui mai lent dintre compone a a Cum procedeaza cei patru pentru a trece podul n timpul stabilit?

2. Dac v putei imagina c desenul alturat reprea a t a a zint un taur care se uit spre est, schimbai poziia a a a t t dou segmente astfel nct acesta s se uite spre vest. a a

Not. Soluiile problemelor 1 i 2 se pot gsi la pagina 43. a t s a 18

Combinatoric . . . algebric a a
Gabriel DOSPINESCU 1

Expansiunea combinatoricii n concursurile de matematic de orice nivel i a cunoaterea unor procedee i metode ct mai variate de abordare a problem s s acest fel. Scopul acestei note este prezentarea, pe un numar de exemple, a m cum pot fi utilizate unele mijloace algebrice: numere complexe, polinoame rezolvarea problemelor de combinatoric. a Cardinalul unei mulimi A va fi notat |A|. Daca A este o mulime de n t t naturale, atunci suma elementelor acesteia se noteaza m (A) i se numete m s s lui A (prin convenie, m () = 0). Mulimea A se numete par / impar dac t t s a a a este numr par / impar ( este par). a a Vom utiliza n mod frecvent urmatoarea 2 2 Lem. Fie = cos a + i sin . Are loc egalitatea n n a0 + a1 + . . . + an1 n1 = 0, a0 , a1 , . . . , an1 R,

daca si numai daca a0 = a1 = . . . = an1 . Demonstraie. Definim polinoamele f, g R [X] prin f = a0 + a1 X + t +an1 X n1 i g = 1 + X + . . . + X n1 . Daca f , g au radacini comune, atunc s divide g. Deoarece g este ireductibil n R [X], rezulta (f, g) = g, adica g | f , s g = kf , k R. n consecina, avem a0 = a1 = . . . = an1 . Reciproca este evi t

1. Cte numere de n cifre 2, 3, 7 sau 9 se divid cu 3? (Concursul "T Lalescu", 2003) Soluie. Fie xn , yn i zn numarul numerelor cu n cifre 2, 3, 7 sau 9 cong t s 2 cu 0, 1 i respectiv 2 modulo 3. Se cere sa se afle xn . Fie = cos s +i 3 X Evident, xn + yn + zn = 4n i xn + yn + 2 zn = a1 +... s

a1 ,...,an {2,3,7,9} n = 1. Deci xn 1 + yn + 2 zn = 0, de unde rezu = 2 + 3 + 7 + 9 1 xn 1 = yn = zn = k. Atunci 3k = xn +yn +zn 1 = 4n 1, de unde k = (4 3 1 n Ca urmare, xn = (4 + 2). 3

t 2. Fie Sn = {1, 2, . . . , 2n} si An (Bn ) familia submulimilor pare (impa mulimii Sn , avnd n elemente. Sa se determine |An | |Bn |. (Polonia, 20 t X (1)m(A Soluie. Ideea eseniala este ca avem |An | |Bn | = t t
ASn , |A|=n i=1

2n i Yh i 1 + (1) X . Cum ultima expresie este coecientul lui X n n dezvoltarea


1

Elev, Liceul "Dimitrie Cantemir", Oneti s

19

n n n produs este egal cu (1 + X) (1 X) = 1 X 2 , deducem c |An ||Bn | es a n s cientul lui X n n dezvoltarea 1 X 2 , adica este 0 pentru n impar i (1)n/ pentru n par.

3. Fie p un numar prim impar, numerele naturale m si n divizibile cu p impar. Pentru fiecare m -upla (c1 , . . . , cm ), unde ci {1, 2, . . . , n}, cu propr m X ci , consideram produsul c1 . . . cm . Sa se demonstreze ca suma ca p | i=1 m n . (Gabriel Dospinescu) produse este divizibila cu p X Soluie. Pentru orice k {0, 1, . . . , p 1}, fie xk = t c1 . . . cm m X lundu-se dupa toate m-uplele (c1 , . . . , cm ) pentru care ci k (mod p). 2 2 = cos + i sin , avem p p
i=1

Cum, printr-un calcul simplu, se obine t + 22 + . . . + nn = rezulta ca nn+2 (n + 1) n+1 + ( 1)2


p1

m + 22 + . . . + nn =

c1 ,...,cm {1,2,...,n}

c1 . . . cm c1 +...+cm =

k=0

p1 X

xk

n , 1

Pe de alt parte, din p1 + . . . + + 1 = 0 deducem c a a 1 1 p2 + 2p3 + . . . + (p 2) + p 1 = 1 p i obinem s t m p2 m nm n + 2p3 + . . . + (p 2) + p 1 . m = p ( 1) Scriind p2 m X + 2X p3 + . . . + (p 2) X + (p 1) = b0 + b1 X + . . . + bm(p2) X m deducem ca nm = ( 1)m X bk . m n y0 + y1 + . . . + yp1 p1 , p

X nm xk k . m = ( 1)
k=0

unde yj =

kj(mod p)

Din (1) i (2), obinem relaia s t t

x0 ry0 + (x1 ry1 ) + (x2 ry2 ) 2 + . . . + (xp1 ryp1 ) p1 = 0 20

unde r =

m n . Din aceasta rezulta ca x0 ry0 = . . . = xp1 ryp p Ramne sa aratam ca r | x0 . Este suficient sa aratam ca r | k. Dar

pk = x0 + . . . + xp1 r (y0 + . . . + yp1 ) = m = (1 + 2 + . . . + n) r b0 + . . . + bm(p2) = m m = (1 + 2 + . . . + n) r (1 + 2 + . . . + (p 1)) . m m n (n + 1) p (p 1) i, cum membrul drept se divide s Deci pk = r 2 2 rezulta ca r | k.

daca si numai daca exista un indice i astfel nct n | ai . (Rookie Contest, Soluie. Sa observam ca au loc relaiile t t m X Y X + X 2 + . . . X ai = X c1 +c2 +...+cm i s
i=1 1ci ai

4. Fie n, a1 , a2 , . . . , am N si f (k) numarul m-uplelor (c1 , . . . , cm ) pent m X 1 ci ai si ci k (mod n). Sa se arate ca f (0) = f (1) = . . . = f
i=1

f (0) + f (1) + . . . + f (n 1) n1 = unde am notat n mod firesc = cos

1ci ai

i=1

indice i astfel nct + 2 + . . . + ai = 0 i, ca urmare, ai = 1, adic n | ai . s a

2 2 + i sin . Atunci f (0) = f (1) = n n = f (n 1) este echivalent cu f (0) + f (1) + . . . + f (n 1) n1 = 0, d m Y + 2 + . . . + ai = 0. Ultima realie are loc dac i numai dac exi t a s a

c1 +...+cm =

i=1

m Y

+ 2 + . . . +

5. Fie p un numar prim si A = {1, 2, . . . , 2p}. Sa se determine numarul milor B A cu p elemente si avnd proprietatea ca p | m (B). (OIM Polonia) 2 +i Soluie. Cazul p = 2 fiind banal, vom considera p 3. Fie = cos t p i xj numarul mulimilor B A cu p elemente i pentru care m (B) j (m t s s Atunci p1 X X X xj j = m(B) = c1 +...+cp ,
j=0 BA, |B|=p p 1c1 ...cp 2p

ultima sum fiind coeficientul lui X n dezvoltarea (X + ) X + 2 . . . X a Cum X p 1 = (X 1) (X ) . . . X p1 , deducem c (X + ) X + 2 a p1 X 2 2p p p = (X + 1) i, deci, coeficientul lui X este 2. Aadar, s xj j = s X +
j=0

21

x0 2 + x1 + . . . + xp1 p1 = 0, de rezult c x0 2 = x1 = . . xp unde a a .= 1 2p 2p Urmeaz c pk = x0 +. . .+xp1 2 = a a 2. n concluzie, x0 = 2+ p p p

6. Fie A o mulime de numere naturale ce conine cel puin doua numere i t t t X (1)m(B) mk (B) = 0 ( B Sa se arate ca pentru k {0, 1, 2} are loc relaia t
BA

fi sau A). (Gabriel Dospinescu) Soluie. Pornim de la observaia c t t a Y X (1 + X a ) = X m(B) .


aA BA

n (1) facem x = 1 i deducem ca s

relaia (1) i obinem (notnd A = {a1 , ..., ap }) t s t X X m(B)X m(B) . a1 X a1 (1 + X a2 ) . . . (1 + X ap ) =


BA

BA

(1)m(B) =

aA

(1 + (1)a ) = 0. D

analog ca

n (2) facem x = 1 i deducem (innd cont c din ipotez fiecare termen a s a a Xt din membrul stng este 0) ca m(B)(1)m(B) = 0. Derivam apoi (2) i o s
BA

BA

(1)

m(B)

m (B) = 0.

n final propunem spre rezolvare pe aceeai cale problemele urmtoare. s a a realiza ca aceasta metoda merita sa fie cunoscuta, sugeram ca att prob anterioare ct i cele urmatoare sa fie rezolvate i "clasic". s s

an determine . (Polonia) bn 8. Sa se calculeze suma elementelor submulimilor lui {1, 2, ..., 3n} care au m t multiplu de 3. (Gabriel Dospinescu) 9. Fie A = {1, 2, .., n}. a) Sa se arate ca familiile submulimilor pare si t ale lui A au acelai numar de elemente si aceeai suma a masurilor elemente s s b) Sa se afle suma masurilor submulimilor pare ale lui A. (Test de se t 1994)

7. Fie an numarul submulimilor B {1, 2, ..., 6n} pentru care m(B) 5 ( t Y t si bn numarul submulimilor C {1, 2, ..., 7n} pentru care x 5 (mod 7)
xC

22

Un procedeu de calcul al limitelor unor irur s de forma (an+1 an )n1


Oana CRJA1

1. Scopul acestei note este prezentarea unei scheme de calcul al limitelo iruri de forma (an+1 an )n1 , schem desprins din soluia dat de M. Tena s a a t a la sirul lui Lalescu [1, p. 443]. Baza teoretica a acestei scheme este data de urmatoarea

Propoziie. Fie (an )n1 un sir de numere strict pozitive ce verifica cond t n an+1 an an+1 = 1; (ii) lim = (i) lim = (0, ) ; (iii) lim n an n n n an Atunci lim (an+1 an ) = ln . n Demonstraie. Avem: t an+1 an n

de unde, prin logaritmare, obinem: t

n (an+1 an ) an an an+1 an an+1 an , n 1, = 1+ an

trecere la limita n (1), obinem ca aceasta limita este ln , q.e.d. t Observaie. Menionm faptul c pentru calculul limitei de la punctu t t a a (unde apare o nedeterminare de tipul 1 ) nu putem utiliza limita fundam 1/x = e, ci trebuie procedat n alt mod. corespunztoare, anume lim (1 + x) a
k

Relaiile (1), (2), (ii) i (iii) arat c exist lim (an+1 an ), finit sau nu, t s a a a a
n

an n an+1 an an+1 an an an+1 (an+1 an ) ln 1 + = , n 1. ln an n an an+1 an Din (i) rezulta imediat ca lim = 0 i, deci, s n an an an+1 an an+1 an = 1. lim ln 1 + n an

2. Aplicaie la iruri remarcabile. Avem vedere urmtoarele trei t s n a s p 1 Sirul lui T. Lalescu Ln = n+1 (n + 1)! n n!, n 2. Notam an = n n!, n 2 i obinem: s t n an n! 1 lim = lim = ; n n n n p e p n+1 (n + 1)! 1 an+1 n n+1 (n + 1)! n + 1 = lim = lim = e1 lim n n n an n n n+1 n e n! n!
1

Elev, cl. a XI-a, Colegiul Na ional "C. Negruzzi", Iai a t s

23

!n p n/(n+1) (n + 1)! n+1 lim = lim = lim = e. n n n n n n! n! 1 1 Conform Propoziiei, lim Ln = ln e = . t n e e 2 Sirul lui R. T. Ianculescu In = (n + 1) n+1 n + 1 n n n, n 2 [1, p. an an+1 Fie an = n n n, n 2. Se obine imediat ca lim t =1 = 1 i lim s n n n an n n n 1 n+1 an+1 (n + 1) n+1 n + 1 n+1 lim = lim = lim n+1 n n n an n n nnn n+ i, deci, lim In = 1 ln e = 1. s an+1 an n
n+1

nn1 (n + 1)n n2 , n 2 [1, p nn1 (n 1) Luam an = nn1 / (n 1)n2 , n 2. Prin calcul, gasim = = e (cu nota t Propoziie). Ca urmare, lim Gn = e. t 3 Sirul lui M. Ghermanescu Gn =
n

C:844, G. M.-11/1988)

yn+1 yn yn+1 Observam ca lim yn = +, lim = 0, lim = 1 i s n n yn n n n yn yn+1 yn yn+1 = lim 1 + = . . . = ec/b (calcul de rutin). a lim n n yn yn a an+1 yn+1 n+1 xn+1 t = lim = 1 Fie an = yn n xn ; obinem: lim n x n an n yn a n n n an yn an+1 yn+1 xn+1 1 n x = lim lim = lim n+1 n = ba, lim n n n n n n an yn xn x t = ec/b . Conform Propoziiei, limita ceruta este egala cu (ba) ln ec/b , adica est Propunem spre rezolvare urmtoarele probleme selectate din G. M.: a (7-8/1998), 24628 (1/2002) i 24708 (5-6/2002). s Bibliografie s 1. D. M. Btineu - Siruri, Ed. "Albatros", Bucureti, 1979. a t 24

3. Probleme de concurs. Cu Propoziia de mai pot fi calculate multe t limitele ce sunt date la concursurile colare. s 1 1 1 1 a t 1 Calculai lim e1+ 2 +...+ n+1 e1+ 2 +...+ n . (Olimpiada local, 20 n 1 an+1 1 1 Luam an = e1+ 2 +...+ n i constatam ca lim = lim e n+1 = 1, s n an n n n an an+1 1 1+ 1 +...+ n ln n lim = lim e n+1 = e i lim s = eC = lim e 2 n n n n n an 1 1 C = lim 1 + + . . . + ln n este constanta lui Euler. Deci, limita est n 2 n 2 Fie (xn )n1 si (yn )n1 doua siruri de numere reale strict pozitive astfe xn+1 yn = a R+ , lim = b R+ si lim (yn+1 yn ) = c R. Demo lim n xn n n n a t ca lim yn+1 n+1 xn+1 yn n xn = ac. (D. M. Btineu-Giurgiu, M. So
n

Asupra unei probleme propuse la Concursul Florica T. Cmpan, martie 200


Horia-Nicolai TEODORESCU 1

La etapa judeeana a concursului amintit, s-a dat i urmatoarea problema t s dusa aici sumar): Pe un biliard dreptunghiular ABCD, o bila pornete din M catre AB si lo s alta bila n N . Unde lovete prima bila latura AB? Se cunosc coordonatele pu s M si N . (Autor necunoscut) Baremul afiat pe ua colii unde s-a desfaurat concursul preciza drept s s s s unic un punct de coordonate corespunztoare datelor problemei, (0, 2). Aceas a a ns numai o soluie particular (este drept, singura care putea fi determinat a t a a din datele problemei). Din figura alaturata se poate constata ca exista o inf de soluii, obinute prin ciocniri multiple fie i numai dupa laturile AB i re t t s s CD (semi-biliardul biliardul semi-infinit creat de dreptele AB i CD, fr s aa AD i BC). s

D M

Deoarece la ciocnire traiectoria face unghiuri egale cu normala la supraf ciocnire, din egalitile de unghiuri, obinem, cu notaiile M (xM , yM ), N (xN at t t xN xM = yM tg + yN tg + 2pa tg ,

unde p este numrul de ciocniri cu latura CD, iar a este distana dintre latur a t i CD. Deci, s xN xM = (yM + yN + 2pa) tg , de unde xN xM tg = , = (p) , p = 0, 1, . . . yM + yN + 2pa In textul problemei, nu se da valoarea a, dar aceasta nu face sa dispara infi de soluii posibile dupa relaia de mai sus. Corespunzator fiecarei valori de un t t determin punctele de lovire a dreptei AB, iar pentru p = 0, se obine soluia in a t t n timpul concursului. Desigur, exist i alte familii de soluii, prin ciocniri as t
1

Prof. dr., Fac. de Electronic i Telecomunica ii, Univ. Tehnic "Gh. Asachi", Iai as t a s

25

cu pereii AD i BC (strict similare cu cele discutate) sau obinute prin cioc t s t toi cei patru perei soluii pe care nu le analizam aici. t t t O anumita ambiguitate a textului se pare ca a produs ncurcaturi unor can Printre exprimrile mai puin fericite se numr se ndreapt spre AB. In a t aa a ar trebui interpretat aceast afirmaie? Orice traiectorie n jos se ndreap a a t AB, caci se apropie de AB distana dintre punctul de pe traiectorie i AB t s Ar fi trebuit spus traiectoria punctului este iniial dupa o dreapta, care interse t segmentul AB. Sau, mai simplu, bila se ciocnete nti de AB. Dac se ma s a i precizarea se ciocnete o singur dat cu AB i nici o dat de alte marg s s a a s a biliardului, nainte de a lovi bila din N , atunci problema ar fi fost nu numa mai clara, dar ar fi avut ca unica soluie chiar soluia indicata participan t t concurs. Unii candidai au fost nclinai s considere c bila ce pornete din M se t t a a s nti de bila din N i abia apoi de marginea AB. Interpretarea nu era greit s s textul problemei nu contrazice aceasta interpretare. Din nefericire, aceasta co problema, cci dou puncte materiale se mic dup ciocnire pe dreapta dup a a s a a s-au micat iniial, dar n sens opus, deci bila din M urma s se ntoarc nt s t a a CD (sau AD, CD funcie de dimensiunile biliardului) i apoi spre AB! t s bilele se presupun nepunctuale, problema este nedeterminat (este necesar s a a precizeze unghiul de contact al bilelor la prima ciocnire); cu precizarea unghi ciocnire, problema devine rezolvabil, dar depete nivelul de cunotine de f a as s s t clasa a VIII-a. In concluzie, problema discutat avea prea multe neclariti i imprecizii a at s a fi inclus ca atare ntr-un concurs de nivel judeean, iar acordarea de note m a t aceast problem probabil a lsat pe unii elevi cu falsa impresie c au neles a a a a t rezolvat corect (i complet) problema daca au dat soluia indicata n ziua concu s t In nal, cred c este meritoriu pentru Comisia concursului amintit c a dat a a unei probleme de tip biliard, dat ind c domeniul biliardelor formale (n par a teoria biliardelor hiperbolice) este dintre cele mai profunde i fertile astzi n s a sistemelor ergodice i a sistemelor cu dinamica neliniara [1], [2]. Biliardele s onale i eliptice n plan i cele cubice i tetraedrice sunt de altfel probleme s s s n geometrie (vezi problema lui Alhazen, datnd din antichitate i intens stud s evul mediu, sau porismul lui Poncelet), cu multiple i importante aplicaii n s t prea puin reectate n literatura de specialitate pentru elevi de la noi din a t t ateptm deci i alte probleme despre biliarde n culegeri i n concursuri co s a s s s

Bibliografie 1. Lai-Sang Young - Developments in Chaotic Dynamics, Notices of AMS, 199 45, nr. 10, pp. 13181328. 2. M. Hasewinkel - Encyclopedia of Mathematics, Vol. 1, pp. 406411 (Pessin T Kluwer Academic, 1997.

26

O problem . . . i opt soluii a s t


Adrian ZANOSCHI 1

La prima ediie a Concursului de matematic "Alexandru Myller", t a avut loc la Iai, n perioada 4 - 6 aprilie 2003, a fost propus elevilor de clasa s a urmtoarea problem [1]: a a

b b Fie ABC si ADE doua triunghiuri dreptunghice cu B = D = 90 si AB Fie F proiecia lui B pe AC si G proiecia lui D pe AE. Sa se arate ca p t t B, F , E sunt coliniare daca si numai daca punctele D, G, C sunt coliniare.

Problema, rezolvat corect de aproximativ 60% dintre concureni, a prileju a t tora etalarea unor variate tehnici i metode de geometrie sintetic, analitic i s a as rial. V prezentm n continuare opt dintre rezolvrile care au fost date n c a a a a i n afara lui, lasndu-va pe dumneavoastra sa decidei care este cea mai fru s t Soluia I (sintetic). Aplicnd teorema t a catetei n triunghiurile ABC i ADE, obinem: s t AF AC = AB 2 = AD2 = AG AE, de unde rezulta ca AF AC = AG AE, adica punctele F , C, E i G sunt conciclice. De aici, s deducem c CF E = 90 dac i numai dac a \ a s a \ EGC = 90 , ceea ce nseamna ca punctele B, F , E sunt coliniare dac i numai dac punctele D, as a G, C sunt coliniare. (Andrei Stefnescu, elev, a Colegiul Naional de informatica "Tudor Vianu", t Bucureti) s

C F B

A
Fig. 1

Soluia a II-a (sintetic). Vom demonstra mai nti urmatoarea t a Lem. Fie un triunghi ABC si un punct M (BC). n aceste condii a t este perpendiculara pe BC daca si numai daca: AB 2 BM 2 = AC 2 CM 2 . Demonstraie. I Dac AM BC, atunci din egat a litile AB 2 BM 2 = AM 2 i AC 2 CM 2 = AM 2 , at s rezulta relaia cautata. t II Sa presupunem ca are loc relaia (1). Notnd cu t \ avem unghiul AM B, AB 2 = AM 2 + BM 2 2 AM BM cos , AC 2 = AM 2 + CM 2 + 2 AM CM cos , de unde, folosind egalitatea (1), deducem AM BM cos = AM CM cos sau AM BC cos = 0, deci = 90 .
1

B M
Fig. 2

Profesor, Colegiul Na ional "C. Negruzzi", Iai t s

27

Revenind la problema noastr, s presupunem c B, F , E sunt coliniare. Ap a a a Lema n triunghiurile ABC, ADE i ACE, obinem s t AD2 AG2 = DE 2 EG2 , AB 2 AF 2 = CB 2 CF 2 ,

Cum AB = AD, din (2) i (3) rezulta ca: s

CE 2 CF 2 = EA2 F A2 .

de unde, avnd n vedere relaia (4), gasim: t sau

CB 2 CF 2 + AF 2 = DE 2 EG2 + AG2 ,

CB 2 + EA2 CE 2 = DE 2 EG2 + AG2 , CB 2 + EA2 DE 2 AG2 = CE 2 EG2 , CB 2 + AD2 AG2 = CE 2 EG2 , CA2 AG2 = CE 2 EG2 ,

ceea ce nseamna, conform Lemei, ca CG AE. Prin urmare, punctele D sunt coliniare. Reciproca se demonstreaz n mod analog. (Adrian Zanosc a

Soluia a III-a (sintetic). Presupunem ca punctele B, F , E sunt co t a Fie G0 proiecia punctului C pe AE. Din faptul ca 4AG0 C 4AF E rez t AG0 AC AF AC AB 2 0 = sau AG = = (teorema catetei n 4ABC). Ca u AF AE AE AE 2 AD s = AG (teorema catetei n 4ADE), adic G0 coincide cu G a AG0 = AE punctele D, G, C sunt coliniare. Implicaia invers se demonstreaz la fel. ( t a a Asaftei, prof., Scoala Normala "V. Lupu", Iai ) s

Soluia a IV-a (sintetic). Vom utiliza rezultatul urmtor: Un pat t a a M N P Q (convex sau concav) este ortodiagonal daca si numai daca este nde relaia M N 2 + P Q2 = M Q2 + N P 2 . t Daca punctele B, F , E sunt coliniare, atunci patrulaterul ABCE est diagonal i are loc relaia AB 2 + CE 2 = AE 2 + BC 2 . Deoarece AB = s t AE 2 = AD2 +ED2 , BC 2 = AC 2 AB 2 = AC 2 AD2 , relaia se scrie AD2 + t = AC 2 + ED2 , adica patrulaterul ADEC este ortodiagonal. Deci D, G, C su niare. Menionam faptul ca n condiiile E BF i E [BF ) patrulaterul A t t s / este concav i c E [BF ] nu poate avea loc. Se arat la fel implicaia i s a a t (Temistocle Brsan)

Soluia a V-a (vectorial). S presupunem c punctele B, F i E sunt co t a a a s n acest caz, avem BE AC, deci BE CA = 0. Sa aratam ca CG AE: CG AE = CB + BA + AG AB + BE = = CB AB + CB BE + BA AB + BA BE + AG AB + AG BE = = CB BE AB 2 + BA BE + AG AB + AG BE = 28

De aici, avnd n vedere c DG AE, rezult c punctele D, G i C sun a a a s niare. Cealalt implicaie se demonstreaz n mod analog. (Cristina Gutue a t a Colegiul Naional "I. C. Bratianu", Piteti i Bianca Milatinovici, eleva, Li t s s Informatica "Gr. C. Moisil ", Iai ) s

= CB + BA BE AB 2 + AG AB + BE = = CA BE AB 2 + AG AE = AB 2 + AG AE = AD2 + AG AE =

Soluia a VI-a (vectorial). Fie xOy un sistem ortogonal de coordonat t a t nct Ox s fie mediatoarea lui BD. Vom nota cu M vectorul de poziie a r punct oarecare M (faa de polul O). Punctele B, F i E sunt coliniare dac i t s as r r daca BE AC, adica ( E B ) ( C A ) = 0 sau r r + = + . r r r r r r r r
B C E A E C A B

i s

r r r Punctele D, G i C sunt coliniare dac i numai dac ( C D ) ( E A s as a r sau + = + . r r r r D r E C A E C A D r r r b = D = 90 , rezulta ca ( B A ) ( C B ) = 0 i ( D b r r r s r Deoarece B r ) = 0, deci rD ( r E + = +2 r r r r r r r
B C A B C A B

+ = +2 . r r r r D r E A D E A D r r Datorita modului de alegere a sistemului de coordonate avem 2 = rB s = i astfel, din (3) i (4), obinem rA rD rA rB s t + = + . r r r r r r r r
B C E A D E C A

r Prin urmare, avnd n vedere relaia (5) i egalitatea A B = A D , t s r r r c relaiile (1) i (2) sunt echivalente, adic ceea ce trebuia demonstrat. (A a t s a Zanoschi)

Soluia a VII-a (cu numere complexe). Deoarece relaiile F BE i G t t s sunt echivalente cu BE AC i respectiv cu DC AE, problema noastra re s a arta echivalena BE AC DC AE. a t Considerm un reper ortogonal n plan cu originea n A i notm cu b, c a s a afixele punctelor B, C, D i E. Cum AB = AD, rezult c |b| = |d|. Dac s a a a cu x y (x, y C) produsul real al numerelor complexe x i y, avem: s BE AC (e b) c = 0 (e b) c + e b c = 0 bc + bc = ce + ce DC AE (c d) e = 0 (d c) e + d c e = 0 de + de = ce + c BC AB (c b) b = 0 (c b) b + c b b = 0 bc + bc = 2bb, DE AD (e d) d = 0 (e d) d + e d d = 0 de + de = 2dd.

Cum bb = |b|2 = |d|2 = dd, din relaiile (3) i (4) rezulta ca bc + bc = de + de, d t s deducem c relaiile (1) i (2) sunt echivalente, deci BE AC, dac i numa a t s as DC AE. (Cezar Chiril, elev, Colegiul Naional "M. Eminescu", Botoa a t s 29

Soluia a VIII-a (analitic). Fie t a xOy un sistem ortogonal de coordonate cu originea n A i (Ox = (AC. n acest s caz, punctele din problem vor avea coordoa natele: A (0, 0), B (b, b0 ), C (c, 0), D (d, d0 ), E (e, e0 ), F (b, 0), G (g, g 0 ). Sa presupunem ca B, F , E sunt coliniare i sa aratam ca s D, G, C sunt coliniare (reciproca se poate demonstra n mod analog, alegnd sistemul de coordonate astfel nct triunghiul ADE sa joace rolul triunghiului ABC). n acest caz e = b. b b Din B = D = 90 , rezult c a a sau

y O=A(0,0) F(b,0)

B(b,b

G(g,g') D(d,d')
Fig. 3

E(e,e

mAB mBC = mAD mDE = 1

b0 b0 d0 e0 d0 = = 1, b bc d ed 2 2 s de unde obinem b2 + (b0 ) = bc i ed + e0 d0 = d2 + (d0 ) . De aici, avnd n t 2 2 a c AB = AD, deci b2 + (b0 ) = d2 + (d0 ) , deducem c: a ed + e0 d0 = bc. Cum DG AE, rezulta ca mDG mAE = 1, sau g 0 d0 e = 0. gd e

n sfrit, innd cont de (1) i (2), putem scrie: s t s g 0 d0 e d0 d0 (2) = 0 = mDG = mDC gd dc e dc care este adevrat, conform ipotezei. (Evelina Sltineanu, elev, a a a a "D. Cantemir", Iai) s ed + e0 d0 = ec bc = ec b = e,
(1)

Bibliografie 1. Concursul de matematic "Al. Myller", ediia I, 2003, Problema X.3 a t creaii matematice, suplimentul nr. 1 (2003) sau n prezentul numar, p. 33). t 2. Liviu Nicolescu, Vladimir Boskoff - Probleme practice de geometrie, Ed. T Bucureti, 1990. s

30

Asupra unor sume cu radicali


Dan POPESCU 1

Scopul acestei note este prezentarea unor proprieti ale radicalilor accesib at vului de gimnaziu i utile n abordarea unitara a unor tipuri de probleme. s Propoziia 1. Daca n N este astfel nct n Q, atunci n N. t p p Demonstraie. Fie n = , cu p, q N, q 6= 0, (p, q) = 1. Atunci n = t q q q 2 | p2 , de unde q | p, adic n N. a Propoziia 2. Fie a, b Q astfel nct a, b R \ Q. Atunci ab t + r a si numai daca Q. b r a 1 ab. = Demonstraie. Imediat, din faptul c t a b b Propoziia 3. Fie a, b Q si , Q astfel nct a + b Q ; t + a Q si b Q. 2 a 2 b Q i Demonstraie. Observm c a b = t a a s a+ b i h 1 a= a + b + a b Q; analog pentru b. 2 Propoziia 4. t Fie a Q , iar b Q astfel nct b R \ Q; a b R \ Q si a b R \ Q. Demonstraie. Daca, prin absurd, a + b Q, atunci b = a + b t 1 a b Q, fals. contradicie. La fel, daca a b Q, atunci b = t a a t a Consecina. Deoarece un numr iraional este nenul, iar inversul oricrui t a 1 s a iraional este tot numr iraional, n condiiile P4 avem i c R\Q, t a t t b a Propoziia 5. Fie a, b Q astfel nct a, b, ab R \ Q, iar m, t + n Atunci m a + n b Q daca si numai daca m2 + 2 = 0. Demonstraie. Dac m = n = 0, atunci m a + n b = 0 Q. Inv t a presupunem ca m a + n b Q. Patratul acestui numar este tot raional i t s mn ab Q. ns ab R \ Q, deci mn = 0. n cazul n care m = 0, av a n b Q, adic n = 0. Analog, pentru n = 0 obinem c i m = 0. n con a t as m2 + n2 = 0. t r Observaie. Tinnd seama de P2 , ipoteza ab R \ Q poate fi nlocu a R \ Q. b Prezentam n continuare cteva aplicaii ale acestor rezultate, iar n ncheie t propuse alte exerciii care pot fi rezolvate asemanator. t
1

Profesor, Colegiul Na ional " tefan cel Mare", Suceava t S

31

Problema 1. Sa se rezolve n N N ecuaiile t 3 2 a+ b = 5; b) a + b = 18. a) 3 4 t s Soluie. a) Aplicnd P3 , obinem ca a, b Q i atunci a, b N, c t . . a .3, b. i cercetnd posib .4 s P1 . n plus, deoarece (3, 4) = 1, avem c a. existente, gsim unica soluie (9, 16). a t b) t s s Ecuaia se scrie echivalent 2a+ 2 2b = 6 i aplicnd din nou P3 i P1 , o ca 2a, 2b N, deci a = 2x2 , b = 2y cu x, y N. nlocuind, gasim ca x + de unde (a, b) {(0, 18) , (18, 0) , (2, 8) , (8, 2)}. Problema 2. Sa se arate ca n + n + 1 R \ Q, n N . Soluie. S observm mai nti c n i n + 1 nu pot fi simultan ptrate p t a a a s a pentru n N . Dac exact unul dintre ele este ptrat perfect, concluzia urmea a a P4p a nici unul nu este patrat perfect, atunci se poate aplica P5 , dat fiind . Dac ca n (n + 1) R \ Q (numarul n (n + 1) este cuprins strict ntre patratele p consecutive n2 i (n + 1)2 ). s t Problema 3. Sa se rezolve n N3 ecuaia 2 p p 2 + n + p2 4 m 3m + 2 n n2 + 5n + 6 = 0.

Soluie. Singura valoare a lui n N pentru care unul dintre radicali d t este n = 0; n acest caz, ecuaia devine p2 4 = 0, iar mulimea soluiil t t t {(a, 0, 2) | a N}. Dac n 6= 0, suntem n ipotezele P5 : a
2 2 2

prin urmare m2 3m + 2 = 0 i p2 4 = 0, deci mulimea soluiilor ecua s t t t S = {(1, a, 2) | a N } {(2, b, 2) | b N }.

n2 < n2 + n < (n + 1) ; (n + 2) < n2 + 5n + 6 < (n + 3) ; 2 2 n + 3n < n2 + n n2 + 5n + 6 = n2 + 3n n2 + 3n + 2 < n2 + 3n +

Probleme propuse. 1. S se rezolve n N N ecuaiile: a t a) 2 x + 3 y = 12; b) m + n = 10. 2. Sa se arate ca ecuaia a b = p, unde p N este prim, are o inf t de soluii n N N. t 3. Sa se arate ca n2 + n + n2 + 3n + 2 R \ Q, n N.

4. Sa se arate ca: 1 1 R \ Q, n N; R \ Q, a) b) 5n + 2 + 5n + 1 5n + 7 11 5. Sa se rezolve n Q ecuaia 2x2 + 5x + 3 11 2x2 + 3x 21 = 0. t n o 6. Fie A = a + b | a, b N , a, b 100 . S se afle cardinalul m a A Q.

32

Concursul "Alexandru Myller"


Ediia I, Iai, 4 - 6 aprilie 2003 t s

Alexandru Myller (1879 - 1965) s-a nscut n Bucureti dintr-o fam a s intelectuali. A absolvit Facultatea de Stiine secia matematica din Bucuret t t s anii 1902 - 1906 se afla n Germania pentru a continua studiile. n 1906, la G susine teza de doctorat sub conducerea lui David Hilbert. Tot aici l cunoa t Felix Klein, fondatorul faimosului "Program de la Erlangen". Rentors n ar t titularizat n 1910 ca profesor de geometrie analitica la Univ. "Al. I. Cuza" d n 1910 pune bazele unei biblioteci, Seminarul Matematic, care se va d treptat, i a unei scoli de geometrie ce va fi renumit n ntreaga lume. Este s a torul unei colecii de modele geometrice i se preocup de problemele nvam t s a at secundar. A adus contribuii de valoare n geometria difereniala, ecuaii difer t t t si integrale i istoria matematicii. s A fost rector al universitatii ieene n perioada 1945 - 46. Din 1949 este m s al Academiei Romne. n 1960 universitatea Humbold din Berlin i-a conferi de doctor honoris causa.

Not. Concursul "Al. Myller" se adreseaz elevilor din clasele VII - XII care au a a premii si meniuni la fazele superioare ale olimpiadelor scolare din anul n curs t anterior. Prima ediie a acestui concurs a fost sprijinit de ctre Univ. "Al. I. C t a a Filiala din Iai a SSMR. Sarcina organizrii i desfurrii acesteia au avut-o I. S. J s a s as a urmtoarele licee: Colegiul Naional i Liceul de Informatica "Gr. Moisil". Aceast a t s a Concursului "Al. Myller" a fost un succes deplin sub aspect calitativ i organizatori s

Clasa a VII-a

1. Determinai numerele ntregi a, b, c, d care verific relaiile a2 + b2 = 2 t a t i c2 + d2 = 2 (a + b). s Gheorghe Iure 2. Fie ABCD un ptrat fix i punctele variabile M (BC), N (CD a s nct M N = BM + DN . Demonstrai ca masura unghiului ]N AM este con t Gheorghe Iure 3. Se considera un triunghi ABC, un punct M (AC) i punctul N BC s nct M N BC. Perpendiculara din C pe AN i perpendiculara dusa n B s se intersecteaz n P , iar dreptele M P i AN se intersecteaz n Q. Demonst a s a AP CQ dac i numai dac AB AC. as a Petru Rducan a 4. Fie a, b, c numere reale astfel nct 0 a 1, 0 b 1, 0 c ab + bc + ca = 1. Demonstrai ca a2 + b2 + c2 2. t Mircea Becheanu, Buc

Clasa a VIII-a

1. Determinai numerele x, y, z care verifica relaiile x+y 2z i x2 +y 2 2 t t s Adrian Zanosch 2. Un tetraedru regulat cu muchiile de lungime 1 se proiecteaza pe un pla Demonstrai ca aria figurii obinute este cel mult 1/2. t t 33

3. Fie tetraedrul ABCD n care AB = CD = a, AC = BD = b, AD = BC GA , GB , GC , GD centrele de greutate ale feelor BCD, ACD, ABD respectiv t Determinai lungimea minima a unui drum care este situat pe feele tetraed t t trece prin GA , GB , GC , GD .

4. Fie n 3 un numar ntreg. Demonstrai ca este posibil ca, eliminnd c t dou dintre elementele mulimii {1, 2, . . . , n}, s obinem o mulime care ar a t a t t elementelor ptrat perfect. a Mihai Blun, Buc a a

Clasa a IX-a

1. Fie ABCD un patrulater convex i O un punct n interiorul ace s Notm cu a, b, c, d, e, f respectiv ariile triunghiurilor formate de a AB, BC, CD, DA, AC i BD. S se arate c |ac bd| = ef . s a a Alexandru M

2. a) Aratai ca exista funcii f : R R, f (x) = ax2 + bx + c, a 6= 0, astfe t t f (f (k)) = k, k {1, 2, 3}. b) Artai c, dac f este o funcie ca la a), atunci numerele a, b, c nu sun a t a a t ntregi. Gheorghe Iure 1 1 1 3. Fie a, b, c numere reale pozitive astfel nct + + = 1. S se a a ab bc ca ab 1 bc 1 ca 1 3 + + < 2. 2 ab + 1 bc + 1 ca + 1 Mircea Becheanu, Buc 2 4. Fie S = (x, y) R | x, y > 0 primul cadran si T : S S, T (x 1 1 o transformare a lui S. Numim "S - dreapt" intersecia dintre a t = , x y dreapta din plan. a) Artai c orice S - dreapt fix a lui T conine punctele fixe ale lui T a t a a a t b) Determinai S - dreptele fixe ale lui T . t (M S se numete submulime fix a lui T dac T (M ) = M ). s t a a Gabriel Pop

Clasa a X-a

s 1. Fie A1 A2 . . . An un poligon regulat nscris n cercul C (O, R) i M un p planul acestuia. S se arate c nR M A1 + M A2 + + M An n (R + OM a a Gheorghe Iure 2.

Fie polinomul f (X) = X n + 2X n1 + 3X n2 + + nX + n 2 2 = cos + i sin . S se arate c f () f 2 . . . f n+1 = (n + a a n+2 n+2 Mihai Piticari, C-lung Moldov

3. Fie ABC i ADE doua triunghiuri dreptunghice cu m (]B) = m (]D) s i AB = AD. Fie F proiecia lui B pe AC i G proiecia lui D pe AE. S se a s t s t a punctele B, F , E sunt coliniare dac i numai dac punctele D, G, C sunt co as a 34

4. La un concurs se dau cinci probe, cu rezultatul admis - respins. Ca numarul minim de participani la concurs pentru care, orice rezultate ar obin t t tia, sa existe doi concureni A i B astfel nct A sa fie admis la toate probele t s a fost admis i B? s

1. Considerm An = A M2 (R) | An+1 = 2003n A , unde n este un a natural nenul fixat. a) Aratai ca An conine o infinitate de elemente. t t b) Determinai A3 A2003 . t Gheorghe Iure 2. Fie A, B M3 (R) doua matrice cu proprietatea ca, oricare ar fi o m X M3,1 (R) pentru care AX = O3,1 , avem c BX = O3,1 . S se arate c e a a a matrice C M3 (R) astfel nct B = CA. Mircea Becheanu, Buc 3. S se arate c pentru orice numr natural n 0 exist numerele ra a a a a strict pozitive a0 < a1 < < an cu urmtoarele proprieti: a at a0 a1 an 1 a) + + ... + = ; 0! 1! n! n! 3 b) a0 + a1 + . . . + an < n . 2 Dorin Andrica, Cluj - N 4. S se determine funciile derivabile f : [0, ) R cu proprietile: a t at a) f (0) = 0. 1 x 2 0 2x + f , x [0, ). b) f 0 (x) = f 0 3 3 3 3 Mihai Piticari, C-lung Moldov

Clasa a XI-a

Clasa a XII-a

1. Fie f i g dou polinoame cu coeficieni raionali, ireductibile n Q s a t t , C astfel nct f () = g () = 0. Sa se arate ca, daca + Q, atunc au acelai grad. s Bogdan Enescu, B Z 2 2. S se calculeze a cos t cos2 (2t) cos3 (3t) . . . cos2002 (2002t) dt.
0

Dorin Andrica, Cluj - N 3. Fie (A, +, ) un inel necomutativ, unitar i a, b, c A astfel nct ab + s Dac exist x A astfel nct a + cx este inversabil, atunci exist y A astfe a a a b + yc este inversabil. Andrei Nedelcu i Lucian Ldunc s a Z t sin x 4. a) S se arate c exist lim a a a dx i aceasta este finit, unde n s a t 1 xn fixat. Z t sin x b) Daca se noteaza cu ln = lim dx, sa se calculeze lim ln . t 1 n xn Mihai Piticari, C-lung Moldov 35

Concursul Florica T. Cmpan, ediia a III-a t


Faza judeean, 1 martie 2003 t a Clasa a IV-a

1. Care este cel mai mare numr care mprit la 10 d ctul 9? a at a 2. Sa se ordoneze numerele din irul urmator n ordinea crescatoare a s cifrelor lor: 132, 456, 199, 897, 1124,9191. 3. Cte triunghiuri sunt n figur? a

4. S se taie 7 cifre din irul 123123123123, astfel nct numrul rmas s a s a a a mai mare posibil. Care este numarul? 5. Pe o farfurie sunt 19 fructe: prune, caise, piersici. Numrul piersicilor a 9 ori mai mare dect cel al prunelor. Cte caise sunt? 6. O coloan de militari, lung de 100 metri, trece pe un pod lung de 100 a a cu viteza de 100 metri pe minut. Ct timp dureaza pna ce coloana parcurge 7. Cnd tu veneai pe lume, eu aveam cu 1 an mai mult dect de 4 ori v de acum. A putea s ajung la 99 ani dac voi mai tri cu 2 ani mai mult d s a a a trait tu pna acum. Ci ani am eu acum? t

Clasa a V-a

1. Suma cifrelor unui numr natural este 23, iar ctul mpririi sale prin a at 96. S se afle numrul. a a 2. n cte zerouri se termina numarul N = 12
56 34

Monica Ne 3. Numerele 1, 2, 3, 4, 5, 6, 7, 8, 9 sunt aezate ntr-un tablou triunghiula s a b c d e f i h g

23

61 45

34

12 56

45

23 61

56

34 12

61

45 23

Daca suma numerelor de pe fiecare latura a triunghiului este 20, sa se ar numrul 5 este unul dintre vrfuri. a Andrei Ne 4. Un colier este format din bile pe care sunt nscrise numere naturale astfel nct pe bilele vecine uneia este nscris un divizor sau multiplu al num nscris pe acea bil, fr ca un acelai numr s apar pe mai multe bile. Ca a aa s a a a
1

Not. Toate subiectele sunt obligatorii. Timp de lucru: 1.5 ore - cl. IV i 2 ore - cl. V a s

36

cel mai lung colier care poate fi format cu numerele naturale mai mici sau e 100? Descriei toate soluiile cu numar maxim de bile! t t Mihaela C

Clasa a VI-a

t 1. Fie S = 2 + 22 + 23 + 24 + . . . + 22003 . Calculai S + 2. s 2. Determinai toate numerele de forma abcd tiind ca t a+b+c b+c+d c+d+a = = . 5 2 3 3. n jurul unui punct O considerm unghiurile cu msurile din figur. a a a t x = 9 , calculai n .
4 3 2 1 x n

4. ntr-un triunghi laturile sunt numere naturale pare. O latur este egal a Artai c triunghiul este isoscel. a t a

Clasa a VII-a

1. Fiecare celula a unui tabel 2003 2003 este colorata la ntmplare cu u 2002 culori. La un pas se permite recolorarea cu o aceeai culoare a unei li s a unei coloane, dac pe aceast linie (coloan) se afl mcar dou celule de a a a a a a a culoare. Prezentai un algoritm cu un numar minim de pai care permite c t s tabel sa devina monocolor. 2. Fie r1 , r2 , r3 , . . . , r2003 o rearanjare a numerelor ntregi 1,2,3,. . . ,2003 demonstreze c produsul P = (r1 1) (r2 2) . . . (r2003 2003) este numr p a a 3. Fie 4ABC i M (BC). Notam cu M 0 , C 0 , A0 , B 0 simetricele punct s M 0 , C 0 , A0 respectiv faa de M , C, A, B. S se arate c punctele M 0 i B 0 t a a s dac i numai dac M este mijlocul lui (BC). as a Gabriel b = 80 , m(C) = 60 i AC = 1. S se d b 4. Se consider 4ABC cu m(A) a s a streze c BC este medie proporional (geometric) ntre AC i (AB + 1). a t a a s

1. Fie n N . Sa se gaseasca partea ntreaga a numarului n2 + 3n. 2. Fie A = {1, 2, 3, . . . , 2003} i fie f : A A o funcie liniar necon s t a Artai c f (1002) = 1002. a t a Gheorghe 3. Dou furnici merg cu vitez constant pe paralelipipedul drept a a a ABCDA0 B 0 C 0 D0 , cu AB = 10 cm, BC = 15 cm, AA0 = 12 cm. Prima pleaca din A i ajunge n A0 traversnd, n ordine, muchiile [BB 0 ], [CC 0 ] i s s 37

Clasa a VIII-a

(pe drumul cel mai scurt). A doua furnic pleac din B 0 i ajunge n B trave a a s n ordine, muchiile [CC 0 ], [DD0 ] i [AA0 ] (pe drumul cel mai scurt). Cunosc s furnicile pornesc n acelai timp i ca ele se ntlnesc, aflai raportul vitezelor s s t 4. Fie o mas de biliard dreptunghiular ABCD la care s-au ales drept a a coordonate laturile AB i AD (AB e axa absciselor). Se dau 2 bile situate n p s M (5, 6) i N (1, 2). Bila din M pleaca liniar catre AB astfel nct sa loveas s din N . S se gseasc punctul n care bila lovete latura AB. a a a s

Faza interjudeean, 24 mai 2003 t a Clasa a IV-a

1. Pe fiecare dintre cele cinci cari, numarul de jos este ntr-o aceeai le s t ascuns cu numrul de sus. Care este al doilea numr scris pe a cincea carte? a a a

93 3

84 2

55 1

62 3

51

2. Un elev trebuie s nvee pentru a doua zi la istorie, matematic i engl a t as cte moduri i poate stabili ordinea disciplinelor la care nvaa? Precizai-le! s t t 3. Dac un pahar i o sticl cntresc ct o can, sticla respectiv cn a s a a a a ct paharul i o farfurie, iar dou cni cntresc ct trei farfurii, atunci cte s a a a cntaresc ct o sticla? 4. Am vizitat grdina zoologic. Am vzut urii, leii, lupii i maimuele, a a a s s t n aceast ordine. n prima cuc animalele dormeau i erau uri sau maimu a s a s s t doua cuca nu erau lupi i nici lei. n a treia cuca animalele se uitau n alta s s s nu la mine. n a patra cuca nu erau maimue i nici uri. Maimuele nu do s t s s t Lupii se uitau la mine. n ce ordine am vizitat animalele?

Clasa a V-a

1. Determinai cel mai mic numr scris n baza 10 numai cu cifrele 0 i 1, d t a s cu 225. 2. Aratai ca numerele 1, 2, 3 . . . , 16 nu pot fi aranjate pe o circumferina t t nct suma oricror dou numere vecine s fie ptrat perfect. Este posibil o a a a a a de aranjare pe o linie? Justificai. t 56 3. Fie fracia 2003 . t 2 a) Justificai ca fracia este zecimala finita; t t b) Care sunt ultimele dou zecimale nenule? Dar ultimele trei? a 4. Un grup de prieteni hotrsc s fac o cltorie la Viena. Fiecare aa a a aa ei prezinta la vama acelai numar de bancnote, unele de 100 , altele de s Organizatorul grupului deine un sfert din bancnotele de 100 i o esime d t s s de 100 $. Cte persoane sunt n grup i care e minimul numrului total de ban s a tiind c fiecare trebuie s aib cel puin 5 astfel de bancnote? s a a a t Mihaela C

Clasa a VI-a
1. Pe ase recipiente avem scrise capacitaile lor: 8 l, 13 l, 15 l, 17 l, s t 38

respectiv 31 l. Recipientele sunt umplute cu ulei sau oet. Un client cump t 840000 lei oet i tot de 840000 lei ulei, golind cinci din cele ase recipiente i t s s s unul singur neatins. Care recipient a fost neatins? Care este preul unui litru d t tiind c preul uleiului este de dou ori mai mare dect preul oetului? s a t a t t Nicu 2. Fie numerele naturale nenule a1 < a2 < a3 < a4 < a5 < a6 . Spun mulimea {a1 , a2 , . . . , a6 } are proprietatea P daca k {3, 4, 5, 6}, i, j {1, 2, 3 t a t i 6= j astfel nct ak = ai + aj . S se afle cte mulimi cu proprietatea P s forma {1, 2, a, b, c, d}. Petru A 3. Se considera triunghiul ABC dreptunghic n A. Daca AB = 2AC, ara b msura unghiului C este mai mic dect 67 300 . a a Petru A 4. Sase drepte se afla n acelai plan. Aratai ca cel puin doua dintre s t t drepte fac ntre ele un unghi cu msura mai mic dect 31 . a a

Clasa a VII-a

1 7 1 + = 1. S se rezolve ecuaia a t . 2+4 2 + 11 12 x x 2. Sa se arate ca toate dreptele care mpart un dreptunghi n doua pari t egale sunt concurente. 3. Un cerc este mprit n n pri egale. Plecnd din fiecare punct de div at at se numr m puncte consecutive i se unete punctul iniial cu cel obinut (p aa s s t t unite nu sunt diametral opuse). Sa se demonstreze ca nu exista trei drepte fie concurente n interiorul cercului. 4. Alice i Bob au un scule cu 2003 bile. Alice scoate ntre una i trei b s a t s scule dup care Bob are dreptul s scoat i el ntre una i trei bile. Proce a t a a as s repeta pna la extragerea tuturor bilelor din sacule. Aratai ca Alice poate p t t n aa fel nct s extrag ea ultima bil, indiferent de felul n care acioneaz s a a a t

Clasa a VIII-a
2 2

(n + 7) (n + 6) (n + 5) (n + 4) (n + 3) (n + 2) (n + 1) n c) Fie A = {2003, 2004, . . . , 20042}. Artai c exist mulimile B i C di a t a a t s astfel nct B C = A, suma elementelor lui B este egal cu suma elementelo a i suma patratelor elementelor lui B este egala cu suma patratelor elementelor s 2. La o balana braele n care se pun greutaile i marfa trebuie sa fie n ech t t t s Un cumprator a sesizat faptul c balana este defect, deoarece punnd m a a t a un taler i greutile pe cellalt taler i apoi invers, balana nu este n ech s at a s t Cumparatorul, care vrea sa achiziioneze 2 kg, a cerut sa i se cntareasca 1 t marfa ntr-un mod i 1 kg de marfa n celalalt mod. A ieit n pierdere sau n s s 3. Folosind dou butoaie cilindrice, unul de 50 l, altul de 60 l i avnd or a s mult ap la dispoziie, prin mai multe msurtori s se obin 55 l de ap. a a t a a a t a a Ctlin Bu a a 39

1. a) Demonstrai c (n + 3)2 (n + 2)2 (n + 1)2 + n2 = 4, n N. t a b) Aratai ca putem alege semnele astfel nct n N sa aiba loc egalitate t
2 2 2 2 2

a a a 4. Fie ABCA0 B 0 C 0 o prism triunghiular regulat cu toate muchiile ega a) Determinai poziia punctului M pe segmentul [BB 0 ] astfel nct A4A t t fie minima. b) Dac M este mijlocul segmentului [BB 0 ], s se determine unghiul dintre a a (ABC) i (AM C 0 ). s

Not de cititor a

Muli dintre nvatorii ieeni au fost probabil contrariai n primele mome t at s t fel ca i mine, cnd au aflat de organizarea unui concurs de matematic, la s a IVIII avnd pe generic numele "Florica T. Cmpan". Dupa ce surpriza a trecut, curiozitatea i-a facut loc printre gnduri de t s i m-a ndemnat s aflu ce zvcnire de spirit se ascunde n spatele acestui num s a chip nc pentru mine, dar care a determinat o mobilizare considerabil de fo a a Aveam s constat n scurt timp c bibliotecile aveau suficiente materiale a a ma ajute sa gasesc raspunsuri convingatoare. Din paginile crilor rsfoite sau citite cu aviditate, se contura personalitate at a om de cultur, profesor de prestigiu i dttor "de cari atractive si lamur a s aa t (D. Brnzei ) ale geometriei, ale irurilor de numere i ale ptratelor magi s s a istoriei matematicii. Nu mi-am propus n aceast not o incursiune n bibliografia acestei Do a a a matematicii. Au fcut-o alii cu mai mult rvn i pricepere nainte a t a a s M-am gndit doar c sfritul toamnei poate constitui pentru ieeni (i nu a s s s un prilej de aducere aminte a faptului ca pe 26 noiembrie 1906 la Iai, pe t s matematicii o "aleas a Domnului" se ivea s-i mplineasc harul. Cci a as a a profesor doctor-docent Florica T. Cmpan matematica nu este o simpl a Ea reprezint, ca i credina, o cale prin care poi s fii mai aproape de div a s t t a Rndurile mele se doresc a fi mai mult un prilej de a scrie despre ceva dra redescoperirea prin lectur a universului matematicii. a Si poate atunci cnd iarna i va intra n drepturi, vei gsi o clip de rgaz s s t a a a despre "Istoria numarului ", despre "Probleme celebre din istoria matemati traii "Aventura geometriilor neuclidiene", sa aflai cine sunt "Licuricii din ad t t i s simii c "Dumnezeu si matematica" au aceeai esena. s a tt a s t Supleea si persuasiunea discursului matematic, profunzimea discursului fi t savoarea dialogului te fac sa uii ariditatea "terenului" pe care te afli, i aduc t t matica macar mai aproape de suflet, daca nu de minte. Chiar dac, personal, nu am excelat n domeniu i nici timp s-o fac nu m a s m-am aflat prin intermediul d-nei Florica T. Cmpan ntr-un dialog cu mate dincolo de catalog, dincolo de folosirea ei marunta i lenea, la interferena din s t s i divin. s A venit apoi firesc ntrebarea: un nvator aproape netiut poate aduce ce at s n lumea matematicii? Rspunsul a venit prompt. Da, poate veni cu pute a de patrundere, cu puina lumina n mintea copiilor, iar daca nu are nimic din t acestea, poate veni cu sufletul... Pentru ca ea, MATEMATICA, este preten VREA TOTUL!

nv. Luminia Murariu, Scoala "Elena Cuza", Ia t 40

Concursul "Adolf Haimovici", ediia a VII-a1 t


pentru liceele economice, industriale i agricole s

Faza judeean, 22 februarie 2003 t a Clasa a IX-a

1. Sa se demonstreze ca exista o singura funcie f : R R, f (x) = ax2 + t unde a, b, c R, a 6= 0, astfel nct a, , P i S s fie numere ntregi consecu s a aceast ordine. a 2. Sa se arate ca daca a, b, c [0, +) i a + b + c = 1, atunci s 8 abc (a + b) (b + c) (c + a) . 729 3. S se scrie ecuaiile laturilor unui triunghi ABC dac se cunosc A ( a t a ecuaiile a doua mediane: x 2y + 1 = 0 i y 1 = 0. t s

Clasa a X-a

1. a) Se considera numerele reale strict pozitive a1 , a2 , a3 cu produsul p = diferit de 1. Dac m = logp a1 , n = logp a2 , s se exprime n funcie de m i n n a a t s logp aq , unde q R. 3 b) Daca m = log70 2, n = log70 5, calculai log70 49. t c) Rezolvai ecuaia 3|x+1| 2 |3x 1| = 3x + 2. t t 1 1 1 1 < 2. a) Aratai ca < 2 , a (0, ). t a a+1 a (a + 1)2 1 1 1 1 + + + + < 102 . b) Demonstrai c t a 1012 1022 20022 20032 3. Fie O mijlocul laturii BC a triunghiului ABC, M un punct pe perpend n O pe planul (ABC). Fie D proiecia pe BC a lui A, E proiecia pe M B a t t 1 F proiecia pe M C a lui A. Artai c dac M O = BC, atunci (ADE) ( t a t a a 2

1 c = 0, unde a, b, c C. D c2 c3 2 x 2. Valorile parametrului real a pentru care matricea A = x 1 3 x+2 a este inversabila pentru orice x R sunt: 1 1 a) , 2 ; b) , (2, ); c) (, 1) (2, ); d) ; e) R. 2 2 n Q 1 1 k , n 1. II. 1. Sa se studieze convergena irului an = t s 2 k=1 1 cos x cos 2x . . . cos nx 2. Fie an = . x2 1 a a2 a3 1 b b2 b3
1

Clasa a XI-a

1 x I. 1. S se rezolve ecuaia n x: 2 a t x 3 x

Not. Toate subiectele sunt obligatorii. Timp de lucru: 2 ore. a

41

a) S se determine lim a1 . a
x0

b) Sa se demonstreze ca an = an1 cos nx + c) Sa se calculeze lim an .


x0

1 cos nx . x2

a) a = c = 1, b = 0; b) a = 0, b = 1, c < 0; c) a > 0, c < 0, b = 0; d) / e) a = b = c = 0.

(x + ex )1/x , x 6= 0 III. 1. Studiai continuitatea funciei f (x) = t t n x 1, x = 0 2. Valorile a, b, c R aa nct lim x ax cx2 + bx 2 = 1 sunt: s
x

Clasa a XII-a

I. Fie mulimea A = (0, ) {1}, a A, R . Definim pe A le t compoziie x y = x loga y . Notm cu Ga, = (A, ). Demonstrai c Ga, est t a t a abelian. 1 + z + z2 II. Artai c dac z C \ R i a t a a s R, atunci |z| = 1. 1 z + z2 III. Sa se determine k R astfel nct funcia f : R R, 1. t ln x2 + 1 , x > 0 s admit primitive i s se gseasc o primitiv a a s a a a f (x) = xe2x + k, x 0 x2 + 1 2. Determinai primitivele funciei f : D R, f (x) = 2 t t x 2x2 cos 2 + 1 este oarecare din intervalul (0, /4), iar D R.

x2 + (m + 1) x 5 1. S se determine m R astfel nct 7 < a < 3, x R x2 x + 1 2. Fie a R fixat. S se rezolve ecuaia 3 x + 3 a3 x = a. a t 1 3. a) S se demonstreze identitatea sin 10 cos 20 cos 40 = . a 8 DM b) Se da paralelogramul ABCD. Fie M [DC] astfel nct = k i N s DC astfel nct M N = k N A. Sa se arate ca D N B coliniare.

Faza interjudeean, Iai, 9 - 11 mai, 2003 t a s Clasa a IX-a

Clasa a X-a
2. Fie numerele a, b, c (1, 2]. S se demonstreze egalitatea: a loga (3b 2) + logb (3c 2) + logc (3a 2) 6.

a a 4 1. Fie (an )n1 o progresie aritmetic. Dac 3a2 + a2 + 6 (a4 + a6 + 2) = 0 6 a) a1 , r; b) a8 , S8 .

3. a) Daca a, b R , n dezvoltarea (a + b)n nu exista trei termeni cons egali; n b) S se arate c partea ntreag a numrului 3 + 2 2 este un numr n a a a a a impar, n N . 42

Clasa a XI-a

q i f : R R, f (x) = 3 (x a)2 (x b). 1. Fie a, b R distincte s a) Sa se determine mulimea A = {x R | f are derivata n x}; t b) S se arate c funcia f are dou puncte de extrem local x1 , x2 , iar dac a a t a atunci x1 , x2 [a, b). 1p + 2p 2! + 3p 3 3! + + np n n! . 2. S se calculeze lim a n np+2 a 0 ... 0 b 0 0 ... 0 0 3. Fie A Mn (R), A = . . . . . . . . . . . . . . ., 0 a2 + b2 < 1. 0 0 ... 0 0 b 0 . . . 0 a arate c Am 6= On , m N , dar lim Am = On . a
m

Clasa a XII-a

1. Fie G = R \ {1} i operaia x y = xloga y , x, y G, a > 0, a 6= 1 s t + arate ca (G, ) este grup abelian i (G, ) (R , ). s = Z sin x 2. Sa se calculeze dx. 3 sin x + 4 cos x 3. Fie G = a + b 5 | a, b Z, a2 5b2 = 1 . a) S se arate c (G, ) este grup abelian; a a b) S se arate c G are cel puin 2003 elemente. a a t

Recreaii matematice
Soluiile problemelor enunate la pagina 18. t t 1. Cei patru oameni procedeaza astfel: a i b trec podul (2 minute); s a se ntoarce (1 minut); c i d trec podul (10 minute); s b se ntoarce (2 minute); a i b trec podul (2 minute). s n acea noapte, dup 17 minute, a, b, c i d trec astfel podul. a s

2. Modificarea necesar se vede pe figur. a a

43

Concursul Traian Lalescu, ediia a IV-a1 t


mai 2003, Iai s

1. Sa se calculeze: 1 (6 + 12 : 3) : 10. 2. Suma a doua numere este 60. Daca unul dintre ele este de 3 ori ma dect cellalt, s se afle diferena lor. a a t 3. S se afle x din egalitatea: 33 + 3 [(3 + 99 : x) 9 33] = 96. a 4. S se afle restul mpriri numrului a = 1903 + 1904 + 1905 + . . . + 2 a at a 2003. 5. Sa se calculeze: (900 1 2 . . . 40) 80 80 80. 6. Sa se determine numerele naturale nenule a, b, c tiind ca: a[7 + 4 (3b + s = 35. 7. Sa se determine cte numere de trei cifre abc au proprietatea: abc = cb 8. Elevii unei clase, n numr de 30, au participat la un concurs de r a probleme. Stiind c 25 elevi au rezolvat bine prima problem, 24 pe a doua, 2 a a treia i 22 pe a patra, s se determine numrul minim de elevi care au rezolv s a a toate problemele. 9. ntr-o sala de spectacole scaunele sunt aezate cte 25 pe rnd. Daca s ocup locul 630 pe rndul din mijloc, ce loc ocup Cristina, care este pe ultimu a a n dreptul Ioanei? 10. Ana i Maria au mpreun 63 de ani. Ana are n prezent de dou ori ma s a a ani dect a avut Maria atunci cnd Ana avea ct are Maria acum. Sa se det ce vrsta are acum Ana i ce vrsta are Maria. s 11. ntr-o clas fiecare biat este prieten cu trei fete i fiecare fat este p a a s a cu doi biei. Dac n clas sunt 19 bnci (de cte dou locuri) i 31 de ele a t a a a a s pasionai de matematic, ci elevi sunt n clas? t a t a 12. Elevii prezeni la Concursul de matematica "Traian Lalescu" au fost t tizai n mod egal n 18 sali de clasa, astfel nct n fiecare sala numarul elev t fie mai mare dect 11 i mai mic dect 17. Dac numrul bieilor este de pa s a a a t mai mic dect numrul fetelor, s se afle numrul concurenilor. a a a t 13. n ptratul alturat suma numerelor de pe fiecare linie, a a b a de pe fiecare coloana i de pe fiecare din cele doua diagonale este s c 5 aceeai. Sa se determine numerele a, b, c, d, e. s
22 e

14. Fiecare numr nscris ntr-un ptrat din figura alturat a a a a este egal cu suma numerelor din ptratele pe care se sprijin. S a a a se determine numerele a, b, c.
3

21 b a

Not. Fiecare subiect va notat cu cinci puncte. Timp de lucru: 2 ore. a

44

Olimpiada de matematic cl. a V-a i a VIa s


Etapa judeean, 10 mai 2003 t a
Not. Toate subiectele sunt obligatorii. Timp de lucru: 2 ore. a

Clasa a V-a
I.

Petru A 2. Avem la dispoziie un numr nelimitat de jetoane pe care sunt t a numerele naturale 5, 7 sau 11. Spunem c "am obinut numrul n" dac pute a t a a jetoane cu suma numerelor de pe ele egala cu n. Aratai ca 13 este cel ma t numr care nu poate fi obinut. a t Valerica III. La un stadion cu capacitatea de 10000 locuri, vin spectatorii. n minut vine un spectator, n al doilea minut vin trei spectatori, n al treilea min cinci spectatori i aa mai departe. S se afle dup cte minute se umple stad s s a a

1. Suma a k numere naturale consecutive poate fi o putere a lui 2 (k 2. Sa se gaseaca restul mparirii numarului n = 10001000 la 27. t Aurel II. 1. Determinai cel mai mic numr k N astfel nct 93 divide 999 . t a | {z

k cif

Clasa a VI-a

I. Artai c dac x + y, y + z, z + x sunt direct properionale cu numerele a t a a t x z 2 a + 2 i a + 3, a N , atunci + 4 . s y x 3 t II. Fie a1 , a2 , a3 , b1 , b2 , b3 , c1 , c2 , c3 Z . Demonstrai ca numerele: a2 b3 c1 , a3 b1 c2 , a1 b3 c2 , a2 b1 c3 , a3 b2 c1 nu pot fi simultan pozitive. III. 1. n triunghiul ABC, bisectoarea unghiului B intersecteaz nlime a at M (BC), n punctul O. Construim OP AB, P (AB). a) Daca P este mijlocul lui [AB], demonstrai ca masura unghiului BA t 30 . b) Dac O este centrul de greutate al triunghiului, artai c triunghiul AB a a t a echilateral. Marius F 2. Se consider triunghiul ABC i punctele M (AB), N (AC) astfe a s \ [BM ] [AN ]. Sa se calculeze masura unghiului CP N , unde BN CM = {P

ERRATA

2. n enunul problemei V.40 (RecMat 1/2003, p. 80) n loc de "2n + t mulimi " se va citi "4n + 3 submulimi ". t t

1. n finalul soluiei problemei XII.26 (RecMat 1/2003, p. 64) au fost t rndurile: ". . . n cazul A = R. Daca A = Q, atunci (M, ) (Q, +), nsa gr = s (Q, +) si Q , nu sunt izomorfe. n sfrit Z , nu este grup.". + +

45

Olimpiada Balcanic de Matematic pentru Jun a a


Ediia a VII-a, Izmir (Turcia), 20 - 25 iunie 2003 t

A. Problemele de concurs - enunuri i soluii t s t 1. Fie n N . Numarul A este format din 2n cifre de 4, iar numarul format din n cifre de 8. S se arate c A + 2B + 4 este ptrat perfect. a a a S. Grkovska, Mace 2. Fie n puncte n plan, oricare trei necoliniare, cu propietatea (P ): am numerota aceste puncte A1 , A2 , . . . , An , linia frnta A1 A2 . . . An nu se a tersecteaza. Gasii valoarea maxima a lui n. t D. Serbnescu, Ro a

2. Exist mulimi de 4 puncte cu proprietatea (P ), de exemplu mulim a t t furilor unui patrulater concav. Vom arta c nu exist mulimi cu n 5 a a a t satisfcnd (P ). S observm c dac printre cele n puncte exist patru A, a a a a a a D astfel ca ABCD sa fie patrulater convex, cu renotarea A1 = A, A2 = C, A A4 = D am avea [A1 A2 ] [A3 A4 ] 6= , deci (P ) nu ar avea loc. Artm c aa a n 5, putem selecta 4 puncte care s fie vrfurile unui patrulater convex. a arbitrar 5 puncte din mulime i considerm nchiderea lor convex. Dac acea t s a a a este triunghi, problema este rezolvata. Daca este triunghi, dreapta determin cele dou puncte din interiorul lui taie exact dou laturi ale triunghiului; fie A a a comun al acestor dou laturi. Cele patru puncte rmase determin un pat a a a convex, ceea ce ncheie soluia. t b b 3. a) Notam m(A) = m(BD) = m(DC) = , m(B) = m(AE) = m(EC 1 1 b b m(C) = m(AF ) = m(F B) = . Atunci m(D) = ( + ) = (180 ) = 9 2 2 \ ) = 90 , iar m(DF E) = 90 . Pe de alt parte, m(E \ \ Analog, m(DEF a 2 2 46

/ / s C AB, A BC, B CA i fie F , D, respectiv E mijloacele acestor arce. No / G, H punctele de intersecie ale lui DE cu CB, respectiv CA i cu I, J punc t s intersecie ale lui DF cu BC, respectiv BA. Fie M , N mijloacele lui [GH], re t [IJ]. a) Gasii unghiurile 4DM N funcie de unghiurile 4ABC. t t b) Dac O este centrul cercului circumscris 4DM N i {P } = AD EF , a s c O, P , M i N sunt conciclice. a s Ch. Lozanov, Bu 1 + x2 1 + y2 1 + z2 4. Fie x, y, z (1, ). Sa se arate ca + + 1 + y + z 2 1 + z + x2 1 + x + y L. Panaitopol, Ro 1. Soluia redaciei. Se constata uor ca t t s 102n 1 10n 1 .. } .. } + 16 + A + 2B + 4 = 4 11 {z . 1 + 16 11 {z . 1 + 4 = 4 | | 9 9 n 2n 2 4 102n + 16 10n + 16 2 10n + 4 = 66 .{z. 68 2 . = = | . } 9 3
n

3. Pe cercul circumscris 4ABC, arcele AB, BC, CA se consider astfe a

a b c a2 b2 c2 (a + b + c) + + = + + 2c + b 2a + c 2b + a 2ac + ab 2ab + bc 2bc + ac 3 (ab + bc + a cu egalitate daca i numai daca a = b = c. Egalitatea n inegalitatea iniiala se t s pentru x = y = z = 1. Altfel, () se poate demonstra renotnd numitorii A = 2c + b, B = 2 C = 2b + a; dup calcule, se obine a t B C A B C A + + +4 + + 15, A, B, C > 0. A B C A B C r C A B C A B Este nsa clar ca + + 33 = 3 i analoaga. s A B C A B C

i i A 1h 1h m(AE) + m(CD) = m(CE) + m(BD) = 2 2 P L \ \ \ = m(CGE), deci CHG CGH, adica 4CGH este F K isoscel. Cum CF este bisectoare, ea va fi mediana i s \ ) = 90 . H nlime, prin urmare M CF i m(EM F at s J \ Analog se arat c m(F N E) = 90 , deci patrulaterul a a M N \ EM N F este inscriptibil i atunci m(DM N ) = s B I G \ \ \ = m(DEF ) = 90 , iar m(DM N ) = m(DF E) = 2 90 . D 2 b) Fie {K} = AB EF , {L} = AC EF ; ca la punctul a) se arata ca AP \ \ \ [ m(F P N ) = m(EP M ) = 90 . nsa m(AKP ) = m(ALP ) = 90 2 2 \ \ AB k P N i AC k P M , de unde m(M P N ) = m(BAC) = . Avem ca 4DM s \ s ascuitunghic (i. e. O este punct interior lui), iar m(M DN ) = 90 i t 2 \ \ \ m(M ON ) = 180 . Urmeaza ca m(M ON ) + m(M P N ) = 180 , adica p O, P , M , N sunt conciclice. 1 + y2 1+x 4. Deoarece y (cu egalitate pentru y = 1), avem ca 2 1+y+ 2 1 + x2 . Scriind analoagele i adunndu-le, cu notaiile a = s t 2 (1 + z 2 ) + (1 + y 2 ) b = 1 + y 2 , c = 1 + z 2 , inegalitatea de demonstrat devine a b c + + 1, a, b, c > 0. 2c + b 2a + c 2b + a Pentru a demonstra (), folosind Cauchy-Schwarz i binecunoscuta a2 + b2 s ab + bc + ac, avem =

B. Probleme aflate n atenia juriului - enunuri t t a b c a 1. Fie a, b, c lungimile laturilor unui triunghi, p = + + , iar q = + b c a c Sa se arate ca |p q| < 1. 2. Fie a, b, c R cu a2 +b2 +c2 = 1. S se arate c ab+bc+ca2 (a + b + c) a a 47

r 1 c3 1 1 + = . Artai c a t a Q. a + bc b + ac a+b c+1 4. 2 Fie a, b, c 2N lungimile laturilor unui triunghi neisoscel. Demonst ab + bc2 + ca2 a b b2 a c2 a 2. 5. Fie a, b, c (0, ) cu ab + bc + ca = 3. Aratai ca a + b + c abc + 2. t 6. Demonstrai c exist mulimi disjuncte A = {x, y, z} i B = {m, n t a a t s numere naturale mai mari ca 2003 astfel nct x + y + z = m + n + p i x2 + y 2 s = m2 + n2 + p2 . 7. Numerele 1, 2, 3, . . . , 2003 sunt renotate a1 , a2 , a3 , . . . , a2003 . Definim b b2 = 2a2 , b3 = 3a3 , . . . , b2003 = 2003a2003 i fie B cel mai mare dintre bk , k = 1 s a) Dac a1 = 2003, a2 = 2002, . . . , a2003 = 1, gsii valoarea lui B. a a t b) Demonstrai ca B 10022 . t 8. Fie M = {1, 2, 3, 4}. Fiecare punct al planului este colorat n rou sau al s Artai c exist cel puin un triunghi echilateral de latur m M cu vrfu a t a a t a aceeai culoare. s 9. Exista un patrulater convex pe care diagonalele sa-l mparta n patru ghiuri cu ariile numere prime distincte? 10. Exist un triunghi cu aria 12 cm2 i perimetrul 12 cm? a s t 11. Fie G centrul de greutate al 4ABC, iar A0 simetricul lui A faa de C arate c G, B, C, A0 sunt conciclice dac i numai dac GA GC. a as a 12. Trei cercuri egale au n comun un punct M i se intersecteaz cte d s a A, B, C. Demonstrai ca M este ortocentrul 4ABC 1 . t 13. Fie 4ABC cu AB = AC. Un semicerc de diametru [EF ], cu E, F este tangent laturilor AB i AC n M , respectiv N , iar AE retaie semicercu s Demonstrai c dreapta P F trece prin mijlocul coardei [M N ]. t a 14. Paralelele la laturile unui triunghi duse printr-un punct interior mpart interiorul triunghiului n 6 pari cu ariile notate t c b a b c 3 ca n figur. Demonstrai c + + . a t a 2 a 3. Fie a, b, c Q cu

Echipa Romniei a fost condusa de prof. Dan Brnzei, asistat de prof. Serbnescu. n clasamentul neocial pe naiuni, Romnia a ocupat primul a t 205 puncte din 240 posibile, urmat de Bulgaria (182 puncte) i Turcia (124 p a s n continuare, s-au situat R. Moldova, Serbia, Macedonia, Grecia i Ciprul. C s nenii echipei Romniei au obinut urmatoarele punctaje i medalii (cu men t t s c primii doi sunt singurii elevi care au realizat punctajul maxim!): a Drago Michnea (Baia Mare) s 40 p Aur Adrian Zahariuc (Bacau) 40 p Aur Lucian Turea (Bucureti) s 38 p Aur Cristian Tlu (Craiova) aa 37 p Aur Sebastian Dumitrescu (Bucureti) 29 p Argint s Beniamin Bogoel (Arad) s 21 p Bronz.
1

t Identicat drept problema piesei de 5 lei a lui Ti eica. a

48

Un nou concurs internaional de matematic t a

n primvara anului 2001, Romnia a lansat prin "Fundaia pentru Inte a t European Sigma", un nou concurs internaional de matematica. Competi a t t tulata "MCM - Multiple Choice Contest in Mathematics", este prevazu desfura pe echipe de cte 4-6 elevi ce aparin la patru categorii de vrst: 11as t a 13-14 ani, 15-16 ani i 17-18 ani. n fiecare echip pot intra cel mult dou pe s a a din aceeai grupa de vrsta i este posibila colaborarea ntre membrii ei. Fieca s s ticipant primete cte 20 probleme-grila gradate pe trei nivele de dicultate, s de lucru ind de 90 minute. Oricare dintre probleme are 5 variante de rspun a singur ind corect. Un rspuns bun aduce 4 puncte, unul incorect scade 2 p a a a n timp ce cazurile netratate aduc diminuari de cte 1 punct. Clasamentul este dat de media aritmetica a punctelor obinute de componenii ei. t t Jocul-concurs "MCM" s-a dorit a fi faza nal internaional a ctig a t a s a naionali ai jocului-concurs "Cangurul", adus din Australia n Europa prin t mediul asociaiei pariziene "Kangourou sans Frontires" (n Romnia a t competiie s-a introdus ncepnd cu anul 1994). Competiia a avut loc pe 1 t t 2001 la Poiana Pinului (Buzu) i au participat urmtoarele ari: Austria, B a s a t Bulgaria, Frana, Georgia, Italia, Lituania, R. Moldova, Romnia, Spania, U t i Ungaria. Pe primele trei locuri s-au situat: 1. Romnia, 2. Frana, 3. Bu s t Echipa Romniei a fost formata din elevii: Eduard Dogaru (12 ani), R Leonte (12 ani), Gabriel Kreindler (14 ani), Bogdan Buca (16 ani), T s Pristavu (16 ani) i Bogdan Stan (18 ani). s Vom prezenta mai jos 12 probleme, cte 3 pentru fiecare categorie de vrs

Grupa 1 (11-12 ani)


1.
9 6 A) 9 6 12 3 B) 9 6 12 3 C) 9 6 300 12 3 9 6 12 3 D) 9 6 12 3 315 12 3 9 1730 12

?
6

E) 9

12 3 6

2. a, b, c {0, 1, . . . , 9} ab + bc + ca =? A) 132 B) 48 C) 72 D) 51 E) 37 3. n =?
1 2 3 21 n-2

Laureniu M t
n-1 n

n-1

n-2

A) 28 B) 25 C) 27 D) 29 E) 31 49

Grupa 2 (13-14 ani)


4.
x
7 11 3

x =? A) 15 B) 20 C) 18 D) 16 E) 25

1 1 1 + + ... + 1+ 2 2+ 3 2001 + 2002 A) E < 1 B) 2001 < E < 2002 C) 43 < E < 44 D) 33 < E < 34 E) E 5. E = 6. S =? A) 2 +
1

B)

1 C) + 1 D) 3 + 2 2 4

E)
V

1 3

Grupa 3 (15-16 ani)

G3

7. M1 A1 = M1 A2 , M2 A2 = M2 A3 , M3 A1 = M3 A3 , G1 G2 G2 M2 G3 M3 1 SG1 G2 G3 G1 M1 A1 =? = = = , M3 G1 V G2 V G3 V 2 SA1 A2 A3 M2 M1 1 1 2 4 1 A) B) C) D) E) A2 3 9 4 9 3 8) a2 1 x4 + a2 3 x3 3a2 + 1 x2 + 5a2 + 3 x2 a2 1 = 0, a R x1 = x2 = x3 = 1. a =? A) a = 0 B) a 2 C) a D) a = 2 E) a = 1/2 Laureniu Modan i Dinu Teodo t s


A

9.

Grupa 4 (17-18 ani)

3n+2 + x + 2 = x2 + 3x + 3 C (x) + R (x), n N, 0 grad R (x 10. (3x + 1) A) R (x) = x2 1 B) R (x) = 2x+3 C) R (x) = 0 D) R (x) = 1 E) R (x) = Traian L 0 1 2001 11. C2001 + C2001 + . . . + C2001 = a1 . . . an , ai {0, . . . , 9}, i = 1, n. A) an = 2 B) an = 4 C) an = 8 D) an = 1 E) an = 6 Laureniu M t 1, x (0, 1) 0 12. f : R R, f (ln x) = , f (0) = 1, s = f (1) + f (1) x, x [1, ) A) s < 0 B) s = e C) s = 2 D) s = e 2 E) s = (e + 1) /2 Laureniu Modan i Dinu Teodo t s Rspunsuri: 1 A; 2 A; 3 C; 4 A; 5 C; 6 B; 7 A; 8 C; 9 A; 10 C; 11 A; 12 a

b b b BC = 2 AB, A = 3 C, B =? b = 60 B) B = 30 C) B = 50 b b A) B b E) B = 40

b D) B = 90

t Conf. dr. Laureniu Modan Fac. de Cibernetic i Informatic Ecoo as a A. S. E., Bucureti s
50

Soluiile problemelor propuse n nr. 2 / 2002 t


Clasele primare

P.33. Care este cel mai mare numar pe care l spunem atunci cnd nu crescator din doi n doi, din trei n trei sau din cinci n cinci, pornind de la 1 sa depasim 100? ( Clasa I ) Raluca Popa, elev Soluie. Sunt spuse irurile de numere (1, 3, 5, . . . , 97, 99), (1, 4, 7, . . . , 97, t s (1, 6, 11, . . . , 91, 96). Numrul 100 ndeplinete cerina problemei. a s t P.34. Numarul merelor de pe o farfurie este cu 3 mai mare dect cel ma numar natural scris cu o cifra. Numarul perelor de pe aceeai farfurie nu de s numarul merelor, dar este mai mare dect jumatate din numarul acestora. C pot fi pe farfurie? ( Clasa I ) nv. Maria Rac Soluie. Numrul merelor este 9 + 3 = 12. Jumtatea lui 12 este 6. Pe f t a a pot fi: 7, 8, 9, 10, 11 sau 12 pere. P.35. Care dintre numerele 3132, 8182, 3435, 3932, 2021, 5960 este intru ( Clasa a II-a) Matei Luca, ele Soluie. Toate numerele, cu excepia lui 3932, sunt formate cu cifrele t t numere consecutive. Intrusul este 3932. P.36. Cum putem realiza egalitatile 4 4 4 4 = 28 si 4 4 4 4 4 4 = 28

nsernd ntre cifrele 4 de mai sus semnele grafice +, , , : , ( )? ( Clasa a II-a) Alexandru-Gabriel Tudorache, ele Soluie. Pentru prima egalitate avem (4 + 4) 4 4 = 28. Pentru t egalitate avem (4 + 4) 4 4 : (4 : 4) = 28 sau (4 + 4) 4 4 + 4 4 = P.37. Suma a doua numere naturale este 109. Daca l dublam pe prim triplam pe al doilea, suma devine 267. Care sunt numerele? ( Clasa a II-a) nv. Galia Paraschiv Soluie. t

109

267

Dublul sumei numerelor a i b este 1092 = 218. Al doilea numr este 26721 s a iar primul numar este 109 49 = 60. P.38. Cte nmuliri de tipul abc 9 = 8d1e sunt posibile? t ( Clasa a III-a) Sergiu Diaconu, ele Soluie. Avem abc = 8d1e : 9 = 8010 + d0e : 9 = 890 + d0e : 9. Dis t cazurile: d = 1, e = 8; d = 2, e = 7; . . .; d = 8, e = 1; d = 9, e = 0, 9; n to cazuri. Adugnd i cazul d = 0, e = 0, 9, obinem 12 nmuliri posibile. a s t t P.39. Scriei cel mai mic numar natural de sase cifre care ndeplinete, n t s timp, condiiile: a) nu are cifre care se repeta; b) suma cifrelor sale este 30; t 51

mai mare dect 900000. ( Clasa a III-a) nv. Maria Rac Soluie. Cel mai mare numar natural scris cu ase cifre distincte este t s care are suma cifrelor 39. Numrul cutat este 987510. a a P.40. Emilia are de rezolvat un numar de probleme. A hotart sa rezolve probleme pe zi. Ea lucreaza nsa mai mult cu 2 probleme pe zi si termina de r cu 5 zile mai devreme. Cte probleme a avut de rezolvat si n cte zile le-a term ( Clasa a III-a) nv. Doinia Spn t Soluie. Emilia rezolva cte 4+2 = 6 probleme pe zi. n ultimele 5 zile tre t rezolve 5 4 = 20 probleme. Emilia a terminat de rezolvat problemele n 20 : zile i a rezolvat 10 6 = 60 probleme. s P.41. Stiind ca data de 1 Decembrie din anul 2001 a fost ntr-o zi de s sa se afle care va fi urmatorul an n care ziua de 1 Decembrie se va sarbatori zi de duminica. ( Clasa a IV-a) nv. Rodica Rotaru, B Soluie. De la 1.12.2001 pn la 30.11.2002 sunt 365 zile. Deoarece t a = 752+1, ziua de 30.11.2002 este ntr-o smbt. Ziua de 1.12.2002 se va s aa a ntr-o zi de duminica. P.42. Danila Prepeleac i-a propus dracului sa se ntreaca la trnta, dar a-l pune la ncercare i-a spus ca are un unchi, mo Ursila, batrn de 999 an s

saptamni, si de-l va putea trnti pe dnsul, se vor ntrece apoi amndoi . D 5 din vrsta lui mo Ursila depasete cu 220 ani s din vrsta nepotului, ce vr s 8 Danila? ( Clasa a IV-a) nv. Valerica Beldima Soluie. Vrsta lui mo Ursil este 999 ani +52 sptmni = 1000 ani. O p t s a a a din vrsta lui mo Ursil este 1000 : 4 = 250 ani. Cinci optimi din vrsta nep s a reprezinta 250 220 = 30 ani. Vrsta nepotului este 30 : 5 8 = 6 8 = 48 P.43. Primele douasprezece numere dintr-un sir de numere sunt: 1, 2, 0 1, 5, 6, 2, 7, 8, 0. a) Scriei urmatoarele 6 numere din sir; t b) Calculai suma primelor 111 numere din sir. t ( Clasa a IV-a) Alina Stan, elev Soluie. a) Observam ca: (1 + 2) : 3 = 1 (rest 0); (3 + 4) : 3 = 2 (rest 1); (5 t : 3 = 3 (rest 2). Dupa fiecare grupa de doua numere naturale consecutive a fo restul mpririi sumei lor la 3. Urmtoarele ase numere sunt: 9, 10, 1, 11, 12 at a s b) Avem 111 : 3 = 37 grupe de cte trei numere n care intr primele 74 n a naturale cu suma S1 = (1 + 74) 74 : 2 = 2775. n trei grupe consecutiv resturile 0, 1, 2 care au suma 3. Cum 37 = 3 12 + 1, nseamna ca suma res S2 = 12 3 = 36. Suma celor 111 numere este S = S1 + S2 = 2775 + 36 = 28

Clasa a V-a

V.31. Sa se arate ca

1 1 7 1 + + ... + > . 101 102 200 12 52

Petru Asafte

Soluie. Grupnd termenii din sum obinem t a t 1 1 1 1 1 1 1 + > + + ... + = + ... + + ... + 101 102 200 101 150 151 200 50 50 1 1 7 > + = + = . 150 200 3 4 12 V.32. Determinai numerele prime a, b, c pentru care 5a + 4b + 7c = 107. t Mihai Crciun, P a Soluie. Deoarece 5a + 7c trebuie s fie impar, a i c au pariti diferit t a s at prime, unul dintre ele este egal cu 2. Dac a = 2, atunci 4b + 7c = 97, cu s a b = 19, c = 3 i b = 5, c = 11. Daca c = 2, atunci 5a + 4b = 93, cu soluiile s t b = 17; a = 13, b = 7 i a = 17, b = 2. s

cu suma cifrelor 1. La a doua ntrebare, raspunsul este negativ, deoarece n posibil ca att numrul ct i succesorul su s fie multipli de 3. a s a a

V.33. Un numar natural scris n baza 10 are suma cifrelor 603. Este po succesorul sau sa aiba suma cifrelor 1? Dar ca acesta sa aiba suma cifrelor 3 Matei Luca, ele a Soluie. Numrul 99 {z . 9 are suma cifrelor 603, iar succesorul su este 1 t a | | .. }
67 cifre

V.34. Aflai numarul abc, stiind ca abc = 2n ab + 3n bc + 5n ca, unde t Nicolae Stnic, a a Soluie. n mod evident, a, b, c 1. Pentru n = 0, obinem c 89a = b + t t a cum b + 10c 99, rezulta ca a = 1. De aici, b + 10c = 89 i deci b = 9, c = 8. s Pentru n = 1 obinem ca 75a = 22b + 52c, de unde 25 | 25 (b + 2c) + (2c t i deci 25 | 2c 3b. Cum 2c 3b < 25 i 2c 3b 25, obinem 2c 3b = s s t 2c 3b = 25. Dac 2c = 3b, atunci 3 | c. Cum 3 | 75a i 3 | 52c, rezult c a s a a deci 3 | b i de fapt c este multiplu de 9. Obinem deci c = 9, b = 6, a = 8 s t 2c 3b = 25, atunci c = 1, b = 9 i atunci 75a = 250, deci a nu este ntreg. s Pentru n = 2 obinem c 25a = 84b + 258c. Cum 25a 225 i 84b + 2 t a s 342, ecuaia nu are soluie n acest caz. t t Pentru n 3, 2n ab + 3n bc + 5n ca 11 (2n + 3n + 5n ) 1760, deci ecuaia t soluie. n final, abc {198, 869}. t

t V.35. Se da numarul N = 77 . . . 7 cu 2002 cifre. Cercetai daca N se poat ca suma a doua sau trei patrate perfecte impare. Tamara Cula Soluie. N = a 100+77, deci N = M4 +1. Dac N = x2 +y 2 +z 2 , cu x, y t a impare, atunci N = (2k + 1)2 + (2l + 1)2 + (2n + 1)2 = M4 + 3, contradicie t N = x2 +y 2 , cu x, y N impare, se obine n mod analog ca N = M4 +2, contra t

Clasa a VI-a
VI.31. Fie S = a1 a1 . . . a1 + a2 a2 . . . a2 + . . . + an an . . . an , unde k1 , | {z } | {z } | {z }
k1 cifre k2 cifre kn cifre

kn 2. Aratai ca S se divide cu 4 daca si numai daca a1 + a2 + . . . + an se t cu 4. Dumitru Gherman, P 53

i analoagele, obinem ca S = M4 (a1 + a2 + . . . + an ), de unde concluzia. s t VI.32. Aflai ab stiind ca ab = (a b)! ba 3 (unde 0! = 1, iar t = 1 2 3 . . . n, n 1). Nicolae Stnic, a a Soluie. Pentru a b = 0 sau a b = 1 obinem ab = ba 3, deci 9a = t t contradicie. Pentru ab = 2 obinem ab = 2ba6, deci 8a = 19b6. Cum a t t rezulta, ca a = 4, b = 2. Pentru ab =3, avem ab = 6ba18, adica 55b = 30, a Pentru a b 4 obinem ca (a b)! ba 3 24 ba 3 24 (10b + b + t deci (a b)! ba 3 264b 264, contradicie. n final ab = 42. t

Soluie. Deoarece a1 a1 . . . a1 = a1 11 {z . 1 = a1 11 . . . 100 + 12 1 = M t | .. } | {z }


k1 cifre k1 cifre

VI.33. ntr-o urna sunt bile albe, roii, negre si albastre. Numarul bilel s 3 este din numarul celorlaltor bile; bilele roii reprezinta jumatate din celelal s 5 iar bilele negre a treia parte din numarul celorlaltor bile. Daca extragem calculai probabilitatea ca aceasta sa fie roie sau albastra. t s Marcel Rotaru, B Soluie. Notm numrul total de bile cu n. Cu ajutorul proporiilor d t a a t 3n n , iar al bilelo obinem c numrul bilelor roii este , al bilelor albe este t a a s 3 8 3n n este . De aici rezult c numrul bilelor albastre este a a a . Probabilitatea 4 24 3 n/3 + n/24 = . este atunci p = n 8 AB VI.34. n 4ABC, fie M mijlocul laturii [BC]. Daca d (M, AC) = , 2 b ca m(A) = 90 . N. N. Hra t Soluie. Fie M N AC i BA0 AC, M, A0 AC. Atunci M N = d (M, t s BA0 . Rezult de aici c BA0 a a M N este linie mijlocie n 4BA0 C, deci M N = 2 0 b = 90 . i deoarece BA AC, rezulta ca m(A) s b b VI.35. n 4ABC, m(A) = 60 si m(C) = 45 . Bisectoarea [AD si na AM [BE] se intersecteaza n M (cu D [BC], E [AC]). Sa se arate ca = EC Romeo Cerna \ Soluie. n 4ABC, m(EBC) = 180 90 45 = 45 , deci 4EB t \ \ isoscel cu EC = BE. Deoarece m(ABM ) = 75 45 = 30 = m(BAM ), 4 \ ) = 90 i m(EAM ) = 30 \ este isoscel cu AM = BM . n 4AEM , m(AEM s AM AM AM AM AM 2 ME = . De aici, = = = = . 2 EC EB BM + M E AM + AM/2 3

Clasa a VII-a

VII.31. Sa se rezolve ecuaia 1 + a2x + b2x = ax + bx + ax bx , cu a, b R , t Dumitru Neag Soluie. nmulind egalitatea cu 2 obinem, dupa gruparea convenabila a t t t 54

nilor, (ax 1) (bx 1) + (ax bx ) = 0, de unde ax = bx = 1, deci x = 0 VII.32. Fie a si b, c lungimile ipotenuzei si respectiv catetelor unui t 1 2 1 1 2 2 dreptunghic. Sa se arate ca a + b + c + 2 + 2 10. a2 b c Claudiu - Stefan Pop t Soluie. Deoarece a2 = b2 + c2 , obinem ca t 1 2 1 1 1 b2 + c2 a + b2 + c2 + 2 + 2 10 2 b2 + c2 + 2 2 a2 b c b2 + c2 b c 2 2 2 b + c2 4 b2 c2 0, 2 c2 b ultima inegalitate fiind evidenta. VII.33. Triunghiul ABC are unghiul A obtuz si semiperimetrul p. Cercu diametre [AB] si [AC] delimiteaza o suprafaa comuna S. Aflai valoarea de t t a propoziiei: Exista P S astfel nct d1 + d2 + d3 = p, unde di sunt dis t de la P la laturile triunghiului ABC. Ctlin Calistr a a Soluie. Este cunoscut ca ntr-un triunghi dreptunghic mediana ce plea t vrful unghiului drept este jumatate din ipotenuza, n timp ce ntr-un triun tuzunghic mediana ce pleac din vrful unghiului obtuz este mai mic stric a a jumtate din latura ce se opune unghiului obtuz. a Fie P S i A1 , B1 , C1 mijloacele laturilor [BC], [CA], [AB]. Atunci d1 +d2 s \ [ \ P A1 +P B1 +P C1 . Cum m(AP B) 90 , m(AP C) 90 , m(BP C) 90 , puin una din inegaliti este strict, prin aplicarea rezultatului menionat a t at a t c a b s t obinem c d1 + d2 + d3 < + + , deci d1 + d2 + d3 < p i propoziia din t a 2 2 2 este falsa. VII.34. Fie ABC un triunghi, iar I un punct interior lui. Daca ce nscrise n triunghiurile AIB, BIC si CIA sunt congruente si tangente dou doua, atunci 4ABC este echilateral. Ioan Scleanu, H a a Soluie. Fie C1 (O1 , r), C2 (O2 , r), C3 (O3 , r) cercurile nscrise n 4BIC, 4 t 4AIB i notm T1 = C2 C3 , T2 = C1 C3 , T3 = C1 C2 ; evident, T1 AI, T2 s a s T3 CI. Atunci O1 O2 = O1 T3 + T3 O2 = 2r i analog O1 O3 = O2 O3 = 2 4O1 O2 O3 este echilateral. n patrulaterul IT1 O3 T2 , \ m(T1 IT2 ) = 360 90 90 60 = 120 ,

[ [ [ adic m(AIB) = 120 i analog demonstrm c m(BIC) = m(CIA) = 12 a s a a 4AIB, b b m m(A) + m(B) [ [ [ = 90 + m(AIB) = 180 m(IAB) m(IBA) = 180 2 b b b t a s de unde m(C) = 60 i analog m(A) = m(B) = 60 , de unde obinem c 4 este echilateral. VII.35. Fie ABCD un paralelogram, O centrul cercului circumscris 4AB H ortocentrul 4BCD. Sa se arate ca punctele A, O, H sunt coliniare. Constantin Cocea i Dumitru Neag s 55

\ Soluie. Deoarece BH CD i AB k CD, BH AB i deci m(ABH) t s s \ = 90 . De aici rezult c ABHD este inscriptibil analog obinem c m(HDA) t a a a \ m(ABH) = 90 , [AH] este diametru pentru cercul circumscris triunghiului deci l conine pe O. t

Clasa a VIII-a
VIII.31. Sa se determine mulimea A = t Soluie. Deoarece t

4n 1 (m, n) N N | N mn + 1 A. V. Mihai, Buc

4n 1 N, rezult c 4n1 mn+1. De aici, n(4m) a a mn+1 5 4n 1 i deci m < 4, adic m {1, 2, 3}. Pentru m = 1 obinem c = 4 s a t a mn + 1 n+ 3 4n 1 = 2 N, deci ceea ce implica n = 4. Pentru m = 2, avem ca mn + 1 2n + 1 4n 1 n2 Pentru m = 3 obinem ca t = 1+ . Cum 3n + 1 > n 2, este n mn + 1 3n + 1 ca n 2 = 0, deci n = 2. n final, A = {(1, 4) , (2, 1) , (3, 2)}.

Cum x2 x + 1, x2 x + 2, . . . , x2 x + 2002 > 0, x R, rezult c x {0 a a VIII.33. Determinai funciile f : R R pentru care t t 1 + f (x + y) f (x) + f (y) x + y + 2, x, y R.

VIII.32. Sa se rezolve ecuaia t 1 2 2002 + + ... + 2 = 2002. x2 x + 1 x2 x + 2 x x + 2002 Mihaela Predescu, P Soluie. t 1 2 2002 + 2 + ... + 2 = 2002 2x+1 x x x+2 x x + 2002 1 2 2002 1 + 1 + ... + 1 = x2 x + 1 x2 x + 2 x2 x + 2002 1 1 1 = 0. x x2 + 2 + ... + 2 x2 x + 1 x x + 2 x x + 2002

VIII.34. Fie AB dreapta soluiilor ecuaiei x y = 5 si CD dreapta so t t ecuaiei x + y = 3, cu A, C Ox, B, D Oy. a) Aratai ca AB C t t Calculai aria si perimetrul triunghiului BCD; c) Aratai ca AD BC. t t Vasile Solcanu, Bogdneti (Su a s Soluie. a) Printr-un calcul imediat se deduce c A (5, 0), B (0, 5), C t a D (0, 3). Fie AB CD = {E} i EF BD, F Oy. Atunci E (4, 1), F ( s 56

Gheorghe Iure Soluie. Pentru x = y = 0 obinem 1 + f (0) 2f (0) 2, de unde f (0 t t Fiindc f (x) + f (y) x + y + 2, pentru y = 0 gsim c f (x) x + 1, a a a Deoarece f (x) + f (y) 1 + f (x + y), pentru y = x obinem ca f (x) 2 f t Dar f (x) x + 1, de unde f (x) 1 + x, x R, ceea ce implica f (x) =

Deoarece EF = BF = F D = 4 i EF BD, 4BEF i 4F ED sunt dreptu s s \) = m(F ED) = 45 , de unde m(BED) = 90 i deci AB \ \ isoscele i m(BEF s s b) SBCD = 12, PBCD = 34 + 3 2 + 8. c) Deoarece AO i DE sunt nlimi n 4BAD i AO DE = {C}, s at s ortocentrul 4BAD i deci BC este de asemenea nalime n 4BAD. s t VIII.35. Fie ABCDA0 B 0 C 0 D0 un cub de muchie a. Determinai poziiile p t t lor M (BB 0 ) si N (CC 0 ) pentru care perimetrul patrulaterului strmb AM este minim; aflai aceasta valoare minima. t Mihaela Buctar a Soluie. PAMN D0 = AM + M N + t A' B' C' +N D0 + AD0 este minim AM + M N + N +N D0 este minim. Desfurnd suprafaa a as t M lateral a cubului ca n gur, S = AM + a a +M N + N D0 este minima A, M, N, D0 sunt coliniare. Atunci Smin = AD0 = a 10, B C iar valoarea minim a PAMN D0 este AD0 + A a +Smin = a 2 + a 10.

a a Urmeaz c dac m Z \ {1, 0, 1, 2}, atunci (2n + 1) se afl ntre ptratele a a a numere ntregi consecutive, ceea ce este imposibil. Pentru m = 1, obinem n = 0 sau n = 1 i u = 0. Pentru m = 0 obin t s t nou n = 0 sau n = 1 i u = 0. Pentru m = 1 nu exist n cu proprietatea c s a iar pentru m = 2 obinem n = 5 sau n = 6 i u = 30. n concluzie, A C = { t s 3 4 Fie acum u = n +n = 2m BD; m, n Z. Atunci (n + 1)4 (n 1)4 = ecuaie care nu are soluii nebanale, conform teoremei lui Fermat. Atunci n t t deci m = 1 sau m = 1, de unde u = 2, sau 2m = 0, deci n = 0, de unde u = concluzie, B D = {0, 2}. Analog se trateaza A D i A B. s IX.32. Fie fi , gi : R R, i {1, 2, . . . , n}, funcii care pastreaza t variabilei. Sa se rezolve sistemul: 57

2 (2n + 1)2 = 2m2 + m + 1 + 2m m2 . Folosind semnul funciei de gradul al doilea, obinem t t 2 2 2 (1) (2n + 1) > 2m + m m Z \ {1} , 2 2 2 m Z \ {0, 1, 2} . (2) (2n + 1) < 2m + m + 1
2

x IX.31. Fie mulimile A x2 + x | Z , B = x3 + x | x Z , C = t = t +x3 + x2 + x | x Z , D = 2x4 | x Z . Determinai A C, B D, A D, Andrei Nedelc Soluie. Fie u = n2 + n = m4 + m3 + m2 + m A C; m, n Z. t 2 (2n + 1) = 4m4 + 4m3 + 4n2 + 4m + 1, i grupnd n dou moduri terme s a membrul drept obinem t 2 (2n + 1)2 = 2m2 + m + 3m2 + 4m + 1 , respectiv

Clasa a IX-a

x1 + f1 (x1 ) + g1 (x1 ) = x2 x2 + f2 (x2 ) + g2 (x1 + x2 ) = x3 ... xn + fn (xn ) + gn (x1 + x2 + . . . + xn ) = x1 . Obinei sisteme diverse prin particularizarea funciilor! t t t Iuliana Georgescu i Paul Georgesc s Soluie. Pentru x1 > 0, deducem din prima ecuaie ca x2 > x1 > 0. t t asemntor, 0 < x1 < x2 < . . . < xn < x1 , contradicie. Pentru x1 < 0, o a a t analog c 0 > x1 > x2 > . . . > xn > x1 . De aici, x1 = 0 i analog x2 = . . . = x a s Exemple funcii care pastreaza de t semnul variabilei: f (x) = x2n+1 , f (x) = ar f (x) = x 1 + x2 , f (x) = ln x + 1 + x2 etc. hni hni hni h n i h n i + + = 11 IX.33. Rezolvai n N ecuaia n t t 2 3 12 18 36 h n i h n i hGheorgheiIure i hn n + + Soluie. Fie f : N Z, f (n) = n t 11 2 3 12 18 Avem ca f (n + 36) = f (n), n N, deci f periodica de perioada 36. De ase hni hni n n hn 5n f (0) = 0, iar pentru n = 1, 2, . . . , 35, + + = < n, iar 2 3 2 3 6 36 deci 1, 2, . . . , 35 nu sunt soluii ale ecuaiei. n concluzie, mulimea soluiilor e t t t t este S = {36k; k N}. IX.34. Fie n N, n 2. Sa se determine x1 , x2 , . . . , xn R stiind ca n+1 2 n 2 2 2 x1 + x2 + . . . + xn + = 2 x1 sin + x2 sin + . . . + xn sin 2 n+1 n+1 n Vladimir Martinu s Soluie. Egalitatea din enun revine la t t 2 X n n X n+1 k k xk sin = sin2 . n+1 n+1 2 k=1 k=1 na (n sin cos n P 1 cos 2a 2 2 i cos ka = s Tinnd seama de formulele sin a = a 2 k=1 sin 2 n 2 P k k xk sin = 0, de unde xk = sin , k = 1, n. obinem ca t n+1 n+1 k=1 x+y , x, y 0, . IX.35. Aratai ca sin (cos x) + sin (cos y) < 2 cos t 2 2 DanPopescu, Su , obine t Soluie. Tinnd seama c sin x < x i 0 < cos x < 1, x 0, t a s 2 x+y xy x+ sin (cos x) + sin (cos y) < cos x + cos y = 2 cos cos 2 cos 2 2 2 ceea ce trebuia demonstrat.

Clasa a X-a

X.31. Consideram sirurile (Fn )n0 , (Ln )n0 definite prin F0 = 0, F Fn+2 = Fn+1 + Fn , n 0, respectiv L0 = 2, L1 = 1, Ln+2 = Ln+1 + Ln , qP qP pPn n n 2 2 L2 Fk1 + Sa se arate ca k=1 k k=1 k=1 Fk+1 . Mihail Bencze, B 58

Soluie. Se arat (prin inducie) c Ln = Fn1 + Fn+1 , n N . C t a t a inegalitaii lui Minkowski, urmeaza concluzia. t

X.33. Fie funciile f, g : R (0, ), f (x) = ax , g (x) = bx , unde a t b (0, 1), ab 6= 1. Sa se arate ca exista o infinitate de paralelograme cu vrfu reuniunea graficelor celor doua funcii. t Petru Asafte Soluie. Fie x1 > 0 i B (x1 , g (x1 )). Deoarece f ((, 0)) = (0, 1), x2 < t s fel nct f (x2 ) = g (x1 ) i atunci i f (x2 ) = g (x1 ). Construim A (x2 , f s s A0 (x2 , f (x2 )), B 0 (x1 , g (x1 )). Deoarece ab 6= 1 rezult a x2 6= a c f (x2 )+f 0 0 0 0 ABA B este nedegenerat. Segmentele AA i BB au mijloacele M 0, s 2 g (x1 ) + g (x1 ) 0 0 respectiv N 0, , deci M N i ABA B este paralelogram s 2 x1 a fost arbitrar, problema este rezolvata.

X.32. Fie (an )n0 un sir de numere reale si pozitive. Pentru orice n noteaza cu Pn graficul funciei fn (x) = an x2 + an+1 x + an+2 . Determinai l t t definire a sirului stiind ca parabolele Pn au vrfurile pe axa Ox. Temistocle Brsa Soluie. Condiia ca parabolele sa fie cu vrfurile pe Ox revine la n = t t t 4an an+2 = 0, n N. Prin logaritmare, obinem 2 ln an+1 = ln an + ln an+2 s n N, sau tn+1 = tn ln 4, n N, unde tn = ln an+1 ln an . Aad este o progresie aritmetic cu raia r = ln 4, i deci tn = t0 + nr, ceea a t s a1 plic an+1 = t0 +nr an , n N. Rezult c an = en ln a0 n(n1) ln 2 a0 , d a e a a n a0 a1 . an = n(n1) a0 2

X.34. Sa se arate ca ecuaiile 4 9x + (4x 45) 3x + 11 x = 0 si 4 t 3 6x 128 3x + 2x + 32 = 0 sunt echivalente. t Marcel Chiria, Buc 1 x x (3 11 + x) = 0, deci 3 Soluie. Din prima ecuaie obinem ca 3 t t t 4 t a cu soluia x1 = 2 log3 2, sau 3x = 11 x, cu soluia x2 = 2, unic deoarece f1 t (0, ), f1 (x) = 3x este strict crescatoare, iar f2 : R R, f2 (x) = 11x est descrescatoare. Din cea de-a doua ecuaie obinem ca (4 3x 1) (6x 2x 3 t t 1 x x x a t deci 3 = , cu soluia x3 = 2 log3 2 sau 6 = 2 + 32, echivalent cu 3x = 1 4 Cum f3 : R (0, ), f3 (x) = 3x este strict cresctoare iar f4 : R R, f4 (x) = a

este strict descresctoare, ecuaia f3 (x) = f4 (x) are o singur soluie, anume x a t a t

X.35. Fie n N, n 3 si z1 , z2 , . . . , zn C de module egale astfe zk 1 Q 2 zk + zj , j = 1, n. A C\ {1, i} 1 k 6= t n. Notam aj = zt zj k6=j ca daca doua dintre numerele aj sunt reale, atunci toate sunt reale. n plus n Q zk = 1 (n legatura cu problema X.86 din R.M.T. nr. 3-4/2 n 6= 4, atunci k=1 Daniel Jinga, P 59

Soluie. Fie P = t

k=1

Fie p, q 1, n astfel nct ap , aq R i notam bp = s

de unde, prin trecere la module, deducem c |bp | = |bq |. a 2 s t a 2 Dac bp 6= 0, atunci bp = bq i conform (1) obinem c zp = zq , d a 4 zp zp = 1. De aici, s {1, i}, contradicie. Atunci bp = 0 i din (1) de t zq zq 4 |zj | 2 2 ca P = r4 , de unde j = 1, n, aj = zj + 2 = zj + zj 2 R. Fie acum zj t a s Deoarece P = r4 , prin trecere la module obinem c rn = r4 , deci r = 1 i P = 1.

P P2 zp 2 , bq = 2 z 2 . q 2 zp zq 2 2 ap = zp + zp 2 + bp i cum zp + zp 2 R, avem ca bp R. Analog obinem ca s t t s Din definiiile lui bp i bq obinem ca t 2 2 bp zp = bq zq = P r4 ,

n Q

2 zk i r = |zk |, k = 1, n. Atunci aj = zj + s

P 2 , j zj

Clasa a XI-a

t XI.31. Fie A, B Mn (Q). Daca det A 6= det B, demonstrai ca det (A + 6= det (B + A). Paul Georgescu i Gabriel Pop s Soluie. Fie P () = det (A + B) det (B + A). Evident, P Q [ t supunem prin reducere la absurd ca det (A + B) = det (B + A). Atunci P ( i cum nu este algebric, P 0. De aici, det (A + B) = det (B + A), s Pentru = 0 obinem det A = det B, contradicie. t t i Pk Pn h 2 p p k t (1) Ck (k p) . Calculai XI.32. Fie sn = k=1 k + k + 1 p=0 p lim n+1 1 + sn n 1 + sn1 .
n k P

Stefan Alexe, P

Soluie. Suma k = t

p=0

p (1) Ck (k p) reprezint numrul funciilor a a t

tive f : A A, unde A are k elemente. Cum orice asemenea funcie este i in t s rezulta ca k = k! i deci s n n n n i X Xh X X (k + 1)2 k k! = k 2 + k + 1 k! = (k + 1) (k + 1)! k sn =
k=1 k=1 k=1 k=1
n+1

exemplu, aplicnd teorema lui Lagrange funciei f : (0, ) (1, ), f (x) = t 60

1 + sn n 1 + sn1 . Atu De aici, sn = (n + 1) (n + 1)! 1. Notam un = p n un = n+1 (n + 1) (n + 1)! n n! = p n n! n n! n+1 n n+1 n+1 n+1 n (n + 1) n+1 (n + 1)! n! + n+1n n = n n n p 1 1 n! n n+1 Este cunoscut ca lim (n + 1)! n! = , lim = i lim n s n n e n n e De asemenea se poate demonstra uor c lim (n + 1) n+1 n + 1 n n n = s a
n

pe intervalul [n, n + 1] i estimnd cantitile obinute). De aici, limita din s at t 1 este egala cu . e XI.33. Fie (xn )n1 un sir de numere reale pozitive cu proprietatea ca p N si numerele a1 , a2 , . . . , ap > 0 cu a1 + a2 + . . . + ap > 1 astfel nc = a1 xn+1 + a2 xn+2 + . . . + ap xn+p , n 1. Sa se arate ca inf {xn | n N } Marian Tetiva, B a Soluie. Deoarece xn 0, n N , exist m = inf {xn | n N } i m t s a ipotez i din faptul c xn m, n N , deducem c xn a1 m + a2 m + a s a +ap m = am, n N , unde am notat a = a1 + a2 + . . . + ap > 1. n cont xn a1 am + a2 am + . . . + ap am = a2 m, n N i prin inducie urmeaza i s t c xn ak m, n, k N . Dac am presupune prin absurd c m > 0, am ob a a a xn lim mak = +, contradicie. Rmne c m = 0, ceea ce trebuia demo t a a k XI.34. Fie a (0, 1) (1, ) si b un numar ntre 2 ln a si 2 ln a. S 1 semnul funciei f : R R, f (x) = (ax + ax ) bx. t 2 Gheorghe Costovic Soluie. Artm mai nti c t aa a
2

x2 (ln a)2 + 1 bx > 0, x 0. 2 ax + ax > bx, x R, deci f este strict pozitiv. a Din (1) i (2) obinem c s t a 2 n n XI.35. Fie f : [0, ) R o funcie continua, iar (ak )k=1 , (bk )k=1 progres t metrice astfel nct ak < bk , k = 1, n. Daca f (a1 a2 an ) < 1 si f (b1 b2 b aratai ca exista (ck )n progresie geometrica, ck (ak , bk ) astfel nct f (c1 c2 t k=1 Doru - Dumitru Buza Solu Notm pa = a1 a2 . . . an , pb = b1 b2 . . . bn . Considerm g : [0, 1 tie. a a g (t) = f pt p1t ; observam ca g (0) = f (pb ) > 1, iar g (1) = f (pa ) < 1. De a b g este continua, t0 (0, 1) astfel ca g (t0 ) = 1. n n Fie (ck )k=1 definit prin ck = at0 b1t0 . Se observ c (ck )k=1 este de ase a a k k progresie geometric i ak < ck < bk , k = 1, n. n plus, deoarece g (t0 ) = 1, as ca f (c1 c2 . . . cn ) = 1, ceea ce trebuia demonstrat. 61

ln a + ax/ 2 ln a 0, x 0, deci g este cresctoare i g (x) g (0) = 0, x 0. a s 2 i b x2 h x2 (ln a)2 2 2 2 (ln a) b + + 1 bx = x 1 , deci De asemenea, 2 4 2 Fie g : R R, g (x) = ax/2 ax/2 x ln a. Atunci g 0 (x) = ax/2

ax + ax x2 (ln a) + 1. 2 2 ax + ax x2 (ln a)2 i f2 : R R, f2 (x) = s + Deoarece f1 : R R, f1 (x) = 2 2 funcii pare, este suficient s demonstrm (1) doar pentru x [0, ). Atunci t a a 2 2 (1) ax/2 ax/2 (x ln a) , x 0 ax/2 ax/2 x ln a, x

XII.32. Fie (G, ) grup, iar a G\{e} fixat. Aratai ca numarul morfi t surjective de la G la (Z3 , +) cu proprietatea ca f (x) = b x = a este 2 numarul subgrupurilor H ale lui G care nu-l conin pe a si care au propriet t x3 H, x G. Dana Stan, elev Soluie. A se vedea Nota de la p. 17 din acest numr al revistei. t a

t XII.31. Fie M = A M2 (C) | A2 = I2 . Determinai toate subgrupu (M2 (C) , ) coninute n M . t a a Angela Tigeru i Ctlin Tigeru, Su s a a Soluie. Prin calcul direct observam ca M = M1 M2 , cu M1 = {I2 , t a b | a2 + bc = 1, a, b, c C . n plus, M1 M2 = . Demo M2 = c a acum urmtorul rezultat auxiliar: a Lem. a) Dac A1 , A2 M astfel nct A1 A2 M , atunci A1 A2 = A2 A1 a a b) Daca A1 , A2 M2 astfel nct A1 A2 M , atunci A1 = A2 sau A1 = Demonstraie. a) Fie A1 , A2 M astfel ca A1 A2 M . Atunci A2 = A2 t 1 2 t t de asemenea A1 A2 A1 A2 = I2 . nmulind ultima relaie la stnga cu A1 i la d s t cu A2 obinem egalitatea cerut. a a1 b1 a2 b2 , A2 = b) Fie A1 = M2 . Deoarece A1 A2 M c1 a1 c2 a2 t A1 A2 = A2 A1 , obinem ca b1 c2 = b2 c1 , a1 b2 = a2 b1 , a1 c2 = a2 c1 , de unde A = (a1 a2 + b1 c2 ) I2 . Cum A1 A2 M i (a1 a2 + b1 c2 ) I2 M2 , obinem ca A1 A2 / t s deci A1 A2 = I2 , de unde b). Singurele subgrupuri H coninute n M1 sunt H1 = I2 , H2 = {I2 , I2 }. Fi t H un subgrup al lui M , H M2 6= . Exista atunci A1 H M2 ; pentru matrice A2 H M2 obinem ca A1 A2 H M deci, conform b), A1 = t A1 = A2 . Obinem de aici c, alturi de H1 i H2 , singurele subgrupuri t a a s t s M2 (C) coninute n M sunt cele de tipurile H3 = {I2 , A} i H4 = {I2 , I2 , A cu A M2 .

Clasa a XII-a

XII.33. Fie f : R R o funcie monotona al carei grafic are un cen t simetrie ce nu aparine graficului. Aratai ca f nu admite primitive. t t Oana Marangoci, elev, P a Soluie. Deoarece A (x0 , y0 ) este centrul de simetrie al graficului lui f , t egalitatea f (2x0 x) = 2y0 f (x), x R. Presupunem prin reducere la t ca f este continua n x0 . Trecnd la limita n egalitatea de mai sus obin lim f (2x0 x) = lim (2y0 f (x)), deci f (x0 ) = y0 , contradicie. De aic t
xx0 xx0

este continua n x0 i, deoarece f este monotona, x0 este punct de discontinui s spea nti. De aici rezulta ca f nu admite primitive. t

XII.34. Fie f : [0, 1] (0, ) o funcie care admite primitive si fie F o pr t c 2 a sa cu F (1) = 0. Aratai ca exista c (0, 1) astfel nct F (c) > ec f (c) t 2 Rodica Luca Tudorach 2 Soluie. Fie G : [0, 1] R, G (x) = 1 ex F (x). Se observa ca t continua pe [0, 1], derivabila pe (0, 1), iar G (0) = G (1) = 0. Conform teore 62

continu, Im f = R, deci h este surjectiv i n final bijectiv. a as a Atunci g = h f , deci g este injectiv. Cum o funcie injectiv admite pr a t a daca i numai daca este continua, cu aceasta observaie cerina problemei se t t s sub forma "f este continu g este continu". Dar acest lucru este imediat, a a seama c h = g f , cu h continu i bijectiv. a as a

Rolle, exist c (0, 1) astfel nct G0 (c) = 0, ceea ce nseamn c 2cec a a a 2 2 c x2 + e t 1 f (c) = 0. Deoarece e 1 > x , x (0, 1), obinem i inegalitatea din enun. t Pn XII.35. Fie f, g : R R, unde f este o funcie injectiva si g (x) = k=0 a t 2k+1 + sin f (x), cu a1 [1, ) si a2k+1 (0, ), k = 1, n. Arata [f (x)] admite primitive daca si numai daca g admite primitive. Lucian Georges Ldunc a Soluie. Fie h : R R, h (x) = a1 x + a3 x3 + . . . + a2n+1 x2n+1 + sin x. t h0 (x) = a1 + 3a2 x2 + . . . + (2n + 1) a2n+1 x2n + cos x > 0, x R, prin urmare strict cresctoare i deci injectiv. Cum lim h (x) = , lim = + i a s a s
x x

IMPORTANT

n scopul unei legaturi rapide cu redacia revistei, pot fi utilizate urm t toarele adrese e-mail: tbi@math.tuiasi.ro, popagabriel@go.com Pe aceast cale colaboratorii pot purta cu redacia un dialog privitor a t materialele trimise acesteia, procurarea numerelor revistei etc.

La problemele de tip L se primesc soluii de la orice iubitor de matemat t elementare (indiferent de preocupare profesionala sau vrsta ). Fieca dintre soluiile acestor probleme - ce sunt publicate n revist dup u t a a an - va fi urmata de numele tuturor celor care au rezolvat-o.

Adresm cu insistena rugmintea ca materialele trimise r a t a vistei s nu fie (s nu fi fost) trimise i altor publicaii. a a s t

63

Soluiile problemelor pentru pregtirea concursu t a din nr. 2 / 2002


A. Nivel gimnazial
n

G22. Sa se arate ca ntre n si n! exista cel puin un numar prim, oricar t n N\ {0, 1, 2}.

G21. Sa se afle restul mparirii prin 43 a numarului 62002 , n N. t Ctlin - Cristian Budean a a n Soluie. Avem c 2002n = (2001 + 1) = 3k + 1, cu k N . Atunci 62 t a n k = 6 63k = 6 (5 43 + 1) , de unde restul mparirii lui 62002 la 43 este 6. t

Soluie. Fie p un divizor prim al numrului n! 1 (care evident exist, ev t a a fiind n! 1). Daca p n, atunci p | n!, i cum p | n! 1 rezulta ca p | 1, absu s concluzie p > n. G23. Sa se rezolve n Z2 ecuaia x2 + 10x + y! = 2002. t

Soluia a III-a. Deoarece 7! = 5040 > 2002, avem c y < 7; problema se t a la rezolvarea a apte ecuaii de grad II. s t

Soluia a II-a (dat de t a Alexandru Bejinariu, elev, Iai). Pentru s u. c. (y!) = 0. n plus, u. c. x2 + 10x = u. c. x2 {0, 1, 4, 5, 6, 9}, dec 2 s x + 10x + y! nu ar fi 2. Ramne ca y < 5 i considernd pe rnd cele cinci obinem soluiile de mai sus. t t

Adrian Zanosch Soluia I (a autorului). Evident, trebuie impusa condiia y 0. De t t x (x + 1) nu poate fi dect de forma 3k sau 3k + 2, k Z, de asemenea x2 + = x (x + 1)+9x este de una din aceste forme. Dac y 3, atunci 3 | y! i x2 +1 a s este tot de forma 3k sau 3k + 2, dar 2002 = 3 667 + 1, absurd. n con y {0, 1, 2}. Daca y = 0 sau y = 1, obinem ca x2 + 10x = 2001, ecuaie care t t soluii n Z. Daca y = 2, atunci x2 + 10x = 2000, cu soluiile x1 = 40 i x2 = t t s n concluzie, (x, y) {(40, 2) , (50, 2)}.

G24. Aflai cte numere de 4 cifre au proprietatea ca cifrele citite de la t spre dreapta sunt invers proporionale cu cifrele citite de la dreapta spre t Aceeai problema pentru numerele de 3, respectiv 5 cifre. s Gabriel Pop Soluie. Pentru a putea vorbi despre inversa proporionalitate, este nec t t fiecare caz ca toate cifrele sa fie nenule. a (i) Dac abcd este un numr cu proprietatea din enun, atunci a a t = 1/d c d = = , deci ad = bc. Dac {a, d} = {b, c} iar a = d, obinem 9 n a t 1/b 1/a Daca {a, d} = {b, c} iar a 6= d, obinem 4 36 = 144 numere (36 este n t perechilor de cifre nenule (a, d) cu a < d, iar corespunztor fiecrei perechi a a a a obine 4 numere abcd distincte prin diverse permutri). Dac {a, d} 6= {b, c}, t {{a, d} , {b, c}} {{1, 4} , {2, 2}}, {{1, 6} , {2, 3}}, {{1, 8} , {4, 2}}, {{2, 6} , { t {{2, 8} , {4, 4}}, {{2, 9} , {3, 6}}, {{3, 8} , {4, 6}}, {{4, 9} , {6, 6}}. Obinem +5 8 = 52 numere (pentru fiecare element subliniat obinem 4 numere abcd t 64

celelalte cazuri cte 8). n total obinem 9 + 144 + 52 = 205 numere. t (ii) Daca abc este un numar cu proprietatea din enun, deducem ca ac t Daca a = c, obinem 9 numere. Daca a 6= c, atunci b = 2, {a, c} = {1, 4}; t {a, c} = {1, 9}; b = 4, {a, c} = {2, 8}; b = 6, {a, c} = {4, 9}. Obinem 4 t numere. n total obinem 9 + 8 = 17 numere. t t (iii) Daca abcde este un numar cu proprietatea din enun, deducem ca ae = Daca a = b = c = d = e, obinem 9 numere. n caz contrar, t c = 2, c = 3, c = 4, c = 6, {{a, e} , {b, d}} {{{1, 4} , {2, 2}} , {{1, 4} , {1, 4}}} ; {{a, e} , {b, d}} {{{1, 9} , {3, 3}} , {{1, 9} , {1, 9}}} ; {{a, e} , {b, d}} {{{2, 8} , {4, 4}} , {{2, 8} , {2, 8}}} ; {{a, e} , {b, d}} {{{4, 9} , {6, 6}} , {{4, 9} , {4, 9}}} .

Obinem 4 (4 + 4) = 32 numere. n total, obinem 9 + 32 = 41 numere. t t G25. Sa se arate ca pentru orice x, y, x R, exista a, b, c Z astfe 3 2 2 2 (x a) + (y b) + (z c) . Generalizare. 4 Vladimir Martinu s 1 Soluie. Daca : R Z, (x) = x + , este funcia de rotunjire, t t 2 1 t | (x) x| , x R. Pentru a = (x), b = (y), c = (z), obinem co 2 s problemei. Cu acelai raionament obinem ca daca n N i x1 , x2 , . . . , x s t t atunci exist a1 , a2 , . . . , an Z astfel ca (x1 a1 )2 +(x2 a2 )2 + +(xn an ) a

G26. Daca suma, produsul si ctul a doua numere iraionale sunt numere t nale, calculai suma cuburilor celor doua numere. t Claudiu - Stefan Pop a a Soluie. Daca + 1 6= 0, atunci a + b = b t + 1 R \ Q, deoarece b b b a s Rmne c + 1 = 0, deci a = b i a3 + b3 = 0. a a b Observaie. Ipoteza "ab Q" nu este necesar. t a G27. Determinai n N minim pentru care 2n 1 este divizibil cu 125. t Gheorghe Iure Soluie. Fie n = 4q + r, q, r N, 0 r < 4. Atunci 2n 1 = 2r 24q t = 2r 16q 1. Cum ultima cifr a lui 16q este 6 (q = 0 nu convine), ca 2n a divida cu 5 e necesar r = 0, n caz contrar ultima cifra a lui 2n 1 fiind 1, 3 Atunci 2n 1 = 24q 1 = 15 16q1 + 16q2 + + 1 i deci ca 2n s se divid cu 125 este necesar i suficient ca 25 | 16q1 + 16q2 + + 1. a s p 16p = (15 + 1) = M5 + 1 pentru p 0, ca suma s se divid cu 5 este nec a a suficient ca numarul termenilor sa se divida cu 5, deci q = 5k, k N . Atunc 2n 1 = 220k 1 = 10242k 1 = 1048576k 1 = 1048575 1048576k1 +

Cum 1048575 este divizibil cu 25, dar nu i cu 125, este necesar i suficient s s divida al doilea factor. Deoarece 1048576p = M5 + 1 pentru p 0, este nec suficient ca numarul termenilor sa se divida cu 5, deci k = 5m, m N i n = s a m N . Numrul n minim este 100. G28. Sa se arate ca ecuaia t 65

are cel puin doua soluii reale pentru orice a, b R. t t Marian Teler, Costeti (A s Soluie. Ecuaia data se descompune n x2 ax + b 1 x2 bx + a t t Se obin ecuaiile x2 bx + a 1 = 0 i x2 ax + b 1 = 0. Atunci t t s 1 + 2 = b2 4a + 4 + a2 4b + 4 = (a 2)2 + (b 2)2 0,

x4 (a + b) x3 + (a + b + ab 2) x2 a2 + b2 a b x + (a 1) (b 1)

s t a t t deci 1 0 sau 2 0 i cel puin una dintre cele dou ecuaii are soluii re b b G29. Fie triunghiul ABC cu m(A) < m(C). Pe bisectoarea interioara a u b luam un punct E astfel nct EAB ACB. Se prelungete latura [B \ \ lui B s segmentul [BD] [AB], B ntre C si D. Sa se arate ca mijlocul M al segm [AC] se afla pe dreapta DE. Constantin Chiril Soluia I (a autorului). Construim prin E o dreapt paralel cu A t a a \ s \ \ intersecteaz AD i CD n P i Q. n 4EBA i EQB, EBA EBQ i E a s s s \ \ EQB ( ACB). Cum [EB] [EB], 4AEB 4BEQ (ULU), deci [EA] \ \ \ \ t Deoarece 4ABD este isoscel, BAD BDA. Cum EAB ACB, obin [ \ s \ [ EAP CAD, i deoarece CAD EP A, 4EAP este isoscel, deci [EA] Urmeaz c [EP ] [EQ], adic [DE] este median n 4DP Q i atunci dreap a a a a s njumtete segmentul [AC] paralel cu "baza" P Q. a at s

Soluia a II-a. Fie {A0 } = BC AE t A i {F } = AB DE. Deoarece 4ABC s c a b 4A0 BA, putem scrie: 0 = = 0 , M AB c AA F bc c2 0 0 i A B = . Conform s de unde A A = a a DB EA0 = teoremei lui Menelaus, avem: C B D DA0 EA c DB EA0 BA0 c FB i = s (4ABA0 i DE). Cum s = = (t = FA DA0 c + c2 /a EA BA a c FB bisectoarei), rezult c a a = . Pentru a dovedi c M DE, art a a FA a+c punctele D, M , E sunt coliniare utiliznd reciproca teoremei lui Menelaus re c a+c DB M C F A = 1 = 1. 4ABC; ntr-adevr, avem a DC M A F B c+a c G30. Fie triunghiul AB0 B1 dreptunghic n B0 si triunghiurile ABi B \ Bi Bi+1 ABi , i N , iar m(Bi ABi+1 ) = 30 , i N . a) Demonstrai ca punctele A, Bq si Br sunt coliniare, unde q = 2 n3 t r = 32002 32000 + 31998 31996 + . . . + 32 30 . b) Aflai aria triunghiului AB2001 B2002 funcie de a = B0 B1 . t t Romana Ghia i Ioan Ghia t t s t Soluie. a) Observam mai nti ca daca i N, i = 12q1 + r1 , q1 , r1 t 0 r1 < 12, atunci Br1 [ABi ]. Deoarece q = 2n (n + 1) (n 1) + 2, q = M1 B2 [ABq ]. n plus, r = 8 32000 + 31996 + + 30 = M12 + 8 i B8 [ABr s B2 , A, B8 sunt coliniare, Br , A i Bq sunt de asemenea coliniare. s 66

a a raportul de asemnar a b) Observm c 4ABi Bi+1 4ABi+1 Bi+2 , i N, 4001 22001 2 3 2 3 2 . Atunci SAB2001 B2002 = SAB0 B1 = 2001 a2 3. 3 3 3 G31. Se considera un triunghi isoscel ABC cu baza BC = 2 2 cm. Fie p variabile M (AB) si N (AC) astfel nct [AM ] [CN ]. Fie O m segmentului [M N ] si P intersecia dreptelor AO si BC. Aflai perimetrul 4 t t stiind ca aria minima a reuniunii suprafeelor triunghiulare [M BP ] si [N C t 3 cm2 . Adriana Maxiniuc, Bo \ \ \ i cum ABC Soluie. Fie N Q k AB, Q (BC). Atunci N QC ABC s t 4N QC este isoscel, deci [N C] [N Q]. Deoarece [N C] [AM ], rezult c [N Q a a i cum N Q k AM , AN QM este paralelogram. Notam AQ M N = {O0 }. Atu s njumtete [M N ], deci O0 O i atunci P Q, deci AM P N este paralelo a at s s t Deoarece SMBP +SN CP +SAMN P = SABC = constant, aria din enun este m \ cnd SAMNP este maxima. Cum SAMN P = AM AN sin BAC = AM (AC A AC \ sin BAC, iar AM (AC AM ) este maxim cnd AM = , rezulta ca S 2 este maxima atunci cnd M i N sunt mijloacele segmentelor [AB], respectiv s SABC SABC cu valoarea . Atunci minimul ariei din enun este de asemenea t 2 2 lungimea nalimii din A este 6 cm. Rezulta AB = 2 2 cm, iar PABC = 6 t b b G32. Fie patrulaterul convex ABCD cu m(A) = m(D) = 90 , M un pu dreapta AD, iar N BC astfel nct M N BC. Aratai ca SCMB SAND t Neculai Roman, Mirceti s Soluie (Constantin Tonu, elev, Iai). Patrulaterul ABCD este drep t s 1 sau trapez dreptunghic. Fie N 0 proiecia lui N pe AD; atunci SCMB = M N t 2 1 SAN D = N N 0 AD. ns M N N N 0 , iar BC AD, de unde concluzia. a 2 b G33. n triunghiul ABC cu m(A) = 2, fie D (BC) piciorul bise b iar M , N puncte pe (AB) respectiv (AC). Daca {P } = AD unghiului A, 1 2 cos 1 demonstrai ca t + = (n legatura cu C:2402 din G.M. 5-6/2 AM AN AP Mihaela Buctar a A 2 AM AN cos , egalitate adevrat, de a a Soluie. Relaia se scrie AP = t t AM + AN 2 [AP ] este bisectoare n triunghiul AM N .

\ G34. n triunghiul ABC avnd m(BAC) = 135 se nscrie patratul M N M, N (BC), P (CA) si Q (AB). Aratai ca: t b+c 2 c AM AM BM ; 2 1 = = . AN CN b AN c+b 2 Temistocle Brsa \ = 135 , A este pe cercul circumscris patr Soluie. Deoarece m(BAC) t \ \ \ Observm de aici c m(BAM ) = m(M AN ) = m(N AC) = 45 . Notm l = M a a a 1 Deoarece AP M Q este inscriptibil, AM P Q = AP M Q + AQ M P 67

AP b AQ AP AQ i . Deoarece 4AQP 4AP C, + 2 = s PQ PQ PQ a PQ l l b + 2 c . Analog obinem ca AN = t 2 b + c , de unde Atunci AM = a a concluzia. BM AM i s = s 2 Utiliznd teorema sinusurilor n 4ABM i 4ACN , sin 45 sin B si AN BM AM sin C c AM = . Atunci = = . sin C CN AN sin B b AN G35. Fie [ABCD] un tetraedru. n planele (ABC), (ADC), (ADB) cons tangentele n A la cercurile circumscrise triunghiurilor ABC, ADC respectiv AM = l

care intersecteaza dreptele BC, CD, DB n M , N respectiv P . Notam x = NC PD 1 1 1 y= ,z= . Sa se arate ca + + 1. ND PA 1+x 1+y 1+z Marian Ionescu, P M B AB = Soluie. n planul (ABC), observam ca 4AM B 4CM A, deci t M A AC 2 MB AB 2 MB MA AB i analog deducem ca y = Atunci , deci x = s = = MC MA MC AC AC 2 2 AD 1 1 z= , prin urmare xyz = 1. Observam ca + = 1, deci inega 2 AB 1 + x 1 + yz 1 1 1 de demonstrat se reduce la + , care se obine prin calcul t 1+y 1+z 1 + yz

B. Nivel liceal

t L21. Rezolvai n N2 ecuaia a2 + 3b2 = 2n , unde n N este fixat. t Gheorghe Iure Soluie (dat de Andrei Nedelcu, Iai). Dac a i b au pariti d t a s a s at atunci n = 0 i avem soluia a = 1, b = 0. Dac n > 0, atunci a, b au s t a paritate i atunci fie a + b = 2x, x N i a b = 2y, y Z. nlocuind, o s s 4 x2 xy + y 2 = 2n , de unde n 2. Fie d = (x, y); avem ca d = 2s , 2s n s s 2 x = 2 x1 , y = 2 y1 , cu (x1 , y1 ) = 1. nlocuind din nou, gsim x2 x1 y1 +y1 = 2 a 1 i cum membrul stng este impar, n = 2 (s + 1) este numar par, iar x2 x1 y1 + s 1 Fiindca x1 , y1 N, cu x1 6= 0, obinem ca x1 = y1 = 1 sau x1 = 1, y1 = 0, cu s t n2 corespunzatoare a = 2n/2 , b = 0, respectiv a = b = 2 2 . Cu aceast tehnic putem aborda rezolvarea ecuaiei n Q2 . S observ a a t a a nti c este suficient s gsim soluiile (a, b) Q2 , celelalte soluii fiind de a a a t t + x y (a, b), (a, b), (a, b). Fie a = , b = , cu x, y, z, w N, (x, z) = (y, w z w Se observ uor c z = w i ecuaia devine x2 + 3y 2 = 2n z 2 (). Pentru a s a s t putem scrie x2 y 2 = z 2 4y 2 , i. e. (x y) (x + y) = (z 2y) (z + 2y). Daca 1 xy z + 2y m atunci z = 2y, deci a = b = . Dac x 6= y, atunci a = = ,d 2 x + y n z 2y 2 2 2 2 2 2 x = k m 4mn + n , y = k m n , z = 2k m mn + n , cu k, m, astfel nct x, y, z 0. Fie n 6= 0; atunci x, y au aceeai paritate i fie x + y = 2, x y = 2 s s nlocuire n (), obinem 4 2 + 2 = 2n z 2 . Daca n impar, aceasta t 68

nu are soluii, deoarece n membrul stng i n z 2 factorul 2 apare numai t s teripare. Daca n este par, fie (, ) = 2s , z = 2r z1 cu z1 impar i atu s 2 22s 2 1 1 + 2 = 2n 22r z1 deci 2s + 2 = n +2r, iar 2 1 1 + 2 , 1 1 1 1 cu soluia z1 = k m2 mn + n2 , iar (1 , 1 ) = k m2 n2 , k 2mn n2 t invers etc. Not. Soluie corect s-a primit de la Alin Iacob, elev, Iai. a t a s L22. Fie n N un numar scris n baza 10. Acestui numar i adaugam la 147, numarului obinut i adaugam din nou la sfrit 147 si aa mai departe. A t s s ca printre numerele astfel obinute exista numere compuse. t Adrian Zanosch Soluie. Avem n1 = 1000n + 147 = 37 (27n + 4) + n 1, de unde o t ca n1 n 1 (mod 37). Apoi n2 = 1000n1 + 147 = 37 (27n1 + 4) + n deci n2 n1 1 (mod 37) n 2 (mod 37) i aa mai departe. Astfel obin s s t numerele din irul construit sunt congruente, modulo 37, cu n 1, n 2, . . . , s . . . , deci exista printre ele numere divizibile cu 37, adica numere compuse. L23. Fie f : R R o funcie periodica de perioada principala T astfel t [0, T ] f se anuleaza de un numar finit de ori si fie (xn )n1 un sir de numer nenule. Aratai ca exista R astfel nct f (xn ) f ( + xn ) 6= 0 n N t Paul Georgescu i Iuliana Georgesc s Soluie. Fie {a1 , a2 , . . . , ak } mulimea pe care f se anuleaz n [0, T ] i fi t t a s ! ! [ [ k [ [ ai T + (ai xn + T Z) . A= Z xn xn n=1 i=1 n=1 i=1 ai T ns a + t a Z i ai xn + T Z sunt mulimi numrabile, i 1, k, n N , s xn xn A este numrabil. Cum R este nenumrabil, R \ A 6= . a a a a ai T + / Z, i 1, k, n N , deci xn ai Fie R \ A. Atunci / xn xn a a i 1, k, n N , ceea ce nseamn c f (xn ) 6= 0, n N . Analog dedu f ( + xn ) 6= 0, n N , de unde rezult concluzia problemei. a L24. Fie A, B Mn (R) matrice nesingulare cu det A + det B = 0. Exista astfel nct A2 B B 2 A = A? Ctlin Calistr a a Soluie. Presupunem prin reducere la absurd ca exista > 0 astfel ca t B 2 A = A. nmulind la dreapta cu A1 , obinem ca A2 BA1 = B 2 + In t t 2 det B + In = det B + i In det B i In = det B + i In 2 t iar det A2 BA1 = det A det B = (det A) < 0, contradicie. L25. Fie A Mn (R) o matrice cu proprietatea ca exista m N, m si R, || 1, astfel nct Am+1 Am A + In = On . Sa se a |det A| = 1. Lucian Georges Ldunc a Soluie. Fie PA (X) = det (XI A) = (X x1 )k1 (X x2 )k2 . . . (X t polinomul caracteristic al matricei A. Cum x1 , x2 , . . . , xl sunt rdcini i a a s mA , polinomul minimal al matricei A peste C, iar mA divide X m+1 X m rezulta ca x1 , x2 , . . . , xl au modulele egale cu 1, deoarce ultimul polinom ar 69

rdcinile de modul 1. Dar PA (0) = det (A) = (1) a a


n Pl k

det (A) = (1) det A = (1) i=1 i xk1 xk2 . . . xkl , de unde |det A| = 1. 1 2 l L26. Fie f : Sn Sn endomorsm astfel nct exista Sn pentr (f f ) () = 1 , Sn . Aratai ca f are un punct fix (i.e. t f () = ). Ovidiu Munteanu, B Soluie. Fie Sn cu proprietatea din enun. Este uor de observat ca t t s : Sn Sn , () = 1 este bijectiv. Relaia din enun se scrie sub a t t f f = , de unde rezult c f este bijectiv. a a a Pentru Sn , (f f f ) () = f ( ()) = f 1 = f ( ) f () f i, de asemenea, (f f f ) () = (f ()) = f () 1 . Rezulta de s f ( ) f () f 1 = f () 1 , Sn i, deoarece f este surj s f ( ) f 1 = 1 , Sn . Deci 1 f ( ) = 1 f ( ) , Sn 1 f ( ) comut cu toate elementele lui Sn , 1 f ( ) = e. Obinem c f ( ) a t a L27. Fie (A, +, ) un inel cu unitate si n, k N , k impar, astfel nct xn+k x A. Sa se arate ca xk+1 = x, x A (n legatura cu C: 1896 din G.M. nr.1/ Drago Deliu i Marian Tetiva, B s s Soluie. Daca n = 1, nu avem nimic de demonstrat. Fie acum n 2. De t (1)n+k = (1)n , iar n i n + k au paritai diferite, avem ca 1 = 1, deci 1 + s t i n concluzie 2a = a + a = 0, a A. s Demonstrm acum c n inelul A este valabil implicaia x2 = 0 x = 0 a a a t adevar, fie x A astfel ca x2 = 0. Atunci xs = 0, s N, s 2 i aplicnd f s binomiala rezulta ca (1 + x)p = 1 + px, p N. Cum (1 + x)n+k = (1 + x ipotez, obinem c 1 + (n k) x = 1 + nx, de unde kx = 0. Deoarece k este a t a i 2x = 0, rezult imediat c x = 0, deci implicaia este dovedit. s a a t a Din ipoteza, xn+k = xn , x A, deci i xm+k = xm , x A, pentru m s Vom avea atunci 2 n+k1 x xn1 = x2n+k2+k x2n+k2 x2n2+k x2n2 = 0 ceea ce implic xn+k1 xn1 = 0. Am obinut c xn+k1 = xn1 ; re a t a raionamentul obinem ca xn+k2 = xn2 , . . . , xk+1 = x, q.e.d. t t L28. Fie ABC si A0 B 0 C 0 doua triunghiuri ascuitunghice. Daca t a 2 b 2 c 2 S 0 0, a0 b0 c S aratai ca triunghiurile sunt asemenea. t Ioan Scleanu, H a a sin A sin B sin C Soluie. Notam cu x = t , y = , z = . Folosind te sin A0 sin B 0 sin C 0 2 2 sinusurilor i formula S = 2R sin A sin B sin C, obinem c x y 2 z 2 x s t a aici, x y z i xy z. Atunci i xy y, de unde x 1 i deci z y s s s t t Deoarece 4ABC i 4A0 B 0 C 0 sunt ascuitunghice, iar funcia sinus este cresc s h i b c b t pe 0, , din x 1, y 1, z 1 obinem ca m(A) m(A0 ), m(B) 2 b c b b b c c c m(C) m(C 0 ). Cum m(A) + m(B) + m(C) = m(A0 ) + m(B 0 ) + m(C 0 ) = b c b c b c rezult c m(A) = m(A0 ), m(B) = m(B 0 ), m(C) = m(C 0 ), deci cele dou triu a a a sunt asemenea. 70

Pl

i=1

ki

xk1 xk2 . . . xkl . 1 2 l

Not. Soluie corect s-a primit de la Alin Iacob, elev, Iai. a t a s L29. Fie ABC un triunghi, iar C un cerc tangent laturilor [AB] si [AC respectiv E si care intersecteaza latura [BC] n M si N . Fie X un punct i triunghiului astfel nct exista un cerc C1 tangent laturilor [XB] si [XC] n spectiv Y si care taie [BC] tot n M si N . Demonstrai ca patrulaterul EF Z t inscriptibil. Neculai Roman, Mirceti, s Soluie. Dac [AB] [AC], punctul X va fi n mod necesar pe axa triun t a ABC (altfel C1 n-ar exista). Ca urmare, patrulaterul EF ZY este trapez isosc este inscriptibil. Fie acum [AB] 6 [AC] i fie {K} = F E BC, {K 0 } = ZY s a (K 0 exist deoarce triunghiul XBC nu va fi nici el isoscel). Aplicnd teore KB EC F A Menelaus pentru 4ABC cu transversala KF E obinem t = KC EA F B FB KB = . Aplicnd teorema lui Menelaus pentru 4XBC cu transversala KC EC0 K 0B ZB K B Y C ZX = 1, deci 0 = . Dar F B = ZB i Y C s obinem 0 t K C Y X ZB KC YC KB s (deoarece B i C aparin axei radicale a cercurilor C1 i C), prin urmare s t = KC 0 a a de unde K = K . Deoarece K se afl pe axa radical a cercurilor C1 i C s KE KF = KY KZ, deci punctele E, F , Y , Z sunt conciclice. Not. Soluie corecta s-a primit de la Alin Iacob, elev, Iai. a t s L30. Fie ABC un triunghi echilateral, iar P un punct n planul triunghiulu tam cu A1 , B1 , C1 simetricele lui P faa de BC, CA si respectiv AB. Sa s t ca se poate forma un triunghi avnd lungimile laturilor egale cu AA1 , BB1 , Constantin Coce Soluie. Fie A (a), B (b), C (c), P (p) i fie O (0) centrul cercului circu t s 4ABC. Putem presupune, fr a restrnge generalitatea, c a = 1, b = , c aa a bc bc Avem BC = = = bc, de unde P A1 = bc, deoarece P A1 1/b 1/c bc 0 t Fie {A } = P A1 BC. Ecuaia dreptei BC este zb = bc z b i poate fi r s t sub forma z + bcz = b + c. Ecuaia dreptei P A1 este z p = bc (z p). D 1 obinem c afixul a0 al punctului A0 este dat de egalitatea a0 = (b + c + p t a 2 p + a1 Cum a0 = a a , unde a1 este afixul punctului A1 , urmeaz c a1 = b + c 2 n mod similar, b1 = c + a cap, c1 = a + b abp. De aici, (a a1 )+(b b1 )+(c c1 ) = 0 i se obine ca |a a1 |+|b b1 | | s t t s de unde AA1 +BB1 CC1 . Analog obinem ca AA1 +CC1 BB1 i BB1 +CC1 ceea ce trebuia demonstrat. L31. Fie (an )n1 un sir de numere reale astfel nct a1 = a > 1, a p p p1 = an + an 1, n N , unde p N, p 2. Sa se calculeze lim
n

legatura cu C:1463 din G.M. nr.11/1993 ). Viorel Cornea i Dan Stefan Marinescu, Hune s Soluie. Deoarece a > 1, putem demonstra prin inducie c an > 1, t t a p p de unde, innd seama c an+1 an = ap1 1, obinem c irul (an )n t a t a s n strict crescator. Daca (an )n1 ar fi marginit superior, atunci ar avea o limita 71

p l. Trecnd la limit n relaia de recurena s-ar obine c l = l + lp1 a t t t a l = 1, contradicie. n concluzie, lim an = +. t r n an+1 1 t a = 1+ p mprind acum n relaia de recurena prin a1 obinem c at t t an an p a an+1 n cu ajutorul lem de unde lim = 1. Calculm acum limita lim a n an n n Cesaro - Stolz. Observam ca p a p a a an n+1 n q n+1 lim = = lim q p p n n p p1 p2 n+1n p a + a a + . . . + ap1
n+1

1 1 p p1 1 an = lim s = . p1 s p2 n p an+1 an+1 p + p + ... + 1 an an p a 1 1 an n = , deci lim p = p . De aici, lim n n n n p p xn+1 L32. Fie (xn )n1 un sir de numere naturale care satisface condiia t xn xn n 1, iar x1 = 2. Sa se arate ca exista > 1 pentru care lim 2n = 1. n Cristinel Mortici, Trg xn+1 < xn + 1, deci x2 xn+1 < x2 + xn ; n par Soluie. Avem c xn t a n n xn ln xn 2n1 putem deduce prin inducie ca xn 2 t , n 1. Notam yn = . 2n 2 ln xn+1 /xn ln (1 + 1/xn ) 1 , deci 0 yn+1 yn < . < yn+1 yn = n+1 n+1 2 2 xn 2n+1 a s Se obine c (yn )n1 , este monoton cresctor i, n plus, t a n n X X 1 (yk+1 yk ) y1 + , yn+1 = y1 + 2k+1
k=1 k=1

n+1 n

1 obinem ca yn+p yn < t . Cum (yn )n1 este monoton crescator, pentru xn 2n ln xn 1 1 obinem ca 0 ln yn t . De aici, 0 ln n xn 2n 2 xn 2n n n 2 2 xn 1 0 ln . Pentru n obinem ca lim ln t = 0, deci lim 2n n n xn xn xn L33. Fie m N, m 2 fixat. Aratai ca soluiile continue ale e t t funcionale t m ! ! m X X X f xi + f (xi xj ) = m f (xi ) , xi R, i = 1, m
i=1 1i<jm i=1

de unde yn+1 < y1 + 1/2, adica (yn )n1 este marginit superior. Deducem ca ( este convergent i fie ln limita sa, > 1. Deoarece s p1 p1 X X 1 (yn+k+1 yn+k ) < , yn+p yn = xn+k 2n+k+1
k=0 k=0

72

Adrian Cordunean 2 Soluie. Pentru x1 = x2 = . . . = xm = 0, obinem ca f (0) + Cm f (0) = m t t deci f (0) = 0. Pentru x1 = x2 = . . . = xm1 = 0 i xm = x, obin s t f (x) + (m 1) f (x) = mf (x), x R, deci f (x) = f (x), x R i s funcie para. Este deci sucient sa determinam f pe (0, ). t Notam f (1) = c. Pentru x1 = x2 = 1 i x3 = . . . = xm = 0, obin s t f (2) + (m 2) f (1) = 2mf (1), deci f (2) = 4c = c 22 . Pentru x1 = n, x2 x3 = . . . = xm = 0, obinem c f (n + 1) + (m 2) f (n) + f (n 1) + (m 2) t a = m (f (n) + f (1)) i deci f (n + 1) = 2f (n) f (n 1) + 2f (1). De aici se d s streaza uor prin inducie ca f (n) = cn2 , n N. t s Pentru x1 = x2 = . . . = xm = x R obinem c f (mx) = m2 f (x). D t a x n x + f (x) f (x) i, prin inducie, f = = = c f , de unde f s t m m2 m m2k mk o k n n ; n, k N . Din cele demonstrate mai sus, f (x) n N. Notm M = a mk x M . Demonstrm c x > 0 i > 0, x M astfel ca |x x | < a a s 1 a k N astfel ca k < min {x, }. Conform axiomei lui Arhimede, exist n m n n 1 n n + 1 , deci 0 x k < k < , de unde x k astfel ca k x < m mk m m m 1 n Rezult c putem alege x = k . Pentru = , l N, obinem c (xl )l a a t a m l 1 astfel ca |x xl | < , l 1, deci xl x pentru l . Cum f (xl ) = cx2 l l este continu, obinem c f (x) = cx2 . n concluzie, f (x) = cx2 , x R. a t a

sunt funciile de forma f (x) = cx2 , cu c R. t

Aratai ca f (x) definete o funcie pe [2, ) si cercetai daca aceasta funcie s t t t t primitive. Stefan Alexe, P Soluie. Fie mai nti x [2, 2]. Se observ c funcia g : [0, ] t a a t g (x) = cos x este bijectiv. n concluzie, exist un unic t = t (x) [0, ], t = ar a a x astfel nct cos t = , deci f0 (x) = 2 cos t. Atunci f1 (x) = 2 + 2 cos t = 2 c 2 t se demonstreaz prin inducie c fn (x) = 2 cos n . a t a 2 Fie acum x (2, +). Se observ c funcia h : (0, ) (1, ), h a a t ex + ex = = ch x este bijectiva. n concluzie, exista un unic t = t (x) ! 2 24 x x x + astfel nct ch t = . Atunci f1 (x) = 2 + 2 t = ln 2 2 2 t = 2 + et + et , deci f1 (x) = et/2 + et/2 = 2 ch i se demonstreaz p s a 2 73

L34. Se considera sirul de funcii (fn )nN , fn : [2, ) R definit prin f t p = x, fn (x) = 2 + fn1 (x), n 1. Notam 4n fn (x) 1 + lim , daca x [2, 2] n 2 f (x) = . n lim [3 fn (x)]4 , dac x (2, ) a
n

2 a c f (x) = 1 + e 2 arccos 2 , dac x [2, 2], respectiv f (x) = e ln a x (2, +). Cum lim f (x) = 2, iar lim f (x) = 1, x = 2 este punct de disco

t ducie ca fn (x) = 2 ch n . t 2 Gsirea expresiei funciei f se reduce la calculul unor limite elementare. Se a t
1 2 x 2 x+

x2 4

x2 x<2

x2 x>2

(se stie ca 0, 45 < e/4 < 0, 46). Gabriel Popa i Paul Georgesc s Z /4 Z /4 ex sin x Soluie. Notam I = I () = t dx, J = J () = ex + sin x ex + si 0 0 a a se deduce imediat c I + J = , > 0. Observm c a 4 2 1 2 I I = I= . IJ = 64 4 64 8 Rmne s demonstrm c exist un unic > 1 astfel ca I () = . Fie 0 < a a a a a 8 < 2 < . Deoarece Z /4 ex sin x I (1 ) I (2 ) = (2 1 ) dx, x + sin x) (ex + sin x) (e 1 2 0 obinem ca I (1 ) > I (2 ) i |I (1 ) I (2 )| (2 1 ), deci I () t s 4 funcie continua strict descrescatoare pe (0, ). t h i x e Fie > 0 i f : 0, s R, f (x) = x . Atunci 4 e + sin x h 2ex sin (x /4) 0 , < 0, x 0, f (x) = 2 4 (ex + sin x) h i deci f este strict descresctoare pe 0, . Aplicnd teorema de medie int a 4 astfel ca I (1), obinem ca exista c 0, t 4 ec e/4 > I (1) = c > . 2 4 e + sin c 4 e/4 + 7 2 h i Fie acum 0, . Atunci 8 Z Z /4 ex ex e I () = dx + dx + x + sin x x + sin x e 4 e + si 0 e Pentru obinem ca lim I () < . Tinnd seama de (1), de co t 8 tatea lui I () i de stricta sa monotonie, obinem c exist un unic > 1 a s t a a I () = , de unde obinem concluzia problemei. t 8 74

tate de spea nti pentru f , deci f nu admite primitive pe [2, +). t L35. Sa se arate ca exista si este unic > 1 astfel nct Z /4 Z /4 x 2 e sin x dx dx = x + sin x x + sin x e e 64 0 0

Probleme propuse
Clasele primare

P.54. Calculai a i b daca 46 a = 36 + a i b 3 = 17 b. t s s ( Clasa I ) nv. Doinia Spn t P.55. n cte moduri pot fi aranjate n linie dreapta 9 mingi roii i una ga s s ( Clasa I ) Georgiana Ciobanu, elev

P56. Cu cinci ani n urma, suma vrstelor a trei copii era de 11 ani. Ca suma vrstelor acelorai copii peste 6 ani? s ( Clasa a II-a) nv. Rodica Rotaru, B

P.57. n cte moduri pot fi mprii 8 biei n dou echipe de cte 4 ju a tt a t a dac Petru vrea s fie n echip cu Mihai i Dan, dar nu vrea s fie cu Avram a a a s a ( Clasa a II-a) Adina Dohotaru, elev

P. 58. S se arate c suma 1 + 4 + 7 + . . . + 100 mprit la 3 d restul a a at a a ( Clasa a III-a) Alexandru - Gabriel Tudorache, ele P.59. Fie a i b doua numere consecutive. Suma acestor numere mpre s numerele obinute marind cu 12 fiecare dintre vecinii lor este 939. Care su t dou numere? a ( Clasa a III-a) nv. Maria Rac

P.60. Din 16 bile, una este mai grea dect celelalte 15, care au mase egale este cel mai mic numr de cntriri prin care se poate stabili bila mai grea? a a ( Clasa a III-a) Carmen Ciolacu, elev

P.61. Suma a doua numere este un numar de doua cifre al caror produs Diferena dintre cele doua numere este 7. Care sunt cele doua numere? t ( Clasa a IV-a) nv. Maria Rac

P.62. Doua ceasuri au nceput sa funcioneze la aceeai ora. Se constat t s fiecare 30 minute (faa de ora exact) unul rmne n urm cu un minut iar t a a a avanseaz cu un minut. La un moment dat orele indicate de aceste ceasuri su a h 36 min i 19 h 24 min. La ce ora au nceput sa funcioneze? s t ( Clasa a IV-a) Felicia Amihiesei, elev a

P.63. Alege un numar format din trei cifre. Scrie la dreapta lui un numar din dou cifre. Scoate din numrul format de 99 ori numrul format din tre a a a Din rezultat scoate diferena dintre numrul de trei cifre i numrul de dou t a s a a scrie rezultatul. Eu i ghicesc numarul format din doua cifre. Cum se explic t lucru? ( Clasa a IV-a) Prof. Petru Asafte

Clasa a V-a

V.41. Fie a numar natural compus astfel nct daca p | a, cu p prim, p + 1 | a. S se arate c 12 | a i s se afle cel mai mare numr a de trei cifre a a s a a Ciprian Baghi

V.42. Se dau numerele xy, ab scrise n baza 10 astfel nct xy divide ab arate c x = y dac i numai dac a = b. a as a Ioan Scleanu, H a a 75

s V.43. S se afle cifrele a i b tiind c a b = cd i ab = dc. a s s a Romana Ghia i Ioan Ghia t t s t n n n V.44. S se afle x, y, z Q+ pentru care x = yz, y = xz, z = xy, cu n a N. N. Hra t V.45. Se dau ase urne, unele coninnd bile. Fie operaia: se aleg trei s t t se pune cte o bil n fiecare dintre ele. a a) Compoziia urnelor fiind 0, 0, 4, 6, 6, 8, s se indice o succesiune de ope t a urma carora toate urnele sa conina acelai numar de bile. t s b) Compoziia urnelor fiind 0, 1, 2, 3, 4, 4, sa se arate ca nu exista o succesi t operaii n urma crora toate urnele s conin acelai numr de bile. t a a t a s a Gheorghe Iure

Clasa a VI-a
2

VI.41. Pe opt cartonae sunt nscrise cte unul din numerele 1, 2, 22 , s 3 , 33 , 34 . Dac P (k) este probabilitatea ca, extrgnd dou cartonae, nu a a a s obinute sa aiba n total k divizori distinci, sa se rezolve inecuaia P (k) 1 t t t Dumitru Dominic Bucesc VI.42. Fie x, y, z N pentru care 84x + 91y + 98z = 2002. S se afle v a maxima a sumei x + y + z. Adrian Zanosch VI.43. Fie {a1 , a2 , . . . , an } Z pentru care k {1, 2, . . . , n}, i, j {1, 2, . i 6= j, astfel nct ak = ai + aj . Sa se arate ca n 6. Petru Asafte VI.44. Fie ABCD un paralelogram i M AB, N AD triunghiuri echilatera s struite n exteriorul acestuia. Demonstrai c [M N ] [BD] dac i numa t a a s N D k M B. Ciprian Baghi VI.45. Fie E, F picioarele nlimilor din B i C ale triunghiului ascuit at s t ABC. Dac P , N sunt mijloacele laturilor [AB], respectiv [AC], iar {Q} = P E a b \ s se arate c m(P QF ) = 180 3 m(A). (n legatura cu Q1086 din Par a a nr. 3/2000 ) Titu Zvonaru, Buc

Clasa a VII-a
t VII.41. Rezolvai n N2 ecuaia t

a b + = 1. b+1 a+1 Alexandru Negrescu, elev, Bo VII.42. S se arate c a2 + 1 b2 + 1 c2 + 1 (|a| + |b|) (|b| + |c|) (|c| a a a, b, c R. Dorin Mrghidanu, Co a VII.43. Pentru n N, notm cu s (n) numrul de reprezentri distin a a a lui n ca sum de dou numere naturale (n = a + b i n = b + a constituie a a s reprezentare). Sa se arate ca: n hni hn + X 1 mn s (k) = a) s (m + n) = s (m) + s (n) [1 + (1) ]; b) 2 2 2 k=0 Petru Minu 76

VII.44. Fie [AB] diametru al cercului C de centru O, N, M C astfe \ ) = 36 , iar [OM este bisectoare pentru N OB. Dac T este simetr \ m(AON a O faa de M N , s se arate c proiecia lui T pe AB este mijlocul lui [AO]. t a a t Valentina Blende VII.45. Fie 4ABC echilateral, iar P (BC). Notm cu D, E simetricel a faa de AC, respectiv AB. S se arate c dreptele AP , BD i CE sunt concu t a a s Constantin Cocea i Julieta Grigora s

Clasa a VIII-a

Lucian Tuescu, C t VIII.43. Dac un triunghi dreptunghic are laturile numere naturale, ia a catetelor este ptrat perfect, atunci suma cuburilor catetelor este sum d a a patrate. Andrei Nedelc VIII.44. Pe laturile [AB], [CD], [BC], [AD], [AC] i [BD] ale tetr s BP lui ABCD se iau respectiv punctele M , N , P , Q, R, S astfel ca = BC DN AR DS AM = , = . Notam cu V1 , V2 , V3 , V4 , V respectiv volumele tetra AB DC AC BD AM RQ, BP M S, CP N R, DN QS i ABCD. S se arate c 212 V1 V2 V3 V4 V s a a Viorel Cornea i Dan Stefan Marinescu, Hune s VIII.45. Fie A1 , A2 , . . . , Ak puncte pe un cerc C. S se determine o c a necesar i suficient pentru a putea nscrie n C un poligon regulat cu n lat as a admite punctele date ca vrfuri (nu neaprat consecutive). a Irina Mustaa, elev t

t VIII.41. Fie f1 , f2 , f3 funcii liniare ale caror grafice sunt drepte conc doua cte doua. Cele trei drepte sunt concurente daca i numai daca exist s R i exist u 6= v R astfel ca s a f1 (u) f2 (u) f3 (u) = = , cu fi (v) 6= , i {1, 2, 3} . f1 (v) f2 (v) f3 (v) Claudiu Stefan Pop VIII.42. Fie x, y, z (0, ). Sa se arate ca yz + zx xy + xz p p + yz + zx + x2 + y 2 + z 2 yz zx xy + xz + x2 + y 2 + z 2 xy xy + yz p 2. + xy + yz + x2 + y 2 + z 2 xy yz

Clasa a IX-a

IX.41. Pentru n N, n 10, notam cu u2 (n) numarul format din ultime cifre ale lui n. S se arate c: a a a) u2 a20k+p = u2 (ap ), p {4, 5, . . . , 23}, k N, a {2, 3, 8}; b) u2 a10k+p = u2 (ap ), p {2, 3, . . . , 11}, k N, a {4, 9}; c) u2 (5n ) = n N; 25, d) u2 65k+p = u2 (6p ), p {2, 3, . . . , 6}, k N; 77

Ovidiu Pop, Satu 1 1 1 IX.42. Fie a, b, c R astfel nct a + b + c = + + . a b c a) Daca |abc| > 1, sa se arate ca unul dintre numere este mai mare n mo 1, iar altul mai mic n modul ca 1. b) S se afle numerele dac |abc| = 1. a a Marius Pachiariu, ele t IX.43. Fie funcia f : R R, iar a (1, ). Stiind c t a 1 1 f x2 + ax a f 2 + 1, x (, 0) , f x x s se arate c f nu este injectiv. a a a Titu Zvonaru, Buc IX.44. Dac 4ABC a este ascuitunghic, s se gseasc maximul ex t a a a E = sin A cos A + sin B cos B + sin C cos C. Cezar Lupu, elev, i Tudorel Lupu, Con s IX.45. Demonstrai ca 4ABC n care are loc egalitatea t X ha hb mc = 1, ma mb mc + ha hb mc + ma mb ic suma fiind obinut prin permutri circulare, iar notaiile fiind cele uzual t a a t echilateral. Iuliana Georgescu i Paul Georgesc s

e) u2 74k+p = u2 (7p ), p {2, 3, 4, 5}, k N.

2 F2n 2n , where the Fibonacci numb n Fn1 Fn are defined by F0 = F1 = 1, Fn+1 = Fn + Fn1 , n 1. Zdravko Starc, Vrac, Serbia and Munte X.42. S se rezolve ecuaia 2[x] + 6[x] + 7[x] = 3[x] + 4[x] + 8[x] . a t Daniel Jinga, P X.43. Fie f o funcie reala nenula cu proprietatea ca t

Clasa a X-a

X.41. Prove the inequality

Adrian Zanosch X.44. Urnele U1 , U2 , . . . , Un conin fiecare cte a bile albe i b bile negr t s fiecare urn se extrage cte o bil care se depune ntr-o alt urn U . Din urn a a a a scoate o bil i se constat c este alb. Care este compoziia cea mai prob as a a a t urnei U ? Petru Minu X.45. Se considera triunghiul ascuitunghic ABC. Sa se arate ca exi t \ triunghi A0 B 0 C 0 astfel nct A0 (BC), B 0 (AC), C 0 (AB), iar m(AC \ \ = m(BA0 C 0 ) = m(CB 0 A0 ) = (0, 90]. Daca n plus 4ABC este echilatera calculeze lungimile laturilor 4A0 B 0 C 0 n funcie de a = BC i . (n legatur t s problema propusa la O. N. M., 2002 ) Dan Popescu, Su 78

f (x + y xy) = f (x + y) f (x) f (y) , 2003 . S se calculeze f a 2002

x, y R.

Clasa a XI-a
XI.41. Fie A1 , A2 , . . . , Ak Mn (Z) astfel nct

A(1) A(2) . . . A(k . unde Sk este mulimea permutrilor de ordin k. S se arate c n . k!. t a a a . Vladimir Martinu s XI.42. Prin punctele M1 i M2 ale unei elipse se duc normalele la elips s 0 0 intersecteaza una din axele de simetrie ale acesteia n M1 , respectiv M2 . 0 0 arate c mediatoarea segmentului [M1 M2 ] trece prin mijlocul lui [M1 M2 ]. R a proprietatea adevrat pentru hiperbol sau pentru parabol? a a a a Gheorghe Costovic XI.43. Consideram irul de funcii fn : (0, ) R, fn (x) = nx + ln x t s i fie xn soluia unic a ecuaiei fn (x) = 0. S se calculeze limitele irurilor ( s t a t a s i ((xn )n )n1 . s a Angela Tigeru, Su XI.44. S se determine funciile continue f : (0, ) (0, ) pentru care f a t 2x2 2x + 1 , x > 0. =f Marian Ursrescu, R a XI.45. Fie k N, k 2 i numerele reale pozitive a1 , a2 , . . . , ak , b1 , b2 , . . . s p a1 < a2 < < ak . Definim xn = n b1 an + b2 an + . . . + bk an . 1 2 k a) Sa se demonstreze ca lim xn = ak ;
Sk n

b) Sa se arate ca lim n (xn ak ) = ak ln bk ; n n ak c) Dac bk = 1, are loc lim n a (xn ak ) = ak bk1 . n ak1 Marian Tetiva, B Clasa a XII-a Z (1 + x) 1 + x2 . . . 1 + x2n1 dx, unde x XII.41. Sa se calculeze x2n nN . Oana Marangoci, student Z 5 4 5 f (x) sin 2x d XII.42. Fie f : , R o funcie continu pentru care t a 4 5 astfel nct f (c) (1, 2). S se arate c exist c , a a a 4 Mihai Haiva XII.43. S se arate c a a Z 1 ln a arctg ln a x2 1 , a > 1. a dx 3 ln a 0 Petru Rducan a XII.44. S se afle numrul rdcinilor reale ale polinomului P Z [X] d a a a a minim, care admite rdcina 2 + , unde verific ecuaia x3 x + 1 = 0. a a a t Laureniu Modan, Buc t 2 XII.45. Fie S5 . Sa se arate ca are puncte fixe daca i numai daca s puncte fixe. Paul Georgescu i Gabriel Pop s 79

Probleme pentru pregtirea concursurilor a


A. Nivel gimnazial
G46. Determinai ultimele cinci cifre ale numrului t a A = 72000 + 72001 + 72002 + 72003 .

Ovidiu Pop, Satu 3 a a G51. Fie a, b, c , cu a + b + c = 1. S se arate c 10 2 3 a a + bc + b b + ca + c c + ab < . 3 4 Gabriel Dospinescu, elev, O G52. Se consider o piramid format din ptrate 1 1, a a a a avnd n trepte, pe treapta k existnd 2k 1 ptrate (n figur, a a n = 4). Aflai numarul minim de dreptunghiuri, fiecare alcatuit t numai din casue ntregi, n care poate fi mparita tabla. t t Adrian Zahariuc, elev, G53. Fie ABCD un patrat de latura 70. Sa se arate ca exista o mul t patrate Pk = Ai Bi Ci Di | Ai Bi = i, i = 1, k care sa aiba suma ariilor egala ptratului dat. Putem acoperi ptratul ABCD cu elementele mulimii Pk ? a a t Petru Asafte G54. Sa se arate ca nu putem alege nici un punct n interiorul triun echilateral ABC de latur l 10, care s aib distanele la vrfuri numere a a a t distincte. Doru Buza G55. Printr-un punct situat n interiorul unui tetraedru se duc planele para feele tatraedrului. Daca V1 , V2 , V3 , V4 sunt volumele tetraedrelor unic deter t 80

Viorel Cornea i Dan Stefan Marinescu, Hune s G47. Determinai valorile parametrilor a, b Z pentru care soluiile siste t t y x x=a ; y=b y+1 x+1 sunt n Z Z. Temistocle Brsa 2002 i avnd proprietatea c G48. Fie A (0, ) o mulime care conine t t s 2003 a+1 a a A (a, b N ), atunci A i s A. Sa se arate ca A Q . + b b 2b Gheorghe Iure G49. Fie x1 , x2 , . . . , xn+1 R astfel nct x1 + x2 + . . . + xn+1 (n + 2) + n+4 2 2 2 2 a M , unde m = min xi , M = max xi . S se a x1 + x2 + . . . + xn+1 4 exact n dintre numerele date sunt egale. Eugen Jeca G50. Fie a N, a 3. Sa se arate ca an + 1 = an + 2 = . . . = an + a 1 , n N a {3, 4}

de aceste plane, iar V este volumul tetraedrului dat, s se arate c a a V 16 (V1 + V2 + V3 + V4 ) . Neculai Roman, Mirceti s

bs L46. Fie ABCD un patrulater inscriptibil. Bisectoarele unghiurilor A intersecteaza ntr-un punct situat pe latura [CD]. Sa se arate ca CD = AD Mircea Becheanu, Buc L47. Dac un triunghi are ptratele laturilor n progresie aritmetic, a a a simetricul centrului de greutate faa de latura mijlocie se afl pe cercul circu t a triunghiului. Gabriel Popa i Paul Georgesc s L48. Fie R, r, R1 raza cercului circumscris 4ABC, raza cercului nscris 4 respectiv raza cercului circumscris 4DEF determinat de picioarele bisect interioare ale 4ABC. Sa se arate ca R/2 R1 r. Marian Tetiva, B L49. ntr-un ptrat 10 10 se nscriu numerele 1, 2, 3, . . . , 100 n aa fe a s oricare dou numere consecutive s se afle n csue vecine. Demonstrai c e a a a t t a linie sau o coloana ce conine macar doua patrate perfecte. t Adrian Zahariuc, elev, a L50. Fie (an )n1 o progresie aritmetic avnd a1 = 5, r = 2002. Pen element b al progresiei, s se arate c bm aparine progresiei dac i numa a a t a s 60 | m 1. Mihai Piticari, C-lung Moldov L51. Fie A, B M2 (R) dou matrice care comut i pentru care det A2 + a as 2 < (det A + det B) . S se arate c xA + yB este matrice nesingular, x, y a a a Ctlin Calistr a a L52. Fie Q C [X] un polinom de grad m avnd radacinile distincte. determine cardinalul mulimii t E = {P C [X] | A Mn (C) a. . Q (A) = On i P (X) = det (XIn A s Ovidiu Munteanu, B L53. Fie n 2 i (A, +, ) un inel comutativ cu n2 elemente, care are ce s n 2 divizori ai lui zero. S se arate c A este corp. a a Gabriel Dospinescu, elev, O L54. Fie f : R R o funcie cu derivata continu pentru care f (x) 6= 0, t a Sa se determine funciile continue : R R care satisfac identitatea t Z y Z x 1 1 f (x) (t) dt (y) = f (y) (t) dt (x) , x, y R a a 0 0 unde a 6= 0 este o constant dat. a a Adrian Cordunean a1 ; xn = L55. Fie a (0, ) \ {1}. Definim irul (xn )n1 prin x0 = s ln a n a t as s t s xn1 , n 1. Artai c irul este convergent i calculai lim xn i lim n n ln a Gheorghe Iure 81

B. Nivel liceal

Premii acordate de FUNDA TIA POIANA

Fundaia Poiana, prin d-l Dan Tiba, pune la dispoziia revistei "R t t matematice" suma de 100 care se constituie ca fond de premii acordate el colaboratori care se disting prin calitatea articolelor, notelor i problemelor or s aparute n paginile revistei. Redacia revistei decide premierea cu cte 1 000 000 lei a urmatorilor ele t 1. DOSPINESCU Gabriel (Liceul "D. Cantemir", Oneti ) s Combinatorica . . . algebrica (RecMat 2/2003, 1922), probleme propuse: G.51, L.53;

2. PACHITARIU Marius (Colegiul Naional, Iai ) t s Cteva aplicaii ale teoremei lui Casey (RecMat 1/2003, 303 t probleme propuse: IX.37, IX.42;

3. CRJA OANA (Colegiul Naional "C. Negruzzi", Iai ) t s Aplicaii ale rotaiei planului complex (RecMat 2/2002, 2123 t t Un procedeu de calcul al limitelor unor iruri de forma (an+1 s (RecMat 2/2003, 2324 Premiile se pot ridica direct de la redacie sau pot trimise prin mandat t

Premii acordate rezolvitorilor

Pentru apariia de trei ori la rubrica "Pagina rezolvitorilor" redacia r t t "Recreaii matematice" acorda o diplom i un premiu n cari elevei t as t

SOFICU Crina Maria (Scoala nr.3 "Al. Vlahua", clasa a IV-a, Iai ): t s RecMat 1/2002 (6 pb), 2/2002 (6 pb), 1/2003 (5 pb). Crile au fost oferite de ctre at a

Editura PARALELA 45

82

Pagina rezolvitorilor
BOTO ANI S Scoala nr. 7 "Octav Bancila". Clasa a VIII-a. NEGRESCU Alexandru: 37), VII(31,36,38), VIII(31,33,37), IX.31, G(23,26,27,33,38-41).

CRAIOVA Colegiul Naional "Fraii Buzeti". Clasa a V-a. AL KHATIB Anne t t s P(51,53), V(36,37,39); BAZA-VERDE Daniela: P(50-53), V.36; DECA Ale Maria: P(51-53), V(37,40); PATRA CU Andrada: P(50-53), V.37. Clasa a S POPESCU Mihnea: V.37, VI(37-40). Clasa a VII-a. VASILE Teodor: VI( VII(39,40). Scoala nr. 22 "M. Eliade". Clasa a III-a (nv. STAICU Angela). STA Ioan: P(44-50,52,53).

IA I S Colegiul Naional. Clasa a VII-a. CO BUC Mircea: VI(36-40), VII( t S VIII.37; IANU Andrada: VI(36-39), VII.40; PRELIPCEAN Cristina: VI(37, S VII(36-40), G.40. Clasa a VIII-a. ANDRIE Delia: VI(36-39), VII.36, S APETROAEI Georgiana: VII(36,39,40), VIII(39,40); BALAN Doru: VI( VII.38, VIII.40, G.40; BALANIUC Drago: VI(36-38,40), VII.39; BATRNU s lina: VI(36-40), VII(36,39), VIII.37; BELCESCU Cosmin Cezar: VI(37,38), V 38,40); CHELSAU Ariel: VI(37,38), VII(36,38,39); CHIRUTA Marta: VI( VII(36,38-40), VIII.37; CHITIC Ionu: VI(36,38,40); VII(36-39); VIII.37; t STANTIN Diana: VII.39, VIII(37-40); CROITORU Cosmina: VI(36-39), V 38), VIII.36, G(37,40,42); DOBRILA Tudor: VI(36,38,40), VII(38,40), VIII.3 DEA Andreea: VI.40, VII(38,40), VIII.36, G.40; FALTICEANU Paul: VI( VII.40; FLORESCU Darian: VI(37-40), VII(36,38,40); GRADINARIU Ioana A dra: VII(36,39,40), VIII(37,38); GRADINARU Andrei: VI(36,37,39,40), V 40); ILIESCU Alca-Iolanda: VI(36,38,39), VII.39, VIII.37; LUCA Paul: VI( VII(38-40); MACUC Andra: VI(36-39), VII.39; MATEI Silvia: VII(36-40), V MARARI Cezarina: VI(36-38), VII(36,37,39); MARTINU Luciana: VII( S G.40; SAVENCU Ramona-Irina: VI(36-38), VII.39, VIII.37; TOMESCU Seb VI(36,38-40), VII.36; TUCALIUC Vlad: VI(36,38,40), VII(38-40); TURCAN xana: V.37, VI(36,38), VII(38,40); VNTU Calin: VII(36-40), VIII.40; ZANO Delia: VI(36,39, 40), VIII(36,38); ZANOSCHI Iulia: VI(36,38), VII.38, VIII( Clasa a IX-a. PACHITARIU Marius: VIII(36-40), IX(36-40), X(37-40), G 41,42), L(36,39). Clasa a X-a. MUSTATA Irina: VIII(36-38,40), IX(36-40) 38), XII.36. Colegiul Naional "C.Negruzzi". Clasa a V-a. HARAGA Anca-Elena: t 52), V(37,38), VI(37,38). Clasa a VII-a. DICU Ciprian-Dinu: VI(38,39), V 38). Clasa a X-a. BEJINARIU Alexandru: VIII(31-35), IX(32-35), X( G(23,25,26,28,30,32,33). Liceul "Garabet Ibraileanu". Clasa a IV-a (nv. LISNIC Sebastian). TIBA ius P(47-53). Clasa a VI-a. BUDEANU Stefana: P(52,53), V(37,39), VI.38; Matei: P(52,53), V(37,39), VI.38; PLACINSCHI Oana: P(41,42,50), V(31,33). a VIII-a. TANASE Ioana: VI(37-40), VII.40, VIII.40. 83

Liceul Teoretic "M.Eminescu". Clasa a V-a. CIURARU Ionela: P(42,43, V(31,33,37-39); MA TALERU Alexandra: P(50,52), V(36,37,39). S Scoala nr. 17 "I. Creanga". Clasa a VII-a. IFTODE Andreea: VI( VII.32, IX.31; PAULIN Ana Maria: VI(33-35), VII(31,32,34); SOFRONEA Ga VI(33-35), VII(31,32); TANANA Irina-Eliza: VI(33-35), VII(31,32). Scoala "G.Cobuc". Clasa a II-a (nv. GALIA Paraschiva). ALUPEI s P(37,44-47); CIOABA Oana-Catalina: P(37,44-47); GHERCA Catalin: P 47); HOMEA Liviu: P(37,44-47); HUIDE Gina: P(37,44-47); IGNAT A S P(37,44-47); MIHAILESCU Laura: P(37,44-47); PISICA Alexandru: P(37, SCUTARU Constantin: P(37,44-47). Clasa a II-a (nv. RACU Maria). B BULA Ioana: P(37, 44-47); BULGARU Ionela: P(37, 44-47); CALOIAN A P(37, 44-47); CALIN Georgiana: P(37, 44-47); CRACIUN Mdlina: P(37, a a IFROSA Adriana: P(37, 44-47); IOJA Petru-Alexandru: P(34,44-47); LE Crina-Alexandra: P(37, 44-47); MOISA Bogdan: P(37, 44-47); PINTILIE R P(37, 44-47); RUSU Flavia: P(37, 44-47). Scoala "Alexandru cel Bun". Clasa a II-a (nv. SPNU Doinia). BUR t Ionu-Mihai: P(44-47,53); DAMIAN Daniel: P(44-47,53); FLOREA Roxanat P(44-47,53); FURTUNA Marta: P(44-47,53); IFTENIE Ioana-Catalina: P(44 RUSU Alexandru: P(44-47,53); URSU Gina-Ioana: P(44-47,53). Scoala "N.Tonitza". Clasa I (nv. TUDOSE Elena). ANCHIDIN Alex P(33,34,44-46); CRNU Alina: P(33,34,44-46); DOBRIN Diana: P(33,34, LEONTE Anca: P(33,34,44-46); POSTICA Simona: P(33,34,44-46); ROT Larisa-Maria: P(33,34,44-46). Clasa I (nv. MELINTE Rodica). BACIU C P(33,34,44-46); BRZU Constantin: P(33,34,44-46); BOTO ANU Bianca-M S a t P(33,34,44-46); BUZDUGAN Petru-Ctlin: P(33,34,44-46); CEUCA Dnu-V aa P(33,34,44-46); CONSTANTINESCU Diana-Gabriela: P(33,34,44-46); CUCU NU Paul-Ctlin: P(33,34,44-46); GU OVATE Diana- tefana: P(33,34,44-46) aa S S GAN Larisa-Diana: P(33,34,44-46); MIRON Vlad- tefan: P(33,34,44-46); M S Geanina-Diana: P(33,34,44-46); ROTARIU Marian: P(33,34,44-46); SUCIUC P(33,34,44-46); TEIU-COSTIN Andra: P(33,34,44-46). Clasa a III-a (nv. M Monica). BUTNARU Valentin: P(44-49); ONUTA Alin: P(44-49). Scoala "B.P.Hadeu". Clasa I (nv. TRZIORU Iuliana). ADASCALITE s tor: P(33,34,44-46); BALAN Andrei: P(33,34,44-46); CUBERSCHI PAUL: 35,44-46); E ANU Georgiana: P(33,34,44-46); GREIEROSU Claudiu: P(33 S 46); LAMATIC Ioana: P(33-35,44-46); REBEGEA Andrada: P(33,34,44-46) sa I (nv. TUTU Laura). BUHU Vlad: P(33,34,44-46); BUZA Eduard-A Alexandra-Elena: P(33,34,44-46); GURAU R P(33,34,44-46); CHICHIRAU Claudia: P(33,34,44-46); HATESCU Iustina: P(33,34,44-46); NASTASE A P(33,34,44-46); SIMIRAD Andrei: P(33,34,44-48). Clasa a IV-a (nv. ST Liviu). PINTILIE Liviu: P(44-53); PINTILIE Nicoleta: P(44-53); STERBU Daniel: P(44-53). Scoala "T.Maiorescu". Clasa a III-a (nv. CHIRILA Beatrice). TUDOR Alexandru-Gabriel: P(44-53).

84

Anul VI, Nr. 2

Iulie Decembrie

RECREAII MATEMATICE

REVIST DE MATEMATIC PENTRU ELEVI I PROFES

e i = 1

Editura Crengua Gldu IAI, 2004

Semnificaia formulei de pe copert: i ntr-o form concis, formula e = 1 leag cele patru ramuri fun
tale ale matematicii: ARITMETICA GEOMETRIA ALGEBRA ANALIZA MATEMATIC reprezentat reprezentat reprezentat reprezentat de de de de 1 i e

Redacia revistei :

Petru ASAFTEI , Temistocle BRSAN, Dan BRNZEI, Ctlin - Cristian BUD Constantin CHIRIL, Eugenia COHAL, Adrian CORDUNEANU, Mihai CR (Pacani), Paraschiva GALIA, Paul GEORGESCU, Dumitru GHERMAN (P Gheorghe IUREA, Lucian Georges LDUNC, Mircea LUPAN, Dan MARINESCU (Hunedoara), Gabriel MRANU, Andrei NEDELCU, Gabriel Dan POPESCU (Suceava), Florin POPOVICI (Braov), Maria RACU, Ioan ER (Ortie), Dan TIBA (Bucureti), Lucian Tuescu (Craiova), Adrian ZANOSCHI.

Responsabili de numr : Temistocle BRSAN, Gabriel POPA, Paul GEORGESCU, Gheorghe IUREA, Lucian Georges LDUNC, Mircea LUPAN, Andrei NEDELCU

Adresa redaciei: Catedra de Matematic Universitatea Tehnic Gh. Asachi Iai Bd. Carol I, nr.11, 700506, Iai Tel. 032 213737 / int. 123 E-mail: acord@math.tuiasi.ro

EDITURA CRENGUA GLDU Toate drepturile rezervate ISSN 1582 - 1765 Bd. N. Iorga, Bl. K2, ap. 4, IAI Tel. / Fax: 032 - 230598

Anul VI, Nr. 2

Iulie Decembrie 2

RECREAII MATEMATICE

REVIST DE MATEMATIC PENTRU ELEVI I PROFES

e i = 1
Apare cu sprijinul

FILIALEI IAI a SOCIETII de TIINE MATEMATIC i INSPECTORATULUI COLAR al JUDEULUI IAI

IAI, 2004

Ctre cititori a dup cinci ani de apariie a revistei a t

Fugit irreparabile

Revista "Recreaii matematice", cu doua apariii pe an (exceptnd primu t t n care a fost publicat un singur numar), a intrat n al aselea an al existe s Numele ei deriv din cel al revistei "Recreaii stiinice", care a apru a t t a perioada 1883-1888 i care este prima publicaie din ara noastr destinat s t t a lui studios. Revista "Recreaii tiinice" a publicat materiale din toate t s t tiinei, dar cu precadere articole, note i probleme de matematica. Este o s t s faptul de a ne lega numele de aceast veche i prestigioas revist i este a s a as rspundere ncercarea noastr de a o continua prin asumarea obiectivelo a a (v. RecMat - 1/2003, pp. 1-5), ramase actuale i acum, la mai mult de o su s Membrii fondatori ai revistei "Recreaii Matematice" sunt cei prezeni t t primului numr: Temistocle Brsan, Catalin Calistru, Alexandru Cara a stantin Cocea, Adrian Corduneanu i Gheorghe Iurea. Redacia revistei s t cat i s-a largit cu timpul; actuala redacie (prezenta pe coperta, interior s t mata din profesori, nvaatori i elevi, care fac din entuziasm i cu pasiun s s t necompensat de vreo rsplat material. Nu-l vom uita pe Alin Spuma a a a a de abnegaie, pasiune i competena. t s t n aceti primi ani de existena, eforturile redaciei s-au dirijat spre cr s t t unei identitai a revistei i dobndirea unui impact favorabil al acesteia c s t interesat de matematica elementar. ntr-adevr, revista are n prezen a a grafic de prezentare definitivat i un coninut structurat pe un numr a as t a bine conturat. Pe de alta parte, punctele de distribuie a revistei sunt ras t o arie ntinsa, iar colaboratorii cu note i probleme originale provin din to s Aceste preocupri sunt n legtur strns cu atingerea obiectivelor rev a a a a constituie n fapt raiunea apariiei sale. t t Revista se adreseaza elevilor de la cei mici, din clasele primare, pn venii liceelor studenilor preocupai de viitoarea lor munca la catedra, p t t t i tuturor celor ce ndrgesc matematica elementar. s a a Prima parte a fiecrui numr ce ocup aproape jumtate din spaiu a a a a t destinata articolelor de informare, studiilor i notelor originale, chestiunilo s i din istoria matematicii. A fost publicat n aceti cinci ani un numar de ap s s o sut de articole de acest fel, care a oferit cititorilor un material variat, pe a nivelurile de pregtire, n bun parte accesibil i elevilor. Credem c existen a a s a spaiu larg de publicare a stimulat posibilitaile creative ale cititorilor no t s t n scopul cultivarii gustului pentru matematica, au fost publicate un portrete de matematicieni ilutri (Fermat, Abel, Kolmogorov etc.) s s prezentate unele probleme celebre (postulatul V al lui Euclid, marea teo Fermat, problema celor patru culori, trisecia unghiului etc.). Au fost evoc t nalitai remarcabile ale matematicii romneti (Spiru Haret, Gh. Vrnce s a t reviste cu un aport major n cultura tiinic a arii (Recreaii stiinice, R s t a t t t inica "V. Adamachi"), instituii cu un rol important n nvamntul i t t at s romneasca (Seminarul matematic "Al. Myller", Observatorul astronomic

momente de vrf ale matematicii din ara noastr (Al V-leaCongres intern t a matematicienilor romni ), ct i multe guri de matematicieni ieeni, dis s s n viaa (I. Creang, Gh. Gheorghiev, R. Miron, C. Corduneanu et t a de prestigiu ale nvamntului liceal (N. Colibaba). at Menionm n mod special rubrica "Nota elevului", care a devenit p t a ncepnd cu nr. 1/2001 al revistei i care a gazduit deja 11 articole. Ele s buni au n aceasta rubrica un spaiu destinat ncercarilor lor pe un anume s t pe o idee propice. La recomandarea redaciei revistei, Fundaia culturala t t (director d-l Dan Tiba) ofer anual premii n bani tinerilor autori ai celor a astfel de note. Pna n prezent au fost premiai un numar de cinci elevi. t Partea a doua cea mai dinamica parte a revistei este destinata con problemelor propuse i soluiilor acestora i se ncheie cu o list a rez s t s a Sunt publicate cu regularitate subiectele date la urmtoarele concursuri a organizate de ieeni: Concursul "Al. Myller" (concurs naional, cl. VII-X s t cursul "F. T. Cmpan" (concurs interjudeean, cl. IV-VIII), Concursul t matematice" (concurs n cadrul Taberei naionale de matematic, cl. VII t a cursul "A. Haimovici" (concurs interjudeean pentru liceele economice, t i agricole, cl. IX-XII). Tot cu scopul informarii elevilor competitivi, sunt s i concursuri organizate n alte centre: Concursul "R. Miron" (Vaslui), s "Unirea" (Focani), Concursul "O. Onicescu" (Botoani) toate ind inte s s Pentru elevii din gimnaziu talentai au fost publicate enunurile i solu t t s lemelor date la OBM (juniori), ct i problemele aflate n atenia juriului s t Subiectele date la examenul de admitere n cteva faculti din un at ieene ofer o orientare candidailor care opteaz pentru facultile respec s a t a at Desigur, n centrul demersului nostru se afl rubricile "Probleme p a "Soluiile problemelor propuse" prin care se urmarete atragerea elevi t s studiul matematicii i o bun pregtire a celor care reuesc s devin s a a s a a constani i pasionai. n aceti cinci ani de apariie, s-au publicat 73 prob t s t s t tru clasele primare i cte 50 probleme pentru fiecare clas de gimnaziu sa s a Sunt rezolvate toate problemele propuse dupa un an de la publicarea lor, s multe soluii (atunci cnd acestea apar) i se menioneaz autorii acestora t s t a Rubrica "Probleme pentru pregatirea concursurilor", deschis n nr. a revistei i mprit n dou nivele (gimnazial i liceal), pune la dispoziia s at a a s t nclinaii speciale probleme cu un grad de dificultate sporit. t Revista se ncheie cu "Pagina rezolvitorilor", o list a elevilor rezolvit a bleme menit s-i ncurajeze i s-i ambiioneze. Orice elev menionat de a a s a t t aceast rubric primete din partea redaciei o diplom i un premiu n ca a a s t as Printre note i probleme au fost presarate i un numar de "Recreaii s s t matice" pentru a dovedi c matematica poate lumina feele celor pasiona a t t cu idei inspirate, ci i cu zmbete. s Dup aceti cinci ani de apariie, desprindem o concluzie: s continum a s t a a pasiune pentru a ndrepta ceea ce am greit, a completa ceea ce am facut m s i a tinde spre perfeciune n acele direcii n care am reuit s facem ceva s t t s a

Redacia revistei "Recreaii matema t t

Alexandru Myller, ctitorul colii matematice i s (3 decembrie 1879 - 4 iulie 1965)

La 3 decembrie a. c. se mpl sferturi de veac de la naterea lu s dru Myller, savant de reputa naionala si eminent profesor t sitatii din Iai. s S-a nascut n Bucureti, und s coala primar, liceul (1896) i s a s de Stiine (1900), avnd ca pr t ilutrii matematicieni romni S. s Pangrati, N. Coculescu i D. Em s Dup un scurt stagiu ca p a Liceul "V. Alecsandri" din Gala n 1902, la studii la Gttingen, u ca profesori pe celebrii matemat Klein i David Hilbert. Prelu s noua teorie a lui Hilbert asupra integrale, Myller publica un ciclu printre care i teza de doctorat ( s borat sub ndrumarea lui Hilbe a Lucrrile lui Myller n domen a matematice, elaborate la Gttin tinuate la Bucureti, au marcat, s metodele i rezultatele obinute, un moment important n dezvoltarea i s t s matematicii romneti pe plan naional i internaional. Acestea, precum s t s t elev al lui Hilbert, l-au consacrat ca matematician de prima marime. La I i schimba direcia cercetarilor matematice, trecnd la geometrie, domen s t fascinat nc din tineree. a t A obinut numeroase rezultate n domeniul teoriei ecuaiilor difer t t integrale prin: extinderea unor rezultate ale lui Hilbert la cazul unor ecua tiale de ordin arbitrar; ecuaii integrale cu nucleu antisimetric; probleme t limita si de periodicitate pentru ecuaii difereniale ordinare si cu derivat t t utilizarea metodelor funcionale n rezolvarea unor probleme de fizica mat t n domeniul geometriei difereniale contribuiile sale se refera la: t t algebrica si geometrie riglata; definirea noiunii de concurena a vectoril t t variani ca o generalizare a paralelismului Levi Civita; paralelismul ntr t de plane, care a condus la noiunea de conguraie Myller; dezvoltarea, m t t O. Mayer a geometriei difereniale centroane. Aceste cercetari au fost de t impulsionate de apariia teoriei relativitai generale, unde se folosea n m t t tent teoria conexiunilor afine i alte noiuni geometrice legate de conexiu s t preocupari au marcat momentul intrarii scolii de matematica de la Ia s mondiala a cercetarii stiinice. t

n domeniul istoriei matematicii a reuit s repun n valoare co s a a originale ale unor precursori precum D. Asachi, St. Botez, E. Bacaloglu e Numit n 1910 profesor titular la catedra de geometrie analitica a Unive Iai, Alexandru Myller pune bazele nvatamntului matematic modern s instituie de nvatamnt prin: fondarea vestitei biblioteci de specialitate (18 t ca fundament al cercetarilor originale; ncadrarea unui corp profesoral de loare; atragerea unor tineri cu care creeaza prima scoala romneasca de m cunoscuta sub numele de Seminarul Matematic din Iai; iniierea n s t la Iai a unor studii de istoria matematicii romneti si universale; in s s unor cursuri libere de specialitate si a lucrarilor de licena; aporturi o t geometria difereniala, care au lansat scoala ieeana n competiie inter t s t Aceste evenimente s-au petrecut, prima oar n 1922 cnd au fost publicat a legate de paralelismul lui Levi Civita, a doua oar, n 1933, cnd a aprut a a de geometrie difereniala centroafina elaborat n colaborare cu O. Mayer. t Ca profesor, Al. Myller a fost un maestru n arta comunicarii cu st aplicat, pentru prima oar n ara noastr, metoda euristic n predarea m a t a a la nivel universitar. Leciile sale erau adevrate momente de creaie n care t a t ghida pe studeni sa descopere, mpreuna, adevarurile tiinei predate. A t s t cu dragoste i generozitate fiecrei generaii de studeni tot ce a acumulat s a t t de vedere tiinific i metodic. s t s Sub ndrumarea sa, a fost obinut n 1920 primul doctorat n matematic t universitate romneasca, de catre Octav Mayer, iar n 1925 a fost obinut t n matematici de ctre Silvia Creanga, care devine prima femeie doctor n m a la o universitate romneasc. a Dup rzboi, a funcionat ca rector n dicila perioad 1944-1945, a a t a deschida cursurile universitare n mai 1945, dupa refugiul n Transilvan preocupat de reconstrucia cldirii, de revenirea la Iai a laboratoarelor t a s s cilor, precum i a profesorilor i studenilor. s s t n 1947 iese la pensie, dar rmne legat de activitatea Seminarului M a unde continua cercetarile tiinice, se preocupa de instruirea tinerilor s t i de bunul mers al bibliotecii urmrind cu asiduitate obinerea unor pub s a t greu de procurat. A mai desfurat variate activiti i la Institutul de Matematic de la as at s a a Academiei Romne. A continuat sa lucreze pna la stingerea sa din viaa t la 4 iulie 1965. Pentru meritele sale de excepie, Academia Romn l-a ales me t a tular n 1938. n 1959 Universitatea Humbold din Berlin i-a decer de Doctor Honoris Causa pentru straduini deosebite de a creea o m t romneasca de sine statatoare. A primit numeroase distincii i decoraii t s t statului romn. La mplinirea a 125 ani de la naterea savantului Alexandru Mylle s elevii elevilor sai l omagiaza n semn de adnca preuire pentru opera sa t dezvoltrii tiinei i nvamntului modern n ara noastr. a s t s at t a

Prof. dr. Gheorghe BANT Prof. dr. Vasile OPROIU

Henri Poincar la 150 de ani de la naterea sa s


La pense nest quun clair, au milieu dune longue nuit, mais cest cet clair qui est tout. H. P.

Este anevoie pentru un om sa compare coloii; adncim s ite tulbur vederea detaliilor a nu reuete s completeze s s a de neles. Exista consensul t Poincar a fost ultimul ma universal si s-a dovedit genia a ntreprins. Acum, la 150 d naterea sa, zabovim un pic s el. Henri Poincar s-a Nancy, pe 29 aprilie 1854, ntr familie de intelectuali. Amb miop, a suferit de o slaba c muscular. Din coala elemen a s celat n compoziii scrise. n t nscris la Liceul din Nancy 11 ani s-a dovedit aici un e cit la toate materiile. Profeso matematic l considera un m a matematicii. La concursurile generale ale liceelor din Frana a ctigat p t s ti. n 1873 este admis la cole Polytechnique pe care o absolva n 1875, d i clar colegii n domeniile matematicii. Citea mult i variat, realiznd s s neateptate i dovedind o memorie vizual excelent. i completeaz cu m s s a a s a studiile la cole des Mines i profeseaza ca inginer n timp ce i elaboreaz s s de doctorat, sub conducerea tiinica a lui Charles Hermite, n domeniul s t difereniale. Devine doctor n matematic n 1879, convingnd dar i uimin t a s Pred analiz matematic 2 ani la Caen, din 1881 primete catedr a a a s a tatea de Stiine din Paris, din 1886 trece la Sorbona pe Catedra de ecua t matematice i probabilitai, preda i la cole Polytechnique. Leciile sa s t t s domenii variate, mereu n schimbare, dar nu sunt uor de urmrit de ctr s a a din cauza abundenei de idei. Este ales n Academia de Stiine n 1887; t t este ales n fiecare din cele cinci secii ; n 1906 devine secretar permanent a t n 1908 este ales n Academia Franceza al carei secretar permanent devi prematurului su deces, la 17 iulie 1912. a Activitatea sa de creaie tiinic se desfura cu precizie: ntre 10 i 12 t s t a as s ntre 5 i 7 dupa amiaza. Dupa ora 7 se informa. Pornea cte o lucra s

plan prestabilit, fr calcule preliminare; dup primii pai urmau natural aa a s Aborda i abandona subiecte din unghiuri de vedere diferite dar subcont s s continua studiile, ntregind imaginea. Dicultatea de a opri cercetarea l s nu ntreprind lucrri importante dup ora 7 seara, spre a nu-i tulbur a a a a s A dezvoltat studiul funciilor automorfe dup o idee ce susine c i-a t a t a nici o pregatire prealabila n momentul cnd urca ntr-un autobuz; folosea complet diferit transformarile din geometria neeuclidiana. Prin lucrare situs, publicat n 1895, ntemeiaza domeniul topologiei algebrice n care a conjectur este nc actual. Este considerat fondatorul teoriei funciilo a a a t de mai multe variabile complexe. A adus contribuii eseniale n geom t t brica, dezvoltnd metode ce au permis deduceri directe ale unor rezultate ce se bazau pe o idee de demonstraie greit. n 1901 a rezolvat o prob t s a cial de teoria numerelor: cutarea punctelor de coordonate raionale p a a t algebrica f (x, y, z) = 0 cu coecieni raionali. Geometria hiperbolica, t t catre Lobacevski i Bolyai, a devenit de neles pe modelul de univers im s t catre Poincar. Este considerat alturi de Einstein (i Lorentz ) fondator a s relativitatii. Reinem atenia cu cteva detalii referitoare la problema celor 3 co t t presupun date la un moment iniial 3 corpuri date prin trei puncte de mase t prin poziia lor, viteze i acceleraii. Se presupune c asupra lor nu int t s t a o fora exterioar, dar c ele evolueaz respectnd legea atraciei univ t a a a t cere sa se evalueze comportarea lor n timp. Problema abordata de catr n 1889 era ct pe ce s fie compromis de o eroare comis de editorul a a a Mathematica. Un intens schimb de scrisori cu Mittag-Leler a lmurit a i un memoriu al lui Poincar a aprut n 1890, fiind considerat act de s a teoriei haosului. Motivul este ca aceasta problema a celor 3 corpuri re sistem de ecuaii difereniale (cu necunoscutele cele 9 funcii ce dau co t t t punctelor) care nu este stabil. Aceasta nseamn c modificri infinite a a a datelor problemei conduc la modificri substaniale ale traiectoriilor. a t H. Poincar a adus contribuii eseniale n numeroase domenii ale t t cilor aplicate: mecanica cereasca, cosmologie, mecanica cuantica, optica tate, hidrodinamica, telegrae, termodinamica, teoria elasticitatii, electrom capilaritate. n special, modul diferit de a concepe matematica la Poincar i Hilber s la filosofii distincte ale matematicii. n opoziie cu punctul de vedere t formalist, H. Poincar a susinut un punct de vedere intuiionist: p t t demonstram, dar prin intuiie creem, logica ramne sterila fara a fi fer t intuiie. t H. Poincar este un maestru al popularizrii tiinei; crile sale: a s t at ipoteza (1902), Valoarea stiinei (1905), Stiina si metoda (1908), Ultime t t (1908, postum) sunt disponibile i n limba romn. s a La funeraliile sale s-a spus: ... a fost matematician, geometru scriitor, poet al infinitului, bard al tiinelor. s t

Prof. dr. Dan BRN

Trecerea planetei Venus prin faa Soarelu t

Planeta Venus, a doua planeta dupa Mercur ca departare de Soare, cu sub numele de Luceafarul de dimineaa sau de seara, trece din cnd n cn t Soarelui (se spune c n acel moment planeta se afl n conjuncie inferioa a a t "din cnd n cnd" reprezint un interval de timp de cel puin 100 de an a t de 8 iunie a acestui an fenomenul se va desfaura din nou, iar noi cont s fenomenului vom avea prilejul sa-l vedem, daca cerul va fi favorabil obser Trecerea lui Venus zeia frumuseii i dragostei la romani, numit t t s Afrodita, ca ica lui Zeus i Dionei a prezentat o importana deosebit s t a astronomiei i de ce nu i n istoria civilizaiei. Omul, aceasta creaie extra s s t t legilor lumii materiale, prin existena i natura sa, s-a ntrebat de multa vr t s aprut; s-a nscut aici pe Pmnt, a venit de undeva, a fost creat de cineva a a a locul su n lumea Universului observabil? Iat ntrebri, devenite fascin a a a de secole, la care astronomia, dei a raspuns doar parial, totui a dat rasp s t s mult sau mai puin convingatoare. t Poziia Pmntului n Univers poate fi determinat dac se cunoat t a a a s Pamnt - Soare. Aceast distana, msurat prin procedee astronomic a t a a 149 597 870 km. Ei bine, aceasta distana a fost ameliorata treptat prin t timp de cteva secole si atunci cnd Venus trecea prin faa discului sol t exist metode ce permit determinarea distanei Pmnt - Soare cu precizie a t a zeci de kilometri, ns pentru acele vremuri metoda trecerii a reprezentat a remarcabil de la trecere la trecere. Nici acum observarea fenomenului nu este lipsita de interes daca obse fac cu suficient precizie (precizia n timp n determinarea fenomenului, a fie de cel puin o sutime de secund; important este s se cunoasc la fe t a a a poziia locului din care se fac observaiile). t t Trecerea planetei Venus prin faa Soarelui are loc o data la 115 ani t urmeaza o alta trecere dupa 8 ani. Urmatoarea trecere urmnd sa aiba loc de ani, urmata de o alta dupa nca 8 ani. Aa se succed aceste treceri. D s traiectoriei lui Venus ar fi acelai cu planul orbitei terestre, atunci trecerea s ar avea loc dupa fiecare 584 de zile i 22 de ore. Acest interval de timp s perioada sinodica a lui Venus (intervalul dintre dou poziii asemntoare a a t a a n raport cu Soarele i Pmntul). s a ntr-o viaa de om, trairea acestui eveniment nu poate fi dect de d t singura data sau deloc, cu excepia celor ce au o longevitate mai mare t sau 122 de ani. Penultima trecere vizibila la noi a avut loc n anul 1874 urmrit de profesorul Neculai Culianu, fost decan al Facultii de Stiin a at rector ntre anii 1888 - 1898 al prestigioasei universitai ieene. Ultima tr t s putut fi observata dupa 8 ani (1882). Fenomenul se ncadreaza n categoria eclipselor de Soare cnd Luna, M Venus si Pamntul sunt n linie dreapta. Spre deosebire de eclipsele de S Luna interpunndu-se ntre Pamnt i Soare poate sa-l acopere complet, t s

Venus nu are aceast calitate. Venus fiind mult deprtat de Pmnt (n ac a a a a departarea va fi de 43 228 162 km) i mica n raport cu Soarele, va lasa o s un mic disc ntunecat reprezentnd a 33-a parte din diametrul Soarelui. Intrarea peste discul solar (primul contact exterior) va avea loc la ora 47 s ca dup 19 m si 27 s Venus s intre complet peste discul solar la ora a a 14 s (primul contact interior). Urmatoarele etape vor fi: ultimul contact ora 14 h 03 m 39 s, iar ultimul contact exterior va avea loc la ora 14 h 2 Momentele caracteristice menionate mai sus au fost calculate de ctre t a de Mecanica Cereasca si Calculul Efemeridelor din Frana, pentru Buc t teritoriul arii noastre aceste momente vor diferi de la localitate la localit t mai mult de cteva secunde. NASA a fcut calcule i pentru Iai i iat momentele caracteristice: a s s s a 34 s primul contact exterior, 8 h 39 m 11 s primul contact interior, 15 s ultimul contact interior i 14 h 22 m 37 s ultimul contact exterio s nceputul intrarii lui Venus va avea loc prin partea stnga (partea e vecintatea marginii inferioare a discului solar aa cum se poate vedea n d a s turat. Tot n desen se arat traiectoria umbrei i momentele caracteristice m a s mai sus. Durata fenomenului, de la primul contact interior la ultimul conta

va fi de 5 h 24 m 25 s. Pentru observarea fenomenului trebuiesc luate masuri de protecie a o t comandam ca atunci cnd se ndreapta privirea spre Soare sa se foloseas afumat sau o sticl ce se utilizeaz la ochelarii pentru sudur electric. a a a a a ndemn ar fi folosirea foliei metalizate utilizat drept ambalaj de ctre v a a a de flori. Aceasta ultima protecie trebuie confecionata din cel puin dou t t t folii suprapuse, funcie de sensibilitatea ochilor celor ce vor sa observe fen t Observaii se pot face i cu o lunet dotat cu un ocular prin care s se t s a a a Soarele pe un ecran situat la o distana convenabil. Imaginea Soarelu t a ajunge la dimensiunile unui cerc cu raza 10 - 15 cm i chiar mai mult. Evid s trebuie instalata pe un trepied pentru a-i asigura stablitate i urmarire s Soarelui. O urmrire asemntoare a fenomenului poate fi fcut proiectnd a a a a a Soarelui ca mai sus, folosind un binoclu. Amatorii care vor sa aduca contribuii la observarea trecerii, avnd n t unele observatoare specializate pot sa nu aiba vreme favorabila observaiilo t s in seama de urmtoarele: 1) s posede un ceas cu cronometru i pus l at a a a s un post de radio naional, 2) s urmreasc momentul primului contact t a a a sa-l cronometreze i la fel sa procedeze pentru celelalte trei momente car s (primul contact interior, al doilea contact interior i al doilea contact ex s plus faa de aceste recomandri, trebuie s apeleze la o cunotina ce t a a s t aparat numit GPS cu ajutorul cruia s se determine coordonatele locu a a facut observaiile (longitudinea i latitudinea geografica). Daca masurat t s fcute cu o eroare n timp de plus sau minus 0,1 secunde, iar coordonatele a fi determinate cu o eroare de maximum 8 metri n jurul punctului unde msurtorile, atunci observaiile pot fi luate n consideraie. a a t t Observaiile pot fi transmise n scris la Observatorul Astronomic din t Sadoveanu nr. 5, textul urmnd s conin: mijloacele cu care s-au fcut ob a t a a momentele caracteristice msurate i coordonatele locului. a s Operaia, pe plan mondial, este coordonat de ctre Observatorul Eu t a a Sud cu sediul n Garching (Germania) iar calculele centralizate vor fi fac stitutul de Mecanica Cereasca si Calculul Efemeridelor cu sediul la Paris. Urmrirea fenomenului nu este lipsit de interes dac avem n veder a a a lui, importana tiinific, ncrederea i seriozitatea ce se poate acorda t s t a s astronomice, spre deosebire de ideile avansate de prezicatori i astrologi s interpretri apocaliptice unor astfel de fenomene. Din anul 3000 . H. a prezent, au avut loc 64 de treceri i alte evenimente astronomice deosebite, s constatat, n afara unor fenomene naturale izolate, uneori devastatoare (c inundaii, razboaie provocate de om etc.), sa se fi produs acel eveniment a t mult proferat de prezictori. a Iulian BREAHNA

Membru al Uniunii Astronomice Interna ex-director al Observatorului Astronomi

Cteva noi aplicaii ale unei idei consacrat t


Gabriel DOSPINESCU 1

Nu credem c exagerm spunnd c 99% din inegalitile care sunt p a a a at probleme de concurs pot fi demonstrate direct, aplicnd alte inegaliti at sacrate. Exist ns o categorie aparte de probleme foarte greu de rezolv a a mod. Aceste probleme au soluii frumoase, bazate pe raionamente ind t t extrem de eciente. Vom prezenta n continuare ideea comuna aflata n spatele tuturor aces S presupunem c avem de demonstrat o inegalitate de forma g (x1 ) a a + + g (xn ) 1 n ipoteza c variabilele x1 , x2 , . . . , xn veric o rela a a t f (x1 ) + f (x2 ) + + f (xn ) = 1. Presupunem prin reducere la absurd ca g (x1 ) + g (x2 ) + + g (xn ) < 1 t a S = g (x1 ) + g (x2 ) + + g (xn ), obinem c S = pentru un anum k t s Punem kg(xi ) = ai , i = 1, n, rezolvam ecuaiile n xi i introducem ac t a a ca funcii de ai n relaia f (x1 ) + f (x2 ) + + f (xn ) = 1. Gsim c var t i = 1, n satisfac de asemenea o relaie de forma h(x1 ) + h(x2 ) + + t precum i a1 + a2 + + an = 1. Problema se reduce acum la a dovedi ca s h(x1 ) + h(x2 ) + + h(xn ) = 0 este imposibila, lucru care se realizea demonstrnd c h(x1 ) + h(x2 ) + + h(xn ) > 0 (sau < 0) pentru orice n a a2 , . . . , an cu sum 1. a Credem ca noua problema este adesea mai uoara dect cea iniiala s t s exemplifica acest lucru vom rezolva cteva probleme de concurs utiliznd de mai sus. Prima dintre ele, Problema 3, OM China 2003, este faimoasa datorita sale, fiind de asemenea nrudit cu o alt binecunoscut problem care va f a a a a n cele ce urmeaz, Problema 2, OIM 2001. a Exemplul 1. Fie x1 , x2 , . . . , xn (0, /2) astfel ca tg x1 tg x2 . . . tg Determinai cel mai mic kn pentru care inegalitatea cos x1 +cos x2 + +c t este ntotdeauna adevarata. Soluie. Substituind tg2 xi = 2ai , problema revine la a determina su t 1 1 1 + + + pentru a1 , a2 , . . . , an expresiei 1 + 2a1 1 + 2a2 1 + 2an produsul egal cu 1. Nu vom discuta cazul n = 1, acesta fiind trivial. Pentru n = 2, ramn 1 x + , x > 0. Studiind ceea ce se ntm valoarea maxima a lui 1 + 2x x+2 2 x = 1, x i x 0, deducem c valoarea cutat este . Pentru a do s a a a 3 1 lucru, este suficient s se studieze monotonia funciei f (x) = a t + 1 + 2x
1

Student, Facultatea de Matematic i Informatic, Bucureti as a s

x > 0, cu ajutorul derivatei. Sa studiem ce se ntmpla pentru n > 2. Daca toi ai , i = 1, n, sunt t n valoarea expresiei de mai sus este . Acum, dac ncercm s "apropiem a a a 3 muli ai de 0, observam ca putem "apropia" de 0 cel mult n 1 dintre ei, t n valoarea expresiei va tinde la n 1. Deoarece n 1 > , este clar c va a 3 1 1 1 dovedim ca + + + n 1. 1 + 2a1 1 + 2a2 1 + 2an Este uor de vzut c raionnd ca n problema precedent ajungem re s a a t a un impas. Observm ns c este suficient s demonstrm inegalitatea d a a a a a doar pentru n = 3. De ce acest lucru? Daca n > 3 i a1 a2 an = 1, atu s alege ai , aj , ak al caror produs este cel puin 1. Atunci t 1 1 1 1 1 + + + < n3+ +p + 1 + 2a1 1 + 2a2 1 + 2an 1 + 2ai 1 + 2aj

Dar

presupunnd c inegalitatea este adevrat pentru n = 3, iar din relaia p a a a t rezult c ea este adevrat i pentru n oarecare. a a a as Sa dovedim deci inegalitatea pentru n = 3. Presupunem ca ea nu este 1 1 1 2 + + = pentru o tripleta a1 , a2 , a3 . Atunci p 1 + 2a1 1 + 2a2 1 + 2a3 3 3 Q Q p 1 p2 1 , obinem c 1 = t a < Notnd xi = 2 2 2 2 1 + 2ai k=1 8xk k=1 8xk 1 deci suficient sa demonstram ca pentru orice 0 < x, y, z < cu x + y + z 2 1 1 1 1 1 1 1, inegalitate echivalenta cu ca 8x2 2 8y 2 2 8z 2 2 (1 2x)(1 2y)(1 2z)(1 + 2x)(1 + 2y)(1 + 2z) 83 x2 y 2 z 2 . O alta schimbare de variabila este acum necesara. Desigur, aceasta este 1 2y = b, 1 2z = c. Ramne deci sa demonstram ca, pentru orice a, suma 1, avem 8(1 a)2 (1 b)2 (1 c)2 abc(2 a)(2 b)(2 c). Avem ns c a a

1 1 1 1 1 1 +p + +p +q 1 + 2ai 1 + 2ak 1 + 2ai 1 + 2aj 1 + 2aj 1 + 2a

8 8(1 a)2 (1 b)2 (1 c)2 = 8(b + c)2 (c + a)2 (a + b)2 8 (a + b + c)(ab + 9 512 512 512 2 = (ab + bc + ca) abc (a + b + c) = abc. 81 27 27 512 , ceea ce e Este suficient, deci, sa demonstram ca (2 a)(2 b)(2 c) < 27 deoarece (2 a)(2 b)(2 c) < 8, iar problema este acum rezolvata.

A fost menionat anterior, n trecere, frumoasa Problema 2, OIM 200 t a de Hojoo Lee. Problema n cauza fiind att de mult discutata i pop s s-ar putea crede ca nu mai este nimic nou de spus despre ea. Totui, s urmtoarea demonstraie a unei generalizri a problemei este nou. a t a a

Exemplul 2. Demonstrai ca daca a1 , a2 , . . . , an sunt numere reale s t tive astfel nct a1 a2 an = 1, iar k > 1 este un numar natural, atunci 1 1 1 p + p + + p k k k 1 + (nk 1) a1 1 + (nk 1) a2 1 + (nk 1) an

i, din inegalitatea mediilor, s n Y (x1 + x2 + + xi1 + xi+1 + + xn ) (n 1)n x1 x2 . . . x


i=1

Vasile Soluie. Presupunem c t a 1 1 1 p + p + + p = k k k k 1) a k 1) a 1 + (n 1 + (n 1 + (nk 1) an 1 2 p pentru un anumit p > 1. Notam p = xi , i = 1, n i o s k 1 + (nk 1) ai k n n k n Q p Q 1 n 1 = 1 > 1 , iar x1 + x2 + + xn = 1. k k i=1 xi i=1 xi n Q 1 Pentru a ajunge la o contradicie, vom dovedi ca t 1 k i=1 xi pentru orice x1 , x2 , . . . , xn > 0 astfel nct x1 +x2 + +xn = 1. Pentru dem acestei inegalitai, sa observam ca t n n Q Q (x1 + + xi1 + xi+1 + + 1 + xi + + xk1 n i Y 1 i=1 1 = i=1 xk (x1 x2 . . . xn )k i i=1

Desigur, pentru minorarea celuilalt produs din formula (1), aplicarea dir galitii mediilor nu mai este la fel de ecient. Conform inegalitii med at a at q n n X n (x1 x2 . . . xn )i xi j qQ , i = 0, k n k1 n 1 + xj + + xk1 j j=1 j=1 1 + xj + + xj Sumnd inegalitaile de mai sus, obinem t t n Y n 1 + xj + + xk1 1 + G + G2 + + Gk1 , j
j=1

unde G =

1 n x1 x2 . . . xn . Din (1), (2) i (3) deducem c s a n n n Y 1 (n 1)n Gn 1 + G + G2 + + Gk1 1 , Gnk xk i i=1

iar pentru a naliza soluia problemei este sucient s demon t a 1 + G + G2 + + Gk1 1 + n + + nk1 , ceea ce este trivial, deoare Gk1 Este acum momentul s discutm o alt problem deosebit, propus la a a a a a a anual al Gazetei Matematice de catre Vasile Crtoaje. Soluia autorului t pe aplicarea succesiva a ctorva identitai, fiind aproape imposibil de ga t independent. Credem c soluia ce urmeaz este mai natural din acest a t a a vedere.

Exemplul 3. Demonstrai ca pentru orice a, b, c, d > 0 astfel ca a2+b2+ t este satisfacuta inegalitatea (1 a)(1 b)(1 c)(1 d) abcd. (1 a) (1 b) (1 c) (1 d) Soluie. S presupunem c t a a = p4 , cu p abcd P P 1b 1c 1d 1 1a = x, = y, = z, = t. Atunci 1 = 2 > pa pb pc pd (1 + px) iar xyzt = 1. Problema se reduce, deci, la a demonstra ca 1 1 1 1 1 2 + 2 + 2 + (1 + x) (1 + y) (1 + z) (1 + t)2

pentru orice x, y, z, t > 0 verificnd xyzt = 1. Profitnd de faptul c enunul problemei utilizeaz 4 numere, vom a t a merele n 2 grupe i demonstra c s a 1 1 1 1 1 1 i s 2 + 2 1 + xy 2 + 2 1 + zt (1 + x) (1 + y) (1 + z) (1 + t)

Prima inegalitate se reduce la xy (x y)2 + (1 xy)2 0, care es evident adevarata, pentru a doua raionndu-se n mod analog. Prin sum t dou inegaliti i innd seama de faptul c xyzt = 1, rezult concluzia. a at s t a a

n ncheierea acestui articol vom discuta alte doua probleme deosebite, pot da soluii rapide utiliznd ideile de mai sus. Prima dintre ele a fost pu t revista "American Mathematical Monthly", fiind propus de ctre Vasile C a a Exemplul 4. Fie x1 , x2 , . . . , xn > 0 astfel nct x1 + x2 + + xn =

1 1 1 1 . Demonstrai ca t + + + xn n 1 + x1 n 1 + x2 n 1 + xn 1 1 1 Soluie. Presupunem ca t + + + n 1 + x1 n 1 + x2 n1+ p 1 , i = 1, n i deducem c ai < s a p < 1. Notm ai = a , i n 1 + xi n1 t t a1 + a2 + + an = 1. Condiia din enun se transcrie sub forma X n n X p 1 n+1 = . p ak ak n + 1 + +
k=1 k=1

Deoarece p < 1, putem scrie X X n n n X 1 p n+1 > n+1 = ak ak


k=1 k=1

p k=1 ak

X 1 > n+1

1 k=1 ak

n final vom discuta o alta frumoasa problema de concurs, propusa la selecie a lotului Romniei pentru OIM, 1999, de ctre Gheorghe Eckst t a Exemplul 5. Demonstrai ca daca x1 , x2 , . . . , xn > 0 satisfac x1 x2 . t 1 1 1 + + + 1. atunci n 1 + x1 n 1 + x2 n 1 + xn Soluie. Ca mai sus, problema se reduce la a demonstra ca pentru orice t n Q 1 1 xn < n astfel nct x1 +x2 + +xn = 1 are loc inegalitatea n1 i=1 xi Cu substituia bi = 1 (n 1) xi problema de demonstrat se reduce la t Conform inegalitaii mediilor, t n n n Y YX Yh (1 bi ) = bj (n 1)
i=1 i=1 j6=i i=1

Rmne, deci, s demonstrm c pentru orice a1 , a2 , . . . , an > 0 cu a1 +a2 + a a a a n n P 1 P 1 are loc inegalitatea n+1 , care este echi 1 ak n+1 k=1 k=1 ak n n P P 1 (n 1) ak ak . ak k=1 1 (n 1) ak k=1 O alta substituie este acum necesara, anume 1 (n 1)ak = bk , k t observ c avem de asemenea b1 + b2 + + bn = 1 i rmne deci s a a s a n n P 1 bk bk 2 P c a (n 1) , rezultat care se poate obine prin su t bk k=1 k=1 1 bk P bk bk 1 bk =P . inegaliti de forma at 2 1 bk bi (n 1) i6=k bi i6=k

(1 b1 ) (1 b2 ) . . . (1 bn ) (n 1) b1 b2 . . . bn . sY
j6=i

n1

i n bj = (n 1) b1 b2 .

ceea ce trebuia demonstrat. n ncheiere, in s mulumesc prof. Marian Tetiva pentru lectura aces t a t i observaiile preioase facute, care au contribuit la mbunatairea formei s t t t

ERRATA

n articolul "Combinatorica . . . algebrica" de Gabriel Dospinescu, p nr. 2/2003 al revistei, forma corecta a Lemei de la pag. 19 este: 2 2 Fie n numar natural prim si = cos + i sin . Are loc egalitatea n n a0 + a1 + + an1 n1 = 0, a0 , a1 , . . . , an1 Q,

daca si numai daca a0 = a1 = = an1 . Diferenele ce apar ne-au fost semnalate de prof. Sergiu Romacu, Va t s tea nu afecteaz cu nimic restul articolului. De aceast neglijena se fac a a t redacia i nu autorul articolului. t s

Cteva proprieti ale medianelor at


Temistocle BRSAN 1

1. Ne propunem s indicm un numr de proprieti ale medianelor a a a at de nelegere al unui elev bun de gimnaziu. Instrumentele principale de l t teorema lui Thales i teorema lui Menelaus. s Pentru o exprimare scurt, vom folosi noiunile de puncte izotomice a t conjugate armonic. Fie A, B, X, Y patru puncte coliniare. Punctele X izotomice n raport cu A i B daca sunt simetrice faa de mijlocul segmen s t (evident, X i Y sunt fie interioare, fie exterioare acestui segment); aceast s revine la egalitatea AX = BY . Punctele X i Y sunt conjugate armonic s XA YA cu A i B daca mpart segmentul [AB] n rapoarte egale: s = (e XB YB este interior segmentului i Y exterior sau invers). s

Propoziia 1. Fie ABC un triunghi oarecare, A0 , B 0 , C 0 mijloacel t [BC], [CA], respectiv [AB] si M, N BC doua puncte izotomice n rapo furile B si C, cu M diferit de mijlocul segmentului [A0 C]. Atunci, sunt armaiile: t 1 dreptele B 0 M si C 0 N se intersecteaza ntr-un punct X AA0 ; 2 paralela prin M la AC si paralela prin N la AB se intersecteaza nt X 0 AA0 ; 3 punctele X si X 0 sunt conjugate armonic n raport cu A si A0 . Demonstraie. Avem patru situaii distincte ilustrate n figurile d t t demonstraia este nsa aceeai. t s
X A C X B M N B A C M B C X A C A B N B M A X N C B N C A X B C

Deoarece M nu-i mijlocul lui [A0 C] rezulta ca N nu-i mijlocul lui ca urmare, dreptele B 0 M i C 0 N intersecteaz AA0 . Fie {X} = AA0 s a {X1 } = AA0 C 0 N . Conform teoremei lui Menelaus, aplicat la 4AA0 C a s sala B 0 M i la 4AA0 B i C 0 N , avem s s X1 A N A0 C 0 B XA M A0 B 0 C 0 = 1 i s = 1, 0 XA M C B A X1 A0 N B C 0 A
1

Prof. dr., Catedra de matematic, Univ. Tehnic "Gh. Asachi", Iai a a s

XA MC NB X1 A i = = . Punctele M , N fiind izotomice n s XA0 M A0 X1 A0 N A0 X1 A XA B i C, avem M A0 = N A0 i M C = N B. Rezult c s = , de s a a XA0 X1 A0 t X1 coincide cu X. Afirmaia 1 este dovedita. 0 Fie X 0 , X1 interseciile dreptei AA0 cu paralela prin M la AC, respect t X 0A = prin N la AB. Conform teoremei lui Thales, au loc relaiile: t X 0 A0 0 NB X1 A = . Din acestea i din izotomia punctelor M i N faa d s s t 0 X1 A0 N A0 0 deducem egalitatea rapoartelor din membrii din stnga. Ca urmare, X1 c 0 X , deci 2 este demonstrata. MC MC X 0A XA i 0 0 = = , deci X i X 0 m s s Mai sus s-a aratat ca 0 0 XA 0 MA XA M A0 n acelai raport, adic X, X sunt conjugate armonic faa de A, A0 . Afi s a t deci i propoziia, este demonstrat. s t a de unde

Observaie. Sa urmarim deplasarea punctului X pe AA0 , atunci cn t curge BC. Notm cu G centrul de greutate al triunghiului ABC i cu A a s medianei [AA0 ]. Dac M este n C, atunci X coincide cu A. Dac M se nd a a de C, la "dreapta" acestuia, atunci X (AA ). Daca M se apropie "stnga" acestuia, atunci X (A G). Pentru M situat n B, X coincide c M (BA0 ], atunci X (GA0 ]; punctul M n poziia A0 coincide cu X. t ntre A0 i mijlocul segmentului [A0 C], X parcurge semidreapta de origin s conine A. n sfrit, daca M este ntre mijlocul lui [A0 C] i C, atunci t s s este situat pe semidreapta de origine A ce nu conine A0 i se apropie de t s

O poziie particular interesant a punctului M este semnalat n urm t a a a

Corolar. Daca sunt ndeplinite condiiile din Propoziia 1 si n plu t t a = N B = , atunci X este izotomicul punctului G n raport cu A si A0 , i 2 0 simetricul lui A faa de A. t XA MC a/2 s = = Demonstraie. Avem X [AA0 ] i t = XA0 M A0 a/2 + a/2 1 1 s a XA0 = 2XA sau XA = AA0 . Cum avem i GA0 = AA0 , rezult XA 3 3 0 XA s prima afirmaie este dovedita. Pe de alta parte, X 0 [AA0 ] i 0 0 = t XA s deci 2X 0 A = X 0 A0 , de unde X 0 A = AA0 . Aadar X 0 i A0 sunt simetrice f s

2. Aplicaii. Fie D, Da , Db , Dc punctele de tangena a cercurilor t t exnscris, B-exnscris respectiv C-exnscris triunghiului ABC cu dreapta B tim c au loc egalitile [1], p.30: a at

t s primele spun c punctele D i Da sunt izotomice faa de B i C, iar u a s aceast proprietate o au i punctele Db i Dc . Putem presupune c A a s a s pentru a evita cazul trivial n care 4ABC ar fi isoscel cu vrful A.

DB = Da C = p b i Db C = Dc B = p a (2p = a + b + c) s

s adic are loc 3 . n sfrit, avnd n vedere Corolarul, X este izotomicul a a a i numai daca Db C = , adica p a = sau 2a = b + c. s 2 2 Observaie. Triunghiurile ce satisfac condiia t t A 2a = b+c (o latur este media aritmetic a celorlaltor a a doua) sunt speciale, cu multe proprietai cunoscute t X (n [2], p. 242, sunt date opt proprieti). Armaia at t C Y 4 indic o nou proprietate caracteristic lor. a a a U Un rezultat similar se obine dac lum punctele t a a Dc B D A izotomice D, Da n locul punctelor Db , Dc .

Propoziia 2. Relativ la punctele Db , Dc , sunt adevarate afirmaiile: t t 1 B 0 Db si C 0 Dc se intersecteaza ntr-un punct X (AA ); 2 B 0 Dc si C 0 Db se intersecteaza ntr-un punct Y (A G); YA XA + = 2; 3 XA0 Y A0 4 X este izotomicul lui G n raport cu A si A0 daca si numai daca 2 Demonstraie. 1 i 2 decurg direct din Propoziia 1 i observaia d t t s t s De asemenea, avem YA Db C Dc C pa p 2p a XA + = + = + = XA0 Y A0 Db A0 Dc A0 (p a) + a/2 p a/2 p a/2

Propoziia 3. Daca 4ABC satisface condiia t t a |b c| 6= , atunci relativ la punctele D, Da avem: 2 1 B 0 D si C 0 Da se intersecteaza n U AA0 ; 2 B 0 Da si C 0 D se intersecteaza n V AA0 ; VA UA 3 = 2 (+ n cazul b > c si n cazul b < c). U A0 V A0 Omitem demonstraia, ce urmeaza pe cea din Propoziia 2, dar men t t a a prin condiia |b c| 6= se evit ca D sau Da s fie mijlocul segmentului t a 2 Observaie. Punctele X 0 , Y 0 , U 0 , V 0 conjugatele armonic ale punct t U , V n raport cu A i A0 sunt alte patru puncte pe dreapta AA0 care po s conexiune cu punctele de tangena D, Da , Db , Dc , aa cum se indica n Pr t s Rmne n seama cititorului examinarea lor. a Bibliografie

1. T. Lalescu - Geometria triunghiului, Ed. Tineretului, Bucureti, 1958. s 2. V. Gh. Vod - Vraja geometriei demodate, Ed. Albatros, Bucureti, 198 a s

O construcie geometric a mediilor (II) t a


Claudiu - Stefan POPA1

n [2] am prezentat o construcie geometric a mediilor armonic, g t a a aritmetica, patratica i ponderata a lungimilor bazelor unui trapez, ca se s capetele pe laturile neparalele ale trapezului i paralele cu bazele lui. Ne s n continuare dezvoltarea acestor idei, fapt ce va conduce la o serie de c cu interesante aplicaii geometrice. t

Demonstraie. Deoarece AA0 k CC 0 i t s D D a a AA0 = CC 0 , rezult c AC 0 CA0 este paralelogram, deci AC 0 k A0 C, AC 0 = A0 C. Analog, K BD0 k B 0 D, AD k A0 D0 i BC k B 0 C 0 , cu egalis K E F tile de segmente corespunztoare. Unghiurile at a \ s \ s D0 A0 E i DAF au laturile respectiv paralele i vor \ \ fi congruente; la fel, A0 D0 E ADF . Avem nc a 0 0 A D = AD, prin urmare 4A0 D0 E 4ADF , A A deci A0 E = AF . nsa AE 0 k AF i atunci AF EA0 s este paralelogram. Rezulta ca EF k AA0 i EF = AA0 , primele doua a s concluziei. Fie {K 0 } = EF A0 D0 . Deoarece A0 C i BD0 sunt diagonale n trapezu s iar K 0 L este paralela la baze prin punctul de intersecie a diagonalelor t t urmeaza ca EL = EK 0 . nsa KK 0 = AA0 + EF , deci EL EF = EK 0 K F L = KE, ceea ce ncheie demonstraia. t

Propoziie. Fie ABCD un trapez cu AB k CD si punctele A0 , B 0 , C t nct A (A0 B), B (AB 0 ), C (C 0 D), D (CD0 ), iar AA0 = BB 0 = CC Notam {E} = A0 C BD0 , {F } = AC 0 B 0 D, {K} = EF AD, {L} = n aceste condiii, EF k AB, EF = AA0 , iar KE = F L. t

Observaie. Concluzia se pstreaz, cu demonstraie asemntoare, t a a t a a care punctele A0 , B 0 , C 0 , D0 se afl pe semidreptele [AB, [BA, [CD, resp a

n cele ce urmeaz vom folosi notaiile: AB = a, CD = b, AA0 = x. Stu a t blema variaiei lungimii segmentului [KL] funcie de x; vom gndi lungime t t tului [AA0 ] ca ind pozitiv n cazul n care A (A0 B) i negativ pentru a s a 2ab Pentru x = 0, avem KL = (= mh ), iar pentru x = ab (= m a+b KL = mg , dupa cum s-a demonstrat n [2]. Desennd figurile pentru c ale lui x > 0, observam ca segmentul [KL] "coboara" pe masura ce x cre "atinge" ns linia mijlocie a trapezului. n momentul n care vom demons a acest lucru, vom avea o (probabil) nou demonstraie pentru inegalitatea a t doua numere reale pozitive. n trapezul A0 BCD0 , segmentul [K 0 L] are ca lungime media armonica 2 (a + x) (b + x) 0 0 0 s . A B = a + x i CD = b + x, deci K L = a + b + 2x
1

Profesor, S coala "Alecu Russo", Iai s

Atunci, avem KL = K 0 LKK 0 =

ma x + m2 2(a + x)(b + x) x(a + b) + 2ab g = x = = a + b + ax 2x + (a + b) x + ma Suntem astfel condui la studiul funciei s t x + mh y . f : R\ {ma } R, f (x) = ma x + ma Aceasta este o funcie omograca, strict t ma s crescatoare pe (, ma ) i pe (ma , +), mg al crei grafic este o hiperbol de asimptote a a mg mh ma y = ma i x = ma . Prin calcul, stabilim c a s mh O m valorile x = mg sunt puncte fixe ale funciei. t g Deoarece f ([0, )) = [mh , ma ), iar restricia t mg lui f la [0, ) este strict cresctoare, justia carea armaiilor anterioare este complet. t a Consideraiile precedente conduc la urmt a toarele interpretari geometrice: 1. Faptul ca f (mh ) = 0 arata ca, daca ABCD este trapez cu A A0 [AB, D0 [DC sunt astfel nct AA0 = DD0 = mh , iar {E} = A atunci paralela prin E la bazele trapezului dat trece prin punctul de con prelungirilor laturilor neparalele ale acestuia. 2. Faptul ca f (mg ) = mg se interpreteaza geometric astfel: daca A un trapez cu AB k CD, iar A0 [AB, D0 [DC sunt astfel nct AA0 = D atunci A0 C, BD0 i AD sunt concurente ntr-un punct E, iar paralela prin s intersecteaz BC n F astfel nct EF este media geometric a lungimilo a a a + kb 3. n sfrit, sa observam ca pentru x = s , obinem f (x) t 1+k (calculul se efectueaza cu uurina). Prin urmare, considernd segmentu s t lungime egal cu media ponderat a bazelor cu ponderile 1 i k, segmentu a a s reprezenta media ponderat a bazelor, cu ponderile 1 i k. Acest fapt n a s sa construim cu rigla i compasul conjugatul armonic al unui punct; citit s dezvolta singur ideile. Bibliografie

1. L. Constantinescu - O interpretare geometrica a inegalitatii mediilor, 1/1982, 30. 2. C. - St. Popa - O construcie geometrica a unor medii, Rec. Mat. - 2/200 t

O generalizare a teoremelor de baz a ale calculului diferenial t


Florin POPOVICI 1

Nota de faa i propune sa extinda teoremele de baza ale calculului difer t s impunerea condiiei de derivabilitate bilateral n locul condiiei clasice t a t bilitate. Rezultatele obinute se exprim sub forma unor condiii de ap t a t care pot fi, nsa, interpretate geometric. Teorema 1 (Teorema lui Fermat generalizata ). Fie f : [a, b] R o fu Daca c (a, b) este un punct de extrem local al funciei f si aceasta este t bilateral (la stnga si dreapta) n punctul c, atunci la 0 0 0 0 0 min f (c) , f+ (c) , max f (c) , f+ (c) . Demonstraie. Fie c punct de maxim local. Aadar, > 0 astfel n t s f (x) f (c) 0 x (c , c) [a, b] f (x) f (c) 0 f (c) xc f (x) f (c) 0 x (c, c + ) [a, b] f (x) f (c) 0 f+ (c) xc 0 0 i, ca urmare, 0 f+ (c) , f (c) , adica are loc (1). s Exemplu. Funcia f (x) = max x, 1 x2 , x 0, este t y 51 , pentru care derivabil cu excepia punctului x0 = a t 2 0 f (x0 ) = 1 5 i f+ (x0 ) = 1. Relaia (1) revine la 0 s 0 t 1 5, 1 , adic semitangentele la grafic n M0 (x0 , x0 ) sunt a x0 de pari diferite faa de paralela prin acest punct la axa Ox. t t Teorema 2 (Teorema lui Rolle generalizata ). Daca f : [a, b] R este continua pe [a, b], derivabila bilateral pe (a, b) si f (a) = f (b), atunci exist c (a, b) astfel nct are loc relaia (1). t Demonstraie. Conform teoremei lui Weierstrass, funcia f este mrg t t a atinge marginile, adica c1 , c2 [a, b] astfel nct f (c1 ) f (x) f (c2 ), Daca {c1 , c2 } {a, b}, atunci rezulta ca f este funcie constanta i ( t s pentru orice c (a, b). Dac {c1 , c2 } 6 {a, b}, fie c {c1 , c2 } \ {a, b} a Teoremei 1 pentru acest punct c are loc (1). Teorema 3 (Teorema lui Lagrange generalizata ). Daca f : [a, b] o funcie continua pe [a, b] si derivabila bilateral pe (a, b), atunci exista t c (a, b) astfel nct 0 0 f (b) f (a) 0 0 min f (c) , f+ (c) , max f (c) , f+ (c) . ba Demonstraie. Se aplica Teorema 2 funciei auxiliare g : [a, b] R de t t f (b) f (a) g (x) = f (x) x, x [a, b]. ba Corolarul 1 (Teorema lui Lagrange pentru funcii convexe). Fie f : t interval deschis) o funcie convexa. Atunci, pentru orice puncte a, b t exista c (a, b) astfel nct
1

Profesor, Liceul Teoretic "N. Titulescu", Braov s

f (b) f (a) 0 f+ (c) . ba Demonstraie. Deoarece f este convex pe I (deschis), rezult c f est t a a a 0 pe I, derivabil bilateral pe I i f (x) f+ (x), x I. Prin aplicarea T a s 0 pe intervalul [a, b], obinem rezultatul cerut. t Corolarul 2. Fie f : I R ( I interval deschis) o funcie continua t bila bilateral pe I. Atunci funcia f este crescatoare pe I daca si numai t satisfacuta condiia t 0 0 min f (x) , f+ (x) 0, x I. Demonstraie. Necesitatea condiiei este uor de dovedit. Pentru suf t t s x1 , x2 I cu x1 < x2 . Conform Teoremei 3, aplicat restriciei funciei f a t t 0 f (x2 ) f (x1 ) 0 min f (c) , f+ (c) c (x1 , x2 ) astfel nct x2 x1 De aici i din (3), deducem c f (x1 ) f (x2 ); ca urmare, funcia f este cr s a t Teorema 4 (Teorema lui Cauchy generalizata ). Daca f, g : [a, b] R funcii continue pe [a, b], derivabile bilateral pe (a, b) si t 0 0 0 0 0 min g (x) , g+ (x) , max g (x) , g+ (x) , x (a, b) , / atunci g (a) 6= g (b) si exista un punct c (a, b) astfel nct 0 0 0 0 f (c) f+ (c) f (c) f+ (c) f (b) f (a) min , 0 , max , 0 . 0 0 g (b) g (a) g (c) g+ (c) g (c) g+ (c) Demonstraie. Daca am avea g (a) = g (b), atunci, t conform Teoremei 2 0 0 0 0 un punct c (a, b) astfel nct 0 min g (c) , g+ (c) , max g (c) , g+ ce contrazice (4). Deci are loc g (a) 6= g (b). Considerm acum funcia h : [a, b] R definit prin a t a f (b) f (a) h (x) = f (x) g (x) , x [a, b] . g (b) g (a) Observam ca h este continua pe [a, b], derivabila bilateral pe (a, b f (a) g (b) f (b) g (a) h (a) = = h (b). Conform Teoremei 2, exista c ( (b) 0 g 0 g (a) 0 nct 0 min h (x) , h+ (x) , max h (x) , h0 (x) , adica avem + f (b) f (a) 0 f (b) f (a) 0 0 0 g (c) 0 f+ (c) g (c) f (c) g (b) g (a) g (b) g (a) + sau f (b) f (a) 0 f (b) f (a) 0 0 0 g (c) 0 f (c) g (c) . f+ (c) g (b) g (a) + g (b) g (a) Presupunem ca are loc (6) (se procedeaza analog, daca ar avea loc (7)) ipotezei (4), putem scrie 0 0 0 0 sau max g (c) , g+ (c) < 0 min g (c) , g+ (c) > 0 (8) Dac are loc (8), atunci (6) ia forma a 0 0 f (c) f+ (c) f (b) f (a) 0 (c) g (b) g (a) g 0 (c) , g + deci (5) este adevarata. Daca are loc (9), atunci (6) se scrie 0 f 0 (c) f+ (c) f (b) f (a) , 0 0 g+ (c) g (b) g (a) g (c) deci (5) este adevrat i n acest caz. Demonstraia este complet. a as t a
0 f (c)

Asupra unei inegaliti at


Alexandru NEGRESCU 1

La a V-a ediie a Concursului interjudeean de matematica "Radu Miro t t brie 2003, elevilor clasei a IX-a li s-a propus urmtoarea problem: a a 1 1 Fie x1 , x2 , . . . , xn (2, +) astfel nct + + + x1 1 x2 1 xn n Demonstrai ca x1 x2 . . . xn (n + 1) . t Vom da 5 demonstraii acestei inegaliti. t at

Dar, conform cu inegalitatea dintre mediile armonica i geometrica, avem s p n n (x1 1) (x2 1) (xn 1 1 1 1 + + + x1 1 x2 1 xn 1 sau, innd seama de condiia din enun, t t t p n n (x1 1) (x2 1) (xn 1). Din (1) i (2) rezulta ca x1 x2 . . . xn (1 + n)n , q.e.d. s Soluia II. Scriem t

Soluia I. Notam xi 1 = ai , i = 1, n; deci ai > 1. Aplicam inega t Huygens: n (1 + a1 ) (1 + a2 ) (1 + an ) (1 + n a1 a2 an ) , a1 , a2 , . . . , an Obinem t n p x1 x2 . . . xn 1 + n (x1 1) (x2 1) (xn 1) .

s x xi 1 xi 1 xi 1 n+1 xi = 1 + (xi 1) = 1 + + + + (n + 1) n n n Ca urmare, r n n n n n+1 (x1 1) (x2 1) (xn 1) x1 x2 . . . xn (n + 1) nn2 n Din (2), avem (x1 1) (x2 1) (xn 1) n , deci r n n n n+1 (x1 1) (x2 1) (xn 1) 1. 2 nn Combinnd (3) i (4), obinem inegalitatea ceruta. s t 1 Soluia III. Notm t a s = yi , i = 1, n; deci yi (0, 1) i y1 + y2 + xi 1 yi + 1 Rezult c xi = a a , 1, n i avem: s yi y1 + 1 y2 + 1 yn + 1 ... = x1 x2 . . . xn = y1 y2 yn
1

Elev, cl. a IX-a, Colegiul Na ional "A. T. Laurian", Botoani t s

2y1 + y2 + + yn y1 + 2y2 + + yn y1 + y2 + + 2y ... y1 y2 yn q n n+1 (n + 1) n+1 (y1 y2 yn ) n = (n + 1) . y1 y2 yn 1 Soluia IV. Apelam la metoda lui Sturm. Deoarece xi = 1 + t xi 1 i = 1, n, inegalitatea de demonstrat se scrie "n # Y n Y 1 1 1+ (n + 1)n . xi 1 xi 1 i=1 i=1 =

1+a 1+b Sa analizam comportarea produsului , cu a, b > 0 i a + s a b stant sububitar, atunci cnd a i b "se apropie". Presupunem a < b a a s s numerele a i b cu a + t i respectiv b t, unde 0 < t < b a. Atunci s s t (1 + a + b) (a b + t) 1+a+t 1+bt 1+a 1+b = < a+t bt a b ab (a + t) (b t)

1+a 1+b ceea ce arat c apropiind numerele a i b produsul a a s descret s a b 1 1 1 Daca printre numerele , , ... , exista doua ineg x1 1 x2 1 xn 1 1 1 i celalalt este strict mai mare ca ; fie unul este strict mai mic ca s n n x 1 1 1 1 1 1 i i s > . nlocuim s prin i respectiv s + x2 1 n x1 1 x2 1 n x1 1 x 1 1 1 1 1 1 Suma numerelor , + , , ... , rmn a n x1 1 x2 1 n x3 1 xn 1 dar membrul stng n (5), obinut prin aceast nlocuire, este mai mic. t a 1 de numere avem unul egal cu , iar, daca printre celelalte exista doua n 1 procedeaza la fel pna cnd se obine un set de numere egale cu . n t n membrul stng are valoare minim, anume, a n 1 .1 n = (n + 1) . 1+ n n Soluia V. Vom dovedi mai nti rezultatul urmator: t Lem. Daca ai , bi > 0, i = 1, n, atunci a v v v un un un uY uY uY n n n t (ai + bi ) t ai + t bi .
i=1 i=1 i=1

Demonstraie. Inegalitatea se poate scrie astfel: t r r an bn a1 b1 n 1 ... + n ... a1 + b1 an + bn a1 + b1 an + bn

i rezult aplicnd inegalitatea mediilor: s a b1 a1 1 an bn 1= + + + + + n a1 + b1 an + bn a1 + b1 an + bn r r an bn a1 b1 n ... + n ... . a1 + b1 an + bn a1 + b1 an + bn Lund n (6) ai = xi 1 i bi = 1, i = 1, n, obinem s t (2) p n x1 x2 . . . xn n (x1 1) (x2 2) (xn 1) + 1 n + 1 de unde rezult inegalitatea dorit. a a

Observaie. Aceasta problema poate fi uor generalizata astfel: t s 1 1 Fie x1 , x2 , . . . , xn (2, +) astfel nct + + + x2 1 xn n x1 1 k+n Demonstrai ca x1 x2 . . . xn t . k Bibliografie

1. Gh. Andrei i colab. - Exerciii si probleme de algebra pentru concursu s t piade scolare, Partea I, Constana, 1990. t 2. M. Ganga - Manual pentru clasa a IX-a, Profil M1,M2, Ed. Mathpres 2003. 3. M. Ganga - Probleme elementare de matematica, v. II, Ed. Mathpres 2003. 4. D. St. Marinescu, V. Cornea - Doua inegalitati si unele aplicaii ale t Gazeta Matematic, CVI (2001), nr. 3, 102-104. a 5. I. Nedelcu - Probleme de matematica pentru liceu, Ed. Mathpress, Ploie 6. L. Panaitopol, M. Lascu, V. Bndil - Inegalitati, Ed. GIL, Zalau, 1 a a

1 (Problema de cntrire a lui Bachet). Care este cel mai mic a greutai care pot fi folosite pentru a cntari cu o balana ori numar ntreg de t t de la 1 la 40?

2. Gsii dou numere care se scriu n baza 10 numai cu ajutorul cifre a t a au suma egala cu produsul lor.

3. Un urs pleac din brlogul su 1 km spre sud, se ntoarce i parc a a s spre est, apoi 1 km spre nord, revenind astfel n punctul de plecare. Ce c ursul?
Not. Rspunsurile la aceste probleme se gsesc la p. 110 i la p. 123 a a a s

Asupra problemei VII.41 din RecMat - 2/2

n nr. 2/2003 al revistei Recreaii matematice este publicata urmatoarea t propus de elevul Alexandru Negrescu din Botoani: a s VII.41. Rezolvai n N2 ecuaia t t

a b + = 1. b+1 a+1 Aceast problem a fost apoi propus elevilor de cl. a VII-a n cadrul C a a a "Recreaii matematice", ediia a III-a, 2003, Iai. t t s Aceste mprejurari, ct i accesibilitatea problemei, au facut ca ace s bucure de atenia elevilor. Ca rezultat, au fost date mai multe solui t t sau variante ale lor. Nota de faa colecteaz aceste soluii. Cititorul v t a t entuziasmul i pasiunea cu care elevii au atacat problema VII.41. s

Soluia I (Alexandra Ciofu, eleva, Hrlau). Daca perechea (a, b) e t a b a ecuaiei, atunci t 1 i s 1, de unde a b + 1 i b a + 1. s b+1 a+1 I Daca a b, rezulta ca 0 a b 1, deci a b {0, 1}. n cazul 2a = 1. De aici, obin t urmeaz a = b i, deoarece (a, b) este soluie, a s t a+1 i, deci, perechea (1, 1) va fi soluie a ecuaiei date. n cazul a b = 1, n s t t b = 1, deci b = 0. Aadar, n s ecuaia dat pe a cu b + 1 obinem 1 + t a t b+2 obinem soluia (1, 0). t t II Dac a b, rolurile numerelor a i b se schimb i (a, b) va fi (1, 1) a s as Rezumnd, mulimea soluiilor ecuaiei date este {(0, 1) , (1, 0) , (1, 1)}. t t t

Soluia II (Maria Crciun, eleva, Hunedoara). Ecuaia data este e t a t cu a (a + 1) + b (b + 1) = (a + 1) (b + 1), deci cu a2 ab + b2 = 1. Aceasta poate fi scris sub forma a (a b) b (a b) = 1 ab sau (a b)2 = 1 a a numarul 1 ab este un patrat perfect cel mult egal cu 1, daca perechea soluie a ecuaiei. Ca urmare, 1 ab {0, 1}, adic ab {0, 1}. t t a Dac ab = 1, atunci perechea (a, b) va fi (1, 1) i aceasta verific ecua a s a t Dac ab = 0, atunci a = 0 sau b = 0. Dac a = 0, nlocuind n ecua a a t b = 1; perechea (0, 1) verifica ecuaia din enun. Daca b = 0, obine t t t asemntor soluia (1, 0). Dac a = 0 i b = 0, verificm c perechea a a t a s a a soluie. t n concluzie, mulimea soluiilor ecuaiei este {(0, 1) , (1, 0) , (1, 1)}. t t t

Soluia III (Bogdan-Alexandru Burican, elev, Hrlau). Ecuaia e t t 2 2 2 2 2 lent cu a ab + b = 1 i apoi cu a + b + (a b) = 2. Rezult c a2 a s a a a {0, 1}. Daca a = 0, din ecuaia data obinem b = 1. t t Dac a = 1, din relaia a2 ab + b2 = 1 obinem b2 b = 0, de unde a t t b = 1. Prin urmare, perechile (0, 1), (1, 0) i (1, 1) sunt soluiile ecuaiei date. s t t

Soluia IV (Diana Prodan, eleva, Iai). Ecuaia a2 ab + b2 = 1, e t t s

cu ecuaia din enunul problemei, se pune n forma 3a2 + (a 2b) = 4 t t avem 3a2 4, deci a {0, 1}. Se continua ca n Soluia III. t

Soluia V (Adrian Hamciuc, elev, Iai). Ca mai sus, obine t s t a2 ab + b2 = 1. Cum a2 + b2 2ab, rezulta ca ab 1, adica ab ncheie ca n Soluia II. t

Soluia VI (Diana Timofte, eleva, Iai). Fie (a, b) o soluie a t ecuaiei t s t i a ecuaiei a2 ab + b2 = 1. Ca urmare, ecuaia b2 ab a2 1 = 0, c s t t n b, are soluii reale i atunci = a2 4 a2 1 0. Rezulta ca 3a2 t s a {0, 1}. Se continu ca n Soluia III. a t

Soluia VII (Alexandru Negrescu, elev, Botoani). Conform i t s Cauchy-Buniakovski-Schwarz, avem 2 a b [(b + 1) + (a + 1)] a+ b , + b+1 a+1 2 a + b (b + 1) + (a + 1), adica a + b + 2 ab a + b + 2 sau de unde De aici ab {0, 1} etc.

Observaie. Soluiile prezentate mai sus au comun faptul ca n prim t t obine, cu tehnici diverse de calcul, una dintre relaiile a {0, 1}, a t t ab {0, 1}. n partea a doua sunt utilizate aceste relaii (una dintre e t determinarea mulimii soluiilor ecuaiei. t t t

Soluia Problemei de cntarire a lui Bachet (p. 108) t

Dac greutile se pun pe un singur taler, sunt necesare ase greuti: a at s at 4 kg, 8 kg, 16 kg, 32 kg. ntr-adevar, orice greutate de la 1 la 40 poat astfel: 1 = 1, 6 = 4 + 2, 2 = 2, ............... 3 = 2 + 1, . . . . . . . . . . . . . . . 4 = 4, 39 = 32 + 4 + 2 + 1, 5 = 4 + 1, 40 = 32 + 8. Daca greutaile pot fi puse pe ambele talere, sunt necesare numai patr t 1 kg, 3 kg, 9 kg, 27 kg. ntr-adevar, avem: 1 = 1, 6 = 9 3, 2 = 3 1, ............... 3 = 3, ............... 4 = 3 + 1, 39 = 27 + 9 + 3, 5 = 9 3 1, 40 = 27 + 9 + 3 + 1.

Asupra unei probleme de concurs


Dumitru MIHALACHE, Marian TETIVA1

ntr-o prima faza, negasind nici o alta idee, am ncercat o rezolvare a bazata pe calcule trigonometrice. Rezultatul acestei cautari este Metoda I de rezolvare a problemei. Din \ \ D ipotez rezult c m(DBA) = m(DBC) = a a a \ = 60 si, apoi, ca m(ADB) = 40 , iar \ m(BDC) = 70 . S considerm punctele a a M AB (A (M B)) i N (BC) asts fel nct triunghiurile M BD i BN D sa fie s P \ echilaterale (prin urmare m(M DA) = 20 i s \ m(N DC) = 10 ). Fie a lungimea laturilor M A B acestor triunghiuri. Cu teorema sinusurilor n triunghiul M AD obinem t MA a a sin 20 a sin 20 = MA = = , sin 20 sin 100 sin 100 sin 80 iar din triunghiul N CD, a a sin 10 NC = NC = . sin 10 sin 50 sin 50 Atunci 2 sin 30 cos 50 cos sin 80 sin 20 =a =a AB = M B M A = a sin 80 sin 80 sin i s cos 20 sin 50 + sin 10 =a , BC = BN + N C = a sin 50 sin 50 prin urmare 2 cos 20 sin 80 sin 40 BC cos 20 sin 80 = = 2 cos 20 = = AB sin 50 cos 50 sin 100 sin 20 (este evident c am aranjat acest raport, tiind unde vrem s ajungem) a s a
1

n aceast not ne propunem s prezentm dou modaliti de abord a a a a a at probleme de geometrie i s obinem o generalizare a sa (rostul ghilime s a t vedea la vremea potrivita). Problema a fost propusa de C. Apostol la Naional de Matematica Laureniu Duican (ajuns, iata, la cea de a X t t felicitri i succes n continuare!) i enunul ei poate fi citit n [1]; iat a a s s t a (modificat pentru a reine doar esenialul): t t b b Problema 1. n patrulaterul convex ABCD avem m(A) = 80 , m(B b b = 50 , m(D) = 110 . Sa se arate ca, daca (BD este bisectoarea un m(C) b atunci (AC este bisectoarea unghiului A.

Profesori, Colegiul Na ional "Gh. Roca Codreanu", Brlad t s

\ \ Acum, fie x = m(BAC), deci m(BCA) = 60 x; tot cu teorema (acum n triunghiul ABC) i innd seama de calculul anterior, avem s t sin x sin 40 sin x sin 20 = sin (60 x) sin 40 = x) sin (60 sin 20 transformam produsele n sume i avem s deci cos (x 20 ) cos (x + 20 ) = cos (20 x) cos (100 x) ,

cos (100 x) cos (x + 20 ) = 0 sin 60 sin (40 x) = 0 Cum, evident, 0 < x < 80 , de aici rezulta x = 40 i problema este rezo s

Desigur, asemenea calcule nu sunt pentru cl. VII-a (iar problema a fo elevilor acestei clase); s vedem aadar i o soluie a problemei la acest ni a s s t

Metoda a II-a se bazeaz pe observaia urmtoare: dac P (BC) es a t a a \ = 50 , atunci triunghiul P DC este isoscel cu P pentru care m(P DC) \ (evident), iar triunghiul ADP este echilateral. ntr-adevr, unghiul BP D a imediat, are masura de 100 , de aceea patrulaterul ABP D este inscripti \ \ dou unghiuri opuse suplementare). Atunci m(AP D) = m(DBA) = 60 , a \) = m(ADC) m(P DC) = 110 50 = 60 , deci \ \ alt parte, m(ADP a ADP are dou unghiuri de msur 60 . a a a Rezult c avem AP = DP = P C i triunghiul P AC este tot isoscel; p a a s [ 180 m(AP C) [ [ m(P AC) = m(P CA) = = 20 , 2 \ de unde rezult imediat m(BAC) = 40 i demonstraia se ncheie. a s t Fr ndoial, aceast a doua variant de soluionare a problemei este aa a a a t simpl dect prima; totui, ideea de a alege punctul P nu vine prea uor. a s s rolul sau n problema este fundamental; de fapt, noi credem ca proble construita pornind de la cele doua triunghiuri isoscele lipite, ADP i s triunghiul ADP este chiar echilateral nu este esenial i de asta v pute t s a t rezolvnd urmtoarea generalizare a primei probleme. a b b Problema 10 . Fie ABCD un patrulater n care m(A) = + , m(B) b = , m(D) = + , m(DBC) = , unde , , > 0 si 2 + + b \ m(C) Atunci: a) 90 > > . b b) (AC este bisectoarea unghiului A. b > m(DBC), avem 2+ > i 2 < 2+ \ Soluie. a) Deoarece m(B) t s \ \ < 90 . Apoi, m(ABD) = 2+ i m(ADB) = 180 (2 + + s = , deci > . b) Alegem un punct P (BC) astfel nct \ \ m(P DC) = m(P DB) = .

\ Acest punct exist pe segmentul (BC), deoarece m(BDC) = + . M a demonstraia decurge ca mai sus: observai ca triunghiul P CD este isosc t t

ABP D este patrulater inscriptibil i c triunghiul P AD este isoscel (cu m s a \ = ). Rezult triunghiul P AC isoscel, de unde se va put = m(P DA) a [ i apoi se ajunge la m(BAC) = + , ceea ce ncheie re \ masura lui P AC s 2 Nu e o generalizare efectiv, totul se bazeaz pe aceeai idee (Prob a a s a s regsete pentru = 50 , = 60 i = 20 ): iat de ce am pus cuv a s ghilimele. Iar pentru a ncheia lasam ca tema o problema nrudita cu cele d (eventual ncercai i o generalizare a ei). t s b b Problema 2. Fie ABC un triunghi cu m(B) = 110 si m(C) = 50 \ \ ram punctele M (AC) si N (AB) astfel nct m(M BC) = 70 si m(N \ ). (Rspuns: 60 .) Sa se calculeze m(AM N a Bibliografie

1. F. Diac - A XI-a ediie a Concursului Naional de Matematica Laureniu t t t Braov, 2003, G. M. 11/2003. s

LISTA MEMBRILOR FILIALEI IA I a S. S S


continuare din nr. 1/2000, 1/2001, 1/2002, 1/2003 i 1/2004 s 126. 127. 128. 129. 130. 131. 132. 133. 134. 135. 136. 137. 138. 139. 140. 141. 142. 143. 144. 145. 146. 147. DIMITRIU Gabriel RADUCANU Petru IUREA Gheorghe LAZAR Cristian PETCU Alina Emilia POPA Gabriel VATAMANUTA Laura NEDELCU Andrei S CARAU U Alexandru ROMAN Neculai CALIN Ionela S GOLAE Angelica BEJAN Tinua t BUZAC Gabriela - Tamara PADURARU Adriana LUCHIAN Dorel COZLAC Magda LUCA TUDORACHE Rodica POPA Antoaneta ARBONE Dorina IONESCU Mihaela SAVA Radu

I. M. F., Iai s Liceul "D. Cantemir", Iai s Liceul "D. Cantemir", Iai s Colegiul Naional, Iai t s Liceul Energetic, Iai s Colegiul Naional, Iai t s Scoala Waldorf, Iai s Liceul "Gr. Moisil", Iai s Univ. Tehnic "Gh. Asachi", I a Scoala "V. Alecsandri", Mirce s Liceul "D. Mangeron", Iai s Gr. c. ind. uoar "Victoria", s s a Scoala "Al. Vlahua", Iai t s Liceul Economic nr. 1, Iai s Scoala "B. P. Hasdeu", Iai s Liceul "M. Costin", Iai s Liceul ind. nr. 7, Iai s Univ. Tehnica "Gh. Asachi", I Scoala Mnzateti (Iai) s s Scoala "Mircea cel Btrn", Ia a Scoala "I. Ghica", Iai s Colegiul "C. Negruzzi", Iai s (continuare la p

Exponentul numrului natural a n produsu a


Mihai CRACIUN 1

n cele ce urmeaz, date ind a, n N , vom indica o formul de calc a a nentului lui a n n!, notat expn! (a), mpreun cu cteva aplicaii. a t n n n Teorema 1. Daca p este prim, atunci expn! (p) = + 2 + 3 p p p Demonstraie. Mai nti, se observ c suma din enun este finit i s t a a t as pn cnd termenul curent are partea ntreag egal cu 0. a a a n Dintre produsului n!, un numr de factorii a vor fi multipli de p. Di p n s numar de 2 vor fi multipli de p2 , iar dintre acetia din urma, un numa p 3 vor fi multipli ai lui p etc. Suma numerelor indicate va fi exponentul cerut fiecare factor al produsului n! care este multiplu al lui pm fr a fi i mul aa s pm+1 se socotete n modul indicat de m ori, ca multiplu al lui p, p2 , . . . , p s 50 50 50 50 Exemplul 1. exp50! (3) = + + + = 16 + 5 + 1 3 9 27 81 expn Teorema 2. Daca p este prim iar N , atunci expn! (p ) = Demonstraia este evident. t a 7 exp50! 3 22 = = 3. Exemplul 2. exp50! 3 = 7 7 Teorema 3 (Legendre). Daca a N este descompus n factor primi a = p1 p2 pk , atunci expn! (a) = min {expn! (p1 ) , expn! (p2 ) , . . . , ex 1 2 1 2 k Demonstraia este imediat. t a Exemplul 3. exp253! 2 exp exp253! (108) = min exp253! 22 , exp253! 33 = min , 2 s Deoarece exp253! 2 = 246 i exp253! 3 = 125, urmeaza ca exp253! (108) = 41 Problema 1. Determinai cu cte zerouri se termina 1958! t Soluie. Numrul de zerouri este dat de exp1958! (10). Se observ c t a a a Atunci

exp1958! (10) = min {exp1958! 2, exp1958! 5} = exp1958! (5) . 1958 1958 1958 1958 1958 + + + =4 + exp1958! (5) = 5 52 53 54 55 deci 1958! se termin cu 467 zerouri. a Problema 2. Demonstrai ca n! nu se divide cu 2n , n N. t Soluie. Fie n N i fie k N astfel ca 2k n < 2k+1 . Atunci t s hni h n i h n i n n n expn! (2) = + 2 + + k+1 + 2 + + k+1 = n 1 2 2 2 2 2 2 2k
1

Profesor, Liceul "M. Sadoveanu", Pacani s

5n 1 Problema 3. Determinai cu cte zerouri se termin t a !, und 4 5n 1 . Ca mai sus, numarul de zerouri e Soluie. Sa notam N = t 4 expN ! 5. n plus n n n 5 1 5 1 5 1 + + + = expN ! 5 = + 45 4 52 4 5k n n n 5 5k + 5k 5 5+51 5 52 + 52 1 + + = + 45 4 52 4 5k n1 n2 nn n 1 5 1 1 5 5 5 4n 1 = + + + = , 4 4 4 16 5k 1 5k 1 deoarece [0, 1). De aici, N ! se t N pentru k N , iar 4 4 5k n 5 4n 1 zerouri. 16 Problema 4. Factorialul caror numere se termina exact n 1000 zero Soluie. Fie n N un numar al carui factorialhse termina exact n 10 t h hni n i ni i fie k N astfel ca 5k n < 5k+1 . Atunci + 2 + + k+1 = s 5 5 5h i n n n n 1 n s + 2 + + k = 1 k > 1000 i n > 1000. n plus, < 5 5 5 4 5 5 n < 5005, ceea implic k = 5. Se observ c pentru n 4004, n! se term a a a ce 4004 4004 4004 4004 4004 mult + + + = 999 zerouri, n vreme + 2 3 4 5 5 5 5 5 5 4010 4010 4010 40 4010 n 4010, n! se termin n cel puin a t + + + 2 + 5 5 53 54 5 zerouri. Numerele cerute sunt 4005, 4006, 4007, 4008, 4009. (2m)! (2n)! Problema 5. Demonstrai ca t N, unde m, n N. m!n! (m + n)! Soluie. Mai nti, se poate demonstra inegalitatea t [x] + [y] + [x + y] [2x] + [2y] , x, y R. Exponentul unui factorprim p din descompunerea canonica a numarato X n m m+n s1 = + k + , n vreme ce exponentul lui p di pk p pk X 2m 2n k punerea canonic a numrtorului este s2 = a aa + k . Fo pk p k (2m)! (2n)! s N. obinem c s1 s2 i deci t a m!n! (m + n)!

deci expn! (2) < n i n! nu se divide cu 2n . s

Probleme propuse. 101001! ? 1. Cu cte zerouri se termin numrul a a 2004! 2. Sa se demonstreze ca numerele A = 111! 222! 333! 444! i B = 5 s 777! 888! sunt divizibile cu 10268 , respectiv 10715 . 3. Factorialul cror numere se termin cu exact 2004 zerouri? a a 4. Aflai exponentul lui k n 1 + k + k 2 + + kn !, unde k, n N , t

Concursul de matematic Al. Myller a


Ediia a II-a, Iai, martie 2004 t s

Not (pentru clasele IV-VI ). Toate subiectele sunt obligatorii. Timp efect a 90 min. Se acord din oficiu 30 puncte, cte 6 puncte pentru problemele 1-5, c a pentru problemele 6-10 i cte 10 puncte pentru problemele 11-15. s

Clasa a IV-a

1. Un elev rezolva fiecare dintre primele 5 probleme ale acestui test minute, iar pe fiecare dintre urmtoarele 5 n cte 5 minute. Cte minute a pentru a rezolva una dintre ultimele 5 probleme, presupunnd c fiecare a solicita acelai timp? (timpul total de lucru este 1h 30 min.) s 2. Calculeaza (100 99) + (98 97) + (96 95) + + (2 1). 3. Calculeaz (5 + 55 + 555 + 5555 + 55555) : (1 + 1 + 111 + 1111 + 1 a 4. Cei 41 de elevi ai unei clase urc n ir pe munte. Mircea observ a s a lui sunt un sfert dintre colegii sai. Al ctalea n ir este Mircea? s 5. Delia calculeaza suma cifrelor pe care le afieaza ceasul ei digital (d s la ora 14:28 ea obine 1 + 4 + 2 + 8 = 15). Care este suma maxim pe ca t a obine? t 6. Care sunt ultimele trei cifre ale numarului 1 2 3 4 5 . . . 2004 + 7. Aflai suma dintre demparit, mparitor, ct i rest, tiind ca res t s s t t 18 mai mic dect ctul, ctul este 25, dempritul este impar, iar mpri at at singur cifr. a a 8. Un numar se mparte la 3 i da restul 2. Ctul se mparte din s obinnd restul 2. Noul ct se mparte iar la 3 i gasim ctul 2 i restul t s s fost numrul iniial? a t 9. La un magazin se aduc 301 kg de mere n lzi de 25 kg i 21 kg. C a s folosesc n total? 10. 58 de elevi sunt aezai pe 4 rnduri, fiecare rnd avnd cu 3 elevi s t dect rndul din faa sa. Ci elevi sunt pe ultimul rnd? t t 11. Dan vrea s cumpere mingi. Dac ar cumpra 5, i-ar mai rmne 1 a a a a iar daca ar dori sa cumpere 7, ar mai avea nevoie de 220 000 lei. Ct costa 12. Marinarii de pe un vapor au hrana pentru 60 de zile. Ei gasesc p 30 de naufragiai i astfel hrana le va ajunge tuturor doar 50 de zile. C t s t erau pe vapor? 13. 12 baei i 8 fete sunt membri ai cercului de matematica. n fiecare s t s nca 2 fete i 1 baiat sunt acceptai ca membri ai cercului. Ci membri va a s t t de matematic atunci cnd numrul bieilor va fi egal cu numrul fetelo a a a t a 14. Cte numere naturale de patru cifre au ultima cifr 3? a 15. Pe o insula locuiesc numai arici, erpi i vulpi. Fiecare animal m s s data pe zi, astfel nct orice arici mannca la micul dejun cte un arpe, o s mnnc la prnz cte un arici, iar orice arpe mnnc la cin cte o a a s a a a sfritul zilei de miercuri, pe insul a rmas un singur animal. Cte anim s a a pe insula luni, nainte de micul dejun?

Clasa a V-a

1. Sa se calculeze suma 4 + 8 + 12 + + 2000. 2. Determinai numerele naturale x astfel nct mulimile A = {2x; 6x + t t i B = {2x 1; 2x + 1; 5x + 6} s aib un singur element comun. s a a 3. Care sunt numerele prime a, b, c pentru care a + 10 b + 12 c = 82 4. Aflai suma cifrelor numarului A = 102004 1. t 5. Sa se determine perechile de numere naturale (x, y) pentru ca 15 este echiunitar. a (x + 1) (y 4) 6. Sa se determine x N astfel nct numarul a = 5x + 5x+1 + 5x+2 aiba exact 48 divizori. 1 1 1 1 1 1 7. Sa se calculeze suma S = + + + + + + . 2 6 12 20 30 360 3a + 16 8. Aflai numerele a N pentru care t N. 2a 5 9. Care este suma ultimelor trei cifre ale produsului 1 2 3 . . . 25? 10. S se afle cel mai mic numr natural de 2 cifre pentru care suma dint a a i cubul lui este patrat perfect. s 11. Aezai n ordine cresctoare numerele a = 250 , b = 247 + 224 , c = 2 s t a +223 + 222 , d = 248 + 223 . 12. n peter erau dragoni roii i dragoni verzi. Fiecare dragon ro s a s s capete, 8 picioare i 2 cozi. Fiecare dragon verde avea 8 capete, 6 picioar s n total dragonii aveau 44 cozi. Sunt, de asemenea, cu 6 picioare verzi dect capete roii. Ci dragoni roii sunt n peter? s t s s a 13. Dan spal o main n 40 de minute, iar Ionu spal o main n 2 a s a t a s a timp vor spala mpreuna 3 maini? s 14. Dintre cei 101 de dalmaieni, 56 au o pat neagr pe urechea stn t a a pat neagr pe urechea dreapt, iar 29 au ambele urechi albe. Ci dalm a a a t pete negre pe ambele urechi? 15. Fie a = 2214 + 3143 i b = 3143 . Care dintre numere este mai mare? s

Clasa a VI-a
1. Dac a

7a 2b 2 b = , atunci este .............................. 5a + 4b 15 a 2. Numarul triunghiurilor din gura este .............................. 3. Diferena dintre masurile suplementului i complement s tului aceluiai unghi este .............................. s 1 1 1 1 4. Rezultatul calculului + + + + este ......... 13 35 57 2003 2005 0, 2 ( 5. Doua unghiuri complementare au raportul masurilor lor egal cu 0, 4 ( msura unghiului mai mare a este .............................. 6. Rezultatul calculului ab0 ba : 99 este .............................. 7. n cte moduri putem aeza patru persoane ntr-un rnd? s 8. Ultima cifr a numrului 22004 + 32004 + + 92004 este ................ a a 9. Avem la dispoziie timbre de 4000 lei i de 9000 lei. Pentru a t s scrisoare sunt necesare timbre n valoare de 35000 lei. Numarul total

este .............................. 10. Suplementul unui unghi i complementul sau au masurile invers pro s cu 2 i 5. Suma masurilor lor este .............................. s 11. Numrul maxim de unghiuri n jurul unui punct cu msuri numer a a diferite este .............................. 12. Un produs se scumpete cu 10% i apoi cu 20%. Acelai produs se s s s cu 20% i apoi cu 10%. n ce caz preul final este mai mare? s t 13. Soluia ecuaiei 1, (1x) + 2, (2x) + + 9, (9x) = 50 este ................ t t 14. Cte cifre de 0 are la sfrit numrul 1 2 3 4 . . . 2004? s a 15. Pe o tabla de ah 4 4 se trage o linie dreapta. Cel mai mare s patraele 1 1 care pot fi taiate n doua pari este .............................. t t

Not (pentru clasele VII-XII ). Toate subiectele sunt obligatorii. Timp efect a 3 ore. Pentru fiecare subiect se acord 7 puncte. a

Clasa a VII-a

1. a) Demonstrai c dac m i n sunt numere naturale, atunci 25n t a a s divizibil cu 3. b) Determinai cel mai mic numr de forma |25n 7m 3m |, unde m t a numere naturale nenule. Marius Ghergu 2. Fie a, b dou numere reale avnd modulele cel puin 2. Demo a t (a2 + 1)(b2 + 1) (a + b)(ab + 1) + 5. Cnd are loc egalitatea? Marius Du 3. n triunghiul ABC se consider nlimea [CM ], M AB, iar N a at tricul punctului M faa de BC. Paralela prin punctul N la CM intersec t n P i AC n Q. s a) Demonstrai ca M Q AP daca i numai daca [AB] [AC]. t s b) Artai cum pot fi obinute poziiile punctelor A, B, C atunci cnd a t t t doar poziiile punctelor M , N , P . t Petru Rduc a 4. Se consider un triunghi ABC, n puncte distincte A1 , A2 , . . . , An a (BC), n puncte distincte B1 , B2 , . . . , Bn pe latura (AC) i n puncte dis s C2 , . . . , Cn pe latura (AB). Fie M mulimea punctelor care se obin la t t a cel puin doua din segmentele (AAi ), (BBj ), (CCk ). Determinai numa t t i numrul maxim de elemente pe care le poate avea mulimea M . s a t Dan Br

Clasa a VIII-a
1. Determinai numerele ntregi a, b pentru care, oricare ar fi x real, t

(2a a2 )x4 + (2a + 2b 4)x3 + (3b 3)x2 + (2b2 2b)x + b + 2

Gheorghe Iu

2. Un zar este un cub de latur 1, pe feele cruia sunt impria t a mate puncte ca n gura, astfel nct suma numerelor de puncte de pe feele opuse sa fie 7. Din opt zaruri alcatuim un cub de t latur 2. a a) Ce valori poate avea numrul punctelor care sunt vizibile a pe feele cubului de latura 2? t b) Este posibil sa aezam zarurile astfel nct oricare doua fee care sunt s t s aib un numr egal de puncte? a a a 3. Fie ABCD o piramida n care AC = BC = 1 i AB = AD = BD = s Determinai distana de la punctul A la planul (BCD). t t

4. Fie cubul ABCDA0 B 0 C 0 D0 de latura a i puncte M (AB), N s P (D0 A0 ), astfel ca AM = CN = D0 P = x. a) Calculai M P . t b) Aratai ca triunghiul M N P are centrul de greutate pe segmentul [B t Dan

Clasa a IX-a
n

1. Fie n un numar natural i numerele reale a, b, c astfel nct an s b = b + c i cn = c + a. S se arate c a = b = c. s a a Gheorghe Iu 2. Pe laturile [AB], [BC], [CD], [DA] ale unui patrulater convex MA NB PD considera punctele M , N , P , respectiv Q astfel nct = = = MB NC PC unde k 6= 1. Sa se arate ca S[ABCD] = 2S[M N P Q] daca i numai daca s = S[BCD] . Petre Asa 3. Fie ABC un triunghi dreptunghic n A i D un punct aparin s t [BC]. Bisectoarele unghiurilor ADB i ADC intersecteaz laturile AB s a punctele M i N. S se arate c unghiul dintre dreptele BC i M N a s a a s 1 b b m(B) m(C) daca i numai daca D este piciorul perpendicularei din s 2 Bogdan Enesc p 4. S se determine numerele reale x, x > 1 pentru care n [xn ]este ntr a ar fi n 2. (Se noteaz cu [a] partea ntreag a numrului real a). a a a Mihai Piticari, C-lung Moldovenesc, Dan Popescu

Clasa a X-a

1. Fie A = {1, 2, 3, 4, 5}. Sa se determine numarul funciilor f : A t proprietatea c nu exist numere distincte a, b, c A astfel nct f (a) = f a a Adrian Zano 2. Se consider tetraedrul ABCD n care medianele din A n triunghiu a ABD i ACD sunt perpendiculare doua cte doua. Sa se arate ca much s sunt egale. Dinu Serbnescu, B a 3. Se considera numerele reale x, y, z cu proprietatea ca cos x+cos y +c

cos 3x + cos 3y + cos 3z = 0. S se demonstreze inegalitatea cos 2x cos 2y a Bogdan Enesc 4. Fie a 2 un numar natural. Consideram mulimea A = { a, 3 a, 4 a t a) S se arate c A nu conine o progresie geometric infinit neconsta a a t a a b) S se arate c pentru orice n 3, exist n elemente din A care sunt n a a a geometrica. Bogdan Enesc

Clasa a XI-a
1. a) Fie (xn ) un ir de numere reale, cu proprietatea |xn+1 xn | s

Artai c irul (xn ) este convergent. a t as b) S se construiasc un ir de numere reale (yn ), care s aib simult a a s a a etaile: t (i) lim (yn+1 yn ) = 0; (ii) (yn ) este marginit; (iii) (yn ) este div n Eugen P 2. Se da paralelogramul ABCD, cu laturi inegale. Vrful B se p pe AC n punctul E. Perpendiculara n E pe BD intersecteaza drept AB n punctele F , respectiv G. S se arate c EF = EG dac i nu a a a s \ = 90 . m(ABC) Mircea Becheanu, B 3. Fie matricele A, B M2 (Z) astfel nct AB = BA i det B = 1. S s c dac det(A3 + B 3 ) = 1, atunci A2 = O. a a Mircea Becheanu, B 4. Fie f : R R o funcie cu proprietatea lui Darboux. S se arate t a este cresctoare pe R \ Q, atunci f este continu. a a Mihai Piticari, C-lung Mol

1 2

Clasa a XII-a

1. Fie p un numar prim, p > 3. Sa se arate ca ecuaia (x + y)1 = x1 t soluii n corpul Zp dac i numai dac 3 divide p 1. t as a Mihai Piticari, C-lung Mol 2. Sa se calculeze limita X ln 1 + i ln 1 + j n n p lim . n n4 + i2 + j 2 1i<jn

Gabriel Mranu i Andrei Ned s s 3. Fie n 3 un numr impar i A un inel comutativ cu 3n elemente. S a s c numrul elementelor nilpotente ale lui A este cel mult n. (Elementul a a numete nilpotent daca an = 0, pentru n > 0 convenabil). s 4. S se arate c, pentru orice numr natural p, a a a Z 1 lim np+1 enx ln (1 + xp ) dx = p!.
n 0

Gheorghe Iu

Concursul de matematic Florica T. Cmp a


Etapa interjudeean, 8 mai 2004 t a
Not. Toate subiectele sunt obligatorii. Timp de lucru: cl. a IV-a 90 a cl. V-VIII 2 ore.

Clasa a IV-a

1. Precizeaz regula de formare a numerelor urmtoare i identific c a a s a lipsesc n ultimul numar: 2798, 5783, 3574, 7862, 54 . 2. ntr-o urn sunt bile albe, galbene i roii. Dac mprim numrul b a s s a at a la numarul bilelor galbene obinem ctul 2 i restul 2. mparind numarul b t s t la suma celorlalte obinem ctul 3 i restul 3. Aflai cte bile sunt n urn t s t a diferena dintre numrul bilelor roii i dublul sumei celorlalte este 17. t a s s 3. n timpul unui campionat de ah, doi participani care jucaser ace s t a de partide s-au mbolnvit i s-au retras, iar ceilali au continuat turne a s t sfrit. Este adevarat ca cei doi participani au ajuns sa joace ntre ei, dac s t n total s-au jucat 23 de partide? (Turneul s-a jucat n sistemul "fiecare c cte o singur partid.) Justificai rspunsul. a a t a

Clasa a V-a

1. Aflai vrstele tatalui, ului i nepotului, tiind ca sunt exprimate pr t s s mere prime, iar peste cinci ani vrstele lor vor fi exprimate prin trei numer ptrate perfecte. a 2. Determinai toate numerele de forma abbc, cifrele a, b, c fiind distin t c sunt ndeplinite condiiile: a t a) ba se divide cu 13; b) ab este numr prim; c) c este ptratul unui num a a Cte soluii are problema? t 3. Pe trei jetoane aezate cu faa n jos sunt scrise trei numere natur s t i distincte a caror suma este 13. Jetoanele sunt aezate n ordine cres s s la stnga la dreapta. Ana ridic, prima, jetonul din stnga, apoi Dan a dreapta, iar ultimul, Stefan, pe cel din mijloc, declarnd fiecare, n aceas ca nu are suficiente informaii pentru a descoperi celelalte numere. Dar t poi spune ce numar a vazut Stefan? t Mihaela Cia

Clasa a VI-a

1. Se consider mulimea A = {1, 2, 3, . . . , 2004}. Determinai: a t t a) probabilitatea ca, alegnd la ntmplare un element din A, acesta sa f cu 167. b) probabilitatea ca, alegnd la ntmplare o submulime B 6= a lui A t tuturor elementelor mulimii B s fie egal cu produsul tuturor elementelo t a A \ B. Ioan Lung \ 2. Se consider triunghiul ABC cu [AB] [AC], m(BAC) 90. Fie a \ = 60 i fie punctul E astfel nct D (AE) i [AE astfel nct m(ADC) s s \ Sa se determine m(EBD). Mihai Gavrilu t

3. Un pilot de avion parcurge o anumit distana cu avionul i efectue a t s torul calcul: aduna numerele naturale nenule ce reprezinta distana (n km t (n km/h) i cu timpul (n h) i obine 6008. Determinai ct timp a dura s s t t Cristian L

Clasa a VII-a

1. S se arate c oricum am alege 51 de numere naturale distincte de l a a printre ele exista doua numere naturale distincte a i b astfel nct a | b. s 2. Un casier de banca distrat, platindu-i un cec lui Lucian Georges, euro cu cenii (1 Euro = 100 ceni), dndu-i acestuia euro n loc de ceni t t t loc de euro. Dup ce i-a cumprat o acadea de 5 ceni, Lucian Georges a a s a t c i-a mai rmas exact o sum reprezentnd dublul sumei iniiale de pe a a a t este suma scrisa pe cec? Ctlin Bude a a 3. 26 de pisici sunt nchise ntr-un labirint care are forma din figura alaturata (fiecare latura are lungimea de 1 m). Dac dou pisici se afl la disa a a tana mai mic de 1, 5 m se vor zgria! Artai c t a a t a oricum am aeza cele 26 de pisici n labirint, macar s doua se vor zgria. Monica Nedelcu, Iai s
1m 1m

Clasa a VIII-a

1. Fie tetraedrul ABCD. Vrfului A i asociem numarul natural n, iar B, C, D le asociem numrul 0. Numim "mutare" alegerea a dou vrfuri a a mrirea numerelor asociate lor cu cte o unitate. S se arate c dup un n a a a a de "mutari" putem face ca fiecarui vrf sa-i fie asociat acelai numar, dac s daca n este numar par. Gheorghe Iu 2. Pentru a R consideram funcia fa : R R, fa (x) = ax + 2 a, t fie Ma simetricul originii faa de graficul funciei fa . t t a) Artai c graficele funciilor fa trec printr-un punct fix, oricare ar a t a t b) Sa se arate ca, oricare ar fi a i b R, lungimea segmentului Ma M s mica sau egala cu 2 5. Gabriel P 3. Un corp gol n forma de tetraedru regulat este aezat cu o faa pe s t sufer dou rsturnri instantanee, consecutive, pe alte dou fee ale sale. O a a a a a t lastic, aflat iniial n vrful ce nu atinge pmntul, se mic sub aciune a a t a s a t pamntului naintea primei rasturnari i pna dupa ultima rasturnare, im s fiecare rasturnare plecnd spre alta faa din punctul n care a ajuns. t a) Artai c bila cade de fiecare dat (nu se rostogolete pe fee). a t a a s t b) Calculai lungimea parcursului bilei, dac latura tetraedrului are lu t a 9 6 m. 32 Claudiu- tefan P S

Concursul Traian Lalescu


Ediia a V-a, Iai, 2004 t s
Not. Fiecare subiect va fi notat cu cinci puncte. Timp de lucru: 2 ore. a

1. Calculai: [62 + 8 (24 24 : 4 3)] : 5. t 2. Aflai valoarea lui x din egalitatea: 1500 [(409 307) 4 + 315 : 1 t = 1322. 3. Determinai cel mai mic numr natural n astfel nct: n+3n+5n+7 t a > 2004. 4. Cte numere de trei cifre au suma cifrelor mai mare sau egala cu 2 5. Pe tabla s-a scris de douazeci i trei de ori numarul 13 i de tr s s ori numrul 23. Cte numere trebuie terse de pe tabl pentru ca suma a s a rmase s fie 464? a a 6. Gasii cel mai mic numar natural care are suma cifrelor 102. t 7. Mulimea numerelor naturale diferite de zero se mparte n grupe t (2, 3), (4, 5, 6), (7, 8, 9, 10), . . . . Cu ce numr ncepe grupa cu numrul 51 a a 8. Suma anilor de natere ai Raluci i Elenei este 3961. Stiind c la s a s a mai 1998 Raluca era de ase ori mai mare dect Elena, ci ani are astazi s t 9. De-a lungul unui gard sunt opt tufe de zmeura. Numarul fructelor d tufe vecine difer cu 1. Numrul fructelor de pe toate tufele poate fi: a a a) 101; b) 213; c) 225; d) 229; e) alt numr. a 10. ntr-o clas, elevii politicoi sunt de dou ori mai muli dect cei n a s a t Numarul fetelor politicoase i al baieilor nepoliticoi este de doua ori mai m s s t cel al baieilor politicoi i al fetelor nepoliticoase, iar numarul fetelor polit s s t egal cu numrul tuturor bieilor. Ci biei sunt n clas, dac aceasta a a a t t a t a a i 30 de elevi? s 11. mparatul Rou lasa motenire celor patru fii herghelia de cai astf s s mare ia o treime din herghelie, al doilea ia trei optimi din rest, al treilea t din noul rest, iar mezinul a primit cei 10 cai rmai. Ci cai are hergheli a s t 12. Pe distana de 124 de metri s-au instalat 18 conducte de ap. S t a depozit se gaseau 15 conducte de 6 m i 10 conducte de 8 m, determinai s t masoara mpreuna conductele care au mai ramas n depozit.

Soluiile problemelor enunate la p. 108. t t 1. Numerele cautate sunt 11 i 1, 1, caci 11 + 1, 1 = 11 1, 1. s 2. Ursul se aa la Polul Nord. El are culoarea alba.

Concursul Adolf Haimovici, ediia a VII t


pentru liceele economice, industriale i agricole s

Faza interjudeean, Iai, 8 - 9 mai, 2004 t a s


Not. Toate subiectele sunt obligatorii. Timp de lucru: 2 ore. a

Clasa a IX-a

1. a) Se d graficul funciei f : R R, f (x) = x2 + ax + a, din figu a t este a? b) n figura 2 sunt reprezentate graficele a trei trinoame de gradul al d s fie acestea urmtoarele trinoame: ax2 + bx + c, cx2 + ax + b, bx2 + cx + a a a ntrebare pentru figura 3.
y y

x O O

Figura 1.

Figura 2.
2

Figura 3.

c) Se consider trinoamele de gradul al doilea de forma x + px + q, a p + q = 30. Care dintre ele au rdcini ntregi? a a 2. Se d triunghiul ABC n care AB = 3, AC = 4 i BC = 5. a s a) Sa se calculeze aria triunghiului ABC. b) Sa se afle raza cercului nscris i raza cercului circumscris triunghiu s c) S se calculeze sin B, cos B, sin 2B, cos 2B. a d) S se arate c cos nB Q, n N. a a e) Argumentai ca cos nB 6= 0 i ca exista o infinitate de numere nat t s proprietatea ca cos nB > 0. 3. a) Sa se arate ca (n + 1)2 (n + 2)2 (n + 3)2 + (n + 4)2 = 4, n b) S se arate c pentru orice k Z, exist n N i o alegere con a a a s semnelor + i astfel nct k = 12 22 ... n2 . s

Clasa a X-a

1. Fie z1 , z2 , z3 numere complexe astfel nct |z1 | = |z2 | = |z3 | |z1 + z2 + z3 | = 2004. Atunci (z1 + z2 )(z1 + z3 )(z2 + z3 ) = 0. 2. Codul Morse, care a fost mult utilizat n comunicaii telegrafice, t punctele i liniile pentru a codifica anumite caractere. De exemplu E s printr-un punct, T printr-o linie, zero prin 5 linii etc. Cte caractere se p prin cel mult 5 semnale Morse (puncte sau linii)? 3. Se consider polinoamele f = 8X 3 6X 1, g = 4X 3 3X. a a) S se arate c f nu are rdcini raionale. a a a a t b) Sa se afle ctul i restul mparirii lui f la g. s t

c) Artai c f cos a t a = 0. 9 d) Este cos iraional? Argumentai. t t 9 . e) Daca h Q[X] i h cos s = 0, sa se arate ca h . f . . 9

Clasa a XI-a
1. Se d graficul funciei a t f : R R, f (x) = ax4 x2 + bx + c.
O

Sa se afle semnele numerelor a, b i c. s 2. Fie dreptele d1 i d2 perpendiculare n A. Un punct F aezat n interi s s din unghiurile formate de cele dou drepte, este situat la distanele 1 i r a t s faa de acestea. S se determine lungimea minim a unui segment [BC t a a C d2 ) care trece prin punctul F . a b c 3. Fie mulimile M = c a b | a, b, c N i t s b c a K = n N | n = a3 + b3 + c3 3abc; a, b, c N . a) S se arate c dac A, B M, atunci AB M. a a a b) Pentru orice m, n K, avem mn K. a b c c) Exist o matrice E M cu proprietatea c c a b = aI3 + a a b c a a, b, c N? d) Fie A M, nesingular, cu d = det A. S se arate c a a a 22004 2004 i det . . . (A ) . . . = d2 s . . . (A ) . . . = d 3 {z } | {z } |
2004 2004

Clasa a XII-a

1. Pe mulimea G = {a, b, c, d, e} operaia definete t t s o structura de grup. Completai tabla grupului G. t

e 2. Fie polinoamele f = 8X 3 6X 1 i g = 4X 3 3X. s b a) Sa se arate ca f nu are radacini raionale. t b) S se calculeze ctul i restul mpririi lui f la g. a s at c) Artai c f cos a t a = 0. 9 d) Este iraional numrul cos ? t a 9 . = 0, sa se arate ca h . f . e) Daca h Q[X] i h cos s . 9 3. a) Calculai aria subgraficului funciei f : [1, e] R, f (x) = ln x. t t b) Dac a, b, c sunt numere reale oarecare din intervalul [1, e] astfel nct a atunci (b a) ln a + (c a) ln b + (c b) ln c < 2.

a b c d e

b d

Olimpiada Balcanic de Matematic (juniori) a a


A. Problemele de concurs - enunuri i soluii t s t 1. Pentru x, y R, nu simultan nule, s se demonstreze inegalitatea a x+y 2 2 p . x2 xy + y 2 x2 + y 2

Ediia a VIII-a, Novi Sad (Serbia i Muntenegru t s

3. Fie x, y N astfel nct 3x + 4y i 4x + 3y s fie ambele ptrate pe s a a se arate c numerele x i y sunt multipli de 7. a s

2. Fie 4ABC isoscel (AC = BC), M mijlocul segmentului [AC], nlimea din C. Cercul ce trece prin B, C i M intersecteaz a doua oa at s a CH n Q. Sa se afle raza cercului circumscris 4ABC funcie de m = CQ t

4. Se consider un poligon convex cu n 4 vrfuri. Descompunem a poligonul n triunghiuri ale cror vrfuri sunt printre cele ale poligonului, a a oricare doua triunghiuri sa nu aiba puncte interioare comune. Coloram triunghiurile ce au doua laturi care sunt i laturi ale poligonului, cu rou tri s s ce au exact o latur care este i latur a poligonului, celelalte triunghiuri a s a albe. S se demonstreze c numrul triunghiurilor negre este cu 2 mai m a a a numarul triunghiurilor albe.

1. Observam ca cei doi numitori sunt ntotdeauna pozitivi. Daca x inegalitatea este evident (i strict). Dac x + y > 0, se demonstreaz im a s a a a p x2 + y 2 x + y 2 (x2 + y 2 ), x2 xy + y 2 , 2 prin urmare p 2 (x2 + y 2 ) x+y 2 2 x+y 2 p . x2 xy + y 2 x xy + y 2 x2 + y 2 x2 + y 2 2 Egalitate se atinge pentru x = y > 0.

C 2. Fie P i O centrele cercurilor circumscrise tris unghiurilor BCM , respectiv ABC, K mijlocul lui [CM ], iar {L} = CH P K. Evident c O CH i a s L K cum OM i P K sunt perpendiculare pe AC, rezulta s P M ca OM k P K. Urmeaza ca LK este linie mijlocie n O 4CM O, deci CL = LO. [ ) = 90 Deoarece OP BC, avem ca m(LOP Q \ [ \ m(OCB). Apoi, m(OLP ) = m(CLK) = 90 A H [ [ m(OCA). ns OCB OCA, deoarece nlimea a \ at [ [ s CH este bisectoare n 4ABC isoscel, deci P OL P LO i P L = P O. Ob 4P CQ este isoscel cu P C = P Q, prin urmare va rezulta c CL = OQ. a 2 2 n concluzie, CL = LO = OQ, deci R = OC = CQ = m. 3 3

4. Notam cu x, y, z numarul triunghiurilor colorate n negru, rou, res s atunci x + y + z = n 2, acesta fiind numrul triunghiurilor din desco a poligonului. Deoarece fiecare latur a poligonului este latur a exact unu a a din descompunere, avem ca 2x + y = n. Combinnd cele doua relaii, o t x z = 2, ceea ce trebuia sa demonstram.

3. Fie m2 = 3x + 4y, n2 = 4x + 3y; atunci m2 + n2 = 7 (x + y), deci m Un ptrat perfect d la mprirea prin 7 unul din resturile 0, 1, 2, 4 i atun a a at s ca o suma de patrate perfecte sa se divida cu 7, trebuie ca fiecare n parte s . . . cu 7. ns, dac m2 . 7, atunci m2 . 72 i analog n2 . 72 , adic 7 (x + y) = m a a . . s . a . . deci x + y . 7. Urmeaza ca x = (4x + 3y) 3 (x + y) . 7, y = (3x + 4y) 3 ( . .

2. Gasii toate numerele naturale A care se scriu cu patru cifre n b t 2 1 a proprietatea c A + 2000 = A. (Cu A am notat rsturnatul lui A). a 3 3 3. Gsii toate numerele naturale n 3 cu proprietatea c n divide (n a t a 4. Pentru a, b, c [1, +), demonstrai inegalitatea t 1 1 1 (1 + abc) + + 3 + a + b + c. a b c 5. Pentru x, y, z R, demonstrai inegalitatea t

B. Probleme aflate n atenia juriului - enunuri t t 1. Fie a, b, p, q N cu p, q 3, iar a, b relativ prime i de pariti d s at se arate ca numarul N = 2ap b 2abq nu poate fi patrat perfect.

y2 z2 z 2 x2 x2 y 2 + 2 + 2 (x + y + z)2 . 2x2 + 1 2y + 1 2z + 1

8. Fie E, F dou puncte distincte n interiorul paralelogramului ABC a minai numarul maxim posibil de triunghiuri avnd aceeai arie, cu vrfu t s dintre punctele A, B, C, D, E, F .

a < b < 2a, sa se arate ca b 2 2 2ab a 2ab b2 1 a b + 1+ . 7ab 3b2 2a2 7ab 3a2 2b2 4 b a s 7. Dou cercuri C1 i C2 sunt secante n A i B. Un cerc C cu centru a s C1 n M i P , iar pe C2 n N i Q, astfel nct N i Q sunt situate de o p s s s \ alta a dreptei M P , iar AB > AM . S se demonstreze c M BQ N BP . a a \ 6. Daca 0 <

9. Fie 4ABC nscris n cercul C. Cercurile C1 , C2 , C3 sunt tangen cercului C n punctele A1 , B1 , respectiv C1 i tangente laturilor [BC], [CA s s punctele A2 , B2 , respectiv C2 , astfel nct A, A1 sunt de o parte i de alta etc. Dreptele A1 A2 , B1 B2 i C1 C2 intersecteaz a doua oar cercul C n pu a a s \ B 0 , respectiv C 0 . Dac {M } = BB 0 CC 0 , demonstrai c m(M AA0 ) = 9 a t a b 10. Fie 4ABC cu m(C) = 90 i punctele D [AC], E [BC]. Spre s triunghiului construim semicercurile C1 , C2 , C3 , C4 de diametre [AC], [B

respectiv [CE] i fie {C, K} = C1 C2 , {C, M } = C3 C4 , {C, L} = C2 C3 , s C1 C4 . Aratai ca punctele K, L, M , N sunt conciclice. t 11. Pe o tabl dreptunghiular cu m linii i n coloane, n fiecare cs a a s a scris semnul "". Putem efectua urmtoarele operaii: a t (i) schimbarea tuturor semnelor de pe o linie (din "+" n "", iar din " (ii) schimbarea tuturor semnelor de pe o coloana. a) Dac m = n = 100, artai c nu putem obine 2004 semne "+" u a a t a t un numr finit de ori operaiile descrise; a t b) Daca m = 100, gasii cea mai mica valoare a lui n > 100 pentru c t obine 2004 semne "+". t (continuarea tabelului din p. 113) 148. 149. 150. 151. 152. 153. 154. 155. 156. 157. 158. 159. 160. 161. 162. 163. 164. 165. 166. 167. 168. 169. 170. 171. 172. 173. 174. 175. 176. 177. PRECUPANU Codrin DUREA Magdalena BURGHELEA Diana FARCA Marius S BAGHIU Ciprian ION Elena BUCESCU Dominic PA A Narcisa S SUFITCHI Viorica POPA Gabriela RUSU Virginia DASCALESCU Diana GHERGHELA Liliana S FERENT Olimpia LEONTIE Rodica S PANAINTE Ecaterina Bronia POPA Dumitru CARBUNE Ioan BRE UG Constantin S CAILEANU Sorin DOCA Laurenia t BRNZILA Cristina S PLAE U Dan CONTU Valentin OLENIUC Mariana ROMILA Amalia - Patricia GRADINARU Daniela ARBONE Ion NECHITA Remus BUJOR Lorena

Colegiul "C. Negruzzi", Iai s Scoala "T. Maiorescu", Iai s Scoala "T. Maiorescu", Iai s Scoala "T. Maiorescu", Iai s Liceul "D. Cantemir", Iai s Liceul "G. Ibrileanu", Iai a s Scoala "I. Creang", Iai a s Liceul de arta, Iai s Scoala gen. nr. 26, Iai s Scoala gen. nr. 43, Iai s Liceul "M. Costin", Iai s Liceul "M. Eminescu", Iai s Gr. c. "Victoria", Iai s s Gr. c. "Victoria", Iai s s Liceul "Al. I. Cuza", Iai s Scoala "Gh. I. Brtianu", Iai a s Scoala " t. Brsanescu", Iai S s Liceul "I. Neculce", Tg. Frumo Liceul "I. Neculce", Tg. Frumo Liceul "I. Neculce", Tg. Frumo Liceul "I. Neculce", Tg. Frumo Liceul "D. Cantemir", Iai s Scoala normala "V. Lupu", Ia s Scoala "I. Cantacuzino", Paca s Gr. c. "D. Mangeron", Iai s s Scoala normal "V. Lupu", Ia a s Gr. c. "A. Saligny", Iai s s informatician Liceul "M. Costin", Iai s Scoala "I. Teodoreanu", Iai s (continuare la p

Cercul de matematic Leonard Euler a organizat la Universitatea Humboldt, Berl


Probleme pentru clasa a VIII-a Holger STEPHAN 1

Not. Rubrica "Corespondene" are ca scop informarea elevilor i profe a t s ara noastr cu privire la activitatea de performana din alte coluri ale lum t a t t orice "corespondena", soluiile problemelor vor publicate n numrul urmtor t t a a Materialul de faa a fost obinut prin strdania d-lui Dan Tiba, cercettor t t a a Institutul de Matematic al Academiei Romne. a

1. Patru numere adunate doua cte doua dau sumele 4, 7, 9, 14, 16 sunt cele patru numere? 2. Demonstrai c prin "rotirea ctre dreapta" a unui numr de 8 cifr t a a a cu 73, se obine tot un numr divizibil cu 73. (Se spune c un numr na t a a a "rotit catre dreapta", daca ultima cifra este mutata n faa primei cifre t 1234 4123.) 3. Aflai cifrele necunoscute x, y, z din egalitatea t 20 058 473 11! = x00yz0055046400.

4. Un numr x format din cinci cifre diferite i nenule este divizibil cu a s c suma tuturor numerelor de cinci cifre distincte ce se pot forma cu aceste a (inclusiv x) este divizibila cu 2399976. 5. Gasii toate perechile de numere ntregi x i y care sunt soluii a s t t diofantice 2x2 + 7xy + 3y 2 = 228. 6. Gsii toate perechile de numere ntregi x i y care sunt soluii a a t s t diofantice 2x2 + 3y 2 = 77. 7. Considerm numrul natural n, 1000 n < 5000. Formm num a a a a sau 13 cifre) obinut scriind n ordine cifrele lui 3n, 2n i respectiv n. t s acest numar este divizibil cu 28 + 1. 8. Sase numere prime 7 < p1 < p2 < p3 < p4 < p5 < p6 formeaz a de numere prime", dac p2 , p3 i p4 , p5 sunt numere prime gemene (adic a a s = p5 p4 = 2), iar p2 p1 = p4 p3 = p6 p5 = 4. Demonstrai c sum t a divizibila cu 630. 9. Este posibil ca suma a apte patrate perfecte succesive sa fie un patr s

Cercettor dr., Institutul Weierstrass, Berlin (e-mail: stephan@wias-berlin.de) a

Soluiile problemelor propuse n nr. 2 / 20 t


Clasele primare

P.54. Calculai a si b daca 46 a = 36 + a si b 3 = 17 b. t ( Clasa I ) nv. Doinia Sp t Soluie. 46 a = 36 + a se scrie 36 + 10 a = 36 + a, de unde 10 t a + a = 10, deci a = 5. Analog, b 3 = 17 b se scrie b = 3 + 17 b b + b = 20, deci b = 10. P.55. n cte moduri pot fi aranjate n linie dreapta 9 mingi roii si un s ( Clasa I ) Georgiana Ciobanu, e Soluie. Toate mingile ocup 10 locuri. Mingea galben poate ocupa t a a din cele 10. Sunt 10 moduri. P56. Cu cinci ani n urma, suma vrstelor a trei copii era de 11 ani. suma vrstelor acelorai copii peste 6 ani? s ( Clasa a II-a) nv. Rodica Rotaru Soluie. In prezent suma vrstelor copiilor este 11 + 3 5 = 26 ani, i t ani aceasta va fi 26 + 3 6 = 44 ani. P.57. n cte moduri pot fi mparii 8 baiei n doua echipe de cte tt t daca Petru vrea sa fie n echipa cu Mihai si Dan, dar nu vrea sa fie cu A ( Clasa a II-a) Adina Dohotaru, e Soluie. n gruparea Petru, Mihai, Dan mai trebuie un singur bia t a poate fi luat din restul bieilor, cu excepia lui Avram. Al patrulea elev a t t poate fi ales n patru moduri. P. 58. Sa se arate ca suma 1 + 4 + 7 + . . . + 100 mparita la 3 da re t ( Clasa a III-a) Alexandru - Gabriel Tudorache, Soluie. Suma se scrie 1 + (3 1 + 1) + (3 2 + 1) + . . . + (3 33 + 1 t mpririi la 3 este acelai cu restul mpririi sumei | + 1 + 1 + . . . + 1 at s at 1 {z } 34 ori este 1. P.59. Fie a si b doua numere consecutive. Suma acestor numere m numerele obinute marind cu 12 fiecare dintre vecinii lor este 939. Care t doua numere? ( Clasa a III-a) nv. Maria R a Soluie. Considerm b = a + 1. Folosim metoda gut a a 1 rativ. Primul numr este a a a Al doilea numar este 148 + 1 = 149. [939 (11 + 13 + 1 + 1 + 11 + 1 + 13)] : 6 = 888 : 6 = 148.
a a a
1 1 1

P.60. Din 16 bile, una este mai grea dect celelalte 15, care au mase e este cel mai mic numar de cntariri prin care se poate stabili bila mai gre ( Clasa a III-a) Carmen Ciolacu, e Soluie. Aezm cte 8 bile pe fiecare taler. Dup prima cntrire se t s a a a grupul de 8 bile care conine bila mai grea. Din acestea aezm cte 3 t s a taler. Daca balana este n echilibru, atunci bila mai grea se afla n perech t

Printr-o nou cntrire se afl bila mai grea. Dac balana nu este n a a a a t atunci bila mai grea se afla ntr-o grupare de 3 bile. Aeznd cte o bila s taler se determina bila mai grea. Numarul minim de cntariri este 3. P.61. Suma a doua numere este un numar de doua cifre al caror prod Diferena dintre cele doua numere este 7. Care sunt cele doua numere? t ( Clasa a IV-a) nv. Maria R Soluie. Suma celor dou numere poate fi 13 sau 31. Cum diferena t a t este 7, n primul caz numerele sunt 3 i 10, iar n al doilea numerele sunt s P.62. Doua ceasuri au nceput sa funcioneze la aceeai ora. Se con t s fiecare 30 minute (faa de ora exacta) unul ramne n urma cu un minut t avanseaza cu un minut. La un moment dat orele indicate de aceste ceasur h 36 min si 19 h 24 min. La ce ora au nceput sa funcioneze? t ( Clasa a IV-a) Felicia Amihiesei, e a Soluie. Cele dou ceasuri se abat cu acelai numr de minute faa de o t a s a t unul prin lipsa iar celalalt prin adaos. n momentul citirii abaterea este (1 18 h 36 min) : 2 = 48 min : 2 = 24 min. Ceasurile au fost citite la ora 1 24 min = 19 h. Numrul de ore n care ceasurile au funcionat este 24 a t Ceasurile au nceput s funcioneze la ora 19 12 = 7. a t P.63. Alege un numar format din trei cifre. Scrie la dreapta lui un num din doua cifre. Scoate din numarul format de 99 ori numarul format din Din rezultat scoate diferena dintre numarul de trei cifre si numarul de do t scrie rezultatul. Eu i ghicesc numarul format din doua cifre. Cum se ex t lucru? ( Clasa a IV-a) Prof. Petru Asa Soluie. Fie abc numarul de trei cifre i xy numarul de doua cifre. Ave t s = abc00 + xy = 100 abc + xy, 100 abc + xy 99 abc = abc + xy. n folosim metoda figurativ. a

n urma efecturii operaiilor indicate n problem se obine dublul num a t a t doua cifre.

Clasa a V-a

V.41. Fie a numar natural compus astfel nct daca p | a, cu p pr p + 1 | a. Sa se arate ca 12 | a si sa se afle cel mai mare numar a de trei Ciprian Ba Soluie. Fie p un divizor prim al lui a, conform enunului. Atunci p + t t p (p + 1) | a i, cum p (p + 1) este numr par, deducem c 2 | a. Din 2 s a a 3 | a i apoi 4 | a; prin urmare 12 | a. s Cel mai mare numr de trei cifre divizibil cu 12 este 996 = 22 3 83; cum a deducem ca 996 nu este soluie. La fel 984 nu este soluie. Numarul 972 t t verific cerinele problemei. a t V.42. Se dau numerele xy, ab scrise n baza 10 astfel nct xy divide

arate ca x = y daca si numai daca a = b.

Ioan Scleanu a a Soluie. Fie x = y. Atunci din xx = 11x | ab rezulta ca 11 | ab, deci ab t 33, . . . , 99}, adic a = b. a Fie a = b. Atunci xy | 11a i, cum a este cifr nenul, deducem c xy s s a a a 11. Ca mai sus, avem x = y.

V.43. Sa se afle cifrele a si b stiind ca a b = cd si ab = dc. Romana Ghia i Ioan Gh t t s Soluie. Evident, a {2, 3, . . . , 9}. Daca a = 2, atunci b 5 i cum 2 t s 25 = 32, 2 6 = 12 i 26 = 64, iar 27 > 100 rezult c nu avem soluii. D s a a t urmeaz b 4 i din 3 4 = 12 i 34 = 81, iar 35 > 100 deducem c nici n a s s a nu avem soluii. Considernd i verificnd toate posibilitaile, obinem sol t s t t i b = 2. s

V.44. Sa se afle x, y, z Q pentru care xn = yz, y n = xz, z n = xy, + N. N. H Soluie. Avem xn+1 = y n+1 = z n+1 = xyz, deci, n mod necesar, x t Condiiile iniiale se reduc la xn = x2 , prin urmare, x = y = z = 1 daca n t t i x = y = z Q daca n = 2. s +

V.45. Se dau sase urne, unele coninnd bile. Fie operaia: se aleg t t t se pune cte o bila n fiecare dintre ele. a) Compoziia urnelor fiind 0, 0, 4, 6, 6, 8, sa se indice o succesiune de t urma carora toate urnele sa conina acelai numar de bile. t s b) Compoziia urnelor fiind 0, 1, 2, 3, 4, 4, sa se arate ca nu exista o suc t operaii n urma carora toate urnele sa conina acelai numar de bile. t t s Gheorghe Iu Soluie. a) O succesiune de operaii ce atinge scopul este (0, 0, 4, t t (1, 1, 5, 6, 6, 8) (2, 2, 6, 6, 6, 8) (3, 3, 7, 6, 6, 8) (4, 4, 8, 6, 6, 8) (5, 5 (6, 6, 8, 8, 6, 8) (7, 7, 8, 8, 7, 8) (8, 8, 8, 8, 8, 8). b) Presupunem c dup n operaii urnele conin acelai numr m de b a a t t s a ca numarul total de bile din urne este 6m = 3n + 14, imposibil.

Clasa a VI-a

VI.41. Pe opt cartonae sunt nscrise cte unul din numerele 1, 2, s 32 , 33 , 34 . Daca P (k) este probabilitatea ca, extragnd doua cartonae, s obinute sa aiba n total k divizori distinci, sa se rezolve inecuaia P (k) t t t Dumitru Dominic Buc Soluie. Considernd cele 28 de posibilitai de extragere a cartonaelo t t s

rnd divizorii distinci ai numerelor extrase, obinem: P (1) = 0; P ( t t 5 8 7 3 2 1 i P (3) = ; P (4) = ; P (5) = ; P (6) = ; P (7) = ; P (8) = s 28 28 28 28 28 28 1 k 9. Prin urmare, mulimea soluiilor inecuaiei P (k) este S = {3, t t t 7 VI.42. Fie x, y, z N pentru care 84x + 91y + 98z = 2002. Sa se af maxima a sumei x + y + z. Adrian Zano

Soluie. Relaia din enun este echivalent cu 12x + 13y + 14z = t t t a 12 (x + y + z) = 286 (y + 2z). De aici, rezulta ca x + y + z este m y + 2z este minim i 12 divide pe 286 (y + 2z). Obinem x + y + s t pentru y + 2z = 10. Valoarea maxim a expresiei este 23 i se obi a s t (x, y, z) {(18, 0, 5) ; (17, 2, 4) ; (16, 4, 3) ; (15, 6, 2) ; (14, 8, 1) ; (13, 10, 0)}.

Not. D-l Titu Zvonaru, Comaneti (Bacau), stabilete valoarea a s s sumei x + y + z. ntr-adevr, dac x + y + z 19, atunci y + 2z 57 a a s 12 (x + y + z) + y + 2z 12 19 + 57 = 285; prin urmare, x + y + z x + y + z = 20, atunci y + 2z = 46. Pentru y < 15, z < 15, avem y + 2z < 4 16 y 20, avem ca z 4, deci y + 2z 28. Pentru 16 z 20, avem y y + 2z 44. Rezult c valoarea minim a sumei x + y + z este 21, care a a a de exemplu, pentru x = 2, y = 4, z = 15.

VI.43. Fie {a1 , a2 , . . . , an } Z pentru care k {1, 2, . . . , n}, i, j {1 i 6= j, astfel nct ak = ai + aj . Sa se arate ca n 6. Petru Asa Soluie. Putem presupune c a1 < a2 < < an . Considernd nu t a exista i, j {1, 2, . . . , n}, i 6= j, astfel ca an = ai + aj . Daca ai < 0, aj < an < ai , imposibil. La fel, ai < 0, aj > 0 implica an < aj , imposibil. De sunt pozitive i atunci mulimea conine cel puin trei numere pozitive. Co s t t t numrul a1 deducem c mulimea conine cel puin trei numere negative, p a a t t t n 6. Un exemplu de mulime cu 6 elemente este {3, 2, 1, 1, 2, 3} iar un t elemente este {3, 2, 1, 1, 2, 3, . . . , n 3}.

VI.44. Fie ABCD un paralelogram si M AB, N AD triunghiuri echila struite n exteriorul acestuia. Demonstrai ca [M N ] [BD] daca si n t N D k M B. Ciprian Ba Soluie. Presupunem [M N ] [BD]. Din cont D gruena triunghiurilor BAD i M AN deducem ca t s 360 120 \ \ m(BAD) = m(M AN ) = = 120 , deci N 2 \ A a m(BAN ) = 180 , adic punctele B, A, N sunt coli\ \ niare. Cum M BN DN B, rezulta M B k N D. \ Presupunem M B k N D. Ca urmare, m(ABD)+ \ \ +m(ADB) = 180 120 = 60 , deci m(BAD) = \ ) = 120 i atunci tri= 120 ; totodata m(M AN s unghiurile M AN i BAD sunt congruente, de unde rezult c [BD] [M s a a

VI.45. Fie E, F picioarele nalimilor din B si C ale triunghiu t tunghic ABC. Daca P , N sunt mijloacele laturilor [AB], respectiv [AC], b \ = P E F N , sa se arate ca m(P QF ) = 180 3 m(A). (n legatura din Parabola, nr. 3/2000 ) Titu Zvonaru, B

Soluie. Fie Q exterior triunghiului ABC. Din t A 4AEB dreptunghic n E, cu (EP ) mediana, deducem [ [ E c triunghiul AEP este isoscel cu AEP P AE; deci a P b \) = 2m(A). La fel, din triunghiul dreptunghic m(QP F N F \ \ AF C cu mediana (F N ) deducem N F A F AN . Din b \ 4P QF rezult c m(P QF ) = 180 3m(A). S remara a a B b cm c punctul Q este exterior dac m(A) < 60 . Cazul a a a b n care Q este interior triunghiului ABC, ce corespunde situaiei m(A) t b = 60 , dreptele P E i s trateaz folosind aceleai argumente. Dac m(A) a s a \ paralele i putem considera ca m(P QF ) = 0 . s

Clasa a VII-a

VII.43. Pentru n N, notam cu s (n) numarul de reprezentari di lui n ca suma de doua numere naturale (n = a + b si n = b + a constitu reprezentare). Sa se arate ca: n hni hn X 1 a) s (m + n) = s (m) + s (n) [1 + (1)mn ]; b) s (k) = 2 2 k=0 Petru M hni Soluie. Artm c s (n) = t aa a + 1. ntr-adevr, pentru n par, n = a 2 hni + n = 0 + n = 1 + (n 1) = . . . = k + k, deci s (n) = k + 1 = 2 n impar, n = 2k + 1, avem: n = 0 + n = 1 + (n 1) = . . . = k + (k hni s (n) = k + 1 = + 1. 2 a) Relaia se demonstreaza analiznd paritatea numerelor m i n. t s b) Daca n = 2k + 1, avem: s (0) + s (1) + . . . + s (n) = 1 + 1 + 2 + hni hn + 1i +k + k = k (k + 1) = . Dac n = 2k, avem s (0) + s (1) + . . a 2 2 hni hn + 1i = 2 (1 + 2 + . . . + (k 1)) + k = k2 = . 2 2

a b + = 1. b+1 a+1 Alexandru Negrescu, elev, Not. Mai multe soluii ale acestei probleme sunt date n articolul a t acest numr la p. 109. a VII.42. Sa se arate ca a2 + 1 b2 + 1 c2 + 1 (|a| + |b|) (|b| + |c|) a, b, c R. Dorin Mrghidanu, a Soluie. Notnd |a| = |b| = y, |c| = z, x, y, z R+ , inegalitatea e t x, lent cu x2 + 1 y 2 + 1 z 2 + 1 (x + y) (y + z) (z + x), x, y, z R a p 2 2 x + 1 y 2 + 1 = x2 y 2 +x2 +y 2 +1 x2 +y 2 +2xy = (x + y) , deci (x2 + 1 p p x + y. La fel, au loc: (x2 + 1) (z 2 + 1) x + z i (y 2 + 1) (z 2 + 1 s Prin nmulirea ultimelor trei relaii se obine inegalitatea dorit. t t t a Egalitate se obine pentru xy = 1, yz = 1, zx = 1, deci pentru x = y = t |a| = |b| = |c| = 1. VII.41. Rezolvai n N2 ecuaia t t

VII.44. Fie [AB] diametru al cercului C de centru O; N, M C a \ ) = 36 , iar [OM este bisectoare pentru N OB. Daca T este sim \ m(AON O faa de M N , sa se arate ca proiecia lui T pe AB este mijlocul lui [AO t t Valentina Blen \ Soluie. n triunghiul T N O isoscel, m(T N O) = 180 t T 1 \ \ 2m(N OT ) = 180 m(N OB) = 108 . n triunghiul 2 1 \ isoscel OAN , m(AN O) = (180 36 ) = 72 , deci N 2 \ \ m(T N O) + m(AN O) = 180 , prin urmare T , N , A sunt [ [ coliniare. Cum m(T AO) = m(T OA) = 72 , triunghiul T AO O A este isoscel i concluzia este imediat. s a

VII.45. Fie 4ABC echilateral, iar P (BC). Notam cu D, E simetr faa de AC, respectiv AB. Sa se arate ca dreptele AP , BD si CE sunt c t Constantin Cocea i Julieta Grig s Soluie. Fie {M } = BD AC, {N } = CE AB; t A AB = a, BP = x, P C = a x. Din asemnarea tria CM CD ax unghiurilor DM C i BM A gsim s a = = , MA AB a iar din asemanarea triunghiurilor BEN i ACN gasim E N s BN x BE = = . AC AN a B P BP CM AN n triunghiul ABC avem = 1; conform reciprocei te P C MA NB Ceva, dreptele AP , BM , CN sunt concurente.

Clasa a VIII-a

VIII.41. Fie f1 , f2 , f3 funcii liniare ale caror grafice sunt drepte c t doua cte doua. Cele trei drepte sunt concurente daca si numai daca e R si exista u 6= v R astfel ca f1 (u) f2 (u) f3 (u) = = , f1 (v) f2 (v) f3 (v)

cu fi (v) 6= , i {1, 2, 3

Claudiu- tefan P S Soluie. Fie fi : R R, fi (x) = ai x + bi , i = 1, 3. Graficele func t concurente doua cte doua daca i numai daca a1 , a2 , a3 sunt distincte. s b1 = Prin proporii derivate relaia din enun este echivalent cu t t t a a1 b3 = , cu convenia c, dac unul dintre numerele ai este zero, atunc t a a a3 este zero. Graficele funciilor fi sunt concurente dac i numai dac exist (, t as a a astfel nct fi () = , i = 1, 3, adic a1 + b1 = a2 + b2 = a3 + b3 = a b2 b3 b1 = = , ceea ce ncheie rezolvarea problemei. = a1 a2 a3 VIII.42. Fie x, y, z (0, ). Sa se arate ca

Lucian Tuescu t Soluie. Avem x2 + + z 2 xy yz zx 0 deci x2 + y 2 + z 2 yz t y2 yz + zx yz + zx p i atunci s . yz + zx + xy yz + zx + x2 + y 2 + z 2 yz zx aceasta cu nc dou inegaliti asemntoare, obinem concluzia problem a a at a a t

yz + zx xy + xz p p + yz + zx + x2 + y 2 + z 2 yz zx xy + xz + x2 + y 2 + z 2 xy + yz p + 2. xy + yz + x2 + y 2 + z 2 xy yz

VIII.43. Daca un triunghi dreptunghic are laturile numere naturale, catetelor este patrat perfect, atunci suma cuburilor catetelor este suma de trate perfecte. Andrei Ned Soluie. Fie (a, b, c) un triplet de numere naturale nenule care satis t a2 + b2 = c2 . Urmeaz c toate trei sunt pare sau numai unul este a a cb+a c+ba , N . 2 2 2 2 1 1 Avem a2 + b2 ab = (c b + a) + (c + b a) . Deoarece a + 2 2 hp i2 h p 2 3 3 2 3 a +b = (a + b) a + b ab , rezulta ca a +b3 = (c b + a) + (c + 2 2 VIII.44. Pe laturile [AB], [CD], [BC], [AD], [AC] si [BD] ale BP lui ABCD se iau respectiv punctele M , N , P , Q, R, S astfel ca BC DN AR DS AM = , = . Notam cu V1 , V2 , V3 , V4 , V respectiv volumele t AB DC AC BD AM RQ, BP M S, CP N R, DN QS si ABCD. Sa se arate ca 212 V1 V2 V3 V Viorel Cornea i Dan Stefan Marinescu, H s V1 V2 Soluie. Notnd cele trei rapoarte cu , , , avem: t = , = V V V3 V4 (1 ), = (1 ) (1 ) (1 ), = (1 ) . Atunci V V 2 2 2 1 1 1 V1 V2 V3 V4 2 2 2 2 2 2 = (1 ) (1 ) (1 ) 4 V 4 4 4 unde am folosit inegalitatea x (1 x)

1 , x [0, 1]. 4 VIII.45. Fie A1 , A2 , . . . , Ak puncte pe un cerc C. Sa se determine necesara si suficienta pentru a putea nscrie n C un poligon regulat ce admi date ca vrfuri (nu neaparat consecutive). Irina Mustaa, e t Soluie. Presupunem c P este un poligon regulat cu n laturi ce ar t a A1 , A2 , . . . , Ak ca vrfuri. Dac ntre Ai i Ai+1 sunt mi laturi ale l a s 2 unghiul la centru dintre dou vrfuri consecutive ale lui P este a , de n

\ 2 m(Ai OAi+1 ) \ m(Ai OAi+1 ) = mi t , deci Q, i = 1, k (cu convenia Ak n 2 \ m(Ai OAi+1 ) mi S artm c aceast condiie este i sucient. Fie a aa a a t s a , = 2 ni i = 1, k. Alegnd n multiplu comun al numerelor n1 , n2 , . . . , nk A1 B2 . . . Bn este poligon regulat care conine vrfurile A1 , A2 , . . . , Ak t m1 nm1 2 nm1 \ i 2 = N, deci A2 n aceast situaie m(A1 OA2 ) = a t s n1 n1 n n1

Clasa a IX-a

IX.41. Pentru n N, n 10, notam cu u2 (n) numarul format di doua cifre ale lui n. Sa se arate ca: a) u2 a20k+p = u2 (ap ), p {4, 5, . . . , 23}, k N, a {2, 3, 8}; b) u2 a10k+p = u2 (ap ), p {2, 3, . . . , 11}, k N, a {4, 9}; c) u2 (5n ) = n N; 25, d) u2 65k+p = u2 (6p ), p {2, 3, . . . , 6}, k N; e) u2 74k+p = u2 (7p ), p {2, 3, 4, 5}, k N. Ovidiu Pop, Sa Soluie. n rezolvarea problemei vom folosi faptul c u2 (a) = u2 ( t a numai daca a b se divide cu 100. 2k a) 220k+p = 210 2p = 10242k 2p = (1025 1)2k 2p = (M25 + p = M100 + 2 ; k k 320k+p = 910k 3p = 815 3p = 01 3p = (M100 + 1) 3p = M1 820k+p = 260k+3p = 10246k 23p = (M25 1)6k 8p = (M25 + 1)8p = M Egalitile b) e) se demonstreaz analog. at a 1 1 1 IX.42. Fie a, b, c R astfel nct a + b + c = + + . Sa se afle a b c a, b, c, daca |abc| = 1. (enun scurtat) t Marius Pachiariu, t a Soluie. Ecuaia |abc| = 1 este echivalent cu a2 b2 c2 = 1. Observm c t t a

(1 ab) (1 bc) (1 ca) = 1 ab bc ca + abc (a + b + c) a2 b2 c2 = 1

(s-a inut seama de relaia din enun). Ca urmare, suntem condui t t t s (1 ab) (1 bc) (1 ca) = 0. Dac ab = 1, gsim c = 1 i, deci, a a s (x, 1/x, 1), x R , sunt soluii ale problemei. Celelalte soluii se obin d t t t prin permutari circulare. IX.43. Fie funcia f : R R, iar a (1, ). Stiind ca t 1 2 2 1 f x + ax a f x f x + 1, x (, 0) ,
2

Titu Zvonaru, B 1 Soluie. Ecuaia x + ax a = , x (, 0) este echivalent c t t a x 2 2 x + ax + x + 1 = 0 sau cu x + x (a + 1) + 1 = 0, x (, 0). Cu S < 0, P > 0 ea are dou rdcini reale, distincte, negativex1 ,x2 . a a a 1 1 1 Pentru x = x1 , relaia din enun devine f x1 f 2 x1 f x1 t t

sa se arate ca f nu este injectiva.

IX.44. Daca 4ABC este ascuitunghic, sa se gaseasca maximul t E = sin A cos A + sin B cos B + sin C cos C. Cezar Lupu, elev, i Tudorel Lupu, C s Soluie. t Putem presupune A B C, deci sin A sin B sin C i s cos B cos C. Folosind inegalitatea lui Cebev, vom avea s sin A + sin B + sin C E cos A + cos B + cos C . 3 p 3 3 Dar sin A+sin B+sin C , cos A+ cos B+ cos C 3(cos A+cos 2 r 3 3 6 3 t . Cum pentru A = B = C = 60 obinem 3 = . Deci E 2 4 2 3 6 . deducem c maximul lui E este a 4 IX.45. Demonstrai ca 4ABC n care are loc egalitatea t X ha hb mc = 1, ma mb mc + ha hb mc + ma mb ic suma fiind obinuta prin permutari circulare, iar notaiile fiind cele uz t t echilateral. Iuliana Georgescu i Paul Georg s Soluie. Avem ha ia ma i analoagele; deci suma din enun este t s t X ha hb mc ha . Notnd x = sau egal cu S 0 = a ma mb mc + ha hb mc + ma mb hc ma X hc xy z= , avem 0 x, y, z 1 i S 0 = s . mc 1 + xy + z X Cum 1 + xy x + y pentru x, y [0, 1] i analoagele, avem S 0 s x X x = 1; egalitate are loc daca i numai daca x = y = z s x+y+z ha = ma , hb = mb , hc = mc , adica daca i numai daca triunghiul este ech s

2 1 1 1 f x1 1 0, de unde deducem f x1 = 1. La fel f x2 = 1. Pr 1 1 f x1 = f x2 , x1 6= x2 ; deci f nu este injectiv. a

2 F2n 2n X.41. Prove the inequality , where the Fibonacci nu n Fn1 Fn are defined by F0 = F1 = 1, Fn+1 = Fn + Fn1 , n 1. Zdravko Starc, Vrac, Serbia and Mo Soluie. Conform inegalitii Cauchy-Schwarz, avem t at " # 2 2 2 n n 2 n n n 2 2 F0 + F1 + + Fn + + + F + F + 1 n 1 1 0 0 0 n n n P n 2 P 2 P n 2n = , Fk = Fn Fn+1 , Fk = F2n (v Deoarece n k=0 k k=0 k k=0 2n 2 2/2002, p. 70), obinem t Fn Fn+1 F2n . Egalitate apare pentru n = n

Clasa a X-a

X.42. Sa se rezolve ecuaia 2[x] + 6[x] + 7[x] = 3[x] + 4[x] + 8[x] . t Daniel Jing Soluie. Notam [x] = y, y Z. Avem de rezolvat n Z ecuaia 2y + t t = 3y + 4y + 8y . Dac y < 0 avem 7y > 8y i 2y + 6y > 3y + 4y (deoarece a a s echivalent cu (2y 1) (3y 2y ) > 0, y < 0). Prin urmare nu avem solui a t Se verifica uor faptul ca y = 0, y = 1, y = 2 sunt soluii. t s Pentru n 3, n N, demonstram ca 3n + 4n + 8n > 2n + 6n + 7n . Est s artm prin inducie c 8n + 4n > 6n + 7n , n 3, care este un exerciiu a aa t a t Prin urmare y {0, 1, 2} i atunci x [0, 1) [1, 2) [2, 3). Mulimea s t ecuaiei este intervalul [0, 3). t X.43. Fie f o funcie reala nenula cu proprietatea ca t f (x + y xy) = f (x + y) f (x) f (y) , 2003 Sa se calculeze f . 2002 x, y R.

Adrian Zano Soluie. Rezolvam problema n mai multe etape. t 1. Pentru x = y = 0, din relaia dat gsim f (0) = 0. t a a 2. Pentru y = x = 1, deducem f (1) + f (1) f (1) = 0, adic f (1 a f (1) = 1. Dac f (1) = 0, pentru y = 1 relaia dat implic f (x a t a a x R, adica f = 0, contrar ipotezei. 3. Ne ocupm de cazul f (1) = 1 i f (1) = a 6= 0. Pentru y = 1, g a s a f (x + 1) = af (x) + a, x R.

Pentru y = 1 i x x + 1, obinem f (2x + 1) = f (x + 1) + f (x s t af (2x) + a = af (x) + a + f (x), deci a+1 f (2x) = f (x) , x R. a Din (1) i (2) pentru x = 1, deducem f (2) = a2 + a = a + 1, de und s Daca a = 1, relaia (2) implica f (2x) = 0, x R, fals. Prin urmare a = t 4. Din (1), gasim f (x + 1) = f (x) + 1, x R. Prin inducie, gasim f t = f (x) + k, x R i k Z. Pentru x = 0 deducem c f (k) = k, k s a p 5. n relaia dat nlocuim x cu i y cu q (p, q Z, q 6= 0) i gsim f t a s s a q 2003 2003 adic f (x) = x, x Q. Prin urmare, f a = . 2002 2002 t X.44. Urnele U1 , U2 , . . . , Un conin fiecare cte a bile albe si b bile n fiecare urna se extrage cte o bila care se depune ntr-o alta urna U . D se scoate o bila si se constata ca este alba. Care este compoziia cea mai p t urnei U ? Petru M Soluie. Notam cu Ek evenimentul constnd n faptul ca n U ar fi dep t k nk a b k albe (i n k bile negre). Avem P (Ek ) = Cn s ,k= a+b a+b E evenimentul ca bila extrasa din U sa fie alba. Conform formulei lui Ba

k P (Ek ) PEk (E) kCn ak bnk kC k ak bnk PE (Ek ) = Pn = Pn n i i ni = na (a + b)n i=1 P (Ei ) PEi (E) i=1 iCn a b k Aceasta probabilitate este maxima cnd f (k) = kCn ak bnk , k = 0, n est f (k + 1) na f (k + 1) na i Observam ca >1k < s <1k> f (k) a+b f (k) a+b este punctul de maxim pentru f (k), avem f (k0 + 1) f (k0 ) i f (k0 ) s na na na din care deducem k0 , + 1 . Prin urmare, k0 = , dac a a+b a+b a+b na na i k0 = s + 1, dac a N. / a+b a+b X.45. Se considera triunghiul ascuitunghic ABC. Sa se arate ca t triunghi A0 B 0 C 0 astfel nct A0 (BC), B 0 (AC), C 0 (AB), iar m( \ \ = m(BA0 C 0 ) = m(CB 0 A0 ) = (0, 90]. Daca n plus 4ABC este echilat calculeze lungimile laturilor 4A0 B 0 C 0 n funcie de a = BC si . (n leg t problema propusa la O. N. M., 2002) Dan Popescu Soluie. Fie D proiecia punctului C pe AB, deci D (AB). Fie C 00 t t 00 A00 B) \ E (AC) nct m(EC 00 A) = i A00 (BC) nct m(C\ = . De s \ fie A1 i C1 , A (BA1 ), C (BC1 ) i F (A1 C1 ) cu A1 C1 k AC i m(A00 s s s 00 00 00 00 0 n aceste condiii, (C E (A F = {B }, (BB (AC) = {B }, iar om t a centru B i raport , raport definit de BB 0 = BB 00 , transform A00 n A s i C 00 n C 0 (AB). Prin urmare A0 B 0 C 0 satisface enunul. s t n cazul n care triunghiul ABC este echilateral se deduce relativ uor ca s este echilateral i (AB 0 ) (CA0 ) (BC 0 ). Cu teorema sinusurilor n s a deducem B 0 C 0 = . cos + 3 sin

Clasa a XI-a
XI.41. Fie A1 , A2 , . . . , Ak Mn (Z) astfel nct

A(1) A(2) . . . A . unde Sk este mulimea permutarilor de ordin k. Sa se arate ca n . k!. t . Vladimir Marti Soluie. Deoarecetr (AB) = tr (BA), A, B Mn (C), inductiv s t tr A(1) A(2) A(k) = tr (A1 A2 Ak ), Sk . Tinnd cont c tr ( a = tr A + tr B, A, B Mn (C) deducem c a ! ! P P tr A(1) A(k) = tr (In ) tr A(1) A(k) = n
Sk Sk

Sk

i cum tr (A1 A2 Ak ) Z, concluzia problemei este imediata. s

tr A(1) A(k) = n k! tr (A1 A2 Ak ) = n

Sk

XI.42. Prin punctele M1 si M2 ale unei elipse se duc normalele la e 0 intersecteaza una din axele de simetrie ale acesteia n M1 , respectiv M 0 0 arate ca mediatoarea segmentului [M1 M2 ] trece prin mijlocul lui [M1 M2 ] proprietatea adevarata pentru hiperbola sau pentru parabola? Gheorghe Costo

x2 y2 + 2 = 1, = 1 i Mi (xi , yi ) s a2 b 2 a yi t (x xi ), i i = 1, 2. Normala n Mi la curba are ecuaia y yi = 2 b xi 2 a b2 0 xi , 0 intersecteaza una din axele simetrie, de exemplu Ox, n Mi a2 2 2 a b 0 0 Mijlocul M 0 al segmentului [M1 , M2 ] are coordonatele (x1 + x2 2a2 y1 + y2 x2 x1 diatoarea segmentului M1 M2 are ecuaia y t x = 2 y2 y1 Se verica uor ca M 0 este situat pe mediatoarea segmentului [M1 M2 ]. s Prin calcul se verifica ca proprietatea ramne valabila i pentru parab s XI.43. Consideram sirul de funcii fn : (0, ) R, fn (x) = nx + ln x t si fie xn soluia unica a ecuaiei fn (x) = 0. Sa se calculeze limitele sirurilo t t n si ((xn ) )n1 . a Angela Tigeru a s Soluie. Evident, funciile fn sunt strict cresctoare i surjective, ceea t t existena i unicitatea soluiei xn . t s t x1 Folosim n continuare inegalitaile ln x x 1, x (0, ). t x n 1 n 1 n = + ln + 1 = 0 i s fn n+1 n+1 n+1 n+1 n+1 n+1 n+1 2 n+1 n+2 2 fn = ln + 1 + > 0, n+2 n+2 n+2 n+2 n+1 n+2 n n+1 n i atunci fn (xn ) fn s xn Prin urmare, fn n+1 n+2 n+1 1 s n 1. Gsim lim xn = 1 i lim xn = . a n n n e XI.44. Sa se determine funciile continue f : (0, ) (0, ) p t f (x) = f 2x2 2x + 1 , x > 0. Marian Ursrescu a p Soluie. Fie x0 (0, 1] fixat i irul (xn )n0 definit prin xn+1 = 2x2 t s s n n N. Se verifica faptul ca xn (0, 1], n N, (xn )n0 este crescator i li s n p Pentru x = xn , relaia din enun devine f (xn ) = f t t 2x2 2xn + 1 = n n N, deci f (x0 ) = f (xn ). Ca urmare, f (x0 ) = lim f (xn ). Rezult a Soluie. Fie curba de ecuaie t t
n

p 1 + 2x2 1 n s s Fie x0 > 1 i irul definit prin xn+1 = , n N. S 2 xn > 1, n N. (xn )n0 este descrescator i lim xn = 1. Cum rela s n q curena se poate scrie i n forma xn = 2x2 2xn+1 + 1, obinem f t s t n+1 q =f 2x2 2xn+1 + 1 = f (xn ), apoi f (x0 ) = f (xn ) i f (x) = f (1) s n+1 n concluzie, f (x) = f (1), x (0, ), deci f este funcie constanta. t

= f (1), x (0, 1].

XI.45. Fie k N, k 2 si numerele reale pozitive a1 , a2 , . . . , ak , b1 p cu a1 < a2 < < ak . Definim xn = n b1 an + b2 an + . . . + bk an . 1 2 k a) Sa se demonstreze ca lim xn = ak ;
n

ln xn xn e ak 1 xn 1 = ak lim n ln = ak lim lim n (xn ak ) = ak lim n n n n n ak ln xn ak ak n n n a1 a2 ak1 = ak lim ln b1 + b2 + + bk1 + bk = a n ak ak ak c)

b) Sa se arate ca lim n (xn ak ) = ak ln bk ; n n ak c) Daca bk = 1, are loc lim n (xn ak ) = ak bk1 . n ak1 Marian Tetiv p Soluie. a) Avem ak n bk n an b1 + + an bk ak n b1 + b2 + + t 1 k p Deducem ca lim n b1 an + b2 an + + bk an = ak . 1 2 k
n

b)

n n ln xn ak ak e ak 1 xn lim n (xn ak ) = lim ak n ln n n ak1 ak1 ln xn ak ak n n n n ak a1 a2 ak1 ln b1 + b2 + + bk1 = ak lim n ak1 ak ak ak n n n n ak a1 a2 ak1 = ak lim + b2 + + bk1 b1 n ak1 ak ak ak n n a1 a2 = ak lim b1 + b2 + + bk1 = ak bk1 n ak1 ak1

Clasa a XII-a
XII.41. Sa se calculeze nN .

Z (1 + x) 1 + x2 . . . 1 + x2n1 x2n

dx, unde x

1 x2 1 x2 1 x2 1 x2 n ... = 1 + x + x2 + + x2 1 n1 = 2 1x 1x 1 x 1 x2 n1 (1 + x) 1 + x2 1 + x2 1 1 1 = 2n + 2n 1 + + , x2n x x x relaie care se verific i pentru x = 1. Primitivele funciei sunt t as t =

Oana Marangoci, stud n1 Soluie. Pentru x (1, ), avem (1 + x) 1 + x2 1 + x2 t =


2 n n

1 1 1 + + + + ln x + C. (2n 1) x2n 1 (2n 2) x2n 2 x Z 5 4 5 f (x) sin R o funcie continua pentru care t XII.42. Fie f : , 4

Mihai Ha Soluie. Avem t 5 Z 5 Z 5 4 4 4 sin 2x (cos 2x)0 dx = dx = arctg (cos 2x) = 2 1 2 2x 1 + cos 1 2 sin 2x Z 5 Z 5 Z 5 4 4 4 2 sin 2x dx sin f (x) sin 2x dx = f (x) 2 1 2 sin2 x 1 2 sin 2x 5 Aplicnd teorema de medie exista c , astfel nct 4 2 2 sin 2c = 0 f (c) = . f (c) 2 4 2 sin 2c 2 sin2 2c 2 5 Dar (1, 2) pentru c , , adica f (c) (1, 2). 2 4 2 sin 2c XII.43. Sa se arate ca Z 1 arctg ln a ln a x2 , a > 1. a dx 1 3 ln a 0 Petru Rduc a Soluie. Se arat c ax x ln a + 1, x R i a > 1. Obinem astfe t a a s t 2 1 1 x2 ln a + 1 ax = x2 2 , x R, a > 1. x ln a + 1 a Integrnd ntre 0 i 1, obinem inegalitaile cerute. s t t XII.44. Sa se afle numarul radacinilor reale ale polinomului P Z [X t minim, care admite radacina 2 + , unde verifica ecuaia x3 x + 1 Laureniu Modan, B t Soluie. Polinomul n y care se cere, apare prin eliminarea lui x ntr t x3 x + 1 = 0 i y = x2 + x. Deoarece x2 = y x, gsim x (y x) x s a y1 (y 1 a (evident y 6= 0). Deducem y = nlocuind din nou x2 gsim x = y y2 i, n final, P (y) = y 3 2y 2 + 3y 1. Folosind irul lui Rolle, polinomul P s s o singur rdcin real y (0, 1). a a a a a XII.45. Fie S5 . Sa se arate ca 2 are puncte fixe daca si numa are puncte fixe. Paul Georgescu i Gabriel P s Soluie. Folosind descompunerea unei permutari n produs de cicli t se poate observa ca, daca k , 1 k 5, are puncte xe, atunci descompunerea sa cicli de lungime l, cu l | k. Presupunem c 2 are puncte xe. Atunci conine n descompunerea a t lungime 1 (puncte xe) sau 2. Daca are puncte xe, 3 are de asemen xe. Daca conine n descompunerea sa cicli de lungime 2, fara a avea p t atunci conine n descompunerea sa i un ciclu de lungime 3. Element t s ciclu vor toate puncte xe pentru 3 . Implicaia reciproc se demonstrea t a

5 Sa se arate ca exista c , astfel nct f (c) (1, 2). 4

Soluiile problemelor pentru pregtirea concur t a din nr. 2 / 2003


A. Nivel gimnazial

G46. Determinai ultimele cinci cifre ale numarului t 2000 2001 2002 A=7 +7 +7 + 72003 . Viorel Cornea i Dan Stefan Marinescu, H s Soluia I. Scriem A = 72000 400. Se constant c 720 = M1000 + 1. C t a a 72000 = M1000 + 1 i A = (M1000 + 1) 400 = M100000 + 400, deci ult s cifre ale lui A sunt 00400. Soluia II (Irina Mustaa, eleva, Iai). Conform teoremei lui Eule t t s 1(mod n) pentru (a, n) = 1. Considernd a = 7, n = 1000, avem (1000) = 5 7400 1(mod 1000), adic 72000 = 7400 1(mod 1000), de unde A = 72 a 400(mod 100000), deci numrul A se termin cu 00400. a a G47. Determinai valorile parametrilor a, b Z pentru care soluiile s t t y x x=a ; y=b y+1 x+1 sunt n Z Z. Temistocle B Soluie. Fie M mulimea perechilor (a, b) Z Z pentru care este t t cerina problemei. Observm c (0, 0) Z Z este soluie a sistemului i t a a t s este unica soluie daca a = 0 sau b = 0. Ca urmare, (a, 0) M , a Z i ( t s b Z. Pe (R \ {1}) (R \ {1}) sistemul dat, dac facem abstracie de sol a t este echivalent cu (b + 1) x = ab 1, (a + 1) y = ab 1. ab 1 ab 1 I. a 6= 1, b 6= 1. Obinem x = t ,y= . Atunci (x, y) b+1 a+1 b + 1 | ab 1 i a + 1 | ab 1. Cum ab 1 = (a + 1) (b + 1) (a + 1) s deducem c (x, y) Z Z b + 1 | a + 1 i a + 1 | b + 1 b + 1 = a s b + 1 = (a + 1). Prin urmare (a, a) M , a Z, a 6= 1 i (a, a s Z, a 6= 1. II. a = b = 1. Cum rezult ab 1 = 0, sistemul (1) are o infinitate d a nu toate sunt n Z Z. Deci (1, 1) M . / III. a = 1, b 6= 1 (analog a 6= 1, b = 1). Avem ab 1 6= 0, d are soluii, iar sistemul dat are (0, 0) ca unic soluie. Aadar, (a, 1) M t a t s a 6= 1 i (1, b) Z, b Z, b 6= 1. s n concluzie: M = {(a, 0) , a Z} {(0, b) , b Z} {(a, 1) , a Z} {(1, b) , b {(a, a) , a Z} {(a, a 2) , a Z} \ {(1, 1)} . 2002 G48. Fie A (0, ) o mulime care conine t t si avnd propr 2003 a a+1 a daca A (a, b N ), atunci A si A. Sa se arate ca A b b 2b Gheorghe Iu

Soluie. Considerm transformrile t a a a+1 a A A b b i s a a A A. b 2b

Avem: 2002 (2) 1001 (1) 1002 (2) 501 (1) 502 (2) 251 (1) ( A A A A A A 2003 2003 2003 2003 2003 2003

G49. Fie x1 , x2 , . . . , xn+1 R astfel nct x1 + x2 + . . . + xn+1 + n+4 2 2 2 2 iar x1 + x2 + . . . + xn+1 M , unde m = min xi , M = max xi . S 4 ca exact n dintre numerele date sunt egale. Eugen Je Soluie. Daca toate numerele ar fi egale, relaia a doua din en t t n+4 2 t (n + 1) M 2 M , imposibil. Exista cel puin un numar egal cu M , 4 xn+1 . Deducem ca (n + 2) m x1 + + xn + M deci 2m M . De n+4 2 2 a M x1 + + x2 + M 2 nm2 + M 2 sau M 2m. Rezult M n 4 prima relaie devine x1 + x2 + +xn +2m (n + 2) m, deci x1 +x2 + + t sau nca (x1 m)+ +(xn m) 0. Cum x1 m 0, x2 m 0, . . . , x deducem ca x1 = x2 = = xn = m, valori care verica i a doua condii t s n concluzie exact n numere sunt egale cu m i unul egal cu M = 2m. s

2 1 1 (1) (1) (1) 2003 A A A, deci A. 2003 2003 2003 1 1 a b 1 a a Fie Q . Cum A A A A, rezulta ca + b 1 b b b b Q A. +

Ovidiu Pop, Sa 1 = 2 = = Soluie. Pentru n = 0 relaia din enun devine t t t a 1 = 1, echivalent cu 2 a < 5, i cum a N, a 3, avem a s deci Reciproc, arat ca, daca {3, 4}, au loc egalitaile din enun. F t am a t trebuie artat c a a 3n + 1 = 3n + 2 , n N. Pentru un numr n N exist k N (k depinde de n) astfel nct k2 a a < (k + 1)2 . Considernd numarul 3n+2 avem 3n+2 < (k + 1)2 sau 3n+2 = Cum 3n + 2 nu este ptrat perfect, deducem c k2 3n + 1 < 3n 2 < (k a + a unde rezulta k 3n + 1 < 3n + 2 < k + 1, deci 3n + 1 = 3n + 2 4n + 1 = 4n + 2 = Pentru a = 4, trebuie demonstrat c a 2 n N. Considernd k = 4n + 1 avem k 2 4n + 1 < (k + 1) i cu s 2 4n+3 nu sunt ptrate perfecte a pentru nici un n N, deducem k 4n+1 < 4n + 3 < (k + 1)2 i atunci 4n + 1 = 4n + 2 = 4n + 3 = k. s

G50. Fie a N, a 3. Sa se arate ca an + 1 = an + 2 = . . . = an + a 1 , n N a {3

3 G51. Fie a, b, c , cu a + b + c = 1. Sa se arate ca 10 3 2 a a + bc + b b + ca + c c + ab < . 3 4 Gabriel Dospinescu, ele p a+b+a+c 1+a = Soluia. Avem a + bc = (a + b) (a + c) t 2 2 X a2 + a X X 1 2 = 1+ a . Cum a = 1 (b + c) < 1 a a + bc 2 2 X 1 1 4 1 37 3 etc., rezulta ca 1+ a2 < 1+3 = < . 2 2 25 2 25 4 Pentru inegalitatea din stnga avem: a + bc = (a + b) (a + c) a + X X X X a a + bc = 1 2 ab + a bc. Rmne a atunci a a + bc > 2 X (a b) X X X 1 a c 2 a ab+ a bc 0 sau, echivalent, c 3 3 2 2 (a b) a b , echivalenta cu a+ suficient sa aratam ca c 3 sau cu a + b + 2 ab 3 3a 3b. Ultima inegalitate se scrie 4a + 4b + 2 3 este adevarata n condiia a, b t , . 10 G52. Se considera o piramida formata din patrate 1 1, avnd n trepte, pe treapta k existnd 2k 1 patrate (n figura, n = 4). Aflai numarul minim de dreptunghiuri, fiecare alcatuit t numai din casue ntregi, n care poate fi mparita tabla. t t Adrian Zahariuc, ele Soluia I (a autorului). Colorm csuele tablei alternativ n al t a a t (csua din vrf este neagr). Cum modulul diferenei dintre numrul de c a t a t a i numrul de csue negre dintr-un dreptunghi este cel mult 1 i cum s a a t s casue negre mai multe dect albe, trebuie sa avem cel puin n dreptungh t t numr poate fi obinut tind pe nivele. Aadar, rspunsul este n. a t a s a Soluia II (Irina Mustaa, elev, Iai). Unim, ca n figut t a s ra, vrfurile din stnga sus ale patratelor marginale din stnga; se observa ca deasupra acestei drepte nu mai exista vrfuri ale piramidei. Oricare dou ptrate marginale din stnga nu a a pot aparine aceluiai dreptunghi, deoarece ar nsemna c acel t s a dreptunghi va avea colul din stnga sus deasupra dreptei considerate; pr t numarul minim de dreptunghiuri este n, minim atins pentru mparirea p t Soluia III (Marius Pachiariu, elev, Iai). Prin inducie complet t t s t a G53. FieABCD un patrat de latura 70. Sa se arate ca exista o m patrate Pk = Ai Bi Ci Di | Ai Bi = i, i = 1, k care sa aiba suma ariilor eg patratului dat. Putem acoperi patratul ABCD cu elementele mulimii Pk t Petru Asa k(k+1) Soluie. Trebuie s avem 12 +22 + +k2 = 702 , echivalent cu t a 6

= 702 , de unde k = 24. Soluia este unic deoarece k > 24 implic 12 +22 + t a a > 702 iar k < 24, 12 + 22 + + k 2 < 702 . Aratam ca mulimea Pk nu poate acoperi patratul ABCD. Pentru a t observm c ptratele Ai Bi Ci Di , pentru a acoperi Pk , nu au puncte comun a a a fcnd laturile (altfel aria acoperit de acestea este mai mic dect 12 +22 + a a a = 702 ); de asemenea, patratele Ai Bi Ci Di nu lasa "spaii goale" ntre ele t acoperita de ele plus ariile "spaiilor goale" este mai mare ca 12 +22 + +2 t Analiznd poziia ptratului de latur 1 n ptratul ABCD, observm t a a a a o suprafaa ce nu mai poate fi acoperit (rmne o suprafaa dreptung t a a t laturi 1 i l 1 n care nu "ncape" nici unul din patratele ramase, care s mai mari sau egale cu 2). G54. Sa se arate ca nu putem alege nici un punct n interiorul tr echilateral ABC de latura l 10, care sa aiba distanele la vrfuri num t distincte. Doru Bu Soluie. t Presupunem, prin absurd, c exist a a A M Int (ABC) astfel nct M A, M B, M C s fie nua mere prime. Cum M A, M B, M C < l i M A, M B, M C pot fi s 7 laturile unui triunghi (teorema lui Pompeiu), deducem c AM, BM, CM {3, 5, 7}. Fie AM = 7, BM = 3, a M 3 CM = 5. \ Fie N astfel nct BN = 3 i m(M BN ) = 60 . Evi- B s dent, triunghiul BM N este echilateral. Din congruena t 3 triunghiurilor AM B i CN B deducem N C = 7. Atunci s N M N 2 +M C 2 N C 2 1 \ \ \ s cos N M C = = , deci m(N M C) = 120 i m(BM C 2 MN MC 2 fals. Prin urmare, nu exista M Int (ABC) cu proprietatea ceruta. G55. Printr-un punct situat n interiorul unui tetraedru se duc planele feele tetraedrului. Daca V1 , V2 , V3 , V4 sunt volumele tetraedrelor unic de t de aceste plane, iar V este volumul tetraedrului dat, sa se arate ca 27V 16 (V1 + V2 + V3 + V4 ) . Neculai Roman, Mirce Soluie. Notm cu xi distana de la punctul considerat la faa te t a t t A1 A2 A3 A4 opusa vrfului Ai i cu hi nalimea corespunzatoare acestei s t p hi xi xi ai = 3 Vi /V , i = 1, 4. Avem ai = = 1 , i = 1, 4, dec hi hi 4 4 4 P xi P Si xi 1 P =4 =4 Vi = 4 1 = 3. =4 V i=1 i=1 hi i=1 Si hi Relaia de demonstrat se scrie 27 16 a3 + a3 + a3 + a3 i decurge t 1 2 3 4 s 3 a3 + a3 + a3 + a3 33 27 a1 + a2 + a3 + a4 2 3 4 c 1 a = 3 = . 4 4 4 64

B. Nivel liceal

L46. Fie ABCD un patrulater inscriptibil. Bisectoarele unghiurilor

intersecteaza ntr-un punct situat pe latura [CD]. Sa se arate ca CD = A Mircea Becheanu, B b Soluia I (Irina Mustaa, eleva, Iai). Fie m(A) = 2, D t t s E b = 2, iar P (CD) astfel nct AD = DP ; avem ca m(B) 1 b \ a 180 m(D) = . Deosebim dou cazuri, m(AP D) = 2 dup cum E (DP ) sau E (P C); ne plasm n prima a a [ \ situaie. Deoarece EP A EBA, patrulaterul ABP E este t \ \ inscriptibil, deci m(CP B) = . Atunci m(CBP ) = 180 A b = , deci 4CP B este isoscel cu CP = CB, de unde concluzi m(C) Soluia II. Folosind teorema sinusurilor n triunghiurile ADE, BEC t obinem: t B A sin sin 2 2 AB, CE = AB DE = B A A+B A+B 2 cos sin 2 cos sin 2 2 2 2 A B sin B sin A 2 2 AD = AB, BC = AB B A A+B A+B 2 cos sin 2 cos sin 2 2 2 2 din care deducem sin A + sin B CD = DE + CE = AD + BC = AB. A B A+B 4 cos cos sin 2 2 2 Not. S-a mai primit soluie corecta de la Marius Pachiariu, elev, a t t L47. Daca un triunghi are patratele laturilor n progresie aritmeti simetricul centrului de greutate faa de latura mijlocie se afla pe cercul c t triunghiului. Gabriel Popa i Paul Georg s Soluie. Fie a, b, c lungimile laturilor triunghiului ABC, c < a < b cu 2a t Simetricul cercului circumscris triunghiului ABC faa de BC este cerc t scris triunghiului BHC i atunci simetricul lui G faa de BC este pe cerc s t scris daca i numai daca BHGC este patrulater inscriptibil, echivalent cu s b \ = m(BHC) = m(A). \ m(BGC) a2 \ n triunghiul BGC se determina cos BGC = (se folosete relaia 2 s t 2bc 2 a \ i formula medianei). Cum cos A = s , deducem c avem cos BGC = a 2bc b \ m(BGC) + m(A) = , ceea ce ncheie soluia. t Not. Soluii corecte au dat Irina Mustaa i Marius Pachiariu, a t t s t L48. Fie R, r, R1 raza cercului circumscris 4ABC, raza cercului nscr respectiv raza cercului circumscris 4DEF determinat de picioarele bisect terioare ale 4ABC. Sa se arate ca R/2 R1 r. Marian Tetiv Soluie (Titu Zvonaru, Comneti (Bacu)). Inegalitatea r R1 t a s a varata oricare ar fi punctele D (BC), E (CA), F (AB) (Liliana N

L49. ntr-un patrat 10 10 se nscriu numerele 1, 2, 3, . . . , 100 n a s oricare doua numere consecutive sa se afle n casue vecine. Demonstra t t o linie sau o coloana ce conine macar doua patrate perfecte. t Adrian Zahariuc, ele Soluie. Observam ca avem 10 patrate perfecte dintre care 5 sunt p t supunem ca patratele perfecte sunt situate pe linii i coloane diferite. Colo s ca pe o tabl de ah n alb i negru. Numerele pare vor fi situate pe csu a s s a t culoare, la fel numerele impare. Fie (x1 , y1 ), (x2 , y2 ), . . . , (x10 , y10 ) coordonatele casuelor n care su t ptratele perfecte. Datorit presupunerii c ptratele perfecte sunt pe linii a a a a diferite, numerele xi , i = 1, 10 snt diferite dou cte dou i {x1 , x2 , . a a s = {1, 2, . . . , 10}; la fel pentru numerele yi , i = 1, 10. Obinem (x1 + y1 ) + ( t + + (x10 + y10 ) = 2 (1 + 2 + + 10) = numar par i rezulta ca un num s perechi are suma par. Cum perechile cu suma combinaiilor par au acea a t a deducem c exist un numr par de ptrate perfecte pare, absurd. a a a a Not. Soluie corect a dat Marius Pachiariu, elev, Iai. a t a t s

O metoda de demonstrare a unor inegalitati geometrice, GM - 2-3/1993; T p. 51). ntr-adevar, fie Q centrul cercului circumscris 4DEF i d1 , d2 , d3 s de la Q la laturile BC, CA, AB. Cum fiecare dintre dreptele BC, CA s secant sau tangent cercului circumscris 4DEF , avem d1 R1 , d2 R1 a a deci ad1 bd2 cd3 aR1 bR1 c pr = A[ABC] = A[QBC] +A[QCA] +A[QAB] = + + + + 2 2 2 2 2 adica r R1 . Inegalitatea R1 R/2 este adevrat, dac punctele D, E, F aparin se a a a t determinate de picioarele nlimilor i mijloacele laturilor respective (a at s ntmpl cu picioarele bisectoarelor). n acest caz aceste puncte sunt n a cercului lui Euler al 4ABC, a carui raza este R/2, i rezulta R1 R/2. s

L50. Fie (an )n1 o progresie aritmetica avnd a1 = 5, r = 2002. element b al progresiei, sa se arate ca bm aparine progresiei daca si n t 60 | m 1. Mihai Piticari, C-lung Mol Soluie. S demonstrm mai nti urmtoarea t a a a

Lem. Daca a, m, n sunt numere naturale nenule astfel nct n | am a este cel mai mic numar cu aceasta proprietate, atunci n | ak 1 daca si n m | k. ntr-adevr, dac m | k, atunci k = sm i din n | am 1 | asm a a s n | ak 1. Presupunem n | ak 1. Conform teoremei mpririi cu r at c i r numere naturale astfel nct k = mc + r, r < m. Din n | ak s n | amc+r 1 = ar (amc 1) + ar 1 i deducem n | ar 1. Cum r < m s minimalitii lui m rezult r = 0, deci m | k. at a

t Revenim la problema data. Evident, bm aparine progresiei daca i n s 2002 | bm b. Cum b = 5+2002p, rezult c 2002 este prim cu b i atunci b a a s progresiei daca i numai daca 2002 | bm1 1. Din bm1 1 = (5 + 20 s

5m1 + 5m1 1, urmeaz c 2002 | bm1 1 dac i numai dac 2002 | a a as a Avem 2002 = 2 7 11 13 i 6, 10, 4 sunt minime cu proprietaile 7 s t 11 | 510 1, 13 | 54 1. Deci, pe baza lemei, 2002 | 5m1 1 daca i n s 6 | m 1, 10 | m 1, 4 | m 1, adic dac i numai dac 60 | m 1. a as a Not. Soluie corect a dat Marius Pachiariu, elev, Iai. a t a t s L51. Fie A, B M2 (R) doua matrice care comuta si pentru care det A < (det A + det B)2 . Sa se arate ca xA + yB este matrice nesingulara, x Ctlin Cali a a Soluie. Se arat cu uurina c t a s t a prin urmare det (xA + yB) = x2 det A + y 2 det B + xy [det (A + B) det A det B] ,

L52. Fie Q C [X] un polinom de grad m avnd radacinile distinc determine cardinalul mulimii t

Condiia din enun se rezum la a arta c x, y R , avem det (xA + yB t t a a a ecuaia x2 det A + y 2 det B + xy [det (A + B) det A det B] = 0 nu adm t reale nebanale. ntr-adevr, discriminantul acestei ecuaii este a t 2 = [det (A + B) det A det B] 4 det A det B = det A2 + B 2 2 (det A det B) 4 det A det B = det A2 + B 2 (det A + det B)

Cum AB = BA, avem (A + iB) (A iB) = A2 + B 2 i relaia anterioar s t a 2 2 2 det A + B = (det A det B) + (det (A + B) det B det A)

det (A + iB) det (A iB) = (det A det B)2 + [det (A + B) det A d

Ovidiu Munteanu Soluie. Fie P E. Exist A Mn (C), Q (A) = On i P (x) = det t a s Fie C o radacina a lui P . Rezulta ca sistemul AX = X are i o soluie s t notat X0 . Este uor de vzut c Q (A) X0 = Q () X0 i prin urmare Q ( a s a a s rdcinile lui P sunt n numere din mulimea {1 , 2 . . . m }, unde i sunt a a t lui Q. Prin urmare, P este determinat de n numere, nu neaprat distinc a . . . , n din mulimea {1 , 2 , . . . , m }. t Invers, dnd un polinom P care are ca rdcini n numere 1 , 2 , . a a mai sus, fie A matricea care are pe diagonala principal 1 , 2 , . . . , n a s Evident, det (xIn A) = (x 1 ) (x n ) = P (x), iar Q (A) este care are pe diagonala principala Q (i ) = 0 i 0 n rest, deci Q (A) = On . s P E. Prin urmare, numrul de elemente al lui E este egal cu numrul de p a a de a alege n numere oarecare dintr-o mulime cu m elemente, fr a cont t aa adic Cm . a n L53. Fie n 2 si (A, +, ) un inel comutativ cu n2 elemente, care ar n 2 divizori ai lui zero. Sa se arate ca A este corp. Gabriel Dospinescu, ele Soluie. Presupunem c A nu este corp. Fie T mulimea divizorilo t a t urmeaza ca T 6= . Presupunem ca T are k elemente, 1 k n 2. C

E = {P C [X] | A Mn (C) a. . Q (A) = On si P (X) = det (XIn

x T i a A \T . Exist d 6= 0 nct xd = dx = 0. Atunci (ax) d = d (ax s a ax = 0 sau ax T . Daca ax = 0, avem a T , fals. Deci ax T i prin urm s defini f : A \T T , f (a) = ax. Cum A \T are n2 k1 elemente, T are n2 k 1 i > n, exist elementele diferite a1 , a2 , . . . , an+1 A \ T a a s k f (a1 ) = f (a2 ) = = f (an+1 ). Deducem ca (an+1 ai ) x = x (an+1 i = 1, n. Deci an+1 ai T pentru i = 1, n. Aadar T are cel puin n s t contradicie. t L54. Fie f : R R o funcie cu derivata continua pentru care f (x) 6= t Sa se determine funciile continue : R R care satisfac identitatea t Z y Z x 1 1 f (x) (t) dt (y) = f (y) (t) dt (x) , x, y a a 0 0 unde a 6= 0 este o constanta data. Adrian Cordune Soluie. Relaia dat pote fi scris sub forma t t a a Rx Ry 1 1 (t) dt (y) (t) dt (x) 0 0 a a = , x, y6=0. f (y) f (x) Rx 1 Z x (t) dt (x) 0 1 a (t) dt (x) Prin urmare, = c, c R, deci f (x) a 0 egalitatea avnd loc i pentru x = 0, n baza continuitaii. Pentru x = s t (0) = acf (0). 1 Prin derivare obinem ecuaia (x) 0 (x) = cf 0 (x), ce a t t a Z x eat (acf 0 (t)) dt , unde k = (0) = acf (0) (x) = eax k + 0 Z x (x) = aceax f (0) + eat f 0 (t) dt , c R.
0

a1 x0 = . Pentru a (0, 1) avem xn a xn ax xn , x [0, 1]; ln a Z 1 a 1 obinem t ax xn dx . Deci lim xn = 0. Pentru a ( n n+1 n+1 0 1 a xn , n N. C xn xn ax xn a, x [0, 1], deducem n+1 n+1 lim xn = 0.
n

a1 L55. Fie a (0, ) \ {1}. Definim sirul (xn )n1 prin x0 = ; x ln a n t xn1 , n 1. Aratai ca sirul este convergent si calculai lim xn si t n ln a Gheorghe Iu Z 1 Soluie. Prin inducie matematic se arat c xn = t t a a a ax xn dx,
0

n concluzie, pentru orice a (0, ) \ {1}, lim xn = 0. Din relaia de t n rezult lim nxn1 = a, de unde deducem c lim nxn = a. a a
n n

Probleme propuse1
Clasele primare

P.74. Descopera regula de formare, apoi completeaza irurile urmatoa s a) 1,2,3; 2,3,5; 3,,; 5,,. b) 11,10,12; 13,12,14; 15,,; 17,,. c) 2,6,4; 3,7,5; 4,8,6; 5,,; 6,,. ( Clasa I ) nv. Maria R P.75. Rspundei la urmtoarele ntrebri: a t a a a) De cte suprafee este mrginit cubul? t a b) Ce forma au feele cuboidului? t c) Ce forma are un obiect care se aseamana cu sfera? ( Clasa I ) Aliona Loghin, e P.76. Completai casetele din expresia 654321 cu semnele g t sau "" pentru a obine cel mai mic rezultat posibil. t ( Clasa a II-a) nv. Gheorghe Toma, Muncelu de S P.77. Un corp este format din trei cuburi a, b, c ca n 4 5 figura alturat. Fiecare cub are feele numerotate de la a a t 1 la 6, iar suma numerelor de pe oricare dou fee opuse a t 2 4 ale sale este 7. Stiind c pe feele lipite ale cuburilor a i a t s b este scris acelai numar i ca aceeai proprietate o au i s s s s a b cuburile b i c, s se afle suma tuturor numerelor scrise pe feele corpului s a t vd. a ( Clasa a II-a) Oxana Pascal, e P.78. a) Verific egalitile: 1 + 3 + 5 + 7 = 4 4, 1 + 3 + 5 + 7 + 9 + 1 a at b) Scrie rezultatul la fel ca la punctul a) pentru 1 + 3 + 5 + 7 + + 1 ( Clasa a III-a) Andreea Surugiu, stud P.79. 7 elevi mnc 7 ingheate n 6 minute. Ci elevi vor mnca 24 a a t t n 36 minute? ( Clasa a III-a) Alexandru Tudorache, P.80. Dou orae sunt legate printr-o linie de cale ferat. La fiecare a s a un tren din fiecare ora ctre cellalt. Toate trenurile merg cu aceeai vitez s a a s calatorie de la un ora la altul dureaza 6 ore. De cte ori fiecare tren, car s distana dintre orae, se ntlnete cu trenuri care merg n sens opus? t s s ( Clasa a IV-a) Alexandru Tudorache, P.81. Sa se arate ca din feele unui cub confecionat din carton putem t t fr resturi, feele a ase cuburi. aa t s ( Clasa a IV-a) Petru Asa P.82. Sa se afle cel mai mare numar natural de forma abcd cu pro a 6= d, b + c = 5 (a + d). ( Clasa a IV-a) Adrian Andronic, P.83. Mircea mpreuna cu fratele sau au un numar de bomboane mai 30. Mircea are de 3 ori mai multe dect fratele su. Aflai cte bomboane a t
1

Se primesc solu ii pn la data de 1 iunie 2005. t a

i dea Mircea fratelui su pentru a rmne cu un numr de dou ori mai m a a a a al fratelui. Cte bomboane avea Mircea la nceput i cu cte a ramas? s ( Clasa a IV-a) Inst. Tudor Tudorache

Clasa a V-a

V.51. ntre oricare doua numere naturale definim operaia a b = ab t a) Sa se rezolve ecuaia 2 (x + 1) = 34. t b) Este operaia dat comutativ? t a a Vasile Solcanu, Bogdneti ( a s V.52. Un dreptunghi se poate descompune n 1344 patrate de arie 25 c perimetrul dreptunghiului daca acesta este: a) maxim posibil; b) minim p Romana Ghia i Ioan Gh t t s V.53. Determinai n N pentru care t 3 1 1 1 + + + n 2 + 6 + + 98 2 3 3 = . 1 1 5 1 + 3 + + 17 1 + + + n 4 5 5 Viorel Cornea, H V.54. S se arate c nu exist numere raionale pozitive a, b, c a a a a t b+c c+a a+b s = 22003 , = 22004 i = 22005 . ab bc ca Andrei - Sorin Cozma, 2 3 2004 1 + + + + , unde V.55. Fie numrul raional N = a t a1 a2 a3 a2004 1 i = 1, 2004. Sa se arate ca exista a1 , a2 , . . . , a2004 astfel nct N = 200 ralizare. Petru Asa

Clasa a VI-a

VI.51. Pentru efectuarea unei lucrri, trei muncitori au fost retribui a de bani direct proporionale cu numerele 16, 14, 17. Unul dintre muncitor t ca daca sumele primite ar fi fost invers proporionale cu numerele 3, 4, t primit mai puin cu 1000000 lei. Aflai ce sum de bani a primit fiecare m t t a Ion Vian s

VI.52. Determinai numerele ntregi n care pot fi scrise sub forma n = t cu a, b, c Z . Gheorghe Iu d i punctele A, B (Ox, C, D VI.53. Se da unghiul ascuit xOy s t nct A (OB), C (OD), AB 6= CD i t OA + s AB = t OC + s s, t R . Atunci mediatoarele segmentelor [AB] i [CD] i bisectoarea lui s s trei drepte concurente daca i numai daca t = 2s. s Ioan Scleanu a a VI.54. Fie 4ABC isoscel (AB = AC), N mijlocul lui [AC], iar D un prelungirea lui [BC] astfel nct CD < BC. S se arate c ntre triunghiu a a i N CD nu exist nici o congruena. s a t Romana Ghia i Ioan Gh t t s

VI.55. Fie punctele O, A1 , A2 , A3 , . . . astfel nct OA1 = OA2 = OA \ \ \ = 1 cm, iar m(A1 OA2 ) = 1 , m(A2 OA3 ) = 2 , m(A3 OA4 ) = 3 etc. (to urile se consider n sens orar). S se arate c exist k 6= l astfel nct Ak a a a a Cristian L

Clasa a VII-a

VII.51. Fie a, b N astfel nct n2004 a se divide cu n b, pentru or n 6= b. Sa se arate ca a = b2004 . Alexandru Negrescu, elev, VII.52. Fie a, b, c R cu a + b + c = 0; s se arate c a a 3 2 3 3 3 3 3 3 3 3 a b + b c + c a = 3(a b)(b c)(c a) a bc b2 ac Anca Tuescu, elev t a VII.53. Determinai m, n, p Z astfel nct soluia inecuaiei |mx t t t fie [p, p + m + 1]. Ciprian Ba d de masura 10 i un segment [M N ] d VII.54. Se considera unghiul xOy s a. S se construiasc, folosind numai rigla i compasul, un triunghi dr a a s OAB, A (Ox, B (Oy, avnd o catet de lungime a. a Florin Asvoaie, a VII.55. Fie ABCD patrulater convex, iar {O} = AC BD. Bisec \ \ \ \ terioare ale unghiurilor AOB, BOC, COD, DOA taie laturile (AB), (B respectiv (DA) n M , N , P , respectiv Q. Sa se arate ca dreptele M Q, N sunt concurente sau paralele. Constantin Cocea i Dumitru Ne s

Clasa a VIII-a

VIII.51. Se considera funciile f, g, h : R R definite prin f (x) = t x = , h (x) = 3. Notm {A} = Gg Gh , {B} = Gf Gh , iar C i D sunt p s a 3 t intersecie ale dreptei x = 2 cu Gf , respectiv Gg . Determinai masurile u t perimetrul i aria patrulaterului ABCD. s Dumitru - Dominic Buc 2 2 VIII.52. Fie E (x, y) = 2004 2x 5y + 2xy + 6y, cu x, y N. D valoarea maxima a lui E. Gheorghe Iu VIII.53. S se arate c pentru orice a, b, c R, are loc inegalitatea a a a4 + b4 + c4 + 3a2 b2 + 3a2 c2 + 3b2 c2 2 a3 b + ab3 + a3 c + ac3 + b3 c + Marian Tetiv VIII.54. Pentru a, b, c (0, ), s se demonstreze inegalitatea a p p p 2a b2 + c2 2 + c2 2 + b2 + c2 < a+ b a < a + b2 + c2 a2 + b2 + c2 Radu Frunz i Mircea Cobuc, e as s VIII.55. O piramida triunghiulara regulata este tetraedru regulat dac dac unghiurile fcute de o faa lateral cu planul bazei, respectiv cu o a a t a lateral, sunt congruente. a Claudiu - Stefan P

Clasa a IX-a

IX.51. Fie irul (an )n1 definit prin: a1 = 1 + 2 3; a2 = a1 + 4 + 5 + s a3 = a2 + 9 + 10 + 11 + 12 13 14 15 etc. a) S se determine semnele cu care apar 100 n a100 , respectiv 91 n a9 a b) S se afle formula termenului general al irului. a s Lidia Nicola IX.52. S se determine funciile f, g : Z R cu proprietile: f (0 a t at f este par, g este a impar i exist a, b N astfel ncat f (x) = f x2 a s a 2 g (x) = g x + x + b , x Z. D. M. Btineu-Giurgiu, B a t IX.53. Exista funcii f : R R pentru care t |f (x + y + z + t) + cos x + cos y + cos z + cos t| < 4, x, y, z, t

Lucian Tuescu t [ IX.54. Fie ABCD un patrat de latura a, iar T (AD) astfel nct m(A Notam {S} = AC BT i fie R punctul n care perpendiculara n S pe s secteaz AB. a a) S se arate c 4RST este isoscel. a a b) Sa se exprime RS funcie de a i . t s Gheorghe Costo IX.55. Fie ABC un triunghi cu c < b. Notm cu M i N mijloacel a s [AB], respectiv [AC] i cu D i E punctele de tangena a cercurilor nscris s s t s A- exnscris triunghiului cu latura [BC]. Aratai ca t (i) M E i N D se intersecteaza pe mediana din vrful A; s (ii) M D k N E a = 2 (b c); (iii) dac a 6= 2 (b c), atunci M D i N E se intersecteaz pe prelungirea a s a din A. Temistocle B

Clasa a X-a

S se arate c a a

X.51. Fie OABC un tetraedru cu OA OB OC, circumscris un 3+1 R raz r. Dac R este raza cercului circumscris 4ABC, atunci a a r 2 Cezar Lupu, elev, C X.52. Fie polinomul X 6n+1 12n+2 = ak X k , n N. P (X) = 1 + X + X 2
k=0 2n P

a6k =

Ctlin Cali a a X.53. Fie a, b, c (1, ) astfel nct a + b + c = 9. S se arate c a a 3 3 3 3 3 3 loga 2b + c + logb 2c + a + logc 2a + b 12. a Angela Tigeru X.54. Definim mulimile Ak , k 1, prin t n n o A1 = | n = 1, 2, . . . , 10000 \ {1} ; Ak = (Ak1 \ {a, b}) a 2003

k=0

2n P

k=0

a6k+2 .

cu a, b Ak1 arbitrare, k 2. S se determine A9999 . a Marius Pachiariu, t 2 X.55. Fie a, b Z cu a 4b < 0, iar o soluie a ecuaiei x2 + ax t t Definim funcia f : Z Z Z, f (x, y) = x2 axy + by 2 . Pentru ori t 1 (x, y) f 1 (1), sa se arate ca (x + y)card f (1) = 1. Andrei Ned

Clasa a XI-a

S se calculeze limita irului (an )n3 , an = a s

XI.51. S se calculeze determinantul unei matrice ptratice de ordinul a a are toi minorii de ordin trei egali. t Lucian - Georges Ldu a 2 XI.52. Fie funcia f : M2 (R) [0, ), f (A) = det A + I2 , A t 2 2 a) S se arate c f (A) = (det A 1) + (tr A) , A M2 (R). a a b) Sa se demonstreze ca f este surjectiva, dar nu este injectiva. Ovidiu Pop, Sa XI.53. Fie R; pentru n 3, definim cos + 2k cos sin + 2k sin n n k = , k 2(k+1) cos sin + 2(k+1) sin cos + n n
n2 P k=1

|k |.

Gheorghe Croitoru i Gabriel P s n+k n n1 t x x x XI.54. Fie k N ; sa se arate ca ecuaia x o singura soluie pozitiva, pe care o notam xn . Sa se arate apoi ca irul (x t s convergent; ce se poate spune despre limita sa? Dumitru Mihalache i Marian Tetiv s XI.55. Determinai toate funciile f : R R pentru care t t 2n+1 f x + x x f 2n+1 (x) + f (x) , x R,

unde n N. (n legtur cu problema 2811 din Crux Mathematicorum, n a a Titu Zvonaru, C Z


1

Clasa a XII-a
XII.51. S se arate c irul (an )n1 , an = a as

sin x dx, p < 2, este c p 1/n x Rodica Luca Tudora XII.52. Fie f : [0, 1] R derivabil, cu derivata continu, astfel nct a a f (x) + f 0 (x) = 0, x [0, 1]. S se arate c a a Z Z 1 (2e 5) f (1) 4 2 xex f (x 1) f (x) dx dx. + 5 e 1 (x2 + 1)2 0 Mihail Bencze XII.53. Prove that Z x t 2 (ex 1) e dt x, x 0. ex + 1 et + 1 x Zdravko Starc, Vrsac, Serbia and Mo

unde este constant, iar b = b (t) este continu pe [0, a]. a a Adrian Cordune

XII.54. S se afle funciile continue u = u (t), soluii ale ecuaiei a t t t Z t Z a u (t) = + b (s) u (s) ds + b (s) u (s) ds, 0 t a,
0 0

XII.55. S se arate c pentru orice n N , exist monoizi care nu su a a a i care conin exact n elemente inversabile. s t Paul Georgescu i Gabriel P s

Probleme pentru pregtirea concursurilo a


A. Nivel gimnazial

G66. Se considera mulimea A = {1, n + 1, 2n + 1, . . . , mn + 1}, m t m > n. S se afle cte valori distincte poate lua suma a1 + a2 + + a a1 , a2 , . . . , an A. Petru Asa

G67. Fie b N, b 2. Spunem ca un numar natural este decompozab poate scrie ca suma a dou numere cu aceei sum a cifrelor n baza b. S a s a c exist o infinitate de numere care nu sunt decompozabile. a a Adrian Zahariuc, ele

G68. Fie N N ; s se arate c exist n N astfel nct factorialu a a a numr natural s nu se termine cu n, n + 1, . . . , n + N zerouri. a a Iuliana Georg

G69. Fie E (x) = ax2 + bx + c, a, b, c Q, x R. Dac a + b + c Z, a exist o innitate de numere ntregi n astfel nct E (n) s e numr ntre a a a Gheorghe Iu

G70. S se arate c ecuaia x2 + y 2 + 3x + y 707 = 0 nu are soluii a a t t Dan Popescu

a G71. Fie (m, n) N2 \ {(0, 0)}. S se demonstreze inegalitatea a b c + + (m + n) a2 + mb2 + nc2 (m + n) b2 + mc2 + na2 (m + n) c2 + ma2 1 1 1 1 + + , a, b, c (0, ) . 2 (m + n) a b c Titu Zvonaru, C

G72. Fie 4ABC circumscris cercului de centru I. Cercul de diam bs b intersecteaza bisectoarele unghiurilor B i C n M , respectiv N . Sa se ara N se afl pe dreapta suport a liniei mijlocii paralele cu BC. a Doru Bu

G73. Fie ABCD un dreptunghi de centru O. Considerm N (AO), M a lui [AD], {P } = M N CD, {E} = OP BC. S se arate c N E BC. a a Andrei Ned

G74. Fie n puncte n spaiu astfel nct oricare patru s formeze te t a volum cel mult 1. Sa se arate ca exista un tetraedru de volum cel mult conina n interior toate cele n puncte. t Tudor Chiril, a

G75. Fie A1 A2 . . . An un poligon regulat de latur 1, n 4. Pe latu a se considera punctul P1 cu P1 A1 = a (0, 1). Din punctul P1 se propaga lumina care se reecta de laturile [A2 A3 ], [A3 A4 ], . . . , genernd pe latur de incidena P2 , P3 , . . . (presupunnd c raza de lumin nu ajunge nicioda t a a 2 \ vrf al poligonului) astfel nct m(A2 P1 P2 ) = , . S se a a n n minima a lui l pentru care Pl i Pl+1 nu aparin la doua laturi consecutiv t s Irina Mustaa, e t

B. Nivel liceal

L67. Dreptele paralele t1 i t2 sunt tangente cercului C de centru O. s de centru O1 este tangent la t1 i C, iar cercul C2 de centru O2 este tange s i C1 ; cele trei cercuri sunt exterioare unul celuilalt. Sa se arate ca unghi s este ascuit i s se afle valoarea maxim a msurii acestuia. t s a a a Neculai Roman, Mirce L68. a) Pentru x, y, z (0, ), sa se demonstreze inegalitatea s r r 1 1 1 x y (x + y + z) + + 1+ + . x y z y x

L66. Fie ABC un triunghi, D i Da punctele n care cercurile n s exnscris sunt tangente la BC i Eb , Fc punctele n care cercurile B-exn s exnscris sunt tangente la AC i respectiv AB. Sa se arate ca punctele D s b Fc sunt conciclice dac i numai dac AB = AC sau m(A) = 90 . as a Temistocle B

b) Folosind eventual a), s se arate c n orice triunghi, cu notaiile u a a t loc inegalitatea s r r pa pb R 1+4 1+ + . r pb pa Marian Tetiv

L69. Pentru ce numere naturale n 3, exista n plan n puncte alb puncte roii, oricare trei necoliniare, astfel nct n interiorul oricrui t s a vrfurile albastre s existe cel puin un punct rou, iar n interiorul oricru a t s a cu vrfurile roii s existe cel puin un punct albastru? s a t Adrian Zahariuc, ele

s L70. Fie k, p N i un dreptunghi de dimensiuni 82k 2p, acoperit fara suprapuneri cu dreptunghiuri 7 5 i 6 4. Sa se arate ca numarul p s (x, y), x par, y impar, ale dreptunghiului mare, care sunt coluri n dre t 75, este egal cu numrul de dreptunghiuri 75. (Prin dreptunghiuri 75 a dreptunghiuri cu lungimea egal cu 7 i limea egal cu 5.) a s at a Marius Pachiariu, t

Gabriel Popa i Paul Georg s s a L74. Fie n N , n 2 i a, b R, a < b. Dac f : [a, b] R este c Rb k x f (x) dx = 0 pentru 0 k n, atunci f are cel puin n + 1 zerouri d t a (a, b). Andrei Ned L75. Sa se determine n N pentru care este adevarata inegalitatea i 1 cos < p , 0, . 8 2 1 + n sin4

unde m, M nu depind de a1 , a2 , . . . , an , valabil pentru orice numere a a an pozitive i cu produsul 1. s Gabriel Dospinescu, B L72. Fie a, b numere raionale, pozitive, distincte, astfel nct an bn t o infinitate de numere naturale n. Sa se arate ca a i b sunt ntregi. s Gabriel Dospinescu, B L73. Fie k N , k 3. S se determine n N \ {0, 1} pentru care a r q a1 + a2 + + ak n a1 a2 . . . ak , a1 , a2 , . . . , ak [0,

L71. Fie n N, n 2 fixat. S se determine cea mai tare inegalitate a n q n X X a2 + n2 1 m ak + M, k


k=1 k=1

Ctlin Cali a a

Training problems for mathematical contes


Junior high school level

G66. Considering the set A = {1, n+1, 2n+1, . . . , mn+1}, m, n N , m the number of distinct values taken by the sum a1+a2+. . .+an , when a1 , a2 , . Petru Asa G67. Let b N, b 2. It is said that G N is decomposable if we can a sum of two numbers such that their expansions in the basis b have the sa digits. Prove that there exist innitely many numbers which are not deco Adrian Zahariuc, high school studen G68. Let N N . Prove that there is n N such that no factorial n + 1, . . . , n + N zeros. Iuliana Georg G69. Let E (x) = ax2 + bx + c, a, b, c Q, x R. If a + b + c is an inte that there exist innitely many integers n such that E (n) is also an integ Gheorghe Iu 2 2 G70. Prove that the equation x + y + 3x + y 707 = 0 has no soluti Dan Pop

G71. Let (m, n) N2 \ {(0, 0)}. Prove that b c a + + (m + n) a2 + mb2 + nc2 (m + n) b2 + mc2 + na2 (m + n) c2 + ma2 1 1 1 1 + + , a, b, c (0, ) . 2 (m + n) a b c

Titu Zvonaru, C

G72. Let I be the incenter of a triangle ABC. The circle with diam b b meets the bisectors of B and C in M , respectively N . Prove that M an the line joining the midpoints of [AB] and [AC]. Doru Bu

G73. Let ABCD be a rectangle with center O. Let N (AO), let midpoint of [AD] and let {P } = M N CD, {E} = OP BC. Prove that N Andrei Ned

G74. Let us consider n points such that any given four are the ve tethraedron with volume at most 1. Prove that there is a tethraedron wi at most 27 which contains all n points in its interior. Tudor Chiril, high school stud a

G75. Let A1 A2 . . . An be a regular n-gon with side 1, n 4. We c on the side [A1 A2 ] such that P1 A1 = a (0, 1). A ray of light is emitte towards and is reected by the sides [A2 A3 ], [A3 A4 ], . . . , generating the points P2 , . . . , Pn (supposing that the ray never meets the vertices A1 , A2 \ 2 ) = , 2 . Find the minimal value of l such t such that m(A2 P1 P n n Pl+1 do not belong to adjacent sides. Irina Mustaa, high school stud t

High school level

L66. Let ABC be a given triangle and let D, Da be the points in incircle, respectively the A-escribed circle are tangent to the side BC. L Fc be the points in which the B-escribed and C-escribed circles are tang side AC, respectively to the side AB. Prove that D, Da , Eb , Fc are concy b only if AB = AC or m(A) = 90 . Temistocle B

L67. The parallel lines t1 and t2 are tangent to the circle C with cen circle C1 with center O1 is tangent to t1 and C and the circle C2 with ce tangent to t2 , C and C1 ; C, C1 and C2 being exterior to each other. Prov \ angle O1 OO2 is acute and nd the minimum value of its measure. Neculai Roman, Mirce L68. a) Prove that, for x, y, z (0, ), s r r x y 1 1 1 (x + y + z) + + 1+ + . x y z y x

b) Using a), prove that r

R 1+ 1+4 r

for any given triangle with the usual notations.

pa + pb

pb pa

for any a1 , a2 , . . . , an > 0 satisfying a1 a2 . . . an = 1. Gabriel Dospinescu, B n L72. Let a, b Q, a, b > 0, a 6= b such that a bn Z for innitely ma Prove that a, b Z. Gabriel Dospinescu, B L73. Let k N , k 3. Find n N \ {0, 1} such that r q a1 + a2 + + ak n a1 a2 . . . ak , a1 , a2 , . . . , ak [0,

Marian Tetiv L69. Find n N, n 3, such that there are n blue points and n red the same plane, no three points being collinear, such that the interior of an with blue vertices contains at least one red point and the interior of any tri red vertices contains at least one blue point. Adrian Zahariuc, high school studen L70. Let k, p N and let a 82k 2p rectangle which is completely co 7 5 and 6 4 rectangles with no superpositions. Prove that the number (x, y) with side 1, x even, y odd, which are vertices of 7 5 rectangles total number of 7 5 rectangles. (By a 7 5 rectangle we mean a rect length 7 and height 5). Marius Pachiariu, high school stud t L71. Let n N, n 2. Find the best constants m, M such that n n Xq X 2 + n2 1 m ak ak + M
k=1 k=1

Gabriel Popa and Paul Georg L74. Let n N , n 2 and a, b R, a < b. If f : [a, b] R is a c Rb function such that a xk f (x) dx = 0 for any k N, 0 k n, then f h n + 1 distinct zeros in (a, b). Andrei Ned L75. Find n N such that i 1 cos < p , 0, . 8 2 1 + n sin4 Ctlin Cali a a

Pagina rezolvitorilor

BOTO ANI S Scoala nr. 7 "O. Bancila". Clasa a IV-a. IFTODE Cozmin: P(54-5 67,68,70). Colegiul Naional "A. T. Laurian". Clasa a IX-a. NEGRESCU A t VII(43,47,48), VIII(47,49), IX(42,46,48), X.42, G46. CRAIOVA Scoala nr. 22 "M. Eliade". Clasa a IV-a (inst. VANTU Angela). Ioan: P (64-73) IA I S Scoala nr. 3 "Al. Vlahua". Clasa a V-a. COJOCARU Ioana: P t 49),VI.46; DODU Corina: P.71, V(47-49),VI.46; IRIMIA Andreea: P 49),VI.46; STIRBAN Ioana: P.71, V(47-49),VI.46; UNGURU George Clau V(47-49),VI.46. Scoala nr. 7 "N. Tonitza". Clasa a II-a (nv. TUDOSE Elena). DOBR Maria: P(64-68,70,72); LEONTE Anca: P(64-68,70,72); POSTICA Simona dra: P(64-68,70,72); ROTARU Larisa-Maria: P(64-68,70,72); SAVIN Raz 68,70,72). Clasa a II-a (nv. MELINTE Rodica). BACIU Ciprian: P(64 BRZU Constantin: P(64-69,71,72); BOTO ANU Bianca-Mihaela: P(64 S a t BUZDUGAN Petru-Ctlin: P(64-69,71,72); CEUCA Dnu-Vasilic: P aa a 72); CONSTANTINESCU Diana-Gabriela: P(64-69,71,72); CUCUTEA Ctlin: P(64-69,71,72); GU OVATE Diana- tefana: P(64-69,71,72); aa S S Larisa-Diana: P(64-69,71,72); MIRON Vlad- tefan: P(64-69,71,72); MOT S na-Diana: P(64-69,71,72); ROTARIU Marian: P(64-69,71,72); SUCIU P(64-69,71,72); TEIU-COSTIN Andra-Mihaela: P(64-69,71,72). Clasa (nv. PA ANIUC Maria). ATASIEI Vldu: P(64,65,67,68,72); PASAROI S a t P(66-68,70,71); SOLOMON Ana-Maria: (65,68-71); TINCU Andrei: P 70,72); ZAMFIR Loredana-Cristiana: P(64,65,69-71). Clasa a III-a (nv. Gabriela). ADASCALITEI IONUT: P(65,68-71); GHIARASIM Olivia: P 68,72); MACOVEI Alina: P(64,65,68-70,72); NEGRESCU Vlad-Petru: P 71); TROCIN Monica-Andreea: P(66,68-71). Scoala nr. 13 "Alexandru cel Bun". Clasa a IV-a (inst. COJOCARIU A a a BILITA Ctlina-Elena: P(64-68,70); CURMEI Renata-Maria: P(64-68,70 Radu: P(64-68,70); ILIE Laura: P(64-68,70); MAGDICI Magda-Otilia: P( MARDARI Claudiu: P(64-68,70); LUCAN Mihaela-Alexandra; P(64-68,70 FOR Stefan-Marian: P(64-68,70); NICULAE-GRIGORESCU Andrei: P( PRISACARU Carmen-Georgiana: P(64-68,70); PETREA Silvia: P(64-68 FAN Teodora-Ioana: P(64-68,70); TRLAGEANU Ingrid-Maria: P(64-68 Scoala nr. 17 "I. Creanga". Clasa a VIII-a. DUMITRIU Vlad: VI(46 VIII.46. Scoala nr. 22 "B. P. Hasdeu". Clasa I (nv. STEFAN Liviu). DA drin P(55,64,65,67,71); IGNAT Andreea: P(54-56,64,65); NICOLA De P(55,64,65,67,71); PURICE Dumitru-Ciprian: P(64-67,71). Clasa a II-a

HOTARU Liliana). TURCU Andrei-Daniel: P(64-67,71); Clasa a II-a (n ORU Iuliana). ADASCALITEI Victor: P(64-69,71,72); APOSTOL A P(64-68,71,72); BALAN Andrei: P(64-68,71,72); BURUIANA Catalina: P( CUBERSCHI Paul: P(64-67,70-72); E ANU Georgiana: P(64-68,71); GR S Claudiu: P(64-68,72); GNDU Alexandra-Livia: P(64-66,68,71,72); LAM na: P(61,64-68,71); MOGA Alexandru: P(64-68,71,72); REBEGEA Andra 68,71); UNGUREANU Teofana: P(64-68,71,72). Clasa a II-a (nv. TUT AILENEI-OPREA Adriana: P(64-68); ANDRONICIUC Ana-Miruna: P(6 LADEANU Claus-Alex: P(64-66,68,71); BOARU Adrian: P(64-66,68,71); NA Sebastian-Andrei: P(64-68,71): BUHU Vlad: P(64-68,71); CEOBAN Nicolae: P(64-67,71); CHICHIRAU Alexandra-Elena: P(64-68,71); COSTA Ivona: P(64-68,71); DANILA Alexandru: P(64-66,68,71); DIACONESC P(64-68,71); DOROHOI Ovidiu: P(64-68,71); GHERAN Ana-Maria: P(64 GRIGORE Georgiana: P(64-68,71); GURAU Raluca-Claudia: P(64-68,71) CU Iustina: P(64-68,71); HORBOVANU Bianca-Alexandra: P(64-68,71); N Andrei-Ionu: P(64-68,71); ONOFREI Liviana Ana-Maria: P(64-68,71 t Andrei: P(64-66,68,71); SIMIRAD Andrei: P(54,55,57-59,61,62,64-72); Alexandra-Arina: P(64-66,68,71). Clasa a V-a. PINTILIE Mina-Liviu: P 73); PINTILIE Nicoleta-Livia: P(61,62,71-73); STERBULEAC Daniel: P 73),V(42,43). Scoala nr. 23 "T. Maiorescu". Clasa a IV-a (nv. CHIRILA Beatrice RACHE Alexandru-Gabriel: P(64-73). Scoala nr. 26 "G. Cobuc". Clasa a II-a (nv. BUCATARIU Rica) s Robert-Ionu: P(54-57,64-67); IVANCIUC Dumitru-Florin: P(55,64-67 t Adrian-Bogdan: P(54-57,64-67); SANDU Ioana-Luiza: P(54-57,64-67); S Ionela-Cristina: P(54-57,64-67). Clasa a III-a (nv. RACU Maria). B Ionela-Alexandra: P(64-68,70-72); BURLACU Stefan-Claudiu: P(64-68,7 LIN Andreea-Georgiana: P(64-68,70-72); IFROSA Adriana: P(64,66-72); tru-Alexandru: P(64-70,72); MOISA Bogdan: P(64-68,70-72); PINTILIE Florin: P(64-68,70-72); RAZLOG Ionu: P(64-68,70-72). Clasa a III-a (n t Paraschiva). ALUPEI Andra-Madalina: P(64-67,68,70,71); CIOABA Oana P(64-68,70,71); GHERCA Marius-Catalin: P(64-68,70,71); HOMEA Liv 68,70,71); HUIDE Gina: P(64-68,70,71); MANOLIU Mdlina: P(64 S a a MIHAILESCU Laura; P(64-68,70,71); PISICA Alexandru: P(64-68,70,7 Florin: P(64-68,70,71); SCUTARU Constantin: P(64-68,70,71,73). Colegiul Naional "C. Negruzzi". Clasa a V-a. ANDRIESCU Gabriela t 59,61,63),V.42; TIBA Marius: P(71-73),V(46,47,49,50),VI.46. Colegiul Naional Iai. Clasa a V-a. VLCU Maria Caterina: V(40, t s Liceul "M. Eminescu". Clasa a VI-a. CIURARU Ionela: V(46,48), VI IPATE Cristina: V(46,48),VI(47,48,50). Clasa a IX-a. AVRAM Mircea: 44,46,48). Clasa a X-a. COMAN Stefan: VIII(46,49), IX(46,47), X.47.

Premii acordate rezolvitorilor

Pentru apariia de trei ori la rubrica "Pagina rezolvitorilor" redac t t "Recreaii matematice" acorda o diplom i un premiu n cari elevilor t as t

Scoala nr. 22 "M. Eliade", Craiova STANCIU Ioan (cl. a IV-a): 2/2003 (9pb), 1/2004 (10pb), 2/2004 (10p

Scoala nr. 7 "N. Tonitza" BACIU Ciprian (cl. a II-a): 2/2003 (5pb), 1/2004 (5pb), 2/2004 (8pb); BRZU Constantin (cl. a II-a): 2/2003 (5pb), 1/2004 (5pb), 2/2004 (7 BOTO ANU Bianca-Mihaela (cl. a II-a): 2/2003 (5pb), 1/2004 (5pb S (7pb); BUZDUGAN Petru-Ctlin (cl. a II-a): 2/2003 (5pb), 1/2004 (5pb), a a (7pb); a t CEUCA Dnu-Vasilic (cl. a II-a): 2/2003 (5pb), 1/2004 (5pb), 2/20 a CONSTANTINESCU Diana-Gabriela (cl. a II-a): 2/2003 (5pb), 1/20 2/2004 (8pb); CUCUTEANU Paul-Ctlin (cl. a II-a): 2/2003 (5pb), 1/2004 (5pb), a a (8pb); DOBRIN Diana-Maria (cl. a II-a): 2/2003 (5pb), 1/2004 (5pb), 2/200 GU OVATE Diana- tefana (cl. a II-a): 2/2003 (5pb), 1/2004 (5pb), 2 S S (8pb); LEOGAN Larisa-Diana (cl. a II-a): 2/2003 (5pb), 1/2004 (5pb), 2/200 LEONTE Anca (cl. a II-a): 2/2003 (5pb), 1/2004 (5pb), 2/2004 (7pb); MIRON Vlad- tefan (cl. a II-a): 2/2003 (5pb), 1/2004 (5pb), 2/2004 ( S MOTAN Geanina-Diana (cl. a II-a): 2/2003 (5pb), 1/2004 (5pb), 2/2 POSTICA Simona-Alexandra (cl. a II-a): 2/2003 (5pb), 1/2004 (5pb (7pb); ROTARIU Larisa-Maria (cl. a II-a): 2/2003 (5pb), 1/2004 (5pb), 2/20 ROTARIU Marian (cl. a II-a): 2/2003 (5pb), 1/2004 (5pb), 2/2004 (8p SUCIUC Raluca (cl. a II-a): 2/2003 (5pb), 1/2004 (5pb), 2/2004 (8pb) TEIU-COSTIN Andrada-Mihaela (cl. a II-a): 2/2003 (5pb), 1/2004 2/2004 (8pb).

Scoala nr. 22 "B. P. Hasdeu" ADASCALITEI Victor (cl. a II-a): 2/2003 (5pb), 1/2004, (5pb), 2/20 BALAN Andrei (cl. a II-a): 2/2003 (5pb), 1/2004 (5pb), 2/2004 (8pb); BUHU Vlad (cl. a II-a): 2/2003 (5pb), 1/2004 (5pb), 2/2004 (6pb); CHICHIRAU Alexandra-Elena (cl. a II-a): 2/2003 (5pb), 1/2004 (6p (6pb); CUBERSCHI Paul (cl. a II-a): 2/2003 (5pb), 1/2004 (6pb), 2/2004 (7 E ANU Georgiana (cl. a II-a): 2/2003 (5pb), 1/2004 (5pb), 2/2004 (5p S GREIEROSU Claudiu (cl. a II-a): 2/2003 (5pb), 1/2004 (6pb), 2/2004 GURAU Raluca-Claudia (cl. a II-a): 2/2003 (5pb), 1/2004 (5pb), 2/2

NASTASE Andrei-Ionu (cl. a II-a): 2/2003 (5pb), 1/2004 (6pb), 2/2 t HATESCU Iustina (cl. a II-a): 2/2003 (5pb), 1/2004 (5pb), 2/2004 (6p LAMATIC Ioana (cl. a II-a): 2/2003 (5pb), 1/2004 (6pb), 2/2004 (7pb REBEGEA Andrada (cl. a II-a): 2/2003 (5pb), 1/2004 (5pb), 2/2004

Scoala nr. 23 "Titu Maiorescu" TUDORACHE Alexandru-Gabriel (cl. a IV-a): 2/2003 (10pb), 1/200 2/2004 (10pb).

Scoala nr. 26 "G. Cobuc" s ALUPEI Andra-Mdlina (cl. a III-a): 2/2003 (5pb), 1/2004 (5pb), 2/2 a a BULGARU Ionela-Alexandra (cl. a III-a): 2/2003 (5pb), 1/2004 (5pb (8 pb); BURLACU Stefan-Claudiu (cl. a III-a): 1/2003 (5pb), 1/2004 (5pb), 2/2 GHERCA Marius-Ctlin (cl. a III-a): 2/2003 (5pb), 1/2004 (5pb), 2/2 a a HOMEA Liviu (cl. a III-a): 2/2003 (5pb), 1/2004 (5pb), 2/2004 (7pb); HUIDE Gina (cl. a III-a): 2/2003 (5pb), 1/2004 (5pb), 2/2004 (7pb); S IFROSA Adriana (cl. a III-a): 2/2003 (5pb), 1/2004 (5pb), 2/2004 (8p IOJA Petru-Alexandru (cl. a III-a): 2/2003 (5pb), 1/2004 (5pb), 2/20 PISICA Alexandru (cl. a III-a): 2/2003 (5pb), 1/2004 (5pb), 2/2004 (7 Colegiul Naional "C. Negruzzi" t TIBA Marius (cl. V-a): 1/2003 (6pb), 2/2003 (7pb), 1/2004 (7pb).

Liceul "M. Eminescu" CIURARU Ionela (cl. VI-a): 2/2003 (11pb), 1/2004 (5pb), 2/2004 (5pb

(continuarea tabelului din p. 128) 178. 179. 180. 181. 182. 183. PIFTOR Rositta SPIRIDON Doina MARTINU I Vladimir S TIMOHE TUMAC Gabriel ANTON Florina Cristiana PRICOP Vasile Liceul economic nr. 2, Iai s Liceul de arta, Iai s Colegiul Naional "E. Racovia", t t Liceul "G. Ibraaileanu", Iai s Colegiul Naional "E. Racovia", t t Pacani s

Premiile pe anul 2004 acordate de FUNDA TIA CULTURALA "POIANA

Fundaia Cultural "Poiana" (director d-l Dan Tiba) acord an t a a elevilor - colaboratori ai revistei "Recreaii matematice" care se disting pri t articolelor, notelor i problemelor originale publicate n paginile acesteia. s Redacia revistei decide ca pentru anul 2004 premiile oferite, n valoa t 1 000 000 lei, s e atribuite urmtorilor elevi: a a

1. LUPU Cezar (Colegiul Naional "Mircea cel Batrn", Constana) t t Asupra unei inegaliti condiionate (RecMat 1/2004, 27-28), at t probleme propuse: IX.36 (1/2003), IX.44 (2/2003), IX.48, XI.46 X.51 (2/2004).

2. NEGRESCU Alexandru (Colegiul Naional "A. T. Laurian", Boto t s Asupra unei inegalitai (RecMat 2/2004, 106-108), t probleme propuse: VI.38 (1/2003), VII.41 (2/2003), VIII.46 (1 VII.51 (2/2204).

Premiile se pot ridica direct de la redacie sau pot trimise prin man t la adresa elevului premiat.

IMPORTANT

n scopul unei legturi rapide cu redacia revistei, pot fi utilizate u a t toarele adrese e-mail: tbi@math.tuiasi.ro, popagabriel@go. Pe aceast cale colaboratorii pot purta cu redacia un dialog privi a t materialele trimise acesteia, procurarea numerelor revistei etc.

La problemele de tip L se primesc soluii de la orice iubitor de matem t elementare (indiferent de preocupare profesionala sau vrsta ). F dintre soluiile acestor probleme - ce sunt publicate n revist du t a an - va fi urmat de numele tuturor celor care au rezolvat-o. a

Adresm cu insistena rugmintea ca materialele trimis a t a vistei s nu fie (s nu fi fost) trimise i altor publicaii. a a s t

Revista RECREAII MATEMATICE apare de dou ori pe datele de 1 martie i 1 septembrie) i se adreseaz elevilor, profes studenilor i tuturor celor pasionai de matematicile elementare. n atenia tuturor colaboratorilor Materialele trimise redaciei spre publicare (note i articole, chesti metodic, probleme propuse etc.) trebuie prezentate ngrijit, clar i conc trebuie s prezinte interes pentru un cerc ct mai larg de cititori. Se reco ca textele s nu depeasc patru pagini. Evident, ele trebuie s fie ori i s nu fi aprut sau s fi fost trimise spre publicare altor reviste. Problemele originale destinate rubricii Probleme propuse redactate pe foi separate cte una pe fiecare foaie, cu enu demonstraie/rezolvare, fiind nsoite de numele autorului, coala i loca unde lucreaz/nva. Redacia revistei va decide asupra oportunitii publicrii mater primite. n atenia elevilor Numele elevilor care vor trimite redaciei soluii corecte la exerci problemele din rubrica Probleme propuse vor fi menionate n P rezolvitorilor. Elevii vor ine seama de urmtoarele reguli: 1. Pot trimite soluii la minimum cinci probleme propuse n nu prezent i cel anterior al revistei; pe o foaie va fi redactat solui singure probleme. 2. Elevii din clasele VI-XII au dreptul s trimit soluii la prob propuse pentru clasa lor, pentru orice clas mai mare, din dou clase mai m imediat anterioare. Elevii din clasa a V-a pot trimite soluii la prob propuse pentru clasele a IV-a, a V-a i orice clas mai mare, iar elevii c I-IV pot trimite soluii la problemele propuse pentru oricare din clasele p i orice clas mai mare. Orice elev poate trimite soluii la problem concurs (de tip G i L). 3. Vor fi menionate urmtoarele date personale: numele i prenu clasa, coala i localitatea. 4. Plicul cu probleme rezolvate se va trimite prin pot (sau va f direct) la adresa Redaciei: Prof. dr. Temistocle Brsan Catedra de Matematic Universitatea Tehnic Gh. Asachi Iai Bulevardul Carol I nr. 11, 700506, Iai E-mail: tbi@math.tuiasi.ro

CUPRINS

Ctre cititori ................................................................................................................ ALEXANDRU MYLLER, ctitorul colii matematice ieene................................ HENRI POINCAR la 150 de ani de la naterea sa ...................................... Trecerea planetei Venus prin faa Soarelui .............................................................

ARTICOLE I NOTE

G. DOSPINESCU Cteva noi aplicaii ale unei idei consacrate ...................... T. BRSAN Cteva proprieti ale medianelor..................................................... C. - t. POPA O construcie geometric a mediilor (II)..................................... F. POPOVICI O generalizare a teoremelor de baz ale calculului diferenial ........

NOTA ELEVULUI

A. NEGRESCU Asupra unei inegaliti................................................................ *** Asupra problemei VII.41 din RecMat 2/2003 .......................

CHESTIUNI METODICE

D. MIHALACHE, M. TETIVA Asupra unei probleme de concurs....................

CHESTIUNI COMPLEMENTARE MANUALELOR CONCURSURI I EXAMENE


Concursul Concursul Concursul Concursul Olimpiada

M. CRCIUN Exponentul numrului natural a n produsul n!.........................

Alexandru Myller ed. a II-a, Iai, 2004 ............................................. Florica T. Cmpan, ed. a IV-a, 2004 ................................................. Traian Lalescu, ed. a V-a, Iai, 2004 ................................................ Adolf Haimovici, ed. a VIII-a, 2004 .................................................. Balcanic de Matematic (juniori), ed. a VIII-a, 2004 .......................

CORESPONDENE PROBLEME I SOLUII

H. STEPHAN Probleme pentru clasa a VIII-a .....................................................

Soluiile problemelor propuse n nr. 2/2003............................................................. Soluiile problemelor pentru pregtirea concursurilor din nr. 2/2003 .................... Probleme propuse........................................................................................................ Probleme pentru pregtirea concursurilor ................................................................. Training problems for mathematical contests ..........................................................

Pagina rezolvitorilor....................................................................................................

Anul VIII, Nr. 2

Iulie Decembri

RECREAII MATEMATIC

REVIST DE MATEMATIC PENTRU ELEVI I PROF

e i = 1

Editura Recreaii Matematic IAI - 2006

Semnificaia formulei de pe copert: i ntr-o form concis, formula e = 1 leag cele patru ramuri fun
ale matematicii: ARITMETICA GEOMETRIA ALGEBRA ANALIZA MATEMATIC reprezentat reprezentat reprezentat reprezentat de de de de 1 i e

Redacia revistei :

Petru ASAFTEI, Dumitru BTINEU-GIURGIU (Bucureti), Cornelia - Livi Temistocle BRSAN, Dan BRNZEI, Ctlin - Cristian BUDEANU, CRUU, Constantin CHIRIL, Eugenia COHAL, Adrian CORDUNEAN CRCIUN (Pacani), Gabriel DOSPINESCU (student, Paris), Marius FARCA, GALIA, Paul GEORGESCU, Mihai HAIVAS, Gheorghe IUREA, Lucian LDUNC, Mircea LUPAN, Gabriel MRANU, Andrei NEDELCU, Gabriel P POPESCU (Suceava), Florin POPOVICI (Braov), Maria RACU, Ioan SC (Hrlu), Ioan ERDEAN (Ortie), Dan TIBA (Bucureti), Adrian ZAHARIU Adrian ZANOSCHI.

Adresa redaciei: Catedra de Matematic Universitatea Tehnic Gh. Asachi Iai Bd. Carol I, nr.11, 700506, Iai Tel. 032 213737 / int. 123 E-mail: recreatii.matematice@gmail.com http://www.recreatiimatematice.uv.ro

COPYRIGHT 2006, ASOCIAIA RECREAII MATEMATICE Toate drepturile aparin Asociaiei Recraii Matematice. Reproducerea in parial a textului sau a ilustraiilor din aceast revist este posibil numai cu acordu scris al acesteia. TIPRIT LA SL&F IMPEX IAI Bd. Carol I, nr. 3-5 Tel. 0788 498933 E-mail: simonaslf@yahoo.com

Anul VIII, Nr. 2

Iulie Decembrie

RECREAII MATEMATICE

REVIST DE MATEMATIC PENTRU ELEVI I PROF

e i = 1
Revist cu apariie semestrial publicat de

ASOCIAIA RECREAII MATEMATIC

IAI - 2006

Mendel Haimovici i Scoala matematic din s a

Scoala matematic din Iai, fondat a s a n cadrul Seminarului Matematic de ctr a Al. Myller, s-a dezvoltat i consolidat prin o s mari personaliti, ca Octav Mayer, Mendel at i prin contribuia unor strlucii continuato s t a t Departamentul de mecanic al acestei S a creat de profesorul Mendel Haimovici, m Academiei Romne. Calitile sale de Om, at Profesor, lupttor neobosit pentru dreptate a admiraia opiniei publice i a colegilor, colab t s i studenilor si. Contribuia sa la dezvolta s t a t matematice moderne este cu totul remarc treaga sa activitate poart marca personal a caracterizat prin inteligena, curiozitate a t creativitate, vast cultur i etic profesiona a as a M. Haimovici s-a nscut la Iai, n 30 noiembrie 1906. A fcut studiil a s a Colegiul (Liceul) Naional. Si-a continuat pregtirea la Facultate de Stii t a Matematic) pe care a absolvit-o n 1930. A fost imediat numit asistent a s colaborator al fotilor si profesori. A fcut stagii de specializare la Rom s a a Londra. n iulie 1933 i-a luat doctoratul n Italia, cu o valoroas tez de s a a fluidelor, sub ndrumarea lui Tullio Levi-Civita. ntors n ar, i-a reluat t a s asistent. n 1940, dup promulgarea legilor rasiale, a fost ndeprtat ci a a t Universitate. La ncetarea rzboiului a fost numit profesor i a devenit ef a s s de mecanic, pe care a condus-o pn la sfritul vieii, adic timp de 28 a a s t a n 1948, M. Haimovici a fost ales membru corespondent, iar civa ani t a devenit membru titular al Academiei Romne. Cnd s-a nfiinat Filiala t demiei, a fost numit director al Institutului de Matematic, pe care l-a co a la sfritul vieii. A fost invitat s ia parte la numeroase conferine intern s t a t a organizat la Iai manifestri tiinifice de mare prestigiu. A ncetat din s a s t martie 1973, lsnd o oper valoroas i o amintire luminoas de remarca a a as a Cercetrile lui M. Haimovici se raporteaz la trei domenii principale al a a ticii: geometrie, teoria ecuaiilor cu derivate paiale i mecanic. Cu to t t s a inuena exercitat de Myller i Mayer n domeniul geometriilor cu grup t a s tal, a abordat un nou subiect din geometria acelui timp. Dup vizita lui E a la Iai, n 1931, atras de ideile acestuia a eleborat importante lucrri as s a probleme dicile. Astfel, a descoperit clase remarcabile de spaii Finsler t ile fundamentale ale hipersuprafeelor din aceste spaii. Mai trziu, mate t t japonez Makoto Matsumo a inclus lucrrile lui M. Haimovici n celebra sa m a asupra fundamentelor geometriei Finsler. M. Haimovici a fost pionier i n s geometriei integrale, descoperind o formul care generalizeaz teorema cu a a lui Crofton. A descoperit conceptul de quasigrup difereniabil i a constru t s complet i unitar; a demonstrat c aceste obiecte se comport faa de a s a a a t Lie n aceeai manier n care varietile plate se comport n raport cu ce s a at a Actualmente, noiunea de quasigrup este studiat din punct de vedere t a

topologic n coli renumite de matematic din SUA i Rusia. s a s O lung perioad de timp, M. Haimovici s-a ocupat de geometria v a a neolonome. Importana lor const n faptul c ele conduc la cel mai adec t a a care descrie micarea sistemelor mecanice neolonome. Se poate spune c s a intrinsec a varietilor neolonome n spaii Riemann este creaia lui M. a at t t n 1998, a aprut volumul I al operelor sale complete, iar volumul al doilea a cu prilejul Centenarului naterii savantului. s Prin contribuiile sale, M. Haimovici a scos la lumin noi domenii de t a n care au lucrat i lucreaz continuatorii operei sale, printre care i s s a s acestei Note, care este totodat i primul cercettor care a obinut doct as a t ndrumarea lui M. Haimovici. Profesorul M. Haimovici a avut vocaie de geometru. Pentru cei ca t familiarizai cu personalitatea sa, poate prea curios cum a putut el de t a specialist n mecanic. Pentru a lmuri acest aspect, se cuvine s facem a a a istoric al predrii mecanicii la Universitatea din Iai. La nfiinarea acestei a s t o catedr de mecanic. Primul curs "Mecanic elementar i raional" a fo a a a as t a Nicolae Culianu. Ulterior, aceast catedr a fost ilustrat de Ioan Melik a a a Tzony (absolvent de la Sorbona), care a publicat acum 125 de ani o c probleme de mecanic n revista "Recreaii Stiinifice". Manuscrisul cur a t t de Tzony se pstreaz i astzi la Biblioteca Seminarului Matematic. T a a s a datoreaz i nfiinarea Laboratorului. Tzony rmne unul dintre cei mai i as t a oameni care au pus bazele unui nvamnt de nalt valoare n ara n at a t condus catedra de mecanic pn n anul 1898. n 1907, Dimitrie Pompe a a n matematic de la Sorbona, a preluat aceast catedr. El era un excele a a a dar a fost i primul n Iai, care a publicat lucrri originale de mecani s s a civa ani (cinci) a fost chemat la Bucureti, pentru a lua postul rmas v t s a retragerea lui Spiru Haret. Ulterior, catedra de mecanic a fost ilustrat s a a Victor Vlcovici, Simion Sanielevici i Ioan Pl cineanu. s a t n 1945, M. Haimovici a devenit eful catedrei de mecanic. Avnd va s a tine de geometrie, ecuaii difereniale i cu derivate pariale, domenii n c t t t s t contribuii de valoare, a gsit acum oportunitatea de a materializa n m t a sintez a experienei predecesorilor i a sa personal. A inut cursuri de a t s a t hidrodinamic i mecanica mediilor continue. A publicat un curs de e a s i a organizat seminarii de specialitate i conferine naionale. A cond s s t t de mecanicieni de la Institutul de Matematic. A condus teze de docto a subiecte din teoria mediilor continue, hidrodinamic, magneto-hidrodinam a ticitate isotropic i neisotropic n medii omogene i neomogene, termoe as a s plasticitate, medii continue cu microstructur i mecanic analitic. as a a Aceast activitate intens care a dus la crearea unei Scoli originale de a a este, fr ndoial, rezultatul muncii de o viaa a profesorului M. Haimo aa a t nefericire, activitatea nentrerupt, plin de abnegaie i permanentul su a a t s a satisface toate exigenele au contribuit la sfritul su prematur. t s a Druirea sa pentru tiina este concretizat n cele dou mari realiz a s t a a a Scoala de Mecanic din Iai i Institutul de Matematic (care a renscut d a s s a a

Acad. Radu MIR

Florica T. Cmpan

Gndim c, n luna noiembrie a acestui a multe clopote de aur i argint vor suna s a s a 100 de ani de la naterea Florici T. Cmp s a nu ntr-o zi anume, cci socotitorii de vreme au a stil vechi pe altul nou, ncurcnd un 13 cu un nu n clopotnie ociind ntre cer i pmnt, t s a bre aule academice, ci doar n inimi ce adun a cu raiunea. Poate nu cu frecvene i inten t t s s zguduie timpane, ci profund, grav, linit a s mngie i s ndemne suete. Gndim c s a a Recreaiilor Matematice de a marca momentu t ticat i va reverbera ecouri. as Muli oameni cred despre ei nii c tiu s t ss as a s preuiasc i s iubeasc tot ce i nconjoar a t as a a oameni, obiceiuri, gnduri, deveniri, cri. De fapt sunt puini; o larg at t a i concentreaz nelegerea spre agonisiri de folos personal. (Citez ns s a t a matematician din Focani: Dumnezeu i arat dispreul faa de avere prin s s a t t le-o distribuie). Dintre cei puini care pot s neleag, s preuiasc i s t a t a a t as sunt tare puini care dovedesc generozitate, capacitate i profunzime comp t s ale Florici. a Florica a neles i iubit zica i mai ales matematica, dobndind licene t s s t 1929. Din matematic a neles profund i a iubit cu nelepciune mai ales G a t s t Fundamentele i Istoria (matematicii). n geometrie i-a mplinit i doc s s s 1942, sub bagheta renumitului furitor de coal Alexandru Myller ), cu o t a s a suprafee paralele i asemenea, ilustrat cu elegante interpretri geometric t s a a Nu putem uita aici inspirata carte Aventura geometriilor neeuclidiene adnci i originale asupra Fundamentelor matematicii s-au nchegat n re s carte Licuricii din adncuri. Generoasa aplecare spre Istoria matema valoricat n numeroase lucrri i comunicri tiinice dar i n cri a a s a s t s at numrului , Probleme celebre din istoria matematicii. a Florica a neles i iubit cartea. Semnatarul acestor rnduri nu poat t s unele imagini din amintire. Era deja pensionar cnd ajungea la ua a s Seminarului matematic cu mult naintea harnicei bibliotecare. Se scuz reinut prea bine nite gnduri din anume cri. Auzeam apoi (cu tresriri t s at a un scrit de scar lung, dar subire (spre a uoar), spre rafturi n t a a t s a nerbdtoare cu o minuscul batist feminin n mn. Dup ce scotea o c a a a a a a a o tergea grijuliu de un bnuit colb, apoi o mngia cu ochii i cu mna i a s a s s cobora de pe scar (linitindu-mi temeri). Nu accepta s e ajutat; era a s a a inalienabil al ei, instaurat nc de Alexandru Myller. Stergea cu aceeai b a s poate cu alta asemntore) i masa pe care aeza cartea. O deschidea nc a a s s cu pagin, i adsta ndelung notnd ades ntr-un caiet de elev. Pn s a s a a pe mas se aduna un teanc de cri i de reviste care aproape o ascundea a at s de nchidere cerea ngduina s pstreze pe acea mas i cri care s le a t a a as at a

a doua zi. Cnd socotea c dialogul nu are pe cine tulbura, mprte a a as bucuriile ei: iat ce idee grozav a avut X, sau iat ce frumos o spune Y. a a a stngciile: cum poate Z s scrie despre asta f a a-l cita pe...? Pentru co a a ar vzut frecvent muli care se ncruntau cnd descopereau idei prin revist a t mi-a venit mie ideea asta? Mai cu seam, Florica a neles i iubit adolescenii n strdaniile l a t s t a apropia de matematic. Din dragoste faa de ei a fost o excelent profe a t a la Liceul "Oltea Doamna", unde fusese i eleva primei femei din Romnia s matematic, Silvia Creang . Mai apoi a fost confereniar i profesor la Fac a a t s matematic din Iai. Din aceeai dragoste a scris i cunoscutele i mult-c a s s s s cri. Alii au conceput cri asemntoare nu din dragoste faa de cititor at t at a a t zgndri glorii; s ne mirm c nu au avut ecou? Parc spunea cineva c a a a a a a este ca o minge pe care cu ct o arunci mai departe de tine, revine mai repe Un mic dar ginga, semn c ieenii nu au uitat-o pe Florica, sunt cele s a s (judeene i interjudeene) ale Concursului de matematic "Florica T. C t s t a rndurile de mai sus doresc a semnala adnc i nentinat preuire. as a t

Prof. dr. Dan BRN

Eclipsele de Soare

Se spune c un corp ceresc fr lumin proprie este eclipsat atunci c a aa a intr n conul de umbr al unei planete, ind astfel lipsit de lumina Soa a a poate fi vzut. Aa se produc eclipsele de Lun sau eclipsele sateliilor a s a t Fenomenul de dispariie a unui astru din cmpul nostru de vedere, cnd nt t tru i observator se interpune un alt astru, se numete ocultaie. Astfel, s s t sa n jurul Pmntului, Luna poate s acopere parial sau total Soarele. F a a t produs este impropriu spus eclips de Soare, cnd de fapt este vorba de a Soarelui de ctre Lun. Trecnd peste aceste mici nenelegeri lingvis a a t mrim cum se produce o eclips de Soare. a a Dispariia treptat a Soarelui din cmpul de vizibilitate are loc at t a Soarele, Luna i Pmntul sunt aproximativ n vecintatea aceleiai dr s a a s au loc eclipse pariale, totale sau inelare. ntruct Luna se rotete n jur t s tului de aproximativ 12,368 ori pe an, ar rezulta c n fiecare lun s-ar a a eclips de Soare i una de Lun. Totui, nu este aa i iat de ce: P a s a s s s a a rotete n jurul Soarelui pe o elips, Soarele aflndu-se ntr-unul din foca s a treia a lui Kepler ). Elipsa, curba plan descris de Pmnt, este foarte a a a a un cerc; distana minim a Pmntului faa de Soare (Pmntul la perihel t a a t a 147 099 000 km, iar cea maxim (Pmntul la afeliu) este de 152 097 090 a a alt parte, Luna se rotete n jurul Pmntului tot pe o elips, planul or a s a a (planul elipsei) face, ns, cu planul orbitei terestre un unghi de 5 grade, 8 a 48 secunde, aa c, nu la fiecare rotaie cele trei obiecte pot fi coliniare. s a t

n fig.1, SA reprezint raza Soarelui, egal cu 696 260 km, LB raza L a a cu 1738 km, iar V un punct situat n vrful conului de umbr. Din calcu a c nlimea conului variaz n limitele 363 200 km i 375 900 km, n timp c a at a s de la Pmnt la Lun rmne cuprins ntre 363 100 km (perigeu) i 4 a a a a s (apogeu). Rezult de aici c, n cazul n care Luna se afl la distana ma a a a t 375 900 km, vrful conului de umbr nu poate atinge nici un punct de pe a Pmntului i, n consecina, nu poate avea loc nici o eclips total. D a s t a a

atunci cnd Luna se afl ntre cele dou limite menionate pot avea loc toa a a t de eclipse (totale, pariale i inelare), vizibile n mod diferit n funcie de l t s t tului de observaie. t

n fig. 2 se arat cazul unei eclipse totale de Soare de o anumit durat a a a depinde de lungimea conului de umbr. Durata maxim pentru un loc da a a depi 8 minute, dar poate fi i instantanee. as s

n fig. 3 este exemplificat o eclips inelar cnd de pe Pmnt Soare a a a a la un moment dat sub forma unui inel luminos. Aa cum se poate constata din gurile de mai sus, Luna las pe suprafa s a tului o umbr (mulimea punctelor de intersecie a conului de umbr cu sfer a t t a al crui contur este o curb ce numai ntr-un singur caz poate fi cerc: at a a centrele Soarelui, Lunii i Pmntului sunt colineare iar Luna se afl la s a a versitatea de manifestare a fenomenului depinde nemijlocit de raza Soarelu precum i de cele dou distane, SL i LV . s a t s n desfurarea unei eclipse totale de Soare se deosebesc cinci momen as tante. Astfel, momentul n care discul Lunii este tangent discului Soarel partea vestic sau, altfel spus, momentul la care ncepe eclipsa parial s a t a

momentul primului contact exterior. Urmeaz acoperirea treptat a Soa a a cnd acesta nu se mai vede, momentul primului contact interior, dup ca a st n totalitate un timp, care nu poate fi mai mare de 8 minute. n a discul Lunii devine tangent exterior, moment numit ultimul contact inte care Soarele iese din totalitate. Dup momentul cnd cele dou discuri a a nou tangente exterior, ultimul contact exterior, eclipsa ia sfrit. Ansa s turor punctelor de pe suprafaa Pmntului pe unde trece pata de umbr s t a a banda de totalitate, a crei lime maxim poate ajunge la 260 km, cnd L a at a perigeu. n afara benzii de totalitate, unde eclipsa este parial, cele dou t a a interioare lipsesc. Determinarea momentelor caracteristice, la care se adaug un al cinci a momentul fazei maxime, ofer informaii importante pentru mecanica cer a t Acum circa 3000 de ani, caldeenii au notat datele la care observau fiec i au constatat c eclipsele de Soare i Lun se reproduc n condiii ident s a s a t interval de timp de 18 ani, 11 zile i 8 ore. Aceast perioad de repetare a s a a a primit numele de ciclu Saros (saros n limba arab nseamn repetiie a a t ciclu Saros au loc 71 de eclipse din care 43 de Soare si 28 de Lun. Ulte a mbuntirea mijloacelor de observare i dezvoltarea mecanicii cereti, s-a a at s s c perioada ciclului Saros nu a fost determinat cu suficinent precizie a a a condus la abateri mari n calcularea datei celei mai mari srbtori a cre a a srbtoarea Patelui. Ca exemplu, n anul acesta cretinii catolici au a a s s Patele la data de 16 aprilie, iar ortodocii la 23 aprilie. Diferenele dintre s s t date pot depi uneori i o perioad sinodic a Lunii (perioada sinodic as s a a a timpul necesar Lunii ca s treac prin dou faze succesive de acelai fel i ar a a a s s de 29 zile, 12 ore, 44 minute i 3 secunde). s n ziua de 29 martie a acestui an a avut loc o eclips total de Soar a a la Iai ca eclips parial , pentru care momentele caracteristice calculate (p s a t a sunt: nceputul eclipsei pariale - 12h 50m 53s, faza maxim - 14h 02m 02 t a eclipsei pariale 15h 12m 01s, iar acoperirea maxim a discului solar a fost t a Chiar i acum observarea eclipselor prezint interes tiinic pentru o s a s t mai bun a drumului parcurs de Pmnt n jurul Soarelui precum i pent a a s unor fenomene solare ce nu pot fi observate dect n timpul eclipselor. To vaiile fcute cu mare rigurozitate sunt binevenite. t a

Iulian BREAHN A Membru al Uniunii Astronomice Internai t Ex-director al Observatorului Astronomic d

OLIMPIADA 57

Cel care nsil aceste rnduri socotete (de multior) c olimpiadele d a a s s a tic sunt evenimente majore, pline de semnicaii pentru mult lume: pen a t a eroi care i ncearc puterile n munci mai relevante dect cele ale lui Hercu s a (venic tinerii) care lefuiesc cu migal subiecte sau evalueaz redactri, p s s a a a moii care asigur detalii organizatorice, pentru sponsorii care au avut nel s a t plaseze bine bnui trebuitori. Lista de mai sus a fost nevoit s sar peste a t a a a pregtesc bune prestaii ale elevilor n coal i acas: profesori (n genera a t s as a pect eforturi, profesori de matematic aprinztori de scntei, prini care a a a a t iubitori, editori care adun strdanii trecute spre a fi de folos celor care u a a S mai spunem c apelativul de olimpiad s-a conturat prin anii 50, ma a a a strdanii ale neasemuitului Grigore C. Moisil. (n 2006 s-a evocat n lu a centenarul naterii sale.) Exista buna experiena a concursurilor gazetei s t ce cu suflet, inim i minte donate de Gheorghe Tieica. Dar a exist a s t potrivnic dinspre rsrit care ar fi preferat un nume de spartakiad. Inst aa a a asigurat geneza acestor olimpiade a fost Societatea de Stiine Matematic t Este benefic s ne minunm de vigoarea cu care au crescut i s-au a a s olimpiadele. Erau 50, apoi o sut, apoi dou de finaliti; acum ar veni (e a a s cheltuial proprie) cam 10 mii de finaliti; spre a preveni auto-sufocri sa s a prin anul 2000 algoritm prin care s nu se treac de 600 de participani. a a t Buna experiena a olimpiadelor naionale a ndreptit Romnia s iniie t t at a t ade Internaionale, apoi Olimpiade Balcanice. Acum nala naional nu m t t a ultim scop; urmeaz ndat prime baraje, calicri la loturi (de seniori i d a a a s alte baraje, calicri n echipe naionale, variate ntreceri internaionale. a t t Dulcele Trg al Ieilor se ntlnise cu Olimpiada Naional de Matemati s t a (A fost atunci creditat la treab i semnatarul acestor rnduri.) Au mai ex as de rentlnire; ce susineau unii, boicotau alii. Iaul se armase prin anii t t s cial prin trio-ul de elevi Turinici Gheba Brsan, iar prin 99 prin multipl Marius Beceanu. Dintre vechi olimpici (dar nu numai) fuseser invitai a t Naionale distini profesori ieeni. Exist o familie olimpic ieean; citm t s s a a s a a pe ultimul: campionul Pachi. Loturi naionale se pregtiser n Bucium. t a a n calendar naional i Tabere organizate de ieeni. Se aprobase i se desf t s s s succes Concursul Naional "Al. Myller". Factori diriguitori susinuser t t a Naional de Matematic la Iai n 2005. A fost aprobare, dar s-a acordat t a a s Olimpiadei Balcanice; se tia c Olimpiada Naional din 2006 va fi la Ia s a t a s A fost i a ieit bine. Climatul intelectual al Iaului a constituit un fond s s s Profesori dotai i inimoi au contribuit cu capacitate i druire. Colegiil t s s s a i Naional au colaborat excelent ca gazde. Facultatea de matematic a a s t a academic; preedinte de onoare a fost academicianul Radu Miron. Au fos s ca propuntori sau evaluatori cam 15 profesori ieeni. A funcionat im a s t secretariat ce a nsumat cam 30 de profesori destoinici. Elevii ieeni au m s ceva din avantajul terenului, cifra pentru olimpiada naional crescnd de t a nc nu putem face un bilan al comportrilor internaionale, dar suntem a t a t

Prof. dr. Dan BRN

Pseudoinvers i invers generalizat as a a ale unei aplicaii liniare t


Adrian REISNER1

1. Pseudoinvers a unui endomorsm ntr-un spaiu vectori a t mensiune finit. Fie S un R-spaiu vectorial de dimensiune finit. Are a t a

Teorema 1. Fiind date dou endomorfisme u, v ale spaiului S, dac a t a cele trei condiii urm toare: t a a) uvu = u, b) vuv = v, c) rang u = rang v

sunt verificate, atunci a treia este de asemenea verificat . a Demonstraie. a) i b) c) Avem: rang u = rang(uvu) rang(uv t s din condiia a), rang v = rang(vuv) rang(vu) rang u din condiia b). t t condiia c). t a) i c) b) Condiia a) conduce, nmulind la dreapta cu v, la egalit s t t uv = (uvu)v = u(vuv).

Tinnd seama de a), avem, pe de alt parte, c rang u rang(uv) a a rang(uv) rang u, deducem egalitatea rang(uv) = rang u. Dar rangul smului uv este rangul restriciei endomorsmului u la subspaiul Im v. A t t stricie u/ Im v este deci injectiv i egalitatea () poate fi simplicat cu u t as a ceea ce nchide demonstraia. t b) i c) a) rezult imediat, endomorsmele u i v avnd roluri simet s a s laiile a), b), c). Teorema este demonstrat. t a

Corolar. Fiind dat un endomorsm u al spaiului vectorial S, exist t a morsm v (nu unic) vericnd condiiile a), b) si c). t Demonstraie. Tinnd seama de teorem, ne propunem s constr t a a morsmul v vericnd condiiile b) i c). Fie v un astfel de endomors t s pentru y Im v : vu(y) = y. Deducem c, dac x Im v Ker u, x = a a Im v Ker u = {0}. Suma acestor dou spaii vectoriale este deci direct a t a condiia rang u = rang v, deducem atunci c Im v Ker u = S. t a Fiind dat endomorsmul u, fie F un suplementar al subspaiului vecto t i G un suplementar al spaiului Im u. Aplicaia u0 : F Im u, x 7 u( s t t izomorsm. Definim aplicaia v prin restriciile sale la cele dou subsp t t a mentare Im u i G prin: s v/ Im u = (u0 )1 , v/G = 0. Aceast aplicaie v este evident liniar i verific: a t as a Im v = Im(u0 )1 = F , deci rang v = dim F = rang u i rezult c v v s a a x S, v(x) F , deci vuv(x) = v {u0 [v(x)]} = (vu0 )[v(x)] = v( verific vuv = v, adic b). a a Corolarul este astfel stabilit.
1

Cercettor, Centrul de Calcul E.N.S.T., Paris a

Observaie. Endomorfismul v nu este unic: el depinde de alegerea s t F si G din demonstraia corolarului. t

Definiii. Fiind dat endomorfismul u, orice endomorfism verificnd con t numete invers generalizat a lui u. Fiind dat endomorfismul u, orice en s a a verificnd condiiile a), b) se numete pseudoinvers a lui u. t s a

Cu aceleai notaii ca n corolarul precedent, o invers generalizat v a s t a a smului u este definit prin restriciile sale la subspaiile suplementare a t t prin: v/ Im u = (u0 )1 , v/G morfism oarecare aparinnd spaiului L(G, S). t t O invers generalizat a endomorfismului u depinde deci de alegerea subs a a G i de alegerea morfismului v/G . Vom numi Ing(u) = {v L(S) |uvu = s Observaii. 1) Inversa generalizat a unui automorfism u este unic t a a cu inversa u1 a lui u : Ing(u) = u1 . 2) Fie v Ing(u). Avem: endomorfismul w = uv este un proiector (w2 = w) de imagine Im u: w2 = (uv)(uv) = (uvu)v = uv = w, endomorfismul w0 = vu este un proiector (w02 = w0 ) de nucleu Ker u: w02 = (vu)(vu) = v(uvu) = vu = w0 .

Aplicaie. Vom nota cu aceeai liter un endomorfism al spaiulu t s a t Rn i matricea acestui endomorfism n baza canonic a lui Rn . Fiind s a endomorfisme A, B, C ale spaiului vectorial Rn , fie A Ing(A) i C In t s loc

Teorema 2. Ecuaia matriceal AXC = B admite o soluie X dac t a t a dac avem AAB CC = B. a Demonstraie. Presupunnd c AAB CC = B, deducem c matricea t a a verific AXC = B. Vom nota X0 = AB C. a Invers, dac exist X L(Rn ) vericnd AXC = B, atunci avem (AAA a a B i, n consecina, B = AA(AXC)CC = AAB CC. s t s A i C fiind dou matrice oarecare aparinnd respectiv la Ing(A) a t notm a n o K(A, C) = Y AAY C C | Y Mn (R) .

Teorema 3. Soluia general a ecuaiei matriceale AXC = B este X t a t Z, unde Z K(A, C). Demonstraie. Soluia general a ecuaiei matriceale AXC = B t t a t X0 + Z, unde X0 = AB C (v. demonstraia Teoremei 2) i Z este soluia t s t ecuaiei AZC = O. Ne propunem s justificm echivalena urmtoare: t a a t a a) AZC = O a) b) Z = Z O = Z A(AZC)C; b) Z K(A, C).

b) a) Y Mn (R), matricea Z = Y AAY C C verific a i Teorema 3 este demonstrat. s a

AZC = AY C AAAY C CC = AY C (AAA)Y (C CC) = O 0 0 0 0 , unde k R. 0 0

X M3 (R) a acestei ecuaii. t 0 0 1 0 0 i A = 1 0 0 Ing(A); C = s 0 0 0 0 k 0 0 i C = C Ing(C). Cum B = 0 0 0 = AAB CC, ecuaia (1) adm s t 0 0 0 0 0 0 o soluie fiind matricea X = AB C = k 0 0. Tinnd seama de T t 0 0 0 soluia general a ecuaiei (1) este t a t 0 0 0 X = AB C + Y AAY C C = k 0 0 + Y AAY C C, 0 0 0

S gsim soluia a a t general a 0 1 Avem: A = 1 0 0 0

Exemplu. Fie ecuaia matriceal t a 0 1 0 1 0 0 k 1 0 0 X 0 0 0 = 0 0 0 0 0 0 0 0

unde Y este o matrice oarecare aparinnd spaiului de matrice M3 (R). t t t Considernd a, b, c, d, e, f , g, h, i numere reale oarecare, obinem a b c 0 d e f , c soluia general a ecuaiei matriceale (1) este X = k a t a t g h i g unde b, c, e, f , g, h, i sunt numere reale oarecare.

2. Pseudoinversa unui endomorfism ntr-un spaiu euclidian t un spaiu euclidian, produsul scalar va fi notat h, i. [Aceleai definiii t s t s sunt valabile dac spaiul E este un spaiu hermitian, adic un C - spai a t t a t nzestrat cu un produs scalar, form hermitian pozitiv definit pe E.] a a a definiia i teorema urmtoare: t s a

Definiie i teorem. Fiind dat un endomorfism al spaiului euclidian t s a t un singur endomorfism u al spaiului E verificnd t Acest endomorfism se numete adjunctul endomorfismului u. s Avem Ker u = (Im u) si Im u = (Ker u) . Demonstrm c Ker u = (Im u) ; ntr-adevr, a a a Asemntor procedm pentru egalitatea Im u = (Ker u) . a a a y (Im u) x E, hu(x), yi = 0 x E, hx, u (y)i = 0 y x, y E, hu(x), yi = hx, u (y)i .

Fie u un endomorfism al spaiului euclidian E . Notm K = (Ker u) o t a nucleului lui u i I = (Im u) ortogonalul imaginii endomorfismului u. s Notm P proiecia ortogonal de imagine K (de nucleu Ker u), P a t a ortogonal de imagine Im u (de nucleu I = (Im u) ). Avem: P 0 u = a P 0 (y) Im u, y E; u(1) P 0 (y) (imaginea reciproc a elementului P a o clas modulo Ker u a spaiului euclidian E (atenie: u(1) nu este o a t t Imaginea acestei clase prin proiecia P este un element unic al spaiului K t t

s Definiie. Aplicaia liniar u+ = P u(1) P 0 se numete pseudoinvers t t a morsmului u.

Teorema 4. Aplicaia pseudoinvers u+ a endomorfismului u verific t a a) uu+ u = u, b) u+ uu+ = u+ , c) uu+ si u+ u sunt endomorfisme ortogonale, i.e. (uu+ ) = uu+ , (u+ u Mai mult, u+ este singurul endomorfism al spaiului E verificnd aseriu t t si c). Demonstraie. P i P 0 fiind proiectori, avem P u+ = u+ P 0 = u+ . At t s u+ u = P u(1) P 0 u = P u(1) u = P, proiector ortogonal, uu+ = uP u(1) P 0 = uu(1) P 0 = P 0 , proiector ortogonal, i, n consecina, uu+ u = uP = u; u+ uu+ = u+ P 0 = u+ . n plus, s t Im u+ = Im P = K = (Ker u) = Im u ,

Ker u+ = Ker P 0 = I = (Im u)

Demonstrm acum unicitatea endomorsmului u+ . Fie u0 un alt en a verificnd condiiile a), b) i c). Avem: t s u0 uu+ = u u0 u+ = (u u+ u )u0 u+ = (u+ u)(u0 u)u+ = u+ uu+ = Deci u0 = u+ , c.c.t.d.

u0 = u0 uu0 = u0 u0 u = u0 u0 (u u+ u ) = u0 (uu0 )(uu+ ) = u0 uu+

Observaie. Am vzut n prima parte c o pseudoinvers a lui u d t a a a particular de alegerea subspaiilor suplementare F i G ale spaiilor Ker u t s t s Im u. n cazul n care E este un spaiu euclidian F i G sunt unic de t s F = (Ker u) i G = (Im u) , de unde unicitatea pseudoinversei. s

Dac y este un vector oarecare al spaiului euclidian E, notm x0 = u+ a t a

a Teorema 5. Vectorul x0 = u+ (y) verific : a) kux0 ykeste minimum, i.e. kux0 yk kux yk, x E; b) kx0 k este minim din toate elementele x verificnd kux yk = kux0 + Demonstraie. a) kux0 yk = uu y y = kP 0 y yk, t
2

kux yk2 = ku(x x0 ) + ux0 yk2 = ku(x x0 ) + P 0 y yk = = ku(x x0 )k2 + kP 0 y yk kux0 yk2 .
2

b) Avem kux yk = kux0 yk numai cnd u(x x0 ) = 0. n caz 2 2 x = x0 + z, unde x0 K = (Ker u) i z Ker u. Deci kxk = kx0 k + kzk s i deducem b). s

Observaie. Din teorema precedent deducem c x0 = u+ (y) este cea t a a soluie apropiat n norm a ecuaiei y = ux. t a a t

Pseudoinversa unei matrice. Dac alegem n spaiul euclidian a t ortonormat B, obinem U -matricea endomorsmului u n aceast baz: M a t a a T T U = U .[n cazul n care E este hermitian avem M at(u , B) = U = U Traducnd cele de mai sus n limbaj matriceal, obinem t

Fiind dat o matrice U Mn (R), exist o singur matrice V verificnd a a a urm toare: a U V U = U, V U V = V, (U V ) = U V, (V U ) = V U.

Aceast unic matrice V se numete pseudoinversa matricei U si se note a a s

X0 = U + Y este cea mai bun aproximaie cuadratic a ecuaiei U X = a t a t

Exemplu de calcul al pseudoinversei unei matrice. Fie matricea 1 0 1 U = 0 1 0 M3 (R). 1 0 1 S calculm matricea U + . a a Matricea U fiind simetric, avem Ker u = (Im u) . a

Fie B = {e1 , e2 , e3 } baza canonic a spaiului R3 nzestrat cu produ a t canonic. Pentru endomorfismul u asociat matricei U n aceast baz, avem a a Endomorfismul v de matrice U Im u = Vect {e2 , e1 + e3 } ,
+

Ker v = (Im u) = Ker u;

n baza canonic B verific atunci a a

Ker u = R(e1 e3 ).

Im v = (Ker u) = Im u.

Pe de alt parte: a uvu(e2 ) = u(e2 ) uv(e2 ) = u(e2 ) v(e2 ) = e2 + (e1 e3 ); v(e2 ) Im u = 0, deci v(e2 ) = e2 ; e1 + e3 uvu(e1 ) = u(e1 ) uv(e1 + e3 ) = e1 + e3 = u( ) v(e1 + e3 ) = 2 e1 + e3 (e1 e3 ); v(e1 + e3 ) Im u = 0, deci v(e1 + e3 ) = . Da 2 e1 + e3 v(e1 e3 ) = 0 [Ker v = Ker u], deducem c v(e1 ) = v(e3 ) = a . 4 + Finalmente matricea U este 1 0 1 1 0 4 0 . U+ = 4 1 0 1

O soluie parial a unei probleme a lui N. Pa t t a


C t lin TIG aa AERU 1

Profesorul Nicolae Papacu analizeaz n [1] recurena ptratic, subl a t a a tul c natura irului este dictat de alegerea termenului iniial i las ca a s a t s a deschis cazul (III.3.d). Ne propunem s dm un rspuns parial acestei p a a a a t

I. O problem deschis i cteva rezultate generale. Recurena a as t xn+1 = ax2 + bxn + c, x0 R, a 6= 0, n b 2 t se reduce, cu substituia yn = axn + , la recurena yn+1 = yn + , t 2 1 t a a a (2b ), cu = b2 4ac. Situaia propus ca problem deschis, notat 2 n articolul citat, se refer la cazul n care < 0 i y0 (L2 , L2 ) \ { a s 1 a t a L1,2 = 1 1 4 reprezint punctele fixe ale recurenei. Dac p 2 2 a t , atunci ne rmne de studiat recurena y0 [L2 , L2 ] , yn+1 = f (yn ) , n N, unde f : R 2 , , f (x) = x2 este funcia ataat recurenei, care este surjectiv, strict descresctoare pe t s a t a a strict cresctoare pe [0, ) i care are punctele fixe a s p p 1 1 L1 = 1 1 + 4 2 , L2 = 1 + 1 + 4 2 . 2 2 Introducem notaia: dac g : M M , atunci g g . . . g = g t a compunerea s-a efectuat de n ori. Astfel, constatm c recurena (1) se m a a t [n] y0 [L2 , L2 ] , yn+1 = f (y0 ) , n N. Analiza care urmeaz va stabili urmtoarele tipuri distincte de comport a a irurilor studiate: (i) iruri convergente, care nu sunt constante de la un s s (ii) iruri convergente, constante de la un loc ncolo; (iii) iruri mrginite, f s s a cu mulimea valorilor finit; (iv) iruri mrginite, fr limit, cu mulime t a s a aa a t infinit; (v) iruri nemrginite, cu limita egal cu . a s a a

O condiie suficient, care s asigure mrginirea irului (yn )nN , este d t a a a s a Propoziia 1. Dac 0 < 2, atunci sirul (yn )nN este m rginit. t a Demonstraie. Pornim cu observaia c f ([L2 , L2 ]) = 2 , L2 ; d t t a L2 2 (), atunci 2 , L2 [L2 , L2 ], de unde 2 2 f , L2 = f , 0 [0, L2 ] = f ([0,L2 ]) = 2 , L2 . 2 Demonstrm, prin inducie, c f [n] ([L2 , L2 ]) = , L2 , n N . a t a [n1] dac presupunem c f a a ([L2 , L2 ]) = 2 , L2 , atunci f [n] ([L2 , L2 ]) = f f [n1] ([L2 , L2 ]) = f 2 , L2 = 2 , L Deducem c f [n] ([L2 , L2 ]) = 2 , L2 , n N , adic, dac este a a a condiia (), atunci irul (yn )nN este mrginit. Rezolvnd inegalitatea i t s a s t s t a de > 0, obinem 0 < 2 i demonstraia este ncheiat. t
1

Lect. dr., Universitatea S tefan cel Mare, Suceava

Tinnd cont de acest rezultat, mprim analiza noastr n dou ca at a a 2 i > 2. Teoremele materialului prezent aduc lmuriri doar n p s a n al doilea limitndu-ne la cteva comentarii i la formularea unei conjec s

Lema 1. Dac (yn )nN este sirul definit de (1), atunci yn > 0, n a numai dac y0 = L2 . a Demonstraie. S presupunem prin reducere la absurd c exist (yn )n t a a a de (1) astfel nct yn > 0, n N, cu y0 (0, L2 ). Demonstrm c, a a ipotez, irul ar fi strict descresctor: n adevr, dac y0 (0, L2 ), atunc a s a a a f (x) < x, x (0, L2 ), ar rezulta c y1 < y0 , de unde, innd cont de f a t este strict cresctoare pe (0, ), s-ar deduce, prin inducie, c yn+1 < y a t a Mai departe, ar rezulta c irul este covergent, deci limita ar fi unul di a s fixe L1 sau L2 . Cum irul este strict descresctor, s-ar deduce c lim yn s a a n ceea ce contrazice faptul c yn > 0, n N. Constatarea faptului c, dac a a atunci yn = L2 , n N, ncheie demonstraia. t

a Lema 2. Fie (yn )nN sirul definit de (1); dac > 1, atunci yn < 0 dac si numai dac y0 = L1 . a a Demonstraie. Considerm un invers la dreapta al funciei f : t a t p 2 h : , L2 [L2 , 0] , h (x) = x + 2 , unde > 1; evident h satisface (f h) (x) = x, x 2 , L2 . Deoar deducem c 2 < < L1 < 0, de unde rezult c h ([, 0]) [, 0 a a a constatare permite construirea irului (n )nN , definit astfel: s 0 = 0, Tinnd cont de faptul c h este strict descresctoare i de faptul c L1 a a s a punct fix al funciei h, se demonstreaz prin inducie c t a t a s a a i c lim n = L1 . Definim irul de intervale dup cum urmeaz: s a
n

n+1 = h (n ) , n N.

= 1 < 3 < < 2n1 < < L1 < < 2n < < 2 <

I0 = (1 , 3 ), In = (2n1 , 2n+1 ), n N i J0 = (2 , 0 ), Jn = (2n+2 , s

Constatnd c h (In ) = Jn , n N , h (Jn ) = In+1 , n N, obinem (h h a t [n] In , (h h) (J0 ) = Jn , ceea ce conduce la (f [2n1] ) (In ) = J0 , (f [2n] ) (Jn ) = J0 , n N .

t Dac y0 [L2 , 0), atunci avem de analizat situaiile: a (a) y0 [L2 , ); atunci y1 = f (y0 ) > 0; (b) y0 {n | n N}; atunci exist un m N astfel ca y0 = m = h[m a unde obinem c ym = f [m1] h[m1] (0) = 0; mai departe ajungem la y t a (c) Dac y0 [, 0] \ {{n | n N} {L1 }}, atunci: (c.1) dac y0 < a a un m N astfel nct y0 Im , deci y2m1 = f [2m1] (y0 ) J0 , de u f (J0 ) = 2 , , ceea ce conduce la y2m+1 f 2 , (0, L2 ); [2m] y0 > L1 , exist un m N astfel nct y0 Jm , deci y2m = f a (y0 ) J y2m+2 (0, L2 ).

S-a demonstrat c, dac y0 [L2 , 0] \ {L1 }, atunci m N astfel nc a a Deoarece, dac y0 = L1 , atunci yn = L1 < 0, n N, deducem c yn < a a dac i numai dac y0 = L1 . as a Urmtorul rezultat marcheaz locul apariiei irurilor periodice. a a t s

Propoziia 2. Dac 1 , atunci: t a a) ecuaia f (x) = h (x) are o unic soluie pozitiv a [0, ); t a t a b) dac y0 = a, atunci sirul (yn )nN , definit de (1), este periodic, cu y2 a y2n+1 = f (a) < 0. p Demonstraie. a) Fie funcia g (x) = f (x) h (x) = x2 + x + t t p este strict cresctoare pe [0, ). Deoarece g (0) = 2 0, g () = a (egalitatea apare pentru = 1), deducem c ecuaia g (x) = 0 are o sol a t a [0, ) [0, L2 ). b) se deduce din (f h) (x) = x, x 2 , L2 . n adevr, dac y0 = a a y2 = f (f (a)) = f (h (a)) = a, de unde, prin inducie obinem y2n = a t t Analog vom obine y2n+1 = f (a), n N . q.e.d. t

II. Analiza cazurilor corespunztoare lui 0 < 1. a 1. Cazul 0 < < 1. Comportamentul irului (1) este descris de s L1 ; y0 (L2 , L Teorema 1. Dac 0 < < 1, atunci lim yn = a L2 ; y0 {L2 , L n Demonstraie. n primul rnd, demonstrm c, dac y0 (L2 , L t a a a exist n0 2 astfel nct yn 2 , 0 , n n0 . n adevr, dac y0 a a a atunci y2 2 , 0 ; demonstrm prin inducie c yn 2 , 0 , n a t a 2 yn1 2 , 0 , atunci yn = yn1 2 4 2 = 2 2 1 < 0; cum yn 2 2 2 , obinem yn , 0 , n 2. S-a demonstrat c, dac y0 t a a atunci yn 2 , 0 , n 2. Dac yn (L2 , ) (, L2 ), din Lema 1 deducem c n0 astfel ca a a Deoarece y1 > 0,rezult c n0 2. Pentru c yn0 = f (yn0 1 ), cu n0 a a a deducem c yn0 2 , 0 , de unde, ca mai sus, rezult c yn 2 , 0 , a a a a a a Am demonstrat c, dac y0 (L2 , L2 ), atunci exist n0 2 astfel 2 , 0 , n n0 . Mai departe, dac y0 (L2 , L2 ) \ {L1 }, atunci, deoa a strict descresctoare pe 2 , 0 , obinem, prin inducie, urmtoarele con a t t a 2 (a) n cazul yn0 , L1 , avem m N yn0 < yn0 +2 < < yn0 +2m < < L1 < < yn0 +2m+1 < < yn0 +3 (b) n cazul yn0 (L1 , 0), avem m N
n

a n ambele cazuri se deduce lim yn = L1 . Dac y0 = L1 , atunci yn = L

yn0 +1 < yn0 +3 < < yn0 +2m+1 < < L1 < < yn0 +2m < < yn0 +

deci, din nou lim yn = L1 . Tinnd cont c, dac y0 {L2 , L2 }, at a a n L1 , n 2, deci lim yn = L2 , demonstraia se ncheie. t
n

2. Cazul = 1. Conform Propoziiilor 1 i 2, vor apare iruri mrg t s s a limit. Pentru a enuna rezultatul, construim mulimea M0 astfel: pentru a t t

n N , considerm mulimile M0 = x [L2 , L2 ] | f [n] (x) = 0 i fie M0 = a t s

Teorema 2. Dac = 1, atunci sirul (yn )nN este convergent dac a a L1 ; y0 (L2 , L2 ) \ M0 , dac y0 [L2 , L2 ] \ M0 si lim yn = a L2 ; y0 {L2 , L2 } . n Dac y0 M0 , atunci (yn )nN nu are limit si exist n1 N a a a a 1 n yn = ((1) 1), n n1 . 2 Demonstraie. Se demonstreaz prin inducie c, dac y0 (1, 1), a t a t a a (1, 0), n 1. Mai departe, judecnd ca n Teorema 1, deducem c lim a

2n S notm faptul c, pentru fiecare n N , avem 0 M0 , {1} M0 a a a mult, dac M0 , atunci avem i incluziunea x [L2 , L2 ] | f [n] (x) = a s

s a S notm deocamdat c L1 ML1 , L2 ML2 i c ML1 ML2 = . a a a a Teorema 3. Dac 1 < 2, atunci sirul (yn )nN este conver a si numai dac y0 ML1 ML2 si lim yn = L1 dac y0 ML1 si = a a n y0 ML2 . Dac y0 [L2 , L2 ] \ {ML1 ML2 }, atunci sirul (yn )nN este m rgin a a limit si conine o infinitate de termeni negativi si o infinitate de termen a t Mai mult, exist valori ale lui y0 pentru care mulimea valorilor sirului es a t valori ale lui y0 pentru care mulimea valorilor sirului este infinit . t a Demonstraie. S presupunem c y0 [L2 , L2 ] \ {ML1 ML2 } t a a supunem prin absurd c exist n0 N astfel ca yn > 0, n n0 . Atu a a considera irul (n )nN definit astfel s y y0 = yn0 , yn+1 = f (n ) , n N . y Deoarece yn = yn+n0 > 0, n N, deducem, pe baza Lemei 1, c a adic yn0 = L2 ; dar aceasta conduce la y0 ML2 , fals. Presupunem p a c exist n0 N astfel ca yn < 0, n n0 . Lund n considerare iru a a s definit de (), deducem c yn = yn+n0 < 0, n N, ceea ce, n virtutea a conduce la y0 = L1 , adic yn0 = L1 , aceasta nsemnnd c y0 ML1 , fal a a S-a demonstrat astfel c, dac y0 [L2 , L2 ] \ {ML1 ML2 }, atunci ir a a s este mrginit, nu are limit i conine o infinitate de termeni pozitivi i o a as t s de termeni negativi. Dac y0 ML1 ML2 , deducem c lim yn {L1 , L a a n Propoziia 2 semnaleaz faptul c, n acest caz, exist iruri periodic t a a as general, iruri cu mulimea valorilor finit. Demonstrm n continuare c p s t a a a

Dac y0 M0 , atunci exist n0 N astfel nct yn0 +1 = f [n0 ] (y0 a a 1 s t continuare obinem yn = ((1)n 1), n n1 = n0 + 1 i demonstraia t 2 3. Cazul 1 < 2. Pentru fiecare n N , considerm mulimile a t n o [ [ n [n] n ML1,2 = x [L2 , L2 ] | f (x) = L1,2 , ML1 = ML1 , ML2 =
n=1

S presupunem c y0 (L2 , 1) (1, L2 ) \ M0 ; conform Lemei 1, exi a a astfel ca yn0 < 0. Deoarece y0 M0 , rezult c f [n0 ] (y0 ) 6= 0, deci yn0 + / a a unde 1 < yn0 < 0. La fel ca mai sus, obinem lim yn = L1 . t
n

n=

y0 astfel nct mulimea valorilor irului s fie infinit. Pentru aceasta, c t s a a aplicaia A : N [X], unde R [X] este inelul polinoamelor cu coeficien t N = N N \ {(n, n) | n N}, definit prin A (n1 , n2 ) = f [n1 ] (X) f [n2 ] (X a Considerm mulimea S care are ca elemente toate rdcinile din intervalu a t a a ale polinoamelor ce aparin imaginii funciei A. Cum S este numrabi t t a y0 = b [L2 , L2 ] \ S; demonstrm c funcia Y : N [L2 , L2 ], Y (n) a a t injectiv: n adevr, dac ar exista n1 , n2 N, n1 6= n2 , astfel ca Y (n1 ) a a a atunci am avea f [n1 ] (b) = f [n2 ] (b), ceea ce ar nsemna c polinomul g a g (X) = f [n1 ] (X) f [n2 ] (X), are rdcina b S, ceea ce este fals. Q.e.d. a a

III. Completri i comentarii finale. 1. Propoziia 2 asigur, pen a s t a existena irurilor periodice, de perioad 2. Fr a intra n detalii, preciz t s a aa a se pot construi iruri care au perioade mai mari dect 2. s 2. Pentru = 2 se cunoate forma analitic a termenului genera s a situaie fcnd obiectul mai multor probleme de concurs (vezi [1], [3] ). t a 3. n cazul > 1, putem constata: (a) dac y0 ML1 ML2 , atunci lim a n dac y0 ML1 i = L2 dac y0 ML2 . a s a (b) Conform propoziiei 2, exist iruri periodice sau iruri periodice d t as s ncolo, deci iruri mrginite, cu un numr finit de valori. s a a s (c) Exist posibilitatea de a alege y0 [L2 , L2 ] astfel nct irul (yn )n a limita egal cu ; de exemplu, dac este y0 = 0, atunci y1 = 2 , y2 = 4 a a de unde lim yn = . Ceea ce nu putem preciza este dac exist posib a a
n

a alege y0 [L2 , L2 ], astfel nct irul s fie mrginit i mulimea valor s a a s t infinit. n legtur cu acest subiect propunem conjectura: a a a 1 p p 1 Sirul (yn )nN , definit de recurena y0 1 + 1 + 4 2 , 1 + t 2 2 2 a a a yn+1 = yn 2 , > 2, este m rginit dac si numai dac este periodic d ncolo. 4. Sintetizm concluziile articolului a Valorile lui (0, 1) ; = 1; 2 1, ; 2, ; n urmtorul tabel: a Tipul de iruri s (i) , (ii) ; (i) , (ii) , (iii) ; (ii) , (iii) , (iv) ; (ii) , (iii) , (v) , (iv?) .

5. Dl prof. Gheorghe Marchitan a citit cu atenie materialul, mbun t prin observaii. Acelai efect benefic l-au avut i discuiile purtate cu dl pr t s s t Marchitan. Amndurora, autorul le mulumete i pe aceast cale. t s s a

Bibliografie 1. N. Papacu - Asupra unui sir recurent, Arhimede, 5-6/2000, 1-4. 2. Gh. Oprian - On a class of real sequences defined by recurence, Gazeta M s (seria informare tiinific i perfecionare metodic), XIV(XCIII), 2/1996 s t as t a 3. C. Tigeru - Recurena an = 2a2 1 si problema XI.3 a Concursulu a t n1 "Sinus", Sinus, II.1(4)/2006.

Criterii de congruena a triunghiurilor t

Marius APETRII 1 si Cristian-C t lin BUDEAN aa

n RecMat - 2/2005, elevilor de clasa a VII-a le este propus urmtoarea a a

VII.63. Fie triunghiurile ABC si A0 B 0 C 0 cu AB = A0 B 0 , AC = A0 C toarele [AD], [A0 D0 ] congruente. S se arate c 4ABC 4A0 B 0 C 0 (enun a a Petru Asa

n cele ce urmeaz, dorim s stabilim condiii suficiente pentru congrue a a t triunghiuri care au dou bisectoare congruente. Vom considera mereu a ipoteza A (I) Triunghiurile ABC si A0 B 0 C 0 au bisectoarele [AD] si [A0 D0 ] congruente, cu D (BC), D0 (B 0 C 0 ), iar \ \ m(ADB) 90 , m(A0 D0 B 0 ) 90 . \ Propoziia 1. n ipoteza (I), dac BAC B 0 A0 C 0 si t a \ \ B 0 D0 A0 , atunci 4ABC 4A0 B 0 C 0 . \ BDA Demonstraie. Din congruena triunghiurilor ADB i t t s \ A0 D0 B 0 (U.L.U.) rezult c AB = A0 B 0 i ABD A0 B 0 D0 , a a s \ de unde concluzia urmeaz conform U.L.U. a

B
A

\ Propoziia 2. n ipoteza (I), dac BAC B 0 A0 C 0 si t a \ D B AB = A0 B 0 , atunci 4ABC 4A0 B 0 C 0 . 0 0 0 Demonstraie. Obinem c 4ABD 4A B D (U.L.U.), de und t t a \ a A0 B 0 C 0 . Folosind U.L.U., deducem c 4ABC 4A0 B 0 C 0 .

\ Propoziia 3. n ipoteza (I), dac BAC B 0 A0 C 0 si BD = B 0 t a \ 0 0 0 4ABC 4A B C . \ \ Demonstraie. Deoarece BAD B 0 A0 D0 , AD = A0 D0 , BD = B 0 D t 0 D0 B \ 0 sunt ambele neobtuze, conform L.L.U. rezult c 4ADB 4 a a A acum concluzia este imediat. a

\ Propoziia 4. n ipoteza (I), dac AB = A0 B 0 si ABC A0 B 0 t a \ 4ABC 4A0 B 0 C 0 . \ \ Demonstraie. Deoarece AB = A0 B 0 , AD = A0 D0 , ABC = A0 B 0 C t 0 D0 B \ 0 sunt ambele neobtuze, conform L.L.U. rezult c 4ADB 4A a a A unde concluzia anunat. t a

\ Propoziia 5. n ipoteza (I), dac ADB A0 D0 B 0 si BC = B 0 t a \ 0 0 0 4ABC 4A B C . Demonstraie. Considerm punctul A00 n acelai semiplan cu A fa t a s \ astfel nct 4A00 BC 4A0 B 0 C 0 . Fie [A00 D00 bisectoarea lui BA00 C, D00 00 00 00 \ \ {E} = A B AD, {F } = A D AC. Din ADB A00 D00 B rezult c A a a
1 2

Asist. drd., Facultatea de Matematic, Univ. Al. I. Cuza, Iai a s Profesor, C. N. Emil Racovi a, Iai t s

i cum AD = A00 D00 , obinem c ADD00 A00 este paralelogram. Astfel, AA s t a 00 \ \ s \ \ deci A Ext ABC, de unde m(A00 CB) m(ACB) i m(ABC) m(A invers; ne plasm n prima situaie. a t [ A A Din AD k A00 D00 obinem c m(EAC) = t a 00 \ ) = i m(AEA00 ) = m(EA00 F ) = . \ \ m(AF A s \ Atunci m(ABE) = 180 (180 ) = i s E F \ m(A00 CF ) = 180 (180 ) = . Cum \ 0, deducem c = . Astfel, m(A00 CA) = 0, a \ deci C, A00 , A sunt coliniare i m(ABA00 ) = 0, deci s B, A00 , A sunt coliniare. n concluzie, A00 = A, de unde 4ABC 4A0 B 0 C 0 .

D D

Propozia 6. n ipoteza (I), dac AB = A0 B 0 si AC = A0 C 0 , atunci t a 4A0 B 0 C 0 . (Problema VII.63 din RecMat - 2/2005) C Demonstraia 1. Considerm punctul C 00 n acet a lai semiplan cu C faa de AB, astfel nct 4ABC 00 s t \ 4A0 B 0 C 0 . Fie [AD00 bisectoarea lui BAC 00 , D00 D BD AB a (BC 00 ). Din teorema bisectoarei avem c = CD AC AB BD00 00 i cum AC = AC, deducem c s a i 00 00 = s C D AC 00 A BD BD00 s = 00 00 , deci DD00 k CC 00 . Triunghiurile ACC 00 i ADD00 sunt CD C D atunci A se afl att pe mediatoarea lui [DD00 ], ct i pe cea a lui [CC 00 ] a s dou mediatoare sunt paralele sau coincid; rmne cea de-a doua varian a a rezult c A aparine medianei din B n 4BCC 00 , fals. Deducem c C = a a t a 4ABC 4A0 B 0 C 0 .

Observaia 1. n aceast demonstraie nu folosim explicit faptul c A t a t a BD B 0 D0 sunt bisectoare, ci doar egalitatea rapoartelor = 0 0 . Astfel, auto DC DC mei, Petru Asaftei, observ c rmne adevrat concluzia dac nlocuim a a a a a a a a bisectoarele [AD] i [A0 D0 ] cu dou ceviene ce determin rapoarte egale pe s care cad. n particular, concluzia rmne adevrat pentru [AD], [A0 D a a a sau simediane. Observaia 2. Am putea denumi rezultatul Propoziiei 6 cazul de c t t Latur -Bisectoare-Latur . Claudiu- tefan Popa ridic problema funcio a a S a t cazuri Bisectoare-Latur -Bisectoare sau Bisectoare-Bisectoare-Bisectoare; a cititorului demonstrarea sau invalidarea acestora. Pentru o eventual justi a promiatoare pare calea urmat n t a 2bc 2b0 A Demonstraia 2. Cu notaiile uzuale, la = t t cos i la = 0 s 0 b+c 2 b + A A0 0 0 0 t Cum la = la , b = b , c = c , obinem cos = cos , de unde A = A0 . 2 2 rezult din L.U.L. a \ Propoziia 7. n ipoteza (I), dac ABC A0 B 0 C 0 si BC = B 0 t a \

4ABC 4A0 B 0 C 0 . Demonstraie. Mai nti, s observm c: t a a a \ < 90 AB < AC. ntr-adevr: (i) m(BDA) a \ m(BDA) < 90 BD > c cos B

(ii) n 4ABC, AB < AC, considerm A00 cu A (BA00 ) i fie [A a s \ respectiv BA00 C, D, D00 (BC); pe segme \ bisectoarele unghiurilor BAC, avem atunci ordinea B D D00 C. ntr-adevr, dac AC = b, BC = a a a a a \ A00 B = c00 , A00 C = b00 i m(ABC) = , avem c + 00 > 2 cos . R s a c c evident pentru 90 , iar pentru < 90 , folosind teorema cosinusului a c < b, avem: a a a b2 = a2 + c2 2ac cos a2 > 2ac cos > 2 cos + 00 > c c c Atunci a a a a a a 00 + 00 > 00 2 cos c c c c c c a2 b002 b D00 C b00 DC > 002 > 00 00 < D (BD00 ) . c2 c c c D B DB A Revenim la demonstraia propoziiei; putem pret t 0 0 E supune fr a restrnge generalitatea c AB A B . aa a A Considerm A00 [BA astfel nct 4A00 BC 4A0 B 0 C 0 a 00 0 0 00 (deci A B = A B ). Cum A (BA ), conform observaiei (ii), pe (BC) avem ordinea BDD00 C i atunci t s \ \ \ \ m(BAC) > m(BA00 C), adic m(BDA) < m(BD00 A00 ). a \ Construim DE k A00 D00 , E AB. Cum m(BAD) < 90 , B D D \ s a a atunci m(EAB) > 90 i deci ED > AD. Din DE k D00 A00 rezult c

ac a2 + c2 b2 > b+c 2a h i 2 2 2a2 c > a2 (b + c) + (c b) (b + c) (b c) (b + c) a2 > 0 b

BE < 1, de unde DE < D00 A00 . Obinem astfel c AD < A00 D00 , fals. R t a BA00 00 0 0 0 AB = A B, prin urmare 4ABC 4A B C .

Propoziia 8. n ipoteza (I), dac AB = A0 B 0 si BC = B 0 C 0 , atunci t a 4A0 B 0 C 0 . \ \ Demonstraie. Presupunem prin absurd c m(ABC) < m(A0 B 0 C 0 ), t a A0 C 0 . Cu notaiile uzuale, avem: t i h " # 2 bc (b + c)2 a2 2bc a2 A 2 = = bc 1 < AD = cos b+c 2 (b + c)2 (b + c)2 h i # " 2 2 b0 c (b0 + c) a2 a 2b0 c A0 = = 0 < b0 c 1 cos =A 2 2 b +c 2 (b0 + c) (b0 + c)

deci, AD < AD0 , ceea ce contrazice ipoteza. Presupunerea fcut este a a urmare 4ABC 4A0 B 0 C 0 .

Asupra problemei 3639 din Gazeta Matematic, v. XXXIII (19271 a


D. M. B ATINETU-GIURGIU 1

n volumul XXXIII (1927-1928) al Gazetei Matematice, la pagina 28 Ionescu2 a propus problema 3639 cu urmtorul enun: a t m e 1 e = , 1+ lim m m m 2 m e 1 11e e + lim m m 1+ = , m m 2 24 m e 1 11e 21e e + 1+ lim m m m = , m m 2 24 48 e fiind baza logaritmilor neperieni. S se demonstreze c a a

Soluia acestei probleme a fost dat de d-nii G. G. Constantinescu, t a Olteanu, George Sila n Gazeta Matematic, vol. XLII (1936-1937), pag. s a Mai nti se arat c: a a m 1 1 21 am 11 =e 1 + , + em = 1 + m 2m 24m2 48m3 m4 am = 0. Din (4) rezult limitele din enun. a t unde lim m m4 vergent de numere reale cu lim xn = x = D0 (xn ) R.
n not

n aceast Not ne propunem s generalizm limita (1). Fie (xn )n1 u a a a a

a a a Definiie [1]. Spunem c irul (xn )n1 admite o derivat dac exist t as
not n

a as lim (n (xn x)) = D (xn ) R. Dac D (xn ) R, spunem c irul (x derivabil. r s Propoziia 1. Sirul (un )n1 , un = 1 + t , unde r R , s + n derivabil. Demonstraie. Dac s = 0, atunci un = 1, n N; deci n (un t a n N , deci lim (n (un 1)) = 0. n r s Dac s R , atunci D0 (un ) = lim un = lim 1 + a = 1. Deci + n n n un 1 ln un = 1 lim (ln un ) = ln lim lim (n (un 1)) = lim n n n n n n ln un sn n rs r r r = ln lim 1 + 1+ = ln ers = rs = ln lim n n n n
1 2

Profesor, Colegiul Na ional "Matei Basarab", Bucureti t s Matematician romn (19011985)

ceea ce demonstreaz propoziia. a t Propoziia 2. Dac (xn )n1 este un sir convergent cu D0 (xn ) = t a

= x R , atunci sirul (xn )n1 este derivabil dac si numai dac exist lim a a a n = d R . + Demonstraie. Este evident c t a x ! n(xn x) x x n 1 xn x xn x lim (n(xn n lim = lim = e x n 1+ n n x x x n 1 li n = d R , atunci d = e x n Relaia (5) ne arat c, dac exist lim t a a a a + n x Prin urmare exist i D (xn ) = lim (n (xn x)) = x ln d R, adic iru as as n este derivabil. Reciproc, relaia (5) ne arat c, dac irul (xn )n1 este derivabil, atu t a a as x n 1 lim ( n D (xn ) = lim (n (xn x)) R, deci exist i d = lim as = e x n n n x 1 e x D(xn ) R . Cu aceasta propoziia este demonstrat. t a +
n n

n continuare vom nota lim xn = D0 (xn ) = x R i lim yn = D0 (yn s

Propoziia 3 [2]. Dac sirurile (xn )n1 , (yn )n1 sunt derivabile, a t a (xn yn )n1 este derivabil si D (xn yn ) = D (xn ) y + xD (yn ). Demonstraie. Conform enunului avem: t t D0 (xn ) = x = lim xn ,
n

D0 (yn ) = y = lim yn R
n

i, de asemenea, s D (xn ) = lim (n (xn x)) ,


n

D (yn ) = lim (n (yn y)) R.


n

Prin urmare, trebuie de artat c a a D (xn yn ) = lim (n (xn yn xy)) R.


n

ntr-adevr, a n (xn yn xy) = n (xn yn xyn + xyn xy) = yn n (xn x)+xn (yn y) , de unde, prin trecere la limit cu n , deducem: a
n n n n

D(xn yn ) = lim (n (xn yn xy)) = lim (n (xn x)) lim yn + x lim (n (y = D (xn ) D0 (yn ) + D0 (xn ) D (yn ) .

Cu aceasta propoziia este demonstrat. t a n continuare, fie r R , s R+ i vom adopta notaiile: s t + n+s r r n , en (r) = en (r, 0) = 1 + , en (r, s) = 1 + n n n r s 1 , en = 1 + , n N . un = 1 + n n

r n Se observ c lim en (r, s) = er , lim en (r, 0) = lim 1 + a a = er , li n n n n n lim en = e.


n

Propoziia 4. Sirul (en (r))n1 este derivabil si t D (en (r)) =

r2 er . 2 Demonstraie. Conform inegalitilor Bencze-Tth din [3] avem t at er er 2 < er en (r) < , n N , unde a = 2 ; b, c R+ an + b an + c r Prin nmulire cu n, deducem c t a er n er n < n (er en (r)) < , n N . an + b an + c De aici, prin trecere la limit cu n , rezult c a a a er r2 er D (en (r)) = lim (n (en (r) er )) = = . n a 2 Teorem. Sirul (en (r, s))n1 este derivabil si a D (en (r, s)) = lim (n (en (r, s) er )) =

rer (2s r) . n 2 Demonstraie. S observm c en (r, s) = en (r) un , n N . Deoare t a a a (en (r))n1 i (un )n1 sunt iruri derivabile, conform Proprietii 3 rezul s s at (en (r, s))n1 = (en (r))n1 (un )n1 este un ir derivabil i s s
n n

D (en (r, s)) = D (en (r) un ) = D (en (r)) lim un + lim en (r) D (un r2 er rer = 1 + er rs = (2s r) . 2 2

Observaie. n particular, dac r = 1, s = 0 obinem relaia (1) din t a t t 3639 a lui D. V. Ionescu, iar dac r = s = 1 obinem c a t a !! n+1 e 1 lim n e = . 1+ n n 2 Bibliografie

1. M. Btineu-Giurgiu, D. M. Btineu-Giurgiu, M. Bencze - Siruri a t a t Gazeta Matematic, nr. 9/2005, 410-420. a 2. M. Btineu-Giurgiu, D. M. Btineu-Giurgiu, M. Bencze - Der a t a t quences, Octogon Mathematical Magazine, 13 (2005), no. 2, 936-945. 3. M. Bencze, L. Tth - O generalizare a inegalit tii Plya-Szeg, Arh a 3-4/2005, 7-10.

O problem despre suma cifrelor unui numr n a a n baze de numeraie oarecare t


Adrian ZAHARIUC 1

1. Introducere. Studiul cifrelor puterilor unui numr ntreg este o a dificil de teoria numerelor. Pentru ultimele progrese n acest sens, citit a consulta [2]. Exist multe probleme, evidente din punct de vedere int a nc sunt departe de a fi rezolvate. Chiar i faptul c suma cifrelor lui a a s a infinit nu este att de evident, fiind un rezultat binecunoscut al lui Sch Printre ntrebrile ce apar, una pare adevrat dincolo de orice ndoial, d a a a a imposibil de atacat. Propoziia 1. a si b sunt numere naturale nenule astfel nct t s(an ) = s(bn )

pentru orice n. Atunci lg a lg b Z. (s (N ) noteaz suma cifrelor num a a

Observaie. Desigur, lg a lg b = lg a/b, deci lg a lg b Z nseamn t a a = b sau c numrul mai mare dintre a i b este obinut din cellalt prin a a s t a ctorva 0-uri la sfrit. Este evident c reciproca acestui fapt este adevr s a a Propoziia 2. a si b sunt numere naturale nenule astfel nct t s(an) = s(bn) pentru orice n. Atunci lg a lg b Z.

Este clar c aceasta este o variant mai slab a problemei anterioare a a a (2) este mult mai restrictiv dect (1); n caz c (2) este adevrat, avem a a a a s(an ) = s(an1 b) = s(an2 b2 ) = = s(abn1 ) = s(bn ),

deci (1) este de asemenea adevrat. Astfel, am devenit interesai de ace a a t bare. Am crezut c nu avem de vericat dect un lucru simplu, dar ad a dovedit a fi altul.

2. Observaii preliminare. O modalitate natural de a ataca proble t a vedem care termeni ai irurilor s(an) i s(bn) pot fi calculai uor. Aceas s s t s nu se va dovedi eficient, dar ne va da o imagine asupra problemei. Este a urmtoarea a Lem. Dac N 10k 1, atunci a a s 10k 1 N = 9k.
Elev, cl. a XI-a, Colegiul Na ional "Ferdinand I", Bacu t a

Demonstraie. Putem presupune c 10 nu divide N . Fie N = b1 b2 . t a primele cifre pot fi i nule. Este mai convenabil i mai uor s calculm o s s s a a
1

dect un produs: s 10k 1 N = s 10k N N = s(b1 b2 . . . bk 00 .{z. 00 b1 b2 . . . bk | . }


k cifre

deci lema este demonstrat. a

= s b1 b2 . . . bk1 (bk 1)(9 b1 )(9 b2 ) . . . (9 bk1 )(10 bk ) =

Ce se ntmpl dac folosim aceasta n problema noastr? Dac 10k a a a a atunci s 10k 1 a = 9k i s 10k 1 b = 9k, s deci, de fapt, s(an) = s(bn) pentru toi n = 10k 1 suficient de mari t ipoteza noastr. Aadar, din aceast observaie, mpreun cu observaia c a s a t a t a 0-urilor n numerele n nu are efect, concluzionm c toate irurile de form a a s structuri similare i chiar valori comune pentru anumite valori ale lui n. s este clar c alegerea unor valori "frumoase" ale lui n nu este o tehnic ut a a Dar ce s-ar ntmpla dac am reui s gsim forme simple pentru a a s a a Aceasta este prima idee util. a Desigur, putem tia 0-urile de la sfritul numerelor a i b, deci 10 nu a s s a nici b, i ne ramne s artm c a = b. Presupunem c nu este adevr s a aa a a a a > b. Atunci, evident, pentru orice n, an > bn. Vrem s gsim un num a a are suma cifrelor mai mare dect cea a tuturor numerelor mai mici dect el aceasta va contrazice s(an) = s(bn) i problema este rezolvat. Dar care s s a numere? Sunt numerele care conin doar cifra 9, eventual, cu excepia pr t t Nu este greu s vedem c a are un astfel de multiplu dac i numai dac ( a a as a Deci, n acest caz, problema este rezolvat. Din pcate, nu putem rezolva a a idee cazul n care a sau b sunt divizibile cu puteri mari ale lui 2 sau 5 dac am fi folosit o baz prim n locul lui 10, problema ar fi fost rezol a a a acest motiv considerm potrivit s generalizm problema pentru o baz de a a a a oarecare.

Propoziia 3. Fie b 2, iar m si n dou numere naturale nenule as t a sb (km) = sb (kn), pentru orice k. Atunci logb m logb n Z.

3. Soluia problemei. Pasul 1. Vom arta ca fracia n/m are un n t a t de cifre dup virgul. Pentru fiecare i, exist un ki Z astfel nct a a a bi ki m bi + m 1. Atunci, deoarece funcia sb este subaditiv, t a

sb (ki m) sb (bi ) + s(ki m bi ) = 1 + sb (ki m bi ) 1 + ki m bi

S fixm j N, foarte mare. Vom demonstra c pentru orice i suficien a a a primele j 1 cifre ale lui ki n coincid cu primele j 1 cifre ale lui n/m. P

a lui n/m este prima cifr din stnga diferit de 0 indiferent dac este a a a dup virgul. Fie a a jn k bj . C = bj m Avem n 1 ki n 1 C = < C + j ki mC ki n < ki m C + j m ki m b b i i 1 b 1 bi C ki n < b + m C + j = bi C + j + m C + j b b b s Ca s stabilim faptul c primele j 1 cifre ale lui ki n sunt aceleai a a j 1 cifre ale lui n/m, trebuie s artm c a aa a bi C ki n < bi C + bi+1 . bj

care este adevrat pentru i suficient de mare. Atunci, pentru orice j, exis a a ai irului ki n care au suma cifrelor cel puin egal cu suma primelor j s t a lui n/m. Cum sb (ki m) m, pentru orice i rezult c irul sb (ki n) este mrgin a as a deducem c toate cifrele n baza b ale lui n/m sunt 0, eventual cu exc a numr finit. a

Conform (), este suficient s artm c a aa a 1 bi (b 1) m C+ j < , b bj

Pasul 2. Vom ncepe cu un rezultat simplu, dar important. Nu-l vom d fiind binecunoscut.

Lem. A si B au proprietatea c A/B are un num r finit de cifre du a a a n baza b. Atunci, toi factorii primi ai lui t B (A, B)

se num r printre factorii primi ai lui b. Altfel spus, dac p este prim, a a a atunci expp m expp n. (expp A noteaz exponentul numrului p n desco a a lui A.)

Am vzut c n/m are un numr finit de cifre dup virgul. Dar, desig a a a a a demonstra analog c i m/n are un numr finit de cifre dup virgul. Atunc as a a a lemei, pentru orice p care nu l divide pe b, expp m = expp n. Cu alte cuv dac b = a
t Y 1

pi , atunci m = N i

cu (N, b) = 1. S fixm f N suficient de mare i s lum a a s a a k1 =


t Y 1

t Y 1

p ri i n = N s i

t Y 1

psi , i

pf i si . i

Pentru simplitate, fie

nlocuind k din ipoteza sb (km) = sb (kn) cu kk1 , obinem t ! t Y f s +r pi i i i N = sb kbf N = sb (kN ), sb (k1 km) = sb (k1 kn) sb k
1

K=

t Atunci, sb (kN K) = sb (kN ) pentru toi k N, care reduce problema cu m important s inem minte c (N, b) = 1. at a

t Y 1

pf i si +ri . i

Pasul 3. S artm c logb K Z. Deoarece sb (kN K) = sb (kN ) p a aa a k N, deducem c a sb (N ) = sb (KN ) = sb (K 2 N ) = sb (K 3 N ) = . . .

n particular, aceasta nseamna c mulimea {sb (K u N ) : u N} este mar a t Dac logb K R \ Q, vom arta c pentru orice niruire de cifre, e a a a s astfel nct K u N ncepe exact cu acea secvena de cifre, ceea ce ar con t contradicie. Pentru aceasta, este suficient s artm c mulimea t a aa a t {{logb K u N } : u N},

unde {x} = x bxc, este dens n [0, 1]. Dar, deoarece logb K u N = u logb K a aceasta rezult imediat din lema lui Kronecker. Amintim c lema lui a a afirm faptul c pentru orice R \ Q, mulimea {{n} : n N} est a a t (0, 1). Aadar, am artat c logb K Q. s a a S presupunem, prin absurd, c logb K Q\Z. Este foarte interesant s a a a c acest lucru nu este posibil dect atunci cnd b este o putere netriv a demonstraia este fcut n toate celelalte cazuri. Observm c logb K Q t a a a a de fapt c, pentru orice p, expp K = logb K expp b. Fie r = blogb Kc; atunci a r expp b, deci br | K. Fie K 0 = K/br ; este clar c K 0 < b, deci K 0 este a baza b. Deoarece K 0 este obinut din K prin tergerea 0-urilor n baza b, t s sb (kN K 0 ) = sb (kN ), pentru orice k. Deoarece (N, b) = 1, nu este greu s a N are un multiplu kN de forma 11 . . . 11 n baza b (exist dou numere a a form congruente modulo K; deci K divide diferena lor, care, dup omiter a t a de la sfrit este de forma 11 . . . 11). Deci sb (kN ) = a i sb (kN K 0 ) = K 0 s s K 0 = 1. Aadar, K = br , deci logb K Z i, deoarece logb K = f +(logb m s s Z, va rezulta c armaia din Propoziia 3 este adevrat. a t t a a Bibliografie

1. G. Dospinescu, A. Zahariuc - Suma cifrelor unui num natural, ar Revist de cultur matematic (Bucureti), 2004, nr. 3-4, 2-15. a a a s 2. R. Blecksmith, M. Filaseta, C. Nicol - A result on the digits of an , A 64(3), 1993, 331-339. 3. http://www.mathlinks.ro

Generalizri ale unor inegaliti din RecM a at


Alexandru NEGRESCU 1
Ne propunem s generalizm urmtoarele inegaliti aprute n revista a a a at a Matematice":

X.53. (nr. 2/2004, p. 155). Fie a, b, c (1, ) astfel nct a + b + c = arate c a loga 2b3 + c3 + logb 2c3 + a3 + logc 2a3 + b3 12. Angela IX.48 (nr. 1/2004, p.77). Fie a, b, c (0, ) cu a + b + c + abc = arate c a a2 b2 3 c2 + + . 2 a + bc b + ca c + ab Ce

VII.17 (nr. 1/2001, p. 74). a) Fie x, y, z [2, ). Ar tai c a t a 2 2 2 x + y y + z z + x 27xyz. b) Fie x, y, z [3, ). Ar tai c x2 + y y 2 + z z 2 + x 64xyz. a t a Lucian

Soluii ale acestor probleme pot fi gsite n numerele 2/2005, 1/2005 t a s 1/2002 ale revistei.

loga1 (2an + an + + an ) + loga2 (an + 2an + + an ) + + 2 3 n 1 3 n n n n n + logan 2a1 + a2 + + an1 + n2 , logn S 1 S cu egalitate dac si numai dac a1 = a2 = = an = . a a n Soluie. Notm cu E membrul stng al inegalitii. Utiliznd inegalit t a at ilor, avem: E loga1 na2 a3 an + loga2 na1 a2 an + + logan na2 a2 a 2 3 1 n X logai n + 2 loga1 a2 + loga2 a3 + + logan a1 + =
i=1

Problema 1. Date numerele a1 , a2 , . . . , an (1, ), n 2, fie S = + + an . S se arate c a a

Dar

+ loga1 (a3 an ) + loga2 (a1 a4 an ) + + logan (a2 an1 n q X logai n + 2n + n n loga1 (a3 an ) logan (a1 an1 ).
i=1

loga1 (a3 an ) =
1

ln a3 + ln a4 + + ln an (n 2) (ln a3 ln an ) ln a1 ln a1

Elev, cl. a XI-a, Colegiul Na ional "A. T. Laurian", Botoani t s

n2 n n + n2 = logn S a1 + a2 + + an logn n i astfel inegalitatea este dovedit. Cazul n care are loc egalitate se obi s a t rina. t =

Cu aceast inegalitate i analoagele ei, vom avea a s n n X X 1 n2 logai n + 2n + n (n 2) = + n2 Pn + n2 E logn ai i=1 logn ai i=1 i=1 n2 + n2 logn (a1 a2 an )

Observaie. Inegalitatea din Problema X.53 se obine pentru n = 3 t t s

a2 + + 2 a1 + k a1 a3 an 1 1+ n S 2 S n+1 an + , n1 a a a an + k k+1 1 2 n1 cu egalitate dac si numai dac a1 = a2 = = an = 1 si S = n + 1. a a Soluie. Notnd cu E primul membru al inegalitii i utiliznd in t at s Cauchy-Buniakowski-Schwarz i cea a mediilor, obinem s t
n1 n1

Problema 2. Date numerele a1 , a2 , . . . , an (0, ), n 3, fie S = + + an + n1 a1 a2 an . S se arate c a a a2 + 1 a2 a3 an a2 + k

(a1 + a2 + + an ) (a + a2 + + an ) 1 E P n1 a a a P a2 + a3 + + an a1 + k 2 3 n a1 + k n1 Pn Pn 2 ai ( i=1 ai ) Pn = i=1 . = k+1 (k + 1) i=1 ai Pe de alt parte, pornind de la S = a1 + a2 + + an + n1 a1 a2 an a din nou inegalitatea mediilor, vom avea v s un uY n Q n+1 a, S (n + 1) t a n1
i i i=1 i=1

de unde

Ca urmare,

n X

S n+1

n+1

i=1

n Y

ai

! n n1

Introducnd n (), vom obine inegalitatea din enun. Egalitate avem dac t t S n1 a a s , adic ai = 1, i N i S a dac a1 = a2 = = an = a 1 n = n+1

v un n + 1 uY S n t ai S ai = S n1 . n+1 i=1 i=1

Observaii. 1) Mai general, putem considera a1 , a2 , . . . , an (0, ) t S = a1 + a2 + + an + p a1 a2 an , p 3; se va obine t p(n+1) p+1 1 S . E S k+1 n+1 2) Pentru n = 3 i k = 1 regsim inegalitatea din Problema IX.48. s a

n privina Problemei VII.17, vom spune mai nti c n nr. 1/2002, l t a sunt prezentate dou demonstraii pentru urmtoarea generalizare a aces a t a Fie x, y, z [n, ), n N. Ar tai c x2 + y y 2 + z z 2 + x (n a t a O inegalitate i mai general este dat de s a a

Cu egalitate dac si numai dac xi = k = 1, i = 1, n, sau xi = k, n = 3, a a Soluie. S notm din nou cu E membrul nti al inegalitii de de t a a at Observm c x1 k implic xn1 kxk2 i a a a 1 s 1

Problema 3. Fie x1 , x2 , . . . , xn [k, ), k, n N si n 2. Ar tai a t k(n n1 n1 n1 n x1 + x2 x2 + x3 xn + x1 (k + 1) (x1 x2 xn ) k+

xn1 + x2 kxn2 + x2 = xn2 + xn2 + + xn2 + x2 . 1 1 1 1 |1 {z } k q k+1 k(n2) Deci xn2 + x2 (k + 1) x1 x2 . Utiliznd aceast inegalitate i a s 1 ei, obinem c t a q k+1 k(n2)+1 k(n2)+1 k(n2)+1 E (k + 1)n x1 x2 xn = (k + 1)n (x1 x2 xn cu egalitate aa cum este specificat n enun. s t

Observaie. Inegalitile din Problema VII.17 se obin pentru k = 2 t at t respectiv k = 3, n = 3, iar generalizarea lor, dat n revist, pentru n = 3 a a

Cnd se nate, omul zice AA. . . . Cnd moare, zice MOR. . . . Aadar, s s vieii este [A, M OR], adic AMOR. t a

Vizitai pe Internet revista "Recreaii Matematice" la t t

http://www.recreatiimatematice.uv.ro

Cea mai bun inegaliate de acest tip . . . a


Marian TETIVA1

1. Introducere. Ai ntlnit cu sigurana asemenea enun (mai ale t t t preocup i probleme mai grele, "de olimpiad"). Iat trei astfel de exem as a a 1. Cea mai bun inegalitate de tipul a se obine pentru k = 4 (este aa-numita inegalitate a lui Schur ). t s 2. Cea mai bun inegalitate de tipul a p kR + hr (x + y + z)3 27xyz k[(x + y + z)(xy + xz + yz) 9xyz],

x, y,

care are loc n orice triunghi se obine pentru k = 2 si h = 3 3 4 (s t inegalitatea lui Blundon n acest caz). 3. Cea mai bun inegalitate de forma a a2 + b2 + c2 kR2 + hr2

si care are loc, iar si, n orice triunghi este cea obinut pentru k = 8 si a t a Cum se abordeaz o asemenea problem? Rspundem imediat la aceast a a a rezolvnd-o pe prima (a se vedea i [2], problema 9.22, un pic altfel formul s celelalte sunt soluionate n [3], respectiv [1]. Iat mai departe soluia pro t a t Trebuie s nelegem bine enunul; observm c expresia din membrul a t t a a (x + y + z)(xy + xz + yz) 9xyz,

are numai valori nenegative, conform unei inegaliti cunoscute. De aceea at

s a a pentru x, y, z > 0 i k1 k2 , aa c problema noastr este, de fapt, proble s celui mai mare numr k astfel nct inegalitatea a (x + y + z)3 27xyz k[(x + y + z)(xy + xz + yz) 9xyz]

k1 [(x + y + z)(xy + xz + yz) 9xyz] k2 [(x + y + z)(xy + xz + yz)

s fie adevrat pentru orice x, y, z > 0. De obicei ntr-o asemenea chestiun a a a mai nti cea mai mare valoare posibil a lui k (cea mai mic, n alte caz a a valori particulare variabilelor, apoi se demonstreaz inegalitatea rezulta a valoarea extrem depistat pentru k. Cel mai comun (i comod) este a a s variabile; n cazul de faa, s punem y = z = 1 i x > 0 arbitrar. Obinem in t a s t

care trebuie s aib loc pentru orice x > 0 (pentru c inegalitatea iniia a a a t aib loc pentru orice x, y, z > 0). De fapt asta nseamn x + 8 2k, pe a a x 6= 1 i, n ultim instana, pentru orice x > 0. Aici are loc de obicei s a t de trecere la limit, pentru a obine cel mai bun k. Limita se calculeaz, a t a puncte care sunt capete ale intervalelor pe care rezult inegalitatea final a de faa, pentru x 0, obinem k 4. (Evident, aici trecerea la limit t t a
1

(x + 2)3 27x k[(x + 2)(2x + 1) 9x] (x 1)2 (x + 8) 2k(x

Profesor, Colegiul Na ional "Gheorghe Roca Codreanu", Brlad t s

strict necesar, cci x, y, z pot fi considerate 0 - nu neaprat strict po a a a raionamentul funcioneaz la fel). Astfel am aflat c, probabil, cea mai bu t t a a a lui k este 4; numai dac reuim s demonstrm inegalitatea pentru k = a s a a trage concluzia final, cum c "cea mai bun inegalitate de tipul" a a a (x + y + z)3 27xyz k[(x + y + z)(xy + xz + yz) 9xyz], x3 + y 3 + z 3 + 3xyz x2 y + xy 2 + x2 z + xz 2 + y 2 z + yz 2 ,

se obine pentru k = 4. Ori, nlocuind aceast valoare a lui k, ajungem la t a

x, y,

inegalitate bine cunoscut (numit uneori "inegalitatea lui Schur", aa cu a a s spus), care apare n multe cri (inclusiv n [2]), deci nu o mai demonstr at a

x, y, z

2. Din nou despre normare. Iniial aceast lucrare s-a vrut o co t a notei [4]. ntre timp, s-a ivit aceast nou inegalitate (vom arta la urm a a a concurs de mprejurri am ajuns la ea) care s-a lsat rezolvat cu metod a a a (nu ezitai s o folosii i pentru inegalitatea lui Schur! de fapt, aa e f t a t s s a [2]) i, n plus, ilustreaz bine i genul de probleme pe care l-am descri s a s de ce am decis s amnm publicarea soluiilor exerciiilor din [4] (mai al a a t t aa?, "cititorii notri sunt la fel de inteligeni ca i noi", ba chiar mai) s s t s problemei pe care o prezentm chiar acum. a Problema 1. Care este cea mai bun inegalitate de tipul a k(a + b + c)(a2 b + ab2 + a2 c + ac2 + b2 c + bc2 6abc), (a2 + b2 + c2 )(a2 + b2 + c2 ab ac bc)

Cu alte cuvinte, deoarece i aici a doua parantez din membrul drept s a nenegative i a, b, c sunt strict pozitive, trebuie s determinm cea mai ma s a a pe care o poate lua k astfel nct inegalitatea de mai sus s fie adevrat p a a a a, b, c > 0. Si de aceast dat ncepem cu particularizarea b = c = 1 care n a a la (a2 + 2)(a 1)2 2k(a + 2)(a 1)2 , deci, pn la urm a2 + 2 2k(a a a ar trebui s fie adevrat pentru orice a > 0, a 6= 1, adic (pe baza con a a a a pentru orice a > 0. Facem iar pe a s tind la 0 pentru a gsi 2k 1 dar a a a Pentru k = 1/2 i b = c = 1 inegalitatea devine a(a 1)3 0 i, prin ur s s departe de a fi adevrat pentru orice a, b, c > 0. Trebuie deci s rafinm a a a a lui k, observnd c, de fapt, am obinut a t 2k

a, b, c >

a2 + 2 , a > 0; a+2 ceea ce nseamn c, n mod necesar, 2k este cel mult egal cu valoarea a a x2 + 2 funciei f (x) = t pe intervalul (0, ). (De aici i deosebirea fa s t x+2 studiat anterior: minimul nu se atinge n captul intervalului.) Aceasta se a a fi ceva mai mic a dect limita funciei n origine, adic dect 1; valoar t a a a a funciei f este 2 6 4 (se realizeaz pentru x = 6 2), astfel c ar t avem k 6 2 (fr s fie deloc clar deocamdat dac aceasta este, n aa a a a valoarea care produce cea mai bun inegalitate). Oricum, putem cons a departe k [0, 62] (n mod clar valorile negative ale lui k nu pot fi luate drept candidate pentru "cel mai bun" k); n particular, avem k 6 2

Acum urmeaz partea neplcut (dar inevitabil cnd folosim metoda a a a a n care trebuie s facem calculele; pe care nu le vom reproduce aici (dar a s le verificai!. . . ). Inegalitatea de demonstrat capt forma a t aa x4 (k + 1)x3 y + (2 2k)x2 y 2 (k + 1)xy 3 + y 4 +

1 2k > 0. Mai departe ncercm, cu metoda normrii, s vedem ce se n a a a inegalitatea noastr pentru asemenea k i a, b, c numere pozitive oarecare a s simetriei putem presupune c = min{a, b, c}; notm atunci a a b = 1 + x, = 1 + y, c c unde x i y trebuie s fie nenegative. Inegalitatea mai poate fi scris n fo s a a X X X X 4 2 2 2 a b + (4k 1) a bc (k + 1) (a3 b + a a + (2 2k) a b (toate sumele sunt ciclice) sau nlocuind cu 1 + x i cu 1 + y, s c c (1 + x)4 + (1 + y)4 + 1 + (2 2k) (1 + x)2 (1 + y)2 + (1 + x)2 + (1 + +(4k 1) (1 + x)2 (1 + y) + (1 + x)(1 + y)2 + (1 + x)(1 + y) (k + 1) (1 + x)3 (1 + y) + (1 + x)(1 + y)3 + (1 + x)3 + (1 + y)3 + 1 + x

sau nc a

+(2 2k)x3 3x2 y 3xy 2 + (2 2k)y 3 + (3 6k)(x2 xy + y 2 )

1 1 (x y)4 + (x2 y 2 )2 + (1 k)xy(x y)2 + 2(1 2k)x2 y 2 + 2 2 +2(1 k)(x + y)(x y)2 + (2 5k)xy(x + y) + 3(1 2k)(x y)2 + 3(1 Acum putem demonstra inegalitatea pentru k 6 2; oricum, ea est pentru k < 0, deci putem considera k 0. S observm mai nti c, pentru aceste valori ale lui k, avem a a a pentru orice t R, deoarece discriminantul trinomului este 4(2 5k)2 24(1 2k)2 = 4(k2 + 4k 2) = 4(k + 6 + 2)(k 6 + 2(1 2k)t2 2(2 5k)t + 3(1 2k) 0,

n plus, cum am mai artat, k este mai mic dect 1/2, deci 2(1 2k) > 0 a n membrul stng al inegalitii de demonstrat toi termenii sunt n m at t nenegativi, cu excepia lui (2 5k)xy(x + y) care poate fi luat n grupul t 2(1 2k)x2 y 2 + (2 5k)xy(x + y) + 3(1 2k)xy = = xy [2(1 2k)xy + (2 5k)(x + y) + 3(1 2k)] ,

pe care-l mai putem scrie ! 2 x+y x+y 1 2k xy 2(1 2k) + 2(2 5k) + 3(1 2k) xy(x 2 2 2

Dar paranteza mare este nenegativ, conform observaiei referitoare la tr a t gradul al doilea, iar ultimul termen este "anihilat" de (1 k)xy(x y)2 : 1 2k 1 (1 k)xy(x y)2 xy(x y)2 = xy(x y)2 0 2 2

i inegalitatea este demonstrat; valoarea cea mai bun (maxim) a lui k s a a a adevr, 6 2. a

3. Poate c nu e lipsit de interes s vedem cum am ajuns la aceas a a tate: desigur, din ntmplare (multe ies astfel . . . )! Am vrut s vedem ce a dac aplicm inegalitatea lui Jensen funciei radical n felul urmtor: a a t a p p p b c a (b + c)2 + (a + c)2 + (a + b)2 a+b+c a+b+c a+b+c r b c a (b + c)2 + (a + c)2 + (a + b)2 . a+b+c a+b+c a+b+c (Asta mai rezult i folosind inegalitatea Cauchy-Buniakowski-Schwarz ). as care la ptrat i nc cteva calcule ajungem la a s a P P 30abc 4 a3 3 a(b2 + c 4 4(ab + ac + bc)2 (a + b + c)3 + a+b+c 9 9 sau, totuna,

Mai inem cont acum de dou lucruri: prima parantez din membrul st t a a pete 6(a2 + b2 + c2 ), i a doua este nenegativ conform inegalitii as s s a at a2 + b2 + c2 ab + ac + bc,

4(a2 + b2 + c2 + 5ab + 5ac + 5bc)(a2 + b2 + c2 ab ac bc) = X X = 4 (a + b + c)4 9(ab + ac + bc)2 (a + b + c) 4 a3 + 3 a(b2 + c2 )

pe cnd cea de-a doua parantez din membrul drept se poate minora (pe a galit tii lui Schur ) cu a X X X a(b2 + c2 ) 30abc = 7 a(b2 + c2 ) 4 a(b2 + c2 ) 12abc + 3 astfel suntem condui la inegalitatea s 24(a2 + b2 + c2 )(a2 + b2 + c2 ab ac bc)

7(a + b + c)(a2 b + ab2 + a2 c + ac2 + b2 c + bc2 6abc), a, b, c i ntrebarea din problema 1 vine n mod natural (dup ce am demonstrat in s a 7 pentru k = ). Putem vedea acum c valoarea lui k de la care am pornit a 24 de departe de cel mai bun k; avem, totui, o demonstraie fr prea mult s t aa unei inegaliti de acest tip, care ne poate da sperane c s-ar putea face at t a mai bun dintre aceste inegaliti". De asemenea, v propunem spre rezo a at a Problema 2. G sii cea mai bun inegalitate de forma a t a (x + y)2 (x + z)2 (y + z)2 + (27k 64)x2 y 2 z 2 kxyz(x + y + z)3 , Bibliografie 1. 2. 3. 4.

x, y

L. Panaitopol - O inegalitate geometric , Gazeta Matematic seria B, 4/ a a L. Panaitopol, V. Bndil, M. Lascu - Inegalit ti, Editura GIL, Zal a a a a V. Vjitu - Asupra unei inegalit ti, Gazeta Matematic seria B, 7/1984 a a M. Tetiva - Metoda norm rii, RecMat - 1/2006, 30-34. a

Societatea de Stiine Matematice din R. Mol t

Anul 2006 aduce cu sine dou date marcante n cercetarea i n nv a s superior din Republica Moldova. Dou instituii ce constituie imaginea a t postbelice n tiina contemporan - Academia de Stiine a Moldov s t a t versitatea de Stat din Moldova - mplinesc 60 de ani de la fonda matematic din R. Moldova datoreaz n mare parte acestora evoluia sa. a a t vante despre istoria acestor instituii pot gsite n volumele: Institutul de t a tic si Informatic . File din istorie, editat de Acad. de St. a Moldovei a a Istoria Universit tii de Stat din Moldova (1946-1996), editat de USM n a La armarea colii matematice basarabene au contribuit un ir de s s ti de autentic valoare, fondatori de coli tiinice: Vladimir Andru at a s s t Valentin Belousov (algebr), Constantin Sibirschi (ecuaii difereniale), I a t t berg (analiz funcional), Alexandru Zamorzaev (geometrie), Alexandru a t a (logic matematic) .a. Scolile tiinice create n Republica Moldova a a a s s t se impun n lumea matematic prin realizri valoroase, devenind centre d a a a a specialitilor din republic i de peste hotare timp de cteva decenii. s as Societatea Matematic a fost prezent n Basarabia n diferite form a a nizare, reunind n rndurile sale cercettori, profesori de matematic din u a a i coli i chiar elevi. n actualul su statut Societatea Matematic din s s s a a Moldova (SMM) activeaz din anul 1993. n prezent SMM ntrunete 180 a s bri, incluznd matematicieni consacrai, tineri cercettori, profesori de la t a superioare de nvamnt, licee i coli. n anul 2004 Adunarea General at s s a reales n calitate de preedinte pe academicianul Mitrofan Ciobanu, iar vice s pe profesorii universitari Nicolae Jitarau i Alexei Cau. s s s Activitatea de baz a SMM ine de promovarea cercetrilor tiinice n a t a s t matematicii i al informaticii, de monitorizarea sistemului de nvamnt s at la toate nivelele i de consolidarea comunitii matematice. s at Un element important n consolidarea comunitii matematice din at Moldova, pe parcursul anilor, au fost conferinele tiinice. n ultimii 50 au t s t nizate peste 50 conferine tiinice, inclusiv ediii ale unor conferine inte t s t t t ex-sovietice i europene. La organizarea acestor manifestri tiinice a s a s t cietii Matematice a fost esenial, cu toate acestea prima conferina SM at t t organizat abia n anul 2001. Beneciind de sprijinul nanciar al UNESC a conferina a Societii Matematice din Republica Moldova a reuit s n t at s a peste 160 de participani, inclusiv 40 de oaspei de peste hotare, specialit t t s niul matematicii i al informaticii din Romnia, Frana, Germania, Rusia s t Ucraina, Belarus, Serbia i Muntenegru. La conferina au fost prezenta s t rapoarte iar ulterior au fost publicate materialele conferinei n dou num t a vistei Buletinul Academiei de Stiine a Republicii Moldova. Matematic , t a al revistei Computer Science Journal of Moldova i o culegere de articol s niul didacticii matematicii i al informaticii. n anul 2004 a fost organiza s conferina SMM, ind dedicat anivers rii a 40-a de la fondarea Instit t a a Matematic i Informatic al A. . R.M. La aceast conferina au a s a S a t

peste 170 de specialiti n matematic i informatic, inclusiv 30 de invita s as a hotare (SUA, Romnia, Germania, Ucraina, Rusia, Israel, Canada .a.). s n anul 2002, susinnd iniiativa societilor matematice din Grecia, R t t at Bulgaria, SMM a participat la fondarea Societii Matematice din S at Europei (MASSEE) cu sediul la Atena, Grecia. Fondarea MASSEE a c nou etap n cooperarea regional a matematicienilor din Balcani. a a a Criza extins prin care a trecut R. Moldova n primii si ani de existe a a repercursiuni negative asupra tuturor sferelor de activitate i n primul r s dezvoltrii tiinelor fundamentale. Astfel, SMM i-a propus drept scop id a s t s unor modaliti de depire a dicultilor ce afecteaz cercetarea i e at as at a s R. Moldova. Prima conferina SMM a adresat conducerii R. Moldova t care au fost formulate propuneri concrete viznd probleme de stringent a a n domeniile cercetare-dezvoltare i nvamnt. Iniiativele SMM, inclu s at t apel, au inuenat politica Guvernului i a altor structuri statale abilit t s drept urmare anunarea unor concursuri naionale de granturi, inclusiv g t t cercetare n matematic i n informatic, granturi pentru organizarea co as a i a simpozioanelor tiinice, pentru editarea revistelor de specialitate et s s t n anul 2004 a intrat n vigoare Codul cu privire la stiina si inovare al R t ce reglementeaz raporturile juridice ce in de elaborarea i promovarea p a t s stat n sfera tiinei i inovrii, de acreditarea organizaiilor din sfera s t s a t s atestarea cadrelor .a. Codul admite participarea comunitii tiinice s at s t rarea politicii de stat n domeniul tiinei i inovrii. Si nvamntul, n s t s a at universitar i postuniversitar, la etapa actual este n proces de reformare s a mitate cu cerinele Programului de la Bologna, la care recent a aderat R. t n contextul celor menionate conducerea SMM i propune monitori t s cesului de reformare n domeniul matematicii i al informaticii, promov s acceptate de comunitatea matematic internaional. a t a Comunitatea matematic din Republica Moldova marcheaz n anul a a nizarea ediiei a 50-a a Olimpiadei republicane de matematic pentru el t a eveniment este deosebit de semnicativ i graie rezultatelor Lotului olim s t publicii Moldova, care a obinut doar n ultimii cinci ani 66 de medalii la t internaionale, inclusiv trei medalii de aur la olimpiadele internaionale d t t tic i nou medalii de aur la olimpiadele balcanice. n anul 2006 Chi a s a s gazd a Olimpiadei Balcanice de Matematic pentru juniori. a a Evoluia tiinei i a nvamntului n Basarabia dup 1990 a fost pute t s t s at a enat de deschiderea cultural ctre Romnia. Un loc aparte l ocup c t a a a a dezinteresat a Romniei n oferirea de burse pentru liceeni, studeni, ma a t doctoranzi din R. Moldova. Participarea larg la conferine tiinice, inc a t s t nizarea unora din ele n comun, a extins i a concretizat cadrul de colabor s drept urmare susineri de teze de doctorat, schimb de profesori invitai, t t lotului olimpic, tabere de var, editare de carte i multe alte activiti d a s at benece pentru cultura, cercetarea i educaia din Basarabia. s t

Florin DAMIAN, Parascovia SRBU Fac. de Mat. si Inf., Univ. de Stat din Mold

Concursul de Matematic Al. Myller a


Ediia a IV-a, Iai t s Clasele IV-VI, aprilie 2006

Not. Toate subiectele sunt obligatorii. Timp efectiv de lucru: 90 min. Se a oficiu 30 de puncte, cte 6 puncte pentru problemele 1-5, cte 8 puncte pentru 6-10 i cte 10 puncte pentru problemele 11-15. s

Clasa a IV-a

1. Ce numr trebuie sczut din 9 pentru ca diferena obinut nmul a a t t a t devin 40? a

2. ntr-o clas sunt biei i fete. Numrul bieilor este cu 3 mai m a a t s a a t numrul fetelor. Dac n clas ar veni 4 biei i ar pleca 4 fete, atunc a a a a t s bieilor ar fi de 2 ori mai mare dect numrul fetelor. Ci elevi sunt n a t a t

4. Suma dintre un numr i succesorul su este cu 2006 mai mare dec a s a sorul su. Care este numrul? a a X 5. Fie ptratul magic (sumele elementelor de pe linii, coloane a i diagonale sunt egale). Aflai ce numr trebuie nscris n csua s t a a t marcat cu X? a 9 6. mprind numrul natural a la numrul natural b, obinem ctul at a a t 31. S se afle a tiind c a b < 196. a s a

3. Se dau numerele: a = 1 + 2 + 3 + + 9; b = 11 + 22 + 33 + c = 111 + 222 + 333 + + 999. S se calculeze ctul mpririi lui a + b a at

7. ntr-un co sunt 28 fructe: mere, pere i caise. Cte fructe sunt de s s dac mere sunt de 6 ori mai multe dect pere, iar n co se afl cel puin u a s a t fiecare fel? 8. Cte numere de 3 cifre se mpart exact la 21? 9. Aflai cel mai mic numr natural cu suma cifreler 56. t a

10. S se afle 3 numere naturale tiind c: produsul primelor dou a s a produsul ultimelor dou este 192, iar suma dintre primul i ultimul este 4 a s

11. Un tat, dorind s-i ncurajeze fiul s rezolve probleme, i promi a as a da 8 monede pentru fiecare problem bine rezolvat, dar pentru problem a a nu a rezolvat-o sau a rezolvat-o greit, fiul va trebui s-i plteasc 5 mon s a a a 26 de probleme, fiul nu trebuie s plteasc nimic, dar nici s primeasc a a a a a probleme a rezolvat corect? 12. Ioana culege o ldia de cpuni n 40 minute, iar Luiza culege o a t a s ore. n ct timp vor culege mpreun 3 ldie de cpuni? a a t a s

13. Mama are cu 14 lei mai mult dect Paul i cu 10 lei mai mult de s Ci lei va da fiecruia din biei pentru a avea toi 3 aceeai sum? t a a t t s a

14. M-am nscut n secolul XX. Dac n 1999 am avut o vrst egal a a a cifrelor anului meu de natere, ce vrst am acum? s a 15. Sony car n fiecare zi cte o piatr din vrful muntelui. n prima zi a a 7 ore urcnd i cobornd, a doua zi a petrecut 8 ore urcnd i cobornd. n s s urc de dou ori mai ncet dect n ziua precedent, dar coboar de 2 ore m a a a a Ct timp va munci n cea de-a treia zi?

Clasa a V-a
1. Care este cel mai mare numr natural impar de 4 cifre distincte? a 2. Determinai x N pentru care 5 3x 2 3x = 81. t 4. Cte ptrate perfecte conine mulimea {x N | x 1000}? a t t 3. Ci termeni are suma 26 + 32 + 38 + + 2006? t

6. Cte numere xy scrise n baza 10 verific relaia x + xy = y + yx + a t 7. Dac a b + a c = b c + c2 , b + c = 1003, calculai a + 2b + c. a t

5. Cte mulimi X verific {1, 2} X = {1, 2, 3, 4, 5}? t a

8. S se determine un numr de 4 cifre al crui produs cu 9 se termin a a a

9. Dac restul mpririi lui 2a + 3b + 5c la 13 este 7, aflai restul m a at t 34a + 51b + 85c la 13.

10. Cte numere care se scriu cu 4 cifre n baza 10 au suma cifrelor 3?

11. Se consider 5 numere naturale distincte avnd suma 60. Gsi a a t dac suma diferenelor dintre cel mai mare i fiecare dintre celelalte 4 nu a t s 10.

12. Care sunt ultimele dou cifre ale lui 52006 + 72006 ? a o na | a, b N , a + b = 2006 . Care este produsul elem 13. Fie M = b M?

at 14. Suma a dou numere naturale este 3n + 5, n N . mprind un a dublul celuilalt obinem ctul n i restul 0. Care sunt numerele? t s 15. Se scriu n ordine cresctoare toate numerele nenule din baza 10 a scriere nu apar alte cifre afar de 0, 1, 2 i 3. Care este cel de-al 2006-lea a s

Clasa a VI-a

1. Care este cel mai mic numr natural de nou cifre divizibil cu trei? a a

2. Fie a, b, c, d msurile a 4 unghiuri formate n jurul unui punct. a a b c d aceste msuri dac = = = . a a 1 2 3 4 3. Preul unui obiect se micoreaz cu 20%. Cu ct la sut trebuie s se t s a a a noul pre pentru a se ajunge la preul iniial? t t t

4. Calculai msura unui unghi tiind c triplul complementului su e t a s a a 300 mai mare dect jumtatea suplementului su. a a

5. Care este cea mai mare valoare pe care o poate lua numrul n = a1 a a4 + + a9 a10 , unde a1 , a2 , . . . , a10 sunt numere naturale distincte din {1, 2, 3, . . . 10}?

6. Fie A, B, C, D puncte pe o dreapt, n aceast ordine, astfel nct a a 7 BC AD = . Calculai t . BD 5 AD 7. Se dau dou vase cu ap astfel nct dac turnm jumtate din pr a a a a a doilea i jumtate din cantitatea de ap ce se afl acum n al doilea o s a a a primul i lund apoi jumtate din cantitatea aflat n primul i o turnm s a a s a obinem (n al doilea vas) 10 litri ap. Aflai ci litri de ap se afl n f t a t t a a tiind c ambele cantiti sunt numere ntregi. s a at

8. Artai c numrul a = 3 + 32 + 33 + + 34n se divide cu 120, n a t a a

9. S se calculeze a 2007-ea zecimal a numrului n = 0, 00 (300)+0, 00 a a a

10. S se afle x, tiind c a s a 1 2 3 n 1 %din %din %din . . . %din x . . . = 2 3 4 n+1 100n (n +

11. Fie n un numr natural, n > 2. Numrtorii i numitorii fraciilor a aa s t 555 . . . 56 i s au cte n cifre. Care este fracia mai mare? t 666 . . . 67 12. Cte unghiuri cu msurile numere naturale consecutive se pot form a unui punct?

13. Msurile unghiurilor unui triunghi sunt direct proporionale cu tr a t invers proporionale cu trei numere direct proporionale cu trei numere p t t secutive ale lui 3. Gsii msurile unghiurilor. a t a

14. Fie S = 1 + 2 + 3 + + 102006 . La ce putere apare 2 n descom factori primi a lui S? 2006! 15. S se afle cardinalul mulimii A = (x, y) N N | x a t 7 11y n! = 1 2 3 n.

Clasele VII-XII, 19 aprilie 2006 Juniori


1. S se arate c ecuaia a a t

1 1 1 1 + + = are o infinitate de soluii n t a b ab c

2. Fie 4ABC dreptunghic n C i D (BC), E (CA) astfel nc s AE \ = k. Dreptele BE i AD se intersecteaz n O. S se arate c m(BO s a a a CD dac i numai dac k = 3. as a

3. Se dau n plan 5 puncte cu proprietatea c oricare dintre cele 10 t a cu vrfurile n aceste puncte are aria cel mult egal cu 1. S se arate c a a a trapez de arie cel mult 3 ce conine n interior sau pe laturi cele 5 puncte t

Seniori
1. Fie p 3 numr prim. Demonstrai c a t a
p1 P

kp k p+1 (mod p) p 2 k=1 2. Fie p 5 numr prim. Determinai numrul polinoamelor de fo a t a pX k + pX l + 1, k > l, k, l {1, 2, . . . p 1} care sunt ireductibile n Z [X] 3. Mediana AM a triunghiului ABC taie cercul nscris n K i L. Dre s prin K i L paralele la BC taie a doua oar cercul nscris n X i Y . s a s interseciile dreptelor AX i AY cu BC. Artai c BP = CQ. t s a t a

8 3 9 10 1

11 6 2 5 2

ARITMOGRIF 1 6 3 3 6 12 2 1 1 13 14 12 4 12 12

?
8 5 13 15 16 7 6 1 13 17 18 6 15 2 8 * 7 19 4 10 8 8 8 5 10 1 7 12 2 6 19 15

9 2 8 8 2

9 4 7 2 15

6 4 3 13 20

4 1 5 6 17

21 15 2 6 17

nlocuind numerele cu litere vei obine numele a 18 matematicieni. t t Pe orizontal A-B: numele unei reviste de matematic pentru elevi i p a a s

Prof. Valeriu Br C. N. "Gh. Roca Codrean s (Rezolvarea aritmogrifului se gsete la pag. 162) a s

Vizitai pe Internet revista "Recreaii Matematice" la t t

http://www.recreatiimatematice.uv.ro

Concursul de matematic Florica T. Cmp a


Etapa judeean, 18 februarie 2006 t a Clasa a IV-a

1. S se mpart la trei persoane 24 sticle de suc identice ca mrime, a a a sunt pline, 11 umplute pe jumtate i 8 goale, nct fiecare s aib acelai a s a a s sticle, dar i aceeai cantitate de suc. s s nv. Lin 2. ntre cele 9 numere de mai jos exist un "intrus". Acesta nu respe a dintre cifre ce exist la fiecare din celelalte opt numere. Descoper i sc a as precum i numrul "intrus": 9334, 4862, 6148, 5132, 7835, 3524, 9963, 97 s a nv. Fnic a a 3. Cntarul pe care vor s se cntreasc trei copii nu msoar mase m a a a a a 40 kg. Fiecare din cei trei copii cntresc ntre 25 i 30 kg. a s Cum a reuit fiecare copil s se cntreasc? s a a a Inst. Iulia

Clasa a V-a

1. Cifrele care alctuiesc vrsta bunicului reprezint vrstele celor d a a Dac mprim vrsta bunicului la suma vrstelor nepoilor, se obine a at t t restul 12. Aflai vrsta bunicului i vrstele nepoilor. t s t Mihael 2. Se consider nmulirea urmtoare, unde literele nu reprezint a t a a obligatoriu cifre distincte: a) Determinai e. t b) Artai c bd = 63. a t a c) Reconstituii nmulirea. t t Gabriel Popa 3. Se consider mulimea A = {2, 3, 4, . . . , 13}. a t a) Determinai B, C disjuncte astfel nct B C = A i suma element t s este egal cu sum elementelor din C. a a b) Artai c nu exist M , N disjuncte cu M N = A i produsul e a t a a s din M egal cu produsul elementelor din N . c) Gsii dou mulimi X, Y disjuncte cu X Y = A, X avnd dou a t a t a cu produsul elementelor lui X egal cu suma elementelor lui Y . Ionel

Clasa a VI-a

1. Pe o tabl s-au scris trei numere naturale. Cnd n locul lor s-au sc a produsul i suma produselor cte dou, s-a vzut c pe tabl au apru s a a a a a numere ca i cele iniiale. Care este produsul lor? Explicai! s t t 2. Pe o tabl este scris numrul 12. La fiecare minut numrul se nmu a a a se mparte fr rest fie la 2, fie la 3, iar rezultatul se scrie pe tabl n locul aa a iniial. S se arate c numrul scris pe tabl dup exact o or nu poate fi t a a a a a a \s \ 3. Unghiurile proprii AOB i BOC sunt adiacente suplementare. Fie [ [ [ bisectoarele acestora. Dac m(BOy) N i m(COx) = p m(BOy), u a s

numr prim, aflai numrul p. a t a

Vasile

Clasa a VII-a

1. Fie a un numr natural arbitrar, divizibil prin 9, avnd 2007 cifre. a s (a) numrul care reprezint suma cifrelor lui a. Gsii s (s (s (a))). a a a t Gabriel

2. Vrfurile unui cub se noteaz cu 8 numere ntregi consecutive, i a fiecrei fee se noteaz cu media aritmetic a vrfurilor feei respective. a t a a t a) S se demonstreze c suma centrelor oricror dou fee opuse este ac a a a a t se afle ct este aceasta n funcie de cel mai mic dintre numerele din vrfu t b) S se gseasc n ce condiii centrul unei fee este numr ntreg. a a a t t a vrfurile feelor ale cror centre sunt numere ntregi. t a c) Artai c, dac centrele a trei fee care au un vrf comun sunt a t a a t numere ntregi, atunci i centrele celorlaltor fee sunt notate tot cu numer s t Julieta

3. n ara TI a triunghiurilor isoscele era mprat, firesc, triunghiul e t a El decretase c este singurul care binemerit numele de Prearostogolibil ; a a trebuiau s fie numii tepoi dac au o latur mai scurt dect cele egale a t s a a a turtii dac au o latur mai lung dect cele egale. (Vorba congruent era s t a a a ocar pe acele meleaguri.) Nite unghiuri umblau venetice prin TI cut a s a triunghi isoscel la al crui vrf s slujeasc. a a a Teposule, zise un unghi . Eu i vecinii mei de pribegie blbit s nemsuratul de ne cutm stpni n TI. Ne-ai fi de mare folos dac ai a a a a a ne spui dac nu cumva ai o bisectoare interioar a ta exact att de lung c a a a Dup vorbire se cunoate c venii de pe coclauri unde lucrurile nu s a s a t din linii drepte bine limitate. ntrebi de lucruri la care nu gndete nim s nu sunt de niciun folos. Dar, pn cercetez pentru rspuns, fii bun m a a a tule i spune-mi dac aa se obinuiete pe la voi: s-i ponegreti colegi s a s s s at s necuviincioase? Nu e necuviina, prea-limitatule. Eu, , m exprim frumos n grade t a sunt purttor de cuvnt; nu cunoate fracii ordinare ci doar zecimale i a s t s grozav cnd ncearc s spun cte grade are; nc nu tie dac este m a a a a s a grade. Dar bag seam c ntrzii cu rspunsul la ntrebarea mea; o fi cap a a a mult ascuit dect ncptor? t a a Bine, mi vorbreule. Am cercetat i rspund precis: am exact dou a a t s a a interioare exact aa de lungi ca laturile mele egale. s Am neles. Te rog s m ieri c i-am zis epos; neleg c eti t t a a t at t t a s personal nu mi eti de folos, dar iat c pentru eti bun de stpn. D s a a s a toarele tale egale erau ct latura ta scurt, te recunoteam de stpn. Dac a s a a bisectoare a ta era ct latura ta scurt, te-ar fi slujit cu credina. a t a) Exprimai cu fracii ordinare gradele lui i . t t s b) Exprimai cu fracii zecimale numrul de grade, minute i secunde a t t a s c) Argumentai c triunghiurile isoscele care convin lui , respectiv t a poase, respectiv turtite.

d) Desenai un triunghi isoscel cu unghiurile de la vrf . Este el tepos t Dan

Clasa a VIII-a

1. ntr-o clas sunt 20 de elevi. Fiecare fat ofer fiecrui biat trei flor a a a a a fete o floare, iar fiecare biat ofer cte trei flori fiecrei fete i cte o floa a a a s biat. a a) Artai c numrul maxim de flori oferite este 780. a t a a b) Cte fete ar trebui s fie n clas, astfel nct s fie oferite exact 780 a a a Monica Ned 2. Pe fiecare faa a unui cub este scris cte un numr natural nenul, i t a vrf i corespunde produsul numerelor de pe cele trei fee adiacente acest t suma numerelor corespunztoare tuturor vrfurilor este 2006, artai c a a t a puin dou fee pe care este scris acelai numr. t a t s a 3. Fie paralelipipedul dreptunghic ABCDM N P Q i punctele E ( s (DQ) astfel nct suma AE + AF + P E + P F este minim. Artai c a a t a dac i numai dac ABCDM N P Q este prism regulat. as a a a Valentina Blendea i Gheorghe Blen s

Faza interjudeean, 20 mai 2005 t a Clasa a IV-a

1. Lungimea laturii unui ptrat este de 17 m. O persoan pleac dintr a a a ptratului i, mergnd n acelai sens pe laturile acestuia, parcurge o dista a s s m. Din punctul n care a ajuns se ntoarce i parcurge 773 m. Aflai la c s t se va situa n final persoana, faa de punctul de plecare. t RecMat - 1/2 2. Avem mai multe vase identice. Stim c 50 vase pline cu ap cntre a a a iar 10 vase goale cntresc cu un kilogram mai puin dect apa dintr-un v a t se afle ct cntresc 100 de vase goale. a Petr 3. n cte moduri diferite pot fi scrise numerele 1, 2, 3, 4 n ptratele di a figura alturat, cte unul n fiecare ptrat, astfel nct s nu existe dou a a a a ptrate alturate n care suma s fie 5? Justificai. (Ptratele care au doa a a a t a un vrf comun nu sunt considerate alturate, iar ptratul mare este fix.) a a Petr

Clasa a V-a

1. Din produsul tuturor numerelor naturale de la 1 la 2006 se ex numerele care se divid cu 5. n ce cifr se termin produsul celorlaltor nu a a 2. Pe o tabl sunt scrise numerele 1,3,4,6,8,9,11,12,16. Doi copii au a patru numere i s-a observat c suma numerelor terse de unul este de t s a s mare dect suma numerelor terse de cellalt. Ce numr a rmas scris pe s a a a 3. Fie numrul 123456789. O operaie nseamn s alegem dou cifre a t a a a crora s li se scad o unitate i s li se schimbe locurile (de exemplu: 123 a a a s a 123436789 ...). Care este cel mai mic numr ce se poate obine ca a t acestor operaii? Dup cte operaii se obine cel mai mic numr? t a t t a

Clasa a VI-a

1. Se pot aeza pe muchiile unui cub numerele 1, 2, 3, . . . , 12 (cte un s fiecare muchie) astfel nct suma numerelor aflate pe cele trei muchii care acelai vrf s fie aceeai pentru fiecare vrf al cubului? s a s Constanti \ 2. Se consider triunghiul ABC cu [AB] [AC] i m(BAC) = a s \ \ M, P (AC) astfel nct i m(ABM ) = m(CBP ) = 20 i N (AB) a s s \ ) = 50 . S se afle m(AM N ). \ m(BCN a 3. Numerele naturale 22, 23, 24 au urmtoarea proprietate: descompune a tori primi ale numerelor din ir au exponenii factorilor numere impare (22 s t 23 = 231 , 24 = 23 31 ). Care este cel mai mare numr de numere naturale c a care au aceast proprietate? a Cristian - Ctlin a a

Clasa a VII-a

1. a) Artai c din oricare 3 numere naturale, putem alege dou a a t a a suma lor s fie un numr par. a a b) Fiind date apte numere naturale, artai c putem alege patru dintr s a t a nct suma lor s fie divizibil cu 4. a a Cristi 2. Fie ABC isoscel cu BC = 2a, AB = AC = b, a, b N . S se a toate triunghiurile ABC, dac a = 2r, unde r este raza cercului nscris n a apoi gsii ABC cu aria minim. a t a Dan 3. Un trapez ABCD are baza mare [AB] i [AC] [BD] = {O}. Lin s a trapezului intersecteaz pe AC n E i pe BD n F . a s a) Demonstrai c ABCD este trapez isoscel dac i numai dac [OE] t a as a b) Vrfurile trapezului i punctul O reprezint 5 orae, iar laturile i d s a s s sale sunt osele de legtur. Dou maini pleac din D, respectiv C pe ru s a a a s a scurt spre A, respectiv spre B i alte dou maini pleac din A respectiv a s a s a respectiv C, trecnd prin O pe ruta cea mai scurt. Cele 4 maini au acee a s constant, pe ntreg parcursul. Demonstrai c primele 2 maini ajung s a t a s D, respectiv C. Pot ajunge, toate patru, n acelai timp la destinaie? s t Claudiu- te S

Clasa a VIII-a

1. Aflai perechile de numere ntregi cu proprietatea c diferena cu t a t este egal cu ptratul diferenei lor. a a t 2. Un poliedru are 17 muchii i 9 fee. s t a) Desenai dou astfel de poliedre diferite. t a b) Dac feele poliedrului sunt doar triunghiuri echilaterale sau ptrate a t a muchiile sunt de lungime 1 cm, calculai aria sa total. t a Gab 3. Un cub cu latura de n cm (n N ) se mparte n cuburi cu latur i se coloreaz toate cuburile situate pe diagonalele feelor cubului iniia s a t t astfel ca numrul cuburilor colorate s fie 2006. a a Julieta

Soluiile problemelor propuse n nr. 2 / 20 t


Clasele primare

P.94. Aflai numerele de dou cifre cu proprietatea c diferena dintre t a a t r sturnatul lui este egal cu cel mai mare num r scris cu o singur cifr . a a a a a (Clasa I ) Mara Neicu, elev a

Soluie. Dac un numr de dou cifre ndeplinete condiia din proble t a a a s t diferena dintre numrul reprezentat de cifra zecilor i numrul reprezent t a s a unitilor este egal cu 1. Numerele care verific cerina sunt: 21, 32, 43, at a a t 87, 98.

P.95. La ora de educaie fizic , fetele unei clase sunt aliniate de la cea t a la cea mai scund , iar b tii dup ele, n aceeai ordine. Ana este cea m a aie a s Ene este cel mai nalt, iar Sorin cel mai scund. ntre Ana si al doilea b i a copii, iar ntre Ene si Sorin sunt 10 b ti. Ci elevi sunt aliniai la educa aie t t (Clasa I ) nv. Constana Cristea i Inst. Iulian Cri t s Soluie. Numrul fetelor este 151+1 = 15, iar numrul bieilor este t a a a t 12. La ora de Educaie fizic sunt aliniai 27 elevi. t a t P.96. Descoper a 2 5 6 a) 3 8 4 9 7 8 b) 2 5 1 (Clasa a II-a) regula 10 14 7 6

si completeaz c suele libere n cazurile: a a t 5 7 1 3 ; 2 6 13 4 3 3 5 3 . 0 1 2 0 Inst. Maria R

a b , atunci regula est c d a + c = b i b + c = d. n tabelul al treilea completm csua liber cu 2 + s a a t a al patrulea cu 13 6 = 7, iar n ultimul tabel cu 3 + 4 = 7, respectiv 4 + Soluie. a) Dac tabelul este de forma t a

b) n acest caz regula este dat de a c = b i b c = d. Tabelele se co a s cu 3 0 = 3; 5 3 = 2; b = 0 + 2, a = b + 2 = 2 + 2 = 4.

P.97. n cte moduri putem forma un sir indian compus din 4 b ie a t astfel nct dou fete s nu stea una lng alta, iar sirul s nu nceap cu a a a a a (Clasa a II-a) Andrei Burdun,

Soluie. Sirul poate ncepe cu cel mult doi biei. Avem posibilitil t a t at bfbbfbf; bfbfbbf; bbfbfbf. Sirul poate fi format n 4 moduri.

P.98. La Cr ciun, copii au mpodobit bradul cu 35 globuri albe, galben a Andrei a observat c , dac mparte num rul globurilor albe la cele galb a a a ctul 3 si restul 2, iar dac mparte num rul globurilor roii la cele galb a a s ctul 2 si restul 3. Cte globuri de fiecare fel au mpodobit bradul? (Clasa a III-a) nv. Rica Buc a Soluie. Folosim metoda figurativ. t a

Numrul globurilor galbene este a [35 (2 + 3)] : 6 = 5. 35 Numrul globurilor albe este 5 3 + a 3 Numrul globurilor roii este 5 2 + a s P.99. La concursul de alergare organizat de clasa a II-a, cei mai buni Radu, Cezar, Tudor, Dan si Mihai. Care a fost clasamentul final, dac a: nu a luat locul nti, 2) Cezar s-a clasat n urma lui Mihai, 3) Radu naintea lui Mihai, 4) Tudor nu s-a clasat al doilea, 5) Dan este al tr Tudor. (Clasa a III-a) nv. Constana Cristea i Inst. Iulian Cri t s Soluie. Deoarece Tudor nu s-a clasat al doilea i Dan este al treilea du t s deducem c Tudor s-a clasat pe locul I. Tinnd cont i de ordinea RM a s clasamentul final TRMDC. P.100. Trei numere naturale au suma 60. S se afle numerele stiind c a a du-l pe primul la al doilea obinem ctul 4 si restul 3, iar al treilea este t mare dect dublul celui de-al doilea. (Clasa a III-a) Vasile Solcanu, Bogdneti a s Soluie. Utilizm metoda figurativ. t a a Al doilea numr este a 3 a b = [60 (3 + 1)] : 7 = 8. b 60 Primul numr este a = 48+3 = 3 a 1 Al treilea numr este c = 2 8 + 1 a P.101. Exist numere naturale care mp rite la 12 s dea ctul 7, iar a a t a la 15 s dea restul 2? a (Clasa a IV-a) Alexandru-Gabriel Tudorache, Soluie. Fie n = 12k + 7 = 15p + 2. Atunci n = 3 (4k + 2) + 1 = t adic pe de o parte n d la mprirea prin 3 restul 1, pe de alt parte d a a at a imposibil. P.102. S se arate c p tratul de latur 36 poate fi acoperit cu piese a a a a 1 . 1 (Clasa a IV-a) Andrei Burdun, Soluie. Prin mbinarea a dou piese putem obine o pies drept a t a tunghiular de forma alturat. Pentru acoperirea ptratului sunt a a a a necesare (36 : 4) (36 : 2) = 162 piese dreptunghiulare. Ptratul a poate fi acoperit cu 162 2 piese iniiale. t P.103. Un dreptunghi are perimetrul de 624 cm, iar lungimea este dub Poate fi mp rit dreptunghiul ntr-o reea de p trate egale astfel nct sum a t t a trelor lor s fie 6656 cm? a (Clasa a IV-a) Petru Asa Soluie. Limea dreptunghiului este 624 : 6 = 104 cm. Dac mp t at a a tunghiul n n ptrate egale, atunci n poate fi 2, 8, 32, 128, 512, . . . . Sum a trelor este 4nl = 6656 cm, de unde nl = 1664 cm. Pentru n = 2 obinem l t

a b

> 104 cm, fals. Pentru n = 8 obinem l = 208 cm > 104 cm, fals. Pent t obinem l = 52 cm, ceea ce nseamn c limea poate fi acoperit cu d t a a at a latura unui ptrat, ceea ce este fals, deoarece dreptunghiul trebuie acop a ptrate. Pentru n = 128 obinem l = 13 cm. Atunci dreptunghiul poate a t cu (104 : 13) (208 : 13) = 8 16 = 128 ptrate. Pentru n > 128 nu avem a

Clasa a V-a

V.61. Determinai x, y, z N n fiecare din cazurile: t 25 25 a) x y = ; b) x2 + y 2 = ; 2z + 1 2z + 1 Vasile Solcanu, Bogdneti a s Soluie. a) Deoarece xy N, atunci 2z + 1 | 25, adic 2z + 1 {1, 5, 25 t a mare z {0, 2, 12}. Dac z = 0, atunci xy = 25, deci (x, y) {(1, 25) ; (5, 5 a Dac z = 2, atunci xy = 5, deci (x, y) {(1, 5) ; (5, 1)}. Dac z = 12, atun a a deci (x, y) = (1, 1). n concluzie, b) Obinem tot z {0, 2, 12} i, analiznd toate cazurile, gsim c t s a a

(x, y, z) {(1, 25, 0) ; (5, 5, 0) ; (25, 1, 0) ; (1, 5, 2) ; (5, 1, 2) ; (1, 1, 12)

(x, y, z) {(0, 5, 0) ; (5, 0, 0) ; (3, 4, 0) ; (4, 3, 0) ; (1, 2, 2) ; (2, 1, 2) ; (0, 1, 12) ; ( S notm c a) i b) nu pot avea loc simultan. a a a s

V.65. Vom numi "numr p" un num r natural care are exact 4 diviz a a a) Dai exemplu de trei numere p consecutive. t b) S se arate c nu exist trei numere p consecutive astfel nct primu a a a s fie par. a Ovidiu Pop, Sa Soluie. a) De exemplu, putem considera numerele 33, 34, 35. t

V.64. S se determine x, y, z N, z 6= 0, nct 5x +5y = z!, unde z! = a Doru Turb Soluie. Notm U2 (A) ultimele dou cifre ale numrului A. Dac x t a a a a atunci U2 (5x + 5y ) = 50, n timp ce U2 (z!) 6= 50, z N. Dac x = 1, y a U2 (5x + 5y ) = 30, n timp ce U2 (z!) 6= 30, z N. Analog se arat c n a a cazul x 2, y = 1. Rmn de studiat cazurile (x, y) {(0, 0) , (1, 0) , (0, a Analizndu-le pe rnd, obinem soluiile (x, y, z) {(0, 0, 2) , (1, 0, 3) , (0, 1 t t

V.62. S se scrie num rul 12321 ca diferena a dou p trate perfecte. a a t a a Andrei-Sorin Cozma, t t Soluie. Observm c 12321 = 32 372 . nmulim relaia 32 + 42 = 5 t a a obinem c 32 372 + 42 372 = 52 372 , de unde 32 372 = 52 372 42 t a 12321 = 34225 21904. 3 V.63. S se scrie n ordine cresc toare numerele 33 ; 333 ; 333; 333 ; a a Ion Vian s Soluie. Observm c t a a 3 3 3 3 333 > 327 = 33 = 39 = 813 > 333 > 273 = 33 > 32 = 93 = 729 3 3 deci 333 < 33 < 333 < 33 < 333 .

b) Presupunem prin absurd c exist a = 2k, b = 2k + 1, c = 2k + 2 = a a k N, trei numere p consecutive. Cum 2k i 2 (k + 1) au fiecare exact s numerele k i k + 1 trebuie s fie ambele prime. Singurele numere prime c s a sunt 2 i 3, deci k = 2, apoi a = 4, b = 5, c = 6. Evident ns c 5 nu este s a a

Clasa a VI-a

prin urmare |b| + b = |c| + c = 0.

VI.61 Dac a = x9 y 3 z 4 t10 , b = xy 5 z 6 t8 , c = x5 y 7 z 10 t12 si |a| + a a arate c |b| + b = |c| + c = 0. a Cristian - Ctlin Bude a a Soluie. Evident c |a| + a = 0 a 0. Deoarece a = xy x4 yz t a rezult c xy 0. Atunci a a 2 b = xy y 2 z 3 t4 0, c = xy x2 y 3 z 5 t6 0, VI.62. Fie a1 , a2 , . . . , an {1, 1} astfel nct an1 an a1 an a1 a2 a1 a2 a3 a2 a3 a4 + + + + = 0. a4 a5 a2 a3

Ioana Olan, e Soluie. Suma dat are n termeni, fiecare dintre ei fiind +1 sau 1. t a fiind 0, numrul termenilor egali cu +1 este acelai cu al celor egali cu a s urmare n = 2k. Produsul celor n fracii este, pe de o parte, (1)k (+1)k = t

S se arate c n se divide cu 4. a a

de alta este a2 a2 a2 = +1; rezult k = 2l, l N. n concluzie, n = 4l, d a n 1 2 S remarcm c pentru n = 4 nu exist numere a1 , a2 , a3 , a4 care a a a a ipotezele problemei, ns pentru n = 8 putem considera numerele 1, 1, a 1, 1.

VI.63. Se poate completa un p trat 10 10 cu numerele de la 1 la 100 a pentru fiecare coloan s se poat forma (cu numerele din acea coloan a a a a) astfel ca sumele numerelor din fiecare grup s fie egale? a a Bogdan Andrei Ciacoi, elev 100 101 Soluie. Suma tuturor numerelor din ptrat este t a = 5050. 2 putea completa ptratul n acest fel, atunci datorit faptului c pe fieca a a a suma numerelor este multiplu de 3, suma numerelor de la 1 la 100 ar fi mul Cum 5050 nu este multiplu de 3, rezult c ptratul nu se poate completa a a a dorit.

VI.64. Pentru fiecare n N, n 4, ar tai c interiorul oric a t a arui convex se poate descompune n reuniune de n triunghiuri dreptunghice cu i disjuncte. Gabriel P Soluie. Observm nti c interiorul oricrui triunghi poate fi mpr t a a a a triunghiuri dreptunghice, ducnd nlimea din vrful unghiului cel mai m at cauie necesar dac triunghiul iniial este obtuzunghic). Dac n = 4, o t a a t a mparte interiorul patrulaterului n dou triunghiuri, iar dup procedeul a a

obinem 4 triunghiuri dreptunghice. Dac se poate face o descompuner t a unghiuri dreptunghice, putem realiza descompunerea n k + 1 triunghiu nlimea ipotenuzei n unul din primele k i cu aceasta rezolvarea este n at s

VI.65. Fie 4ABC, DE k BC, cu E (AC) si D (AB), iar F si {G} = F E DC. Demonstrai c , dac dou dintre urm t a a a atoarele afir adev rate, atunci este adev rat si a treia: a a a (i) BD = 2 BF ; (ii) AC = 2 CE; (iii) EF = 2 F G. Claudiu - Stefan P Soluie. Fie H mijlocul lui [CE], iar {L} = BE t A F H, {K} = CD F H. (i), (ii) (iii). Folosim n mod repetat teoremele liniei mijlocii ntr-un triunghi/ trapez i reciprocele lor. s D E BC G Cum AE = EC i DE k BC, atunci DE = s . Apoi, 2 K F din DF = F B i EH = HC, obinem F H k BC, s t L deci [F L], [F K] vor fi linii mijlocii n 4BDE, respectiv DE BC 4DBC. Atunci F L = = , LK = F K F L = B 2 4 BC BC BC = , adic F L = LK = KH. n 4F EC, F H este m a 2 4 4 2 FK = , deci K va fi centru de greutate; rezult c CK este medi a a KH 1 F G = GE.

(i), (iii) (ii). n 4CEF , F H i CG sunt mediane, deci K va fi s BC DE greutate. Rezult c F K = 2 KH i cum F K = a a s , KH = , atu 2 2 2 DE. Conform unei a doua reciproce a teoremei liniei mijlocii n triunghi, cunoscut, rezult c [DE] este linie mijlocie n 4ABC (altfel [D0 E 0 ] ar f a a a BC t a mijlocie i cum DE = D0 E 0 = s , DE k D0 E 0 k BC, am obine c DE 2 0 0 paralelogram, adic DD k EE i atunci AB k AC, fals!). a s

(ii), (iii) (i). Presupunem prin absurd c F nu este mijlocul lui [BD] a F 0 acest mijloc, iar {G0 } = F 0 E DC. Din (ii), EF 0 = 2 F 0 G0 , deci [GG0 mijlocie n 4EF F 0 , prin urmare GG0 k BD, adic DC k BD, absurd. R a are loc (i).

Clasa a VII-a
VII.61. Fie n N fixat si p, k N, p < k. S se arate c a a 1 1 2 (k p + 1) 1 + + + > . n+p n+p+1 n+k 2n + k + p

(n legtur cu V.31 din RecMat - 2/2002). a a Soluie. Avem: t (n + p) + (n + p + 1) + + (n + k) =

Gigel Buth, Sa

(n + k) (n + k + 1) (n + p 1) 2 2

(2n + k + p) (k p + 1) . 2 Inegalitatea dintre media armonic i cea aritmetic arat c as a a a = (n + p) + (n + p + 1) + + (n + k) > kp+1
1 n+p

kp+1 1 + n+p+1 + +

1 n+k

Din (1) i (2) obinem inegalitatea dorit. Problema V.31 se obine p s t a t 100, p = 1, k = 100. VII.62. Fie a R astfel nct a10 a6 + a2 = 4. S se arate c 7 < a a Alexandru Negrescu, elev,

a s Soluie. Faptul c a10 a6 + a2 = 4 implic a 6= 0 i a 6= 1, de unde a t a 4 8 1 a4 + 1 10 a a i a4 + 4 > 2. Din a12 + 1 = a + 1 a a4 + 1 = s a a2 1 4 a2 + 2 , avem a12 + 1 > 4 2, deci a12 > 7. Din a10 + a2 = a6 a 1 4 4 4 a + 4 = 1 + 6 , de unde 1 + 6 > 2, deci a6 < 4, prin urmare a12 < 16 a a a VII.63. Fie triunghiurile ABC si A0 B 0 C 0 cu AB = A0 B 0 , BC = B 0 C toarele [BE], [B 0 E 0 ] congruente. S se arate c 4ABC 4A0 B 0 C 0 . a a Petru Asa Soluie. A se vedea n acest numr nota Criterii de congruena a triu t a t pag. 107-109.

VII.64. n triunghiul echilateral ABC consider m cevienele AM , B a concurente ntr-un punct O, interior triunghiului. Ar tai c , dac 4M a t a a echilateral, atunci O este centrul 4ABC. Temistocle B Soluie. Unghiurile marcate sunt congruente. ntrt A b \ \ adevr, m(A) = 60 implic m(AN P ) = 120 m(AP N ); a a \ \ \ m(N P M ) = 60 implic m(BP M ) = 120 m(AP N ), a \ \ \ deci AN P BP M . Analog artm c BP N CM N ; aa a \ P \ BM P CN M . Se arat \ \ vom avea i egalitile AP N s at a O uor c 4AN P 4BP M 4CM N , de unde AP = s a BM = CN i AN = BP = CM . Conform teoremei lui s MB NC P A M Ceva, are loc relaia t = 1, care revine B MC NA P B 3 MB la = 1 sau M B M C. Rezult c M , N , P sunt mijloacel a a MC triunghiului ABC, deci O este centrul acestuia.

VII.65. Fie ABCD paralelogram. O dreapt variabil ce trece pr a a dreptele BC si CD n F , respectiv G si taie paralela prin C la BD se arate c AE 2 AF AG. a Ctlin Cali a a

A Soluie. Fie {O} = AC BD, {P } = AE BD; t cum [OP ] este linie mijlocie n 4ACE, vom avea c a O AE = 2 AP . Din asemnrile evidente 4AP D a a P 4F P B, 4ABF 4GCF , 4GCF 4GDA, 1 BF 1 CF obinem respectiv t = , = , F B AP P F BC AF GF BF E 1 CF = . De aici, G AG GF BC 1 CF 1 1 CF BF + BC 1 PF B 1 + = + = AF AG GF BF BC GF BF BC AP BF 1 CF BF + BC P F = . AP F G BF BF PF BC + BF FB = = Din 4DP A 4BP F obinem t BC + F B AP + P F BF 1 1 CF AF P F 1 2 1 + = = = AF AG AP F G P F BF AP AE i de aici concluzia rezult imediat, aplicnd inegalitatea mediilor. s a

Clasa a VIII-a

VIII.61. Rezolvai n numere naturale ecuaia x! = y 2z + 1. t t Denisa Floric i Lucian Tuescu as t Soluie. Pentru x {0, 1}, obinem y = 0, z N. Pentru x = 2, g t t a z N sau y N, z = 0. Pentru x 3 nu mai avem soluii, deoarece x! se t 3, pe cnd y 2z + 1 nu (dac y = 3k, atunci y 2z + 1 = M 3 + 1, iar dac y a a atunci y 2 = M 3 + 1, deci y 2z + 1 = M 3 + 2). VIII.62. If a, b, c > 0 and a + b + c 3, prove that 2 (a + b + c)2 3 (ab + bc + ca) + 3 (a + b + c) .

Condiia din ipotez i inegalitatea Cauchy - Schwarz arat c t as a a 2 a + b + c a + b + c 3. 3 (a + b + c)

Babis Stergiou, Chalkida Soluie. Inegalitatea de demonstrat revine la t 2 a2 + b2 + c2 + (ab + bc + ca) 3 (a + b + c) .

Atunci: 2 a2 + b2 + c2 + (ab + bc + ca) a2 + b2 + c2 + 2 (ab + bc + ca) = (a + b + c)2 = (a + b + c) (a + b + c) 3 (a + b + c) ,

deci (1) este adevrat. Egalitatea se atinge pentru a = b = c = 1. a a

VIII.63. Let a, b be distinct nonzero real numbers. Find all solutions ( 4ab of the equation (x 1)2 + y 2 + 2 (x 1) y = 0. a + b2 Jos Luis Daz-Barrero, Barcelon

Soluie. Ecuaia devine succesiv: t t 2 a + b2 x 12 + y 2 + 4ab (x 1) y = 0 2 2 a x + b2 y 2 + a2 + 2abxy 2a2 x 2aby + + b2 x2 + a2 y 2 + b2 + 2abxy 2b2 x 2aby = 0 (ax + by a)2 + (bx + ay b)2 = 0.

Evident atunci c ax + by a = bx + ay b = 0, de unde obinem x = a t VIII.64. S se afle num rul de p trate perfecte p astfel nct 2n p a a a Marian Pan t Soluie. Dac n = 2k, k N, atunci primul ptrat perfect va fi t a a Dac notm cu m numrul de ptrate perfecte cutat, atunci acesta ver a a a a k 2 2 2k+1 inegalitate 2 + m 1 < 2 < 2k + m , de unde m 1 < 2k 2 ceea ce ne arat c m 1 partea ntreag a numrului 2k 2 2k , a a a este a a 22k i 22k+1 se gsesc 2k 2 2k + 1 ptrate perfecte. Analog tratm s a a a 2k + 1, k N, obinnd m = 2k+1 2k 2 . t VIII.65. Fie M un punct interior tetraedrului ABCD, iar dA , dB distanele de la M la planele (BCD), (ACD), (ABD), respectiv (ABC t MA MB MC MD arate c a + + + 12. dA dB dC dD D.M. Btineu-Giurgiu, B a t Soluie. Fie MA , HA proieciile punctelor M , respectiv t t A, pe planul (BCD); evident c dA = M MA , ha = AHA a i analoagele. Mai notm sA = SBCD , sB = SACD etc. s a Observm c: a a M AM + M MA AMA AHA sA M A + sA dA sA hA = 3VABCD B sA M A + sA dA sA dA + sB dB + sC dC + sD dD MA sB dB sC dC sD dD MA > + + . C dA sA dA sA dA sA dA Grupnd convenabil termenii, obinem: t X M A X sA dA sB dB 6 2 = 12, + dA sB dB sA dA cu egalitate cnd tetraedrul este regulat, iar M este centrul su de greuta a

Clasa a IX-a

IX.61. The radii of the three escribed circles of a triangle ABC ar rb = 6, rc = 12. Find the lenghts of the sides of the triangle. Jos Luis Daz-Barrero, Barcelon Soluie. Folosim succesiv binecunoscutele relaii t t 1 1 2 1 1 + + ; S = rra rb rc ; S = ra (p a) = rb (p b) = rc (p = r ra rb rc Obinem r = 2, apoi S = 24, prin urmare a = 6, b = 8, c = 10. t

IX.62. Rezolvai sistemul n necunoscutele x1 , x2 , . . . , xn R , n N t 2 (n 1)


k=1 n Y n X X 2 n1 2 2 2 xk = 1 + x1 xk = 1 + xn xk . k=2 k=1

ilor, 4 0. ns x1 R prin urmare trebuie s avem 4 0, fapt care se a a cnd x2 = x2 = = x2 . Analog obinem c x2 = x2 = = x2 t a 1 2 3 n 2 n (n 1 2 2 2 s a a x1 = x2 = = xn = t R+ . nlocuim n (1) i gsim c x1 = (n 1 n3 , de unde t = 1. n concluzie, soluiile sistemului au forma (x1 , x2 , t t |xk | = 1, k 1, n.

2 n n Q P 2 Discriminantul este 4 = (n 1) s xk xk i, conform inegali


k=2 k=2

Silviu Boga Soluie. Considernd o soluie (x1 , x2 , . . . , xn ), din prima egalitate ob t t n n ! ! n X Y X 2 2 xk x1 2 (n 1) xk x1 + x2 = 0. k


k=2 k=2 k=2

Atunci s2 2p 1 + p, deci 2 2 1 s2 2p p s2 2p p2 (1 + p) p (1 + p) p2 = 2 1 5 5 = p2 + p + 1 = p + . 2 4 4 5 s Deoarece 1 p2 +p+1 i p2 +p+1 Z, atunci p2 +p+1 = 1, deci 4 2 Ne amintim c s > 0 i s 3p 1 i obinem soluiile (x, y) {(0, 1) ; (1, a s s t t Analog, dac s < 0 gsim soluiile (x, y) {(0, 1) ; (1, 0) , (1, 1)}. a a t dac s = 0, evident c irul de inegaliti din ipotez este adevrat cu egal a as at a a pereche (n, n), n Z constituie soluie a ecuaiei. t t

IX.63. Determinai x, y Z pentru care x3 + y 3 x + y x5 + y 5 . t Romeo Ilie Soluie. Notm x + y = s, xy = p. Dac s > 0, avem: t a a (x + y) x2 xy + y 2 x + y (x + y) x4 x3 y + x2 y 2 xy 3 + y 2 (x + y)2 3xy 1 x2 + y 2 x2 y 2 xy x2 + y 2 2 s2 3p 1 s2 2p p s2 2p p2 .

IX.64. Fie ABCD un patrulater, iar HA , HB , HC , HD ortocentrele urilor BCD, ACD, ABD, respectiv ABC. Demonstrai c AC = HC H t a numai dac BD = HD HB . a Ioan Scleanu a a Soluie. Notm cu OA , OB , OC , OD centrele circumscrise triunghiur t a ACD, ABD, respectiv ABC. Tinnd cont de relaia Sylvester, se obine t t OA HA + HC OC = OA B + OA C + OA D + AOC + BOC + DOC

OA C + CHA + HC A + AOC = OA C + AOC + OA B + BOC + OA D + CHA + HC A = 2OA OC 2OC OA = AHC + HA C. Analog se deduce c 2OB OD = DHB + HD B. Atunci a AC = HC HA AHC = CHA AHC + HA C = 0 OC = OA ABCD patrulater inscriptibil OB = OD DHB + HD B = 0 BD IX.65. Fie a, b R astfel nct sin x + sin a cos x cos b, x R. S ab c a Z. Adrian Zano 3 relaia dat devine 1 + si t a Soluie. Dac sin a < 1, pentru x = t a 2 sin a < 1, contradicie. Rmne c sin a = 1, adic a = + 2k, k Z. In t a a a 2 din enun devine: t sin x + 1 cos x cos b sin x cos x cos b 1 cos b 1 1 sin x cos x 2b 2b 1 + cos 1 + cos 1 + cos2 b 1 1 sin x cos sin cos x sin (x ) 1 + cos2 b 1 + cos

pentru orice x R (am notat [0, 2) numrul pentru care cos = a

cos b 3 sin = ). Lund x = + n aceast inegalitate, obin a t 2b 2 1 + cos 1 cos2 b 0 cos b = 0, deci b = + l, l Z. Astfel 2b 2 1 + cos 2k l Z.

x X.61. If x 0, prove that log3 (1 + 3x ) > log4 4x + 3 2 . Oleg Faynshteyn, Leipzig, G Soluie. Ineglitatea dat se scrie echivalent t a !x ! x 3 2 1 x x log3 3 1 + > log4 4 1 + 3 4 !x ! x 3 1 2 log3 1 + . > log4 1 + 3 4 3 2 1 Aceast ultim inegalitate rezult din observaia c > a a a t a , iar log3 a 3 4 a log4 a > log4 a > log4 b, de ndat ce a > b. X.62. Se d sirul (zn )nN C cu proprietatea c zn+2 = zn + izn+1 a a si se noteaz cu V mulimea termenilor s i. Dac U3 V , s se arate c a t a a a a (Am notat Uk = z C | z k = 1 .) Monica Ned

Clasa a X-a

Soluie. Observm c zn+3 = zn+1 + izn+2 = zn+1 + i (zn + izn+1 ) = t a a urmare zn+12 = izn+9 = i2 zn+6 = i3 zn+3 = i4 zn = zn , deci irul este p s perioad 12. n plus, iz V pentru orice z V . a Pentru a arta c V = U12 , este suficient s demonstrm c U12 V . a a a a a U12 = {zk | zk = cos 2k 2k + i sin , k = 0, 11}. 12 12

Evident c {z0 , z4, z8 } = U3 V , apoi z3 = iz0 V , z6 = iz3 V , z9 = a z7 = iz4 V , z10 = iz7 V , z1 = iz10 V , z11 = iz8 V , z2 = z5 = iz2 V i demonstraia este ncheiat. s t a

X.63. Un num r de n juc tori arunc succesiv o moned avnd fe a a a a t jocul este ctigat de acela care obine primul faa s. S se afle probab s t t a juc atorul de pe locul k (1 k n) s ctige jocul n primele np + k ar a s monedei, unde p N este dat. Petru Asa Soluie. Problema extinde, folosind aceeai idee, cazul n = 2 prezen t s manuale. Notm cu Ai evenimentul ca juctorul de pe locul k s ctig a a a s exact ni + k aruncri ale monedei; pentru realizarea lui Ai , feele monedei a t ni+k 1 . Evenimentele A1 , apar n ordinea bb {z . } s, deci P (Ai ) = a | .. b 2
ni+k1ori

sunt incompatibile n totalitatea lor, iar A1 A2 Ap reprezint even a crui probabilitate este cerut de problem. Avem: a a a P (A) = P (A1 ) + P (A2 ) + + P (Ap ) =

1 1 1 + k+m + + k+n 2k 2 2

1 1 1 2np 1 2np 1 = n(p1)+k . = k 1 2 1 2n 2 1 2n

este suficient s artm c g (x) 1, x R . Vom demonstra acest luc a aa a etape: i) Dac |x| > 2, atunci a 0 < f (x) 2005 |x|4 2005 |x| + 3 |x|3 = 4 |x|3 < 4 1 = , 8 2

2005 |x| 2005 X.64. Fie f : R (0, ), f (x) = 4 + sin 2004 x + 2005 |x| | 1 9 1 , x R . arate c f (x) + f (x) a 2 2 1 + |x|3 Ioan Serdean Soluie. Construim funcia g : R (0, ), t t 1 + |x|3 f (x) + 1 f (x) 1 , x R ; g (x) = 9 2 2

deci f (x) +

1 1 = f (x). Rezult c a a 2 2 ! 3 3 8 8 1 1 + |x| 4 1 1 + |x| +1 2f (x) 2 3 = + g (x) + 9 9 9 |x|3 9 8 |x|

1 = f (x) + 1 , iar f (x) 2 2

Rmne de demonstrat c exist un semicerc S C astfel nct M G 1 a a a Dac G = O, inegalitatea este adevrat pentru orice M C. Dac G 6= O a a a a diametrul lui C perpendicular pe OG, iar S semicercul delimitat de aces \ lui G. Atunci m(M OG) 90 , M S, deci M G M O = 1, ceea rezolvarea.

X.65. Fie C un cerc de raz 1 si fie P1 , P2 , . . . , Pn puncte ale discului c a tor. Ar tai c exist un semicerc nchis S C astfel nct M P1 +M P2 + a t a a n, M S. Adrian Zahariuc, ele Soluie. Fie O centrul cercului, iar G centrul de greutate al sistemului t P1 , P2 , . . . , Pn . Vom demonstra c M P1 + M P2 + + M Pn nM G, pe a punct M din plan. Raportm planul complex la un reper cu originea n a afixul lui G, pi afixele punctelor Pi , i = 1, n. Atunci p1 + p2 + + pn g= n |g| = |p1 + p2 + + pn | |p1 | + |p2 | + n n M G M P1 + M P2 + + M Pn .

ii) Dac 0 < |x| 2, folosind faptul c |a| |b| |a b|, avem: a a 3 1 + |x|3 f (x) + 1 f (x) + 1 = 1 + |x| 1 + 8 = 1 g (x) 9 2 2 9 9

Clasa a XI-a

XI.61. Fie a R\{1} si A Mn (C). Dac exist k N a a a Ak+1 = aAk , s se arate c In A este inversabil . a a a Gheorghe Iu Soluie. Fie X0 Mn,1 (C) o soluie a sistemului (In A) X = O t t AX0 = X0 , deci Ak X0 = Ak+1 X0 = X0 , de unde X0 = aX0 i cum a 6= s c X0 = O. Astfel, sistemul omogen (In A) X = O are doar soluia a t atunci det (In A) 6= 0. D XI.62. Fie ABCD un p trat de centru O, iar M un a punct variabil pe [AB]. Not m {S} = CM AD, {E} = a SO M D. Se cere locul geometric al punctului E. O Petru Rducanu, Iai a s Soluia 1 (analitic). Raportm planul la un reper t a a cu originea n A, avnd dreptele AB i AD axe de coordos E nate. Putem considera A (0, 0), B (2, 0), C (2, 2), D (0, 2), O (1, 1), iar M (a, 0), cu a 2). Intersectnd dreapta A L (0, M 2a CM cu AD, obinem c S 0, t a . Atunci ecuaia t a2

a+2 (x 1) + y 1 = 0 i cum DM : 2x + a (y 2) s a2 eliminarea parametrului a ntre cele dou ecuaii gsim ecuaia locului a t a t a t a x2 + y 2 2y = 0. Aceast ecuaie reprezint cercul de diametru [AD] trebuie reinut doar semicercul interior ptratului (fr capete), deoarece t a aa Soluia 2 (sintetic). n 4AM D cu transversala ELS, conform te t a EM LA SD Menelaus obinem c t a = 1. Apoi, n 4AM C cu transversal ED LM SM SC SD CD EM LA OC SM = 1. Cum = = , atunci = obinem t LM OA SM SM SA AM ED aici rezult c AE M D, adic AEOD este patrulater inscriptibil, deci E a a a cercului de diametru [AD]. a Rmne s artm c orice punct E 0 de pe semicercul interior ptratulu a a aa a locului. Fie {M 0 } = DE 0 AB, {S 0 } = CM 0 DA, {T } = AC M 0 D. A dreptei SO este OA OD \ \ = = tg(OT D) = tg(AT E 0 ); OT OT \ ctg(AT E 0 ) AM 0 E0T E0D E0T \ : = = = ctg(AT E 0 ). E0D AE 0 AE 0 AD \ ctg(ADM 0 )

AD OA E 0 T S 0 D S0D CD = , rezult c a a = = 1. Din reciproc S 0A AM 0 AM 0 OT E 0 D 0 S 0 A 0 lui Menelaus n 4AT D urmeaz c punctele O, E , S sunt colinare, ceea a a demonstraia. t Cum

XI.63. Fie f, g : [a, b] R funcii continue astfel nct g(a)+f (b) < 1+ t S se arate c exist c (a, b) pentru care a a a 3 3 3 g (c) 2c a b 3 (2c a b) f (c) g f (c) + + = bc ac (b c) (a c) (b c)2 (a c)2

Valeriu Braoveanu s Soluie. Florin Popovici, Braov, d urmtoarea generalizare: t s a a

ntr-adevr, din ipotez rezult c (g n (a) 1) (f n (a) 1) > 0. C a a a a funcia u : [a, b] R, u (x) = f n (x) (a x)n +g n (x) (b x)n +(2x a b t Evident c u este continu pe [a, b], iar a a u (a) = (g n (a) 1) (b a)n ; u (a) u (b) = (g n (a) 1) (1 f n (b)) (b a) u (b) = (1 f n (b)) (b a)n
2n

Fie f, g, h : [a, b] R trei funcii continue pe [a, b], astfel nct ( t (f (b) 1) > 0 si h (a) = h (b) = 0. Dac n N este impar, exist c ( a a nct f n (c) (a c)n + g n (c) (b c)n + (2c a b)n = h (c).

< 0.

Urmeaz c exist c (a, b) astfel nct u (c) = 0, de unde concluzia. a a a Problema se obine pentru n = 3 i h (x) = 3 (2x a b) (a b) (b x) t s f (x) XI.64. Fie f : (0, 1) (0, ) o funcie cresc toare cu lim t a = 1 si x0 x 1 x (0, 1). Definim sirurile (an )n1 , (xn )n1 prin an = (a + b) (2a + b)

Avem c xn = eln xn = e n . Pentru a calcula limita exponentului, ne o a raportul (Stolz pentru ) f (a2n+1 ) + f (a2n+2 ) f (an+1 bn+1 ln bn+1 ln bn = ln 1 + = ln (n + 1) n bn bn bn Dar a2n+1 + a2n+2 f (a2n+1 ) + f (a2n+2 ) 1 a2n a2n+1 a2n+ + = bn nf (a2n ) n f (a2n ) a2n a2n f (x) i innd seama de faptul c lim f (x) = lim s t a x = 0, x0 x0 x bn bn+1 bn f (a2n+1 ) + f (a2n+2 ) f lim = lim = lim n f (an+1 ) n f (an+2 ) f (an+1 ) n f (a2n+2 ) f (an+1 f (a2n+2 ) a2n+2 f (an+1 ) f (a2n+1 ) a2n+1 + : lim n a2n+1 an+1 a2n+2 an+1 an+1 f (an+2 ) an+2 f (an+1 ) : =1 an+2 an+1 an+1 0 ln bn (am folosit Stolz pentru ). Rezult lim a = i, prin urmare, li s n n n 0 a t XI.65. Pentru x1 , x2 , . . . , xn R+ , x1 + x2 + + xn = nA, ar tai x x x x2 3 xn 3 x1 3 1 2 n +3 +3 +3 +2 + 2 ... + A A A A A A S se obin de aici inegalitatea dintre media aritmetic si cea geometric a t a a a Dumitru Mihalach x Soluia 1 (a autorului). Este cunoscut inegalitatea ex 1 + + t a 1! x R, egalitate pentru x = 0. (Inegalitatea se poate obine ignornd t dezvoltarea Taylor a lui ex , sau direct prin derivri repetate). Trecem pe a i gsim c s a a ex1 1 +

(a > 0, b > 0) si xn = [f (an+1 ) + f (an+2 ) + + f (a2n )] . S se a lim xn . n Gheorghe Costo Soluie. S observm nti c problema este consistent, n sensul t a a a a funcii f ca n enun; de exemplu f1 (x) = x, f2 (x) = sin x, f3 (x) = arctg x t t ln (1 + x) etc. Notm bn = f (an+1 ) + f (an+2 ) + + f (a2n ) i observm c bn 0 a s a a 1 0 < bn nf (an ) n 0. (a + b) (2a + b) (na + b)
ln bn

1/n

x 1 (x 1)2 x3 + 3x + 2 (x 1)3 + + ex1 , x 1 2 6 6 x1 x2 xn cu egalitate pentru x = 1. Dm lui x valorile a , ,... i nmulim m s t A A A x1 + x2 + xn membru inegalitile obinute; innd seama de ipoteza at t t A

unde P (x) = x3 + 3x + 2. ns a P (a1 ) P (a2 ) P (an ) 6n ln P (a1 ) + ln P (a2 ) + + ln P (an ) ln P (a1 ) + ln P (a2 ) + + ln P (an ) a1 + a2 + + an ln P n n a1 + a2 + + an deoarece P = P (1) = 6. Ultima inegalitate ar fi n dac funcia ln P (x) ar fi concav. Cum a t a 2 2 00 0 P (x) P (x) [P (x)] = 6x x3 + 3x + 2 3x2 + 3 = = 3 x4 4x + 3 = 3 (x 1)2 x2 + 2x + 3 0, demonstraia este ncheiat. t a Not. Cum domeniul de definiie al funciei ln P (x) este de forma ( a t t x0 (1, 0), putem spune c inegalitatea din enun are loc pentru x1 , x2 a t (x0 , +), cu x1 + x2 + + xn 6= 0.

x1 x2 xn obinem concluzia. Egalitatea se atinge pentru t = = = = A A A x2 = = xn . Vom demonstra c x3 + 3n + 2 6x, x R ; ntr-adevr, acest fap a a + 2 n x x . . . x , ob (x 1) (x + 2) 0, evident adevrat. Cu notaia G = a t t 1 2 n x x x x1 3 x2 3 xn 3 1 2 n 6n +3 +3 +3 +2 + 2 A A A A A A x x x Gn Gn 1 2 n 6 6 6 = 6n n n 1 G A A A A A A adic tocmai inegalitatea mediilor. a Soluia 2 (Marian Tetiva, Brlad). Fie x1 = a1 A, x2 = a2 A, . . . , xn t x1 + x2 + + xn a1 + a2 + + an = = n; inegalitatea de demonstrat d 3 3 A 3 a1 + 3a1 + 2 a2 + 3a2 + 2 . . . an + 3an + 2 6n P (a1 )P (a2 )

Clasa a XII-a
x&0 x&0

Mihai Ha 1 Soluie. (dat de Florin Popovici, Braov). Cu notaia u = F t a s t 2 din enun se scrie sub forma u00 (x) u0 (x) = x 1, x (0, ), c t ecuaie diferenial liniar de ordin 2, cu coeficieni constani, neomogen t t a a t t x2 1 2 x sa general este u (x) = c1 + c2 e , x (0, ), adic F (x) = c1 + a a 2 2 1 t a x (0, ). Din condiia lim F (x) = 1 obinem c c1 + c2 = . Apoi, d t x&0 2 1 ambii membri i folosind din nou condiia iniial, gsim c c2 = , deci s t t a a a 2 concluzie, F 2 (x) = ex x2 , x (0, ).

F (x) f 0 (x) = F (x) f (x) f 2 (x) + x 1, x (0, ).

XII.61. S se determine funciile derivabile f : (0, ) (0, ) c a t o primitiv F : (0, ) (0, ) pentru care lim F (x) = 2 lim f ( a

1 = f (0), pentru orice 0 < a < f (0), exist b) Deoarece lim nF n n 1 1 a pentru care n F > a, n > n1 , altfel spus F > , n n n Atunci n1 n1 n n n X 1 X 1 X 1 X 1 X 1 = + > F F F F +a k k k k k k=1 k=1 k=n1 +1 k=1 k=1 n1 n1 n X X1 X 1 1 , n > n1 > +a F a k k k k=1 k=1 k=1 n n P 1 P 1 = +. i cum lim a s F = +, rezult c lim a a n k=1 k n k=1 k XII.63. Fie f, g : R R dou funcii surjective. Dac g (g f ) este a t a toare si exist L > 1 astfel nct |g (x) g (y)| L |x y|, x, y R, s s a a f are un unic punct fix. Sorin Pupan s a Soluie. Din |g (x) g (y)| L |x y|, x, y R, rezult c g est t a a i, cum g surjectiv, rezult g bijectiv; atunci g 1 este o contracie de s a a a t 1 < 1, deci continu. Rezult c g este continu i cum este i injectiv a a a as s a L este strict monoton. Folosind acum faptul c g (g f ) este descresctoa a a a c g f este monoton, iar g i g f au monotonii diferite. Distingem si a a s i) Fie g descresctoare, iar g f cresctoare; dac x, y R, x < y, atu a a a

Pentru x (0, 1], avem ex x2 ex 1 > 0. Pentru x (1, x 00 0 e x2 = ex 2 > e 2 > 0. Apoi, ex x2 > 0, x (1, ) i cum s deducem c ex x2 > 0, x (1, ). Astfel, F (x) = ex x2 , x (0 a ex x2 urmare f (x) = F 0 (x) = , x (0, ) este unica soluie a prob t 2 ex x2 t XII.62. Fie f : R R o funcie continu , iar F primitiva sa care s + a n P 1 1 . F ; b) lim n origine. S se calculeze: a) lim xF a x n k=1 x k Dan Popescu Soluie. a) Avem: t 1 F (y) F (y) F (0) lim xF = lim = lim = F 0 (0) = f (0) x y0 y0 x y y0
y>0 y>0

(g f ) (x) (g f ) (y) g (x) g (y) f (x) f (y) |f (x) f (y)| |g (x) g (y)| |f (x) f (y)| L |x y| , x, y (g f ) (x) (g f ) (y) f (y) f (x) g (y) g (x) |f (x) f (y)| L |x y| , x, y R.

ii) Fie g cresctoare, g f descresctoare; dac x, y R, x < y, atunc a a a

n ambele cazuri am obinut c |f (x) f (y)| L |x y| , t a


1

este contracie de constan t surjectiv, ca i n cazul lui g, obinem c f a s t a deci are un unic punct fix, acesta fiind unicul punct fix i pentru f . s

x, y R

XII.64. Fie M mulimea funciilor f : R R derivabile de dou t t a a t a a proprietatea c exist R astfel nct f 00 = f + 1. Ar tai c exist o a a grupuri {(G , ); R} ce verific simultan: a 1) (G , ) ' R2 , + , R; 2) {G ; R} este o partiie a mulimii M . t t Temistocle B Soluie. Pentru R fixat notm cu G mulimea soluiilor ecuaiei d t a t t t f 00 = f + 1. Ecuaia caracteristic asociat este 2 = , cu rdcinile: 1 t a a a a dac > 0, 1,2 = i , dac < 0 i 12 = 0 (dubl), dac a a s a a 1 observm c o soluie particular a acestei ecuaii este f (x) = , daca a a t a t 2 x , dac = 0. Ca urmare, avem: a f0 (x) = 2 n o a G = ae x + be x 1/; a, b R , dac > 0; a G = a cos x + b sin x 1/; a, b R , dac < 0 2 x G0 = ax + b + ; a, b R , dac = 0. a 2 Evident, M = S

C sunt distincte, atunci polinoamele Pk (t) sunt identic nule pe R, i = 1, n ncheie demonstraia punctului 2). t Pe mulimea G , cu 6= 0, definim operaia prin (f g) (x) t t 1 g (x) + , x R, iar pe G0 considerm 0 dat de (f 0 g) (x) = f (x) + g a a s a x R. Stabilitatea operaiilor , R, i axiomele de grup se verific c t (elementele neutre sunt funciile i f0 definite mai sus). t f s n sfrit, dac fi (x) = ai e x + bi e x 1/, i = 1, 2, avem (f1 s a (a1 + a2 ) e x + (b1 + b2 ) e x 1/ i deducem c (G , ) ' R2 , s a > 0. La fel se procedeaz n cazurile < 0 i = 0. a s

t s {G ; R}. Mulimile G sunt nevide i G1 G2 n P k t 1 6= 2 (se aplic rezultatul: dac a a Pk (t) e = 0, t R, unde 1 , 2
k=1

XII.65. Fie p1 < p2 < < pn numere prime, iar P mulimea polino t 1 grad n cu termenul liber 1 si ceilali termeni distinci n mulimea t t t |i= pi S se arate c pentru orice x N \ {1} relativ prim cu toate numerele a a 37xn + 42 q, r P astfel nct 0 |q (x) r (x)| < . Dac , n plus, x a 42n! inegalitatea de mai sus este strict . a Marius Pachiariu, t Soluie. Avem: t

1 n 1 n1 1 1 1 n max k (x) = x + x + + 1 x + + + n1 kP p1 p2 p1 2p2 2 pn 1 1 1 1 1 xn + + + + + ... +1< 2 2 3 22 5 78 89 1 1 1 1 37 n < xn + + + +1< x + 1. 2 6 20 7 42 (Am utilizat inegalitatea evident pn 2n1 n (n 1), precum i faptul a s 1 1 1 1 1 1 + + + = < ). 78 89 m (m + 1) 7 m+1 7 Cum mulimea P conine n! polinoame, putem gsi dou q, r distincte p t t a a s aib loc inegalitatea din enun. a a t Pentru partea a doua, fie q 6= r i x N \ {1} relativ prim cu toate n s i astfel nct x pn 1. S presupunem prin absurd c q (x) = r (x), de s a a 1 n1 1 n 1 n1 1 n x + x + + 1 = x + x + + 1, pi1 pi2 pj1 pj2 unde (i1 , i2 , . . , in ) i (j1j2 , . . . , j) sunt permutri distincte ale mulimii { . s , n a t 1 1 1 1 1 1 Atunci xn1 +xn2 + + = 0 i, s pi1 pj1 pi2 pj2 pin pjn p1 p2 pn (pjn pin ) aceast relaie cu p1 p2 pn , obinem c x divide a t t a ,d pin pjn a a necesar pin = pjn . Analog se demonstreaz c pik = pjk , k = 1, n, dec astfel ajungem la o contradicie. t

IMPORTANT

n scopul unei legturi rapide cu redacia revistei, pot fi utilizate urm a t a adrese e-mail: tbirsan@math.tuiasi.ro, profgpopa@yahoo.co. aceast cale colaboratorii pot purta cu redacia un dialog privitor l a t rialele trimise acesteia, procurarea numerelor revistei etc. Sugerm a ratorilor care trimit probleme originale pentru publicare s le numer a s-i rein o copie xerox a lor pentru a putea purta cu uurina o as t a s t prin e-mail asupra acceptrii/neacceptrii acestora de ctre redacia r a a a t

La problemele de tip L se primesc soluii de la orice iubitor de mat t elementare (indiferent de preocupare profesional sau vrst ). Fiecar a a soluiile acestor probleme - ce sunt publicate n revist dup un an t a a urmat de numele tuturor celor care au rezolvat-o. a

Adresm cu insistena rugmintea ca materialele trimise r a t a s nu fie (s nu fi fost) trimise i altor publicaii. a a s t

Rugm ca materialele tehnoredactate s fie trimise pe adresa redac a a A soite de ierele lor (de preferina n L TEX). t s t

Soluiile problemelor pentru pregtirea concur t a din nr. 2 / 2005


A. Nivel gimnazial

restul i la mprirea prin n. Dac cel puin o mulime Ai este vid, cum n2 at a t t a (n 1) (n 1) + 1, din principiul cutiei rezult c exist mcar o mulim a a a a t conine cel puin n elemente, iar suma acestora se divide evident cu n. D t t mulimile Ai , i = 0, n 1, sunt nevide, alegem cte un element din fiecar t n (n 1) Suma acestora, modulo n, este 0 + 1 + + (n 1) (mod n) 2 n (n 1) 0 (mod n). este impar, n 1 este par i rezult c s a a 2 G87. Ar tai c exist o infinitate de numere naturale n pentru care p a t a a P ai {1, 1}, i = 1, n astfel nct 2005 = ai aj . Aflai valoarea minim t
1i<jn

G86. Fie n 3 un num r natural impar, iar A N o mulime cu n a t elemente. S se arate c putem alege n numere din mulimea A cu prop a a t suma lor se divide cu n. Titu Zvonaru, C n1 S Soluie. Scriem A = t Ai , unde Ai conine toate elementele lui A t
i=0

i cum 4010 = 642 86, putem alege a1 + a2 + + an = 64, a2 + a2 + s 1 2 Aceste dou egaliti sunt realizate pentru n = 86, a1 = a2 = = a at a76 = a77 = = a86 = 1. Bineneles, exist o infinitate de posibiliti de alegere ale lui n. De t a at scriind 4010 = 652 109, ar trebui ca a1 + a2 + + an = 65, a2 + a2 + + 1 2 deci putem considera n = 109, a1 = a2 = = a88 = 1, a89 = a90 = = a Se poate continua! Demonstrm c valoarea minim a lui n este 86. Fie p numere egale a a a numere egale cu 1, p + q = n. Atunci 4010 = (a1 + a2 + + an )2 a2 + a2 + + a2 = (p q)2 p 1 2 n 2 (2p n) n = 4010 4p2 4np + n2 n 4010 = 0.

Gheorghe Iu Soluie. Putem scrie: t X 4010 = 2 ai aj = (a1 + a2 + + an )2 a2 + a2 + + a 1 2


1i<jn

Cum p N, trebuie ca discriminantul ecuaiei de grad II n p de mai sus s t a perfect. Avem c = 16 (n + 4010) i cel mai mic ptrat perfect de acea a s a se obine pentru n = 86, cnd = 16 4096 = 2562 . t

a G88. Spunem c o mulime M R+ are proprietatea (P ) dac ori a t din M este media geometric a dou elemente distincte ale lui M . a a a) S se arate c exist o infinitate de mulimi cu proprietatea (P ). a a a t b) G sii mulimile cu 2005 elemente avnd proprietatea (P ). a t t Gabriel Popa i Paul Georg s

Soluie. a) Pentru orice x R \ {1}, mulimea M = {xn | n Z} a t t + n 2n , iar 1 = n0 xn0 , cu n Z arbitr etatea (P ), deoarece x = 1 x x 0 b) Fie M o mulime finit cu proprietatea (P ), iar a M cel mai ma t a al acesteia; atunci a este media geometric a elementelor b, c M , cu b < a c b < a < c, deci c ar fi un element al lui M strict mai mare dect a! n a nici o mulime finit nu are proprietatea (P ). t a G89. Pentru a, b R , s se demonstreze inegalitatea a + 1 1 1 1 1 1 1 + + + + . a2 + ab + b2 a2 + a + 1 b2 + b + 1 3 a b ab Marius Pachiariu, t Soluie (semnalat de Titu Zvonaru, Comneti). Inegalitatea t a a s 1 1 (a b)2 0 conduce la a2 + ab + b2 3ab, deci 2 a + ab + b2 3ab 1 1 1 1 obinem c 2 t a , 2 i prin sumare obinem s t a +a+1 3a b + b + 1 3b problemei. Egalitatea se atinge pentru a = b = 1. Din pcate, aceast rezolvare evident nu a fost observat n mome a a a a trii problemelor, fapt care a condus la includerea ei n seciunea Proble a t preg tirea concursurilor. a G90. Fie n 1 un num r natural. S se arate c mulimea A = {n+1, n a a a t se poate partiiona n dou submulimi, fiecare cu aceeai sum a element t a t s a si numai dac n = 4k, k N sau n = 4k + 1, k N \ {1}. a Marian Tetiv Soluie. Dac A se poate partiiona cum cere enunul, suma element t a t t n (3n + 1) (n + 1) + (n + 2) + + 2n = trebuie s fie numr par, deci n (3 a a 2 Cum n i 3n + 1 au pariti diferite, unul dintre ele este multiplu de 4, de s at k N sau n = 4k + 1, k N . n al doilea caz, nu putem avea k = A = {6, 7, 8, 9, 10} i suma elementelor fiecarei submulimi din partiie a s t t fie 20; submulimea care-l conine pe 10 nu poate avea ns suma element t t a Reciproc, fie nti n = 4k, k N. Atunci A = {4k + 1, 4k + 2, . . . , 8k} p prit n 2k submulimi cu aceeai sum a elementelor: {4k + 1, 8k}; {4k + at a t s a . . . ; {6k, 6k + 1}. Reuniunea a k dintre aceste submulimi constituie u t clasele partiiei, iar cele rmase cealalt clas. t a a a Fie acum n = 4k + 1, k 2; atunci A = {4k + 2, 4k + 3, . . . , 8k A = B C partiia dorit. n B punem, pentru nceput, numerele 4k + t a 4k + 4, 4k + 6, 4k + 8, iar n C numerele 4k + 5, 4k + 7, 4k + 9, 8k + 2; p elementele din B i cele din C au aceeai sum 20k + 23. Numerele rm s s a a 4k + 10 la 8k + 1, le mpim n k 2 grupe de cte 4 numere consecutive at {p, p + 1, p + 2, p + 3}. Numerele p i p + 3 din fiecare grup le punem n B s a i p + 2 le punem n C, realiznd astfel partiia dorit. s t a G91. G sii cel mai mic num r natural k pentru care, oricum am colo a t a de sah 8 8 cu k culori astfel nct fiecare culoare s fie folosit cel pu a a t exist 6 c sue de culori diferite care se afl pe aceeai linie sau coloan . a a t a s a Adrian Zahariuc, ele

Soluie. Rspunsul este 35. Dm mai nti un exemplu de colorare t a a culori care nu are proprietatea dorit: mprim tabla n 4 table 4 4, a at stnga-sus o colorm n albastru, pe cea din dreapta-jos o colorm n rou a a s csuelor, n numr de 32, vor fi colorate fiecare cu cte una dintre cele 3 a t a rmase. n acest fel, pe fiecare linie i pe fiecare coloan vom avea ct a s a diferite. Demonstrm n continuare c numrul k = 35 veric cerinele proble a a a a t supunem prin absurd c ar exista o colorare cu 35 de culori care nu are pr a dorit. Numerotm culorile de la 1 la 35. Fie li , respectiv ci , numrul de lin a a a pe care se afl cel puin o csua de culoare i. Cum intersecia a li linii cu a t a t t 35 P are li ci elemente, rezult c a a li ci 64 (). Din faptul c pe fiecare lin a
i=1

avem cel mult 5 culori diferite, rezult c a a

i=1

s ci 1. Fie xi = li 1 0, yi = ci 1 0 i atunci
i=1 35 P

35 P

li 40,

i=1

35 P

ci 40 i, desig s
i=1 35 P

xi =

i=1

yi =

i=1

35 P

at Putem presupune, din simetrie, c max xi max yi . Cele trei inegaliti a avem la ndemn nu sunt suficiente pentru a obine o contradicie, dar nu a t t 3 cazuri: i) xi = yi = 5 i xk , yk = 0 pentru k 6= i (pentru un i fixat); atun s culoare care ocup (nu n ntregime!) 6 linii i 6 coloane, iar restul culo a s exact o dat, deci vor rmne dou linii i dou coloane care conin csu a a a s a t a t diferite dou cte dou, contradicie cu presupunerea asumat. a a t a ii) xi = 5, yi = 4 i xk , yk = 0 pentru k 6= i (pentru un i fixat); ace s evident mai defavorabil dect primul. iii) xi = 5, yi = 4, yj = 1, xk = 0 pentru k 6= i, yk = 0 pentru k 6= i, j; nu putem evita existena unei linii sau coloane alctuit numai din csu t a a a t diferite, deoarece 5 + max {1, 2} = 7 < 8. Rezult c presupunerea fcut este fals i rmne adevrat conclu a a a a as a a a mei. G92. Fie 4ABC, cu [AB] cea mai scurt latur Bisectoarea un a a. intersecteaz paralela dus prin B la AC n M , iar latura AB n C 0 . Pa a a C 0 la AC intersecteaz M A n P si BC n Q. Ar tai c n limile 4A a a t a at laturi ale unui triunghi dac si numai dac 2AB > P Q. a a Valentina Blendea i Gheorghe Blen s 2S M P A Soluie. Cu notaiile uzuale, ha = t t , hb = a 2S 2S C , hc = . Cum c min {a, b}, atunci hc b c este cea mai lung nlime, prin urmare ha , hb , a at hc pot fi laturi ale unui triunghi dac i numai as
B

ci 35 5. Din () obinem: t 35 35 X X (xi yi + xi + yi + 1) 64 xi yi 19.


i=1 i=1

35 P

li

dac hc < ha + hb . ns a a hc < ha + hb 1 1 1 < + 2c > c a b


1 a

2 +

1 b

2 AB > P Q.

b Cum m(A) 90 , atunci b2 + c2 < a2 , iar a2 < a (b + c), prin urmare ptrat este strict pozitiv. Rmne c b c = 0, deci 4ABC este isosce a a a a a b b = 105 , m(B) = 30 . Se consider DE m a G94. Fie 4ABC cu m(A) \ lui [BC], D BC, E AB, [CF bisectoarea lui BCE, F AB, i CF DE, {G} = CE AI. S se arate c 4DF G este echilateral. a a Gabriel Mr s \) = Soluie. t Evident c m(BCF a A \ [ E m(F CE) = m(ECA) = 15 . Apoi, 4EBC G b este isoscel cu axa de simetrie ED i m(B) = s F \ \ 30 , deci m(BED) = m(DEC) = 60 , de I [ unde m(CEA) = 60 . Rezult c 4AEC a a D 4IEC (U.L.U), prin urmare EA = EI i B s AC = IC. n 4EAI isoscel, EG este bisectoare, deci va fi i median: AG s a 1 Aplicnd teorema bisectoarei n 4EBC, cum ED = BE (4DEB dr 2 b t m(B) = 30 ), obinem: EF EC 2 ED ED = = = . FB BC 2 BD BD Reciproca teoremei bisectoarei n 4EDB arat c DF este bisectoare, deci a a \ \ \ m(F DB) = 45 . Cum BDF ACB, atunci F D k AC. Punctul D est 1 lui [BC], prin urmare DF va fi linie mijlocie n 4BAC: DF = AC 2 b \ s AF = F B (3). n plus, m(BF D) = m(A) = 105 .Din (1) i (3) rezu 1 [ [ este linie mijlocie n 4ABI, deci F G = BI i m(AF G) = m(ABI) = 15 s 2

Pentru ultima echivalena, am folosit faptul cunoscut c segmentul ce se t a laturile neparalele ale unui trapez, paralel cu bazele i care trece prin p s intersecie al diagonalelor, are ca lungime media armonic a bazelor. t a b 90 , M mijlocul lui [BC], iar N G93. Fie 4ABC cu m(A) \ a contact al cercului nscris cu BC. Dac BAN M AC, s se arate c 4 a \ a isoscel. Doru Bu Soluie. Dreptele AN i AM fiind izogonale, putem utiliza relaia l t s t BN BM BN AB 2 c2 , de unde a s = = 2 . Pe de alt parte, BN = p b i C NC MC AC 2 NC b deci pb c2 = 2 b2 (a + c b) = c2 (a + b c) pc b 2 a b c2 + bc (b c) b3 c3 = 0 (b c) a (a + b) + bc b2 + bc + c2 = 0 (b c) a (b + c) b2 + c

I se afl pe mediatoarea lui [BC], deci BI = IC i am artat c IC = A a s a a 1 c F G = AC. Conform (2), urmeaz c F G = F D, adic 4F GD este a a a a 2 plus,

\ [ \ m(DF G) = 180 m(BF D) m(AF G) = 180 105 15 = 6 deci 4F GD este echilateral.

G95. Fie 4ABM dreptunghic n B. Fie C pe [M A] astfel nct M S se arate c n 4 ABC, bisectoarea AD, mediana BE si n limea a a at concurente. Dan Br Soluiea 1 (a autorului). Fie {K} = ADCF ; t A vom arta c punctele B, K, E sunt coliniare, de a a E unde rezult concluzia. Conform reciprocei teorea K mei lui Menelaus n 4ADC, coliniaritatea punctelor F C KA BC 0 B, K, E revine la = (1). Fie {D } = D KD BD AD BM ; atunci B D KA KA KD0 CA DD0 = = 1+ = KD KD0 KD CM KD BD0 AC BD0 D0 M AC 1+ = 1+ 0 = = AB KC AB D M KC AC AB AM AC AB AM = 1+ = 1+ = . AB AM AC AB AC AB Pe de alt parte, a

egalitii (1). at Soluia 2 (dat de Marius Tiba, elev, Iai). Notm {T } = EB C t a s a E EBAD. Cum CF k BM , cu teorema lui Thales n 4EBD obinem c t a T EU AE Din teorema bisectoarei n 4AEB avem c a = . ns EC = AE, C a UB AB i atunci T = U , de unde concluzia problemei. s

BC DC AC AM = 1+ = 1+ = , ceea ce ncheie j BD BD AB AB

A. Nivel liceal

L86. Cercurile de centre Ia , Ib , Ic exnscrise 4ABC sunt tangent [BC], [AC] respectiv [AB] n D, E, F . Bisectoarea interioar a unghiu a intersecteaz latura BC n M ; fie {P } = F E AM , iar Q F D, S D a definite. Ar tai c dreptele DP , EQ si F S sunt concurente. a t a Neculai Roman, Mirc Soluie. Cu notaiile uzuale, avem: BD = AE = p c, CD = AF t t BF = CE = p a. Teorema bisectoarei i teorema sinusurilor n 4 s cos C IA B MB 2 . Aplicm acum relaia (R1 ) din nota Rapoarte de a t = = MC Ia C cos B 2

de o cevian si o secant ntr-un triunghi din RecMat 1/2005; obinem: a a t AF M B AC P E PE AB M C AE =1 = AB M C AE P F PF AF M B AC cos B p c PE c PE c(p c) cos C cos2 B PE 2 2 2 = = = C PF p b cos 2 b PF b(p b) cos B cos2 C PF 2 2 cos A SD cos B P E QF SD QF 2 2 , , prin urmare Analog, = = =1 s QD P F QD SE cos C SE cos A 2 2 rezult din reciproca teoremei lui Ceva. a Not. Vlad Emanuel, elev, Sibiu, evalueaz rapoartele necesare p a a carea reciprocei teoremei lui Ceva prin considerente de arii. L87. S se arate c un tetraedru cu muchiile n progresie geometric a a a perechile de muchii opuse sunt perpendiculare, este regulat. Marius Olteanu, Rm Not. Ulterior publicrii n revista noastr, problema a aprut cu nr. a a a a G.M. 6/2005. Soluia ei poate fi gsit n G.M. 12/2005, pp. 640. t a a Soluie corect s-a primit de la Vlad Emanuel, elev, Sibiu. t a L88. Fie 4ABC de arie S, perimetru 2p, avnd razele cercurilor circ 8+3 3 3 nscris R, respectiv r. Not m k = a p + r. Fie A, B, C discuri 12 4 A, B, C si aceeai raz < r. Pentru punctele D A, E B, F C, s a 4DEF . S se arate c , dac = k, atunci |T S| < . a a a Dan Br Soluie. nlimea din A taie cercul frontier a t at a A lui A n A0 , A00 , iar paralelele la ea prin B, C taie A celelalte dou cercuri frontier B, C n B 0 , B 00 , C 0 , a a 00 0 00 0 00 0 00 0 00 C , notate aa nct A B B A i A C C A s fie s s a A paralelograme. nlimea din D a 4DEF , notat at a B hD , satisface hA 2 hD hA + 2. Avem i s c a 2 EF a + 2. nmulind aceste iruri a t s B de inegaliti ntre numere pozitive i notnd La = at s B a + hA , obinem: t (hA 2)(a 2) < 2T < (hA + 2)(a + 2) S La < T < S + La |S T | < La .

Rmne de demonstrat c La K; de fapt, ar trebui artat c u a a a a numerele La , Lb sau Lc (Lb , Lc construindu-se analog cu La ) poate fi Avem: 1 1 1 L = La + Lb + Lc = a + b + c + hA + hB + hC = 2p + +2S + + a b c 2S p2 + r2 + 4rR p = 2p + (ab + bc + ac) = 2p + = 2+ p+ 2+ abc 2R 2R Vom fi asigurai c putem face alegerea anunat dac artm c L t a t a a aa a p galitate la care se ajunge uor folosind cunoscutele 2p 3 3R ( s 2R

r 1 ). 2R 4 L89. Cte drumuri de la A la B exist , dac din orice a a C punct de pe traseu ne putem deplasa doar spre stnga sau F n jos? D Irina Mustaa, elev, Iai t a s Soluie. Notm cu an numrul drumurilor de la A t a a la B i ncercm s gsim o relaie de recurena pentru s a a a t t an . Notm cu bn1 numrul drumurilor de la E la B; B G a a numrul drumurilor de la C, F , respectiv D, pn la B va fi an1 , an2 a a bn2 . De la A la C putem ajunge n dou moduri, iar de la A la D put a ntr-un singur mod fr a trece prin C, prin urmare aa R 2r ( an = 2an1 + bn2 . bn1 = 2an1 + bn2 . Din (1) i (2) obinem s t (Acel 1 din final reprezint singurul drum posibil de la G la B.) Trecem a i scdem egalitile obinute; gsim: s a at t a an+1 an = 2an an1 an+1 = 3an an1 .

Cum din E putem pleca fie spre C, fie n jos, apoi stnga, spre D, atunci

an = 2n1 + bn2 = 2an1 + an2 + bn3 = . . . = 2an1 + an2 + . . . +

t t a Se verific uor c a1 = 2, a2 = 5, obinnd astfel o recurena liniar om a s a ordin II pentru an . Mai mult, se demonstreaz imediat prin inducie c an a t a unde Fk este al k-lea termen din irul lui Fibonacci. s Not. Soluie corect s-a primit de la Vlad Emanuel, elev, Sibiu. a t a L90. Fie n 1 un num natural. S se arate c mulimea A = {n+1, n ar a a t se poate partiiona n trei submulimi, fiecare cu aceeai sum a elementel t t s a numai dac n este multiplu de 3, n 6. a Marian Tetiv Soluie. Pentru ca A s poat fi partiionat n trei clase avnd ace t a a t a n(3n + 1) a elementelor, trebuie ca suma elementelor sale, care este , s fi a 2 de 3, ceea ce conduce la 3 | n. Pentru n = 3, evident c A = {4, 5, 6} n a partiiona cum cere enunul, deci n 6. t t Fie acum n = 3k, k 2; dac k este par, k = 2p, p 1, atunci A = a 6p + 2, . . . , 12p}. Scriem A = {6p + 1, 12p} {6p + 2, 12p 1} {9p, 9p + 1},

numrul de submulimi fiind 3p, fiecare cu aceeai sum a elementelor a t s a partiiei se obin reunind p dintre aceste submulimi (prima clas), apoi t t t a urm cele p care au rmas. a a Pentru k impar, k = 2m + 1, m 1, avem de partiionat mulimea A = t t 6m + 5, . . . , 12m + 6}. Remarcm c suma tuturor elementelor lui A este a a (9m + 5), iar suma elemntelor lui A care dau restul 2 la mprirea cu 3 at (6m + 5) + (6m + 8) + + (12m + 5) = (2m + 1)(9m + 5),

prin urmare o clas a partiiei poate fi construit din numerele care da a t a mprirea cu 3; fie P aceast mulime. Mai notm cu M mulimea elem at a t a t A ce dau restul 1 la mprirea cu 3 i cu N mulimea elementelor lui at s t multipli de 3. Suma elementelor lui M este (2m + 1)(9m + 5) (2m + 1) elementelor lui N este (2m+1)(9m+5)+(2m+1); vom ncerca s schimb a a elemente din N cu elemente din M astfel nct s egalizm sumele. nti a a schimbarea 12m + 1 12m + 6, care crete suma elementelor lui M cu s efectum m 2 schimbri, fiecare mrind suma elementelor lui M cu 2 (i a a a s corespunztor suma elementelor lui N ): a 12m 2 12m; 12m 5 12m 3; . . . ; 9m + 7 9m + 9. Cu aceasta, soluia problemei este ncheiat. t a Not. O frumoas soluie prin inducie matematic s-a primit de a a t t a Emanuel, elev, Sibiu. L91. Punctele planului se coloreaz n 3 culori astfel nct fiecare a fie folosit cel puin o dat . Spunem despre un triunghi c este aproape a t a a dac m surile unghiurilor sale sunt cel mult 60, 001 . Ar tai c exist u a a a t a a aproape echilateral cu vrfurile de culori diferite. Adrian Zahariuc, ele Soluie. S numim albastru, rou i verde cele 3 culori. Vom scrie M t a s s sau M (v) dup cum este punctul M albastru, rou sau verde. Fie A(a), B a s i D1 un disc de centru O1 i raz r > 0 astfel nct A, B, C D1 . Fie d > s s a Fie D discul de centru O1 i raz d + r. s a S lum mai nti cazul cnd mulimea de puncte D nu este monocroma a a t am notat cu T complementara mulimii T (faa de plan). Fie P, Q D t t diferite. Este clar c exist o niruire finit de puncte P = M0 , M1 , . . . a a s a s astfel nct [Mi Mi+1 ] D i |Mi Mi+1 | 2r, i = 0, n 1. Cum M0 i Mn s diferite, rezult c exist i {0, 1, 2, . . . , n 1} astfel nct Mi i Mi+1 a a a s diferite. n concluzie, exist T, S D cu culori diferite, s zicem T (a), a a s nct [T S] D i |T S| 2r. Fie D2 discul de raz r i centru O2 , und a s mijlocul lui [T S]. Este clar c T, S D2 i c |O1 O2 | d. Fie un punct a s a nct triunghiul O1 O2 O3 s fie echilateral. S demonstrm acum urmtoa a a a a a Lem. Fie O1 , O2 si O3 vrfurile unui triunghi echilateral de latur l a si D1 , D2 si D3 , 3 discuri de raz r si centre O1 , O2 , respectiv O3 . Dac a Y D2 si Z D3 , atunci m surile unghiurilor 4XY Z sunt cel mult 60, a Demonstraie. Avem XY XO1 + O1 O2 + O2 Y l + 2r i O2 O3 t s Y Z + ZO3 Y Z + 2r, sau Y Z l 2r. Atunci XY l + 2r 1000002 sin Z < 1, 00001 < 1, 00001. YZ l 2r 999998 sin X Relaiile analoage se demonstreaz ntr-o manier asemntoare etc. t a a a a

S revenim la problem. Fie D3 discul de centru O3 i raz r. Fie a a s a oarecare. Dac W este albastru, conform lemei, triunghiul CSW este apr a lateral. Dac W este rou, triunghiul CT W este aproape echilateral. n fin a s este verde, triunghiul ASW este aproape echilateral i demonstraia pen s t caz se ncheie.

S lum acum cazul cnd mulimea de puncte D este monocromatic, s a a t a toate punctele ei sunt albastre. Fie L = {|RV | : R(r), V (v) R2 }. Din pre c toate punctele din exteriorul discului D sunt albastre, rezult c mulim a a a t mrginit. Atunci exist l L astfel nct 1, 000001l > l0 pentru orice l a a a punctele R1 (r), V1 (v) astfel nct |R1 V1 | = l. Fie punctul Q astfel nc QV1 = 1, 000001l. Dac Q este rou, OV1 = 1, 000001l contrazice aleg a s Analog, Q verde ne conduce la o contradicie. Rezult c Q este albastru. t a a imediat c triunghiul QR1 V1 este aproape echilateral i problema este rez a s

Gheorghe Molea, Curtea Soluie. Considerm o ordonare a numerelor ai (1, ); putem presu t a a restrnge generalitatea c a1 a2 an . Atunci ap ap a 1 2 amp amp amp , m > p 1. Aplicm inegalitatea lui C a n 1 2 inegalitatea mediilor:

L92. Dac ai (1, ), i = 1, n, n 3, iar m, p N cu m > p a demonstreze inegalitatea am + + am am + am + am 2 n 1 3 n + + loga1 mp mp + loga2 mp a2 + + an a1 + amp + amp n 3 am + + am 1 n1 + logan mp np. a1 + + amp n1

am + am + + am amp ap + amp ap + + amp ap 2 3 n n n 2 3 3 = 2 n1 n1 amp + amp + + amp ap + ap + + ap n n 3 3 2 2 n1 n1 q mp amp + a3 + + amp n1 p p n a2 a3 ap 2 n n1 q m m m a2 + a3 + + an p n1 (a2 a3 an ) . np mp a2 + a3 + + amp n Dac notm cu S membrul stng al inegalitii din enun, folosind aceast a a at t a i analoagele obinem: s t p S loga1 (a2 a3 an ) + loga2 (a1 a3 an ) + logan (a1 a2 an n1 p X logai aj + logaj ai . S n1
i6=j

a Cum logai aj > 0, atunci logai aj + logaj ai 2. Numrul grupelor n(n 1) (logai aj + logaj ai ) este i concluzia problemei urmeaz imediat s a 2 Not. Pentru n = 3, m = 4, p = 3, obinem problema 22814 din G.M. a t Pentru n = 3, m = 2, p = 1 obinem problema C:1605 din G.M. 11/1994. t Principial aceeai soluie a dat Vlad Emanuel, elev, Sibiu. s t

L93. Exist vreun polinom f (X, Y ) n dou nedeterminate astfel nc a a {f (m, n) | m, n Z} N = x2004 | k 1 , k

unde xn =

Gabriel Dospines Soluie. Rspunsul este afirmativ! De exemplu, putem considera f t a s X 2004 (2 (X 2 2Y 2 )2 ) i atunci, pentru m, n Z, f (m, n) > 0 2 > (m2 2n2 )2 |m2 2n2 | = 1. Obinem astfel dou ecuaii Pell; rezolvndu-le, obinem c, de fapt, t a t t a ceea ce arat c {f (m, n) | m, n Z} N = {x2004 | k 1}. a a k {m2004 | exist n a.. |m2 2n2 | = 1} = {x2004 | k 1}, a k

n n 1+ 2 + 1 2 ? 2

L94. Fie A Mn(C) o matrice cu coeficieni ntregi, inversabil si a t a mulimea Ak | k N este finit . S se demonstreze c aceast mulime a t a a a a t 2 3n elemente. R mne rezultatul adev rat dac suprim m condiia ca a a a a t matricei s fie ntregi? a Gabriel Dospines Soluie. Din ipotez rezult c exist p 1 astfel nct Ap = In . S t a a a a a 2 mai mic cu aceast proprietate i s observm c dac p 3n atunci am a s a a a a prima parte a problemei. S presupunem deci contrariul i s considerm a s a a din mulimea H = {I, A, . . . , Ap1 }. Ele sunt n mod evident diferite or t i, mai mult, fiecare element al lui H are proprietatea c X p = In . S ob s a a 2 din faptul c H are cel puin 3n + 1 elemente, exist dou elemente n a t a a diferena este o matrice cu toate elementele multipli de 3 (principiul lui t 2 observnd c exist 3n matrice cu elemente 0, 1 sau 2). Deci exist B a a a 1 i < j p 1 astfel nct Aj Ai = 3B. Avem atunci Aj1 In = Notnd s = j i, X = BAp1 avem X M3 (Z) i (In + 3X)p = In . Fie s 2 p valorile proprii ale lui X. Avem (1 + 3i ) = 1 de unde |i | ; atunci 3 n m P m i n 2 < 1 de la un rang m0 . Tinnd cont c tr X m este ntre a 3 i=1 n P m i = 0 de la acel rang. Din for c de la acel rang el este 0 i deci a s
i=1

a Newton rezult imediat c 1 = = m =0. Pe de alt parte, din (In + a a obinem X n + p 32 X + + p X p1 = On . Din faptul c i = 0 t 3pI 3 a 2 n det 3pIn + p 32 X + + 3p X p1 = (3p) 6= deci matricea 3pIn + 0, 2 + 3p X p1 este inversabil i cum X 3pIn + p 32 X + + 3p X p1 = as 2 n mod necesar X = On , adic Aj1 = In , contradicie cu minimalitatea a t justificat astfel prima parte a problemei. Pentru a doua parte, mai mult o glum, raspunsul este evident nu. Est a s lum o matrice diagonal ce are ca elemente pe diagonal o rdcin pr a a a a a a a 2 ordinul 3n + 2 a unitii. at

L95. Fie f : R R periodic , m rginit si astfel nct exist x0 a a a a care ls (x0 ), ld (x0 ) exist , sunt finite si distincte. Determinai a R pent a t x R exist lim (f (t) + a) dt. a
x 0

Paul Georgescu i Gabriel P s

x R Soluie. Fie T R o perioad a lui f i notm F (x) = (f (t) + a)d t a s a + 0 Z x+(n+1)T Z T F (x + (n + 1) T ) F (x + nT ) = (f (t) + a) dt = f (t) dt x+nT 0 Z ! T

F (x + nT ) = F (x) + n

f (t) dt + aT

n N ,

x [0,

T R Cum f este mrginit, rezult c F este mrginit pe [0, T ], iar dac f (t a a a a a a a 0 ! T R f (x) dt + aT . 0, rezult imediat c lim F (x) = (+) sgn a a x 0 T 1R f (t) dt; atunci Fie acum a = T 0 Z x+T (f (t) + a) dt = F (x) , F (x + T ) = 0

x R,

deci F este periodic de perioa T . Presupunem prin absurd c F este a a a x R2 Atunci F (x1 ) = F (x2 ), x1 , x2 R, deci (f (t) + a) dt = 0, x1 , x2
x1 x1

pentru x0 x1 < x2 < x0 , deci ls + a 0 ls + + a. Cum er obinem c ls (x0 ) + a = 0. Analog, ld (x0 ) + a = 0, deci ls (x0 ) = ld (x t a contradicie cu ipoteza. Rmne c F este periodic i neconstant i d t a a as as T R limit la + n cazul n care f (t) dt + aT = 0. a
0

> 0 fixat, putem gsi > 0 astfel ca a Z x2 (ls + a) (x2 x1 ) (f (t) + a) dt (ls + + a) (x2 x1

Rezolvarea aritmogrifului de la pag. 129.


S C H U A R V I R E I F T C E H E C R E I R S O S E
ARITMOGRIF

P F A F F

S T O L Z

M W G I I * M A R L U O E S N S S

M K H A L H S F A I R E O T E M A T I C U I E C I N R G L O N M L B

Probleme propuse1
Clasele primare
P.114. n piramida alturat unele numere s-au ters de-a a a s lungul timpului. Putem s le punem la loc? a (Clasa I ) Ionela Brgan, elev, Iai a a a s
3 8

20 1

P.115. Dac din prima lad iau 2 mere i le pun n lada a doua, din a a a s iau 3 mere i le pun n lada a treia, iar din lada a treia iau 4 mere i le pu s s lad, atunci n fiecare lad voi avea cte 34 mere. Cte mere au fost n fie a a (Clasa I ) Mariana Nastasia, e

P.116. Luni, mama pune ntr-un co cteva mere. Joi, ea gsete n c s a s de mere. Cte mere a pus mama n co, tiind c, n fiecare zi din acea s s s a Mihai, cel mai mare dintre frai, mparte frailor mai mici cu un mr mai t t a ziua precedent i c joi el mparte 5 mere? Cte mere mai rmn n co as a a s dup mprirea merelor? a at (Clasa II-a) Inst. Maria R

P.117. n exerciiul 1111111 = fiecare csua poate fi nlocu t a t sau . Ct poate fi rezultatul acestui exerciiu? t (Clasa II-a) Diana Tnsoaie, e a a

P.118. Calculai a + b + c + d, tiind c a b = 5 i c d = 15. G t s a s a posibilitile. at (Clasa III-a) nv. Rica Buct a

P.119. Produsul a 10 numere naturale este 40. Aflai cea mai mic t a mare valoare a sumei celor 10 numere. (Clasa III-a) nv. Mirela Buburuzanu, Tome

P.120. Se consider numerele 1, 2, 3, 4, 5, . . . , 49. Care este cel a numr de numere pe care putem s alegem dintre acestea astfel nct sum a a trei dintre ele s se mpart exact la 9. a a (Clasa III-a) nv. Felicia-Petronela Leanu, Ceplen

P.121. Ceasul lui Andrei o ia nainte cu 20 secunde pe or. El a potr a luni la ora 8 i a citit din nou ceasul lunea urmtoare la aceeai or. S s a s a aceast durat ceasul nu a funcionat permanent, iar la ultima citire ar a a t a 50 min, s se afle ct nu a funcionat ceasul. a t (Clasa IV-a) Paula Boranu, e s

P.122. La Concursul de matematic "F.T.Cmpan", etapa judeean, a t a pat 100 elevi de clasa a IV-a, care au avut de rezolvat 3 probleme. Dac 7 a rezolvat bine prima problem, 69 a doua problem i 64 a treia problem, a as a c mcar 3 elevi au rezolvat corect toate cele trei probleme. a a (Clasa IV-a) Anca Cornea, e

P.123. ntr-o cutie sunt 34 bile, din care unele cntresc cu 1 g mai m a fiecare bil cntrete un numr natural de grame, iar masa tuturor bilelor a a s a
1

Se primesc solu ii pn la data de 1 iunie 2007. t a

s se afle cte bile sunt mai grele. a (Clasa IV-a)

Petru Asa

Clasa a V-a
V.71. Comparai numerele 3300003 i 2450004 . t s

Lucian Tuescu t

V.72. Fie mulimile A, B astfel nct A B, |P (A)| 60, |P (B)| t determine |A| i |B|. (Prin |X| am notat cardinalul mulimii X.) s t Petru Asa

V.73. S se scrie numrul 20062005 ca o sum de ase ptrate perfecte a a a s a Ionel Nech

V.74. Se consider irul de numere naturale 1, 1, 2, 5, 12, 27, 58, . . . . as suma primilor 100 de termeni ai irului. s Marius Damia V.75. Fie x, k N, x 2, k < x. S se arate c a a (x 1) 12 . . . k (x) + k + 1 = 11 {z . 1 (x) . | .. }
k+1 cifre

Doru Bu

Clasa a VI-a

VI.71. Fie x, y, z N astfel nct 100x 2006y 2 + 15z = 0. S s a . . 85. y (x + z) . Dan Nedeianu, Dr.-Tr

VI.72. Fie p N un numr prim. S se determine x, y N a a a p y + N . x2 x D. M. Btineu-Giurgiu, B a t 2 An 7n 7n mulimea M = n N | t N . 9


2n cifre

a VI.73. Fie An = 14 + 1414 + 141414 + + 1414 . . . 14, n N . S se | {z }

Valeriu Braoveanu s

VI.74. Considerm dou axe perpendiculare Ox i Oy, precum i cel a a s s sectoare ale unghiurilor drepte care se formeaz. Fie A oarecare, iar B, C a sale faa de prima bisectoare, respectiv faa de Ox. Rotim segmentul [OC t t lui O cu 90 n sensul acelor de ceasornic i notm cu D extremitatea se s a rotit. S se arate c: a a a) B, O, D sunt coliniare i O este mijlocul lui [BD]; s b) D este simetricul lui A faa de a doua bisectoare. t Adrian Cordune

VI.75. Fie ABCD un patrulater convex, O intersecia diagonalelor, M t lui [AB], iar N mijlocul lui [OD]. S se arate c 2PBCN M < PBDC + PAB a a Bogdan Posa i Marius Drgoi, elev s a

Clasa a VII-a

VII.71. Fie 4ABC, AB < AC i D (AC). Fie AE bisectoarea s E (BD), F mijlocul lui [AD], {O} = AE BF , {G} = DO AB. S s a GD k BC AB = CD. Carmen Daniela Tama VII.72. Fie ABCD paralelogram, E (CD), {M } = AE BD, {N AC, {O} = AC BD. S se arate c AMEN = 2AMON . a a Mirela M VII.73. Fie a < b c razele a trei cercuri tangente ntre ele i tan s 1 1 1 aceeai dreapt n trei puncte distincte. S se arate c = + . s a a a a c b Dan Radu, B VII.74. Fie M mulimea multiplilor lui 36 n a cror scriere n baza 1 t a alte cifre dect 4, 6 sau 9. Cte numere cel mult egale cu 100 000 conine t Gabriel P VII.75. Fie m 3 un numr natural impar i a1 , a2 , . . . , am Z astfe a s

|a1 a2 | = |a2 a3 | = = |am1 am | = |am a1 | . Demonstrai c a1 + a2 + + am se divide cu m. t a Maria Mihe, T t

Clasa a VIII-a

VIII.71. Pe planul 4ABC se ridic perpendiculara AM . Fie P proi a pe planul (M BC), iar E, F proieciile punctului P pe M B, respectiv M t \ arate c M EF M CB. a \ Otilia Neme, Ocn s VIII.72. Fie a, b, c numere reale distincte. S se afle partea ntreag a a a a2 + bc b2 + ca c2 + ab A= + + . (a b) (a c) (b c) (b a) (c a) (c b) Mihail Bencze VIII.73. S se demonstreze c a N este ipotenuz a unui triunghi dr a a a cu laturile exprimate prin numere naturale dac i numai dac exist n as a a s a nct a2 n i a2 + n sunt ptrate perfecte. Ctlin Cali a a xy : xy VIII.74. Fie (0, 1]; s se arate c + a a 1 xy 1 xy fiecare din situaiile: t x+ . a) x, y [0, ); b) < x 0 y < 1 + x Gheorghe Costo 2n+1 + VIII.75. Determinai valorile lui n N pentru care fracia t t 3 2n + ductibil. a Gheorghe Iu

Clasa a IX-a

IX.71. Fie a < b numere reale; s se determine x, y, z R pentru care a

= 2yz + a, iar

Dan Pl IX.73. Fie a1 , a2 , . . . , an (0, ) cu a1 a2 an = 1. Pentru orice m arate c are loc inegalitatea a m1 + + am1 . am + am + + am am1 + a2 1 2 n n 1 Marius Tiba, IX.74. Fie a, b, c lungimile laturilor 4ABC i m, n (0, +). C s punctele A0 , B 0 , C 0 astfel nct C (AA0 ), A (BB 0 ), B (CC 0 ) i CA0 s AB 0 = mb + n, BC 0 = mc + n. S se arate c 4ABC i 4A0 B 0 C 0 au ace a a s de greutate dac i numai dac 4ABC este echilateral. as a Dumitru Mihalach \ IX.75. Fie ABCD patrulater inscriptibil, {O} = AC BD, m(AO MA NC O i punctele M (AB), N (CD). Notm k = s a , r = , p= MB ND O 2 1 \ p 2k, , , s se arate c m(M ON ) 6= 90 . a a r 2r Mihai Ha

Ce egalitate se obine pentru x, y, z (, 1]? t

Andrei Nedelcu i Lucian Ldu s a x y z IX.72. Fie x, y, z [1, +) aa nct s = = . S se arate c a a [x] [y] [z] q q p 2 2 2 2 2 2 [x] + [y] + [z] + {x} + {y} + {z} = x2 + y 2 + z 2 .

x a + b x = y.

Clasa a X-a

X.71. Fie a, b, c lungimile laturilor unui triunghi. S se arate c a a 4 4 4 a + 2 bc b + 2 ac c + 2 ab + > 3. + a+ c b+ c a+ b Lucian Tuescu t X.72. S se rezolve sistemul a x2 log2 15 + y 2 log3 10 + z 2 log5 6 = 2 (xy + yz + zx) , x + y + z = 5. Marius Damia X.73. Determinai funciile f : N cu proprietatea c pentru oric t t R a f (x) + f (y) x+y = . astfel nct 3 | x + y, are loc egalitatea f 3 3 Eugenia Rou, e s X.74. Fie d1 , d2 dou drepte perpendiculare i l1 , l2 dreptele suport a s toarelor celor dou perechi de unghiuri opuse formate de ele. Determina a cu A d1 , B d2 , C l1 i O, H l2 (O, H cu semnicaiile uzuale). s t Temistocle B X.75. a) Fie C . Dac z1 , z2 , z3 sunt trei numere complexe d a fel nct Re (1 ) = Re (2 ) = Re (3 ), atunci punctele de afixe z1 , z z z z coliniare. Notm cu d dreapta pe care sunt situate aceste puncte. a

b) Dac z1 , z2 , z3 sunt trei numere complexe diferite cu proprietatea c I a 0 0 0 a 0 Im (2 ) = Im (3 ), atunci punctele cu afixele z1 , z2 , z3 sunt colinia z z0 dreapta ce le conine. t c) S se arate c dreptele d i d0 sunt perpendiculare. a a s Constantin Co

Clasa a XI-a
XI.71. Fie irul (xn )n1 , unde x1 > 1, xn+1 = s
n

lim xn . n

xn 1 , n N . ln xn

Dan Popescu

XI.72. Este posibil ca o funcie f : R R, care veric 1+f (x)+f (x) f t a x R, s fie continu pe R? a a Dorin Mrgidanu, a

XI.73. Fie f : [0, 1] R o funcie nenul de clas C k+1 pe [0, 1], k t a a ca f (0) = f (1) = 0. Dac pentru orice 1 j k exist aj {0, 1} a a a f (j) (aj ) = 0, atunci exist x1 , x2 (0, 1) astfel ca f (k+1) (x1 ) f (k+1) (x2 ) a Gheorghe Moroanu i Paul Georg s s

XI.75. Fie A, B M2 (C). S se arate c dac AB BA comut cu a a a a atunci AB = BA. Dorel Mihe, T t

XI.74. Fie a, b, c R, a 6= 0, n 2N . Urm atoarele afirmaii sunt ech t (1) b2 4ac 0, (2) det aA2 + bA + cIn 0, A Mn (R). Marian Ursrescu a

Clasa a XII-a

XII.72. Fie funcia f : [0, 1] R, indefinit derivabil pe [0, 1], cu pr t a c exist M > 0 astfel nct f (n) (x) M , x [0, 1] i n N. Artai a a s a t a) Pp k+1 (n + p) (n + p 1) (n + p k + 2) f (n+pk) (1) k=1 (1) lim =0 p+1 n (1) (n + p) (n + p 1) (n + 1) b)
p

XII.71. Fie f : [a, b] R o funcie derivabil cu derivata continu. S t a a Rb c lim a f (x) sin nx dx = 0. a n Dan Radu, B

lim

Pp

k=1

(1)k+1 (n + p) (n + p 1) (n + p k + 2) f (n+pk) ( (1)


p+1

1 0

(n + p) (n + p 1) (n + 1) xn f (n) (x) dx, n N.

Ovidiu Pop, Sa

XII.73. S se arate c a a Z 1 Z 1 n (1 + x) + (1 x)n 1 ln (1 + x) ln dx = n dx = n ln 2, n 1 + x2 1 + xn 1 + x2 8 0 0

S se calculeze apoi a

Gabriel Necula XII.74. Fie (G, ) un grup comutativ cu proprietatea c exist n N a a a din xn = y n rezult x = y, cux, y G. Dac f , g sunt dou endomo a a a lui G, atunci ecuaia f (x) = g x1 are soluie unic dac i numai da t t a as h : G G, h (x) = f (xn ) g (xn )este injectiv. a D. M. Btineu-Giurgiu, B a t XII.75. Fie A Mn (C) de rang r i S = {B Mn (C) | ABA = On s c S este subspaiu vectorial n Mn (C) i c dim S = n2 r2 . a t s a Adrian Reisn

/4 R 0

ln

1 + tg3 x dx. 1 + 3 tg2 x

Premiu pe anul 2006 acordat de ASOCIA TIA RECREA MATEMATI TII


Se acord un premiu n bani n valoare de 100 lei elevului a

ZAHARIUC Adrian Colegiul Naional "Ferdinand I", Bac t

pentru nota O problem despre suma cifrelor unui num natural n ba a ar meraie oarecare aprut n acest numr la pagina 113. t a a a Acest premiu este acordat de Asociaia "Recreaii Matematice" ca t t contractului de sponsorizare cu Fundaia Cultural "Poiana" (directo t a Tiba).

Premii pe anul 2006 acordate unor elevi din Republica Moldova participani la faza fi t a ONM, Iai, 2006 s
Se acord cte un premiu n valoare de 100 lei elevilor a GUZUN Ion BOREICO Iurie din lotul Republicii Moldova, care au obinut cele mai bune rezultate. t

Premiile sunt acordate de Asociaia "Recreaii Matematice", ia t t bani provine din contractul de sponsorizare cu Fundaia Cultural t a (director d-l Dan Tiba).

Probleme pentru pregtirea concursurilo a


A. Nivel gimnazial

Zdravko Starc, Vrac, Serbia i Mu s s s G113. Fie segmentul [AB] de mijloc O i semicercurile C1 i C2 de diam s respectiv [AO] situate n acelai semiplan faa de AB. Perpendiculara n s t pe AB intersecteaz C1 n E i C2 n D. Dac AD C1 \ {F }, s se arate c a s a a tangent cercului circumscris 4DEF . a Alexandru Negrescu, elev, \ G114. Fie ABCD un paralelogram care nu este romb cu m(BAD) = M, N (AC), P (BC) i Q (CD) sunt astfel nct [BM , [DN , [A s \ \ \s \ sunt bisectoarele unghiurilor ABC, ADC, BAC i respectiv DAC, atunc perpendicular pe N Q. a Andrei Ned G115. Fie ptratul M N P Q nscris n ptratul ABCD, M (AB), N a a P (CD), Q (BC) i fie {E} = P N AB, {F } = P Q AB. Notm s a S3 ariile ptratului ABCD, ptratului M N P Q, respectiv 4P EF . S se a a a

G106. Fie m N \ {0, 1} i k N, k m. Pentru fiecare x N, c s propoziiile: x > 1; x > 2; . . . ; x > m. Aflai x N pentru care k d t t propoziii sunt adevrate, iar celelalte m k sunt false. t a Maria Mihe, T t G107. Mulimea A N de cardinal n N are proprietatea c, or t a patru elemente ale sale, putem alege dou cu suma 22006 + 1. Aflai valoare a t a lui n. Dan Nedeianu, Dr. Tr G108. Fie m, n N . S se arate c mulimea numerelor ntregi de a a t mult egal cu n poate fi partiionat n m submulimi cu aceeai sum a el t a t s a dac i numai dac n + 1 m. as a Marian Tetiv G109. La un concurs se dau ase probleme evaluate cu 1, 2, 3, 4, 5 s 6 puncte. Dac un elev nu rezolv o problem, primete 1 punct; dac a a a s a primete punctajul corespunztor. Fiecare elev obine mcar 11 puncte. S s a t a c o problem a fost rezolvat de cel puin o treime dintre elevi. a a a t Gabriel Dospinescu, stude G110. Fie mulimile A = {k + n | k Z, n N} i B = (0, 1/10). t s A B este infinit. a Petru Asa G111. Fie 0 < a < b numere reale date i x,y [a, b]. Dac s = x + a s ab s2 + ab s se afle maximul expresiei E = p + a . p Vlad Emanuel, el G112. Dac a, b, c sunt lungimile laturilor triunghiului ABC, atunci a q q q (a + b)2 c2 + (b + c)2 a2 + (c + a)2 b2 < 2 (a + b + c)

p a) S1 S2 = 4 SAEN SBF Q ;

b) S3 S1 ;

1 1 1 = . S3 S3 S3 Claudiu Stefan P c)

B. Nivel liceal

L106. Fie I centrul cercului nscris n 4ABC. Dreptele AI, BI i s secteaz a doua oar cercurile circumscrise triunghiului BCI, CAI i ABI a a s respectiv C 0 . Dac notm cu |XY Z| perimetrul 4XY Z, s se demonstre a a a CA AB BC + + = 1. |BCA0 | |CAB 0 | |ABC 0 | Titu Zvonaru, C a L107. Fie M1 N dou puncte situate n interiorul 4ABC, avnd dista la laturile AB, BC, CA egale cu 3, 2, 7, respectiv 9 , 5, 5 . Dac raz a 2 2 circumscris 4ABC este R = 8, s se calculeze M N . a Vlad Emanuel, el L108. S se arate c n orice 4ABC are loc inegalitatea a a 3A 3 3 (sin A + sin B + sin C) 4 3 sin2 . 2 12 Marian Tetiv L109. Se dau numerele reale pozitive subunitare a1 , a2 , . . . ,a2n2 n , n S se demonstreze inegalitatea (sumarea se face prin permutri circulare) a a 2n1 X 2n 1 a1 < . 2n + 1 a2n1 + a2n1 + . . . + a2n1 + 2n + 1 2 3 2n2 n

Ioan Serdean L110. Fie a, b, c (0, ) i n, k N. Demonstrai identitatea s t

bn+k cn+k 4n(a b)2 an+k + n + n ak + bk + ck + . bn c a k(a2k + b2k + c2k )

(n legtur cu o problem propus la OBM 2005). a a a a Titu Zvonaru, Comneti and Bogdan Ionia, B a s t L111. Se dau m numere naturale distincte din mulimea {1, 2, . . ., n}. S t c putem alege cteva dintre ele, cu suma S, astfel nct a m(m + 1) 0S n + 2n m. 2 Adrian Zahariuc, ele L112. Pentru n N, notm cu a(n) numrul modurilor n care n se p a a ca sum a unui numr par de puteri ale lui 2 i cu b(n) numrul moduri a a s a n se poate scrie ca sum a unui numr impar de puteri ale lui 2. S s a a a a(n) = b(n), n 2. Adrian Zahariuc, ele L113. Determinai numerele reale a, b pentru care mulimea A = t t n N } este finit. a Gheorghe Iu

L114. Considerm o parabol i dou drepte secante parabolei, paralel a as a dar neparalele cu axa de simetrie a parabolei. Folosind doar rigla negrad construiasc tangenta la parabol care este paralel cu dreptele date. a a a Titu Zvonaru, C L115. Determinai P R[X], grad P 2, astfel nct funcia f t t f (x) = p({x}) + {p(x)} s fie periodic (unde p este funcia polinomial a a t a P , iar {} desemneaz partea fracionar). a t a Paul Georgescu and Gabriel P

Training problems for mathematical contes


Junior highschool level

G106. Let m N\ {0, 1} and k N, k m. For any x N, co assertions: x > 1; x > 2; . . . ; x > m. Find x N such that k out assertions are true while the other m k are false. Maria Mihe, T t G107. The set A N of cardinal number n N has the property that four of its elements, we can choose two elements whose sum is 22006 + 1. maximum value of n. Dan Nedeianu, Drobeta-Turnu G108. Let m, n N . Show that the set of integer numbers of abso at most equal to n can be partitioned into m subsets with the same su elements if and only if n + 1 m. Marian Tetiv G109. Six problems are proposed for a school contest and they are re evaluated with 1, 2, 3, 4, 5, 6 points. If a schoolchild does not solve a p receives 1 point for it; if he (she) solves it he gets the corresponding poin schoolchild receives at least 11 points. Show that a problem was solved b one third of the participating schoolchildren. Gabriel Dospines G110. Let us consider the sets A = {k + n | k Z, n N} and B = Show that A B is infinite. Petru Asa G111. Let 0 < a < b be two given real numbers and x, y [a, b] . If ab(s 2 + p = x y, determine the maximum value of the expression E = p + p Vlad Emanuel, highschool stude G112. Let a, b, c be the sides of the triangle ABC. Prove that p p p (a + b)2 c2 + (b + c)2 a2 + (c + a)2 b2 < 2(a + b + c)

Zdravko Starc, Vrac, Serbia and Mo G113. Let [AB] be a line segment of midpoint O and let C1 and halfcircles of diameters [ AB], respectively [ AO] situated in the same half

respect to line AB. The perpendicular line at C (AB) on AB meets C1 C2 at D. If AD C1 = {F }, show that AE is tangent to the circle circum DEF . Alexandru Negrescu, highschool student,

G114. Let ABCD be a parallelogram that is not a rhomb with m 60 . If M, N (AC), P (BC) and Q (CD) are such that [B \ \ \ [AP and [AQ are the bisectrix lines of the angles ABC, ADC, BAC a \ tively DAC then M P is orthogonal on N Q. Andrei Ned G115. Let M N P Q be the square inscribed in the square ABCD, with M N (AD), P (CD) and Q (BC) and let {E } = P N AB, {F } = We denote by S1 , S2 , S3 , the areas of the square ABCD, of the square M N of P EF , respectively. Show that: p 1 1 1 a) S1 S2 = 4 SAEN SBF Q ; b) S3 S1 ; c) = . S3 S3 S3 Claudiu- tefan P S

Highschool level

L106. Let I be the centre of the circle inscribed in the arbitrary trian The straight lines AI, BI, CI meet for the second time the circles circu to the triangles BCI, CAI and ABI at the points A0 , B 0 and respectively denote by |XY Z| the perimeter of the triangle XY Z, show that BC CA AB + + = 1. |BCA0 | |CAB 0 | |ABC 0 | Titu Zvonaru, C L107. Let M , N be two points situated in the interior of ABC

distances up to the sides AB, BC, CA are respectively equal to 3, 2, 7 an If the radius of the circumscribed circle to ABC is R = 8, determine th from M to N . Vlad Emanuel, highschool stude L108. Show that the following inequality holds in any ABC : 3A 3 3 (sin A + sin B + sin C) 4 3 sin2 . 2 12 Marian Tetiv L109. The real positive and less than 1 numbers a1 , a2 , . . . ,a2n2 n , n are given. Prove the following inequality (with summation being perform cular permutations): X 2n 1 a2n1 1 < . 2n1 2n1 2n1 2n + 1 a2 + a3 + . . . + a2n2 n + 2n + 1

Ioan Serdean L110. Let a, b, c (0, ) and n, k N. Prove the inequality bn+k cn+k 4n(a b)2 an+k + n + n ak + bk + ck + . bn c a k(a2k + b2k + c2k )

(A problem connected with another problem proposed at OBM 2005). Titu Zvonaru, Comneti and Bogdan Ionia, B a s t L111. We are given m distinct natural numbers contained in the set {1 Show that it is possible to select a couple of them, with their sum equal m(m + 1) that 0 S n + 2n m. 2 Adrian Zahariuc, highschool studen

L112. For n N, we denote a(n) the number of ways under which th n can be written as a sum of an even number of powers of 2 and let b( the number of ways under which n can be written as the sum of an odd powers of 2. Show that a(n) = b(n), n 2. Adrian Zahariuc, highschool studen L113. Determine the real numbers a, b such that the set A = {an + bn is finite. Gheorghe Iu L114. We consider a parabola and two parallel straight lines that parabola but are not parallel to its axis of symmetry. Using a non-meas build the tangent to the parabola which is parallel to the given lines. Titu Zvonaru, C L115. Determine the polynomials P R[X] of degree 2 so that th f : R R, f (x) = p({x}) + {p(x)} be periodical (where p is the polynomia corresponding to P , and {} denotes the franctional part function). Paul Georgescu and Gabriel P

ASOCIA TIA RECREA MATEMATIC TII

La data de 14.02.2005 a luat ina ASOCIATIA RECREATII t MATICE, cu sediul n Iai (str. Aurora, nr. 3, sc. D, ap. 6), avnd ca s jinirea activit tilor de matematic specice nv t mntului preuniversita a a aa zarea si desf surarea de activit ti care s contribuie la dezvoltarea gustu a a a matematic n rndurile elevilor, profesorilor si iubitorilor de matematic a a larea preocup rilor si cercet torilor originale. a a

Obiectivele majore pentru atingerea scopului propus sunt: 1. editarea unei reviste destinat elevilor i profesorilor revista " a s Matematice"; 2. fondarea unei biblioteci de matematic elementar biblioteca " a a Matematice"; 3. alctuirea unei colecii de cri de matematic elementar, cri d a t at a a at i aate la prima apariie Colecia "Recreaii Matematice". s t t t

Poate deveni membru al Asociaiei, printr-o simpl completare a une t a orice perosan care ader la obiectivele acesteia si sprijin realizarea lor. a a a

Pagina rezolvitorilor

BRA OV S Liceul "N. Titulescu". Clasa a X-a. ANTONACHE Alexandru: VIII IX(61,70), X.67; BARBU Cosmin: VIII(61-64), IX(61,70), X.69; BAR VIII(61-64), IX(61,70), X.69; BRNZEA Alexandru: VIII(61-64), IX. NARIU Anda: VIII(61-64), IX.61; DAVID Dana: VIII(61,64), IX(61,70), X Sandor: VIII(61-64), IX.61; GHICA Alexandra: VIII(61,63,64), IX(61, ILIE Andreea: VIII(61,62,64), IX(61,70), X.67; MURE IANU Raul: V S IX.61; OBANCEA Drago: VIII.64, IX(68,70), X(67,69); SZCS Daniel s 64), IX(61,70), X.69; SEITAN Marius: VIII(61-64), IX.61; SOIM Mihaela 64), IX(61,68,70), X.67; VASILE Ctlin: VIII(61-64), IX.61; VRD aa VIII(61-64), IX.61. Clasa a XI-a. DEMNY Ana-Maria: IX(63,65,68,70) MUNTEAN Vlad: IX(61,63,65), X(62,69).

CARANSEBE S Liceul teoretic "Traian Doda". Clasa a IV-a. SZALO Ildiko: P(94-10

Liceul pedagogic "G. D. Loga". Clasa a VIII-a. VLAD Adina A VI(61-65), VII(61,64), VIII(61,62), IX.63.

CRAIOVA Colegiul Naional "Carol I". Clasa a VI-a. STANCIU Ioan: V(66-7 t 70), VIII.68, G(96,97). HRLAU Scoala "P. Rare". Clasa a III-a (nv. BUDACEA Maria). NEI s P(100,104,105,108,110). Liceul "Stefan cel Mare". Clasa a VII-a. CURCA Ioana: VI(66 VII(68,70); PINTILII Anda: VI(66,67,69,70), VII(68,70).

IA I S Scoala nr. 3 "Al. Vlahua". Clasa a III-a (inst. MAXIM Gabriela t Ionela-Lavinia: P(94,96,98,100,103-105,108); CELMARE Raluca: P(94 103-105,108); GHEMU Laura: P(94,96,98,100,103-105,108); NEAGU Ram 96,98,100,103-105,108); POPOVICI Ionu: P(94,96,98,100,103-105,108); R t Andreea: P(94,96,98,100,103-105,108); RUSU Mdlina Andreea: P(94 a a 103-105,108); SAVA Vlad: P(94,96,98,100,103-105,108); VECHIU Mdlin a a 98,100,103-105,108), ZANCHI Georgiana-Alexandra: P(94,96,98,100,103 Clasa a III-a (nv. MARIUTA Valentina). BRLADEANU Andrei: P(9 105,108); CULEA Alina: P(98,100,103-105,108); HADARAG Ana-Maria: 103-105,108); NASTASE Cosmin: P(98,100,103-105,108); POPA Iulian: 103-105,108); PROCA Ancua-Ioana: P(98,100,103-105,108); TNCU A t P(98,100,103-105,108).

Scoala nr. 13 "Alexandru cel Bun". Clasa a II-a (inst. COJOCAR AGAFITEI Elena-Roxana: P(104-107,109); CARAMALAU Andra: P(104 CALIN Andreea-Claudia: P(104-107,109); COJOCARIU Andreea: P(104 DUDUMAN Luisa- tefania: P(104-107,109); LELEU Alexandrina- tefan S S 107,109); LUPA CU Diana-Maria: P(104-107,109); MANOLACHE Mdl S a a

ea: P(104-107,109); MIHAILA Narcisa-Lorena: P(104-107,109); PASCU P(104-107,109); PADURARU Tiberiu- tefan: P(104-107,109); RADUCE S Andrei: P(104-107,109); SAVIN Cristina-Simona: P(104-107,109); STEFA Vasile: P(104-107,109); STIUBEI Cosmin-Ionu: P(104-107,109). t Scoala nr. 26 "G. Cobuc". Clasa I (inst. RACU Maria). APACHIT s Georgiana: P(94-97,105,110); BURA Emma-Andreea: P(94-97,105,110); grid- tefania: P(94-97,105,110); GRADINARIU Georgiana: P(94-97,105,1 S CU Ovidiu-Constantin: P(94-97,105,110); HUZA Mdlina: P(94-97,105, a a XIM Alexandra-Camelia: P(94-97,105,110); VASILE Bogdan-Andrei: P( 110). Clasa a IV-a (nv. BUCATARIU Rica). AELENEI George-Cipria 105,108-111); SCUTARU Ionela-Cristina: P(104-112). Colegiul Naional "Emil Racovia". Clasa a II-a (inst. CALINESCU t t PETREA Mdlina: P(104-107,110); UNGUREANU George: P(104,10 a a 110). Clasa a V-a. ENEA Maria: V(66-70); TURCANU Andreea Bian 70); Clasa a VI-a. TUDORACHE Alexandru Gabriel: V(66-70), VI(66 Colegiul Naional "C. Negruzzi". Clasa a V-a. PAVALOI Alexandru: t Clasa a VII-a. TIBA Marius: VII(66-68,70), G(96,97,100,103). PETRO ANI S Liceul de informatic . Clasa a X-a. PALITA Florian: VIII(66,68,6 a X.68, G(97,102,103). SIBIU Colegiul Naional "Gh. Laz r". Clasa a XI-a. VLAD Emanuel: X( t a XI(62,63), L(86,87,89,90,92). SUCEAVA Scoala general nr. 3. Clasa a II-a (nv. TABARCEA Silvestru). a Stefan: P(94-97,99).

Premii acordate rezolvitorilor

ASOCIATIA "RECREATII MATEMATICE" n colaborare c revistei RECREATII MATEMATICE acord cte o diplom i un p a as cri pentru trei apariii la rubrica Pagina rezolvitorilor urmtorilor elev a t t a Liceul "N. Titulescu", Braov s BARDA Dan (cl. a X-a): 1/2005(5pb), 2/2005(5pb), 2/2006(7pb);

BUTNARIU Anda (cl. a X-a): 1/2005(6pb), 2/2005(5pb), 2/2006(5pb SZCS Daniel (cl. a X-a): 1/2005(6pb), 2/2005(5pb), 2/2006(7pb); Liceul "Stefan cel Mare", Hrlu a CIUCA Ioana (cl. a VII-a): 2/2005(5pb), 1/2006(5pb), 2/2006(6pb);

PINTILII Anda (cl. a VII-a): 2/2005(5pb), 1/2006(5pb), 2/2006(6pb);

Colegiul Naional "Gh. Laz r", Sibiu t a Scoala nr. 3 "Al. Vlahua", Iai t s SAVA Vlad (cl. a III-a): 2/2005(6pb), 1/2006(5pb), 2/2006(8pb); Scoala nr. 13 "Alexandru cel Bun", Iai s

VLAD Emanuel (cl. a XI-a): 2/2005(5pb), 1/2006(7pb), 2/2006(12pb);

RUSU Mdlina-Andreea (cl. a III-a): 2/2005(6pb), 1/2006(5pb), 2/2 a a

AGAFITEI Elena-Roxana (cl. a III-a): 2/2005(5pb), 1/2006(5pb), 2/2 CARAMALAU Andra (cl. a III-a): 2/2005(5pb), 1/2006(5pb), 2/2006

CALIN Andreea-Claudia (cl. a III-a): 2/2005(5pb), 1/2006(5pb), 2/2

COJOCARIU Andreea (cl. a III-a): 2/2005(5pb), 1/2006(5pb), 2/200

DUDUMAN Luisa- tefania (cl. a III-a): 2/2005(5pb), 1/2006(5pb), 2/ S

LELEU Alexandrina- tefana (cl. a III-a): 2/2005(5pb), 1/2006(5pb), 2 S

LUPA CU Diana-Maria (cl. a III-a): 2/2005(5pb), 1/2006(5pb), 2/20 S

MANOLACHE Mdlina-Andreea (cl. a III-a): 2/2005(5pb), 1/2006 a a 2/2006(5pb);

PASCU Gabriela (cl. a III-a): 2/2005(5pb), 1/2006(5pb), 2/2006(5pb) PADURARU Tiberiu- tefan (cl. a III-a): 2/2005(5pb), 1/2006(5pb), 2 S

RADUCEA Marin-Andrei (cl. a III-a): 2/2005(5pb), 1/2006(5pb), 2/2

SAVIN Cristina-Simona (cl. a III-a): 2/2005(5pb), 1/2006(5pb), 2/20

STEFAN Bogdan-Vasile (cl. a III-a): 2/2005(5pb), 1/2006(5pb), 2/20

STIUBEI Cosmin-Ionu (cl. a III-a): 2/2005(5pb), 1/2006(5pb), 2/20 t

ASOCIA TIA RECREA MATEMATIC TII

Lista membrilor asociaiei - continuarea listei din nr. 1/2006 t 40 41 42 43 44 45 46 47 48 49 50 51

Numele si prenumele Locul de munc a Data Calistru Ctlin aa Univ. Tehn. "Gh. Asachi", Iai s 2 Nechita Vasile Col. Na. "C. Negruzzi", Iai t s 0 Kern Edgar S. C. SELGROS, Braov s 1 Btineu-Giurgiu Maria Acad. Tehnic Militar, Buc. a t a a 1 Btineu-Giurgiu Dumitru Col. Na. "Matei Basarab", Buc. a t t 1 Picioroag Gabriel a Univ. of Southern, Denmark 2 Anica Rzvan s a Lakehead University, Canada 0 Chean Corneliu t CITRIX, INC., USA 0 Maftei V. Ioan Col. Na. "Sf. Sava", Buc. t 0 Gheorhia V. Vitalii t Univ. Tehn. "Gh. Asachi", Iai s 2 Ionescu Marius Vasile Dartmounth College, USA 2 Ciacoi Bogdan Andrei Lic. T. "Ana Iptescu", Gherla, (elev) 2 a

Revista semestrial RECREAII MATEMATICE este ed ASOCIAIA RECREAII MATEMATICE. Apare la datele de 1 1 septembrie i se adreseaz elevilor, profesorilor, studenilor i tutu pasionai de matematica elementar. n atenia tuturor colaboratorilor Materialele trimise redaciei spre publicare (note i articole, che metodic, probleme propuse etc.) trebuie prezentate ngrijit, clar i co trebuie s prezinte interes pentru un cerc ct mai larg de cititori. Se reco textele s nu depeasc patru pagini. Evident, ele trebuie s fie origin nu fi aprut sau s fi fost trimise spre publicare altor reviste. Rugm rialele tehnoredactate s fie nsoite de fiierele lor. Problemele destinate rubricilor: Probleme propuse i Problem pregtirea concursurilor vor fi redactate pe foi separate cu enun i de ie/rezolvare (cte una pe fiecare foaie) i vor fi nsoite de numele autoru la i localitatea unde lucreaz/nva. Redacia va decide asupra oportunitii publicrii materialelor prim n atenia elevilor Numele elevilor ce vor trimite redaciei soluii corecte la proble rubricile de Probleme propuse i Probleme pentru pregatirea conc vor fi menionate n Pagina rezolvitorilor. Se va ine seama de regulile: 1. Pot trimite soluii la minimum cinci probleme propuse n prezent i cel anterior al revistei; pe o foaie va fi redactat soluia une probleme. 2. Elevii din clasele VI-XII au dreptul s trimit soluii la pr propuse pentru clasa lor, pentru orice clas mai mare, din dou clase m imediat anterioare. Elevii din clasa a V-a pot trimite soluii la problemel pentru clasele a IV-a, a V-a i orice clas mai mare, iar elevii claselor trimite soluii la problemele propuse pentru oricare din clasele primare i s mai mare. Orice elev poate trimite soluii la problemele de concurs (tip 3. Vor fi menionate urmtoarele date personale: numele i pr clasa, coala i localitatea. 4. Plicul cu probleme rezolvate se va trimite prin pot (sau v direct) la adresa Redaciei: Prof. dr. Temistocle Brsan Str. Aurora, nr. 3, sc. D, ap. 6, 700 474, Iai Jud. IAI E-mail: tbirsan@math.tuiasi.ro

CUPRINS

Mendel Haimovici i coala matematic din Iai .............................................. Florica T. Cmpan ................................................................................................... Eclipsele de Soare..................................................................................................... Olimpiada 57 .............................................................................................................

ARTICOLE I NOTE

A. REISNER Pseudoinvers i invers generalizat ale unei aplicaii liniare.. C. IGERU O soluie parial a unei probleme a lui N. Papacu ................. M. APETRII i C.-C. BUDEANU Criterii de congruen a triunghiurilor ........... D. M. BTINEU-GIURGIU Asupra problemei 3639 din Gazeta Matematic, v. XXXIII (1927-1928).................

NOTA ELEVULUI

A. ZAHARIUC O problem despre suma cifrelor unui numr natural n baze de numeraie oarecare.............................................. Al. NEGRESCU Generalizri ale unor inegaliti din RecMat...........................

CHESTIUNI METODICE
M. TETIVA Cea mai bun inegaliate de acest tip

...................................

CORESPONDENE CONCURSURI I EXAMENE

Societatea de tiine Matematice din R. Moldova .................................................

Concursul de Matematic "Al. Myller", Ediia a IV-a, Iai.................................. Concursul de matematic "Florica T. Cmpan" ...........................................................

PROBLEME I SOLUII

Soluiile problemelor propuse n nr. 2/2005............................................................. Soluiile problemelor pentru pregtirea concursurilor din nr. 2/2005 .................... Probleme propuse........................................................................................................ Probleme pentru pregtirea concursurilor ................................................................. Training problems for mathematical contests ..........................................................

Pagina rezolvitorilor ................................................................................................. ASOCIAIA RECREAII MATEMATICE

Anul IX, Nr. 2

Iulie Decembri

RECREAII MATEMATIC

REVIST DE MATEMATIC PENTRU ELEVI I PROF

e i = 1

Asociaia Recreaii Matemati IAI - 2007

Semnificaia formulei de pe copert: i ntr-o form concis, formula e = 1 leag cele patru ramuri fun
ale matematicii: ARITMETICA GEOMETRIA ALGEBRA ANALIZA MATEMATIC reprezentat reprezentat reprezentat reprezentat de de de de 1 i e

Redacia revistei :

Petru ASAFTEI, Dumitru BTINEU-GIURGIU (Bucureti), Cornelia - Livi Temistocle BRSAN, Dan BRNZEI, Ctlin - Cristian BUDEANU, CRUU, Constantin CHIRIL, Eugenia COHAL, Adrian CORDUNEAN CRCIUN (Pacani), Gabriel DOSPINESCU (student, Paris), Paraschiva GAL GEORGESCU, Mihai HAIVAS, Gheorghe IUREA, Lucian - Georges LDUNC LUPAN, Gabriel MRANU, Andrei NEDELCU, Gabriel POPA, Dan (Suceava), Florin POPOVICI (Braov), Maria RACU, Neculai ROMAN (Mirc SCLEANU (Hrlu), Ioan ERDEAN (Ortie), Dan TIBA (Bucureti), Maria (Brlad), Lucian TUESCU (Craiova), Adrian ZAHARIUC (Bacu), Adrian ZA Titu ZVONARU (Comneti).

Adresa redaciei: Catedra de Matematic Universitatea Tehnic Gh. Asachi Iai Bd. Carol I, nr.11, 700506, Iai Tel. 032 213737 / int. 123 E-mail: recreatii.matematice@gmail.com http://www.recreatiimatematice.uv.ro

COPYRIGHT 2007, ASOCIAIA RECREAII MATEMATICE Toate drepturile aparin Asociaiei Recraii Matematice. Reproducerea in parial a textului sau a ilustraiilor din aceast revist este posibil numai cu acordu scris al acesteia. TIPRIT LA SL&F IMPEX IAI Bd. Carol I, nr. 3-5 Tel. 0788 498933 E-mail: simonaslf@yahoo.com

Anul IX, Nr. 2

Iulie Decembrie

RECREAII MATEMATICE

REVIST DE MATEMATIC PENTRU ELEVI I PROF

e i = 1
Revist cu apariie semestrial publicat de

ASOCIAIA RECREAII MATEMATIC

IAI - 2007

Al VI-lea Congres Internaional t al Matematicienilor Romni

n zilele de 28 iunie 4 iulie, 2007, s-au desfurat la Bucureti (Unive as s crarile celui de-al VI-lea Congres Internaional al Matematicienilor t sub auspiciile Academiei Romne i ale Universitatilor din Bucureti, Pite s s s soara. Un rol central n organizarea Congresului l-a avut Institutul de M "Simion Stoilov" al Academiei Romne. Amintim ca precedentele congrese, similare acestuia, s-au inut la C t Turnu Severin (1932), Bucureti (1945), din nou Bucureti (1956) i Pite s s s s Ideea organizrii unor astfel de congrese s-a datorat lui Petre Sergescu a fesor la Universitile din Cluj i Bucureti, precum i la Politehnica din at s s s (unde a avut i funcia de rector). Petre Sergescu va deveni, dupa stabi s t Frana (1946), preedinte al Academiei Internaionale de Istorie a Stiinei t s t t t sprijinit i ncurajat de ctre Gheorghe Tieica i Dimitrie Pompeiu s a s tani proemineni ai matematicii romneti n prima jumtate a sec. al XX t t s a ticiparea matematicienilor straini a fost din ce n ce mai numeroasa, nregi un record de peste 100 la prezentul congres. De asemenea, numarul ma nilor romni din diaspor a fost considerabil, remarcndu-se n special tin a pregtesc doctoratul n diverse ari. a t Menionam ca toate arile europene, din est i din vest, au avut partic t t s numar de peste 20 participani au prezentat celelalte continente ale globul t n cele opt secii ale Congresului au fost prezentate peste 400 comu t deschiderea Congresului au fost prezentate trei rapoarte, ntocmite d desemnate de Comitetul de organizare: I Cercetarea matematica n Ro nvaamntul matematic n Romnia; III Diaspora matematica romne t raportat Vasile Brnzanescu (Director al Institutului de Matematic "S. a Constantin Niculescu (Universitatea Craiova), Constantin Corduneanu ( Romna) i Dan Timotin (Institutul de Matematica). s Dintre participanii din strainatate (incluznd Diaspora), menionam t t putai matematicieni ca H. T. Banks (SUA), Daniel Tataru (SUA), Do t nescu (Frana), Izu Vaisman (Israel), Vicenzo Cappasso i Mimo Iannel t s Jean Mawhin (Belgia), Mitrofan Cioban (R. Moldova), Tudor Raiu (Elve t t Grebenikov (Rusia), Alexandra Bellow (SUA), Preda Mihailescu (German Diacu (Canada). Muli participani tineri au prezentat comunicri, din cele mai noi d t t a cercetare matematica. Nici informatica nu a fost neglijata, precum i ma s aplicate la diverse alte domenii de cercetare: Mecanica, Fizica matematica, tate si Statistica, Cercetare operaionala. t Un alt fapt demn de remarcat i foarte mbucurtor pentru noi este pa s a cadrelor didactice din (practic) toate universitaile romneti n care m s t apare ca disciplina de studiu. Congresul s-a bucurat de atenia forurilor rspunztoare pentru tii t a a s neasc, ncepnd cu Preedinia (mesaj trimis la deschidere), continund cu a s t Romna (prin preedintele ei, acad. Ionel Haiduc), cu Universitatea Bucu s

rector, care a oferit i o recepie mpreun cu Ministerul de Externe). s t a O excursie a fost organizata n ziua de duminica 1 iulie, pentru mai suta de participani. S-au vizitat obiective turistice precum oraul Sinaia t s Bran, iar la sfrit Universitatea din Piteti, unde a avut loc i banch s s s gresului. Muzica i dansul (cu specic romnesc), costumele naionale i s t s placuta ce a fost prezenta n timpul excursiei (minunatele priveliti de s Bran-Cmpulung-Muscel), au contribuit n mare masura la reuita acest s menit s aminteasc oaspeilor de frumuseile arii noastre. a a a t t t Sperm ca urmtorul congres, Al VII-lea Congres Internaional al a a t ticienilor Romni, sa aiba loc n anul 2011, daca se va respecta intervalu ani ntre congrese consecutive.

Constantin CORDUNEAN

University of Texas at Arlin

Conjectura lui Poincar

Termenul conjectura nseamna n matematica presupunere, ipoteza, unei armaii nedemonstrate, care poate fi adevrat cu o probabilitate t a a mare (spre exemplu, este adevrat n mai multe cazuri particulare, ca n a a duciei incomplete). Conjectura lui Poincar se refera la caracteriz t 3-dimensionale cu ajutorul unor proprietai topologice uor de intuit i a s s t pus de Poincar n 1904. Jules Henri Poincar (29 aprilie 1854 -17 a a fost unul dintre cei mai mari matematicieni francezi; a fost, n acela s mare fizician teoretic i un filosof al tiinei. Poincar este descris adesea s s t matematician universal, deoarece a excelat n toate domeniile matematic vrat importante, ce existau n perioada vieii sale (relativ scurte). La m a t a fost caracterizat ca matematician, geometru, filosof si om de litere; a fo al innitului, un fel de bard al stiinei. t Pn n momentul formulrii conjecturii, existau suficiente informaii a a t s legate de caracterizarea suprafeelor (varieti 2-dimenionale) orientabile, t at i nchise din spaiul euclidian 3-dimensioanal. Aceste suprafee pot fi ca s t t de genul lor. Acesta este un numar ntreg nenegativ (g 0) care poate intuitiv ca numrul de gauri ale suprafeei. Spre exemplu, sfera uzual, a t a locul geometric binecunoscut n geometria euclidin clasic (sau ca fron a a are genul zero pentru c nu are nici o gaur. Torul, asimilat cu suprafaa u a a t are genul 1 pentru ca are o gaura. Mai departe, se pot imagina covrigi cu guri, iar suprafeele lor ne furnizeaz suprafee de genuri mai mari. a t a t destul de uor c doua suprafee orientabile, nchise si marginite (compa s a t acelai gen pot fi puse n corespondena biunivoca si bicontinua (sunt hom s t n particular, sfera apare ca singura suprafaa orientabila, compacta de ge t Problema ce apare n mod natural, este dac exist caracterizri de a a a pentru sfera 3-dimenional, gndit ca frontiera unei bile 4-dimensionale a a a imaginat o operaie intuitiv deosebit de fructuoas pentru dezvoltarea t a a topologiei algebrice, o disciplina matematica noua, extrem de importanta n actual. Este vorba despre deformarea continua n interiorul unei anumit a unei curbe continue sau difereniabile din acea mulime. O astfel de t t poart numele de homotopie. n particular, este important cazul cnd a simpla nchisa i deformarea se face catre un punct al ei. Intuitiv, pu s imaginm procesul realizat cu un lasou lansat de un paznic de vite. Cnd a a nu prinde gtul niciunei vite, el se strnge napoi n mna paznicului. C prinde o vit (sau un ciot de copac), a ntlnit o singularitate (o gaur) i pa a a s nevoit sa deschida lasoul (adica sa renune la continuitate) ca sa-l poata t Acum putem formula mai precis coninutul conjecturii lui Poincar: t Daca o varietate M 3 , neteda, compacta, de dimensiune 3, are propr orice curba simpla nchisa situtata n aceasta varietate poate fi deformata la un punct, rezulta ca M 3 este homeomorfa cu o sfera? Chiar Poincar a notat oarecum prevztor: Mais cette question nous e a a trop loin. Conjectura lui Poincar a inspirat muli matematicieni i tentati t s demonstra au condus la multe progrese n nelegerea topologiei varietailor t t

siune trei i nu numai. O extindere natural a acestei conjecturi a fost form s a de Poincar, care a armat, n mod eronat, ca: orice varietate poliedrala avnd omologia unei sfere n-dimensionale este homeomorfa cu sfera n-dim Noiunea de omologie a fost abordat, la nceput, n contextul topologiei com t a disciplin ce studiaz complexele simpliciale sau, mai general, celulare, ap a a noiune a fost studiata n condiii mai generale, obinndu-se invariani t t t t interesani. t Pe la sfritul anilor 50 i nceputul anilor 60 s-au obinut rezultate c s s t n studiul conjecturii lui Poincar, realizndu-se c studiul varietilor de d a at mai mari era mai uor de facut dect al celor de dimensiune 3. Con s Poincar a fost demonstrata n cazul unei dimensiuni mai mari dect 4, ctre S. Smale. Alte contribuii au fost aduse de ctre J. Stallings, E. a t a A. Wallace. 20 de ani mai trziu, M. Freedman a folosit cup-produsul i i s Kirby Siebenmann, pentru a demonstra conjectura lui Poincar n dimens Pentru dimensiunea 3, toate tehnicile dezvoltate anterior nu au dat re mod curios, la fel ca n problema gsirii structurilor difereniabile pe spai a t t ene (unde s-au folosit aa numitele teorii de etalonare (gauge), specice fizi s a venit din partea geometriei difereniale. In geometria difereniala problem t t centrat pe studierea proprietilor diverselor structuri geometrice pe varie a at puin pe probleme de tipul conjecturii lui Poincar. A fost R. Hamilton car t pentru studiu fluxul Ricci, pentru care a oferit i unele interpretri zice. s a sa construiasca, n 2003, o metrica de curbura constanta pe orice 3-varie curbura Ricci pozitiv. a Ceva mai trziu, G. Perelman de la Sankt Petersburg a oferit o solu jecturii lui Poincar, n cteva articole postate pe Internet. Aceast solu a t intersul mai multor grupuri de cercetatori care au nceput sa aprofundez tehnice ale demonstraiilor propuse de Perelman. t Oricum, comunitatea matematic internaional s-a artat convins a t a a a mentele lui G. Perelman i, la Congresul Internaional al Matematicienilor s t 2006 de la Madrid, lui G. Perelman i s-a oferit medalia Fields (un premiu tor cu premiul Nobel, care nu exist pentru domeniul matematicilor). Tr a c G. Perelman a refuzat, pentru prima dat n lumea matematicienilo a a Fields din diverse motive. n momentul actual, G. Perelman trece printrextrem de rea din viaa sa, adoptnd o atitudine de respingere a oricare t de apropiere din partea confrailor si (o atitudine asemntoare a adopt t a a a american R. Fisher n anii 70, dup obinerea titlului de campion mondial) a t man a refuzat i un premiu al Societatii Europene de Matematica (EMS ) s cale sa refuze i Premiul Mileniului. S-a retras de la Institutul Steklov, u s i triete izolat, alturi de mama sa. s a s a ntr-o ierarhie a celor mai importante descoperiri tiinice din anul 2006 s t de prestigioasa revist Science, soluia dat de G. Perelman s-a situat pe p a t a devansnd o alta descoperire tiinica extrem de importanta din domeniu s t

Prof. dr. Vasile OPRO Univ. "Al. I. Cuza", I

Tipurile subgrupurilor finite din GL2 (Z)


Gabriel DOSPINESCU 1

Studiul care urmeaza este o continuare a celui nceput n [1]. Cu ace demonstrat o serie de rezultate care ne-au permis s gsim majorri pentr a a a subgrupurilor finite ale lui GLn (Z). Ne-am bazat pe o teorema de mare profunzime, cunoscuta sub numel lui Serre. De fapt, rezultatul a fost obinut de Minkowski i extins de Sel t s pentru care l vom numi n cele ce urmeaz Lema lui Selberg. a Ne-a mai rmas, din planul nostru, s demonstrm c n GL2 (Z) ex a a a a noua tipuri de grupuri finite (pna la un izomorfism). Am adus deja marea teorema Jordan-Zassenhaus, care asigura ca n GLn (Z) exista un n de clase de conjugare ale subgrupurilor finite, rezultat mult mai puternic s de demonstrat dect ceea ce am numit noi "versiunea slab" a teorem a Zassenhaus. De asemenea, pentru n 3, studiul subgrupurilor lui GLn foarte laborios i complicat; de exemplu, in [2] se demonstreaza ca exista 7 s de conjugare ale subgrupurilor finite din GL3 (Z). Chiar studiul claselor de ale subgrupurilor finite din GL2 (Z) este dificil (de altfel, in finalul artico discuta problema conjugarii subgrupurilor ciclice ale lui GL2 (Z)). Teorema 5. Exista exact noua clase de izomorfism ale subgrupurilor lui GL2 (Z). Demonstraie. Desigur, aici considerm i subgrupurile triviale. Deja t a s teorema 2) ca ordinul oricarui subgrup finit al lui GL2 (Z) divide pe 24; pr ordinul unui subgrup finit al lui GL2 (Z) poate fi doar unul dintre numere 4, 6, 8, 12 sau 24. S observm c apte clase de izomorfism se gsesc destul de repede. n a a as a subgrupuri cu un element doua se gasesc fara probleme, cu trei eleme sau 0 1 lua subgrupul generat de , pentru patru elemente putem alege 1 1 0 1 generat de (izomorf cu Z4 ) i subgrupul format din matricil s 1 0 1 0 1 0 i s (izomorf cu Z2 Z2 ), iar pentru ase elemente s 0 1 0 1 0 1 subgrupul generat de (izomorf cu Z6 ) i subgrupul generat de s 1 1 0 1 0 1 i s (izomorf cu grupul S3 al permutarilor de grad 3). 1 1 1 0 teoremele de structur ale grupurilor cu cel mult ase elemente arat c ac a s a a singurele posibiliti pentru subgrupurile lui GL2 (Z) cu cel mult 6 elemen at ramne sa dovedim ca exista cte o singura clasa de izomorfism pentru s din GL2 (Z) cu 8 elemente i tot una pentru cele cu 12 elemente. s Un subgrup abelian cu opt elemente al lui GL2 (Z) (ca, de altfel, oric abelian cu opt elemente) ar trebui s fie izomorf cu Z8 , cu Z2 Z4 , sau cu Z2 a
1

Student, cole Normale Suprieure, Paris

Cum orice element din GL2 (Z) are ordinul 2, 3, 4, sau 6 (nu i 8), nu exist s al lui GL2 (Z) izomorf cu Z8 . Acum sa facem cteva observaii generale despre matricile de ordin t din GL2 (Z). n primul rnd se poate stabili (chiar cu mijloace elementa folosi adic noiuni ca polinom caracterisitic, valori proprii, etc) c orice a t a x y ordin 2 este fie I2 , fie are forma , cu x, y, z numere ntregi a z x 2 x + yz = 1. O matrice A de ordinul patru trebuie sa verifice polinomu deci (fiind cu elemente numere ntregi) trebuie s aib valorileproprii a a x y cum A2 6= I2 , ramne a doua varianta, deci A2 = I2 i A = s z x 2 numere ntregi astfel nct x + yz = 1. n sfrit, argumente asemntoa s a a la concluzia c, dac A are ordinul 3, atunci valorile sale proprii sunt i a a s o radcina cubica diferita de 1 a unitaii), deci A are urma 1 i determi s a t x y A= , cu x, y, z numere ntregi astfel nct x2 + x + 1 + yz z 1 x Acum s presupunem c exist n GL2 (Z) un subgrup izomorf cu Z a a a Z2 i sa privim elementele lui ca matrici complexe. Fiind de ordinul al s sunt diagonalizabile i, deoarece ele comut dou cte dou, exist o baz s a a a a de diagonalizare. n acea baz, matricile din G sunt diagonale i au pe a s principal valorile lor proprii care sunt 1 (cci toate, cu excepia iden a a t ordinul 2). Astfel s-ar obine existena a opt matrici de ordinul al doi t t pe diagonala principal i 0 n rest, evident absurd (acest argument func as t general: ordinul maxim al unui subgrup al lui GLn () care are toate elem ordinul 2 desigur, cu excepia elementului neutru este 2n ; ceea ce fu t o demonstraie eleganta a faptului ca, pentru m 6= n, GLm (Z) nu este i t GLn (Z), o problem greu de rezolvat altfel). a Existena unui subgrup al GL2 (Z) izomorf cu Z2 Z4 ar implica t lui s a b x y a dou matrici A = a , de ordinul al doilea i B = s , c a z x 4, cu a, b, c, x, y, z numere ntregi, a2 + bc = 1 i x2 + yz = 1, car s AB = BA. Condiia aceasta (de comutativitate) ne da sistemul bz = cy t az = cx. Presupunerea c un element, oricare, al celor dou matrici este n a a la o contradicie (de exemplu, a = 0 implic bc = 1 i yz = 1 i acestea t a s s bz = cy). Daca sunt nenule, a i b sunt prime ntre ele, la fel x i y; de s s ay = bx rezulta a = x i b = y, apoi obinem i c = z (semnele c s t s deci A = B, evident o contradicie. De altfel, se constat uor c, tot t a s a cale, se poate obine i afirmaia demonstrat mai sus: orice subgrup al l t s t a format numai din elemente de ordin 2 (cu excepia elementului neutru) ar t patru elemente; desigur, n cazul general nu se poate proceda aa. s Un subgrup necomutativ cu opt elemente este izomorf fie cu grupul die cel al cuaternionilor. Un subgrup al lui GL2 (Z) cu elemente, izomorf opt 0 1 0 1 i diedral, este cel generat de matricile s (omitem de fi 1 0 1 0 verificrile, acestea fiind imediate). Ne mai rmne s demonstrm c G a a a a a

are subgrupuri izomorfe cu grupul cuaternionilor. Presupunem ca exista un asemenea subgrup; asta ar nsemna, de fapt matricile A, B GL2 (Z), astfel nct AB = B 3 A, A2 = B 2 , B 4 = I2 lui B este 4; cum am vzut, asta nseamn c A2 = B 2 = I2 , ceea a a a la AB = BA. Rezult existena unor numere ntregi a,b, c, x, y, z a a t a b 2 2 i B = a + bc + 1 = x + yz + 1 = 0 i astfel nct A = s s c a Egalitatea AB = BA implica i 2ax + bz + cy = 0; eliminnd c, z din pri s relaii i folosind-o pe a treia, rezult (bx ay)2 + b2 + y 2 = 0, deci b = y t s a ce, evident, conduce la contradicie. t S ne ndreptm acum atenia asupra subgrupurilor cu 12 elemente ale lu a a t consideram mai nti un asemenea subgrup G neabelian. Sa presupunem numerele x2 , . . . , xq nu apare i 1; cum (2 (1))(2 0) = 6 i cum s s x2 ) (2 xq ), trebuie s avem xq = 2. Deci exist a, b , cu a + b = 10 a a a 12|2k + a 0k + b (1)k + (2)k , oricare ar fi k numr natural. Este clar a sa avem atunci b = 0, iar alegerea k = 2 conduce iarai la o contradicie. t s trebuie s existe o matrice A G cu urma egal cu 1. a a Fie u, v valorile proprii ale matricii A (care verific u + v = 1). Cum a 1 1 1 u i v au modulul 1, deci s = + = u + v = u + v = 1. Din u + v uv u v 3 3 rezult u = v = 1, deci A3 = I2 , adic A are ordinul 6. S mai c a a a B G {Ak | 0 k 5} i sa observam ca vom avea atunci G = {Ak s 5} {BAk | 0 k 5} i ca AB = BA5 . ntr-adevar, este clar (din mod s l-am ales pe B) c exist k 0 astfel nct AB = BAk , ceea ce nseamn c a a a Ak = B 1 AB i A sunt similare, deci au aceleai valori proprii. Atunci s s 1 1 {uk , uk } i {u, u } coincid, ceea ce duce imediat la posibilitaile k 1 (m s t k 5 (mod 6). Cum G este neabelian ramne k 5 (mod 6) i AB = BA s se mai scrie (innd cont de A2 A + I2 = 02 , adic de teorema Cayleyt a AB + BA = B, iar de aici obinem tr(B) = 2 tr(AB). Cum matricile de o t 6 au urma impara, B trebuie sa aiba ordinul 2 sau 4. Sa presupunem ca ordinul 4; ca mai sus va rezulta ca B 2 = I2 . Un scurt moment de refle existena unor numere ntregi a, b, c, x, y, z astfel nct a2 + bc + 1 = x2 + t x y a b i A = s ,B= . Atunci, din AB + BA = B, dupa un z 1x c a y 2 ay obinem relaia u2 u + 1 + t t = 0 (u = x), egalitate evident i b b Astfel rezulta ca B are ordinul 2, deci singurul subgrup necomutativ de or a a a a exist a GL2 (Z) ar putea fi cel diedral. Si nici nu e greu s artm c n G 1 0 1 i subgrup izomorf cu D6 : este cel generat de matricile: s 1 1 1 0 n fine, s presupunem c G ar fi un subgrup abelian cu 12 elemente al lu a a Dup cum am amintit (v. mai sus problema de olimpiad), ordinul oric a a a din GL2 (Z) poate fi doar 1, 2, 3, 4 sau 6; prin urmare G ar putea fi izomor doua tipuri de grupuri comutative, adica Z12 i Z2 Z2 Z3 ) doar cu Z2 s (deoarece Z12 este generat de un element de ordin 12).

S zicem c ar fi n GLn (Z) un subgrup izomorf cu Z2 Z2 Z3 ; aces a a mai mult de un element de ordin 2 (prin urmare i o matrice de ordin 2 s

c(2x + 1) t s ; nlocuim n x2 + x + 1 + yz = 0, mai inem seama i de b 2a i ajungem iar la o evident contradicie: 3a2 + (2x + 1)2 = 0; deci GL2 s a t subgrupuri comutative cu 12 elemente. Ne-a mai rmas s demonstrm c nu exist subgrupuri cu 24 de e a a a a a GL2 (Z). Din pacate, orict ne-am straduit, nu am reuit sa gasim o astfel s straie care s utilizeze ideile de mai sus. Se poate ns arta acest fapt, t a a a un nivel superior al ediciului matematic. Avem, mai nti Teorema 6. Orice subgrup finit al lui GLn (Z) este conjugat cu un sub On (R). Demonstraie. Prin On (R) nelegem grupul matricilor ortogonale di t t G fiind subgrupul finit despre care este vorba n enun, vom defini un n t scalar pe Rn (unde elementele se considera ca vectori coloana) prin X hgx, gyi, hx, yi = c=
gG

I2 ) , precum i elemente de ordin trei, deci atunci ar exista matricile A = s x y 2 de ordin 2 (a, b, c Z, a + bc = 1) i B = s de ordin 3 (x z 1 x t t x2 +x+1+yz = 0) astfel nct AB = BA. Obinem relaiile bz = cy, 2ay = i 2az = c(2x + 1); vedem imediat c de aici s-ar obine, dac am presup s a t a b( c b = c = 0, imposibil. De aceea a 6= 0 i putem exprima y = a s

unde h, i este produsul scalar obinuit din Cn . Deoarece fiecare matrice s inversabila, se verifica uor faptul ca i h, i este un produs scalar, precu s s hgx, gyi = hx, yi , x, y Cn ,

adica orice element din G este izometrie faa de acest nou produs scalar t mare, matricile din G sunt matrici ortogonale ntr-o baza ortonormala relat produs, ceea ce reprezint (reformulat) concluzia lemei. Acum putem de a a Teorema 7. Orice subgrup finit al lui GL2 (Z) este fie ciclic, fie diedr Demonstraie. Fie, iar, G un subgrup finit al lui GL2 (Z), pe care t teoremei anterioare l putem considera direct subgrup al lui O2 (R), i fie H s lui G cu SO2 (R), adic cu grupul (matricilor) rotaiilor lui R2 (matrici a t cu determinantul 1). Indicele lui H n G este cel mult egal cu 2 (deoare orice x, y G, xy 1 este o matrice ortogonala care are determinantul 1 sa plus, H este un grup ciclic, deoarece este un grup finit de rotaii i se ara t s orice asemenea grup este generat de o rotaie a sa de unghi minim (tot a t arata ca orice subgrup al lui Z este generat de un element al sau de modu Acum, daca intersecia H are indicele 1 n G, evident vom avea G = t este ciclic. Dac indicele este 2, considerm un element s G H, care tre a a o simetrie faa de o dreapt ce trece prin origine. Se verific atunci cu u t a a s prin calcul, fie recurgnd la interpretarea geometrica pe un desen) ca

r fiind generatorul lui H. Cum s2 este identitatea i G este generat de s (indicele lui H n G fiind 2) obinem imediat ca, n acest caz, G este died t n fine, Teorema 5 rezulta din nou, sub forma:

Teorema 8. Orice subgrup finit al lui GL2 (Z) are cel mult 12 elemen doar o clasa de izomorfism a subgrupurilor lui GL2 (Z) cu 12 elemente. Demonstraie. Totul rezulta din teorema anterioara si vesnica obs t matricile din GL2 (Z) au ordin 1, 2, 3, 4 sau 6. Evident, teorema 7 implic c exist doar 9 tipuri de clase de izomorfism pentru subgrupurile lui GL a a noi ne-am straduit mai sus (i nu am reuit n totalitate) sa aratam ca se po s s acelai rezultat i pe cale "elementara". s s

Terminam acest articol inndu-ne o promisiune: stabilirea claselor de t ale matricilor de ordin 3, 4, 6 din GL2 (Z). Nu vom trata cazul matricilor d din simplul motiv c necesit o cu totul alt metod. Vom folosi fr dem a a a a aa un rezultat clasic de geometria numerelor, anume faimoasa teorem a lui M a orice mulime convexa, simetrica n raport cu originea i de arie strict mai m t s 4 din R2 conine macar un punct laticial nenul. Vom ncepe cu urmatoare t direct a teoremei lui Minkowski, care se va dovedi crucial n studiul c a a conjugare ale matricilor de ordin finit:

Lema 1. a) Daca a,b, c sunt numere ntregi astfel nct bc = a2 + t ecuaia cx2 2axy + by 2 = 1 are soluii n numere ntregi. t 2 b) Daca bc = a + a + 1, atunci ecuaia cx2 (2a + 1)xy + by 2 = 1 t ntregi. Demonstraia este uoar dac folosim teorema lui Minkowski i f t s a a s altfel. Ideea este urmatoarea (vom rezolva doar a), punctul b) ind absolu 2 dac considerm A mulimea punctelor (x, y) din plan pentru care cx 2a a a t 2, un calcul imediat arata ca A are aria strict mai mare decat 4. ntr-adev n mod evident presupune b, c > 0 i atunci condiia se scrie z 2 + t2 < s t a y z = x c y i t = . Deci A este imaginea cercului de arie 2 pri s c c z + at t a a liniara (z, t) ( , ct). Or, aceasta aplicaie conserv ariile, cci ma c ciat are determinantul 1. Deci A are aria 2 > 4 si totul rezult acum di a a lui Minkowki: A conine un punct laticial nenul (x, y) i pentru acest pun t s n ipoteza b, c > 0, avem n mod evident cx2 2axy + by 2 = 1. Cum a urmeaz exact aceeai cale de demonstraie, deci rmne n seama cititor a s t a

Acum putem ncepe sa studiem conjugarea matricilor de ordin 3, 4, 6. M vom demonstra urmatoarea:

Teorema 9. Exista exact o clasa de conjugare in GL2 (Z) pentru m ordin 3 din GL2 (Z). Aceeai concluzie este valabila pentru matricile de s pentru cele de ordin 6. Demonstraie. Dac A GL2 (Z) este de ordin 3, respectiv 6, am t a timpul studiului tipului de izomorfism al subgrupurilor finite c A2 + A a i A2 A + I2 = O2 , respectiv. Sa consideram o matrice A de ordinul 3. s

a b ca A se poate scrie sub forma unde a, b, c Z verifica a2 + a c 1 a Aplicnd lema, rezulta ca exista un vector nenul e = (x, y) astfel nct |det( (aici (e, Ae) este matricea care are prima coloan egal cu e i pe cea dea a s Ae): un calcul imediat a c de fapt condiia |det(e, Ae)| = 1 este echi arat a t 2 cx (2a + 1)xy + by 2 = 1. Aceasta nseamna ca (e, Ae) este o baza a sensul ca matricea (e, Ae) este n GL2 (Z). Or, A(Ae) = A2 e = Ae 0 1 aceasta baza matricea lui A este exact . Am aratat astfel ca 1 1 de ordin 3 sunt toate conjugate ntre ele. Cu exact aceleai argumente s s elementele de ordinul 4, respectiv 6 au aceeai proprietate. s Cititorul ar putea sa se ntrebe n acest moment: de ce am inclus ultim n acest articol? Raspunsul este simplu: cum n tot articolul am oscilat nt i teoria numerelor i cum am nceput cu algebra, preferm s terminm s s a a a numerelor. Si, ntr-adevar, din ultima teorem obinem cteva rezultate a t moase din acest domeniu. Sa luam, de exemplu, iarai cazul matricilor d s a b Am vzut ca pentru o astfel de matrice, scris sub forma a a ,p c 1 a 0 1 x y P GL2 (Z) astfel nct A = P 1 P . Scriind P = , un s 1 1 z t arat c exist = det P {1, 1} astfel nct a a a a = (yz xy zt), b = (y 2 yt + t2 ) i s c = (z 2 xz + x2 ), d = (zt xt + xy),

unde xt yz = . Aceasta este deci soluia general a ecuaiei a2 + a t a t n numere ntregi. ncercai sa demonstrai aceasta prin alte mijloace i t t s avantajele acestei metode. Sa mai subliniem un rezultat, deloc banal, car obine de aici. S presupunem c p este un numr prim de forma 3k + 1 t a a a p1 u o rdcin primitiv modulo p. Notnd x = u 3 , obinem imediat c a a a a t x3 = 1, deci x2 + x + 1 = 0 (lucrm in Z/pZ). Aceasta arat existena u a a t ntreg a pentru care p | a2 + a + 1. Din cele observate anterior, rezulta ca scrie sub forma x2 xy + y 2 pentru nite numere ntregi x i y. Am arta s s a orice numr prim de forma 3k + 1 poate fi exprimat ca x2 xy + y 2 (cu x, a Evident, lucrnd cu matricile de ordin 4, ajungem cu aceleai argumente la s teorema a lui Fermat: orice numar prim de forma 4k + 1 se scrie ca sum ptrate de numere ntregi. a ncheiem aici scurta noastr vizit n lumea subgrupurilor lui GL2 (Z), a a de a sugera cititorului temerar un studiu al claselor de izomorfism ale sub finite ale lui GL3 (Z). Bibliografie

1. G. Dospinescu - Cteva proprietati ale subgrupurilor finite din GLn (Z) Matematice 1/2006. 2. Ken-Ichi Tahara - On the finite subgroups of GL3 (Z), Nagoya Math. J

O problem cu cifrele unui numr a a


Titu ZVONARU 1
Problema G.116, propus de Maria Mihe n RecMat - 1/2007, are a t enun: t

Aflai toate numerele naturale N de patru cifre nenule distincte cu pr t ca diferena dintre cel mai mare numar obinut prin permutarea cifrelor lu t t mai mic asemenea numar este tocmai N . Ne propunem sa rezolvam urmatoarea problema mai generala:

Aflai toate numerele naturale N cu cifre dictincte cu proprietatea ca t dintre cel mai mare numar obinut prin permutarea cifrelor lui N si ce t asemenea numar este tocmai N . Fie n numrul cifrelor lui N . Avem de analizat urmtoarele cazuri: a a A. n = 2. Fie a > b cifrele lui N . Cel mai mare numar scris cu cifrele ab, iar cel mai mic este ba. Avem posibilitile: at i) ab ba = ab ba = 0, ii) ab ba = ba 8a = 19b i nu obinem soluii. s t t

B. n = 3. Fie a > b > c cifrele lui N . Avem abccba = xyz, cu {a, b, c} i obinem z = 10 + c a, y = 9 + b b = 9, x = a 1 c. s t Cum a este cea mai mare cifra, rezulta a = 9 i {b, c} = {8 c, c + s 8 c = c, b = c + 1. Obinem soluia 954 459 = 495. t t

C. n = 4. Fie a > b > c > d cifrele lui N . Avem ca abcd dcba = xyz C = {a, b, c, d} = {x, y, z, t}. Obinem: t t = 10 + d a, z = 9 + c b, y = b 1 c, x = a d. Observm c y + z = 8, x + t = 10 i deducem c a a s a {y, z} {{0, 8} , {1, 7} , {2, 6} , {3, 5}} , {x, t} {{1, 9} , {2, 8} , {3, 7} , Avem de analizat urmatoarele posibilitai: t {y, z} {x, t} (a, b, c, d) {0, 8} {1, 9} (9, 8, 1, 0) b 1 c = 6 C / {0, 8} {3, 7} (8, 7, 3, 0) 9 + c b = 5 C / {0, 8} {4, 6} (8, 6, 4, 0) b 1 c = 1 C / {1, 7} {2, 8} (8, 7, 2, 1) b 1 c = 4 C / {1, 7} {4, 6} (7, 6, 4, 1) {2, 6} {1, 9} (9, 6, 2, 1) b 1 c = 3 C / {2, 6} {3, 7} (7, 6, 3, 2) a d = 5 C / {3, 5} {1, 9} (9, 5, 3, 1) a d = 8 C / {3, 5} {2, 8} (8, 5, 3, 2) a d = 6 C / {3, 5} {4, 6} (6, 5, 4, 3) b 1 c = 0 C. /
1

Comneti, e-mail: tzvonaru@hotmail.com a s

Obinem o singur soluie: 7641 1467 = 6174. t a t

D. n = 5. Fie a > b > c > d > e cifrele lui N . Avem c abcde edcb a i notam C = {a, b, c, d, e} = {x, y, z, t, u}. Obinem u = 10 + e a, t = s t z = 9, y = b 1 d, x = a e. Rezult c a = 9 i e 6= 0 (dac e = 0 ar ur a a s a deci x = z). Observm c y + t = 8, x + u = 10 i, cum 0 C, deducem c a a s / adic a {y, t} {{1, 7} , {2, 6} , {3, 5}} , {x, u} {2, 8} {4, 6} {3, 7} {2, 8} {4, 6} {y, t} {1, 7} {1, 7} {2, 6} {3, 5} {3, 5} Nu obinem soluii. t t (a, b, c, d, e) (9, 8, 7, 2, 1) (9, 7, 6, 4, 1) (9, 7, 6, 3, 2) (9, 8, 5, 3, 2) (9, 6, 5, 4, 3)

{x, u} {{2, 8} , {3, 7} , {4, 6}} b1d=4C / ae=8C / 9+db=5C / ae=7C / b 1 d = 2 C. /

E. n = 6. Fie a > b > c > d > e > f cifrele lui N . Avem c a i notam C = {a, b, c, d, e, f } = {x, y, z, t, u, v}. Obinem s t v = 10+f a, u = 9+eb, t = 9+dc, abcdef f edcba = xyztuv

z = c1d,

y = be,

Observam ca z + t = 8, y + u = 9, x + v = 10 i deducem ca s {z, t} {{0, 8} , {1, 7} , {2, 6} , {3, 5}} {y, u} {{0, 9} , {1, 8} , {2, 7} , {3, 6} , {4, 5}} {x, v} {{1, 9} , {2, 8} , {3, 7} , {4, 6}} . c1d=4C / be=5C / be=7C / be=7C / 10 + f a = 2 C / cd1=4C / 9+eb=3C / be=5C / c1d=0C / be=5C / be=5C / af =8C / c1d=0C / be=6C / 9+dc=8C / af =7C / af =8C / c 1 d = 0 C. /

Avem de analizat urmtoarele posibiliti: a at {z, t} {y, u} {x, v} (a, b, c, d, e, f ) {0, 8} {2, 7} {1, 9} (9, 8, 7, 2, 1, 0) {0, 8} {2, 7} {4, 6} (8, 7, 6, 4, 2, 0) {0, 8} {3, 6} {1, 9} (9, 8, 6, 3, 1, 0) {0, 8} {4, 5} {1, 9} (9, 8, 5, 4, 1, 0) {0, 8} {4, 5} {3, 7} (8, 7, 5, 4, 3, 0) {1, 7} {0, 9} {2, 8} (9, 8, 7, 2, 1, 0) {1, 7} {0, 9} {4, 6} (9, 7, 6, 4, 1, 0) {1, 7} {3, 6} {2, 8} (8, 7, 6, 3, 2, 1) {1, 7} {4, 5} {2, 8} (8, 7, 5, 4, 2, 1) {2, 6} {0, 9} {3, 7} (9, 7, 6, 3, 2, 0) {2, 6} {1, 8} {3, 7} (8, 7, 6, 3, 2, 1) {2, 6} {4, 5} {1, 9} (9, 6, 5, 4, 2, 1) {2, 6} {4, 5} {3, 7} (7, 6, 5, 4, 3, 2) {3, 5} {0, 9} {2, 8} (9, 8, 5, 3, 2, 0) {3, 5} {0, 9} {4, 6} (9, 6, 5, 4, 3, 0) {3, 5} {1, 8} {4, 6} (8, 6, 5, 4, 3, 1) {3, 5} {2, 7} {1, 9} (9, 7, 5, 3, 2, 1) {3, 5} {2, 7} {4, 6} (7, 6, 5, 4, 3, 2)

Nu obinem soluii. t t E. n = 7. Fie a > b > c > d > e > f > g cifrele lui N . Avem abcdef g gf edcba = zyxtuvw

i la fel ca n cazul n = 5 deducem ca a = 9 i g 6= 0. s s Diferena dintre un numar i rasturnatul sau este multiplu de 9; cum su t s din baza 10 este de asemenea multiplu de 9, deducem c suma celor trei cifre a la scrierea lui N trebuie s fie multiplu de 9. a Deoarece tim ca cifra 0 nu este folosita, ramnd de analizat doar ur s posibilitai: t cifre nefolosite 0,1,8 9765432 2345679 = 7419753 0,2,7 9865431 1345689 = 8519742 0,3,6 9875421 1245789 = 8629632 0,4,5 9876321 1236789 = 8639532 i nu obinem soluii. s t t

F. n = 8. Din acelai motiv ca n cazul n = 7, avem doar posibilitaile s t cifre nefolosite 0,9 87654321 12345678 = 75308643 1,8 97654320 2345679 = 95308641 2,7 98654310 1345689 = 97308621 3,6 98754210 1245789 = 97508421 4,5 98763210 1236789 = 97527421 cu soluia 98763210 1236789 = 97527421. t G. n = 9. Deoarece n este impar, rezult c singura cifr nefolosit a a a a lui N este 0. Obinem soluia 987654321 123456789 = 864197532. t t H. n = 10. Obinem soluia 9876543210 123456789 = 9753086421. t t

ERATA

n numrul 1/2007 al revistei Recreaii Matematice s-au strecurat ur a t greeli: s 1. La pag. 2, r. 1 n loc de 1866 se va citi 1766. 2. n enunul problemei XII.76 se va considera c funcia f : [a, b] t a t continua, condiie care a fost omisa. t

O problem de construcie a unui triungh a t


Temistocle BRSAN 1

Problemele de construcie cu rigla i compasul au un specific i un farm t s s Pe aceasta linie, menionam remarcabila monografie Probleme de construc t trice cu rigla si compasul a lui Gh. Buicliu. Problema de care ne vom ocupa este de tipul urmtor: a

Se pornete de la o gura F (un triunghi, de exemplu) si prin diverse s auxiliare se obine o conguraie F 0 . Reinnd cteva elemente ale lui t t t stergnd o buna parte din conguraia F 0 ), cum putem reconstrui cu rig t pasul figura iniiala F. t

Vom da cteva exemple cunoscute de acest fel: 1. S se construiasc un triunghi ABC cunoscndu-i centrele cercuril a a scrise triunghiurilor HBC, HCA i HAB, unde H este ortocentrul ace s Problema 624 ).

2. Sa se construiasca un triunghi cunoscnd punctele de intersecie a bis t interioare cu cercul circumscris triunghiul ([4], p. 105, Problema 88 ).

3. S se construiasc triunghiul ABC cunoscnd poziiile vrfurilor A0 , a a t triunghiurilor echilaterale construite pe laturile lui i n exterior ([1], Probl s

4. S se construiasc triunghiul ABC cunoscnd centrele , , ale a a adjuncte (CA), (AB), (BC) sau centrele 0 , 0 , 0 ale cercurilor adjun (CB), (AC) (se numete cerc adjunct (CA) cercul care trece prin B i es s s n C laturii CA) ([1], Problema 640 ).

5. S se construiasc un triunghi cunoscndu-i punctele O, H, I (cu s a a uzuale); n ce condiii exist un astfel de triunghi? ([3]) t a

S ncepem prin a preciza notaiile (de altfel uzuale, v. [2]) utiliza a t (fig. 1): I, Ia , Ib , Ic centrele cercurilor nscris i exnscrise triunghiului; s H, Ha , Hb , Hc ortocentrul i proieciile lui pe laturile BC, CA s t s AB; D, E, F punctele de contact ale cercului nscris I (I, r) I cu la CA i respectiv AB; s Da , Ea , Fa etc. punctele de contact ale cercului exnscris Ia (Ia , ra cu dreptele suport ale acelorai laturi; s A0 , B 0 , C 0 punctele date de {A0 } = Db Fb Dc Ec , {B 0 } = Ec D {C 0 } = Fa Ea Fb Db . Ne propunem s rezolvm urmtoarea a a a Problem. Sa se construiasca cu rigla si compasul triunghiul ABC a poziiile punctelor A0 , B 0 , C 0 . t
1

Prof. dr., Univ. Tehnic "Gh. Asachi", Iai a s

Vom face analiza problemei, adic pentru configuraia construit, fo a t a 4ABC dat, 4Ia Ib Ic al centrelor cercurilor exnscrise i 4A0 B 0 C 0 al pu s contact exterioare (Fa , Ea ; Db , Fb i Ec , Dc ) vom indica un numar de p s Ca urmare, construcia triunghiului A0 B 0 C 0 va decurge cu uurina. t s t

A Ec
Ic

Fb A
Ib

Fc I Dc B Fa B Da H Ha

Eb

Db

Ea
Ia

Fig. 1

Propoziia 1. Triunghiul A0 B 0 C 0 are proprietatile: t


0 0 0 0

2) B C si Db Dc sunt antiparalele, 3) B C k Ib Ic , ca si analoagele aces Demonstraie. 1) Deoarece BDb i BFb sunt tangente la cercul Ib , t s B \ s 4BDb Fb este isocel i, deci, m(BDb Fb ) = 90 . La fel, considernd 2 \c ) = 90 C . Ca urmare, n 4A0 Db D isoscel, deducem ca m(CDc E 2 A B c c c i m(C 0 ) = m(A0 ) = 90 . Analog, se stabilete c m(B 0 ) = 90 s a s 2 2

c 1) m(A0 ) =

a s Observaie. 4A0 B 0 C 0 i 4Ia Ib Ic sunt omotetice, cci au laturile par t cte dou. Centrul lor de omotetie este punctul de concurena a dreptelor A a t C 0 Ic , pe care-l notam cu K 0 .

B 0D D c 2) ntruct m(B 0 ) = 90 = m(A\ c ), dreptele B 0 C 0 i Db Dc sun s b 2 alele n raport cu 4A0 B 0 C 0 . 3) Consideram dreapta d determinata de punctele B i Ib ca o secanta a s B 0 C 0 i Ib Ic i aratam ca se formeaza unghiuri alterne interne egale. s s A b b i analoagele (m(Ia ) Se tie c n 4Ia Ib Ic avem m(Ia ) = 90 s a s 2 B A C [ m(BIC) = + = 90 ). Atunci, din faptul ca d Ia Ic de 2 2 2 C C b \ m(d, Ib Ic ) = 90 m(Ic ) = 90 90 = , iar din d A0 C 2 2 C c \ m(d, B 0 C 0 ) = 90 m(C 0 ) = . 2

Propoziia 2. Dreptele AA0 , BB 0 , CC 0 sunt concurente n punctul H t Demonstraie. Fie A punctul n care AA0 intersecteaza dreapta B t a arata ca AA0 trece prin H, vom demonstra ca A coincide cu Ha . Conform teoremei lui Menelaus aplicat 4BDb Fb i transversalei AA0 a s laia t

AB A0 Fb A B = . D A b AFb A0 Db Cum AB = c i AFb = BFb AB = pc (p fiind semiperimetrul 4ABC), s calculm A0 Db i A0 Fb . S mai observm c Db Dc = BDb + CDc BC = a a a a s b + c. Acum, cu teorema sinusurilor n 4A0 Db Dc i innd cont de punct s t ale Propoziiei 1, obinem relaia t t t cos C Db Dc A0 Db 2 = A0 Db = (b + c) , C A cos 2 cos 2 cos A 2 iar A0 Fb = A0 Db Db Fb = (b + c) cos C B 2 2p sin (din 4BDb Fb ). 2 cos A 2 Ca urmare, relaia de mai sus se scrie t A B c = A Db pc A Utiliznd formele de tipul cos = 2 r (b + c) cos C B 2 2p sin 2 cos A 2 . C cos 2 (b + c) cos A 2

p (p a) , dupa calcule de rutina rez bc

a2 b2 + c2 A B = , A Db (b + c) (a + b c)

de unde

a2 b2 + c2 A B a2 b2 A B = = A B + A Db (b + c) (a + b c) + a2 b2 + c2 BDb a2 2 2 2 A B a b +c = A B = c cos B, p 2 2p adic A este piciorul nlimii AHa . Deci AA0 trece prin H. Analog se a at BB 0 , CC 0 trec prin H, ceea ce ncheie demonstraia. t

Observaie. 4A0 B 0 C 0 i 4ABC sunt omologice, H fiind centru de om t s remarc uor c ele sunt i ortologice. a s a s
0

Propoziia 3. Punctul H este centrul cercului circumscris 4A0 B 0 C t R a acestui cerc este data de R0 = HA + ra = HB + rb = HC + rc . \ Demonstraie. Deoarece B 0 H AEa , avem c m(HB 0 C 0 ) = m(H t a A \ 90 m(AEa Fa ) = . Din CH 0 AFa i procednd la fel, deducem c m( s a 2 A a . Deci 4HB 0 C 0 este isoscel, adic HB 0 = HC 0 . n mod analog, s 2 0 0 HB = HA . Rezulta ca H este centrul cercului circumscris 4A0 B 0 C 0 . Pentru partea a doua din enun s artm c AA0 = ra . ntr-adevr, t a aa a a B 0F ) = \ avem: AFb = p c, m(AA b (analoaga uneia de mai sus) i m( s 2 B 90 + t (unghi exterior 4BDb Fa isoscel). Cu teorema sinusurilor, obin 2 r r B pc S p (p b) ac AA0 = cos = (p c) = B 2 sin 2 ac (p a) (p c) pa 0 0 0 Atunci, R = A H = HA + AA = HA + ra etc.

Propoziia 4. Dreptele A0 Ia , B 0 Ib , C 0 Ic trec prin mijloacele laturilor [ t si repsectiv [AB] ale 4ABC. Demonstraie. Vom dovedi numai afirmaia relativ la A0 Ia . Fie {U t t a a BC. Revine la a arta c BU = . a a 2 Din faptul ca 4Ia Da U 4A0 Ha U rezulta ca U Da ra U Da ra a cb c = = U Da = p = U Ha ra + ha Da Ha 2ra + ha 2p a 2 B cb cci Da Ha = BHa BDa = a cos B (p c) = 2c cos2 p = p a ,i 2 a . . 1 a ha pa 2S p a ra = =1 2+ =1 2+ ha 2ra + ha (p a) ra a S a 2+ r
a

Consecina. Dreptele A0 Da , B 0 Eb , C 0 Fc sunt concurente n ortocen t 4A0 B 0 C 0 . Demonstraie. Segmentele [AA0 ] i [Ia Da ] au lungimi egale cu ra i su t s s (ca fiind perpendiculare pe BC). Prin urmare, patrulaterul A0 AIa Da est gram i, deci, A0 Da k AIa . Din aceasta i faptul ca AIa B 0 C 0 , rezulta c s s B 0 C 0 , adic A0 Da este nlimea n 4A0 B 0 C 0 . n acelai fel, B 0 Eb , C 0 Fc su a at s n 4A0 B 0 C 0 i demonstraia este complet. s t a

cb a = i demonstraia este s t 2 2 Consecina. A0 Ia , B 0 Ib , C 0 Ic sunt simediane att n 4A0 B 0 C 0 ct si n t Demonstraie. Mijlocul U al segmentului [BC] este i mijlocul lui [D t s CDb = BDc = p a. Tinnd cont de punctul 2) al Propoziiei 1, n t avem c A0 Ia trece prin mijlocul antiparalelei Db Dc la B 0 C 0 . Cum n oric a o simediana este locul mijloacelor antiparalelelor la latura opusa [2], p.55 c A0 Ia este simediana prin A0 a 4A0 B 0 C 0 . n sfrit din omotetia observa a s a triunghiurilor A0 B 0 C 0 i Ia Ib Ic , rezult c A0 Ia este simediana prin Ia a a s (faptul rezulta i observnd ca BC i Ib Ic sunt antiparalele n 4Ia Ib Ic ). s s n sfrit, BU = BDa + U Da = (p c) + s

Revenim la problema de construcie propus la nceput. Rezultatele t a fac posibila construcia 4ABC, atunci cnd se da 4A0 B 0 C 0 . ntr-adevar, t compasul putem parcurge fiecare dintre paii urmatori: s

1) se construiete centrul H al cercului circumscris triunghiului A0 B 0 C s fi ortocentrul triunghiului ABC, conform Propoziiei 3, iar conform Propo t trebui ca A, B, C s se afle pe A0 H, B 0 H, C 0 H, respectiv); a 2) n 4A0 B 0 C 0 se construiete simediana corespunztoare vrfului A0 ; s a 3) se ia n mod arbitrar un punct C1 (HC 0 ) i prin el construim perp s pe A0 H care intersecteaza (B 0 H) ntr-un punct B1 ; 4) unim H cu mijlocul segmentului [C1 B1 ] pentru a avea locul mijlo mentelor ce se sprijin pe HC 0 i HB 0 i sunt perpendiculare pe A0 H; a s s 5) intersectm acest loc geometric cu simediana prin A0 construit l a a (conform Propoziiei 4 i Consecinei sale, acest punct va fi mijlocul la t s t cautate); 6) obinem vrfurile C i B ale triunghiului de construit ca intersecii a t s t dicularei pe A0 H dusa prin punctul construit la 5) cu (HC 0 ) i respectiv s 7) se construiete vrful A ca intersecie cu (HA0 ) a perpendicularei s t C 0 H. Bibliografie

1. Gh. Buicliu - Probleme de construcii geometrice cu rigla si compasul, Ed t Bucureti, 1957. s 2. T. Lalescu - Geometria triunghiului, Ed. Tineretului, Bucureti, 1958. s 3. F. Lo Jacomo - Enonc 245, APMEP, Buletin no. 408, 1997, 57-79. 4. D. Smaranda, N. Soare - Transformari geometrice, Ed. Acad. R.S.R., 1988.

Un ir strns legat de irul lui Wallis s s


Scopul propus este studiul irului (an )n1 definit prin s
0 1 an = Cn Cn

Adrian CORDUNEANU 1 , Gheorghe COSTOVIC


1!! n n (2n 1)!! 2 3!! 3 5!! + Cn Cn + + (1) Cn . 2!! 4!! 6!! (2n)!!

Vom stabili c este monoton descresctor i convergent la zero i vom pune a a s s legatura acestuia cu sirul lui Wallis (wn )n1 dat de 2 1 (2n)!! wn = (2n 1)!! 2n + 1 (despre care tim c lim wn = ). s a n 2

Lem ([1], p. 124 sau [2], p. 350). Pentru orice k N avem a (k 1)!! Z /2 Z /2 , k par k k k!! 2 cos x dx = sin x dx = (k 1)!! 0 0 , k impar k!! Demonstraie. Dac n una din cele dou integrale efectum schim t a a a t a x , vom obine pe cealalt; deci integralele sunt egale. Fie Ik valoare 2 lor. Integrnd prin pari, gasim: t Z /2 Z /2 k cos x dx = (sin x)0 cosk1 x dx = Ik =
0

= (k 1) = (k 1) de unde Z

/2

sin x cosk2 x ( sin x) dx = 1 cos2 x cosk2 x dx = (k 1) (Ik2 Ik ) Ik = k1 Ik2 , k k 2.

0 /2

Aceast relaie de recurena i faptul c I0 = a t t s a Propoziie. Are loc identitatea t

i I1 = 1 conduc la rezult s 2

(2n 1)!! 0 1 1!! 2 3!! n (2n 1)!! = Cn Cn + Cn + + (1)n Cn . (2n)!! 2!! 4!! (2n)!!

a Demonstraie. Obinem (3) exprimnd integrala I2n n dou moduri. t t


1 2

Prof. dr., Catedra de matematic, Univ. "Gh. Asachi", Iai a s Conf. dr., Catedra de matematic, Univ. "Gh. Asachi", Iai a s

conform Lemei, avem I2n = I2n = = Z


/2

(2n 1)!! . Apoi, folosind formula binomulu (2n)!! 2


/2

cos2n x dx =
/2

t Egalnd aceste doua valori ale lui I2n , obinem (3). Corolarul 1. Sunt adevarate afirmaiile: t (2n 1)!! 1) an = , n N ; (2n)!! 2) (an )n1 este strict descrescator.

0 1 2 3 n = Cn I0 Cn I2 + Cn I4 Cn I6 + + (1)n Cn I2n = n n (2n 1)!! 0 1 1!! 2 3!! 3 5!! = Cn Cn + Cn Cn + + (1) Cn . 2 2!! 4!! 6!! (2n)!!

0 n n 1 2 3 Cn Cn sin2 x + Cn sin4 x Cn sin6 x + + (1) Cn sin2

n 1 sin2 x dx =

Demonstraie. Punctul 1) decurge din (1) i (3), iar 2) din faptul c t s 2n + 1 < 1, n N . 2n + 2 Corolarul 2. Sunt adevarate afirmaiile urmatoare: t 1 1 1) an = , n N ; wn 2n + 1 2) (an )n1 converge la zero.

Demonstraie. Punctul 1) rezulta din (2) i (4), iar 2) prin trecere l t s (5) pentru n . Observaie. Faptul c lim an = 0 poate fi stabilit i utiliznd dubla t a s n 2 3 (2n 1)!! 1 1 < < , n > 1 ([3], p. 48). 2 (2n)!! 2 2n 2n Bibliografie.

1. G. M. Fihtenhol - Curs de calcul diferenial si integral, vol. II, Ed. t t Bucureti, 1964. s 2. Gh. Sirechi - Calcul diferenial si integral, vol. I, Ed. Stiinifica i Enc t t t s Bucureti, 1985. s 3. *** - Probleme de matematica traduse din Kvant, vol. I, E. D. P., Bucure

Asupra rdcinilor polinomului X 3 + pX + q a a


Adrian REISNER1

I. Considerm polinomul P = X 3 + pX + q Q [X], avnd rdcinile a a a 1 , 2 , 3 . Notm, ca de obicei, cu rdcina primitiv de ordin 3 a a a a 3 2 2 1 = cos + i sin ). ( = + i 2 2 3 3 Propoziia 1. Pentru , C, urmatoarele afirmaii sunt echivalent t t (i) 3 = p si P ( + ) = 0; p3 (ii) 3 si p3 sunt radacinile polinomului Q = X 2 + qX . 27 Demonstraie. Cum P ( + ) = 3 + 3 + (3 + p) ( + ) + q, a t p3 (i) 3 + 3 + q = 0, 3 = p 3 + 3 = q, 3 3 = 27 Consecina 1. Daca , C satisfac (i) sau (ii), atunci radacinile p t P sunt + , + 2 si 2 + . Demonstraie. Dac i sunt rdcinile cubice produs rea t a s a a cu dou rdcini ale polinomului Q, atunci perechile , 2 i 2 , a a a a s proprieti, deci are loc (ii). Deducem c P + 2 = P 2 + = at a concluzia. 4p3 Observaie. Daca = q 2 + t este discriminantul polinomului Q a 27 1) Daca > 0, atunci Q admite doua radacini reale i distincte, fie ac s B. Notnd cu = 3 A, = 3 B rdcinile cubice reale, atunci + = a a este unica rdcin real a lui P . a a a a 2) Daca = 0, atunci Q admite radacina reala dubla A. Notnd = R, radacinile polinomului P vor fi + = 2 3 A i + 2 = 2 + s (rdcin dubl), toate reale. a a a a 3) Dac < 0, atunci Q admite dou rdcini complexe conjugat a a a a Notnd cu una dintre radacinile cubice ale lui A i cu conjugatul lui , s lui P vor fi + , + i 2 + 2 , toate reale. s II. n cele ce urmeaza, vom studia submulimile lui C definite prin: t Ai = Q (i ) = {R (i ) | R Q [X]} , i = 1, 2, 3; A = Q (1 , 2 , 3 ) = {R (1 , 2 , 3 ) | R Q [X1 , X2 , X3 ]} .

Propoziia 2. n raport cu operaiile de adunare si nmulire a nume t t t plexe, Ai se structureaza ca un corp comutativ. n plus, Ai este un Q-spai t de dimensiune cel mult 3. Demonstraie. Faptul c Ai este un inel integru, unitar i Q-spaiu v t a s t verific imediat. Dac R Q [X], din teorema mpririi cu rest rezult c a a at a P (X) C (X) + + X + X 2 i, cum P (i ) = 0, deducem ca s Ai = + i + 2 | , , Q , i
1

Cercettor, Centrul de Calcul E. N. S. T., Paris a

prin urmare 1, i , 2 constituie un sistem de generatori pentru Q-spaiu t i Ai ; astfel , dimQ Ai 3. n sfrit, pentru orice a Ai \ {0}, aplica s t Ai Ai definite prin x 7 ax i x 7 xa sunt injective (Ai fiind inte s surjective (Ai fiind spaiu vectorial de dimensiune finita) i atunci exista t s pentru care aa0 = a00 a = 1. Urmeaza uor ca a0 = a00 , deci Ai este corp. s Propoziia 3. Mulimea A se structureaza canonic drept corp comu t t Q-spaiu vectorial de dimensiune cel mult 6. t Demonstraie. Cum 1 +2 +3 = 0, deducem ca A = Q (1 , 2 ) = Q t P (X) = X 2 + X + 2 + p A Dar 2 este radacina a polinomului 1 X 1 t urmare A va fi un A1 -spaiu vectorial de dimensiune cel mult 2. Rezulta ca dimA1 A dimQ A1 6. Faptul ca orice element nenul al lui A este in demonstreaz ca n Propoziia 2. a t

III. Automorfismele algebrei A. S observm c orice endomorfism al spaiului vectorial A cu propr a a a t u (1) = 1, u (xy) = u (x) u (y), x, y A este n mod necesar bijectiv: dac a cu x 6= 0, atunci 1 = u (1) = u xx1 = u (x) u x1 = 0, contradicie, d t injectiv, iar surjectivitatea urmeaz din faptul c dim A este finit. a a a Un astfel de endomorfism (de algebre) u va fi numit automorfism, iar tuturor acestor automorfisme o vom nota Aut A. n raport cu compunerea Aut A se structureaza n mod evident ca grup abelian.

Propoziia 4. n cazul n care P este un polinom reductibil peste t dim A {1, 2}; mai mult exista doi indici distinci i, j astfel nct A = t Daca dim A = 1, atunci |Aut A| = 1, iar daca dim A = 2, atunci |Aut A| Demonstraie. Cum P este reductibil peste Q, atunci toate cele tr t sunt raionale i atunci A = A1 = A2 = A3 = Q, iar dimQ A = 1, sau t s radacina, sa zicem 1 , este raionala, caz n care A1 = Q, A = A2 = t A1 -spaiu vectorial de dimensiune 2, deci dimQ A = 2. t Pentru orice u Aut A i x A, avem c u (xn ) = [u (x)]n , deci u s a R (u (x)), R Q [X]. n particular, P (u (i )) = u (P (i )) = u (0) i {1, 2, 3}, j {1, 2, 3} a. . u (i ) = j ; altfel spus, un automorfism u o permutare a rdcinilor. a a Dac dim A = 1, deci cnd A = Q, atunci pentru orice x A avem a u (x 1) = xu (1) = x, deci Aut A = {1A }. Daca dim A = 2, fie i Q, 2 6= 3 . Pentru u Aut A vom avea c u (1 ) = 1 . Dac u (2 ) = 2 , u a a iar x = y + z2 , y, z Q, este un element al lui A, atunci u (x) = y + zu deci u = 1A . Dac u (2 ) = 3 , u (3 ) = 2 , atunci u (x) = y+ z3 a s avem u (xx0 ) = u (x) u (x0 ), x, x0 A, ceea ce revine la u 2 = [u a 2 aceast egalitate este realizat: am vzut n demonstraia Propoziiei 3 a a a t t sunt rdcinile polinomului X 2 +1 X + 2 + p A1 [X], deci 2 = 1 a a 1 2 prin urmare u 2 = 1 3 2 p = 2 = [u (2 )]2 . Astfel, n acest ca 2 1 3 Aut A = {1A , u}. Propoziia 5. Daca P este ireductibil peste Q, atunci: t a) dim A {3, 6};

b) Daca dim A = 6 si u Aut A, atunci exista S3 astfel nct u ( i {1, 2, 3}, iar |Aut A| = |S3 | = 6; c) Daca dim A = 3, atunci |Aut A| = 3 si singurele elemente invaria orice automorfism sunt elementele lui Q. Demonstraie. a) P fiind ireductibil, admite trei rdcini distincte t a a (o eventuala radacina dubla ar anula P i P 0 , deci (P, P 0 ) Q [X] ar divi s astfel nu ar fi ireductibil). Cum i nu este radacina a unui polinom din grad 2, atunci 1, i , 2 este sistem liniar independent i astfel dimQ A s i 2 A1 , atunci 3 = 1 2 A1 , deci A = A1 = A2 = A3 i dimQ A s 2 A1 , atunci dimA1 A = 2, deci dimQ A dimA1 A dimQ A1 = 6. / = b) Daca dimQ A = 6, o baza a lui A fiind 1, 1 , 2 , 2 , 1 2 , 2 2 (v. 1 1 3), nseamn c un automorfism u este bine determinat prin cunoaterea e a a s u (1 ) i u (2 ), pentru c u realizeaz i aici o permutare a rdcinilor s a a s a a Propoziia 4). Din comportarea lui u faa de baza, rezulta ca t t 2 2 u (1) = 1, u 1 = [u (1 )] , u (1 2 ) = u (1 ) u (2 ) , u 2 2 = [u (1 1 aceste condiii sunt suficiente pentru a demonstra ca u este automorfism t Mai nti,sa aratam ca u este automorfism al lui A1 : o baza a acestui s 3 1, 1 , 2 , ar fi destul s artm c u 3 = [u (1 )] i u 4 = [u ( a aa a s 1 1 1 adevr, avem: a 3 u 3 = u (p1 q) = pu (1 ) q = [u (1 )] ; 1 u 4 = u p2 q1 = p [u (1 )]2 qu (1 ) = [u (1 )]4 . 1 1 Fie acum x = y + z2 , x0 = y 0 + z 0 2 , cu y, z, y 0 , z 0 A1 ; trebuie sa ca u (xx0 ) = u (x) u (x0 ). Cum aceasta proprietate are loc daca x A2 sa 2 rmne s justificm c u 2 = [u (2 )] . Avem: a a a a 2 2 = 1 2 2 p u 2 = 2 3 2 p = 2 = [u (2 ) 2 1 2 2 3 P (X) a a a = X 2 + X + 2 + p c dat fiind c 3 este rdcin pentru a 2 X 2 1 2 3 permutatrea asociat lui u este = a . 2 3 1 Obinem astfel toate automorfismele lui A ca fiind asociate cte unei t din S3 , deci |Aut A| = |S3 | = 6. c) Conform celordemonstrate la a), daca dim A = 3, atunci A = A1 = baze a lui A fiind 1, i , 2 , i = 1, 2, 3. Fie u Aut A. Daca exis i a nct u (i ) = i , atunci u invariaz toate elementele unei baze, deciu = u 6= 1A , avem c u (1 ) = 2 i u 2 = 2 sau u (1 ) = 3 i u 2 = a s s 1 2 1 automorfisme u sunt astfel unic determinate, deci |Aut A| = 3 (automorfi asociate ciclurilor de lungime 3 din S3 ). Cutm acum elementele invariante pentru orice automorfism. Dac a a a x2 1 + x3 2 A, atunci pentru u 6= 1A avem c u (x) = x1 + x2 2 a 1 u2 (x) = x1 + x2 3 + x3 2 . Egalitatea x = u (x) = u2 (x) conduce, dupa 3 x2 + x3 (2 + 1 ) = 0, x2 + x3 (3 + 1 ) = 0. Daca x2 i x3 nu sunt am s atunci 2 + 1 = 3 + 1 , deci 2 = 3 , absurd. Rezult c singurele a a invariante pentru orice automorfism sunt elementele lui Q.

Inegaliti stabilite cu un procedeu de reduc at a numrului de variabile - Mixing variabl a


Iurie BOREICO 1 , Andrei CIUPAN 2

Prezentm n cele ce urmeaz un procedeu mai recent de rezolvare a ine a a folosit adesea pentru rezolvarea inegalitailor de tip olimpiada. Ideea ce s t acestuia este urmmatoarea: cnd avem de-a face cu inegalitai, este con t transformm o problem ce comport trei variabile ntr-una numai cu dou a a a a sau s reducem inegalitatea la cazul cnd unul din numere este egal cu 0. V a acest procedeu prin cteva probleme rezolvate n acest fel. Pornim cu o simpla i binecunoscuta: s 1 Exemplul 1. Daca a + b + c = 1, sa se arate ca a2 + b2 + c2 . 3 1 Soluie. S considerm funcia f (a, b, c) = a2 + b2 + c2 i s t a a t s a 3 a+b a+b , , c au ace f (a, b, c) 0. Observm c tripletele (a, b, c) i a a s 2 2 i evalum diferena s a t a+b a+b , ,c . D = f (a, b, c) f 2 2 a+b a+b 1 a , ,c Se vede uor c D = (a b)2 0, adic f (a, b, c) f s a 2 2 2 a+b a+b fi de ajuns sa demonstram ca f , , c 0, ceea ce este mai c 2 2 2 (a + b) 1 Acest lucru este echivalent cu + c2 . Fiindc a + b + c = 1, a 2 3 2 a+b = 1c, deci ne rmne s demonstrm c (1c)2 +2c2 (c 3 a a a a 3 evident adevarata. Menionam ca inegalitatea de mai sus are multe alte s t

S ne ocupm de probleme mai dificile, care au soluii surprinztor a a t a prin aceast procedeu. b c a + + Exemplul 2. Daca a, b, c 0, sa se arate ca b+c c+a a+b a b c 3 Soluie. Fie f (a, b, c) = t + + . Vom arta c f a a b+c c+a a+b 2 b+c b+c , . Explicitnd i apoi desfcnd parantezele, se obine in s a t f a, 2 2 echivalenta b3 + c3 + ab2 + ac2 2abc + b2 c + bc2 , inegalitate imediat inegalitaii mediilor: b3 + c3 b2 c + bc2 i a(b2 + c2 ) a 2bc. Prin urmar s t b+c sa aratam ca f (a, t, t) 0, unde t = . Aceasta relaie este echi t 2
1 2

Elev, Chiinu s a Elev, Bucureti s

2t 3 a a a + (a t)2 0, adevrat. Deci problema este rezolvat. 2t a + t 2 Exemplul 3. Daca a, b, c > 0 si abc = 1, atunci a2 +b2 +c2 +3 2(ab Soluie. Fie f (a, b, c) = a2 + b2 + c2 + 3 2(ab + bc + ca). Cons t triplet convenabil, carea pstreze condiia abc = 1. Un astfel de triple s a t ( ab, ab, c). Avem f ( ab, ab, = + ab + c2 + 3 2(ab 2 c) = c2 c) ab + Mai departe, D = f (a, b, c) f ( ab, ab, c) = a2 + b2 + 4 c 2(ab + b (a b)2 + 2c(2 ab a b), deci (a b)2 . D = (a b)2 2c( a b)2 = (a b)2 2c ( a + b)2

Fiindc relaia din t este simetric, putem presupune c c = min{a, b, a t enun a a deducem ca c < 2. Asta nseamna ca c2 2 2ab c ( a (a b)2 2c (a b)2 2c 0, deci D 0. Aadar, rmne s s a a ( a + b)2 f ( ab, ab, c) 0 c2 + 3 4 c, adevarata din inegalitatea medii patru numere.

Exemplul 4. Daca a, b, c > 0 si abc = 1, atunci a2 + b2 + c2 + 3 a ab + bc + ca. P P P Soluie. Sa consideram funcia f (a, b, c) = a2 a ab + 3 i t t s strm c f (a, b, c) 0. Vom alege un triplet convenabil pe care s-l interc a a a f (a, b, c) i 0. Un astfel de triplet, la care i produsul e invariant este ( a s s Se vede c f ab, ab, c) = c2 + ab 3 2 ab c(1 + 2 ab). a ( At f (a, b, c) f ( ab, ab, c) = a2 + b2 (a + b) (bc + ca) 2ab + 2 ab Prin grupare convenabil a termenilor, se obine D = (a b)2 ( a b) a t (a b)2 c+1 (c + 1), de unde D = (a b)2 1 (a b)2 ( a + b)2 ( a + b)2 inegalitatea din enun este simetrica, putem presupune ca c = t min{a, b, c c 1 i ab 1, de unde obinem succesiv c + 1 < 4 4 ab ( s t c+1 1 < 0 D 0. Deci, s demonstrm c f ( ab, ab a a a deci ( a + b)2 2 2 t c +(1/ c 1) +2 c+2 c. Dar aceasta relaie din urma este adevarata c2 + 2 2c + 1 c + 2 c. n concluzie, problema este rezolvat. a Exemplul 5. Se dau a, b, c 0, cu a + b + c = 3. Atunci a + ab + bc + ca. P P (Indicaie. Consideram f (a, b, c) = t a ab. Atunci

Cu presupunerea a = min{a, b, c} i dup efectuarea calculelor, se obine f s a t b+c b+c f (a, , ) 0.) 2 2

p b+c b+c (b c) f (a, b, c) f (a, , ) = b + c 2(b + c) bc + 2 2 2 p 2 !2 ( b + c) ( b + c + 2(b + c)) 4 b c p . = 2 b + c + 2(b + c)

Exemplul 6 (Inegalitatea lui Schur ). Daca a, b, c 0 si r 0, atunc ar (a b)(a c) + br (b a)(b c) + cr (c a)(c b) 0.

Soluie. S considerm f (a, b, c) = ar (ab)(ac)+br (ba)(bc)+cr (c t a a i sa demonstram ca f (a, b, c) 0. Se poate observa ca f (a, a, c) = cr (c s Rmne s artm c f (a, b, c) f (a, a, c) f (a, b, c) f (a, a, c) 0. D a a a aa a calcul ne arat c f (a, b, c) f (a, a, c) = (a b) [(a c)(ar cr ) + br (c a a daca consideram ordonarea a c b, permisa de simetria inegalitaii t Prin urmare, f (a, b, c) f (a, a, c) 0 i inegalitatea este demonstrata. s

Exemplul 7. Daca a, b, c 0, a + b + c = 1, atunci a2 b + b2 c + c2 a Soluie. Se observa ca alegerile de triplete folosite n exemplele prec t mai duc la rezolvarea problemei; este nevoie de alegerea unui triplet inspir 2 1 c "ghicim" i verificm c egalitatea are loc dac a = 0, b = , c = a s a a a 3 3 un alt triplet care s ilustreze acest caz de egalitate. Mai nti, consider a a 4 2 2 2 a t f (a, b, c) = a b + b c + c a , apoi evalum diferena D = f (a, b, c) f ( 27 Obinem D = a (bc (c a)(c b)). Atunci cnd c este ntre a i b, t s D 0 (putem face aceasta presupunere, deoarece inegalitatea din enun e t n cele trei variabile). Deci ar fi de ajuns sa demonstram ca f (0, a +

Deoarece a + b + c = 1, relaia precedent este echivalent cu (a + b)2 c t a a 8 2c(1 c)(1 c) , care rezult direct din inegalitatea mediilor. a 27 Exemplul 8. Daca a, b, c 0 si a + b + c = 3, sa se afle maximul exp E(a, b, c) = ab(a + b) + bc(b + c) + ca(c + a).

Soluie. Daca a = b = c = 1, atunci expresia are valoarea 6, care nu est t

ei. ns vedem c, dac lum un numar egal cu 0 i celelalte dou egale cu a a a a s a 27 expresia ia valoarea . Vom arata ca acesta este maximul expresiei. C 4 27 funcia f (a, b, c) = ab(a + b) + bc(b + c) + ca(c + a) . Cu presupunerea t 4 obinem f (a, b, c)f (a+b, c, 0) = a2 (b+c)+b2 (c+a)+c2 (a+b)(a+b)2 c t 27 ab(a + b 2c) 0. Cum f (a + b, c, 0) 0 (a + b) c(a + b+ c) (a 4 (adevarata din inegalitatea mediilor!), problema este rezolvata. Exemplul 9 (Inegalitatea lui Turkevici ). Daca a, b, c, d 0, atunci

a4 + b4 + c4 + d4 + 2abcd a2 b2 + a2 c2 + a2 d2 + b2 c2 + b2 d2 + c2 d P 4 P (Indicaie. Notam f (a, b, c, d) = t a + 2abcd a2 b2 . Presupunn a b c d, se arat c f (a, b, c, d) f (a, b, c, c) f (a, b, b, b) f (a, a, a a 5 Exemplul 10. Daca a, b, c 0, sa se arate ca (a + b + c) 81abc a2 Soluia 1. Inegalitatea din enun fiind omogen, putem presupune c t t a c = 3. Astfel, ramne sa demonstram ca abc a2 + b2 + c2 3. Fie f

2 b+c b+c 2 2 abc a + b + c 3. Se arata ca f a, , f (a, b, c) 0 2 2 b+c b+c , f a, 0 n urma unor calcule de rutin. a 2 2 Soluia 2. Se ine cont din nou de omogenitatea inegalitaii din e t t t presupune abc = 1. Pentru a demonstra ca (a + b c)6 81(a2 + b + procedeaz astfel: artm c f (a, b, c) f (a, bc, bc) 0, unde f a aa a (a + b + c)6 81(a2 + b2 + c2 ). Soluia 3. Vom demonstra inegalitatea din enun fara a folosi metod t t variables", pentru a arata ca, dei aceasta metoda este eficienta pentru s foarte mare de probleme, pot exista i soluii mai simple. Folosind bine s t (ab + bc + inegalitate (ab + bc + ca)2 3abc(a + b + c), deducem ca abc 3(a + b + 6 2 ar fi de ajuns s demonstrm c (a + b + c) 27 (ab + bc + ca) a2 + b2 a a a putea finaliza rezolvarea tot prin mixing variables, dar alegem inegalitatea q X X X 3 2 (a + b + c) = a2 + ab + ab 3 (a2 + b2 + c2 ) (ab + bc +

de unde, prin ridicare la puterea a treia, rezult chiar inegalitatea cutat a a a

Exemplul 11. Se dau a, b, c 0, astfel nct ab+bc+ca = 1. Sa se af 1 1 1 expresiei + + . a+b b+c c+a 5 s t (Indicaie. Minumul este , i se obine cnd doua din numere sunt t 2 i celalalt este 0. Putem considera doua triplete pentru compararea cu s p i 0: a) f (a, c) f (a, t, t) 0, t = (a + b)(a + c) a; b) f s cu b, 1 f 0, , a + b 0.) a+b

Aplicaii t 1. Se dau x, y, z 0 i x + y + z = 3; s se arate c x2 + y 2 + z 2 + xy s a a 2. Se dau a, b, c > 0, abc = 1; sa se arate ca 3(a2 + b2 + c2 ) + 23 4 (a + 1) (b + 1) (c + 1) . 1 1 1 6 3. Dac a, b, c > 0 i abc = 1, s se arate c + + + a s a a a b c a+b+c 4. Fie a, b, c 0, a + b + c = 1. S se arate c a2 b + b2 c + c2 a + abc a a 1 1 1 (ab+ + + 5. Dac a, b, c 0, s se arate c a a a (a + b)2 (b + c)2 (c + a)2 9 (Olimpiada Iran, 1996 ). 4 1 a b c 7 6. Dac a, b, c [ ; 3], s se arate c a a a + + . 3 a+b b+c c+a 5 7. Fie a, b, c > 0 i abc = 1. Sa se arate ca 2(a2 +b2 +c2 )+18 3(a+1)(b s 8. Numerele reale a, b, c satisfac relaia a2 + b2 + c2 = 9. Sa s t 2 (x + y + z) xyz 10 (Test de selecie, Vietnam). t 9. Daca a, b, c 0, sa se arate ca (a + b + c)4 16 a2 b2 + b2 c2 + c2 a2

11. Dac a, b, c 0 cu a + b + c = 3 i k 0, s se afle valoarea a s a expresiei

10. Dac a, b, c 0 i a + b + c = 1, atunci s se arate c a s a a 2 2 2 2 2 2 a + b2 b + c2 c + a2 8 a b + b2 c2 + c2 a2 .

Ea,b,c = ab(a + b + k) + bc(b + c + k) + ca(c + a + k) (Andrei Ciupan,

12. Se dau numerele reale pozitive a1 , a2 . . . an astfel nct a2 +a2 + + 1 2 se afle maximul expresiei E = (1a1 )(1a2 ) (1an ) (Baraj,Seniori,Rom Bibliografie 1. V. Crtoaje - Algebraic Inequalities, Editura GIL, 2006. 2. M. O. Drmbe - Inegalitati - Idei si Metode, Editura GIL, 2003. 3. Site MathLinks: www.mathlinks.ro .

Din partea redaciei t

De-a lungul celor noua ani de apariie a Recreaiilor Matematice, am av t t de a urmri evoluia multor elevi de excepie. Probleme i note semnat a t t s Dospinescu, elev, Oneti, au fost urmate de altele semnate de acelai, da s s Bucureti, i apoi student, Paris. Ali tineri, precum Oana Crj, Cez s s t a Irina Mustaa, Marius Pachiariu (pentru a-i aminti macar pe premia t t tei) i-au fcut ucenicia n paginile Recreaiilor, ind acum studeni emine s a t t universiti prestigioase. at

Parc ns nicio promoie nu ne-a adus atia colaboratori ca aceea ca a a t t i-a ncheiat studiile liceale n 2007. Ne facem o placuta datorie de a ami s acestor proaspei studeni (majoritatea la faculti de matematic): t t at a

Adrian Zahariuc, Bacu (o not matematic, 15 probleme propuse); a a a Alexandru Negrescu, Botoani (dou note matematice, 10 probleme s a Iurie Boreico, Chiinau (o nota matematica, 2 probleme propuse); s Vlad Emanuel, Sibiu (6 probleme propuse, cel mai bun rezolvitor); Bogdan Ciacoi, Gherla (dou note matematice, 1 problem propus) a a a

Alturi de acetia, au mai publicat o not Anca Timofte i Alexan a s a s canu (Botoani), au propus probleme originale Cristian Svescu (Focan s a s dra Vlcu, Iulia Zanoschi, Florin Asvoaie (toi din Iai), au fost m a t s cu soluii deosebite ale unor probleme Dana Timofte, Diana Prodan t Hamciuc (Iai), au primit premii acordate rezolvitorilor Ciprian Cost s Ciucanu, Mihai Dnil, Mircea Avram, Andrei Tofan, Clin Tur a a a a fana Brniteanu, Alina Andriescu i Irina Pruteanu. a s s Tuturor le mulumim, le dorim succes n viitor i pe toi i ateptm n t s t s a alturi de Recreaii Matematice! a t

Asupra unei probleme de concurs


Julieta GRIGORAS 1 , Claudiu-Stefan POPA2

n cele ce urmeaz vom vedea cum se face o Nota plecnd de la o proble a i comun i utiliznd mijloace elementare. Aadar, un ndemn la crea s a s s Punctul de pornire este cea de-a doua problema propusa la Concursul " Cmpan", etapa judeeana, clasa a VIII-a, al carei enun poate fi gasit n ac t t al revistei la pag. 116 Cerina de la primul punct al acestei probleme este, dup cum au o t a civa candidai, o consecina imediata a faptului ca, n ipoteza problemei t t t considerata este una regulata. Vom indica i alte condiii, asemanatoar s t problema surs, care s implice o piramid regulat. Mai nti ns, vom a a a a a i vom rezolva problema iniial. s t a Problema 1. Fie V ABC o piramida triunghiulara regulata cu V A V C V A. Notam x = V A, y = V B, z = V C, a = BC, b = AC si c ipoteza ca xa = yb = zc, piramida V ABC este regulata.p Soluie. t Cum a = y2 + z2, b = x2 + z 2 , p z 2 2 2 + y2 c = x (), condiia (1) devine x y + z 2 = 2 t b y 2 x2 + z 2 = z 2 x + y 2 . Dupa desfacerea parantezelor V 2 2 i reducerea lui x y , din prima egalitate obinem c x = y s t a x i analog y =z, prin urmare V A = V B = V C. Apoi, s c a = b = c = x 2 i deducem c V ABC este piramid regus a a A lata. Problema 2. Daca n ipoteza Problemei 1 nlocuim condiia (1) cu t x y z 3 = = , a b c atunci V ABC este piramida regulata. Soluie. Din (2) obinem, folosind () i proprietatea irului de rapo t t s s x2 y2 z2 x2 + y 2 + z 2 1 ca 2 = 2 = 2 = = , de unde 2x2 = y + z2 x + z2 x + y2 2 (x2 + y 2 + z 2 ) 2 2y 2 = x2 + z 2 i 2z 2 = x2 + y 2 . Scaznd doua cte doua aceste relaii, t s x = y = z i apoi a = b = c = x 2. s Problema 3. Daca n ipoteza Problemei 1 nlocuim condiia (1) cu t x + a = y + b = z + c, 3 atunci V ABC este piramida regulata.
1 2

Profesor, Liceul Teoretic "V. Alecsandri", Iai s Profesor, S coala "A. Russo", Iai s

3 Nota redac iei. Un tetraedru (arbitrar) care veric condi ia V A V B t a t se numete tridreptunghic, care veric (2) izodinamic, iar care veric (3) Crelle (D. s a a Ani a, C. Cocea - Planul s i spaiul euclidian, pp. 245, 255, 251). t t

Soluie. Observm, innd seama de (), c t a t a 2 2 2 2 (3) x + a + 2xa = y + b + 2yb = z 2 + c2 + 2zc 2 2 x + y + z 2 + 2xa = y 2 + x2 + z 2 + 2yb = z 2 + x2 + y 2 + 2zc i astfel Problema 3 se reduce la Problema 1. s Problema 4. Daca n ipoteza Problemei 1 nlocuim condiia (1) cu t |x a| = |y b| = |z c| , atunci V ABC este priramida regulata. Soluie. Calculele sunt analoage cu cele din soluia Problemei 3. t t Apare ntrebarea: ce se ntmpl dac nlocuim a, b, c cu o permutare a a lor? Vom arta c i n aceast situaie piramida este regulat. a as a t a 0 Problema 1 . Daca n ipoteza Problemei 1 nlocuim condiia (1) cu t xb = yc = za, priramida V ABC este regulata. Soluie. Observam nti ca, daca doua dintre numerele a, b, c sunt ega t uor ca sunt egale toate trei; prin urmare, a, b i c sunt fie distincte doua s s fie egale ntre ele. Presupunem prin absurd c ne putem situa n primul a ordona numerele a, b, c; fie a < b < c. Atunci: a2 < b2 < c2 z 2 + y 2 < x2 + z 2 < x2 + y 2 z < y < x. Pe de alta parte, din (10 ) rezulta ca z > x > y i ajungem astfel la o co s Rmne c a = b = c, apoi x = y = z, deci V ABC este piramid regulat a a a a Problema 20 . Daca n ipoteza Problemei 1 nlocuim condiia (1) cu t x y z = = , b c a atunci V ABC este piramida regulata. Soluie. Avem succesiv: t x2 y2 z2 x2 y2 = 2 = 2 2 = 2 (20 ) 2 2 2 2 2 x2 x +z x +y y +z x +z x + y2 y x2 y2 z2 x2 + y 2 + z 2 z2 2 = 2 = 2 = 2 = 1, = 2 2 z2 y +z z x y x + y2 + z2 de unde x = y = z, apoi a = b = c. x y z Observaie. Dac nlocuim (20 ) cu t a = = (200 ), concluzia se c a b x y z cu aceleai calcule. Daca nsa nlocuim (20 ) cu s = = (2000 ), co a c b pastreaza, cu o justificare puin diferita: t x2 y2 z2 x2 + y 2 + z 2 1 (2000 ) 2 = 2 = 2 = = z + y2 x + y2 x + z2 2 (x2 + y 2 + z 2 ) 2

2x2 = y 2 + z 2 , 2y 2 = x2 + y 2 , 2z 2 = x2 + z 2 x = y = z. Lsm n seama cititorului formularea problememlor 30 i 40 ; soluiil aa t s similare cu cele ale Problemei 10 . Concluzionam ca, daca x u = y v = z t, unde "" este una dintre aritmetice +, , , :, iar u, v, t rezprezint o permutare oarecare a numere a atunci piramida este regulat. Se obin n acest fel 24 de probleme distinc a t

Intuirea proprietilor operaiilor aritmeti at t utiliznd metodele figurative


Petru ASAFTEI 1

Este cunoscut faptul c n clasele primare se evideniaz, fr utilizarea a t a aa giei, unele proprieti ale operaiilor lor. Cunotinele clare despre aceste p at t s t se obin daca se pornete de la un suport material intuitiv. n cazul de f t s rialul intuitiv va fi format din reele de puncte n plan, structurate n a t s elevii s sesizeze proprietile operaiilor, bineneles, cu ajutorul explica a at t t de nvaator. Vom construi modele gurative pentru urmatoarele propriet t tativitatea nmulirii, asociativitatea nmulirii, distributivitatea nmuli t t t adunare i distributivitatea nmulirii faa de scdere. s t t a

2. Asociativitatea nmulirii t Raionamentul matematic are la t baz mprirea reelei n dou suba at t a mulimi de puncte, fiecare format din 3 t a linii i 4 coloane. Prin analogie cu cazul s 2x3 2x3 2 precedent, constatam ca 2 (3 4) i s (2 3) 4 reprezint tot att i scriem a s 4x(2x3) sau (2x 2 (3 4) = (2 3) 4. Generalizarea acestei proprietai nu este evidenta. Produsul 2 3 trec t ceea ce nseamna un tablou cu a coloane i b puncte n fiecare coloana. Rep s tablou de c ori i obinem c (a b) = (a b) c. Pe de alt parte, s c s t a a din fiecare tablou o singur coloan; formm n acest fel a tablouri cu a a a puncte n fiecare linie. Ca urmare, obinem a (b c). Numarul total t nu s-a schimbat, ceea ce nseamna ca (a b) c i a (b c) reprezinta s scriem (a b) c = (a b) c.

1. Comutativitatea nmulirii t Aceast configuraie de puncte poate fi privit n dou moduri: a t a a primul, de jos n sus, care pune n evidena c se repet 3 t a a linii cu cte 4 puncte n fiecare linie, ceea ce nseamna 3 4; al doilea de la stnga la dreapta, care pune n evidena ca t se repet 4 coloane cu cte 3 puncte n fiecare coloan, ceea ce nseamn a a a Generalizarea se face n mod natural, tot pe baza unei reele de puncte t formata din a linii i b coloane. Prin analogie cu raionamentul preceden s t a b = b a.

2x(3x4)

3x4

3. Distributivitatea nmulirii faa de t t adunare Se constat c 2 (3 + 4) i (2 3) + a a s (2 4) reprezinta tot att i scriem 2 s (3 + 4) = (2 3) + (2 4).
1

3x4

3x4

2x3

2x4

(2x3)+(2x4)

Profesor, S coala Normal "Vasile Lupu", Iai a s

4. Distributivitatea nmulirii faa de scadere t t Expresiile 2 (4 3) i (2 4) (2 3) reprezs int tot att i scriem 2 (4 3) = (2 4) (2 3). a s Pentru generalizare trecem 2 4 n a b i 2 3 n s a c, cu b c. n continuare, raionamentul este t analog. Obinem 2 (4 3) = (2 4) (2 3). t

Pentru generalizare trebuie s trecem 23 n ab, iar 24 n ac i vom a s (a b)+(a c) i a(b + c) reprezinta tot att. Scriem a(b + c) = (a b s

4 (2x4)-(2x3)

Observaie. Utilizarea modelelor gurative, de exemplu cazurile 3 t tribuie la nelegerea algoritmului de rezolvare a exerciiilor care conin p t t t

ASOCIA TIA RECREA MATEMATIC TII

La data de 14.02.2005 a luat ina ASOCIATIA RECREATII t MATICE, cu sediul n Iai (str. Aurora, nr. 3, sc. D, ap. 6), avnd ca s jinirea activitatilor de matematica specice nvatamntului preuniversita zarea si desfasurarea de activitati care sa contribuie la dezvoltarea gustu matematica n rndurile elevilor, profesorilor si iubitorilor de matematica larea preocuparilor si cercetarilor originale. Obiectivele majore pentru atingerea scopului propus sunt: 1. editarea unei reviste destinata elevilor i profesorilor revista " s Matematice"; 2. fondarea unei biblioteci de matematic elementar biblioteca " a a Matematice"; 3. alcatuirea unei colecii de cari de matematica elementara, cari d t t t i aate la prima apariie Colecia "Recreaii Matematice". s t t t Poate deveni membru al Asociaiei, printr-o simpla completare a une t orice perosana care adera la obiectivele acesteia si sprijina realizarea lor. Membri de onoare ai Asociaiei, academicienii: t

Constantin Cord Radu Miron

Vizitai pe Internet revista "Recreaii Matematice" la t t

http://www.recreatiimatematice.uv.ro

Jensens inequality for non-convex functio


Holger STEPHAN 1

Jensens inequality is well known: Let P be a convex function on x f (x) x1 , ..., xn I and qi 0 are weights with i qi = 1. Then, Jensens inequ
n X i=1

qi f (xi ) f

holds. This inequality is in some sense equivalent to the denition of convexi why, it is not well known that Jensens inequality holds even for non-convex (moreover, this is believed to be false). Lets start with an easy contest problem (from the journal Die wur 05/2005, problem 19): Let x1 ,. . . , xn be positive real numbers with 0.075 h(x) 0.05 x1 + +xn = n. Prove the following inequality.
n X i=1

n X i=1

qi xi

We use Jensens inequality with the function h(x) =

X 2 1 1 + xi 3 + xi i=1

0.025

(1)

10 -0.025 -0.05 -0.075

20

2 1x 1 = . 1+x 3+x (1 + x)(3 + x)

and therefore The result is true but the proof is false, because h(x) is n (1). for x > 1 + 3 2 + 3 4 = 6.69464 . . . (see the picture). Why does the holds, nevertheless? To investigate this question, we try to prove Jensens n P deriving an identity for it. We x x0 = qi xi and dene a function
i=1

1 and the weights qi = n . We get X n n n 1 X 2 1 1 1X xi = h(1) = = h(xi ) h n i=1 1 + xi 3 + xi n n i=1 i=1

g(x) =

(for x = x0 we dene g(x0 ) = f 0 (x0 )). Now, we set n n n X X X X= qi f (xi ) f qi xi = qi f (xi ) f (x0 )
i=1 i=1 i=1
1

f (x) f (x0 ) x x0

and check, when does X 0 hold. Using easy calculations, we obtain


Cercettor dr., Institutul Weierstrass, Berlin (e-mail: stephan@wias-berlin.de) a

X= =

n X i=1 n X

qi f (xi ) f (x0 ) =

n n X X f (xi ) f (x0 ) qi f (xi ) f (x0 ) = qi (x xi x0 i=1 i=1 n X i=1 n n X X g(xi )qi xi qj qj xj = j=1 n X j=1

= =

i,j=1

i=1 n X

qi g(xi ) (xi x0 ) = qi qj xi g(xi )


n X

qi qj xj g(xi ) =

1j<in

We see that X 0 holds, if the function g(x) is monotone increasing. Th g(x) is called slope function. g(x) is the slope of the secant through the x0 and x. If f (x) in convex, g(x) is increasing for any point x0 . But this only case. Function g(x) is increasing, if looking from the point (x0 , f (x graph of f (x), no point of f (x) "lies in the shadow" of the graph. This c for some points x0 even if the function is not convex. The function h(x) a example (here is x0 = 1). An other amazing example are polynomials of fourth order typical non-convex functions. We f(x) consider such a function f (x) and its inection points (the x-coordinates are x1 and x2 ). The e2 inection point tangents e1 and e2 intersect the graph of f (x) in points with x-coordinates x3 and x4 . Now, Jensens inequality holds for x0 x3 or x0 x4 . x4 x2 x1 The typical proof of Jensens inequality starts from the inequality functions (xi x0 )f 0 (x0 ) f (xi ) f (x0 ) (xi x0 )f 0 (xi ) Multiplying by qi and adding up over i we get n n n n X X X X 0 qi (xi x0 )f (x0 ) qi f (xi ) f (x0 ) qi qi (xi x0 )f
i=1 i=1 i=1 i=1

Thus, we get the identity n n X X X= qi f (xi ) f qi xi =


i=1 i=1

qi qj (xi xj ) g(xi ) g(xj ) .

i,j=1

i,j=1

qi qj xi g(xi ) xj g(xi ) =

1j<in

qi qj (xi xj ) g(xi ) g(x

The left hand side in zero because of

i=1

and transforming the right hand sind in a similar way as above (3) we ob n n X X X 0 qi f (xi ) f qi xi qi qj (xi xj ) f 0 (xi ) f 0 (x
i=1 i=1 1j<in

n P

qi = 1 and

i=1

n P

qi xi = x0 . The m

This is similar to identity (3), but holds only for convex functions. Moreover, identity (3) is very useful, if we are not only interested i inequality, but if we want to estimate the dierence on the left hand side

Concursul de Matematic Al. Myller a


Ediia a V-a, Iai, martie 2007 t s Clasa a IV-a

1. Daca a = 2 i b + c = 5, calculai a b + a c. s t 2. Suma a doua numere este 75. Daca din primul scadem 45 i la al doile s 10, obinem numere egale. S se afle cele dou numere. t a a 3. Aflai a b c, unde b este dublul lui a, a este dublul lui c, iar a + t 4. Produsul a 7 numere naturale este 7. Aflai suma numerelor. t 5. Pentru numerotarea paginilor unei reviste sunt necesare 135 cifre. C are revista? 6. Numerele naturale nenule distincte a i b sunt cele mai mici p s mpririle a : 2 : 2 : 2 : 2 i b : 2 : 2 : 2 : 2 se efectueaz exact. Aflai a + b at s a t 7. Aflai ce numar se marete cu 2007 cnd adaugam la dreapta lui ci t s 8. Un profesor are un numar de caiete i jumatate din numarul acestor s Distribuind cte 3 caiete fiecrui elev mai rmn 3 caiete, iar distribu a a creioane fiecrui elev, rmn 3 elevi fr creioane. Care este numrul elev a a aa a 9. Alba ca Zapada i cei apte pitici au suma vrstelor 185 de ani. Stiin s s ca Zapada este cu patru ani mai tnara dect cel mai tnar pitic, iar vrste sunt numere naturale consecutive, aflai vrsta Albei ca Zpada. t a 10. Setil bea la o mas obinuit 5 butoaie de ap, iar cnd este ns a a s a a butoaie. Daca a baut 39 de butoaie, la cte mese a fost Setila nsetat? 11. Stefan trebuie sa nmuleasca numarul 223 cu un numar format t consecutive. Din greeal, el a schimbat ordinea acestor cifre consecutive s a s alt produs. Care este diferena celor dou produse? t a 12. Primul termen al unui ir de numere naturale este 1 + 2 + 3, al s 2 + 3 + 4 + 5, al treilea este 3 + 4 + 5 + 6 + 7 i aa mai departe. Aflai val s s t de-al 50-lea termen al irului. s 13. Suma a 2 numere A i B este 150. Dac tergem una din cifrel s a s obine B. Gsii toate numerele A i B cu aceast proprietate. t a t s a 14. O minge se ridica la trei sferturi din distana de la care cade. D t data i se da drumul de la 64 metri, care este distana totala parcursa de m t cnd atinge pmntul a patra oar? a a 15. Pe o tabl sunt scrise numerele 1, 2, 3, . . . , 100. Lucian i Dan a s rnd numerele de pe tabla astfel: mai nti Lucian terge numerele de s impare, apoi Dana terge numerele de pe locurile pare din irul ramas. Lu s s din nou numerele de pe locurile impare din noul ir i aa mai departe. s s s ultimul numr rmas pe tabl? a a a

Clasa a V-a
1. Care sunt elementele mulimii A = {2x N | 100 x < 103}? t

2. Care este paritatea numrului N = (3n + 2)(2n + 3), dac n este a a natural impar? 3. Determinai suma celor mai mari doua patrate perfecte impare de t 4. Gsii numrul natural de dou cifre egal cu triplul sumei cifrelor s a t a a 5. Care este cardinalul mulimii M = {x N | 2x < 1025}? t 42 4242 424242 6. Calculai suma S = t + + . 43 4343 434343 7. La un concurs se dau 30 de probleme. Pentru fiecare raspuns corect 5 puncte, iar pentru fiecare rspuns greit se scad 3 puncte. Cte rspuns a s a a dat un elev care a obinut 94 de puncte? t 8. Scriei numrul 200 ca sum de puteri ale lui 2. t a a 9. Determinai toate perechile de numere naturale pentru care m2 (n + t 10. Care este cea mai mare fracie de forma t

ab mai mica dect 1 x3y

2007 la scrierea lui n n baza 10? 13. Considerm numerele a = 22005 , b = 22006 , c = 22007 . Care este u a a numrului N = 2a + 3b + 4c? a 14. Stiind ca numerele naturale a i b dau la mparirea prin 2001 rest s t i respectiv 1999, determinai restul mparirii la 667 a numarului 2a + 3b s t t 15. Se d irul de numere naturale 1, 2 3, 4 5 6, 7 8 9 10, . . . . C as 50-lea termen al irului? s

multiplu de 10? 11. Restul mpririi numrului 3a + b la 23 este 11, iar ctul este c at a paritatea ctului mpririi lui 3a + b c la 11. at 12. Fie numarul n = 9 + 99 + 999 + + | 99999 99. Cte cifre de 1 {z }

Clasa a VI-a

1. Numrul x pentru care x2 + x y 4 = 0, unde x, y N i y nu a s este . . . 2. Raportul dintre un numar natural i inversul sau este 9; atunci su s numar i invers este . . . s 3. Dintre numerele 729100 i 246150 mai mare este . . . s 4. Trei frai vor s-i mpart ntre ei 240 de nuci. Ei hotrsc s t as a aa a n pari direct proporionale cu numerele 2, 3 i 5 i apoi sa dea cte 25 t s s t primite surorii mai mici, care astfel a primit . . . nuci. 5. Numrul de trei cifre egal cu de 15 ori suma cifrelor sale este . . . a 6. Cel mai mic multiplu de 5 cu suma cifrelor 44 este . . . 7. Un muncitor lucreaza piese trei zile astfel nct 20% din producia f t 1 t este ct din producia zilei urmatoare. Astfel, a treia zi a lucrat cu 55 6 mult ca n prima zi. A treia zi a lucrat . . . piese.

11. Numarul natural care are exact trei divizori naturali, iar suma este 183 este . . . 12. Cel mai mic numar natural de trei cifre care mparit pe rnd la 2 t d resturi diferite nenule este . . . a 13. Un unghi XOY cu masura de 179 este mparit de 17 semidr t unghiuri, cu msurile exprimate prin numere naturale nenule, distincte a mare unghi posibil dintre cele 18 este . . . 14. Fie triunghiul ABC, (AM bisectoarea unghiului BAC i AD s \ = m(DAM ) i m(BAC) + m(AM D) = 18 \ s \ \ D, M (BC). Daca m(ABC) \ BAC are msura . . . a 15. Fie mulimea A = {1, 4, 7, 10, 13, . . . , 100} i B o submulime t s t elemente avnd proprietatea c, la orice alegere a celor n elemente din A a cel puin doua cu suma 104. Valoarea minima a lui n este . . . t

8. Stiind c raportul dintre suplementul sumei a dou unghiuri adiacen a a 1 suplementelor lor este , masura unghiului format de bisectoarele lor este 3 9. Cel mai mic numr de forma xyz (x < y < z) divizibil cu 12 este . . a o n n+80 10. Cardinalul mulimii M = x N | x = n10 , n N este . . . t

Juniori (Clasele VII-VIII)

1. a) Artai c oricare ar fi n N, numrul n2 +2n+2007 nu este ptr a t a a a b) Fie k un numar natural par, k 4. Sa se arate ca exista un numar a a astfel nct numrul n2 + 2n + k s fie ptrat perfect. a 2. Consideram n drepte concurente n punctul P . Dreptele determin punctului 2n unghiuri cu interioare disjuncte, fiecare unghi avnd msura a de 17 . a) Sa se afle n. b) Sa se arate ca cel puin doua dintre cele n drepte sunt perpendicula t 3. Fie ABC un triunghi dreptunghic n A i fie M mijlocul laturi s perpendiculara n A pe AM consideram un punct D astfel nct segment AB s aib un punct comun, notat P . Fie E proiecia punctului D pe dr a a t [ Sa se arate ca BP M = EAC. \

4. La un concurs de matematic particip n elevi, n 5, iar proba a a probleme. Fiecare elev a rezolvat exact 3 probleme. Pentru orice grup exista o aceeai problema rezolvata de fiecare elev din grup. Sa se arate c s aceeai problem rezolvat de toi concurenii. s a a t t

Seniori (Clasele IX-XII)

1. Rezolvai n mulimea numerelor ntregi ecuaia x3 y 3 = 2xy + 7. t t t 2. Fie a 2 un numr natural. Considerm irul (xn )n1 , unde xn a a s +a2 + + a2n . Demonstrai c exist o infinitate de numere prime car t a a nici un termen al irului. s

\ 3. Pentagonul convex ABCDE are proprietile: AB = BC, m(ABE)+ at \ si m(AEB) + m(BDC) = 180 . Demonstrai c ortocentrul tr \ \ m(DBE) t a BDE se afl pe dreapta AC. a

4. Fie n 2 un numar ntreg. Demonstrai ca, oricum am colora cu d t n3 + 5n o mulime de t numere ntregi consecutive, exist o submulime m a t 6 {a1 , a2 , . . . , an } astfel nct 1 a2 a1 a3 a2 an an1 .

Concursul de matematic Florica T. Cmp a


Etapa judeean, 17-18 februarie 2007 t a Clasa a IV-a

1. Suma a patru numere este 282. Primul numr, ptrimea celui de-a a a dublul celui de-al patrulea sunt cu 4 mai mari dect al treilea numr. Aflai a t 2. Gasii cel mai mic i cel mai mare numar cu cifre distincte, tiin s s t cifrelor fiecaruia este 17. 3. Koallo este un copil care locuite n drguul orel Oloko din Niger s a t as iubete matematica, recent a observat ca daca atribuie cte o alta cifra fieca s literele K, O, A, L i nmulete numarul de 5 cifre corespunzator numelui s t s cu 11, obine numrul corespunztor numelui su. Care sunt cifrele atribu t a a a litere?

Clasa a V-a

1. a) Fie numrul 1234567891011121314...200520062007. S se supri a a cifre astfel nct numrul rmas s fie cel mai mare posibil. a a a b) Sa se determine cel mai mic numar natural de forma a1 a2 ...ak , k verifica relaia 7a1 a2 ...ak = 5 a1 a2 ...ak 7. t 2. Pentru rezolvarea temei de vacana, bunica i d Anei cte o surpr t a imediat ce termin de rezolvat o nou problem. Ana constat de fiecar a a a a adunnd cifrele numarului de surprize primite pna atunci cu cifrele num probleme care i-au rmas de rezolvat, obine 11. Cte surprize Barbie va a t la terminarea temei? Mihael 3. n figura alaturata avem un sistem de drumuri care F leag localitile A, B, C, D, E, F , G. Fiecare drum existent a at ntre dou localiti vecine are lungimea un numr ntreg de a at a A kilometri. O localitate se numete "nod impar" daca suma s lungimilor drumurilor care pleaca din ea este un numar impar de kilometri. S se arate c localitile nu pot fi toate "noduri a a at B impare". Petr

Clasa a VI-a

1. O reprezentana a unui constructor de autoturisme a vndut n 2005 t de 200 de maini. n 2006 vnzarile au crescut cu 28%, iar n 2007 este pre s scdere cu 15% faa de 2006. Cte autoturisme se prognozeaz a fi vndut a t a Gab 2. n figura alaturata este desenat un pentagon ABCDE n care DAC DBE, ACE BEC i [AC] [BE]. s Fie {I} = AD CE i {J} = BD CE. s a) Aratai ca AIC BJE. t b) Demonstrai c [AD] [BD]. t a
B A I E

3. a) Cte numere naturale a1 a2 ...an (n 2), formate din cifre nenu prietatea ca toate numerele a1 a2 , a2 a3 , ..., an1 an sunt patrate perfecte? Adrian b) Alin arunca doua zaruri, iar Vlad arunca trei zaruri. Fiecare vrea c numerelor obinute de el sa fie patrat perfect. n care dintre cele dou t probabilitatea atingerii obiectivului este mai mare? Monica

Clasa a VII-a

1. a) Se dau dou numere ntregi x i y. Cu ajutorul lor se formeaz a s a numere n felul urmtor. Primul numr este egal cu x. Al doilea numr e a a a x + y. Al treilea numar este egal cu diferena dintre al doilea i primul n t s patrulea numar este egal cu diferena dintre al treilea i al doilea numar. t s numr este egal cu diferena dintre al patrulea i al treilea numr i aa m a t s a s s S se afle primele 12 numere ale irului i al 2007-lea numr. a s s a b) Alegei n faa fiecaruia dintre numerele 1, 2, 3, ..., 2006 unul dintre se t t sau "" astfel nct numarul |1 2 3 ... 2006| sa ia cea mai mic Determinai aceast valoare. t a 2. Un iaht trebuie s parcurg un traseu sub form de triunghi echilat a a a plecnd din A. Pe iaht se afla Sam, Bob i John, care ncearca sa n s viteza vasului, dar avnd toi trei ru de mare, nu reuesc s rein dect t a s a t a incomplete. Astfel, Sam a observat c iahtul a parcurs primele trei sfertur a n 3 ore i jumtate, John i-a notat doar c ultimele trei sferturi ale dr s a s a fost parcurse n 4 ore i jumatate, iar Bob a observat ca pentru a parcurg s de la B la C au fost necesare cu 10 minute mai mult dect pentru distana t B. Presupunnd c, pe fiecare latur a triunghiului, iahtul a avut vitez a a a s se determine durata parcurgerii ntregului traseu. a 3. Consideram un patrat ABCD cu latura de 9 cm i punctele E AD s astfel nct A (ED), C (BF ), AE = 9 cm, CF = 3 cm. O furnic cel mai scurt drum de la E la F care traverseaz ptratul dup o parale a a a Construii drumul pe care l parcurge furnica i aflai lungimea lui. t s t

Clasa a VIII-a
1. Considerm 9 puncte dispuse ca n figura alturat: a a a

O furnic pleac din A i ajunge n B trecnd prin fiecare punct a a s o singura data, pe un drum fara autointersecie i mergnd pe lat s turile sau diagonalele patratelor mici care se pot forma cu punctele din reea. Dac lungimea laturii ptratului mic este 1, artai c t a a a t a lungimea minim drumului strbtut de furnic este 8 i cea A a a a a a s maxima este 4 + 4 2. Gheorg

2. Fie piramida triunghiular V ABC astfel nct AV BV , BV CV a i care are produsul oricaror doua muchii opuse egal cu P . Asociem fieca s a piramidei cea mai mica dintre ariile triunghiurilor care au drept baza m spectiv i vrful pe muchia opus a piramidei. as a a) Demonstrai c V este egal deprtat de muchiile bazei ABC. t a a b) Daca suma muchiilor piramidei este S i distana de la V la una dintr s t bazei este d, calculai n funcie de S i d suma celor ase arii asociate t t s s piramidei. Julieta

3. Fie n A = a R | a = a1 3 + a2 ( 3)2 + + a2007 ( 3)2007 ; ai {1, 1} , i =

a) Determinai numrul de elemente raionale din mulimea A. t a t t b) Determinai numrul elementelor mulimii A. t a t Cristian Lazr i Claudiu- te a s S

Etapa interjudeean, 1-3 iunie 2007 t a Clasa a IV-a

1. Suma unor numere naturale consecutive este 90. Unul dintre nume Care sunt celelalte numere? Justificai rspunsul! t a 2. Se d irul de numere naturale 5, 10, 15, 20, 25, . . . Care este prim as din ir cu suma cifrelor 27? s 3. Lucian-Georges ia dintr-o cutie de fiecare data mai multe bombo luase data precedent i, n 5 di, a luat n total 31 bomboane. Cte bo as at luat a patra oar, dac prima dat a luat de trei ori mai puine bomboa a a a t cincea oara?

Clasa a V-a

1. Un dreptunghi cu n linii i m coloane este mparit n patraele 11 s t t linie coloram primul patrael, pe a doua linie coloram primele doua patr t treia linie primele patru ptrele, pe a patra linie primele opt ptrele a at a at pn cnd pe a n-a linie se vor colora toate ptrelele. Stiind c numr a a at a a patraele colorate este 1023, aflai cte linii i cte coloane are dreptungh t s t Mihaela

2. Pe o tabl de ah se gsesc 31 de pioni. Artai c pe tabl exist a s a a t a a a avnd una dintre formele din figura de mai jos, pe care nu se afla niciun p

Tama 3. Un tlhar mparte prada cu tovaraul sau de rautai. Dintr-un sacu s t monede de 10 bani, el scoate pe rnd cte o moned, numrnd: "Una la t a a mine; a doua la tine, una, dou la mine; a treia la tine, una, dou, trei la a a i la fiecare numr rostit, aeaz cte o moned n faa sa sau a tovarului s a s a a t as n sacule sunt 6000 de monede, aflai ce suma (n lei) revine fiecarui tlh t t Gab

Clasa a VI-a

1. Dou coli de hrtie care au aceleai dimensiuni, L = 9 dm i l = a s s taie n dreptunghiuri, prima prin drepte paralele cu lungimea, iar a doua p paralele cu laimea, numarul i dimensiunile acestor dreptunghiuri ind arb t s se arate c exist cel puin o situaie n care suma perimetrelor tuturor d a a t t urilor obinute prin tiere din prima coal este egal cu suma perimetrel t a a a dreptunghiurilor obinute din a doua coala. t Petr 2n + 7 2. a) Fie mulimea A = n N | t fracie reductibil . S s t a a 3n + 1 trei elemente din mulimea A. t Enache b) Cum planteaza un pomicultor 10 copaci pe 5 rnduri, astfel nct sa f pe fiecare rnd?

3. Un evantai are 37 de spie, astfel nct unghiul dintre oricare dou s t a rate are msura de 5 . S se arate c, dac se aleg oricare 12 spie, excep a a a a t prima, se formeaza cel puin trei unghiuri de masuri egale. t Do

Clasa a VII-a

1. Dan are trei jetoane pe care sunt scrise trei numere reale pozitive, ia alte trei pe care sunt scrise inversele lor. Dan i da Anei un jeton pe ca l asociaz cu un altul dect inversul su. Apoi, Ana i d lui Dan un a a a care acesta l asociaz cu un altul dect inversul su. Stefan primete de a a s copii jetoanele ramase i constata ca nici numerele sale nu sunt inverse unu s Fiecare anuna suma numerelor pe care le are pe cele doua jetoane, respe t 7 . Artai c produsul numerelor scrise pe cele trei jetoane deinute ini a t a t t 3 este 1. Mihael 2. Fie I punctul de intersecie al diagonalelor trapezului ABCD, A t AB = a cm, CD = b cm, a > b. Paralela prin I la AB intersecteaza pe a n punctele P , respectiv Q. Trei mobile M1 , M2 i M3 pleac simultan, s cel mai scurt, din punctele B, Q, C spre punctele A, P respectiv D, ast viteza constanta b cm/s, M2 cu viteza constanta a cm/s pna n I i apoi, f s

cu viteza constant b cm/s de la I la P , iar M3 cu viteza constant a cm a a ordinea n care ajung cele trei mobile n punctele A, P , respectiv D. Claudiu Ste

3. Se considera n plan noua puncte, astfel nct din oricare trei se po puin doua cu distana dintre ele mai mica sau egala cu 1. Aratai ca exis t t t de raz 1 care conine cel puin 5 puncte din cele considerate. a t t Gheorghe

1. Pe tabla sunt scrise numerele 3 1, 3 + 1, 2. Se terg cele trei s se scriu n locul lor cele trei medii geometrice a cte doua dintre numere. procedeul cu noile numere. Este posibil ca dup mai muli pai pe tabl s a t s a numerele: p p a) 2 3 2, 2, 2 3 + 2; b) 2 3, 2 + 3, 4? Monica 2. Triunghiul alaturat este considerat fix. n cte moduri putem aeza numerele 1, 2, 3, 4, 5, 6 n cerculee, astfel nct suma nus t merelor de pe fiecare latur a triunghiului s fie aceeai? a a s Petru Asaftei

Clasa a VIII-a

3. Exist o ar K a cubarzilor. Un cubard are corpul de forma u a t a anten ce pornete dintr-un vrf al cubului, o coad ce este diagonal a a s a a cubului i o gur care este exact la mijlocul unei muchii a cubului. Un s a poate rostogoli dupa voie, se poate umfla sau strnge dupa plac, i poate r s cum vrea. Nu exist doi cubarzi pe care i-am putea suprapune nct sa a cozile, gurile i antenele. Care este numrul maxim al cubarzilor? s a Dan

Olimpiada Balcanic de Matematic Juni a a (JMBO)

Ediia a XI-a, Sumen (Bulgaria), 23-30 iunie 200 t

La aceasta ediie a concursului au participat 11 ari: Albania, Bosnia t t govina, Cipru, Macedonia, Grecia, Moldova, Muntenegru, Romnia, Serb Bulgaria 1 i trei echipe n afara concursului: Bulgaria 2, Kazahstan i s s Varna. Delegaia Romniei a fost condusa de prof. Dinu Serbanescu i pro t s Fianu. Echipa arii noastre a fost compusa din urmatorii elevi: Chinde t Bumbacea Radu (din Bucureti) medaliai cu aur (ambii au acumulat s t de puncte), Tiba Marius (Iai), Ciolan Emil Alexandru (Slatina), s Alexandru (Bucureti) i Filip Laurian (Arad) medaliai cu argint. s s t

Elevul Tiba Marius a fost primul din medaliaii cu argint, printre toi c t t ce au luat aceasta medalie. Se preconizeaza ca Albania sa gazduiasca urmatoarea ediie a JMBO. t

Enunurile problemelor t
2

1. Fie a un numr real pozitiv astfel nct a3 = 6 (a + 1). S se arate a a x + ax + a2 6 = 0 nu are soluii reale. t 2. Fie ABCD un patrulater convex avnd DAC = BDC = 36 , C i BAC = 72 . Diagonalele AC i BD se intersecteaz n punctul s a s determine msura unghiului AP D. a 3. Se consider 50 de puncte n plan, oricare trei necolineare. Fiec a aceste puncte este colorat folosind una dintre patru culori date. Sa se arat o culoare i cel puin 130 de triunghiuri scalene cu vrfurile n puncte s t culoare. 4. S se arate c dac p este un numr prim, atunci 7p + 3p 4 nu e a a a a perfect.

Not. Timp de lucru: 4 ore i 30 minute. Fiecare problem este no a s a puncte.

Soluiile problemelor t

prezentate de elevul Marius Tiba 6 1. Observam ca a3 = 6 (a + 1) a a2 6 = 6 a2 6 = (a est a Atunci, ecuaia din enun se scrie sub forma t t 6 x2 + ax + = 0 a 24 i are discriminantul D = a2 s . S presupunem, prin reducere la a a a ecuaia dat sau ecuaia (1) are soluii reale. Acest fapt este echivalent c t a t t D 0. Avem D 0 a3 24 0 6 (a + 1) 24 0 6 (a 3) 0 a 3. 6 = a a2 6 3 (9 6) = 9,

Atunci,

adic 6 9, absurd. a

2. Fie un punct Q pe dreapta AC ce respect ora \ = 18 . ndinea A C Q i este astfel nct m(CBQ) s \ \ truct QAD QBD, patrulaterul ABCD este inscriptibil. \ \ Ca urmare, m(QDB) = m(QAB) = 72 . Deducem c a \ = 36 = m(CDB). Aadar, C este centrul cercu\ m(QDC) s 1 \ \ lui nscris n 4DBQ i, deci, m(DQP ) = m(DQB) = 36 . s 2 \ n sfrit, m(AP D) = 108 (unghi exterior triunghiului s P DQ).

D P

3. Se constat uor c exist cel puin 13 puncte colorate la fel (dac a s a a t a culoare ar exista cel mult 12 puncte colorate cu aceasta, atunci n plan a mult 48 puncte). Vom arata ca exista 130 triunghiuri scalene cu vrfu aceste 13 puncte. ntr-adevr, cu cele 13 puncte putem forma 13 12 11 : 6 = 286 triun a numrul celor 13 12 11 triunghiuri, un triunghi este socotit de 6 ori). a acum ca, daca consideram un segment [AB], exista cel mult doua puncte a [AB] sa fie baza de triunghi isoscel formate cu ele. Cum cu 13 puncte p 13 12 = 78 segmente, rezult c exist cel mult 78 2 = 156 triunghiuri is a a a 2 urmare, exist cel puin 286 156 = 130 triunghiuri scalene cu vrfurile a t de aceast culoare. a

4. Daca p este par, atunci p = 2 i se verifica direct ca numarul din s ptrat perfect. a Dac p este impar, atunci p este fie de forma M4 + 1, fie M4 + 3. n ca a p = M4 + 1, rezulta ca numarul 7p + 3p 4 = M4 + 2 (ultima cifra a lui 3p nu este patrat perfect. Daca, nsa, p = M4 + 3, sa notam k 2 = 7p + 3p 4. imediat c k2 = Mp 1 (se utilizeaz mica teorem a lui Fermat), ech a a a p | k2 + 1. De aici deducem c p | 1 (tim c p prim i p | a2 + b2 p | a s a s ceea ce este absurd. Deci, nici n acest caz numarul din enun, nu-i patra t

Vizitai pe Internet revista "Recreaii Matematice" la t t

http://www.recreatiimatematice.uv.ro

Soluiile problemelor propuse n nr. 2 / 20 t


Clasele primare

20 P.114. n piramida alaturata unele numere s-au sters de-a 8 1 lungul timpului. Putem sa le punem la loc? 3 (Clasa I ) Ionela Brgan, elev, Iai a a a s 1 Soluie. 20 = 8 + 12, 28 = 12 + 16 etc. t P.115. Daca din prima lada iau 2 mere si le pun n lada a doua, din a iau 3 mere si le pun n lada a treia, iar din lada a treia iau 4 mere si le pu lada, atunci n fiecare lada voi avea cte 34 mere. Cte mere au fost n fie (Clasa I ) Mariana Nastasia, e Soluie. n prima lad erau 34 4 + 2 = 32, n a doua 34 2 + 3 = 3 t a treia 34 3 + 4 = 35 (mere). P.116. Luni, mama pune ntr-un co cteva mere. Joi, ea gasete n c s s de mere. Cte mere a pus mama n co, stiind ca, n fiecare zi din acea s s Mihai, cel mai mare dintre frai, mparte frailor mai mici cu un mar mai t t ziua precedenta si ca joi el mparte 5 mere? Cte mere mai ramn n co dupa mparirea merelor? t (Clasa a II-a) Inst. Maria R Soluie. Merele luate pn joi: 5 + 4 + 3 + 2 = 14. Merele existente t a n co: 20 + 14 = 34. Merele mparite vineri i smbata: 6 + 7 = 13. Mer s s t smbata n co: 20 13 = 7. s P.117. n exerciiul 1111111 = fiecare casua poate fi nlocui t t sau . Ct poate fi rezultatul acestui exerciiu? t (Clasa a II-a) Diana Tnsoaie, e a a Soluie. Deoarece avem un numar impar de 1, putem obine ca rezult t t sau 5 sau 7. Exemplu: 1 1 + 1 1 + 1 1 + 1 = 1; 1 1 + 1 1 + 1 + 1 1 + 1 + 1 + 1 + 1 + 1 = 1; 1 + 1 + 1 + 1 + 1 + 1 + 1 = 1. P.118. Calculai a + b + c + d, stiind ca a b = 5 si c d = 15. G t posibilitatile. (Clasa a III-a) nv. Rica Buct a Soluie. a + b + c + d poate fi: 1 + 8 + 1 + 15 = 25, 1 + 8 + 5 t 2 + 4 + 1 + 15 = 22, 2 + 4 + 3 + 5 = 14. P.119. Produsul a 10 numere naturale este 40. Aflai cea mai mica t mare valoare a sumei celor 10 numere. (Clasa a III-a) nv. Mirela Buburuzanu, Tome Soluie. 40 = 2 2 2 5 = 2 2 10 = 2 20 = 4 10. Dup t a numerelor este 9 1 + 40 = 49, 6 1 + 2 3 5 = 17, 7 1 + 2 2 + 8 1 + 2 + 20 = 30, 8 1 + 4 10 = 22. Suma minim este 17, iar cea m a 49. P.120. Se considera numerele 1, 2, 3, 4, 5, . . . , 49. Care este cel mai m de numere pe care putem sa alegem dintre acestea astfel nct suma or

dintre ele sa se mparta exact la 9? (Clasa a III-a) nv. Felicia-Petronela Leanu, Ceplen Soluie. Numerele 3, 12, 21, 30, 39, 48 satisfac condiia ceruta la proble t t mai introducem cel puin un numr, de exemplu 17, atunci putem gsi t a a trei numere care nu se mparte exact la 9, de exemplu 3 + 12 + 17. Num a de numere este 6.

P.121. Ceasul lui Andrei o ia nainte cu 20 secunde pe ora. El a potr luni la ora 8 si a citit din nou ceasul lunea urmatoare la aceeai ora. St s aceasta durata ceasul nu a funcionat permanent, iar la ultima citire ara t 50 min, sa se afle ct nu a funcionat ceasul. t (Clasa a IV-a) Paula Boranu, e s Soluie. De luni, ora 8, pna lunea urmatoare, ora 8, sunt 168 ore. D t ar fi funcionat permanent, atunci ar fi avut un avans de 168 20 : 60 = 56 t n realitate, avansul este de 50 minute. Ceasul nu a funcionat (56 50)60 t (ore).

P.122. La Concursul de matematica "Fl.T.Cmpan", etapa judeeana, t pat 100 elevi de clasa a IV-a, care au avut de rezolvat 3 probleme. Daca 7 rezolvat bine prima problema, 69 a doua problema si 64 a treia problema, ca macar 3 elevi au rezolvat corect toate cele trei probleme. (Clasa a IV-a) Anca Cornea, e Soluie. Dac fiecare elev a rezolvat bine numai cte dou problem t a a numarul de rezolvari corecte este 100 2 = 200. n realitate numarul d corecte este 70 + 69 + 64 = 203. Deoarece 203 200 = 3, nseamna ca ma au rezolvat corect toate cele trei probleme. P.123. ntr-o cutie sunt 34 bile, din care unele cntaresc cu 1 g mai m fiecare bila cntarete un numar natural de grame, iar masa tuturor bilelor s sa se afle cte bile sunt mai grele. (Clasa a IV-a) Petru Asa Soluie. Daca o bila mai grea cntarete 3g , atunci masa maxima est t s 3g 33 = 101g < 113g. Dac o bil mai uoar cntrete 4g, atunci ma a a s a a s este 4g 33 + 5g 1 = 132g > 113g. nseamn ca o bil mai grea cn a a Daca fiecare bila cntarete 3g, atunci masa tuturor bilelor este 3g 3 s 113g 102g = 11g; 4g 3g = 1g. Numarul bilelor mai grele este 11 : 1 =

Clasa a V-a
V.71. Comparai numerele 3300003 si 2450004 . t Soluie. Avem: t

Lucian Tuescu t

3300003 = 3 3300002 = 3 9150001 > 3 8150001 = 3 2450003 > 2 2450003 =

V.72. Fie mulimile A, B astfel nct A B, |P (A)| 60, |P (B)| t se determine |A| si |B|. (Prin |X| am notat cardinalul mulimii X.) t Petru Asa Soluie. Cum A B, atunci |A| |B|, de unde 60 2|A| 2|B| t nem ca 6 |A| |B| 8, deci (|A| , |B|) {(6, 6) ; (6, 7) ; (6, 8) ; (7, 7) ; (7,

V.73. Sa se scrie numarul 20062005 ca o suma de sase patrate perfect Ionel Nech Soluia 1. Avem: t 20062005 = 2006 20062004 = 12 + 52 + 72 + 92 + 252 + 352 20062004 2 2 2 = 20061002 + 5 20061002 + 7 20061002 + 9 200610 2 2 + 25 20061002 + 35 20061002 . Soluia 2 (Emanuel Petrescu, elev, Iai). t s 2 2 20062005 = 20061002 4 + 52 + 102 + 112 + 122 + 402 = 2 2 = 20061002 4 + + 20061002 40 .

V.74. Se considera sirul de numere naturale 1, 1, 2, 5, 12, 27, 58, . . . suma primilor 100 de termeni ai sirului. Marius Damia Soluie. Adunnd 0 la primul termen, 1 la al doilea, 2 la al treilea et t irul puterilor lui 2. Deducem ca termenii irului sunt de forma 2n n, n s s primilor 100 de termeni va fi S = 20 0 + 21 1 + 22 2 + + 299 99 = 99 100 = 20 + 21 + 22 + + 299 (1 + 2 + + 99) = 2100 1 =2 2 V.75. Fie x, k N, x 2, k < x. Sa se arate ca (x 1) 12 . . . k (x) + k + 1 = 11 {z . 1 (x) . | .. }
k+1 cifre

Doru Bu

(x1) 12 . . . k (x) + k + 1 = (x1) xk1 + 2xk2 + + (k1) x + k + = xk + xk1 + xk2 + + x + 1 = 11 {z . 1 (x) . | .. }


k+1 cifre

Soluie. Avem: t

= xk xk1 + 2xk1 2xk2 + + (k 1) x2 (k 1) x + kx k +

Clasa a VI-a

VI.71. Fie x, y, z N astfel nct 100x 2006y 2 + 15z = 0. Sa s . . 85. y (x + z) . Dan Nedeianu, Drobeta-Tr . Soluie. Cum 2006y 2 = 5 (20x + 3z) i (5, 2006) = 1, atunci y 2 . 5, t . s Apoi, 15 (x + z) = 17 118y 2 5x i cum (15, 17) = 1, obinem c x + s t a . . ducem c y (x + z) . 5 17, adic y (x + z) . 85. a . a .

VI.72. Fie p N un numar prim. Sa se determine x, y N a p y + N . x2 x D. M. Btineu-Giurgiu, B a t

p y + = a N ; atunci p + xy = ax2 , deci p = x (ax x2 x este prim, atunci x {1, p} i corespunztor gsim soluiile s a a t Soluie. Fie t (x, y) = {(1, a p) ; (p, ap 1) | a N } .

An 7n2 7n mulimea M = n N | t N . 9

VI.73. Fie An = 14 + 1414 + 141414 + + 1414 . . . 14 , n N . Sa se | {z }


2n cifre

Valeriu Braoveanu s

\s \ Atunci unghiurile AOm i DOm0 au complementele egale, deci sunt congruente. Din 4AOB isoscel avem ca AOm \ \ \ prin urmare DOm0 BOm, adic punctele B, O, \ a BOm, O D D sunt coliniare. Cum AO = BO = CO = DO, deducem m i c O este mijlocul lui [BD]. s a \ b) Din relaia deja demonstrat AOm DOm0 rezult t a\ a ca AOn0 DOn0 , deci [On0 este bisectoare n 4AOD isoscel. Obinem c t \ \ mediatoarea lui [AD], adic D este simetricul lui A faa de a doua bisecto a t

n (n + 1) . . 9. Rezulta de aici ca A 5n (n + 1) 9n (n + 1) . 9, . 5 . . n 2 2 2 An 7n (n + 1) N, n N . n concluzie, M = N . 9 VI.74. Consideram doua axe perpendiculare Ox si Oy, precum si cel sectoare ale unghiurilor drepte care se formeaza. Fie A oarecare, iar B, C sale faa de prima bisectoare, respectiv faa de Ox. Rotim segmentul [OC t t lui O cu 90 n sensul acelor de ceasornic si notam cu D extremitatea se rotit. Sa se arate ca: a) B, O, D sunt coliniare si O este mijlocul lui [BD]; b) D este simetricul lui A faa de a doua bisectoare. t Adrian Cordune [ Soluie. a) Din 4AOC isoscel avem ca AOx COx, t [ n y \ \ de unde AOn0 COm0 (aceeai diferena pn la 135 ). s t a

Soluie. Notnd cu s (k) suma cifrelor numrului natural k, avem k t a . . 9, 1414 5 2 . 9, . . . , 1414 . . . 14 5n . 9, prin urm . . Deducem c 14 5 . a . . | {z }
2n cifre

VI.75. Fie ABCD un patrulater convex, O intersecia diagonalelor, M t lui [AB], iar N mijlocul lui [OD]. Sa se arate ca 2PBCN M < PBDC + PA Bogdan Posa i Marius Drgoi, elev s a Soluie. Fie T mijlocul lui [OB]. Cum M , N , T nu pot fi colini t AO + BD c M N < M T + T N = a . n 4CDO, [CN ] fiind median, a 2 CO + CD CN < . Atunci: 2

PBCN M = BC + CN + N M + M B < CO + CD AO + BD AB < BC + + + = 2 2 2 BC + AC + AB BC + BD + CD = + = 2 2 1 = (PABC + PBCD ) . 2

A
M

O
T

Clasa a VII-a

VII.71. Fie 4ABC, AB < AC si D (AC). Fie AE bisectoarea E (BD), F mijlocul lui [AD], {O} = AE BF , {G} = DO AB. Sa s GD k BC AB = CD. Carmen Daniela Tama Soluie. Cu teorema bisectoarei n 4ABD, obinem c t t a BE AB AG BE = . Folosind apoi teorema lui Ceva, DE AD GB DE DF AG AD = 1, deducem c a = . Atunci GD k BC FA GB AB AD AD AD AG = = AB = CD. GB DC AB DC
A
F G

O E

VII.72. Fie ABCD paralelogram, E (CD), {M } = AE BD, {N AC, {O} = AC BD. Sa se arate ca AMEN = 2AMON . Mirela M Soluie. Cu teorema lui Menelaus n 4M EB t D E (O N A transversal) i 4AEN (M O B a s M N E OB AM transversal) obinem c a t a = 1, N N B OM AE M E OA BN O respectiv = 1. nmulind aceste t M A ON BE OA OB relaii membru cu membru, rezulta ca t A OM ON NE ME AAOB AMEN = 1. ns evident c d (E, AB) = 2 a a = 1, deci AE BE AMON AAEB s adica AAEB = 2AAOB i de aici urmeaza concluzia.

VII.73. Fie a < b c razele a trei cercuri tangente ntre ele si tan 1 1 1 aceeai dreapta n trei puncte distincte. Sa se arate ca = + . s a c b Dan Radu, B t Soluie. Fie T1 , T2 , T3 punctele de tangena ale celor trei cercuri c t 2 2 2 Un calcul imediat arata ca T2 T3 = 4bc i analog T1 T3 = 4ac, T1 T2 = 4a s c T2 T3 = T1 T2 + T1 T3 , rezult c bc = ac + ab i, prin imprire a a a s at obinem egalitatea dorit. t a Evident, concluzia nu are loc n cazul n care T1 = T2 = T3 .

VII.74. Fie M mulimea multiplilor lui 36 n a caror scriere n baza 1 t alte cifre dect 4, 6 sau 9. Cte numere cel mult egale cu 100 000 conine t Gabriel P Soluie. Elementele lui M se divid cu 4 i cu 9, deci se termin n t s a 96 i, cum au cel mult 5 cifre, au suma cifrelor 9, 18, 27 sau 36. Notm s a tuturor cifrelor afara de ultimele doua ale unui numar din M . Daca un numar se termina n 44, atunci s {10, 19}. Suma 10 se for 4 + 6, iar suma 19 din 4 + 6 + 9. Obinem astfel 2 + 6 = 8 elemente ale lui t 6444, 46944, 49644, 64944, 69444, 96444, 94644. Dac un numr se term a a atunci s {8, 17}. Suma 8 se formeaza din 4 + 4, iar suma 17 din 4 + 4 + alte 1 + 3 = 4 elemente ale lui M . n sfrit, daca un numar se termina n s s {12, 21}. Avem c 12 = 4 + 4 + 4 = 6 + 6, iar 21 = 9 + 6 + 6 i astfel a s 1 + 1 + 3 = 5 elemente ale lui M . n total, M conine 8 + 4 + 5 = 17 elemente cel mult egale cu 100000. t |a1 a2 | = |a2 a3 | = = |am1 am | = |am a1 | .

VII.75. Fie m 3 un numar natural impar si a1 , a2 , . . . , am Z astf Demonstrai ca a1 + a2 + + am se divide cu m. t

Maria Mihe, T t Soluie. Fie d valoarea comun a modulelor din enun; atunci fiec t a t numerele a1 a2 , a2 a3 , . . . , am1 am , am a1 este d sau d, deci sum un multiplu impar de d. Pe de alta parte, suma celor m numere este 0, d Rezult c a1 = a2 = = am , de unde a1 + a2 + + am = ma1 . a a

Clasa a VIII-a

VIII.71. Pe planul 4ABC se ridica perpendiculara AM . Fie P proi pe planul (M BC), iar E, F proieciile punctului P pe M B, respectiv M t \ arate ca M EF M CB. \ Otilia Neme, Ocn s
M

B VIII.72. Fie a, b, c numere reale distincte. Sa se afle partea ntreaga a b2 + ca c2 + ab a2 + bc + + . A= (a b) (a c) (b c) (b a) (c a) (c b) Mihail Bencze Soluie. Dup calcule de rutin, se arat c A = 2, deci [A] = 2. t a a a a

Soluie. Conform teoremei celor trei perpendiculare, t obinem c AE M B i AF M C. Cu teorema catetei n t a s 4M AB i 4M AC, deducem ca M A2 = M E M B = M F s ME MF M C, deci = . Rezult c 4M EF 4M CB, a a MC MB \ \ de unde M EF M CB.

E A

VIII.73. Sa se demonstreze ca a N este ipotenuza a unui triunghi d cu laturile exprimate prin numere naturale daca si numai daca exista n nct a2 n si a2 + n sunt patrate perfecte. Ctlin Cali a a

Soluie. Fie b, c N cu a2 = b2 + c2 ; considernd n = 2bc, avem c t a (b c)2 i a2 + n = (b + c)2 , Reciproc, daca a2 n = x2 , a2 + n = y 2 , cu s atunci 2a2 = x2 + y 2 , deci x, y au n mod necesar aceeai paritate. O s 2 2 x+y xy x+y xy a2 = + , cu , numere naturale. 2 2 2 2 xy : x VIII.74. Fie (0, 1]; sa se arate ca + 1 1 xy n fiecare din situaiile: t x+ a) x, y [0, ); b) < x 0 y < . 1 + x Gheorghe Costo Soluie. Este suficient sa demonstram, n fiecare dintre ipoteze, ca t xy 1 xy este strict pozitiv. a) Fr a restrnge generalitatea, s presupunem c x y; atunci aa a a xy xy 1 xy > 0 1 xy > 0 ( x) + y (1 x) > 0 nsa x < x < 2 1 1 x > 0 y (1 x) 0, de und imediata. x+ b) Mai nti, sa verificam ca 0 < , adica ipoteza nu este cont 1 + x Avem: x+ 1 < x 1 2 < x 1 + x > 0 > 1 + x x+ 2 1 x 2 1 0 x 2 x x + (1 + x) 1 + x Acum: xy yx 1 xy > 0 1 xy > 0 (1 xy) y + x > 0 +x ( + x) y (1 + x) > 0 y < , 1 + x adevrat conform ipotezei. a x Not. Concluzia are loc i dac x, y (, 0] sau a s a <y0 1x 2n+1 + VIII.75. Determinai valorile lui n N pentru care fracia t t 3 2n + ductibila. Gheorghe Iu s Soluie. Daca d | 2n+1 + 3 i d | 3 2n + 1, atunci d | 3 2n+1 + 3 2 (3 t adic d | 7. Cum fracia este reductibil, ea se simplific prin 7. Vom art a t a a a lucru se realizeaz dac i numai dac n = M3 + 1. a as a Dac n = M3 , atunci 2n+1 + 3 = 2 23k + 3 = 2 (7 + 1)k + 3 = 2 (M7 + a M7 + 5. Daca n = M3 + 2, atunci 2n+1 + 3 = 23k+3 + 3 = (7 + 1)k (M7 + 1) + 3 = M7 + 4. n sfrit, pentru n = M3 + 1, avem c 2n s a k 3k+2 n 2 + 3 = 3 (7 + 1) + 3 = 4 (M7 + 1) + 3 = M7 i 3 2 + 1 = 3 23 s 3 2 (7 + 1)k + 1 = 6 (M7 + 1) + 1 = M7 .

Clasa a IX-a

IX.71. Fie < b numere reale; sa se determine x, y, z R pentru care a = 2yz + a, iar x a + b x = y. Andrei Nedelcu i Lucian Ldu s a Soluie. Din a doua ecuaie deducem c x [a, b], iar y 0. Cu in t t a Cauchy-Schwarz, obinem t 2 2 2 y2 = xa+ bx xa + bx 12 + 12 = 2 (b nmulind prima relaie cu 2 i adumnd y 2 , gsim c t t s a a y 2 + 4z 2 + 2b = 4yz + 2a + y 2 (2z y) + 2b 2a = y 2
2

Ce egalitate se obine pentru x, y, z (, 1]? t Soluie. Fie t x2 [x]


2

y 2 = 2 (b a) + (2z y)2 2 (b a) . p Din (1) i (2) deducem c y = 2z = 2 (b a) i, cum avem egalitate n s a s 1 x a = b x, deci x = (a + b). 2 x y z IX.72. Fie x, y, z [1, +) aa nct s = = . Sa se arate ca [x] [y] [z] q q p 2 2 2 2 2 2 [x] + [y] + [z] + {x} + {y} + {z} = x2 + y 2 + z 2 . x y z = = = k [1, 2); atunci [x] [y] [z] = x2 + y 2 + z 2 [x] + [y] + [z]
2 2 2

Dan Pl

y2 [y]
2

z2 [z]
2

= k2

q p x2 + y 2 + z 2 = k [x]2 +

y [y] z [z] x [x] = = =k1 [x] [y] [z] {x}2 [x]


2

IX.73. Fie a1 , a2 , . . . , an (0, ) cu a1 a2 an = 1. Pentru orice m se arate ca are loc inegalitatea


m1 am + am + + am am1 + a2 + + am1 . 1 2 n n 1

Scaznd membru cu membru relaiile (1) i (2) obinem concluzia. Cn t s t (, 1], vom avea k (0, 1] i, relund raionamentul, gasim ca s t q q p 2 2 2 2 2 2 [x] + [y] + [z] {x} + {y} + {z} = x2 + y 2 + z 2 .

[y] [z] [x] + [y] + [z] q q 2 2 2 2 2 2 {x} + {y} + {z} = (k 1) [x] + [y] + [z] .

{y}2
2

{z}2
2

{x}2 + {y}2 + {z}2


2 2 2

= (k 1)

Marius Tiba,

inegalitate echivalenta cu cea dorita, ntruct a1 a2 an = 1.

Scriind nc n 1 inegaliti similare, dup adunare i mprire cu mn, a at a s at m1 am + am + + am a1 + am1 + + am1 n a1 a2 an 1 2 n n 2

Soluia 1 (Titu Zvonaru, Comneti). Conform inegalitii mediilor t a s at q mn m(mnn+1) m a1 a2 am = am + + am + am + + am mn n 1 1 2 n q mn mn(m1) m = mn a1 (a1 a2 an ) = mnam1 n a1 a2 an . 1

Soluia 2. Putem presupune fr a restrnge generalitatea c a1 a2 t aa a m1 atunci am1 am1 an si aplicam inegalitatea lui Cebev: s 1 2 am + am + + am = a1 am1 + a2 am1 + + an am1 1 2 n n 1 2 m1 1 + am1 + + am1 . (a1 + a2 + + an ) a1 n 2 n 1 s a ns (a1 + a2 + + an ) n a1 a2 an = 1 i astfel rezult concluzia. a n se atinge pentru a1 = a2 = = an = 1.

Not. Demonstraia inegalitaii (1) ca n soluia a doua (folosind Ce a t t t prezenta n RecMat 2/2006, pg. 160, n cadrul soluiei problemei L92. t IX.74. Fie a, b, c lungimile laturilor 4ABC si m, n (0, +). C punctele A0 , B 0 , C 0 astfel nct C (AA0 ), A (BB 0 ), B (CC 0 ) si CA0 AB 0 = mb + n, BC 0 = mc + n. Sa se arate ca 4ABC si 4A0 B 0 C 0 au ace de greutate daca si numai daca 4ABC este echilateral. Dumitru Mihalach ma + n + b mb + n + c 0 0 Soluie. Observam ca AA = t AC, BB = B b c mc + n + a mc + n + a CA + AB . Atunci 4ABC i 4A0 B 0 C 0 s CB = a a centru de greutate dac i numai dac as a AA0 + BB 0 + CC 0 = 0 mc + n + a mb + n + c ma + n + b mc + n + a AB + AC = a c b a mb + n + c ma + n + b mc + n mb + n ma + mc + n + a = = = = a c b a c b Dac a = b = c, relaia (1) este evident. Reciproc, s presupunem (1) n a t a a Dac dou dintre numerele a, b, c sunt egale, din (1) rezult imediat c a a a a n caz contrar, putem considera ca a < b < c i din (1) deducem ca s (mc + n) (mb + n) (mb + n) (ma + n) cb ba = = ac cb ac cb unde primul raport este negativ, iar al doilea pozitiv, ceea ce constituie o co \ IX.75. Fie ABCD patrulater inscriptibil, {O} = AC BD, m(AO MA NC O si punctele M (AB), N (CD). Notam k = , r = , p= MB ND O

2 1 \ p 2k, , , sa se arate ca m(M ON ) 6= 90 . r 2r

Mihai Ha Soluie. Cum ABCD este inscriptibil, avem c OA OC = OB OD. t a \ m(M ON ) = 90 OM ON = 0 k 1 r 1 OA + OB OC + OD = 0 k+1 k+1 r+1 r+1 OA OC + r OA OD cos 60 + k OB OC cos 60 kr OB OD 1 (1 + rk) OA OC = (r OA OD + k OB OC) 2 OD OB k 2 (1 + rk) = r +k 2 (1 + rk) = rp + . OC OA p 2 1 \ Dac p 2k, , a , evident c relaia (1) nu este verificat, deci m(M O a t a r 2r

Clasa a X-a

X.71. Fie a, b, c lungimile laturilor unui triunghi. Sa se arate ca 4 4 4 a + 2 bc b + 2 ac c + 2 ab + > 3. + a+ c b+ c a+ b Lucian tescu Tu Soluie. Se observa ca a < b + c (deoarece a < b + c+ 2 bc), b < t i c < a Prin urmare, dac a, b, c sunt lungimile laturilor unu s a + b. atunci a, b, c sunt de asemenea lungimile laturilor unui triunghi. raionamentul anterior, deducem ca 4 a, 4 b, 4 c sunt lungimile latu t si a + 4 bc a triunghi. De aici, rezult c a a > 1 (1). (ntr-adevr, (1) b+ c 2 4 bc > b + c a > 4 b 4 c 4 a > 4 b 4 c). Acum, nu dect sa remarcam faptul ca inegalitatea din enun se obine adunnd rel t t inegalitile derivate din ea prin permutri circulare ale laturilor a, b i c. at a s X.72. Sa se rezolve sistemul x2 log2 15 + y 2 log3 10 + z 2 log5 6 = 2 (xy + yz + zx) , x + y + z = 5.

Marius Damia t Soluie. Adunnd x2 + y 2 + z 2 n ambii membri ai primei ecuai t x2 y2 2 2 2 2 + + x log2 30 + y log3 30 + z log5 30 = (x + y + z) , sau log30 2 log30 3 (x + y + z)2 . Pe de alta parte, conform inegalitaii lui Cauchy-Schwarz, a t 2 2 x y z2 (log30 2 + log30 3 + log30 5) + + (x + y + log30 2 log30 3 log30 5 x2 y2 z2 2 + + (x + y + z) . log30 2 log30 3 log30 5

Egalitatea are loc dac i numai dac x, y, z sunt proporionale cu log30 as a t log30 5. Astfel, putem scrie y z x+y+z x = = = = 5, log30 2 log30 3 log30 5 log30 2 + log30 3 + log30 5 de unde rezulta ca soluia sistemului este x = 5 log30 2, y = 5 log30 3, z = t

X.73. Determinai funciile f : N cu proprietatea ca pentru oric t t R x+y f (x) + f (y) astfel nct 3 | x + y, are loc egalitatea f = . 3 3 Eugenia Rou, e s Solu Fie f o funcie cu proprietatea considerat. Pentru x = y tie. t a f (0) + f (0) 0+0 f = , deci f (0) = 0. Dac lum pe rnd x = 2 a a 3 3 apoi x = 3, y = 0 n egalitatea dat, obinem f (2) = 2f (1) i f (3) = 3f a t s demonstra prin inducie propoziia P (n): f (n) = nf (1), n N. Am a t t ca P (0), P (1) i P (2) sunt adevarate. Presupunem acum ca P (k) este s oricare ar fi k {0, 1, . . . , m}, m N. Sa demonstram valabilitatea P (m + 1). Distingem trei cazuri. I. Dac m + 1 = 3l + 1, l N, atunci lund x = 3l + i y = 3l 1 n a 1s 3l + 1 + 3l 1 f (3l + 1) + considerat (pentru c 3 | x+y), avem f a a = 3 3 deci 3 (2l) = f (3l + 1) + f (3l 1). Cum 2l 3l m i 3l 1 m, rezult s ipotezei de inducie, c f (3l + 1) = 3f (2l) f (3l 1) = 6lf (1) (3l t a (3l + 1) f (1) = (m + 1) f (1). II. Daca m + 1 = 3l + 2, l N, consideram x = 3l + 2 i y = 3l + 1 n s 3l + 2 + 3l + 1 f (3l + 2) + f (3l din ipotez (3 | x + y) i obinem f a s t = 3 3 f (3l + 2) = 3f (2l + 1)f (3l + 1) = 3 (2l + 1) f (1)(3l + 1) f (1) = (3l + (m + 1) f (1). III. Daca m + 1 = 3l 3, l N, atunci pentru x = 3l + 3 i y = 3l ( + s 3l + 3 + 3l f (3l + 3) + f (3l) relaia iniial devine f t t a = , de unde de 3 3 f (3l + 3) = 3f (2l + 1) f (3l) = (3l + 3) f (1) = (m + 1) f (1). Cu aceasta demonstraia propoziiei P (n) este terminata. Cum func t t t nf (1), n N satisface proprietatea ceruta, rezulta ca aceasta este soluia t X.74. Fie d1 , d2 doua drepte perpendiculare si l1 , l2 dreptele suport toarelor celor doua perechi de unghiuri opuse formate de ele. Determina cu A d1 , B d2 , C l1 si O, H l2 ( O, H cu semnicaiile uzuale). t Temistocle B Soluie. Faa de reperul cartezian xOy cu d1 luat ca ax Ox i d2 t t a a s A (a, 0), B (0, b), C (c, c) (nu este esenial faptul c l1 este luat ca prima t a a b a a reperului). Mediatoarele laturilor [AB] i [BC] au ecuaiile y = s t 2 b b+c c c respectiv y = x . Punnd condiia ca punctul lor t 2 bc 2 t t aparina dreptei l2 : x + y = 0, obinem relaia (a + b) 2c2 ab = 0 t

c a nlimile din A i C au ecuaiile y = at s t (x a), respectiv y c = ( bc b condiia H l2 conduce la c a2 + b2 = 0 (2). Distingem patru cazuri: t (i) a + b = 0, c = 0, adic b = a, c = 0; atunci 4ABC este dreptung a situat n cadranele II sau IV, avnd catetele pe axele reperului. (ii) a + b = 0, a2 + b2 = 0, adica a = b = 0; atunci 4ABC este degene (iii) 2c2 ab = 0, c = 0; din nou 4ABC este degenerat. (iv) 2c2 ab = 0, a2 + b2 = 0, adic tot triunghi degenerat. a X.75. a) Fie C . Daca z1 , z2 , z3 sunt trei numere complexe d fel nct Re (1 ) = Re (2 ) = Re (3 ), atunci punctele de afixe z1 , z z z z coliniare. Notam cu d dreapta pe care sunt situate aceste puncte. b) Daca z1 , z2 , z3 sunt trei numere complexe diferite cu proprietatea ca I 0 0 0 0 Im (2 ) = Im (3 ), atunci punctele cu axele z1 , z2 , z3 sunt coliniar z z0 dreapta ce le conine. t c) Sa se arate ca dreptele d si d0 sunt perpendiculare. Constantin Co Soluie. a) Avem: t 1 + z1 z 2 + z2 z 3 + z Re (1 ) = Re (2 ) = Re (3 ) z z z = = 2 2 2 z3 z1 z2 z1 = = . z2 z1 z3 z1 z2 z1 z2 z1 z2 z1 = R, ceea ce , adic a Din relaia (1), obinem t t z3 z1 z3 z1 z3 z1 ca punctele de afixe z1 , z2 , z3 sunt coliniare. b) Deoarece 0 z1 z 0 z2 z 0 z 0 0 Im (1 ) = Im (2 ) = Im (3 ) 1 z0 z0 z0 = 2 = 3 2i 2i 2i 0 0 0 0 z3 z1 z2 z1 = 0 = , z2 z1 0 0 z3 z1 0 0 0 z z1 0 0 0 rezult c 2 a a 0 0 R. Prin urmare, punctele cu afixele z1 , z2 , z3 sunt co z3 z1 0 z2 z1 z 0 z1 z2 z1 c) Din relaiile (1) i (2) deducem ca t s = 2 0 z 0 sau z 0 z 0 = z2 z1 z2 1 2 1 0 0 z2 z1 0 s de unde rezulta ca 0 0 iR+ , aadar dreptele d si d sunt perpendicu z3 z1

Clasa a XI-a
XI.71. Fie sirul (xn )n1 , unde x1 > 1, xn+1 =
n

lim xn . n

xn 1 , n N . ln xn

Dan Popescu x1 Soluie. Din inegalitatea 1 < t < x, x (1, ), rezulta ca ln x h in(xn 1) 1 strict descresctor i are limita 1. Scriem xn = (1 + xn 1) xn 1 a s n

criteriul StolzCesaro, 1 1 1 xn ln xn xn + 1 lim = lim = lim n n (xn 1) n xn+1 1 n (x 1)2 (x 1) l xn 1 n n (din lim
x1

= ). Urmeaz c xn e0 = 1. a a n (x 1)2 (x 1) ln x XI.72. Este posibil ca o funcie f : R R, care verica 1+f (x)+f (x)f t x R, sa fie continua pe R? Dorin Mrgidanu, a Soluie. Rspunsul este negativ. Vom demonstra c f nu are propr t a a Darboux. Firete f (x) 6= 0, x R. Daca f ar avea proprietatea lui s atunci ar fi negativ pe R (pozitiv pe R nu poate fi!): f (x) < 0, x R. a a imediat c 1 + f (1 + x) > 0, x R, deci f (x) > 1, x R. Pe de a din 1 < f (x) < 0, x R, rezult c 0 < f (x) + 1 < 1, x R, a a f (x) f (x + 1) < 1, x R, contradicie cu f (x) < 0, x R. t XI.73. Fie f : [0, 1] R o funcie nenula de clasa C k+1 pe [0, 1], k t ca f (0) = f (1) = 0. Daca pentru orice 1 j k exista aj {0, 1} a f (j) (aj ) = 0, atunci exista x1 , x2 (0, 1) astfel ca f (k+1) (x1 ) f (k+1) (x2 ) Gheorghe Moroanu i Paul Georg s s Soluie. Vom ine seama c, dac g : [0, 1] R este de clas C 1 pe [0 t t a a a semn constant pe (0, 1), iar g (0) = 0 sau g (1) = 0, atunci g pastreaza sem pe [0, 1] i n plus este monoton pe [0, 1]. Dac presupunem c f (k+1 s a a a x (0, 1), aplicnd succesiv rezultatul precedent pentru g = f (k) , g = f ( g = f obinem c f este monoton pe [0, 1]. Cum f (0) = f (1) = 0 rezult t a a pe [0, 1], contradicie; deci exista x1 (0, 1) astfel ca f (k+1) (x1 ) < 0. An t x2 (0, 1) astfel ca f (k+1) (x2 ) > 0, de unde concluzia. XI.74. Fie a, b, c R, a 6= 0, n 2N . Urmatoarele afirmaii sunt ech t (1) b2 4ac 0, (2) det aA2 + bA + cIn 0, A Mn (R). Marian Ursrescu a Soluie. Pentru (1) (2) avem succesiv: t b c det aA2 + bA + cIn = an det A2 + A + In = a a " # 2 2 b 4ac b = an det A + In + In = an det X 2 + Y 2 0 2 2a 4a 4ac b2 b unde X = A + In , iar Y = In ; X, Y Mn (R), XY = Y X. 2a 2a Pentru (2) (1) sa presupunem ca b2 4ac > 0. Notnd cu x1 , x2 reale i distincte ale ecuaiei ax2 +bx+c = 0, vom considera dou numere s t a i (, x1 ) (x2 , ). Fie s 2 a + b + c 0 ... 0 a 2 + b + c 0 ... 0 ; aA2 + bA + cIn = A= .. . . . . . . . . . . . . . 0 0 ... 0 a 2 + b

x ln x x + 1

n1 Acum det aA2 + bA + cIn = a2 + b + c a 2 + b + c < 0, in t de alegerea numerelor i , de semnul funciei de gradul al doilea i de f s t s este par. Avem deci contradicie, ceea ce ncheie demonstraia. t t

XI.75. Fie A, B M2 (C). Sa se arate ca daca AB BA comuta c B, atunci AB = BA. Dorel Mihe, T t Soluie. Vom utiliza urmatorul rezultat: "Fie A M2 (C) i mulime t s t {X M2 (C) | AX = XA}. Atunci: a) Dac A = kI2 , k C, atunci C (A) = M2 (C); a b) Daca A 6= kI2 , k C atunci C (A) = {A + I2 | , C}". Sa presupunem ca AB BA nu este de forma I2 . Atunci , , , nct A = (AB BA) + I2 i B = (AB BA) + I2 . Dac = a s

s A = I2 , deci AB BA = O2 , absurd. Deci 6= 0 i atunci AB BA = 1 deci B = a A I2 + I2 = A + bI2 , ceea ce implic AB = BA = adica AB BA = O2 , absurd. Astfel, exista C astfel nct AB B Cum tr (AB BA) = 0, obinem = 0, adic AB = BA. t a

Clasa a XII-a

XII.71. Fie f : [a, b] R o funcie derivabila cu derivata continua. S t Rb ca lim a f (x) sin nx dx = 0. n Dan Radu, B Soluie. Integrnd prin pari, ajungem la relaia t t t Z b Z f (a) cos na f (b) cos nb 1 b 0 f (x) sin nx dx = f (x) cos nx + n n a a R b t Daca M = sup |f (x)|, M 0 = sup |f 0 (x)|, obinem ca 0 a f (x) si x[a,b] x[a,b] R 2M + M 0 (b a) b . Rezult c lim a f (x) sin nx dx = 0, de unde conclu a a n n XII.72. Fie funcia f : [0, 1] R, indefinit derivabila pe [0, 1], cu pr t ca exista M > 0 astfel nct f (n) (x) M , x [0, 1] si n N. Arata t a) P p k+1 (n + p) (n + p 1) (n + p k + 2) f (n+pk) (1) k=1 (1) lim =0 p+1 n (1) (n + p) (n + p 1) (n + 1) b) lim Pp
k=1

(1)k+1 (n + p) (n + p 1) (n + p k + 2) f (n+pk) ( (1)


p+1

1 0

(n + p) (n + p 1) (n + 1) xn f (n) (x) dx, n N.

Ovidiu Pop, Sa

Soluie. Integrnd prin pri, obinem succesiv: t at t Z 1 Z 1 1 n+p (n+p) n+p (n+p1) x f (x) dx = x f (x) (n+p) xn+p1 f (n+p1)(x 0 0 0 Z 1 1 = f (n+p1) (1)(n+p) xn+p1 f (n+p2) (x) (n+p1) xn+p2 f (n+p2)
0 0 (n+p1) (n+p2) 0 0

i aa mai departe. Deducem c s s a Z 1 xn+p f (n+p) (x) dx =


0

(1) (n + p) f (1) + (n + p) (n + p 1) =f Z 1 1 xn+p3 f (n+p3) (x) dx xn+p2 f (n+p3) (x) (n + p 2)

k=1

p X

(1)

k+1

(n + p) (n + p 1) (n + p k + 2) f (n+pk) (1) + Z
1

+ (1)p (n + p) (n + p 1) (n + 1) Aratam ca
n

xn f (n) (x) dx,


0

n, p N,

lim

R1 M a a Din f (m) (x) M , x [0, 1], m N, rezult c 0 xm f (m m+1 M , m N i trecnd la limit, obinem relaia (2). s a t t m+1 Din (1) i (2) rezult concluzia problemei. s a XII.73. Sa se arate ca Z 1 Z 1 n n (1 + x) + (1 x) 1 ln (1 + x) ln dx = n dx = n ln 2, n 1 + x2 1 + xn 1 + x2 8 0 0 Sa se calculeze apoi
/4 R 0

xm f m (x) dx = 0.

ln

Gabriel Necula 1y Soluie. Cu substituia x = t t avem succesiv 1+y Z 1 Z 1 n (1 + y) + (1 y 1 2 n ln (1 + x ) dx = 2 2 ln 2 (1 + y)n 0 1+x 0 (1 + y) + (1 y) Z 1 Z 1 1 1 n n n = ln ((1 + y) + (1 y) ) dy ln (1 + y) dy 1 + y2 1 + y2 0 0 Z 1 Z 1 1 1 ln ((1 + x)n + (1 x)n ) dx ln (1 + xn ) dx 1 + x2 1 + x2 0 0 Z 1 ln (1 + x) dx = n ln 2 =n 1 + x2 8 0

1 + tg3 x dx. 1 + 3 tg2 x

1 + tg3 x 2 dx = 8 ln 2. 1 + 3 tg x 0 XII.74. Fie (G, ) un grup comutativ cu proprietatea ca exista n N a din xn = y n rezulta x = y, unde x, y G. Daca f , g sunt doua endomo lui G, atunci ecuaia f (x) = g x1 are soluie unica daca si numai da t t h : G G, h (x) = f (xn ) g (xn )este injectiva. D. M. atineu-Giurgiu, B B t Soluie. Sa presupunem ca ecuaia f (x) = g x1 are soluie unic t t t aceasta va fi x = e) i s artm c h este injectiv. Succesiv avem: s a aa a a Cu substituia x = tg t rezulta ca t ln h(x) = h(y) f (xn ) g(xn ) = f (y n ) g(y n ) g(xn ) (g(y n )) = (f (xn )) g(xn ) g y n = f xn f (y n ) 1 1 g xn y n = f xn y n = f y n xn = f xn y n ,
1

Z 1 (1 + x)n + (1 x)n 1 ln (1 + x) ln dx = n dx = n ln 2 n 2 1+x 1+x 8 0 1+x 0 1 R ln (1 + x) dx = ln 2.) Pentru n (Presupunem cunoscut rezultatul 1 + x2 8 0 3 3 1 1 R R 2 1 + 3x2 (1 + x) + (1 x) 1 1 ln dx = 3 ln 2, sau ln dx 2 2 1 + x3 8 1 + x3 0 1+x 0 1+x adic a Z 1 Z 1 1 + 3x2 1 ln 2 ln dx = 3 ln 2 dx = 3 ln 2 ln 2 = 2 3 2 1+x 8 8 4 0 1+x 0 1+x Z
1 /4 R

de unde deducem ca xn y n = e xn = y n x = y, adica h este imjectiv Reciproc, daca h este injectiva, sa presupunem ca exista a G \ {e} a f (a) = g a1 . Dar h (an ) = f (an ) g (an ) = g (an ) g (an ) = g (an an ) = prin urmare h (an ) = h (en ), ceea ce contrazice injectivitatea lui h. U ecuaia f (x) = g x1 are numai soluia x = e. t t XII.75. Fie A Mn (C) de rang r si S = {B Mn (C) | ABA = On ca S este subspaiu vectorial n Mn (C) si ca dim S = n2 r2 . t Adrian Reisn Soluie. Faptul c S este subspaiu vectorial n Mn (C) este o simpl t a t a

Matricea A fiind de rang r, exist P, Q GLn (C) astfel nct P 1 AQ = a Acum B S ABA On P 1ABAQ = On P 1AQ Q1 BP P 1 = Ir 0 Ir 0 1 , = On , unde am notat Q BP = 0 0 0 0 Ir 0 matrice ptratice de ordin r, respectiv n r. ns a a 0 0 Ir 0 0 0 a , de unde = Or . Deducem c B S Q 0 0 0 0 0 0 , adica dim S = n2 r2 .

Soluiile problemelor pentru pregtirea concur t a din nr. 2 / 2006


A. Nivel gimnazial

G106. Fie m N \ {0, 1} si k N, k m. Pentru fiecare x N, c propoziiile: x > 1; x > 2; . . . ; x > m. Aflai x N pentru care k d t t propoziii sunt adevarate, iar celelalte m k sunt false. t Maria Mihe, T t Soluie. Pentru k = 0, toate propoziiile sunt false. n particular, x t t falsa i daca x {0, 1}. Pe de alta parte, pentru x = 0 i x = 1 evident ca s s m propoziii sunt false, deci pentru k = 0 rspunsul este x {0, 1}. t a Pentru 1 k m 1, cele k propoziii adevrate trebuie s fie x > 1; t a a x > k, iar cele false vor fi x > k + 1; . . . ; x > m. Avem aadar x > k i s s deci x = k + 1. Cnd k = m, toate propoziiile trebuie s fie adevrate. n particular, t a a fi adevrat, deci x {m + 1, m + 2, . . . }. a a

G107. Mulimea A N de cardinal n N are proprietatea ca, or t patru elemente ale sale, putem alege doua cu suma 22006 + 1. Aflai valoare t a lui n. Dan Nedeianu, Dr. Tr Soluia 1 (a autorului). Pentru m = 6, putem considera A = {1, 2, 3 t 22006 1, 22006 }. Sa presupunem prin absurd ca ar exista mulimi A c t Conform principiului cutiei, exist o submulime B A de cardinal 4 a t elementele de aceeai paritate. Suma oricror dou numere din B va fi s a a diferita de 22006 + 1.

Soluia 2 (Marius Tiba, elev, Iai). Pentru a demonstra ca nu p t s m 7, putem proceda astfel: dac n 7, exist n A fie 4 elemente cel a a cu 22005 , fie 4 elemente cel puin egale cu 22005 + 1. Alegnd dou nume t a patru, obinem o suma fie mai mare, fie mai mica dect 22006 + 1. t

G108. Fie m, n N . Sa se arate ca mulimea numerelor ntregi de t mult egal cu n poate fi partiionata n m submulimi cu aceeai suma a el t t s daca si numai daca n + 1 m. Marian Tetiv Soluie. Sa presupunem mai nti ca n+1 m. Sa notam M = {x Z t {x Z | n x n} i Mk = {k, k} pentru k = 0, n. Clasele parti s atunci mulimile M0 , M1 , . . . , Mm2 , Mm1 Mm Mn ; este eviden t elementelor din fiecare astfel de mulime este aceeai i anume 0. t s s Reciproc, sa presupunem ca avem partiia M = A1 A2 Am u t elementelor din fiecare mulime Aj este aceeai, j = 1, m. Se obine fr t s t aa c aceast sum trebuie s fie 0 (deoarece i suma elementelor mulimii M a a a a s t Observam atunci ca fiecare mulime a partiiei, cu excepia celei care-l con t t t are cel puin doua elemente. (Orice element nenul al mulimii respective t t fie adunat cu un element nenul pentru a da suma zero). Avem atunci c a m P card M = card Ak 2 (m 1) + 1, de unde deducem ca n + 1 m.
k=1

G109. La un concurs se dau sase probleme evaluate cu 1, 2, 3, 4, 6 puncte. Daca un elev nu rezolva o problema, primete 1 punct; daca s primete punctajul corespunzator. Fiecare elev obine macar 11 puncte. S s t ca o problema a fost rezolvata de cel puin o treime dintre elevi. t Gabriel Dospinescu, stude Soluie. Fie n numarul elevilor; punctajul total este cel puin 11n. t t problema, de punctaj a, i notam cu A (a) mulimea elevilor care rezol s t problem. Astfel, exact |A (a)| elevi primesc a puncte i exact n |A (a)| a s punct, deci punctajul obinut la aceast problem va fi (a 1) |A (a)| + n. t a a total va fi X [(a 1) |A (a)| + n] max |A (a)| (1 + 2 + 3 + 4 + 5) + 6n,
a a

deci max |A (a)| 15 + 6n 11n, de unde concluzia problemei. a G110. Fie mulimile A = {k + n | k Z, n N} si B = (0, 1/10). t A B este infinita. Petru Asa Soluia 1. Considerm C = x R | x = m2 + 1 m, m N, m t a 1 1 < dent c C A i, dac x C, atunci 0 < x = a s a 2+1+m m m urmare C A B. S mai artm c mulimea C este infinit. Dac pr a aa a t a a contrariul, exista s 6= t, s, t 10 astfel nct s2 + 1 s = t2 + 1 t. succesiv: p p p t s = t2 + 1 s2 + 1 s2 2st + t2 = t2 + s2 + 2 2 (t2 + 1) (s p (t2 + 1) (s2 + 1) = st + 1 t2 + 1 s2 + 1 = (st + 1)2 s2 + t2

Soluia 2 (Marius Tiba, elev, Iai). Pentru fiecare k ntreg negativ d t s k 1 5, consideram n = k 2 ; ar fi suficient sa aratam ca 0 < k + n < . 5 10 r r k k k k2 + k > 0 k2 > k k 2 > k 2 k < 0; 5 5 5 r 2 k k 1 (1 10k) k2 + k < k2 < 100k 2 20k < 100k 2 2 5 10 5 100 ambele relaii la care am ajuns fiind evident adevrate. t a

prin urmare t = s i ajungem astfel la o contradicie. Rmne c C est s t a a prin urmare A B este infinit. a

G111. Fie 0 < a < b numere reale date si x, [a, b]. Daca s = x + y ab s2 + ab sa se afle maximul expresiei E = p + . p Vlad Emanuel, el Soluie. Cum x [a, b], avem ca (x a) (x b) 0 x2 (a + b) x t deci (a + b) x x2 + ab. Scriem o inegalitate analoag n y i nmulind m a s t membru, obinem: t (a + b)2 xy x2 + ab y 2 + ab (a + b)2 p p2 + ab s2 2p + a

G113. Fie segmentul [AB] de mijloc O si semicercurile C1 si C2 de [AB], respectiv [AO] situate n acelai semiplan faa de AB. Perpend s t C (AO) pe AB intersecteaza C1 n E si C2 n D. Daca AD C1 = { arate ca AE este tangenta cercului circumscris 4DEF . Alexandru Negrescu, elev, \ \ E Soluie. Cum m(DCB) + m(DF B) = 90 + 90 = t 180 , rezult c punctele B, F , D, C sunt conciclice. a a Scriind puterea punctului A faa de cercul determinat t D de ele, obinem ca AC AB = AD AF . Pe de alta t 2 parte, AC AB = AE din teorema catetei n 4AEB. Deducem c AE 2 = AD AF , de unde concluzia. a A CO \ G114. Fie ABCD un paralelogram care nu este romb cu m(BAD) = M, N (AC), P (BC) si Q (CD) sunt astfel nct [BM , [DN , [A \ \ \ \ sunt bisectoarele unghiurilor ABC, ADC, BAC si respectiv DAC, atunc perpendiculara pe N Q. Andrei Ned Soluie. Lum R (AD) astfel nct [CR t a B P \ este bisectoarea unghiului ACD, adica cea de a S N treia bisectoare interioara a triunghiului ACD. Deoarece triunghiurile ABC i ADC sunt cons gruente i simetrice faa de centrul O al paralels t M O ogramului ABCD, rezulta ca [AP ] este paralel i s congruent cu [CR]. De asemenea, din congruena A t R i simetria triunghiurilor AM P i CN R rezult s s a c P M este paralel cu RN . Prin urmare, dac vom arta c N Q R a a a a a rezulta ca N Q M P . Punctul R este centrul cercului exnscris, tangent la \ n triunghiul CDN . Prin urmare, [N R este bisectoarea unghiului AN D. este centrul cercului exnscris, tangent laturii [N D] n triunghiul AN D;

Soluie. Evident c (a + b) c2 < (a + b) = a + b; scriind t a inegaliti similare i sumnd, obinem concluzia. at s t Not. O mai bun evaluare a membrului stng al inegalitii, este i a a at Titu Zvonaru: Daca a, b, c sunt lungimile laturilor unui triunghi, atunci q q q 2 2 2+ 2+ (a + b) c (b + c) a (c + a)2 b2 3 (a + b + c

ab s2 + ab p+ a2 + b2 + 4ab E a2 + b2 + 4ab. p Pentru a se atinge egalitatea, trebuie ca x2 (a + b) x + ab = y 2 (a + b) y fapt care se realizeaz pentru (x, y) {(a, a) ; (a, b) ; (b, a) ; (b, b)}. Rezult a a a2 + b2 + 4ab. G112. Daca a, b, c sunt lungimile laturilor triunghiului ABC, atunci q q q 2 2 2 (a + b) c2 + (b + c) a2 + (c + a) b2 < 2 (a + b + c) Zdravko Starc, Vrac, Serbia i Mu s s q q
2 2

\ \s \ [N Q este bisectoarea unghiului CN D. Deoarece unghiurile AN D i CN D comun N i laturile [N A i [N C n prelungire, rezulta ca bisectoarele lor [N s s sunt perpendiculare, ceea ce ncheie rezolvarea. G115. Fie patratul M N P Q nscris n patratul ABCD, M (AB), N P (CD), Q (BC) si fie {E} = P N AB, {F } = P Q AB. Notam S3 ariile patratului ABCD, patratului M N P Q, respectiv 4P EF . Sa se p 1 1 1 b) S3 S1 ; c) = . a) S1 S2 = 4 SAEN SBF Q ; S3 S2 S1 Claudiu Stefan P Soluie. t a) Din congruena triunghiurilor t D P C AM N , BQM , CP Q i DN P (I.U.) rezulta ca AN = s Q BM = CQ = DP i AM = BQ = CP = DM . s Notm cu S aria 4AM N ; evident c 4S = S1 S2 . a a N Apoi, din asemnrile evidente 4AEN 4DP N i a a s 2 SAEN AN EA M B 4CP Q 4BF Q, obinem ca t = = S DN 2 CQ S = , deci S 2 = SAEN SBF Q . Concluzia de la a) este imedi BQ SBF Q S CQ AN = ; atunci SAEN = k 2 S, SBF Q = 2 , prin urmare b) Fie k = ND BQ k S S3 = S2 + 2S + SAEN + SBF Q = S2 + 2S + k2 S + 2 = S2 + S k + k 1 Cum k + 2, rezulta ca S3 S2 + 4S, adica S3 S1 . k c) Cum S3 = SP EM + SP F M , avem succesiv: 1 P M P E sin 45 P M P F sin 45 2P E PE PF = + PM = 2 2 2 PE + 2 PM P E2 P F 2 2 2 2 = BQ + BM = M Q = 2 2 (P E + P F ) 2 EF 2 + 2P E P F 1 EF 1 = = + BQ2 + BM 2 P E2 P F 2 AM 2 + BM 2 PE PF P 2 1 1 1 + = 2 AD S3 (AM + M B) 2AM M B 1 1 1 1 1 1 1 1 = + = = 2 AM AN AD S3 S3 S1 4S S1 S3 S2 AB 2 4 2

B. Nivel liceal

L106. Fie I centrul cercului nscris n 4ABC. Dreptele AI, BI si secteaza a doua oara cercurile circumscrise triunghiului BCI, CAI si A B 0 , respectiv C 0 . Daca notam cu |XY Z| perimetrul 4XY Z, sa se demon

CA AB BC + + = 1. |BCA0 | |CAB 0 | |ABC 0 | Titu Zvonaru, C Soluie. Notm cu R raza cercului circumscris 4ABC. Patrulaterul B t a 1 b [ inscriptibil, deci m(\ = m(IBC) = m(B). Aplicnd teorema sinusu CA0 I) 2 n 4AA0 C i n 4ABC, avem: s B B A 4R sin cos AC 2R sin B A0 C 2 2 = 4R cos B , = = = A B B B 2 sin sin sin sin 2 2 2 2 I A B 0 deci A C = 4R sin cos . Atunci: B 2 2 A B A C |BCA0 | = 4R sin cos + 4R sin cos + a = 2 2 2 2 A B C A cos + cos + cos = 4R sin 2 2 2 2 A A A 4R sin cos cos BC 2 2 2 = = A B A A B C |BCA0 | cos + cos + cos 4R sin cos + cos + cos 2 2 2 2 2 2 Egalnd analog celelalte doua rapoarte din concluzie, se obine cerina pr t t Observaie. 1) Dac 4ABC este ascuitunghic i n locul punctulu t a t s erm punctul O, relaia rmne valabil i constituie o parte a probleme a t a as Crux Mathematicorum 1/2006. 2) Relaia este adevrat i pentru ortocentrul H n cazul triunghiu t a a s tunghic. Not. n aceeai maniera a rezolvat problema Vlad Emanuel, elev, a s L107. Fie M , N doua puncte situate n interiorul 4ABC, avnd pna la laturile AB, BC, CA egale cu 3, 2, 7, respectiv 9 , 5, 5 . Daca ra 2 2 circumscris 4ABC este R = 8, sa se calculeze M N . Vlad Emanuel, el Soluie. Fie A0 , B 0 , C 0 i A00 , B 00 , C 00 proiecit t s A ile punctelor M , respectiv N pe laturile AB, BC, CA. Consideram punctul I [M N ] ast MI C fel = 2 i fie M Q k BC, Q N A00 , s B IN N B 0 0 I = PrBC I, {P } = II M Q. Avem c N Q = a IP MI 2 I 00 0 s N A M A = 5 2 = 3 i = = , C NQ MN 3 deci IP = 2, apoi II 0 = 2 + 2 = 4. Daca M P Q I 00 = PrAC I, I 000 = PrAB I, analog se arata ca 00 000 II = II = 4, prin urmare I este centrul cerA I A cului nscris n 4ABC, iar r = 4. Rezult c n B a a inegalitatea lui Euler R 2r se atinge egalitatea i astfel 4ABC este ech s

Notm x = A0 B, y = C 0 B. Din teorema lui Pitagora, BM 2 = BC 02 + a BA02 + A0 M 2 . Apoi, cu teorema cosinusului, A0 C 02 = M C 02 + M A02 2M cos 120 = BC 02 +BA02 2BC 0 BA0 cos 60 . Obinem astfel sistemul x2 + t 8 3 7 3 2 2 t ,y= . x + y xy = 19, cu unica soluie admisibila x = 3 3 0 Latura 4ABC echilateral este l = 8 3. Cum I este mijlocul lui BC, 8 3 MI 4 3 A0 I 0 0 0 0 = . Stim ca 0 00 = ca BI = 4 3, de unde A I = 4 3 3 3 IA IN A0 A00 = 2 3. Din trapezul dreptunghic M A0 A00 N gasim M N = 21.

Not (Mihai Haivas). Dac M, N Int ABC au distanele pn a a t a triunghiului egale cu a, b, c, respectiv a0 , b0 , c0 , iar raza cercului circumsc 2 (ab0 a0 b) , atunci condiia t este R = a b a0 + b0 (ab0 a0 b) (b c b0 + c0 ) = (bc0 b0 c) (a b a0 + b0 )

este suficienta pentru ca triunghiul sa fie echilateral. n cazul nostru, cele d sunt verificate.

Soluia 2 (a autorului). Mai general, vom demonstra c oricare ar t a (0, ), are loc inegalitatea x+y x+y+z sin x + sin y + sin z 8 x+y+z sin sin sin2 3 3 3 3 12

A cu egalitate cnd B = C = , rmne s demonstrm c a a a a 2 2 3A 3 3 A sin A 2 cos 2 3 1 cos 2 2 6 3 3 A A A A 2 sin cos 2 cos 2 3 2 3 cos cos + sin sin 2 2 2 2 6 2 6 A A Notam x = cos (0, 1), 1 x2 = sin ; avem de aratat ca 2 2 ! p p 3 3 3 3x 3 1 x2 x 2 x 2x 1 x2 2x 2 2 2 3 3 2 , evident, iar daca x < Daca x > ramne ca 1 2 1 x ra 2 2 2 , iari evident. Egalitatea se atinge n cazul triunghiului echilat as 2 1x

L108. Sa se arate ca n orice 4ABC are loc inegalitatea 3 3 3A (sin A + sin B + sin C) 4 3 sin2 . 2 12 Marian Tetiv Soluia 1 (Vlad Emanuel, elev, Sibiu). Deoarece t BC A A cos 2 cos , sin B + sin C = 2 sin 2 2 2 2

Plecm de la egalitatea imediat a a x + y sin x + sin y x+y xy sin = 2 sin sin2 , 2 2 2 4 pe care o rescriem sub formele z + t sin z + sin t z+t zt sin = 2 sin sin2 , 2 2 2 4 x+y z+t x+y x + y + z + t sin 2 + sin 2 x+y+z+t sin = 2 sin sin2 4 2 4 Din relaiile (2), (3) i (4) deducem c t s a sin

x + y + z + t sin x + sin y + sin z + sin t x+y x = sin sin2 4 4 2 4 x+y+z+t z+t 2 zt 2 x+yzt sin + 2 sin sin . + sin 2 4 4 8 Pentru x, y, z, t (0, ), primii doi termeni din membrul drept sunt nenega cu 0 cnd x = y, z = t), deci are loc inegalitatea sin

x+ x + y + z + t sin x + sin y + sin z + sin t x+y+z+t 2 sin sin2 4 4 4 x+y+z , obinem exact relaia (1). Egalitatea se atin t t nlocuind aici t = 3 x+y+z x = y, z = , deci cnd x = y = z. 3 Cerina problemei se obine din (1) pentru x + y + z = . Egalitatea s t t cazul triunghiului echilateral.

L109. Se dau numerele reale pozitive subunitare a1 , a2 , . . . , a2n2 n , n Sa se demonstreze inegalitatea (sumarea se face prin permutari circulare) X 2n 1 a2n1 1 < . 2n1 2n1 2n + 1 a2 + a3 + . . . + a2n1 + 2n + 1 2n2 n Soluie. Cum 1 > a2n+1 , atunci t 1 X < X

Ioan Serdean

a2n1 1 = a2n+1 + a2n+1 + + a2n+1 + 2n + a2n+1 2 3 1 2n2 n = a2n+1 + a2n+1 + + a2n+1 + 2n 1 2 2n2 n a2n1 + a2n1 + + a2n1 1 2 2n2 n .

a2n1 1 < a2n+1 + a2n+1 + + a2n+1 + 2n + 1 2 3 2n2 n

Pe de alta parte, din inegalitatea mediilor M G M A, avem ca q 2n+1 2n1 2n+1 (2n 1) a2n+1 + 1 + 1 (2n + 1) 1 1 = (2n + 1) a1 1

i nc 2n2 n 1 inegaliti analoage pentru a2 , a3 , . . . , a2n2 n . Prin at s a acestora membru cu membru, obinem ca t (2n 1) a2n+1 + a2n+1 + + a2n+1 + 2 2n2 n 2 n 1 2 2n (2n + 1) a2n1 + a2n1 + + a2n1 1 2 2n2 n a2n+1 1 a2n1 + a2n1 + + a2n1 1 2 2n2 n + a2n+1 2 + + a2n+1 2n2 n + 2n 2n 1 . 2n + 1

Din (1) i (2) rezult inegalitatea dorit, care este strict ntruct (1) este s a a a L110. Fie a, b, c (0, ) si n, k N. Demonstrai inegalitatea t

bn+k cn+k 4n(a b)2 an+k + n + n ak + bk + ck + . bn c a k(a2k + b2k + c2k )

(n legtur cu o problem propus la OBM 2005). a a a a Titu Zvonaru, Comneti i Bogdan Ionia, B a s s t Soluie. Consideram polinomul P (t) = ktn+k (n + k) tk + n, t > 0 t schema lui Horner, obinem c t a 2 n+k2 + + 2nt + n n (t 1)2 . P (t) = (t 1) kt a t n inegalitatea P (t) n (t 1)2 facem t = ; obinem b an+k ak n (a b)2 an+k n k n+k (n + k) k n + k n (n + k) ak nbk + b b b2 b b c Scriem nca doua relaii analoage pentru t = i t = i le sumam: t s s c a n+k a bn+k cn+k (b c)2 ( (a b)2 + n + n + 2k + k ak + bk + ck +n k n 2k b c a b c ns, din inegalitatea Cauchy-Schwarz, avem c a a
2 2 2 2

(b c) (c a) (a b + c b + a c) 4 (a (a b) + 2k + = 2k 2k 2k 2k + b2k + c2k b c a a a + b2k de unde concluzia problemei.

L111. Se dau m numere naturale distincte din mulimea {1, 2, . . . , t arate ca putem alege cteva dintre ele, cu suma S, astfel nct m(m + 1) 0S n + 2n m. 2 Adrian Zahariuc, ele Soluie. Fie a1 < a2 < < am cele m numere date. Notam cu t m (m + 1) i cu i indicele max minim pentru care a1 + a2 + + aj s 2 m (m + 1) care ai + ai+1 + + aj . Pentru S = ai + ai+1 + + aj e 2 m (m + 1) ndeplinit condiia 0 S a t . 2

m (m + 1) , deci 2 m (m + 1) S ai + 1. 2 Din minimalitatea lui j, deducem ca m (m + 1) ai + ai+1 + + aj aj > a1 + a2 + a1 + a2 + + aj1 < 2 nsa ak k, k N , deci i (i 1) n 1 a1 + a2 + + ai1 1 + 2 + + (i 1) = i 2 Apoi, ak n m + k, k {1, 2, . . . , m}; n particular ai n m + i n m + 2n + 1. m (m + 1) Din relaiile (1) i (2) obinem c S t s t a + n m + 2n, ceea 2 rezolvarea. L112. Pentru n N, notam cu a(n) numarul modurilor n care n se ca suma a unui numar par de puteri ale lui 2 si cu b(n) numarul moduri n se poate scrie ca suma a unui numar impar de puteri ale lui 2. Sa s a(n) = b(n), n 2. Adrian Zahariuc, ele Soluia 1. Fie N mulimea tuturor irurilor de numere naturale c t t s numr finit de termeni nenuli. Pentru fiecare n N, definim a Din maximalitatea lui i, avem S ai Atunci se observ c a a a(n) = |Mn (A)| , b(n) = |Mn (B)| , Mn = {(a0 , a1 , . . . ) N | n = a0 + 2a1 + 4a2 + } .

Evident ca

unde Mn (A) = {(a0 , a1 , . . . ) Mn | a0 + a1 + = unde Mn (B) = {(a0 , a1 , . . . ) Mn | a0 + a1 + =

a (2n + 1) = a (2n) i b (2n + 1) = b (2n) . s S evalum a (2n). Este clar c a0 este tot timpul par. Cardinalul lui M a a a a0 = 2k, k {0, 1, 2, . . . , n}, este b (n k). Atunci Analog se arata ca

a (2n) = b (n) + b (n 1) + + b (0) a (2n) = a (2n 2) + b (n

b (2n) = b (2n 2) + a (n) . Folosind relaiile (1), (2) i (3), se arat prin inducie c a (n) = b (n), n t s a t a Not. Asemanator a rezolvat problema Vlad Emanuel, elev, Sibiu. a Soluia 2 (a autorului). Vom folosi funcii generatoare. Fie t t 1 f (x) = 1 x + x2 x3 + = . 1+x Atunci, din definiia numerelor a (n) i b (n) dat n prima soluie, t s a t X 1 1 1 (a (n) b (n)) xn = f (x) f x2 f x4 = 2 1 + x4 1+x 1+x n=0

de unde a (n) b (n) = 0, n 2, ceea ce trebuia demonstrat. L113. Determinai numerele reale a, b pentru care mulimea A = t t n N } este finita. Gheorghe Iu Soluia 1 (a autorului). Notm Sn = an + bn , n N . Deoarece A t a ea este marginita: exista m = min A, M = max A, deci m Sn M , Pentru |a| > 1, avem c a n M S2n = a2n + b2n a2n = 1 + a2 1 1 + n a2 1 > M, n > a Din contradicia obinut rezult c |a| 1. La fel se arat c |b| 1. t t a a a a a Dac |a| < 1, a 6= 0 i |b| 1, atunci a2n+2 < a2n i b2n+2 b2n , dec a s s S2n , n N . Rezulta astfel ca mulimea A este infinita, fals. Analog t contradicie daca |b| < 1, b 6= 0. Deducem ca a, b {1, 0, 1} i pentru ac t s se verific imediat c A este finit. a a a Soluia 2 (Vlad Emanuel, elev, Sibiu). Daca A, B sunt mulim t t |A| = m, |B| = n, atunci AB = {x y | x A, y B} i AB = {xy | x s sunt tot nite, avnd cel mult mn elemente. Fie xn = an + n ; cum A b mulimile {x2n | n N } i x2 |n N sunt nite. Atunci x2 x2n | t s n n n o
2

{2an bn g | n N } este finit, apoi (an bn ) | n N este finit, deci {an a a (an + bn ) + (an bn ) este finita. Cum an = , deducem ca {an | n N } 2 de unde se deduce imediat ca a {1, 0, 1}. Analog obinem ca b { t pentru aceste valori se veric imediat c A este finit. a a a L114. Consideram o parabola si doua drepte secante parabolei, paralele dar neparalele cu axa de simetrie a parabolei. Folosind doar rigla negrad construiasca tangenta la parabola care este paralela cu dreptele date. Titu Zvonaru, C Soluie. Fie P : y 2 = 2px parabola data, iar d i d0 cele doua dr t s 2 2 P (x1 , y1 ), Q (x2 , y2 ) interseciile lui d cu P, cu y1 = 2px1 , y2 = 2px2 , ia t punctul cutat n care tangenta la parabol este paralel cu dreapta P Q. a a a p n M are ecuaia by = px + pa i panta , iar dreapta P Q are panta t s b 2p (y1 y2 ) 2p (y1 y2 ) 2p y1 y2 = = = . 2 y2 x1 x2 2px1 2px2 y1 y1 + y2 2 y1 + y2 Din egalitatea pantelor obinem ca b = t , deci M are aceeai or s 2 mijlocul segmentului [P Q]. Folosind teorema lui Ceva, se arata ca putem construi mijlocul unui seg avem dat o dreapt paralel cu el, apoi putem construi o nou paralel a a a a a printr-un punct dat, ambele construcii efectundu-se doar cu sigla negra t de exemplu, A.Tth - Noiuni de teoria construciilor geometrice, E.D.P., t t Cu segmentul [P Q] i dreapta d0 , construim mijlocul M al lui [P Q] s taie parabola n P 0 i Q0 , construim mijlocul M 0 al lui [P 0 Q0 ]. Dreapta s parabola n punctul T (ea fiind paralel cu axa de simetrie a parabole a paralela prin T la d este tangenta dorita.

Not. Soluie corect s-a primit de la Vlad Emanuel, elev, Sibiu. a t a L115. Determinai P R[X], grad P 2, astfel nct funcia f t t f (x) = p({x}) + {p(x)} sa fie periodica (unde p este funcia polinomiala t P , iar {} desemneaza partea fracionara). t Paul Georgescu i Gabriel P s Soluie. Vom arata ca nu exista polinoame P cu proprietaile din en t t supunem contrariul i fie T R o perioada a lui f . Cum discontinuitai s t + ale lui f sunt numere ntregi i soluiile ecuaiei p (x) = n, n Z, rezult c s t t a [0, T ] un numr finit de puncte de discontinuitate fie acesta N . Funcia a t acelai numar de puncte de discontinuitate n orice interval [kT, (k + 1) T s Fie n = grad P ; atunci g : R R, g (k) = p ((k + 1) T ) p (kT ) e polinomial de grad n 1 n k, deci lim g (k) = . Pentru fixarea a
k k

Cum p ((k + 1) T ) p (kT ) N + [T ] + 2, ntre p (kT ) i p ((k + 1) T ) s minim N + [T ] + 2 valori ntregi, iar din continuitatea funciei polinomi t rezult c p ia toate aceste valori pe [kT, (k + 1) T ]. n intervalul [kT, (k a a afl cel mult [T ] + 1 numere ntregi, posibile puncte de discontinuitate pen a i cum p ({x}) este continu pe restul intervalului, rezult c f are cel pu s a a a puncte de discontinuitate pe [kT, (k + 1) T ], contradicie. t

presupunem c lim g (k) = +. Exist atunci k Z pentru care g (k) N a a

IMPORTANT

n scopul unei legaturi rapide cu redacia revistei, pot fi utilizate urm t adrese e-mail: tbirsan@math.tuiasi.ro sau t_birsan@yahoo profgpopa@yahoo.co.uk . Pe aceast cale colaboratorii pot purta c a ia un dialog privitor la materialele trimise acesteia, procurarea nu t revistei etc. Sugeram colaboratorilor care trimit probleme originale publicare s le numeroteze i s-i rein o copie xerox a lor pentru a s as t a purta cu uurina o discuie prin e-mail asupra acceptrii/neaccept s t t a a tora de ctre redacia revistei. a t

La problemele de tip L se primesc soluii de la orice iubitor de ma t elementare (indiferent de preocupare profesionala sau vrsta ). Fiecar soluiile acestor probleme - ce sunt publicate n revist dup un a t a a urmata de numele tuturor celor care au rezolvat-o.

Adresm cu insistena rugmintea ca materialele trimise r a t a s nu fie (s nu fi fost) trimise i altor publicaii. a a s t

Rugm ca materialele tehnoredactate s fie trimise pe adresa reda a a A soite de ierele lor (de preferina n L TEX). t s t

Probleme propuse1
Clasele primare

P.134. De la apartamentul meu cobor 7 etaje, apoi urc 4 etaje i obse s la etajul 9. La ce etaj locuiesc? (Clasa I ) Drago Iacob, s

P.135. n trei vase sunt 36 nuci. Daca din primul vas se iau 3 nuc treilea o nuc i se pun n al doilea vas, atunci n fiecare vas va fi acelai as s nuci. Cte nuci au fost la nceput n fiecare vas? (Clasa I ) nv. Rica Buct a P.136. Aflai vrsta tatalui meu tiind ca este un numar cuprins ntr t s dublul lui ntre 73 i 77, iar triplul lui este cuprins ntre 112 i 118. s s (Clasa a II-a) Iurie Juc, P.137. Dorin, Oana i Claudia se pregtesc pentru Concursul "Floric s a pan". Oana a rezolvat 15 probleme. Dorin a rezolvat un numar de proble faa de Oana, egal cu numrul de probleme rezolvate n plus de Oana faa d t a t Cte probleme au rezolvat mpreun cei trei copii? a (Clasa a II-a) Inst. Maria R i trei fii au mpucat fiecare cte un iepure. Cnd i-au P.138. Doi tai s t s au vzut c au doar patru iepuri. De ce? a a (Clasa a III-a) Inst. Elena N P.139. Mutai un singur chibrit pentru a obine o egalitate: t t

(Clasa a III-a) 10 0 11 1 12 2 13 14

Nicolae Ivchescu as 98 72 99 81

P.140. Descoper regula de completare a jetoanelor a ...

i calculeaz cte numere diferite sunt scrise pe aceste jetoane pe locul de s a (Clasa a III-a) Lenua Zaharia, e t

P.141. Fiul observa ca, atunci cnd i mai trebuia un an pna la vrstei din prezent, tatl avea vrsta de 12 ori mai mare dect a sa, iar c a 11 ani, vrsta lui va fi de 4 ori mai mic dect a tatlui. S se afle vrst a a a prezent. (Clasa a IV-a) Petru Asa

P.142. Paginile unei cri sunt numerotate de la 1 la 336. Din aceas at rup, la ntmplare, 111 foi. S se arate c: a a a) suma numerelor de pe foile ramase nu se mparte exact la 10; b) produsul numerelor de pe foile ramase se mparte exact la 3. (Clasa a IV-a) Maria Frangoi, e
1

Se primesc solu ii pn la data de 1 iunie 2008. t a

P.143. Aezai numerele 2, 3, 4, . . . , 10 n ptratul alturat astfel s t a a nct, pe fiecare linie, suma numerelor din primele doua casete sa fie egala cu numarul din ultima caseta. n cte moduri pot fi aezate aceste numere? s (Clasa a IV-a) Ionela Brgan, e a a

Clasa a V-a

V.81. Demonstrai ca putem completa cu numere naturale ntr-o in t moduri casuele libere din figura de mai jos, astfel nct sa se poata efec t operaiile indicate: t

Amalia Cantemir, e V.82. ntr-o ferma sunt gaini, oi i vaci, n total 324 de picioare i s s impar de capete: a) S se arate c n ferm nu pot fi 101 gini. a a a a b) Sa se arate ca numarul oilor nu poate fi egal cu numarul vacilor. Petru Asa V.83. Sa se demonstreze ca 13 | abc daca i numai daca 13 | 3 ab c. s Otilia Neme, Ocn s V.84. Determinai cel mai mic i cel mai mare numar natural de 9 t s divizibile cu 90 i avnd suma cifrelor 90. s Carmen Daniela Tama V.85. Fie a, b N; s se arate c dac ultima cifr a numrului a2 + a a a a a 2 atunci ultima cifr a lui (a + b) este tot 9. Reciproca este adevrat? a a a Ioan Scleanu a a 2 2 V.86. a) S se rezolve n numere naturale ecuaia x + y = 625. a t b) S se arate c ecuaia x2 + y 2 = 2007 nu are soluii n N2 . a a t t Valerica B V.87. S se arate c 751 > 389 . a a Nela Cice

Clasa a VI-a

VI.81. Stiind c 13 | 2a + 3b + 4c + 5d, artai c 13 | 43a + 45b + 47 a a t a 13 | 46a + 30b 64c 54d (a, b, c, d N). Norbert-Traian Ionia, t VI.82. Fie A = 3m 5n , m, n N. Notm cu a, b, c numrul divizorilor a a A, 3A, respectiv 5A. Stiind ca a i b sunt direct proporionale cu 3 i 4 t s s sunt invers proporionale cu 15 i 16, s se determine A. t s a Mihai Ha VI.83. Dac p este numr prim, iar n N , s se arate c p4n 3 nu e a a a a perfect. Mirela M

as a a) 3 | An dac i numai dac 3 - n 1; 9n 10n A2n n < 10n + 1 + b) 10 + 1 + < , n 3. 10 An 11

VI.84. Pentru n N , considerm An = 1 + 11 + 111 + + | {z . 1. A a 11 . . }


n

Temistocle B

VI.85. Pe latura Ox a unghiului drept xOy considerm un punct a bisectoarea unghiului considerm un punct B. Perpendiculara n B p a dreapta Oy n C. S se arate c AB = BC. a a Petru Asa

VI.86. a) Fie 4ADC i M (AC). S se arate c PADM < PADC . s a a b b) Fie 4ABC i (AD) bisectoarea unghiului A, D BC. Dac PABD s a sa se arate ca 4ABC este isoscel. Gheorghe Iu

VI.87. n figura alaturata sunt desenate 6 puncte, care unite doua cte doua dau natere la 9 drepte. Avem voie sa tergem unul dintre s s puncte i s-l desenm oriunde n alt parte. s a a a a) Efectuai operaia descris astfel nct, prin unirea cte dou a t t a a noilor puncte, sa se obina 11 drepte. t b) Care este numarul minim i cel maxim de drepte care se pot ob s configuraie permis? t a Gabriel P

Clasa a VII-a

VII.81. Se consider abc i xyzt numere r a s naturale scrise n baza 10. S s a q p q p s numerele naturale A = 6 2 abc i B = 3 2 xyzt. Bogdan Chiriac, stud

VII.82. Fie a, b numere reale strict pozitive. S se arate c: a a a) dac a3 b3 = a + b, atunci a2 + b2 > 1; a b) dac a3 + b3 = a b, atunci a2 + b2 < 1. a Ionel Nech

VII.83. Determinai numerele ntregi a, b, c, d pentru care ac + b t ad + bc = 2. Gheorghe Iu

VII.84. Fie patratul ABCD cu latura de lungime a, iar E, F , G a a laturile [BC], [CD], respectiv [AB] astfel nct CE = , CF = , iar BG 4 3 se arate c dreptele AE, BF i CG sunt concurente. a s Claudiu Stefan P

VII.85. Fie O intersecia diagonalelor patrulaterului ABCD. Daca t CD AB AABC = ACOD , sa se arate ca = 1. AB CD Doru Bu

VII.86. Fie A un punct pe manta unei mese de biliard circulare cu ra O bila pleaca din A i ajunge napoi n A lovind manta de cel puin trei o s t

bilei se face considernd c aceasta lovete un perete plan tangent la cerc a s de contact. Sa se arate ca exista o infinitate de traiectorii posibile i sa se s traiectoria de lungime minima. Cristian L

VII.87. O tabla are forma unui dreptunghi 4 5, format din 20 de 1 1. Avem la dispoziie doua jetoane, fiecare putnd acoperi cte un p t cte moduri putem aeza jetoanele pe tabl, astfel nct ele s nu se a s a a aceeai linie, nici pe aceeai coloan? Generalizare. s s a Gabriel P

Clasa a VIII-a

VIII.81. Considerm fixate numerele a, b Z , m, n N , m 6= n i a s f : N Z, f (x) = ax +b. Dac f (1) +f (2)+ + f (m) = f (1) + f (2) + a sa se calculeze suma S = f (1) + f (2) + + f (m + n). Dan Nedeianu, Dr.T

VIII.82. S se arate c |3xy + x + y| 1, x, y [0, 1]. a a Ovidiu Pop, Sa

VIII.83. Sa se arate ca nu exista x, y Z pentru care 147x2 = 1 + 4y Mihai Crciun a 8 VIII.84. Laturile a, b, c ale unui triunghi verific egalitatea 2 a + a 4 4 4 2 a + b + c . Sa se arate ca triunghiul este dreptunghic. Corina Elena Vian s

VIII.85. Dac a, b, c sunt numere reale pozitive, s se arate c a a a q p p 1 2 3 a2 + b2 ab + b2 + c2 bc 3 = a2 + c2 + = . a c b Liviu Smarandache

VIII.86. O piramid hexagonal regulat V ABCDEF are muchia baz a a a cm i nlimea V O = 4 2 cm. Fie M mijlocul lui V D, {P } = AD s at {Q} = P M (V CF ). S se arate c: a a a) dreptele V P i DQ sunt concurente; b) DQ (V BF ). s Gabriel P

VIII.87. Considerm prisma triunghiular regulat ABCA0 B 0 C a a a AM CN A0 M 0 C 0 N 0 , unde M este punct interior triunghiului ABC. Fie F 0 mijloacele muchiilor [AB], [BC], [A0 B 0 ], respectiv [A0 C 0 ]. a) Aflai msura unghiului dintre dreptele EF 0 i E 0 F . t a s b) Aflai msura unghiului format de planele (M CC 0 ) i (BCC 0 ). t a s Claudiu Stefan P

Clasa a IX-a

IX.81. Fie a, b R, n N . Daca ecuaia x2 + ax + b + 2 = 0 are ambe t ntregi, artai c numrul 2a2 + b2 este natural compus. a t a a Dorin Mrghidanu, a IX.82. Determinai funciile f : R R pentru care t t

f x4 + y 3 + z 2 + t = f (x) + f y 2 + f z 3 + f t4 , x, y, z, t Lucian Tuescu i Liviu Smarandache t s IX.83. Pentru a 9, s se demonstreze c are loc inegalitatea a a q q 3 + 3a + 9 1 + 1 + a. Marian Tetiv IX.84. Fie ABC un triunghi. Determinai numerele ntregi a, b, c nen t ntre ele doua cte doua, astfel nct punctele M , N , P sa fie coliniare, un P sunt determinate prin condiiile AM = aAB; CN = bCA; CP = cBC. t Ioan Scleanu a a IX.85. Fie ABC un triunghi ascuitunghic i D = prBC A, E = prC t s prAB C. Demonstrai echivalena afirmaiilor urmatoare: t t t (i) 4ABC este isoscel; (ii) DB + EC + F A = DC + EA + F B; 1 1 1 1 1 1 (iii) + + = + + . DB EC FA DC EA F B Examinai cazurile n care 4ABC este obtuzunghic sau dreptunghic. t Temistocle B

Clasa a X-a
X.81. S se rezolve n R R R sistemul a x y 2/3 = z 1/3 ; x4/3 y = z 2/3 ;

z 5/3 y 4/3 = z. Vasile Chiria

X.82. Solve the equation aex + b ex 3 = ax3 + b x3 2 + a, a > b > 0. Zdravko Starc, Vra X.83. n exteriorul triunghiului ABC se construiesc triunghiurile isosc \ AN C i CP B de baze AB, AC i respectiv BC, astfel nct m(M AB s s \ = 45 , iar m(P BC) = 30 . S se arate c m(M P N ) = 60 . \ \ m(N AC) a a a Angela Tigeru X.84. Fie ABC un triunghi n care (tg B 1) (tg C 1) = 2. Daca M picioarele nlimilor din B, respectiv C, s se arate c segmentele BM , C at a a se pot constitui n laturi ale unui triunghi. Ctlin Cali a a X.85. Se prelungete diametrul [M N ] al unui cerc C cu segmentul [N P ] s cu [M N ]. Fie d perpendiculara n P pe M N i R d oarecare. Tangentele s R la C intersecteaza tangenta n M la C n S i T . Sa se arate ca centrul d s al 4RST este un punct fix. Adrian Reisn

Clasa a XI-a

{x} XI.81. Dac m Z, s se studieze existena limitei lim a a t . xm sin x Dan Popescu

XI.82. Considerul irul (an )nN , definit prin a0 = 0, a1 = 1 i an+2 = 3 s s 3 5 n N. Pentru x 0, , aratai ca irul xn = a0 + a1 x + a2 x2 + t s 2 n N, este convergent i calculai limita sa. s t Vlad Emanuel, el XI.83. Fie f : [0, ) [0, ) pentru care are loc relaia f (f (x)) + 9x t x [0, ). Aratai ca f (x) 3x, x [0, ). t Bogdan Poa i Marius Drgoi, elev s s a XI.84. Determinai numerele a R pentru care exist o funcie t a t f : R R astfel nct (f f ) (x) = a2 f (x) 2a4 x, x R. Andrei Ned XI.85. Fie A = (aij )20072007 o matrice patratica n care aij {1, 0, 1, 2007. S se arate c determinantul matricei 2008I2007 + A este nenul. a a Paul Georgescu i Gabriel P s

Clasa a XII-a

Dumitru Mihalach XII.84. Fie polinomul f Z [X], f = a0 X + a1 X 2n + + a2n X pentru care n este impar, a0 a2n+1 este impar, iar a1 a2 este par. Sa se arat f are toate rdcinile reale, cel puin una este iraional. a a t t a Mihai Ha XII.85. Fie n N, n 2. Artai c exist P Z [X] de grad n a a t a a toate mulimile Ak = {P (i) (mod k) | i Z}, k N, k 2, sa aiba cardin t mai mic dect k. Vlad Emanuel, el
2n+1

XII.81. Dintre toate parabolele y = ax2 + bx + c, s se determine a trece prin punctele A (0, 1), B (1, 2), satisface condiia y 0 pentru 0 t realizeaza minimul ariei determinata de graficul parabolei, Ox i drepte s respectiv x = 1. Adrian Cordune XII.82. Determinai primitivele funciei f : (1, ) R, t t 4 3 x 5x + 3 (ln x 1) f (x) = . 3 (x4 1) Dan Nedeianu, Dr. Tr XII.83. S se determine funciile continue f : R R pentru care a t Z x et f (x t) dt, x R. f (x) = |x| +
0

Probleme pentru pregtirea concursurilo a


A. Nivel gimnazial

G126. Sa se determine numerele naturale care au proprietatea ca med tric a tuturor divizorilor lor este un numr natural. a a Petru M

G127. Dac a, b, c, x, y, z, t sunt numere reale pozitive, s se de a a inegalitatea 1 1 1 1 8 3 p + + + > ax+by+cz ay+bz+ct az+bt+cx at+bx+cy a2 +b2 +c2 x2 +y

D. M. Btineu-Giurgiu, B a t

G128. Fie a, b, c numere reale pozitive astfel nct abc = 1 i fie t [1 s arate c a a b c 3 + + . a2 + t b2 + t c2 + t t+1 Titu Zvonaru, Comneti i Bogdan Ionia, B a s s t 1 = {xy} + G129. Sa se determine y R pentru care {x} + x + y (Cu {} am notat partea fracionara.) t Alexandru Negrescu, elev,

G130. Fie a, b, c lungimile laturilor unui triunghi ABC. Dac a2007 a 2007 b 2 + 1 c2007 , s se arate c unghiul C este ascuit. a a t Lucian Tuescu t

G131. Fie n, k 2 numere naturale i mulimea M = { (n 1) , . . . , s t 2, . . . , n}. Sa se arate ca M se poate partiiona n k submulimi avnd ec t t sum a elementelor dac i numai dac n se divide cu k. a as a Marian Tetiv

G132. n fiecare cmp unitate al unei livezi m n se afl cte un a numar de k arici pornesc, pe rnd, din cmpul stnga-sus al livezii i se s cmpul din dreapta-jos. La fiecare micare, un arici se poate deplasa cu s spre dreapta sau n jos, fr a iei din livad. Ariciul poate s culeag aa s a a a cmpul pe care l viziteaz, dac nu a fost cules deja de alt arici. Care est a a minim k, pentru care k arici pot sa culeaga toate merele? Iurie Boreico, elev,

G133. Fie 4ABC echilateral i D un punct astfel nct BD = DC, m s \ 30 , iar BC separ A i D. Dac E (BD) cu m(BAE) = 15 , s s a s a a CE AC. Enache Ptracu a s

G134. Se considera patrulaterul convex ABCD nscris ntr-un cerc d b b cm, avnd m(A) = 60 i m(B) = 45 . S se arate c aria patrulaterul a a s 2 mult egal cu 3 6 cm . a Constantin Apostol, R

G135. Fie tetraedrul ABCD cu AB = CD, AC = BD, AD = BC. S ca cel puin doua dintre unghiurile diedre formate de faa (ABC) cu fee t t t (ACD), (ABD) sunt ascuite. t Dan Br

B. Nivel liceal

L126. Fie ABC un triunghi ascuitunghic. Madiatoarea laturii AB int t latura AC n T , iar mediatoarea laturii AC intersecteaza latura AB n S. S ca paralela prin T la AB, paralela prin S la AC i simediana din A sunt c s Titu Zvonaru, C a a L127. Fie A1 A2 A3 A4 A5 A6 un hexagon inscriptibil. S se arate c unde rXY Z este raza cercului nscris n 4XY Z.

rA1 A2 A3 + rA4 A5 A6 + rA1 A3 A6 + rA3 A4 A6 = rA3 A4 A5 + rA1 A2 A6 + rA2 A3 A6 +

Dorel Bian i I.V.Maftei, B at s L129. n planul raportat la un reper cartezian xOy consideram vec n O: v1 (a1 , b1 ), v2 (a2 , b2 ), v3 (a3 , b3 ). S se arate c exist un tetraed a a a regulat, de muchie 1 i astfel nct OA, OB, OC se proiecteaza pe planul s v2 , respectiv v3 daca i numai daca se verifica simultan relaiile: t s 3 2 3 2 a1 + a2 + a2 a1 a2 a1 a3 a2 a3 = b1 + b2 + b2 b1 b2 b1 b3 2 3 2 3 2 2 3 (a1 b1 + a2 b2 + a3 b3 ) (a1 b2 + a2 b1 + a1 b3 + a3 b1 + a2 b3 + a3 b2 ) 2 Irina Mustaa, student t a L130. S se arate c pentru orice x, y 1 are loc inegalitatea a a (xy x y)2 + 6 3 10 xy 6 3 9.

Ctlin Cali a a L128. Sa se arate ca ntre medianele unui triunghi are loc inegalitatea ih X X hY Y X m2 m2 m4 m2 m2 (ma + mb ) 2 8 ma a b a b a

Gabriel Dospinescu, Paris i Marian Tetiv s L131. S se afle valoarea minim a numrului real k astfel nct, oric a a a b, c reale pozitive cu a + b + c = ab + bc + ca, sa aiba loc inegalitatea 1 1 1 (a + b + c) + + k k. a+b b+c c+a Andrei Ciupan, elev, B L132. Fie a, b, c, x, y, t R i A = ax+by +cz, B = ay +bz +cx, C = a s Daca |A B| 1, |B C| 1 i |C A| 1, aratai ca a2 + b2 + c2 x2 + y s t Adrian Zahariuc, ele L133. Determinai funciile f : N N pentru care t t 2f (n + 3)f (n + 2) = f (n + 1) + f (n) + 1, n N. Gheorghe Iu

L134. Avem un colier cu n mrgele, numerotate consecutiv 1, 2, . . a n 3. n cte moduri putem sa le coloram cu trei culori, astfel nct or margele consecutive sa aiba culori diferite? Iurie Boreico, elev, L135. Se consider un poligon cu 3n laturi, n 2, nscris ntr-un cerc a Aratai ca cel mult 3n2 dintre segmentele avnd capetele n vrfurile poli t lungimea strict mai mare dect 2. Bianca-Teodora Iordache, elev a

Training problems for mathematical contes


Junior highschool level

G126. Determine the natural numbers such that the arithmetic mean divisors is a natural number as well. Petru M G127. If a, b, c, x, y, z, t are positive real numbers, prove the inequa 1 1 1 1 8 3 p + + + > ax+by+cz ay+bz+ct az+bt+cx at+bx+cy a2 +b2 +c2 x2 +y D. M. Btineu-Giurgiu, B a t G128. Let a, b, c be positive real numbers such that abc = 1 and let Show that b c 3 a + + . a2 + t b2 + t c2 + t t+1 Titu Zvonaru, Comneti and Bogdan Ionia, B a s t 1 1 G129. Determine y R such that {x} + x + = {xy} + , x y y {} denoting the fractional part.) Alexandru Negrescu, highschool student, G130. Let a, b, c be the side lengths of a triangle ABC. If a2007 2007 b 2 + 1 c2007 , show that the angle C is acute. Lucian Tuescu t G131. Let n, k 2 be natural numbers and consider the set M = { (n 2, 1, 1, 2, . . . , n}. Show that M can be partitioned into k subsets with sum of the elements in each of them if and only if n is divisible by k. Marian Tetiv G132. A rectangular garden has m n unit squares. On each of the an apple. A number of k hedgehogs start successively from the rst left t moving to the right bottom square. At each step, any hedgehog may move right or below (still remaining in the garden) and picking up the apple on t unless the apple was not earlier picked up. Which is the least number k of able to pick up all the apples? Iurie Boreico, highschool student,

G133. Let 4ABC be an equilateral triangle and D a point such that B \ = 30 , and BC separates A and D. If E (BD) with m(BA \ m(BDC) show that CE AC. Enache Ptracu a s G134. It is considered the convex quadrilateral ABCD inscribed in b b of radius of 6 cm, with m(A) = and m(B) = 45 . Show that the a 60 quadrilateral is at most equal to 3 6 cm2 . Constantin Apostol, R G135. Let ABCD be a tetrahedron with AB = CD, AC = BD, A Show that at least two of the angles between the surface (ABC) with th (BCD), (ACD), (ABD) are acute. Dan Br

B. Highschool level

L126. Let ABC be an acute-angled triangle. The mid-perpendicu side AB intersects the side AC at point T , and mid-perpendicular line o intersects the side AB at point S. Show that the parallel line to AB, thro parallel line to AC, through S and the simedian from A are three concur Titu Zvonaru, C L127. Let A1 A2 A3 A4 A5 A6 be an inscriptible hexagon. Show that

rA1 A2 A3 + rA4 A5 A6 + rA1 A3 A6 + rA3 A4 A6 = rA3 A4 A5 + rA1 A2 A6 + rA2 A3 A6 +

Dorel Bian and I.V.Maftei, B at L129. In the Cartesian system xOy, let three vectors v1 (a1 , b1 ), v3 (a3 , b3 ) be sitting with their origin in the point O. Prove that there exis tetrahedron OABC with the edges of length 1, such that the projections o OC (onto the plane xOy) are equal with v1 , v2 , v3 , respectively, if and equations below are simultaneously satised: 3 2 3 2 a1 + a2 + a2 a1 a2 a1 a3 a2 a3 = b1 + b2 + b2 b1 b2 b1 b3 2 3 2 3 2 2 3 (a1 b1 + a2 b2 + a3 b3 ) (a1 b2 + a2 b1 + a1 b3 + a3 b1 + a2 b3 + a3 b2 ) 2 Irina Mustaa, student t L130. Show that the following inequality holds for any x, y 1: (xy x y)2 + 6 3 10 xy 6 3 9.

where rXY Z is the radius of the circle inscribed in XY Z. Ctlin Cali a a L128. Show that the following inequality involving the bisectors of th medians) of a triangle holds: Y ih X X hY X 8 ma m2 m2 m4 m2 m2 (ma + mb ) 2 a b a b a

Gabriel Dospinescu, Paris and Marian Tetiv

L131. Find the minimum value of the real number k such that any wo real positive numbers a, b, c with a + b + c = ab + bc + ca, the following holds : 1 1 1 (a + b + c) + + k k. a+b b+c c+a Andrei Ciupan, highschool student, B L132. Let a, b, c, x, y, t R and A = ax+by+cz, B = ay+bz+cx, C = a If | A B | 1, | B C | 1 and | C A | 1, show that a2 + b2 + c2 x2 + Adrian Zahariuc, highschool studen L133. Determine the functions f : N N satisfying the equation 2 f (n + 3) f (n + 2) = f (n + 1) + f (n) + 1, n N.

Gheorghe Iu L134. A necklace consist of n pearls consecutively numbered 1, 2, . . . n 3. How many ways exist to colour these pearls using three colours s two neighbor pearls have dierent colours? Iurie Boreico, highschool student, L135. It is considered a polygon with 3n sides, such that n 2 and th is inscribed into a circle of radius 1. Show that at most 3n2 line segments ends at pairs of polygons vertices have their length strictly greater than Bianca-Teodora Iordache, highschool student

Premiu pe anul 2007 acordat de ASOCIA TIA RECREA MATEMATI TII


Se acord un premiu n bani n valoare de 100 lei elevului a

CIACOI Bogdan Liceul teoretic "Ana Ipatescu", Gherla

pentru nota O propoziie echivalenta cu conjectura lui Goldbach aparuta t 1/2007 al revistei Recreaii Matematice, la pagina 27. t Se acord, de asemenea, un premiu n bani n valoare de 100 lei elevil a

BOREICO Iurie, elev, Chiinu i CIUPAN Andrei, elev, Buc s a s

pentru nota Inegalitati stabilite cu un procedeu de reducere a numarului d - Mixing variables aprut n acest numr la pagina 100. a a a

Aceste premii sunt acordate de Asociaia "Recreaii Matematice" t t a contractului de sponsorizare cu Fundaia Cultural "Poiana" (directo t a Tiba).

Pagina rezolvitorilor

BRA OV S Colegiul Naional de Informatica "G. Moisil". Clasa a IX-a. AMBR t dra: VII(71,73,77,80), VIII(72,73,74), IX.80; BACIU Dan: VII(71,80), V IX.80; BAUM Bianca: VII(78,79), VIII(77,78), IX.80; DAINEANU Catalin 77-80), IX.80; FARSCH Cristina: VII(79,80), VIII(77,78), IX.80; FIN ana: VII(77,78,80), VIII(71-74,77,78), IX.72; HERMENEANU Horia: V VIII(77,78), IX.80; MAN Andrada: VII(77,79,80), VIII(77,78), IX.80; Laureniu: VII(71,73,77), VIII(72,74); NEGREA Andrei: VII.80, VIII(71t OANCEA Mihai: VII.80, VIII(71-74); OGLINDA Sorin: VII(71,73), V IX.80; POTEC Ionela: VII(77,79), VIII(77,78), IX.80; SIMION Adrian VIII(71-73), IX.80; TURTUREANU Veronica: VII(71,79), VIII(71-73,77,7 BOGDANE TI (SUCEAVA) S Scoala de arte si meserii. Clasa a VI-a. SOLCANU Mdlina Vasilica: a a 76,78,79), VII.75, VIII.75. HRLAU Liceul "Stefan cel Mare". Clasa a V-a. BARAU Larisa Ionela: P(121-1 VI.71; BUZILA Andreea: P(121,122), V(72,73), VI.71; CEUCA Razvan: P V.73, VI(71,72); IVANUTA Andreea Simona: P(121-123), V.73, VI.71; Adina-Diana: P(121-123), V.72, VI.72.

IA I S Scoala nr. 3 "Al. Vlahua". Clasa a IV-a (nv. MARIUTA Valentina t Alina: P(125-129,131); DIACONITA Theodor: P(125-129,131); HADAR Maria: P(125-129,131); NASTASE Cosmin: P(125-129,131); POPA Iulia 129,131); PROCA Ancua: P(125-129,131); TNCU Alexandra Ioana: P t 131). Clasa a IV-a (inst. MAXIM Gabriela). BACIU Ionela-Lavini 128); CELMARE Raluca-Iuliana: P(124-128); POPOVICI Ionu: P(124-12 t Mdlina-Andreea: P(124-128); SAVA Vlad: P(124-128); VECHIU Mdli a a a a 128).

Scoala nr. 13 "Alexandru cel Bun". Clasa a III-a (inst. COJOCAR AGAFITEI Elena-Roxana: P(124,125,127-129); CARAMALAU Andra: 127-129); CALIN Andreea-Claudia: P(124,125,127-129); COJOCARIU P(124,125,127-129); DUDUMAN Luisa- tefania: P(124,125,127-129); LE S xandrina- tefana: P(124,125,127-129); LUPA CU Diana-Maria: P(124,125 S S MANOLACHE Mdlina-Andreea: P(124,125,127-129); MIHAILA Narci a a P(124,125,127-129); PASCU Gabriela: P(124,125,127-129); PADURARU Stefan: P(124,125,127-129); RADUCEA Marin-Andrei: P(124,125,127-12 Cristina-Simona: P(124,125,127-129); STEFAN Bogdan-Vasile: P(124,125 STIUBEI Cosmin-Ionu: P(124,125,127-129). t Scoala nr. 14 "Gh. Mrzescu". Clasa a II-a (inst. NUTA Elena). B dor: P(114-118, 124-128); CHIRILU A George- tefan: P(114-118, 124-1 T S a a ORGHITA Mdlina-Gabriela: P(114-118; STOICA Adriana: P(114-118 TMBALARIU Ioana-Vasilica: P(114-118).

Scoala nr. 26 "G. Cobuc". Clasa a II-a (inst. RACU Maria). AP s Aura-Georgiana: P(124-129); BURA Emma-Andreea: P(124-129); CIORN dra: P(124-127,129); CRACIUN Ioana-Daniela: P(124-127,129); FILIP In nia: P(124-129); GHEORGHITA Narcis-Eugen: P(124-127,129); GRA Georgiana: P(124-129); HRISCU Ovidiu-Constantin: P(124-129); HUZA P(124-129); MARICIUC Drago-Claudiu: P(124-127,129); MAXIM Alexan s lia: P(124-129); TUCA Cosmin: P(124-127,129); VASILE Bogdan-Andr 129). Colegiul Naional, locaia "Gh. Asachi". Clasa a III-a (inst. CA t t Rodica). BERECHET Alexandru: P(126-130); GHENARU Brdia: P a t CHIVULESCU Alexandru: P(126-130); PETREA Mdlina: P(126-130); U a a ANU George: P(126-130). Colegiul Naional. Clasa a VII-a. CEUCA Razvan: V(71-75), VI( t VII(71-75), VIII(72,73,75); MOCANU Dan Mihai: V(76-79), VI(76,78,79), PETRESCU Emanuel: V(76-79), VI.76. Colegiul Naional "C. Negruzzi". Clasa a VI-a. PAVALOI Alexand t 123), VI(71-74). Colegiul Naional "Emil Racovia". Clasa a VII-a. TUDORACHE A t t VII(76-80), G(117,118). SUCEAVA Scoala cu clasele I-VIII, nr. 3. Clasa a II-a (nv. TABARCEA Silve CHET Stefan: P(114-118). Clasa a III-a (inst. NECHITA Daniela). Mircea: P(114,115,117-119).

Premii acordate rezolvitorilor

ASOCIA TIA "RECREATII MATEMATICE" n colaborare c revistei RECREATII MATEMATICE acord cte o diplom i un p a as cri pentru trei apariii la rubrica Pagina rezolvitorilor urmtorilor elev a t t a Scoala cu clasele I-VIII, nr.3, Suceava FECHET Stefan (cl. a II-a): 2/2006(5pb), 1/2007(5pb), 2/2007(5pb);

FECHET Mircea (cl. a III-a): 2/2006(9pb), 1/2007(6pb), 2/2007(5pb) Scoala nr. 3 "Al. Vlahua", Iai t s

HADARAG Ana-Maria (cl. a IV-a): 2/2006(6pb), 1/2007(5pb), 2/200

NASTASE Cosmin (cl. a IV-a): 2/2006(6pb), 1/2007(5pb), 2/2007(6pb

TNCU Alexandra-Ioana (cl. a IV-a): 2/2006(6pb), 1/2007(5pb), 2/2

BACIU Ionela-Lavinia (cl. a IV-a): 2/2006(8pb), 1/2007(6pb), 2/2007

RUSU Mdlina-Andreea (cl. a IV-a): 2/2006(8pb), 1/2007(6pb), 2/2 a a

SAVA Vlad (cl. a IV-a): 2/2006(8pb), 1/2007(6pb), 2/2007(5pb); Scoala nr. 26 "Gh. Cobuc", Iai s s APACHITEI Aura-Georgiana (cl. a II-a): 2/2006(6pb), 1/2007(5pb),

BURA Emma-Andreea (cl. a II-a): 2/2006(6pb), 1/2007(5pb), 2/2007

FILIP Ingrid- tefania (cl. a II-a): 2/2006(6pb), 1/2007(5pb), 2/2007( S

GRADINARIU Georgiana (cl. a II-a): 2/2006(6pb), 1/2007(5pb), 2/2

HRISCU Ovidiu-Constantin (cl. a II-a): 2/2006(6pb), 1/2007(5pb), 2/ HUZA Mdlina (cl. a II-a): 2/2006(6pb), 1/2007(5pb), 2/2007(6pb); a a

MAXIM Alexandra-Camelia (cl. a II-a): 2/2006(6pb), 1/2007(5pb), 2/

VASILE Bogdan-Andrei (cl. a II-a): 2/2006(6pb), 1/2007(5pb), 2/200 Colegiul Naional, locaia Scoala "Gh. Asachi", Iai t t s

PETREA Mdlina (cl. a III-a): 2/2006(5pb), 1/2007(7pb), 2/2007(6p a a

UNGUREANU Georghe (cl. a III-a): 2/2006(5pb), 1/2007(7pb), 2/20 Colegiul Naional, Iai t s

a CEUCA Rzvan (cl. a VII-a): 1/2005(6pb), 2/2005(5pb), 1/2006(5pb) Colegiul Naional "C. Negruzzi", Iai t s

PAVALOI Alexandru (cl. a VI-a): 2/2006(5pb), 1/2007(5pb), 2/2007( Colegiul Naional "Emil Racovia", Iai t t s TUDORACHE Alexandru (cl. a VII-a): 2/2006(10pb), 1/2007(10pb),

Vizitai pe Internet revista "Recreaii Matematice" la t t

http://www.recreatiimatematice.uv.ro

Revista semestrial RECREAII MATEMATICE este ed ASOCIAIA RECREAII MATEMATICE. Apare la datele de 1 1 septembrie i se adreseaz elevilor, profesorilor, studenilor i tutu pasionai de matematica elementar. n atenia tuturor colaboratorilor Materialele trimise redaciei spre publicare (note i articole, che metodic, probleme propuse etc.) trebuie prezentate ngrijit, clar i co trebuie s prezinte interes pentru un cerc ct mai larg de cititori. Se reco textele s nu depeasc patru pagini. Evident, ele trebuie s fie origin nu fi aprut sau s fi fost trimise spre publicare altor reviste. Rugm rialele tehnoredactate s fie nsoite de fiierele lor. Problemele destinate rubricilor: Probleme propuse i Problem pregtirea concursurilor vor fi redactate pe foi separate cu enun i de ie/rezolvare (cte una pe fiecare foaie) i vor fi nsoite de numele autoru la i localitatea unde lucreaz/nva. Redacia va decide asupra oportunitii publicrii materialelor prim n atenia elevilor Numele elevilor ce vor trimite redaciei soluii corecte la proble rubricile de Probleme propuse i Probleme pentru pregatirea conc vor fi menionate n Pagina rezolvitorilor. Se va ine seama de regulile: 1. Pot trimite soluii la minimum cinci probleme propuse n prezent i cel anterior al revistei; pe o foaie va fi redactat soluia une probleme. 2. Elevii din clasele VI-XII au dreptul s trimit soluii la pr propuse pentru clasa lor, pentru orice clas mai mare, din dou clase m imediat anterioare. Elevii din clasa a V-a pot trimite soluii la problemel pentru clasele a IV-a, a V-a i orice clas mai mare, iar elevii claselor trimite soluii la problemele propuse pentru oricare din clasele primare i s mai mare. Orice elev poate trimite soluii la problemele de concurs (tip 3. Vor fi menionate urmtoarele date personale: numele i pr clasa, coala i localitatea. 4. Plicul cu probleme rezolvate se va trimite prin pot (sau v direct) la adresa Redaciei:

Prof. dr. Temistocle Brsan Str. Aurora, nr. 3, sc. D, ap. 6, 700 474, Iai Jud. IAI E-mail: tbirsan@math.tuiasi.ro sau t_birsan@yahoo.

CUPRINS

Al VI-lea Congres Internaional al Matematicienilor Romni......................... Conjectura lui Poincar ...........................................................................................

ARTICOLE I NOTE

G. DOSPINESCU Tipurile subgrupurilor finite din GL2 ( ) .............................. T. ZVONARU O problem cu cifrele unui numr ............................................. T. BRSAN O problem de construcie a unui triunghi .................................... A. CORDUNEANU i Gh. COSTOVICI Un ir strns legat de irul lui Walli A. REISNER Asupra rdcinilor polinomului X 3 + pX + q [ X ] ...................

NOTA ELEVULUI

I. BOREICO i A. CIUPAN Inegaliti stabilite cu un procedeu de reducere a numrului de variabile Mixing variables .....................

CHESTIUNI METODICE MATEMATICA N CLASELE PRIMARE

J. GRIGORA i C.-t. POPA Asupra unei probleme de concurs .....................

P. ASAFTEI Intuirea proprietilor operaiilor aritmetice utiliznd metodele figurative .................................................

CORESPONDENE CONCURSURI I EXAMENE

H. STEPHAN Jensen's inequality for non-convex functions ..............................

Concursul de Matematic "Al. Myller", Ediia a V-a, Iai ................................... Concursul de matematic "Florica T. Cmpan", 2007 ............................................ Olimpiada Balcanic de Matematic Juniori, Ediia a XI-a, umen (Bulgaria)

PROBLEME I SOLUII

Soluiile problemelor propuse n nr. 2/2006............................................................. Soluiile problemelor pentru pregtirea concursurilor din nr. 2/2006 .................... Probleme propuse........................................................................................................ Probleme pentru pregtirea concursurilor ................................................................. Training problems for mathematical contests ..........................................................

Pagina rezolvitorilor .................................................................................................

Anul X, Nr. 2

Iulie Decembri

RECREAII MATEMATIC

REVIST DE MATEMATIC PENTRU ELEVI I PROF

125 de ani de la apariia revistei Recreaii tiinifice (1883 1888)

e i = 1

Asociaia Recreaii Matemati IAI - 2008

Semnificaia formulei de pe copert: i ntr-o form concis, formula e = 1 leag cele patru ramuri fun
ale matematicii: ARITMETICA GEOMETRIA ALGEBRA ANALIZA MATEMATIC reprezentat reprezentat reprezentat reprezentat de de de de 1 i e

Redacia revistei :

Petru ASAFTEI, Dumitru BTINEU-GIURGIU (Bucureti), Temistocle BRS BRNZEI, Ctlin - Cristian BUDEANU, Constantin CHIRIL, Eugenia COHA CORDUNEANU, Mihai CRCIUN (Pacani), Paraschiva GALIA, Paul GEO Mihai HAIVAS, Gheorghe IUREA, Mircea LUPAN, Gabriel MRANU, NEGRESCU (student, Iai), Gabriel POPA, Dan POPESCU (Suceava), Florin P (Braov), Maria RACU, Neculai ROMAN (Mirceti), Ioan SCLEANU (Hr ERDEAN (Ortie), Dan TIBA (Bucureti), Marian TETIVA (Brlad), Lucian (Craiova), Adrian ZANOSCHI, Titu ZVONARU (Comneti).

COPYRIGHT 2008, ASOCIAIA RECREAII MATEMATICE Toate drepturile aparin Asociaiei Recreaii Matematice. Reproducerea in parial a textului sau a ilustraiilor din aceast revist este posibil numai cu acordu scris al acesteia. TIPRIT LA SL&F IMPEX IAI Bd. Carol I, nr. 3-5 Tel. 0788 498933 E-mail: simonaslf@yahoo.com ISSN 1582 - 1765

Anul X, Nr. 2

Iulie Decembrie

RECREAII MATEMATICE

REVIST DE MATEMATIC PENTRU ELEVI I PROF

e i = 1
Revist cu apariie semestrial

EDITURA RECREAII MATEMATICE

IAI - 2008

O sut de ani de la naterea lui Gheorghe Gheo a s

Gheorghe Gheorghiev a fost elev al ilutrilor savani A. Myller i O. Mayer i a des t s s venit, pentru o buna perioada de timp, liderul colii de geometrie de la Universitatea din Iai. A s s fost un reprezentant al unei generaii de dascli t a ieeni care au vegheat la destinele matematis cilor romneti, ntr-o perioada de mari schimbari s politice. n acelai timp, a cautat sa reia legaturile s ntrerupte cu matematicile din lumea ntreag. A a fost un adevrat constructor de coal tiinic a s as t a la Facultatea de matematica din Iai, dovedindus se un merituos continuator al eforturilor i stras duinelor profesorului su A. Myller. t a S-a nscut la 27 iunie 1907 la Bolgrad, judeul a t Ismail. A urmat liceul din Bolgrad n anii 19161925, dupa care a susinut examenul de bacalureat t la Ismail n iunie 1925. Imediat a devenit student la Facultatea de tiine din Iai, secia matematic, unde a urmat cursu s t s t a de profesorii A. Myller, Vera Myller Lebede, O. Mayer, Simeon Saniele Culianu, Constantin Popovici, Mendel Haimovici, Gheorghe Vranceanu avut asistent pe Ilie Popa, cu care a rmas prieten pentru toat viaa. a a t ultimului an de studiu i dup examenul de licena, n martie 1929, a deven s a t i s-a mprietenit cu mai muli tineri matematicieni: I. Schoenberg, P. C s t D. Mangeron, E. Stihi, i cu studentul A. Climescu. Tot n 1929 a urmat s specializare la Universitatea din Hamburg, cu cunoscutul profesor W. Bla n 1929 s-a declanat criza economica pe plan mondial, care a afect s noastra. n perioada 1929-1938, Gh. Gheorghiev a fost profesor de mat liceele din Aiud, Ismail, Chilia Noua, Cetatea Alba i Iai. In perioada 19 s s fost nscris la doctorat la Universitatea din Iai. n aceeai perioad a fost s s a i mobilizat timp de peste 5 ani. A participat la rzboi ca ofier de geniu s a t circa un an i jumatate, a fost prizonier. A existat chiar pericolul sa fie m s momentul n care a fost luat prizonier, pentru ca era ofier. t n 1946 a susinut teza de doctorat cu titlul Suprafee ale caror familii r t t de curbe sunt asemenea (comisia de doctorat, fiind compus din A. Myller, a V. Myller, I. Popa). A funcionat la Universitatea din Iai ca lector (1946-1948), confereni t s t de matematici generale (1946-1948), profesor suplinitor de geometrie anali 1954), profesor titular de geometrie (1955-1975). ntre anii 1953-1968 a fost decan al Facultaii de matematica din Ia s t 1960-1975, a fost eful catedrei de geometrie. n aceste posturi i poziii de s s t Gh. Gheorghiev a avut o influena hotrtoare asupra dezvoltrii facu t a a privete stabilirea unor legturi cu direcii de cercetare tiinific existen s a t s t a colile de matematica din occident ct i cu cele existente n Uniunea Sov s s

A colaborat la elaborarea unui tratat de geometrie intitulat simplu C ometrie analitica (1951); recunoaterea meritelor acestui tratat s-a realizat s darea Premiului de Stat. A mai colaborat la realizarea unei cari Geometri t si difereniala, aprut n dou volume n 1968 i 1969 la Editura Didact t a a a s agogic. n aceast carte ncep s se reecte schimbrile de concepie i a a a a t s de investigare n ce privete studiul geometriei difereniale. Ideile au fost s t n alte doua cari, elaborate n colaborare, Varietati difereniabile finit si t t mensionale, aprut n dou volume n 1976 i 1979 la Editura Academie a a a s i Geometrie difereniala aprut la Editura Didactic i Pedagogic. s t a a as a A scris circa 180 de lucrari tiinice n domeniul geometriei diferen t s t meroase articole de popularizare a matematicii. A confereniat la mai mu t t sitai de prestigiu din lume. A primit premiul "Gh. Tieica" al Academie t a primit titlul de profesor emerit, mai multe diplome i medalii ale uno s profesionale i numeroase ordine i medalii, din partea statului romn. s s Dupa pensionare, n 1975, a ramas profesor consultant la Facultatea d tic (cu unele ntreruperi) pn la deces, n 28 iunie 1999. a a A abordat teme de cercetare tiinic din geometria diferenial euclidi s t a t a pe suprafee, cmpuri de vectori pe suprafee) geometria difereniala afina t t t tiv (cmpuri de conuri, conguraii Myller), geometria diferenial a v a t t a modelate de spaii Banach, teoria grupurilor Lie, teoria G-structurilor i g t s ale acesteia. Circa 30 de tineri studioi i-au elaborat tezele de doctorat sub s s rea tiinica a profesorului Gheorghiev. A colaborat, n elaborea carilor s t t colegi apropiai: R. Miron, D. Papuc, V. Oproiu. t A fost profesor vizitator i a confereniat la numeroase universitai di s t t Jena, Padova, Sofia, Londra, Moscova, Debrecen etc. A predat cursuri de geometrie analitica, difereniala, teoria grupurilor t diverse nivele, la Facultatea de matematic a Universitii din Iai. A m a at s i cursuri de matematici generale pentru studenii de la facultile Instit s t at litehnic din Iai. Leciile sale erau renumite prin densitatea informaiilor tr s t t pentru solicitarea intelectuala extrema la care erau supui studenii ce le s t n perioada ct a fost decan, n facultate s-a petrecut o extraordinar sc a argumentelor predate la cursuri i a celor abordate n cercetare, realizndu s de la aspectele tradiionale existente n matematicile din perioada interbe t matematicile moderne, cercetate i predate n stilul structuralist, promova s grupul Bourbaki. n ultimii ani ai vieii era preocupat de istoria matem t probleme matematice ale filosofiei fizicii. Meritele la catedr, n cercetarea tiinific, de om care, pe tot parcu a s t a sale, a practicat cultul muncii i al cinstei au facut din profesorul Gh. Gh s o personalitate n domeniul matematicilor, respectat att n facultate i u a s ct i pe plan naional i internaional. s t s t Activitatea sa n slujba colii romneti i n servirea matematicii a fost s s s prin mai multe distincii, titlul de Profesor Emerit (1964), Premiul de St t t premiul "Gh.Tieica" al Academiei Romne (1981), Ordinul Muncii (1960 Meritul Stiinific (1960), Medalii ale Muncii etc. t

Prof. dr. Vasile OP

Ilie Popa - 100 de ani de la natere s

Ilie Popa a ramas n contiina noastra ca s t unul dintre matematicienii remarcabili ai colii de s geometrie din Iai, cunoscut i recunoscut i peste s s s hotarele arii, ca un subtil i profund cercetator t s n domeniul istoriei matematicii, ca un profesor care, cu leciile sale expuse cu tact, claritate i t s farmec, a format numeroase generaii de profesori t i viitori cercetatori, care, de la nivelul funciilor s t de conducere asumate, a militat cu daruire i pris cepere pentru un nvamnt de calitate. at Intreaga viaa i activitatea sa variat i rodt s as nica sunt legate de oraul Iai. s s n Iai s-a nascut, la 20 iulie 1907. A urs mat coala primar "C. Negri". Apoi, n pes a rioada 1919-1927, Liceul Internat astzi Colegiul a Naional "C. Negruzzi" -, avnd ca profesor de t matematica pe Ion Raianu (pe care-l va evoca mai trziu cu recunotina i preuire). Pe placa de onoare a elor de p s t s t s liceului, n dreptul anului 1927, este scris numele su (pe aceeai plac, pe a s a aparea i numele ului sau, Eugen, la rndul lui distins matematician). E s cu faptul ca absolvise Liceul Internat i o afirma cu placere cnd avea pri s A continuat studiile la Facultatea de tiine (secia matematic) din Ia s t t a inut examenul de licena n 1931. A susinut doctoratul n matematici n t t t conducerea lui O. Mayer, cu teza Contribuii la geometria centro-afina di t La aceasta nirare de date se impune sa menionam rolul avut n form t s ambiana Seminarului matematic din Iai, de condiiile propice de munc t s t a oteca acestuia, ct i de nrurirea favorabil exercitat de A. Myller i O s a a s conducatorii de atunci ai colii matematice ieene. Ilie Popa s-a ncadrat s s de cercetare ale colii de geometrie din Iai, contribuind cu rezutatele o s s construcia geometriei centro-afine, considerat mai trziu o creaie pur ro t a t ntre 1936 i 1938, Ilie Popa a fcut stagii de specializare la Roma, cu En s a piani, i la Hamburg, cu Wilhelm Blaschke, doi renumii matematicieni ai s t Prin munca i merit a urcat treptele ierarhiei universitare, ocupnd dive s i prednd cursuri variate la Universitatea "Al. I. Cuza" i la Politehnica "G s s din Iai. n 1942, n urma plecrii prof. Gr. Moisil la Bucureti a fost titu s a s postul ramas vacant, ca profesor la Facultatea de tiine, catedra de calcul s t i integral. A fost eful catedrei de analiza matematica din 1948 i pna n s s s an n care, prin retragere la pensie, a devenit profesor consultant. La 26 iulie 1983 a ncetat din viaa dup o lung i grea suferina, fiin t a as t n cimitirul Eternitatea. Nu vom prezenta nici macar sumar contribuia adusa de Ilie Popa n t centro-afin diferenial, contribuie care l-a consacrat i fcut cunoscut a t a t s a nitatea matematic; vom observa doar colaborarea n acest domeniu cu a ieean Gheorghe Gheorghiev i prietenia durabila care s-a legat ntre ace s s

siderm c pentru cititorii prezentei reviste sunt mai accesibile, mai intere a a un folos mai mare alte aspecte ale activitaii variate pe care a desfaurat-o t s Avnd un spirit patrunzator, o gndire profunda i o mare putere de s beneciind de o bogat cultur general, Ilie Popa a abordat teme diverse a a a matematicii romneti aducnd claricri ntr-un material faptic din trecu s a destinat cu precadere uitarii. A scos la lumina meritele unor precursori a ticii romneti ca Dimitrie Asachi fiul cel mare al lui Gh. Asachi , prim s ce public n strintate (Mnchen, 1841) o lucrare de matematici supe a a a N. St. Botez, care d n 1872 o formul pentru o bucat a seriei armonice, a a a atenia matematicianului belgian E.-Ch. Catalan. Un alt studiu este d t Amlochie Hotiniul, care tiparete n Iai, la 1795, prima aritmetica romn s s menti aritmetice aratate fireti, o compilaie dup cteva cri italieneti. s t a at s o munc dicil de restaurare, atunci cnd reconstituie manuscrisul de geom a a dut al lui Gh. Lazar. A pus n evidena rolul jucat de soii Alexandru i V t t s n crearea colii matematice ieene, cu numele generic de Seminarul mate s s prilejul centenarului Universittii din Iai (18601960), public, n al d a s a consacrat evenimentului, studiul Dezvoltarea matematicii o ampl i do as contribuie privind realizarile colii matematice ieene pe parcursul unui s t s s S mai spunem cititorilor c Ilie Popa a publicat n 1955 studiul Rec a a inice precursoare a Gazetei matematice n care evideniaz rolul aceste t t a prima de acest fel din ar, strbun a actualei Recreaii Matematice n d t a a a t nvaamntului i netezirea drumului ce duce la cercetari originale n mat s t Ca profesor, Ilie Popa a fost un maestru al artei comunicrii. Leciil a t oficiate dup un ritual neabtut. ncepeau cnd profesorul punea ceasul d a a pe mas i se terminau cnd acesta era repus la locul lui. Era omis pa as a ore, dar timpul acordat prelegerii era respectat cu strictee. Cu un timbru t vorbire blnd, moldoveneasc, cu un ritm al expunerii care antrena dar a a profesorul reuea imediat s captiveze auditoriul. Ideile i argumentele er s a s ntr-o nlnuire uor de urmrit. Erau scoase n evidena semnificaiile c a t s a t t i rezultatelor. Faa magistrului se acoperea de o lumina discreta, cea s t de a drui. Cte un zmbet involuntar, reinut cu grij, marca un mome a t a leciei sau nsoea un rezultat de o rar frumusee. Calmul, echilibrul, t t t a t ntotdeauna prezente. George St. Andonie spune inspirat: i alege detalii s si obiectivitate si lasa impresia ca-i suna n prealabil cuvintele, spre a-i s s de rezonana lor (Istoria matematicii n Romnia, vol. III). t A avut numeroase funcii de conducere n diverse structuri ale nv t lui superior: prorector al Universitaii din Iai, director al Seminarului M t s "Al. Myller", director general n Ministerul nvaamntului, rector al I t Pedagogic de 3 ani din Iai .a. Cu devotament i competena, a folosit s s s t capacitate de munc n scopul ridicrii nvamntului romnesc. a a at La mplinirea unui secol de la naterea sa, ne gndim cu profund re s a la cel ce a fost omul i profesorul Ilie Popa i cu toata admiraia la op s s t geometru, de istoriograf al matematicii i nvamntului matematic i d s at s nvamntului din ara noastr. at t a

Prof. dr. Temistocle B

Simpozion

dedicat revistei "Recreaii Stiinifice" (1883-1 t t

Academia Romna - liala din Iai a gazduit, n ziua de 15 martie s pozionul "125 de ani de la apariia revistei Recreaii stiinice". Festi t t t desfurat sub auspiciile Academiei Romne, Facultii de matematic a Un as at a "Al. I. Cuza", Catedrei de matematic a Universitii Tehnice "Gh. Asa a at Asociaiei "Recreaii matematice". t t Cuvntul de deschidere a aparinut acad. Viorel Barbu, preedintele fil t s Academiei Romne, care a subliniat faptul c revista Recreaii stiinifice a t t publicaie din ar cu acest profil adresat tineretului. Alturi de alte e t t a a a ieene remarcabile ale anului 1883 apariia ediiei T. Maiorescu a Po s t t Mihai Eminescu, inaugurarea statuii lui Stefan cel Mare .a. apariia R s t tiinifice reprezint un moment important al culturii i spiritualitii rom s t a s at Fondatorii, distini profesori ai Universitii din Iai sau ai colilor ie s at s s s colaboratorii revistei au asigurat o nalta inuta tiinifica acesteia. Revista t s t n ntregul Regat al Romniei de atunci i se remarca prin varietatea s s abordate, rigoare, frumoasa limb romn folosit i o grafic excelent pe a a as a a timpuri. Acad. V. Barbu menioneaza ca n cadrul simpozionului este lansat t integrala a revistei Recreaii stiinifice, reeditata n forma originara, nem t t Realizarea acestui proiect de reeditare se datorete sprijinului material e s doamnei Marinela Ghigea, director al firmei Kepler Systmes dInformatio i muncii depuse de dr. Dan Tiba, cercetator la Institutul de matematica al s Romne i de prof. dr. Temistocle Brsan, Universitatea Tehnic "Gh. As s a Cuvntrile inute de acad. Radu Miron, prof. dr. Vasile Oproiu, prof. a t Iean, m.c. al Academiei, prof. dr. Teodor Precupanu, n aceast ordine, su s a tate mai jos. Programul simpozionului se ncheie cu proiecia unor d t privind revista Recreaii tiinice i epoca n care a aprut aceasta pre t s t s a prof. dr. Temistocle Brsan.

Recreaii Stiinifice t t 125 ani de la apariia primului numr t a


Acad. prof. dr. doc. Radu MIRON

La 15 ianuarie 2008 s-au mplinit 125 de ani de la publicarea numa primul volum al revistei Recreaii tiinif ice. Menita a face educaie t s t t tineretului din Regatul Romniei, revista, care a avut o existena de numai t depit graniele n toate zonele locuite de romni. nfiinat de zece oame as t t a din vechea capitala a Moldovei, ea avea sa imprime n contiina locuitori s t

pamnt primele capitole elevate de istorie a tiinelor din ara i cele s t t s pentru un nvamnt modern n domeniul tiinelor exacte. Peste veac at s t nrurirea covritoare a ideilor vehiculate n cuprinsul acestei reviste, n s toate gradele i n cercetare, conducnd la integrarea noastr n rndul s a t aveau deja tradiii seculare. t Datorit coninutului preponderent matematic se poate afirma cu de a t c revista a deschis prima pagin a matematicilor romneti. Aa cum s-a a a s s mai trziu, dac publicaia s-ar fi numit Recreaii matematice, ea ar fi a t t prima publicaie din lume n domeniu, care se adreseaza tineretului. Est t c, la apte ani de la dispariia "Recreaiilor tiinifice", n 1895 a fost a s t t s t Gazeta Matematica cu adres special pentru tineretul romn. Dar acea a a este considerat a doua din lume ca profil i destinaie. a s t t Evident, apariia "Gazetei", aa cum sublinia Gheorghe Tieica a fost t s nat de "Recreaiile tiinifice". a t s t Acum, la 125 de ani de la nfiinare a celebrei reviste se cuvine s t a omagii profunde memoriei fondatorilor: N. Culianu, C. Climescu, I. M Universitatea "Al. I. Cuza", G. I. Lucescu, V. Paladi, G. I. Roiu, I. D s G. Zarifopol, I. V. Praja i I. M. Dospinescu din nvamntul preu s at ieean. Prin competena, pasiune i sacrificii personale fcute cu generoz s t s a reuit s trezeasc interesul pentru tiina n general i s stimuleze gus s a a s t s a matematici n special. Sunt emoionante cuvintele scrise ntr-un editorial al revistei: Credem t tras cea ntai brazda care conduce catra lucrari originale. Brazda-i mica dar exista! Personalitatea fondatorilor este bine cunoscuta. Ei sunt prezentai de t Andonie n volumul I din Istoria Matematicii n Romnia. Majoritate oameni de tiina cu studii nalte fcute n Frana, Italia, Olanda i Germ s t a t s gnd de recunotina colaboratorilor, nu mai puin celebri: M. Tzony, V s t t P. Tanco, C. Gogu i rezolvitorilor pasionai, elevi pe vremea aceea, E. s t i D. Pompeiu. s Nu trebuie s-i uitm pe oamenii de tiina care au susinut peste timp i a a s t t revistei i impactul ei n cultura romneasca. Citam doar civa dintre ei: s t Myller, Octav Mayer, Ilie Popa, Gheorghe Gheorghiev, Gheorghe Banta, s Tieica, G. St. Andonie, N. N. Mihaileanu etc. t Condiiile istorice n care a aprut n 1883 revista "Recreaii tiinific t a t s t dintre cele mai favorabile. Unirea Principatelor abia se nfaptuise, Regatul era abia ntemeiat, Razboiul de Independena din 1877 lasase urme adnc t ina romnilor, alfabetul chirilic fusese nlocuit cu cel latin, limba rom t abia i definitivase procesul de unificare, romnii i afirmau n mod de s s raia spre o societate moderna. n atari condiii, dei aparusera cu 23 de t t s universitaile din Iai i Bucureti, coala de toate gradele trebuia profund s s s s t Era nevoie imperioas de regndit programarea curricular, de pregtit a a a

didactic, de scris manuale bune n limba romna, de construit coli etc. s n atari condiii spirituale, materiale i sociale dure, a pune bazele u t s de cultur tiinific era un act de curaj, de patriotism. El constituia o i as t a realizare destinat poporului nostru. Soliditatea acestui edificiu este dat d a a tiinifica, didactica i educaionala a subiectelor publicate, de limbaju s t t s adoptat, de grafica de excepie utilizat n acea vreme. Am prezentat aces t a n articolul "Centenarul revistei Recreaii stiinif ice", Probleme de istoria t t tiinei, vol. X, 1984, Filiala Iai. Valabilitatea afirmaiilor fcute atunci s t s t a s temeiul i astazi. Din acest motiv reproduc o parte din text. s Tonul ntregii producii matematice, cuprinznd mai bine de 90% din t de pagini ct nsumeaza aceasta revista, a fost dat n primul rnd de fon care, prin prestigiul lor, au atras foarte curnd valoroi colaboratori: Milti s profesor de mecanica teoretica la Universitatea din Iai, Candide (proba s Costin, pe atunci student la Paris), Iacob Solomon inginer, Paul Tanc sor de matematica si f izica la Gimnaziul Superior din Nasaud, Constant profesor de geometrie analitica la Universitatea din Bucureti, Vasile Bu s profesor de mineralogie si petrograe la Facultatea de stiinte din Iai s. a s n paginile revistei sunt publicate articole, note, probleme si soluii di t ca: aritmetica, algebra, geometrie elementara, geometrie analitica si difere cul diferenial si integral, mecanica, astronomie, istoria matematicii, chim t geograf ie. Apare prima traducere a carii nti din celebrele "Elemente" t clid. Miltiade Tzony tiparete n coloanele ei o remarcabila culegere de s de mecanica teoretica. Geometria proiectiva, domeniu de mare actualita vreme, este prezenta prin traducerea primelor opt paragrafe din vestita lu ometria de poziie" a lui Criristian von Staudt. ntile elemente din ist t maticilor n antichitate sunt transpuse n limba romna de Iacob Solomon La succesul binemeritat al Recreaiilor stiinif ice a contribuit si prezenta t t excelenta. Scrisa ntr-o limba literara elevata, revista are, cu excepia uno t matematici n formare, ceva din culoarea si prospeimea revistelor actuale t Privita global, ca act de cultura stiinif ica, revista rivalizeaza cu cele t publicaii de acest gen tiparite acum un secol pe plan mondial. t nchei aici relatarea din articolul amintit. Dar ultima fraza trebuie co "acum un secol si un sfert pe plan mondial". Subliniez faptul c n tot cuprinsul celor ase volume ale revistei imp a s grija pentru rigoarea prezentrii, acuratea exprimrii n limba romn, ac a t a a expunerilor, informaia de ultima ora, profunzimea raionamentelor i, nu t t s rnd, atenia acordata contribuiilor personale ale tinerilor rezolvitori sau t t problemelor propuse spre publicare. A fost realizat astfel n premier, o revist romneasc extrem de i a a a a pentru nvaamnt i cercetare n tiinele exacte, de acelai nivel cu revis s s t s t cosacrate i vestite din lume. Dupa apte ani de la stingerea activitaii aces s s t ideea de a rspndi n rndul tineretului pasiunea pentru matematic va a a

de "Gazeta Matematica". La 125 de ani de la apariia revistei "Recreaii tiinifice", generaiile t t s t t omagiaz acest eveniment ca semn de adnc recunotina adus nainta a a s t a s pentru contribuia lor inestimabil la tezaurul tiinei i culturii romnet t a s t s s Si un scurt adaos: Centenarul apariiei revistei "Recreaii tiinifice" a t t s t nizat n 1983 de matematicieni ieeni n Seminarul Matematic "Alexandr s iar srbtorirea celor 125 de ani de la apariie a fost iniiat tot n cad a a t t a Seminar pregatit fiind de Asociaia "Recreaii Matematice", Facultatea a t t matica i Institutul de Matematica "Octav Mayer" de la Filiala din Iai a s s Romne. Asociaia "Recreatii Matematice" i face un titlu de onoare t s itarea integral, exclusiv prin grij proprie, a coleciei revistei "Recreaii a a t t s Acest fapt l datorm prof. univ. Temistocle Brsan de la Universitate a "Gheorghe Asachi" din Iai, cercetatorului dr. Dan Tiba de la Institutul d s atic al Academiei Romne din Bucureti i doamnei Marinela Ghigea a s s firmei Kepler Systmes d Information. i asigurm de toat preuirea i gr a a t s noastr. a

Rolul i ponderea geometriei s n revista Recreaii Stiinifice t t


Prof. dr. Vasile OPROIU

n revista Recreaii tiinif ice, scrisa i editata de un grup de oamen t s t s i cultura inimoi (N. Culianu, C. Climescu, I. Melik, G. I. Lucescu, V s s G. I. Roiu, I. D. Rallet, G. Zarifopol, I. V. Praja, i I. M. Dos s s s-au adunat i publicat diferite materiale din domeniile matematicii, fizic s geografiei, cosmografiei, topografiei, mineralogiei, istoriei matematicii et se adresa elevilor din clasele de gimnaziu i liceu, dar i altor categorii d s s interesate de cunoatere: profesori, studeni, funcionari, militari etc. s t t Geometria, ca ramura a matematicilor are o pondere destul de ns paginile revistei. Trebuie sa menionam, de la nceput, ca G.I. Roiu p t s tre anii 1883-1885 prima carte a Elementelor lui Euclid (traducere dup italian). Am regsit cu o anumit emoie i nostalgie multe formulri a a a t s a ntlnisem cnd eram student i apoi le citisem n crile lui Efimov (edii s at t romna i cea n franceza) i n cartea lui I.Vaisman. Astfel (n volumul II a s s ilor), printre definiiile lui Euclid am regasit formulari precum: Punctul es t nu are pari, adeca nu are nici o marime, Linia este lungime fara largime, t plana este aceea care este aezata egal n respectul tuturor liniilor sale dr s tulatele i axiomele au fost publicate anterior, n primul volum, ntr-o ordi s

de cea cu care suntem obinuii. Astfel, faimosul postulat V al lui Eucli s t axioma XII. Autorul prezint i definiiile i axiomele, aa cum au fost a s t s s de Legendre n Geometria sa, ediia V, Paris, 1804. Legendre nu define t s i, n legatur cu definiia, formuleaz urmtoarea aseriune: Def iniia s a t a a t t este exprimarea raporturilor sale catra lucruri cunoscute. Dupa care, se n definesc dreapta: Linia dreapta este drumul cel mai scurt de la un pun a Mai sunt prezentate comentarii critice ale diverilor matematicieni relativ s definiii, inclusiv noi definiii: Cea mai simpla din toate liniile este linia t t caria noiune este familiara la toi si despre care ni da o idee un f ir n t t sunt prezentate propoziiile de la I la XXII. Cum spuneam, prezentarea E t continu n volumul III cu propoziiile rmase i cu exerciiile la cartea I. a t a s t n revist sunt prezentate numeroase aspecte ale geometriei elemen a elevilor, profesorilor i altor persoane interesate: maxime i minime geometr s s i extrema raie, calculul lungimilor unor linii importante din triunghi, calc s t patrulaterul inscriptibil (n primul volum), proprietai ale poligoanelor i d t s Simson (n vol. II), calcularea volumelor piramidei trunchiate i al conului s proprietai sintetice ale elipsei (n vol. III), teoria transversalelor, diviziu t nic, fasciculul armonic, poli i polare, geometria de poziie a lui Staudt ( a s t aceasta din urma se continua i n volumul V. Chestiunile de geometrie ana s considerate separat i se refer la: seciuni plane n conul drept, construci s a t t cu exemplificari din clasele curbelor celebre, tratarea acestora n coordon plane principale la suprafeele de gradul al doilea. t O seciune important n revist este cea a problemelor propuse (de t a a jur de 10 probleme la fiecare numr), la care se adug, pe parcurs, cea cu a a i listele de rezolvitori. Trebuie sa menionam ca, n Regatul Romniei s t existau cteva zeci de gimnazii i licee (oricum, sub 30) i c numrul s s a a rezolvau probleme era destul de mic. Moda rezolvrilor de probleme la a matematica nu prinsese nca. Menionam ca existau i colaborari venite t s din Transilvania, Banat i alte regiuni ale viitoarei Romnii Mari. s Ca o apreciere cu caracter general, coninutul revistei Recreaii St t t destul de ridicat din punct de vedere al nivelului chestiunilor de geometr Subiectele erau interesante i atractive pentru numeroi cititori. Cred ca, s s revistei, erau persoane care doreau sa faca revista ct mai atractiva. Rsfoind cele ase volume am dat i peste un articol fascinant de la a s s mografie, scris de G.I. Lucescu, n care se explica n ce manier au fost a calendarele iulian i gregorian i ca motivul pentru care s-a facut trecerea s s la altul a fost legat de ideea ca, n acord cu hotarrea Conciliului de la anul 325, punctul de plecare pentru fixarea zilei de Pati trebuia s fie e s a de primvar i acesta trebuia s fie mereu la 21 martie. Dup aceea, s a as a a prima noapte cu luna plina i Patele se fixa n duminica imediat urmato s s n 2008, noaptea cu luna plina cade exact n 21 martie i Patele catolic es s s 23 martie; fixarea Patelui ortodox este mult mai complicat i ine de nit s as t s

calendarul iudaic). De la data Conciliului de la Niceea pna n 1582 calend rmsese n urm cu 10 zile faa de calendarul real (anul din calendarul a a a t puin mai lung dect anul real). Acest lucru influena foarte mult diverse t t practice, de exemplu, unele lucrri agricole ce se fceau n strns legtur a a a a a torile religioase. n 1582, papa Grigore al XIII-lea a dat o bula prin care avansarea calendarului iulian, existent, cu 10 zile i c anii multipli de s a puin cei multipli de 400) nu sunt biseci. Acest calendar mai are o mica e t t const n rmnerea n urm cu o zi n circa 3300 ani, eroare considerat a a a a i care va fi corectata n viitor. n ara la noi, calendarul grigorian a fost s t 1923 (s-a trecut la stilul nou!), dat cnd ntrzierea calendarului iulian a real, sau cel grigorian, ajunsese la 13 zile. Revenind la revista Recreaii tiinifice, apreciez c un cititor interesa t s t a gaseasca n cuprimsul ei lucruri incitante, att n domeniul geometriei, c domenii ale matematicilor i ale altor tiine. s s t

Despre problemele de mecanic a


Prof. dr. Dorin IESAN, m.c. al Academiei

Pe lnga alte chestiuni interesante, n revista Recreaii tiinif ice t s t i un numr de probleme de mecanic raional, semnate de Miltiade T s a a t a vremea cnd a publicat aceste probleme, M. Tzony era profesor de mecanic la Universitatea din Iai (a funcionat n aceasta calitate n perioada 23 s t 15.III.1898). Miltiade Tzony este autorul unui curs de mecanic, prezentat n ma a dou volume. Primul volum a fost scris n 1869, iar al doilea volum n a prima pagina a acestui curs autorul scrie "Curs de Mecanica raionale s u t Profesat la Universitatea de Jasy; Dupa cei mai buni autori francezi: Sturm, Duhamel, Bellanger, Bresse, Bour, Collignon, Mesat, Chasles; d Tzony, Liceniat n stiine matematice de la Sorbona din Paris, Ingineriu t t de poduri din Paris, Vechiu elevu al scoalei politechnice din acestu orau, P s Universitatii de Jasy si a Lyceului Nou din Jasy". Leciile de mecanica ra t la Universitatea din Iai se fceau dup acest curs. s a a In perioada 1885-1888, M. Tzony public Un curs de probleme n revista a tiinifice". El marturisete ca acest lucru l face "n scopul de a uura s t s s Universitatilor noastre completa pricepere a cursului de mecanica raionala t ta ntrebuinare a principiilor acestei nsemnate stiini" (vol. III, pp. 7 t t vedem care este originea problemelor i a rezolvrilor date. M. Tzony ne s a "problemele sunt lucrate dupa diveri autori ntre care gureaza n primul s

Jullien, a carui carte n aceasta materie a devenit clasica". Am constata cele 98 de probleme publicate de Tzony n "Recreaii tiinifice" sunt luate t s t clugrului P.M. Jullien Problmes de mcanique rationnelle, aprut a a a a anul 1855. Cartea lui P.M. Jullien conine att probleme originale ct i pr t s altor autori. In "Recreaii tiinifice" M. Tzony prezinta 36 probleme ale c t s t este P.M. Jullien, 24 de probleme datorate lui W. Walton i 28 probleme al s tori (printre care Euler, Bernoulli, Leibniz, Laplace, Gauss, Mbius). Men problemele datorate lui W. Walton se gsesc n cartea acestuia A Collecti a blems in Illustration of the Principles of Theoretial Mechanics, aparuta la C n anul 1842. La nceputul prezentrii problemelor, M. Tzony afirm c "de cte a a a f i posibil vom indica la f inea f iecarei probleme autorul caruia se datore blemele publicate de Tzony sunt i rezolvate i cu "toate indicaiunile neces s s t a putea f i cuprinse cu uurina de tnarul public cetitoriu caruia este s t destinat". M. Tzony susine, pe drept cuvnt, c "opera abatelui Jullien t a ntr-un mod att de laconic nct cetirea ei de ncepatori este foarte labor unele puncte aproape cu totul neneleasa". Dintre problemele publicate t n "Recreaii tiinifice" un numr de 51 sunt rezolvate n cartea lui P.M t s t a Celelalte sunt probleme pe care Jullien le-a propus spre rezolvare. O pa problemele nerezolvate de Jullien sunt ns nsoite de figuri i rspunsur a t s a lui W. Walton. Am comparat aceste rezolvari i figuri cu cele date de M s "Recreaii tiinifice". Se poate spune cu certitudine c Tzony nu s-a in t s t a cartea lui W. Walton. Fiecare capitol din culegerea de probleme este prefaat cu "o scurta t rezultatelor f inale ale teoriei, ntovarasite de cteva noiuni istorice pe car t altor documente, le vom mprumuta pentru cea mai mare parte din opera de care facem meniune". Este uor de vzut c aceste comentarii sunt t t s a a cartea lui P.M. Jullien. Menionam ca rezolvarile prezentate de M. Tzony sunt clare, iar fig t ngrijite i binevenite. Un lucru remarcabil este faptul c i n cazul p s a s rezolvate de Jullien, M. Tzony face figuri suplimentare i adaug explicai s a t lemele publicate de Tzony n "Recreaii tiinifice" sunt 88 de figuri, dint t s t nu se afla n cartea lui Jullien. Referitor la repartiia pe ani a problemelor se constat c n anul t a a publicate 18 probleme, n anul 1886 apar 33 probleme, n anul 1887 sunt 23 probleme, iar n anul 1888 apar 18 probleme. In anul 1888 revista tiinifice" i nceteaza apariia. Acest lucru a curmat publicarea fireas s t s t probleme. Menionm c problemele aprute se refer doar la partea de Static t a a a a a predat de Tzony la Universitate. Dintre acestea, 18 se refera la echilibrul material, 39 la echilibrul corpului rigid, 6 trateaza echilibrul unui siste articulate, 27 sunt dedicate echilibrului firelor, iar 8 se refer la principiu a

mecanic virtual". Cursul lui M. Tzony (n manuscris) i problemele publicate de el n s tiinifice" au stat la baza nvamntului Mecanicii din ara noastr. s t at t a n afar de activitatea de profesor, Miltiade Tzony s-a remarcat prin a depusa n vederea propairii Romniei. A fost senator, secretar de stat s terul Construciilor Publice, director al C.F.R. Printre altele, oraul Iai i t s s pavarea strzilor. a Despre M. Tzony se pot spune multe lucruri. Un fost elev de-al su, Pe a lianu, care a urmat cursurile de mecanica de la Universitatea din Iai n s l descrie pe Miltiade Tzony astfel: "Cu mintea agera, cu f igura frumoa luat ca model de pictorul Grigorescu pentru unele f iguri din biserica de la m Agapia), ce corespund ntru totul nobleei caracterului sau, el a fost cu to t datoriei si naltei misiuni a profesorului".

Problematica de algebr i analiz matemat as a n revista Recreaii Stiinifice t t


Prof. dr. Teodor PRECUPANU

Pentru matematica romneasca, apariia revistei Recreaii tiinif i t t s t iativa unei elite de profesori ai universitii i ai liceelor din Iai, rep t at s s moment important, de nceput, pentru crearea unei atmosfere propice dezv inelor matematice, de atragere a tinerilor, stimulndu-i i amplificndu-le t s calauzindu-i spre problemele moderne ale acelei perioade. Este de remarcat faptul c iniiatorii revistei erau la curent cu mult a t ocuprile existente n matematica european, avnd o bun informare, fa a a a accesul la o serie de reviste importante, ndeosebi din Frana, Italia i Germ t s semnalate nu numai apariia unor rezultate importante, ci i diverse even t s comunitii tiinifice, cum ar fi, spre exemplu, apariia revistei Acta M at s t t fondat de Mittag-Le ler prin casa regal suedo-norvegian, solemnitatea a a a din nvaamnt la vrsta de 70 de ani a marelui matematician Eugn t Catalan, apariia unor tratate importante de matematica, discuiile ge t t proiectul Turnului Eiel, ce urma s se construiasc n Paris. a a Adresndu-se n primul rnd elevilor din nvamntul secundar, rev at mulat, de asemenea, preocuparile profesorilor pentru modernizarea nva t matematic, gazduind n paginile sale dezbateri interesante cu caracter meto programelor analitice i a metodelor prin care s fie atrai elevii pent s a s matematicii, s se asigure o ct mai bun accesibilitate, punnd pe pr a a intuiia i dezvoltarea abilitailor de rezolvitori de probleme. t s t

Sa remarcam faptul ca n acea perioada, sfritul secolului al XIX-lea, s cipline componente ale matematicii nu erau nc bine conturate, n acord a care ele erau concepute la marile universiti europene, influenate de cr at t a tante de matematica din acei ani. Abia aparuse cursul de analiza matema Sturm (1880) i cel de calcul diferenial i integral al lui Catalan (1878), s t s unei cri despre serii, unic la acel moment, cri ce urmau tratatelor cel at a at de Leibniz, Bernoulli sau Serret. Algebra i analiza matematica este prezenta n paginile revistei att pr s cu caracter teoretic informativ asupra unor chestiuni importante, ct i prin s variat de exerciii i probleme ce vizau i pe studenii anilor pregtitori pen a t s s t a politehnice din ar sau din strintate. Semnalm astfel mai mult articol t a a a a C. Climescu dedicate numerelor complexe (numite cantitai imaginare), nu t nca erau evitate de muli matematicieni. ntlnim, de asemenea, demonstr t mei fundamentale a algebrei precum i unele consideraii asupra dreptelor s t sau asupra unor funcii complexe, date ns sub form implicit prin relaii t a a a t ale. Mai mult, n unele probleme apar integrale ale unor funcii complexe, t a se depai aspectele dificile de multivocitate i determinndu-se cu clarita s s lor principale. In cadrul algebrei sunt cuprinse i seriile numerice, dezvoltrile taylo s a cepute ca sume (numerice sau polinomiale) infinite. Convergena acestor t intuitiv se reduce de fapt la extinderea operaiilor numerice algebrice i t s (nu reiese dac se folosea i ), fiind cunoscute limitele fundamentale. S a s ns pentru convergena criterii fine, aproape toate cele cunoscute astzi. a t a articolelor respective sunt C. Climescu, I.D. Rallet i I.V. Praja. Erau s mod uzual dezvoltarile n serii de puteri ale funciilor elementare. Tot ca fa t din algebr, sunt prezentate cunoscutele formule Lagrange i Newton de i a s ntr-un articol foarte interesant scris de Alex. Sadoveanu. In primul volum, din 1883, problema dezvoltarii funciilor n serie es t facnd parte din Analiza algebrica iar Calculul integral era considerat Analiza matematic. De fapt, analiza matematic era conceput la ace a a a numai ca teorie a derivabilitii avnd ca principale rezultate teorema d at lui Lagrange i condiiile suficiente de extrem la funcii de una sau mai m s t t abile. Problemele de calcul de arii sau volume erau considerate ca facnd geometrie sau intervenind n probleme de mecanic. a Noiunea de derivat era acceptat prin interpretrile ei: geometric - de t a a a a sau cele din mecanica. Mai mult, trebuie avut n vedere ca nsai noiunea t s fundamentala pentru ntreaga matematica, era conceputa n acele timpuri eulerian, ceea ce corespunde astzi funciilor elementare. a a t Menionm c ecuaiile difereniale sunt frecvent ntlnite n partea de m t a a t t de geometrie a curbelor plane (probleme concrete de aflare a unor curbe d anumite proprietai metrice legate de tangente) i nu sunt prezentate ca a s t disciplinei de astzi Ecuaii difereniale, ceea ce este normal, ntruct ace a t t

plina avea sa se contureze n matematica mult mai trziu. Este nsa pre cadrul Analizei matematice problema schimbrii de variabil cu suportu a a ecuaiile difereniale i schimbrile de coordonate (ndeosebi polare i sferic t t s a s de importante n mecanic, geometrie i astronomie. a s Problemele de algebra i analiza matematica prezente n cele ase v s s revistei sunt deosebit de frumoase i ilustrative, de mare diversitate, oferind s exact a coninutului disciplinelor de matematic din acea perioad. Apr a t a a pot fi regsite de fapt n culegerile de probleme de astzi. a a n ncheiere, subliniem nca odata rolul remarcabil avut de revista Re inif ice n dezvoltarea i impulsionarea nvamntului matematic romn t s at cordana cu cel european, ce era urmrit ndeaproape. t a Nu aveau s treac muli ani dup ncetarea brusc a apariiei aceste a a t a a t la universitatea ieeana un fost rezolvitor al Recreaiilor tiinifice elabo s t s t lucrri originale de matematic. Este vorba de marele matematician rom a a Pompeiu, ale crui rezultate privitoare la teorema creterilor finite, obin a s t rioada cnd funciona ca profesor al Universitii din Iai, sunt citate i t at s s presionnd prin profunzimea i elegana lor. Cercetarile sale de analiza m s t sunt de fapt primele cercetri tiinifice originale de matematic la Univer a s t a Iai. s Invocnd anterior numele lui Catalan, matematician cu vaste preocupr a tice (analiza, algebra, geometrie, teoria numerelor), se cuvine a aminti nu ia dintre primii autori romni ai unor cercetri tiinifice originale de m a s t N.St. Botez, care stabilete o frumoas identitate legat de seria armonic, s a a a azi ca identitatea Catalan-Botez, rolul lui Catalan fiind acela de a o menio t din articolele sale cu precizarea lui Botez ca autor. Revista Recreaii stiinif ice constituie pasul premergtor apariiei t t a t stiinif ice ale Universitatii "Al.I. Cuza", n 1900, revist dedicat cercet a a a t nale ale profesorilor universitaii ieene, dar care a publicat nca din primele s t articole ale unor recunoscui matematicieni straini ca Lucien Godeaux, Mau t Kentaro Yano, T.J. Willmore .a. s

Semnalam cititorilor reeditarea coleciei complete a revistei t

RECREA TII STIIN TIFICE (1883-1

la 125 de ani de la apariia primului numar, cu respectarea formei n c t publicata iniial. Revista prezinta i astazi interes prin culoarea limbii t s terminologiei folosite, prin coninutul interesant i de un nalt nivel tiinif t s s t i prin forma grafic frumoas. Cei interesai pot consulta site-ul revistei s a a t de unde se poate prelua gratuit.

http://www.recreatiistiintifice.ro

Despre calendar
Filip REICHER1

Omagiu adus revistei Recreai t la 125 de ani de

Nevoia msurrii timpului. Din cele mai vechi timpuri oamenii sim a a t de a msura trecerea timpului, pentru a ti ct timp a trecut de cnd a s un anumit eveniment. Prima posibilitate de a msura timpul, a fost de a fazele succesive ale Lunii. Astronomii antichitaii au reuit sa determine c s t exactitate timpul dintre fazele de acelai fel ale Lunii (Luna noua, prim s Luna plin, al treilea ptrar). De cea mai mare importana era ns de a a t a determina apariia anotimpurilor succesive. t Calendarul egiptenilor din antichitate. n Egipt se ntmpla un f naturii la nceputul fiecrei veri, care era de cea mai mare importana pe a t cultura: revarsarea uviului Nil pe o parte a pamntului, care facea teren Egiptenii au observat, spre marea lor surprindere, ca trei fenomene ale na tmplau cu regularitate la acelai timp: nceputul verii, revrsarea Nilului s a anumit a stelei Sirius pe cer. (Apariia stelei la orizont, naintea rsrit a t aa lui). Astfel, preoii egipteni au putut sa determine lungimea anului; a fo t un calendar, care cuprindea 365 de zile. S-au stabilit 12 luni cu cte 30 de zile i suplimentar 5 zile, dup cele 360 s a celor 12 luni. Primul calendar egiptean, bazat pe micarea Soarelui pe ce s a poziiilor stelelor, a fost adoptat n anul 4241 .Hr. Mai trziu a fost det t anul cu lungimea de 365, 25 zile, prin adaugarea unei zile odata la 4 ani. apropiat lungimea anului egiptean de lungimea exact a anului astonomic, a determinat mai trziu de astronomii moderni, anume de 365 zile, 5 ore, a i 46 secunde (ca numar zecimal: 365, 2422 zile.) s Calendarul iulian. Acesta este denumit dup mpratul roman C a a Caesar i are 365 de zile ntr-un an ordinar i, o data la 4 ani, 366 zile a s s Abaterea calendarului iulian. n raport cu anul adevarat, anul astronom i anul tropic, care cuprinde 365, 2422 zile, calendarul iulian are o diferena s t care face ca, n conformitate cu acest calendar, anotimpurile s se schimb a dupa o data ntrziata. Pentru a calcula abaterea, luam valoarea medie a anului, conform acestu (4 365 + 1) : 4 = 365, 25. Diferena faa de anul adevrat (astronom t t a 365, 25 365, 2422 = 0, 0078 zile ntr-un an. Dup ci ani acest calendar a t urma cu o zi? Rezultatul se obine din mparirea 1/0, 0078 = 128, 2 ani. t t Deoarece dupa calendarul iulian era pierduta (ntrziata) o zi la fiecar de 128 ani i nceputul primverii 21 martie era indicat mereu mai trz s a o piedic n a stabili din timp srbtorile, mai ales Patele. a a a s Acest lucru s-a remediat prin adoptarea unui calendar nou, calendarul
1

Inginer dr., Facultatea de chimie, Univ. Tehnic "Gh. Asachi", Iai a s

Calendarul gregorian. Acesta este denumit dup papa Grigore a a Deoarece abaterea calendarului iulian era prea mare, s-a adoptat un cale la care abatarea faa de anul astronomic este mult mai mica. t Conform acestui calendar anii de fine ai secolelor, la care numrul nu es a prin 400, nu sunt ani biseci. Deci, anii 1700, 1800, 1900, 2100, 2200 etc t ani biseci, n timp ce anii 1600, 2000, 2400, etc. sunt ani biseci. t t Abaterea calendarului gregorian. n intervalul de 400 de ani, acest ca are 100 de ani biseci o dat la 4 ani ci numai 97 de ani biseci. Valoa t a t a anului este de aceea (303 365 + 97 366) : 400 = 365, 2425 zile. Abate anul astronomic este de 365, 2425 365, 2422 = 0, 0003 zile. n ci ani ram t calendar n urma cu o zi? Raspunsul rezulta din mparirea 1 : 0, 0003 = 3 t Calendarul gregorian este cel mai exact dintre calendare (mai exact de darul iudaic, aceasta o vom vedea mai jos).

Calendarul iudaic. Ca i alte popoare, evreii au ales sa masoare timp s micrii Lunii n jurul Pmntului, anume, potrivit fazelor Lunii. ncepu s a a luni Lunare era determinat prin apariia secerii subiri a Lunii pe cer, ime t t Luna Noua. n fiecare luna se ntrunea "Consiliul Calendarului" i apar s subiri a Lunii pe cer numita "Moled", adica naterea Lunii trebuia sa t s mat de ctre doi martori credibili. "Consiliul Calendarului" se ntrunea d a a zile; dac secera subire a Lunii nu aprea, ziua a 31-a era stabilit drep a t a a a lunii urmatoare. Apoi erau trimii oameni n ara i se aprindeau focuri s t s pentru ca toi oamenii sa ia la cunotina ca a nceput noua luna. t s t ncepnd din sec. IV .Hr., calendarul Lunar a fost nlocuit n mod calendarul Luni-Solar. Deoarece n biblie este prevzut c eliberarea e a a robia egipteana a avut loc primavara, evreii au dorit sa potriveasca astfel c ca sarbatoarea Patelui Pesah , care amintete de acest eveniment, s s s ntotdeauna n primvar. Deci, trebuia construit un calendar care s p a a a anul iudaic cu anul astronomic. O lun astronomic dureaz 29 zile, 12 ore, a a a i 2,8 secunde (sau, ca numar zecimal: 29, 53059 zile). Dar 12 luni Luna s mai puin dect un an astronomic: 12 29, 53059 = 354, 36708 zile, aa t s s fie intercalat o lun suplimentar, din cnd n cnd. a a a a Aceast problem a fost rezolvat n antichitate de astronomul Meton a a a Soluia sa se cheama Ciclul lui Meton, care cuprinde 19 ani, n care, pe ln t pe an, se intercaleaza 7 luni suplimentare. Se are n vedere calculul: durata unui an, conform calendarului cunoscut n acea vreme: 365, 25 19 ani solari = 19 365, 25 zile = 6939, 75 zile; (19 12) + 7 luni = 235 luni Lunare; 235 29, 53059 zile = 6939, 6886 Este o diferena foarte mica, deci acceptabila. n ciclul lui Meton, urma t cuprind a 13-a lun: cele cu numrul de ordine 0, 3, 6, 8 11, 14, 17 din cadr a a Calendarul iudaic a adoptat exact aceast succesiune de ani, cnd se a luna suplimentara. Aceasta luna se cheama ADAR II sau VE-ADAR. Calendarul iudaic are doua particularitai. Lungimea unui an poate a t 353, 354 sau 355 zile ntr-un an obinuit (ordinar) i 383, 384 sau 385 zile s s cu 13 luni (an bisect). A doua particularitate a calendarului iudaic const ca data de 1 Tiri (Anul Nou iudaic) nu poate fi niciodata ntr-o dumin s

miercuri sau ntr-o vineri. Este uor de stabilit cnd avem un an bisect. Se mparte numarul s calendarul iudaic prin 19. Daca restul este unul din numerele de mai su 3, 6, 8, 11, 14 sau 17, acel an este bisect. De exemplu: anul 5744 (19 n caledarul gregorian) este un an bisect, deoarece mprirea prin 19 d at a i restul este 6. Mult mai dificil este de a se stabili lungimea unui an iu s 354 sau 355 de zile ntr-un an ordinar, respectiv 383, 384 sau 385 de zile bisect). Calendarul iudaic are o ntziere de o zi n 216, 4 ani.

Addendum. La catolici i protestani, data Patelui este aceeai. Ea se s t s s dupa o regula simpla: Duminica Patelui este prima duminica de dupa p s plina ce apare dupa echinoxiul de primavara. 2005 2006 2007 2008 Luna plina 05.03 13.04 02.04 21.03 Data Patelui s 27.03 16.04 08.04 23.03 2009 2010 2011 2012 Luna plina 09.04 30.03 18.04 06.04

Data 1 0 2 0

Data Patelui pentru ortodoci se stabilete dup un algoritm mult mai s s s a Interesant este faptul c acest algoritm se bazeaz tot pe ciclul de 19 ani, c a a calendarul iudaic. De aceea, nu ntmpltor, Duminica Patelui pentru orto a s ntotdeauna n ultima zi, puin naintea ultimei zile sau imediat dupa ult t cele 8 zile ale srbtorii Patelui (PESAH) din calendarul iudaic. a a s Ultima zi de PESAH dum. 01.05 joi 20.04 mari 10.04 t dum. 27.04 Patele s ortodox 01.05 23.04 08.04 27.04 Ultima zi de PESAH joi 16.04 mari 06.04 t mari 26.04 t smb. 14.04

2005 2006 2007 2008 1. 2. 3. 4.

2009 2010 2011 2012

Bibliografie

G. Petrescu - Astronomie elementara, Bucureti, 1962 s G. Stnil - Sisteme calendaristice, Bucureti, 1980 a a s www.jewfaq.org: Judaism 101 LUAH 5743 (Calendar 1982 -1983), editat de Federaia Comunitailor ev t t Romnia

Nota Redaciei. n revista Recreaii Stiinice probleme ca: msura t t t a lui, alcatuirea unui calendar, stabilirea datei Patelui au fost ndelung de s aprofundate. ntr-un ciclu de noua articole (aparute n vol. I(1883), vol. vol. III(1885)), G. I. Lucescu, profesor la Liceul Naional din Iai, face u t s calendarului, din cele mai vechi timpuri i pn la adoptarea calendarului s a n cuprinsul a cinci scrisori publicate n vol. VI(1888), Constantin Go sor la Universitatea din Bucureti, se ocupa de regulile pentru gasirea zile s Paul Tanco din Nsud, primul romn cu titlul de doctor n matema aa problema periodicitii cu care Patele cade a doua zi dup Sf. Gheorghe at s a

Cteva probleme de teoria numerelor a cr a rezolvare se bazeaz pe identiti a at


Marian TETIVA1

Cnd vorbim despre utilizarea identitilor n teoria numerelor, prob at gndim n primul rnd la ecuaii diofantice, n special la demonstrarea t soluiilor unor asemenea ecuaii. De exemplu, identitaile t t t i s (m2 n2 )2 + (2mn)2 = (m2 + n2 )2

(m2 2mn n2 )2 + (m2 + 2mn n2 )2 = 2(m2 + n2 )2 arata ca ecuaiile x2 + y 2 = z 2 i, respectiv, x2 + y 2 = 2z 2 au, fiecare, o t s de soluii ntregi. Totui, n cele ce urmeaza, vom rezolva alte tipuri de pr t s teoria numerelor cu ajutorul identitilor. at Avem n minte mai ales dou identiti, binecunoscute cititorilor. Este a at (x y)(x + y)(x2 + y 2 ) (x2
n1

+ y2

n1

i de s

) = x2 y 2
n n1

(x2 + xy + y 2 )(x2 xy + y 2 )(x4 x2 y 2 + y 4 ) (x2 x2 =x


2n+1

y2

n1

+x y

2n 2n

+y

2n+1

Ambele sunt valabile pentru orice numere complexe x i y (dar, desigur ca s vor interesa pentru numere ntregi) i orice numr natural n 1. Se just s a prin aplicarea repetat a formulei (a b)(a + b) = a2 b2 . a

Problema cea mai cunoscuta care utilizeaza (1) este probabil Problema 1. Fie n 1 un numar natural. Sa se arate ca numarul N scris n baza 10 cu 2n cifre de 1 are cel puin n divizori primi distinci. t t Soluie. ntr-adevr, avem t a N = (102 1)/9 = (10 + 1)(102 + 1) (102
n n1

+ 1)

deci (1) ne permite s scriem pe N ca produsul a n numere; dac reuim a a s c acestea sunt prime ntre ele dou cte dou problema ar fi rezolvat a a a a fiecare din aceste n numere ar aduce (cel puin) un factor prim n descomp t N ca produs de numere prime. i j Avem, pentru 0 i < j, c 102 + 1 divide pe 102 1 (tot pe baza a i j (1)), deci daca d este un divizor comun pentru 102 + 1 i 102 + 1, atu s j j divizor i al lui 2 = 102 + 1 (102 1). Cum d este impar, rezulta d = 1 s se ncheie.

Problema care urmeaz a aprut acum ceva vreme n Recreaii matem a a t 1/2005, pag. 42, Problema 2, cl. a X-a, Lucian Tuescu. t Problema 2. Exista o innitate de numere n N astfel nct n2+n+1 di Soluie. Sa pornim de la un caz particular al identitaii (2): t t 8 4 2 2 m + m + 1 = (m + m + 1)(m m + 1)(m4 m2 + 1)
1

Profesor, Colegiul Na ional "Gheorghe Roca Codreanu", Brlad t s

i s observm c avem s a a a m2 m + 1 < m2 + m + 1 < m4 m2 + 1 < m4 pentru orice numar natural m 2. Aceste inegalitai arata ca (m4 )! se t (m2 + m + 1)(m2 m + 1)(m4 m2 + 1), deci cu (m4 )2 + m4 + 1 pentru o numr natural. Problema este aadar rezolvat: putem alege n = m4 , m a s a exemplu, 16! se divide cu 162 + 16 + 1 = 7 3 13; de fapt, n = 16 este c cu proprietatea din enun.) t

O problem asemntoare a aprut mai demult n American Mathematic a a a a Problema 3. Exista o inf initate de numere n N astfel nct n2+1 di Soluie. De data asta ne vom folosi de identitatea t 4m4 + 1 = (2m2 2m + 1)(2m2 + 2m + 1), unde vom ncerca s mai descompunem i cel de-al doilea factor. Pent a s observm c a a 4m2 + 4m + 2 = (2m + 1)2 + 1 = 2p2 2m2 + 2m + 1 = p2 , daca 2m + 1 i p sunt soluii ale ecuaiei x2 2y 2 = 1. s t t Dar acest lucru este cunoscut: ecuaia menionat ntr-adevr, o t t a are, a de soluii. Mai precis, dac notm xk + yk 2 = (1 + 2)2k+1 , cu xk i t a a s naturale, atunci avem i xk yk 2 = (1 2)2k+1 , deci s 2 x2 2yk = (xk + yk 2)(xk yk 2) = ((1 + 2)(1 2))2k+1 = (1)2k k pentru orice k; de exemplu, primele trei soluii (care corespund lui k = t respectiv k = 2) sunt (1, 1), (7, 5) i (41, 29). n plus, se arata destul de s este impar pentru orice k. Atunci, este suficient s alegem perechea (2m + 1, p) ca fiind una dint a (xk , yk ) ale ecuaiei x2 2y 2 = 1, pentru a avea t (2m2 )2 + 1 = (2m2 2m + 1)p2 . n aceste condiii, t 4m4 + 1 = (2m2 2m + 1)p2 > 17p2 16p2 + 1, dac m este suficient de mare (ceea ce se poate, deoarece irul (xk ) ti a s de fapt, pentru k 2, avem m = (xk 1)/2 20, ceea ce asigura va inegalitaii care ne trebuie), de unde obinem t t m2 > 2p 2m2 2m + 1 m2 > 2p. Astfel c 2m2 > 2m2 2m + 1 > 2p > p, deci produsul (2m2 )! coni a t (distinci) 2m2 2m + 1, 2p i p, deci se divide cu (2m2 2m + 1)p2 = (2 t s prin urmare sunt soluii toate numerele n = 2m2 , unde m = (xk 1)/2, k t Se poate vedea prin calcul direct c soluia cea mai mic este n = 18 (18 a t a cu 182 + 1 = 52 13). Aceast soluie face parte din irul de mai sus; se ob a t s t k = 1, cnd m = 3 i p = 5 (inegalitatea 2m2 2m + 1 > 2p are loc i n s s chiar daca nu are loc m2 > 2p).

Problema 4. Pentru un numar natural n 2 notam cu h(n) cel divizor prim al lui n. Sa se arate ca exista o inf initate de numere n a h(n) < h(n + 1) < h(n + 2).

Soluie. De ast dat, pe lng identitatea (2), vom folosi i o idee t a a a s subtila. Anume, sa fixam un numar prim impar p i sa observam ca, l s Problema 1, oricare doua dintre numerele p + 1, p2 + 1, . . . , p2 + 1, . . .
k

au cel mai mare divizor comun 2. De aceea exist unul dintre ele care a un factor prim mai mare dect p. S considerm primul dintre aceste num a a k j fie k acel numr natural pentru care h(p2 + 1) > p i h(p2 + 1) < p p a s s 1, 2, . . . , k 1 (clar, nu putem avea h(p2 + 1) = p, deoarece nici unul din k numere nu poate avea factorul p). Numarul n = p2 1 are atunci propri k enun. ntr-adevar, h(n + 1) < h(n + 2) nseamna p < h(p2 + 1), deci r t alegerea lui k; i tot din alegerea lui k rezult i cealalt inegalitate, deoa s as a h(p2 1) = h((p 1)(p + 1)(p2 + 1) (p2
2
k k1

+ 1)) =
k1

Astfel vedem ca pentru fiecare numar prim impar p exista un numar nt k k astfel nct n = p2 1 sa aiba proprietatea h(n) < h(n + 1) < h(n + 2

= max{(h(p 1), h(p + 1), h(p + 1), . . . , h(p2

+ 1)} <

Exerciii pentru cititor. t Problema 5. Fie a, b, c numere ntregi astfel nct ab nu este ptrat a a, b sunt pozitive. Artai c dac ecuaia ax2 by 2 = c are o soluie n a t a a t t numerelor ntregi, atunci ea are o infinitate de asemenea soluii. t Problema 6. Sa se dea o alta soluie Problemei 3 folosind identitatea t Verificai aceasta identitate! t De asemenea, putei obine o soluie a problemei folosind identitatea t t t (x2 + 1)((x + 1)2 + 1) = (x2 + x + 1)2 + 1. (4x2 2x + 1)2 + 1 = 2(4x2 + 1)(2x2 2x + 1).

Problema 7. Fie p > 0 un numr prim. S se arate c din oricare 2p a a a ntregi x1 , . . . , x2p1 se pot alege p a caror suma se divide cu p. Indicaie. Aceasta nu e o problema uoara. De fapt ea este valabila pe t s numr natural n (adic din oricare 2n 1 numere ntregi x1 , . . . , x2n1 se a a n a cror sum se divide cu n) i n aceast form se numete teorem a a s a a s Ginzburg-Ziv (a fost pentru prima data demonstrata de cei trei matem 1961). Se poate arata ca acest enun are proprietatea de multiplicativitate t c dac este adevrat pentru n = a i n = b, atunci este adevrat i pent a a a s a s (a se vedea cartea lui Horea Banea de Probleme traduse din revista Kv urmare demonstrarea sa pentru n numar prim i asigura valabilitatea pent Si ajungem acum i la indicaia promisa: utilizai identitatea s t t X X p X (x1 + + xp )p1 x1 (x1 + + xp1 )p1 + + (1)p1

pentru care se poate consulta, de exemplu, Ioan Tomescu, Probleme natorica si teoria grafurilor, E. D. P., Bucureti, 1981. Sumele se fac d s posibilitaile de a alege din cele 2p 1 numere cte p, p 1, . . . , respectiv t

O caracterizare a punctului Mathot


Ctlin TIGAERU 1 aa

Punctul lui Mathot (sau anticentrul) unui patrulater inscriptibil este punctul de intersecie al perpendicularelor duse din mijlocul fiecrei laturi t a laterului pe latura opusa. Numeroase proprietai ale acestui punct au fo t evidena, cele mai spectaculoase ind n legtur cu cele patru triunghiu t a a de cte dou laturi adiacente ale patrulaterului i cte o diagonal. a s a n acest not punem n evidena o caracterizare a punctului Mathot ca a a t la cele patru triunghiuri formate de cte o latura i cte doua segmente de s pe diagonale de punctul lor de intersecie. Mai precis, demonstrm t a Teorema 1. Se considera patrulaterul inscriptibil ABCD, nscris n centru O, n care se noteaza cu E intersecia diagonalelor AC si BD si c t H3 , H4 ortocentrele triunghiurilor AEB, BEC, CED si respectiv DEA. (a) Patrulaterul H1 H2 H3 H4 este paralelogram. (b) Intersecia diagonalelor paralelogramului H1 H2 H3 H4 coincide cu pu t thot al patrulaterului ABCD. Pentru demonstraie folosim t Lema 1. Daca O1 , O2 , O3 , O4 sunt centrele cercurilor circumscrise urilor AEB, BEC, CED si respectiv DEA, atunci (a) Patrulaterul O1 O2 O3 O4 este paralelogram. (b) Daca este punctul de intersecie a diagonalelor paralelogramului O t atunci punctele O, , E sunt coliniare, punctul fiind mijlocul segmentu Demonstraie. Cititorul poate verifica imediat fapt tul c patrulaterul O1 O2 O3 O4 este paralelogram. Notm a a cu F piciorul perpendicularei din E pe AB i unim E cu s [ \ O3 . Pe de o parte avem m(F EA) = 90 m(BAE); pe de \ \ \ alt parte m(CEO3 ) = 90 m(CDE); cum m(BAE) = a \ rezult c m(F EA) = m(CEO3 ), adic F , [ \ m(CDE), a a a E, O3 sunt coliniare, deci EO3 AB, deci O3 E k OO1 ; analog se demonstreaza ca O1 E k OO3 , O4 E k OO2 , O2 E k OO4 , de unde rezulta ca patrulaterele O1 EO3 O, O2 EO4 O sunt paralelograme. Cum diagonalele paralelogramelor se njumtesc, rezult c punctul este mijlocul segmentulu a at a a Demonstraia Teoremei 1. Vom folosi i urmatoarele rezultate: t s (A) Dac U V W Z este un paralelogram, S este intersecia diagonalelor a t este un punct oarecare din plan, atunci 4M S = M U + M V + M W + M Z (B) (Sylvester) Daca M este centrul cercului circumscris triunghiulu dac S este ortocentrul triunghiului, atunci M S = M U + M V + M W ; a (C) Daca ABCD este inscriptibil, daca O este centrul cercului circumsc este punctul Mathot al patrulaterului, atunci OA + OB + OC + OD = 2O .
1

Lect. dr., Univ. " tefan cel Mare", Suceava S

Se demonstreaz imediat c H1 H2 H3 H4 este paralelogram. Putem sc a a OO4 + O4 A = OO1 + O1 A, OB = OO1 + O1 B = OO2 + O2 B, OC = OO2 OO3 + O3 C, OD = OO3 + O3 D = OO4 + O4 D, de unde, prin nsumare, 2 OA + OB + OC + OD = 2 OO1 + OO2 + OO3 + OO4 +

+ O1 A + O1 B + O1 E + O2 B + O2 C + O2 E + O3 C + O3 D + O (B) + O4 D + O4 A + O4 E + EO1 + EO2 + EO3 + EO4 =


i=1 i=1 i=1 i=1 i=1 i=1

4 4 4 4 4 4 X X X X X X = 2 OOi + Oi Hi + EOi = OOi + Oi Hi + OOi + EOi 4 X = (A) + lema = OHi + 4O + 4E. i=1

Conform lemei, rezult c O + E = Dac a a 0. a notam cu 0 punctul de intersecie a diagonalelor parat lelogramului H1 H2 H3 H4 i, innd cont din nou de (A), s t n P 0 OHi = 4O . Ca urmare, obinem t avem
i=1

Din (1) i (2) obinem ca 4O = 4O0 , de unde deducem s t c 0 i teorema este demonstrat. (Cele spuse se a s a pot urmri pe figura alturat.) a a a O consecina imediat a teoremei este i relaia vectorial t a s t a
4 X Oi Hi = 4, i=1

2 OA + OB + OC + OD = 4O0 .

(2)

care se deduce imediat folosind (A).

Este posibil ca rezultatul notei sa nu fie nou, dar sigur nu este trecut, d printre proprietaile punctului Mathot, demonstrate n capitolul consacrat t lui, din monograa "Problems in plane and solid geometry", scris de V a solov, care este accesibil pe Internet. Precizm c autorul nu a gsit rezu a a a a n carile citate n bibliograe, nici n alte cari clasice de geometrie, scris t t romna. Bibliografie

1. D. Mihalcea, I. Chiescu, M. Chiria - Geometria patrulaterului, E t t Seria Bacalaureat-Admitere, nr. 24, 1998. 2. C. Mihalescu - Geometria elementelor remarcabile, Bibl. Soc. St. Ma S.S.M.R., Ed. Tehnica, Bucureti, 2007. s

Unsprezece ptrate perfecte a


Dan POPESCU 1

ptrate perfecte. a Se va vedea ca rezultatul obinut are drept consecine un numar mare de t t publicate n reviste de specialitate destinate elevilor (de gimnaziu). Din modul cum s-a formulat problema, rezult c ptratele perfecte a a a gsesc printre cele ce corespund unei valori particulare a lui n. Pentru a gasesc, cu ajutorul calculatorului urmatoarele 11 patrate perfecte de form 1) 2) 3) 4) 5) 6) 7) 8) 9) 10) 11) 1111022224 = 333322 ; 1111088889 = 333332 ; 1111155556 = 333342 ; 1111222225 = 333352 ; 4444222225 = 666652 ; 4444355556 = 666662 ; 4444488889 = 666672 ; 4444622224 = 666682 ; 9999400009 = 999972 ; 9999600004 = 999982 ; 9999800001 = 999992 ; a = 1, b = 0, c = 2, d = 4, a = 1, b = 0, c = 8, d = 9, a = 1, b = 1, c = 5, d = 6, a = 1, b = 2, c = 2, d = 5, a = 4, b = 2, c = 2, d = 5, a = 4, b = 3, c = 5, d = 6, a = 4, b = 4, c = 8, d = 9, a = 4, b = 6, c = 2, d = 4, a = 9, b = 4, c = 0, d = 9, a = 9, b = 6, c = 0, d = 4, a = 9, b = 8, c = 0, d = 1.

Scopul acestei note este determinarea numerelor naturale n baz forma aa {z . a b | {z . } d, n N , care, pentru orice numr natura a | . . } cc . . c
n cifre n cifre

perfecte pentru orice n N , anume, acelea ce se scriu cu sistemele de cif d) pe care le-am ntlnit mai sus (n cazul n = 4), adic a (1) (2) (3) (4) (5) (6) 11 {z . 1 0 22 {z . 2 4 = 33 {z . 3 2 , | .. } | .. } | .. }
n n n 2

Vom arata ca exista exact 11 numere de forma aa{z . a b | {z . } d care su | . . } cc . . c


n n

(7) (8) (9) (10) (11)

11 {z . 1 0 88 {z . 8 9 = 33 {z . 3 , | .. } | .. } | .. }
n n n+1

11 {z . 1 55 {z . 5 6 = 33 {z . 3 4 , | .. }| .. } | .. }
n+1 n n

44 {z . 4 6 22 {z . 2 4 = 66 { | .. } | .. } | .
n n n n

44 {z . 4 88 {z . 8 9 = 66 {z | .. }| .. } | ..
n+1 n

11 {z . 1 22 {z . 2 5 = 33 {z . 3 5 , | .. }| .. } | .. }
n n+1 n

99 {z . 9 4 00 {z . 0 9 = 99 { | .. } | .. } | .
n n

44 {z . 4 22 {z . 2 5 = 66 {z . 6 5 , | .. }| .. } | .. }
n n+1 n

99 {z . 9 6 00 {z . 0 4 = 99 { | .. } | .. } | .
n n

44 {z . 4 3 55 {z . 5 6 = 66 {z . 6 , | .. } | .. } | .. }
n n n+1

99 {z . 9 8 00 {z . 0 1 = 99 | .. } | .. } |

Profesor, Colegiul Na ional " tefan cel Mare", Suceava t S

xn = aa {z . a 10n+1 a 10n+1 + b 10n+1 + cc {z . } + d c = | .. } | .. c


n+1

n scopul propus, s notm xn = aa {z . a b | {z . } d, n N . Putem scr a a | . . } cc . . c


n n

adica

= aa {z . a 10 | .. }
n+1

n+1

1 + (b a) 10n+1 1 + aa {z . a + cc {z . } + d | .. } | .. c
n+1 n+1 2 n+1

n+1

Acum, s observm c n () coecienii 9a, 9b + c 8a i b + d a c s a a a t s pentru orice n N (ei reectnd numai forma lui xn ). Ca urmare dac din membrul doi este patrat perfect pentru o valoare particulara a lui n, avea aceast proprietate pentru orice n N . Acest fapt vericndu-se dir a n = 4, vom deduce c numerele (1) (11) sunt cele cutate. a a

xn = 9a11 {z . 1 + (9b + c 8a) 11 {z . 1 + b + d a c, | .. } | .. }


n+1

nN

pentru n = 2: 3442 = 118336. Mai precis, pentru n = 2, exista 18 patra de forma enunat, iar pentru n = 3, numrul lor este 12. n cazul n = 1 t a a lor este mult mai mare, cci problema se reduce la identicarea ptratelo a a cu patru cifre ale sistemului zecimal. ii) Elevul Aursulesei Tudor, cruia i mulumim i cu acest prilej, a ver a t s intermediul calculatorului faptul c, pentru 4 n 14, singurele ptrat a a de forma aa{z . a b | {z . } d sunt exact cele unsprezece prezentate mai sus. . . } cc . . c |
n n n+1 n

Observaia 1. i) Cititorul poate observa c, pentru 1 n 3, exis t a perfecte de forma aa{z . a b | {z . } d care nu apar printre cele unsprezece. U . . } cc . . c |
n n

sunt: 11 {z . 1 55 {z . 5 6, n N i 44 {z . 4 88 {z . 8 9, n N . s | .. }| .. } | .. }| .. }
n+1 n n+1 n

Observaia 2. i) Singurele ptrate perfecte de forma aa {z . a bb {z . } c t a | .. }| .. b

n N .

juniori, Atena, 1998) [1].

Aplicaii t 1. Sa se arate ca numarul N = 11 {z . 1 22 {z . 2 5 este patrat perfec | .. }| .. }


1997 1998

ii) Singurul patrat perfect de forma aa{z . a bb {z . } c, n N este 44 {z . | .. }| .. b | ..


n n+1 n

Este un caz particular al rezultatului (4); N = 33 {z . 3 5 . | .. }


1997 n

perfect, pentru orice n N . Rezolvarea problemei decurge din (8) i (10), deci avem (x, y) {(4, 2) s

2. Sa se determine cifrele x si y, x 6= 0, daca xx{z . x 6 yy . . . y 4 e | .. } | {z }


n

Observaia 3. i) n lista scurta cu problemele propuse la Olimpiada t de Matematica, ediia 2005 [6], E. Velcea a propus problema care fac t rezultatului de la (6). ii) Rezultatul de la (4) a constituit o problem de la Concursul Int a "Gh.Tieica", ediia 2004. t t iii) Autorul acestei note nu a identicat enunuri legate de rezultatele t (9) i (11). s n nal, propunem urmatorul exerciiu (poate cu o alta abordare): t

aplic (6). a 6. Sa se arate ca exista o innitate de numere cu terminaia 0004 t patrate perfecte. Se poate utiliza egalitatea (10). 7. Este numarul a = | 4{z . 43 55 {z . 5 6 natural? (P. Btrneu - O 4 .. } | .. } a t
2007 2007

patrate perfecte, n N, n 2. a Se arat c a = 11 {z . 1 0 88 {z . 8 9 i se aplic (2), iar b = 44 {z . 4 3 55 { a a | .. } | .. } s | .. } | .


n1 n1 n1

Rezult din cele prezentate mai sus; o alt abordare poate gsit n a a a a 4. Rezultatele de la (3) i (7) sunt prezente n [3]. s 5. Sa se arate ca numerele a = 11 {z . 1 22 {z . 2 si b = 44 {z . 4 88 | .. } | .. } | .. } |
2n n 2n

3. Nu exista patrate perfecte n baza zece de forma aa{z . a; a 6= 0, n | .. }


n

scurta), Piteti, ediia 2007 [6]). s t

care, pentru care numar natural nenul n, sa e patrate perfecte. Bibliografie

Sa se arate ca nu exista numere n baza zece cu scrierea poziionala aa {z t | ..

1. D. Brnzei .a. - 10 ani de Olimpiade Balcanice ale Juniorilor, Paralela s 2. N.B. Vasiliev, A.A. Egorov - Zadaci vsesoiuzni matematiceskih olimp Nauka, Moscova, 1988. 3. A.P. Ghioca, L.A. Cojocaru - Matematica gimnazala dincolo de ma Zalu, 2005. a 4. I. Cucurezeanu - Patrate si cuburi perfecte de numere ntregi, Gil, Zalu a 5. Gazeta Matematica, Seria B, nr. 12/2005, Problema E:13095. 6. Romanian Mathematical Competitions, Theta, Bucureti, 2005. s 7. Romanian Mathematical Competitions, Theta, Bucureti, 2007. s

Cercuri seminscrise i puncte de tip Gergonne sau Nagel s


Temistocle BRSAN 1

Fie ABC un triunghi oarecare. Pentru cercurile circumscris, nscris, A etc. folosim notaiile uzuale: C (0, R), C (I, r), C (Ia , ra ) etc. Punctele de t a dreptei BC cu cercurile C (I, r) i C (Ia , ra ) se noteaz D i D0 ; cu E, E s a s notm punctele cu semnificaii similare relativ la dreptele CA i, respecti a t s Este cunoscut faptul ca dreptele AD, BE si CF sunt concurente (ntr punctul lui Gergonne) i, de asemenea, faptul c dreptele AD0 , BE 0 si s a concurente (ntr-un punct N punctul lui Nagel ). Se asociaz triunghiului ABC trei cercuri seminscrise: C (J1 , 1 ), a C (J3 , 3 ) (C (J1 , 1 ) ind cercul tangent dreptelor AB i AC i tangent in s s cului circumscris triunghiului etc.), precum i trei cercuri ex-seminscrise: s C (Jb , b ), C (Jc , c ) (C (Ja , a ) ind cercul tangent dreptelor AB i AC s exterior cercului C (O, R) etc.). Observm c avem un singur cerc nscri a a cercuri seminscrise; pe de alta parte, numarul cercurilor exnscrise este eg celor ex-seminscrise. Privitor la cercurile seminscrise, un numr de prop a lor sunt date n [3] i [1]. s Ne propunem n aceast Not s a a a "trecem" cele doua rezultate mai sus menionate la cercurile seminscrise t i ex-seminscrise. s n scopul propus, s notm D1 i a a s t Da punctele de tangena a cercurilor C (J1 , 1 ) i, respectiv, C (Ja , a ) cu s C (O, R); E1 , Eb i F1 , Fc au sems nicaii analoage. t Odata cu trecerea de la cercul C (I, r) la cele trei cercuri seminscrise C (Ji , i ) (i = 1, 2, 3), este firesc sa consideram n rolul ce vienelor Gergonne AD, BE i CF s cevienele AD1 , BE1 i, respectiv, s CF1 . Similar, n locul cevienelor Nagel AD0 , BE 0 i CF 0 sa consides ram cevienele ADa , BEb i, ress pectiv, CFc legate de cercurile exseminscrise C (Ja , a ) etc.

Vom arata ca rezultatelor clasice de mai sus le corespund cele din urm

Teorem. a) Cevienele AD1 , BE1 si CF1 sunt concurente n cen a omotetiei directe a cercurilor C (O, R) si C (I, r).
1

Prof. dr., Universitatea Tehnic "Gh. Asachi", Iai a s

b) Cevienele ADa , BEb si CFc sunt concurente n centrul S 0 al omotet a cercurilor C (O, R) si C (I, r). k Demonstraie. a) Evident, omotetia HA , cu k = 1 , transforma cer t r R 0 k a n C (J1 , 1 ), pe cnd omotetia HD1 , cu k 0 = , transform C (J1 , 1 ) n C ( 1 R k0 k urmare, produsul HD1 HA are centrul pe AD1 i raportul kk 0 = 1 = s r 1 transforma C (I, r) n C (O, R), acest produs coincide cu omotetia directa cercuri. n consecina, AD1 trece prin S centrul omotetiei directe a t R C (I, r) i C (O, R) (situat pe OI i definit de relaia SO = SI). Simila s s t r s c dreptele BE1 i CF1 trec prin S. a t b) Se procedeaz la fel. HA , cu t = a , transform C (I, r) n C (J a a r R 0 t HDa , cu t0 = , transforma C (Ja , a ) n C (O, R). Omotetia produs H a R cu raportul tt0 = , coincide cu omotetia inversa a cercurilor C (I, r) s r 0 t a s ADa conine centrul S al acestei din urm omotetii (situat pe OI i dete R 0 0 S 0 O = S I). Se arat similar c i BEb , CFc trec prin S . Q.e.d. a as r Observaia 1. Demonstraia standard pentru concurena cevienelor t t t (sau Nagel) se bazeaza pe reciproca teoremei lui Ceva. Acest instrumen utilizat i pentru stabilirea afirmaiilor a) i b), dar cu preul unor calcule s t s t BX c2 p b Astfel, dac notm X = BC AD1 , se gsete c a a a s a = 2 (2p = XC b pc Aceasta relaie i cu analoagele ei fac posibila aplicarea reciprocei teorem t s i, deci, dovedirea concurenei dreptelor AD1 , BE1 , CF1 . Faptul ca S es s t lor de concurena devine o chestiune de rutin, care cere noi calcule; de e t a poate utiliza Propoziia 2 din [2] i lista de coordonate triliniare din [4]. n t s este preferabila demonstraia data pe baza produsului a doua omotetii. t

Observaia 2. n [3], sub form de problem propus cititorilor spre t a a a este afirmata concurena dreptelor AD1 , BE1 , CF1 (cu alte notaii), fara t t vreo precizare asupra punctului lor de concurena. t

Observaia 3. n [5], ntr-o interesanta Nota de geometria triunghiulu t de omotetie S i S 0 apar ca puncte de concurena ale altor dou triplete s t a asociate unui triunghi dat. Bibliografie 1. 2. 3. 4.

R. Bairac - Cercuri seminscrise n triunghi, Delta, 1/2006, 12-15. T. Brsan - Ceviene izogonale si puncte de concurena remarcabile, 9/2002 t A. Girici - Cteva probleme despre triunghiuri si cercuri, Kvant, 11/1990 C. Kimberling - Centrul Points and Central Lines in the Plane of a Mathematics Magazine, 67(1994), no.3, 163-187. 5. I. V. Maftei - Doua puncte remarcabile ntr-un triunghi, G.M. (B) 1/2

O rafinare a inegalitii lui Jensen at


Florin POPOVICI 1

Cu o demonstraie simpla, stabilim un criteriu de monotonie a funciilo t t caie, prezentm o ranare a inegalitii lui Jensen, despre care credem c t a at a

Propoziia 2 (de tip Rolle). Daca f : [a, b] R este o funcie continu t t care are derivata la dreapta n R pe [a, b) si f (a) = f (b) , atunci exista c1 , c2 [a, b), astfel nct 0 0 f+ (c1 ) 0 f+ (c2 ) .

Propoziia 1 (de tip Fermat). Fie f : [a, b] R o funcie data. Daca t t este un punct de maxim local (respectiv de minim local) al funciei f si f a t 0 0 la dreapta n R n punctul x0 , atunci f+ (x0 ) 0 (respectiv f+ (x0 ) 0).

1. Preliminarii. Fie a, b R, cu a < b. Se stabilete n mod obinuit s s

Demonstraie. Presupunem ca f nu-i constanta (cazul contrar fii t Conform teoremei de mrginire a funciilor continue a lui Wieirstrass, exis a t [a, b], nct f (c1 ) = max {f (x) | x [a, b]} i f (c2 ) = min {f (x) | x [a, b s rezulta ca putem alege c1 , c2 [a, b). Conform Propoziiei 1, rezulta ca ar t

Propoziia 3 (de tip Lagrange). Daca f : [a, b] R este o funcie c t t [a, b], care are derivata la dreapta n R pe [a, b), atunci exista c1 , c2 [a, f (b) f (a) 0 0 f+ (c2 ) . f+ (c1 ) ba Demonstraie. Aplicam Propoziia 2 funciei g : [a, b] R, definita t t t f (b) f (a) x, x [a, b] . g (x) = f (x) ba Propoziia 4 (criteriu de monotonie). Daca f : [a, b] R este o funci t t pe (a, b], care are derivata la dreapta n R pe [a, b) si
0 f+ (x) 0,

atunci funcia f este crescatoare. t Demonstraie. Fie x1 , x2 (a, b], cu x1 < x2 . Conform Propoziiei 3 t t f (x2 ) f (x1 0 , exist c1 [x1 , x2 ), astfel nct f+ (c1 ) a restriciei f t x2 x1 (a,b] a este cresctoare. Urmeaz c lim f a a a 0 f (x2 ) f (x1 ), adic f
(a,b] x&a

x [a, b),

0 R {}. Conform ipotezei (3), avem f+ (a) 0. Rezult c f (a) l. a a c avem f (a) f (x), x (a, b], deci funcia f este cresctoare. a t a

2. Rezultatul principal. Putem acum stabili urmtoarea a

Teorem. Fie I R un interval dat. Daca f : I R este o funci a t atunci pentru orice a1 , . . . , an I, cu a1 a2 an si pentru orice p1
1

Prof. dr., Colegiul Na ional "Gr. Moisil", Braov t s

(0, ), are loc inegalitatea lui Jensen rafinata: p1 a1 + . . . pn an p1 f (a1 ) + + pn f (an ) f p1 + + pn p1 + + pn (p1 + p2 ) f (a2 ) + + pn f (an ) (p1 + p2 ) a2 + + pn an f p1 + + pn p1 + + pn (p1 + + pn1 ) f (an1 ) + pn f (an ) (p1 + + pn1 ) an1 + pn f p1 + + pn p1 + + pn Demonstraie. Stabilim prima inegalitate din (4). Dac a1 = a2 , atu t a inegalitate din (4) are loc cu egalitate. Dac a2 = an , atunci prima ineg a (4) rezulta direct din definiia convexitaii. Consideram cazul a1 < a2 t t g : I R funcia definita prin t p1 x + p2 a2 + + pn an p1 f (x) + p2 f (a2 ) + + pn f (an ) f g (x) = p1 + + pn p1 + + pn Deoarece funcia f este convexa rezulta (a se vedea [1], 1.3) ca funcia f t t a a tinu pe (a1 , a2 ], este derivabil la dreapta pe (a1 , a2 ), are derivat la drea a 0 0 f+ (a1 ) R {} i derivata f+ : [a1 , a2 ) R {} este funcie cr s t Urmeaza ca funcia g este continua pe (a1 , a2 ], este derivabila la dreapta p t are derivata la dreapta n a1 (n R) i avem s p1 p1 x + p2 a2 + + pn an 0 0 0 f+ (x) f+ , x g+ (x) = p1 + + pn p1 + + pn Deoarece p2 a2 + + pn an p1 x + p2 a2 + + pn x [a1 , a2 ) x < x< p2 + + pn p1 + + pn rezulta ca g+ (x) 0, x [a1 , a2 ). Conform Propoziiei 4, aplicata t 0 , rezult c funcia g a a t este descresctoare. Urmeaz c avem a a a g
[a1 ,a2 ] [a1 ,a2 ]

g (a2 ), care este prima inegalitate din (4). Prin inducie finit descendent se obin i celelalte inegaliti din ( t a a t s at inegalitate din (4) se obine direct din definiia convexitii. t t at

Observaia 1. n particular, din (4) se obine inegalitatea lui Jensen t t p1 f (a1 ) + + pn f (an ) p1 a1 + + pn an . f p1 + + pn p1 + + pn Observaia 2. Criteriul de monotonie de mai sus (Propoziia 4) este t t diferite situaii. De exemplu, pe baza lui poate fi obinuta o rafinare a i t t lui Tiberiu Popovici (a se vedea [2]). Bibliografie

1. C. P. Niculescu, L. E. Persson - Convex Functions and Their Applic Contemporary Approach, CMS Books in Mathematics, vol. 23, Springer-V York, 2006. 2. C. P. Niculescu, F. Popovici - A Refinement of Popoviciu s Inequality, Sci. Math. Roum. 49 (97), No.3, 285-290.

Asupra unor inegaliti geometrice at


Gheorghe IUREA1

Rezultatul principal al notei [1] este urmtoarea a Propoziie. Fie a, b, c lungimile laturilor unui triunghi; atunci, pe t x 0, au loc inegalitatile:
3 3

n cele ce urmeaz, vom demonstra c inegalitile (1), (2) i (3) au a a at s orice a, b, c numere reale pozitive.

(a+bc) x + b+ca (b+ca) x + c+ab (c+ab) x + a+bc p + p + p (ax + b) (bx + c) (bx + c) (cx + a) (cx + a) (ax + b) (b + c) x + a + c (c + a) x + b + a (a + b) x + c + b + + cx + a ax + b bx + c 2 ax + b + bx + c + cx + a ,

(a + b + c) (x + 1) 27 [(ab) (1x) + c (1+x)] [(bc) (1x) + a (1 [(c a) (1 x) + b (1 + x)] ,

Cu substituiile = (a b) (1 x) + c (1 + x), = (b c) (1 x) + t i = (c a) (1 x) + b (1 + x), observnd ca + + = (a + b + s 3 inegalitatea (1) se scrie sub forma ( + + ) 27 (10 ). Dac a atunci (10 ) este evident. Dac 0, cum + , + i + sunt n a a s rezulta ca , , sunt nenegative i atunci (10 ) urmeaza imediat din i s mediilor (M A M G). Egalitatea se atinge pentru a = b = c i x [0, s

Notnd ax + b = 2 , bx + c = 2 , cx + a = 2 , cu , , > 0, inega devine 2 + 2 2 2 + 2 2 2 + 2 2 + + 3 care, dup calcule, poate fi scris sub forma a a ( ) ( ) + ( ) ( ) + ( ) ( ) 0.

Aceasta este ns cunoscuta inegalitate Schur. Egalitatea se atinge cnd a a Folosind aceleai substituii, inegalitatea (3) este echivalent cu s t a 2 + 2 2 + 2 2 + 2 + + 2 ( + + ) , 2 2 2 2 + + + + i s + + + . Egalitatea are loc pentru a = b = c. care rezult prin sumarea inegalitilor a at Bibliografie 1.

I. V. Maftei, M. Haivas - Tehnici de stabilire a unor inegalitati g Recreaii Matematice 1/2008, 22-23. t
1

Profesor, Liceul Teoretic "Dimitrie Cantemir", Iai s

Metoda deligamentrii i rafinarea unor inega a s


Titu ZVONARU 1

Scopul acestei note este de a prezenta demonstraii elementare pentru t egalitai, ca i obinerea unor rafinari ale acestora. Descrierea metodei deli t s t poate fi gsit n [2]. a a Pentru nceput, o demonstraie prin metoda deligamentrii a unei t a cunoscute: a b c 3 1. + + , a, b, c > 0. b+c c+a a+b 2 Soluie. Avem t

1 2a b c ab ac a i, anal = = + s b+c 2 2 (b + c) 2 (b + c) 2 (b + c) b 1 bc ba c 1 ca cb = + , = + . Grupn c+a 2 2 (c + a) 2 (c + a) a + b 2 2 (a + b) 2 (a + b) n funcie de numaratorii lor, obinem: t t

ba (a b) (c + a b c) (a b)2 ab + = = 2 (b + c) 2 (c + a) 2 (b + c) (c + a) 2 (b + c) (c + a) mpreuna cu relaiile similare: t

cb (b c)2 ca ac (c bc + = , + = 2 (c + a) 2 (a + b) 2 (c + a) (a + b) 2 (a + b) 2 (b + c) 2 (a + deducem valabilitatea inegalitaii de demonstrat. t

Metoda deligamentarii, folosita n demonstraia urmatoarelor inegalita t la obinerea unor rafinari ale acestora. Chiar daca sunt necesare unele calcu t sunt uor de condus ctre rezultatul dorit. s a 2. a
2

3 (a + b + c) , a, b, c > 0. 4 (ab + bc + ca) (b + c) (c + a) (a + b) Darij Grinberg i Ce s Soluie. Demonstraia din [1] face apel la inegalitatea lui Cebev i t t s s tatea lui Gerretsen. Avem a 4ab + 4bc + 4ca 3b2 3c2 6bc 3a a 2 4 (ab + bc + ca) = 2 (b + c) 4 (ab + bc + ca) (b + c) +
2

= i, analog, s b
2

4 (ab + bc + ca) (b + c)

(3ab + ac) (a b)

4 (ab + bc + ca) (b + c)

(3ac + ab) (a c)

3b (3ab + bc) (b (3bc + ab) (b c) = 2 + 4 (ab + bc + ca) (c + a) 4 (ab + bc + ca) (c + a) 4 (ab + bc + ca) 3c (3bc + ac) (c c (3ac + bc) (c a) 2 4 (ab + bc + ca) = 2 + (a + b) 4 (ab + bc + ca) (a + b) 4 (ab + bc + ca)
1

Comneti, e-mail: tzvonaru@yahoo.com a s

Grupnd convenabil, obinem t 4 (ab + bc + ca) (b + c)2 = (3ab + ac) (a b) + 4 (ab + bc + ca) (c + a)2
2

(3ab + bc) (b a)

=
2

ab (3ab + ac) (c + a) (3ab + bc) (b + c) , 2 2 4 (ab + bc + ca) (b + c) (c + a)

i cum s (3ab + ac) (c + a)2 (3ab + bc) (b + c)2 = 3a3 b + a3 c + 3abc2 + ac3 + 6

deducem ca

+2a2 c2 3ab3 b3 c 3abc2 bc3 6ab2 c 2b2 c2 = 2 = 3ab a b2 + c a3 b3 + c3 (a b) + 6abc (a b) + 2c2 a2 b = (a b) 3a2 b + 3ab2 + a2 c + abc + b2 c + c3 + 6abc + 2ac2 + 2bc2 = (ab) a2 b+ ab2 + b2 c+ bc2 + a2 c+ ac2 + 3abc+ 2a2 b+ 2ab2 + 4abc+ c3 + = (a b) (a + b + c) (ab + bc + ca) + 2a2 b + 2ab2 + 4abc + c3 + bc2 + 4 (ab + bc + ca) (b + c) (3ab + ac) (a b)
2

(a b)2 (a + b + c) (ab + bc + ca) + 2a2 b + 2ab2 + 4abc + c3 + bc2 + 4 (ab + bc + ca) (b + c)2 (c + a)2 = (a b) (a + b + c) (ab + bc + ca) 4 (ab + bc + ca) (b + c) (c + a)2
2

4 (ab + bc + ca) (c + a)2

(3ab + bc) (b a)

(a b) (a + b + c) 4 (b + c)2 (c + a)2

Prin permutri circulare obinem nc dou relaii similare. Rezult u a t a a t a rafinare a inegalitaii date: t b c 3 (a + b + c) a 2 + 2 + 2 4 (ab + bc + ca) (b + c) (c + a) (a + b) 2 2 2 a+b+c (b c) (c a) (a b) + + 4 (b + c)2 (c + a)2 (c + a)2 (a + b)2 (a + b)2 (b + c) b2 c2 a a2 b c + 2 + 2 + + , a, b, c > 0. 2 + c2 2 2 b c +a a +b b+c c+a a+b Vasile Soluie. Avem t 3. a ab (a b) ac (a c) a2 = + , 2 + c2 2 + c2 ) b b+c (b + c) (b (b + c) (b2 + c2 ) b bc (b c) ab (b a) b2 = + , 2 + a2 2 + a2 ) c c+a (c + a) (c (c + a) (c2 + a2 ) c2 c ac (c a) bc (c b) = + 2 + b2 2 + b2 ) a a+b (a + b) (a (a + b) (a2 + b2 ) i mai departe s

ab (a b) ab (b a) ab (a b) + = 2 + c2 ) 2 + a2 ) (b + c) (b (c + a) (c (b + c) (c + a) (b2 + c2 ) (c2 + 3 2 2 c + ac + a c + a3 b3 b2 c bc2 c3 = ab (a b)2 c2 + ac + bc + a2 + ab + b2 ab (a b)2 c2 + ac + bc + = (b + c) (c + a) (b2 + c2 ) (c2 + a2 ) (b + c) (c + a) (b2 + c2 ) (c2 ab (a b) ab (a b) (b + c) (c + a) = 2 . (b + c) (c + a) (b2 + c2 ) (c2 + a2 ) (b + c2 ) (c2 + a2 ) Obinem urmtoarea rafinare a inegalitii n discuie: t a at t b2 c2 a b c a2 + 2 + 2 2 + c2 2 2 b c +a a +b b+c c+a a+b 2 2 2 ab (a b) bc (b c) ca (c a) 2 + 2 + 2 (b + c2 ) (c2 + a2 ) (c + a2 ) (a2 + b2 ) (a + b2 ) (b2 + c2 ) =
2 2

n ncheiere, propunem cititorilor demonstrarea i, eventual, rafinarea urm s inegaliti: at 4.

y n+1 z n+1 xn + y n + z n xn+1 + + , x, y, z > 0, n N. y+z z+x x+y 2 5. a, b, c fiind laturile unui triunghi, are loc inegalitatea bc ca ab + + a + b + c. a+bc b+ca c+ab Gabriel Do ab a+b Indicaie. t = . . . etc. a+bc 2 6. a, b, c fiind laturile unui triunghi, avem c2 + a2 a2 + b2 1 1 1 b2 + c2 + 3 + 3 + + . 3 + abc a b + abc c + abc ab bc ca Titu Zvonaru i Bogd s b2 + c2 1 1 Indicaie. 3 t = . . . etc. a + abc 2ab 2ac Bibliografie

1. C. Lupu - Asupra inegalitatii lui Gerretsen, R.M.T., 4/2006, 3-100. 2. T. Zvonaru - Inegalitati ligamentate si neligamentate, Arhimede, 5-6/20 Semnalm cititorilor reeditarea coleciei complete a revistei a t

RECREA TII STIIN TIFICE (1883-1

la 125 de ani de la apariia primului numar, cu respectarea formei n c t publicata iniial. Revista prezinta i astazi interes prin culoarea limbii t s terminologiei folosite, prin coninutul interesant i de un nalt nivel tiinif t s s t i prin forma grafic frumoas. Cei interesai pot consulta site-ul revistei s a a t

http://www.recreatiistiintifice.ro

O problem i . . . nou soluii as a t


Gheorghe IUREA1 , Gabriel POPA2

n numrul 2/2007 al Recreaiilor Matematice, Enache Ptracu a p a t a s rezolvare problema

G133. Fie 4ABC echilateral si D un punct astfel nct BD = DC, m \ 30 , iar BC separa A si D. Daca E (BD) cu m(BAE) = 15 , sa s CE AC.

Soluia autorului problemei (prezentata mai jos) recurge la o constru t toare interesant, dar greu de gsit. ncercrile de a aborda problema n a a a diferit au fost ncununate de succes ntr-o msur mai mare dect ne a a a s cele ce urmeaz pot fi gsite nou soluii ale problemei, iar cititorul prob a a a t mai observa i altele. s

Soluia 1. Notm cu A0 simetricul lui A faa de BC. Observm c m t a t a a \ m(A0 BC) = 75 60 = 15 , prin urmare EBA0 EAA0 . D \ \ \ m(EBC) \ \ \ patrulaterul ABEA0 este inscriptibil, de unde EA0 B EAB, adic m(EA a Obinem astfel ca 4EBA0 este isoscel cu EB = EA0 i de aici rezulta ca t s pe mediatoarea segmentului [BA0 ], deci CE BA0 . Este nsa clar ca B prin urmare CE AC (fig. 1).

Fig. 1 Fig. 2 Fig. 3 Soluia 2 (Sergiu Prisacariu, Cristian Lazr). Fie F (BD) a t a \ \ CF AC; atunci m(BCF ) = 90 60 = 30 i cum m(CBF ) = 75 s \ = 75 . Rezulta ca CB = CF , de unde CA = CF , adica 4 ca m(CF B)
1 2

Profesor, Liceul Teoretic "Dimitrie Cantemir", Iai s Profesor, Colegiul Na ional, Iai t s

[ \ dreptunghic isoscel, cu m(CAF ) = 45 . Astfel, m(BAF ) = 60 45 = \) = 15 i astfel F = E, de unde concluzia problemei (fig. urmare m(BAF s

Soluia 3 (Enache Ptracu). Considerm punctul S pentru care A t a s a \ [ AB BS, iar AB separa C i S. Cum m(ABE) = m(SBE) = 135 , d s [ 4ABE 4SBE (L.U.L.), de unde AE = SE. ns m(SAE) = 45 + 1 a deci 4ASE este echilateral. Rezulta ca 4ABS 4ACE (SA = AE, A [ [ [ [ iar m(SAB) = m(EAC) = 45 ), prin urmare m(ACE) = m(ABS) = 90

Soluia 4 ( dup o idee dat de Ctlin Budeanu). Vom calcu t a a a a 4ACE n funcie de a = AB. Mai nti, observam ca, daca {M } = AD t a 3 a 3 a a = a 1+ = ca AD = AM + M D = + + 2 2 tg 15 2 2 2 3 BE AB 31 i, pe de cum AE este bisectoare n 4ABD, atunci = = s ED AD 2 p 2 2a 1 + 3 2+ 3 2 AB AD cos 15 = = a 2. Folosind AE = AB + AD 2 a 2+ 3 Stewart n 4BCD, obinem ca CE 2 BD = BC 2 DE+CD2 BEBEDE t 31 ED; dupa calcule de rutina, deducem c vazut mai sus ca BE = 2 n concluzie, CA = CE = a i AE = a 2 i, din reciprova teoremei lui s s rezult c EC AC (fig. 4). a a

Fig. 4 Fig. 5 Soluia 5. Fie {N } = AD CE i G centrul triunghiului ABC; ca t s BE 31 precedenta, avem ca = , de unde, folosind proporiile derivat t ED 2 DE ca = 31. Aplicam teorema lui Menelaus n 4BM D cu transversa DB BE DN M C DN C; deducem c a a = 1, de unde = 2 3 + 1 . Dup ED N M CB NM

Soluia 6. Folosim teorema sinusurilor n triunghiurile ABE i BCD t s BE AB BC BD c a = , respectiv = . Se verific prin calcu a sin15 sin 30 sin 30 sin 75 sin 15 sin 30 BE BC = , deci = . Cum AB = BC i CBE CBD, u s \ \ sin 30 sin 75 AB BD [ \ 4CBE 4DBC, prin urmare m(BCE) = 30 , de unde m(ACE) = 90

a 3+ 3 a 3+2 3 DN rutina, rezulta ca DN = , DG = , i atunci s 3 2 DG DE DN i din reciproca teoremei lui Thales obinem c n concluzie, = s t a DB DG prin urmare N E AC, tocmai concluzia problemei (fig. 5).

ordonata lui E va fi media aritmetica a ordonatelor punctelor A i D, adica s xE xB yE yB = , de u Cum punctele B, E, D sunt coliniare, avem ca yD yB x xB D a 1 3 yA yC = 3, iar panta d . Panta dreptei AC este mAC = 2 xA xC yC yE 1 este mCE = = i, cum mAC mCE = 1, rezulta ca CE A s xC xE 3 Soluia 8. Folosim reperul din soluia precedent, lucrnd ns cu nu t t a a plexe. Pentru simplitate, vom considera c AB = 2; atunci C (1), B (1) a 31 BE zB + kzD = = k, avem zE = = 1 D 2 + 3 i . Cum ED 2 1+k zE zC = i, prin urmare CE AC (i, n plus, CE = AC) ( s Rezulta ca zA zC CB + 31 CD BE 31 2 Soluia 9. Din t = = , obinem c CE = t a ED 2 1 + 31 2 3 1 CD 2CB + 1 = 2CB CA + 3 1 C , deci CE CA = 3+1 3+1 a2 ns CB CA = CB CA cos 60 = a , CD CA = CA + AD CA 2 \ = 3 + 1 a2 . Astfel, CE = 0, de unde CE C CA AD AC cos CAD 2

Soluia 7. Raportam planul un reper cu originea n M , unde M t la a 3 a a a BC; dac a = AB, atunci A 0, a , B ,0 , C , 0 , D 0, 2 2 2 \ \ Observm c 4EAD este isoscel, deoarece m(EAD) = m(EDA) = 15 , p a a

Not. Soluia 6 sugereaz urmtoarea extindere: a t a a Se considera 4ABC isoscel ( AB = BC) si triunghiul BCD cu A, D \ \ plane opuse faa de BC, iar m(ABC) = 2m(BDC). Daca E (BD t x sin 2 sin \ , unde = m(BAE), = m prietatea ca = sin ( + + x) sin + x 2 \ x = m(ABC), atunci CE AC. s Problema G133 se obine pentru = 15 , = 75 i x = 60 . t

Sur les matrices magiques


Adrien REISNER1

Toutes les matrices considres ici appartiennent lespace vectoriel r Cet espace est de dimension 9. Nous nous proposons dtudier certaines de lensemble des matrices 3 3 dites magiques dont la dnition est la s Dnition. Une matrice A = (aij ) M3 (R) est dite magique si les hu ai1 + ai2 + ai3 , a1j + a2j + a3j , a11 + a22 + a33 , a31 + a22 + a13 sont i, j : 1, 2, 3. On appelera somme magique cette somme commune. Considrons les trois matrices suivantes (videmment magiques): 1 1 1 1 2 1 0 0 , N = MT . L = 1 1 1 , M = 0 1 1 1 1 2 1 Proposition 1. Une matrice quelconque A est somme dune matrice s A0 et dune matrice antisymtrique A00 , la dcomposition A = A0 + A00 ta 1 Dmonstration. On a de faon unique: A = A0 + A00 , o A0 = A 2 0 1 A00 = A AT ; A est symtrique et A00 est antisymtrique (i.e. A00T = 2 Proposition 2. La somme de deux matrices magiques est une matrice La transpose dune matrice magique est magique. Enn le produit dun maqique et dun scalaire est une matrice magique. Si A est une matrice m deux matrices A0 et A00 dnies plus haut sont elles mmes magiques.

On se propose de construire toutes les matrices magiques antisymtr 0 0 , alors: + = + = + = eet, si A00 = 0 0 1 = = 0. On en dduit la solution gnrale: A00 = 1 0 1 1 est un scalaire arbitraire. On se propose de construire toutes les matrices magiques symtriques. que prcdemment toutes les matrices symtriques A0 = ij ) vriant: (a s 1 1 1 0 1 a11 + a22 + a33 = 0 sont de la forme suivante: A0 = 1 0 1 0 1 1 un scalaire arbitraire. Si s 6= 0, la forme gnrale des matrices symtriques s sobtient en ajoutant aux lments aij de la matrice prcdente. On en 3 solution gnrale: 1 1 0 1 1 1 1 + 1 1 1 , A0 = 1 0 0 1 1 1 1 1
1

Centre de Calcul E.N.S.T., Paris; e-mail: adrien.reisner@enst.fr

o , sont des rels arbitraires. Compte tenu de la Proposition 1, immdiatement la forme gnrale des matrices magiques: 0 1 1 1 1 0 1 1 1 1 + 1 0 1 + 1 1 1 = A = 1 0 1 1 0 0 1 1 1 1 1 1 1 = ( ) M ( + ) N + L. 2 2

et par suite lquivalence: AB est une matrice magique 0 = 0 obtient les quatre cas suivants: = = 0 L 0 M + 0 N + 0 L = 3 0 L (1) = 0 = 0 (N + L) 0 N + 0 L = 3 0 L (2) = 0 = 0 (M + L) (0 M + 0 L) = 3 0 L (3) 0 = 0 = 0 0 L (M + N + L) = 3 0 L (4) Les deux cas (1), (4) nen forment quun seul. Les cas (2), (3) sont id un change de matrices prs. On en dduit immdiatement la deuxime p Proposition 4. Proposition 5. Le produit dune matrice magique par une combinais de L et I est une matrice magique. Dmonstration. De faon vidente la matrice: (M + N + L) (0 I + 0 L) = 0 M + 0 N + (0 + 3 0 ) L est magique. Proposition 6. A = M + N + L est inversible si et seulement si De plus, dans le cas o 6= 0 la matrice A1 est elle mme magique. Dmonstration. La proposition est immdiate puisque det (M + N 1 3 3 4 1 36. Dans le cas o 6= 0 il vient: A = M+ N+ L 36 bien une matrice magique.

Les trois matrices M , N et L tant linairement indpendentes on a, d Thorme 3. Lensemble des matrices 33 magiques est un sous-espac de M3 (R). Ce sous-espace vectoriel a pour dimension 3, une base tant f les trois matrices L, M et N . Etant donn les deux matrices A = M + N + L et B = 0 M + nous avons: Proposition 4. Le produit AB est une matrice magique si et seuleme 0 = 0 = 0. La matrice L est la seule matrice ( un facteur scalaire soit magique et produit de deux matrices magiques. Dmonstration. Les relations videntes: M 2 = N 2 = M L = LM LN = 0, L2 = 3L et M N + N M = 12I 4L conduisent immdiatement AB = (M + N + L) 0 M + 0 N + 0 L = 20 + 6 0 40 20 6 0 80 40 = 3 0 L + 40 0 0 0 2 6 4 20 + 6 0

Cette proposition se gnralise avec le thorme suivant: Thorme 7. Soit A = M + N + L une matrice magique. Alors, n N la matrice A2n+1 est magique. Si 6= 0, pour tout p N A(2p+1) est magique. Dmonstration. Avec A = M +N +L on obtient A2 = 12I+ 3 et la Proposition 5 permet alors par une rcurrence vidente sur n N de les implications suivantes: A2n1 est une matrice magique A2n = kI A2n + 1 est une matrice magique. On conclut alors grce la proposition 2p+1 puisque A(2p+1) = A1 . Remarques. Mis part le cas o = 0 voir la Propozition 4 le A2n ne sont pas magiques. Si = 0 les matrices A et An = 3n1 n L avec n > 1 sont magiques. Si 6= 0 on obtient immdiatement par rcurrence sur n N: i 1h n 2n n A2n = (12) I + (3) (12) L 3 nest pas magique. n 2n A2n+1 = (12) (M + N ) + (3) L est une matrice magique. Si 6= 0 ces mmes formules sont vries aussi pour n < 0. Gnralisation. Les rsultats prcdents peuvent tre gnraliss pou trices magiques appartenant lespace vectoriel rel Mn (R). On considr deux sous-espaces vectoriels S0 = Mg0 Symn (R) et A0 = Mg0 Asymn (R est lensemble de matrices magiques ayant une somme magique nulle et [resp. Asymn (R)] est lensemble des matrices symtriques [resp. antisym Mn (R). Mgn (R) tant le sous-espace vectoriel des matrices magiques d on dmontre que Mgn (R) = Mg0 , o est la droite engendre par (cij ) avec cij = 1 (i, j : 1, . . . , n) voir le Thorme 3. De plus les sous-es toriels S0 et A0 admettent chacun une base forme des matrices dont les c appartiennent lensemble {0, 1, 2}.

Carrs magiques. Un cas particulier des matrices magiques A Mgn (R) est le "carr magique". Pour construire un carr magique on la condition supplmentaire: aij 1, 2, . . . , n2 . Exemples clbres des c iques: Le carr magique "lo-shu" attribu au philosophe chinois Kung Tzu (C 4 9 2 (VI-me sicle avant J.C.): 3 5 7 est un carr magique connu pro 8 1 6 bien avant Confucius. (voir: http://membres.lycos.fr/fusionbfr/JHM/CM/ 16 3 2 13 5 10 11 8 Un autre carr magique clbre est: 9 6 7 12 qui appar 4 15 14 1 clbre tableau (eau-forte) de Albrecht Drer (1471 1528) intitul M (voir: http://users.skynet.be/litterature/lecture/melancholia.htm).

Concursul de matematic Al. Myller a


Ediia a VI-a, Iai, martie 2008 t s Clasa a VII-a

1. Numerele reale distincte x, y, z au propietatea c x3 x = y 3 y a Sa se arate ca x + y + z = 0. 2. a) S se arate c, dintre cinci numere naturale oarecare, se pot a a numere cu suma divizibil cu 3. a b) Sa se arate ca, dintre 17 numere naturale oarecare, se pot alege no cu suma divizibila cu 9. 3. Fie AD nalimea triunghiului ascuitunghic ABC. Consideram mu t t \ = ACX. \ a punctelor X (AD) cu proprietatea c ABX a a) S se arate c mulimea M este nevid. a a t a b) Daca M conine cel puin doua elemente, sa se demonstreze ca mu t t conine o infinitate de elemente. t Cristi 4. Fie segmentul AB i semidrepta (Ox, unde O (AB) i A, B s s / pendicularele n A i B pe dreapta AB intersecteaza bisectoarele (Oy s [ s [ unghiurilor AOx i BOx n punctele M , respectiv N . Perpendiculara din intersecteaza perpendiculara din B pe (Oz n punctul P . Sa se arate ca pu N , P sunt coliniare. Mirc

Clasa a VIII-a

1. Consideram cubul ABCDA0 B 0 C 0 D0 i M , N , P mijloacele muchiilor s respectiv AA0 . Sa se determine masura unghiului dintre dreapta A0 C 0 i d s intersecie a planelor (M N P ) i (BCC 0 ). t s 2. Fie a, b numere ntregi distincte cu proprietatea ca exista n numar nct a3 a = b3 b = n. S se arate c n = 0. a a 3. Se dau ase puncte n plan, oricare trei necoliniare. Consideram zece s fiecare avnd capetele n cte doua dintre aceste puncte. Sa se arate ca puin un triunghi avnd ca laturi trei dintre cele zece de segmente. t 4. Fie SABC un tetraedru regulat. Punctele A1 , B1 , C1 aparin much t (SB), (SC), respectiv, astfel nct A1 B1 = B1 C1 = C1 A1 . S se arate a (A1 B1 C1 ) i (ABC) sunt paralele. s

Clasa a IX-a
1. Determinai numarul soluiilor ecuaiei t t t [x] 2007x = . {x} 2008
6

Miha 2. Rezolvai n mulimea numerelor ntregi ecuaia x + x + 4 = y 2 . t t t Ioan Cuc


5

3. Fie ABCDE un pentagon convex. Demonstrai c t a aria(ABC) aria(CDE) + < 1. aria(ABCD) aria(BCDE)

Dan Is s 4. Fie C1 , C2 doua cercuri concentrice distincte i [AB] un diametru C1 . Considerm dou puncte variabile M C1 , N C2 , nesituate pe drea a a a) Artai c exist i sunt unic determinate punctele P , Q, situate p a t a as M A, respectiv M B, astfel nct N sa fie mijlocul segmentului [P Q]. b) Aratai ca suma AP 2 + BQ2 este constanta, unde P , Q sunt definit t Mihai Piticari, Miha

Clasa a X-a

1. Fie O centrul cercului circumscris triunghiului ABC i A1 punctul s diametral opus lui A. Notm cu G, G1 centrele de greutate al triunghiuril a PG 2 A1 BC i cu P intersecia dreptelor AG1 i OG. Sa se arate ca s t s = . PO 3 Gabriel Popa, Paul G 2. Sa se arate ca nu exista numere ntregi a, b, c astfel nct (a+bi 3)17 Dorin Andrica, Miha 3. Sa se determine poligoanele convexe, inscriptibile, cu proprietate triunghi determinat de trei dintre vrfurile acestora este isoscel. Gheorg n 1 1 n 4. Fie r un numar real cu proprietatea ca 2 r 4 , 2 r + 4 Z 6= orice n N. Sa se arate ca r este numar ntreg. Cipria

Clasa a XI-a

1. Fie A M4 (R) astfel nct det(A2 I4 ) < 0. Sa se arate ca exista || < 1, astfel nct matricea A + I4 sa fie singulara. Mih a 2. Fie A, B, S M3 (C), S fiind o matrice nesingular nct B = S 1 arate ca tr(B 2 ) + 2 tr(B ) = (tr(A))2 . Mih 3. Fie a > 1 un numr real. Pentru fiecare numr natural nenul n, k( a a

a mai mic numr natural k pentru care (n + 1)k ank . S se calculeze lim a n Necula 4. Fie f : R R o funcie continu pe Q, cu proprietatea c f (x) < t a a pentru orice x R i n N . Sa se demonstreze ca f este strict crescatoa s Gabriel Mranu, Miha s

Clasa a XII-a
1. Se consider irul (an )nN , an = as
n R 1 dx (1+x2 )n .

Calculai lim n 2n an t n Bogda

Miha 4. Fie A un inel finit n care numarul elementelor inversabile este egal c elementelor nilpotente. S se arate c numrul elementelor inelului este a a a lui 2. (Un element x A se numeste nilpotent dac exist k natural cu x a a Dinu Se

2008, pentru orice n N.

2. Determinai numerele n N, n 3 i a R pentru care polinomul X t s are un divizor de foma X 2 + X + cu , Z. Miha 1 R 3. Determinai funciile crescatoare f : [0, 1] R pentru care f (x t t
0

Concursul de matematic Florica T. Cmp a


Etapa judeean, 16-17 februarie 2008 t a Clasa a IV-a
31.

1. a) Gsii regula de formare a irului 3, 8, 13, . . . i scriei termenul d a t s s t

b) Dupa un concurs de matematica, un elev nu i-a amintit rezultatul s leme. Totui, i-a adus aminte c numrul are ase cifre, ncepe cu 1 i d s s a a s s cifr se mut la sfrit, atunci numrul obinut va fi de trei ori mai mare a a s a t iniial. Care a fost rezultatul problemei? t 2. Mergnd cu maina, un ofer observa la ora 09:10 ca pe kilometr s s bord apare numrul 12921. La ora 11:00, pe kilometraj apare urmtorul n a a coincide cu rsturnatul su. La ce or va observa oferul din nou un astfel a a a s presupunnd ca se deplaseaza cu viteza constanta? Gabriel Mranu, Recreaii Matematic s t 3. Pe o foaie este scris numrul A = 1234xy. Cinci elevi joac urm a a fiecare dintre primii patru citete numarul, i fixeaza cte o regula de tra s s a lui i scrie pe tabla numarul transformat. Al cincilea, care cunoate do s s patru cifre ale lui A, trebuie s ghiceasc regula fiecruia dintre colegi i a a a s s afle numrul. Stiind c primii patru au scris pe tabl numerele 12350 a a a a 123460, 120000, se cere: a) Care sunt regulile de transformare observate de al cincilea elev? b) Poate al cincilea elev s afle cu exactitate numrul? Care sunt valor a a ale numrului A? a Petr

Clasa a V-a

1. La un concurs se acord cinci puncte pentru premiul I, trei puncte a doilea i doua puncte pentru al treilea. Aflai numarul de premii primit s t unei coli, tiind ca au obinut n total 25 de puncte i cel puin cte doua s s t s t fiecare categorie. 2. Un numr natural se numete simpatic dac este format din cifr a s a nenule, a caror suma se divide cu 10.

a) Determinai cel mai mic i cel mai mare numr simpatic. t s a b) Precizai cte numere de trei cifre sunt simpatice i divizibile cu 4. t s

3. Dac a N i b N , notm ab = ab +ba (de exemplu, 32 = 32 + a a s a) Determinai numarul n N astfel nct 1 1 + 2 1 + 3 1 + + t b) Comparai numerele 3 18 i 2 27. t s c) Aflai ultima cifr a numrului 2 (2 2008). t a a Adrian

Clasa a VI-a

1. Un parinte i mparte averea astfel: primul copil primete 10 00 s s o cincime din rest; al doilea copil primete 20 000 lei plus o cincime din s al treilea copil primete 30 000 lei plus o cincime din noul rest i aa ma s s s S se afle suma mprit de printe, precum i numrul copiilor, tiind a at a a s a s moteniri egale. s Mihai Grtan, Recreaii Matematic t

2. Ctlin este faianar i trebuie s paveze podeaua unei ncperi n aa t s a a dreptunghi avnd lungimea de 3 metri i laimea de 2 metri, folosind dale s t cu latura de 50 centimetri. El are la dispoziie 6 dale roii, 6 albastre, 6 g t s verzi, iar cerina este ca orice dou dale de aceeai culoare s nu se ating t a s a a a) Indicai un exemplu de pavare corect. t a b) Catalin sparge o dala roie i primete n loc una verde. Poate acu s s s n aa fel nct s respecte cerina? Justificai rspunsul. s a t t a Do

3. Un numar natural N se scrie n baza 10 folosind 6 cifre nenule i s Se tie c, oricum am schimba ordinea cifrelor numrului N , numrul N s a a a numerele obinute sunt toate multipli de p, unde p este un numr prim. t a a) Determinai cte numere se pot obine din N prin schimbarea ordin t t b) Daca p = 3, dai exemplu de un numar N care sa verifice condiiile t t c) Dac p 6= 3, demonstrai c nu exist numere N care s verifice con a t a a a enun. t R

Clasa a VII-a

1. Se considera irurile definite prin: a1 = 91204; a2 = 9012004; a3 = 9 s . . . ; b1 = 91504; b2 = 9015004; b3 = 900150004; . . . a) Aflai numrul de cifre al sumei an + bn , unde n N ; t a b) Aratai ca an + bn Q, n N ; / t c) Demonstrai ca an Q, n N , nsa bn Q, n N . t / Constantin Chiril, Recreaii Matematic a t

2. Doua blocuri de locuine care au nalimea de 21 m fiecare sunt t t un teren plat. La ora 10, umbra primului bloc proiectat pe cel de-al a nlimea de 15 m, iar umbra celui de-al doilea bloc pe pmnt are lungi at a m. Ce nalime are umbra primului bloc pe cel de-al doilea la ora 11, d t celui de-al doilea bloc pe pamnt este de 31,50 m?

3. Pardoseala unei bi de dimensiuni L = 45 dm, l = 35 dm este acoperi a de gresie n forma de patrat cu latura de 1 dm. Daca se ndeparteaza cte

din cele patru coluri, se poate acoperi suprafaa rmas cu plci dreptung t t a a a dimensiuni 1 dm 2 dm?

Clasa a VIII-a
1. a) Fie suma

1 1 1 +p + + p S=p 21 21 1+ 1 3+ 3 2007 + 20072 Aflai cel mai mic numar natural nenul n pentru care numarul S n est t b) Dac a este lungimea ipotenuzei i b, c lungimile catetelor unui triu a s tunghic, demonstrai c 2a > b + c + ha , unde ha este lungimea nlimii t a at zatoare ipotenuzei. Claudiu Ste 2. Cte plane pot fi duse la egala distana de patru puncte necoplan t Justificai rspunsul. t a 3. n cetatea N N a numerelor naturale se organizeaz o mare petrecere a numrului 0. La poarta castelului bate unul din locuitorii cetii. a at Sunt numarul 83. mi permitei sa intru la petrecere? ntreaba acest t La petrecere sunt invitate doar numerele fantastice, i rspunse o voce a cealalt. a Dar ce nseamn numr fantastic? ntreab numrul 83. a a a a Sa va explic, spune vocea stranie. Daca n este un numar natural dect 1 i notm An = {x N | (x, n) 6= 1}, numrul n se numete fant s a a s pentru orice dou numere x, y aparinnd mulimii An , suma lor x + y e a t t element al mulimii An . Ai priceput? t t Am neles, raspunde lamurit vizitatorul. t a) Stabilii voi dac numrul 83 este invitat la petrecere. Aceeai cerin t a a s t numrul 2008. a b) Gsii toate numerele pare invitate la petrecere. a t Alexandru

Etapa interjudeean, 22-23 martie 2008 t a Clasa a IV-a

1. George este mai mic dect Andrei cu o ptrime din vrsta lui And a un an, Andrei va fi mai mare dect George cu o ptrime din vrsta lui G a vrsta au acum Andrei i George? s 2. Se considera mparirea: t a) Dai un exemplu de astfel de mparire. t t b) Cte mpriri de acest tip se pot efectua? Justi- = = at cai rspunsul! t a = = 3. Un elev de clasa a IV-a are n total 100 de fructe, nuci i mere. El schimba cu un prieten cte noua nuci s = pentru doua mere, terminnd toate nucile dupa un numar de schimburi i rmnnd n final cu 44 de mere. s a a) Cte nuci a avut iniial elevul? t b) Cte schimburi s-au facut i cte mere a primit de la prietenul sau? s

Clasa a V-a

1. Un numar se numete fiul unui alt numar daca este format cu d s cifrele numarului iniial, numit tata. Dintre numerele de trei cifre cu ultim t aflai toi taii cu 891 mai mari dect unul dintre fiii lor. t t t 2. Se considera urmatorul tablou cu 200 de linii. a) Ce numr se afl n mijlocul ultimei linii a a a tabloului? b) Cte numere conine tabloul? t c) De cte ori apare numrul 100 n acest tablou? a 2 2 4 2 2 4 6 4 2 4 6 8 6 .................

3. La concursul "Florica T. Cmpan", etapa locala, au luat parte to t clasa a V-a dintr-o coal. Elevii din clasa a V-a D au obinut urmtoarele s a t a prima problem au rezolvat-o 9 elevi, a doua problem au rezolvat-o 7 ele a a problema au rezolvat-o 5 elevi, a patra problema au rezolvat-o 3 elevi, ia problema a rezolvat-o un singur elev. Toi elevii clasei, n afara de Petrica, a t acelai numr de probleme, n timp ce Petric a rezolvat cu una mai mult d s a a si. Poate s fie el premiant al concursului, dac premianii concursului au a a a t care au rezolvat 4 sau 5 probleme?

Clasa a VI-a
1. Se considera mulimea A = t mulimii A N. t

2008 2009 2010 , , , . . . . Determinai t 7 8 9

2. Fie dreapta AB, O un punct ntre A i B i, de aceeai parte s s s \ semidreptele [OA1 , [OA2 ,. . . , [OAn , n aceasta ordine, astfel nct m(AO \2 ) = a + 2,. . . , m(An OB) = a + 2n, unde a N, n N, n 2. D \ m(A1 OA numrul unghiurilor i msura fiecruia dintre ele. a s a a

3. a) Artai c, oricum am alege cinci numere naturale, exist print a t a a cu suma divizibila cu 3. b) Aratai ca, oricum am alege 25 de numere naturale, exista printre e t suma divizibil cu 9. a

1. a) Se considera numerele 1, 3 2, 3 + 2 i 5. Dupa un pas, fiec s se nlocuiete cu media aritmetic a celorlaltor s a trei. Este posibil ca, dup a de pai, s obinem numerele 5 2 2, 3, 3 2 i 2? s a t + 2s b) Sa se arate ca daca numerele a, b i a + b sunt raionale, atunc s t sunt raionale. t

Clasa a VII-a

2. Fie ABC, AB < AC i D (AC). Fie AE bisectoarea unghiu s E (BD), F mijlocul lui [AD], {O} = AE BF , {G} = DO AB. S s a GD k BC AB = CD. Daniela Tama, Recreaii Matematic s t

3. Ionel i Gigel au trasat cu creta pe parchet, n dou coluri diferit s a t camere, cte un segment de dreapta cu capetele la marginea pereilor, t

dou triunghiuri dreptunghice. Dac cele dou triunghiuri au aceeai arie a a a s perimetru, sa se arate ca ele sunt congruente.

x+y , tiind ca 0 < x < y i (x y)(3x s s xy 2. Doi puti au un cornet transparent care, ca i lichidul omogen pentru s s coninut n el, au forme de piramide patrulatere regulate de vrf V i baze X t s respectiv A1 A2 A3 A4 , Ai (V Xi ), i = 1, 4. Deoarece, din motive care seaz aici, nu-l pot nghea n poziie vertical, ei nclin cornetul, fr a t t a a aa astfel nct noua suprafaa a lichidului este A0 A0 A0 A0 , A0 (V Xi ), i t 1 2 3 4 i (A1 A2 A3 ) (A0 A0 A0 ) = M1 M2 , unde M1 (A2 A3 ), M2 (A1 A4 ), ia 1 2 3 A1 A2 . a) Artai c patrulaterul A0 A0 A0 A0 este trapez isoscel. a t a 1 2 3 4 b) Cum vor controla cei doi puti nclinarea cornetului, nainte de a s ngheat, pentru ca, dupa aceea, planul (V M1 M2 ) sa mparta ngheata t t egale, tiind ca pot sa masoare doar ariile suprafeelor A1 A2 A0 A0 i A3 A s t 2 1 s Claudiu Ste 1. Sa se afle valoarea fraciei t

Clasa a VIII-a

3. Se dau zece numere naturale nenule care au suma egala cu 100. De c putem alege trei dintre numerele date care s poat fi lungimile latu a a a triunghi. Adrian Zanoschi, Recreaii Matematic t

Concursul "Student pentru o zi" prezentare de Ctlin TIGAERU 1 aa

ncepnd din acest an, Universitatea " tefan cel Mare" din Suceava or S n lunile martie i aprilie, suita de concursuri care poart denumirea generi s a pentru o zi. Concursurile se adreseaza elevilor de clasa a XII-a i se des s acele discipline al cror studiu este aprofundat n facultile Universitii. a at at matematic a avut loc n ziua de 9 martie i a constat n rezolvarea a patru a s n timp de trei ore, programa fiind anunat n prealabil. Au participat 2 t a de la cteva colegii renumite din judeele Neam, Botoani i Suceava. Pr t t s s i III au fost n valoare de 500, 300, respectiv 200 de lei. Ctigtorii ace s s a ediii sunt urmtorii: t a Premiul I Cepoi Alexandru, C.N. " tefan cel Mare", Suceava S Premiul II Bozianu Rodica, C.N. "A. T. Laurian", Botoani s Premiul III Brescu Lucian, C.N. "Mihai Eminescu", Botoani a s Problemele date la concurs au fost urmtoarele: a ln x Problema 1. Se consider funcia f : (0, ) R, f (x) = . a t x
1

Lector dr., Universitatea " tefan cel Mare", Suceava S

a) Artai c funcia F : (0, ) R, F (x) = 2 x (ln x 2), este o a t a t pentru funcia f . t b) Demonstrai ca orice primitiva G a funciei f este crescatoare pe [1 t t c) Aflai aria suprafeei cuprinse ntre graficul lui f , axa Ox i dreptele t t s 1 x = i x = e. s e Subiect propus pentru Bacalau

Problema 2. Fie (A, +, ) un inel de caracteristic 3 (3x = 0, x A a s un element oarecare. Fie M = x A | x3 = ax = xa i b M cu prop b comuta cu orice element din A. a) Demonstrai ca b x M , oricare ar fi x M . t b) S se arate c, dac a A are n plus proprietatea c exist n a a a a a care an = 1, atunci x + (b x)2n+1 M , oricare ar fi x M . Ion Bursuc Problema 3. Dac f : [0, 1] R este funcia continu, atunci a t a Z 1 Z 1 x (f (x) + f (1 x)) dx = f (x) dx.
0 0

Dumitru Crciun, a Problema 4. Se consider numerele prime distincte p i q, astfel nct a s cu r 3 prim. a) Aratai ca p q | pq q p daca i numai daca p q | q pq 1. t s b) Demonstrai ca p q | pq q p daca i numai daca q 2 1 (mod (p t s a Ctlin Tigeru a a

Prima problem fiind una cunoscut, prezentm doar soluiile celorlalt a a a t

Problema 2. a) Observam ca ( x)3 = 3 32 x + 3x2 x3 = 3 ax. Cum M , atunci 3 = a = a, de unde deducem c ( a a ax = a xa = a ( x) = ( x) a. b) Demonstram prin inducie ca x2m+1 = am x, pentru orice x M t s Pentru m = 1, relaia este adevrat. Presupunem c, dac x M , atun t a a a a ak1 x; atunci x2k+1 = x2k1 x2 = ak1 x x2 = ak1 x3 = ak1 ax = ak ncheie demonstaia afirmaiei iniiale. t t t Deoarece x M , atunci pentru orice numar m N , rezulta ca ( 2n+1 m t a = x, de unde x+( a ( x). Lund m = n, obinem c ( x) M , ceea ce trebuia demonstrat. Problema 3. Fcnd o schimbare de variabil, se arat c a a a a a =
b R a b R a

f (x) dx

f (a + b x) dx, f : [a, b] R continu. Aplicnd de dou ori a a a


1 R 0

tat, obinem c t a
1 R 0

f (x) dx =

1 1 R R (1 x) (f (x) + f (1 x)) dx, de unde 2 x (f (x) + f (1 x)) dx = f ( 0 0

1 R 0

f (1 x) dx, respectiv

1 R 0

x (f (x) + f (1

cluzia problemei. Problema 4. a) Putem scrie c pq q p = (p q) pq1 + qpq2 + a q q (q pq 1); deoarece (p q, q q ) = 1, deducem ca p q | pq q p daca i n s p q | q pq 1. b) Rezultatul de la punctul a) se reformuleaz astfel: p q | pq q p dac a dac q pq 1 (mod (p q)). Dar (p q, q) = 1, deci q (pq) 1 (m a (teorema lui Euler); deoarece (p q) = (2r) = r 1, deducem ca (mod (p q)) i q 2r 1 (mod (p q)). Dar (r 1, 2r) = 2, deci p q | pq s i numai dac q 2 1 (mod (p q)). s a Particulariznd, se pot obine o serie de probleme interesante; iata ctev t 34 | 67101 10167 , 34 | 137103 103137 , 22 | 881859 859881 , 6 | 20112017

1 R 0

1 1 R R f (x) dx. Deducem ca 2 x (f (x) + f (1 x)) dx = 2 f (x) dx, adica to 0 0

1. La data de 15 ianuarie 1883 a aprut primul numr al revistei Re a a inice (18831888). n anul acesta, 2008, se mplinesc 125 de ani de t acesteia. Scriei 2008 folosind numrul 125 i (numai) operaiile de adunare i t a s t s Care este numrul maxim de operaii cu care putei face aceast scriere a t t a minim? (Nu se accepta termeni nuli!) 2. Se dau trei cifre i un rezultat. Indicai operaiile necesare pentru s t t egalitatea! 1 2 3 4 5 6 7 8 9 1 2 3 4 5 6 7 8 9 1=8 2=8 3=8 4=8 5=8 6=8 7=8 8=8 9 = 8.

Not. Raspunsurile pot gasite la pag. 160. a

Soluiile problemelor propuse n nr. 2 / 20 t


Clasele primare

P.134. De la apartamentul meu cobor 7 etaje, apoi urc 4 etaje si obse la etajul 9. La ce etaj locuiesc? (Clasa I ) Drago Iacob, s Soluie. 9 + (7 4) = 9 + 3 = 12. Eu locuiesc la etajul 12. t P.135. n trei vase sunt 36 nuci. Daca din primul vas se iau 3 nuc treilea o nuca si se pun n al doilea vas, atunci n fiecare vas va fi acelai s nuci. Cte nuci au fost la nceput n fiecare vas? (Clasa I ) nv. Rica Buct a Soluie. Din 36 = 12 + 12 + 12 obinem: n primul vas erau 12 + 3 = t t al doilea vas 12 3 1 = 8 nuci, iar n al treilea vas 12 + 1 = 13 nuci. P.136. Aflai vrsta tatalui meu, stiind ca este un numar cuprins ntr t dublul lui ntre 73 si 77, iar triplul lui este cuprins ntre 112 si 118. (Clasa a II-a) Iurie Juc, Soluie. Numerele cuprinse ntre 35 i 41 sunt 36, 37, 38, 39 i 40. N t s s nu verific a doua condiie, iar numrul 37 nu verific a treia condiie. N a t a a t verific toate condiiile, iar dac vrsta este mai mare ca 38, atunci dub a t a depaete pe 77. Tatal are 38 ani. s s P.137. Dorin, Oana si Claudia se pregatesc pentru Concursul "Floric pan". Oana a rezolvat 5 probleme. Dorin a rezolvat un numar de proble faa de Oana, egal cu numarul de probleme rezolvate n plus de Oana faa d t t Cte probleme au rezolvat mpreuna cei trei copii? (Clasa a II-a) Inst. Maria R Soluie. Cei trei copii au rezolvat mpreun 5 + (5 + a) + (5 a) = 15 t a P.138. Doi tai si trei fii au mpucat fiecare cte un iepure. Cnd i-au t s au vazut ca au doar patru iepuri. De ce? (Clasa a III-a) Inst. Elena N Soluie. Echipa de vnatori este formata din doi feciori, tatal lor i bu t s doi feciori. P.139. Mutai un singur chibrit pentru a obine o egalitate: t t

(Clasa a III-a) Nicolae Ivchescu as Soluie. Mutm chibritul orizontal de la semnul de operaie plus i ob t a t s

P.140. Descopera regula de completare a jetoanelor 10 11 12 13 14 98 ... 0 1 2 72

99 81

si calculeaza cte numere diferite sunt scrise pe aceste jetoane pe locurile (Clasa a III-a) Lenua Zaharia, e t Soluie. Regula este data de: 1 0 = 0, 1 1 = 1,. . . , 9 9 = 81. t diferite scrise pe locurile de jos sunt generate de produsele: 1 0, 1 1, 1 2, . . . , 1 9 2 5, 2 6, . . . , 2 9 3 5, 3 7, 3 8, 3 9 4 5, 4 7, 4 8, 4 9 5 5, 5 6, . . . , 5 9 6 7, 6 8, 6 9 7 7, 7 8, 7 9 8 8, 8 9 99 n total sunt scrise 37 numere diferite. 10 numere 5 numere 4 numere 4 numere 5 numere 3 numere 3 numere 2 numere 1 numr. a

P.141. Fiul observa ca, atunci cnd i mai trebuia un an pna la vrstei din prezent, tatal avea vrsta de 12 ori mai mare dect a sa, iar c 11 ani, vrsta lui va fi de 4 ori mai mica dect a tatalui. Sa se afle vrst prezent. (Clasa a IV-a) Petru Asa Soluie. Diferena dintre vrsta tatalui i vrsta fiului este 4 11 11 t t s Figurarea marimilor din problema:

(*) vrsta fiului n prezent (**) vrsta fiului cnd mai avea un an p jumatatea vrstei din prezent.

Valoarea unui segment este 33 : 11 = 3 ani. Vrsta fiului n prezent est 2 = 8 ani.

P.142. Paginile unei cari sunt numerotate de la 1 la 336. Din aceas t rup, la ntmplare, 111 foi. Sa se arate ca: a) suma numerelor de pe foile ramase nu se mparte exact la 10; b) produsul numerelor de pe foile ramase se mparte exact la 3. (Clasa a IV-a) Maria Frangoi, e Soluie. a) n total cartea are 336 : 2 = 168 foi. Numrul foilor r t a a 168 111 = 57. Suma numerelor de pe fiecare foaie este un numr imp a unui numar impar de numere impare este un numar impar, deci nu se poa exact la 10. b) n irul 1, 2, 3,. . . , 336 avem 112 numere care se mpart exact la 3 s sunt: 1 3, 2 3, 3 3,. . . , 112 3. Pe paginile celor 111 foi putem ave 111 numere care se mpart exact la 3. Pe paginile ramase vom ntlni ce numar care se mparte exact la 3, deci i produsul numerelor se mparte e s

P.143. Aezai numerele 2, 3, 4, . . . , 10 n patratul alaturat asts t fel nct, pe fiecare linie, suma numerelor din primele doua casete sa fie egala cu numarul din ultima caseta. n cte moduri pot fi aezate aceste s numere? (Clasa a IV-a) Ionela Brgan, e a a Soluie. Suma numerelor de pe primele doua coloane este egala cu t 2 suma numerelor de pe ultima coloana. Suma tuturor numerelor este 4 54, deci suma numerelor de pe ultima coloan este 27. Singura situaie a t 3 care satisface condiia de pe ultima coloan este 8 + 9 + 10 = 27. Un t a exemplu de aezare este prezentat alaturat. Numerele 8, 9 i 10 pot fi s s 3 2 1 = 6 moduri pe ultima coloana. Deoarece 2+6=6+2=8 3+5=5+3=8 4+5=5+4=9 i 2+7=7+2=9 , s 3 + 7 = 7 + 3 = 10 4 + 6 = 6 + 4 = 10 nseamn c avem (8 2) 6 = 96 moduri de aezare a celor 9 numere. a a s

Clasa a V-a

V.81. Demonstrai ca putem completa cu numere naturale ntr-o in t moduri casuele libere din figura de mai jos, astfel nct sa se poata efec t operaiile indicate: t

Amalia Cantemir, e Soluie. t

Not. Explicaia este aceea ca "schema" se poate descompune n ct a t "nule", n sensul ca suma numerelor care sunt scrise pe traseu i care se a s egal cu suma numerelor care se scad: a I II III

V.82. ntr-o ferma sunt gaini, oi si vaci, n total 324 de picioare si impar de capete:

a) Sa se arate ca n ferma nu pot fi 101 gaini. b) Sa se arate ca numarul oilor nu poate fi egal cu numarul vacilor. Petru Asa Soluie. a) Notm cu g, o, v numrul ginilor, oilor, respectiv vacilor. t a a a 2g + 4o + 4v = 324, de unde g + 2 (o + v) = 162, deci g trebuie s fie num a b) Cum g + o + v este numar impar, iar g este par, rezulta ca o + v e impar, prin urmare o i v au n mod necesar paritai diferite. s t

V.84. Determinai cel mai mic si cel mai mare numar natural de 9 t divizibile cu 90 si avnd suma cifrelor 90. Carmen Daniela Tama Soluie. Un numr cu suma cifrelor 90 este oricum divizibil cu 9; p t a divizibil i cu 10, el trebuie sa se termine n 0. Cel mai mare numar cu pr s dorita va fi 99 {z . 9 00 {z . 0, iar cel mai mic 1 00 {z . 0 8 99 {z . 9 0. .. }| .. } .. } | .. } | |
10 80 78 9

V.83. Sa se demonstreze ca 13 | abc daca si numai daca 13 | 3 ab c Otilia Neme, Ocn s Soluie. Avem: t 13 | abc 13 | 10 ab + c 13 | 13 ab 10 ab + c 13 | 3 ab

V.85. Fie a, b N; sa se arate ca daca ultima cifra a numarului a2 + 2 atunci ultima cifra a lui (a + b) este tot 9. Reciproca este adevarata? Ioan Scleanu a a Soluie. Ultima cifra a unui p trat perfect poate fi 0, 1, 4, 5, 6 sau t a U a2 + b2 = 9, obligatoriu U a2 = 0, U b2 = 9 (sau invers) sau U 2 U b = 5 (sau invers). n primul caz , vom avea U (a) = 0, U (b) {
2

invers), deci U (a + b) {3, 7} i atunci U (a + b) = 9. n al doilea caz s U (a) {2, U (b) = 5 (sau invers), deci U (a + b) {3, 7} i din nou o 8}, s 2 U (a + b) = 9. Reciproca este fals; de exemplu, pentru a = 2, b = 1 avem c (a + b) a a a + b2 = 5.
2

V.86. a) Sa se rezolve n numere naturale ecuaia x2 + y 2 = 625. t b) Sa se arate ca ecuaia x2 + y 2 = 2007 nu are soluii n N2 . t t Valerica B Soluie. a) Scdem din 625, pe rnd, fiecare ptrat perfect care nu-l t a a rezultatul este tot patrat perfect n cazurile 625 49 = 576, 625 225 = 400 = 225 i 625576 = 49. Obinem soluiile (x, y) {(7, 24) ; (15, 20) ; (20 s t t b) Un ptrat perfect d la mprirea prin 4 fie restul 0, fie restul 1, a a at y 2 poate fi M4 , M4 + 1 sau M4 + 2. Cum 2007 = M4 + 3, ecuaia dat nu t a n N2 . V.87. Sa se arate ca 751 > 389 .

Nela Cice

Soluia 2 (M. Haivas). Inegalitatea se scrie echivalent t 4 51 7 89 51 89 7 4 > 3 7 (2401) 4 > (2187) 7 , 51 89 ceea ce este evident adevrat deoarece 2401 > 2187, iar a > . 4 7

Soluia 1 (a autoarei). Avem: t 17 = 34317 > 34217 = (9 38)17 > (9 36)17 = 368 234 = 751 = 73 4 4 = 368 22 28 > 368 3 2564 > 369 2434 = 369 35 = 38

Clasa a VI-a

VI.81. Stiind ca 13 | 2a + 3b + 4c + 5d, aratai ca 13 | 43a + 45b + 47 t 13 | 46a + 30b 64c 54d ( a, b, c, d N). Norbert-Traian Ionia, t Soluie. Avem: t 13 | 2 (2a + 3b + 4c + 5d) + 13 (3a + 3b + 3c + 3d) 13 | 43a + 45b + 4 13 | 13 (4a + 3b 4c 3d) 3 (2a + 3b + 4c + 5d) 13 | 46a + 30b 6 VI.82. Fie A = 3m 5n , m, n N. Notam cu a, b, c numarul numerelor A, 3A, respectiv 5A. Stiind ca a si b sunt direct proporionale t iar b si c sunt invers proporionale cu 15 si 16, sa se determine A. t Mihai Ha Soluie. Avem ca a = (m + 1) (n + 1), b = (m + 2) (n + 1), iar c = (m t a b Din = obinem c m = 2, apoi din 15b = 16c deducem c n = 3, pr t a a 3 4 2 3 A = 3 5 = 1125. VI.83. Daca p este numar prim, iar n N , sa se arate ca p4n patrat perfect. Mirela M Not. Domnul Titu Zvonaru atrage atenia asupra faptului c ipotez a t a numar prim nu este importanta, fiind suficient sa consideram p N \ {0 lucru a fost observat n redacie n momentul selectarii problemei spre pub t s-a preferat pstrarea enunului dat de autor. Prezentm mai jos soluiile a t a t Zvonaru. Soluia 1. Urmrim ultima cifr a numrului p4n 3: t a a a U (p) 0 1 2 3 4 5 6 7 8 9 U p4 0 1 6 1 6 5 6 1 6 1 U p4n 0 1 6 1 6 5 6 1 6 1 U p4n 3 7 8 3 8 3 2 3 8 3 8

Deducem astfel ca p4n 3 nu poate fi patrat perfect. 2 2 Soluia 2. Are loc dubla inegalitate p2n 1 < p4n 3 < p2n . n t inegalitatea din dreapta este evident, iar cea din stnga se scrie succesiv a p4n 2 p2n + 1 < p4n 3 2 p2n > 4 p2n > 2,

fapt adevrat pentru p 2. Astfel p4n 3 este strict cuprins ntre dou a perfecte consecutive, prin urmare nu poate fi patrat perfect.

Temistocle B Soluie. a) Restul mparirii unui numar prin 3 este acelai cu restul t s t prin 3 a sumei cifrelor sale. Rezult c oricum am considera trei termeni c a a s a a din An , ei vor fi de forma M3 , M3 + 1 i M3 + 2 (nu neaprat n aceast atunci suma lor se va divide cu 3. . Dac n = 3k, grupnd cte trei termenii lui A , deducem c A . 3. a . a
n n 3k+1

a) 3 | An daca si numai daca 3 - n 1; 9n 10n A2n n < 10n + 1 + b) 10 + 1 + < , n 3. 10 An 11

VI.84. Pentru n N , definim An = 1 + 11 + 111 + + | {z . 1. Ar 11 . . }


n

3k + 1, atunci An = M3 + | {z . 1 = M3 + 1. Dac n = 3k + 2, atu 11 . . } a


3k+1 3k+2

de la a). b) Observam ca

M3 + | {z . 1 + | {z . 1 = M3 + (M3 + 1) + (M3 + 2) = M3 i de aici rezulta 11 . . } 11 . . } s 11 . . } 11 . . } 11 . . } A2n =An + | {z . 1 + + | {z . 1 = An + 10n + | {z . 1 +


n+1 2n n n n n

A2n 11 . . . 1 9 11 . i rmne s artm c Rezult c a a = 1 + 10n + n s a a aa a < An An 10 A n 3. Scrise dezvoltat, aceste inegaliti revin la at 9 + 99 + 999 + + | .{z. 9 < 10 + 100 + 1000 + + 1 | .{z. 0; 9 . } 0 . }
n n n1 n

=An (10n + 1) + n 11 {z . 1 . | .. }
n

11 . . } 11 . . } .. } + 11 10n + | {z . 1 + + 11 {z . 1 10n + | {z . 1 = |

i sunt adevarate pentru n 3. s

11 + 110 + 1100 + + 11 | .{z. 0 < 10 + 110 + 1110 + + | {z . 1 0 . } 11 . . }

VI.85. Pe latura Ox a unghiului drept xOy consideram un punct bisectoarea unghiului consideram un punct B. Perpendiculara n B pe dreapta Oy n C. Sa se arate ca AB = BC. Petru Asaftei, Iai s Soluie. Fie E i F proieciile lui B pe dreptele Ox, t s t \ respectiv Oy. Observam ca ABE CBF (au acelai s \ complement), BE = BF (deoarece OB este bisectoarea \ s lui EOF ) i, conform C.U., rezulta ca 4ABE 4CBF , de unde AB = BC.

VI.86. a) Fie 4ADC si M (AC). Sa se arate ca PADM < PADC . b b) Fie 4ABC si (AD) bisectoarea unghiului A, D BC. Daca PABD

sa se arate ca 4ABC este isoscel. PADM = AD + AM + DM < AD + AM + (DC + M C) = = AD + DC + (AM + M C) = PADC . Soluie. a) Folosind inegalitatea triunghiului n 4CDM , t

Gheorghe Iu

b) Sa presupunem prin absurd ca AB 6= AC; sa zicem ca AB < AC i fie M (AC) astfel nct AM = AB. Avem ca s 4ABD 4AM D (L.U.L), deci PABD = PADM < PACD i s astfel am ajuns la o contradicie. t VI.87. n figura alaturata sunt desenate 6 puncte, care unite doua cte doua dau natere la 9 drepte. Avem voie sa stergem unul dintre s puncte si sa-l desenam oriunde n alta parte. a) Efectuai operaia descrisa astfel nct, prin unirea cte doua a t t noilor puncte, sa se obina 11 drepte. t b) Care este numarul minim si cel maxim de drepte care se pot ob configuraie permisa? t Gabriel P Soluie. a) De exemplu, putem proceda ca n figura 1. t

Fig. 1

Fig. 2

Fig. 3

b) Numrul minim de drepte este 8, deoarece nu vom putea face n a a s sa obinem mai mult de 4 puncte coliniare; a se vedea figura 2. t Numrul maxim de drepte este 13: numrul maxim posibil de drepte pri a a 65 este = 15 i el nu poate fi atins dect dac oricare trei puncte sunt n s a 2 La nlocuirea a trei puncte necoliniare cu trei puncte coliniare, numrul a scade cu 2 i atunci maximul posibil este 13, atins de exemplu n config s figura 3.

Clasa a VII-a

VII.81. Se considera abc si xyzt numere naturale scrise n baza r q q p p compare numerele naturale A = 6 2 abc si B = 3 2 xyzt. Bogdan Chiriac, stud q p p . Soluie. Fie 6 p abc = k N; atunci 6 2 abc = k 2 , iar k2 . 6, pr t . 2 k = 6l. Rezult c 2 abc = 6l2 , deci abc = 18l4 , de unde abc = 3 a a 100 324l8 999 obinem c l = 1, adic A = 6. Cu un raionament as t a a t gasim ca B = 6 i astfel A = B. s VII.82. Fie a, b numere reale strict pozitive. Sa se arate ca:

Ionel Nech Soluie. Mai nti, s observm c nu putem avea a = b, altfel din amb t a a a ar rezulta ca a = b = 0. Avem: a3 b3 a3 + b3 a) =1 > 1 a2 ab + b2 > 1 a2 + b2 > 1; a+b a+b a3 + b3 a3 b3 b) =1 < 1 a2 + ab + b2 < 1 a2 + b2 < 1. ab ab VII.83. Determinai numerele ntregi a, b, c, d pentru care ac + bd t ad + bc = 2. Gheorghe Iu Soluie. Scznd membru cu membru relaiile din ipotez, deducem c t a t a ac bd = 1, deci (a b) (d c) = 1, de unde a b = d c = 1 sau a b = d n primul caz, substituind a = b + 1 i d = c + 1 n prima dintre relai s t avem succesiv: ac + bd = 1 c (b + 1) + b (c + 1) = 1 2bc + b + c = 1 4bc + 2b + 2c + 1 = 3 (2b + 1) (2c + 1) = 3 (b, c) {(1, 0) ; (0, 1) ; (1, 2) ; (2, 1)} .

a) daca a3 b3 = a + b, atunci a2 + b2 > 1; b) daca a3 + b3 = a b, atunci a2 + b2 < 1.

Obinem soluiile (a, b, c, d) {(1, 2, 1, 0) ; (0, 1, 2, 1) ; (1, 0, 1, 2) ; t t Similar, n al doilea caz gasim soluiile t

(a, b, c, d) {(2, 1, 0, 1) ; (1, 0, 1, 2) ; (0, 1, 2, 1) ; (1, 0, 1, 2)

VII.84. Fie patratul ABCD cu latura de lungime a, iar E, F , G a a laturile [BC], [CD], respectiv [AB] astfel nct CE = , CF = , iar BG 4 3 se arate ca dreptele AE, BF si CG sunt concurente. Claudiu Stefan Popa, Iai s Soluie. Fie {P } = AC BF ; din asemanarea 4CF P t CP CF 1 CP AG 4ABP , deducem ca = = . Atunci PA AB 3 P A GB 1 1 3 BE = = 1 i din reciproca teoremei lui Ceva urmeaza s EC 3 1 1 concluzia.

VII.85. Fie O intersecia diagonalelor patrulaterului ABCD. Daca t CD AB AABC = ACOD , sa se arate ca = 1. AB CD Doru Bu Soluie. Notam S1 = AAOD , S2 = AAOB , S = ADOC . t Cum AABD = AABC , rezulta ca ABCD este trapez cu AB k CD, prin urmare ABOC = S1 . Din ipotez vom a CD i atunci, cum a avea c S = S1 + S2 . Notm c = a s AB S = c2 . Pe de alt parte, a 4OAB 4OCD, deducem c a S2

2 OD dist (A, BD) S1 S S1 = = , prin urmare S1 = S = c i astfel s S2 OB dist (A, BD) S2 Sr 2 S S obinut c S = S S2 + S2 , deci t a = 1+ , adic c2 = 1 + c. R a S2 S2 1 c = 1 + (c este tocmai numarul de aur ), de unde concluzia problemei. c VII.86. Fie A un punct pe manta unei mese de biliard circulare cu ra O bila pleaca din A si ajunge napoi n A lovind manta de cel puin trei o t bilei se face considernd ca aceasta lovete un perete plan tangent la cerc s de contact. Sa se arate ca exista o infinitate de traiectorii posibile si sa se traiectoria de lungime minima. Cristian L Soluie. Considernd un poligon regulat cu n laturi, n 4, nscris t dat, vrfurile acestuia pot fi punctele de contact cu manta ale bilei ntr-o dorita; poligonul poate fi chiar unul stelat! Lungimea minima a traiectorie n cazul triunghiului echilateral nscris n cerc, al carui perimetru va fi 3

VII.87. O tabla are forma unui dreptunghi 4 5, format din 20 de 1 1. Avem la dispoziie doua jetoane, fiecare putnd acoperi cte un pa t cte moduri putem aeza jetoanele pe tabla, astfel nct ele sa nu se a s aceeai linie, nici pe aceeai coloana? Generalizare. s s Gabriel P Soluie. Numaram nti n cte moduri putem aeza jetoanele pe ta t s sena restriciei din enun. Dac jetoanele ar fi numerotate, ar exista 2019 t t t a 20 19 de aezare a lor; cum nu conteaz ordinea, avem s a = 190 modaliti d at 2 Dintre acestea, 10 conin cele dou jetoane pe prima linie, 10 pe a doua t a deci 40 de aezri au jetoanele pe aceeai linie. Apoi, exist 6 aezri cu je s a s a s a prima coloana etc., deci 6 5 = 30 de aezari cu jetoanele pe aceeai coloa s s n final, avem 190 40 30 = 120 aezari ce verifica enunul. s t Generalizare. n cazul unei table m n, avem

mn (mn 1) mn (n 1) mn (m 1) mn = (m 1) (n 1 2 2 2 2 modalitai de aezare. s t

Clasa a VIII-a

VIII.81. Consideram fixate numerele a, b Z , m, n N , m 6= n si f : N Z, f (x) = ax+b. Daca f (1)+f (2)+ +f (m) = f (1)+f (2)+ sa se calculeze suma S = f (1) + f (2) + + f (m + n). Dan Nedeianu, Dr.T 2 m2 + m a n + Soluie. Din relaia din ipoteza deducem ca t t +mb = 2 2 Trecem totul ntr-un membru i simplificam prin mn 6= 0; rezulta ca a (m s

2b = 0. Atunci f (1) + f (2) + + f (m + n) =

(m + n) (m + n + 1) a + (m + n) 2 (m + n) (2b) = + (m + n) b = 0. 2 VIII.82. Sa se arate ca |3xy + x + y| 1, x, y [0, 1]. Ovidiu Pop, Sa Soluie. Avem de artat c 1 3xy + x + y 1, x, y [0, t a a inegalitate este echivalent cu 0 x (1 y) + y (1 x) + (1 xy), adevr a a x, y [0, 1] (toi cei trei termeni sunt pozitivi). A doua inegalitate este e t cu 0 (1 x) (1 y) + 2xy, din nou adevarata pentru x, y [0, 1]. VIII.83. Sa se arate ca nu exista x, y Z pentru care 147x2 = 1 + 4y h Mihai Crciun i a 2 2 Soluia 1. Scriem relaia din enun sub forma 3 (7x) + y = 4y + t t t patrat perfect este sau M4 , sau M4 + 1; pentru x, y Z, paranteza patrat M4 + 1 sau M4 + 2 i atunci membrul stng este M4 , M4 + 3 sau M4 + s membrul drept este M4 + 1, urmeaz concluzia. a Soluia 2 (Ioan Stanciu, elev, Craiova). Daca exista x, y cu pr t cerute, n mod necesar 1 + 4y 3y 2 0. Deducem ca y {0, 1}, valori p x nu este ntreg. VIII.84. Laturile a, b, c ale unui triunghi verifica egalitatea 2 a8 + 4 2 a + b4 + c4 . Sa se arate ca triunghiul este dreptunghic. Corina Elena Vian s Soluie. Avem succesiv: t

i de aici concluzia. s VIII.85. Daca a, b, c sunt numere reale pozitive, sa se arate ca q p p 1 2 3 a2 + b2 ab + b2 + c2 bc 3 = a2 + c2 + = . a c b Liviu Smarandache Soluie. Fie [OA], [OB], [OC] trei segmente de lungimi t \ \ a, b, respectiv c, astfel nct m(AOB) = 60 , iar m(BOC) = 30 . Avem: q p p a2 + b2 ab + b2 + c2 bc 3 = a2 + c2 p p a2 + b2 2ab cos 60 + b2 + c2 2bc cos 30 = p = a2 + c2 AB + BC = AC A, B, C coliniare

a8 + b8 + c8 2a4 b4 2b4 c4 2c4 a4 = 0 2 a4 + b4 c4 4a4 b4 = 0 a4 + b4 + 2a2 b2 c4 a4 + b4 2a2 b2 c 2 2 a2 + b2 c4 a2 b2 c4 = 0 a2 + b2 + c2 a2 + b2 c2 a2 b2 + c2 a2 b2 c2 = 0

VIII.87. Consideram prisma triunghiulara regulata ABCA0 B 0 C AM CN A0 M 0 C 0 N 0 , unde M este punct interior triunghiului ABC. Fie F 0 mijloacele muchiilor [AB], [BC], [A0 B 0 ], respectiv [A0 C 0 ]. a) Aflai masura unghiului dintre dreptele EF 0 si E 0 F . t b) Aflai masura unghiului format de planele (M CC 0 ) si (ECC 0 ). t Claudiu Stefan P Soluie. a) Fie l = AB, h = AA0 ; cum AC = AM 2 t i AM = AA0 = h, deducem c l = h 2. n 4A0 B 0 C 0 echis a l 3 h 6 = , iar din 4CC 0 F dreplateral, avem c C 0 E 0 = a 2 2 h 6 tunghic obinem ca C 0 F = t . Se observa uor ca EF C 0 F 0 s 2 0 0 0 0 este paralelogram, deci EF k F C , EF = F C . Rezulta ca \ \ m(EF 0 , E 0 F ) = m(F C 0 , E 0 F ) = C 0 F E 0 ). Din motive de m( \ h 6 i astfel 4F C 0 E 0 este echilateral, pr simetrie, E 0 F = EF 0 = F C 0 = s 2 msura unghiului dorit este de 60 . a 0C0M 0C0M b) Unghiul planelor este E\ 0 , a carui masura este m(A\ 0 ) m( 45 30 = 15 .

AAOB + ABOC = AAOC ab sin 60 1 bc sin 30 ac 3 + = ab 3 + bc = 2ac + = 2 2 2 a c VIII.86. O piramida hexagonala regulata V ABCDEF are muchia baz cm si nalimea V O = 4 2 cm. Fie M mijlocul lui V D, {P } = AD t {Q} = P M (V CF ). Sa se arate ca: a) dreptele V P si DQ sunt concurente; b) DQ (V BF ). Gabriel P Soluie. a) Cum (V AD) (V CF ) = V O, P M (V AD), t iar {Q} = P M (V CF ), nseamna ca Q V O. Astfel dreptele V P i DQ sunt ambele incluse n planul (V AD), s cu Q Int V P D i de aici urmeaz concurena dorit. s a t a b) Observm c BF AD i BF V O (V O (ABC)), a a s de unde BF (V AD), prin urmare BF DQ. Vom mai arata ca DQ V P i atunci va rezulta ca DQ (V BF ). s Avem c P O = 2 cm, deci V P = V O2 + OP 2 = 6 cm, iar a P D = P O + OD = 6 cm. Deducem c 4P DV este isoscel, a iar mediana bazei P M va fi i nalime. Astfel, Q va fi ortocentru n 4V s t urmare DQ V P .

Clasa a IX-a

IX.81. Fie a, b R. Daca ecuaia x2 +ax+b+2 = 0 are ambele radac t aratai ca numarul 2a2 + b2 este natural compus. t Dorin Mrghidanu, a Soluie. Daca x1 , x2 Z sunt soluiile ecuaiei date, atunci a = (x t t t

i concluzia se impune. s

b = x1 x2 2, de unde rezult c a, b Z, deci 2a2 + b2 N. n plus, a a 2 2 2 2 2a + b = 2 (x1 + x2 ) + (x1 x2 2) = x2 + 2 x2 + 2 1 2

Not. ntr-o forma puin modificata, problema a aparut n RMT 2 a t numrul IX.216, sub semntura aceluiai autor. a a s IX.82. Determinai funciile f : R pentrucare t t R f x4 + y 3 + z 2 + t = f (x) + f y 2 + f z 3 + f t4 ,

Lucian Tuescu i Liviu Smarandache t s Solu tie. Pentru x = z = t = 0, gsim f (0) = 0. Dac y = z =y a a = a a f x4 + t = f (x) + f t4 , x, t R. Pentru t = x4 , gsim c f x16 4 16 4 x R. Cum f x = f (x), x R, deducem c f x = a = f x4 f (x), x R. Prin urmare f (x) = f (x), x R, adic f (x) = 0, x a IX.83. Pentru a 9, sa se demonstreze ca are loc inegalitatea q q 3 + 3a + 9 1 + 1 + a.

x, y, z, t

Marian Tetiv Soluie. Pentru a = 9 avem egalitate; sa aratam ca are loc strict in t din enun pentru a > 9. Prin ridicare la patrat i cu notaia a = x2 , x > 3 t s t succesiv: q p 1 + 3a + 9 > 2 1 + a + a 1 + 3x2 + 9 > 2 1 + x + x p 2x (x 3) 2 (x 3x2 + 9 (x + 3) 2 1 + x 2 > 1+x 3x2 + 9 + x + 3 p x 1 x 1 + x + 1 > 3x2 + 9 > 1+x+2 3x2 + 9 + x + 3 Aceasta din urma inegalitate rezulta adunnd x > 3 i x 1 + x > 3x2 s revine la x3 + x2 > 3x2 + 9, evident pentru x > 3).

IX.84. Fie ABC un triunghi. Determinai numerele ntregi a, b, c nen t ntre ele doua cte doua, astfel nct punctele M , N , P sa fie coliniare, un P sunt determinate prin condiiile AM = aAB; CN = bCA; CP = cBC t Ioan Scleanu a a Soluie. Exprimnd vectorii N P i M N n funcie de AC i BA, o t s t s N P = (b + c) AC + cBA i M N = (1 b) AC aBA. Deoarece punctele s sunt coliniare, rezult c vectorii N P i M N sunt coliniari; folosind rela a a s dente, gasim condiia ab + ac + bc = c. De aici rezulta ca c | ab i cum c t s cu a i b, deducem c c {1, 1}. Pentru c = 1 gsim soluiile a = s a a t sau a = 2, b = 3. Pentru c = 1 gsim soluiile a = 1, b Z (n a t M = P = B) sau a Z, b = 1 (n acest caz N = A i P = B). s

IX.85. Fie ABC un triunghi ascuitunghic si D = prBC A, E = prC t prAB C. Demonstrai echivalena afirmaiilor urmatoare: t t t (i) 4ABC este isoscel;

care, dupa transformari, este echivalenta cu (a b) (b c) (c a) (a + b + Prin urmare, (iii) este echivalent cu (i). a Analog se arat c (ii) este echivalent cu (a b) (b c) (c a) (a + b a a a deci cu (i). n cazul n care triunghiul este obtuzunghic i A este unghiul obtuz, av s b lena condiiilor: t t (j) 4ABC este isoscel de vrf A, (jj) DB + EC F A = DC EA + F B, 1 1 1 1 1 1 (jjj) + = + . DB EC FA DC EA F B Daca 4ABC este dreptunghic n A, atunci condiia (iii) nu se mai poat t iar (ii) devine DB + b = DC + c, care este echivalent cu faptul c 4 a a dreptunghic i isoscel de vrf A. s

(ii) DB + EC + F A = DC + EA + F B; 1 1 1 1 1 1 (iii) + + = + + . DB EC FA DC EA F B Examinai cazurile n care 4ABC este obtuzunghic sau dreptunghic. t Temistocle B Soluie. Implicaiile (i) (ii) i (i) (iii) sunt triviale. Pentru i t t s inverse acestora, utilizm relaiile BD = c cos B, DC = b cos C etc. a t cosinusului. Astfel, avem: X X 1 X X 1 2a 2a = = (iii) 2 c2 a2 2 a2 c cos B b cos C b c

Clasa a X-a
X.81. Sa se rezolve n R R R sistemul x y 2/3 = z 1/3 ; x4/3 y = z 2/3 ; z 5/3 y 4/3 = z.

Vasile Chiria Soluie. Notnd x1/3 = t, y 1/3 = u, z 1/3 = v, sistemul devine t3 t 2 t4 u3 = v 2 ; t5 u4 = v 3 . Avem c t8 = u3 + v 2 = u2 + v u4 + v 3 a gsim c uv (u v)2 = 0. Prin urmare, u = 0 sau v = 0 sau u = v. Analiz a a cazuri, gasim tiile u, v) : (1, 0, 1); (1, 0, 1); 0, 0); (1, 1, 0); ( solu (t, (0, 1 + 5 1 + 5 1 + 5 1 5 1 5 1 5 i , , s , , . Corespunzato 2 2 2 2 2 2 soluiile (x, y, z) ale sistemului dat. t X.82. Solve the equation aex + b ex 3 = ax3 + b x3 2 + a, a > b > 0.

Considernd funcia f : R R, f (t) = at + b |t 1|, ecuaia devine f (e t t f x3 1 . Se verific uor c f este strict cresctoare, deci injectiv. Ecu a s a a a se reduce la ex 2 = x3 1, adica x3 ex + 1 = 0. Cum funcia g t

Zdravko Starc, Vra Soluie. Scriem ecuaia sub forma t t a ex 2 + b ex 3 = a x3 1 + b x3 2 .

g (x) = x3 ex + 1 este strict cresctoare i g (0) = 0, concluzionm a s a x3 ex + 1 = 0 are soluia unica x = 0. Prin urmare, ecuaia data are so t t x = 0. X.83. n exteriorul triunghiului ABC se construiesc triunghiurile isosc \ AN C si CP B de baze AB, AC si respectiv BC, astfel nct m(M AB \ = 45 , iar m(P BC) = 30 . Sa se arate ca m(M P N ) = 60 . \ \ m(N AC) a Angela Tigeru Soluie. Vom nota afixul fiecrui punct cu litera mic t a a ce i corespunde. Deoarece M A se obine din M B n urma t 5 unei rotaii n jurul lui M de unghi t , avem c a m = a 6 5 2a+b 3bi 5 . Analog , de unde m = (bm) cos +i sin 6 6 2+ 3i 2b + c c 3i gasim afixele punctelor P i N , anume p = s , 3 3i c ai mp 3 1 cos + i sin respectiv n = . Prin calcule, = 1i np 3 3 concluzia problemei. X.84. Fie ABC un triunghi n care (tg B 1) (tg C 1) = 2. Daca sunt picioarele nalimilor din B, respectiv C, sa se arate ca segmentele t si M N se pot constitui n laturi ale unui triunghi. Ctlin Cali a a Soluie. Problema este nrudit cu VI.30, publicat de acelai autor t a a s b 1/2002. Ca i acolo, cheia rezolvarii este aceea de a arata ca m(A) = 4 s triunghiurile ABM i ACN vor fi dreptunghice isoscele, cu BM = AM i C s s deci segmentele din enun se constituie n laturi ale 4AM N . t Deoarece tg A + tg B + tg C = tg A tg B tg C (identitate cunoscut), a ipoteza este echivalenta cu 1+tg B+tg C = tg B tg C, i.e. (tg A 1) (tg B t 0. Se observ uor c al doilea factor nu se poate anula, deci rmne c a s a a a b adic m(A) = 45 . a X.85. Se prelungete diametrul [M N ] al unui cerc C cu segmentul [N s ent cu [M N ]. Fie d perpendiculara n P pe M N si R d, oarecare. T duse prin R la C intersecteaza tangenta n M la C n S si T . Sa se arate de greutate al 4RST este un punct fix. Adrian Reisn Soluie. Raportm planul la un reper ortogonal cu origit a nea n M , avnd dreapta M N ca ax Ox i tangenta n M a s a la C drept ax Oy. Fie a raza cercului C; atunci M (0, 0), 2 N (a, 0), P (2a, 0), iar R (2a, ), cu variabil i fie S (0, s). s Ecuaia dreptei RS este ( s) x 2ay + 2as = 0 i, t s a impunnd condiia c d (O, RS) = , obinem: t a t 2 a q ( s) + 2as a 2 q = | + 3s| = ( s)2 + 4a2 2s2 + 2s a 2 ( s)2 + 4a2

Dac T (0, t), se obine pentru t ecuaia 2t2 + 2t a2 = 0. Din relaiile lui a t t t 2a + 0 + 0 ducem c t + s = . Centrul de greutate al 4RST este G a , 3 2a i.e. G , 0 , deci este un punct fix. 3

Clasa a XI-a

{x} XI.81. Daca m Z, sa se studieze existena limitei lim t . xm sin x Dan Popescu [x] [x] Soluie. Observm c sin x = (1) sin (x [x]) = (1) sin t a a m (1) 1 {x} {x} = = xm lim . Pentru calcu R. Atunci xm lim [x] x>m sin x x>m sin {x} (1) {x} 1 la stnga, dac m = 2k, k Z, obinem c lim a t a = = x2k sin x 0
x<2k

1 {x} = = +. n concluzie, limit m = 2k + 1, k Z, atunci lim x2k sin x 0+ x<2k exista pentru nicio valoare a lui m Z.

XI.82. Considerul sirul (an )nN , definit prin a0 = 0, a1 = 1 si an+2 3 5 , aratai ca sirul xn = a0 + a1 x + a2 an , n N. Pentru x 0, t 2 n an x , n N, este convergent si calculai limita sa. t Vlad Emanuel, el Soluie. Folosind ecuaia caracteristic pentru irul (an ), stabilim t t # a s " " n n 1 3 5 3 3+ 5 , n N. Rezult c an 0, a a 2 2 5 n N. Evident ca irul (xn ) este strict crescator i cum s s n 3+ 5 1 3+ 5 , 0 xn < 1 + xn < x + + 2 2 1 x 3+2 5
n

(xn ) este marginit. n concluzie, (xn ) este convergent. Fie l = lim xn

xn+2 3xxn+1 + x2 xn = a0 + (a1 3a0 ) x = x, prin trecere la limita x l= 2 . x 3x + 1 XI.83. Fie f : [0, ) [0, ) pentru care are loc relaia f (f (x)) + 9x t x [0, ). Aratai ca f (x) 3x, x [0, ). t Bogdan Posa i Marius Drgoi, elev s a 9 s Soluie. Din ipotez deducem c f (6x) 9x, de unde f (x) x i f t a a 9 2 6 2 +9 9 x, x [0, ). Atunci, conform ipotezei, f (x) 6 x, x 6 6 9 Prin inducie se arat c f (x) un x, x 0, n N , unde u1 = , i t a a 6

u2 + 9 n s . Deoarece (un ) este convergent i lim un = 3, deducem ca f (x) n 6 deci f (x) 3x, x [0, ). XI.84. Determinai numerele a R pentru care exista o funcie t t f : R R astfel nct (f f ) (x) = a2 f (x) 2a4 x, x R. Andrei Ned Soluie. Pentru a = 0 exist f : R R, f (x) = 0 care veric t a Vom arata ca nu exista astfel de funcii pentru a 6= 0. Presupunnd cont t ipoteza se obine ca f este injectiva, prin urmare f va fi strict monotona. t f f este strict cresctoare; din x < y deducem c (f f ) (x) < (f f ) (y a a 2a4 (y x) < a2 (f (y) f (x)) i de aici deducem c f este strict cresctoa s a a mare, i funcia f f f este strict crescatoare. Cum (f f f ) (x) = a4 f s t x R, n membrul stng am avea o funcie strict crescatoare, iar n c t funcie strict descresctoare. n concluzie, rmne c a = 0. t a a a XI.85. Fie A = (aij )20072007 o matrice patratica n care aij i, j = 1, 2007. Sa se arate ca determinantul matricei 2008I2007 + A este Paul Georgescu i Gabriel P s Soluie. Fie B = 2008I2007 + A; atunci bii = 2008 + aii , iar bij = t i 6= j. Observam ca 2007 X |bij | , i = 1, 2007. |bii | = |2008 + aii | 2007 > 2006
j=1,j6=i

Relaiile (1) i (2) intr n contradicie, prin urmare rmne c det B 6= 0 t s a t a a

Presupunem prin absurd ca det B = 0; atunci exista c1 , c2 , . . . , c2007 nuli, astfel nct c1 C1 + c2 C2 + + c2007 C2007 = O, undeC1 , C2 , . . . , 2007 sunt coloanele lui B. Fie k 1, 2007 pentru c C max |ci | ; i = 2007 ; atunci c1 c2 ck1 ck+1 c2007 Ck = C1 C2 Ck1 Ck+1 C20 ck ck ck ck ck c1 c2 ck1 ck+1 c2007 bkk = bk1 bk2 bk,k1 bk,k+1 ck ck ck ck ck De aici deducem ca 2007 2007 2007 X cj X X cj |bkj | bkj |bkj | . |bkk | = ck j=1,j6=i ck j=1,j6=i j=1,j6=i

Clasa a XII-a

XII.81. Dintre toate parabolele y = ax2 + bx + c, sa se determine trece prin punctele A (0, 1), B (1, 2), satisface condiia y 0 pentru 0 t si realizeaza minimul ariei determinata de graficul parabolei, Ox si drept respectiv x = 1. Adrian Cordune Soluie. Din condiiile y (0) = 1 i y (1) = 2, gsim y = ax2 + (1 t t s a cu a R . Cum y (0) = 1 > 0, y (1) = 2 > 0, condiia y 0 pentru x t

scrie sub forma

obinem ca A este minima pentru a maxim, deci a = 3 + 2 2, iar Amin = t

1+x > a, x (0, 1), deci a sup h (x), unde h : ( x x2 x(0,1) 1+x . Gsim cu uurina a 3 + 2 2, prin urmare y = ax2 + (1 a s t h (x) = x x2 1 R a t a , 3 + 2 2 \ {0}. Cum aria cerut n enun este A = y (x) dx
0

XII.82. Determinai primitivele funciei f : (1, ) R, t t 4 3 x 5x + 3 (ln x 1) . f (x) = 3 (x4 1) Dan Nedeianu, Dr. Tr Soluie. Cu substituia x4 1 = t, avem de calculat t t Z 1 (5t + 8) (ln (t + 1) 4) dt = 16 t3 Z Z 1 5t + 8 5t + 8 = dt + ln (t + 1) dt . 4 16 t3 t3 Calculnd a doua integrala prin pari i nlocuind t cu x4 1, gasim ca t s Z 4 8 4 x + 3x 1 x ln x + ln x4 1 + C. f (x) dx = 4 1)2 4 1)2 16 (x 4 (x XII.83. Sa se determine funciile continue f : R R pentru care t Z x et f (x t) dt, x R. f (x) = |x| +
0

Dumitru Mihalach Soluie. Cu schimbarea de variabil x t = u, aducem ecuaia f t a t x x R x+u R u la forma f (x) = |x| + e f (u) du. Notnd e f (u) du = F (x), d
0 0

XII.84. Fie polinomul f Z [X], f = a0 X 2n+1 + a1 X 2n + + a2n pentru care n este impar, a0 a2n+1 este impar, iar a1 a2 este par. Sa se arat f are toate radacinile reale, cel puin una este iraionala. t t Mihai Ha Soluie. Dac x1 , x2 , . . . , x2n+1 ar fi rdcini raionale ale lui f , a t a a a t a0 xi , i = 1, 2n + 1 vor fi radacini raionale ale polinomului g = Y 2n+1 + t

s t ex f (x) = F 0 (x), x R i ecuaia devine ex F 0 (x) ex F (x) = |x|, x ( 2 x , x < 0 0 x 2 ,op nc (e F (x)) = |x|, x R. Considernd G (x) = a 2 x 2 ,x 0 funciei g (x) = |x|, x R, obinem c ex F (x) = G (x) + C, C constant t t a ( 2 x , x < 0 0 x 2 + C. Cum f (0) = 0 f (x) = F (x) e , deci f (x) = |x| + x2 2 ,x 0 ( 2 x x , x < 0 2 C = 0 i atunci f (x) = s , funcie care verifica ecuaia d t t 2 x + x ,x 0 2

a0 a2 Y 2n1 + + a2n a2n+1 . Rezult c yi Z i yi | a2n a2n+1 , i = a a s 0 0 Cum a0 a2n+1 este impar, atunci a2n a2n+1 este impar, deci yi , i = 1, 2 0 2n+1 P numere impare. Din prima relaie Vite, t yi = a1 , rezulta ca a1 e i=1 P t Apoi, din i<j yi yj = a0 a2 , obinem ca a0 a2 este numar impar, fiind 2 C2n+1 = n (2n + 1) termeni impari, iar n fiind impar. Deducem ca a2 est astfel produsul a1 a2 va fi impar, contradicie. Rezult c f are cel puin t a a t iraional. t a XII.85. Fie n N, n 2. Aratai ca exista P Z [X] de grad n, a t toate mulimile Ak = {P (i) (mod k) | i Z}, k N, k 2, sa aiba cardin t mai mic dect k. Vlad Emanuel, el Soluie (Gheorghe Iurea). Deoarece P (mk + i) = P (i) (mod k) t i k N, k 2, i = 1, k 1, mulimea Ak conine, cel mult, elemen s t t P (1), . . . , P (k 1) (mod k). Trebuie determinat un polinom P Z [X] a printre elementele enumerate mai sus, cel puin doua sa fie egale (pentr t Polinomul P = X n X, verific cerinele date, ntruct P (0) = P (1), k a t

pe cnd cel minim este 24: 2008 = 125 + + 125 + 125 + + 125 : 125. | {z } | {z }
16 termeni 8 termeni

Rspunsuri (la recreaiile de la pag. 142) a t 1. Numarul maxim cerut este 4015: 2008 = | : 125 + 125 : 125 + + 125 : 125, 125 {z }
2008 termeni

2.

5 + 5!! : 5 = 8 p (6 + 6)!! : 6! = 8 7+7:7=8 8+88=8 99:9=8 (Pentru orice n N , factorialul se denete prin n! = 1 2 3 n, ia s torialul astfel: (2n)!! = 2 4 6 (2n) i (2n 1)!! = 1 3 5 (2n 1).) s

32 3 : 3 = 8 4+ 4 4=8

(1 + 1 : 1)3 = 8 222=8

Soluiile problemelor pentru pregtirea concur t a din nr. 2/2007


A. Nivel gimnazial

D. M. Btineu-Giurgiu, B a t Soluie. Observam ca produsul (a + b + c) (x + y + z + t) este, dupa t parantezelor, tocmai suma numitorilor din membrul stng ai inegalitii at strat. Notm acest membru stng cu S; folosind inegalitatea dintre media a i cea armonica, obinem s t 16 (a + b + c) (x + y + z + t) S 16 S (a + b + c) (x + y + z + t r r a2 + b2 + c2 x + y + z + t x2 + y 2 + z 2 + t2 a+b+c i s (in nsa 3 3 4 4 dintre media aritmetica i cea patratica) i atunci concluzia problemei urm s s litatea se atinge cnd a = b = c i x = y = z = t. s G128. Fie a, b, c numere reale pozitive astfel nct abc = 1 si fie t se arate ca b c 3 a + 2 + 2 . 2+t a b +t c +t t+1 Titu Zvonaru, Comneti i Bogdan Ionia, B a s s t Soluie. Deoarece abc = 1, exist numerele reale pozitive x, y, z a t a x y z a = , b = , c = . Avem ca y z x x xy xy x2 a = = 2 = 2 2+t 2 a x + ty 2xy + (t 1) y 2x + (t 1) y 2x + (t 1) 2 2 b c x y z a + + 2 + + 2 a + t b2 + t c2 + t 2x + (t1) xy 2y 2 + (t1) yz 2z 2 + (t (x + y + z)2 . 2 (x2 + y 2 + z 2 ) + (t 1) (xy + yz + zx)

G126. Sa se determine numerele naturale care au proprietatea ca med trica a tuturor divizorilor lor este un numar natural. Petru M Soluie. Fie 1 = d1 < d2 < < dk = n divizorii lui n; atunc t n ..., este irul acelorai divizori, scris descrescator, prin urmare d1 d s s dk nk 2 , de unde (d1 d2 dk ) = nk . Media geometric a tuturor div a d1 d2 k d n este k d1 d2 dk = n i este numr natural dac i numai dac n e s a a s a perfect. G127. Daca a, b, c, x, y, z, t sunt numere reale pozitive, sa se de inegalitatea 1 1 1 1 8 p + + + ax+by+cz ay+bz+ct az+bt+cx at+bx+cy 3 a2 +b2 +c2 x2 +

Prin urmare, este suficient s demonstrm c a a a 3 (x + y + z)2 . 2 (x2 + y 2 + z 2 ) + (t 1) (xy + yz + zx) t+1 Dup efectuarea calculelor, acesta se reduce la a (5 t) x2 + y 2 + z 2 xy yz zx 0,

evident adevrat. a a

Not. Pentru t = 2 se obine o problem propus la concursul Baltic a t a a 1 1 = {xy} + G129. Sa se determine y R pentru care {x} + x + y y (Cu {} am notat partea fracionara.) t Alexandru Negrescu, elev, 1 2 2 1 1 Soluie. Pentru x = obinem c t t a + = , deci = y y y y y 2 1 2 1 1 [0, 1), iar Z, deducem c a = = 0, prin urmare y y y y y 2 2 2 N. Astfel, (0, 2) N, deci = 1 i atunci y = 2. s y y y 1 = {2x} + Pentru y = 2, egalitatea din enun devine {x} + x + t 2 R, iar aceasta este adevarata ntruct revine la cunoscuta identitate a lu 1 [x] + x + = [2x], x R. 2

de unde a > 2c b + c, ceea ce contrazice inegalitatea triunghiului. procedeaza daca am presupune ca c a.

G130. Fie a, b, c lungimile laturilor unui triunghi ABC. Daca a2007 2007 b 2 + 1 c2007 , sa se arate ca unghiul C este ascuit. t Lucian Tuescu t Soluie. Vom arata ca c este cea mai mica latura a triunghiului, de unde t este imediat. S presupunem prin absurd c c b; atunci a a a 2007 a2007 + c2007 a2007 + b2007 > 22007 + 1 c2007 a2007 > (2c)

G131. Fie n, k 2 numere naturale si mulimea M = { (n 1) , . . . , t 2, . . . , n}. Sa se arate ca M se poate partiiona n k submulimi avnd fiec t t suma a elementelor daca si numai daca n se divide cu k. Marian Tetiv Soluie. Condiia este necesara: daca M admite o partiie ca n enu t t t suma elementelor sale (care este n) va fi egal cu sk (s fiind suma eleme a fiecare clas a partiiei). a t Pentru a demonstra suficiena, vom construi efectiv o partiie n caz t t

i este evident ca M = M1 M2 Mk , Mi Mj = , i 6= j, iar suma e s oricrei mulimi Mi este s. a t

M1 = {2, 3, . . . , s, s + 1, 1, 2, . . . , (s 1) , s} ; M2 = {s + 2, . . . , 2s, 2s + 1, (s + 1) , . . . , (2s 1) , 2s} ; . . . . . . Mk1 = {(k 2) s + 2, . . . , (k 1) s, (k 1) s + 1, ((k 2) s + 1) , . . . , Mk = {1, (k 1) s + 2, . . . , ks, ((k 1) s + 1) , . . . , (ks 1)}

n = ks, cu s N. Considerm mulimile: a t

G132. n fiecare cmp unitate al unei livezi m n se afla cte un numar de k arici pornesc, pe rnd, din cmpul stnga-sus al livezii si se cmpul din dreapta-jos. La fiecare micare, un arici se poate deplasa cu s spre dreapta sau n jos, fara a iei din livada. Ariciul poate sa culeaga s cmpul pe care l viziteaza, daca nu a fost cules deja de alt arici. Care est minim k, pentru care k arici pot sa culeaga toate merele? Iurie Boreico, elev, Soluie. Numerotm cmpurile (x, y), cu x {1, 2, . . . , m}; y {1 t a ncepnd din colul stnga-sus. Fiecare micare a unui arici duce la ma t s donatei x sau y a cmpului pe care se afla cu 1, adica suma coordonat cu 1 la fiecare micare a unui arici. n particular, un arici poate s v s a mult un cmp de pe diagonala x + y = k. Cea mai lung diagonal are a a min (m, n) (diagonalele cu aceasta lungime sunt x + y = m + 1, x + y = m x + y = n + 1 daca, de exemplu, asumam ca m n), prin urmare avem ne puin min (m, n) arici care s culeag toate merele de pe aceast diagona t a a a k min (m, n). Pe de alt parte, un numr de min (m, n) arici sunt su a a cazul m n, putem considera ca primul arici merge spre dreapta pna la livezii, apoi coboara pna la destinaie; al doilea merge o unitate n jos, t dreapta pn la marginea livezii, dup care coboar; al treilea merge dou a a a a jos .a.m.d. s Raspunsul este deci k = min (m, n).

G133. Fie 4ABC echilateral si D un punct astfel nct BD = DC, m \ 30 , iar BC separa A si D. Daca E (BD) cu m(BAE) = 15 , sa s CE AC. Enache Ptracu a s Not. A se vedea nota O problema si. . . noua soluii din acest numr a t a pag. 128.

G134. Se considera patrulaterul convex ABCD nscris ntr-un cerc d b b cm, avnd m(A) = 60 si m(B) = 45 . Sa se arate ca aria patrulaterul 2 mult egala cu 3 6 cm . Constantin Apostol, R Soluie (Gabriel Popa). Cu teorema sinusurilor n 4ABC i n t s obinem c AC = 2R sin 45 = 2 3 cm, respectiv BD = 2R sin 60 = t a 1 \ Dac = m(AC, BD), aria patrulaterului este S = AC BD sin i es a s 2

cnd = 90 . n cazul nostru, cum AC 2 + BD2 > (2R) (relaia revine la t deducem ca exista un patrulater cu diagonalele perpendiculare i de lungim s 1 i 3 2 cm, prin urmare maximul ariei se atinge i este Smax = 2 3 3 s s 2 cm2 .

G135. Fie tetraedrul ABCD cu AB = CD, AC = BD, AD = BC. S ca cel puin doua dintre unghiurile diedre formate de faa (ABC) cu feel t t t (ACD), (ABD) sunt ascuite. t Dan Br Soluie (Rzvan Ceuc, elev, Iai). Este evit a a s dent ca cele trei diedre nu pot fi toate neascuite; sa t presupunem prin absurd c dou dintre ele sunt neasa a cuite (anume cele de muchii BC i AC, ale cror mt s a a suri vor fi , respectiv , cu , 90 ), iar al treilea, anume cel de muchie AB, are masura < 90 . Se ob serv uor c proiecia O a lui D pe planul (ABC) a s a t aparine interiorului sau laturilor unghiului opus la t \ vrf lui ACB. Deducem ca AABC = AABO AACO ABCO , pr AABC = AABD cos AACD cos (180 ) ABCD cos (180 ). Tetra echifacial, deci AABC = AABD = AACD = ABCD i, dup simplicare, o s a 1 + cos (180 ) + cos (180 ) = cos . Cum cele trei cosinusuri sunt n intervalul [0, 1), ajungem la o contradicie. t

B. Nivel liceal

L126. Fie ABC un triunghi ascuitunghic. Mediatoarea laturii AB int t latura AC n T , iar mediatoarea laturii AC intersecteaza latura AB n arate ca paralela prin T la AB, paralela prin S la AC si simediana d concurente. Titu Zvonaru, C Soluia 1 (n maniera autorului). Notm cu b, c lungimile latu t a \ [ respectiv AB i fie = m(BAQ), = m(CAQ), unde Q este interseci s t prin T la AB cu paralela prin S la AC. Notm, de asemenea, {D} = AQ a Folosind teorema sinusurilor n 4ABD i n 4ACD, gasim ca s c sin BA = . (1) DC b sin Cum patrulaterul ASQT este paralelogram, folosind triunghiul ASQ, deducem c a c sin c SQ AT = = = 2 cos A = . (2) b sin AS AS b 2 cos A BD c2 Din (1) i (2) obinem ca s t = 2 , de unde AD este simediana DC b din A n triunghiul ABC.

Soluia 2 (Vlad Emanuel, student, Bucureti). Este cunoscut faptu t s diana dusa dintr-un vrf este locul geometric al mijloacelor antiparalelelo

opus (vezi, de exemplu, L. Niculescu i V. Bosko - Probleme practice de a s Ed. Tehnica, 1990). Cum noi dorim sa aratam ca diagonala AQ a parale ASQT este simediana, ar fi destul sa demonstram ca ST este antiparale AT AS deci ca = . Acest lucru este nsa evident, deoarece AT = AB AC 2 AC i astfel rezolvarea este ncheiata. AS = s 2 cos A L127. Fie A1 A2 A3 A4 A5 A6 un hexagon inscriptibil. Sa se arate ca rA1 A2 A3 + rA4 A5 A6 + rA1 A3 A6 + rA3 A4 A6 = rA3 A4 A5 + rA1 A2 A6 + rA2 A3 A6 + unde rXY Z este raza cercului nscris n 4XY Z. Ctlin Cali a a Soluie. Dac R este raza cercului circumscris hexagonului, este cunosc t a X Y Z rXY Z = 4R sin sin sin , unde XY Z este un triunghi avnd vrfuri 2 2 2 cu hexagonul. Vom demonstra nti urmatoarea Lem. Daca A1 A2 A3 A4 este un patrulater inscriptibil, atunci a rA1 A2 A3 + rA1 A3 A4 = rA1 A2 A4 + rA2 A3 A4 . ntr-adevar, cu notaiile din figura, vom avea: t + rA1 A2 A3 + rA1 A3 A4 = 4R sin sin sin + 2 2 2 + + sin sin sin ; 2 2 2 + rA1 A2 A4 + rA2 A3 A4 = 4R sin sin sin + 2 2 2 + + sin sin sin 2 2 2 + i dezvoltnd sin = sin cos + sin cos etc., obinem concluzia t s 2 2 2 2 2 s Aplicnd lema patrulaterelor inscriptibile A1 A2 A3 A6 i A3 A4 A5 A6 membru cu membru egalitile obinute, gsim tocmai relaia de demonst at t a t Not. Aceeai soluie a fost data de Vlad Emanuel, student, Bucur a s t L128. Sa se arate ca ntre medianele unui triunghi are loc inegalitatea X hY ih X Y X m2 m2 (ma + mb ) 2 m2 m2 m4 8 ma a b a b a Soluie. Avem, folosind cunoscutele x2 + y 2 + z 2 xy + xz + yz, sin t i 4R sin s A B C sin sin = r, ca 2 2 2

Dorel Bian i I.V.Maftei, B at s

a2 b2 + b2 c2 + c2 a2 abc (a + b + c) (b + c) sin = sin

A B (c + a) sin (a + b) sin 2 2

A B C sin sin 2p (a + b) (b + c) (c + a) = 2 2 2 r 2S S = (a + b) (b + c) (c + a) = (a + b) (b + c) (c + 4R r 2R

Apoi, s observm c are loc identitatea a a a

a4 +b4 +c4 +(a + b + c) (a + b + c) (a b + c) (a + b c) = 2 a2 b2 + b2 c de unde obinem ca t a4 + b4 + c4 + 16S 2 = 2 a2 b2 + b2 c2 + c2 a2

Pentru triunghiul de laturi a, b, c, exist un triunghi dual, avnd laturi a e P 2 2 S Q mc . Relaia (1) aplicat n triunghiul dual ne d c 2 t a a a ma mb e R i putem scrie, folosind (2), ca s P 2 2 P 4 e e e 2 ma mb ma S 16S 2 S = ma mb mc = = . e 4ma mb mc 4ma mb mc R e 4S Combinnd aceste relaii se obine concluzia problemei. t t

Not. Principial aceeai soluie a dat Marius Olteanu, inginer, Rm a s t

L129. n planul raportat la un reper cartezian xOy consideram vec n O: v1 (a1 , b1 ), v2 (a2 , b2 ), v3 (a3 , b3 ). Sa se arate ca exista un tetraed regulat, de muchie 1 si astfel nct OA, OB, OC se proiecteaza pe plan v1 , v2 , respectiv v3 daca si numai daca se verifica simultan relaiile: t 3 2 3 2 a1 + a2 + a2 a1 a2 a1 a3 a2 a3 = b1 + b2 + b2 b1 b2 b1 b3 2 3 2 3 2 2 3 (a1 b1 + a2 b2 + a3 b3 ) (a1 b2 + a2 b1 + a1 b3 + a3 b1 + a2 b3 + a3 b2 ) 2 Irina Mustaa, student t a Soluie. Completam reperul din plan la unul n spaiu Oxyz i fie A ( t t s B (a2 , b2 , c2 ), C (a3 , b3 , c3 ) astfel nct OABC este tetraedru regulat de Din OA = OB = OC = 1 deducem c a2 + b2 + c2 = a2 + b2 + c2 = a2 + a 1 1 1 2 2 2 3 \ \ [ 1, iar din m(AOB) = m(BOC) = m(COA) = 60 rezulta, via produs 1 a1 a2 + b1 b2 + c1 c2 = a1 a3 + b1 b3 + c1 c3 = a2 a3 + b2 b3 + c2 c3 = . Acest 2 pot fi scrise sub forma matriceala astfel: 1 a1 a2 a3 1 1 2 a1 b1 c1 2 a2 b2 c2 b1 b2 b2 = 1 1 1 . 2 2 1 1 a3 b2 c3 c1 c2 c3 1 2 2 3 1 1 1 a1 b1 c1 2 2 2 1 Fie X = a2 b2 c2 , iar A = 2 1 1 ; vom avea A1 = 1 2 2 1 1 a3 b2 c3 1 1 2 2 2 deci X X T A1 = I3 s Evident, de aici avem ca X T A1 X = I3 i, dupa efectuarea calculelor, se exact cele trei condiii din enunul problemei. t t S artm acum c aceste condiii sunt suficiente, adic s demonstrm a aa a t a a a gasi c1 , c2 , c3 care sa dea restul condiiilor din egalitatea (2). Ecuaiile n t t

c1 , c2 , c3 sunt: 3a1 a2 a3 a1 a1 a2 3a2 a3 3a3 c1 + c2 + + c3 + 2 2 2 2 2 2 2 2 2 3b1 b1 3b2 b1 b2 b2 b3 b3 3b3 c1 + c2 + + c3 + 2 2 2 2 2 2 2 2 2 2 3 c + c2 + c2 c1 c2 c1 c3 c2 c3 = 1. 2 3 2 1 Primele doua sunt ecuaii omogene cu 3 necunoscute, despre care se t t s t variabil liber, deci o soluie netrivial (c1 , c2 , c3 ). Cum a treia ecuaie est a a t a e e e i omogen n c1 , c2 , c3 , putem nmuli c1 , c2 , c3 cu un factor k astfel nct s a t e e e sa fie ndeplinita i rezolvarea problemei este ncheiata. s L130. Sa se arate ca pentru orice x, y 1 are loc inegalitatea 2 (xy x y) + 6 3 10 xy 6 3 9.

Gabriel Dospinescu, Paris i Marian Tetiv s Soluie. Lum x = a + 1, y = b + 1, cu a, b 0; inegalitatea de d t a devine a2 b2 + 6 3 10 (a + b + ab) 2ab 0. Cum a + b 2 ab i 6 3 10 > 0, ar fi suficient s artm c s a aa a a2 b2 + 6 3 10 2 ab + ab 2ab 0. a Cu notaia t = ab 0, am avea de justificat c t 4 2 t + 6 3 10 t + 2 6 3 10 t 0 f (t) 0, unde f : [0, ) R, (t) = t3 + 6 3 10 t + 2 6 3 10 . Deriva f 2 funciei este f 0 (t) = 3 t2 t 3 1 , care are ca singur rdcin p a a a a 3 1. E uor de vzut c acesta este punct de minim pentru f pe interva s a a prin urmare f (t) f 3 1 = 0, t 0, ceea ce ncheie demonstraia. t 2 Nota autorilor. De fapt, avem ca f (t) = t 3 + 1 t+2 32 conduce la concluzia dorit f (t) 0, t 0, ns aceast descompuner a a a greu de vzut. a Not. Soluii asemanatoare celei prezentate s-au primit de la Vlad a t student, Bucureti, precum i de la dl. Marius Olteanu, inginer, Rm. V s s L131. Sa se afle valoarea minima a numarului real k astfel nct, or a, b, c reale pozitive cu a + b + c = ab + bc + ca, sa aiba loc inegalitatea 1 1 1 (a + b + c) + + k k. a+b b+c c+a Andrei Ciupan, elev, B Soluii. n t particular, inegalitatea din enun trebuie sa aiba loc pentr t 3 9 9 c = 1; astfel, 3 k k k . Vom arta c este valoarea minim a a 2 8 8

(a + b + c)2 + (a + b + c) (a + b + c)2 + ab + bc + = a+b+c a+b+c (a + b) (b + c) + (b + c) (c + a) + (c + a) (a + b) = ; a+b+c 9 (b + c) (c + a) 1 astfel, ar fi suficient s demonstrm c (a + b + c) a a a a+b 8 (a + b + c) scriind nca doua inegalitai similare i sumndu-le, obinem chiar ceea s t t Aceasta ultima inegalitate se scrie succesiv: 2 9 (a + b) (b + c) (c + a) 8 (a + b + c) 9 (a + b) (b + c) (c + a) 8 (a + b + c) (ab + bc + ca) 9 (a + b) (b + c) (c + a) 8 (a + b) (b + c) (c + a) + 8abc (a + b) (b + c) (c + a) 8abc fapt care rezulta din inegalitatea mediilor. Not. Soluie corect a dat dl. Marius Olteanu, inginer, Rm. Vlce a t a L132. Fie a, b, c, x, y, z R si A = ax+by+cz, B = ay+bz+cx, C = a Daca |A B| 1, |B C| 1 si |C A| 1, aratai ca a2 + b2 + c2 x2 + t Adrian Zahariuc, ele Soluie. Deoarece distana pe axa real ntre oricare dou dintre nu t t a a B i C este cel puin 1, distana dintre cel mai mare i cel mai mic dint s t t s cel puin 2, deci cel puin unul dintre ele se afla la distana de cel puin t t t t origine. Putem presupune ca |A| = max {|A| , |B| , |C|} 1. Folosind iden Lagrange i inegalitatea CBS, obinem: s t 2 2 2 2 a + b2 + c2 x2 + y 2 + z 2 = (ax + by + cz) + (ay bx) + (bz cy) + ( a+b+c+1= (ax + by + cz) +
2

a lui k; pentru aceasta, ar trebui s artm c a aa a 1 1 1 9 (a + b + c) + + (a + b + c + 1) , a+b b+c c+a 8 oricare ar fi a, b, c R+ cu a + b + c = ab + bc + ca. Observam ca

(ay + bz + cx bx cy az)2 |B C| 2 = |A| + 3 3

n N. Gheorghe Iu Soluie. Notam an = f (n), n N; atunci 2an+3 an+2 = an+1 + an t 2an+4 an+3 = an+2 +an+1 +1, deducem c 2an+3 (an+4 an+2 ) = an+2 a a Prin urmare, 2an+3 |an+4 an+2 | = |an+2 an |, n N. Deoarece 2an an+1 + an + 1 1, rezulta ca an 6= 0, n 2. Daca exista n0 N cu an folosind relaiile anterioare gasim ca an0 +4 6= an0 +2 , an0 +6 6= an0 +4 , . . . , t s |an0 +2 an0 | > |an0 +4 an0 +2 | > |an0 +6 an0 +4 | > > 0, contradicie. Prin urmare, an+2 = an , n N. Notnd a1 = a3 = a5 = t a2 = a4 = a6 = b N, gasim 2ab = a + b 1, de unde a = 1, b = 2 = + 1, n par 2, n par b = 1, deci f (n) = sau f (n) = . 2, n impar 1, n impar

L133. Determinai funciile f : N N pentru care t t 2f (n + 3)f (n + 2) = f (n + 1) + f (n) + 1,

L134. Avem un colier cu n margele, numerotate consecutiv 1, 2, . . n 3. n cte moduri putem sa le coloram cu trei culori, astfel nct or margele consecutive sa aiba culori diferite? Iurie Boreico, elev, Soluie. Notam cu an numarul modalitailor de colorare i vom calc t t s cursiv. Evident ca a2 = a3 = 6. Fie n 4; putem alege culoarea ma 3 moduri, iar culorile mrgelelor 2, 3, . . . , n n cte dou moduri, obin a a t 3 2n1 modaliti de colorare n care mrgelele 1 i 2, 2 i 3, . . . , n at a s s culori diferite. Mai avem nsa o condiie: ca margelele n i 1 sa aiba culo t s atunci 3 2n1 = an + bn , unde bn este numarul colorarilor de tipul descr pentru care mrgelele 1 i n au aceeai culoare. Observm c bn = an1 , s a s s a a mrgelei n dnd o corespondena bijectiv ntre numrul colorrilor coresp a t a a a prin urmare an1 + an = 3 2n1 . Avem ca an + an+1 = 3 2n i, prin scadere, an+1 an1 = 3 2n1 . D s 2k1 a2k+1 = 3 2 + 22k3 + + 23 + a3 = 6 a2k2 + + 22 + 1 = 6

22k+1 2. Cum a2k + a2k+1 = 3 22k , vom avea ca a2k = 22k + 2. Raspu n fi scris sub forma an = 2n + 2 (1) . L135. Se considera un poligon cu 3n laturi, n 2, nscris ntr-un ce 1. Aratai ca cel mult 3n2 dintre segmentele avnd capetele n vrfurile p t au lungimea strict mai mare dect 2. Bianca-Teodora Iordache, elev a Soluie. Evident c oricum am alege 4 puncte pe cercul de raz t a a dou printre acestea situate la o distana cel mult egal cu 2. Consid a t a ful G (X, U ), unde X este mulimea vrfurilor poligonului iniial, iar doua t t fi unite printr-o muchie daca i numai daca distana dintre ele este strict t s dect 2. Conform observaiei iniiale, oricum am alege 4 vrfuri ale grafu t t dou care nu sunt unite printr-o muchie, deci G nu conine subgrafuri co a t ordin 4. Aplicam acum urmatorul rezultat: Teorema lui Turan. Daca G = (X, U ) este un graf neorientat cu n nu conine subgrafuri complete de ordin p, iar r este restul mparirii lui t t atunci p 2 n2 r2 r (r 1) |U | + . p1 2 2 n cazul nostru avem 3n vrfuri, p = 4, r = 0, prin urmare |U | 3n mult 3n2 distane formate cu vrfurile poligonului iniial sunt strict mai m t t Not. Soluie asemntoare a dat Vlad Emanuel, student, Bucuret a t a a s

Probleme propuse1
Clasele primare

P.154. Dorina are 15 baloane roii i albastre. Cte baloane roii p s s s dac numrul acestora este mai mic dect numrul baloanelor albastre a a a puin egal cu 3? t (Clasa I ) Inst. Maria R

P.155. Dintr-o carte lipsesc cteva pagini, de la numarul 71 la numaru foi lipsesc din aceasta carte? (Clasa I ) Ionela Brgan, e a a

P.156. La concursul "Desene pe asfalt", elevii claselor I-IV de la Sco Cazimir" au acumulat 50 de puncte i cel puin 2 premii din fiecare categ s t este cel mare numr de premii pe care-l pot primi elevii, dac pentru prem a a acordat 10 puncte, pentru premiul al II-lea s-au acordat 6 puncte, iar pent al III-lea s-au acordat 4 puncte? (Clasa a II-a) nv. Elena Po

P.157. Prin golirea unui singur vas, ales dintre cele de mai jos, pute restul vaselor s aib cantiti egale de lichid. Care vas trebuie golit? a a at

(Clasa a II-a)

Amalia Cantemir, e

P.158. Aflai trei numere naturale tiind c, adunndu-le dou cte dou t s a a 100, 89, respectiv, 141. (Clasa a III-a) Inst. Maria R

P.159. Se consider numerele: a = 1 + 4 + 7 + 10 + + 2008, b = 2 a + + 2009, c = 3 + 6 + 8 + + 2010. Aratai ca suma a + b + c se mp t la 3, fara sa calculai aceasta. t (Clasa a III-a) Iuliana Moldovanu, e

P.160. Numarul a este de forma xy0, iar numarul b este de forma uv. a i b tiind c a + b = 22 zeci. s s a (Clasa a III-a) Drago Toma, s

P.161. Fie a i b doua numere naturale astfel nct diferena lor este d s t mic dect suma lor. S se arate c numrul cel mai mare se mparte exa a a a a cel mai mic se mparte exact la 2. (Clasa a IV-a) Diana Tnsoaie, e a a

P.162. Maria are 9 sculei cu monede. Cel puin un scule cn a t t a t kilogram. n orice grupare de 5 saculei, cel puin 3 saculei au aceeai m t t t s orice grupare de 6 saculei, cel mult 5 saculei au aceeai masa. Care es t t s mare numr de sculei de un kilogram pe care l poate avea Maria? a a t (Clasa a IV-a) Petru Asa
1

Se primesc solu ii pn la data de 1 iunie 2009. t a

P.163. Jumtatea produsului a dou numere naturale consecutive m a a 3, nu poate da niciodata restul 2. Recreaii Stiinifice, Anul I (1883), nr. 4, t t

Clasa a V-a

V.95. Doua numere naturale se scriu n baza 10 folosind doar cifrele 1 Poate fi unul dintre numere de 2008 ori mai mare dect cellalt? a Ctlin Bude a a V.96. Determinai k, n N astfel nct t (1 + 1 n) + (2 + 2 n) + + (k + k n) = 3 4 5 6.
n n n

Petru Asa a V.97. Artai c numrul N = 17 +21 +25 , n N, nu poate fi ptr a t a a Virginia Grigorescu V.98. Fie n N . S se demonstreze c numrul N = 5050 . . . 505 (2n a a a se scrie ca suma a 4n + 2 patrate perfecte distincte. Veronica Pleu i Dan Pl a s s V.99. Se consider numrul N = 1 + 11 + 101 + 1001 + + | {z . 0 a a 100 . .

n cifre a) Pentru n N, n 5, aratai ca 5 | N 5 | n. t b) Precizai care dintre propoziiile "3 | n 3 | N " i "3 | N 3 t t s adevrat pentru orice n 3. a a Temistocle B V.100. Determinai numerele naturale nenule a i b pentru care exi t s a 3n + 2 i 3a + 2b < 100. astfel nct = s b 7n + 5 Gheorghe Iu

an + b , unde n, a, b, c, d N , astfel nc cn + d paritai diferite, iar a i c au aceeai paritate. Aratai ca, daca ad bc = 2 s s t t atunci fracia este ireductibila. t Cosmin Manea i Drago Petric s s V.101. Considerm fracia a t

Clasa a VI-a

VI.95. Determinai numerele naturale nenule a1 , a2 ,. . . , a2008 , tiind t s a2007 a2088 a2 a3 = = , iar a1 + a2008 = 2009. 23 2007 2008 Gheorghe Iu

VI.96. Determinai p N pentru care numerele p, p + 12, p + 22, p + t p + 102 i p + 132 sunt prime. s Damian Marinescu, T VI.97. a) Daca a, b, c, d, e, f N sunt astfel nct (a, b) = (c, d) = a c e (b, d) = 1, iat t = + N, aratai ca f = bd. t b d f 7 4 1 b) Determinai a, b N pentru care t b + N. 2a + 1 2 6 Cosmin Manea i Drago Petric s s

VI.98. Determinai cel mai mic numr natural n cu proprietatea c t a zerourilor n care se termina numarul (n + 10)! este cu 2008 mai mare dec zerourilor n care se termina n! (unde n! = 1 2 3 n). Ctlin Bude a a

VI.99. Un patrulater convex are doua laturi opuse congruente i d s congruente. Aratai ca patrulaterul este trapez isoscel sau dreptunghi. t Ioan Scleanu a a b b 90 . Sa se arate ca m(B) = 2m(C VI.100. Fie 4ABC cu m(A) numai daca exista M [BC] astfel nct AB = AM = M C. Petru Asa b = 90 i CD b VI.101. Fie ABC un triunghi dreptunghic cu m(A) s b unghiului C, D (AB). Perpendiculara din D pe bisectoarea unghiulu secteaza ipotenuza BC n E. Daca P este punctul de intersecie a bis t unghiurilor triunghiului ABC, iar M este punctul de intersecie dintre E t artai c M P A P BE. a t a \ \ Nela Ciceu, Bacu i Titu Zvonaru, C a s

Clasa a VII-a

VII.95. Fie ABCD ptrat, M un punct oarecare pe (AB), iar N a astfel nct M N M D. Aratai ca AM AB + CN CB = DM 2 . t Ovidiu Pop, Satu Mare i Gh. Szllsy, Sighetul M s

VII.96. Fie [AD] median n 4ABC, M mijlocul lui [AD], {E} = B a iar punctul F pe dreapta AB este astfel nct CF k AD. Demonstrai c t D, E i F sunt coliniare. s Mirela M

VII.97. Fie C1 (O1 , r1 ) i C2 (O2 , r2 ), r1 < r2 , dou cercuri tangent s a Considerm punctele A0 C1 , B 0 C2 , de aceeai parte a dreptei O1 O2 , a a s A0 O1 k B 0 O2 . Daca AB este tangenta comuna exterioara a cercurilor B C2 ), demonstrai c dreptele AB, A0 B 0 i O1 O2 sunt concurente. t a s Romana Ghia i Ioan Gh t t s

VII.98. Sa se determine numerele naturale nenule a i b, tiind ca s s s proporionale cu b 6 i a i invers proporionale cu a + 12 i b. t s s t s Constantin Apostol, R

s VII.99. Fie a, b Z i numerele A = 119a5 + 5b3 4a i B = 119b5 + s Sa se arate ca A se divide cu 120 daca i numai daca B se divide cu 120. s Dan Nedeianu, Dr. Tr

VII.100. Aratai ca 2a2 + 15b2 + 7c2 10ab 6ac + 20bc, a, b, c t Alexandru Negrescu, stud VII.101. Pentru n N , notm cu d (n) numrul divizorilor primi ai a a a) Determinai cardinalul mulimii A = {n N | n 208, d (n) = 3}. t t b) Aflai cel mai mic i cel mai mare element al mulimii t s t B = {k N | n N , n 2008, a.. d (n) = k} .

Gabriel P

Clasa a VIII-a
VIII.95. Pentru a, b, c R , notam = numarul x = b3 c3 a3 + 3 + 3 n funcie i . t s b3 c a

a b c a c b + + ,= + + . b c a c b a

Elena Nicu, Malu-Ma VIII.96. Rezolvai n numere naturale ecuaia x2 + y 2 + xy = x2 y 2 . t t Mihail Bencze VIII.97. Fie d1 , d2 , d3 , d lungimile diagonalelor feelor, respectiv diago t 2d2 d2 2 3 paralelipiped dreptunghic. Daca d2 = 2 1 2 , sa se arate ca paralelipip d2 + d3 d 3 . muchie de lungime cel puin egal cu t a 3 Gheorghe Molea, Curtea

VIII.98. Fie VABCD piramid patrulater regulat. Notm u = m((V a a a a \ CD)) i t = m((V BC) , (V AD)). Artai c u + v + t > \ v = m((V BC) , (V s a t a Claudiu Stefan P VIII.99. Pentru n N , considerm A = 12 , 22 , 32 , . . . , n2 . Dete a tiind c exist o funcie f : A A astfel nct f (x) f (y) = x y, s a a t Cristian L 2 2 n VIII.100. Rezolvai n n N ecuaia x 8 + 1287 = 0. t t Mihai Crciun a VIII.101. Se calculeaza suma cifrelor pentru fiecare dintre numerele n, n > 10. Pentru fiecare suma dintre cele n se calculeaza din nou sum repetndu-se aceast operaie pn cnd obinem n numere formate din ct a t a t cifr. S se afle n, tiind c n mulimea astfel obinut cifrele 1, 2, 3 i 4 a a s a t t a s de cte 101 ori fiecare, iar cifrele 5, 6, 7, 8 i 9 de cte 100 ori fiecare. s Mihai Ha

Clasa a IX-a

( + 1) ( n++ Gheorghe Costo AB 3 b IX.93. Fie 4ABC dreptunghic cu m(A) = 90 i s = , iar D m AC 2 [AC]. Notam cu E punctul de intersecie a cercurilor C1 (A, AD) i C2 (B, t s de aceeai parte a dreptei AB ca i punctul C. Determinai msura unghi s s t a a Ctlin Tigeru a a ( + 1) ( + i 1) + (1) ( + n 2) = (1)
n n

IX.91. Fie a, b, c, p R, p > 0. Daca ax2 + bx + c p, x [1 2 cx + bx + a 2p, x [1, 1]. Dorin Mrghidanu, a IX.92. Fie n N, n 3, iar , R astfel nct n + + 6= 0. Art a n2 X (1 + ) (n + ) i+1 (1) (1 + ) (n i (1 + ) (n 1 + ) + n++ i=1

IX.94. n 4ABC, I este centrul cercului nscris, iar {M } = AI BC \ strai c bisectoarea unghiului AM C, BI i AC sunt trei drepte concuren t a s b = 120 . numai dac m(A) a Vlad Emanuel, student i Andrei Cozma, elev, s

s t a IX.95. Dac xi [0, a], i = 1, n i xn+1 = x1 , demonstrai c a n X na2 xi+1 (a xi ) < . 4 sin2 4 i=1 Gigel Buth, Sa

Clasa a X-a
X.91. Artai c a t a 2 3 2 2 1 4 3 4 1 2 + arctg + = 2 arctg + arctg + arctg 7 4 16 7 4

D. M. Btineu-Giurgiu, B a t

X.92. Fie a, b C. Demonstrai ca ecua z 2 az + b = 0 are ambele t tia 2 modul 1 dac i numai dac |b| = 1 i |a| + a2 4b = 4. (n legatura cu as a s RecMat - 1/2007.) Marian Tetiv

X.93. Dac a1 , a2 , . . . , an (0, 1) sau a1 , a2 , . . . , an (1, ), iar f, g : {1 a {1, 2, . . . , n} sunt funcii injective, sa se arate ca t X X n n logak af (k) ak n2 . ag(k) k=1 k=1 Dan Popescu X.94. a) S se arate c a a p p p 2n + y 2n + xn y n + x2n + z 2n + xn z n x y 2n + z 2n + y n z n , x, y, z

b) Demonstrai ca, daca n este par, inegalitatea este stricta, iar daca n e t atunci exista x, y, z R pentru care se atinge egalitatea. Bogdan Victor Grigoriu, X.95. Consideram funcia f : R3 R3 , t

Determinai maximul i minimul funciei f . t s t

f (x, y, z) = sin x + sin y + sin z + sin (x y) + sin (y z) + sin (z

Ctlin Cali a a

Clasa a XI-a

XI.91. Fie matricele A, B, C, D Mn (R) astfel nct AC + BD AD = BC. Demonstrai c CA + DB = In i DA = CB. t a s I. V. Maftei, Bucureti i Mihai Ha s s 1 2 XI.92. Determinai matricele X Mn (R) pentru care X + X = t 1 Adrian Reisn

Gheorghe Costovici i Adrian Cordune s XI.94. S se demonstreze c pentru orice n N , exist numerel a a a 4 n s . x1 , x2 , . . . , xn (1, 2), aa nct x1 x2 xn = e Dan Pl 21 1 n1 1 1 + + 1 + XI.95. Calculai lim 1 + + 1 + t n n n n 1 este fixat. (n legatura cu L83 din RecMat-1/2005.) Marius Olteanu, Rm

n N .

XI.93. Studiai convergena irului (un )n1 definit prin u1 0, un+1 t t s

Clasa a XII-a
XII.91. Prove that
1 R 0

(1 + x) e(1+x)e dx = ee 1.

Zdravko Starc, Vra s XII.92. Fie b > a > 0, iar f : [a, b] R o funcie continu pe [a, b] i t a s c R pe (a, b); s se arate c exist c (a, b) astfel nct b f (x) dx = c (b c) a a a
a

Dan Nedeianu, Dr. Tr

2 R sin x 2 co XII.93. Demonstrai c exist c (2, ) pentru care t a a dx x c 1 Constantin Micu, Meline 2n R xa + b dx, unde a (0, ) i b R. s XII.94. Calculai lim n t n n x2a+4 + 1 Liviu Smarandache XII.95. Fie (A, +, ) un inel n care 0 6= 1 i 1 + 1 + 1 + 1 + 1 = 0. S s c, dac x3 y 2 = y 2 x3 , x, y A, atunci inelul este comutativ. a a I.V. Maftei, Bucureti i Mihai Ha s s

Semnalam cititorilor reeditarea coleciei complete a revistei t

RECREA TII STIIN TIFICE (1883-1

la 125 de ani de la apariia primului numr, cu respectarea formei n c t a publicat iniial. Revista prezint i astzi interes prin culoarea limbii a t a s a terminologiei folosite, prin coninutul interesant i de un nalt nivel tiinif t s s t i prin forma grafica frumoasa. Cei interesai pot consulta site-ul revistei s t de unde se poate prelua gratuit.

http://www.recreatiistiintifice.ro

Probleme pentru pregtirea concursurilo a


A. Nivel gimnazial

G146. Fie x, y, z (0, ) astfel nct xyz = 1. Aratai ca t 3 3 3 yz zx xy + 4 + 1. x4 + y + z y + z + x z4 + x + y Liviu Smarandache i Lucian Tuescu s t G147. Fie n N, n 2, fixat, iar a, b, c sunt numere naturale a hn 1i a + b + c n. na + (n + 1) b + 2nc = n2 + 1. Aratai ca n t 2 Gheorghe Iu a a a G148. Fie a1 a2 . . . ap N. S se arate c orice numr natural are u de forma a1 a2 . . . ap a1 a2 . . . ap . . . a1 a2 . . . ap 0 . . . 0. Marian Pan t G149. a) Determinai dou numere prime p, q astfel nct p < q, iar t a mai muli divizori naturali dect q 2 1. t b) Determinai toate numerele prime p pentru care p2 1 are exact o t naturali. Dan Popescu G150. Fie m i n numere naturale nenule cu proprietatea c m 1 + 2 s a S se arate c m poate fi scris ca suma ctorva numere distincte dintre 1, a a Marian Tetiv G151. Bazele unei prisme sunt poligoane cu 2008 vrfuri. Numero 2,. . . , 2008 vrfurile bazei inferioare i, corespunztor, cu a1 , a2 ,. . . , a200 s a bazei superioare, unde {a1 , a2 , . . . , a2008 } = {1, 2, . . . , 2008}. a) Demonstrai ca putem gasi o numerotare pentru baza superioara a t . . 8, i {1, 2, . . . , 2008}. i + ai . b) Demonstrai ca nu putem gasi o numerotare pentru baza superioara a t . . 9, i {1, 2, . . . , 2008}. i + ai . Gabriel Popa i Gheorghe Iu s G152. n triunghiul isoscel ABC (AB = AC) notm cu B 0 , C 0 picioa a milor din B, respectiv C. Dac AB = 2 B 0 C 0 , s se determine unghiurile tri a a Nela Ciceu, Bacu i Titu Zvonaru, C a s \ G153. n triunghiul ABC, M este mijlocul laturii [BC], m(ABC) \ m(ACB) = 105 . Perpendiculara din C pe AM taie AB n Q. Calcula QA raportului . QB Neculai Roman, Mirce G154. Fie D mijlocul laturii [BC] a triunghiului echilateral ABC d iar P un punct mobil pe [CD]. Notm cu M i N proieciile pe AP ale pu a s t respectiv C. Aflai aria locului geometric descris de segmentul [M N ]. t Marius Olteanu, Rm G155. Fie C cercul circumscris 4ABC ascuitunghic. Notm cu P p t a intersecie al tangentelor duse la cerc n B i C, {D} = AP C, iar M t s

mijloacele arcului mic BC, respectiv arcului mare BC. S se arate c dre a a DN i BC sunt concurente. s Gabriel P

B. Nivel liceal

L147. Se consider un poligon convex cu n laturi, n 4, avnd prop a oricare dou diagonale nu sunt paralele i oricare trei nu sunt concurente a s diferite de vrfurile poligonului. Se noteaz cu ni numrul punctelor de in a a diagonalelor interioare poligonului i cu ne cel al punctelor de intersecie s t poligonului. a) S se arate c exist exact opt poligoane care verific relaia ni > n a a a a t b) S se arate c exist exact trei poligoane pentru care ni + ne = kn2 a a a Mihai Ha

L146. n plan se considera dreptele d1 , d2 ,. . . , dn+1 , oricare doua n \ Notm cu k = m(dk , dk+1 ), k 90 , k = 1, n. Pe d1 se consider u a a de lungime 2 care se proiecteaza pe d2 , apoi segmentul obinut se proi t d3 i tot aa, pna cnd p dn+1 se obine un segment de lungime 1. t s s pe n tg min i | i = 1, n = 4 1, determinai unghiurile k , k = 1, n. t Cristian Svescu, student, B a

L148. Pe latura (AB) a triunghiului ABC considerm punctul D a a AB = 4 AD. De aceeai parte a laturii AB ca i punctul C, lum un s s a \ ACB i P B = 2 P D. Demonstrai ca patrulaterul A \s astfel nct P DA t inscriptibil. Nela Ciceu, Bacu i Titu Zvonaru, C a s

L149. S se determine poziia punctului P pe directoarea parabole a t nct aria triunghiului P T1 T2 sa fie minima, unde T1 i T2 sunt punctele s cu parabola ale tangentelor duse din P la P. Adrian Cordune

L150. Fie tetraedrul A1 A2 A3 A4 , iar P un punct n interiorul su. a Aij (Ai Aj ) proieciile ortogonale ale lui P pe muchiile Ai Aj ale te t Demonstrai ca t 1 VP A12 A13 A23 + VP A12 A14 A24 + VP A13 A14 A34 + VP A23 A24 A34 VA1 A2 A 4 Cnd se atinge egalitatea? Marius Olteanu, Rm

S a a a s h L151.2n+1 ise demonstreze ic nu exist numere naturale n i k a h k 2+ 3 = 4 + 15 . Cosmin Manea i Drago Petric s s L152. Pentru a, b, c R i x R+ , demonstrai inegalitatea s t
2 4

3 (x + 1) (a + b + c) 1 9 i h a2 + b2 + c2 a 2 + 1)(a2 + b2 + c2 ) + 2x(a + b + c)2 (ab + bc + ca)2 3(x

I. V. Maftei i Dorel Bian, B s at

Adrian Zahariuc, student, P L154. Fie P R [X] un polinom de gradul n i p : R R funcia s t al asociat. Stiind c mulimea {x R | p (x) = 0} are k elemente (dist a a a t funcia f : R R, f (x) = |p (x)| este derivabila pe R, aratai ca numarul t h i t n radacini complexe nenule ale lui P este egal cu 2 2k. 2 Vlad Emanuel, student, B a L155. Fie A, B M2 (C) dou matrice astfel nct matricea AB inversabila. Sa se arate ca urma matricei (I2 + AB) (AB BA)1 este eg Florina Crlan i Marian Tetiv s

L153. Gsii toate funciile f : R R cu proprietatea c a t t a 2 2 f x + xy + yf (y) = xf (x + y) + f (y) , x, y R.

Training problems for mathematical contes


A. Junior highschool level
G146. Let x, y, z (0, ) such that xyz = 1. Prove that

xy 3 yz 3 zx3 + 4 + 4 1. x4 + y + z y +z+x z +x+y Liviu Smarandache and Lucian Tuescu t G147. Let n N, n 2 be a xed number and let a, b, c be natura hn 1i such that na + (n + 1) b + 2nc = n2 + 1. Show that n a+b 2 Gheorghe Iu G148. Let a1 a2 . . . ap N. Show that every natural number has a m the form a1 a2 . . . ap a1 a2 . . . ap . . . a1 a2 . . . ap 0 . . . 0. Marian Pan t 2 G149. a) Determine two prime numbers p, q so that p < q, and p 1 natural divisors than q 2 1. b) Determine all the prime numbers p such that p2 1 has exactly eig divisors. Dan Popescu G150. Let m and n be nonzero natural numbers with the property 1 + 2 + + n. Show that m may be written as the sum of a couple numbers among 1, 2,. . . , n. Marian Tetiv G151. The bases of a prism are polygons with 2008 vertices. We n 1, 2, . . . , 2008 the vertices of the lower basis and by a1 , a2 ,. . . , a2008 the the upper basis, where {a1 , a2 , . . . , a2008 } = {1, 2, . . . , 2008}.

a) Show that we can nd a numbering for the upper basis so that i {1, 2, . . . , 2008}.

b) Show that we cannot nd a numbering for the upper basis so that i {1, 2, . . . , 2008}. Gabriel Popa and Gheorghe Iu G152. In the isosceles triangle ABC (AB = AC), B 0 , C 0 denote t the altitudes from B, respectively C. If AB = 2 B 0 C 0 , determine the an triangle. Nela Ciceu, Bacu and Titu Zvonaru, C a \ G153. In the triangle ABC, M is the midpoint of the side [BC], m(AB \ = 105 . The perpendicular from C on AM cuts AB at Q. and m(ACB) QA the value of the ratio . QB Neculai Roman, Mirce

G154. Let D be the midpoint of the side [BC] in the equilateral tria of side length 1, and let P be a moving point on [CD]. Denote by M a projections of the points B, respectively C on AP Find the area of the locus described by the segment [M N ]. Mariu Olteanu, Rm G155. Let C be the circumcircle of the acute-angled triangle 4ABC. P the intersection point of the tangents to the circle at B and C, {D} =

while M and N are the midpoints of the small arc BC, respectively of t

BC. Show that the straight lines AM , DN and BC meet at a point. Gabriel P

B. Highschool level

L146. The straight lines d1 , d2 ,. . . , dn+1 , are considered in the plane \ any two lines are not parallel. We denote by k = m(dk , dk+1 ), k 90 A segment of length 2 is cosidered on d1 that is projected on d2 , then th segment is projected on d3 and so on, until a segment of length 1 is ob p n 4 1, determine the dn+1 . Knowing that tan min i | i = 1, n = k = 1, n. Cristian Svescu, student, B a L147. A convex polygon with n sides, n 4, is considered such that a diagonals are not parallel and and any three diagonals do not meet at ot except the vertices of the polygon. Let us denote by ni the number of in points of the diagonals inside the polygon and by ne the number of intersec of the diagonals outside the polygon. a) Show that exactly eight polygons exist such that the inequality n satised. b) Show that exactly three polygons exist such that ni + ne kn2 , k N Mihai Ha L148. A point D is considered on the side (AB) of the triangle ABC AB = 4 AD. In the same halfplane as point C with respect to the side AB \ \ a point P such that P DA ACB and P B = 2 P D. Prove that the qu ABCP is inscriptible, that si it admits a circumscribed circle. Nela Ciceu, Bacu and Titu Zvonaru, C a

L149. Determine the position of the point P on the directrix line of th P, so that the area of the triangle P T1 T2 be minimum, where T1 and contact points with P of the tangents drawn from P to P. Adrian Cordune L150. Let us consider the tetrahedron A1 A2 A3 A4 , and a point P Denote by Ai j (Ai Aj ) the orthogonal projections of P on the edge(s) A tetrahedron. Prove that 1 VP A12 A13 A23 + VP A12 A14 A24 + VP A13 A14 A34 + VP A23 A24 A34 VA1 A2 A 4 When the equality is attained? Marius Olteanu, Rm h L151. Prove than no natural numbers n and k exist such that 2 + h k i 4 + 15 . Cosmin Manea and Drago Petric s L152. For a, b, c R and x R+ , prove the inequality

3 (x + 1)2 (a + b + c)4 1 9 i h a2 + b2 + c2 a 2 + 1)(a2 + b2 + c2 ) + 2x(a + b + c)2 (ab + bc + ca)2 3(x

Adrian Zahariuc, student, P L154. Let P R [X] a polynomial of degree n and p : R R its polynomial function. Knowing that the set {x 6 R | p (x) = 0} consists of k elements, and the function f : R R, f (x) = | p (x)| is dierentiable o hni that the maximum number of nonzero complex roots of P equals 2 2 Vlad Emanuel, student, B L155. Let A, B M2 (C) be two matrices such that the matrix A B invertible. Show that the trace of the matrix (I2 + AB) (AB BA)1 is Florentina Crlan and Marian Tetiv

I. V. Maftei and Dorel Bian, B at L153. Find all functions f : R R with the property that f x2 + xy + yf (y) = xf (x + y) + f 2 (y) , x, y R.

Concursul omagial "Recreaii Stiinifice" t t

Acest concurs este organizat cu prilejul mplinirii a 125 de ani de la apa tei "Recreaii Stiinice" (1883-1888), prima revist tiinic (predomina t t as t a atic) din ar adresat tineretului. a t a a

Organizatorii concursului: Asociaia "Recreaii matematice". t t Premiile prevzute de concurs: a Premiul I 200 lei (un premi Premiul II 100 lei (dou pre a Premiul III 50 lei (trei prem Participanii la concurs: orice elev al colilor de orice nivel. t s Obligaiile concurenilor: t t se cere rezolvarea celor cinci probleme enunate mai jos, selectate di t Recreaii Stiinifice i Recreaii matematice; t t s t elevii vor trimite soluiile prin pot (plic simplu timbrat) pe adresa t sa Asociaia "Recreaii matematice" t t str. Aurora, nr. 3, sc. D, ap. 6 700474, Iai s

cu meniunea (pe plic): Concursul "Recreaii Stiinifice"; t t t data limit de participare este 1.02.2009; a concurenii vor redacta ngrijit soluiile problemelor trimise (cte t t problema pe foaie, cu enun, figura etc.). t Acordarea premiilor se face pe baza punctajelor obinute de concur t fiecare problema este notata maxim cu 10 puncte; se acorda cte 2 puncte suplimentare pentru alte soluii ale problem t lizri etc; a se depuncteaz soluiile incomplete sau redactate nengrijit; a t concurenii trebuie sa obina t t minim 42 puncte, pentru premiul I, minim 35 puncte, pentru premiul II, minim 25 puncte, pentru premiul III. Sponsor: Fundaia culturala "Poiana" (director, Dan Tiba). t Premianii concursului vor fi anunai n nr. 1/2009 al revistei Recre t t t matice (ce va aprea n martie 2009). a Problemele concursului

1. Ion si Constantin merg la cumparaturi cu soiile lor, Maria si Elen t din aceste patru persoane cumpara un numar de obiecte ce le platete pe s atia lei cte obiecte a cumparat. Ion cumpara noua obiecte mai mult d t si fiecare so cheltuiete cu 21 lei mai mult dect soia sa. Care este soia t s t t care este a lui Constantin? Care este numarul de obiecte cumparate de fie aceste persoane? Care este suma cheltuita de fiecare dintre ele?

2. Sa se rezolve sistemul de ecuaii t (x + 2y) (x + 2z) = a, (y + 2x) (y + 2z) = b, (z + 2x) (z + 2y) = c (0 < a < b < c) . 0 3. Fie O, I, I centrele cercului circumscris triunghiului ABC, cercu acestuia si, respectiv, al cercului exnscris tangent laturii BC. Sa se demo 2 |sin B sin C| [ tg IOI 0 = . 2 cos A 1 4. Sa se taie o sfera cu un plan astfel nct diferena volumelor conur t ce au ca baze seciunea planului cu sfera si vrfurile pe sfera sa fie maxim t

5. Fie M un punct exterior cercului C de centru O si raza R. Notam punctele de contact ale tangentelor duse din M la C si cu A punctul de in dreptei OM cu cercul C care verifica condiia A [OM ]. Sa se determine t / punctelor M pentru care se poate construi un triunghi cu segmentele [M T si [M O], dar nu se poate construi un triunghi cu [M T1 ], [M T2 ] si [M A].

IMPORTANT

n scopul unei legturi rapide cu redacia revistei, pot fi utilizate urm a t adrese e-mail: t_birsan@yahoo.com i profgpopa@yahoo.co s aceasta cale colaboratorii pot purta cu redacia un dialog privitor t rialele trimise acesteia, procurarea numerelor revistei etc. Sugeram ratorilor care trimit probleme originale pentru publicare s le nume a s-i rein o copie xerox a lor pentru a putea purta cu uurina o as t a s t prin e-mail asupra acceptarii/neacceptarii acestora de catre redacia t

La problemele de tip L se primesc soluii de la orice iubitor de ma t elementare (indiferent de preocupare profesionala sau vrsta ). Fiecar soluiile acestor probleme - ce sunt publicate n revist dup un a t a a urmata de numele tuturor celor care au rezolvat-o.

Adresm cu insistena rugmintea ca materialele trimise r a t a s nu fie (s nu fi fost) trimise i altor publicaii. a a s t

Rugm ca materialele tehnoredactate s fie trimise pe adresa reda a a A soite de ierele lor (de preferina n L TEX). t s t

Pentru a facilita comunicarea redaciei cu colaboratorii ei, autorii t alelor sunt rugai s indice adresa e-mail. t a

Pagina rezolvitorilor

BRA OV S Colegiul Naional de Informatica "Gr. Moisil". Clasa a IX-a (prof. P t Florin). DARIE Flavius: VII.88, VIII(88-90), IX(88,89).

CRAIOVA Colegiul Naional "Carol I". Clasa a VIII-a (prof. STANCA Monica t CIU Ioan: VII(81,82), VIII(84-86), G(126,129,130,133).

IA I S Scoala nr. 11 "Otilia Cazimir". Clasa a III-a (inst. HUZUM Lina). Laura: P(144-148); HUZUM Andrei: P(144-148); MARIUTA Simina: P STOIAN Ioana: P(144-148). Scoala nr. 14 "Gh. Mrzescu". Clasa a III-a (inst. NUTA Elena). B George- tefan: P(144-148); POSTUDOR G dor: P(144-148); CHIRILUTA S Mdlina: P(144-148); STOICA Adriana: P(144-148). a a Scoala nr. 26 "G. Cobuc". Clasa a III-a (inst. RACU Maria). AP s Aura Georgiana: P(144-149); BURA Emma-Andreea: P(144-149); FIL Stefania: P(144-149); GHEORGHITA Narcis-Eugen: P(144-149); HRISC Constantin: P(144-150); HUZA Mdlina: P(144-150); LE OVSCHI A a a S Ioana: P(144-149); LUPU Roxana-Elena: P(144-149); MARICIUC Drago P(144-149); MAXIM Alexandra-Camelia: P(144-149); TUDOSE Ema-Alin 150); TUCA Cosmin: P(144-149); VASILE Bogdan-Andrei: P(144-149). Cl a (nv. HRIMIUC Valeria). BRUMA Andrei-Alexandru: P(144-150); DU Bianca: P(144-150); HARAPCIUC Eduard-Gabriel: P(144-150); MANTA Adrian: P(144-150); OLARU Alexandra: P(144-150). Colegiul Naional. Clasa a V-a (prof. POPA Gabriel). STOLERU t Ingrid: V(88-90,93,94). Colegiul Naional "C. Negruzzi". Clasa a VII-a (prof. SAVA Radu) t Norbert Traian: VII(81-86), G126.

SUCEAVA Scoala cu clasele I-VIII, nr. 3. Clasa a III-a (nv. TABARCEA FECHET Stefan: P(136-139,141,143); Clasa a IV-a (inst. NECHITA Dan CHET Mircea: P(134-141,143).

Premii acordate rezolvitorilor


Scoala nr. 14 "Gh. Mrzescu", Iai s

BACIU Tudor (cl. a III-a): 2/2007(10pb), 1/2008(6pb), 2/2008(5pb), CHIRILUTA George- tefan (cl. a III-a): 2/2007(10pb), 1/2008(6pb), 2 S STOICA Adriana (cl. a III-a): 2/2007(10pb), 1/2008(6pb), 2/2008(5pb

Scoala nr. 26 "G. Cobuc", Iai s s GHEORGHITA Narcis-Eugen (cl. a III-a): 2/2007(5pb), 1/2008(6pb),

Revista semestrial RECREAII MATEMATICE este ed ASOCIAIA RECREAII MATEMATICE. Apare la datele de 1 1 septembrie i se adreseaz elevilor, profesorilor, studenilor i tutu pasionai de matematica elementar. n atenia tuturor colaboratorilor Materialele trimise redaciei spre publicare (note i articole, che metodic, probleme propuse etc.) trebuie prezentate ngrijit, clar i co trebuie s prezinte interes pentru un cerc ct mai larg de cititori. Se reco textele s nu depeasc patru pagini. Evident, ele trebuie s fie origin nu fi aprut sau s fi fost trimise spre publicare altor reviste. Rugm rialele tehnoredactate s fie nsoite de fiierele lor. Problemele destinate rubricilor: Probleme propuse i Problem pregtirea concursurilor vor fi redactate pe foi separate cu enun i de ie/rezolvare (cte una pe fiecare foaie) i vor fi nsoite de numele autoru la i localitatea unde lucreaz/nva. Redacia va decide asupra oportunitii publicrii materialelor prim n atenia elevilor Numele elevilor ce vor trimite redaciei soluii corecte la proble rubricile de Probleme propuse i Probleme pentru pregatirea conc vor fi menionate n Pagina rezolvitorilor. Se va ine seama de regulile: 1. Pot trimite soluii la minimum cinci probleme propuse n prezent i cel anterior al revistei; pe o foaie va fi redactat soluia une probleme. 2. Elevii din clasele VI-XII au dreptul s trimit soluii la pr propuse pentru clasa lor, pentru orice clas mai mare, din dou clase m imediat anterioare. Elevii din clasa a V-a pot trimite soluii la problemel pentru clasele a IV-a, a V-a i orice clas mai mare, iar elevii claselor trimite soluii la problemele propuse pentru oricare din clasele primare i s mai mare. Orice elev poate trimite soluii la problemele de concurs (tip 3. Vor fi menionate urmtoarele date personale: numele i pr clasa, coala i localitatea. 4. Plicul cu probleme rezolvate se va trimite prin pot (sau v direct) la adresa Redaciei: Prof. dr. Temistocle Brsan Str. Aurora, nr. 3, sc. D, ap. 6, 700 474, Iai Jud. IAI E-mail: t_birsan@yahoo.com

CUPRINS

O sut de ani de la naterea lui GHEORGHE GHEORGHIEV ...................... ILIE POPA 100 de ani de la natere..................................................................... Simpozion dedicat revistei "Recreaii tiinifice" (1883-1888) ............................

ARTICOLE I NOTE

F. REICHER Despre calendar................................................................................ M. TETIVA Cteva probleme de teoria numerelor a cror rezolvare se bazeaz pe identiti .............. C. IGERU O caracterizare a punctului Mathot .............................................. D. POPESCU Unsprezece ptrate perfecte ........................................................... T. BRSAN Cercuri seminscrise i puncte de tip Gergonne sau Nagel.......... F. POPOVICI O rafinare a inegalitii lui Jensen .............................................. Gh. IUREA Asupra unor inegaliti geometrice .................................................. T. ZVONARU Metoda deligamentrii i rafinarea unor inegaliti...................

CHESTIUNI METODICE CORESPONDENE CONCURSURI I EXAMENE

Gh. IUREA, G. POPA O problem i nou soluii.......................................

A. REISNER Sur les matrices magiques..............................................................

Concursul de matematic "Al. Myller", ed. a VI-a, 2008....................................... Concursul de matematic "Florica T. Cmpan", 2008 ............................................ Concursul "Student pentru o zi", Suceava................................................................

PROBLEME I SOLUII

Soluiile problemelor propuse n nr. 2/2007............................................................. Soluiile problemelor pentru pregtirea concursurilor din nr. 2/2007 .................... Probleme propuse........................................................................................................ Probleme pentru pregtirea concursurilor ................................................................. Training problems for mathematical contests ..........................................................

Concursul omagial "Recreaii tiinifice" ............................................................

Pagina rezolvitorilor ................................................................................................. ISSN 1582 1765

S-ar putea să vă placă și